You are on page 1of 1011

2164_IFC_C2 30/03/12 12:25 PM Page 1

CONTENTS IN BRIEF

Chapter 1: Orientation and Pre-Tests 1 Chapter 9: Physiological Integrity: Basic Care


and ComfortNutrition 689
Sally Lambert Lagerquist Janice Lloyd McMillin
Robyn Marchal Nelson Denise Wall Parilo Sally Lambert Lagerquist Janice Lloyd McMillin
Kathleen E. Snider Janis Ryan Wisherop Robyn Marchal Nelson Denise Wall Parilo
Kathleen E. Snider
Chapter 2: Guidelines and Tips for International
Nurses and Repeat Test-Takers 59 Chapter 10: Psychosocial Integrity: Behavioral/
Mental Health Care Throughout
Sally Lambert Lagerquist
the Life Span 711

Chapter 3: Safe, Effective Care Environment: Sally Lambert Lagerquist Janis Ryan Wisherop (Reviewer
Management of Care, Cultural Diversity, and Contributor to Selected
Ethical and Legal Aspects of Nursing, Sections)
Nursing Trends, and Safety and
Infection Control 69 Chapter 11: Reduction of Risk Potential: Common
Procedures: Diagnostic Tests, Oxygen
Sally Lambert Lagerquist Janice Lloyd McMillin
Therapy, and Hands-on Nursing Care 825
Robyn Marchal Nelson Denise Wall Parilo
Kathleen E. Snider Janice Lloyd McMillin Robyn Marchal Nelson

Chapter 4: Health Promotion and Maintenance: Chapter 12: Final Tests 1 and 2 865
Nursing Care of the Childbearing
Sally Lambert Lagerquist Janice Lloyd McMillin
Family 115
Robyn Marchal Nelson Denise Wall Parilo
Janice Lloyd McMillin Kathleen E. Snider Janis Ryan Wisherop

Chapter 5: Health Promotion and Maintenance: Appendices


Nursing Care of the Pediatric Client 249
Appendix A: Laboratory Values 909
Kathleen E. Snider Denise Wall Parilo
Appendix B: Memory Aids 915
Appendix C: Common Acronyms and
Chapter 6: Physiological Integrity: Nursing
Abbreviations 919
Care of the Adult Client 347
Appendix D: Quick Guide to Common Clinical
Robyn Marchal Nelson Signs 925
Appendix E: Complementary and Alternative
Chapter 7: Physiological Integrity: Nursing Care Therapies 927
of the Geriatric Client
579 Appendix F: Index: Content Related to NCLEX-RN
Test Plan: Four General Client Needs
Robyn Marchal Nelson
and Six Client Subneeds (With Detailed
Chapter 8: Physiological Integrity: Knowledge, Skills, and Abilities) 933

Pharmacological and Parenteral Appendix G: Index: Questions Related to Client


Therapies 603
Needs/Subneeds 939
Appendix H: Reduction of Risk Potential:
Sally Lambert Lagerquist Janice Lloyd McMillin Index to Diagnostic Tests and
Robyn Marchal Nelson Kathleen E. Snider Procedures 943
Index 945
2164_FM_i-xxviii 06/04/12 5:30 PM Page i

Daviss

NCLEX-RN
Success
Third Edition

Money
Back Guarantee
If you are a graduate of a nursing program accredited in the United States,
take the NCLEX-RN for the first time, and do not pass after using
Daviss NCLEX-RN Success, return the book to F. A. Davis Company,
Customer Service, 404-420 N. 2nd Street, Philadelphia, PA 19123.
Enclose your original receipt for purchase of the book and copies of your
official test results notification and your certification of graduation.
We will refund the price you paid for the book. If you have
any questions, please call 800-323-3555.
2164_FM_i-xxviii 06/04/12 5:30 PM Page ii

...
Online Resource Center
DavisPlus is your online source for a wealth of learning resources and
teaching tools, as well as electronic and mobile versions of our products.

Students Instructors
Unlimited free access. Upon Adoption.
No password. Password-protected library of
No registration. title-specific, online course content.
No fee.

Visit http://davisplus.fadavis.com

Explore more online resources from f.a.davis...


www.drugguide.com
powered by is Daviss Drug Guide Online, the complete Daviss Drug Guide for Nurses
Unbound Medicine database of over 1,100 monographs on the web.

www.tabersonline.com
powered by delivers the power of Tabers Cyclopedic Medical Dictionary on the web.
Unbound Medicine Find more than 60,000 terms, 1,000 images, and more.

www.LabDxTest.com
is the complete database for Daviss Comprehensive Handbook of
powered by Laboratory and Diagnostic Tests with Nursing Implications online.
Unbound Medicine Access hundreds of detailed monographs.

www.fadavis.com
2164_FM_i-xxviii 06/04/12 5:32 PM Page iii

Daviss

NCLEX-RN
Success
Third Edition

Edited by
Sally Lambert Lagerquist, RN, MS
Written by
Sally Lambert Lagerquist, RN, MS Denise Wall Parilo, RN, MSN, PhD(c)
Former Instructor of Undergraduate, Graduate, and Associate Professor, School of Nursing, California State
Continuing Education in Nursing, School of Nursing, University, Sacramento
University of California, San Francisco Per Diem Staff Nurse, Pediatrics, Sutter Memorial
President, Review for Nurses, Inc., and RN Tapes Company, Hospital, Sacramento, California
San Francisco, California
Kathleen E. Snider, RN, MSN, CNS
Janice Lloyd McMillin, RN, MSN, EdD Professor of Nursing, Los Angeles Valley College,
Former Lecturer in Perinatal Nursing, School of Nursing, Valley Glen, California
California State University, Sacramento
Staff Nurse, Labor and Delivery, Methodist Hospital,
Janis Ryan Wisherop, RN, MSN
Professor, College of San Mateo
Sacramento, California
Home Health Hospice Nurse, San Francisco, California
Robyn Marchal Nelson, DNSc, RN
Dean, College of Nursing, West Coast University, Irvine, California
2164_FM_i-xxviii 06/04/12 5:32 PM Page iv

F. A. Davis Company
1915 Arch Street
Philadelphia, PA 19103
www.fadavis.com

Copyright 2012 by F. A. Davis Company

Copyright 2012 by F. A. Davis Company. All rights reserved. This book is protected by copyright. No part of it may be reproduced, stored in a
retrieval system, or transmitted in any form or by any means, electronic, mechanical, photocopying, recording, or otherwise, without written permis-
sion from the publisher.
A modified version of this text was previously published as Addison-Wesleys Nursing Examination Review by Addison-Wesley Nursing, a Division of
the Benjamin/Cummings Publishing Company, Inc. Copyright 1977, 1981, 1987, and 1991; and Little, Browns NCLEX-RN Examination Review
by Little, Brown and Company. Copyright 1996 by Sally L. Lagerquist.

Printed in the United States of America

Last digit indicates print number: 10 9 8 7 6 5 4 3 2 1

Publisher, Nursing: Robert G. Martone


Director of Content Development: Darlene D. Pedersen
Sr. Project/NCLEX Editor: Padraic J. Maroney
Illustration and Design Manager: Carolyn OBrien
Project Manager, Electronic Publishing: Tyler Baber

As new scientific information becomes available through basic and clinical research, recommended treatments and drug therapies undergo changes.
The author(s) and publisher have done everything possible to make this book accurate, up to date, and in accord with accepted standards at the time
of publication. The author(s), editors, and publisher are not responsible for errors or omissions or for consequences from application of the book, and
make no warranty, expressed or implied, in regard to the contents of the book. Any practice described in this book should be applied by the reader
in accordance with professional standards of care used in regard to the unique circumstances that may apply in each situation. The reader is advised
always to check product information (package inserts) for changes and new information regarding dose and contraindications before administering
any drug. Caution is especially urged when using new or infrequently ordered drugs.

Library of Congress Cataloging-in-Publication Data

Daviss NCLEX-RN success / edited by Sally Lambert Lagerquist ; written by Sally Lambert Lagerquist . . . [et al.]. 3rd ed.
p. ; cm.
NCLEX-RN success
Includes bibliographical references and index.
ISBN-13: 978-0-8036-2164-0
ISBN-10: 0-8036-2164-7
I. Lagerquist, Sally L. II. Title: NCLEX-RN success.
[DNLM: 1. Nursing CareExamination Questions. 2. Nursing CareOutlines. WY 18.2]

610.73076dc23 2011038536

Authorization to photocopy items for internal or personal use, or the internal or personal use of specific clients, is granted by F. A. Davis Company
for users registered with the Copyright Clearance Center (CCC) Transactional Reporting Service, provided that the fee of $.25 per copy is paid
directly to CCC, 222 Rosewood Drive, Danvers, MA 01923. For those organizations that have been granted a photocopy license by CCC, a
separate system of payment has been arranged. The fee code for users of the Transactional Reporting Service is: 8036-2164-0/12 0 + $.25.
2164_FM_i-xxviii 06/04/12 5:32 PM Page v

DEDICATIONS

This book is dedicated to:


TOM my husband,
43 years isnt enough to share all the 1001 things we want to experience. Heres to taking more time
for enjoying our three granddaughters (Kaya, Kasey and Audrey), organizing our museum and for
finally using our wedding Satsuma chinaware. Your love and devotion, your Scandinavian patience com-
bined with your Irish good humor have been my mainstay while I cranked out 29 editions of seven dif-
ferent nursing books during my professional lifetime.
ELANA our daughter,
Your creativity and talent, energy and commitment to always giving your all to all that you do be
it Stagewrite, parenting your two daughters, Kaya and Kasey, or fostering family unity, make you so very
special.
DAN our son-in-law,
A special thank you for putting reviewfornurses.com and the 6-eBookLibrary online, on smart phone
or tablet device, and on DVD making us the oldest independent, successful all-in-one nursing exam
prep company. Your job has been never-ending and always innovative!
KALEN our son,
Your name brings to mind some of your very special gifts:
Kindness to and respect for generations behind you.
Ambition to seek and grow with opportunities.
Living with passion and laughter.
Embracing change, not fearing it.
Nurturing those you love (Lara and Audrey especially).
With gratitude for sharing your love, celebrations and care and concern.

To our third generationthe 3 granddaughters: Kaya, 9, Kasey, 5, Audrey, 3 (and another grandchild
on the way)each of whom is very precious to me in her own very special way:
KAYA for your sweet, kind and thoughtful ways, and a gentle manner. You are fun and enjoyable to
be with, and insightful, sensitive, sentimental and creative at the same time. (We love your notes and
artwork that you leave around for us to find all over the house.)
KASEY for your enthusiasm and boundless energy as you laugh and play with us, with friends, and
with your collection of dolls and stuffed animals. You delight us with your remarkable singing (on pitch!
and you pick up new melodies in Swedish, Russian, Jewish, and Hawaiian really fast!). We love your
affectionate and sharing ways.
AUDREY for your giant hugs and loving smiles, as well as your growth and linguistic development
in the way you seek and quickly grasp and enjoy new experiences (e.g., picking the correct numerical
days on your Advent calendar, learning Italian phrases, and retaining how to say I love you in Russian).
May you continue to always enjoy both Chanukah and Christmas as part of your own cultural diversity.
May you also continue to enjoy playing with and bonding with your cousins, Kaya and Kasey.
Sally Lambert Lagerquist

First, I thank God for His unending grace, for the many blessings in my life, and for the gift of salvation
in Christ Jesus.
To my husband, Mike, who supports me in everything I do, thank you for your humor and your love.
We are a great team. I look forward to a bright future with you. You are truly the best thing that ever
happened to me.
To my sweet little Leilani, the light of our life, the girl who makes us smile every day, and I am truly
blessed that she calls me Mommy.
To my daughter Rachel, I am proud of the nurse you have become and enjoy watching you grow in
your practice.
To my son Justin, keep moving forward. I know God has a plan for your life.

v
2164_FM_i-xxviii 06/04/12 5:32 PM Page vi

vi Dedications

To my parents, John and LaVerne Mahony, I miss you both very much and one day, I will see you again.
You both always believed I was capable of achieving whatever I could dream of and for that, I am
eternally grateful.
To my big brother John, thank you for being my example. You made me want to achieve even more
to keep up with your great accomplishments.
To my brother Brian, who has been making God and all of heaven laugh since 1993, I miss your smile
and practical jokes.
To my mentors, Dr. Mollyn Bohnen and Dr. Louise Timmer, who helped shape me as a nurse and a
faculty member, I am forever grateful for your attention and encouragement.
To all the nursing students whose enthusiasm inspires me, every time I see a student cry at the birth of a
child, I remember why I chose to be a perinatal nurse. It is a pleasure to mentor the next generation of
nurses. I have never had a day I wanted to be anything but a nurse, and I am proud to say I am a nurse.
Jan Lloyd McMillin

This book is dedicated


To those who have chosen nursing as their passion, as their life work . . . one education and 1000
career choices! Thanks for choosing NCLEX-RN Success to increase your knowledge for the practice of
nursing. Thanks for choosing a career as a professional nurse. Nurses are the glue of healthcare . . . and
year after year they are voted the most trusted profession by consumers. Nurses make a difference in the
lives of those they care for.
This book is dedicated
To my family which is composed of members from four generational groupsmy husband
Dean (the mature generation), daughters Tina (the nurse and a baby boomer) and Kelly (the musician-
graphic designer and Gen Xer), and my grandsons Stirling and Sage (the millennials). Nurses are
prepared to give high quality care across all generationsnot just one. Our lives and our health will be
better because of nursesbecause of you!
Thanks to each of you for touching my life daily!
Robyn Marchal Nelson

To my husband Chris, for always being the eager listener for a new test question and for support through-
out the editing process.
To Mom, Jeri, and Joe, who took such good care of Will and Zach when I needed to write.
To Sally and Robyn, mentors and friends, who supplied humor and heart every day of this book
project. You two are the best!
To nursing students, both present and past, for choosing this profession, for always doing your best,
and for continuing to give me such a richly rewarding career. I hope this book helps you succeed in the
next phase of your journey!
Denise Wall Parilo

This book is gratefully dedicated:


To Terry Oleson, PhD. Thank you for your endless patience and guidance in helping me find the
answers to my lifes questions.
To my nursing instructors, religious and secular, who taught me that I wanted to be a pediatric nurse
and had the right stuff with which to do it.
To my pediatric patients and their families who taught me how to be a pediatric nurse.
To my pediatric nursing students who taught me how to teach pediatric nursing.
To my friends and colleagues in the pediatric community who, with their combined experience and
wisdom, gave me insight into the writing of pediatric content and taught me that there is always a light
at the end of the tunnel and that this book was not an everlasting project.
And, finally, to Steevio Bardakjian, my resident computer genius and guru who taught me the
mysteries of the computer world.
Thank you!
Kathy Snider
2164_FM_i-xxviii 06/04/12 5:32 PM Page vii

Dedications vii
I dedicate my work involved in this book first and foremost to my beautiful and loving daughters Molly
and Katie. You both have always been my inspiration in everything I do and always willI love you both
more than you will ever know.
During the time in editing, authoring the chapters and developing questions for this book, my life
was occupied with personal emotional distress and the most challenging experiences ever. I thank Sally
Lagerquist for giving me this opportunity to have this great distraction and time-consuming endeavor
as I coped and began a new journey in life. Sally, you are not only my partner in this book, you are a
dear and loving friend. Without your ongoing support, insistence on perfection and keeping me sane
through it all, I never could have been successful in achieving the final edited and authored copy.
Lastly, I dedicate this to my family, co-workers and friends, who encouraged me and cheered me on
during some of the lowest points of my personal life. Without you, I could not have done this.
And, finally to my mother, Marilyn Evans, a nurse at heart and the most loving, God inspired woman
on the face of the Earth. You are my role model in everything I do as a nurseI have become who I am
and care for my patients the way I do because of your example. Thanks for being my mom and, most of
all, my very best friend.
I am forever grateful to all the wonderful people in my life.
Janis Ryan Wisherop
2164_FM_i-xxviii 06/04/12 5:32 PM Page viii
2164_FM_i-xxviii 06/04/12 5:32 PM Page ix

ABOUT THE EDITOR/AUTHORS

Sally Lambert Lagerquist, RN, MS, is currently the author Robyn Marchal Nelson, DNSc, RN, is Dean of the College
and editor of three editions of Daviss NCLEX-RN of Nursing at West Coast University in Irvine, California.
Success; four editions of Practice Questions and Answers for Prior to her position at WCU, she was Chairperson and
NCLEX-RN, published by Review for Nurses Tapes Co.; Professor in the Division of Nursing at California State
How to Pass Nursing Exams, published by Review Press University, Sacramento, and Dean, College of Health and
and later by ATI, and now available as an e-book at Human Services at Touro University in Nevada. She
www.reviewfornurses.com; four editions of Addison received her BS in Nursing from Loma Linda University,
Wesleys Nursing Examination Reviews (19771991); her MS from Boston University, and a Doctor of Nursing
four books for the NURSENOTES, Core Content Science from the University of California, San Francisco.
At-A-Glance series: Psychiatric-Mental Health, Maternal- In addition to Daviss NCLEX-RN Success, she was
Newborn, Pediatrics, and Medical-Surgical, published by coauthor of NURSENOTES: Medical-Surgical Nursing,
Lippincott and later by ATI, and now available as e-books at Core Content At-A-Glance (Lippincott) and a contributor
www.reviewfornurses.com; and Little, Browns NCLEX-RN to Nursing Q and A (Lippincott), and has developed a
Examination Review, and Little, Browns Nursing Q&A: number of audio and visual aids for students preparing for
Critical-Thinking Exercises, published by Little, Brown and the licensure examination. Dr. Nelson was a lecturer for
Company and later by ATI, and also available as e-books Review for Nurses, Inc., for over 20 years, both at on-site
at www.reviewfornurses.com. She is president and course national review courses and as one of the faculty for the
coordinator of Review for Nurses, Inc. She has coordinated first ever nationally televised satellite review course that
RN licensure exam review courses on campuses nation- was offered for 7 years. She holds memberships in Sigma
wide since 1976. She is presently a lecturer on test-taking Theta Tau International, ANA, ANA/California, and Phi
techniques at workshops held for graduating senior nurs- Kappa Phi. Dr. Nelson is a member of the Nurse Week
ing students. She has produced and developed the (California) Advisory Board, and the academic representa-
NCLEX-RN Board Game and audio CDs, CD-ROMs, tive to the Association of California Nurse Leaders
DVDs, and streaming videos on Nursing Review and (ACNL) Board of Directors.
Successful Test-Taking Techniques for Nurses. She origi-
nated, developed, and has presented national satellite tele- Denise Wall Parilo, RN, MSN, PhD(c) is an Associate
courses for NCLEX-RN review since June 1989. She has Professor in the School of Nursing at California State
developed an online (reviewfornurses.com) NCLEX-RN University, Sacramento. In addition to lecturing on pedi-
mobile review course (also available for smart phone or atric nursing, she is an advisor to the Sacramento State
tablet device to use anywhere, anytime), and an online Pre- Chapter of the California Nursing Students Association
NCLEX-RN Assessment Series for Success (PASSTM) to (CNSA) and has served as coordinator of the undergradu-
help candidates assess and test with the test bank to fill in ate nursing program. Her experience ranges from pediatric
gaps in knowledge as well as increase their test-taking skills. intensive care to public health nursing, and she continues
She has been a marriage, family, and child counselor and is to work as a per diem nurse in acute care pediatrics at
a member of Sigma Theta Tau International. She has been Sutter Memorial Hospital in Sacramento. She is a con-
a faculty member at the University of California at San tributing author to How to Pass Nursing Exams by Sally
Francisco School of Nursing for over 10 years, where she Lagerquist, RN, MS, and has been a lecturer for Review
also received her BS and MS degrees. for Nurses, Inc. She received both her BS in Nursing
and MS in Nursing from California State University,
Janice Lloyd McMillin, RN, MSN, EdD, is a former lec- Sacramento, and has been a Fulbright Scholar in Finland,
turer in perinatal nursing in the Division of Nursing at doing community health nursing research. She is presently
California State University, Sacramento. In addition, she a doctoral student at the University of California,
currently works in Labor and Delivery at Methodist San Francisco School of Nursing.
Hospital in Sacramento. She has been a lecturer for sever-
al years for Review for Nurses, Inc. She received her BA in Kathleen E. Snider, RN, MSN, CNS, has been a pediatric
Biology, BS in Nursing, and MS in Nursing from CSU, nurse for over 40 years and has a broad range of clinical
Sacramento. She completed her doctorate at the and educational experience. She received her diploma in
University of San Francisco. Her dissertation study exam- nursing from Saint Vincents College of Nursing, Los
ined the transition experiences of first-year labor and Angeles; her baccalaureate in nursing from Mount Saint
delivery nurses. She has been a perinatal nurse for over Marys College, Los Angeles; and her masters degree in
27 years. nursing from California State University, Los Angeles. She

ix
2164_FM_i-xxviii 06/04/12 5:32 PM Page x

x About the Editor/Authors

has been teaching pediatric nursing since 1976 in diplo- she was awarded the Certificate of Distinguished Service by
ma, associate of arts, and baccalaureate nursing programs. the Los Angeles Valley College Academic Senate.
She began teaching national licensure examination reviews
in the mid-1980s and has remained active in that area ever Janis Ryan Wisherop, RN, MSN, has been a nurse for
since. She is a contributing author to two editions 25 years. Ms. Wisherop has been a professor with the RN
of Daviss NCLEX-RN Success book and to ATIs program at the College of San Mateo for 18 years and has
NURSENOTES: Pediatrics, Core Content At-A-Glance. She held the title of Content Expert for the Psychiatric Nursing
also served as a reviewer for the 1996 Little, Browns course for that length of time. She teaches in the
Nursing Q&A Critical-Thinking Exercises book. Kathy is Medical/Surgical, Advance Medical/Surgical, Leadership/
currently a professor of nursing at Los Angeles Valley Management, and Pediatric courses as well. Ms. Wisherop
College, Valley Glen, California, and is the lead teacher has been the Faculty Advisor for the Nursing Student
and content expert for pediatric nursing. She is an active Association and is Assistant Director for the Nursing
member in the American and California Nurses Program. Along with teaching, Ms. Wisherop is dedicated
Associations, in national and local chapters of the to serving the community and inspiring her students to do
Association of Pediatric Nurses and Practitioners, and in the same. She has created a blood pressure clinic for seniors,
the Los Angeles County School Nurses Association. She partnered with the African American Community Advisory
has served as a District Academic Senator for the Los Committee in promoting better health through blood
Angeles Community College District and as an Academic pressure screenings and other health-related events.
Senator at Los Angeles Valley College for over 15 years. In Ms. Wisherop serves with passion as Board Secretary for
addition, she serves on numerous departmental and cam- Heart and Soul, a consumer operated mental health agency
pus committees. In 1999 she received the One of Our in San Mateo County. In addition, Ms. Wisherop is a per
Own award from the National Association of Pediatric diem nurse with Crossroads Home Care and Hospice, serv-
Nurse Associates and Practitioners, Los Angeles Chapter, ing patients and their families since 2009. This is her first
for service and excellence in pediatric nursing, and in 2007, time working as a co-author/co-editor.
2164_FM_i-xxviii 06/04/12 5:32 PM Page xi

PREFACE

W H AT S N E W A N D D I F F E R E N T I N T H E 3 R D P R O V E N R E S U LT S
EDITION This is the only review book for NCLEX-RN that
has evolved from and been tested with over 300,000
Questions are coded to correspond to the latest NCLEX-RN candidates who have taken our national
NCLEX-RN TEST PLAN! nursing exam review courses over the last 40 years and
Compact disk containing 3500 questions and passed! The review course content, the framework, the
answers, including all the questions in the book sequence of topics, the test-taking guidelines, and the
and 1000 new questions practice exam questions and answers have been published
Many more questions in ALTERNATE-ITEM in this book. This gives the book an authenticity and
FORMATS relevance that is difficult to attain any other way.
TWO NEW PRE-TESTS with 75 questions each
(150 total), each with rationales and unique test- CONCEPTUAL FRAMEWORK
taking tips The conceptual framework of this book concentrates on
Content totally reorganized under the 13 body body systems, as well as on nursing concerns for client needs
systems (e.g., cardiac, GI, GU, etc.). and the essential requirements for safe, effective, competent
TWO NEW FINAL TESTS with 75 questions nursing care. The text emphasizes practical application of
each (150 total) to aid in identifying topics that clinically relevant data. Separate chapters emphasize the
need extra study content areas of nutrition; pharmacology; common diag-
A new comprehensive index to help quickly locate nostic procedures, treatments, and nursing care; and
specific NCLEX-RN content management of care, including ethical and legal aspects
in nursing. A separate chapter features the geriatric client.
We are very pleased that you have selected our book to
help you on your path to success on NCLEX-RN. S P E C I A L F E AT U R E S
Our aim is to lead you all the way, to have PASSING There are many special features in this abundantly illustrat-
NCLEX-RN MADE EASY for youthe first-time test- ed book (over 205 charts, tables, and illustrations) that
taker, the international nurse, and the repeat test-taker. make it stand out from all other nursing review books.
With a conceptual framework based on body systems as The unique features that help you prepare for exams are:
well as on the components of the latest NCLEX-RN Test test-taking tips for each of the questions, lists of common
Plan (four client needs and their six subcategories), this book abbreviations and clinical signs (Appendices C and D),
differs significantly from other nursing review books. It is and a comprehensive index to guide you to where in the
the only nursing review book that thoroughly covers the book you will find the theory (topics) that specifically
content in the current NCLEX-RN Test Plan and has a relate to the official NCLEX-RN Test Plan. These
test-taking tip for each of the more than 3500 questions! features provide quick, easy, and interesting ways to review.
Each chapter incorporates all the components of the In the Introduction to NCLEX-RN section of Chapter 1,
NCLEX-RN Test Plan. In addition, Chapter 3 specifical- youll also find 60 study tips, anxiety-reducing tech-
ly includes content that focuses on Safe, Effective Care niques, and memorizing ideas.
Environment (with Management of Care and Safety and
Infection Control). Chapters 4 and 5 emphasize Health O T H E R F E AT U R E S
Promotion and Maintenance (growth and development, A practice computer test (CD-ROM) for self-
prevention and early detection of disease). Chapters 6, 7, 8, assessment and familiarization with the NCLEX-RN
and 9 emphasize Physiological Integrity (with Chapter 6 computer testing format includes test-taking tips
specifically addressing Basic Care and Comfort and with each answer.
Physiological Adaptation). Chapter 8 is devoted to Many easy-to-find and easy-to-use tables summa-
Pharmacological and Parenteral Therapies; Chapter 9 is rize information for quick review and emphasize
on Nutrition and Diet Therapy. Chapter 10 focuses specif- nursing responsibilities in a visual way.
ically on Psychosocial Integrity (coping and adaptation), Boxed lab data, diagnostic tests, bold-faced
and Chapter 11 specifically emphasizes Reduction of Risk hazard alerts and emergencies visually stand out.
Potential. Drug information is indicated by a , diets are
In addition, Chapter 2 is a unique chapter that address- indicated by a , client positioning is indicated
es the needs of international nurses and repeat test-takers;
and Chapter 7 addresses the special needs of the geriatric by a to help you focus on important concepts,
client. and memory tips are indicated by a .

xi
2164_FM_i-xxviii 06/04/12 5:32 PM Page xii

xii Preface

Easy-to-find content divisions are marked with HIPAA law, cultural diversity, religious and spiritual influ-
blue page tabs at the outer margin of each page. ences on health, advance directives, advocacy, organ dona-
Emphasis is given to all five steps of the nursing tion, bioethics and client rights, supervision, disaster plan-
process, especially client teaching and specific out- ning, and client restraints. Infection control content
come criteria for evaluation of the effectiveness of includes isolation precautions, with tables covering Types of
nursing care. Precautions and Illnesses Requiring the Precautions, and
Current NANDA-approved nursing diagnoses are Infection Control Conditions that Need Additional
provided as a structure for presenting nursing Precautions.
interventions. Chapter 4, Health Promotion and Maintenance:
A thorough question-and-answer review at the Nursing Care of the Childbearing Family, is streamlined,
end of each chapter, with a critical thinking focus visually enhanced, and up-to-date; it contains the most
that also reflects diverse cultural influences. exam-relevant content for both pregnancy and care of the
neonate. This chapter includes detailed information regard-
This review book has been updated and revised to
ing many important diagnostic tests, such as chorionic villi
incorporate the latest knowledge and current trends in
sampling and biophysical profile testing, to identify the
nursing practice and to parallel the latest NCLEX-RN
woman and fetus at risk. Numerous illustrations are
Test Plan. All content has been submitted to outstanding
included (e.g., amniocentesis, fetal circulation, and ectopic
educators and nursing practitioners for their review and
pregnancy). We include the latest information on emer-
critique. We would like to express our appreciation to this
gency care in labor and birth, perinatal acquired immune
editorial review panel for their contributions, which make
deficiency syndrome, sexually transmitted infections, and
this book the best to use for complete nursing review.
preterm labor. We have updated pharmacological informa-
The books content includes special topics not com- tion regarding new drug therapies during the perinatal
monly covered: period recommended for the at-risk infant, as well as trends
and modes for delivery of contraceptives and birth control
Management of care section including: delegation,
education. Updated information is included on intra-
establishing priorities, HIPAA rules, emergency
partum complications and risk factors, with content on hep-
response plan (fire safety and preparedness).
atitis and TB in pregnancy. Client teaching and nursing inter-
Infection control: animal-borne diseases (SARS,
ventions are delineated for each of these areas of practice.
West Nile virus, mad cow disease, monkeypox,
Chapter 5, Health Promotion and Maintenance:
Lyme disease, rabies, salmonella).
Nursing Care of the Pediatric Client, has a new table
Bioterrorism agents and associated syndromes
with updated immunization information, a pediatric safe-
(anthrax, botulism, plague, smallpox, tularemia,
ty and injury prevention chart, coverage of pediatric emer-
Ebola, Lassa, Hantavirus)
gencies (revised tables for CPR and foreign body airway
Herbal medicines: dietary supplements and herbal
obstructive managementpreviously called Heimlich
products used for psychiatric conditions; herbs and
maneuver), and updated information (nursing care, med-
potential dangers.
ications, tests, etc.) concerning all major physical sys-
Complementary and alternative therapies
tems of the body. Much of this information has been
Chapter 2, Guidelines and Tips for International synthesized into table format for easier reading, recall, and
Nurses and Repeat Test-Takers, is unique. No other book application to client care. Diagrams show genetic trans-
addresses the special concerns of candidates who have mission of sickle cell anemia and hemophilia.
taken the test more than once or who are graduates of Illustrations depict five cardiac defects, airway, CPR, TEF
nursing programs outside the United States. Chapter 2 and esophageal atresia, developmental dysplasia of the hip,
covers: and scoliosis. Coverage is provided for Reye syndrome,
Kawasaki disease, and salicylate, acetaminophen, and lead
Risk factors for not passing the first time poisoning, as well as appreciation of cultural diversity in
Five steps on what to do to pass next time assessment and nursing interventions.
How to know if you are prepared to take the Chapter 6, Physiological Integrity: Nursing Care
exam again of the Adult Client, has been arranged by head-to-toe
Sixty test-taking tips and guidelines to use in body systems, making it easier to remember important
answering questions assessment information and to prioritize the plan of care.
Chapter 3 has been reorganized, with special focus on Risk factors have been identified for all conditions, goals
Safe and Effective Care Environment: Management of of nursing are clearly stated, and a brief description of
Care, Cultural Diversity, Ethical and Legal Aspects of pathophysiology is incorporated with each condition.
Nursing, Nursing Trends, and Safety and Infection Nursing interventions are grouped according to the goal
Control. Content includes: safe use of equipment, delega- of care and identify appropriate treatments and drug ther-
tion, establishing priorities, emergency response plan, apies. Handy tables include: Factors Affecting Vital
2164_FM_i-xxviii 06/04/12 5:32 PM Page xiii

Preface xiii
Signs, SIADH Versus Diabetes Insipidus, Bioterrorism, Chapter 10, Psychosocial Integrity: Behavioral/
Infectious Diseases (including animal-borne), fluid and Mental Health Care Throughout the Life Span, presents
electrolyte imbalances, acid-base disorders, assessment psychiatric disorders that have been organized under
differences with valvular defects, hazards of immobility, DSM-IV-TR guidelines. We have included thorough cov-
complications of diabetes, and malignant disorders. The erage of suicide precautions, midlife crises, mental health
condensed and consolidated content is supplemented by of the elderly, substance abuse, post-traumatic stress disor-
many other tables, charts, and diagrams, such as: cardiac der, amnestic disorders, personality disorders, sleep and
dysrhythmias (new content on atrial flutter), comparison eating disturbances, and affective disorders. Content
of causes of chest pain, burn care, the Glasgow Coma includes a detailed section on psychiatric emergencies and
Scale, chest drainage, comparison of hepatitis types, post- therapeutic communication techniques. Also, three devel-
operative complications, enteric and standard precau- opmental sections are included: a table on development of
tions, respiratory isolation, care of the adult client with body image (includes disturbances and four phases of
medical and surgical emergencies, preventing TPN compli- crises), human sexuality throughout the life cycle (with
cations, and TPN dressing changes. Illustrations show causes of dysfunction and sexual health counseling), and a
12 types of fractures, 8 skin and skeletal tractions, and table on emotional disturbances in children, with sections on
breast and testicular self-exam. Special content covers autistic spectrum disorder, attention deficit-hyperactivity
management of PICC lines, use of metered-dose inhalers, disorder, and child abuse. Nine important sections
pneumonia severity, pheochromocytoma, Raynauds dis- are: domestic violence (abused woman and child; elder
ease, SARS, and Guillain-Barr syndrome. The up-to- abuse/neglect), diagnostic tests (for dementia/cognitive
date content also includes acquired immunodeficiency disorders and alcohol abuse); mental status assessment
syndrome, GERD, latex allergies, JCAHO Requirements (individual, cultural); interviewing; general principles of
for Pain Assessment, Lyme disease, compartment syn- health teaching; self-concept; coping mechanisms; and
drome, Crohns disease, ulcers, external fixation devices treatment modes, with a section on complementary
for fractures, and lithotripsy. and alternative methods. There is also an extensive
Chapter 7, Physiological Integrity: Nursing Care of glossary.
the Geriatric Client, is a concise and highly readable chap- Chapter 11, Reduction of Risk Potential: Common
ter synthesizing 12 key problems associated with care of the Procedures: Diagnostic Tests, Oxygen Therapy, and
older adult. There is a section on assessment of the older adult Hands-on Nursing Care, includes content on commonly
that includes material on normal changes of aging, includ- used tubes, oxygen delivery equipment, and colostomy
ing a table summarizing common concerns in the older care, as well as many diagnostic procedures commonly
adult. Five functional rating scales and a functional screening tested on NCLEX-RN. Also included are diagnostic tests
exam are included to make management of care easier: to evaluate fetal well-being, newborn screening procedures
(with content on Tay-Sachs screening), and bladder and
Social resources rating scale
bowel training. Common client positions and intubation
Economic resources rating scale
and ventilation are illustrated.
Mental health rating scale
Physical health rating scale
F I V E U N I Q U E S E L F - E VA L U AT I O N
Performance rating scale for activities of daily living
TOOLSOVER 3500 TEST QUESTIONS
Chapter 8, Physiological Integrity: Pharmacological A N D T E S T- TA K I N G T I P S
and Parenteral Therapies, contains drug classifications This book contains four special, integrated tests to help you
and drug treatments with emphasis on nursing implica- assess your knowledge before taking the NCLEX-RN.
tions. Content covers drug conversions and calculations, The special two new 75-item Pre-Tests at the end of
pediatric medication administration, fluid and electrolyte Chapter 1 are intended for students to take after reading
therapy, commonly used IV fluids, and obstetric analge- the Introduction to NCLEX-RN section but before
sia. Psychotropic and mind-altering substances are reading the rest of this book. It is a pre-assessment tool to
included, as well as tables on herbal therapies (with let you find your areas of strength and weakness based on
potential dangers). New drugs have been added for this the 10 client needs/subneeds of the NCLEX-RN Test
edition. Plan before beginning focused study. The two integrated
Chapter 9, Physiological Integrity: Basic Care and Final Tests in Chapter 12 are intended to be taken after
ComfortNutrition, contains unique information reviewing each chapter in this book to evaluate your
regarding ethnic food patterns, eating problems, nutrition- progress. These two new 75-item Final Tests, the third and
al needs of the elderly, religious and ethnic food preferences, fourth integrated exam tools, will assess your readiness to
and cultural disease treatments involving food. Special and take NCLEX-RN and identify any last-minute knowledge
therapeutic diets are featured, with coverage of medical gaps.
conditions with dietary management, high-phosphorus and Use the fifth self-evaluation exam tool as a practice
lactose-free diets, and anticancer nutrients. Computer Test on the CD-ROM anytime in your
2164_FM_i-xxviii 06/04/12 5:32 PM Page xiv

xiv Preface

preparation when you need extra practice or when you In the answers for each of the end-of-chapter review
want to become familiar with the NCLEX-RN testing question sections, as well as for the Pre-Tests, Final
format. Tests, and Computer Test, you will find a four-part code
All questions at the end of chapters and in the integrated to help you understand exactly what is being tested by
tests have been field-tested for several years with students each question. The code refers to Integrated Process (e.g.,
a diverse group of candidates who have successfully passed the step of the nursing process), the cognitive level (level of
the examfrom all over the United States. These questions difficulty of the item), the clinical/subject area, and the
have also been reviewed by an editorial panel for appropri- client need and the subcategory of client need that
ateness. The Answers/Rationales/Tips sections in each applies to that question. The Computer Test will track
chapter contain detailed explanations about why a particu- these codes for you to identify your problem areas. Use
lar answer is best and why the other options are incorrect, these codes as a guide for review when you find you did
with a test-taking tip for each question. We expect that not select the best answer. The test-taking tips and
these special integrated tests, end-of-chapter review ques- codes are unique to this book; they are an added study
tions, and corresponding answer sections will prove to be tool to help you assess your strengths and pinpoint prob-
invaluable review tools for each and every student. lem areas as you prepare for the NCLEX-RN.
2164_FM_i-xxviii 06/04/12 5:32 PM Page xv

CONTRIBUTORS

Reviewer and Contributing Author to Special Sections in Chapter 6

Estrella Evangelista-Hoffman, RN, CNL, MEd, DNP


Las Vegas, Nevada

Test Item Writers for This Edition

Cheryl Osborne, RN, EdD


Professor, School of Nursing and Director, Gerontology Program
California State University, Sacramento, California

Connie Overman, RN, MS


Former Clinical Faculty, School of Nursing
California State University, Sacramento, California

Carolyn Van Couwenberghe, RN, PhD


Professor, School of Nursing
California State University, Sacramento, California

xv
2164_FM_i-xxviii 06/04/12 5:32 PM Page xvi
2164_FM_i-xxviii 06/04/12 5:32 PM Page xvii

ACKNOWLEDGMENTS

BONNIE BERGSTROM YOU are the superstar on my winning team of authors in


this 3rd edition. You stepped into the picture at midstream and added your editing
expertise and gave of your very special kind of nurturing when we were all under time
pressure with manuscript deadlines. You are one-of-a-kind and Im deeply grateful!

BERTA STEINER and the Bermedica Production, Ltd. staff (including Megan
Westerfeld, the copyeditor)You were invaluable in helping me maintain a vigilant
eye for quality controlfinding gaps and inconsistencies, and giving shape and form
to a complicated manuscript during stops and starts in the editorial process. Above all,
you were there for me and my co-authorswe worked as a team.

BOB BUTLERYou were there for me as a conduit in the F.A. Davis communica-
tion chain. Your support served as a beacon for decisions in putting my ideas into
print.

BOB MARTONEYou contributed important ideas to make this 3rd edition the
best yet for visual emphasis, with unique features that are designed to help the user
be successful in nursing school exams and NCLEX-RN.

A special personal thank you also to NEIL KELLY and ROB CRAVEN, whose
support for this book over the last eight years and three editions made it a happy
happening. Contacts along the way with each of you went beyond the ordinary
and are much appreciated.
Sally

xvii
2164_FM_i-xxviii 06/04/12 5:32 PM Page xviii
2164_FM_i-xxviii 06/04/12 5:32 PM Page xix

LIST OF TABLES

CHAPTER 1 4.16 Estimation of Gestational Age: Common Clinical


1.1 Description of NCLEX Question Formats 6 Parameters 204
1.2 Examination Weight Given to Each Category of Health 4.17 Infant Stool Characteristics 206
Needs of Clients Based on Current Practice
Analysis Study 8 CHAPTER 5
5.1 Stages of Spiritual Development in Childhood 251
CHAPTER 3 5.2 Child Preventive Care Timeline 252
3.1 Cultural Influences on Health-Care Practices 5.3 Recommended Health Screenings for Infants,
with Children and Adults 74 Children, and Adolescents: Specific Conditions 253
3.2 Guidelines for Isolation Precautions 88 5.4 Pediatric Behavioral Concerns: Nursing Implications
3.3 Contact PrecautionsSummary 90 and Parental Guidance 253
3.4 Standard PrecautionsSummary 91 5.5 Pediatric Sleep and Rest Norms: Nursing Implications
3.5 Airborne and Droplet PrecautionsSummary 91 and Parental Guidance 254

3.6 Infection Control: Conditions That Need Additional 5.6 Normal Vital Signs: Measurements and Variations with Age 255
Precautions 91 5.7 Facts About the DENVER II Developmental Assessment 256
3.7 Infection Control When Caring for Infants, Children, 5.8 Nursing Care of Hospitalized Infants and Children: Key
and Adolescents 92 Developmental Differences 265
3.8 Summary: Types of Precautions and Illnesses 5.9 Pediatric Respiratory Infections 270
Requiring the Precautions 93 5.10 Comparison of Common Oxygen Delivery Systems 271
3.9 Safety Considerations When Caring for Hospitalized 5.11 Guidelines for Home Care of Infant on Apnea Monitor 276
Infants, Children, and Adolescents 94 5.12 SIDS: What to Tell Families 277
3.10 Safety and Injury PreventionHome Safety 94 5.13 Comparison of Acyanotic and Cyanotic Heart Disease 279
5.14 Overview of the Most Common Types of Congenital
CHAPTER 4
Heart Disease 279
4.1 Contraception 119
5.15 Comparison of Cleft Lip and Cleft Palate 284
4.2 Sterilization 121
5.16 Signs and Symptoms of Dehydration in Infants
4.3 Interruption of Pregnancy (Elective/Voluntary Abortion) 123
and Young Children 288
4.4 Hormones of Pregnancy 127
5.17 Comparison of Nephrosis and Acute Poststreptococcal
4.5 Blood Values 129
Glomerulonephritis 292
4.6 Common Discomforts During Pregnancy 129
5.18 Differences Between Type 1 and Type 2 Diabetes Mellitus 294
4.7 Behavioral Changes in Pregnancy 133
5.19 Communicable Diseases of Childhood 300
4.8 Emergency Conditions 149
5.20 Common Pediatric Orthopedic Conditions 307
4.9 Comparison of Placenta Previa and Abruptio Placenta 159
5.21 Comparison of Two Major Types of Spina Bifida 311
4.10 First Stage of Labor 163
5.22 Pediatric Surgeries: Nursing Considerations 316
4.11 Assessment: Differentiation of False/True Labor 165
4.12 Apgar Score 174 CHAPTER 6
4.13 Postpartum Infections 195 6.1 Factors Affecting Vital Signs 349
4.14 Physical Assessment of the Term Neonate 201 6.2 Preventive Care Timelines 352
4.15 Assessment: Normal Newborn Reflexes 204 6.3 Recommended Adult Immunization Schedule 353

xix
2164_FM_i-xxviii 06/04/12 5:32 PM Page xx

xx List of Tables

6.4 Diabetes Insipidus (DI) Versus Syndrome of Inappropriate 6.40 Papanicolaou (Pap) Smear Classes 535
Antidiuretic Hormone (SIADH) 358 6.41 Uterine Cancer: Recommended Treatment, by Stage
6.5 Electrolyte Imbalances 360 of Invasion 535
6.6 Blood Gas Variations with Acid-Base Imbalances 363 6.42 International System of Staging for Cervical Carcinoma 535
6.7 Blood Gas Abnormalities: Causes 363 6.43 Selected Cancer Problems 538
6.8 Acid-Base Imbalances 364 6.44 Nursing Care of the Adult in Medical and Surgical
6.9 Postoperative Complications 370 Emergencies 542

6.10 Required Pain Assessment on Admission for Clients


Who are Hospitalized 376 CHAPTER 7
7.1 Medications That Should Not Be Used by Geriatric Clients 584
6.11 Format for Assessing Pain 377
7.2 Most Frequent Issues in the Care of the Geriatric Client 587
6.12 Levels of Consciousness 378
7.3 Functional Rating Scale for the Geriatric Client 587
6.13 Glasgow Coma Scale 379
7.4 Functional Screening Examination 588
6.14 Differentiating Between Causes of Dyspnea 400
6.15 Comparison of Pneumothorax and Hemothorax 407
CHAPTER 8
6.16 Tracheostomy Suctioning Procedure 409 8.1 Commonly Used Intravenous Fluids 607
6.17 Imbalances in Blood Pressure: Comparative Assessment 8.2 Transfusion with Blood or Blood Products 609
of Hypotension and Hypertension 410
8.3 Complications of IV Therapy 611
6.18 Comparison of Selected Cardiac Dysrhythmias 412
8.4 Complications Associated with Total Parenteral Nutrition 613
6.19 Comparison of Physical Causes of Chest Pain 417
8.5 Antipsychotic AgentsComparison of Side Effects 615
6.20 Comparison of Symptomatology for Valvular Defects 420
8.6 Antianxiety Agents: Comparison 617
6.21 Left Ventricular Compared with Right Ventricular 8.7 Major Substances Used for Mind Alteration 619
Heart Failure 426
8.8 Rates of Absorption by Different Routes 623
6.22 Signs of Hypovolemic Shock 428
8.9 Regional Analgesia-Anesthesia for Labor and Birth 623
6.23 Nursing Responsibilities with Anticoagulant Therapy 434
8.10 Food and Fluid Considerations with Drugs 624
6.24 Etiology, Incidence, and Epidemiological and Clinical
8.11 Antineoplastic Drug Classifications 625
Comparison of Hepatitis A, Hepatitis B, Hepatitis C,
8.12 Common Side Effects of Chemotherapeutic Agents 626
and Delta Hepatitis 437
8.13 Properties of Selected Analgesic Agents 627
6.25 Comparison of Ileostomy and Colostomy 454
8.14 Equianalgesic Dosing for Opioid Analgesics 627
6.26 Colostomy Irrigation 456
8.15 Dietary Supplements and Herbal Products Used
6.27 Comparison of Hemodialysis and Peritoneal Dialysis 460
for Psychiatric Conditions 629
6.28 Comparison of Diabetic Complications 469
8.16 Herbs and Potential Dangers 630
6.29 Infectious Diseases: Animal-Borne 485
8.17 Common Medications 632
6.30 Recognizing Bioterrorism Agents and Associated
Syndromes 487
CHAPTER 9
6.31 Burn Characteristics According to Depth of Injury 489 9.1 Nutrient Needs During Pregnancy 690
6.32 Topical Antimicrobials Used in Burn Care 490 9.2 Eating Problems of the Geriatric Client and Ways
6.33 Wound Coverings 491 to Improve Nutrition 692
6.34 Complications of Immobilization 493 9.3 Cultural Food Patterns 692
6.35 Complications of Fractures 503 9.4 Hot-Cold Theory of Disease Treatment 694
6.36 Assessing Injured Limb: CSM 506 9.5 Physiological Functions of Common Vitamins
6.37 Teaching Crutch Walking 507 and Related Deficiencies 695
6.38 Measuring Crutches Correctly 507 9.6 Essential Nutrients and Potential Deficiencies 697
6.39 Fractures of the Hip 511 9.7 Medical Conditions with Dietary Management 698
2164_FM_i-xxviii 06/04/12 5:32 PM Page xxi

List of Tables xxi


CHAPTER 10 CHAPTER 11
10.1 Freuds Stages of Psychosexual Development 712 11.1 Diagnostic Tests to Evaluate Fetal Well-Being 832
10.2 Eriksons Stages of the Life Cycle 713 11.2 Newborn Screening Procedures 834
10.3 Summary of Theories of Psychosocial Development 11.3 Summary: Oxygen Delivery Equipment 836
Throughout the Life Cycle 713 11.4 Positioning the Client for Specific Surgical Conditions 837
10.4 Piagets Age-Specific Development Levels 716 11.5 Review of the Use of Common Tubes 839
10.5 Four Phases of Body Image Crisis 718 11.6 Emptying Colostomy Appliance 843
10.6 Body Image Development and Disturbance Throughout 11.7 Changing Colostomy Appliance 843
the Life Cycle: Assessment 718 11.8 Care of Dentures 844
10.7 Sexual Behavior Throughout the Life Cycle 723 11.9 Caring for an Artificial Eye 844
10.8 Analysis/Nursing Diagnosis: Altered Feeling States 11.10 Caring for a Hearing Aid 844
Related to Grief 731
11.11 Bladder Training 844
10.9 Individual Assessment 733
11.12 Bowel Training 844
10.10 Emotional Disturbances in Children 748
10.11 Comparison: Anorexia and Bulimia 761
10.12 Comparison of the Two Different Types of Depressive
Disorders 786
10.13 Behaviors Associated with Mania and Depression 789
10.14 Estimation of Time at Which Soft Tissue Injury
Occurred 799
10.15 Curative Factors of Group Therapy 803
10.16 Differences Between Reality Orientation
and Resocialization 804
2164_FM_i-xxviii 06/04/12 5:32 PM Page xxii

LIST OF FIGURES

CHAPTER 1 CHAPTER 6

1.1 Sample appearance of NCLEX-RN question on computer 6.1 Cranial nerves and their distributions 351
screen 7 6.2 Closed drainage system for constant suction 385

CHAPTER 3
6.3 Pneumothorax 405

3.1 Typical procedure for involuntary commitment 85 6.4 Interpretation of normal cardiac cycle 412
6.5 Angina pectoris 418
CHAPTER 4 6.6 Arterial balloon angioplasty 423
4.1 The female pelvis 116 6.7 Bypass 423
4.2 Female internal reproductive organs 117 6.8 Hiatal hernia 442
4.3 The menstrual cycle 117 6.9 Abdominal pain by location 448
4.4 Basal body temperature chart 123 6.10 Colostomy sites 455
4.5 Amniocentesis 147 6.11 AV shunt (cannulae) 461
4.6 Ectopic pregnancy 152 6.12 AV fistula 461
4.7 The fetal head 165 6.13 Types of fractures and terminology 499
4.8 Categories of fetal presentation 166 6.14 Types of skin and skeletal traction 501
4.9 Cardinal movements in the mechanism of labor with 6.15 Rheumatoid arthritis 509
the fetus in vertex presentation 167 6.16 Breast self-examination 534
4.10 Fetal heart rate (FHR) decelerations and nursing 6.17 Testicular self-examination 542
interventions 168
4.11 Fetal circulation 200 CHAPTER 9
9.1 MyPlateguide to daily food choices 704

CHAPTER 5 CHAPTER 10
5.1 CPR recommendations for the healthcare provider 275 10.1 Maslows hierarchy of needs 715
5.2 Foreign body airway obstruction management 276 10.2 Operationalization of the behavioral concept of
5.3 Atrial septal defect 277 masturbation 727
5.4 Ventricular septal defect 278 10.3 Grief versus depression 732
5.5 Patent ductus arteriosus 278 10.4 Operationalization of the behavioral concept of
5.6 Tetralogy of Fallot 278 manipulation 746
5.7 Transposition of the great arteries 278 10.5 Symptoms associated with alcohol withdrawal 767
5.8 Sequelae of strep infections 281 10.6 Adaptation responses to anxiety on a continuum 771
5.9 Esophageal malformations 286 10.7 Levels of depression 787
5.10 Genetic transmission of sickle cell anemia 296 10.8 Victim decisions following a sexual assault 798
5.11 Genetic transmission of hemophilia 298
CHAPTER 11
5.12 Signs of developmental dysplasia of the hip 309 11.1 Common client positions 838
5.13 The four major curve patterns in idiopathic scoliosis 310 11.2 Chest drainage system 843
5.14 Pathophysiological effects of lead poisoning 315

xxii
2164_FM_i-xxviii 06/04/12 5:32 PM Page xxiii

TABLE OF CONTENTS

Dedications v Pre-Test 2: Questions (75) 36


About the Editor/Authors ix Pre-Test 2: Answers/Rationales/Tips 43
Preface xi
Contributors xv Chapter 2: Guidelines and Tips for
Acknowledgments xvii International Nurses and
List of Tables xix Repeat Test-Takers 59
List of Figures xxii Sally Lambert Lagerquist

Tips for NCLEX-RN Candidates Who Must Repeat


Chapter 1: Orientation and Pre-Tests 1 the Examination 60
Sally Lambert Lagerquist Janice Lloyd McMillin
Purpose of This Section 60
Robyn Marchal Nelson Denise Wall Parilo
Kathleen E. Snider Janis Ryan Wisherop What is the Difference Between Taking This
Examination for the First Time and Repeating It? 60
Orientation 2
What are Some Risk Factors for Not Passing the
How to Use This Review Book as a Study Guide 2 First Time? 60
As a Starting Point 2 It is Not a Matter of How Much You Study for the
As an Endpoint 2 Test, but How You Use the Review Material 60
As an Anxiety-Reduction Tool 2 How Do You Know What You Need to Review? 60
As a General Study Guide and as a Refresher for What to Do to Pass Next Time 61
Nurses Not Presently in Practice 2 How Will You Know That You are Well-Prepared
Where to Begin 2 and Ready to Retake the Examination? 61
While Reviewing 3 A Guide for Graduates of Nursing Schools
After Reviewing 3 Outside the United States 61
Key Points to Recall for Better Study 3 Test-Taking Tips and Guidelines: Sixty
Memorization: Purpose and Strategies 4 Strategies to Use in Answering Questions 62
The Mechanics of the National Council Confidence, Performance, Pass! 67
Licensure Examination for Registered
NursesComputer Adaptive Test (NCLEX-RN) 4 Chapter 3: Safe, Effective Care Environment:
How to Prepare for and Score Higher on Management of Care, Cultural Diversity,
Examinations 11 Ethical and Legal Aspects of Nursing,
The Psychology of Test-Taking 11
Nursing Trends, and Safety and
Infection Control 69
Prepare Physically and Mentally 11 Sally Lambert Lagerquist Janice Lloyd McMillin
Tips for NCLEX-RN 12 Robyn Marchal Nelson Denise Wall Parilo
Tips for Other (Pencil-and-Paper) Tests and Examinations 12 Kathleen E. Snider

How to Reduce Anxiety 13 Management of Care 70


Progressive Relaxation 13 Delegation 70
Instructions 13 Priorities 70
Cultural Diversity in Nursing Practice 72
Suggestions for Additional Experiential Vignettes 14
Religious and Spiritual Influences on Health 76
Pre-Test 1 and 2: Introduction to Special Questions
Nursing Ethics 76
Based on Official NCLEX-RN Detailed Test Plan 14
Legal Aspects of Nursing 78
Pre-Test 1: Questions (75) 14
Pre-Test 1: Answers/Rationales/Tips 21

xxiii
2164_FM_i-xxviii 06/04/12 5:32 PM Page xxiv

xxiv Table of Contents

Questions Most Frequently Asked by General Aspects: Nursing Care of the


Nurses About Nursing and the Law 80 High-Risk Infant and Family 217
Ethical and Legal Considerations in Questions (100) 218
Intensive Care of the Acutely Ill Neonate 84 Answers/Rationales/Tips 228
Legal Aspects of Psychiatric Care 85
Legal Aspects of Preparing a Client for Surgery 86 Chapter 5: Health Promotion and
Maintenance: Nursing Care
Trends in Nursing Practice 86
of the Pediatric Client 249
Nursing Organizations 88 Kathleen E. Snider Denise Wall Parilo
Safety and Infection Control: Guidelines for Isolation
and Standard Precautions 88 Management of Care of Infants, Children,
Questions (75) 95 and Adolescents 250
Answers/Rationales/Tips 101 Safety and Infection Control for Infants, Children,
and Adolescents 250
Chapter 4: Health Promotion and Prevention and Early Detection of Disease in
Maintenance: Nursing Care Infants, Children, and Adolescents 250
of the Childbearing Family 115 Coping and Adaptation in Infants, Children,
Janice Lloyd McMillin and Adolescents 250
Growth and Development 116 Growth and Development 251

Biological Foundations of Reproduction 116 Developmental Disabilities 262


Psychosocial-Cultural Functions 264
Female Reproductive Anatomy and Physiology 116
Neurological System 265
Decision Making Regarding Reproduction 118
Respiratory System 268
Childbearing: Pregnancy by Trimester 125 Cardiovascular System 277
Prenatal Management 134 Gastrointestinal System 284

Antepartum 136 Structural Defects 284


Lifestyle Choices and Influences That Obstructive Disorders 287
Impact Health in Pregnancy, Intrapartum, Disorders of Motility 290
Postpartum, and Newborn 145
Genitourinary System 290
Common Complications of Pregnancy 148
Genitourinary Tract Disorder 290
First-Trimester Complications 148
Kidney Tumor 291
Second-Trimester Complications 157
Glomerular Diseases 292
Third-Trimester Complications 159
Endocrine System 294
The Intrapartum Experience 162
Hematological System 294
Complications During the Intrapartum Immunological System 299
Period 180
Streptococcus Infections/Sequelae 299
The Postpartum Period 188
Integumentary System 299
Complications During the Postpartum Musculoskeletal and Neuromuscular Systems 307
Period 193 Accidents: Ingestions and Poisonings 312
The Newborn Infant 200 Pediatric Surgeries: Nursing Considerations 315
Complications During the Neonatal Period: Selected Pediatric Emergencies 316
The High-Risk Newborn 207 Questions (100) 317
Congenital Disorders 214 Answers/Rationales/Tips 326
Emotional Support of the High-Risk Infant 217
2164_FM_i-xxviii 06/04/12 5:32 PM Page xxv

Table of Contents xxv


Chapter 6: Physiological Integrity: Nursing Chapter 8: Physiological Integrity:
Care of the Adult Client 347 Pharmacological and
Robyn Marchal Nelson Parenteral Therapies 603
Sally Lambert Lagerquist Janice Lloyd McMillin
Assessment of the Adult Client 348
Robyn Marchal Nelson Kathleen E. Snider
Subjective Data 348
Conversions and Calculations in Medication Administration 604
Nursing History 348
Medication Administration: Infants and Children 605
Objective Data 348
Intravenous Therapy/Blood Administration 606
Growth and Development 351 Psychotropic Medications 613
Young Adulthood (20 to 30 Years of Age) 351 Mind-Altering Substances 619
Adulthood (31 to 45 Years of Age) 355 Absorption Rates by Different Routes 623
Regional Analgesia-Anesthesia in Labor and Birth 623
Middle Life (46 to 64 Years of Age) 355
Food/FluidMedication Interactions 624
Early Late Years (65 to 79 Years of Age) 356
Antineoplastic Drug Classifications 625
Later Years (80 Years of Age and Older) 356 Common Side Effects of Chemotherapeutic Agents 626
Fluid and Electrolyte Imbalances 356 Properties of Selected Analgesic Agents 627
The Perioperative Experience 365 Equianalgesic Dosing for Opioid Analgesics 627
Pain Management 375 Dietary Supplements and Herbs 629
Neurological System 378 Common Medications 632
Eyes, Ears, Nose, and Throat 384 Questions (93) 664
Respiratory System 392 Answers/Rationales/Tips 671
Cardiovascular System 410
Gastrointestinal System 435 Chapter 9: Physiological Integrity: Basic
Genitourinary System 457 Care and ComfortNutrition 689
Endocrine System 467 Sally Lambert Lagerquist Janice Lloyd McMillin
Hematological System 478 Robyn Marchal Nelson Denise Wall Parilo
Immunological System 483 Kathleen E. Snider

Integumentary System 485 Population-Specific Nutritional Needs 690


Musculoskeletal System 492
Nutrition During Pregnancy and Lactation 690
Neuromuscular System 514
Oncological Disorders 524 Nutritional Needs of the Newborn 691
Emergency Nursing Procedures 537 Nutritional Needs of the Geriatric Client 691
Questions (105) 552 Cultural Food Patterns 691
Answers/Rationales/Tips 560 Religious Considerations in Meal Planning 691
Common Vitamins and Nutrients and Related Deficiencies 694
Chapter 7: Physiological Integrity: Medical Conditions with Dietary Management 694
Nursing Care of the Special Diets 694
Geriatric Client 579 Common Therapeutic Diets 699
Robyn Marchal Nelson Anticancer Nutrients and Nonnutritive Compounds 701
Food List for Menu Planning 703
Introduction 580
Questions (25) 704
Health Assessment of the Geriatric Client 580
Answers/Rationales/Tips 706
General Nursing Diagnoses in the Geriatric Client 582
Problems Associated with the Geriatric Client 582
Questions (50) 590
Answers/Rationales/Tips 594
2164_FM_i-xxviii 06/04/12 5:32 PM Page xxvi

xxvi Table of Contents

Chapter 10: Psychosocial Integrity: Behavioral/Mental Anxiety 770


Health Care Throughout the Life Span 711 Patterns of Adjustment (Defense Mechanisms) 772
Sally Lambert Lagerquist Janis Ryan Wisherop (Reviewer and
Anxiety Disorders (Anxiety and Phobic
Contributor to Selected Sections)
Neuroses) 774
Psychosocial Growth and Development 712 Anxiety Disorders 774
Major Theoretical Models 712 Dissociative Disorders (Hysterical Neuroses,
Body Image Development and Disturbances Throughout Dissociative Type) 776
the Life Cycle 716 Somatoform Disorders 777
Body Image DisturbanceSelected Somatization Disorder 777
Examples 720 Conversion Disorder (Hysterical Neuroses,
Body Image Disturbance Caused by Amputation 720 Conversion Type) 777
Body Image Disturbance in Brain Attack (Stroke) 721 Hypochondriasis (Hypochondriacal Neurosis) 778
Body Image Disturbance in Myocardial Infarction 721 Other Conditions in which Psychological
Body Image and Obesity 722 Factors Affect Medical Conditions
Human Sexuality Throughout the Life Cycle 723 (Psychophysiological Disorders) 778
Sexual-Health Counseling 724 Schizophrenia and Other Psychotic Disorders 779
General Issues 724 Delusional (Paranoid) Disorders 783
Specific Situations 726 Personality Disorders 784
Concept of Death Throughout the Life Cycle 728 Mood Disorders 786
End-of-Life: Death and Dying 729 Major Depressive Disorder 786
Grief/Bereavement 730 Bipolar Disorders 788
Mental Status Assessment 732 Psychiatric Emergencies 790
Interviewing 734 Crisis Intervention 795
General Principles of Health Teaching 734 Selected Specific Crisis Situations:
The Therapeutic Nursing Process 736 Problems Related to Abuse/Violence 796
Common Behavioral Problems 740 Treatment Modes 801
Mental and Emotional Disorders in Milieu Therapy 801
Children and Adolescents 747 Behavior Modification 801
Midlife Crisis: Phase-of-Life Problems 755 Activity Therapy 802
Mental Health Problems of the Geriatric Group Therapy 802
Client 756
Reality Orientation and Resocialization 804
Alterations in Self-Concept 758
Family Therapy 805
Sleep Disturbance 759
Electroconvulsive Therapy 807
Eating Disorders 760
Complementary and Alternative
Anorexia Nervosa/Bulimia Nervosa 760
Medicine (CAM) 807
Sensory Disturbance 762
Glossary 808
Delirium, Dementia, and Amnestic
and Other Cognitive Disorders 762 Questions (50) 810
Answers/Rationales/Tips 815
Substance-Related Disorders 765
Alcohol Use Disorders: Alcohol Abuse and Dependence 766
Other Substance-Related Disorders 769
2164_FM_i-xxviii 06/04/12 5:32 PM Page xxvii

Table of Contents xxvii


Chapter 11: Reduction of Risk Potential: Suggestions for Further Study 866
Common Procedures: Final Test 1: Questions (75) 866
Diagnostic Tests, Oxygen Final Test 1: Answers/Rationales/Tips 873
Therapy, and Hands-on Final Test 2: Questions (75) 887
Nursing Care 825 Final Test 2: Answers/Rationales/Tips 894
Janice Lloyd McMillin Robyn Marchal Nelson

Common Diagnostic Procedures 826


Appendices
Diagnostic Tests to Evaluate Fetal Well-Being 832
Appendix A: Laboratory Values 909
Newborn Screening Procedures 834
Appendix B: Memory Aids 915
Oxygen Therapy 835
Appendix C: Common Acronyms and
Hands-On Nursing Care 836
Abbreviations 919
Positioning the Client 836 Appendix D: Quick Guide to Common
Commonly Used Tubes 836 Clinical Signs 925
Colostomy Care 836 Appendix E: Complementary and
Basic Prosthetic Care 836 Alternative Therapies 927

Bladder Training 836 Appendix F: Index: Content Related to


NCLEX-RN Test Plan:
Bowel Training 844
Four General Client Needs
Questions (73) 845
and Six Client Subneeds
Answers/Rationales/Tips 851
(With Detailed Knowledge,
Skills, and Abilities) 933
Chapter 12: Final Tests 1 and 2 865
Sally Lambert Lagerquist Janice Lloyd McMillin
Appendix G: Index: Questions Related to
Robyn Marchal Nelson Denise Wall Parilo Client Needs/Subneeds 939
Kathleen E. Snider Janis Ryan Wisherop Appendix H: Reduction of Risk Potential:
Index to Diagnostic Tests
Introduction 866
and Procedures 943
What is Final Test 1? 866
What is Final Test 2? 866 Index 945
Selection of Content and Distribution of the Questions 866
Timing 866
Test Results 866
2164_FM_i-xxviii 06/04/12 5:32 PM Page xxviii
2164_Ch01_001-058 29/03/12 12:18 PM Page 1

CHAPTER 1

Orientation and
Pre-Tests
Sally Lambert Lagerquist Janice Lloyd McMillin Robyn Marchal Nelson
Denise Wall Parilo Kathleen E. Snider Janis Ryan Wisherop

1
2164_Ch01_001-058 29/03/12 12:18 PM Page 2

2 chapter 1 Orientation and Pre-Tests


ORIENTATION

O R I E N T AT I O N As a Starting Point
This text can be used in early review when a longer study
How to Use This Review Book period is needed to fill in gaps of knowledge. One cannot
as a Study Guide remember something if one does not know or understand
it. A lengthy review before the examination allows stu-
Although nursing students may know that they are dents time to rework and organize notes accumulated dur-
academically prepared to take the computer adaptive ing 2 to 4 years of basic nursing education. In addition, an
National Council Licensure Examination for Registered early review allows time for self-evaluation. We have pro-
Nurses (NCLEX-RN), many find that reviewing nurs- vided questions and answers to help students identify areas
ing content for the licensure examination itself presents requiring further study and to help them integrate unfa-
special concerns about what and how to study. miliar material with what they already know.
Some typical concerns about what to study are reflected
in the following questions: As an Endpoint
This text can also be used for a quick review (a) to promote
Considering that there will be up to 265 questions
retention and recall and (b) to aid in determining nursing
on the examination, and every candidate gets a
actions appropriate to specific health situations. During
different examination, how does one select what is
the time immediately preceding the examination, the
the most important content for review? How does
main objective might be to strengthen previous learning by
one narrow the focus of study and distinguish the
refreshing the memory. Or a brief overview may serve to
relevant from the irrelevant material?
draw together the isolated points under key concepts and
What areas should be emphasized?
principles in a way that shows their relationships and rela-
How detailed should the review be?
tive importance.
How does one know what areas to review first?
Should basic sciences, such as anatomy, physiology, As an Anxiety-Reduction Tool
microbiology, and nutrition, be included in the In some students, anxiety related to taking examinations in
study? general may reach such levels that it causes students to be
Concerns relating to how to study include: unproductive in study and to function at a lower level dur-
ing the actual examination. Sections of this chapter are
How does one make the best of limited review directed toward this problem and provide simple, practical
time to go over content that may be in lecture approaches to the reduction of general anxiety. For anxiety
and clinical notes compiled during 2 to 4 years of specifically related to unknown aspects of the licensure
schooling? examination itself, the section on the structure, format, and
Is it best to review from all the major textbooks mechanics of the RN examination might bring relief through
used in nursing school? its focus on basic examination information (see pp. 4-11).
Should material be memorized, or should one For anxiety related to lack of confidence or skill in test-
study from broad principles and concepts? taking know-how, the special section on test-taking tech-
We have written this nursing review book with the niques may be helpful (see Chapter 2).
general intent of assisting nurses in identifying what they
need to study in a format designed to use their study time As a General Study Guide
effectively, productively, and efficiently while preparing and as a Refresher for Nurses
for the examination. Not Presently in Practice
The contributing authors have selected content and
Many nursing students will find this review book useful
developed a style of presentation that has been tested
throughout their education as a general study guide as
by thousands of nursing students attending review
they prepare client care plans and study for midterm and
courses coordinated by the editor in various cities
final examinations. It will help them put information into
throughout the United States. This book is the result of
perspective as they learn it. And nurses who have not been
this study.
in practice for several years will find it a useful reference
This review book can be used in a variety of ways:
tool and review device.
(a) as a starting point for review of essential content
specifically aimed at NCLEX or Canadian examination Where to Begin
preparation, (b) as an endpoint of studying for the exam- In using this review book to prepare for the licensure
inations, (c) as an anxiety-reduction tool, (d) as a general examination, the nurse must:
guide and refresher for nurses not presently in practice,
and (e) as a guide for graduates of nursing schools outside 1. Be prepared mentally.
of the United States, as well as repeat test-takers (see a. Know the purpose of the examination.
Chapter 2). b. Know the purpose of reviewing.
2164_Ch01_001-058 29/03/12 12:18 PM Page 3

Key Points to Recall for Better Study 3

ORIENTATION
c. Anticipate what is to come. b. Make meaningful associations.
d. Decide on a good study methodset a study c. Look for implications for nursing actions as
goal before beginning a particular subject area concepts are reviewed.
(number of pages, for example); plan the 4. Take notes on the review outlineuse stars and
length of the review period by the amount of arrows, underscore, highlight with highlighter
material to be covered, not by the clock. pens, and write comments in margins, such as
2. Plan the work to be done. most important and memorize, to reinforce the
a. Select one subject at a time for review, and relative importance of points of information.
establish and follow a sequence for review of
each subject. After Reviewing
(1) Answer the practice questions following 1. Repeat the self-evaluation process as often as
the outline of the selected subject area. necessary to gain mastery of content essential to
(Set a time limit, because pacing is safe nursing practice.
important.) 2. Continue to refer to major textbooks to fill in gaps
(2) Compare your answers with those provid- where greater detail or in-depth comprehension is
ed following the questions as a means of required.
evaluating areas of competence. 3. Look for patterns in your selection of responses to
b. Identify those subjects that will require addi- the multiple-choice practice questionsidentify
tional concentrated study in this review book sources of difficulty in choosing the most appro-
as well as in basic textbooks. priate answers.
c. Study the review text outlines, noting head- 4. Review test-taking strategies (see Chapter 2,
ings, subheadings, and italics and boldface pp. 62-67).
type for emphasis of relative importance.
d. Study the content presented in the shaded
boxes and chart format to facilitate memoriza- Key Points to Recall
tion, understanding, and application.
e. Repeat the self-evaluation process by taking the for Better Study
test again. 1. Schedulestudy time should be scheduled so that
f. Look up the answers for the correct response to review begins close to the time at which it will be
the multiple-choice questions. Do not memo- used. Retention is much better following a well-
rize the answers. Read the rationale explaining spaced review. It may be helpful to group material
why it was the correct response. (These expla- into small learning segments. Study goals should
nations serve to correct as well as reinforce. be set before beginning each period of study
Understanding the underlying principles also (number of pages, for example).
serves as an aid in applying the same principles 2. Organizemany students have better retention
to questions that may be based on similar of material after they have reorganized and
rationales, but phrased differently on the actual relearned it.
examination.) 3. Rephrase and explaintry to rephrase material in
g. If necessary, refer to basic textbooks to relearn your own words or explain it to another person.
any unclear aspects of anatomy, physiology, Reinforce learning through repetition and usage.
nutrition, or basic nursing procedures. Look 4. Decide on order of importanceorganize study time
up unfamiliar terminology in a medical in terms of importance and familiarity.
dictionary. 5. Use mechanical memory aidsmnemonic (memory)
devices simplify recall. For example, in On Old
While Reviewing Olympuss Towering Top a Finn and German
1. Scan the outline for main ideas and topics. Viewed Some Hops, the first letter of each word
a. Do not try to remember verbatim what is on identifies the first letter of a cranial nerve (see
each page. Appendix B).
b. Paraphrase or explain this material to another 6. Associationassociate new material with related
person. concepts and principles from past experience.
2. Refer to basic textbooks for details and illustrations 7. Original learningif an unfamiliar topic is pre-
as necessary to recall specific information related to sented, do more than review. Seek out sources of
basic sciences. additional information.
3. Integrate reading with experience. 8. Make noteslook for key words, phrases, and sen-
a. Think of examples that illustrate the key tences in the outlined review material, and mark
concepts and principles. them for later reference.
2164_Ch01_001-058 29/03/12 12:18 PM Page 4

4 chapter 1 Orientation and Pre-Tests


ORIENTATION

9. Definitionslook up unfamiliar terms in a c. When you have drawn all that you can recall,
dictionary or the glossary of a basic text, or in check with the book and study what you did
Appendix C. correctly and incorrectly.
10. Additional studyrefer to other textbook refer- d. Take another sheet and do it again. Purpose: to
ences for more detailed information. reinforce what you already know and work
11. Distractorskeep a pad of paper on hand to jot with what you want to remember.
down extraneous thoughts; get them out of your 6. Make up and use mnemonic devices to help
mind and onto the paper. you remember the important elements (see
Appendix B).
7. Repetitively explain to another person the
Memorization: Purpose material you want to memorize.
8. Saturate your environment with the material you
and Strategies want to memorize.
You will need to memorize some items before you can a. Purpose: to overcome the minds tendency to
rapidly assess or apply that knowledge to a particular ignore.
situation; for example, you need to be able to recall the b. Tape facts, formulas, concepts on walls.
standard and lethal doses of a drug before deciding to 9. Above all, feel confident in your ability to
administer it. Items you should memorize include, but memorize!
are not limited to:
1.
2.
Names of common drugs.
Lethal and therapeutic doses.
The Mechanics of the
3. Laboratory norms and values. National Council Licensure
4. Growth and development norms.
5. Foods high or low in iron, protein, sodium, Examination for Registered
6.
potassium, or carbohydrates.
Conversion formulas.
NursesComputer Adaptive Test
7. Anatomical names. (NCLEX-RN )
8. List of cranial nerves and their innervations.
Since 1994, the licensure examination for registered
9. Definitions of defense mechanisms.
nurses has been administered year-round by computer, in
To facilitate memorizing these and other essentials, here an adaptive format: NCLEX-RN. Frequently, candidates
are some strategies: for the examination have many questions about the struc-
ture and format of the test itself and the rules and regula-
1. Before you work on training your mind to
tions concerning the examination procedure.
remember, you must want to remember the
As an aid to reducing apprehension and time spent on
material.
speculation, this section is intended to provide informa-
2. You cannot memorize something that you do not
tion that candidates need, in outline form; the last seg-
understand; therefore, know your material.
ment offers a question-and-answer format for specific
3. Visualize what you want to memorize; picture it;
questions frequently asked by nursing students.
draw a picture.
This information was verified as correct at the time
4. Use the familiar to provide vivid mental pictures,
that it was compiled from information provided by the
to peg the unfamiliar.
National Council of State Boards of Nursing, Inc.
a. When needing to remember a sequence, use
(NCSBN), and other parties involved in the testing. If you
your body to turn material into a picture.
have further questions, contact the board of nursing in
Draw a person, then list the first item to be
your state.
memorized on top of the head, the next item
on the forehead, and so on for nose, mouth, I. WHAT NCLEX IS
neck, chest, abdomen, thighs, knees, and feet. A. National Council Licensure Examination, devel-
b. Use what you already know to tie in with oped and administered by the National Council
what you want to remember; make it of State Boards of Nursing, Inc., with services
memorable. provided by VUE (Virtual University Enterprises),
c. Use as pegs the unexpected, the exaggerated. an NCS (National Computer Systems) Pearson
Weird imagery is easiest to recall. Company.
5. Use the blank-paper technique: B. Tests for minimum nursing competence according
a. Place a large blank sheet on the wall. to a national standard.
b. After you have studied, draw on the blank C. Pass/fail result; must pass to be issued license from
paper what you remember. state board of registered nursing.
2164_Ch01_001-058 29/03/12 12:18 PM Page 5

The Mechanics of the National Council Licensure Examination for Registered Nurses 5

ORIENTATION
D. National examination. Application procedure and than his or her ability, then he or she will have a
licensing requirements are determined by each state greater than 50-50 chance of answering the item
board and may differ from state to state. correctly. Conversely, when a candidate encounters
II. CAT: HOW IT WORKS an item that is higher than his or her ability, then
he or she will have a less than 50-50 chance of
A. Stands for computer adaptive testing.
answering the question correctly. The deviation
B. Adaptive because the computer estimates the
from 50-50 is governed by the size of the difference
candidates ability after each question; the next
between the persons ability and the items difficulty.
question is chosen based on the candidates esti-
A candidates ability could be described by illustrat-
mated ability so far. The test is therefore adapt-
ing the type of item that he or she finds challeng-
ed to each candidate; everyone takes a different,
ing. The latest passing standard took effect on
customized test.
April 1, 2010, in conjunction with the 2010
C. Estimated ability is recalculated after each question
NCLEX Test Plan (see Section V).
until it is precise enough to determine whether the
candidate is above or below minimum competence III. SOME SPECIFICS
level, in all areas of the test plan. The test will stop A. Format
as soon as this is determined. 1. Most questions will be multiple choice, each with
D. Example: The first question is at a fairly easy four options from which you choose only one
level. The candidate answers correctly, so the option. In addition to multiple-choice items,
computer estimates that the candidates ability candidates may be administered items written in
level is above the level of this question and selects alternate formats (Table 1.1). The formats may
a harder question; or if the candidate answers include but are not limited to multiple response
incorrectly, it selects an easier question, to pin- (including ordered response), fill-in-the-blank,
point where the candidates ability level lies, in numerical entry, drag and drop, and/or hot spots.
each area of the test plan. All item types may include multimedia such as
E. After consideration of all available information, the charts, tables, graphics, sound, and video.
NCSBN determined that safe and effective entry- 2. One question at a time appears on screen
level RN practice requires a greater level of knowl- (Fig. 1.1).
edge, skills, and abilities than was required in 2007 3. Each question stands alone (no scenarios
(the date that the NCLEX was last reviewed/ with several spin-off questions); no question
revised). The passing standard was increased in will relate to information given in previous
response to changes in U.S. health-care delivery and questions.
nursing practice that have resulted in more acutely B. Answering
ill clients being seen by entry-level RNs. An expert 1. As candidate chooses the answer to each ques-
panel of nine nurses was convened and performed tion, he or she will be asked to confirm that
criterion-referenced standard-setting procedures. answer by pressing the <NEXT> button, or can
The panels findings supported the creation of a change that answer before confirming.
higher passing standard. In addition, the NCSBN 2. Must answer every question (no penalty for
considered the results of national surveys of nursing guessing; will simply count as a correct or incor-
professionals including nursing educators, directors rect answer and adjust difficulty of next question
of nursing in acute care settings, and administrators accordingly).
of long-term care facilities. The NCSBN voted on 3. Cannot go back to previous questions either to
December 10, 2009, to raise the passing standard review or change answers (because the level of
for the NCLEX examination. The new passing subsequent questions administered is based on
standard is 0.16 logits on the NCLEX logistic previous answers).
scale, 0.05 logits higher than the previous standard C. Number of questions
of 0.21 logits. Logits are usually a foreign concept 1. 75 to 265, including about 15 tryout questions
to the general public. Logit is an abbreviation for that are not scored (these are questions being
log odds unit. Basically, logits are a useful unit of field tested for use on future examinations).
probability. Using the candidate responses, the diffi- 2. Depends on number needed to determine
culty of the items relative to each other (not relative whether candidate is definitely above or definitely
to the candidates) is assessed. Once the hierarchy of below minimum competence level.
difficulty for items is established, then the ability of 3. Taking only the minimum can mean either pass
a candidate can be described as the point on this or fail; it indicates that it took fewer questions
item hierarchy, or continuum, where the candidate to determine whether this candidate is above
has a 50-50 chance of correctly answering an item. or below minimum competence, but does not
When a candidate encounters an item that is lower indicate which!
2164_Ch01_001-058 29/03/12 12:18 PM Page 6

6 chapter 1 Orientation and Pre-Tests


ORIENTATION

Table 1.1
Description of NCLEX Question Formats
Multiple Choice
Which clinical feature found on assessment should indicate to a nurse that a client has congestive heart failure? Select the one best
answer.
1. Fatigue and dyspnea.
2. Cheyne-Stokes breathing and orthostatic hypotension.
3. Liver tenderness and peripheral edema.
4. Pulmonary crackles and weak pulses.
Multiple Response
A client with a history of Graves disease is admitted for uncontrolled hyperglycemia and a foot ulcer. This client requires close moni-
toring because of the increased risk of:
Select all that apply.
1. Hypotension.
2. Elevated temperature.
3. Hypothyroidism.
4. Hypoglycemia.
5. Extreme tachycardia.
Fill in the Blank
A child is to receive digoxin 0.07 mg PO twice daily. The label on the bottle of digoxin reads 0.05 mg/mL. A nurse correctly calcu-
lates that this client should receive _______ mL of digoxin. Record your answer using one decimal place.
Hot Spot
A client has mitral regurgitation. Where is the best place for a nurse to auscultate the associated murmur?

D C

Ordered Response (Prioritization; Drag and Drop)


A nurse prepares to administer a Denver II developmental assessment on a toddler. In which order should the nurse complete the
assessment? Place each action in sequential order.
Ask child to name dolls body parts.
Ask caregiver if the child uses a spoon and fork.
Ask the child to build a tower of 2 cubes.
Advise parent that the assessment is not an IQ test.
Have child kick ball forward.
2164_Ch01_001-058 29/03/12 12:18 PM Page 7

The Mechanics of the National Council Licensure Examination for Registered Nurses 7

ORIENTATION
Table 1.1
Description of NCLEX Question Formatscontd
Chart or Exhibit
A nurse is caring for a client with a closed chest drainage system. Which area should the nurse identify that regulates the amount of
suction?

To suction To client

1 2 3

Graphic
When documenting in a clients medical record, which symbol or abbreviation is appropriate for a nurse to use according to the Joint
Commission?
1. >
2. @
3. mL
4. U

D. Timing
Question question question
question question question
1. No time limit for each question.
question question question 2. Maximum time for test is 6 hours.
3. A preprogrammed optional break is given after
1 Answer option 2 hours; a second preprogrammed optional
2 Answer option
break may be taken after another 11/2 hours.
E. The test will stop when any one of the following
3 Answer option occurs:
4 Answer option
1. A pass or fail determination can be made.
2. The maximum number of questions has been
Click inside the box to select the answer taken (265).
3. The test has lasted for 6 hours, including exami-
Figure 1.1 Sample appearance of NCLEX-RN question nation instructions, sample items, and all rest
on computer screen.
breaks.
2164_Ch01_001-058 29/03/12 12:18 PM Page 8

8 chapter 1 Orientation and Pre-Tests


ORIENTATION

F. The computer 3. Candidate must present Authorization to Test


1. Computer experience will not be necessary. and two signed pieces of identification, one
2. A mouse is used to move down the list of with picture; candidate must sign in and be
options and select and confirm answer choice by fingerprinted and photographed.
pressing the <NEXT> button; all keys will be 4. Lockers or other secured storage provided;
turned off. personal items restricted in testing room.
3. An on-screen calculator will be available. V. TEST PLAN, DEVELOPMENT
4. Candidate receives a brief orientation in the
A. Test plan
use of the mouse and on-screen calculator
1. Blooms taxonomy for the cognitive domain is
with sample questions before beginning the
used as a basis for writing and coding items for
examination.
the examination. Since the practice of nursing
IV. ADMINISTRATION requires application of knowledge, skills, and
A. Application and scheduling abilities, the majority of items are written at the
1. Involves three parties: the state board of nursing, application or higher levels of cognitive ability,
VUE, and Pearson Professional Test Center. which requires more complex thought processing.
2. Candidate applies to the board of nursing in the 2. Client needs are the bases, or dimensions, for
state in which licensing is desired; must meet the test plan (Table 1.2).
requirements of that state board. (There is a test 3. Health needs of clients: The test plan primarily
fee, and some state boards may also require an emphasizes meeting the clients physical needs
administrative fee.) in actual or potentially life-threatening, chronic,
3. After state board determines eligibility and fees or recurring physiological conditions and the
are received, candidate will be mailed an needs of clients who are at risk for complications
Authorization to Test (ATT) and information on or untoward effects of treatment. Subneeds
available Pearson Professional Test Centers, include (1) basic care and comfort (6%12%),
which serve as testing sites (at least one per state; (2) pharmacological and parenteral therapies
some states have many more). (13%19%), (3) reduction of risk potential
4. Candidate must schedule an appointment to (10%16%), and (4) physiological adaptation
take the examination by calling NCLEX (11%17%).
Candidate Services or going to the NCLEX The second highest category of emphasis is
Candidate Web site. (Note: These procedures safe, effective care environment, which focuses
differ in some states, such as Massachusetts.
Candidates should follow instructions from
state board.)
5. Candidate contacts Pearson Professional Test
Center (or other designated test site) to set up
Table 1.2
testing appointment (5 days/wk, 15 hr/day, in Examination Weight Given to Each Category
6-hr time slots). Candidates may test at any test of Health Needs of Clients Based on Current
center in any state (they do not have to take the Practice Analysis Study*
test in the same state to which they have applied
Percentage of Items From
for licensure). Client Needs/Subneeds Each Category/Subcategory
6. After testing, results are communicated to state
Safe and Effective Care
board to which the candidate applied, which
Environment
mails results to candidate (time frame will differ Management of Care 16%22%
by state). Candidates who do not pass will Safety and Infection Control 8%14%
receive a Candidate Performance Report, indi- Health Promotion and 6%12%
cating performance in each area of the test plan Maintenance
relative to the passing standard, and information
Psychosocial Integrity 6%12%
about retesting.
B. Test environment: Pearson Professional Test Center Physiological Integrity
Basic Care and Comfort 6%12%
test sites Pharmacological and 13%19%
1. Up to 30 workstations, each with computer Parenteral Therapies
terminal, desk lamp, work surface, and scratch Reduction of Risk Potential 10%16%
paper. Physiological Adaptation 11%17%
2. Designed for security, monitored by a proctor *Source: National Council of State Boards of Nursing web site. 2010
and by videotape. NCLEX-RN Test Plan.
2164_Ch01_001-058 29/03/12 12:18 PM Page 9

The Mechanics of the National Council Licensure Examination for Registered Nurses 9

ORIENTATION
on (1) management of care (16%22%) and j. Communicable diseases (see Chapters 5
(2) safety and infection control (8%14%). and 6).
Health promotion and maintenance (6%12%) k. Natural and behavioral sciences (integrated
covers growth and development throughout into all units).
the life span and prevention and early detection l. Normal growth and development
of disease. Psychosocial integrity (6%12%) (see Chapters 4, 5, and 10).
concerns coping and adaptation in stress- and m. Basic human needs (see Chapter 10).
crisis-related situations throughout the life n. Individual coping mechanisms (see
cycle. Chapter 10).
4. Nursing process (nursing behaviors): The exami- o. Actual or potential health problems
nation integrates steps of the nursing process, (see Chapters 4, 5, 6, and 7).
as applied to client situations from all stages in p. Life cycle.
the life cycle and to common health problems in q. Client environment (there is continual
all the major health areas and based on current reference throughout this book related to
morbidity studies. It is a problem-solving protection from harm against airborne
approach to client care that includes data collec- irritants, cold, and heat; identification of
tion (assessment, analysis), planning, implemen- environmental discomforts such as noise,
tation, and evaluation. odors, dust, and poor ventilation; elimination
5. Levels of cognitive ability: Most items are at the of potential safety hazards; maintenance of
levels of application (application of rules, proce- environmental order; and cleanliness).
dures, principles, ideas, and theories, and using B. Development
a concept in a new situation) and analysis 1. The Examination Committee of the National
(analysis of data to set priorities and see rela- Council prepares the test plan, which is
tionships). Some items include knowledge and approved by the delegates representing the state
comprehension (simple recognition or recall of boards of nursing. It is designed to reflect the
material; restating or reorganizing material to knowledge and skills needed for minimum
show understanding). competence by a newly licensed nurse to be a safe
6. Categories of nursing knowledge and other and effective practitioner in entry-level nursing,
concepts that are commonly tested: as determined by studies of nursing practice
a. Caring ways by which nursing can assist (performed every 3 years to reflect current
individuals to maintain health and cope with nursing practice).
health problems. It involves interaction 2. State boards take turns nominating item writ-
between nurse and client in a collaborative ers (faculty, clinical nurse specialists, or begin-
environment, with mutual respect and trust. ning practitioners); the Board of Directors of
The nurse provides hope, support, and com- the National Council then selects from this
passion to achieve desired outcomes. (See group those who meet the criteria for item
nursing care plan/implementation sections in writing based on expertise in a particular area
each unit.) of nursing, type of nursing program, creden-
b. Communication (see Chapter 10). tials, regional balance, etc. The item writers
c. Effects of age, sex, culture, ethnicity, and write questions based on common clinical
religion on health needs (sociocultural com- situations and according to the test plan. Then
ponents) are integrated into all units; for the questions are researched and reviewed by a
special emphasis see Chapters 3 and 9 panel of experts (nominated by the state
(dietary implications). boards), by those state boards that choose to
d. Documentation; legal and ethical aspects of review, and by the National Councils
nursing, accountability (see Chapter 3). Examination Committee.
e. Self-care. 3. Finally, the questions are field tested to eliminate
f. Teaching-learning; helping client and questions that may be ambiguous, irrelevant, or
significant others to acquire knowledge, not equally applicable to all regions of the
skills, and attitudes that facilitate changes in United States. (Each candidate taking the
behavior. (See health teaching sections [after NCLEX will take 15 of these tryout ques-
the conditions] that are part of Nursing care tions, mixed in with the regular questions; they
plan/implementation.) will not count toward that candidates perform-
g. Nursing fundamentals (see Chapter 11). ance.) The data gained through field testing are
h. Nutrition and diet therapy (see Chapter 9). also used to analyze and determine the difficulty
i. Pharmacology (see Chapter 8). level of a question.
2164_Ch01_001-058 29/03/12 12:18 PM Page 10

10 chapter 1 Orientation and Pre-Tests


ORIENTATION

C. Passing standard: The passing standard is criterion B. Taking the test/after the test
referenced; this means that there is no fixed percent- 1. Can I skip questions? No. You must answer
age of candidates who pass or fail. Passing depends each question in order to move on to the next.
solely on performance in relation to the level of You also will not be able to return to previous
minimum competence. The passing standard is questions to try them again or change the
reviewed every 3 years. answer (because the level of subsequent questions
administered is based on previous answers).
VI. FREQUENTLY ASKED QUESTIONS 2. What if I just dont know the answer? Use the
AND ANSWERS ABOUT NCLEX-RN tips in Test-Taking Tips and Guidelines: Sixty
A. Test questions, test plan Strategies to Use in Answering Questions,
1. How many questions will there be? You will take Chapter 2, pp. 62-67, to make your best guess.
anywhere from 75 to 265 questions. You will need to answer the question in order to
2. Where do the questions come from? The state move on to the next question.
boards of nursing nominate item writers, 3. When will I know my results? Will the computer
who must meet various criteria (see V. B. tell me? You will not learn your pass/fail status
Development, p. 9). at the testing center; the results will be commu-
3. Will there be questions involving conditions with nicated directly to your state board of nursing,
which I may not have had experience in my nurs- who will mail the results to you. The time frame
ing program? Most of the questions are about may differ from state to state, but most candi-
clients with conditions familiar to you and are dates should receive their results within 10 days
representative of common health problems on a to 2 weeks after taking the examination. Some
national basis. Some questions may relate to states provide a Results-by-Phone Service, where
nursing problems with which you may not have unofficial results are available 3 business days
had prior experience; their purpose is to test after taking the examination. There is a flat fee
your ability to apply knowledge of specific prin- for this service. Note: Taking only the mini-
ciples from the physical, biological, and social mum number of questions, or taking the maxi-
sciences to new situations. mum, is not an indication of whether you have
4. Can some questions have more than one answer? passed or failed. It merely indicates that a lesser,
Yes. In some questions, only one of the four or greater, number of items was required to
options is the best answer. In alternate items, you reach a determination.
may be asked to choose more than one option as 4. What percentage must I answer correctly to pass?
your answer. Because of the nature of CAT, a percentage rate
5. Will I get partial credit for selecting the next-to- is not used to determine passing or failure. The
the-best answer? No. Your answer will be treated test determines whether you are above or below
only as correct or incorrect. standard competence by determining not the
6. If everyone takes a different test, how can it be number of questions you answer correctly, but
fair? Each candidate is tested according to the rather the difficulty level you can consistently
same test plan, and the same passing standard. It answer correctly. The process of administering
may simply require more items to reach a stable harder or easier questions, as described in II.
pass/fail determination for one candidate, while CAT: HOW IT WORKS, p. 5, continues
another may require fewer questions to demon- until you reach the level where you answer
strate competence level. approximately 50% of the complex questions
7. Do diploma, associate, and baccalaureate graduates correctly. Note: When practicing with questions
all take the same test? What about graduates from of mixed difficulty, such as those in this book,
schools outside the United States? All candidates you may wish to use 80% correct as a bench-
are held to the same test plan and passing stan- mark goal for yourself.
dard, and take the same test. (Of course, due 5. If I do not pass, can I retest only those areas of the
to the nature of CAT, each will receive an test plan in which I tested poorly? No. The whole
individualized test, without regard to degree, examination is on a pass/fail basis.
or to state or country of education.) 6. Can I repeat the examination before I get my
8. Does the examination differ by state? The exami- results? No.
nation is a national examination; all candidates 7. How many times can the examination be repeated,
are held to the same test plan and passing stan- and when? The National Council allows you to
dard, and take questions from the same pool. repeat the examination not more than once in
(Again, through CAT, each candidate receives an any 45-day period; however, each state board
individualized test, but without regard to state.) may set its own, more restrictive time limits and
2164_Ch01_001-058 29/03/12 12:18 PM Page 11

How to Prepare for and Score Higher on Examinations 11

ORIENTATION
retake requirements. At present, most states judgment in answering questions, and to make educated
allow the examination to be repeated after guesses when you are not sure of the right answer.
45 days; other states still require a 91-day wait- This section offers practical suggestions to help ensure
ing period before repeating the examination. that you are at your best on examination day, and dis-
8. Who grants the nursing license? The license cusses practical strategies for eliminating wrong answers
is granted by the state board of nursing to and for increasing your chances of selecting the best ones.
which you applied and for which you took
the examination. Prepare Physically and Mentally
C. Testing facilities 1. On the morning of an examination, avoid exces-
1. Can I bring any materials into the examination sive oral intake of products that act as diuretics for
room? There are severe restrictions on personal you. If you know that coffee or cigarettes, for
items allowed in the examination room; you example, increase urgency and frequency, it is
will be provided with a locker or other secure best to limit their intake. Undue physiological
place for your belongings. Do not take any study discomforts can distract your focus from the
materials (books, notes, calculators, etc.), pens, examination at hand.
pencils, candy, chewing gum, drinks, food, 2. Increase your oral intake of foods high in glucose and
watches, pagers, cell phones, note pads or Post-it protein. These foods reportedly have been helpful
notes, purses, wallets, or cameras into the exami- to some examinees for keeping up their blood-
nation building. sugar level. This may enhance your concentration
2. How many people will take the test at the same and problem-solving ability at the times when you
time? Up to 30 people may be at computer most need to function at a high level. On the
workstations in your testing room; however, other hand, avoid carbohydrates such as dough-
they may not all be taking the NCLEX, nuts, which slow down thinking.
because Pearson Professional Test Centers 3. Before examination days, avoid eating exotic or
also offer other services. highly seasoned foods to which your system may
3. When and where will I take the test? As soon as not be accustomed. Avoid possible gastrointestinal
you receive your authorization (ATT), you distress when you least need it!
must contact a Pearson Professional Test Center 4. Use hard candy or something similar prior to
facility to set up your own testing appointment entering the examination room, to help relieve
(5 days/wk, 15 hr/day, in 6-hr time slots). There the discomfort of a dry mouth related to a state
is at least one facility per state, although most of anxiety.
states have many more. A candidate must test 5. Wear comfortable clothes that you have worn before.
within the validity dates of the ATT. These dates The day of an examination is not a good time to
cannot be extended. If the ATT expires, you wear new footwear or clothes that may prove to be
may be required to re-register and repay to test. constricting, binding, or uncomfortable, especially
4. What accommodations are available for candidates at the waistline and shoulder seams.
with disabilities? All test centers are accessible to 6. Anxiety states can bring about rapid increases and
candidates with disabilities. Other accommoda- decreases in body temperature. Wear clothing that
tions will be made only with the prior authoriza- can be shed or added on. For example, you might
tion of the board of nursing; contact your state bring a sweater that can be put on when you feel
board before submitting your application. chilled or removed when your body temperature
fluctuates again.
7. Women need to be prepared for late, irregular, or
How to Prepare for and Score unanticipated early onset of menses on examination
day, a time of stress.
Higher on Examinations 8. Examination jitters can elicit anxiety-like reactions,
both physiological and emotional. Because anxiety
The Psychology of Test-Taking tends to be contagious, try to limit your contacts
Many nursing students know the nursing material on with those who are also experiencing examination-
which they are being tested, and can demonstrate their related anxiety or who elicit those feelings in you.
nursing skills in practice, but do not know how to prepare 9. The night before an examination is a good time to
themselves for taking and passing examinations. engage in a pleasurable activity as a means of
It is not just a matter of taking examinations but of anxiety reduction. You need stamina and endurance
knowing how to take them, taking steps to ensure you can for sitting, thinking, and reacting. Give yourself a
function at full capacity, and using the allotted time in the chance for restful, not energy- or emotion-draining,
most productive way. You must learn to use strategy and activities in the days before an examination.
2164_Ch01_001-058 29/03/12 12:18 PM Page 12

12 chapter 1 Orientation and Pre-Tests


ORIENTATION

10. Try a relaxation process if anxiety reaches an 6. When taking practice questions, it is a good idea
uncomfortable level that cannot be channeled to aim for an average of 1 minute per question, so
into the service of learning (see How To Reduce that you will be at a speed to finish the examina-
Anxiety, p. 13). tion even if you do need to take the maximum
11. When you arrive home after an examination, number of questions. For the actual examination,
jot down content areas that were unfamiliar however, the 6-hour time limit is not a problem
to you. This may serve as a key focus for for most candidates, so go ahead and take the time
review. to work through a difficult problem, and make use
of the scratch paper provided (but dont dwell on a
Tips for NCLEX-RN question you just cant get!).
1. Get an early start on the day you take the exami-
nation, to avoid raising your anxiety level before
Tips for Other (Pencil-and-Paper)
the actual examination starts. Allow yourself time Tests and Examinations
for delays in traffic and in public transportation 1. Answer the easy questions first. Too often students
or for finding a parking place. Even allow for a focus on 1 question for 10 minutes; for example,
dead battery, flooded engine, flat tire, or bus instead of going on to answer 20 additional
breakdown. If you are unfamiliar with the area in questions during this time. The main goal in this
which the test center is located, find it the day type of examination is to answer correctly as many
before. questions as possible.
2. Remember that you do not need to get all the 2. Your first hunch is usually a good one. Pay attention
answers right to pass. The examination is to your intuition, which may indicate which
designed to test for minimum competence; answer feels best.
demonstrating a higher level will not earn a 3. Be wise about the timing. Divide your time. For
special designation on your license, or any other example, if you have 90 questions and 11/2 hours
bonus. Moreover, due to the adaptive nature of for the test, aim for an average of 1 question per
the test, you will probably reach a level where minute. Keep working! Do not lose time looking
you are answering only 50% of the questions back at your answers.
correctly; this is normal for this test and should 4. If you cannot decide between two multiple-choice
not in itself be taken as an indication of poor answers, make a note of the numbers of the two
performance (you may be answering 50% of the choices. This will narrow down your focus when you
very difficult questions correctly)! come back to this question. Leave the question; do
3. Because you cannot skip questions and go back not spend much time on those in doubt. When you
to them, or go back to change answers, it is have completed the test, go back and spend more
important that you simply do the best you can on time on those with which you had trouble.
a particular question, using the Test-Taking Tips 5. Exercise care and caution when using electronically
and Guidelines: Sixty Strategies to Use in scored answer sheets or booklets. It is essential that
Answering Questions, Chapter 2, pp. 62-67, you use only the type of pencil or ink specified in
and move on. The adaptive test will give you the instructions. If erasing is possible, be sure to
another chance to show your competence, should erase completely; a mere trace might throw out the
you get that question wrong. answer.
4. Remember that, although you cannot change 6. When using a separate answer sheet or booklet, be
answers you have already confirmed, you may especially careful to mark your answer in the space
change your answer during the selection process. As for the correct question number. It might be helpful
you make your answer selection, you will choose to say to yourself as you answer each question,
an answer (by pressing the enter key once) and Choice No. 4 to question No. 3, to make sure
then confirm it (by pressing the <NEXT> key); that the right answer goes with each question
you will be able to change your mind before con- number.
firming your answer choice. 7. Stay the entire time allotted. If you complete the
5. Although the examination uses different levels of test early, check your answers. On a second look,
difficulty to estimate your competence level, do after you have completed the test, you may find
not try to figure out the difficulty level of each something that you are now sure you marked in
question; likewise, do not try to keep track of the error the first time. If you were undecided
number of questions you are answering. You will between two possible answers on any questions,
only distract yourself and raise your anxiety. use leftover time to reconsider those answers.
Again, simply answer each question to the best of (Also, look for and erase stray marks, if using
your ability and move on. electronically scored answer sheets.)
2164_Ch01_001-058 29/03/12 12:18 PM Page 13

How to Reduce Anxiety 13

ORIENTATION
How to Reduce Anxiety The physiological changes during relaxation include
decreased oxygen consumption, decreased carbon dioxide
Most people have untapped inner resources for achieving elimination, and decreased respiratory rate.
relaxation and tension release in stressful situations (such The basic factors vital to eliciting a relaxation response
as during an examination) when they need to function at include the following:
their highest potential. The goal of this discussion is to
help you experience a self-guided approach to reducing 1. Quiet settingeliminate unnecessary internal and
your anxiety level to one that is compatible with learning external stimuli.
and high performance. 2. Passive, let-it-happen attitudeempty your mind
In anxiety-producing settings whenever you feel over- of thoughts and distractions.
whelmed or blocked, a fantasy experience can be of help 3. Comfortable positionsit or recline in one
in mastering the rising anxiety by promoting a feeling of position for 20 minutes or so.
calm, detached awareness, and a sense of deeper personal 4. Constant stimulus on which to focusa repetitive
coping resources. Through the fantasy you can gain access sound, constant gaze on an object or image, or
to a zone of tranquility in the center of your being. attention to ones own breathing pattern.
Guided imagery often carries with it feelings of serenity, Relaxation training is a procedure that can be defined,
warmth, and comfort. specified, and memorized until you can go through the
Fantasy experiences are, of course, highly individual. exercises mechanically. If you regularly practice relaxation,
Techniques that help one person experience serenity may you will be able to cope more effectively with difficult sit-
frustrate another. Try out the self-guided experiences uations by reaping the physiological and psychological
suggested here, make up your own, and select ones that are benefits of a balanced and relaxed state.
best for you. There are endless possibilities for fantasy
journeys. The best approach is to work with whatever fan- Instructions
tasy occurs to you at the moment. The ideas for a journey Sit comfortably in a chair. Shut your eyes and chase
presented here are meant to be a springboard for variations your thoughts for a minute; go where your
of your own. thoughts go.
A fantasy will be more effective if you take as comfort- Then, let the words go. Become aware of how you
able a physical position as possible, with eyes closed and feel, here and now, not how you would like to feel.
attention focused on the inner experience. Get in touch Shift your awareness to your feet. Do not move
with physical sensations, your pattern and rate of breath- them. Become aware of what they are doing.
ing, your heartbeat, and pressure points of your body as it Spend 20 to 30 seconds focusing progressively
comes in contact with the chair and floor. on different parts of your body. Relax each part
When you take a fantasy journey by yourself, it is in turn:
important for you to read over the instructions several
times so that you will be able to recall the overall structure Relax each of your toes; the tops of your feet; the arch of
of the fantasy. Then, close your eyes and take your trip each foot; the insteps, balls, and heels; and your ankles, calves,
without concern for following the instructions in detail. knees, thighs, and buttocks. Become aware of how your body
is contacting the chair in which you are sitting. Let go of your
Progressive Relaxation abdominal and chest muscles; relax your back. Release the
tension in your shoulders, arms, elbows, forearms, wrists,
Relaxation approaches are used in a variety of anxiety states
hands, and each finger in turn; relax the muscles in your
whenever stress interferes with the ability to function.
throat, lips, and cheeks. Wrinkle your nose; relax your eyelids
Progressive relaxation training was originated in 1929
and eyebrows (first one and then the other); relax the mus-
by Dr. Edmund Jacobson. It is a technique for attaining
cles in your forehead and top and back of your head. Relax
self-control over skeletal muscles in order to induce low-
your whole body.
level tonus in the major muscle groups. The approach
Concentrate on your breathing; become aware of how you
involves learning systematically and sequentially to tense
breathe. Allow yourself to inhale and exhale in your usual way.
and relax various muscle groups throughout the body.
Become aware of the depth of your breathing. Are you expand-
The objectives of this approach are to soothe nerves,
ing the lungs all the way? Or is your breathing shallow?
combat hypertonus in muscles, and substitute relaxing
Increase your depth of breathing. Now focus on the rate at
activities for stressful ones in order to feel comfortable in
which you are breathing. See if you can slow the rate down.
and more alert to the internal and external environments.
When you breathe in, can you feel an inflow of energy that fills
The theory behind this method takes as its basis the idea
your entire body?
that muscular relaxation and anxiety states produce direct-
ly opposite physiological effects and thus cannot coexist. Now concentrate on the sounds in the room.
In other words, it is not possible to be tense in any body Focus on how you feel right now.
part that is completely relaxed. Slowly open your eyes.
2164_Ch01_001-058 29/03/12 12:18 PM Page 14

14 chapter 1 Orientation and Pre-Tests

Suggestions for Additional 3. Identify the clinical areas where you need further
PRE-TEST

review (e.g., Child Health, Behavioral and


Experiential Vignettes Emotional, Adult Health, Geriatrics, Maternity/
Imagine yourself leaving the room. In your minds Newborn).
eye, go through the city and over the fields. Come 4. Read specific content chapters (Chapters 4, 5,
to a meadow covered with fresh, new grass and 6, 7, and 10).
flowers. Look out on the meadow and focus on 5. Then test yourself again on the Pre-test 2.
what you see, hear, smell, and feel. Walk through
the meadow. See the length and greenness of the This time, take all the questions by one clinical area at
grass; see the brilliance and feel the warmth of the a time (starting with the one you missed the most).
sunlight. If you are repeating the NCLEX-RN, also use the
For a more expansive feeling, visualize a mountain Pre-test 1 and 2 a second time but take questions by
in the distance. Fantasize going to the country client needs/subneeds where you scored below the pass-
and slowly ascending a mountain. Walk through a ing standard and/or near the passing standard.
forest. Climb to the top until at last you reach a Pre-Test 1
height where you can see forever. Experience your


awareness.
Focus on a memory of a beautiful place you have
Questions
been to, enjoyed, and would like to enjoy again. Select the one answer that is best for each question, unless
Be there; experience it. otherwise directed.
Imagine that you are floating on your back down 1. A 38-year-old client, who is gravida 4, para 2 with gesta-
a river. It may help at first to breathe deeply and tional diabetes, is scheduled for an amniocentesis
feel yourself sinking. Visualize that you are com- at 33 weeks gestation because of unstable blood sugar.
ing out on a gentle river that is slowly winding its The results of the amniocentesis reveal the lecithin/
way through a beautiful forest. The sun is out sphingomyelin ratio (L/S) is 1:1, and the phosphatidyl-
and the rays feel warm on your skin. You pass glycerol (PG) is negative. How should a nurse best
trees and meadows of beautiful flowers. Smell the interpret this data?
grass and flowers. Hear the birds. Look up in the 1. The infant is at low risk for congenital anomalies.
blue sky; see the lazy tufts of clouds floating by. 2. The infant is at high risk for intrauterine growth
Leave the river and walk across the meadow. retardation.
Enjoy the grass around your ankles. Come to a 3. The infant is at high risk for respiratory distress
large tree . . . syndrome.
Fill in the rest of the tripwhat do you see now? 4. The infant is at high risk for birth trauma.
Where do you want to go from here? 2. A client, who is overweight and has diabetes, has a blood
Sally L. Lagerquist pressure of 148/92 mm Hg. The clients urinalysis reveals
microalbuminuria. The client does not want to take drugs
PRE-TEST 1 AND 2 despite a family history of brain attack (cerebrovascular
Introduction accident [CVA]). The correct response by a nurse is:
1. Find herbal products to reduce blood pressure and
This Pre-test section is uniqueit provides you with weight.
a special roadmap to your examination success with 2. Use a drug while lifestyle changes are instituted.
150 challenging assessment questions divided into two 3. Evaluate barriers to weight loss and make a plan for
75-item examinations. exercising and dieting.
The Pre-test 1 is an initial assessment tool intended to 4. Recheck blood pressure weekly.
help you to assess your strengths and weaknesses in your
ability to apply the material you have learned in specific 3. A school nurse is called to the playground during recess to
clinical areas to any nursing situation. By taking the Pre- see a child with hemophilia who has collided with a friend.
test, you can focus your subsequent review of content, The childs knee is bruised and swollen and the child
based on your own analysis of your results. reports significant pain. Which actions should the nurse
We suggest that you take the Pre-test 1 before you read take? Select all that apply.
any of the content units in this book. After taking the Pre- 1. Call emergency medical services (EMS).
test 1, you should: 2. Wrap the knee in a compression bandage.
3. Provide crutches for mobility.
1. Score your answers. 4. Apply ice packs.
2. Take another look at the questions where your 5. Elevate the injured extremity.
answer was wrong. 6. Suggest that the child stay indoors during recess.
2164_Ch01_001-058 29/03/12 12:18 PM Page 15

Pre-Test 1 15
4. Which clinical feature found on assessment should indicate 10. A nurse is caring for a teenager immediately following

PRE-TEST
to a nurse that a client has congestive heart failure? surgical correction of severe scoliosis. Which interven-
1. Fatigue and dyspnea. tions should the nurse expect to be part of the care plan?
2. Cheyne-Stokes breathing and orthostatic hypotension. Select all that apply.
3. Liver tenderness and peripheral edema. 1. Administer pain medication around the clock.
4. Pulmonary crackles and weak pulses. 2. Check circulation, sensation, and movement (CSM)
frequently in extremities.
5. A nurse is caring for a client at 32 weeks gestation who has 3. Monitor urinary catheter output.
been admitted to an obstetrical unit with complete placenta
4. Assist child to the chair once per shift.
previa. Which symptoms should the nurse identify as being
5. Assess bowel sounds daily.
a result of this clients condition? Select all that apply.
1. Painless vaginal bleeding. 11. A client with syndrome of inappropriate antidiuretic
2. Tetanic uterine contractions. hormone (SIADH) secretion has a Na+ of 128 mEq/L
3. Premature rupture of membranes. and is confused. A nurses primary goal should be to:
4. Decreased hemoglobin. 1. Decrease edema by restricting fluid intake.
5. Rigid, boardlike abdomen. 2. Prevent complications of hyponatremia.
3. Reorient the client to surroundings.
6. A 4-year-old is hospitalized with acute glomerulonephritis. 4. Restore fluid and electrolyte balance.
A nurse should feel most confident that the client teaching
has been effective and the child has a basic understanding 12. A group of clients who are pregnant are attending a
of this condition if the child states: childbirth preparation class. A nurse is discussing the
1. I was mean to my sister and now I am sick. effects of cigarette smoke on fetal development. Which
2. I have a stomachache but it will get better. characteristic should the nurse describe as being associ-
3. I dont have much pee pee come out when I use the ated with babies born to mothers who smoked during
toilet. pregnancy?
4. I have an infection in my stomach. 1. Low birth weight.
2. Large for gestational age.
7. A 34-year-old client, who is gravida 6, para 4, delivered a 3. Preterm birth, but appropriate size for gestation.
healthy infant at full term about 18 hours ago. She reports
4. Macrosomia.
a sharp pain in her left calf while walking to the bathroom.
A nurse assesses the clients left calf, which has a confined 13. The school nurse inspects the toddlers attending the
area of redness, warmth, and tenderness. Which nursing schools day-care center. Which observation by the nurse
actions are appropriate for this clients nursing care? Select is cause for follow-up for pinworms?
all that apply. 1. All of the toddlers are wearing shoes and socks.
1. Instruct the client to massage the calf to relieve the pain 2. Some of the toddlers are wearing one-piece outfits.
and tenderness. 3. Most of the toddlers have short fingernails.
2. Encourage the client to ambulate to increase circulation. 4. Most of the toddlers are wearing cloth diapers.
3. Administer anticoagulation therapy as ordered.
14. A client, who is gravida 4, para 2, is admitted to a
4. Elevate the affected extremity to promote venous
labor and delivery unit. A nurse performs a vaginal
blood flow.
examination and determines that the clients cervix is
5. Have the client sit up at the side of the bed with
6 cm dilated, 75% effaced, and a +1 station. Based
affected leg dangling.
on the nurses assessment, in which stage of labor is
8. The nurse is conducting a seminar on sudden infant death this client?
syndrome (SIDS) with a group of first-time parents of 1. First phase, latent stage.
newborns. The nurses teaching should emphasize: 2. First stage, active phase.
1. Avoiding soft bedding. 3. First stage, transition phase.
2. Using a pillow. 4. First phase, active stage.
3. Using a soft sleep surface.
15. A client is admitted with a history of an abdominal
4. Promoting co-sleeping with the parents.
aortic aneurysm. A nurse should know that the client
9. The laboratory values for an adult client who is NPO are: has an impending rupture of the aneurysm if the client
Na+ 128, K+ 3.5, and glucose 130. Which IV solution reports:
should a nurse expect will be ordered? 1. Severe right upper quadrant abdominal pain.
1. D10W with 40 mEq KCl/L at 150 mL/hr. 2. Persistent or intermittent lower back pain.
2. 1/4NS with 20 mEq KCl/L at 100 mL/hr. 3. Absent pedal pulses in both extremities.
3. D5 1/4NS at 50 mL/hr. 4. Chest pain in the epigastric region.
4. D5NS with 20 mEq KCl/L at 75 mL/hr.
2164_Ch01_001-058 29/03/12 12:18 PM Page 16

16 chapter 1 Orientation and Pre-Tests

16. A clinic nurse is preparing to administer several scheduled 23. A nursing student is performing an initial newborn
PRE-TEST

immunizations to a 6-month-old infant. What are the assessment. The newborn is observed to have a caput
appropriate actions for the nurse to take when preparing succedaneum. What are likely causes of this condition?
and administering immunizations? Select all that apply. Select all that apply.
1. Draw vaccines into one syringe for injection. 1. Scheduled cesarean delivery.
2. Administer injections in either vastus lateralis muscle. 2. Prolonged active phase of labor.
3. Administer injections in either dorsogluteal muscle. 3. Breech presentation.
4. Ask the parent to wait outside the exam room. 4. Vacuum-assisted vaginal delivery.
5. Select a 1-inch needle. 5. Prolonged second stage of labor.
6. Wash hands.
24. A 65-year-old man with a 45-year history of smoking
17. A nursing student is assigned to care for a client who is reports a change in his cough pattern, a nonproductive
2 days posttotal left hip replacement. Which observa- cough, and generally not feeling well. The chest x-ray
tion should be reported immediately to a staff nurse? reveals an infiltrate. A nurse should suspect:
1. Pain rated as 5 on a scale of 010. 1. Pneumonia.
2. Temperature, 99.6F; reddened incision. 2. Chronic obstructive pulmonary disease (COPD).
3. Pain and cramping in the right lower leg. 3. Pulmonary edema.
4. Difficulty tolerating weight-bearing on the left leg. 4. Tuberculosis.
18. A client, who is gravida 2, para 1, ruptures her mem- 25. A nurse is monitoring the progress of a gravida 3,
branes spontaneously with a large amount of clear fluid. para 1 client in active labor. Which finding should be
A nurse performs a vaginal examination and discovers a reported to a physician immediately?
loop of umbilical cord in the vagina. Which immediate 1. Moderate uterine contractions.
action should be taken by the nurse? 2. Frequent urination.
1. Place the client on her left side. 3. The presence of green-tinged amniotic fluid.
2. Attempt to replace the cord. 4. The presence of scant bloody discharge.
3. Elevate the clients hips.
26. The blood pressure of a 28-year-old client who is over-
4. Cover the cord with a dry, sterile gauze.
weight and does not have diabetes is 136/78 mm Hg.
19. When assessing a child for symptoms of neurogenic Client education should include:
diabetes insipidus, what should a nurse anticipate? 1. Losing 20 pounds would bring the blood pressure
1. Elevated blood glucose. down by as much as 20 points and reduce the risk
2. Increased levels of ADH. of stroke.
3. Increased urine output. 2. The need to recheck blood pressure weekly so
4. Decreased serum sodium. medications can be started as soon as it goes over
140/90 mm Hg.
20. A client, who is gravida 1, para 0, delivered an infant at 3. The need for medications in clients who are
39 weeks gestation this morning. The client tells a nurse
overweight with a systolic blood pressure over
that she plans to breastfeed her baby and has taken a class
130/80 mm Hg.
about breastfeeding. The nurse is aware that successful
4. Acknowledging that the client has maintained a blood
breastfeeding is most dependent on:
pressure within normal limits.
1. Mothers educational level.
2. Size of the mothers breast. 27. A group of clients is attending a childbirth preparation
3. Mothers desire to breastfeed. class. A client asks a nurse how to measure the duration
4. Infants birth weight. of the uterine contractions. Which is the best description
by the nurse about the measurement of the duration of
21. Which observation in the newborn of a mother with dia- contractions?
betes would require an immediate nursing intervention?
1. Duration is measured by timing from the beginning
1. Crying.
of one contraction to the beginning of the next
2. Wakefulness.
contraction.
3. Jitteriness.
2. Duration is measured by timing from the end of
4. Yawning.
one contraction to the beginning of the next
22. In teaching a client with vision loss from glaucoma about contraction.
proper lighting, a nurse knows that: 3. Duration is measured by timing from the beginning
1. Overhead or ceiling lights are best. of one contraction to the end of the same
2. Using fluorescent lighting avoids shadows. contraction.
3. Brighter lights are needed. 4. Duration is measured by timing from the peak of one
4. Dim lighting is needed. contraction to the end of the same contraction.
2164_Ch01_001-058 29/03/12 12:18 PM Page 17

Pre-Test 1 17
28. A client with a history of Graves disease is admitted for 34. A 12-year-old child is seen in a pediatricians office for a

PRE-TEST
uncontrolled hyperglycemia and a foot ulcer. This client well-adolescent checkup. The adolescents most important
requires close monitoring because of the increased risk of: vaccine status for a nurse to assess would be:
Select all that apply. 1. Hepatitis B.
1. Hypotension. 2. Hepatitis A.
2. Elevated temperature. 3. MMR (measles, mumps, and rubella).
3. Hypothyroidism. 4. HPV (human papillomavirus).
4. Hypoglycemia.
35. A client, who fell 10 feet onto a concrete floor, presents
5. Extreme tachycardia.
to an emergency department. The client does not speak
29. Which order written by a pediatrician for a 2-month-old English. The correct action by a nurse to determine the
infant during a well-infant visit should a nurse question? severity of the clients pain would be to:
1. Administer DTaP (diphtheria, tetanus, and acellular 1. Call for an interpreter.
pertussis) vaccine. 2. Note the clients facial expression.
2. Administer Hib (Haemophilus influenzae type b) vaccine. 3. Observe the clients response to palpation.
3. Administer IVP (inactivated poliovirus) vaccine. 4. Use an illustrative pain scale.
4. Administer MMR (measles, mumps, and rubella) vaccine.
36. The parents of a 6-month-old infant express their con-
30. Treatment for hypertension is important in the preven- cerns regarding the number of vaccines their infant has
tion of heart failure. A nurse tells a client that the greatest already received. A nurses best action would be to:
decrease in blood pressure is most likely to occur with: 1. Provide the parents with accurate, user-friendly
1. Increasing physical activity. information regarding vaccines.
2. Reducing sodium intake. 2. Reassure the parents that their concern for their
3. Limiting alcohol intake. infants health is appreciated.
4. Weight loss. 3. Respect the parents ultimate decision regarding
vaccines.
31. A client, who is gravida 3, para 1, is diagnosed with dia- 4. Involve the parents in minimizing potential adverse
betes at 28 weeks gestation. A nurse is providing instruc-
effects of the vaccines.
tion about the role of insulin in managing the clients
diabetes. Which statement by the nurse is true regarding 37. An undesired consequence of acute pain is the
insulin needs during pregnancy? progression to a chronic pain syndrome. The correct
1. Insulin requirements are moderate as the pregnancy approach by a nurse to prevent chronic pain syndrome
progresses. would be to:
2. A decreased need for insulin occurs during the second 1. Adequately control acute pain.
trimester. 2. Give ordered opiates infrequently.
3. Insulin requirements will increase as the pregnancy 3. Give pain medication only when requested by
progresses. client.
4. Elevations in human placental lactogen decrease the 4. Prevent physical dependence on the prescribed
need for insulin. drug.
32. A nursing student selects a 1/2-inch needle with which to 38. A 42-year-old, who is gravida 1, para 0, has a successful
administer a DTaP (diphtheria, tetanus, and acellular in-vitro fertilization and is 7 weeks pregnant. Based
pertussis) vaccine to a 4-month-old infant. A nursing on the clients history, a nurse knows the infant is at
instructors best action would be to: risk for:
1. Allow the nursing student to continue preparing the 1. Respiratory distress syndrome.
vaccine. 2. Pathological jaundice.
2. Stop the nursing student from preparing the vaccine. 3. Turners syndrome.
3. Question the nursing student regarding the injection 4. Down syndrome.
route used to administer the vaccine.
39. A client delivers a 4-pound, 8-ounce neonate at 35 weeks
4. Instruct the nursing student to select a shorter needle.
gestation. A meconium drug screen shows the presence of
33. A client, who is gravida 1, para 0 at 38 weeks gestation, heroin. The neonate is admitted to the nursery with nar-
is found to have a blood pressure of 170/90 mm Hg. cotic abstinence syndrome. Nursing care of the neonate
Which action should a nurse take first? should include:
1. Evaluate the urine protein. 1. Teaching the mother to provide tactile stimulation.
2. Administer an analgesic. 2. Wrapping the neonate snugly in a blanket.
3. Assess fetal heart tones. 3. Placing the neonate in the infant car seat.
4. Obtain a diet history. 4. Initiating an early infant-stimulation program.
2164_Ch01_001-058 29/03/12 12:18 PM Page 18

18 chapter 1 Orientation and Pre-Tests

40. A client, who is 32 years old, gravida 4, para 1 at 2. You will produce more milk if you use a breast pump
PRE-TEST

31 weeks gestation, has been recently diagnosed with and then give it to your baby in a bottle.
gestational diabetes. She has attended a class about 3. Milk production is increased by the hormone
managing diabetes during pregnancy. Which statements estrogen. You can ask your doctor for a prescription
should indicate to a nurse that the client understands to take.
what was taught about gestational diabetes? Select all 4. The size of the breasts doesnt matter. All women
that apply. have about the same amount of milk-producing
1. I cant eat any carbohydrates for the rest of my tissue.
pregnancy. 45. An African American infant with an umbilical hernia is
2. I need to keep my blood sugar well controlled so the brought to a well-infant clinic for routine immuniza-
baby wont grow too large. tions. During the visit, a nurse notes that the parent
3. I will have to take insulin for the rest of my life. has taped the umbilical hernia down on the abdomen
4. I need to count the grams of carbohydrate for each to flatten the protrusion. The nurse would be correct in
meal and my snacks. telling the parent that:
5. I will have to check my blood sugar before I eat in 1. Taping is an appropriate practice to follow.
the morning and after each meal. 2. Taping will eliminate the need for a surgical repair
41. A client with a known history of IV drug use is found because the umbilical hernia will resolve spontaneously
to have acute appendicitis with perforation. Following with this treatment.
surgery, the client reports severe pain at the surgical 3. Taping the umbilical hernia down to the abdomen
incision. There are no signs of abscess formation or other does not help.
postoperative complications. The correct approach to 4. Manually reducing the umbilical hernia on a daily
manage the clients pain would be to: basis by the parent will result in resolution of the
1. Give the ordered opioid. hernia.
2. Expect a placebo to be ordered. 46. A client, who is gravida 3, para 2, is asking a nurse for
3. Give an NSAID or acetaminophen. advice concerning her nightly leg cramps. Which
4. Use nondrug therapies such as visualization. response by the nurse is correct?
42. The mother of a school-age child who is isolated at 1. Increase your fluid intake.
home with chickenpox (varicella) informs a school 2. Increase your protein intake.
nurse that, My child is miserable from the itching. 3. Increase your calcium intake.
What can I do? The school nurses best response 4. Increase your fiber intake.
would be to suggest: 47. A child hospitalized with measles (rubeola) cries every
1. Giving diphenhydramine hydrochloride. time a nurse enters the room and turns on the overhead
2. Keeping the child cool. light. The nurse should investigate if the child has:
3. Changing the linens daily. 1. A fear of bright lights.
4. Keeping the childs fingernails short and clean. 2. An intolerance of light.
43. An 84-year-old with diabetes and left leg cellulitis is 3. Irritated eyes from rubbing.
brought to an emergency department. Vital signs are: 4. An enhanced reaction to unanticipated events.
BP 132/90 mm Hg, T 99.4F, P 95, and R 20. Blood 48. A nurse knows that a client with hypercalcemia may
glucose is 98. Left leg is red and swollen. The client is experience:
admitted to a nursing unit. Which order should be a Select all that apply.
priority for the nurse? 1. Thirst.
1. Start ampicillin of 1 gm every 8 hours. 2. Constipation.
2. Obtain an ECG. 3. Diarrhea.
3. Administer insulin for elevated blood sugar. 4. Loss of appetite.
4. Draw blood cultures. 5. Feeling sleepy.
44. A client, who is gravida 4, para 0 at 39 weeks gestation, 6. Hyperactivity.
states, I would like to breastfeed, but my mother-in-law 49. A child is hospitalized with scarlet fever. Which
told me my breasts are too small and I wont have enough food item should the nurse remove from the childs
milk for my baby. Which would be the best response by breakfast tray?
the nurse? 1. Chocolate milk.
1. Your breasts might be small, but if you dont pro- 2. Orange juice.
duce enough milk, you can always give the baby 3. Apple juice.
some formula. 4. 2% milk.
2164_Ch01_001-058 29/03/12 12:18 PM Page 19

Pre-Test 1 19
50. Positioning to prevent skin breakdown is a nursing con- 55. A school-age child is hospitalized with mumps (parotitis).

PRE-TEST
cern. Which site is at greatest risk for breakdown in this Which food item should the nurse remove from the
position? childs lunch tray?
1. Gelatin.
2. Applesauce.
3. An apple.
C 4. A banana.

A D 56. A client with a history of GI bleeding is admitted


B with complaints of fatigue, weakness, and shortness of
breath. Hgb is 7.4 mg/dL, T 97.8F, BP 86/45 mm Hg,
P 118, and R 22. Two units of packed RBCs are
1. A. ordered. As a nurse begins the blood transfusion, the
2. B. client suddenly complains of chills and chest pain.
3. C. Which actions should be taken by the nurse? Select all
4. D. that apply.
1. Taking the clients vital signs.
51. A client comes to the prenatal clinic to begin care. The
2. Giving acetaminophen as ordered.
nurse would advise her that the routine test(s) during the
3. Stopping the blood transfusion.
initial examination is/are:
4. Infusing normal saline to keep vein open.
Select all that apply.
5. Disposing of blood product in biohazard
1. Rubella titer.
container.
2. Complete blood count (CBC).
6. Sending blood product to lab for analysis.
3. Glucose tolerance test.
4. Ultrasound. 57. A client, who is gravida 5, para 4, delivered a 6-pound,
5. Blood type and antibody screen. 5-ounce infant 2 hours ago. The client is complaining
of severe uterine cramping while breastfeeding. A
52. The most appropriate nursing intervention to encour-
nurse knows that these symptoms are most likely
age a child to eat who had a tonsillectomy 24 hours
due to:
ago is to:
1. Mastitis.
1. Promise the child all the ice cream that the child
2. Uterine involution.
can eat.
3. Uterine atony.
2. Apply an ice collar to the throat 30 minutes prior to
4. Postpartum endometritis.
the meal.
3. Have the child gargle with mouthwash to relieve 58. A child is isolated at home with whooping cough
any unpleasant taste in the mouth. (pertussis). Which parental behavior indicates the
4. Tell the child that there will be no television that night need for additional teaching by the nurse?
if some food is not eaten. 1. Confining cigarette smoking to the outdoors.
2. Avoiding sudden changes in household
53. With which conditions is a risk of postpartum hemor-
temperature.
rhage associated?
3. Keeping the childs room well ventilated.
Select all that apply.
4. Using the fireplace as an additional means of heating
1. Pregnancy-induced hypertension.
the house.
2. The birth of a macrosomatic neonate.
3. Retained placental fragments. 59. A client, who is a 22-year-old primigravida, is at
4. Full urinary bladder. 38 weeks gestation and 9 cm of dilation. Her contrac-
5. Advanced maternal age. tions are 2 to 4 minutes apart and her uterus palpates
soft between very firm contractions. She is very irrita-
54. A client with Sjgrens syndrome, an autoimmune
ble and screaming in pain. A nurse should instruct
disorder in women, is scheduled for surgery. The
this client to:
correct nursing action would be:
1. Take deep breaths.
1. Have a humidifier at the bedside when the client
2. Practice patterned breathing with deep cleansing
returns from surgery.
breath.
2. Lubricate the clients eyes with ointment before surgery.
3. Hold her breath and bear down.
3. Tell the client that nothing can be taken by mouth
4. Do shallow chest breathing.
for 12 hours.
4. Withhold antidepressant therapy for 6 hours after
surgery.
2164_Ch01_001-058 29/03/12 12:18 PM Page 20

20 chapter 1 Orientation and Pre-Tests

60. A charge nurse on a pediatric unit is preparing the daily 5. Hypertension.


PRE-TEST

assignment sheet. Which nurse is the best choice to care 6. Bradypnea.


for a child hospitalized with German measles (rubella)? 65. When counseling the parents of a child with severe
1. A 25-year-old woman who is a new graduate from hemophilia, the nurse would correctly inform the
nursing school. parents that:
2. A 32-year-old woman with 8 years of pediatric nursing 1. Primary prophylaxis should be given on a regular basis
experience. before the onset of joint damage.
3. A 45-year-old woman who is a new graduate from 2. Primary prophylaxis should be given on a regular basis
nursing school. after joint damage.
4. A 50-year-old woman with 25 years of pediatric 3. Primary prophylaxis should be given after surgical
nursing experience. procedures.
61. A client with a brain tumor has been unresponsive to 4. Primary prophylaxis is cost prohibitive.
verbal commands and is showing signs of abnormal 66. A child is being prepared for a tonsillectomy. The
motor responses. Which motor change should indicate to laboratory telephones the pediatric unit and reports
the nurse that a critical change in the clients condition that the childs prothrombin time (PT) is 14.1 seconds.
has occurred? The nurses first action should be to:
1. Abnormal flexion of the upper extremities. 1. Note the laboratory result in the childs chart.
2. Extreme extension of arms and legs. 2. Immediately notify the pediatric surgeon.
3. Diminished response to painful stimuli. 3. Call the operating room and cancel the surgery.
4. Disappearance of the Babinski response. 4. Release the child to the operating room
62. A 5-year-old child is diagnosed with acute tonsillitis for transporter.
the third time in a year. The mother asks the nurse, 67. Which assessment findings should alert a nurse to the
Cant the pediatrician just take the tonsils out now? potential of an addisonian crisis in a client? Select all
Why wait until the infection is over? The nurses most that apply.
accurate response would be: 1. Hyperglycemia and rapid and weak pulse.
1. I agree. Lets talk to the pediatrician about this. 2. Hypernatremia and hyperglycemia.
2. You really shouldnt question the pediatricians reason 3. Cyanosis and weak pulse.
for delaying the surgery. 4. Hypoglycemia and hyponatremia.
3. A healthy child is a better candidate for surgery than a 5. Hypotension and pallor.
sick child.
4. The inflammation and infection increase the risk of 68. A client has mitral regurgitation. Where is the best place
bleeding during the surgery. for a nurse to auscultate the associated murmur?
63. A 30-year-old man has been recently diagnosed with
Cushings syndrome. He asks a nurse if he will be able to
continue riding off-road motorcycles. The correct
response by the nurse is:
1. Continuing important activities will be beneficial to
D C
preventing depression.
2. Because of the potential for obesity of your trunk, a
larger bike will be necessary. B
3. Muscle weakness and the increased risk for fractures
A
will require a change in activities.
4. Visual changes from the disease will prevent you from
riding your motorcycles.
64. The care of a client posttraumatic brain injury includes
monitoring for signs of brainstem herniation and occlu-
sion of cerebral blood flow. A nurse should recognize a
deterioration in the client if which vital sign changes
occurred? Select all that apply.
1. Tachycardia. 1. A.
2. Tachypnea. 2. B.
3. Hypotension. 3. C.
4. Bradycardia. 4. D.
2164_Ch01_001-058 29/03/12 12:18 PM Page 21

Answers/Rationales/Tips 21
69. A 92-year-old client, who recently had a below-the-knee 3. How easily the client can be roused from sleeping.

PRE-TEST
leg amputation, is resisting attempts at rehabilitation. 4. The level of carbon dioxide in the blood using
The most likely reason for the resistance is that the client: capnography.
1. Has goals that differ from the rehabilitation care plan. 75. Physical assessment of a client admitted for a bron-
2. Has too many disabilities to realistically plan for choscopy reveals a thin, muscular man with wheezes in
rehabilitation. the left lung. Which area should a nurse identify as most
3. Is too old to undergo rehabilitation. likely to be partially obstructed?
4. Needs more time to feel comfortable with the dressing
change.
70. A nurse prepares to discontinue the IV of a child with a
low platelet count. Special precautions taken by the nurse
when performing this nursing action should include:
1. Applying a sterile pressure dressing to the IV site.
2. Applying direct pressure to the IV site for 5 minutes.
3. Sending the tip of the IV catheter to the laboratory for
a culture. A
4. Restraining the child from moving the arm with the
IV site for a period of 1 hour.
B
71. A nursing student is being mentored by a charge nurse C
on a pediatric unit. The nursing student is assigned to
care for a child who has increased intracranial pressure
following a head trauma. The charge nurse questions the
nursing student regarding positions that would be con-
traindicated or ineffective when caring for the child. The
D
nursing student should correctly identify: E
Select all that apply.
1. Prone position.
2. Supine position.
3. Semi-Fowlers position. 1. A.
4. Right side-lying position. 2. B.
5. Left side-lying position. 3. C.
72. A client is combative and demanding and refuses to 4. D.
swallow any medication following a brain attack (CVA). 5. E.
The correct nursing action is to:
1. Continue to attempt to follow the physicians orders Answers/Rationales/Tips
and give the medication anyway.
2. Determine what the clients personality was before 1. CORRECT ANSWER: 3. Answer 1 is incorrect because the
the CVA. amniocentesis in this case is not used to determine congenital
3. Apply restraints and request a change in the route of anomalies. Answer 2 is incorrect because the amniocentesis does
medication. not establish intrauterine growth retardation. Answer 3 is cor-
4. Wait for the client to become more cooperative. rect because the test is performed to determine fetal lung
maturity. The L/S ratio should be 2:1 with + PG for the
73. A child is to receive digoxin 0.07 mg PO twice daily. infant who would be at low risk for respiratory distress
The label on the bottle of digoxin reads 0.05 mg/mL. syndrome. Answer 4 is incorrect because the amniocentesis is
A nurse correctly calculates that this client should receive not used to test for birth trauma.
______ mL of digoxin. Fill in the blank. TEST-TAKING TIP: Consider the clients condition; an amnio-
74. A nurse is concerned that a client may be at risk for centesis at 33 weeks gestation is used to establish lung maturity
oversedation from opioid therapy using a patient- in an unborn fetus. Eliminate the answers that do not deal
controlled analgesia (PCA) pump. The most reliable directly with this clients condition.
assessment for possible oversedation would be to check: Content Area: Maternity, Antepartum; Integrated Process:
1. Changes in the level of pain reported by the client. Nursing Process, Evaluation; Cognitive Level: Application;
2. The oxygen saturation level recorded by pulse Client Need/Subneed: Physiological Integrity/Reduction of
oximetry. Risk Potential/Diagnostic Tests
2164_Ch01_001-058 29/03/12 12:18 PM Page 22

22 chapter 1 Orientation and Pre-Tests

2. CORRECT ANSWER: 2. Answer 1 is incorrect because herbs Content Area: Adult Health, Cardiovascular; Integrated Process:
PRE-TEST

have not been adequately tested for managing a disease this Nursing Process, Analysis; Cognitive Level: Application;
significant. Answer 2 is correct because this is an impending Client Need/Subneed: Physiological Integrity/Physiological
disaster. The client has genetic risks and risks from hyper- Adaptation/Pathophysiology
tension (HTN), obesity, and diabetes. The client already has 5. CORRECT ANSWERS: 1, 4. Answer 1 is correct because pain-
early renal disease. While losing weight and exercising less vaginal bleeding is the common symptom of complete
potentially could bring down the clients BP to under placenta previa. Answer 2 is incorrect because tetanic uterine
130/80, the client will be exposed to risks over the months contractions are a symptom of placental abruption, not placenta
it will take to change. Answer 3 is incorrect because, while previa. Answer 3 is incorrect because premature rupture of mem-
losing weight and exercising potentially could bring down the branes is a symptom of placental abruption, not placenta previa.
clients BP under 130/80 mm Hg, the client will be exposed to Answer 4 is correct because the clients hemoglobin level will
risks over the months it will take to change. Answer 4 is incor- drop in response to the bleeding caused by a complete placen-
rect because this is an impending disaster. A client has genetic ta previa. It is common to draw serial blood counts to deter-
risks, and risks from HTN, obesity, and diabetes. A client who mine when to transfuse the mother or deliver the fetus.
already has renal disease needs treatment, not just assessment. Answer 5 is incorrect because a rigid, boardlike abdomen is a
TEST-TAKING TIP: Only one option will definitely reduce the symptom of placental abruption, not placenta previa.
BPdrug therapy. TEST-TAKING TIP: The important difference between
Content Area: Adult Health, Cardiovascular; Integrated Process:
abruption and previa is pain. Abruption is painful with many
Communication and Documentation; Cognitive Level: contractions; previa is painless without contractions.
Application; Client Need/Subneed: Psychosocial Integrity/ Content Area: Maternity, Antepartum; Integrated Process:
Therapeutic Communications Nursing Process, Analysis; Cognitive Level: Application; Client
3. CORRECT ANSWERS: 2, 4, 5. Answer 1 is incorrect because Need/Subneed: Physiological Integrity/Physiological
emergency medical services (EMS) are not required for this Adaptation/Pathophysiology
joint injury because the condition is not immediately life 6. CORRECT ANSWER: 3. Answer 1 is incorrect because the
threatening. The child should be seen by the regular physician. childs statement demonstrates the magical thinking of a pre-
Answer 2 is correct because the nurse should apply a wrap
schooler. This child needs additional teaching and reassurance
to the knee for immobility and compression. Answer 3 is that hospitalization is not a punishment. Answer 2 is incorrect
incorrect because the child should not continue to ambulate as because, although the child may be experiencing a stomachache,
it could worsen the injury. Answer 4 is correct because ice this statement is not the best evidence that the child has a basic
should be applied to constrict blood vessels to reduce bleed- understanding of the illness. Answer 3 is correct because the
ing, swelling, and pain. Answer 5 is correct because elevat- childs statement demonstrates a basic knowledge that the
ing the extremity will help to reduce swelling and bleeding. main reason for hospitalization pertains to a urinary prob-
Answer 6 is incorrect because keeping the child indoors is
lem. Acute glomerulonephritis involves altered kidney func-
inappropriate. The child should be permitted to self-limit tion, resulting in decreased urine output. Answer 4 is incor-
activities and lead as normal a life as possible. rect because acute glomerulonephritis is thought to be an
TEST-TAKING TIP: Use the mnemonic RICE (rest, ice, immune-complex response to a previous streptococcal illness.
compression, elevation) to remember the treatment for Although the child may have a persistent infection and may be
hemophilia. receiving antibiotics, Answer 3 demonstrates greater understand-
Content Area: Child Health, Hematological; Integrated Process:
ing of the rationale for hospitalizationso that kidney function-
Nursing Process, Implementation; Cognitive Level: Application; ing may be monitored and restored.
Client Need/Subneed: Physiological Integrity/Reduction of Risk
TEST-TAKING TIP: Note that only one answer describes a
Potential/Potential for Complications from Surgical Procedures symptom specific to kidney functioning while the others
and Health Alterations indicate no understanding (Answer 1) or are too generalized
4. CORRECT ANSWER: 1. Answer 1 is correct because con- (Answers 2 and 4). Select the option that is specific to the
gestive heart failure (CHF) or right-sided heart failure kidneys.
reduces cardiac output; and circulating O2 is reduced, Content Area: Child Health, Genitourinary; Integrated Process:
producing fatigue and shortness of breath. Answer 2 is Nursing Process, Evaluation; Cognitive Level: Analysis; Client
incorrect because Cheyne-Stokes breathing occurs from Need/Subneed: Physiological Integrity/Physiological
neurologic problems or problems with respiratory gas Adaptation/Alterations in Body Systems
exchange. Answer 3 is incorrect because the liver may be 7. CORRECT ANSWERS: 2, 3, 4. Answer 1 is incorrect because
enlarged but not painful. Answer 4 is incorrect because it is contraindicated to massage the site of the deep vein
crackles would be more consistent with left-sided failure, thrombosis (DVT). Massaging the calf can dislodge the clot
not CHF (right-sided failure). and lead to an embolism. Answer 2 is correct because the
TEST-TAKING TIP: Review the clinical differences between client should be encouraged to ambulate in addition to
right-sided versus left-sided heart failure. receiving anticoagulant therapy, since bedrest itself may
2164_Ch01_001-058 29/03/12 12:18 PM Page 23

Answers/Rationales/Tips 23
enhance venous stasis. Answer 3 is correct because the most in fluid overload. A client needs potassium, which is not pres-

PRE-TEST
important goal is to prevent a pulmonary embolism or its ent in this IV. 1/4 NS would dilute the serum sodium further.
recurrence. Incidence of pulmonary embolism depends on Answer 4 is correct because the client needs potassium,
whether or not DVT is adequately treated with anticoagu- which cannot be replaced with enteral feedings. Fluids are
lant therapy. Answer 4 is correct because elevation of the needed when NPO, but the low serum sodium should not
affected extremity is important initially because it promotes be further diluted with rapid administration of hypotonic
venous return and decreases edema. Answer 5 is incorrect solutions; thus, normal saline is appropriate. Even though
because sitting with legs dependent is contraindicated because the blood sugar is slightly elevated, it is appropriate to give
it increases stasis and edema. clients who are NPO some energy source. Dextrose 5%
TEST-TAKING TIP: The main goal of DVT treatment is to contains only 17 calories/100 mL.
prevent clots from traveling to major organs and causing TEST-TAKING TIP: The variables to look at for inclusion are:
mortality and morbidity. Eliminate those options that are dextrose, NS, KCl, and at a moderate rate. Consider the effect
not consistent with this goal. the solution will have on the laboratory values.
Content Area: Maternity, Postpartum; Integrated Process: Content Area: Adult Health, Fluid and Electrolyte Imbalances;
Nursing Process, Implementation; Cognitive Level: Application; Integrated Process: Nursing Process, Implementation; Cognitive
Client Need/Subneed: Physiological Integrity/Reduction of Risk Level: Analysis; Client Need/Subneed: Physiological Integrity/
Potential/System Specific Assessments Pharmacological and Parenteral Therapies/Parenteral/
Intravenous Therapies
8. CORRECT ANSWER: 1. Answer 1 is correct because the use
of soft bedding for a newborn should be avoided. It is 10. CORRECT ANSWERS: 1, 2, 3. Answer 1 is correct because
believed that newborns sleeping on soft bedding may not be the nurse can expect that the child will experience pain
able to move their heads to the side, increasing the risk of related to surgery, and pain medication needs to be provid-
suffocation, which may be a cause of SIDS. Answer 2 is ed on a schedule to prevent and treat pain. Answer 2 is cor-
incorrect because the use of a pillow for a newborn should be rect because circulation, sensation, and movement (CSM)
avoided. It is believed that infants sleeping with a pillow may must be assessed often to detect potential neurological com-
not be able to move their heads to the side, increasing the risk plications of spinal surgery. Answer 3 is correct because the
of suffocation, which may be a cause of SIDS. Answer 3 is child will have a urinary catheter in place and monitoring
incorrect because the use of a soft sleep surface for a newborn of the output is essential to protect against the complica-
should be avoided. It is believed that infants sleeping on a soft tion of urinary retention. Answer 4 is incorrect because initial
sleep surface may not be able to move their heads to the side, care of this child involves careful logroll turning, not placing
increasing the risk of suffocation, which may be a cause of the child in a chair as this could injure the surgical area.
SIDS. Answer 4 is incorrect because co-sleeping with parents Answer 5 is incorrect because bowel sounds should be assessed
should be avoided. Studies reveal that bed-sharing has a posi- frequently to monitor for possible paralytic ileus after surgery.
tive association with SIDS. The leading theory suggests that Monitoring bowel sounds once daily is not sufficient.
suffocation is the cause. Parents who wish to share a bed with TEST-TAKING TIP: The key phrase in the stem of the question
the newborn are advised to follow the same safeguards in the is immediately following surgery. Select priorities that are
bed as in the crib (e.g., avoiding soft bedding, pillows, soft part of the immediate postsurgical period.
sleeping surfaces, and overheating [thermal stress]). Content Area: Child Health, Musculoskeletal; Integrated
TEST-TAKING TIP: Think suffocation when answering Process: Nursing Process, Analysis; Cognitive Level: Analysis;
this question and eliminate answers that might increase the Client Need/Subneed: Physiological Integrity/Reduction of Risk
possibility of this happening. The question asks for what will Potential/Potential for Complications from Surgical Procedures
prevent the possibility of suffocation. and Health Alterations
Content Area: Child Health, Respiratory; Integrated Process:
11. CORRECT ANSWER: 4. Answer 1 is incorrect because the
Teaching and Learning; Cognitive Level: Application; Client
client needs sodium replacement and fluid restriction. Answer 2
Need/Subneed: Health Promotion and Maintenance/Health
is incorrect because the client already has developed complica-
Promotion Programs
tions. Answer 3 is incorrect because the confusion is due to
9. CORRECT ANSWER: 4. Answer 1 is incorrect because when cerebral edema from the hyponatremia, which needs to be cor-
the sugar of D10W is metabolized, free water remains. At a rate rected. Physiological intervention is needed now, not psychoso-
of 150 mL/hr, this would further decrease the serum sodium. cial (orientation). Answer 4 is correct because SIADH causes
Answer 2 is incorrect because, although the fluid rate and water retention and dilutional hyponatremia.
potassium in this order are reasonable, the 1/4 normal saline TEST-TAKING TIP: Only one answer corrects both the fluid
(NS) contains relatively more free water than normal saline and electrolyte problem.
does, which would decrease the serum sodium further. Clients Content Area: Adult Health, Fluid and Electrolyte Imbalances;
who are NPO generally need an energy source order as dex- Integrated Process: Nursing Process, Planning; Cognitive Level:
trose. Answer 3 is incorrect because the rate is a little slow for Application; Client Need/Subneed: Physiological Integrity/
normal fluid balance, but could be acceptable if the client was Physiological Adaptation/Fluid and Electrolyte Imbalances
2164_Ch01_001-058 29/03/12 12:18 PM Page 24

24 chapter 1 Orientation and Pre-Tests

12. CORRECT ANSWER: 1. Answer 1 is correct because Answer 3 is incorrect because first stage, transition phase would
PRE-TEST

smoking has a direct association with low birth weight, mean the client was 8 to 10 cm dilated. This client is 6 cm dilat-
doubling the risk of an infant who may have a low birth ed, in the active phase of labor. Answer 4 is incorrect because
weight. Answer 2 is incorrect because smoking results in low there is no first phase of labor, only the first stage of labor.
birth weight, not large-for-gestational-age infants. Answer 3 is TEST-TAKING TIP: Eliminate Answers 1 and 4 because there
incorrect because, although smoking does have an association are four stages of labor and three phases in the first stage of
with preterm birth, the infant would be low birth weight, not labor. Eliminate Answer 3 because the woman is not yet in
an appropriate size for gestation. Answer 4 is incorrect because transition phase with 6 cm dilation.
smoking results in low birth weight, not macrosomia. Content Area: Maternity, Intrapartum; Integrated Process:
TEST-TAKING TIP: Eliminate macrosomia and large for Nursing Process, Analysis; Cognitive Level: Application; Client
gestational age because they are essentially the same condition. Need/Subneed: Physiological Integrity/Reduction of Risk
Smoking causes low-birth-weight, not large infants. Potential/System Specific Assessments
Content Area: Maternity, Antepartum; Integrated Process:
15. CORRECT ANSWER: 2. Answer 1 is incorrect because
Teaching and Learning; Cognitive Level: Application; Client abdominal pain from an impending rupture is localized in the
Need/Subneed: Health Promotion and Maintenance/Ante/
middle or lower abdomen to the left of the midline. Answer 2
Intra/Postpartum and Newborn Care is correct because the aneurysm is pressing on the lumbar
13. CORRECT ANSWER: 4. Answer 1 is incorrect because the nerves. Severe back or middle or lower abdominal pain are
wearing of shoes and socks is encouraged to prevent toddlers signs of impending rupture. If the aneurysm ruptures, the
from acquiring the infestation from contaminated soil. This back pain will be constant. Answer 3 is incorrect because cir-
observation would not be cause for follow-up by the school culation would not be impaired before rupture. Answer 4 is
nurse. Answer 2 is incorrect because the wearing of one-piece incorrect because the pain is lower abdomen or back. Epigastric
outfits prevents the toddlers from touching or scratching their pain is more consistent with cardiac or gastric problems.
perianal areas and continually reinfecting themselves. This TEST-TAKING TIP: Visualize the location of the abdominal
observation would not be cause for follow-up by the school aorta.
nurse. Answer 3 is incorrect because short fingernails prevent Content Area: Adult Health, Cardiovascular; Integrated Process:
the eggs from being deposited on the hands and under the fin- Nursing Process, Analysis; Cognitive Level: Application; Client
gernails of the toddlers, with continual reinfection through Need/Subneed: Physiological Integrity/Physiological
scratching their perianal areas. This observation would not be Adaptation/Medical Emergencies
cause for follow-up by the school nurse. Answer 4 is correct 16. CORRECT ANSWERS: 2, 5, 6. Answer 1 is incorrect
because the use of superabsorbent disposable diapers, which because the nurse should not combine immunizations together
prevent leakage, are preferred over cloth diapers. Cloth dia- in a syringe. Answer 2 is correct because the appropriate
pers leak, which could result in feces that may be infested injection site for an infant is the thigh muscle. Answer 3 is
with pinworms (enterobiasis) to be spread from toddler to incorrect because the dorsogluteal muscle is contraindicated for
toddler. Cloth diapers would also be taken home to be use in infants due to the risk of injury to the sciatic nerve.
laundered, which is another way to spread the infestation. Answer 4 is incorrect because parents should be given the
The superabsorbent disposable diapers should be disposed option to stay with the child during painful procedures.
of in a closed receptacle as soon as they are soiled. Answer 5 is correct since a 1-inch needle is appropriate for
TEST-TAKING TIP: Ask yourself: What is the relationship infants 4 months of age and older. This length allows the
between diapers and infestations? tip of the needle to penetrate deep into the muscle and
Picture this: Diapers cover Perianal Area Infestations avoids accidental administration into the subcutaneous
occur in tissue. A longer needle (e.g., 11/2 inch) would be better for
Remember that diapers come in direct contact with the adults or older children who are significantly overweight.
toddlers perianal area, which is where the infestation is Answer 6 is correct since the nurse should first wash hands
located. Superabsorbent disposable diapers would be pre- prior to any procedure.
ferred over cloth diapers because they may prevent leakage TEST-TAKING TIP: Recall that immunizations are intramus-
of contaminated feces. cular injections. The best site for injections in an infant is a
Content Area: Child Health, Gastrointestinal; Integrated
well-developed muscle, such as the thigh.
Process: Nursing Process, Analysis; Cognitive Level:
Content Area: Child Health, Immunizations; Integrated
Application; Client Need/Subneed: Health Promotion and Process: Nursing Process, Analysis; Cognitive Level:
Maintenance/Health Screening Application; Client Need/Subneed: Health Promotion and
14. CORRECT ANSWER: 2. Answer 1 is incorrect because there Maintenance/Immunizations
is no first phase of labor, only the first stage of labor. Answer 2 is 17. CORRECT ANSWER: 2. Answer 1 is incorrect because this is
correct because there are four stages of labor and the first not unexpected following surgery. If the client were unresponsive
stage covers all phases of cervical dilation. This client is in to pain medication, there might be a greater concern. Answer 2
the first stage, active phase, which is 4 to 8 cm dilation. is correct because an infection following hip replacement is a
2164_Ch01_001-058 29/03/12 12:18 PM Page 25

Answers/Rationales/Tips 25
serious complication that may require removal of the Content Area:Child Health, Endocrine; Integrated Process:

PRE-TEST
implant. Total joint infections may be disastrous, with pre- Nursing Process, Planning; Cognitive Level: Application; Client
vention of an infection a priority. Usually prophylactic Need/Subneed: Physiological Integrity/Physiological
antibiotics are ordered; therefore, the appearance of a wound Adaptation/Fluid and Electrolyte Imbalances
infection would be a grave concern. Answer 3 is incorrect
20. CORRECT ANSWER: 3. Answer 1 is incorrect because
because a deep vein thrombosis (DVT) would not be expected
successful breastfeeding is not necessarily dependent on the
until 5 to 7 days after surgery. DVTs do occur in 45% to 70%
mothers educational level. Answer 2 is incorrect because suc-
of clients after hip surgery. Further assessment would be needed,
cessful breastfeeding is not dependent on the size of the
such as the presence of a positive Homans sign and calf
mothers breasts. Answer 3 is correct because successful
swelling. Answer 4 is incorrect because the ability to tolerate
breastfeeding is most dependent on the mothers desire
weight-bearing varies with the clients condition, the procedure
and commitment to breastfeed. Answer 4 is incorrect
done, and the type of fixation device used. There is insufficient
because successful breastfeeding is not dependent on the
information to know if this is a problem.
infants birth weight.
TEST-TAKING TIP: When all options are important nursing
TEST-TAKING TIP: This question is asking about factors that
concerns, look for the one where an immediate response could
are important for breastfeeding success. The size of the breasts,
prevent a serious consequence.
infants birth weight, and mothers educational level are not
Content Area: Adult Health, Musculoskeletal; Integrated
factors in breastfeeding. Select the psychosocial factor (desire)
Process: Nursing Process, Assessment; Cognitive Level: Analysis;
over physical factors (breast size, weight).
Client Need/Subneed: Physiological Integrity/Reduction of Risk
Content Area: Maternity, Postpartum; Integrated Process:
Potential/Potential for Complications from Surgical Procedures
Nursing Process, Analysis; Cognitive Level: Comprehension;
and Health Alterations
Client Need/Subneed: Health Promotion and Maintenance/
18. CORRECT ANSWER: 3. Answer 1 is incorrect because Ante/Intra/Postpartum and Newborn Care
placing the client on her left side does not relieve the umbilical
21. CORRECT ANSWER: 3. Answer 1 is incorrect because cry-
cord compression that results in fetal bradycardia. Answer 2 is
ing is not a specific finding for immediate nursing intervention.
incorrect because it is contraindicated for the nurse to attempt
Answer 2 is incorrect because wakefulness is not associated with
to replace the cord. Answer 3 is correct because elevating the
hypoglycemia, which is the main complication in a newborn
clients hips is the immediate intervention that helps to
of a mother with diabetes. Answer 3 is correct because jitteri-
release the pressure from the umbilical cord until an emer-
ness can be indicative of hypoglycemia and the nurse needs
gency cesarean section can be performed. Answer 4 is incor-
to obtain a heel stick blood glucose. Answer 4 is incorrect
rect because covering the cord with a dry, sterile gauze does not
because yawning is a normal response in a newborn and is not
relieve the umbilical cord compression and the fetal bradycar-
associated with hypoglycemia.
dia that results.
TEST-TAKING TIP: Restate the question. This question is
TEST-TAKING TIP: The most important intervention is to
asking for the symptoms of hypoglycemia. Examine the answers
relieve the umbilical cord compression. Covering the cord with
for the observation that is consistent with hypoglycemia.
a dry, sterile gauze causes a delay in relieving the compression.
Content Area: Child Health, Newborn; Integrated Process:
Eliminate the two options (Answers 2 and 4) that specifically
Nursing Process, Assessment; Cognitive Level: Analysis; Client
focus on the visible cord. Focus on positioning the motherone
Need/Subneed: Physiological Integrity/Physiological
position releases cord compression and the other position
Adaptation/Alterations in Body Systems
(Answer 1) does not relieve the compression.
Content Area: Maternity, Intrapartum; Integrated Process: 22. CORRECT ANSWER: 2. Answer 1 is incorrect because over-
Nursing Process, Implementation; Cognitive Level: Application; head or ceiling lights cause glare or shadows. Answer 2 is correct
Client Need/Subneed: Physiological Integrity/Physiological because glaucoma causes tunnel vision (loss of peripheral
Adaptation/Medical Emergencies vision) and low vision. The client needs increased light, and
sees best in natural lighting (sunlight). Fluorescent lighting
19. CORRECT ANSWER: 3. Answer 1 is incorrect because the does not produce glare and reduces shadows. A task light
child should have a normal blood sugar; the child with diabetes
that can be directed is helpful. Answer 3 is incorrect because
mellitus will have elevated blood glucose. Answer 2 is incorrect
bright lights will cause glare and impair vision. Answer 4 is
because the condition is caused by decreased secretion of anti-
incorrect because the client needs increased lighting and prefer-
diuretic hormone (ADH). Answer 3 is correct because the child
ably natural light, not dim lighting.
with neurogenic diabetes insipidus (DI) will experience severe
TEST-TAKING TIP: Remember that the client has lost periph-
diuresis resulting from a decrease in ADH secretion. Answer 4
eral vision, so that lighting needs to enhance central vision
is incorrect because the child is likely to have increased serum
without glare or shadows.
sodium due to massive amounts of fluid loss through the urine.
Content Area: Adult Health, Sensory; Integrated Process:
TEST-TAKING TIP: Recall that neurogenic diabetes
Nursing Process, Analysis; Cognitive Level: Application;
insipidus and diabetes mellitus share the common symptom
Client Need/Subneed: Physiological Integrity/Physiological
of polyuria.
Adaptation/Illness Management
2164_Ch01_001-058 29/03/12 12:18 PM Page 26

26 chapter 1 Orientation and Pre-Tests

23. CORRECT ANSWERS: 2, 4, 5. Answer 1 is incorrect Need/Subneed: Physiological Integrity/Reduction of Risk


PRE-TEST

because a scheduled cesarean section does not increase the Potential/Potential for Complications from Surgical Procedures
pressure on the fetal head or lead to development of caput and Health Alterations
succedaneum. Answer 2 is correct because caput succeda- 26. CORRECT ANSWER: 1. Answer 1 is correct because this
neum is a diffuse swelling of the scalp in a newborn caused client has prehypertension without compelling indications
by pressure from the uterus or vaginal wall during a head- for drug treatment. Lifestyle changes are indicated. The
first (vertex) delivery. Prolonged active phase increases the client is not recognizing the risk. In one sentence, the risk
pressure on the fetal head. Answer 3 is incorrect because a to the client and how to control the risk were addressed.
breech delivery does not increase the pressure on the fetal head Answer 2 is incorrect because this is prehypertension and
or lead to development of caput succedaneum. Answer 4 is should be managed with lifestyle changes to include at least
correct because vacuum-assisted vaginal delivery increases weight loss. Answer 3 is incorrect because this is prehyperten-
the pressure on the fetal head and can result in caput suc- sion without compelling indications for drug treatment.
cedaneum. Answer 5 is correct because caput succedaneum Answer 4 is incorrect because this is prehypertension, not a
is a diffuse swelling of the scalp in a newborn caused by normal blood pressure.
pressure from the uterus or vaginal wall during a head-first TEST-TAKING TIP: For prehypertension, look for a nondrug
(vertex) delivery. Prolonged second stage increases the option first. Eliminate Answers 2 and 3 with medications
pressure on the fetal head. in the options.
TEST-TAKING TIP: Think about the forces that increase Content Area: Adult Health, Cardiovascular; Integrated Process:
pressure on the fetal head and can cause caput succedaneum. Teaching and Learning; Cognitive Level: Application; Client
Content Area: Child Health, Newborn; Integrated Process:
Need/Subneed: Health Promotion and Maintenance/Lifestyle
Nursing Process, Analysis; Cognitive Level: Application; Client Choices
Need/Subneed: Health Promotion and Maintenance/Ante/
Intra/Postpartum and Newborn Care 27. CORRECT ANSWER: 3. Answer 1 is incorrect because tim-
ing from the beginning of one contraction to the beginning of
24. CORRECT ANSWER: 1. Answer 1 is correct because the
the next contraction is the correct method for measuring the
symptoms are characteristic of pneumonia. Answer 2 is frequency and not the duration of a contraction. Answer 2 is
incorrect because the client reports a change. COPD is a incorrect because timing from the end of one contraction to
chronic condition with sputum production. Answer 3 is the beginning of the next contraction is not the measurement
incorrect because the development of pulmonary edema for the duration of the contraction; it is the resting period
includes frothy sputum. The client indicates a nonproductive between contractions. Answer 3 is correct because duration is
cough. Answer 4 is incorrect because tuberculosis includes a measured by timing from the beginning of one contraction
productive cough, fever, sweats, and weight loss. to the end of the same contraction. Answer 4 is incorrect
TEST-TAKING TIP: Only pneumonia has a nonproductive because timing from the peak of one contraction to the end
coughthe others all produce sputum. of the same contraction is not the measure of duration or
Content Area: Adult Health, Respiratory; Integrated Process:
frequency of a contraction.
Nursing Process, Analysis; Cognitive Level: Application; Client TEST-TAKING TIP: There are similar options for answers in
Need/Subneed: Physiological Integrity/Physiological
this question. Visualize each of the options and try drawing
Adaptation/Pathophysiology it out if needed. Focus on beginning . . . end . . . as the
25. CORRECT ANSWER: 3. Answer 1 is incorrect because best answer not beginning . . . beginning . . . and not
moderate uterine contractions are a normal finding in active end . . . beginning . . .
labor. Answer 2 is incorrect because frequent urination is a Content Area: Maternity, Antepartum; Integrated Process:
normal finding in active labor as a result of descent of the Teaching and Learning; Cognitive Level: Application;
fetal head and decreased bladder capacity. Answer 3 is correct Client Need/Subneed: Health Promotion and Maintenance/
because the presence of green-tinged amniotic fluid is not Ante/Intra/Postpartum and Newborn Care
a normal finding and needs to be reported to the physi- 28. CORRECT ANSWERS: 1, 2, 5. Answer 1 is correct
cian immediately. Meconium staining can be indicative of because the client is at risk for thyroid storm because of a
fetal distress. Answer 4 is incorrect because presence of scant second illness, which can lead to hypotension and if
bloody discharge is a normal finding in active labor, called untreated, heart failure. Answer 2 is correct because an
bloody show. It is an indicator of cervical progress. elevated temperature is a sign of thyroid storm, which is
TEST-TAKING TIP: The question is asking about what is not a a life-threatening form of hyperthyroidism. When a client
normal part of active labor. Eliminate the options that are a with hyperthyroidism suffers a second illness, thyroid
part of normal labor: moderate contractions, frequent urina- storm is a risk. Answer 3 is incorrect because Graves disease
tion, and the presence of scant bloody discharge. is a hyperthyroid state. Answer 4 is incorrect because the
Content Area: Maternity, Intrapartum; Integrated Process:
added illnesses of hyperglycemia and a foot ulcer may poten-
Nursing Process, Assessment; Cognitive Level: Analysis; Client tiate a thyroid storm. Hypoglycemia may be a concern later
2164_Ch01_001-058 29/03/12 12:18 PM Page 27

Answers/Rationales/Tips 27
with treatment. Answer 5 is correct because thyroid storm causing the client to be more insulin resistant, increasing

PRE-TEST
is an acute episode of thyroid overactivity. The metabo- the need for insulin. Answer 4 is incorrect because elevations
lism is markedly increased, and extreme tachycardia in human placental lactogen increase, not decrease, the need for
occurs. insulin.
TEST-TAKING TIP: Focus on the options that would result TEST-TAKING TIP: Remember that insulin needs decrease
from a metabolic storma thyroid storm. Recall that during the first trimester, steadily increase during the second
thyroid storm is rare but a life-threatening medical and third trimesters, then decrease after delivery.
emergency. Content Area: Maternity, Antepartum; Integrated Process:
Content Area: Adult Health, Endocrine; Integrated Process: Teaching and Learning; Cognitive Level: Application;
Nursing Process, Analysis; Cognitive Level: Application; Client Need/Subneed: Physiological Integrity/Physiological
Client Need/Subneed: Physiological Integrity/Physiological Adaptation/Alterations in Body Systems
Adaptation/Medical Emergencies
32. CORRECT ANSWER: 3. Answer 1 is incorrect because
29. CORRECT ANSWER: 4. Answer 1 is incorrect because the allowing the nursing student to continue would lead to an
DTaP (diphtheria, tetanus, and acellular pertussis) vaccine is error that could jeopardize the infant. Answer 2 is incorrect
given at 2 months of age. Answer 2 is incorrect because the Hib because just stopping the nursing student is not enough as it
(Haemophilus influenzae type b) vaccine is given at 2 months of will not lead to critical thinking regarding the correct action
age. Answer 3 is incorrect because IPV (inactivated poliovirus) that needs to be taken. Answer 3 is correct because it will
vaccine is given at 2 months of age. Answer 4 is correct stimulate the nursing student to think in a critical manner.
because MMR (measles, mumps, and rubella) vaccine is not DTaP (diphtheria, tetanus, and acellular pertussis) vaccine
administered until the infant is 12 to 15 months of age. The is given intramuscularly and a 1-inch/2.5-cm needle must
second dose of this vaccine is given at 4 to 6 years of age. be used to deposit the vaccine deep in the muscle mass.
TEST-TAKING TIP: Select as the correct answer the order Answer 4 is incorrect because a longer, rather than a shorter
that is not correct. needle, is needed for the correct administration of the vaccine
Content Area: Child Health, Immunizations; Integrated (i.e., deep in the muscle mass).
Process: Nursing Process, Evaluation; Cognitive Level: TEST-TAKING TIP: Choose the assessment option
Application; Client Need/Subneed: Safe and Effective (i.e., question the nursing student).
Care Environment/Safety and Infection Control/Error Content Area: Child Health, Immunizations; Integrated
Prevention Process: Teaching and Learning; Cognitive Level: Application;
Client Need/Subneed: Health Promotion and Maintenance/
30. CORRECT ANSWER: 4. Answer 1 is incorrect because Principles of Teaching and Learning
increasing activity may drop the systolic BP up to 9 mm Hg.
Answer 2 is incorrect because limiting sodium only lowers the 33. CORRECT ANSWER: 1. Answer 1 is correct because a
systolic BP by 8 mm Hg. Answer 3 is incorrect because limit- high blood pressure (170/90 mm Hg) may be indicative
ing alcohol will likely only lower the systolic BP by 2 to 4 mm of preeclampsia, or may be due to gestational hyperten-
Hg. Answer 4 is correct because a weight loss of 10 kg can sion, pain, or severe anxiety. The nurse needs to deter-
reduce the systolic BP up to 20 mm Hg. mine if the client has proteinuria. If there is proteinuria,
TEST-TAKING TIP: All options are correct, but weight loss the high blood pressure is suggestive of preeclampsia.
produces the greatest drop. Diagrammatically visualize these Answer 2 is incorrect because administering an analgesic will
measures: not help to determine the cause of the high blood pressure in
Physical activity weight this client, and may delay the treatment if the client has
Na+ weight preeclampsia. Answer 3 is incorrect because assessing fetal
Alcohol intake weight heart tones is a general assessment for all clients in the pre-
Content Area: Adult Health, Cardiovascular; Integrated Process: natal clinic; it is not a specific evaluation for the clients high
Teaching and Learning; Cognitive Level: Application; Client blood pressure. Answer 4 is incorrect because obtaining a
Need/Subneed: Health Promotion and Maintenance/Health diet history is not the first action to take for this client. A
and Wellness diet history is appropriate for a client with hyperemesis, not
possible preeclampsia.
31. CORRECT ANSWER: 3. Answer 1 is incorrect because TEST-TAKING TIP: Choose the option that will not delay the
insulin requirements increase as the pregnancy progresses.
evaluation of whether or not this client has preeclampsia. The
Increasing amounts of human placental lactogen cause the
question only states the client has a single high blood pressure,
client to be more insulin resistant, increasing the need for
which does not mean that the client has preeclampsia.
insulin. Answer 2 is incorrect because insulin needs decrease
Content Area: Maternity, Intrapartum; Integrated Process:
during the first trimester, but steadily increase as the pregnancy
Nursing Process, Implementation; Cognitive Level: Analysis;
progresses. Answer 3 is correct because insulin requirements
Client Need/Subneed: Physiological Integrity/Physiological
will increase as the pregnancy progresses. The placenta
Adaptation/Alterations in Body Systems
creates increasing amounts of human placental lactogen,
2164_Ch01_001-058 29/03/12 12:18 PM Page 28

28 chapter 1 Orientation and Pre-Tests

34. CORRECT ANSWER: 4. Answer 1 is incorrect because the Content Area: Child Health, Immunizations; Integrated
PRE-TEST

adolescent would have received the hepatitis B vaccine series in Process: Nursing Process, Implementation; Cognitive Level:
infancy (birth, 1 to 2 months of age, and 6 to 18 months of Application; Client Need/Subneed: Health Promotion and
age). Answer 2 is incorrect because the adolescent would have Maintenance/Immunizations
received the hepatitis A vaccine series in infancy (2 doses 37. CORRECT ANSWER: 1. Answer 1 is correct because the
between 12 and 23 months of age). Answer 3 is incorrect most common cause of chronic pain syndrome is failure
because the adolescent would have received the MMR to control acute pain. Chronic pain syndrome consists of
(measles, mumps, and rubella) vaccine series in infancy/ chronic anxiety and depression, anger, and changed
childhood (12 to 15 months of age and 4 to 6 years of age). lifestyle, all with a variable but significant level of genuine
Answer 4 is correct because the HPV (human papillo-
neurologically based pain. Answer 2 is incorrect because
mavirus) vaccine is given between 11 and 12 years of age. opiates are effective in the treatment of acute pain. Answer 3
This would be the most likely vaccine that the adolescent is incorrect because pain medication should be offered as
has yet to receive. ordered. Some clients will not ask. Answer 4 is incorrect
TEST-TAKING TIP: Focus on the age-relevant vaccine for a because physical dependence is determined when the drug is
12-year-old. Vaccines are not limited to infancy/childhood. discontinued. It is expected in all clients who take opioids
Knowing the vaccine schedule for 7- to 18-year-olds is critical continuously. It is not addiction.
for correctly answering this question. TEST-TAKING TIP: Look for the answer that relieves acute
Content Area: Child Health, Immunizations; Integrated
pain.
Process: Nursing Process, Assessment; Cognitive Level:
Content Area: Adult Health, Pain Control; Integrated Process:
Application; Client Need/Subneed: Health Promotion and Nursing Process, Implementation; Cognitive Level:
Maintenance/Immunizations Comprehension; Client Need/Subneed: Physiological Integrity/
35. CORRECT ANSWER: 4. Answer 1 is incorrect because Pharmacological and Parenteral Therapies/Pharmacological
this may delay treatment, and the interpreter may not con- Pain Management
vey the clients pain as accurately as the client with a nonver- 38. CORRECT ANSWER: 4. Answer 1 is incorrect because
bal pain scale. Answer 2 is incorrect because ethnic/cultural respiratory distress syndrome is associated with preterm birth,
groups may mask the response to pain (e.g., by denial, not related to the age of the client at conception. Answer 2 is
rationalization as coping mechanisms). Answer 3 is incorrect incorrect because there are no specific risk factors based on the
because, depending on the culture, responding to pain may information given that would put her infant at a higher risk
not be acceptable. Answer 4 is correct because pain scales, for pathological jaundice. Pathological jaundice usually occurs
like the Oucher Scale, are accurate with many ethnic with an ABO incompatibility. Answer 3 is incorrect because the
groups and languages. mutation that causes the missing or altered X chromosome
TEST-TAKING TIP: Look for an assessment that is the takes place spontaneously. Theres no evidence that Turners
clients own description, even though it is through a projec- syndrome is hereditary or is caused by any other environmen-
tive device. tal or health factors. Answer 4 is correct because the clients
Content Area: Adult Health, Pain Control; Integrated Process:
age (42) puts her at a higher risk for an infant with Down
Nursing Process, Assessment; Cognitive Level: Application; syndrome.
Client Need/Subneed: Psychosocial Integrity/Cultural
TEST-TAKING TIP: The in-vitro fertilization has nothing to
Diversity do with risk factors that affect an infant. The risk factor is
36. CORRECT ANSWER: 1. Answer 1 is correct because the clients age of 42. Do not let the extraneous information
information is always the best way to reduce parents anxi- distract from the true risk factor.
eties and concerns over vaccines. The information should Content Area: Maternity, Antepartum; Integrated Process:
include the need for each vaccine and what disease each Nursing Process, Analysis; Cognitive Level: Application;
vaccine prevents. Lack of information could needlessly pre- Client Need/Subneed: Health Promotion and Maintenance/
vent an infant from getting protection against life-threaten- Ante/Intra/Postpartum and Newborn Care
ing diseases. Answer 2 is incorrect because, while reassurance 39. CORRECT ANSWER: 2. Answer 1 is incorrect because tac-
of the parents is important, it does not replace the need for tile stimulation can increase the hyperirritability associated with
information. Answer 3 is incorrect because, while respect for narcotic abstinence syndrome. Answer 2 is correct because
the parents decisions is important, it does not replace the need wrapping the newborn snugly in a blanket decreases the
for information. Answer 4 is incorrect because, while involving hyperirritability and increases feeling of security. Answer 3
the parents in minimizing potential adverse effects of the vac- is incorrect because putting the neonate in the car seat can
cine (e.g., administering an age-appropriate dose of acetamino- increase physical and sensory stimulation to the infant. The
phen prior to the vaccine, etc.) is important, it does not replace goal is to decrease these effects. Answer 4 is incorrect because an
the need for information. early stimulation program increases physical and sensory stim-
TEST-TAKING TIP: Think knowledge as the parents first ulation to the infant. The goal is to decrease these effects.
need when concerns about their infant arise.
2164_Ch01_001-058 29/03/12 12:18 PM Page 29

Answers/Rationales/Tips 29
TEST-TAKING TIP: A neonate with narcotic abstinence keeping the child cool will assist in reducing the childs itching,

PRE-TEST
syndrome needs decreased tactile stimulation. Eliminate the it is not as effective as diphenhydramine hydrochloride
three options that increase stimulation. (Benadryl). Answer 3 is incorrect because, while changing the
Content Area: Child Health, Newborn; Integrated Process: linens daily will assist in reducing the childs itching, it is not
Nursing Process, Implementation; Cognitive Level: Analysis; as effective as diphenhydramine hydrochloride (Benadryl).
Client Need/Subneed: Psychosocial Integrity/Chemical and Answer 4 is incorrect because, while keeping the childs finger-
other Dependencies nails short and clean will assist in reducing the childs itching,
it is not as effective as diphenhydramine hydrochloride
40. CORRECT ANSWERS: 2, 4, 5. Answer 1 is incorrect
(Benadryl).
because the client will be allowed some complex carbohy-
TEST-TAKING TIP: Choose the direct, most immediate
drates, but simple carbohydrates (such as sugar, juice, and
suggestion (Benadryl) rather than the indirect, supportive
cold cereal) are not allowed in a gestational diabetic diet.
suggestions (focusing on linens and fingernails). Note that
Answer 2 is correct because keeping the blood sugar well
the mothers concern is reducing itching.
controlled has a direct relationship to decreasing the
Content Area: Child Health, Integumentary; Integrated Process:
risk for macrosomia and hypoglycemia in the neonate.
Nursing Process, Implementation; Cognitive Level: Application;
Answer 3 is incorrect because many clients with gestational
Client Need/Subneed: Physiological Integrity/Physiological
diabetes are diet controlled or controlled with glyburide,
Adaptation/Illness Management
and gestational diabetes is resolved after the pregnancy is
over. All clients with gestational diabetes have a follow-up 43. CORRECT ANSWER: 4. Answer 1 is incorrect because
glucose tolerance test scheduled about 6 weeks after delivery. blood cultures are done before treatment. Answer 2 is incor-
Answer 4 is correct because the diet for gestational dia- rect because there is no indication of cardiac involvement.
betes is based on the grams of carbohydrate allowed at Answer 3 is incorrect because blood glucose is not elevated at
each meal and snack. Answer 5 is correct because clients this time (although diabetes is a risk factor for cellulitis).
with gestational diabetes are required to test their fasting Answer 4 is correct because a low-grade temperature and
blood sugar and 1 to 2 hours postprandial. elevated pulse in an older adult are consistent with infec-
TEST-TAKING TIP: Look for phrases like cant eat any tion. The specific bacteria causing the infection of the
(meaning none) and for the rest of my life; those are both skin and subcutaneous tissue should be identified to
very definitive phrases. Be sure they are totally true before prevent sepsis.
choosing these responses. TEST-TAKING TIP: This is a priority questionmore data
Content Area: Maternity, Antepartum; Integrated Process: are needed to identify the cause of the infection (itis).
Nursing Process, Evaluation; Cognitive Level: Analysis; Client Content Area: Adult Health, Integumentary; Integrated Process:
Need/Subneed: Health Promotion and Maintenance/Ante/ Nursing Process, Analysis; Cognitive Level: Application;
Intra/Postpartum and Newborn Care Client Need/Subneed: Physiological Integrity/Physiological
Adaptation/Alterations in Body Systems
41. CORRECT ANSWER: 1. Answer 1 is correct because the
client must be treated for pain as expressed. The IV drug 44. CORRECT ANSWER: 4. Answer 1 is incorrect because the
use may require that a higher dose be given, but the pres- volume of breast milk produced is related to how often the
ence of pain as expressed by the client should not be breasts are emptied of milk. Formula supplementation decreases
ignored. Answer 2 is incorrect because IV drug use may only breast milk production since the infant nurses less often.
alter the drug effectiveness, not the feeling of pain. Answer 3 is Answer 2 is incorrect because an infant is more efficient at emp-
incorrect because these drugs are not effective for moderate to tying a breast than a breast pump. In addition, oral stimulation
severe pain. Answer 4 is incorrect because nondrug therapies of the nipples by the infant stimulates the release of oxytocin,
alone are not adequate for pain management. which triggers the let-down reflex. Answer 3 is incorrect
TEST-TAKING TIP: Remember: pain is what the client says it because estrogen does not stimulate milk production. Oxytocin
issevere. Three of the options involve other measures, but not and prolactin are the hormones responsible for breast milk pro-
what was ordered. Select this option that is different. duction. Answer 4 is correct because the amount of milk-
Content Area: Adult Health, Pain Control; Integrated producing glandular tissue in all women is approximately
Process: Nursing Process, Implementation; Cognitive Level: the same. The size of large breasts is due to increased fatty
Application; Client Need/Subneed: Physiological Integrity/ tissue.
Pharmacological and Parenteral Therapies/Pharmacological TEST-TAKING TIP: Milk production is directly related to the
Pain Management amount of time the infant is nursing. Any other option will
lead to decreased milk production.
42. CORRECT ANSWER: 1. Answer 1 is correct because Content Area: Maternity, Antepartum; Integrated Process:
diphenhydramine hydrochloride (Benadryl) is a nonpre-
Communication and Documentation; Cognitive Level:
scription antihistamine that will safely and effectively
Application; Client Need/Subneed: Health Promotion and
reduce the childs itching for up to 4 to 6 hours following a
Maintenance/Ante/Intra/Postpartum and Newborn Care
dose of the medication. Answer 2 is incorrect because, while
2164_Ch01_001-058 29/03/12 12:18 PM Page 30

30 chapter 1 Orientation and Pre-Tests

45. CORRECT ANSWER: 3. Answer 1 is incorrect because tap- Content Area: Child Health, Infectious Disease; Integrated
PRE-TEST

ing the umbilical hernia down to the abdomen does not aid in Process: Nursing Process, Analysis; Cognitive Level: Analysis;
resolution, and it can produce skin irritation and breakdown. Client Need/Subneed: Physiological Integrity/Reduction of Risk
This practice should be discouraged. Answer 2 is incorrect Potential/Potential for Complications from Surgical Procedures
because taping the umbilical hernia down to the abdomen does and Health Alterations
not replace a surgical repair of the umbilical hernia; it will not 48. CORRECT ANSWERS: 1, 2, 4, 5. Answer 1 is correct
result in resolution of the umbilical hernia, and it can produce because severe thirst may be secondary to the polyuria from
skin irritation and breakdown. Umbilical hernias that do not the high solute (calcium) load. Answer 2 is correct because
resolve spontaneously by 3 to 5 years of age should be corrected constipation is a common side effect from decreased tone in
surgically on an elective basis. Answer 3 is correct because tap- the bowel. Answer 3 is incorrect because diarrhea is not com-
ing the umbilical hernia down to the abdomen does not aid mon with hypercalcemia until levels above 17 mg/dL, which is
in resolution of the hernia. It can produce skin irritation considered a crisis. Answer 4 is correct because the high calci-
and breakdown. This practice should be discouraged. um increases gastric acid secretion and may intensify gas-
Answer 4 is incorrect because manual reduction of the umbilical
trointestinal manifestation. Anorexia, nausea, and vomiting
hernia should never be attempted by the parents. This proce- are intensified by increased gastric residual volume. Answer
dure should only be done by a pediatrician and/or surgeon. 5 is correct because high calcium levels depress brain func-
TEST-TAKING TIP: Select the one option that implies do not tion, leading to activity. Answer 6 is incorrect because
do the taping of the hernia. hypercalcemia depresses neuromuscular excitability, leading to
Content Area: Child Health, Gastroenterological; Integrated
muscle weakness and lethargy.
Process: Teaching and Learning; Cognitive Level: Application;
TEST-TAKING TIP: Look at pairs of contradictory options
Client Need/Subneed: Psychosocial Integrity/Cultural
(Answers 2 and 3, and Answers 5 and 6) where both cant be
Diversity correct answers. Hypercalcemia depresses functions (e.g., bowels
46. CORRECT ANSWER: 3. Answer 1 is incorrect because and activity).
increasing fluid intake is good general advice for pregnancy, but Content Area: Adult Health, Fluid and Electrolyte Imbalances;
will not resolve the clients concern. Answer 2 is incorrect Integrated Process: Nursing Process, Assessment; Cognitive
because increasing protein intake is a good suggestion for Level: Analysis; Client Need/Subneed: Physiological
clients with gestational diabetes. Answer 3 is correct because Integrity/Physiological Adaptation/Fluid and Electrolyte
leg cramps are caused by low blood calcium levels; increas- Imbalances
ing calcium intake can decrease leg cramps. Answer 4 is 49. CORRECT ANSWER: 2. Answer 1 is incorrect because
incorrect because increasing fiber intake is good general advice chocolate milk is not an irritating fluid. Answer 2 is correct
for pregnancy, but will not resolve the clients concern. because scarlet fever is associated with an inflamed oral cav-
TEST-TAKING TIP: Eliminate the answers that are good ity. Because of this, rough foods and irritating fluids are to
general suggestions for pregnancy, but do not address the be avoided. Citrus juices, such as orange juice, would fur-
specific concerns of the client. ther irritate the inflamed oral cavity. Answer 3 is incorrect
Content Area: Maternity, Antepartum; Integrated Process:
because apple juice is not an irritating fluid. Answer 4 is incor-
Teaching and Learning; Cognitive Level: Application; rect because 2% milk is not an irritating fluid.
Client Need/Subneed: Health Promotion and Maintenance/
TEST-TAKING TIP: Three of the choices are bland in nature
Ante/Intra/Postpartum and Newborn Care and one choice is irritating in nature. Select the choice that is
47. CORRECT ANSWER: 2. Answer 1 is incorrect because a different from the rest of the choices.
fear of bright lights is not associated with measles (rubeola) nor Content Area: Child Health, Infectious Disease; Integrated
is it common in childhood. Answer 2 is correct because an Process: Nursing Process, Implementation; Cognitive Level:
intolerance of light (photophobia) is a common side effect Application; Client Need/Subneed: Physiological Integrity/Basic
of measles (rubeola). Dimming lights in the childs room is Care and Comfort/Nutrition and Oral Hydration
suggested if photophobia is present. Answer 3 is incorrect 50. CORRECT ANSWER: 1. Right lateral recumbent posi-
because, while children with measles (rubeola) frequently rub tioning provides for drainage of oral secretions, which is
their eyes, this leads to eye redness and irritation as opposed to important with clients who are immobile following a
intolerance of light (photophobia). Answer 4 is incorrect stroke. However, a pillow must be placed between the lower
because an enhanced reaction to unanticipated events (such as limbs to avoid redness and skin breakdown.
turning on the overhead light) is not associated with measles TEST-TAKING TIP: Think about what happens when two
(rubeola), nor is it common in childhood. surfaces rub together.
TEST-TAKING TIP: Focus on the two answers that refer to Content Area: Adult Health, Integumentary; Integrated Process:
lights (Answers 1 and 2). Note the word light in the stem Nursing Process, Implementation; Cognitive Level: Application;
and in the two answers with the word light, as opposed to Client Need/Subneed: Physiological Integrity/Reduction of Risk
rubbing (Answer 3) and unanticipated events (Answer 4). Potential/Potential for Alterations in Body Systems
Correlate side effects of the disease with the childs behavior.
2164_Ch01_001-058 29/03/12 12:18 PM Page 31

Answers/Rationales/Tips 31
51. CORRECT ANSWERS: 1, 2, 5. Answer 1 is correct because 54. CORRECT ANSWER: 2. Answer 1 is incorrect because

PRE-TEST
rubella titer is part of the initial prenatal laboratory panel. pulmonary dryness is not the problem. Answer 2 is correct
Answer 2 is correct because the CBC is part of the initial pre- because, with Sjgrens syndrome, lacrimal glands are
natal laboratory panel. Answer 3 is incorrect because the glu- destroyed, resulting in dry eyes (keratoconjunctivitis).
cose tolerance test is not usually part of the initial prenatal panel; Answer 3 is incorrect because dry mouth is a problem. Twelve
it is obtained during the 28th week of pregnancy. Answer 4 is hours is too long. Answer 4 is incorrect because antidepres-
incorrect because the ultrasound is not part of the initial prenatal sants are not a treatment for Sjgrens syndrome.
examination. Answer 5 is correct because blood type and Rh TEST-TAKING TIP: Note that three options focus on after
are part of the initial prenatal laboratory panel. surgery (Answers 1, 3, and 4). Select the option that is before
TEST-TAKING TIP: Use the process of elimination to discard surgery. Review Sjgrens syndromeremember dry eyes and
the nonroutine tests for an initial visit (Answers 3 and 4). mouth.
Content Area: Maternity, Antepartum; Integrated Process: Content Area: Adult Health, Immunological; Integrated
Nursing Process, Implementation; Cognitive Level: Application; Process: Nursing Process, Implementation; Cognitive Level:
Client Need/Subneed: Physiological Integrity/Reduction of Risk Application; Client Need/Subneed: Physiological Integrity/
Potential/Diagnostic Tests Reduction of Risk Potential/Therapeutic Procedures
52. CORRECT ANSWER: 2. Answer 1 is incorrect because milk 55. CORRECT ANSWER: 3. Answer 1 is incorrect because
products, such as ice cream, coat the mouth and throat, caus- eating gelatin would require no chewing on the childs part.
ing the child to clear the throat, which may initiate bleeding Answer 2 is incorrect because eating applesauce would
from the surgical site. Answer 2 is correct because an ice col- require no chewing on the childs part. Answer 3 is correct
lar applied to the throat 30 minutes prior to the meal will because mumps (parotitis) is associated with swelling at
serve as a local anesthetic and reduce the childs oral and the jaw line, followed by an earache that is aggravated by
throat pain, which will assist in eliciting the childs cooper- chewing. A soft, bland diet is suggested, as well as avoid-
ation with eating. Answer 3 is incorrect because gargling is to ing foods that require chewing. An apple would require
be avoided in the immediate postoperative period because it can chewing on the childs part. Answer 4 is incorrect because
start bleeding from the surgical site. Answer 4 is incorrect eating a banana would require little, if any, chewing on the
because it is punitive. It is not the nurses role to threaten or childs part.
punish the child for not eating. It is the nurses role to find a TEST-TAKING TIP: Think soft or no chewing in terms of
way to make eating more comfortable for the child. the childs diet. Relate clinical manifestations of the disease to
TEST-TAKING TIP: Think comfort measures to encourage the childs dietary requirements.
the child to eat. Oral and throat pain following a tonsillectomy Content Area: Child Health, Infectious Disease; Integrated
will prevent a child from eating. Eliminate answers that Process: Nursing Process, Implementation; Cognitive
promise or threaten (Answers 1 and 4). Level: Application; Client Need/Subneed: Physiological
Content Area: Child Health, Pediatric Surgery; Integrated Integrity/Basic Care and Comfort/Nutrition and Oral
Process: Nursing Process, Implementation; Cognitive Level: Hydration
Application; Client Need/Subneed: Physiological Integrity/
56. CORRECT ANSWERS: 1, 3, 4, 6. Answer 1 is correct
Basic Care and Comfort/Non-pharmacological Comfort
because a transfusion reaction can lead to anaphylactic
Interventions
shock and life-threatening vasodilation and hypotension.
53. CORRECT ANSWERS: 2, 3, 4. Answer 1 is incorrect Answer 2 is incorrect because treating pain or a fever is not
because pregnancy-induced hypertension is not related to the priority. Answer 3 is correct because the client should
uterine atony or other causes of postpartum hemorrhage. not receive any more blood, which is the source of the
Answer 2 is correct because a macrosomatic (very large) antigen that is causing the allergic response. Answer 4 is
baby can predispose a woman to uterine atony, and thus correct because a patent IV may be needed for emergency
to hemorrhage. Answer 3 is correct because retained pla- treatment. Answer 5 is incorrect because the blood needs to
cental fragments are a cause of postpartum hemorrhage. be returned to the laboratory, not disposed of. Answer 6 is
Answer 4 is correct because a full urinary bladder may correct because the reason for the allergic reaction needs
result in uterine atony, and thus hemorrhage. Answer 5 is to be determined.
incorrect because maternal age is not a risk factor for hemor- TEST-TAKING TIP: Note that Answers 5 and 6 are contradic-
rhage. Increased parity is related to an increased risk of hem- tory. Select Answer 6keep rather than dispose of the blood
orrhage. product. With each option, ask if the statement is a true or false
TEST-TAKING TIP: The stem asks for conditions that are action with a transfusion reaction.
associated with a risk for hemorrhage. Content Area: Adult Health, Hematological; Integrated Process:
Content Area: Maternity, Postpartum; Integrated Process: Nursing Process, Implementation; Cognitive Level: Analysis;
Nursing Process, Analysis; Cognitive Level: Application; Client Need/Subneed: Physiological Integrity/Pharmacological
Client Need/Subneed: Physiological Integrity/Physiological and Parenteral Therapies/Blood and Blood Products
Adaptation/Pathophysiology
2164_Ch01_001-058 29/03/12 12:18 PM Page 32

32 chapter 1 Orientation and Pre-Tests

57. CORRECT ANSWER: 2. Answer 1 is incorrect because technique for the active phase of the first stage of labor
PRE-TEST

mastitis is an infection of the breast that occurs postpartum, when the client is trying to relieve the pain of labor.
but is unrelated to uterine cramping. Answer 2 is correct Using patterned breathing in this phase of labor can lead
because breastfeeding causes the release of prolactin and to bearing down before complete dilation and hyperventila-
oxytocin. Oxytocin is a hormone that causes uterine con- tion. Answer 3 is incorrect because this client is not
tractions. The release of oxytocin while breastfeeding completely dilated and it is inappropriate to bear down
speeds uterine involution. This may also cause cramps with contractions. Pushing before being completely dilated
while breastfeeding in the postpartum period. Cramping can cause cervical lacerations and edema of the cervix.
is more severe in clients who are multiparous. Answer 3 is Answer 4 is correct because this client is in the transition
incorrect because uterine atony is the failure of the uterine phase of the first stage of labor. The proper breathing
muscles to contract normally after the baby and placenta are technique is shallow chest breathing, which prevents
delivered; there is no cramping with uterine atony. Answer 4 is hyperventilation and bearing down before she is com-
incorrect because postpartum endometritis is an infection of pletely dilated.
the endometrium or decidua, extending into the myometri- TEST-TAKING TIP: Remember the stages and phases of
um and parametrial tissues, which is the most common labor and how the client would respond to latent, active,
cause of postpartum fever. It does not cause cramping during and transition phases of labor. Knowing what could be the
breastfeeding. clients expected response will guide appropriate coaching
TEST-TAKING TIP: Cramping is a sign of contracting of the for breathing.
uterus, not uterine atony, which is the opposite (a lack of Content Area: Maternity, Intrapartum; Integrated Process:
cramping). Therefore, eliminate that option. Nursing Process, Implementation; Cognitive Level:
Content Area: Maternity, Postpartum; Integrated Process: Application; Client Need/Subneed: Physiological Integrity/
Nursing Process, Analysis; Cognitive Level: Application; Client Basic Care and Comfort/Non-pharmacological Comfort
Need/Subneed: Health Promotion and Maintenance/Ante/ Interventions
Intra/Postpartum and Newborn Care 60. CORRECT ANSWER: 4. Answer 1 is incorrect because
58. CORRECT ANSWER: 4. Answer 1 is incorrect because German measles (rubella) has a teratogenic effect on fetuses.
it is a desirable behavior by the parents. Confining cigarette At 25 years of age, pregnancy could be a concern for this
smoke to the outdoors can prevent paroxysms of coughing nurse. Answer 2 is incorrect because German measles
associated with whooping cough (pertussis). Answer 2 is (rubella) has a teratogenic effect on fetuses. At 32 years of
incorrect because it is a desirable behavior by the parents. age, pregnancy could be a concern for this nurse. Answer 3
Avoiding sudden changes in household temperature can is incorrect because the nurse is a new graduate, and it may
prevent paroxysms of coughing associated with whooping be unlikely that she had experience in caring for a child
cough (pertussis). Answer 3 is incorrect because it is a desir- with German measles (rubella). However, at 45 years of
able behavior by the parents. Keeping the childs room well age, pregnancy would be an unlikely concern. Answer 4 is
ventilated can prevent paroxysms of coughing associated correct because German measles (rubella) has a terato-
with whooping cough (pertussis). Answer 4 is correct genic effect on fetuses. At 50 years of age, this likely
because using the fireplace as an additional means of would not be a concern. And, the nurses 25 years of
heating the house creates dust and smoke that may cause pediatric nursing experience would be a plus in caring
paroxysms of coughing associated with whooping cough for the child.
(pertussis). TEST-TAKING TIP: The clue is in the age of the nurse.
TEST-TAKING TIP: The question asks for the answer that is Correlate the most severe side effect of this disease (teratogenic)
not a correct action by the parents. Think triggers that pro- with the childbearing ages of the nurses.
mote coughing and select the behavior by the parents that may Content Area: Child Health, Infectious Disease; Integrated
cause paroxysms of coughing. Process: Nursing Process, Analysis; Cognitive Level: Analysis;
Content Area: Child Health, Infectious Disease; Integrated Client Need/Subneed: Safe and Effective Care Environment/
Process: Nursing Process, Evaluation; Cognitive Level: Management of Care/Delegation
Application; Client Need/Subneed: Physiological Integrity/ 61. CORRECT ANSWER: 3. Answer 1 is incorrect
Physiological Adaptation/Alterations in Body Systems because decorticate posturing may result from painful stimu-
59. CORRECT ANSWER: 4. Answer 1 is incorrect because lation. Answer 2 is incorrect because decerebrate posturing,
deep breathing is the correct technique for the latent phase a poor prognostic sign, may occur with painful stimuli.
of the first stage of labor, when the client is trying to Answer 3 is correct because the lack of response to
relax and allow labor to progress. Taking deep breaths in noxious stimuli signals a deepening coma. Answer 4 is
this phase of labor can lead to hyperventilation. Answer 2 incorrect because the Babinski reflex is not normally present
is incorrect because patterned breathing is the correct in adults. If present, it is a pathologic response.
2164_Ch01_001-058 29/03/12 12:18 PM Page 33

Answers/Rationales/Tips 33
TEST-TAKING TIP: Remember that a response to pain is 64. CORRECT ANSWERS: 4, 5, 6. Answer 1 is incorrect

PRE-TEST
desired. because this change is characteristic of hypovolemic
Content Area: Adult Health, Neurological; Integrated Process: shock. Answer 2 is incorrect because respirations would
Nursing Process, Analysis; Cognitive Level: Application; slow with herniation. Rapid respirations would be
Client Need/Subneed: Physiological Integrity/Physiological characteristic of hypovolemic shock. Answer 3 is incorrect
Adaptation/Alterations in Body Systems because blood pressure increases, not decreases. Answer 4 is
62. CORRECT ANSWER: 4. Answer 1 is incorrect correct because bradycardia is one of the vital sign
because the nurse has the necessary knowledge base to changes with Cushings triad, a grave sign, indicating
correctly answer the parents question without referring to herniation of the brainstem and occlusion of the cerebral
the pediatrician. Answer 2 is incorrect because the nurse blood flow if treatment is not initiated. Answer 5 is
would not want to answer the parents question with a correct because hypertension occurs with brainstem
closed-ended response. It does not provide the parent with herniation initially (Cushings triad)a widening pulse
needed information and tends to abruptly terminate any pressure (Cushings reflex) occurs earlier in an attempt
communication between the nurse and the parent. to overcome the increased intracranial pressure (ICP).
Answer 6 is correct because bradypnea accompanies
Answer 3 is incorrect because, while a healthy child is a
better candidate for surgery than a sick child, this is a bradycardia and hypertension (Cushings triad) with
general response rather than a specific response to this par- brainstem herniation.
ents question. Answer 4 is the correct answer because TEST-TAKING TIP: Recognize a widening of the pulse
acute inflammation and infection (itis) of tonsillar tis- pressure with Cushings reflex versus hypertension with
sue increase the risk of bleeding at the time of surgery. Cushings triad.
Content Area: Adult Health, Neurological; Integrated Process:
The operative site for a tonsillectomy is highly vascular,
with the risk of postoperative bleeding already a concern. Nursing Process, Analysis; Cognitive Level: Application; Client
Need/Subneed: Physiological Integrity/Physiological
An active inflammation and infection would increase
this concern. Adaptation/Pathophysiology
TEST-TAKING TIP: The stem in this question asks for the 65. CORRECT ANSWER: 1. Answer 1 is correct because
most accurate response. The most specific response is the most primary prophylaxis (the infusion of factor VIII concen-
accurate response. Choose the specific pathophysiology-based trate on a regularly scheduled basis before the onset of
answer. joint damage) has been proven to be effective in prevent-
Content Area: Child Health, Pediatric Surgery; Integrated ing arthropathy. Answer 2 is incorrect because primary
Process: Nursing Process, Implementation; Cognitive Level: prophylaxis (the infusion of factor VIII concentrate on a
Application; Client Need/Subneed: Physiological Integrity/ regularly scheduled basis before the onset of joint damage)
Reduction of Risk Potential/Potential for Complications from must be given prior to the development of arthropathy.
Surgical Procedures and Health Alterations Secondary, not primary, prophylaxis involves the infusion
63. CORRECT ANSWER: 3. Answer 1 is incorrect
of factor VIII concentrate on a regular basis after the
because the clinical manifestations of weakness in child experiences a first joint bleed. Answer 3 is incorrect
Cushings syndrome will interfere with normal activities. because primary prophylaxis (the infusion of factor VIII
There will be mood alterations as a result of the hormone concentrate on a regularly scheduled basis before the onset
imbalance. Answer 2 is incorrect because continuing the of joint damage) is not associated with special circumstances,
activity will put the client at risk for injury (e.g., fractures). such as surgical procedures, which demand additional
Answer 3 is correct because the overproduction of
factor VIII concentrate at unscheduled times. Answer 4 is
adrenocortical hormone results in excessive protein incorrect because, while primary prophylaxis is very
breakdown, causing compression fractures and costly, organizations such as the National Hemophilia
weakness. Answer 4 is incorrect because the risk for Foundation exist to assist in providing resources for this
falls is greatest because of weakness, not visual problems, therapy.
although cataracts and glaucoma may result from Cushings TEST-TAKING TIP: Think of primary prophylaxis as being
syndrome. before the problem occurs, and secondary prophylaxis as being
TEST-TAKING TIP: The 3 Ss are signs of Cushings after the problem occurs. Note that two answers focus on
syndrometoo much sugar, salt, and sex hormone, which after (Answers 2 and 3) and one answer refers to before
weaken the client. (Answer 1).
Content Area: Child Health, Hematological; Integrated Process:
Content Area: Adult Health, Endocrine; Integrated Process:
Teaching and Learning; Cognitive Level: Application; Client Teaching and Learning; Cognitive Level: Application; Client
Need/Subneed: Safe and Effective Care Environment/Safety
Need/Subneed: Physiological Integrity/Physiological
Adaptation/Pathophysiology and Infection Control/Accident Prevention and Injury
Prevention
2164_Ch01_001-058 29/03/12 12:18 PM Page 34

34 chapter 1 Orientation and Pre-Tests

66. CORRECT ANSWER: 1. Answer 1 is correct because a the tricuspid area. Although the murmur associated with
PRE-TEST

normal prothrombin time (PT) in a child is 11 to mitral regurgitation would be audible in this area, it would
15 seconds. A value of 14.1 seconds is within normal be best heard at the apex. Answer 3 is incorrect because it is
limits and the child is a safe candidate for the surgery. the pulmonic area: the second intercostal space, left sternal
Because the tonsillar operative site is highly vascular, border. Although the murmur associated with mitral regurgi-
bleeding is a concern and coagulation studies are always tation may be audible throughout the precordium, pulmonic
conducted prior to the surgery. Since other health-care and aortic valve dysfunction would be best heard in this
personnel (e.g., the pediatric surgeon, operating room area. Answer 4 is incorrect because it is the aortic area: the
surgical team, etc.) will also need this information before second intercostal space, right sternal border. Although the
proceeding with the surgery, the nurses first action murmur associated with mitral regurgitation may be audible
should be to note the laboratory result in the childs throughout the precordium, aortic valve dysfunction would
chart. Answer 2 is incorrect because the pediatric surgeon be best heard in this area.
will routinely review the childs chart prior to the surgery, TEST-TAKING TIP: Key words in the question stem are: best
and will specifically look for the results of the prothrombin place and mitral.
time (PT) before starting the surgery. There is no need for Content Area: Adult Health, Cardiovascular; Integrated Process:
the nurse to notify the pediatric surgeon at this time because Nursing Process, Implementation; Cognitive Level: Application;
the prothrombin time is within normal limits. Answer 3 is Client Need/Subneed: Physiological Integrity/Reduction of Risk
incorrect because the prothrombin time (PT) is within nor- Potential/System Specific Assessments
mal limits and the surgery can take place as planned. If the 69. CORRECT ANSWER: 1. Answer 1 is correct because
laboratory results were not within normal limits, the nurse resistance or noncompliance is often related to different
would call the operating room and notify them. But, it is priorities or goals. Answer 2 is incorrect because there is no
not the nurses role to cancel the surgery. It is the pediatric evidence to support this choice. Answer 3 is incorrect because
surgeons responsibility to cancel the surgery if needed. chronological age is not the determinant of rehabilitation
Answer 4 is incorrect because, while it is safe to release the
potential. Answer 4 is incorrect because rehabilitation plans
child to the operating room transporter, the nurse should may actually begin before surgery. It is unnecessary to wait for
only do so after noting the results of the prothrombin time the client to feel comfortable changing the dressing.
(PT) in the childs chart. TEST-TAKING TIP: Choose the option that requires further
TEST-TAKING TIP: The stem asks for a priority nursing assessment of the situationa review of the goals.
action. Eliminate the two options that imply that the laboratory Content Area: Geriatrics, Musculoskeletal; Integrated Process:
result is a problem (Answers 2 and 3). Nursing Process, Analysis; Cognitive Level: Application;
Content Area: Child Health, Pediatric Surgery; Integrated
Client Need/Subneed: Psychosocial Integrity/Coping
Process: Nursing Process, Analysis; Cognitive Level:
Mechanisms
Application; Client Need/Subneed: Physiological Integrity/
Reduction of Risk Potential/Laboratory Values 70. CORRECT ANSWER: 2. Answer 1 is incorrect because a
sterile pressure dressing is not necessary for the IV site.
67. CORRECT ANSWERS: 3, 5. Answer 1 is incorrect because
It would also be unnecessarily restrictive for the child.
hyperglycemia would not be expected with adrenal hypofunc- Answer 2 is correct because a child with a low platelet
tion. Answer 2 is incorrect because these findings would be count will have bleeding tendencies. When discontinuing
seen with Cushings syndrome, not addisonian crisis. Answer 3 the IV, the nurse should treat the IV site as an arterial
is correct because addisonian crisis is characterized by puncture and apply direct pressure to the site for a mini-
cyanosis and signs of circulatory collapse. Answer 4 is mum of 5 minutes. Answer 3 is incorrect because sending
incorrect because this describes Addisons disease, not addison- the tip of the IV catheter to the laboratory for a culture
ian crisis. Answer 5 is correct because circulatory collapse would only be necessary when sepsis around the IV catheter
occurs with addisonian crisis. is suspected. It would also be unnecessarily expensive and
TEST-TAKING TIP: Look for the findings that describe the would not help the childs bleeding tendencies. Answer 4 is
worst clinical picturecirculatory collapse. incorrect because it would be unnecessarily restrictive for the
Content Area: Adult Health, Endocrine; Integrated Process:
child and would not help the childs bleeding tendencies.
Nursing Process, Analysis; Cognitive Level: Analysis; Client TEST-TAKING TIP: Remember that a decreased platelet count
Need/Subneed: Physiological Integrity/Physiological
will cause bleeding tendencies.
Adaptation/Medical Emergencies Content Area: Child Health, Hematological; Integrated Process:
68. CORRECT ANSWER: 1. Answer 1 is correct because the Nursing Process, Implementation; Cognitive Level: Application;
murmur associated with mitral regurgitation would be Client Need/Subneed: Physiological Integrity/Reduction of Risk
best heard at the apex, which is the 5th intercostal space, Potential/Potential for Complications of Diagnostic Tests/
left midclavicular line. Answer 2 is incorrect because this is Treatments/Procedures
2164_Ch01_001-058 29/03/12 12:18 PM Page 35

Answers/Rationales/Tips 35
71. CORRECT ANSWERS: 1, 2, 4, 5. Answer 1 is correct Content Area: Child Health, Cardiovascular; Integrated Process:

PRE-TEST
because this position would not promote the drainage of Nursing Process, Implementation; Cognitive Level: Analysis;
cranial fluids that have accumulated due to increased Client Need/Subneed: Physiological Integrity/Pharmacological
intracranial pressure. In addition, placing the child on and Parenteral Therapies/Dosage Calculation
the abdomen (prone) could prevent an adequate airway 74. CORRECT ANSWER: 4. Answer 1 is incorrect because
from being maintained. Answer 2 is correct because this there may be improvement in the clients perception of pain;
position would not promote the drainage of cranial fluids however, the concern is about oversedation by the drug.
that have accumulated due to increased intracranial pres- There are sedation scales available that do help the nurse
sure. In addition, placing the child on the back (supine) identify the onset of sedation in a client. Answer 2 is incor-
could increase the possibility of aspiration. Answer 3 is rect because the client may be receiving supplemental oxy-
incorrect because this position would promote the drainage gen. Answer 3 is incorrect because a client who is oversedat-
of cranial fluids that have accumulated due to increased ed can be awakened and will respond to questions appropri-
intracranial pressure. Semi-Fowlers is a position in which ately. Answer 4 is correct because opioids will depress ven-
the child lies on the back with the trunk elevated at approxi- tilation, causing a buildup of carbon dioxide in the
mately a 45-degree angle. Answer 4 is correct because this blood. If hypoventilation is present, measuring the level
position would not promote the drainage of cranial fluids of carbon dioxide is the most reliable indication of possi-
that have accumulated due to increased intracranial pres- ble respiratory depression. Capnography measures
sure. (However, right side-lying would not prevent an breath-by-breath the level of CO2 and is most often used
adequate airway from being maintained or increase the during anesthesia, in critical care, and in emergency med-
possibility of aspiration.) Answer 5 is correct because this icine to prevent respiratory depression. If capnography is
position would not promote the drainage of cranial fluids not available with the client who is on PCA, monitoring
that have accumulated due to increased intracranial pres- the clients respiratory rate and depth and assessing seda-
sure. (However, left side-lying would not prevent an ade- tion levels are essential.
quate airway from being maintained or increase the pos- TEST-TAKING TIP: This question is asking for the most
sibility of aspiration.) reliable indication. If a client is oversedated, ventilation will
TEST-TAKING TIP: Gravity will assist in the drainage of be affected. Hypoventilation affects CO2.
fluids; select the positions that would prevent or are least likely Content Area: Adult Health, Pain Control; Integrated Process:
to accomplish this. Nursing Process, Assessment; Cognitive Level: Application;
Content Area: Child Health, Neurological; Integrated Process:
Client Need/Subneed: Physiological Integrity/Pharmacological
Nursing Process, Implementation; Cognitive Level: Application; and Parenteral Therapies/Pharmacological Pain Management
Client Need/Subneed: Safe and Effective Care Environment/
Safety and Infection Control/Accident Prevention and Injury 75. CORRECT ANSWER: 3. Answer 1 is incorrect
Prevention because the trachea is larger and a partial obstruction
would most likely produce a crowing sound. Answer 2 is
72. CORRECT ANSWER: 2. Answer 1 is incorrect because the incorrect because the wheezing was heard in the left lung.
client cannot be forced to take an oral medication. Answer 2 is Answer 3, left bronchus, is correct because wheezes
correct because the behavior may be a pattern. Family may occur when there is partial obstruction of the bronchi or
have suggestions that will help. Answer 3 is incorrect because bronchioles. The sound is due to increased vibration of
restraints should be a last resort if at all possible. The route may air molecules as they pass over the area that is narrowed
need to be changed. Answer 4 is incorrect because the behavior or partially obstructed. In this client, the wheezing was
may be the result of the brain injury and may not change. heard in the left lung. Answer 4 is incorrect because the
TEST-TAKING TIP: Choose an option that recognizes the effect sound produced by the pleura would be a grating sound
of the brain attack on behavior. Select the option for further called a friction rub. Answer 5 is incorrect because the sound
assessment (determine personality before . . .) in an attempt produced by the pleura would be a grating sound called a
to understand the behaviors. friction rub.
Content Area: Adult Health, Cardiovascular; Integrated Process:
TEST-TAKING TIP: The question is looking for an area of the
Nursing Process, Implementation; Cognitive Level: Application; lung that, when narrowed, would produce a sound. Think
Client Need/Subneed: Psychosocial Integrity/Behavioral
squeeze with a wheeze as the air moves through a narrow
Interventions tubethe bronchus or bronchioles.
73. CORRECT ANSWER: 1.4. Dose desired (0.07 mg) is to Content Area: Adult Health, Respiratory; Integrated Process:
dose on hand (0.05 mg) as amount desired (x) is to amount on Nursing Process, Assessment; Cognitive Level: Application;
hand (1 mL): 0.07 mg/0.05 mg = x/1 mL; x = 1.4 mL. Client Need/Subneed: Physiological Integrity/Reduction of Risk
TEST-TAKING TIP: Know the standard ratio and proportion Potential/System Specific Assessments
calculations and be careful with decimal points.
2164_Ch01_001-058 29/03/12 12:18 PM Page 36

36 chapter 1 Orientation and Pre-Tests

Pre-Test 2 3. Give prophylactic antibiotics.


PRE-TEST

4. Place on contact isolation.


Questions 7. During the active phase of labor, a client, who is a
Select the one answer that is best for each question, unless gravida 2, para 0, begins to complain of dizziness,
otherwise directed. tingling in her fingers, and numbness in her lips. The
most appropriate nursing intervention for this client
1. The parents of a 2-year-old child ask a nurse how to best would be to:
manage their childs temper tantrums. What advice should 1. Have her breathe into a paper bag.
the nurse give the parents? 2. Help her assume a left lateral position in bed.
1. The frequency of tantrums can be reduced with a 3. Instruct her in shallow chest breathing.
low-sugar diet. 4. Notify the physician immediately.
2. Tantrums may be prevented by alerting the child in
advance about a change in activities. 8. A client has been receiving warfarin (Coumadin) and
3. If the child has a tantrum, it is best to hold the child heparin for 2 days following a total knee arthroplasty.
until he/she is calmed. Laboratory results show that INR is 1.2, Hct is 34%. A
4. Spanking is an effective way to eliminate tantrums. nurse should:
1. Hold the Coumadin, continue the heparin.
2. A nurse prepares to administer cardiopulmonary resuscita- 2. Hold the heparin, continue the Coumadin.
tion (CPR). Which actions should the nurse take? Place in 3. Hold both drugs and call the physician.
sequential order. 4. Continue both drugs as ordered.
1. Begin chest compressions.
2. Open airway. 9. Which assessment finding should a nurse most expect in
3. Call for help. a newborn infant with a cyanotic heart defect?
4. Check for responsiveness and breathing. 1. Clubbing of fingers and toes.
5. Position flat on back. 2. Tachycardia.
6. Check pulse. 3. Polycythemia.
7. Give two breaths. 4. Distended neck veins.

3. A client, who is a 24-year-old gravida 6, para 5, delivered 10. A client will receive a spinal anesthetic for cesarean
a 10-pound, 10-ounce baby 6 hours ago, after a 26-hour birth. As the client discusses the surgery, she asks an
labor. Upon assessment, an RN finds the clients fundus RN about the possibility of postanesthesia headaches.
is 2 fingerbreadths above the umbilicus and slightly devi- Which is the most appropriate response by the RN?
ated to the right. Based on the information provided, for 1. If there is an allergy to the agent used, headaches may
which complication is this client at risk? occur.
1. Postpartum hemorrhage. 2. Headaches should not be a problem with this type of
2. Mastitis. anesthesia.
3. Endometriosis. 3. This problem may be prevented by keeping the head
4. Thrombophlebitis. of the bed flat.
4. Medication is available to counteract headaches and is
4. A client is seen in an emergency department for rotator given as needed.
cuff tendonitis. What is the correct immediate treatment?
1. Hot packs prn. 11. A 50-year-old client, who is overweight, is tested at a
2. Muscle relaxant. health fair and the result of a nonfasting blood sugar is
3. A sling for the arm. 210 mg/dL. A nurse should:
4. ROM every 4 hours. 1. Encourage re-testing in 1 hour.
2. Encourage re-testing in 1 week.
5. A 3-year-old child is admitted to an emergency department 3. Tell the client that follow-up is needed later in the
after being struck by an automobile. A nurses assessment week.
reveals a child with spontaneous eye opening. The child is 4. Tell the client that the results are normal.
confused and does not obey commands but is able to localize
pain. The nurse correctly calculates a score of __________ 12. A clinic nurse prepares to administer an IM injection to a
on the Glasgow Coma Scale. Fill in the blank. school-age child. Which pain management technique
would be best suited for this child and should be applied
6. The correct nursing action to prevent infection in a during the procedure?
client with an external fixation device for an open 1. Guided imagery.
fracture is to: 2. Being held by a parent.
1. Double-glove for dressing changes. 3. Distraction.
2. Scrub pin sites with Betadine. 4. Providing oral sucrose.
2164_Ch01_001-058 29/03/12 12:18 PM Page 37

Pre-Test 2 37
13. What advice should a nurse give to a person with type 3. Rash on face, trunk, and limbs.

PRE-TEST
1 diabetes who is beginning a running program in an 4. White and red spots on buccal mucosa.
effort to get in shape? 19. Health teaching for a client who is newly diagnosed with
1. Administer insulin 30 minutes before running. diabetes should include:
2. Rotate injection sites from one thigh to the other 1. Notify health care provider immediately if the blood
each day. sugar is over 150 mg/dL.
3. Carry a sugar source. 2. Call provider if breath smells fruity.
4. Avoid running. 3. Hold insulin if unable to eat.
14. How should a nurse caring for an elderly client with 4. Check blood sugar every hour if sick.
receptive and expressive aphasia communicate with the 20. A client, who is gravida 3, para 1, is 6 cm dilated,
client? Select all that apply. 80% effaced, and a 0 station. The external fetal
1. Speak loudly. monitor shows baseline heart rate of 145 bpm, mod-
2. Use a picture board or flash cards. erate variability, and variable decelerations. What are
3. Use hands to communicate. appropriate nursing interventions for this client?
4. Speak slowly. Select all that apply.
5. Encourage the client to initiate meaningful 1. Perform a sterile vaginal exam.
communication. 2. Change the clients position.
6. Provide support and encouragement. 3. Prepare the client for an emergency cesarean
15. Which client has the best control of diabetes? delivery.
1. A client who has trouble sticking to diet and exercising 4. Administer a fluid bolus.
but takes meds regularly. FBS is 132 mg/dL, HgbA1c 5. Prepare for a vacuum-assisted delivery.
is 11%. 21. Following a nephrectomy, which complication does a
2. A client who has perfect compliance with diet, exercise, client have the greatest risk of developing?
and meds. FBS is 96 mg/dL, HgbA1c is 11%. 1. Altered nutrition related to NPO status and
3. A client whose diabetes is controlled with diet alone. postoperative ileus.
FBS is 452 mg/dL, HgbA1c is 12%. 2. Possible kidney stones related to urinary stasis.
4. A client who has trouble remembering meds, but good 3. Pedal and periorbital edema related to nephrotic
compliance with diet and exercise. FBS is 110 mg/dL, syndrome.
HgbA1c is 7%. 4. Possible altered gas exchange related to shallow breath-
16. A client, who is 17 years old, has delivered a 6-pound, ing from pain.
6-ounce girl at 38 weeks gestation 3 hours ago. A nurse 22. An infant is admitted for pyloric stenosis. Which symp-
is completing a routine postpartum assessment. Which toms should a nurse expect to find? Select all that apply.
assessments should the nurse report to a physician 1. Metabolic alkalosis.
immediately? Select all that apply. 2. Olive-shaped mass palpable in the upper right
1. Positive Homans sign. quadrant of abdomen.
2. 2+ deep tendon reflexes. 3. History of projectile vomiting.
3. Moderate rubra lochia. 4. Bile-stained vomitus.
4. Expression of orange-sized clots during fundal 5. Decreased urine specific gravity.
massage. 6. Small, infrequent stools.
5. Fundus palpable firm at the level of the umbilicus.
23. A postpartum nurse instructed a client on proper perineal
17. If a clients blood sugar is lowered too rapidly, for what care after delivery of a healthy neonate vaginally after a
manifestation or dangerous outcome should a nurse midline episiotomy. Which client activities should
observe? indicate to the nurse that the client has understood the
1. Glycosuria. instructions? Select all that apply.
2. Hyperkalemia. 1. The client changes her peri pads every 8 hours.
3. Hypertensive crisis. 2. The client consistently uses the peri bottle to clean the
4. Confusion from cerebral edema. perineum after each void or bowel movement.
18. A nurse assesses a child diagnosed with measles. Which 3. The client wipes from back to front after each bowel
symptom is the nurse least likely to observe in a child movement.
diagnosed with measles? 4. The client sprays water from the peri bottle directly
1. Slapped cheek appearance. into the vagina.
2. Fever and general malaise. 5. The client uses the antiseptic spray after rinsing with
the peri bottle.
2164_Ch01_001-058 29/03/12 12:18 PM Page 38

38 chapter 1 Orientation and Pre-Tests

24. Older adults often present with atypical assessment find- 29. Suddenly, during a labor contraction, a pregnant client
PRE-TEST

ings. Which symptom will an older adult with a urinary experiences a gush of amniotic fluid. Which assessment
tract infection (UTI) most often develop first? finding should indicate to a nurse that the client is
1. Anorexia. experiencing a deviation from normal labor patterns?
2. Confusion. 1. Colored amniotic fluid.
3. Fever. 2. Fetal bradycardia with contractions.
4. Restlessness. 3. The client complains of nausea and vomits.
4. The client becomes diaphoretic and irritable.
25. While visiting a local park, a nurse witnesses the
collapse of a preschool-age child. The nurse assesses 30. A nurse auscultates the lungs of a client diagnosed with
the child and finds no pulse and no respirations. The right middle lobe pneumonia. Where should the nurse
nurse begins compressions. At what rate per minute place the stethoscope where crackles would most likely be
should the nurse deliver the compressions? heard?
1. 25 compressions per minute.
2. 50 compressions per minute.
3. 75 compressions per minute.
4. 100 compressions per minute.
26. A client reports sharp pain in the calf of the left leg
when walking to the bathroom. A nurse notes that B
the leg has a confined area of warmth, tenderness, and
redness. Which nursing action is most appropriate for
this client?
1. Encouraging the client to ambulate to increase A
C
circulation.
2. Instructing the client to massage the affected area to D
relieve the tenderness.
3. Applying cold packs to the affected area to decrease
inflammation.
4. Elevating the affected extremity to promote venous
blood flow.
Midline
27. A graduate nurse has been working on a medical unit
for 3 months. The nurse manager has posted the team
leader assignments for the following week. The graduate 1. A.
nurse knows that the major responsibility of the team 2. B.
leader is to: 3. C.
Select all that apply. 4. D.
1. Know the condition of the clients on the team. 31. A nurse provides health teaching to the parents of a
2. Prioritize the needs of the clients on the team. school-age child diagnosed with attention deficit-
3. Supervise the direct care of the nursing assistants. hyperactivity disorder (ADHD). What should the nurse
4. Document the assessments completed by the team include in the teaching plan?
members. 1. One parent should be responsible for discipline.
5. Make the team assignments for the clients care. 2. Prescription bottles should be stored where the child
6. Provide care to the clients who are most acutely ill on can access for self-care.
the team. 3. The homes televisions and radios should be kept at a
28. A woman has chosen a patch applied to her skin as her low to moderate volume.
method to deliver contraceptive medication. A nurse 4. Participation in outdoor sports should be avoided.
should instruct the client to call the clinic immediately if 5. Medications should be taken daily only as needed.
the client experiences: 32. A client with a 6-month history of burning pain and
Select all that apply. weakness to the thumb and first and middle fingers has
1. Mild to moderate nausea during the day following the a carpal tunnel release done with regional anesthesia. In
application of the patch. the PACU, the priority nursing assessment should be:
2. Chloasma. 1. Orientation to person, place, time, situation.
3. Pain in the legs while walking. 2. Brachial pulse.
4. Headache unrelieved by acetaminophen. 3. Color, temperature, movement of the fingers.
5. Swelling of the feet and hands. 4. Pupil reaction to light.
2164_Ch01_001-058 29/03/12 12:18 PM Page 39

Pre-Test 2 39
33. A client, who is a primigravida, is 6 cm dilated, 100% 3. Consult your pediatrician as to why the vaccine was

PRE-TEST
effaced, and a 3 station. She suddenly has a gush of not given.
fluid. Which nursing assessments indicate a complica- 4. This disease is not communicable; therefore, a
tion and should be reported to a physician? Select all vaccine is not necessary.
that apply. 39. The K+ level of a client in acute renal failure is 6.8 mEq/L.
1. Green-tinged fluid. Which treatments will decrease the potassium level? Select
2. Copious amount of vernix in clear fluid. all that apply.
3. Decrease in the fetal heart rate baseline from 140 to 1. Rapid infusion of 50% dextrose.
100 bpm. 2. Sodium bicarbonate IV.
4. Increase in the fetal heart rate baseline to 170 bpm. 3. Regular insulin in D5W.
5. Small amount of clear fluid. 4. Kayexalate retention enema.
34. Which conditions would most likely contribute to an 5. Albuterol sulfate (Ventolin) by nebulizer.
increase in a clients need for oxygen (O2)? Select all that 40. A client, who is 38 years old, gravida 5, para 0, with type 1
apply. diabetes, is 13 weeks pregnant. Her hemoglobin A1c level is
1. Burns over trunk and legs. 8.2%. She is attending a prenatal class for clients with high-
2. Hypothermia from environmental exposure. risk pregnancies. Which possible complications should a
3. Shivering secondary to an elevated temperature. nurse anticipate in this client? Select all that apply.
4. Agitation associated with anxiety. 1. Oligohydramnios.
5. Neuromuscular blockage during anesthesia. 2. Polyhydramnios.
6. Thyrotoxicosis secondary to Graves disease. 3. Macrosomia.
35. A nurse prepares to administer a Denver II developmental 4. Congenital anomalies of the spine.
assessment on a toddler. In which order should the nurse 5. Preeclampsia.
complete the assessment? Place in sequential order. 41. A client sustains multiple fractures following a motor
1. Ask child to name dolls body parts. vehicle accident. The client is intubated and on a
2. Ask caregiver if the child uses a spoon and mechanical ventilator. Chest x-ray reveals diffuse bilateral
fork. pulmonary infiltrates. The ratio of fractional concentra-
3. Ask the child to build a tower of 2 cubes. tion of inspired oxygen to arterial oxygen concentration
4. Advise caregiver that the assessment is not an (PaO2/FIO2 ratio) is 174, central venous pressure (CVP)
IQ test. is 5 mm Hg, and pulmonary capillary wedge pressure
5. Have child kick ball forward. (PCWP) is 10 mm Hg. What interventions would be
36. A primigravida presents to a labor and delivery unit at most appropriate for the client? Select all that apply.
30 weeks gestation. Her major complaints are abdominal 1. Antibiotic therapy.
pain, vaginal bleeding, and dysuria. Which interventions 2. Addition of positive end-expiratory pressure (PEEP) to
or assessments should a nurse perform when caring for ventilator.
this client? Select all that apply. 3. Prone positioning of client.
1. Call the physician for pain medication. 4. Inotropic or vasopressor drugs.
2. Place the client on an external fetal monitor. 5. Heparin drip.
3. Obtain a specimen for fern test. 6. Neuromuscular blockade with pancuronium (Pavalon).
4. Obtain a clean-catch urine specimen. 42. A client is admitted to labor and delivery with a diagnosis
5. Perform a vaginal exam for cervical dilation. of placental abruption. A nursing priority for this client is
to minimize alterations in fetal tissue perfusion. A nurse
37. Which conditions would potentially contribute to
recognizes that this goal has been met when which find-
decreases in cardiac output? Select all that apply.
ing is noted?
1. Early septic shock.
1. Presence of accelerations.
2. Gastrointestinal bleeding.
2. Consistent decreased fetal heart rate variability.
3. Congestive heart failure.
3. Evidence of prolonged fetal bradycardia.
4. Use of positive end-expiratory pressure.
4. Presence of consistent late decelerations.
5. Regular aerobic exercise.
6. Myocardial infarction. 43. Which findings should indicate to a nurse that a
traumatic brain injury has resulted in brain death?
38. The parents of a child isolated with scarlet fever ask a Select all that apply.
nurse. Why didnt our child get a vaccine for this
1. No response to the cold caloric test.
disease? The nurses best response is:
2. Glasgow Coma Scale (GCS) score of 6.
1. The disease is bacterial in nature and responds well to
3. Electroencephalogram (EEG) tracing is flat.
antibiotics.
4. Positive gag reflex.
2. A vaccine is in development.
5. No spontaneous respirations.
2164_Ch01_001-058 29/03/12 12:18 PM Page 40

40 chapter 1 Orientation and Pre-Tests

44. A home health nurse visits a child with hemophilia 3. Chorionic villus sampling (CVS).
PRE-TEST

following a dental extraction. Which medication would 4. Ultrasonography.


the nurse suggest be removed from the familys bathroom 50. The parents of a baby boy, who is newly diagnosed with
medicine cabinet? hemophilia, state This cant be true! No one in our
1. Acetaminophen. family has hemophilia. A nurses most factual response
2. Corticosteroid. would be:
3. Ibuprofen. 1. You are right. Additional testing is necessary to rule
4. Epsilon-aminocaproic acid. out a diagnostic error.
45. Following surgery for appendicitis, a client develops 2. Difficult as it is, you must try and accept the
peritonitis. Which signs or symptoms should a nurse diagnosis.
identify as being consistent with peritonitis? Select all 3. Only about 60% of affected children have a positive
that apply. history for the disease.
1. Shallow respirations. 4. Up to one third of all hemophilia cases may be
2. Abdominal rigidity. caused by a gene mutation.
3. Hyperactive bowel sounds. 51. Which is the most important initial nursing action for a
4. Abdominal pain. client with community-acquired pneumonia?
5. Increased flatus. 1. Maintain IV site used for antibiotic therapy.
46. A nurse is preparing to administer intravenous magne- 2. Assess for indications of antibiotic effectiveness.
sium sulfate to a client diagnosed with preeclampsia. 3. Assist with client mobility.
Which action should the nurse take when administering 4. Assess for pain and treat as ordered.
this medication? 52. A client, who is a primigravida at 39 weeks gestation,
1. Administer a solution of 20 gm magnesium sulfate in is being instructed on when she should come to the
100 mL of lactated Ringers solution. hospital. Which instructions should a nurse include?
2. Monitor maternal VS, FHR, and uterine contractions Select all that apply.
every hour. 1. Come to the hospital if you are feeling decreased fetal
3. Expect the maintenance dose to be approximately movement.
4 gm/hr. 2. Come to the hospital when the contractions are
4. Discontinue the infusion and notify the physician of a regular, about 5 minutes apart, lasting about
respiratory rate of <12 breaths/min. 60 seconds.
47. When would a client with community-acquired pneumonia 3. Come to the hospital when you think your amniotic
be clinically stable enough to change from IV antibiotics to fluid is leaking.
oral antibiotics? When the client has a(n): 4. Come to the hospital when you feel increased fetal
1. Temperature of 96.8F (36C) or lower. movement.
2. O2 saturation of at least 80%. 5. Come to the hospital when the contractions are
3. Systolic blood pressure that stays at 90 mm Hg or 1015 minutes apart lasting about 30 seconds.
higher. 53. What type of dressing should a nurse use on a stage I
4. Heart rate that remains at 120 beats/min or lower. pressure ulcer?
48. An adolescent with hemophilia sustains a bilateral lower 1. Wet to dry.
extremity injury and is hospitalized. Which selection 2. Wet.
on the adolescents lunch menu indicates the need for 3. Dry.
further dietary counseling by the nurse? 4. None.
1. A yogurt and fruit plate. 54. Which pediatricians orders for a newborn boy with a
2. A cheeseburger and coleslaw. family history of hemophilia should a nurse question?
3. A slice of cheese pizza and a tossed green salad. 1. Administer hepatitis B vaccine.
4. A scoop of pasta salad and a cup of vegetable soup. 2. Delay circumcision at this time.
49. A 44-year-old client has just found out that she is 3. Schedule series of well-infant appointments at time of
pregnant. The client knows that she is at a higher risk discharge from birth hospital.
for carrying a fetus with a chromosomal abnormality. 4. Finger or heel punctures for blood draws only.
Which genetic test for chromosomal abnormalities 55. A client, who is a primigravida, delivered a healthy
should a nurse recommend to provide the client with neonate by vacuum-assisted vaginal delivery after
chromosomal results within the first trimester? 24 hours of labor. A nurse is performing a routine
1. Genetic karyotyping. postpartum assessment 12 hours later and finds the
2. Amniocentesis.
2164_Ch01_001-058 29/03/12 12:18 PM Page 41

Pre-Test 2 41
following vital signs: blood pressure, 118/76 mm Hg; 60. A nurse is caring for a client with second-degree burns of

PRE-TEST
temperature, 100.8F; pulse, 102; respirations, 20. both arms. Which care need will be more challenging
Which interventions would be appropriate for this because of the location of the burn?
client? Select all that apply. 1. Reducing risk for immunosuppression and
1. Call the physician. infection.
2. Encourage her to increase her fluid intake. 2. Reducing potential for contractures and
3. Assess her fundal height. deformity.
4. Assess her lochia. 3. Establishing IV access for administration of
5. Encourage her to bottle-feed. fluids.
56. Causation is an element that must be demonstrated by 4. Assessment of vital signs, including blood
the plaintiff in a suit against a nurse for professional pressure.
malpractice. Under what circumstance will the actions 61. A nurse prepares to administer N-acetylcysteine
of the nurse be determined to be within a legal causal (Mucomyst) to a child who ingested a toxic dose of
connection? Select all that apply. acetaminophen (Tylenol). What equipment will the
1. Only if the nurses action directly causes the harm. nurse need to gather in order to administer the
2. If the harm caused was a foreseeable consequence of medication?
the nurses action. 1. An IV setup.
3. If the nurse is named in the lawsuit. 2. An IM setup.
4. If the nurse was involved in any of the clients care. 3. A glass of water.
5. When the nurse did not follow the standards of care 4. A glass of juice.
and harm was caused.
6. If the nurse performed harmful actions while taking 62. A client, who is gravida 1, para 0 at 35 weeks gesta-
narcotic medications. tion, delivered her infant 1 day ago by cesarean section
for breech presentation. The infant was admitted to
57. A toddler, who had a blood lead level (BLL) test result of the nursery for unstable blood sugar. Which action by
<10 mcg/dL 1 year ago, presents in the pediatricians a nurse would be the most appropriate to promote the
office with a level of 48 mcg/dL. The nurse should attachment process between mother and infant?
anticipate the need for a/an: 1. Take pictures of the baby for the mother to see.
1. EEG. 2. Take the mother to the nursery and encourage her to
2. CBC. participate in his care.
3. Abdominal x-ray. 3. Take the mother to the nursery window to see her
4. Chest x-ray. baby.
58. During change of shift, a day shift nurse states that all of 4. Tell the mother her baby is very cute and give her an
a clients urine is to be strained. The evening nurse knows update on his progress.
that straining is ordered because: 63. A 3-year-old child has ingested some pink pills in an
1. Intake and output will be more accurate. unlabeled bottle found on the floor of the garage. In
2. The client has been having difficulty voiding. what order should a nurse correctly perform a gastric
3. The composition of renal calculi needs to be lavage and administration of activated charcoal?
identified. 1. Perform the gastric lavage and then administer the
4. Blood clots have been affecting the accuracy of the activated charcoal.
urinalysis. 2. Administer the activated charcoal and then perform
59. A 22-year-old client, who is gravida 1, para 0, is the gastric lavage.
attending a routine 38-week prenatal visit. A physician 3. Perform the gastric lavage, leave the saline solution
gives the client instructions about when to come to in the stomach, and administer the activated
the hospital for evaluation of onset of labor. A nurse charcoal.
evaluates that the client has understood the instructions 4. Question the order because gastric lavage and activated
when she states: charcoal are not used together.
1. I should go to the hospital when my contractions are 64. A client asks a nurse how soon after bowel surgery
less than 2 minutes apart. normal bowel function will return. The best response
2. I should go to the hospital when I have back pain by the nurse would be:
and an increased vaginal discharge. 1. By 72 hours you should start to pass gas.
3. I should go to the hospital when I have abdominal 2. Around 48 hours, if there are no complications.
pain and painful urination. 3. Some function will return by 12 hours.
4. I should go to the hospital when my contractions are 4. Youll pass gas by 24 hours.
5 minutes apart for an hour.
2164_Ch01_001-058 29/03/12 12:18 PM Page 42

42 chapter 1 Orientation and Pre-Tests

65. A client is in active labor at 38 weeks gestation. She 70. Which types of congenital heart disease are characterized
PRE-TEST

requests an epidural for labor pain. Which actions should by blood flow from right to left? Select all that apply.
a nurse take to prepare for this procedure? Select all that 1. Atrial septal defect.
apply. 2. Transposition of the great vessels.
1. Sit client upright on the side of the bed. 3. Coarctation of the aorta.
2. Start a bolus of IV solution. 4. Tricuspid atresia.
3. Obtain baseline vital signs. 5. Truncus arteriosus.
4. Allow client to empty her bladder. 6. Patent ductus arteriosus.
5. Obtain an informed consent for the procedure. 7. Tetralogy of Fallot.
6. Perform a procedural pause before starting epidural.
71. An older client is scheduled to go home following hos-
66. A teenager with sickle cell anemia tells a clinic nurse that pitalization for thrombophlebitis. When a nurse offers
the school has arranged for a nature hike in the nearby the client the daily warfarin tablet, the client refuses the
mountains. The teen excitedly shows the nurse a new medication. The best response by the nurse would be:
water bottle that has been purchased just for this trip. 1. You must take the pill as ordered by your doctor.
Which initial response by the nurse would be most 2. You have taken this every morning for a week.
appropriate? 3. Do you want to end up back in the hospital for
1. You realize that this kind of hike might send you into another week?
a pain crisis, dont you? 4. Tell me about your concerns with your medication.
2. You should not attend the hike unless you get
72. A client, who is gravida 7, para 6, delivered an infant
approval from the physician first.
at 39 weeks gestation approximately 14 hours ago.
3. Its great that you are preparing to hydrate properly on
Which assessment finding indicates normal postpartum
your hike. How else have you planned for the hike?
progression?
4. That sounds like a lot of fun and is terrific exercise.
1. Firm fundus at the umbilicus and midline with
What do your parents think?
moderate lochia rubra.
67. A client, who is diabetic, is scheduled for discharge. A 2. Firm fundus 12 fingerbreadths above the umbilicus
nurse should recognize the need for further teaching if and midline with moderate lochia rubra.
the client states: 3. Firm fundus 12 fingerbreadths above the umbilicus
1. I take my oral hypoglycemics during the day because and deviated to the right side with large lochia rubra.
I sleep 8 hours at night. 4. Firm fundus 34 fingerbreadths below the umbilicus
2. Ive had diabetes for 15 years, and Ive never had any and midline with scant lochia rubra.
problems until now.
73. A client has not been following the drug regimen for
3. I live alone, so I use the microwave often to prepare
control of gastroesophageal reflux disease (GERD). The
my meals.
best approach by a nurse to improve compliance would
4. I had been taking a name brand drug, but the generic
be to:
brand costs less.
1. Give the client a choice rather than a directive on
68. The lips and oral cavity of a child who ingested a when to take the medication.
corrosive substance are blistered, peeling, and swollen. 2. Ensure that the client understands the expected
The priority action for a nurse in the emergency outcome from the drug regimen.
department is to: 3. Encourage the client to follow the goals set by the
1. Keep child NPO. physician for optimal management of GERD.
2. Administer an analgesic. 4. Ask the physician or pharmacist to meet with the
3. Prepare to induce vomiting. client prior to discharge.
4. Maintain a patent airway.
74. A nurse is performing health teaching with the parents of
69. A client, who is gravida 8, para 7, has saturated three a 16-month-old child who has tetralogy of Fallot. Which
perineal pads with bright red blood during the first hour steps should the nurse advise the parents to take to prop-
after delivery. What are appropriate nursing actions for erly respond to a tet spell? Select all that apply.
this client? Select all that apply. 1. Call EMS.
1. Administer oxytocin. 2. Sit the child in a chair.
2. Massage the fundus. 3. Learn CPR.
3. Have the client empty her bladder. 4. Administer aspirin.
4. Administer tocolytics. 5. Ask the child to cough.
5. Check vital signs. 6. Place child on the side with knees to chest.
2164_Ch01_001-058 29/03/12 12:18 PM Page 43

Answers/Rationales/Tips 43
75. A client, who is 19 years old, gravida 1, para 0, is admitted 3. CORRECT ANSWER: 1. Answer 1 is correct because

PRE-TEST
to labor and delivery at 30 weeks gestation. The client this clients fundus is higher than expected for this
reports nausea and vomiting, epigastric pain, and a stage after delivery and is deviated to the right,
headache. Her vital signs are temperature: 98.8F, pulse: 78, indicating she has a full bladder that can impede the
respirations: 20, and blood pressure: 170/110 mm Hg. uterus from contracting adequately. This can result in a
Her laboratory values are as follows: WBC: 4.5, hematocrit: postpartum hemorrhage. Answer 2 is incorrect because
30, platelets: 88,000, AST: 146, ALT: 120. What other this client is not a high risk for mastitis based on the infor-
assessments should a nurse perform on this client? Select mation provided. Mastitis is a breast infection that can
all that apply. result from cracked or sore nipples or failure to fully drain
1. Deep tendon reflexes. the breast. Answer 3 is incorrect because endometriosis is a
2. Sterile vaginal exam. proliferation of endometrial tissue, not related to the deliv-
3. Sterile speculum exam. ery of an infant. Answer 4 is incorrect because thrombo-
4. Fern test. phlebitis is a risk factor for all women who are pregnant;
5. Urine protein test. this client is not at any increased risk because of her history
with this delivery.
Answers/Rationales/Tips TEST-TAKING TIP: Eliminate complications that are common
to all women who are pregnant, but not directly related to this
1. CORRECT ANSWER: 2. Answer 1 is incorrect because clients specific risk factors.
tantrums are related to the childs developmental stage, not Content Area: Maternity, Postpartum; Integrated Process:
his/her diet. Answer 2 is correct because toddlers may have Nursing Process, Analysis; Cognitive Level: Analysis; Client
temper tantrums to express their frustration. Sudden Need/Subneed: Physiological Integrity/Reduction of Risk
changes in activities should be avoided, and the toddler Potential/Potential for Complications from Surgical Procedures
should be given forewarning about what is coming next. and Health Alterations
Answer 3 is incorrect because tantrums should be ignored. By
4. CORRECT ANSWER: 3. Answer 1 is incorrect because heat
holding the child, the parents will be providing reinforcement increases swelling. Answer 2 is incorrect because pain control
of the tantrum behavior. Answer 4 is incorrect because even may be required, but muscle spasm is not the source of the
negative reinforcement can reinforce the tantrum behavior. pain. Answer 3 is correct because the injured tissue needs
Additionally, spanking should be avoided and it is inappropri- to be immobilized. Answer 4 is incorrect because the injured
ate for the nurse to recommend corporal punishment as a tissue should be immobilized and rested.
behavior modification technique. TEST-TAKING TIP: Look for an option that rests the
TEST-TAKING TIP: The key word to the best answer is extremity.
prevent. Recall that the toddler needs to be autonomous. By Content Area: Adult Health, Musculoskeletal; Integrated
providing the child with information about coming activities, Process: Nursing Process, Implementation; Cognitive Level:
the child will feel more in control and will be less likely to Application; Client Need/Subneed: Physiological Integrity/Basic
become frustrated. Care and Comfort/Assistive Devices
Content Area: Child Health, Growth and Development;
Integrated Process: Nursing Process, Implementation; Cognitive 5. CORRECT ANSWER: 13. To arrive at the correct
Level: Application; Client Need/Subneed: Health Promotion score on the Glasgow Coma Scale (GCS), recall that
and Maintenance/Developmental Stages and Transitions the scale is composed of three scoring areas: eye
opening (ranges from 1 to 4), best motor response
2. CORRECT ANSWERS: 4, 3, 6, 5, 1, 2, 7. After determining (ranges from 1 to 6), and best auditory/visual response
that the client is nonresponsive and not breathing or is (ranges from 1 to 5). This child receives the maximum
gasping for air, the nurse or someone else calls for help. score of 4 for spontaneous eye opening, a score of 5 out
The pulse is palpated for 10 seconds, and if no pulse is of 6 for motor response, and 4 out of 5 for auditory/
palpated, the client is positioned properly to perform visual response.
CPR. The order is: 30 chest compressions at 100/min, TEST-TAKING TIP: Remember that the lowest score an indi-
open the airway, and then 2 breaths. ABC has been vidual can receive on the GCS is 3 (no eye opening and no
changed to CAB; the emphasis is on chest compressions. motor nor auditory/visual response) and the highest is 15.
TEST-TAKING TIP: Remember: the heart is the first Content Area: Child Health, Neurological; Integrated Process:
priority. Nursing Process, Assessment; Cognitive Level: Analysis; Client
Content Area: Adult Health, Cardiovascular; Integrated Process:
Need/Subneed: Physiological Integrity/Physiological
Nursing Process, Implementation; Cognitive Level: Application Adaptation/Alterations in Body Systems
Client Need/Subneed: Physiological Integrity/Physiological
Adaptation/Medical Emergencies
2164_Ch01_001-058 29/03/12 12:18 PM Page 44

44 chapter 1 Orientation and Pre-Tests

6. CORRECT ANSWER: 3. Answer 1 is incorrect because for clots. There is no evidence that a contraindication for
PRE-TEST

infections are generally introduced into the wound during these drugs exists, such as bleeding. Answer 4 is correct
the initial trauma and surgery. Although it is possible that because clients having knee surgery are at very high
infections could be introduced during postoperative wound risk for thrombus formation and need anticoagulation.
care, double-gloving would not reduce the risk. Answer 2 is Coumadin is convenient because it can be taken for sever-
incorrect because pin site care is controversial. Some sur- al days of therapy before achieving therapeutic results (an
geons prefer that pins be surrounded with scabs to protect INR level of 2). Until achieved, the quicker acting drug,
the tissue. Some prefer cleansing with Betadine; however, it heparin, is also used.
is irritating to tissue. Answer 3 is correct because open frac- TEST-TAKING TIP: Look at the verbs: three hold and one
tures create opportunities for microbes to enter the continue. Choose the option that is different in this case.
wound. Antibiotics throughout the perioperative period Content Area: Adult Health, Musculoskeletal; Integrated
reduce the risk. Answer 4 is incorrect because contact isola- Process: Nursing Process, Implementation; Cognitive Level:
tion is used to prevent transmission of infection to other Application; Client Need/Subneed: Physiological Integrity/
clients from the client who is isolated. It could not protect Reduction of Risk Potential/Laboratory Values
the client who is isolated. 9. CORRECT ANSWER: 2. Answer 1 is incorrect because
TEST-TAKING TIP: Look for clues in the stem, such as clubbing in fingers and toes will not be evident in an infant.
prevent, and a similar word in the optionsprophylactic. This symptom develops over time and would be expected in
Content Area: Adult Health, Musculoskeletal; Integrated
an older child. Answer 2 is correct because the newborn
Process: Nursing Process, Planning; Cognitive Level:
with a cyanotic heart defect will most likely experience
Comprehension; Client Need/Subneed: Physiological Integrity/ tachycardia as the heart attempts to improve oxygenation.
Reduction of Risk Potential/Potential for Complications of Answer 3 is incorrect because polycythemia may not develop
Diagnostic Tests/Treatments/Procedures right away, while tachycardia is present at birth. Answer 4 is
7. CORRECT ANSWER: 1. Answer 1 is correct because incorrect because distended neck veins are not always pres-
this client is hyperventilating, resulting in a reduction ent in infants with cyanotic defects even if the infant is in
of carbon dioxide in her blood and causing her symp- congestive heart failure.
toms. Having her breathe into a paper bag will increase TEST-TAKING TIP: The key word in the stem of the question
the carbon dioxide in her blood and alleviate the symp- is newborn.The correct response should take into account the
toms. Answer 2 is incorrect because helping her to assume infants young age and the fact that the child has not suffered
a left lateral position in bed will not increase carbon from congenital heart disease for an extended period of time.
dioxide in her blood. Answer 3 is incorrect because shallow Content Area: Child Health, Cardiovascular; Integrated Process:
chest breathing is a form of hyperventilation and will not Nursing Process, Assessment; Cognitive Level: Analysis; Client
reverse the imbalance between the oxygen and carbon Need/Subneed: Physiological Integrity/Physiological Adaptation/
dioxide levels in the lungs. This imbalance delivers less Pathophysiology
oxygen to the brain, the heart, and the rest of the body. 10. CORRECT ANSWER: 3. Answer 1 is incorrect because,
Answer 4 is incorrect because this situation can be resolved
although medication allergies can cause problems, in this case,
using nursing interventions that do not require calling the the headache is due to a spinal needle that creates a passage
physician. for the spinal fluid to leak out, changing the fluid pressure
TEST-TAKING TIP: When calling the physician is one of the around the brain and spinal cord. If enough of the fluid leaks
options, consider why you are calling the physician and what out, a spinal headache may develop. Answer 2 is incorrect
assessment information you need before calling. If you cannot because spinal headaches are one of the common side effects
identify why you would call the physician and what you of spinal anesthesia. This answer is inappropriate to tell the
want him/her to do about the situation, eliminate the client. Answer 3 is correct because keeping the head of the
option. bed flat can alleviate the headache caused by the difference
Content Area: Maternity, Intrapartum; Integrated Process:
in pressure. Sitting up can make the headache worse by
Nursing Process, Implementation; Cognitive Level: Analysis; increasing the pressure difference. Answer 4 is incorrect
Client Need/Subneed: Physiological Integrity/Physiological
because medication does not provide relief when the origin
Adaptation/Alterations in Body Systems of the headache is a difference in pressure. A blood patch can
8. CORRECT ANSWER: 4. Answer 1 is incorrect because the be used to stop the leakage of spinal fluid, seal the leak, and
goal is a therapeutic international normalized ratio (INR). alleviate the pressure difference.
Coumadin will be needed to maintain an INR of about 2. TEST-TAKING TIP: Eliminate the answer that provides false
Answer 2 is incorrect because the quick-acting drug, heparin, information to the client. Choose the one answer that is a
acts as an anticoagulant bridge until the slower onset of specific action that the nurse can take.
Coumadin achieves therapeutic levels. Answer 3 is incorrect Content Area: Maternity, Intrapartum; Integrated Process:
because the client needs anticoagulation, being at high risk Teaching and Learning; Cognitive Level: Application; Client
2164_Ch01_001-058 29/03/12 12:18 PM Page 45

Answers/Rationales/Tips 45
Need/Subneed: Physiological Integrity/Reduction of Risk TEST-TAKING TIP: First, eliminate the two options that focus

PRE-TEST
Potential/Potential for Alterations in Body Systems on insulin administration. Next, eliminate the answer that
means do not exercise. Choose a prn option: carry a sugar
11. CORRECT ANSWER: 3. Answer 1 is incorrect because source.
this clients high blood sugar needs more immediate follow-
Content Area: Adult Health, Endocrine; Integrated Process:
up, not just retesting. Answer 2 is incorrect because this
Nursing Process, Implementation; Cognitive Level: Application;
clients high blood sugar needs more immediate follow-up by
Client Need/Subneed: Physiological Integrity/Physiological
a health-care provider. Answer 3 is correct because this is
Adaptation/Illness Management
high, even for a nonfasting sample, and is suggestive of
diabetes. The client should be seen soon. Answer 4 is 14. CORRECT ANSWERS: 2, 3, 4. Answer 1 is incorrect because
incorrect because this blood sugar exceeds normal limits and the nurse needs to speak distinctly, not loudly. Answer 2 is
is suggestive of diabetes. correct because a visual mode is helpful when the client has
TEST-TAKING TIP: By recalling the normal range for non- difficulty hearing (auditory aphasia, receptive) or writing
fasting blood sugar levels, Answer 4 can be eliminated as (expressive aphasia, visual). Answer 3 is correct because using
210 mg/dL is not normal. Select between: week, 1 year, gestures such as pointing is one way to establish and encour-
or 3 years, and choose the ASAP time frame. age communication. Answer 4 is correct because facing the
Content Area: Adult Health, Endocrine; Integrated Process: client and speaking slowly allows time to respond. Answer 5
Nursing Process, Implementation; Cognitive Level: Application; is incorrect because, with expressive aphasia, the client will
Client Need/Subneed: Physiological Integrity/Reduction of Risk have difficulty in initiating speech, but may be able to
Potential/Laboratory Values articulate words that have no meaning. Answer 6 is incorrect
because, although support and encouragement need to be
12. CORRECT ANSWER: 1. Answer 1 is correct because offered, these are general nursing approaches and not specific
the school-age child is old enough to understand and
to this client.
comply with instructions regarding guided imagery, a
TEST-TAKING TIP: Be aware that the stem asks for specific
technique to help the child focus on a pleasant mental
approaches.
image during the procedure. Answer 2 is incorrect because
Content Area: Geriatrics, Sensory; Integrated Process: Nursing
this technique would most likely be utilized with a younger
Process, Implementation; Cognitive Level: Application; Client
child. Answer 3 is incorrect because this technique would
Need/Subneed: Psychosocial Integrity/Therapeutic
most likely be utilized with a younger child. Answer 4 is
Communications
incorrect because this technique would most likely be uti-
lized with an infant. 15. CORRECT ANSWER: 4. Answer 1 is incorrect because
TEST-TAKING TIP: Remember that a school-age childs this client has an A1c of 11%, indicating average blood
age range is from 6 to 12 years. If the answer is not suitable sugars of about 300 mg/dL over the last 3 months.
for a child up to 12 years of age, it cannot be the correct Regardless of clients statement of compliance and the
option. immediate fasting blood sugar of 132, the disease, on
Content Area: Child Health, Pain Control; Integrated Process: average, has been out of control for months. Answer 2 is
Nursing Process, Implementation; Cognitive Level: Application; incorrect because the client claims to be compliant with
Client Need/Subneed: Physiological Integrity/Basic Care and diet, medications, and exercise, and the normal fasting
Comfort/Non-pharmacological Comfort Interventions blood sugar is consistent with this. Looking at the A1c
shows that, on average, the clients blood sugar has been
13. CORRECT ANSWER: 3. Answer 1 is incorrect because about 300 mg/dL over the last 3 months. Answer 3 is
exercise facilitates glucose uptake into cells independent of
incorrect because this client is only using diet for diabetes
insulin. Until the client is able to establish his/her own
management. The high fasting blood sugar and A1c indicate
response, it is advisable to use pre-exercise insulin sparingly,
that this is clearly not enough. Answer 4 is correct because
if at all. Answer 2 is incorrect because, although rotation
the A1c measures average blood sugar over the previous
of insulin sites is important, it is not related to exercise. A
3 months. The goal is less than 7%. Short-term compli-
person with diabetes should pick one location (such as the
ance can give the client a normal fasting blood sugar,
right thigh) and rotate injections within that site before
but this is not considered good control of the disease.
switching to another. Answer 3 is correct because people
TEST-TAKING TIP: HgbA1c is the best indication of
with diabetes should carry a sugar source, especially
compliance. Three of the options are very close in value.
when beginning an exercise program. Exercise facilitates
Choose the lowest HgbA1c as an example of best control.
glucose uptake into cells independent of insulin. If the
Content Area: Adult Health, Endocrine; Integrated Process:
client took a regular dose of insulin before exercising, the
Nursing Process, Analysis; Cognitive Level: Analysis; Client
client could become hypoglycemic. Answer 4 is incorrect
Need/Subneed: Physiological Integrity/Reduction of Risk
because exercise is beneficial for diabetes. It should be
Potential/Laboratory Values
monitored but not prevented.
2164_Ch01_001-058 29/03/12 12:18 PM Page 46

46 chapter 1 Orientation and Pre-Tests

16. CORRECT ANSWERS: 1, 4. Answer 1 is correct because a cheek appearance is associated with both fifth disease (ery-
PRE-TEST

positive Homans sign can be a symptom of deep vein thema infectiosum) and scarlet fever. Answer 2 is incorrect
thrombosis and needs to be reported to the physician since the child with measles may first experience a fever and
immediately. Answer 2 is incorrect because 2+ deep tendon malaise. Answer 3 is incorrect because, on or about the third
reflexes are a normal finding, and do not need to be reported day of infection, the child experiences a generalized rash that
to the physician. Deep tendon reflexes that are a 3+ or 4+ may spreads from the face to the trunk and limbs. Answer 4 is
indicate cerebral irritability from preeclampsia and should be incorrect because the child with measles may develop red spots
reported to the physician. Answer 3 is incorrect because mod- with bluish white centers in the buccal mucosa, known as
erate rubra lochia is a normal finding 3 hours after delivery, Kopliks spots.
and does not need to be reported to the physician. Large rubra TEST-TAKING TIP: Answers 2 and 3 are common to many
lochia or expulsion of blood clots during fundal massage pediatric infectious diseases and can be eliminated since they
would indicate a postpartum hemorrhage and should be would not be least likely findings. Of the remaining options,
reported to the physician. Answer 4 is correct because expres- recall that Kopliks spots are a hallmark of only one communi-
sion of orange-sized clots during fundal massage is a sign cable disease (measles) while slapped cheek appearance is
of postpartum hemorrhage and should be reported to the common to more than one. Choose the option that is specific
physician immediately. Answer 5 is incorrect because a to one communicable disease.
fundus that is palpable firm at the level of the umbilicus is a Content Area: Child Health, Infectious Disease; Integrated
normal finding 3 hours after delivery and does not need to be Process: Nursing Process, Assessment; Cognitive Level:
reported to the physician. A fundus that is deviated from the Application; Client Need/Subneed: Physiological Integrity/
midline or palpable above the umbilicus is not a normal find- Physiological Adaptation/Pathophysiology
ing, and should be reported to the physician 19. CORRECT ANSWER: 2. Answer 1 is incorrect because a
TEST-TAKING TIP: When a question asks what should blood sugar level over 150 mg/dL is not imminently danger-
be reported immediately, think of the possible life- ous. Changes to diabetic management are better made based
threatening client outcome if a situation is not reported on trends over days and weeks. Answer 2 is correct because
to the physician. fruity-smelling breath implies the production of ketones,
Content Area: Maternity, Postpartum; Integrated Process:
which would occur with inadequate delivery of glucose to
Nursing Process, Assessment; Cognitive Level: Analysis; Client the cells. This can cause a dangerous acidosis and should
Need/Subneed: Physiological Integrity/Physiological
be corrected immediately. Answer 3 is incorrect because
Adaptation/Alterations in Body Systems clients with diabetes who are unable to eat still need some
17. CORRECT ANSWER: 4. Answer 1 is incorrect because insulin to handle glucose being made in the liver. The dose
sugar in the urine would decrease as the blood sugar came will likely need to be reduced, but without insulin the client
down. Answer 2 is incorrect because, as insulin takes sugar will become acidotic. Answer 4 is incorrect because blood
into the cell, it carries potassium with it. Hypokalemia is a sugar should be checked more frequently when ill, but rarely
risk of treating high blood sugar. Answer 3 is incorrect would this need to be done more frequently than every
because clients with high blood sugars are generally very couple of hours.
dehydrated and require fluids. If blood sugar was reduced TEST-TAKING TIP: Think about what independent action
too quickly, the blood pressure could potentially drop if the client should takecall the provider. Ask which options
fluid resuscitation was inadequate. Answer 4 is correct indicate a serious change in the clients condition.
because sugar adds to the osmolarity of the blood. Content Area: Adult Health, Endocrine; Integrated Process:
Lowering the sugar level too fast makes the intracellu- Teaching and Learning; Cognitive Level: Application; Client
lar osmolarity relatively higher than that in the blood- Need/Subneed: Physiological Integrity/Physiological Adaptation/
stream. Water will move from areas of low concentra- Illness Management
tion to areas of high concentration, swelling the cells 20. CORRECT ANSWERS: 1, 2, 4. Answer 1 is correct
and resulting in confusion or change in level of con- because performing a sterile vaginal examination will
sciousness. If this happens in the brain it could have eliminate the possibility of a prolapsed umbilical cord
life-threatening consequences. that may be causing the variable decelerations. Answer 2
TEST-TAKING TIP: Sugar moves back into the cell. Look for is correct because changing the clients position can help
an option that involves an effect on the cell (cerebral cells). alleviate cord compression that could be causing the vari-
Content Area: Adult Health, Endocrine; Integrated Process:
able decelerations. Answer 3 is incorrect because variable
Nursing Process, Assessment; Cognitive Level: Application; decelerations in the presence of moderate variability and a
Client Need/Subneed: Physiological Integrity/Physiological
normal fetal heart rate baseline is not indicative of fetal dis-
Adaptation/Pathophysiology tress, and does not require an emergency cesarean section.
18. CORRECT ANSWER: 1. Answer 1 is correct because the Answer 4 is correct because administering a fluid bolus
child with measles is likely to exhibit all of the symptoms can increase hydration and decrease the variable decelera-
listed except for a slapped cheek appearance. The slapped tions that may be caused by cord compression. Answer 5
2164_Ch01_001-058 29/03/12 12:18 PM Page 47

Answers/Rationales/Tips 47
is incorrect because the client is not completely dilated and Need/Subneed: Physiological Integrity/Physiological Adaptation/

PRE-TEST
is not ready for a vacuum-assisted delivery. Pathophysiology
TEST-TAKING TIP: Variable decelerations are caused by
23. CORRECT ANSWERS: 2, 5. Answer 1 is incorrect because
umbilical cord compression; think about interventions that
the client needs to change her pads more frequently to stay
can relieve umbilical cord compression. Eliminate the two
fresh and help prevent infection. Changing pads about every
answers that prepare for procedures when the information
3 to 4 hours is the suggested interval for proper hygiene.
supplied is not indicative of a problem and/or the client is
Answer 2 is correct because consistently using the peri
not ready for the procedure (vacuum-assisted delivery).
bottle to clean the perineum after each void or bowel
Content Area: Maternity, Intrapartum; Integrated Process:
movement demonstrates proper perineal care. The nurse
Nursing Process, Implementation; Cognitive Level: Analysis;
should give instructions to keep the perineum clean to
Client Need/Subneed: Physiological Integrity/Reduction of Risk
increase comfort and prevent infection. Using a peri bottle
Potential/Potential for Complications from Surgical Procedures
and water to rinse after each void or bowel movement
and Health Alterations
helps keep the perineum clean and decreases the chance
21. CORRECT ANSWER: 4. Answer 1 is incorrect because post- of infection. Answer 3 is incorrect because the client should
operative ileus would not be expected to last more than a few wipe from front to back to decrease chances of infection due
days. Most people have nutritional stores to tolerate this early. to transmitting bacteria from the rectum to the vagina.
Answer 2 is incorrect because one kidney has been removed Answer 4 is incorrect because the client should spray water
and therefore could not develop stones. The other kidney has over the perineum and not directly into the vagina. Answer 5
increased flow. Stasis would be unlikely. Answer 3 is incorrect is correct because perineal care should include using anti-
because nephrotic syndrome occurs when the glomerulus is septic spray after rinsing. The nurse should give instruc-
unable to conserve protein. This could not occur in the tions to keep the perineum clean to increase comfort and
removed kidney and is unlikely to occur in the remaining kid- prevent infection. Using antiseptic spray after rinsing
ney. Answer 4 is correct because the kidneys are located just with the peri bottle helps to prevent infection and
under the diaphragm. Postoperative pain, particularly with decreases perineal pain.
deep breathing, would be anticipated. The client is there- TEST-TAKING TIP: Select the two options that are correct
fore likely to do shallow breathing. steps when using the peri bottle.
TEST-TAKING TIP: Two options relate to the effects of Content Area: Maternity, Postpartum; Integrated Process:
surgerydecreased peristalsis and pain. Consider the one Nursing Process, Evaluation; Cognitive Level: Analysis; Client
that all clients will experiencepain. Need/Subneed: Health Promotion and Maintenance/Ante/
Content Area: Adult Health, Renal; Integrated Process: Nursing Intra/Postpartum and Newborn Care
Process, Analysis; Cognitive Level: Application; Client Need/
24. CORRECT ANSWER: 2. Answer 1 is incorrect because
Subneed: Physiological Integrity/Reduction of Risk Potential/
the older adult frequently has a poor appetite, so that anorex-
Potential for Complications from Surgical Procedures and
ia would not be a change. Answer 2 is correct because con-
Health Alterations
fusion is often the first indication of infection, which
22. CORRECT ANSWERS: 1, 2, 3, 6. Answer 1 is correct progresses rapidly to urosepsis. Answer 3 is incorrect
because the infant with pyloric stenosis experiences meta- because the temperature may only be 99F. Answer 4 is
bolic alkalosis as a result of excessive vomiting. Answer 2 is incorrect because restlessness is a more characteristic finding
correct because this physical assessment (by inspection) is with hypoxia.
consistent with pyloric stenosis. Answer 3 is correct because TEST-TAKING TIP: Look for an atypical finding (confusion),
this assessment finding is consistent with pyloric stenosis something not usually seen with an infection in a younger
as a result of the pyloric muscle blocking the outflow of client.
stomach contents. Answer 4 is incorrect because the infant Content Area: Adult Health, Renal; Integrated Process: Nursing
with pyloric stenosis has nonbilious vomiting since the obstruc- Process, Assessment; Cognitive Level: Application; Client Need/
tion is above the bile duct. Answer 5 is incorrect because the Subneed: Physiological Integrity/Physiological Adaptation/
infant with pyloric stenosis is expected to have some degree of Alterations in Body Systems
dehydration, which would result in an increase in urine specific
25. CORRECT ANSWER: 4. The appropriate number of
gravity. Answer 6 is correct because this assessment finding
compressions per minute during CPR on a child is
is consistent with pyloric stenosis as a result of poor intake.
100 per minute.
TEST-TAKING TIP: Focus on the name of the diagnosis to
TEST-TAKING TIP: Recall that the rate of compressions for
help identify symptoms. If the pyloric muscle is stenosed,
CPR is always 100 per minute, regardless of the victims age.
expect the infant to have symptoms resulting from stomach
Content Area: Child Health, Cardiovascular; Integrated Process:
contents and with nutrients not able to progress through the
Nursing Process, Implementation; Cognitive Level: Application;
digestive system.
Client Need/Subneed: Physiological Integrity/Physiological
Content Area: Child Health, Gastrointestinal; Integrated Process:
Adaptation/Medical Emergencies
Nursing Process, Assessment; Cognitive Level: Application; Client
2164_Ch01_001-058 29/03/12 12:18 PM Page 48

48 chapter 1 Orientation and Pre-Tests

26. CORRECT ANSWER: 4. Answer 1 is incorrect because a preeclampsia, which also involves swelling of the extremities,
PRE-TEST

client with symptoms of thrombophlebitis should be placed but requires immediate intervention.
on bedrest, not encouraged to ambulate. Answer 2 is incor- TEST-TAKING TIP: The question asks for immediate notifi-
rect because this client has symptoms of thrombophlebitis; cation; think life-threatening complications. Three options
massaging the leg can break the thrombus from the venous (Answers 1, 2, and 5) are annoying, but not life threatening,
wall and cause an embolus. Answer 3 is incorrect because results of increased hormones. Two options (Answers 3 and 4)
local application of heat is one of the treatments for superfi- are potentially life threatening; choose these options.
cial thrombosis, not ice, which constricts blood flow. Answer Content Area: Adult Health, Genitourinary; Integrated Process:
4 is correct because this client has a superficial throm- Nursing Process, Implementation; Cognitive Level: Analysis;
bophlebitis. Elevation of the affected extremity, a few Client Need/Subneed: Physiological Integrity/Pharmacological
days of bedrest, and local application of heat are often all and Parenteral Therapies/Adverse Effects/Contraindications/
that is needed to treat superficial thrombophlebitis. Interactions
TEST-TAKING TIP: Eliminate the responses that make the 29. CORRECT ANSWER: 1. Answer 1 is correct because
condition of thrombophlebitis worse (i.e., massage, Answer 2; normal amniotic fluid is basically colorless; any color
ambulate, Answer 1). See key phrase in the correct option: may signify fetal or maternal problems. For example,
promote venous blood flow. yellow or green amniotic fluid indicates meconium
Content Area: Adult Health, Cardiovascular; Integrated Process:
release due to fetal hypoxia or breech presentation;
Nursing Process, Implementation; Cognitive Level: Application; bloody amniotic fluid may indicate a placental abruption
Client Need/Subneed: Physiological Integrity/Reduction of Risk
or marginal placenta previa. Answer 2 is incorrect because
Potential/Potential for Complications from Surgical Procedures fetal bradycardia commonly occurs at the acme of the con-
and Health Alterations traction, due to head compression, which is a benign condi-
27. CORRECT ANSWERS: 1, 2, 5. Answer 1 is correct tion. Answer 3 is incorrect because nausea with or without
because it describes the role of the team leader. Answer 2 emesis is common in labor, due to physiological stress.
is correct because it describes the role of the team leader. Answer 4 is incorrect because diaphoresis and irritability are
Answer 3 is incorrect because, even though the team leader signs of transitional phase of labor.
may observe specific procedures performed by a nursing TEST-TAKING TIP: Carefully read the stem of the question;
assistant, the team leaders role is to evaluate care after it is it is asking for the answer that is not a normal part of labor.
completed. Answer 4 is incorrect because team members Eliminate the answers that are a normal physiological part of
who are qualified to assess are responsible for their own doc- labor.
umentation. Answer 5 is correct because it describes the Content Area: Maternity, Intrapartum; Integrated Process:
role of the team leader. Answer 6 is incorrect because, even Nursing Process, Assessment; Cognitive Level: Analysis; Client
though this may be true if qualified personnel are not avail- Need/Subneed: Physiological Integrity/Physiological Adaptation/
able, the team leader needs to be available in general as a Alterations in Body Systems
resource throughout the shift. 30. CORRECT ANSWER: 3. Answer 1 is incorrect because the
TEST-TAKING TIP: Note the key words major responsibility client has pneumonia in the right lung, not in the left lung.
in the stem. Answer 2 is incorrect because the right upper lobe is auscul-
Content Area: Management of Care; Integrated Process: Nursing
tated here. The client has right middle lobe pneumonia.
Process, Planning; Cognitive Level: Application; Client Need/ Answer 3 is correct because this is the area where the right
Subneed: Safe and Effective Care Environment/Management of
middle lobe is accessible for auscultation. Answer 4 is
Care/Concepts of Management incorrect because the right lower lobe is auscultated here. The
28. CORRECT ANSWERS: 3, 4. Answer 1 is incorrect because client has right middle lobe pneumonia.
nausea may occur as the body adjusts to the increasing hor- TEST-TAKING TIP: The key words are: right, middle lobe.
mone levels released by the patch. Answer 2 is incorrect Content Area: Adult Health, Respiratory; Integrated Process:
because chloasma is hyperpigmentation caused by the Nursing Process, Implementation; Cognitive Level: Application;
increased hormone levels. Answer 3 is correct because Client Need/Subneed: Physiological Integrity/Reduction of Risk
pain in the legs while walking can be a sign of throm- Potential/System Specific Assessments
bophlebitis, a life-threatening complication of hypercoag- 31. CORRECT ANSWER: 3. Answer 1 is incorrect because
ulability caused by the increased hormone levels. consistency with limit-setting by both parents will help
Answer 4 is correct because a headache unrelieved by
the child with ADHD to meet behavioral expectations.
analgesics can be a symptom of a blood clot in the brain, Answer 2 is incorrect because storing ADHD medications
another life-threatening complication of hypercoagulabil- puts the child at risk for sharing medications with friends or
ity caused by the increased hormone levels. Answer 5 is for potential overdose. The parents can better foster the
incorrect because swelling of the feet and hands is caused by childs independence in self-care by providing access to the
fluid retention, under the influence of increased hormone medication in an observed setting, such as at breakfast time.
levels. Do not confuse this swelling with symptoms of
2164_Ch01_001-058 29/03/12 12:18 PM Page 49

Answers/Rationales/Tips 49
Answer 3 is correct because excess environmental stimula- TEST-TAKING TIP: Eliminate the two options about

PRE-TEST
tion can increase the hyperactivity and distractibility of a clear fluid; a small amount of fluid may be from rupture of
child with ADHD. Answer 4 is incorrect because exercise membranes, and a copious amount of vernix is normal.
can greatly benefit the child with ADHD by providing an Content Area: Maternity, Intrapartum; Integrated Process:
outlet for extra energy. Answer 5 is incorrect because ADHD Nursing Process, Analysis; Cognitive Level: Analysis; Client
medications such as Ritalin should be taken daily to achieve Need/Subneed: Physiological Integrity/Physiological
therapeutic blood levels, not on an as needed basis. Adaptation/Alterations in Body Systems
TEST-TAKING TIP: When the stem of the question does not
34. CORRECT ANSWERS: 1, 3, 4, 6. Answer 1 is correct
ask for a priority answer, go through the options and ask
because burns increase cellular metabolism, resulting in an
whether each statement is a true or false recommendation.
increased oxygen demand. Answer 2 is incorrect because the
Select all the true answers as these should be part of a general
metabolic demand and need for oxygen decrease with
teaching plan.
hypothermia. Answer 3 is correct because shivering raises the
Content Area: Child Health, Behavioral; Integrated Process:
metabolism and the demand for oxygen. Answer 4 is correct
Teaching and Learning; Cognitive Level: Application; Client
because the energy expended with agitation increases oxy-
Need/Subneed: Physiological Integrity/Physiological Adaptation/
gen utilization. Answer 5 is incorrect because the metabolic
Alterations in Body Systems
demand is decreased, and therefore oxygen demand is decreased.
32. CORRECT ANSWER: 3. Answer 1 is incorrect because this Answer 6 is correct because hyperthyroidism increases cellu-
procedure is generally done under regional anesthesia. Priority lar metabolism and increases oxygen demand.
assessment in the postanesthesia care unit (PACU) would TEST-TAKING TIP: Eliminate conditions that decrease O2
involve evaluating whether anesthesia is wearing off and demand.
whether there are complications from surgery, such as damage Content Area: Adult Health, Respiratory; Integrated Process:
to nerves or blood vessels. Answer 2 is incorrect because the Nursing Process, Analysis; Cognitive Level: Application; Client
brachial pulse is above the level of the surgery and would not Need/Subneed: Physiological Integrity/Physiological
indicate whether damage had occurred to nerves or blood Adaptation/Pathophysiology
vessels during the procedure. Answer 3 is correct because
35. CORRECT ANSWERS: 4, 2, 3, 1, 5. Answer 4. Advising the
checking circulation and sensation in the fingers helps to
caregiver is first because the nurse should begin by provid-
establish that no damage to blood vessels or nerves
ing information about this assessment tool. Answer 2. Asking
occurred during surgery, and that postoperative swelling is
the caregiver about use of spoon and fork is next because the
not damaging those structures. This procedure is usually
nurse should attempt to score items that can be determined
done under regional anesthesia; return of sensation and
through interview, before asking the child to perform tasks.
movement indicates anesthesia is wearing off appropriately.
Answer 3. Asking the child to build a tower is next because
Answer 4 is incorrect because this procedure is usually done
the nurse can provide the child with blocks while talking
under regional anesthesia with sedation, neither of which
with the caregiver; the child may spontaneously build a
would create a risk for alteration in pupil response.
tower without prompting. If the nurse has to prompt the
TEST-TAKING TIP: Visualize the operative site and choose
child to build a tower, this is best done after the child is
an option that includes the operative area (fingers).
given an opportunity to warm up during the dialogue
Content Area: Adult Health, Circulatory; Integrated Process:
between the nurse and caregiver. Answer 1. Asking the child
Nursing Process, Assessment; Cognitive Level: Application;
to name body parts is next because language items are best
Client Need/Subneed: Physiological Integrity/Reduction of Risk
administered following fine motor activities when the child
Potential/Potential for Alterations in Body Systems
is more comfortable with the nurse. Answer 5. Having the
33. CORRECT ANSWERS: 1, 3, 4. Answer 1 is correct because child kick a ball is the last step because gross motor activi-
green-tinged fluid is indicative of meconium staining, a ties should be assessed at the end. Administering gross
possible indicator of fetal distress, and should be reported motor items earlier may be a distraction. The child may
to the physician. Answer 2 is incorrect because vernix in the then be unwilling to return to the table to perform fine
fluid is a normal finding after rupture of membranes, and motor activities or engage in an interview.
does not need to be reported to the physician. Answer 3 is TEST-TAKING TIP: The approach for the developmental
correct because a decrease in the fetal heart rate baseline to assessment is similar to that of a physical assessment. A nurse
100 bpm is bradycardia and may indicate umbilical cord should first address the caregiver, and then use a transition
prolapse or compression, and should be reported to the object such as a toy for interaction with the child, speaking
physician. Answer 4 is correct because an increase in the directly to the child only after trust has been established.
baseline fetal heart rate to above 160 bpm can be indicative Content Area: Child Health, Growth and Development;
of an infection, and should be reported to the physician. Integrated Process: Nursing Process, Implementation;
Answer 5 is incorrect because a small amount of clear fluid is a Cognitive Level: Application; Client Need/Subneed: Health
normal finding after rupture of membranes, and does not need Promotion and Maintenance/Developmental Stages and
to be reported to the physician. Transitions
2164_Ch01_001-058 29/03/12 12:18 PM Page 50

50 chapter 1 Orientation and Pre-Tests

36. CORRECT ANSWERS: 2, 4. Answer 1 is incorrect because Answer 3 is incorrect because the nurse has sufficient knowl-
PRE-TEST

the client is probably experiencing preterm labor with a uri- edge about the pathophysiology of scarlet fever to correctly
nary tract infection. Calling the physician to convey infor- answer the parents question without referring the parents to
mation about the clients condition would be important, but the pediatrician. Answer 4 is incorrect because scarlet fever is
not specifically for pain medication. Answer 2 is correct communicable and can be treated with the use of antibiotics
because placing the client on an external fetal monitor is (usually penicillin).
standard nursing procedure to evaluate the fetal well- TEST-TAKING TIP: Eliminate Answers 2, 3, and 4 because
being and the pattern of contractions. With preterm they are theoretically incorrect (i.e., there are no vaccines for
labor, it is important to evaluate uterine activity on the this bacterial disease, and scarlet fever is communicable).
external fetal monitor. Answer 3 is incorrect because the Content Area: Child Health, Infectious Disease; Integrated
client is not complaining of rupture of membranes. The fern Process: Teaching and Learning; Cognitive Level: Application;
test specifically helps to determine if the amniotic sac is Client Need/Subneed: Physiological Integrity/Physiological
leaking. Answer 4 is correct because the client is complain- Adaptation/Alterations in Body Systems
ing of dysuria. Obtaining a clean-catch urine specimen 39. CORRECT ANSWERS: 2, 3, 4, 5. Answer 1 is incorrect
would be appropriate to evaluate the possibility of a uri- because dextrose alone will not directly affect the potassium
nary tract infection. Answer 5 is incorrect because the client level. Insulin also needs to be given. Answer 2 is correct
is preterm and has vaginal bleeding. A vaginal exam is con- because correcting the acidosis with sodium bicarbonate
traindicated when the client has vaginal bleeding of will cause potassium to shift back into the cell. Answer 3
unknown origin. is correct because regular insulin is the first line of
TEST-TAKING TIP: The key to this question is dysuria and treatment for hyperkalemia. Dextrose is used to prevent
preterm. The appropriate interventions for a client who is hypoglycemia, which may occur with the shift of the
preterm are different than those for a full-term gestation. potassium back into the cell. Answer 4 is correct because
Content Area: Maternity, Intrapartum; Integrated Process:
Kayexalate is an exchange resin. Potassium is exchanged
Nursing Process, Analysis; Cognitive Level: Analysis; Client for Na+ in the intestines, and potassium is removed
Need/Subneed: Physiological Integrity/Physiological Adaptation/
via feces. Answer 5 is correct because albuterol has
Alterations in Body Systems been shown to decrease the potassium level by 0.5 to
37. CORRECT ANSWERS: 2, 3, 4, 6. Answer 1 is incorrect 1.5 mEq/L.
because in early septic shock there is actually an increase in TEST-TAKING TIP: Review current treatment approaches for
cardiac output. Answer 2 is correct because circulating hyperkalemia.
blood volume would be decreased with GI bleeding. Content Area: Adult Health, Renal; Integrated Process:
Answer 3 is correct because the ventricles are unable to Nursing Process, Analysis; Cognitive Level: Application; Client
pump the blood efficiently through the heart, so that car- Need/Subneed: Physiological Integrity/Pharmacological and
diac output is decreased. Answer 4 is correct because posi- Parenteral Therapies/Expected Effects/Outcomes
tive end-expiratory pressure (PEEP) increases the intratho- 40. CORRECT ANSWERS: 2, 3, 4, 5. Answer 1 is incorrect
racic pressure; therefore, venous return and cardiac output because oligohydramnios is too little amniotic fluid, and is
would be decreased. Answer 5 is incorrect because aerobic not a complication of type 1 diabetes. Answer 2 is correct
exercise increases the heart rate and thus cardiac output. because polyhydramnios, or too much amniotic fluid, is a
Answer 6 is correct because damage to the myocardium
condition resulting from poor glycemic control during
decreases the pumping effectiveness of the heart and pregnancy, which causes hyperglycemia of the fetus and
impacts cardiac output. fetal polyuria (fetal urine is a major source of amniotic
TEST-TAKING TIP: Eliminate the options that increase blood fluid). This clients hemoglobin A1c is elevated at 8.2%
pressure and heart rate. (the normal is less than 6.0%), indicating poor glycemic
Content Area: Adult Health, Cardiovascular; Integrated
control during early pregnancy. Answer 3 is correct
Process: Nursing Process, Analysis; Cognitive Level: Analysis;
because macrosomia, a fetus who is oversized, is a condi-
Client Need/Subneed: Physiological Integrity/Physiological
tion resulting from poor glycemic control during preg-
Adaptation/Pathophysiology nancy. The fetus produces too much insulin in response
38. CORRECT ANSWER: 1. Answer 1 is correct because scar- to the maternal hyperglycemia; insulin acts as a growth
let fever is bacterial in nature (Group A -hemolytic strep- hormone during fetal development, producing a large
tococci) and responds well to antibiotics (usually peni- fetus. Answer 4 is correct because this clients hemoglobin
cillin). Vaccines are generally developed for communicable A1c is elevated at 8.2% (the normal is less than 6.0%),
diseases that are viral in nature. Answer 2 is incorrect because indicating poor glycemic control during early pregnancy,
vaccines are generally developed for communicable diseases which is highly correlated with spinal anomalies.
that are viral in nature as opposed to bacterial. There is no Answer 5 is correct because preeclampsia is more com-
vaccine for scarlet fever in development at the present time. monly found in clients with preexisting diabetes. Rates
2164_Ch01_001-058 29/03/12 12:18 PM Page 51

Answers/Rationales/Tips 51
of preeclampsia in women with type 1 diabetes are two to indicates the absence of electrical activity in the brain.

PRE-TEST
four times higher than in normal pregnancies. Answer 4 is incorrect because a gag reflex would indicate
TEST-TAKING TIP: Look at two opposite complications brain function. Answer 5 is correct because the absence of
(too much . . . too little amniotic fluid) and eliminate one (i.e., spontaneous respirations indicates the absence of brain
too little). Remember that, if the maternal glucose is poorly function.
controlled, the fetus is at greater risk than with well-controlled TEST-TAKING TIP: Remember that brain death is an
blood glucose. In this case, the hemoglobin A1c is elevated, irreversible loss of all brain functions.
meaning the blood sugar has been poorly controlled. Content Area: Adult Health, Neurological; Integrated Process:
Content Area: Maternity, Antepartum; Integrated Process: Nursing Process, Analysis; Cognitive Level: Application; Client
Nursing Process, Analysis; Cognitive Level: Analysis; Client Need/Subneed: Physiological Integrity/Physiological Adaptation/
Need/Subneed: Physiological Integrity/Reduction of Risk Alterations in Body Systems
Potential/Potential for Alterations in Body Systems
44. CORRECT ANSWER: 3. Answer 1 is incorrect because
41. CORRECT ANSWERS: 2, 3, 4, 6. Answer 1 is incorrect there is no contraindication for the use of acetaminophen
because the client has acute respiratory distress syndrome to control pain in the treatment of hemophilia. Answer 2 is
(ARDS), not an infection, which would call for antibiotic incorrect because corticosteroids are given for the hematuria,
therapy. Answer 2 is correct because PEEP improves acute hemarthrosis, and the chronic synovitis associated
ventilation to the atelectatic alveoli, improving oxygena- with hemophilia. Answer 3 is correct because, while
tion. Answer 3 is correct because prone positioning, if nonsteroidal anti-inflammatory drugs (NSAIDs), such
not contraindicated, improves oxygenation because more as ibuprofen, are effective in relieving pain caused by
even distribution of ventilation occurs. Answer 4 is cor- the chronic synovitis associated with hemophilia, they
rect because these drugs increase contractility of the heart inhibit platelet function, which is essential in controlling
and raise blood pressure, improving perfusion. Answer 5 bleeding episodes. Answer 4 is incorrect because oral use
is incorrect because the client has ARDS, not a coagulation of epsilon-aminocaproic acid (Amicar) prevents clot destruc-
problem that would call for anticoagulant therapy. Answer 6 tion. Its use is limited to mouth trauma or surgery, and a
is correct because neuromuscular blockade decreases dose of factor concentrate must be given first. The child
oxygen demand. may rinse the mouth with this medication and then
TEST-TAKING TIP: Eliminate the options that do not improve swallow it.
oxygenation. TEST-TAKING TIP: Select the drug that would be con-
Content Area: Adult Health, Respiratory; Integrated Process: traindicated. Think prevent bleeding in hemophilia,
Nursing Process, Analysis; Cognitive Level: Analysis; Client Need/ and then select the medication that could cause rather than
Subneed: Physiological Integrity/Physiological Adaptation/ prevent bleeding.
Medical Emergencies Content Area: Child Health, Pharmacology; Integrated
Process: Teaching and Learning; Cognitive Level: Application;
42. CORRECT ANSWER: 1. Answer 1 is correct because Client Need/Subneed: Physiological Integrity/Pharmacological
accelerations are an indication of fetal well-being and a
and Parenteral Therapies/Adverse Effects/Contraindications/
well-oxygenated fetus. Answer 2 is incorrect because consis-
Interactions
tent decreased fetal heart rate variability is an indicator of
decreased placental perfusion and poor oxygenation. Answer 3 45. CORRECT ANSWERS: 1, 2, 4. Answer 1 is correct
is incorrect because prolonged fetal bradycardia is an indica- because deep breathing causes pressure on the peri-
tor of decreased placental perfusion and fetal distress. Answer 4 toneum from the diaphragm and pain. Expect to see shal-
is incorrect because consistent late decelerations are an indi- low breathing, which causes minimal pressure on the
cator of poor uteroplacental perfusion and fetal distress. diaphragm. Answer 2 is correct because peritoneal inflam-
TEST-TAKING TIP: The priority for this client is fetal well- mation causes peritoneal contraction and the classic
being; consider which of the patterns are indicative of good boardlike abdomen. Answer 3 is incorrect because bowel
oxygenation. sounds are typically hypoactive. Answer 4 is correct because
Content Area: Maternity, Intrapartum; Integrated Process: the inflammation and irritation of the peritoneum cause
Nursing Process, Evaluation; Cognitive Level: Analysis; Client pain. Answer 5 is incorrect because the decreased peristalsis
Need/Subneed: Physiological Integrity/Physiological will decrease flatus.
Adaptation/Medical Emergencies TEST-TAKING TIP: Eliminate the two options that describe
increased bowel activity.
43. CORRECT ANSWERS: 1, 3, 5. Answer 1 is correct because Content Area: Adult Health, Gastrointestinal; Integrated
the cold caloric test is a test of the oculovestibular reflex,
Process: Nursing Process, Assessment; Cognitive Level:
and absence of this reflex indicates severe brainstem injury.
Application; Client Need/Subneed: Physiological Integrity/
Answer 2 is incorrect because a GCS of less than 7 indicates
Physiological Adaptation/Pathophysiology
coma, not brain death. Answer 3 is correct because a flat EEG
2164_Ch01_001-058 29/03/12 12:18 PM Page 52

52 chapter 1 Orientation and Pre-Tests

46. CORRECT ANSWER: 4. Answer 1 is incorrect because Content Area: Child Health, Hematological; Integrated Process:
PRE-TEST

this is the wrong concentration of magnesium sulfate. The Nursing Process, Evaluation; Cognitive Level: Application;
usual concentration is 40 gm of magnesium sulfate in 1,000 Client Need/Subneed: Physiological Integrity/Basic Care and
mL of lactated Ringers solution. Answer 2 is incorrect Comfort/Nutrition and Oral Hydration
because, although monitoring vital signs are an important 49. CORRECT ANSWER: 3. Answer 1 is incorrect because
part of every nursing assessment for a client in labor, this genetic karyotyping is the name of the general test done for
client is being treated for preeclampsia, for which the vital analysis of chromosomes; it is not a specific diagnostic test.
signs need to be assessed more frequently than every hour. Answer 2 is incorrect because amniocentesis cannot be per-
Answer 3 is incorrect because the maintenance dose should
formed until after the first trimester. Answer 3 is correct
be approximately 2 gm/hr. The loading dose is usually 4 gm because chorionic villus sampling (CVS) can be performed
over a 30-minute period. Answer 4 is correct because a res- as early as about 9 weeks gestation, providing information
piratory rate of less than 12 breaths/min can be a result before the end of the first trimester. Answer 4 is incorrect
of magnesium toxicity. The nurse should discontinue the because ultrasonography cannot provide diagnostic chromoso-
infusion and notify the physician. mal information; it can only provide general screening for
TEST-TAKING TIP: Magnesium is a central nervous system abnormalities.
depressant and can cause respiratory failure at a toxic level. TEST-TAKING TIP: There is a difference between diagnostic
Watch the respiratory rate closely. and screening tests. Screening tests are clues to possible prob-
Content Area: Maternity, Antepartum; Integrated Process:
lems, but diagnostic tests provide absolute information to make
Nursing Process, Implementation; Cognitive Level: a specific diagnosis. Ultrasound is an example of a screening
Application; Client Need/Subneed: Physiological Integrity/ test, and should be eliminated when the question is asking for
Pharmacological and Parenteral Therapies/Parenteral/ a diagnostic test.
Intravenous Therapies Content Area: Maternity, Antepartum; Integrated Process:
47. CORRECT ANSWER: 3. Answer 1 is incorrect because a Nursing Process, Implementation; Cognitive Level: Application;
temperature this low is subnormal and would not be an accu- Client Need/Subneed: Physiological Integrity/Reduction of Risk
rate indication of antibiotic effectiveness. Answer 2 is incorrect Potential/Diagnostic Tests
because the O2 saturation should be 90% or higher. Answer 3 is 50. CORRECT ANSWER: 4. Answer 1 is incorrect because it
correct because a systolic BP of 90 mm Hg or higher signals supplies the parents with false hope that the diagnosis is inaccu-
stability of the client. Answer 4 is incorrect because a heart rate. The diagnosis would not have been presented to the
rate of 120 is high. A rate of 100 or lower would indicate client parents without thorough confirmation of the positive diag-
stability. nostic tests. Answer 2 is incorrect because it does not address
TEST-TAKING TIP: Look for a clinical indicator within the parents concerns. It is a closed-ended response that does
normal limits. not allow for further dialogue between the parents and the
Content Area: Adult Health, Respiratory; Integrated Process:
nurse. Answer 3 is incorrect because, while it may be accurate,
Nursing Process, Analysis; Cognitive Level: Application; it does not supply the parents with why this happened to their
Client Need/Subneed: Physiological Integrity/Physiological
infant. Answer 4 is correct because it is the response that
Adaptation/Alterations in Body Systems supplies the parents with the most facts/information. It tells
48. CORRECT ANSWER: 2. Answer 1 is incorrect because a the parents why this disease is present in their infant.
yogurt and fruit plate would not be a high-calorie lunch, TEST-TAKING TIP: Look at the two answers with statistical
which could contribute to weight gain while recuperation data (i.e., 60% and 1/3) and choose the lesser percentage.
from the injury takes place. Answer 2 is correct because Content Area: Child Health, Hematological; Integrated
diet is an important consideration in the treatment of Process: Teaching and Learning; Cognitive Level: Application;
hemophilia. Excessive body weight can increase the strain Client Need/Subneed: Physiological Integrity/Physiological
on affected joints, especially the knees, and predispose Adaptation/Pathophysiology
the adolescent to hemarthrosis. Consequently calories 51. CORRECT ANSWER: 1. Answer 1 is correct because
need to be supplied in accordance with energy require- antibiotic therapy is the most important aspect of treat-
ments. Answer 3 is incorrect because a slice of cheese pizza ment. Maintaining IV site patency and integrity is a key
and a tossed green salad would not be a high-calorie lunch, role for the RN and for the clients recovery. Answer 2 is
which could contribute to weight gain while recuperation incorrect because the antibiotics must be administered through
from the injury takes place. Answer 4 is incorrect because a a patent site before effectiveness can be evaluated. Answer 3 is
scoop of pasta salad and a cup of vegetable soup would not incorrect because this is not the first action, although mobility
be a high-calorie lunch, which could contribute to weight is important in improving respiratory function. Answer 4 is
gain while recuperation from the injury takes place. incorrect because IV therapy is the first priority, but the clients
TEST-TAKING TIP: The correct answer is the wrong food comfort is important as pain will affect respiratory status.
choice (i.e., what is not a good choice to eat). Relate calories TEST-TAKING TIP: This is a priority question. All of the
consumed to potential for additional injury. options may be correct but one is priority. The sooner the
2164_Ch01_001-058 29/03/12 12:18 PM Page 53

Answers/Rationales/Tips 53
antibiotics are started, the sooner the client will be ambulating especially those who might have a chronic disease process. The

PRE-TEST
and breathing comfortably. nurse does not need to question this order. Answer 4 is correct
Content Area: Adult Health, Respiratory; Integrated Process: because venipuncture for blood draws are usually preferred in
Nursing Process, Planning; Cognitive Level: Analysis; infants/children with hemophilia. There is usually less bleed-
Client Need/Subneed: Physiological Integrity/Physiological ing after venipuncture than after finger or heel punctures.
Adaptation/Illness Management TEST-TAKING TIP: The question calls for an order that is
not OK. Think prevent blood loss when answering this
52. CORRECT ANSWERS: 1, 2, 3. Answer 1 is correct because question.
decreased fetal movement may be an indication of fetal
Content Area: Child Health, Hematological; Integrated Process:
hypoxia, and the fetus should be evaluated using an external
Nursing Process, Evaluation; Cognitive Level: Application; Client
fetal monitor. Answer 2 is correct because contractions that are
Need/Subneed: Physiological Integrity/Reduction of Risk
regular, every 5 minutes, and lasting 60 seconds are usually
Potential/Potential for Complications of Diagnostic Tests/
indicative of active labor, and the client should come to the
Treatments/Procedures
hospital to be evaluated for labor. Answer 3 is correct because,
if the client thinks her amniotic fluid is leaking, she should be 55. CORRECT ANSWERS: 1, 2, 3, 4. Answer 1 is correct because
evaluated for rupture of membranes and labor. Answer 4 is calling the physician is appropriate when the clients tempera-
incorrect because increased fetal movement is a reassuring sign, ture is above 100.4F. Answer 2 is correct because encouraging
and the client does not need to come to the hospital to evaluate her to increase fluids is an appropriate intervention after a
the fetal well-being. Answer 5 is incorrect because contractions long labor and delivery process. Some of the increased tem-
that are 10 minutes apart, only lasting for 30 seconds, are usually perature may be due to dehydration. Answer 3 is correct
associated with the latent phase of labor, and the client does not because assessing the fundal height is a part of a routine
need to come to the hospital for evaluation of labor. postpartum assessment and should be performed. Answer 4 is
TEST-TAKING TIP: Look for pairs of answers that are opposite correct because assessing the lochia is a part of a routine
of each other and eliminate one of them in each pair. Select postpartum assessment and should be performed. Answer 5 is
Answer 1 and therefore eliminate Answer 4. Select Answer 2 incorrect because there is no reason given in this scenario for
and therefore eliminate Answer 5. encouraging the client to bottle-feed. Her increased temperature
Content Area: Maternity, Intrapartum; Integrated Process: is not a reason to choose bottle feeding over breastfeeding.
Nursing Process, Implementation; Cognitive Level: Analysis; TEST-TAKING TIP: This client has a risk factor for postpar-
Client Need/Subneed: Health Promotion and Maintenance/ tum infection, a lengthy labor, and an operative delivery. Her
Ante/Intra/Postpartum and Newborn Care increased temperature may be the first sign of infection.
Content Area: Maternity, Postpartum; Integrated Process:
53. CORRECT ANSWER: 4. Answer 1 is incorrect because any Nursing Process, Implementation; Cognitive Level: Analysis;
dressing that is wet could potentially cause more damage. Answer
Client Need/Subneed: Physiological Integrity/Physiological
2 is incorrect because a wet dressing could macerate the skin.
Adaptation/Alterations in Body Systems
Answer 3 is incorrect because this probably would not harm the
area, but observation of the site is easier without a dressing. 56. CORRECT ANSWERS: 2, 5, 6. Answer 1 is incorrect
Answer 4 is correct because no dressing is needed for a stage I because the legal causal connection includes indirect as well as
pressure ulcer because the skin is intact. Just turn the client direct cause. Answer 2 is correct because legal cause includes
frequently and keep pressure off the area. both harm caused directly by the actions of the nurse and
TEST-TAKING TIP: Know the depth of tissue involvement harm caused by others when the actions of the nurse know-
with each stage to select the type of dressing. ingly placed the client at risk of harm. Answer 3 is incorrect
Content Area: Adult Health, Integumentary; Integrated because anyone named in a lawsuit is entitled to a defense that
Process: Nursing Process, Implementation; Cognitive Level: shows that the actions were outside of the legal causal connec-
Application; Client Need/Subneed: Physiological Integrity/ tion. Answer 4 is incorrect because the involvement of the
Physiological Adaptation/Illness Management nurse would have to have some direct or indirect connection to
the harm produced. Answer 5 is correct because, if the nurses
54. CORRECT ANSWER: 4. Answer 1 is incorrect because hepati- actions are outside the standards of care and the actions
tis B vaccine should be given at birth, or definitely prior to dis-
cause harm to the client, then the nurses actions are within
charge from the birth hospital. There is no contraindication in
the legal causal connection. Answer 6 is correct because
beginning vaccinations for an infant with hemophilia. The nurse
harm caused by a nurse giving care under the influence of
does not need to question this order. Answer 2 is incorrect
narcotic medications is within the legal causal connection.
because a circumcision would involve a small amount of bleed-
TEST-TAKING TIP: Answers that include absolute words such as
ing, which could be difficult to control and might lead to hemor-
only (Answer 1) and any (Answer 4) are likely to be wrong.
rhage. It should be delayed until the disease is confirmed and the
Content Area: Management of Care, Legal; Integrated Process:
severity of the factor deficiency is determined. The nurse does
Nursing Process, Analysis; Cognitive Level: Application; Client
not need to question this order. Answer 3 is incorrect because s
Need/Subneed: Safe and Effective Care Environment/
etting up a series of well-infant appointments at the time of dis-
Management of Care/Legal Rights and Responsibilities
charge from the birth hospital is a requirement for all infants and
2164_Ch01_001-058 29/03/12 12:19 PM Page 54

54 chapter 1 Orientation and Pre-Tests

57. CORRECT ANSWER: 3. Answer 1 is incorrect because an burn wound tissue shortens because of flexion of muscles in
PRE-TEST

EEG (electroencephalogram) is a study of the electric activity of wound healing. Contractures will need to be prevented with
the brain and would not detect lead levels in the toddlers body. splints or traction. Answer 3 is incorrect because large-bore
Answer 2 is incorrect because a CBC (complete blood count) is central line IV catheters via cutdown can be placed in any
a general study of the hematological system, and would not detect major vessel. Answer 4 is incorrect because a sterile dressing
lead levels in the toddlers body. Answer 3 is correct because a can be applied under the BP cuff, or a Doppler can be used, or
toddler with a sharp rise in blood lead level (BLL) should an arterial catheter can be inserted.
have an abdominal x-ray to determine the presence of lead TEST-TAKING TIP: Visualize the burn location and how
chips, which would be a result of repeated ingestion of lead- treating the wound would interfere with the care need.
based paint chips. If lead chips are discovered in the stomach Content Area: Adult Health, Integumentary; Integrated Process:
or small intestine, the bowel is decontaminated using a Nursing Process, Analysis; Cognitive Level: Application;
cathartic before beginning chelation therapy. Answer 4 is Client Need/Subneed: Physiological Integrity/Physiological
incorrect because a chest x-ray is a general study of the respirato- Adaptation/Alterations in Body Systems
ry system, and would not detect lead levels in the toddlers body 61. CORRECT ANSWER: 4. Answer 1 is incorrect because the
or reveal the presence of lead chips. use of IV N-acetylcysteine (Mucomyst) is still investigational.
TEST-TAKING TIP: Think: Lead poisoning is from ingestion, Answer 2 is incorrect because N-acetylcysteine (Mucomyst) is
ingestion is related to the GI system, the GI system fills the not given IM. Answer 3 is incorrect because the offensive odor
abdomen; therefore, select an abdominal x-ray option. of N-acetylcysteine (Mucomyst) would not be concealed by
Content Area: Child Health, Poisoning; Integrated Process:
water. Answer 4 is correct because N-acetylcysteine
Nursing Process, Analysis; Cognitive Level: Application; Client (Mucomyst) is usually given orally, but is first diluted in
Need/Subneed: Physiological Integrity/Reduction of Risk
fruit juice or soda because of the antidotes offensive odor
Potential/Diagnostic Tests (rotten eggs).
58. CORRECT ANSWER: 3. Answer 1 is incorrect because TEST-TAKING TIP: Think of how to best conceal the drugs
straining does not affect intake and output. Answer 2 is incor- odor...juice is more effective than water.
rect because the voided urine is strained. Straining does nothing Content Area: Child Health, Poisoning; Integrated Process:
to promote voiding. Answer 3 is correct because straining is Nursing Process, Implementation; Cognitive Level: Application;
done to collect a renal stone for analysis of composition. Client Need/Subneed: Physiological Integrity/Pharmacological
Answer 4 is incorrect because a urinalysis can be completed and Parenteral Therapies/Medication Administration
even with blood clots present. 62. CORRECT ANSWER: 2. Answer 1 is incorrect because giv-
TEST-TAKING TIP: Look for the answer that describes the ing the mother pictures of her baby is appropriate only when an
need to find something. Three of the options imply straining infant is too ill for physical contact or the mother is too ill to
will improve urine quantity or quality. visit in the nursery. Answer 2 is correct because the combina-
Content Area: Adult Health, Renal; Integrated Process:
tion of direct physical contact between mother and baby and
Nursing Process, Analysis; Cognitive Level: Comprehension; participation in the infants care helps the mother in recog-
Client Need/Subneed: Physiological Integrity/Physiological
nizing her infant as a distinct individual who is yet a part of
Adaptation/Alterations in Body Systems her. This process is the beginning of attachment. Answer 3 is
59. CORRECT ANSWER: 4. Answer 1 is incorrect because con- incorrect because direct physical contact between mother and
tractions that are less than 2 minutes apart are uterine tetany, baby is most likely to promote attachment. Looking through
not a sign of normal labor. Answer 2 is incorrect because back the window doesnt replace touching and caring for her infant.
pain and increased vaginal discharge are not the signs of normal Answer 4 is incorrect because descriptions of the infant do not
onset of labor. Answer 3 is incorrect because frequent urination replace physical contact between mother and baby.
is a sign of urinary tract infection, not a sign of the normal TEST-TAKING TIP: Facilitating attachment when an infant
onset of labor. Answer 4 is correct because contractions that is in the nursery is an important part of caring for the
are 5 minutes apart and regular for an hour are the signs of mother and helping her to cope with an infant who is ill.
normal onset of labor. Eliminate looking, seeing, and helping. Select direct contact
TEST-TAKING TIP: Eliminate Answer 3 because it includes with the baby.
painful urination, which is never a sign of normal labor. The Content Area: Maternity, Newborn; Integrated Process: Nursing
other two options are not signs of typical labor. Process, Implementation; Cognitive Level: Application;
Content Area: Maternity, Antepartum; Integrated Process: Client Need/Subneed: Health Promotion and Maintenance/
Nursing Process, Evaluation; Cognitive Level: Analysis; Developmental Stages and Transitions
Client Need/Subneed: Health Promotion and Maintenance/
63. CORRECT ANSWER: 1. Answer 1 is correct because the
Ante/Intra/Postpartum and Newborn Care ingested toxin is removed by lavage from the stomach,
60. CORRECT ANSWER: 2. Answer 1 is incorrect because and then the activated charcoal is instilled to aid in the
infection is not a greater concern. Answer 2 is correct because absorption and removal of any medication left after the
2164_Ch01_001-058 29/03/12 12:19 PM Page 55

Answers/Rationales/Tips 55
gastric lavage is completed. To lavage means to wash the Answer 6, performing a procedural pause, is the final step

PRE-TEST
stomach and remove the saline when the lavage is com- before beginning the epidural to ensure that this is the correct
pleted. Answer 2 is incorrect because the ingested toxin must procedure and the correct client.
be first removed by lavage, and then the activated charcoal is TEST-TAKING TIP: Read the answers carefully. Eliminate
instilled to aid in the absorption and removal of any medica- Answer 5 completely because an informed consent is not the
tion left after the gastric lavage is completed. Answer 3 is role of the nurse. The act of obtaining a client signature on the
incorrect because the saline solution contains toxic particles consent form is not the same as informed consent.
which must be removed before the activated charcoal is Content Area: Maternity, Intrapartum; Integrated Process:
instilled, which will help to absorb and remove any medica- Nursing Process, Implementation; Cognitive Level: Analysis;
tion that is left after the gastric lavage is completed. Answer 4 Client Need/Subneed: Physiological Integrity/Reduction of Risk
is incorrect because gastric lavage and activated charcoal are Potential/Potential for Alterations in Body Systems
used together in a sequential pattern when attempting to
66. CORRECT ANSWER: 3. Answer 1 is incorrect because it
remove ingested toxins from the stomach.
sounds as though the nurse assumes the child has not
TEST-TAKING TIP: Focus on the contradictory options here.
thought about the potential hazards of the hike. This state-
Since Answer 1 is the opposite of Answer 2, one of them is
ment may block effective communication between the nurse
wrong. Look at the stem that happens to state the correct
and the teenager. Answer 2 is incorrect because the statement
order: wash and then administer . . . select Answer 1!
implies that the nurse is withholding approval until after a
Content Area: Child Health, Poisoning; Integrated Process:
physicians review. This statement prevents effective exchange
Nursing Process, Implementation; Cognitive Level: Application;
of communication between the nurse and the teenager.
Client Need/Subneed: Physiological Integrity/Reduction of Risk
Answer 3 is correct because the nurse first provides praise
Potential/Therapeutic Procedures
for a positive health behavior, then opens up the conver-
64. CORRECT ANSWER: 1. Answer 1 is correct because the sation into a discussion about additional steps that
bowel should be functioning by 72 hours. Answer 2 is should be taken to promote the childs health. Answer 4 is
incorrect because there may be faint bowel sounds, but the incorrect because, although the nurse may want to know
bowel will not be fully functional. Answer 3 is incorrect how the parents feel about the hike, it is better for the nurse
because the effects of general anesthesia on the bowel are to first explore the teenagers plans and attitudes. If the
still present. Answer 4 is incorrect because the question asks nurse specifically asks about the parents thoughts, the teen
for full function. Gas is not full function. Some faint bowel may feel that the nurse is seeking to block participation in
sounds may be present. the activity.
TEST-TAKING TIP: Think about the magnitude of the TEST-TAKING TIP: Be wary of selecting answers that sound
surgery. Choose the longest time period because the effects of condescending or judgmental. Remember that a teenager with
general anesthesia, pain, medications, and immobility will a chronic health condition should have a reasonable sense of
delay the return of peristalsis. After 3 days (72 hours), there possible activity limitations while also having a developmental
would be concern for paralytic ileus if normal bowel function need to be like peers. The best strategy for the nurse is to
did not return. explore with the teen the challenges of participating in a moun-
Content Area: Adult Health, Gastrointestinal; Integrated tain hike and to work together to meet mutual health goals.
Process: Nursing Process, Implementation; Cognitive Level: Content Area: Child Health, Hematological; Integrated
Application; Client Need/Subneed: Physiological Integrity/Basic Process: Nursing Process, Implementation; Cognitive Level:
Care and Comfort/Elimination Application; Client Need/Subneed: Physiological
Integrity/Reduction of Risk Potential/Potential for
65. CORRECT ANSWERS: 1, 2, 3, 4, 6. Answer 1 is correct Alterations in Body Systems
because the client must be sitting up before beginning the
epidural procedure. Answer 2 is correct because an IV bolus 67. CORRECT ANSWER: 4. Answer 1 is incorrect because this
must be infused before beginning an epidural procedure. is an appropriate action. Answer 2 is incorrect because the
Beginning the infusion is the first step because it takes the client recognizes that there has been a problem. Answer 3 is
longest time and must be completed before the epidural pro- incorrect because this is not a concern. Answer 4 is correct
cedure can be started. Answer 3 is correct because baseline because there may be variations in the effects of a name
vital signs must be recorded before beginning the procedure brand versus a generic brand. The physician should be
so that any changes in vital signs can be treated promptly. advised.
Answer 4 is correct because allowing a client to empty her TEST-TAKING TIP: The question is asking for the statement
bladder will make the procedure more comfortable for the that is a problem. When a decision is based on cost, consider
client. In addition, after the epidural procedure, the client the risk to the client.
will lose sensation to her bladder and may be unable to void. Content Area: Adult Health, Endocrine; Integrated Process:
Answer 5 is incorrect because nurses are not responsible for Nursing Process, Evaluation; Cognitive Level: Analysis; Client
obtaining an informed consent. It is the responsibility of the Need/Subneed: Health Promotion and Maintenance/Self-care
physician or anesthesia provider to provide informed consent.
2164_Ch01_001-058 29/03/12 12:19 PM Page 56

56 chapter 1 Orientation and Pre-Tests

68. CORRECT ANSWER: 4. Answer 1 is incorrect because, heart defect. Answer 7 is correct because right-to-left blood
PRE-TEST

while the child should be placed on NPO precautions to pre- flow is found in tetralogy of Fallot.
vent further damage from the corrosive substance to the esoph- TEST-TAKING TIP: A helpful mnemonic for remembering
agus, it is not the nurses first priority. Answer 2 is incorrect cyanotic heart defects is that the most common types begin with
because, while the child is undoubtedly experiencing pain and the letter T.
will require an analgesic, it is not the nurses first priority. Content Area: Child Health, Cardiovascular; Integrated Process:
Answer 3 is incorrect because corrosive substances burn as they Nursing Process, Analysis; Cognitive Level: Comprehension;
go down the esophagus and will burn coming back up the Client Need/Subneed: Physiological Integrity/Reduction of Risk
esophagus with vomiting. Inducing vomiting (by any means) is Potential/Potential for Alterations in Body Systems
contraindicated when dealing with the ingestion of corrosive 71. CORRECT ANSWER: 4. Answer 1 is incorrect because the
substances. Answer 4 is correct because maintaining a client has the right to refuse. Answer 2 is incorrect because this
patient airway is always the nurses first priority. In the response does not address the reason for refusal. Answer 3 is
presence of swelling, the loss of a patent airway is both a incorrect because this is a threatening response and not thera-
real possibility and the nurses first concern. peutic. Answer 4 is correct because this response attempts to
TEST-TAKING TIP: When dealing with corrosive substances, understand the clients actions.
remember that they burn both ways. Select airway as first TEST-TAKING TIP: Choose the option that seeks elaboration.
priority in the ABCs (Airway, Breathing, Circulation). Content Area: Geriatrics, Hematological; Integrated Process:
Content Area: Child Health, Poisoning; Integrated Process:
Communication and Documentation; Cognitive Level:
Nursing Process, Implementation; Cognitive Level: Application; Comprehension; Client Need/Subneed: Psychosocial
Client Need/Subneed: Safe and Effective Care Environment/
Integrity/Therapeutic Communications
Management of Care/Establishing Priorities
72. CORRECT ANSWER: 2. Answer 1 is incorrect because
69. CORRECT ANSWERS: 1, 2, 3, 5. Answer 1 is correct the fundal height is approximately 2 fingerbreadths below
because administering oxytocin can decrease the bleeding the umbilicus immediately after delivery. The fundal height
after delivery. Answer 2 is correct because massaging the increases to 1 to 2 fingerbreadths above the umbilicus within
fundus is the first step to decrease the bleeding. Making 12 hours after delivery. Answer 2 is correct because fundal
sure the fundus is firm and not deviated is an important height increases to 1 to 2 fingerbreadths above the
postpartum assessment. Answer 3 is correct because a full umbilicus within 12 hours after delivery. Answer 3 is
bladder can impede the contraction of the uterus and can incorrect because fundal deviation to the right side indicates
cause increased bleeding. Having the client empty her blad- a full bladder and risk for uterine atony and hemorrhage.
der can help decrease uterine bleeding. Answer 4 is incorrect Answer 4 is incorrect because the fundal height is 3 to 4
because tocolytics are drugs that reduce contractions in preterm fingerbreadths below the umbilicus by the fourth to fifth
labor clients. Tocolytics would be contraindicated in a client postpartum day.
who is postpartum and actively bleeding. Answer 5 is correct TEST-TAKING TIP: When the options have similarities
because vital signs are an important assessment for a client (in this case, firm fundus), ignore that part of the answer
who is postpartum and having a large amount of bleeding. and focus on the dissimilarities between the choices.
Changes in the heart rate and blood pressure can indicate Content Area: Maternity, Postpartum; Integrated Process:
significant blood loss. Nursing Process, Assessment; Cognitive Level: Application;
TEST-TAKING TIP: Remember that tocolytics are used for Client Need/Subneed: Health Promotion and Maintenance/
preterm labor to decrease hyperstimulation in labor, and not Ante/Intra/Postpartum and Newborn Care
used in clients who are postpartum.
Content Area: Maternity, Postpartum; Integrated Process: 73. CORRECT ANSWER: 2. Answer 1 is incorrect because drug
Nursing Process, Implementation; Cognitive Level: Analysis; therapy needs the directive to take the medication with meals to
Client Need/Subneed: Physiological Integrity/Physiological decrease the release of gastric acid or to accelerate gastric empty-
Adaptation/Alterations in Body Systems ing. Answer 2 is correct because giving the client information
also gives the client control over the condition. Answer 3 is
70. CORRECT ANSWERS: 2, 4, 5, 7. Answer 1 is incorrect incorrect because the clients goals and physicians goals need to
because an atrial septal defect causes left-to-right blood flow be the same for the plan of care to be successful. Answer 4 is
and is a noncyanotic heart defect. Answer 2 is correct incorrect because a referral to the interdisciplinary team may be
because right-to-left blood flow is found in transposition needed if further teaching is needed, or to add reinforcement.
of the great vessels. Answer 3 is incorrect because coarctation TEST-TAKING TIP: Compliance with treatment requires the
of the aorta causes left-to-right blood flow and is a noncyan- clients understanding and agreement.
otic heart defect. Answer 4 is correct because right-to-left Content Area: Adult Health, Gastrointestinal; Integrated
blood flow is found in tricuspid atresia. Answer 5 is cor- Process: Nursing Process, Implementation; Cognitive Level:
rect because right-to-left blood flow is found in truncus Application; Client Need/Subneed: Health Promotion and
arteriosus. Answer 6 is incorrect because a patent ductus Maintenance/Principles of Teaching and Learning
arteriosus causes left-to-right blood flow and is a noncyanotic
2164_Ch01_001-058 29/03/12 12:19 PM Page 57

Answers/Rationales/Tips 57
74. CORRECT ANSWERS: 1, 3, 6. Answer 1 is correct because, incorrect because a sterile vaginal examination is not an

PRE-TEST
after taking action to treat the tet spell, the parents essential part of assessing a client with HELLP syndrome.
should activate emergency medical services (EMS). The client is not complaining of abdominal pain or contrac-
Answer 2 is incorrect because the child should be placed on tions; therefore, performing a vaginal examination is not
the side, not upright in a chair. Answer 3 is correct because, necessary. Answer 3 is incorrect because sterile speculum
before the child might have a tet spell, parents should examination is not an essential part of assessing a client with
first get trained in CPR, in order to be ready should their HELLP syndrome. The client is not complaining of abdomi-
child need such intervention. Answer 4 is incorrect because nal pain, contractions, or leaking amniotic fluid; therefore,
nothing should be placed in the childs mouth during a tet performing a sterile speculum examination is not necessary.
spell. Aspirin is not used to treat this condition; it is given for Answer 4 is incorrect because a fern test is a test for rupture
the treatment of heart attack. Answer 5 is incorrect because of amniotic membranes. The client is not complaining of
coughing will not reverse a tet spell; it is the treatment for leaking fluid; therefore, a fern test is not necessary. Answer 5
supraventricular tachycardia. Answer 6 is correct because the is correct because this client has HELLP syndrome, with
parents first response during a tet spell should be to a decreased hematocrit, elevated liver function tests, and
place the child in the position that will increase systemic low platelets. Urine protein is an essential assessment for
vascular resistance and prevent fainting. a client with HELLP syndrome.
TEST-TAKING TIP: The best sequence of steps is: CPR TEST-TAKING TIP: The key to answering this question is to
training positioning calling for help. determine what the diagnosis is for this client, which is based
Content Area: Child Health, Cardiovascular; Integrated Process: on vital signs and laboratory values. Choosing the correct
Nursing Process, Implementation; Cognitive Level: Application; answers depends on recognizing that this client has HELLP
Client Need/Subneed: Physiological Integrity/Physiological syndrome. Remember that HELLP stands for: decreased
Adaptation/Hemodynamics hematocrit, elevated liver function tests, and low platelets.
Content Area: Maternity, Antepartum; Integrated Process:
75. CORRECT ANSWERS: 1, 5. Answer 1 is correct because Nursing Process, Assessment; Cognitive Level: Analysis;
this client has HELLP syndrome, with a decreased hemat- Client Need/Subneed: Physiological Integrity/Physiological
ocrit, elevated liver function tests, and low platelets. Adaptation/Alterations in Body Systems
Deep tendon reflexes are an essential part of an assess-
ment for a client with HELLP syndrome. Answer 2 is
2164_Ch01_001-058 29/03/12 12:19 PM Page 58
2164_Ch02_059-068 29/03/12 12:19 PM Page 59

CHAPTER 2

Guidelines and Tips


for International Nurses
and Repeat Test-Takers
Sally Lambert Lagerquist

59
2164_Ch02_059-068 29/03/12 12:19 PM Page 60

60 chapter 2 Guidelines and Tips for International Nurses and Repeat Test-Takers

This chapter addresses the special concerns of candidates 2. Using too many study aids, from too many different
who have taken the test more than once, as well as inter- resources. You wind up finding that theory and
national nurses who are graduates of nursing programs questions in books contradict each other, and
outside of the United States. there is no one to referee as to which books
Since 1977, the authors of this book have had a long have the right information when books disagree.
history of success in helping repeat test-takers and interna- 3. Reviewing only with questions and answers in books
GUIDELINES & TIPS

tional nurses to pass because we have developed an will not help you to systematically cover all the
approach that works! We give you guidelines, strategies, and theory that you need to review.
tips that give you the skills and confidence to PASS. 4. Reviewing primarily with computer tests is too
This chapter will help put you in the frame of mind of time-consuming. You can cover more questions in
an examination question writer and an entry-level nurse less time by using a book. In addition, when you
(which is what is tested on the examination), so that logic, get through the thousands of questions on the
practice, and a systematic approach can lead you to the disk or computer test, you cannot be sure that
best answer(s). you have reviewed all that you need to know in
each subject area, because the material is both
limited and fragmented (i.e., it is not organized
Tips for NCLEX-RN Candidates by concepts or systems). Compare this with a
book that has detailed explanations for wrong
Who Must Repeat the Examination and correct answers, where so much more helpful
Purpose of This Section information can be seen at a glance on each page.
5. Taking the examination when you are not ready,
Repeat test-takers have somewhat different needs, differ-
just because you set an examination date.
ent starting points, and a different time frame from
6. Going into the examination with little or no
those who are first-time NCLEX-RN candidates: that
review because of life circumstances (e.g., illness,
is, figuring out why they did not pass, what the
moving, vacations, marriage, baby, job).
Candidate Performance Report (CPR) that they
received means, and what to do next. It is Not a Matter of How Much
Also refer to Chapter 1Orientation in the sections
on How to Use This Review Book as a Study Guide: You Study for the Test, but How
Where to Begin, While Reviewing, After Reviewing; You Use the Review Material
Key Points to Recall for Better Study; Memorization: At this point in your examination prep, do not start with
Purpose and Strategies; the section providing information page 1 and go through page 800+. This is usually over-
about the structure and format of the computer-adaptive whelming and not confidence-building. You need a focused
NCLEX-RN; and the section on how to Prepare review, starting with your weakest area and leaving what
Physically and Mentally. you feel most comfortable with toward the end of your
reviewing. Discern if you mostly need a review of theory,
What is the Difference Between test-taking strategies, or both. This will determine where
Taking This Examination for the you start.
First Time and Repeating It? How Do You Know What You
You are ahead of the game! You have already received Need to Review?
feedback about your examination-related strengths and
This is based on knowing what the NCLEX-RN
weaknesses. You know what the examination is really like
Candidate Performance Report (CPR) means. Look at
and what areas to emphasize. You also know what study
the client needs/subneeds listed in the CPR that came
methods did not work for you. Look at the experience you
with your NCLEX results. Look at the areas where the
had in taking the examination as a dry run for helping
boldface print states that your performance was below
you to pass next time.
the passing standard or near the passing standard (which
indicates the amount of improvement you need). This
What are Some Risk Factors means that these are the areas (#1 through #8) that
for Not Passing the First Time? should be the major focus of your content review to
1. It can readily be what you used to study. Often, PASS NEXT TIME (e.g., Management of Care, Safety
it is a matter of what review materials you used and Infection Control, etc.).
that were not as helpful as other resources could If the computer stopped when you had taken only 75
be. Remember, some study aids are better than to 100 questions, this probably means that you have
others. For next time, get a fresh start. Use significant deficiencies and gaps in certain areas of
different review materials. nursing content.
2164_Ch02_059-068 29/03/12 12:19 PM Page 61

A Guide for Graduates of Nursing Schools Outside the United States 61


If the computer stopped when you had taken more than How Will You Know That You
100 but less than 200 questions, this usually means that you
have some identifiable areas where you need to review are Well-Prepared and Ready
certain content, as well as to improve your test-taking to Retake the Examination?
skills. Appendix F will show you where to go in this book When you are getting 80% correct in all of the questions
to read in your areas where improvement is needed. in this book and on the accompanying disk, you are ready!

GUIDELINES & TIPS


If the computer stopped when you had taken between Most of all, you are ready when you change the tape
200 and 265 (the maximum number) questions, you were recorder in your mind that keeps saying I have failed
close to passing. Your problem area may very well be a to I havent PASSED yet, and from I hope I pass to
difficulty with or an inconsistency in answering the I WILL PASS!
questions when more than one answer could be correct.
In this case, help is on the way! Go right to the Test-
Taking Tips and Guidelines: Sixty Strategies to Use A Guide for Graduates of Nursing
in Answering Questions section in this chapter. These
test-taking strategies are designed to help you to pull the
Schools Outside the United States
question apart to show you: International nurses who are educated outside the
United States can use this book to serve their special
1. How to choose the best answer when all four needs:
options could be right.
2. How to narrow your choices down to two possible 1. To check their experiences, skills, and knowledge
answers. for equivalency to those of nursing candidates
3. How to decide between two options. from U.S. programs, in terms of their ability to
4. What to do when you havent a clue! deliver effective and safe health care as determined
by U.S. standards of practice.
You can learn our proven test-taking techniques for 2. To identify cultural differences in perception of
success on the NCLEX-RN. By following our guidelines client needs and nursing responses and actions
before you repeat the examination, you can get quick feed- (see Chapter 3).
back on your examination-related problem areas in order 3. To learn about the structure and format of the
to better predict your next NCLEX results before you examination (see Chapter 1).
take it (i.e., whether you are at risk for not passing). 4. To learn how to prepare for the examination
What to Do to Pass Next Time (see Chapter 1).
5. To practice taking tests made up of multiple-choice
STEP 1: ASSESS yourself. In this book, take the two
and fill-in-the-blank questions (see questions and
Pre-tests and two Final tests all at once. The book
test-taking tips in Chapters 1, 3, 4, 5, 6, 7, 8, 9,
questions are also included with additional questions
10, 11, and 12).
on the CD if you prefer to test electronically.
6. To assess the level of language difficulty in reading
STEP 2: Score yourself at the end. Determine the
the examination.
percentage of questions that were correct in each of
7. To become skilled in test-taking techniques
these integrated tests.
(see following section).
STEP 3: In any of the tests where you scored less than
80% correct, tally the client need subcategories that If you are an international nurse and wish to compare
your wrong answers represented. Each question is your preparation with that of U.S.-educated nurses, you
coded by various categories; you need to focus primarily will find that the practice questions with detailed
on the client subneed category. The codes are found answers and test-taking tips for each question that are
after the rationale paragraph for each question and included at the end of each major content chapter can
test-taking tips in the answer section of each test. For serve as an effective self-assessment guide. If you find
example, if most of the questions that you missed were that you need further in-depth study after taking the
represented by the client need Management of Care, practice tests and reviewing the essential content presented
Appendix F lists the pages in this book where this in outline format and in tables and figures throughout
content is covered. You should be sure to review this the book, you may wish to seek assistance from online or
content before proceeding. Go to Appendix G and mobile review courses for self-paced review on DVD,
look up where there are specific questions in the book CD-ROM, or mobile device. In addition, Chapter 8
that focus on your most problematic area(s). may help you review drugs used in the United States that
STEP 4: Re-test yourself after reviewing the content may be called by other names outside the United States.
areas in which you demonstrated a deficit. Score your- Cultural differences may be one cause of incorrect
self. Try to achieve a correct score of at least 80%. answers stemming from your different perception of
You can do it! clients needs or nursing action. In addition, Chapter 3
2164_Ch02_059-068 29/03/12 12:19 PM Page 62

62 chapter 2 Guidelines and Tips for International Nurses and Repeat Test-Takers

contains the code of ethics and standards of nursing action or factor that you think should be first, then iden-
practice and legal aspects that pertain to nursing in the tify which should be last.
United States. We suggest that the international nurse We think that the following pointers will assist you to
become familiar with these sections to determine what is narrow down your choices systematically and intelligently.
emphasized in this country.
The Orientation section (see Chapter 1) is designed to 1. Always, all, everyone, never, none, only, every, must.
Answers that include global words such as these
GUIDELINES & TIPS

help the international nurse know what to expect during


the examination, what the examination structure and should be viewed with caution because they imply
format will be like, what content will be covered, and that there are no exceptions. There are very few
how it will be scored. It will also help the examination instances in which a correct answer is that
candidate learn how to study for the test, how to take a absolute.
multiple-choice test, and how to reduce test-taking Example: Nurses should exercise caution in
anxiety. interviewing clients who have an alcohol
If you are concerned about your ability to read and use/dependency problem because:
comprehend English as it might be used in the examina- 1. These clients always exaggerate.
tion, first check yourself by looking at the examination 2. These clients are never consistent.
questions in this book. The terms used here are those
used in the health care field and are considered to be Any such suggested answer should be looked at with
those a nurse needs to know and use. If the vocabulary is care because any exception will make that a false
different from yours or is difficult, look at the list of response. A more reasonable answer to the preced-
common terms in Appendix C. You may also want to ing might be Clients who have an alcohol use/
consult local colleges for courses in English as a second dependency problem may not be reliable historians.
language (ESL courses). 2. Broadest, most comprehensive answers. Choose the
If you are not familiar with or proficient in taking answer that includes all the others, which is
examinations with multiple-choice questions when more referred to here as the umbrella effect.
than one answer looks good, the approximately 3530 test
questions in this book and the accompanying disk will Example: A main nursing function in group
provide you with sufficient practice for taking such a test. therapy is to:
The following section on test-taking tips was specifically 1. Help clients give and receive feedback in the group.
included to help you choose the best answer(s) by narrow- 2. Encourage clients to bring up their concerns.
ing your choices to increase your chances of selecting the 3. Facilitate group interaction among the members.
best answer(s). 4. Remind clients to address their comments to the
group.
Test-Taking Tips and Guidelines: Number 3 is the best choice because all the other
choices fall under it.
Sixty Strategies to Use in Answering
3. Test how reasonable the answer is by posing a
Questions specific situation to yourself. For example, the
In a standard four-answer multiple-choice question, if you question might read, The best approach when
can systematically eliminate false answers, you can reduce interviewing children who have irrational fears is
the four-answer question to a two-answer one and thereby to: (1) Help them analyze why they feel this way.
make your chances as good as those in the true-false type Ask yourself if it is reasonable to use Freudian
of question; that is, the odds will favor your guessing half analysis with 2-year-old children.
of the answers correctly. Other questions will ask you to 4. Focus on the client. Usually the reason for doing
select all that apply from five to seven options by click- something with a client is not to preserve the
ing in the correct boxes to select the answers (multiple- good reputation of the doctor, hospital, or nurse,
response questions). or to enforce rules. Wrong choices would focus on
In the alternate items, you may see charts and tables, or enlisting the clients cooperation for the purpose
you may find pictures and graphs requiring identification of fulfilling orders or because it is the rule. On
of a correct location (hot spot) by either point-and-click seeing a client out of bed against orders, instead
or fill-in-the-blank. In the items requiring a calculation, of just saying, Its against doctors orders for you
determine which numbers are needed to figure out the to get up, you might better respond by focusing
correct numerical answer and use the drop-down calcula- on how the client is reacting to the restriction on
tor to fill in the blank. Another alternate item is the drag mobility, by saying, for example, I can see that
and dropordered response, in which you are asked to you want to get up and that it is upsetting to you
arrange all the correct responses in priority order: identify to be in bed now. Let me help you get back
2164_Ch02_059-068 29/03/12 12:19 PM Page 63

Test-Taking Tips and Guidelines: Sixty Strategies to Use in Answering Questions 63


to bed safely and see what I can do for you. 13. Look for the average, acceptable, safe, common,
Examples of client-centered options are: typical, garden variety responses, not the excep-
acknowledging, offering a choice, and determin- tion to the rule, esoteric, or controversial responses.
ing preferences. 14. Eliminate the response that may be the best for a
5. Eliminate any answer that takes for granted that physician to make. Look for an RN role-appropriate
anyone is unworthy or ignorant. For example, in psychosocial response; for example, psychiatrists

GUIDELINES & TIPS


the question, The client should not be told the analyze the past, and nurses in general focus on
full extent of her condition because . . ., a poor present feelings and situations.
response would be, . . . she would not under- 15. Look for similarities and groupings in responses
stand. Choose an answer that focuses on the and the one-of-a-kind key idea in multiple-choice
client as a worthy human being. responses.
6. When you do not know the best answer, and
Example: At which activity would it be important
need to guess, look for the answer that may be
to protect the client who is on phenothiazines from
different from the others. For example, if all choices
the side effects of this drug?
but one are stated in milligrams and the exception
1. Sunday church services.
reads 1 g, that choice may be a distractor or the
2. A twilight concert.
best choice.
3. A midday movie in the theater.
7. Read the question carefully to see if a negative
4. A luncheon picnic on the hospital grounds.
modifier is used. If the question asks, Which
of the following is least helpful, be sure to gear Choices 1, 2, and 3 all involve indoor activities.
your thinking accordingly. Emphasize a key word Choice 4 involves outdoor exposure during the
such as least, contraindicated, or avoid as you read height of the suns rays. Clients need to be protect-
the questions. In this type of question, a correct ed against photosensitivity and burns when on
answer may reflect something that is false. phenothiazines.
8. Do not look for a pattern in the correct answers.
16. Be sure to note whether the question asks for
If you have already selected option 3 for several
what is the first or initial response to be made or
questions in a row, do not be reluctant to choose
action to be taken by the nurse. The choices listed
option 3 again, if you think that it is the correct
may all be correct, but in this situation selecting
response.
the response with the highest priority is important.
9. Look for the choices that you know either are
If the question asks for an immediate action,
correct or may be incorrect. You can save time and
probably all answers are correct and you need to
narrow your selection by using this strategy. This
choose the priority answer. Identify words that set
strategy is also useful when the question requires
a priority: best, main, primary, greatest, most.
you to select all options that apply. Read each
17. When you do not know the specific facts called
option and determine if it is correct or not; if
for in a question, use your skills of reasoning; for
correct, click on the box to the left of the option.
example, when an answer involves amounts or
There is no partial credit if you select some but
time (mainly numbers) and you do not know the
not all that apply.
answer and cannot find any basis for reasoning
10. In eliminating potentially wrong psychosocial
(all else being equal), avoid the extreme responses
answers, remember to look for examples of what
(the highest or lowest numerical values).
has been included in the nontherapeutic responses
18. Give special attention to questions in which each
list in Chapter 10 (e.g., denying feelings, false
word counts. The purpose of this type of question
reassurance, changing the subject).
may be not only to test your knowledge but also
11. Wrong choices tend to be either very brief or very
to see if you can read accurately and find the
long and involved.
main point (e.g., early vs. late sign of shock). In
12. Better psychosocial nursing responses to select are
such questions, each answer may be a profusion
those responses that (a) focus on feelings (unless
of words, but there may be one or two words
safety is at stake!): How did that make you feel?
that make the critical difference. If the option
(b) reflect the clients comments: You say that
has several aspects, all the parts must be correct
made you angry; (c) communicate acceptance of
for that answer to be correct. If you can eliminate
the client by the nurse rather than criticism or a
one aspect in an answer, you can eliminate the
value judgment; (d) acknowledge the client: I see
other options with that aspect.
that you are wincing; and (e) stay in the here-
19. All else being equal, select the response that you
and-now: What will help now? Examples of
best understand. Long-winded statements are
better choices can be found in the therapeutic
likely to be included as distractors and may be a
responses list in Chapter 10.
2164_Ch02_059-068 29/03/12 12:19 PM Page 64

64 chapter 2 Guidelines and Tips for International Nurses and Repeat Test-Takers

lot of words signifying little or nothing, such as Find out


criteria involved in implementing conceptual Identify
referents for standardizing protocol. You may Look
want to eliminate unusual or highly technical Monitor
language. Relate the situation to something that Observe
is familiar to you. Obtain information
GUIDELINES & TIPS

20. Select all that apply questions require that all Recognize
correct responses must be selected to get credit. 27. Isolate the verbs from the rest of the question
First pick out key words (write them down, if (e.g., ask is better than tell, give, or ignore).
that is helpful to you). Translate, into your own 28. Do not overlook the obvious answer: KIS
words, the gist of what is asked in the question. (Keep It Simple). For example, the best answer
You might close your eyes at this point and see for what to do when there is a malodor in the
if the answer pops into mind. Then, skim the room of a client with a colostomy is to check
answer choices, looking for the response that the stoma for fecal leakage. If it smells like feces,
corresponds to what first came into your mind. check for feces.
Key ideas or themes to look for in psychosocial 29. As you read what is given for assessment findings
responses have been covered in this sectionfor (e.g., signs and symptoms) ask yourself: Is this
example, look for a feeling response, acceptance, OK? or is it not OK?
acknowledgment of the client, and reflection. 30. When two options are correct, choose the one
21. Look for the best answer, not the right answer; that covers them both (i.e., incorporates the other,
for example, incorrect action may be the best like a telescope).
answer if the stem asks for an action that is not
Example: Two hours after a liver biopsy, the
appropriate (e.g., Which of the following is an
nurse finds the client lying on the left side. What
inappropriate action? can be rephrased to say
is the best nursing action at this time?
Which action is wrong?).
1. Check for bleeding.
22. To narrow down the choices, first find two con-
2. Turn the client onto the right side.
tradictory options; for example, hypohyper,
flexextend, givewithhold, dilateconstrict, Both options are correct, but choose option 2
increasedecrease, bradycardiatachycardia; because it incorporates option 1; it is possible to
then focus on which one may be the correct or check for bleeding while turning the client over
best choice. onto the right side (where the liver is), to put
23. Focus on the age-appropriate answer (e.g., When pressure on the site (as a preventive measure when
caring for a toddler, with what safety issue should postbiopsy bleeding is possible).
the nurse be concerned?).
31. Look at root words to give you a clue: for example,
24. Time sequence points to the best choice. For
hemi = one half (hemianopsia = half without
example, ask yourself when is this taking place
vision). Break down unfamiliar words in the stem.
(e.g., prenatal or postpartum; preoperative or
32. Remember Maslowsoma before psyche
postoperative; before, during, or after; early or
physiological needs are before psychosocial needs
late; immediately?).
(i.e., physical needs first). Use Maslows hierarchy
25. In medication administration questions, apply
to establish priorities when more than one answer
the 5 rights:
looks correct.
Right Medication
Right Route Example: What is the priority nursing care for a
Right Client client after ECT?
Right Dosage 1. Reorient to time and place.
Right Time 2. Put the side rails up.
26. When more than one answer looks right, choose 3. Explain that memory loss is an expected
the first step of the nursing process (assess before outcome.
implement). Assessment words and phrases
When all three options are good (as in this case),
indicate priority:
select the physical aspect of care first (option 2)
Ascertain
rather than either of the two psychosocial options.
Assess
Check 33. Think safety as the best choice when more than
Collect one answer could be right. Safety is a priority. See
Detect the preceding example, where putting side rails
Determine up is a safety action.
2164_Ch02_059-068 29/03/12 12:19 PM Page 65

Test-Taking Tips and Guidelines: Sixty Strategies to Use in Answering Questions 65


34. Visualize the condition, behavior, situation, and Paresthesia
the options to help you choose the best answer. Paralysis
Form a mental image (e.g., what flexion looks like WOUND2 healingaffected by:
versus extension); visualize and sound out the Wound dimensions
answers (e.g., to eliminate trite clichs or author- Overweight
itarian-sounding responses such as Thats not Undiagnosed infections

GUIDELINES & TIPS


allowed here). Nutritional deficiencies
35. Would that you could that the ideal be possi- Diabetes, Disabilities
ble. Choose an answer for the ideal, not real, (e.g., immunosuppressed)
world. Do not rely solely on real-world experi- 41. Reword the question if the stem says further
ences to answer NCLEX questions (i.e., on the teaching is necessary (e.g., the best answer will
examination, answer as if you have all the time, have an incorrect statement).
all the staff, and all the equipment). 42. Recognize what is normal. For example, are the
36. When in doubt as to which answer is best, use data presented normal, or is the sign/symptom
the process of elimination first (e.g., eliminate presented an Uh-oh! (meaning that a problem
what you know is incorrect) to narrow your exists)?
choice to two options. The best choice will pro- 43. Do not delegate functions of assessment, evalua-
vide an answer to what the question is asking. tion, and nursing judgment to a Licensed
37. Apply the ABCS when the question calls for Vocational/Practical Nurse (LVN/LPN) or CNA
priorities: (Certified Nursing Assistant) (e.g., do not dele-
Airway gate: admitting a client from the OR to the unit;
Breathing establishing a plan of care; teaching or giving
Circulation telephone advice; handling invasive lines).
Safety 44. Do delegate activities to an LVN/LPN or CNA
38. An important goal is to maximize client actions. for clients who are stable with predictable out-
For example, choose options that have indicator comes (e.g., help ambulate a client who is 2
words for encourage: days postsurgery). Do delegate to an LVN/LPN
Reinforce or CNA activities that involve standard,
Support unchanging procedures (e.g., take vital signs
Facilitate after ambulation, do clean catheterizations,
Assist simple dressing changes, suction chronic tra-
Help cheostomies using clean technique).
Aid 45. In positioning a client, decide what you are
Foster trying to prevent (e.g., contractures) or promote
39. Try to turn options into true-or-false responses if (e.g., venous return).
possible in order to narrow down to two possible 46. To help decide in which position to place a
options. For example, when a question asks about client, form a mental image of each position in
adjusting insulin dosage, ask yourself, What is the options (e.g., picture supine, high Fowlers,
true about adjusting the dose? Is it true or false semi-Fowlers, Sims, prone, Trendelenburg,
that dosage is increased when the client has an lithotomy, dorsal recumbent).
infection? (True). Is it true or false that dosage 47. When none of the options looks good, identify
is decreased when blood glucose level increases? the nursing concept implied in the options given
(False). If you find no true answers, look at the (e.g., risk factors for infection).
choices for a maybe answer. 48. When in doubt, first reread the question stem to
40. Use acronyms and memory aids to help remember obtain clues, then reread the options. When you
theory in selecting an answer (see Appendix B). come across a question that is about unfamiliar
SWISSmanagement of Cushings syndrome: nursing content (e.g., paracentesis), first ask
Sugar (hyperglycemia) yourself, What is the topic of the question?,
Water (fluid retention) then What do the answer choices mean?, and
Infection (prone to ...) then reword the question using the clues from
Sodium (retention) the options.
Sex changes (no menses)
Example: What is most important for the nurse
Five Ps of assessing fracture:
to ask a client immediately after a paracentesis?
Pain
1. Are you in pain?
Pallor
2. Do you feel dizzy?
Pulselessness
2164_Ch02_059-068 29/03/12 12:19 PM Page 66

66 chapter 2 Guidelines and Tips for International Nurses and Repeat Test-Takers

3. Does your underwear fit better around the belt 54. If two options are similar, neither can be the
line? answer because both are distractors.
4. Do you need to urinate?
Example: What might the nurse expect to see
The first clue is in the question stem: most impor- when a client with cirrhosis is hospitalized with
tant, immediately after. Then, based on rereading ascites?
the options, you can reword the question to, 1. Client is likely to be anorexic.
GUIDELINES & TIPS

What is an untoward reaction (complication) 2. Clients intake will be poor, especially if served
after this procedure? The answer choices relate large portions.
to expected outcomes (1, 3), a question that is not
relevant to ask after the procedure (4), and a You can eliminate both of these options because
complication (2), which is the correct option. they are saying the same thing in different words:
the client is likely to not be interested in eating.
49. Recognize expected outcomes of drugs and
treatments/procedures. 55. Dont pass the buckthink what is a nursing
action that an RN can do before calling the MD.
Example: What will indicate improvement in the
Example: After surgery, a client with diabetes
condition of the client who has anorexia nervosa?
complains of nausea, and appears lethargic and
1. The client has gained weight.
flushed, with BP 108/78, P 100, R 24 and deep.
2. The client weighs herself every day.
What is the next action?
3. The client eats all the foods served to her.
1. Call the MD.
4. The client asks the parents to bring her favorite
2. Check the clients glucose.
foods.
3. Give an antiemetic.
Choose the option that shows progress toward 4. Change the IV infusion rate.
the goal (in this condition, it is weight gain that
is expected). The nurse should assess (option 2) before calling
the MD (who may then order an antiemetic,
50. When you do not know the answer, choose what option 3, and alter the IV infusion rate,
will cause the least harm. option 4).
Example: The nurse suspects abdominal wound 56. Focus on key words in the stem of the question as
dehiscence when lifting the edges of the clients your clue:
dressings. What should the nurse do next? Best
1. Tell the client to remain quiet and not cough. Essential
2. Offer a warm drink to help relax the client. Highest
3. Place the client with feet elevated. Immediate
4. Change the dressing. Least likely
Most
Option 1 is the best answer, because it will not
Most appropriate
add damage that could happen with changing the
Most likely
position or the dressing.
Vital
51. Take care of the client first, not the equipment or
the family (unless a family member is the focus 57. Action does not always mean choose an imple-
of the question). mentation type of answer. The question may
52. If one option has generally, usually, tends to but ask: What is the best nursing action? However,
other options do not have these qualifiers, use the the answer may be an assessment option.
one option that does have the qualifier as the best Example: What is the best action for the nurse to
answer. take when a mother at the clinic reports that her
53. Identify clues in the stem, that is, look for a simi- child who has diabetes is hyperglycemic in the
lar word or phrase used in the stem and in one of morning (215 mg/dL), although the child has
the options. For example, if the question states been well controlled with NPH and regular
that the client is on an intermediate-acting insulin insulin before breakfast and dinner?
(NPH), and the stem asks for its peak action, 1. Suggest that the mother give the bedtime snack
look for a middle time. Among choices of 4, 6 to earlier.
12, 12 to 14, or 15 to 18 hours after the injec- 2. Suggest that the insulin be given later in the
tion, choose 6 to 12 hours (as the midpoint). evening.
2164_Ch02_059-068 29/03/12 12:19 PM Page 67

Confidence, Performance, Pass! 67


3. Suggest that they continue with the same Example: The primary objective in ileostomy
regimen. teaching with a client during the early
4. Check the blood sugar now, and suggest that postoperative period is to:
the mother check it during the night. 1. Facilitate maintenance of intake and output
records.
Choose check, which is an assessment
2. Control unpleasant odors.
response (option 4), although the question (the

GUIDELINES & TIPS


3. Prevent skin excoriation around the stoma.
stem) is phrased as an implementation (best
4. Reduce the risk of postoperative wound
action is an implementation).
infection.
58. Remember the nursing hierarchy. Go to the next
Choose prevention (option 3), which in turn
line of nursing authority (e.g., staff charge
may prevent contamination of the abdominal
nurse; LPN/LVN staff nurse) when the
incision (option 4). Options 1 and 2 are secondary
question asks to whom to report a situation. For
objectives.
example, if the question is about an LVN/LPN,
the best answer is to report to the staff nurse.
59. When the question includes laboratory values, Confidence, Performance, Pass!
ask yourself whether the given value is Uh-oh!
Chapter 2 is aimed at helping you to take the NCLEX-RN
(meaning too high or too low), or Uh-huh
with more and improved test-taking skills.
(meaning not a particular problem). For example,
a serum K+ of 8.5 is Uh-oh! (too high). THINK: confidence
60. Prevention is a key concept (e.g., when the THINK: performance
question deals with infection control, and in THINK: Pass NCLEX-RN
health teaching).
2164_Ch02_059-068 29/03/12 12:19 PM Page 68
2164_Ch03_069-114 29/03/12 12:20 PM Page 69

CHAPTER 3

Safe, Effective Care


Environment
Management of Care, Cultural Diversity, Ethical
and Legal Aspects of Nursing, Nursing Trends,
and Safety and Infection Control

Sally Lambert Lagerquist Janice Lloyd McMillin Robyn Marchal Nelson


Denise Wall Parilo Kathleen E. Snider

69
2164_Ch03_069-114 29/03/12 12:20 PM Page 70

70 chapter 3 Safe, Effective Care Environment

MANAGEMENT 2. Macrolevel theories: focus on best ways to make


OF CARE changes within an organization, organizing proj-
ects, obtaining resources, attaining goals.
I. CONCEPTS OF MANAGEMENT 3. Intrapersonal/interpersonal theories:
A. Definitions a. Cognitive: beliefa persons motivation is
1. Case managementprocess that involves compre- based on expectations about what will happen
hensive coordination of activities and services as a result of own behavior; involves goal
provided to the client throughout the continuum setting, with regular feedback to increase
of care or episode. Activities include: case finding, motivation to achieve.
screening, intake, assessment, problem identifica- b. Scientific management: beliefrepetition of
tion, prioritization of clients problems and task will result in expertise.
needs, planning, reassessment, evaluation, docu- c. Neoclassical management: based on Maslow
mentation, designing and monitoring clinical (i.e., person continuously strives to meet
pathways, and identification of variances. higher level needs); ERG (Existence,
MANAGEMENT

2. Continuous quality improvementprocess used to Relatedness, Growth); job redesign (i.e.,


make improvements in client care; indicators of ensures that task has validity, significance,
excellence are identified and process involves autonomy, and feedback).
actively including input from and collaboration d. Social/reinforcement: beliefmotivation
with the client (whose needs are at the center of comes from learning from those with
the process), the family, and all health-care team whom a person identifies; conditioned by
members. reinforcement.
3. Incidents/variance reportspart of quality C. Management behaviors
improvement, where occurrences take place in a 1. Decision maker
health-care agency that are not typical according a. Initiator of new projects.
to medical orders, may be an accident, or may b. Crisis handler (e.g., interpersonal conflicts
be a violation of policy and procedures (e.g., among staff ).
wrong medication dose, a client or visitor falls, c. Resource allocator (people, physical, financial).
needle stick by nurse). These are considered d. Negotiator.
unexpected incidents, exceptions that happen 2. Communicator
during client care. a. Monitor of data collection and processing.
4. Quality assuranceactivities that evaluate the b. Dissemination of collected information.
quality of care provided to clients to ensure that c. Speaker on behalf of agency.
it meets predetermined standards of excellence. 3. Representative
5. Resource managementproviding appropriate a. Institutional figurehead.
number and type of resources needed by clients b. Group leader.
to achieve desired outcomes. c. Liaison between agency and community.
6. Supervisionprocess of guiding, encouraging, II. ESTABLISHING PRIORITIES
and assessing the work of others to whom tasks
were delegated. A. When managing a number of clients at the same
7. Delegate responsibility and direct nursing care pro- time, the nurse needs to set priorities by assessing
vided by othersbased on particular client/ types of care needed:
family needs and on job description, roles, 1. Decide on the most important nursing activity
functions, and skills of other nursing personnel: (giving a medication? performing a treatment?
clients condition (stable or medically fragile), taking vital signs? providing nutrition? measur-
complexity of required care, potential risks for ing I&O? etc.).
harm to client, degree of needed problem-solving 2. Identify the first action the nurse needs to take.
expertise, predictability of outcome, type and 3. Select the best nursing action.
level of client interaction required. Important: 4. Determine which client needs immediate care.
The person who delegated the task has the B. Determine priorities with the guidance of:
responsibility and final accountability for effec- 1. Maslows hierarchy of needs (see Fig. 10.1,
tive completion of the task. p. 715)
B. Management theories a. Choose physiological needs (survival) as the
1. Microlevel theories: clarify and predict behavior highest level of priority.
(e.g., motivation) of the individual, with b. Followed by safety needs.
input on the group/organization (e.g., group c. Then psychological needs (care and belonging).
dynamics). d. Lastly, self-actualization needs.
2164_Ch03_069-114 29/03/12 12:20 PM Page 71

Management of Care 71
2. Steps of the Nursing Process (A2DPIE) F. Level of prevention
a. First is assessment (data collection). 1. Primary prevention: prevention of disaster and
b. Next, analyze the data (nursing diagnoses). limiting consequences when cannot be prevented.
c. Then, plan (goals). a. Nursing activities: identification of factors that
d. Followed by implementation (actions). pose actual or potential problem.
e. Finally, evaluation (outcome). 2. Secondary prevention: responding to the disaster,
3. ABCS halting it, and resolving problems caused by it.
a. Airway (e.g., patent airway) a. Nursing activities: assessment of extent of
b. Breathing injuries; tagging victims for treatments and
c. Circulation evaluation; providing first aid; identifying
d. Safety complications; coordinating activities of
4. RACE (e.g., in event of fire) shelter workers.
a. Remove the client. 3. Tertiary prevention: recovery and prevention of
b. Then sound the alarm. recurrence.

MANAGEMENT
c. Call the fire department. a. Nursing activities: implementing communitys
d. Extinguish the fire. disaster plan; providing continuous assess-
III. DISASTER PLANNING ment, planning, implementation, and evalua-
tion; providing counseling as needed to
A. Definition: any man-made (e.g., toxic material spill,
victims and coworkers; educating the public
riot, explosion, structural collapse) or naturally
about disaster preparedness.
occurring (e.g., communicable disease epidemic,
flood, hurricane, earthquake) event that results in IV. EMERGENCY RESPONSE PLAN: FIRE SAFETY
destruction or devastation that causes suffering, AND PREPAREDNESS
creates human needs, and cannot be alleviated A. Know location of:
without support 1. Escape routes, escape doors. Keep fire exits clear.
B. Goal: reduce vulnerability to prevent recurrence 2. Available equipment.
C. Benefits of a disaster plan: a. Fire alarms.
1. Decrease in costs of damage control. b. Fire sprinkler controls.
2. Decreased extent and duration of injury. c. Fire extinguishers.
3. Decreased loss of life. B. Identify fire hazards.
4. Increased ability to respond to unforeseen 1. Faulty electrical equipment and wiring.
disasters. 2. Overloaded circuits.
D. Health-care components 3. Plugs not properly grounded.
1. Early warning signals, with realistic expectations. 4. Smoking.
2. Brief and succinct assessment of those at risk. 5. Combustible substances spontaneous
3. Simple, flexible rescue chains that unfold in combustion.
organized stages/steps. C. Prevention.
E. Nursing responsibilities 1. Report frayed or exposed electrical wires.
1. Nurses at the scene: assisting with rescue, evacua- 2. Avoid overloaded circuits.
tion, and first aid. 3. Dont use extension cords.
2. Nurses at the hospital: triaging victims and pro- 4. Use only three-pronged grounded plugs.
viding acute care. 5. Avoid clutter.
a. Triage: a system of client evaluation to set up 6. Remove cigarettes and matches from room; con-
priorities, assign appropriate staff, and start trol smoking according to institutional policy;
treatment. limit smoking to designated areas.
(1) In emergencies: greatest risk receives priority. 7. Immediately report smoke odors and burning.
(2) In major disasters: selection is based on D. Action to take in event of fire in immediate vicinity:
doing what can be done to benefit the 1. Move clients to safety (triage those who are not
largest number; those needing highly ambulatory or are otherwise incapacitated).
specialized care may be given minimal or 2. Sound alarm.
no care. First, take care of those needing 3. Close all windows and doors.
minimal care to save their lives, and who 4. Shut off valves for O2.
in turn can be available to help others. 5. Follow agency policy about announcing fire and
3. Nurses at community shelters or health clinics: location, notifying fire company, and evacuation
assessing, planning, implementing, and evaluat- plan.
ing ongoing health-care needs of victims. 6. Avoid using elevators.
2164_Ch03_069-114 29/03/12 12:20 PM Page 72

72 chapter 3 Safe, Effective Care Environment

V. SAFE USE OF EQUIPMENT (see also p. 84) make communication more difficult. Speaking
A. Suspect malfunction in equipment when it: too fast may overload the client and make it
1. Does not work consistently or correctly. difficult for the client to follow. Speaking too
2. Makes unusual noise. slowly may lose the clients attention.
3. Gives off unusual odor. (1) Select the gestures you use with care, using
4. Produces extreme temperature. your nonverbal behavior to underscore
5. Produces sparks. your words and your actions. The proper
B. Replace immediately; dont repair it. use of gestures can clarify a message, and
C. Call maintenance department to check for safety drawings can sometimes be helpful. Be
and repair. careful, however; not all gestures mean the
D. When O2 is in use: same thing in all cultures.
1. Secure the O2 according to institutional policy. B. Cultural communication patterns (see Table 3.1)
2. Remove flammable liquids from the area. 1. Willingness to share their thoughts and feelings.
3. Put up oxygen in use signs. 2. Use and meaning of touch between family,
friends, same sex, opposite sex, with health-care
CULTURAL

provider.
C U LT U R A L 3. Personal space: meaning of distance and physical
DIVERSITY IN proximity.
NURSING PRACTICE* 4. Eye contact: special meaning for staring (rude,
With increasing ethnocultural diversity among health- evil eye); for avoidance of eye contact (e.g., not
care clients and staff, health-care providers must increase caring, not listening, not trustworthy); variation
their sensitivity to and knowledge of cultural concepts, of eye contact among family, friends, strangers,
be aware of both similarities and differences in values and socioeconomic groups.
and beliefs that exist across cultures, and know how this 5. Facial expression: how emotions are shown (or not)
may affect health-care delivery. Important objectives are in facial expressions; use and meaning of smiles.
to increase respect and sensitivity for diversity in order 6. Standing, greeting strangers: what is acceptable.
to minimize potential for transgressing cultural norms, C. Concept of time: past, present, or future oriented;
and to provide culturally conscious health-care and social time vs. clock time
working relationships among clients and staff from dis- D. Names: expected greetings by health-care providers
similar cultures. II. FAMILY ROLES
The purpose of this section is to provide a framework/ A. Gender roles: patriarchal or egalitarian; change in
structure for assessing, planning, and implementing cul- perceived head of household during different life
turally conscious interventions. stages; male/female norms (e.g., stoic, modest)
We have selected 10 essential areas as guidelines for B. Prescriptive (should do), restrictive (should not do),
assessing cultural characteristics that have implications taboo behaviors for children and adolescents
for health and health care: communication, family 1. Prescriptive (e.g., Fat children are healthy).
roles, biocultural ecology, high-risk health behaviors, 2. Restrictive practices (e.g., silence, not anger, at
nutrition, pregnancy and childbearing practices, parents).
death rituals, spirituality, health-care practices, and 3. Taboo (e.g., discussion of sexuality).
health-care practitioners. C. Family roles and priorities
I. COMMUNICATION 1. Family goals and priorities (family needs may
A. Language have priority over individual health needs).
1. What is the usual volume and tone of speech? 2. Developmental tasks.
a. Guidelines: use interpreters (to provide mean- 3. Aged: status and role.
ing behind words) rather than translators (who 4. Extended family (biological and nonbiological):
just restate words); avoid use of relatives and role and importance.
children; use interpreters of same age and 5. How social status is gained: through heritage?
gender when possible. Select the words you Educational accomplishments?
use carefully, avoiding buzz words and jargon. D. Alternative lifestyles
Speak clearly, pacing yourself to be neither too 1. Nontraditional families: single parents, blended
fast nor too slow. Words that are slurred, have families, communal families, same-sex families.
many syllables in them, or are too technical III. BIOCULTURAL ECOLOGY
A. Variations in color of skin and biological variations
*Sally Lagerquist (was in Addison-Wesleys Nursing Examination Review ed. 2, Unit 7,
1. Skin color: special problems/concerns: assess-
p. 633 [out of print]). ment of jaundice, mongolian spots, and
2164_Ch03_069-114 29/03/12 12:20 PM Page 73

Cultural Diversity in Nursing Practice 73


blood/oxygenation levels in dark skin. V. NUTRITION: See also Chapter 9, Cultural Food
Considerations for health care: Patterns.
a. Assessment of anemia: examine oral mucosa A. Meaning of food: symbolic, socialization role;
and nailbed capillary refill. denotes caring and closeness and kinship, and
b. Assessment of jaundice (e.g., in Asian people): expression of love and anger
look at sclera. B. Common foods and rituals
c. Assessment of rashes: palpate. 1. Major ingredients commonly used (high sodium,
d. Get a baseline of skin color from family. fat, spices).
e. Use direct sunlight. 2. Preparation practices (e.g., kosher does not mix
f. Look at areas with least amount of meat with dairy in cooking, eating, serving).
pigmentation. 3. Afternoon tea (British), morning coffee (American).
g. Compare skin in corresponding areas. 4. Fasting (e.g., Muslims, Catholics, Jews).
2. Biological variations in body, size, shape, and 5. Foods not allowed (e.g., no shellfish or pork in
structure: long bones, width of hips and kosher diet).
shoulders, flat nose bridges (relevance for C. Nutritional deficiencies and food limitations

CULTURAL
fitting eyeglasses), shorter builds (at variance 1. Enzyme deficiencies (e.g., in glucose-6-phosphate
with normative growth curves); mandibular dehydrogenase deficiency, fava bean can cause
and palatine dimensions (relevance for fitting hemolysis and acute anemic crisis).
dentures); teeth (peg, extra, natal, large size); 2. Food intolerances (e.g., lactose deficiency).
ears (free, floppy, attached); eyelids (epicanthic 3. Significant nutritional deficiencies, such as
folds). calcium (Southeast Asian immigrants).
3. Diseases and health conditions: 4. Native food limitations that may cause special
a. Specific risk factors related to climate, topog- health difficulties, such as poor intake of lysine
raphy (e.g., air pollution, mosquito-infested and other amino acids (Hindu).
tropical areas). D. Use of food for health promotion, to treat illness,
b. At-risk groups for endemic diseases (those and in disease prevention
that occur continuously in a specific ethnic 1. Hot and cold theories.
group): e.g., malaria, liver and renal impair-
VI. PREGNANCY AND CHILDBEARING
ment, infectious blindness and scleral infec-
PRACTICES
tions, otitis media, respiratory diseases
(e.g., tuberculosis, coccidioidomycosis). A. Fertility and views toward pregnancy, contracep-
c. Increased genetic susceptibility for diseases tion, and abortion
and health conditions (e.g., diabetes, B. Prescriptive, restrictive, and taboo practices related to
dwarfism, muscular dystrophy, cystic pregnancy, birthing practices, and postpartum period
fibrosis, myopia, keloid formation, gout, 1. Pregnancy: foods, exercise, intercourse, and
cancer of stomach is more prevalent in avoiding weather-related conditions.
blood type O, sickle cell anemia, Tay- 2. Birthing process: reactions during labor, presence
Sachs disease). of men, position for delivery, preferred types of
4. Variations in drug metabolism (e.g., cardiovas- health-care practitioners, place for delivery.
cular effects of propranolol in Chinese; 3. Postpartum period: bathing, cord care, exercise,
peripheral neuropathy in Native Americans foods, role of men.
on isoniazid). VII. DEATH RITUALS
5. Variations in blood groups (e.g., Native A. Death rituals and expectations
Americans usually are type O and no type B; 1. Cultural expectations of response to death and
Rh-negative nonexistent in Eskimos, more often grief.
in Caucasians); twinning (dizygote) is highest 2. Meaning of death, dying, and afterlife.
among African Americans. a. Euthanasia.
IV. HIGH-RISK HEALTH BEHAVIORS b. Autopsies.
A. Use of alcohol, tobacco, recreational drugs B. Purpose of death rituals and mourning practices
B. Level of physical activity; increased calorie C. Specific burial practices (e.g., cremation)
consumption VIII. SPIRITUALITY
C. Use of safety measures (e.g., seat belts and helmets A. Use of prayer, meditation, or symbols
and safe-driving practices) B. Meaning of life and individual sources of strength
D. Self-care using folk and magicoreligious practices C. Relationship between spiritual beliefs and health
before seeking professional care practices
2164_Ch03_069-114 29/03/12 12:20 PM Page 74

74 chapter 3 Safe, Effective Care Environment

IX. HEALTH-CARE PRACTICES d. Some ethnic groups do not value or possess


A. Health-seeking beliefs and behaviors qualities required for some psychiatric
1. Beliefs that influence health-care practices. therapies, such as verbal skills, introspection,
a. Perception of illness (e.g., punishment for sin, ability to delay gratification, and ability to
work of persons who are malevolent). discuss personal problems with strangers.
2. Health promotion and prevention practices. e. Therapy resources may not be accessible or con-
a. Acupuncture. sidered useful or relevant for members of some
b. Yin and yang: ethnic groups.
(1) Increased yin results in nervous, digestive f. Common feelings and behavior patterns may
disorders. be shared by many minority groups:
(2) Increased yang results in dehydration, (1) Feelings of inferiority and inadequacy, often
fever, irritability. a result of prejudice and racism.
B. Responsibility for health care (2) Incompetent behavior as an outcome of
1. Acute care: curative or fatalistic. feeling inferior and inadequate.
2. Who assumes responsibility for health care? (3) Suppressed anger, resulting in displaced hos-
CULTURAL

3. Role of health insurance. tility and paranoid ideas.


4. Use of over-the-counter medications. (4) Withholding and withdrawal; not comfort-
C. Folklore practices able with sharing feelings or experiences.
1. Combination of folklore, magicoreligious beliefs, (5) Selective inattention; may block out or
and traditional beliefs that influence health-care deny frustration or insults.
behaviors. (6) Overcompensation in some areas to make
D. Barriers to health care (e.g., language, economics, up for denied opportunities in other areas.
geography) 3. Different perception of mentally and physically
E. Cultural responses to health and illness handicapped.
1. Beliefs and responses to pain that influence 4. Beliefs and practices related to chronic illness and
interventions. rehabilitation.
a. Special meaning of pain. 5. Cultural perceptions of the sick role.
2. Beliefs and views about mental illness/mental F. Acceptance of blood transfusions and organ donation
health care. X. HEALTH-CARE PRACTITIONERS
a. Therapies must include extended families as A. Traditional vs. biomedical care
opposed to individuals or nuclear families. 1. Does the age of practitioner matter?
b. Cultural and racial as well as individual com- 2. Does the gender of practitioner matter?
ponents must be considered when assessing B. Status of health-care provider
precipitating or predisposing causes of illness 1. How different members of health-care practice
(e.g., need to atone for sins). see each other.
c. Values may conflict: for example, individual-
ism versus concern for family or social inter- XI. ADDITIONAL CULTURAL
actions; self-actualization versus survival CONSIDERATIONSfor other cultural influences
needs. related to children and families, refer to Table 3.1.

Table 3.1
Cultural Influences on Health-Care Practices with Children and Adults
Cultural Group Belief Practice
African Americans Health is viewed as harmony with nature Self-care and folk medicine prevalent
(numerous groups from Illness may be viewed as will of God Try home remedies first or consult with an old
varying locales) or a punishment (especially in chil- woman in the community (especially for children)
dren); illness can be caused by natural May make use of root doctors, spiritualist,
(polluted food/water) as well as unnat- voodoo priests
ural (hex) sources May seek opinion of black minister, who is highly
May distrust health-care practitioner influential in health-care decisions
who is from the majority group (shown Prefer use of last name (upon greeting)
by silence)
Reluctant to give permission for organ
donation
2164_Ch03_069-114 29/03/12 12:20 PM Page 75

Cultural Influences on Health-Care Practices 75

Table 3.1
Cultural Influences on Health-Care Practices with Children and Adultscontd
Cultural Group Belief Practice
Americans (usually of Health is viewed as a combination of Believe that infant can tell mother its health-care
Caucasian, European physical and emotional well-being needs
descent) Illness may be viewed in rational/ Early/routine prenatal and well baby/child care
scientific terms; believe in germ and and immunizations
stress theories Increasing reliance on/demand for specialists in
Increasing interest in health promotion child health care
as reflected in lifestyle
May view alternative health care as
possibility/valuable, either independ-
ently or in conjunction with Western
medicine
Asian Americans Health is viewed as a balance between Goal of health-care therapy: restore balance of yin

CULTURAL
(numerous groups from energy forces of yin (cold) and yang and yang
varying locales) (hot); harmony with universal order; Restoration of health with: tai chi, acupressure/
pleasing good spirits/avoiding evil acupuncture, diet, folk healers, herbs, massage,
spirits moxibustion (heat applied to skin over specific
Illness may be viewed as an areas)
imbalance Avoid dairy because of lactose intolerance
Childs good health reflects well upon Trend: use combination of Eastern and
the parent/family Western treatment modalities and prevention
Honor and face important Elderly treated with respect
Suppress emotions Close, extended families; father/eldest son are
primary decision makers
Direct eye contact may show disrespect
Hispanic and Mexican Health is viewed as a reward/good Curandero (folk healer) may be consulted about
Americans (numerous luck mal de ojo (evil eye) and susto (fright) before a
groups from varying Illness may be viewed as punishment health-care practitioner from the majority group
locales) or an imbalance between hot and cold Hot diseases are treated with cold remedies
Individual is passive recipient of dis- (does not refer to temperature)
ease, which is caused by external Use: herbs, prayers, religious artifacts/rituals;
forces (supernatural) visits to shrines (strong association between
religion and health)
Children highly valued/desired and taken every-
where with family, which can lead to interruptions
when consulting with health-care practitioner
Family support during labor
Elderly treated with respect
Men make key decisions in matters outside of the
home
Silence may indicate disapproval of plan of care
Very emotionally expressive when grieving
Native Americans/ Health is viewed as a state of harmony Going to the physician/hospital is associated with
American Indians with nature and the universe illness/disease; may delay seeking care as health
(numerous tribes from Illness may be viewed as a price to be state is viewed as part of a natural process
varying locales) paid for past/future deeds (example: adolescent becomes pregnant)
All disorders believed to have super- Reliance on: diviners/diagnosticians, herbs, medi-
natural aspects/influences cine man, rituals, singers; may carry objects to pro-
tect self against witchcraft; full family involvement
Children who are obstinate are respected, while
children who are docile are considered weak; can
lead to conflict with health-care practitioner from
majority group
Elderly treated with respect
Blood and organ donation generally not accepted
Handshake, light touch OK, but maintain respectful
distance while interacting
Adapted from Hockenberry, M, & Wilson, D: Wongs Nursing Care of Infants and Children, ed 8. Mosby, St. Louis,
2007.
2164_Ch03_069-114 29/03/12 12:20 PM Page 76

76 chapter 3 Safe, Effective Care Environment

RELIGIOUS NURSING ETHICS


AND SPIRITUAL Nursing ethics involves rules and principles to guide
INFLUENCES right conduct in terms of moral duties and obligations to
O N H E A LT H protect the rights of human beings. In nursing, ethical
Religious and spiritual beliefs can have a major impact on codes provide professional standards and formal guide-
health and illness. Each religion has its own rituals and tra- lines for nursing activities to protect both the nurse and
ditions that must be observed, with the belief that if these the client.
are not followed, the outcomes may negatively affect the I. CODE OF ETHICSserves as a frame of reference
clients well-being or their family. when judging priorities or possible courses of action.
I. DEFINITION OF TERMS Purposes:
A. Religionan organized belief system in God or A. To provide a basis for regulating relationships
supernatural, using prayer, meditation, or symbols between nurse, client, coworkers, society, and
B. Spiritualityencompasses more than religious profession.
beliefs; includes values, meaning, and purpose in B. To provide a standard for excluding unscrupulous
CULTURAL

life; can provide inspiration and sustain a person or nursing practitioners and for defending nurses
group during crisis unjustly accused.
C. Values clarificationaligns values and beliefs so that C. To serve as a basis for nursing curricula.
they are consistent with goals D. To orient new nurses and the public to ethical
II. ASSESSMENT OF RELIGIOUS AND SPIRITUAL professional conduct.
BELIEFS II. ANA CODE OF ETHICS FOR NURSES incorpo-
A. Beliefs about birth and what follows death rates the following key elements of what the nurse
B. Code of ethics about right and wrong needs to do*:
C. View of health, causes of illness, or what may be A. Demonstrate respect for human dignity and unique-
the cure for the problem ness of individual regardless of health problem or
D. Dietary laws socioeconomic level
E. Relationship of mind, body, and spirit B. Maintain clients right to privacy and
F. Importance of work and money as they relate to confidentiality
religion C. Protect the client from incompetent, unethical, or
G. Pain: purpose of, response to, treatment for illegal behavior of others
H. Importance of family D. Accept responsibility for informed individual
I. Meaning of life, individual sources of hope and nursing judgment and behavior
strength E. Maintain competence through ongoing professional
J. Religious practices that conflict with health prac- development and consultation
tices and use of health services F. Maintain responsibility when delegating nursing
III. ANALYSIS/NURSING DIAGNOSIS care, based on competence/qualification criteria
G. Work on maintaining/improving standards of care
A. Risk for spiritual distress related to prolonged
in employment setting
pain; health-care choices that are in conflict with
H. Protect consumer from misinformation/
religious practices; anxiety and guilt due to
misrepresentation
violating religious beliefs; lashing out against
the religion III. BIOETHICSa philosophical field that applies
ethical reasoning process for achieving clear and
IV. NURSING CARE PLAN/IMPLEMENTATION
convincing resolutions to issues and dilemmas
A. Acknowledge clients beliefs (conflicts between two obligations) in health care.
1. Provide contact with clergy of choice.
A. Purpose of applying ethical reflection to nursing
2. Provide opportunity to carry out practices not
concerns:
detrimental to clients health.
1. Improve quality of professional nursing
B. Do not impose beliefs and values of health-care
decisions.
system
2. Increase sensitivity to others.
V. EVALUATION/OUTCOME CRITERIA 3. Offer a sense of moral clarity and enlightenment.
A. Increased satisfaction related to medical care
decision
B. Decrease in feelings of stress, guilt, depression, *Adapted from recording at JONA and Nurse Educators Joint Leadership Conference,
anger 1981, A. J. Davis, Ethical Dilemmas in Nursing.
Adapted from AAA Code of Ethics on Nurses, Washington, DC.
2164_Ch03_069-114 29/03/12 12:20 PM Page 77

Nursing Ethics 77
B. Framework for analyzing an ethical issue: on Protection of Human Subjects (with informed,
1. Who are the relevant participants in the voluntary, written consent).
situation? F. Right to be free of restraints and seclusion except in
2. What is the required action? an emergency.
3. What are the probable and possible benefits G. Right to humane environment with reasonable pro-
and consequences of the action? tection from harm and appropriate privacy.
4. What is the range of alternative actions or H. Right to confidentiality of medical records.
choices? I. Right of access to personal treatment record.
5. What is the intent or purpose of the action? J. Right to as much freedom as possible to exercise
6. What is the context of the action? constitutional rights of association (e.g., use of
C. Principles of bioethics: telephone, personal mail, having visitors) and
1. Autonomythe right to make ones own expression.
decisions. K. Right to information about these rights in both
2. Nonmalfeasancethe intention to do no harm. written and oral form, presented in an understand-

ETHICS/LEGAL
3. Beneficencethe principle of attempting to do able manner at outset and periodically thereafter.
things that benefit others. L. Right to assert grievances through a grievance
4. Justicethe distribution, as fairly as possible, of mechanism that includes the power to go to court.
benefits, resources, and burdens. M. Right to obtain advocacy assistance.
5. Veracitythe intention to tell the truth. 1. Definition: an advocate is a person who pleads
6. Confidentialitythe social contract guaranteeing for a cause or who acts on a clients behalf.
anothers privacy. 2. Goals: help client gain greater self-determination
7. Respectacknowledge the rights of others. and encourage freedom of choices; increase
8. Fidelitykeep promises and commitments. sensitivity and responsiveness of the health-care,
IV. CLIENTS BILL OF RIGHTS social, political systems to the needs of the
client.
A. Right to appropriate treatment that is most
3. Characteristics: assertiveness; willingness to
supportive and least restrictive to personal
speak out for or in support of client; ability to
freedom.
negotiate and obtain resources for positive
B. Right to individualized treatment plan, subject to
outcomes; willingness to take risks, and take
review and reassessment.
necessary measures in instances of incompetent,
C. Right to active participation in treatment, with the
unethical, or illegal practice by others that may
risk, side effects, and benefits of all medication and
jeopardize clients rights.
treatment (and alternatives) to be discussed.
N. Right to criticize or complain about conditions or
D. Right to give and withhold consent (exceptions:
services without fear of retaliatory punishment or
emergencies and when under conservatorship).
other reprisals.
1. Advance directives: legal, written, or oral state-
O. Right to referral to complement the discharge plan.
ments made by a person who is mentally
competent about treatment preferences. In V. CONFLICTS AND PROBLEMSETHICAL
the event the person is unable to make these DILEMMAS
determinations, a designated surrogate decision A. Personal values versus professional dutynurses have
maker can do so. Each state has own specific the right to refuse to participate in those areas of
laws with restrictions. nursing practice that are against their personal
a. Living will: legal document that specifically values, as long as a clients welfare is not jeopard-
identifies treatment desires and states that the ized. Example: therapeutic abortions.
person does not wish to have extraordinary B. Nurse versus agencyconflict may arise regarding
lifesaving measures (e.g., DNR) when not whether or not to give out needed information to a
able to make decisions about own care. client or to follow agency policy, which does not
b. Durable power of attorney (health care allow it. Example: a teenager who is emotionally
proxy): legal document giving designated upset asks a nurse about how to get an abortion, a
person authority to make health-care decisions discussion that is against agency policy.
on clients behalf when client is unable to do so. C. Nurse versus colleaguesconflict may arise when
E. Right to be free of experimentation unless follow- determining whether to ignore or report others
ing recommendations of the National Commission behavior. Examples: you see another nurse steal
medications; you know that a peer is giving a false
reason when requesting time off; or you observe a
Adapted from AAA Code of Ethics on Nurses, Washington, DC.
colleague who is intoxicated.
2164_Ch03_069-114 29/03/12 12:20 PM Page 78

78 chapter 3 Safe, Effective Care Environment

D. Nurse versus client/familyconflict may stem from of professional services requiring substantial
knowledge of confidential information. Should you specialized knowledge of the biological, physical,
tell? Example: client or family member relates a behavioral, psychological, and sociological
vital secret to the nurse. sciences and of nursing theory as the basis for
E. Conflicting responsibilitiesto whom is the nurse assessment, diagnosis, planning, intervention,
primarily responsible when needs of the agency and and evaluation in the promotion and mainte-
the client differ? Example: a physician asks a nurse nance of health; the casefinding and management
not to list all supplies used for client care, because of illness, injury, or infirmity; the restoration of
the client cannot afford to pay the bill. optimum function; or the achievement of a dig-
F. Ethical dilemmasstigma of diagnostic label nified death. Nursing practice includes but is not
(e.g., AIDS, schizophrenia, addict); involuntary limited to: administration, teaching, counseling,
psychiatric confinement; right to control individual supervision, delegation, and evaluation of prac-
freedom; right to suicide; right to privacy and tice and execution of the medical regimen,
confidentiality. including the administration of medications and
ETHICS/LEGAL

treatments prescribed by any person authorized


by state law to prescribe. Each registered nurse is
LEGAL ASPECTS directly accountable and responsible to the con-
OF NURSING sumer for the quality of nursing care rendered.
I. DEFINITION OF TERMS (American Nurses Association: Nursing: Scope and
A. Common law: accumulation of law as a result of Standards of Practice. American Nurses
judicial court decisions. Association, Washington, DC, 2004).
B. Civil law (private law): law that derives from C. Revoking a license: Board of Examiners in each state
legislative codes and deals with relations between in the United States and each province in Canada
private parties. has the power to revoke licenses for just cause, such
C. Public law: concerns relationships between an as incompetence in nursing practice, conviction of
individual and the state. The thrust of public law is crime, drug addiction, obtaining license through
to attain what are deemed valid public goals, such fraud, or hiding criminal history (see Section XI.
as reporting child abuse. A., B., C., p. 83).
D. Criminal law: concerns actions against the safety III. CRIMES AND TORTS
and welfare of the public, such as robbery. It is part A. Crime: an act committed in violation of societal
of the public law. law and punishable by fine or imprisonment. A
E. Informed consent: implies that significant benefits crime does not have to be intended (as in giving
and risks of any procedure, as well as alternative a client an accidental overdose that proves to be
methods of treatment, have been explained; per- lethal).
son has had time to ask questions and have these 1. Felonies: crimes of a serious nature (e.g.,
answered; person has agreed to the treatment murder) punishable by imprisonment of
voluntarily and is legally competent to give con- longer than 6 months.
sent; and communication is in a language known 2. Misdemeanors: crimes of a less serious nature
to the client. (e.g., shoplifting), usually punishable by fines
F. Reasonably prudent nurse: nurse must react as a or short prison term or both.
reasonably prudent nurse trained in that specialty B. Tort: a wrong committed by one individual against
area would react. For example, if a nurse works another or anothers property. Fraud, negligence,
with fetal monitors, she must know how to use the and malpractice are torts (e.g., losing a clients
monitors, know how to read the strips, and know hearing aid, or bathing the client in water that
what actions to take based on the findings. causes burns).
II. NURSING LICENSUREmandatory licensure 1. Fraud: misrepresentation of fact with intentions
required in order to practice nursing. for it to be acted on by another person (e.g.,
A. Nurse Practice Act: each state has one to protect falsifying college transcripts when applying for
nurses professional capacity, to set educational a graduate nursing program).
requirements, to distinguish between nursing and 2. Negligence: Omission to do something that a
medical practice, to define scope of nursing prac- reasonable person, guided by those ordinary
tice, to legally control nursing through licensing, considerations which ordinarily regulate human
and to define standards of professional nursing. affairs would do; or doing something which a
B. American Nurses Association: The practice of reasonable and prudent person would not do
nursing means the performance for compensation (Brent N: Nurses and the Law. Saunders,
2164_Ch03_069-114 29/03/12 12:20 PM Page 79

Legal Aspects of Nursing 79


Philadelphia, 1997). Types of negligent acts Therefore, if the client asks the nurse to testify, the
related to: nurse must truthfully give all information. However, if the
a. Sponge counts: incorrect counts or failure to nurse is a witness against the client, without the clients
count. permission to release information, the nurse must keep the
b. Burns: heating pads, solutions, steam information confidential by invoking the privileged com-
vaporizers. munication rule if the state law recognizes it and if it
c. Falls: side rails left down, infant left applies to the nurse.
unattended. VII. ASSAULT AND BATTERYviolating a persons
d. Failure to observe and take appropriate right to refuse physical contact with another.
actionforgetting to take vital signs and
A. Definitions
check dressing in a client who is newly
1. Assaultthe attempt to touch another or the
postoperative.
threat to do so and person fears and believes
e. Wrong medicine, wrong dose and concentra-
harm will result.
tion, wrong route, wrong client.
2. Batteryphysical harm through willful touching

ETHICS/LEGAL
f. Mistaken identitywrong client for surgery.
of person or clothing, without consent.
g. Failure to communicateignore, forget,
B. Implications for nursingneed to obtain consent to
fail to monitor, report, or document clients
treat, with special provisions when clients are
status or to report complaints of client or
underage, unconscious, or mentally ill.
family.
h. Loss of or damage to clients property VIII. GOOD SAMARITAN ACTprotects health
dentures, jewelry, money. practitioners against malpractice claims resulting from
i. Inappropriate use of equipment (e.g., exces- assistance provided at scene of an emergency (unless
sive IV fluids via pump). there was willful wrongdoing) as long as the level of
3. Malpractice: part of the law of negligence as care provided is the same as any other reasonably
applied to the professional person; any profes- prudent person would give under similar circum-
sional misconduct, unreasonable lack of skill, stances (see also Section VII, p. 82).
or lack of fidelity in professional duties, such IX. NURSES RESPONSIBILITIES TO THE LAW
as accidentally giving wrong medication or
A. A nurse is liable for nursing acts, even if directed to
forgetting to give correct medication or instilling
do something by a physician.
wrong strength of eyedrops into the clients eyes.
B. A nurse is not responsible for the negligence of the
Proof of intent to do harm is not required in acts
employer (hospital).
of commission or omission.
C. A nurse is responsible for refusing to carry out an
IV. INVASION OF PRIVACYcompromising a per- order for an activity believed to be injurious to
sons right to withhold self and own life from public the client.
scrutiny. Implications for nursingavoid unnecessary D. A nurse cannot legally diagnose illness or prescribe
discussion of clients medical condition; client has a treatment for a client. (This is the physicians
right to refuse to participate in clinical teaching; responsibility.)
obtain consent before teaching conference. E. A nurse is legally responsible when participating
V. LIBEL AND SLANDERwrongful action of com- in a criminal act (such as assisting with criminal
munication that damages persons reputation by print, abortions or taking medications from clients
writing, or pictures (libel), or by spoken word using supply for own use).
false words (slander). Implications for nursingmake F. A nurse should reveal clients confidential infor-
comments about client only to another health team mation only to appropriate health-care team
member caring for that client. members.
G. A nurse is responsible for explaining nursing
VI. PRIVILEGED COMMUNICATIONS activities but not for commenting on medical
information relating to condition and treatment of activities in a way that may distress the client or
client requires confidentiality and protection against the physician.
invasion of privacy. This applies only to court pro- H. A nurse is responsible for recognizing and protect-
ceedings. Selected person does not have to reveal in ing the rights of clients to refuse treatment or
court a clients communication to him or her. The medication, and for reporting their concerns and
purpose of privileged communication is to encourage refusals to the physician or appropriate agency
the client to communicate honestly with the treating people.
practitioner. It is the clients privilege at any time to I. A nurse must respect the dignity of each client and
permit the professional to release information. family.
2164_Ch03_069-114 29/03/12 12:20 PM Page 80

80 chapter 3 Safe, Effective Care Environment

X. ORGAN DONATION bylaws, regulations, and policies regarding this.


A. Legal aspects to protect potential donors and to Failure to follow the hospitals rules could be
expedite acquisition considered negligence.
1. Prohibits selling of organs (National Organ B. Should I follow a physicians orders if (a) I know it is
Transplant Act). wrong, or (b) I disagree with his or her judgment?
2. Guidelines regarding who can donate, how Regarding (a)no, if you think a reasonable,
donations are to be made, and who can receive prudent nurse would not follow it; but first inform
donated organs (Uniform Anatomical Gift Act). the physician and record your decision. Report it to
3. Legal definition of brain death (Uniform your supervisor. Regarding (b)yes, because the
Determination of Death Act)absence of: law does not allow you to substitute your nursing
breathing movement, cranial nerve reflex, response judgment for a doctors medical judgment. Do
to any level of painful stimuli, and cerebral blood record that you questioned the order and that the
flow; and flat EEG. doctor confirmed it before you carried it out.
B. Donor criteria C. What can I do if the physician delegates a task to me
ETHICS/LEGAL

1. Contraindications for being organ donator: for which I am not prepared? Inform the physician
HIV-positive status and metastatic cancer. of your lack of education and experience in per-
2. Prospective donors of both organs and tissues: forming the task. Refuse to do it. If you inform the
those with no neurological functions, but have physician and still carry out the task, both you and
cardiopulmonary functions. the physician could be considered negligent if the
3. Prospective donors of only tissues: those with no client is harmed by it. If you do not tell the physician
cardiopulmonary function (e.g., can donate and carry out the task, you are solely liable.
corneas, eyes, saphenous veins, cartilage, bones, II. OBTAINING CLIENTS CONSENT FOR
skin, heart valves). MEDICAL AND SURGICAL PROCEDURES: Is a
C. Management of donor nurse responsible for getting a consent for medical-surgical
1. Maintain body temperature at greater than 96.8F treatment? Obtaining consent requires explaining the
with room temperature at 70 to 80F, warming procedure and risks involved, which is the physicians
blankets, warmer for intravenous fluids. responsibility. A nurse may accept responsibility for
2. Maintain greater than 100% PaO2 and witnessing a consent. This carries with it little legal
suction/turn and use positive end-expiratory liability other than obtaining the correct signature and
pressure (PEEP) to prevent hypoxemia caused describing the clients condition at time of signing.
by airway obstruction, pulmonary edema.
3. Maintain central venous pressure at 8 to 10 mm III. CLIENTS RECORDS (DOCUMENTATION)
Hg and systolic blood pressure at greater than A. What should be written in the nurses notes? All
90 mm Hg to prevent hypotension caused by facts and information regarding a persons condi-
complete dilation of systemic vasculature due to tion, treatment, care, progress, and response to
destruction of brains vasomotor center, cessation illness and treatment. Document consent or
of antidiuretic hormone production, and refusal of treatment. Purpose of record: factual
decreased cardiac output. Give fluid bolus and documentation of care given to meet legal
vasopressors, and monitor sodium levels. standards; used to refute unwarranted claims of
4. Maintain fluid and electrolyte balance due to negligence or malpractice.
volume depletion. Monitor for hyponatremia, B. How should data be recorded? Entries should:
hyperkalemia, and hypokalemia, and intake and 1. State date and time given.
output. 2. Be written, signed, and titled by caregiver or
5. Prevent infections due to invasive procedures supervisor who observed action.
(e.g., tubes, catheters) by using aseptic technique. 3. Follow chronological sequence.
4. Be accurate, factual, objective, complete, precise,
and clear.
5. Be legible; use black pen.
Questions Most Frequently 6. Use universal abbreviations.
Asked by Nurses About Nursing 7. Have all spaces filled in on documentation
forms; leave no blank spaces.
and the Law IV. CONFIDENTIAL INFORMATION
I. TAKING ORDERS A. If called on the witness stand in court, do I have to
A. Should I accept verbal phone orders from a physician? reveal confidential information? It depends on your
Generally, no. Specifically, follow your hospitals state, because each state has its own laws pertaining
2164_Ch03_069-114 29/03/12 12:20 PM Page 81

Questions Most Frequently Asked by Nurses About Nursing and the Law 81
to this. Consult a lawyer. Inform the judge and ask to request corrections if they identify errors
for specific directions before relating in court infor- and mistakes. The covered entities must consider
mation that was given to you within a confidential, the changes, but do not have to agree to the
professional relationship. changes, and they may charge clients for the cost
B. Am I justified in refusing (on the basis of invasion of copying and sending the records.
of privacy) to give information about the client to 2. Notice of privacy practices. Covered entities
another health agency to which a client is being must provide a notice to their clients stating
transferred? No. You are responsible for providing how they may use personal medical information
continuity of care when the client is moved from and their rights under the new privacy regula-
one facility to another. Necessary and adequate tion. Clients also may ask covered entities to
information should be transferred between pro- restrict the use or disclosure of their information
fessional health-care workers. The clients consent beyond the practices included in the notice, as
for this exchange of information should be long as the restriction does not interfere with
obtained. Circumstances under which confiden- activities related to treatment, payment, or oper-

ETHICS/LEGAL
tial information can be released include: ations (e.g., family members may not be given
1. By authorization and consent of the client. information about a diagnosis without client
2. By order of the court. permission).
3. By statutory mandate, as in reporting cases of 3. Limits on use of personal medical informa-
communicable diseases or child, elder, or tion. The privacy rule sets limits on how
dependent adult abuse. covered entities may use individually identifi-
V. WHAT IS THE HEALTH INSURANCE able health information. The client has a right
PORTABILITY AND ACCOUNTABILITY to have access to accounting (i.e., the right to
ACT (HIPAA)? know who has been given access to their pro-
tected health information). The health-care
A. Importance of this act for nurses: nurses need to
facility must be able to produce a list describ-
be able to answer client questions regarding the
ing people, companies, or agencies who have
national privacy standards. The principles of the
received protected information. In addition,
law reinforce professional responsibility to avoid
clients must sign a specific authorization
unintentional disclosure of information (e.g., in
before a covered entity can release their
elevators and hallways).
medical information to a life insurer, a bank,
B. Overview of HIPAA: The first-ever federal privacy
or another outside business for purposes not
standards to protect clients medical records and
related to their health care.
other health information provided to health plans,
4. Prohibition on marketing. HIPAA sets new
doctors, hospitals, clinics, nursing homes, pharma-
restrictions and limits on the use of client
cies, and other health-care providers took effect on
information for marketing purposes. Covered
April 14, 2003. Developed by the Department of
entities must first obtain an individuals specific
Health and Human Services (HHS), these new
authorization before disclosing their client
standards provide clients with access to their med-
information for marketing. At the same time,
ical records and more control over how their per-
the rule permits doctors and other covered
sonal health information is used and disclosed.
entities to communicate freely with clients
The standards represent a uniform, federal floor of
about treatment options and other health-related
privacy protections for consumers across the country.
information, including disease-management
State laws providing additional protections to
programs.
consumers are not affected by this new rule.
5. Stronger state laws may remain in effect. The
C. Client protections: The new privacy regulations
new federal privacy standards do not affect
ensure a national floor of privacy protections for
state laws that provide additional privacy
clients by limiting the ways that health plans, phar-
protections for clients. The privacy rule sets a
macies, hospitals, and other covered entities can use
national floor of privacy standards that
clients personal medical information. The regulations
protect all Americans, but any state law that
protect medical records and other individually identi-
provides additional protections would continue
fiable health information, whether it is on paper, in
to apply. When a state law requires a certain
computers, or communicated orally. Key provisions
disclosuresuch as reporting an infectious
and points related to these new standards include:
disease outbreak to the public health
1. Access to medical records. Clients generally
authoritiesthe federal privacy regulations
should be able to see and obtain copies of their
would not preempt the state law.
medical records within 30 days of request, and
2164_Ch03_069-114 29/03/12 12:20 PM Page 82

82 chapter 3 Safe, Effective Care Environment

6. Confidential communications. Under the VI. LIABILITY FOR MISTAKESyours and others.
privacy rule, a client can request that covered A. Is the hospital or the nurse liable for mistakes made
entities take reasonable steps to ensure that by the nurse while following orders? Both the
communications with the client are confidential. hospital and the nurse can be sued for damage
The client has the right to request that communi- if a mistake made by the nurse injures the client.
cations about protected health information The nurse is responsible for own actions. The
remain anonymous if mailed. hospital would be liable, based on the doctrine
7. Minimum necessary rule. It guides the of respondeat superior.
provider to use only the minimum amount of B. Who is responsible if a nursing student or another
information necessary to meet the client care staff nurse makes a mistake? The supervisor? The
needs. The actual diagnosis may not be needed. instructor? Ordinarily the instructor and/or super-
This would relate to the use of e-mail or faxes visor would not be responsible unless the court
to communicate client information. thought the instructor and/or supervisor was neg-
8. Telephone requests for personal health ligent in supervising or in assigning a task beyond
ETHICS/LEGAL

information. Inpatient confidentiality must the capability of the person in question. No one
be protected. The nurse may be able to verify is responsible for anothers negligence unless he
whether a client is in the hospital, but only if or she contributed to or participated in that
the caller asks for the client by name; other- negligence. Each person is personally liable for
wise the caller should be directed to the client his or her own negligent actions and failure to act
or family. as a reasonably prudent nurse.
9. Complaints about violations of privacy. A C. Am I responsible for injury to a client by a staff
health-care facility must identify the privacy member who was observed (but not reported) by me
officer and state how to contact the officer. to be intoxicated while giving care? Yes, you may be
D. There are additional situations where medical responsible. You have a duty to take reasonable
information may be disclosed without authoriza- action to prevent a clients injury.
tion, such as:
VII. GOOD SAMARITAN ACT: For what would I be
1. For workers compensation or similar programs.
liable if I voluntarily stopped to give care at the scene of
2. For public health activities (e.g., reporting
an accident? You would be protected under the Good
births or deaths; injury or disability; abuse or
Samaritan Act and required to live up to reasonable
neglect of children, elders and dependent
and prudent nursing standards in those specific
adults; or reactions to medications), to prevent
circumstances. You would not be treated by the law
or control disease or injury.
as if you were performing under professional standards
3. To a health oversight agency, such as the State
of properly sterile conditions, with proper technical
Department of Health Services.
equipment (see also Section VIII, p. 79).
4. In response to a court or administrative order,
subpoena, warrant, or similar process. VIII. LEAVING AGAINST MEDICAL ADVICE
5. To law enforcement officials in certain limited (AMA): Would I or the hospital be liable if a client left
circumstances. AMA, refusing to sign the appropriate hospital forms?
6. To a coroner, medical examiner, or funeral None of the involved parties would ordinarily be
director. liable in this case as long as (a) the medical risks were
7. To organizations that handle organ, eye, or explained, recorded, and witnessed, and (b) the client
tissue procurement or transplantation. is a competent adult. The law permits clients to make
8. Public health: information may be used or decisions that may not be in their own best health
disclosed to avert a serious threat to health or interest. You cannot interfere with the right and exer-
safety of an individual or the public (Tarasoff cise of the decision to accept or reject treatment.
Principle/Duty to Warn).
IX. RESTRAINTS: Can I put restraints on a client who
9. Food and Drug Administration (FDA): health
is combative even if there is no order for this? Only in
information relating to adverse events with
an emergency, for a limited time (not longer than
respect to immunizations and/or health
24 hours), for the limited purpose of protecting
screening tests may be disclosed.
the client from injurynot for convenience of per-
10. For members of the armed forces, health infor-
sonnel. Notify attending physician immediately.
mation may be disclosed as required by military
Consult with another staff member, obtain clients
command authorities.
consent if possible, and get coworker to witness the
11. To notify a person who may have been exposed
record. Check frequently to ensure restraints do not
to a disease or may be at risk for contracting or
impair circulation, cause pressure sores, or other
spreading a disease or condition.
2164_Ch03_069-114 29/03/12 12:20 PM Page 83

Questions Most Frequently Asked by Nurses About Nursing and the Law 83
injury. Remove restraints at the first opportunity, practicing law without a license. You would be
and use them only as a last resort after other witnessing that (a) the client is signing the document
reasonable means have not been effective (right to as her or his last will and testament; (b) at that time,
least restrictive environment). Restraints of any to the best of your knowledge, the client (testator)
degree may constitute false imprisonment. Freedom was of sound mind, was lucid, and understood what
from unlawful restraint is a basic human right pro- the client was doing (i.e., the client must not be
tected by law. In July 1992, the Food and Drug under the influence of drugs or alcohol or otherwise
Administration (FDA) issued a warning that use of unable to know what she or he is doing); and (c) the
restraints no longer represents responsible, primary testator was under no overt coercion, as far as you
management of a clients behavioral problem. It is could tell, but was acting freely, willingly, and under
necessary to advise the client and family of decision own impetus.
to restrain, explain risks and benefits, and obtain
XI. DISCIPLINARY ACTION
informed consent.
The Joint Commission (formerly known as the A. For what reasons may the RN license be suspended or
revoked?

ETHICS/LEGAL
Joint Commission on Accreditation of Healthcare
Organizations [JCAHO]) has published standards to 1. Obtaining license by fraud (omission of infor-
minimize injury and complications from the use of mation, false information).
restraints: 2. Negligence and incompetence; assuming duties
without adequate preparation.
Use alternatives to physical restraints whenever 3. Substance abuse.
possible (e.g., offer explanations; ask someone to 4. Conviction of crime (state or federal).
stay with client; use clocks, calendars, TV, and 5. Practicing medicine without a license.
radio). 6. Practicing nursing without a license (expired,
Use only under supervision of licensed health-care suspended).
provider. 7. Allowing unlicensed person to practice nursing
Use according to manufacturers direction to avoid or medicine that places the client at risk.
strangulation or circulation impairment. 8. Giving client care while under the influence of
Provide frequent monitoring of client in restraints alcohol or other drugs.
and allow regular, supervised restraint-free periods 9. Habitually using drugs.
to prevent injury. 10. Discriminatory and prejudicial practices in
Obtain written order promptly from practitioner giving client care (pertaining to race, skin
upon initiation of restraints and throughout dura- color, religion, sex, age, or ethnic origin).
tion of use. 11. Falsifying a clients record; failure to maintain a
Documentation must include: time applied and record for each client.
removed; type; medical reason (description of 12. Breach in client confidentiality.
dangerous behavior) and alternatives tried before 13. Physically or verbally abusing a client.
application of restraints; response. 14. Abandoning a client.
A. Nursing responsibilities B. What could happen to me if I am proven guilty of
1. Provide padding to protect skin, bony promi- professional misconduct?
nences, and intravenous lines. 1. License may be revoked.
2. Secure restraints to parts of bed or chair that 2. License may be suspended.
will move with the client and not constrict 3. Behavior may be censured and reprimanded.
movement. Never secure restraints to bed rails 4. You may be placed on probation.
or mattress. C. Who has the authority to carry out any of the afore-
3. Use knots with hitches for easy removal, as mentioned penalties? The State Board of Registered
required. Nursing that granted your license.
4. Maintain proper body alignment when securing D. I am the head nurse. One of my nursing aides has a
restraints. history of failing to appear to work and not giving
5. Remove restraints at least every 2 hours to allow notice of or reason for absence. How should I handle
for activities of daily living. this? An employee has the right to know hospital
policies, what is expected of an employee, and
X. WILLS: What do I do when a client asks me to be a what will happen if an employee does not meet
witness to her or his will? There is no legal obligation the expectations stated in his or her job descrip-
to participate as a witness, but there is a moral and tion or in hospital policies and procedures. As a
ethical obligation to do so. You should not, however, head nurse, you must document behavior factually,
help draw up a will because this could be considered clearly, and concisely, as well as any discussion
2164_Ch03_069-114 29/03/12 12:20 PM Page 84

84 chapter 3 Safe, Effective Care Environment

and decision about future course of action. The If in my judgment the clients pain is not severe, am I
employee must have the chance to read and sign legally covered if I give half of the pain medication dosage
this documentation. The head nurse then sends a ordered by the physician? A nurse cannot substitute his
copy to her or his supervisor. or her judgment for the physicians judgment. If you
XII. FLOATING: Is a nurse hired to work in psychiatry alter the amount of medication prescribed by the
obligated to cover in the intensive care unit (ICU) when physician without a specific order to do so, you may
the latter is understaffed? The issue is the hiring be liable for practicing medicine without a license.
contract (implied or expressed). The contract is a XVII. MALFUNCTIONING EQUIPMENT: At the
composite of the mutual understanding by involved end-of-shift report, the nurse going off duty tells me that
parties of rights and responsibilities, any written the tracheal suctioning machine is malfunctioning and
documents, and hospital policies. If the nurse was describes how she got it to work. Should I plan to use the
hired as a psychiatric nurse, he or she could legally machine in the evening shift and follow her suggestions
refuse to go to the ICU. If the hospital intends to about how to make it work? Do not plan to use equip-
float personnel, such a policy should be clearly stated ment that you know is not functioning properly. You
ETHICS/LEGAL

during the hiring process. Also at this time, the could be held liable because you could reasonably
employer should determine the employees education, foresee that proper functioning of equipment would
skills, and experience. On the other hand, if emergency be needed for your client. You have been put on
staffing problems exist, a nurse should go to the ICU notice that there are defects. Report this to the super-
regardless of personal preference, but should request visor or person responsible for maintaining equipment
orientation and not assume responsibility beyond level in proper working order. (Also see p. 72: Safe Use of
of experience or education. Equipment.)
XIII. DISPENSING MEDICATION: Can a nurse
legally remove a drug from a pharmacy when the
pharmacy is closed (during the night) if the physician Ethical and Legal Considerations
insists that the nurse go to the pharmacy to get the
specifically prescribed medication immediately? Within
in Intensive Care of the Acutely Ill
the legal boundaries of the Pharmacy Act, a nurse Neonate
may remove one dose of a particular drug from
the pharmacy for a particular client during an I. RESPONSIBILITIES OF THE HEALTH AGENCY
unanticipated emergency within a limited time and A. Provide a neonatal intensive care unit (NICU) or
availability of resources. However, the hospital should transfer to another hospital.
have a written policy for the nurse to follow and B. Personneladequate number trained in neonate dis-
should authorize a specific person to use the services eases, special treatment, and equipment.
of the pharmacy under certain circumstances. C. Equipmentadequate supply on hand, function-
ing properly (especially temperature regulator in
XIV. ILLEGIBLE ORDERS: What should I do if I can- incubator, oxygen analyzer, blood-gas machine).
not decipher the physicians handwriting when she or he
persists in leaving illegible orders? Talk to the physician II. INFANTS WHO ARE DYING
regarding the dangers of your giving the wrong A. Decision regarding resuscitation in cardiac arrest,
amount of the wrong medication via the wrong route with brain damage from cerebral anoxia. It is diffi-
at the wrong time. If that does not help, follow appro- cult to predict the effect of anoxia in infancy on
priate channels. Do not follow an order you cannot the childs later life.
read. You will be liable for following orders you B. Decision to continue supportive measures.
thought were written. C. Issue of euthanasia, such as in severe
myelomeningocele at birth.
XV. HEROIC MEASURES: The wife of a client who is 1. Active euthanasia (giving overdose).
terminally ill approaches me with the request that heroic 2. Passive euthanasia (not placing on respirator).
measures not be used on her husband. She has not dis-
cussed this with him but knows that he feels the same III. EXTENDED ROLE OF NURSE IN NICUmay
way. Can I act on this request? No. The client is the raise issues of nursing practice versus medical practice, as
only one who can legally make the decision as long as when a nurse draws blood samples for blood gas deter-
he or she is mentally competent. minations without prior order. To be legally covered:
A. The nurse must be trained to perform specialized
XVI. MEDICATION: A physician orders pain medica- functions.
tion prn for a client. The client asks for the medication, B. The functions must be written into the nurses job
but when I question her she says the pain isnt so bad. description.
2164_Ch03_069-114 29/03/12 12:20 PM Page 85

Legal Aspects of Psychiatric Care 85


IV. ISSUE OF NEGLIGENCEsuch as cross-
contamination in nursery.
V. ISSUE OF MALPRACTICEsuch as assigning
care of an infant who is critically ill on respirator to
a student or aide who is untrained.
A. May be liable for inaccurate bilirubin studies for
neonatal jaundice; may be legally responsible if
brain damage occurs in absence of accurate
laboratory tests.
B. May be liable for brain damage in an infant due to
respiratory or cardiac distress. Nurse must make
sure that there are frequent blood gas determina-
tions to ensure adequate oxygen to prevent brain
damage. Nurse also must make sure that the infant

ETHICS/LEGAL
is not receiving too high a concentration of oxygen,
which may lead to retrolental fibroplasia.

Legal Aspects of Psychiatric Care Figure 3.1 Typical procedure for involuntary commitment.
I. FOUR SETS OF CRITERIA TO DETERMINE
CRIMINAL RESPONSIBILITY AT TIME OF
ALLEGED OFFENSE III. LEGAL AND CIVIL RIGHTS OF CLIENTS
A. MNaghten Rule (1832)a person is not guilty if: WHO ARE HOSPITALIZEDthe right to:
1. Person did not know the nature and quality of A. Wear own clothes, keep and use personal posses-
the act. sions and reasonable sum of money for small
2. Person could not distinguish right from wrong purchases.
if person did not know what he or she was B. Have individual storage space for private use.
doing, person did not know it was wrong. C. See visitors daily.
B. The Irresistible Impulse Test (used together with D. Have reasonable access to confidential phone
MNaghten Rule)person knows right from conversations.
wrong, but: E. Receive unopened correspondence and have access
1. Driven by impulse to commit criminal acts to stationery, stamps, and a mailbox.
regardless of consequences. F. Refuse shock treatments, lobotomy.
2. Lacked premeditation in sudden violent
behavior. IV. CONCEPTS CENTRAL TO COMMUNITY
C. American Law Institutes Model Penal Code MENTAL HEALTH (COMMUNITY MENTAL
(1955) Test HEALTH ACT, 1980)
1. Not responsible for criminal act if person lacks A. Systems perspective: scope of care moves beyond
capacity to appreciate the wrongfulness of it or the individual to the community, with influences
to conform conduct to requirements of law. from biological, psychological, and sociocultural
2. Excludes an abnormality manifested only by forces.
repeated criminal or antisocial conduct B. Emphasis on prevention: primary (reduce incidents
namely, psychopathology. by preventing harmful social conditions); secondary
3. Includes knowledge and control criteria. (early identification and treatment of disorders to
D. Durham Test (Product Rule1954): accused is reduce duration); tertiary (early rehabilitation to
not criminally responsible if act was a product of reduce impairment from disorders).
mental disease. Discarded in 1972. C. Interdisciplinary collaboration: flexible roles based on
unique areas of expertise.
II. TYPES OF ADMISSIONS
D. Consumer participation and control.
A. Voluntary: person, parent, or legal guardian applies E. Comprehensive services: ambulatory care, partial
for admission; person agrees to receive treatment hospitalization, 24-hour hospitalization and
and to follow hospital rules; civil rights are emergency care; consultation and education;
retained. screening services.
B. Involuntary: process and criteria vary among states F. Continuity of care.
(Fig. 3.1).
2164_Ch03_069-114 29/03/12 12:20 PM Page 86

86 chapter 3 Safe, Effective Care Environment

Legal Aspects of Preparing a Client advisable to obtain either oral or written


informed consent from someone who has
for Surgery authority to act for the client.
2. Verbal consents should be recorded in detail,
I. NO SURGICAL PROCEDURE, HOWEVER witnessed and signed by two individuals.
MINOR, CAN PROCEED WITHOUT THE 3. Written or verbal consent can be given by alert,
VOLUNTARY, INFORMED, AND WRITTEN coherent, or otherwise competent adults, by
CONSENT OF THE CLIENT. parents, legal guardian, or person in loco parentis
A. Surgical permits are witnessed by the physician, (one standing in for the parent with the parents
nurse, or other authorized person. rights, duties, and responsibilities) of minors or
B. Surgical permits protect the client against unsanc- adults who are incompetent.
tioned surgery and also protect the surgeon and 4. If the minor is 14 years old or older, consent
hospital staff against claims of unauthorized must be acquired from the minor as well as from
operations. the parent or legal guardian. Emancipated minors
C. Informed consent means that the operation has been
ETHICS/LEGAL

can consent for themselves.


fully explained to the client, including possible
complications and disfigurements, as well as
whether any organ or parts of the body are to be TRENDS IN NURSING
removed. PRACTICE
D. Adults and emancipated minors may sign their own I. OVERALL CHARACTERISTICS
operative permits if they are mentally competent; A. Some trends are subtle and slow to emerge; others
permission for surgery of minor children and adults are obvious and quickly emerge.
who are incompetent or unconscious must be B. Trends may conflict; some will prevail, others get
obtained from a responsible family member or modified by social forces.
guardian.
E. The signed operative permit is placed in a promi- II. GENERAL TRENDS
nent place on the clients chart and accompanies A. Broadened focus of carefrom care of ill to care of
the client to the operating room. sick and healthy, from care of individual to care of
F. Legal issues in the emergency department: record- family. Focus on prevention of illness, promotion
keeping plays an essential role in both the preven- of optimum level of health, holism.
tion and defense of malpractice suits. Detailed B. Increasing scientific basein biological-social-
documentation not only provides for continuity physical sciences, not mere reliance on intuition,
of care but also perpetuates evidence that care was experience, and observation.
appropriately given. Records should: C. Increasingly complex technical skills and use of tech-
1. Be written legibly. nologically advanced equipment, such as monitors
2. Clearly note events and time of occurrence. and computers.
3. Contain all laboratory slips and results of other D. Increased independence in use of judgment, such
tests. as teaching nutrition in pregnancy and providing
4. Describe events and clients objectively. primary prenatal care.
5. Clearly note physicians parting instructions to E. New roles, such as nurse clinician, require
the client. advanced skills in a particular area of practice.
6. Be signed where appropriate, such as with Examples: psychiatric nurse consults with staff
doctors orders. about problems; primary care nurse takes medical
7. Contain descriptions of every event that might histories and does physical assessment; one nurse
lead to a lawsuit, such as fights, injuries, equip- coordinates 24-hour care during hospital stay;
ment failures. independent nurse practitioner has her or his own
G. Consentalthough there is no law requiring written office in community where clients come for care;
consent before performing medical treatment, all case management.
elective procedures can only be performed if the F. Community nursing services rather than hospital-
client has been fully informed and voluntarily con- based; needs of the healthy are served as well as
sents to the procedure. those of the ill.
1. If informed consent cannot be obtained because G. Development of nursing standards to reflect specific
of the clients condition and immediate treat- nursing functions and activities.
ment is necessary to save life or safeguard health, 1. Ensure safe standard of care to clients and families.
the emergency rule can be applied. This rule 2. Provide criteria to measure excellence and
implies consent. However, if time allows, it is effectiveness of care.
2164_Ch03_069-114 29/03/12 12:20 PM Page 87

Trends in Nursing Practice 87


III. TRENDS IN CARE OF CHILDBEARING *10. Screening services (courts).
FAMILY 11. Alcohol abuse services.
A. Consumerism: 12. Drug abuse services.
1. Consumer push for humanization and E. Protecting human rights of persons in need of
individualization of health care during the child- mental health care.
bearing cycle to reflect clients role in decision F. Developing an advocacy program for those who are
making, preferences, and cultural diversity. chronically mentally ill.
2. Emphasis on family-centered care (including G. Improving delivery of services to underserved and
father, siblings, grandparents). high-risk populations (e.g., minorities).
3. Increase in options available for conduct of V. ANA STANDARDS OF CLINICAL NURSING
birth experience and setting for birth: birthing PRACTICE*
homes, alternative birth center (ABC) in hospi- A. Use of nursing process: assessment, nursing diagno-
tals; birthing chairs; side-lying position for sis, planning, implementation, evaluation, outcome
birth; family-centered cesarean birth; health- identification.

ETHICS/LEGAL
care provider (MD, RN, lay midwife); length of B. Performance appraisal review.
postpartum stay. C. Continuing education.
4. Increased consumer awareness of legal issues, D. Collegiality; peer review.
clients rights. E. Ethics.
5. Major nursing role: client advocate. F. Interdisciplinary collaboration.
B. Social trends: G. Research.
1. Alternative lifestyles of familiessingle parent- H. Resource utilizationutilization of community
hood, communal living, surrogate motherhood, health systems.
marriages without children.
2. Earlier sexual experimentationavailability of VI. FOUR LEVELS OF NURSING PRACTICE
assistance to emancipated minors. A. Promotion of health to increase level of wellness.
3. Increase in number of older (>38 years) Example: provide dietary information to reduce
primiparas. risks of coronary artery diseases.
4. Legalization of abortion; availability to emanci- B. Prevention of illness or injury. Example:
pated minors. immunizations.
5. Smaller families. C. Restoration of health. Example: teach how to change
6. Rising divorce rates. dressing, care for wound.
C. Technologies: D. Consolation of dyingassist person to attain peace-
1. Development of genetic and bioengineering ful death.
techniques. VII. FOUR COMPONENTS OF NURSING CARE
2. Development of prenatal diagnostic tech- A. Nursing care activitiesassist with basic needs, give
niques, with options for management of medications and treatments; observe response and
each pregnancy. adaptation to illness and treatments; teach self-care;
3. In vitro fertilization and embryo transplantation. guide rehabilitation activities for daily living.
IV. TRENDS IN COMMUNITY MENTAL HEALTH B. Case managementa process that includes coordina-
(1960s1990s) tion of total client careall health team members
A. Shift from institutional to community-based care. should work together toward common goals.
B. Preventive services. C. Continuity of careprocess of ensuring that
C. Consumer participation in planning and delivery of ongoing physical, medical, and emotional
services. health-care needs are assessed, planned for, and
D. Original 12 essential services (1975) reduced to coordinated with all providers, for desired
5 (indicated by asterisk [*]) (1981). outcomes, without interruption of service when
*1. 24-hour inpatient care. the location of care is transferred.
*2. Ambulatory care. D. Evaluation of careflexibility and responsiveness to
*3. Partial hospitalization (day or night). changing needs: clients reactions and perceptions
4. Emergency care. of their needs.
*5. Consultation and education.
6. Follow-up care.
7. Transitional services. *Adapted from the complete description in American Nurses Association: 2004 Nursing:
Scope and Standards of Practice. Washington, DC, American Nurses Association, 2004.
8. Services for children and adolescents. Reprinted with permission. (Rationale and assessment factors for the Standards of Clinical
9. Services for elderly. Nursing Practice are available from the American Nurses Association.)
2164_Ch03_069-114 29/03/12 12:20 PM Page 88

88 chapter 3 Safe, Effective Care Environment

VIII. THREE MAIN NURSING ROLES in relation to III. AMERICAN NURSES ASSOCIATION (ANA)
care of clients and their families (emphasis of each role national professional association in the United
varies with the situation, with adaptation of skills and States, composed of the nurses associations of the
modes of care as necessary) 50 states, Guam, Virgin Islands, Puerto Rico, and
A. Therapeutic role (instrumental). Function: work Washington, DC
toward cure in acute setting. A. Purpose: to foster high standards of nursing practice
B. Caring role (expressive). Function: provide support and promote the education and welfare of nurses
through human relations, show concern, demon- B. Functions: officially represents professional nurses in
strate acceptance of differences. the United States and internationally; defines prac-
C. Socializing role. Function: offer distractions and tice of nursing; lobbies and promotes legislation
respite from focus on illness. affecting nurses welfare and practice
IV. NATIONAL LEAGUE FOR NURSING (NLN)
composed of both individuals and agencies
INFECTION CONTROL

NURSING
O R G A N I Z AT I O N S A. Purpose: to foster the development and improve-
ment of all nursing services and nursing education
I. INTERNATIONAL COUNCIL B. Functions
OF NURSES (ICN) 1. Provides educational workshops.
A. Purpose: to provide a medium through which 2. Assists in recruitment for nursing programs.
national nursing associations can work together and 3. Provides testing services for both RN and LPN
share common interests; formed in 1899 (LVN) nursing programs.
B. Functions
1. Serves as representatives of and spokespersons for
nurses at international level.
SAFETY AND
2. Promotes organization of national nurses
INFECTION
associations.
CONTROL:
3. Assists national organizations to develop and
GUIDELINES FOR
improve services for public health practice of
I S O L AT I O N A N D
nursing and social/economic welfare of nurses.
S TA N D A R D
PRECAUTIONS
II. WORLD HEALTH ORGANIZATION (WHO)
special intergovernmental agency of the United A key component of the NCLEX-RN test plan is injury
Nations, formed in 1948 prevention in the provision of a safe environment. This
section also identifies potential risks to child safety with
A. Purpose: to bring all people to the highest possible
recommendations for a child-safe environment.
level of health
B. Functions: provides assistance in the form of educa- I. GUIDELINES FOR ISOLATION AND
tion, training, improving health standards, fighting INFECTION CONTROL
disease, and reducing water pollution in member A. Guidelines for Isolation Precautions (Table 3.2)
countries B. Contact PrecautionsSummary (Table 3.3)
(text continues on page 91)

Table 3.2
Guidelines for Isolation Precautions
The Hospital Infection Control Advisory Committee and the Centers for Disease Control and Prevention have concluded that some
fluids, secretions, and excretions not covered under Universal Precautions (UP) represent a potential source of nosocomial and
community-acquired infections. The term Universal Precautions has been replaced in the hospital setting by a two-tiered system of
isolation: Standard Precautions for control of nosocomial infections; and Transmission-based Precautions for clients known or
suspected to be infected with highly transmissible pathogens. Standard Precautions incorporate the major components of blood
and body fluid precautions from Universal Precautions. Transmission-based Precautions may be combined for diseases that
have multiple routes of transmission and are used in addition to Standard Precautions.
Standard Precautions
Use Standard Precautions, or the equivalent, for the care of all clients.
1. Handwashing.
a. Wash hands after touching: blood, body fluids, secretions, excretions, and contaminated items, whether or not gloves are
worn. Wash hands immediately: after gloves are removed, between client contacts, and when otherwise indicated to avoid
transfer of microorganisms to other clients or environments. It may be necessary to wash hands between tasks and procedures
on the same client to prevent cross-contamination of different body sites.
2164_Ch03_069-114 29/03/12 12:20 PM Page 89

Safety and Infection Control 89

Table 3.2
Guidelines for Isolation Precautionscontd
b. Use a plain (non-antimicrobial) soap for routine hand washing.
c. Use an antimicrobial agent or a waterless antiseptic agent for specific circumstances (e.g., control of outbreaks or hyperen-
demic infections), as defined by the infection control program (see Contact Precautions for additional recommendations on
using antimicrobial and antiseptic agents).
2. Gloves. Wear gloves (clean, nonsterile gloves are adequate) when touching: blood, body fluids, secretions, excretions, and con-
taminated items. Put on clean gloves just before touching mucous membranes and nonintact skin. Change gloves between tasks
and procedures on the same client after contact with material that may contain a high concentration of microorganisms. Remove
gloves promptly after use, before touching noncontaminated items and environmental surfaces, and before going to another
client; wash hands immediately to avoid transfer of microorganisms to other clients or environments.
3. Mask, eye protection, face shield. Wear a mask and eye protection or a face shield to protect mucous membranes of the eyes,
nose, and mouth during procedures and client care activities that are likely to generate splashes or sprays of blood, body fluids,
secretions, and excretions.

INFECTION CONTROL
4. Gown. Wear a gown (a clean, nonsterile gown is adequate) to protect skin and to prevent soiling of clothing during procedures
and client care activities that are likely to generate splashes or sprays of blood, body fluids, secretions, or excretions. Select a
gown that is appropriate for the activity and amount of fluid likely to be encountered. Remove a soiled gown as promptly as possi-
ble, and wash hands to avoid transfer of microorganisms to other clients or environments.
5. Client care equipment. Handle used client care equipment soiled with blood, body fluids, secretions, and excretions in a manner
that prevents skin and mucous membrane exposures, contamination of clothing, and transfer of microorganisms to other clients
and environments. Ensure that reusable equipment is not used for the care of another client until it has been cleaned and
reprocessed appropriately. Ensure that single-use items are discarded properly.
6. Environmental control. Ensure that the hospital has adequate procedures for the routine care, cleaning, and disinfection of envi-
ronmental surfaces, beds, bed rails, bedside equipment, and other frequently touched surfaces, and ensure that these procedures
are being followed.
7. Linens. Handle, transport, and process used linens soiled with blood, body fluids, secretions, and excretions in a manner that pre-
vents skin and mucous membrane exposures and contamination of clothing, and that avoids transfer of microorganisms to other
clients and environments.
8. Occupational health and blood-borne pathogens.
a. Take care to prevent injuries when using needles, scalpels, and sharp instruments or devices; when handling sharp instruments
after procedures; when cleaning used instruments; and when disposing of used needles. Never recap used needles, or other-
wise manipulate them using both hands, or use any other technique that involves directing the point of a needle toward any
part of the body; rather, use either a one-handed scoop technique or a mechanical device designed for holding the needle
sheath. Do not remove used needles from disposable syringes by hand, and do not bend, break, or otherwise manipulate used
needles by hand. Place used disposable syringes and needles, scalpel blades, and other sharp items in appropriate puncture-
resistant containers that are located as close as practical to the area in which the items were used, and place reusable syringes
and needles in a puncture-resistant container for transport to the reprocessing area.
b. Use mouthpieces, resuscitation bags, or other ventilation devices as an alternative to mouth-to-mouth resuscitation methods in
areas where the need for resuscitation is predictable.
9. Client placement. Place a client who contaminates the environment or who does not (or cannot be expected to) assist in maintain-
ing appropriate hygiene or environmental control in a private room. If a private room is not available, consult with infection control
professionals regarding client placement or other alternatives.
Transmission-Based Precautions
Airborne Precautions
In addition to Standard Precautions, use Airborne Precautions, or the equivalent, for clients known or suspected to be infected
with microorganisms transmitted by airborne droplet nuclei (small-particle residue [5 microns or smaller in size] of evaporated
droplets containing microorganisms that remain suspended in the air and that can be dispersed widely by air currents within a room
or over a long distance).
1. Client placement. Place the client in a private room that has (1) monitored negative air pressure in relation to the surrounding
area, (2) 612 air changes per hour, and (3) appropriate discharge of air outdoors or monitored high-efficiency filtration of room air
before the air is circulated to other areas in the hospital. Keep the room door closed and the client in the room. When a private
room is not available, place the client in a room with a client who has active infection with the same microorganism, unless other-
wise recommended, but with no other infection (cohorting). When a private room is not available and cohorting is not desirable,
consultation with infection control professionals is advised before client placement.
2. Respiratory protection. Wear respiratory protection when entering the room of a client with known or suspected infectious pul-
monary tuberculosis. People who are susceptible should not enter the room of clients known or suspected to have measles (rube-
ola) or varicella (chickenpox) if other caregivers who are immune are available. If persons who are susceptible must enter the room
of a client known or suspected to have measles (rubeola) or varicella, they should wear respiratory protection. Persons immune to
measles (rubeola) or varicella need not wear respiratory protection.
3. Client transport. Limit the movement and transport of the client from the room to essential purposes only. If transport or move-
ment is necessary, minimize client dispersal of droplet nuclei by placing a surgical mask on the client, if possible.
4. Additional precautions for preventing transmission of tuberculosis. Consult CDC Guidelines for Preventing the Transmission
of Tuberculosis in Health-Care Facilities for additional prevention strategies.
Continued
2164_Ch03_069-114 29/03/12 12:20 PM Page 90

90 chapter 3 Safe, Effective Care Environment

Table 3.2
Guidelines for Isolation Precautionscontd
Droplet Precautions
In addition to Standard Precautions, use Droplet Precautions, or the equivalent, for a client known or suspected to be infected
with microorganisms transmitted by droplets (large-particle droplets [larger than 5 microns in size] that can be generated by the
client during coughing, sneezing, talking, or the performance of procedures).
1. Client placement. Place the client in a private room. When a private room is not available, place the client in a room with a client
who has active infection with the same microorganism but with no other infection (cohorting). When a private room is not available
and cohorting is not achievable, maintain spatial separation of at least 3 ft between the client who is infected and other clients
and visitors. Special air handling and ventilation are not necessary, and the door may remain open.
2. Mask. In addition to standard precautions, wear a mask when working within 3 ft of the client. (Logistically, some hospitals may
want to implement the wearing of a mask to enter the room.)
3. Client transport. Limit the movement and transport of the client from the room to essential purposes only. If transport or move-
ment is necessary, minimize client dispersal of droplets by masking the client, if possible.
INFECTION CONTROL

Contact Precautions
In addition to Standard Precautions, use Contact Precautions, or the equivalent, for specified clients known or suspected to be
infected or colonized with epidemiologically important microorganisms that can be transmitted by direct contact with the client (hand
or skin-to-skin contact) that occurs when performing client care activities that require touching the clients dry skin or indirect contact
(touching) with environmental surfaces or client care items in the clients environment.
1. Client placement. Place the client in a private room. When a private room is not available, place the client in a room with a client
who has active infection with the same microorganism but with no other infection (cohorting). When a private room is not available
and cohorting is not achievable, consider the epidemiology of the microorganism and the client population when determining
client placement. Consultation with infection control professionals is advised before client placement.
2. Gloves and handwashing. In addition to wearing gloves as outlined under Standard Precautions, wear gloves (clean,
nonsterile gloves are adequate) when entering the room. During the course of providing care for a client, change gloves
after having contact with infective material that may contain high concentrations of microorganisms (fecal material and
wound drainage). Remove gloves before leaving the clients environment and wash hands immediately with an antimicro-
bial agent or a waterless antiseptic agent. After glove removal and hand washing, ensure that hands do not touch potentially
contaminated environmental surfaces or items in the clients room to avoid transfer of microorganisms to other clients or
environments.
3. Gown. In addition to wearing a gown as outlined under Standard Precautions, wear a gown (a clean, nonsterile gown is
adequate) when entering the room if you anticipate that your clothing will have substantial contact with the client, environmen-
tal surfaces, or items in the clients room; or if the client is incontinent or has diarrhea, an ileostomy, a colostomy, or wound
drainage not contained by a dressing. Remove the gown before leaving the clients environment. After gown removal, ensure
that clothing does not contact potentially contaminated environmental surfaces to avoid transfer of microorganisms to other
clients or environments.
4. Client transport. Limit the movement and transport of the client from the room to essential purposes only. If the client is trans-
ported out of the room, ensure that precautions are maintained to minimize the risk of transmission of microorganisms to other
clients and contamination of environmental surfaces or equipment.
5. Client care equipment. When possible, dedicate the use of noncritical client care equipment to a single client (or cohort of
clients infected or colonized with the pathogen requiring precautions) to avoid sharing between clients. If use of common equip-
ment or items is unavoidable, adequately clean and disinfect them before use for another client.
Source: Centers for Disease Control and Prevention and Hospital Infection Control Practices Advisory Committee.

Table 3.3
Contact PrecautionsSummary
1. Private room if the clients hygiene is poor. In general, clients with the same infection may share a room.
2. Masks are not indicated.
3. Gowns are indicated if soiling is likely.
4. Gloves are indicated for touching infective material.
5. Hands must be washed before and after touching the client or potentially contaminated articles.
6. Contaminated articles should be discarded or bagged and labeled.
2164_Ch03_069-114 29/03/12 12:20 PM Page 91

Safety and Infection Control 91


C. Standard PrecautionsSummary (Table 3.4) II. GUIDELINES FOR ENVIRONMENTAL SAFETY
D. Airborne and Droplet PrecautionsSummary A. Safety Considerations When Caring for
(Table 3.5) Hospitalized Infants, Children, and Adolescents
E. Infection Control by Clinical Syndrome/Condition (Table 3.9)
(Table 3.6) B. Safety and Injury PreventionHome Safety
F. Infection Control When Caring for Infants, (Table 3.10)
Children, and Adolescents (Table 3.7)
G. Summary: Types of Precautions and Illnesses
Requiring the Precautions (Table 3.8)

Table 3.4
Standard PrecautionsSummary

INFECTION CONTROL
Wash your hands and any other skin surfaces immediately and thoroughly if they become contaminated with blood or other body
fluids. Use lots of soap and hot water.
Wear clean gloves whenever there is potential exposure to blood or other body fluids except sweat. Wear surgical gloves for
performing venipunctures, for touching mucous membranes or nonintact skin, or whenever there is a possibility of exposure to
blood or body fluids. Remove gloves after contact with each client. Discard used gloves immediately after use in an appropri-
ate receptacle (e.g., a plastic bag with a biohazard label). Wash your hands immediately after removing gloves.
Use mask, protective eyewear, face shield, gown during any procedure that is likely to generate splashes of blood or other body
fluids.
Handle all needles, intravenous equipment, and sharp instruments with extreme care:
1. Never recap, remove, bend, or break needles after use or manipulate them in any other way by hand.
2. Dispose of syringes, needles, scalpel blades, and other sharp items in a puncture-resistant container kept within easy reach.

Table 3.5
Airborne and Droplet PrecautionsSummary
When Used
For infectious diseases that are transmitted through inhalation of airborne particles (e.g., tuberculosis) or infected microorganisms in
droplets (e.g., Haemophilus influenzae, pertussis)
Precautions
Isolate in private room (clients infected with same disease can be placed in same room)
Client should wear mask if out of room for testing, etc.
Masks to be worn by personnel when working within 3 ft of client
Gowns not necessary
Contaminated articles need to be labeled before being sent for decontamination
Provide adequate ventilation in clients room
Careful hand washing

Table 3.6
Infection Control
Conditions That Need Additional Precautions
Clinical Syndrome or Condition Potential Pathogens Empirical Precautions
Diarrhea
Acute diarrhea with a likely infectious cause in a client who is incontinent Enteric pathogens Contact
or is in diapers
Diarrhea in an adult with a history of recent antibiotic use Clostridium difficile Contact
Meningitis
Rash or exanthems, generalized, etiology unknown
Petechial/ecchymotic with fever Neisseria meningitidis Droplet
Vesicular Varicella Airborne and Contact
Maculopapular with coryza and fever Rubeola (measles) Airborne
Continued
2164_Ch03_069-114 29/03/12 12:20 PM Page 92

92 chapter 3 Safe, Effective Care Environment

Table 3.6
Infection Controlcontd
Conditions That Need Additional Precautions
Clinical Syndrome or Condition Potential Pathogens Empirical Precautions
Respiratory Infections
Cough/fever/upper lobe pulmonary infiltrate in a client who is HIV negative Mycobacterium Airborne
or a client at low risk for HIV infection tuberculosis
Cough/fever/pulmonary infiltrate in any lung location in a client who is HIV Mycobacterium Airborne
infected or a client at high risk for HIV infection tuberculosis
Paroxysmal or severe persistent cough during periods of pertussis activity Bordetella pertussis Droplet
Respiratory infections, particularly bronchiolitis and croup, in infants and Respiratory syncytial Contact
young children or parainfluenza virus
Risk of Multidrug-Resistant Microorganisms
INFECTION CONTROL

History of infection or colonization with multidrug-resistant organisms Resistant bacteria Contact


Skin, wound, or urinary tract infection in a client with a recent hospital or Resistant bacteria Contact
nursing home stay in a facility where multidrug-resistant organisms are
prevalent
Skin or Wound Infection
Abscess or draining wound that cannot be covered Staphylococcus aureus, Contact
group A streptococcus
Adapted from CDC Healthcare Infection Control Practices Advisory Committee, from www.cdc.gov/ncidod/hip/
ISOLAT/isopart2.htm.

Table 3.7
Infection Control When Caring for Infants, Children, and Adolescents
Type Definition Equipment Clinical Applications
Standard Reduce the risk of transmission of Gloves Blood
precautions infection from all recognized and Protective eyewear Body fluids
unrecognized sources of infection Gown Secretions
(formerly termed universal precautions) Mask Excretions
Any break in skin or mucous
membranes
Airborne Reduce the risk of airborne transmis- Special air handling Measles
precautions sion of infectious agents transmitted systems Varicella
by airborne droplet nuclei Ventilation systems: Tuberculosis
HEPA filter masks,
N95 respirator
Droplet Reduce the risk of airborne transmis- Surgical mask Haemophilus influenzae type b
precautions sion of infectious agents transmitted Meningitis
by large-particle droplets Pertussis
Streptococcal pharyngitis
Scarlet fever
Contact Reduce the risk of transmission of Gloves Enteric infections
precautions infectious agents by direct client Gown Skin/wound infections
contact or indirect contact with Respiratory syncytial virus (RSV)
contaminated items in the clients Herpes simplex
environment Impetigo
Viral hemorrhagic conjunctivitis
2164_Ch03_069-114 29/03/12 12:20 PM Page 93

Summary: Types of Precautions and Illnesses Requiring the Precautions 93

Table 3.8
Summary: Types of Precautions and Illnesses Requiring the Precautions
Standard Precautions
Use Standard Precautions for the care of all clients.
Airborne Precautions
In addition to Standard Precautions, use Airborne Precautions for clients known or suspected to have serious illnesses transmitted
by airborne droplet nuclei.
Examples of such illnesses include:
Measles
Varicella (including disseminated zoster)
Tuberculosis

INFECTION CONTROL
Droplet Precautions
In addition to Standard Precautions, use Droplet Precautions for clients known or suspected to have serious illnesses transmitted by
large-particle droplets.
Examples of such illnesses include:
Invasive Haemophilus influenzae type b disease, including meningitis, pneumonia, epiglottitis, and sepsis
Invasive Neisseria meningitidis disease, including meningitis, pneumonia, and sepsis
Other serious bacterial respiratory infections spread by droplet transmission, including:
Diphtheria (pharyngeal)
Mycoplasma pneumonia
Pertussis
Pneumonic plague
Streptococcal (group A) pharyngitis, pneumonia, or scarlet fever in infants and young children
Serious viral infections spread by droplet transmission, including:
Adenovirus
Influenza
Mumps
Parvovirus B19
Rubella
Contact Precautions
In addition to Standard Precautions, use Contact Precautions for clients known or suspected to have serious illnesses easily trans-
mitted by direct client contact or by contact with items in the clients environment.
Examples of such illnesses include:
Gastrointestinal, respiratory, skin, or wound infections or colonization with multidrug-resistant bacteria judged by the infection control
program, based on current state, regional, or national recommendations, to be of special clinical and epidemiologic significance
Enteric infections with a low infectious dose or prolonged environmental survival, including:
Clostridium difficile
For clients who are in diapers or incontinent: enterohemorrhagic Escherichia coli O157:H7, Shigella, hepatitis A, or rotavirus
Respiratory syncytial virus, parainfluenza virus, or enteroviral infections in infants and young children
Skin infections that are highly contagious or that may occur on dry skin, including:
Diphtheria (cutaneous)
Herpes simplex virus (neonatal or mucocutaneous)
Impetigo
Major (noncontained) abscesses, cellulitis, or decubiti
Pediculosis
Scabies
Staphylococcal furunculosis in infants and young children
Zoster (disseminated or in the host who is immunocompromised)
Viral/hemorrhagic conjunctivitis
Viral hemorrhagic infections (Ebola, Lassa, or Marburg)
Adapted from Fundamentals of Isolation Precautions, CDC Healthcare Infection Control Practices Advisory
Committee, from www.cdc.gov/ncidod/hip/ISOLAT/isopart2.htm.
2164_Ch03_069-114 29/03/12 12:20 PM Page 94

94 chapter 3 Safe, Effective Care Environment

Table 3.9
Safety Considerations When Caring for Hospitalized Infants, Children, and Adolescents
Area of Concern Safety Interventions
Security Be alert to all visitors; restrict prn
Monitor/report suspicious visitors
Assess high-risk children (e.g., custody disputes)
Identify person to whom child is discharged
All staff to wear photo ID, especially when transporting child
Transport one child at a time; never leave child unattended during transport
Do not allow visitors to borrow scrubs or lab coat
Infants placed near nurses station, in direct line of vision
Access to pediatrics unit limited; doors alarmed to indicate when someone enters or leaves the unit
Electrical equipment Maintain all equipment in good working order
INFECTION CONTROL

(e.g., apnea monitor, Remove leads when not attached to monitor


ECG monitor, respirator) Unplug power cord when not connected to equipment
Keep equipment away from moisture (e.g., puddle of water, tub)
Do not turn off alarm for any reason
Always check the child when an alarm sounds; never ignore the alarm
Teach other children not to touch or play with the equipment
Home care:
CPR guidelines posted near bed
Emergency numbers posted near phone
Notify electric company, get on priority service list in case of power outage
Use intercom or monitoring system
Environment Keep unused electrical outlets covered with childproof caps
Secure screens on all windows
Strap infant securely into infant seat, high chair, stroller
Discourage use of walkers
Crib rails up at all times when infant or young child is alone
When a crib rail is down, keep one hand on the infant or young child at all times
Keep plants or flowers out of the reach of infants or young children
No small or sharp objects within reach of infants or young children; no pins on diapers
No small or removable parts of toys of infants or young children
Keep all medications out of the reach of infants or young children
Never leave an infant or young child alone in a bath or near water

Table 3.10
Safety and Injury PreventionHome Safety
These safety measures should be used in homes where children live and in homes they frequently visit, e.g., grandparents or
babysitters.
Burns, Electrical, and Fire Falls
Guards in front of or around heating appliances, fireplace, Exits, halls, stairs free of obstructions and well lighted
furnace (including floor furnace) Nonskid mats and safety strips in tubs and showers and on
Electrical wires in good repair and out of reach stairs
Electrical outlets capped Decals on glass doors/walls
Smoke detectors operational Safety glass in doors/walls/windows
Matches out of reach Gates at doors/stairwells
All heated appliances/objects placed out of childs reach and Guard rails on upstairs windows
disconnected when not in use Safety locks/latches that prevent/limit opening in use on
Hot water heater set at 49C (120F) or lower doors/windows
Pot handles turned toward back of stove or toward center of Crib rails raised to full height and mattress kept in low
table position
Cool, not hot, mist vaporizer used Restraints used in high chairs and other infant/child furniture
Fire extinguisher(s) available and operational Pediatricians phone number posted near phone
Family escape plan (in case of fire) current
9-1-1 and address of home with nearest cross street
posted near phone
2164_Ch03_069-114 29/03/12 12:20 PM Page 95

Questions 95

Table 3.10
Safety and Injury PreventionHome Safetycontd
Aspiration and Suffocation Bodily Injury
Small objects stored out of reach and hanging objects Knives, unloaded firearms, power tools stored in locked
(e.g., mobile) placed out of reach cabinet
Toys inspected for broken or removable parts Pets properly restrained and immunized for rabies
Plastic bags and plastic-covered pillow/mattress not Outdoor play equipment kept in safe, working condition
accessible or in use Yard free of glass, nails, litter, etc.
Cribs with slats meet federal regulations (slats <23/8 inches Nearest emergency departments phone number posted near
[6 cm] apart with snug fitted mattress) phone
Bathroom and kitchen faucets firmly turned off
Toilet seats in down position or latched shut
Locked gates by fenced-in pools, spas, etc.
Proper safety equipment by pools, spas, etc.
Wading pools kept empty when not in use

QUESTIONS
Doors to dishwashers, refrigerators, washing machines, and
dryers kept closed
Food cut into sticks rather than coins
Adult family member trained in CPR and first aid for choking
Poisoning
Toxic substances placed on high shelf in locked cabinet
Extra quantities of toxic substances not stored in the house
Used containers of poisonous substances not accessible by
the child
Household cleaners/disinfectants kept in original containers,
separate from food and out of reach
Medications clearly labeled in childproof containers and
stored out of reach
Poison Control Center phone number and address of home
with nearest cross street posted near phone
Adapted from Hockenberry, M, & Wilson, D: Wongs Nursing Care of Infants and Children. ed 8. Mosby, St. Louis,
2007.

Questions 2. A client in liver failure who needs a blood transfusion.


3. A client who is postop and needs IV pain medication.
Select the one answer that is best for each question, unless 4. A client with GI bleeding who needs procedural
otherwise directed. sedation for a CT scan.
1. An RN would be judged negligent if which event 4. A client dies in the hospital. The drivers license indicates
occurred during client care? that the client wanted to be an organ donor. The family
1. Speaking harshly to a client. is refusing organ donation. What is the correct action by
2. Refusing a care assignment. an RN?
3. Making a medication error resulting in injury. 1. Discuss with the family the legality of the drivers
4. Delegating a dressing change to an LPN/LVN. license indicating organ donation.
2. Respect the familys wishes.
2. Which client is most appropriate for an RN to delegate
3. Seek legal advice to carry out clients wishes.
to a CNA?
4. Activate the donor team to go forward with the organ
1. A client who is 3 days postabdominal surgery with
donation process.
decreased breath sounds.
2. A client who is 8 hours postop thyroidectomy with 5. A client is admitted with an existing advance directive
decreased blood pressure. that states Do Not Resuscitate (DNR). The client goes
3. A client who is stable, 4 days postgunshot wound into cardiac arrest. How should a nurse respond?
with abdominal wound dehiscence. 1. Do not resuscitate the client as detailed in the advance
4. A client who is 2 days post-thoracotomy on an directive.
epidural fentanyl drip. 2. If the client is admitted to the hospital, a new advance
directive is needed; therefore, resuscitate the client.
3. Which client would be appropriate for the RN to
3. Start chest compressions, but do not intubate.
delegate to the LVN/LPN?
4. Intubate only to secure an airway.
1. A client who needs IV antibiotics and a central line
dressing change.
2164_Ch03_069-114 29/03/12 12:20 PM Page 96

96 chapter 3 Safe, Effective Care Environment

6. When assessing a client with peripheral neuropathy, 12. When an RN returns from dinner break, which client
which question is most important for a nurse should be assessed first?
to ask? 1. 82-year-old, 2 days posttotal hip replacement with a
1. Do you have to turn your head to obtain lateral platelet count of 98,000.
vision? 2. 28-year-old, IV drug abuser who is 3 days postaortic
2. Can you feel me touching your feet? valve replacement, with a central line and triple antibi-
3. At what time of day are the tremors at their otics due.
worst? 3. 76-year-old, 1 day postabdominal aortic aneurysm
4. Is the hearing loss worse on one side or the other? repair.
4. 46-year-old, stable, 4 hours after cardiac
7. What isolation precautions are necessary for a client with catheterization.
a history of hepatitis C who is anti-HCV positive?
1. Enteric precautions. 13. A client with MRSA is transferred to a nursing unit. A
2. Resistant-organism contact precautions. charge nurse should place the client in a room with:
3. Reverse isolation. 1. A client who is HIV +.
QUESTIONS

4. Standard precautions. 2. A client who is under treatment for Crohns disease.


3. Another client with MRSA.
8. A charge nurse on a medical-surgical floor observes a cer- 4. A client with a leg fracture.
tified nursing assistant (CNA) taking care of a client with
impaired vascular circulation. What behavior by the aide 14. Under which circumstance can an RN safely delegate
requires immediate action by the RN? tasks to unlicensed assistive personnel (UAP)?
1. Aide places clients feet in warm water. 1. The assignment is given in writing.
2. Aide elevates clients legs. 2. Supervision is given to complete the tasks that have
3. Aide places sheepskin between overlapped toes. been provided.
4. Aide applies lanolin to clients legs. 3. Task competency was verified during orientation.
4. The RN provides verbal instructions for the tasks.
9. During hospital orientation for new graduates, an
instructor covers health-careassociated infections 15. A client is prepared for a bronchoscopic examination.
(HAI), formerly referred to as nosocomial infections. The RN gives an IV sedative. Which activity could be
Which infections are known to be spread to the client delegated to an LPN/LVN?
by health-care providers during care? Select all that 1. Teaching the client about the procedure.
apply. 2. Giving the client smalls sips of water for dry mouth.
1. Clostridium difficile. 3. Walking with the client to the bathroom before the
2. Pseudomonas. procedure.
3. Methicillin-resistant Staphylococcus aureus (MRSA). 4. Checking the clients blood pressure and pulse.
4. Vancomycin-resistant enterococcus (VRE).
16. Four clients walked into an ambulatory clinic at the same
5. Human immunodeficiency virus (HIV).
time. Which client should a nurse assess first?
6. Helicobacter pylori.
1. A 45-year-old client with a swollen, bruised ankle,
10. A client in rehabilitation following a brain attack is who is limping.
receiving tube feedings at 75 mL/hr, Na+ is 148, K+ is 2. A 30-year-old client with a temperature of 102F,
4.2, creatinine is 1.2, glucose is 145. What should a vomiting, and diarrhea.
nurse ask a physician, as part of the SBAR (Situation, 3. A 40-year-old client with a sore throat, temperature
Background, Action, Recommendation) communication? 101F, swollen lymph glands.
1. Do you want to turn the feeding rate down? 4. A 35-year-old client complaining of a severe headache,
2. Do you want to start D5W IV? vomiting, and stiff neck.
3. Do you want to give intermittent tap water through
17. Which client should an RN direct an LPN/LVN to check
the feeding tube?
on first?
4. Do you want to give a more concentrated
1. A client with rheumatoid arthritis who needs splints
formula?
reapplied.
11. The most effective intervention to help a client quit 2. A client with diabetes with a blood sugar of 75, who
smoking, that a nurse could suggest, is: finished breakfast and needs a snack.
1. Acupuncture. 3. A client with GI bleeding who was up to use the
2. Tapering off the number of cigarettes smoked. bedside commode.
3. Counseling. 4. A client with COPD on O2 at 2 L who needs vital
4. Selective serotonin reuptake inhibitors. signs and pulse oximetry checked.
2164_Ch03_069-114 29/03/12 12:20 PM Page 97

Questions 97
18. When relocating an older client from a family home to 5. Give stool softeners to prevent constipation.
an assisted-living community, the correct action by a 6. Change IV solution every 24 hours.
nursing case manager is to: 24. After receiving an end-of-shift report on four clients, a
1. Maintain the clients level of economic independence. nurse plans client care. Which client should the nurse
2. Ensure the clients perceived control with the move to assess first?
an assisted-living setting. 1. Client admitted through the emergency department
3. Select a facility close to remaining family. 3 hours ago with acute abdominal pain.
4. Assess availability of culturally appropriate meals. 2. Client awaiting discharge and the spouse is at the
19. Following report on four clients, which client should a bedside.
nurse assess first? 3. Client in wrist restraints who has a sitter in the room.
1. A client with a chest tube inserted for a spontaneous 4. Client who had a chest tube inserted 24 hours ago for
pneumothorax, with respirations of 18. a pneumothorax.
2. A client with possible acute pyelonephritis. Urine out- 25. Which client should a nurse assess first?
put is 25 mL in the past 2 hours. 1. Client with pancreatitis complaining of jaw pain, 4 on

QUESTIONS
3. A client 2 days postradical mastectomy, who is crying a scale of 10.
and upset. 2. Client dependent on insulin who complains of feeling
4. A client on reverse isolation who needs pain medication. shaky and hungry.
20. Which finding should a nurse report immediately to a 3. Client who is confused, who has activated the bed
physician? alarm used to prevent falls.
1. After 2 days of IV penicillin, client has developed a 4. Client with red-colored urine drainage in Foley bag
rash around neck and face. following TURP surgery.
2. Client has only eaten 20% of the meals for 2 days. 26. Following a major earthquake, clients begin to arrive in
3. Client who has a pulse of 100 and irregular; who was an emergency department. Using the disaster triage sys-
previously 80 and regular. tem, which client should be treated first?
4. No bowel movement for 4 days, and a firm 1. Client with an open pneumothorax and severe dyspnea.
abdomen. 2. Client with major head trauma; pupils fixed and dilated.
21. An RN delegates the administration of a nasogastric tube 3. Client with a closed fracture of the femur.
feeding to a licensed practical/vocational nurse 4. Client with multiple abrasions and contusions.
(LPN/LVN). Which statement about responsibility in 27. Which activities could an RN delegate to a certified nurs-
this situation is most accurate? ing assistant (CNA)? Select all that apply.
1. The RN is responsible for delegated care. 1. Taking routine vital signs.
2. This task should not be delegated. 2. NG tube irrigation for a client who is postoperative.
3. The LPN/LVN is responsible for own actions. 3. Feeding a client for the first time who had a stroke.
4. The LPN/LVN should respectfully refuse to initiate 4. Colostomy irrigation for a client who is comatose.
this care. 5. Walking with a client who has aphasia.
22. Following antibiotic therapy, a client develops bloody 6. Shaving a client with a tracheostomy.
diarrhea. A nurse suspects Clostridium difficile, which 28. Medicare has enacted a new policy that states that, if a
often occurs when antibiotics disrupt the normal intestin- client develops certain conditions while in the hospital,
al flora and injure the lumen of the bowel. Control of the hospital will not be reimbursed for the care of the
this infection will include: client. Which condition does a nurse know that quality
Select all that apply. care can prevent?
1. Nurse wearing mask at all times. 1. Yeast infections.
2. Hand hygiene and wearing gloves. 2. Poor blood sugar control.
3. Keeping clients room door closed. 3. Unexpected weight loss.
4. Cleaning over-the-bed table and side rails daily. 4. Unexpected surgery.
5. Drug treatment with metronidazole (Flagyl).
29. To ensure that the hospital is delivering quality care, a nurse
23. A hospital client with cancer has a low white blood cell participates in collecting data for core measures, which are:
(WBC) count (<1,000/mm3). Preventing infection is a 1. Measurements that are taken on a client when first
high priority. Measures to minimize infection include: admitted to the hospital.
Select all that apply. 2. Standardized, evidence-based performance measures
1. A private room if possible. needed for hospital accreditation.
2. Measuring temperature rectally for greater accuracy. 3. Measurements taken for the infection control nurse.
3. Request that family bring fresh fruits from home. 4. Standardized forms that are used by clients to evaluate
4. Shave with an electric razor if needed. hospitals.
2164_Ch03_069-114 29/03/12 12:20 PM Page 98

98 chapter 3 Safe, Effective Care Environment

30. One of the hospital-acquired conditions that Medicare 36. A nurse prepares to admit a child diagnosed with
will not pay for is falls. The nursing staff is responsible respiratory syncytial virus (RSV). Which infection
for fall prevention. The best predictor of the likelihood of control measure would be most appropriate for this
a client falling while hospitalized is: child?
1. The need to urinate during the night. 1. Place child in a negative pressure room.
2. A history of falling before admission. 2. Place child on contact precautions.
3. Presence of morning stiffness and muscle rigidity. 3. Place child on airborne precautions.
4. Change in medications since admitted. 4. Place child in a positive pressure room.
31. A client is very agitated and needs to have soft restraints 37. A nurse is caring for an infant with acute gastroenteritis
applied. To ensure the safety of the client, the nurse with an estimated 10% weight loss. Which actions
should: should the nurse take when caring for this infant?
1. Tie restraints to the side rails. Select all that apply.
2. Keep the head of the bed elevated 30 degrees to keep 1. Administer IV fluid bolus of 5 mL/kg.
tension on the restraints. 2. Apply antibacterial hand foam.
QUESTIONS

3. Inspect the clients skin under the restraints every 3. Put on gloves and gown.
2 hours. 4. Administer IV antibiotic.
4. Turn on the lights in the room to see the client. 5. Monitor hourly urine output.
6. Initiate IV therapy.
32. A nurse returns to work after vacation and the clients
are all new to the RN. National Patient Safety Goals 38. A school nurse receives a call from the parent of a child
require that at least two pieces of identification must be diagnosed with chickenpox. The parent asks when the
used to establish client identity. The best approach to child may return to school. What is the best information
identifying the clients before giving medications would that the nurse should provide to this parent?
be to: 1. Children with chickenpox may return to school once
1. Have the client spell name and give date of birth. all of the lesions have scabbed over.
2. Check the room number and bracelet. 2. The child may attend classes after being on antibiotics
3. Ask the family and client to verify the name. for 24 hours.
4. Check the clients Medication Administration Record 3. It is recommended that the child remain home until
(MAR) and chart. the fever has subsided.
4. Once the child has a positive varicella titer, school
33. To prevent a catheter-associated bloodstream infection, attendance may resume.
when would hand washing be required? Select all that
apply. 39. A nurse receives morning report on todays client
1. Before and after palpating around the site. assignments. In which order should the nurse assess
2. Before and after a dressing change. these children? Place each client in order of priority
3. When touching the administration set. from first to last.
4. Only when not wearing gloves. 1. A 4-year-old with encopresis and a parent at
5. Before and after insertion of the catheter. bedside.
2. An 8-year-old in traction for fractured
34. An RN observes the hand hygiene routine of a nursing femur, with no parent at bedside.
assistant. Additional teaching will be needed if the RN
3. A 5-year-old who has just returned from
observes that:
bronchoscopy, with parents at bedside.
1. Hands were rubbed together for 10 seconds.
4. A 12-year-old scheduled today for
2. Soap lather dripped off hands.
appendectomy, with no parent at bedside.
3. Water was only lukewarm.
4. Hands were not dried thoroughly. 40. A 4-year-old child is being discharged from a
hospital following treatment for Kawasaki disease.
35. A nurse in an emergency department performs triage of Before the child is discharged, the parents should be
clients in the waiting room. Which child should the
taught to:
nurse send for immediate examination?
1. Auscultate the heart.
1. A 1-month-old infant crying vigorously with a history
2. Administer antibiotics prior to dental work.
of frequent sneezing.
3. Return to the clinic for repeat doses of
2. A 2-year-old child with fever and urinary frequency.
immunoglobulin.
3. A 6-month-old infant with a sunken fontanel and
4. Discuss the postponement of immunizations with the
history of poor feeding.
primary care practitioner.
4. A 9-year-old with complaints of pain and a possible
arm fracture.
2164_Ch03_069-114 29/03/12 12:20 PM Page 99

Questions 99
41. A nurse is counseling the parent of a young child with 47. The parents of a child with sickle cell anemia ask for
HIV. What information is essential for the nurse to information about heredity. What information should a
convey during the first meeting? nurse provide? Select all that apply.
1. Children with HIV have shorter life expectancies than 1. If both parents have sickle cell trait, there is a
adults with HIV. 25% chance their offspring will have sickle cell
2. Growth may be inhibited in the child with HIV. anemia.
3. Adherence to antiretroviral therapy is critical for the 2. If one parent has sickle cell anemia and the other does
childs health. not, their offspring will have a 50% chance of having
4. Immunizations should be delayed until the child sickle cell trait.
enters preschool. 3. If both parents carry sickle cell trait, there is a 50%
chance their offspring will have sickle cell trait.
42. A nurse is assigned to care for multiple clients who had 4. If one parent has sickle cell anemia and the other
surgery 2 days ago. Following the 7 a.m. shift report,
carries trait, their offspring will have a 50% chance
which client should the nurse assess first?
of having sickle cell anemia.
1. A 59-year-old, after a knee arthroplasty, with a WBC
5. If one parent has sickle cell anemia and the other does

QUESTIONS
of 12,000.
not, their offspring will have a 25% chance of having
2. An 88-year-old, with CHF and posthip pinning, who
sickle cell anemia.
was medicated for pain at 0600.
3. A 24-year-old, who had a splenectomy at age 18, with 48. A client, who is a 19-year-old gravida 2, para 1,
a temperature of 38.8C. delivers a 10-pound neonate by cesarean section after
4. A 67-year-old, with an abscess postfemur fracture, 36 hours of labor and failure to progress. The client
with a platelet count of 100,000. has developed abdominal tenderness, a fever of 102.2F
(39C), and foul-smelling lochia. A nurse should
43. A nurse is caring for a postoperative client. Which recognize that the most likely reason for the clients
nursing action will prevent infection?
condition is:
1. Swab IV access port with Betadine for 60 seconds
1. Endometriosis.
before injecting through it.
2. Postoperative wound infection.
2. Maintain perineal skin integrity by inserting an
3. Pelvic thrombophlebitis.
indwelling bladder catheter.
4. Endometritis.
3. Keep skinfolds moist to prevent breakdown.
4. Remove dressing to assess wound only during needed 49. A nurse tells a client that a positive reaction to a tubercu-
dressing changes. losis (TB) test will be determined by:
1. The appearance of redness after 24 hours.
44. Which nursing action is appropriate in the care of a 2. The size of the induration 48 hours after the test.
client admitted with pneumonia who had a right mastec-
3. The diameter of the reddened area at the site.
tomy 3 years ago?
4. The length of time it takes before redness or indura-
1. Check right radial pulse every 4 hours.
tion appears.
2. Start an IV on the left forearm.
3. Keep the right arm below heart level. 50. In making room in a hospital for emergency admission
4. Avoid range of motion on right arm. of clients who were in a disaster, which client can be
discharged?
45. A drop of a clients blood splashes on a nurses skin in an 1. Client with a fractured femur complaining of calf
area where the nurse had a small, dried scab. The appro-
pain.
priate nursing action is to:
2. Client with bronchial asthma with 95% oxygen
1. Cover the scab with a dry gauze.
saturation on room air.
2. Apply ointment to keep scab moist.
3. An elderly client with temperature of 101F (38.3C).
3. Wash the site without disrupting the scab.
4. Client experiencing rebound abdominal pain with
4. Scrub scab with Betadine for 5 minutes.
palpation.
46. A nurse has received an end-of-shift report on four 51. In prioritizing care, to which client should a nurse
clients. Which client should the nurse see first?
respond first?
1. A client who is a stockbroker, admitted with GERD,
1. Client on NG bolus feeding, with NG medication due
with BP of 160/90.
and tube is pulled out.
2. A client who is an athlete, admitted with a torn ACL,
2. Client with IV chemotherapy and the infusion pump
whose pulse is 57.
is alarming.
3. A client who is 78 years old with a temperature of
3. Client who is incontinent and lying in feces.
101F (38.3C).
4. Client is yelling, I want to go home and pressing the
4. A client who is postlaparoscopic cholecystectomy,
call button.
with shoulder pain.
2164_Ch03_069-114 29/03/12 12:20 PM Page 100

100 chapter 3 Safe, Effective Care Environment

52. An HIV-positive client with type 1 diabetes gives birth at during pregnancy. Which client statement indicates that
28 weeks gestation because of severe preeclampsia. She health teaching has not been effective?
is asking a nurse about breastfeeding her infant in the 1. I can continue to swim as long as my membranes
neonatal intensive care unit. The nurse should explain to arent ruptured.
the client that that breastfeeding is contraindicated in the 2. I can relax in a hot tub for 30 minutes after swimming.
client with: 3. I can take a warm bath everyday.
1. Diabetes. 4. I should avoid sitting in the sauna for prolonged
2. Positive HIV. periods.
3. Hypertension. 59. A client, who is a 28-year-old gravida 4, para 3, presents
4. Thyroid disease. to labor and delivery with the fetal head crowning. A
53. In what position should a client in a wheelchair be placed physician is called for an emergent delivery, but has not
to facilitate a safe transfer to bed, if the client has had a yet arrived. After the neonates head is delivered, which
right-sided brain attack (stroke)? nursing intervention would be the most appropriate?
1. Weakened (R) side of the client next to the bed. 1. Check for a nuchal cord.
QUESTIONS

2. Weakened (R) side of the client away from the bed. 2. Place antibiotic ointment in the infants eyes.
3. Weakened (L) side of the client next to the bed. 3. Use a DeLee suction to remove secretions.
4. Weakened (L) side of the client away from the bed. 4. Assess the neonate for heart rate and respirations.
54. Where should a nurse place the call light for a client 60. A family lives in an apartment building built in the
with a right-sided brain attack and left homonymous 1950s, which has lead pipes. The family cannot afford to
hemianopsia? move. To prevent exposure to lead through the water, a
1. On the clients right side. nurse should instruct the family to:
2. On the clients left side. 1. Boil all water and use it immediately.
3. Directly in front of the client. 2. Use hot water from the tap for cooking purposes.
4. Where the client prefers. 3. Use cold water from the tap for cooking purposes.
4. Buy bottled water for cooking purposes.
55. A client has a colostomy following surgery for colon
cancer. Which assessment finding would require 61. What would the appropriate infection control technique
immediate action by a nurse? be for changing a leg dressing on a client who has
1. Stoma is raised 2 cm from the abdominal wall. Pneumocystis jiroveci (formerly carinii) pneumonia (PCP)?
2. Bleeding was noted from the stoma during care. 1. Double-bag all contaminated dressings.
3. Complaint of pain with light touch. 2. Wear a mask during the dressing change.
4. Stoma was pinkish-red in color. 3. Wash hands after removing gloves.
4. Place the client in protective isolation.
56. A nurse prepares a hospital room for the pending admis-
sion of a 1-week-old infant with hyperbilirubinemia. 62. The first nursing concern for a client following
Which piece of equipment would be best suited for this bronchoscopy should be to:
infant? 1. Ambulate as soon as sedation wears off.
1. Isolette. 2. Check for the return of the gag reflex.
2. Open crib. 3. Teach how to use the incentive spirometer.
3. Hospital bed with side rails up. 4. Position the client on the unaffected side.
4. Bassinette. 63. While a client is getting out of bed, the chest tube catch-
57. Following report on four clients, the priority intervention es on the night stand and is pulled out. What should a
for a nurse should be to: nurse do first?
1. Request an antiemetic for a client who has been 1. Cover the opening with sterile gauze.
vomiting during the night. 2. Hold clients gown over opening.
2. Report a K+ level of 5.9 in a client receiving 3. Put the client back to bed.
Kayexalate. 4. Call the MD for help.
3. Complete the assessment of the four clients. 64. Which person should be restricted from visiting a client
4. Notify the family that the client with chest pain was with TB until at least 2 weeks of drug therapy has been
now in ICU. completed?
58. A client, who is gravida 1, para 0, is attending a prenatal 1. The clients 25-year-old son.
visit. She asks about water aerobic exercise class. A nurse 2. The clients 3-year-old granddaughter.
gives the client instructions about swimming and bathing 3. The clients 74-year-old mother.
4. The clients middle-aged neighbor.
2164_Ch03_069-114 29/03/12 12:20 PM Page 101

Answers/Rationales/Tips 101
65. Client teaching on how long the medication for TB 3. Ask the LVN/LPN to describe what is meant by short-
should be taken would include: ness of breath.
1. Until the client feels better. 4. Check the clients chart for a current x-ray report.
2. Daily for at least 9 months. 72. A night RN reports that a client, admitted with a
3. Until the sputum culture is negative. diagnosis of gastric ulcer, is reporting syncope and
4. For the rest of the clients life. dizziness. What should be the initial nursing interven-
66. When a clients family members come to visit, they tion by the RN?
would be adhering to respiratory isolation precautions 1. Keep the client on bedrest.
when they: 2. Call for a STAT Hgb and Hct.
1. Put on gowns, gloves, and masks. 3. Give a STAT dose of sucralfate (Carafate).
2. Keep the door to the clients room open. 4. Check the clients vital signs.
3. Wash their hands when leaving. 73. A client, admitted to the emergency department with a
4. Avoid contact with the clients roommate. myocardial infarction (MI), is experiencing chest pain
67. Which task should a nurse delegate to a certified nursing and becomes profusely diaphoretic and apprehensive.

ANSWERS
assistant (CNA)? What should be the initial intervention by the RN?
1. Irrigation of a nasogastric tube. 1. Increase the nasal oxygen from 2 L/min to 6 L/min.
2. Walking with a client who is postoperative. 2. Check the troponin and myoglobin levels on the chart.
3. Administering a pain medication. 3. Give the prn dose of morphine sulfate 3 mg IV push.
4. Feeding a client for the first time following a brain 4. Get a 12-lead ECG.
attack. 74. An RN observes another RN speaking harshly and repri-
68. A nurse notices that a clients signed consent form states manding a client for being incontinent. What action
amputation of the right leg, although it is the left leg should be taken?
that is to be amputated. The client has already received 1. Intervene and tell the RN that a report on client abuse
preoperative medications. What should the nurse do? will be made.
1. Call the nearest relative to come in to sign a new form. 2. Document the incident on the clients chart.
2. Have the client sign another form. 3. Advise the supervisor by writing an objective account
3. Cross out the error and initial the form. of the incident.
4. Call the MD to reschedule the surgery. 4. Discuss the incident with the RN, and the nurses can
then change the assignment between themselves.
69. What should be the first nursing actions for a client admit-
ted to an emergency department following an accident? 75. Which nursing intervention should be implemented
1. Check respirations, circulation, neurological response. before the deflation of a tracheostomy cuff?
2. Align the spine, check pupils, check for hemorrhage. 1. Have the obturator available.
3. Check respirations, stabilize spine, check circulation. 2. Take a pulse oximetry reading.
4. Assess level of consciousness, circulation. 3. Suction the trachea and mouth.
4. Encourage deep breathing and coughing.
70. A staff RN and a nurses aide are caring for six clients.
The RN delegates the following assignments to the
nurses aide: vital signs, baths, and intake and output Answers/Rationales/Tips
for all clients. What is the RNs accountability for the
1. CORRECT ANSWER: 3. Answer 1 is incorrect because
delegated work?
speaking harshly or yelling at a client is unprofessional conduct.
1. Assume that the nurses aide has completed the
Answer 2 is incorrect because an RN can refuse an assignment
assignment.
for personal ethical reasons or if unqualified to give care.
2. Communicate with the nurses aide every hour as to
Answer 3 is correct because an action that results in an
the status of the assignment.
injury can be judged as negligent. Answer 4 is incorrect
3. Ask the clients if they have received appropriate care
because a dressing change is within the scope of practice for
from the nurses aide.
the LPN/LVN.
4. Define client parameters to the nurses aide that must
TEST-TAKING TIP: Review the definitions of negligence as
be reported to the RN.
compared to unethical or unprofessional behavior.
71. On a skilled nursing unit, an LVN/LPN reports to a staff Content Area: Management of Care, Legal; Integrated Process:
nurse that a client is short of breath. What is the most Communication and Documentation; Cognitive Level:
important initial response by the RN? Comprehension; Client Need/Subneed: Safe and Effective Care
1. Ask the LVN/LPN if this is a new symptom. Environment/Management of Care/Legal Rights and
2. Do an independent nursing assessment of the client. Responsibilities
2164_Ch03_069-114 29/03/12 12:20 PM Page 102

102 chapter 3 Safe, Effective Care Environment

2. CORRECT ANSWER: 1. Answer 1 is correct because Answer 4 is incorrect because no action is to be taken if the
the CNA could encourage use of the incentive spirometer client has a DNR.
and ambulate the client as ordered in a client who is TEST-TAKING TIP: The stem and correct option includes the
postoperative GI surgery and needs routine help with same phrasedo not resuscitate. Know the legality of the
postoperative self-care (breathing exercises). Answer 2 is advance directive.
incorrect because this client is unstable. Hemorrhage is a Content Area: Management of Care, Legal; Integrated Process:
risk. Answer 3 is incorrect because dehiscence may be fol- Nursing Process, Implementation; Cognitive Level: Application;
lowed by evisceration. Close assessment is needed. Answer 4 is Client Need/Subneed: Safe and Effective Care Environment/
incorrect because the RN must assess and monitor epidural Management of Care/Advance Directives
analgesia. 6. CORRECT ANSWER: 2. Answer 1 is incorrect because
TEST-TAKING TIP: Choose the client who is most stablea peripheral neuropathy is a condition caused by nerve damage,
client who needs assistance, but not assessment and monitoring. generally in the feet. It does not affect vision. Answer 2 is cor-
Content Area: Management of Care, Delegation; Integrated
rect because peripheral neuropathy is a condition caused by
Process: Nursing Process, Analysis; Cognitive Level: Application;
nerve damage in the feet, sometimes also in the legs and
Client Need/Subneed: Safe and Effective Care Environment/
upper extremities. Clients have either decreased sensation
ANSWERS

Management of Care/Delegation or neuropathic pain from the condition. Clients with this
3. CORRECT ANSWER: 2. Answer 1 is incorrect because cen- condition need extra assessments and care to protect the
tral lines are not routinely part of the LVN/LPN scope of prac- affected extremities from injury. They may require seizure
tice. Answer 2 is correct because the clients condition is not or psychiatric drugs to assist with the pain. If a client has
going to change suddenly. Answer 3 is incorrect because pain diabetes, avoidance of high blood sugar levels is also impor-
management and assessment of the drug response is done by tant. Answer 3 is incorrect because peripheral neuropathy is a
the RN. Answer 4 is incorrect because this client is unstable condition caused by nerve damage, generally in the feet. It
and will need close assessment of sedation for the computed does not cause tremors. Answer 4 is incorrect because peripheral
tomography (CT) scan. neuropathy is a condition caused by nerve damage, generally in
TEST-TAKING TIP: Look for the option that is different from the feet. It does not cause hearing loss.
the others. The correct option is the client who is the most stable, TEST-TAKING TIP: Look for a clue in the stem, neuro.
and within the scope of practice for the LVN/LPN. Think of nerves, sensations, and feelings.
Content Area: Management of Care, Delegation; Integrated Content Area: Adult Health, Neurological; Integrated Process:
Process: Nursing Process, Analysis; Cognitive Level: Application; Nursing Process, Assessment; Cognitive Level: Application;
Client Need/Subneed: Safe and Effective Care Environment/ Client Need/Subneed: Safe and Effective Care Environment/
Management of Care/Delegation Safety and Infection Control/Accident Prevention and Injury
Prevention
4. CORRECT ANSWER: 2. Answer 1 is incorrect because the
drivers license dot is not a legally binding document of 7. CORRECT ANSWER: 4. Answer 1 is incorrect because the
client wishes. Answer 2 is correct because the familys presence of antibodies against the hepatitis C virus (HCV)
wishes must be followed. If the client had an advance means that the client has had hepatitis C in the past but is
directive, this may not have been an issue. Answer 3 is not necessarily still infected. This is a blood-borne infection;
incorrect because, without a legally executed advance direc- therefore, enteric precautions are not required. Answer 2 is
tive on file, there is no use in pursuing the issue. Answer 4 is incorrect because resistant-organisms contact precautions are
incorrect because no donor procurement agency will go used for bacterial infections that are not easily treated with
against the family. standard antibiotics. Hepatitis C is a viral infection. Answer 3
TEST-TAKING TIP: Look for the option that is different from is incorrect because reverse isolation is an outdated term
the others. Three of the choices continue the question of dona- describing practices that had been designed to protect clients
tion. Only one acknowledges the wishes of the family in the who are vulnerable to infection. Answer 4 is correct because
absence of a health care agent for this client. the presence of antibodies against the hepatitis C virus
Content Area: Management of Care, Legal; Integrated Process: (HCV) means that the client has had hepatitis C in the
Nursing Process, Implementation; Cognitive Level: Application; past, but is not necessarily still infected. Management
Client Need/Subneed: Safe and Effective Care Environment/ of this potential blood-borne infection involves standard
Management of Care/Legal Rights and Responsibilities precautions.
TEST-TAKING TIP: Know the indications for the different
5. CORRECT ANSWER: 1. Answer 1 is correct because the approaches to infection control. Without evidence of active
advance directive is an expression of the clients wishes. disease, standard precautions are used with all clients.
Answer 2 is incorrect because, as long as the advance directive
Content Area: Adult Health, Infectious Disease; Integrated
is current, it covers this admission. Answer 3 is incorrect Process: Nursing Process, Planning; Cognitive Level:
because the clients wishes are cleardo not resuscitate.
2164_Ch03_069-114 29/03/12 12:20 PM Page 103

Answers/Rationales/Tips 103
Application; Client Need/Subneed: Safe and Effective Care Environment/Management of Care/Collaboration with
Environment/Safety and Infection Control/Standard/ Interdisciplinary Team
Transmission-based/Other Precautions
11. CORRECT ANSWER: 3. Answer 1 is incorrect because
8. CORRECT ANSWER: 1. Answer 1 is correct because heat is there are no data supporting the efficacy of acupuncture
contraindicated since the client has reduced sensation from as a quitting strategy. However, it may create another
impaired vascular circulation. Injury may result. Answer 2 is opportunity to quit, so it should not be discouraged if the
incorrect because elevation will improve circulation and venous client is interested. Answer 2 is incorrect because tapering
return. Answer 3 is incorrect because this action will prevent has been associated with nicotine withdrawal symptoms and
breakdown. Answer 4 is incorrect because this action is appro- subsequent failure to quit. Setting a quit day is currently
priate dry skin care in impaired vascular circulation. the preferred strategy. Answer 3 is correct because the
TEST-TAKING TIP: Eliminate the options that are familiar 1-year quit rate for a person completing a group counsel-
and effective strategies to improve venous return (Answer 2) ing program is 20%. Individual counseling has a small
and prevent skin breakdown (Answers 3 and 4). but significant impact on quitting. Answer 4 is incorrect
Content Area: Adult Health, Vascular; Integrated Process: because there is an antidepressant approved for use with
Nursing Process, Analysis; Cognitive Level: Application; smoking cessation (Wellbutrin), but it is not a selective

ANSWERS
Client Need/Subneed: Safe and Effective Care Environment/ serotonin reuptake inhibitor.
Management of Care/Supervision TEST-TAKING TIP: Quitting smoking, like losing weight, is
best accomplished in a supportive environment.
9. CORRECT ANSWERS: 1, 3, 4. Answer 1 is correct because Content Area: Adult Health, Substance Abuse; Integrated
poor hand hygiene spreads this infection. Answer 2 is
Process: Caring; Cognitive Level: Application; Client Need/
incorrect because this organism occurs naturally in the client.
Subneed: Safe and Effective Care Environment/Management
Answer 3 is correct because poor hand hygiene spreads this
of Care/Referrals
infection. Answer 4 is correct because poor hand hygiene
spreads this infection. Answer 5 is incorrect because spread 12. CORRECT ANSWER: 4. Answer 1 is incorrect because
of this organism is through direct contact with a client who is there is not an immediate risk to the client from the low
infected. Answer 6 is incorrect because the organism is found platelets. Answer 2 is incorrect because this client is likely
in the stomach and associated with ulcer disease. stable. Drug withdrawal would have been a concern in the
TEST-TAKING TIP: Consider the mode of transmission. Look early postoperative period. Answer 3 is incorrect because this
for organisms that are controlled by the hand hygiene of the client is not in the immediate postoperative period (first
health-care provider. 24 hours). This client should be seen second. Answer 4 is
Content Area: Adult Health, Infectious Diseases; Integrated correct because there is a risk of bleeding at the insertion
Process: Nursing Process, Analysis; Cognitive Level: site and other postcatheterization complications. Stability
Comprehension; Client Need/Subneed: Safe and Effective of vital signs and ECG need to be assessed.
Care Environment/Safety and Infection Control/Standard/ TEST-TAKING TIP: Select the earlier time frame herehours,
Transmission-based/Other Precautions not days. Also, look for the client who is most unstable, as
indicated by the greatest likelihood of a change in condition.
10. CORRECT ANSWER: 3. Answer 1 is incorrect because Content Area: Adult Health, Triage; Integrated Process:
there is no reason to slow the feedings down. Answer 2 is
Nursing Process, Analysis; Cognitive Level: Analysis; Client
incorrect because, although D5W IV would help keep
Need/Subneed: Safe and Effective Care Environment/
the sodium down, it is not necessary to use the IV route
Management of Care/Establishing Priorities
when the gut is working. Answer 3 is correct because
full-strength tube feedings can cause hypernatremia. This 13. CORRECT ANSWER: 3. Answer 1 is incorrect because this
is generally managed with enteral electrolyte-free water. client is at risk for opportunistic infections. Answer 2 is incor-
Tap water is appropriate directly into the GI system; it rect because this client is at risk for a compromised immune
does not need to be sterile. Answer 4 is incorrect because a system from treatment and a potential infection. Answer 3 is
more concentrated formula would further increase the serum correct because all other choices are vulnerable to infection.
sodium. Answer 4 is incorrect because this client is at risk of developing
TEST-TAKING TIP: Interpret the laboratory data. Consider an infection in the bone.
how a concentrated tube feeding will affect fluid and elec- TEST-TAKING TIP: Choose the option that is different from
trolyte levels. Use SBAR to meet client needs. Eliminate the the others. Three of the clients are vulnerable to infection.
option pertaining to rate (Answer 1). Focus on route: IV Content Area: Adult Health, Infectious Disease; Integrated
(Answer 2) vs. feeding tube (Answer 3). Process: Nursing Process, Implementation; Cognitive Level:
Content Area: Adult Health, Gastrointestinal; Integrated Analysis; Client Need/Subneed: Safe and Effective Care
Process: Nursing Process, Implementation; Cognitive Level: Environment/Safety and Infection Control/Standard/
Analysis; Client Need/Subneed: Safe and Effective Care Transmission-based/Other Precautions
2164_Ch03_069-114 29/03/12 12:20 PM Page 104

104 chapter 3 Safe, Effective Care Environment

14. CORRECT ANSWER: 3. Answer 1 is incorrect because dangerously low. Answer 3 is correct because it is impor-
competency needs to be demonstrated before delegating. tant to determine if the client is actively bleeding. Since
Answer 2 is incorrect because the UAP needs to demonstrate this client is the most unstable, the RN needs to ask the
competency before performing a task. Answer 3 is correct LPN/LVN to check on this client first. Answer 4 is incor-
because, when delegating, the RN needs to confirm that rect because the client has a chronic condition and any
the UAP has met the task competencies. If delegating to a changes would develop gradually.
licensed staff (LPN/LVN), the assignment must be within TEST-TAKING TIP: Consider which client is most unstable
the scope of practice. Answer 4 is incorrect because the and the condition that could change suddenly.
UAP needs to demonstrate competency before the task is Content Area: Management of Care, Delegation; Integrated
delegated. Process: Nursing Process, Analysis; Cognitive Level: Analysis;
TEST-TAKING TIP: Choose the one option that has verify in Client Need/Subneed: Safe and Effective Care Environment/
the option, to ensure that the tasks will be performed safely. Management of Care/Establishing Priorities
Content Area: Management of Care, Delegation; Integrated
18. CORRECT ANSWER: 2. Answer 1 is incorrect because
Process: Nursing Process, Implementation; Cognitive Level:
this is only part of the desired control the client will
Comprehension; Client Need/Subneed: Safe and Effective Care hopefully retain. Answer 2 is correct because preserving
ANSWERS

Environment/Management of Care/Delegation the clients control over the decision is important to a


15. CORRECT ANSWER: 4. Answer 1 is incorrect because successful adjustment. Answer 3 is incorrect because this
teaching is done before sedation by the RN. Answer 2 is incor- is not an essential criterion for placement; however, it would
rect because the client could be NPO. Answer 3 is incorrect be beneficial to the family and client. Answer 4 is incorrect
because the client has received sedation and would not be because meals may be important, but are not most important
ambulatory. Answer 4 is correct because the client has been to adjustment.
given IV sedation. Vital signs (VS) need to be checked after TEST-TAKING TIP: Choose the option that encompasses the
receiving sedation. The LVN/LPN can take the VS, but the other options. If the client is involved with food choices,
RN must assess the findings. finances, and location, the client retains perceived control.
TEST-TAKING TIP: There is only one safe choice. Three of the Content Area: Management of Care, Case Management;
options would not be done after giving sedation. Integrated Process: Nursing Process, Implementation;
Content Area: Adult Health, Respiratory; Integrated Process: Cognitive Level: Application; Client Need/Subneed: Safe and
Nursing Process, Implementation; Cognitive Level: Application; Effective Care Environment/Management of Care/Case
Client Need/Subneed: Safe and Effective Care Environment/ Management
Management of Care/Delegation 19. CORRECT ANSWER: 2. Answer 1 is incorrect
16. CORRECT ANSWER: 4. Answer 1 is incorrect because because there is no indication that the client is unstable
the signs are consistent with a soft tissue injury, which is not or experiencing respiratory difficulty. Answer 2 is correct
immediately life threatening. Answer 2 is incorrect because because pyelonephritis is an infectious process that
the signs and symptoms are not imminently life threatening. can lead to acute renal failure. If kidney function is
The risk of dehydration would be a greater concern in a very inadequate, the client may develop a life-threatening
young child or an older adult. Answer 3 is incorrect because condition, such as hyperkalemia. Answer 3 is incorrect
the signs and symptoms are not imminently life threatening. because this response by the client would be a normal
A throat culture would be indicated to determine if the reaction to the change in body image. Answer 4 is
cause is viral or bacterial. Answer 4 is correct because the incorrect because there is no indication that the client is
classic signs and symptoms of bacterial meningitis in an unstable. Pain management is important. However, the
adult are severe headache and neck stiffness (nuchal client with the inadequate urine output needs further
rigidity). This is a serious condition because of the assessment first.
proximity to the brain and spinal cord and the TEST-TAKING TIP: Look for the client who is most
possibility of sepsis. unstable.
TEST-TAKING TIP: Rule out the conditions that are not life Content Area: Adult Health, Triage; Integrated Process:
threatening in a young adult or middle-aged adult. Nursing Process, Analysis; Cognitive Level: Analysis;
Content Area: Adult Health, Triage; Integrated Process: Nursing Client Need/Subneed: Safe and Effective Care Environment/
Process, Analysis; Cognitive Level: Analysis; Client Need/Subneed: Management of Care/Establishing Priorities
Safe and Effective Care Environment/Management of Care/ 20. CORRECT ANSWER: 3. Answer 1 is incorrect because,
Establishing Priorities if a rash develops after several days of therapy, it is not an
17. CORRECT ANSWER: 3. Answer 1 is incorrect because emergency situation. The rash may or may not be related
the client has a chronic condition that is not life threatening. to the drug. Answer 2 is incorrect because there are inde-
Answer 2 is incorrect because the blood sugar is not pendent nursing actions that need to be considered before
2164_Ch03_069-114 29/03/12 12:20 PM Page 105

Answers/Rationales/Tips 105
contacting the physician. Answer 3 is correct because a with other clients. Answer 2 is incorrect because rectal
change in cardiac status is a priority and potentially temperatures are to be avoided because rectal trauma may
life threatening, which requires the nurse to collaborate occur. Answer 3 is incorrect because fresh fruits do not mini-
with the physician on the appropriate action. Answer 4 is mize susceptibility to infection, and unless peeled before
incorrect because there are additional actions by the nurse given to the client, may be a source of bacteria. Answer 4 is
that can be done before contacting the physicianchecking correct because an electric razor avoids the risk of cuts.
for bowel sounds, a review of the clients dietary intake, Answer 5 is correct because constipation and straining at
activity status, and checking for impaction. stool may cause rectal tears. Answer 6 is correct because
TEST-TAKING TIP: Consider which situation requires the the chance of infection increases after 24 hours for
nurse to collaborate with the physician to meet the clients IV solutions, and 72 hours with IV tubing.
needs. TEST-TAKING TIP: Choose options that reduce exposure to or
Content Area: Adult Health, Triage; Integrated Process: risk from organisms.
Communication and Documentation; Cognitive Level: Analysis; Content Area: Adult Health, Oncological; Integrated Process:
Client Need/Subneed: Safe and Effective Care Environment/ Nursing Process, Analysis; Cognitive Level: Application; Client
Management of Care/Collaboration with Interdisciplinary Need/Subneed: Safe and Effective Care Environment/Safety

ANSWERS
Team and Infection Control/Standard/Transmission-based/Other
Precautions
21. CORRECT ANSWER: 1. Answer 1 is correct because
the RN can never delegate the ultimate responsibility 24. CORRECT ANSWER: 1. Answer 1 is correct because a
for the client. Before delegating, the RN needs to deter- newly admitted client with abdominal pain is not stable
mine that the LPN/LVN is competent. Answer 2 is and is first priority. The cause of the pain is unknown and
incorrect because tube feedings are within the scope of vital signs should be assessed. Answer 2 is incorrect because
LPN/LVN practice, and can be delegated. Answer 3 is incor- the client is going home (fourth priority). Answer 3 is incorrect
rect because, even though the LPN/LVN has accountability, because the client is being observed by someone and would be
the RN can never delegate responsibility. Answer 4 is incor- third priority. Answer 4 is incorrect because it is important to
rect because this is a task appropriate for the LPN/LVN, assess the respiratory status and chest tube functioning soon
assuming that the LPN/LVN has shown competency in the (second priority).
procedure. TEST-TAKING TIP: Start with the client who is most unstable
TEST-TAKING TIP: Remember that only the authority to do or critical.
something is delegatednot the responsibility. Content Area: Adult Health, Triage; Integrated Process:
Content Area: Management of Care, Legal; Integrated Process: Nursing Process, Analysis; Cognitive Level: Analysis;
Nursing Process, Analysis; Cognitive Level: Comprehension; Client Need/Subneed: Safe and Effective Care Environment/
Client Need/Subneed: Safe and Effective Care Environment/ Management of Care/Establishing Priorities
Management of Care/Delegation
25. CORRECT ANSWER: 1. Answer 1 is correct because
22. CORRECT ANSWERS: 2, 4, 5. Answer 1 is incorrect jaw pain is not a typical pain pattern for pancreatitis;
because the bacterium is not spread by air or droplets. therefore, the client needs to be assessed for myocardial
Answer 2 is correct because the bacterium can be spread ischemia. Answer 2 is incorrect because this client would
on the hands of health care providers. Answer 3 is incor- be seen third. The client is able to recognize and verbalize
rect because the bacterium is not airborne. Answer 4 is symptoms of hypoglycemia and the need to eat. The client
correct because C. difficile is relatively resistant to could call the nurse if symptoms were increasing. Answer 3
disinfectants. Therefore, items that are close to the is incorrect because this client would be seen second. This
client should be cleaned daily with a bleach-and-water client is at high risk for falling. The reason for the alarm
solution. Answer 5 is correct because metronidazole and needs to be assessed. Answer 4 is incorrect because this
vancomycin are the drugs of choice. client would be assessed last. Red-colored urine is normal
TEST-TAKING TIP: Know that C. difficile is transmitted by after a transurethral resection of the prostate (TURP),
direct contact with infected clients. and the presence of urine in the bag indicates the urine is
Content Area: Adult Health, Infectious Disease; Integrated flowing.
Process: Nursing Process, Planning; Cognitive Level: TEST-TAKING TIP: Top priority in this situation is a
Application; Client Need/Subneed: Safe and Effective Care client with a potentially life-threatening condition
Environment/Safety and Infection Control/Standard/ (cardiac).
Transmission-based/Other Precautions Content Area: Adult Health, Triage; Integrated Process:
Nursing Process, Analysis; Cognitive Level: Analysis;
23. CORRECT ANSWERS: 1, 4, 5, 6. Answer 1 is correct Client Need/Subneed: Safe and Effective Care Environment/
because a private room would reduce exposure of the
Management of Care/Establishing Priorities
client to other organisms transmitted by close contact
2164_Ch03_069-114 29/03/12 12:20 PM Page 106

106 chapter 3 Safe, Effective Care Environment

26. CORRECT ANSWER: 1. Answer 1 is correct because a 29. CORRECT ANSWER: 2. Answer 1 is incorrect because
client color-coded red (emergency) is the first priority. core measures data are collected during the hospital experi-
This client has a life-threatening problem that needs ence on admission. Answer 2 is correct because these meas-
immediate action. Answer 2 is incorrect because this client ures or standards of care are required by the Joint
would be color-coded black (dead) and is nonsalvageable. Commission to ensure that the best practices are being
Answer 3 is incorrect because this client would be color- used in the hospital. An example of a core measure is the
coded yellow (caution), and is the second priority level standard of care for a client admitted with heart failure.
because the fracture is closed. Answer 4 is incorrect because Answer 3 is incorrect because core measures are documenta-
this client would be color-coded green (walking wounded) tions of the standard of care for certain categories of clients
and would be the third priority. Evaluation would be or services, not a specific nurses role. Answer 4 is incorrect
delayed. because the hospital collects and submits the data to
TEST-TAKING TIP: Memorize the color-coded disaster triage JCAHO, not the client.
system: black = dead; red = emergency; yellow = caution; and TEST-TAKING TIP: Core is central to the operation of the
green = go. hospitalquality performance is the objective of care.
Content Area: Adult Health, Triage; Integrated Process: Nursing Content Area: Management of Care, Quality Improvement;
ANSWERS

Process, Analysis; Cognitive Level: Analysis; Client Need/Subneed: Integrated Process: Nursing Process, Implementation; Cognitive
Safe and Effective Care Environment/Safety and Infection Level: Comprehension; Client Need/Subneed: Safe and Effective
Control/Emergency Response Plan Care Environment/Management of Care/Performance
Improvement (Quality Improvement)
27. CORRECT ANSWERS: 1, 5. Answer 1 is correct because
the activity is technical, and there is no potential for harm 30. CORRECT ANSWER: 2. Answer 1 is incorrect because
to the client. The RN would need to evaluate the findings. there is no history stated in the stem of the question that the
Answer 2 is incorrect because the need for a nasogastric (NG) client is unable to get to the bathroom. In the hospital, this
tube may indicate that the client is not stable. The outcome is would increase the risk. Answer 2 is correct because the cir-
not predictable from client to client. Answer 3 is incorrect cumstances of a previous fall can be used to plan specific
because the RN needs to assess the clients ability to swallow, interventions to prevent a fall. Answer 3 is incorrect because
to prevent aspiration of food. Answer 4 is incorrect because there is an increased risk, but no history here, that the client is
the procedure requires assessment and judgment regarding how unable to ambulate safely. Answer 4 is incorrect because the
much to instill to irrigate and evaluation of the results. drug effects are not known, although medications are a cause
Answer 5 is correct because it is routine and not complex. of falls.
The clients condition is not life threatening. Answer 6 is TEST-TAKING TIP: All of the options are risk factors, but a
incorrect because special precautions are needed to ensure prior history and the cause of the falls is the best predictor.
that hair particles do not enter the tracheostomy. This proce- Content Area: Management of Care, Safety; Integrated Process:
dure requires assessment, which needs to be done by a Nursing Process, Analysis; Cognitive Level: Application; Client
licensed staff member. Need/Subneed: Safe and Effective Care Environment/Safety
TEST-TAKING TIP: Only the RN can assess the condition and and Infection Control/Accident Prevention and Injury
evaluate the response of clients who are unstable. Prevention
Content Area: Management of Care, Delegation; Integrated
31. CORRECT ANSWER: 3. Answer 1 is incorrect because
Process: Nursing Process, Analysis; Cognitive Level: Analysis;
restraints are never tied to the side rails. This increases the
Client Need/Subneed: Safe and Effective Care Environment/
chance of injury. Answer 2 is incorrect because ideally there
Management of Care/Delegation
should be no tension on the restraints. Answer 3 is correct
28. CORRECT ANSWER: 2. Answer 1 is incorrect because because the tightness of the restraints and the effects on
yeast is present on the skin normally, not transmitted because clients skin must be frequently assessed. Answer 4 is incor-
of poor hand hygiene. Answer 2 is correct because poor rect because usually the lights should be dimmed to decrease
blood sugar control is related to the quality of care received stimuli when a person is agitated.
while hospitalized. Answer 3 is incorrect because weight TEST-TAKING TIP: Choose an option that directly relates
loss is a result of illness, not care. Answer 4 is incorrect because to the client; side rails, head of bed, or lights indirectly affect
the need for surgery is determined by diagnosis, not quality the client.
of care. Content Area: Adult Health, Integumentary; Integrated Process:
TEST-TAKING TIP: Look for an option that is most influenced Nursing Process, Implementation; Cognitive Level: Application;
by quality of care. Client Need/Subneed: Safe and Effective Care Environment/
Content Area: Management of Care, Quality Improvement; Safety and Infection Control/Use of Restraints/Safety Devices
Integrated Process: Nursing Process, Analysis; Cognitive Level:
Application; Client Need/Subneed: Safe and Effective Care
32. CORRECT ANSWER: 1. Answer 1 is correct because
spelling the name and stating birth date would only be
Environment/Management of Care/Performance Improvement
possible by the actual client. Answer 2 is incorrect because
(Quality Improvement)
2164_Ch03_069-114 29/03/12 12:20 PM Page 107

Answers/Rationales/Tips 107
clients can wander in and out of rooms. Answer 3 is incorrect severe dehydration, which should be addressed quickly.
because client misidentification has been reported with verbal Answer 4 is incorrect because, although the childs pain
affirmation of name. Answer 4 is incorrect because there is no control is a concern, dehydration is the top priority.
assurance that the client who is receiving the medication is the TEST-TAKING TIP: Follow the ABCs for prioritization. If all
client on the MAR. of the children have stable airways, look for the child that is at
TEST-TAKING TIP: Look for the most reliable form of identity risk of circulatory collapse. In this case, the 6-month-old infant
verification. is at greatest risk due to age and history.
Content Area: Management of Care, Error Prevention; Integrated Content Area: Child Health, Fluid and Electrolyte Imbalances;
Process: Nursing Process, Implementation; Cognitive Level: Integrated Process: Nursing Process, Assessment; Cognitive
Comprehension; Client Need/Subneed: Safe and Effective Care Level: Application; Client Need/Subneed: Safe and Effective
Environment/Safety and Infection Control/Error Prevention Care Environment/Management of Care/Establishing Priorities
33. CORRECT ANSWERS: 1, 2, 3, 5. Answer 1 is correct 36. CORRECT ANSWER: 2. Answer 1 is incorrect since a
because hand washing is the most important technique to negative pressure room should only be used for controlling
reduce the risk of infections. Answer 2 is correct because the spread of an infectious agent that can remain suspended
hand washing is the most important technique to reduce in the air, such as tuberculosis. Answer 2 is correct because

ANSWERS
the risk of infections. Answer 3 is correct because hand RSV is transmitted by contact. The child diagnosed with
washing is the most important technique to reduce the risk RSV should be placed on contact precautions (gown,
of infections. Answer 4 is incorrect because gloves do not gloves) with strict emphasis on hand washing. Answer 3 is
replace the need for frequent hand washing. Answer 5 is cor- incorrect because RSV is not an airborne infectious agent.
rect because hand washing is the most important technique Answer 4 is incorrect since the child with RSV does not need
to reduce the risk of infections. a positive pressure room. Positive pressure rooms keep out-
TEST-TAKING TIP: Choose every opportunity to wash side air from entering the clients space and are designed
hands. specifically to protect clients who are severely immunocom-
Content Area: Adult Health, Integumentary; Integrated Process: promised from acquiring infections while hospitalized.
Nursing Process, Analysis; Cognitive Level: Application; TEST-TAKING TIP: Airborne precautions and negative
Client Need/Subneed: Safe and Effective Care Environment/ pressure rooms are similar responses and can be eliminated.
Safety and Infection Control/Standard/Transmission-based/ Content Area: Child Health, Respiratory; Integrated Process:
Other Precautions Nursing Process, Analysis; Cognitive Level: Application;
Client Need/Subneed: Safe and Effective Care Environment/
34. CORRECT ANSWER: 1. Answer 1 is correct because Safety and Infection Control/Standard/Transmission-based/
hands should be vigorously rubbed together for at least
Other Precautions
15 seconds, not 10 seconds. Answer 2 is incorrect because
the lather is not a contaminant if dripped in the sink. 37. CORRECT ANSWERS: 2, 3, 5, 6. Answer 1 is incorrect
Answer 3 is incorrect because the friction of rubbing is most because an appropriate fluid bolus for a child with severe
important, not the temperature of the H2O. Answer 4 is dehydration is 20 mL/kg. Answer 2 is correct because the
incorrect because inadequate drying is not an immediate nurse should first clean the hands for infection preven-
problem with regard to transmission of organisms from tion. Answer 3 is correct because the nurse should wear
client to client. personal protective equipment for enteric precautions
TEST-TAKING TIP: The stem asks for what is not OK. The after cleaning hands. Answer 4 is incorrect because acute
most important part of effective hand washing is friction. gastroenteritis is typically caused by a viral illness such as
Content Area: Adult Health, Integumentary; Integrated Process: rotavirus, not bacterial illness. Antibiotics would be adminis-
Nursing Process, Evaluation; Cognitive Level: Application; tered for bacterial, not viral, infection. Answer 5 is correct
Client Need/Subneed: Safe and Effective Care Environment/ because the childs urinary output should be monitored
Safety and Infection Control/Standard/Transmission-based/ frequently to determine the response to treatment.
Other Precautions Answer 6 is correct because, once the nurse is protected,
IV fluids should be administered to correct the severe
35. CORRECT ANSWER: 3. Answer 1 is incorrect because dehydration.
frequent sneezing is a normal finding in young infants when
TEST-TAKING TIP: Answers 2 and 3 go together (nurses
they attempt to clear the airway of secretions. Since the
self-protection). Answers 5 and 6 go together (give fluid,
child is crying vigorously, oxygenation appears satisfactory at
monitor output).
this time. Answer 2 is incorrect because, although the child
Content Area: Child Health, Gastrointestinal; Integrated
is at risk for dehydration due to frequent voiding and fever,
Process: Nursing Process, Implementation; Cognitive Level:
a 2-year-old child is better able to compensate for dehydra-
Application; Client Need/Subneed: Safe and Effective Care
tion than an infant, and the child may also have adequate
Environment/Safety and Infection Control/Standard/
oral intake. Answer 3 is correct because the infants history
Transmission-based/Other Precautions
and assessment findings are suggestive of moderate to
2164_Ch03_069-114 29/03/12 12:20 PM Page 108

108 chapter 3 Safe, Effective Care Environment

38. CORRECT ANSWER: 1. Answer 1 is correct because Younger children (the third client) and those without a
the child is not considered contagious once all of the parent (the second client) at the bedside should also be a
chickenpox (varicella) lesions have crusted. Answer 2 is top priority.
incorrect because antibiotics are not effective against chick- Content Area: Child Health, Hospitalization; Integrated
enpox as it is a viral infection. Answer 3 is incorrect because Process: Nursing Process, Implementation; Cognitive Level:
a child with or without fever but who has open lesions is Analysis; Client Need/Subneed: Safe and Effective Care
still considered infectious. Answer 4 is incorrect because a Environment/Management of Care/Establishing Priorities
positive varicella titer indicates that the child has been 40. CORRECT ANSWER: 4. Answer 1 is incorrect because
exposed to varicella, not that the child is no longer conta- having the parents learn to auscultate the heart is not the best
gious to others. way to assist them in monitoring for cardiac complications.
TEST-TAKING TIP: Look for answer options to eliminate Rather, the parents should be taught to look for symptoms of
when you know that they cannot be correct. If you know that impaired circulation, such as fatigue and pallor. Answer 2 is
chickenpox is caused by a virus, you can eliminate Answer 2. incorrect because it is not recommended that children with a
Answer 4 can be crossed out if you recall that a titer only history of Kawasaki disease receive prophylactic antibiotics
provides information about the childs exposure, not a risk prior to dental work. Kawasaki disease affects the coronary
ANSWERS

to others. Of the remaining two choices, you can eliminate arteries, not the endocardium, so the risk of bacterial endo-
Answer 3 since many illnesses are contagious during the carditis is not increased. Answer 3 is incorrect because repeat
prodromal period when fever may or may not be present. doses of immunoglobulin are not indicated for this condition.
Therefore, fever is not related to communicability, and the Answer 4 is correct because a key component in the treat-
best answer is Answer 1. ment of Kawasaki disease is the administration of intra-
Content Area: Child Health, Infectious Disease; Integrated
venous immune globulin (IVIG). Parents should be
Process: Nursing Process, Implementation; Cognitive Level:
advised that immunizations, such as the MMR (measles-
Application; Client Need/Subneed: Safe and Effective Care mumps-rubella), may be delayed for some months since
Environment/Safety and Infection Control/Standard/ IVIG could render the vaccinations ineffective.
Transmission-based/Other Precautions TEST-TAKING TIP: The answer options should help you recall
39. CORRECT ANSWERS: 3, 2, 4, 1. Answer 3. The third that immunoglobulin is a cornerstone of treatment for Kawasaki
client would be seen first because this child has been disease. Since this treatment modulates the immune system,
under recent sedation and has had the airway manipulat- select the answer option that addresses a potential effect of the
ed. A nurse should assess this child for high risk for treatment.
respiratory complications. Answer 2. The second client Content Area: Child Health, Infectious Disease; Integrated
would be seen second because the child should be Process: Teaching and Learning; Cognitive Level: Application;
assessed for potential complications of traction. Client Need/Subneed: Safe and Effective Care Environment/
Additionally, this is a younger school-age child who is Safety and Infection Control/Standard/Transmission-based/
in the room alone, needing the nurses attention to Other Precautions
make sure toys, food, etc. are within the childs reach. 41. CORRECT ANSWER: 3. Answer 1 is incorrect because,
Answer 4. The fourth client would be seen third because
although the younger the child is when the diagnosis of HIV
the child has an unstable condition, at risk for rupture is made, the lower the life expectancy, this information is
of the appendix. The child also needs to be assessed, with best discussed when the parent has questions regarding prog-
documentation done and a presurgical checklist complet- nosis. This should not be the nurses main focus during an
ed, before the child can be transported to surgery as initial counseling session. Answer 2 is incorrect because,
soon as the operating room is ready. This child is usually although children with HIV are at risk for developing failure
ambulatory and is old enough to call the nurse for urgent to thrive, nutrition is not the most important topic of an
needs; therefore, this child is a lower priority than the initial meeting. Answer 3 is correct because it is essential
child immobilized by traction. Answer 1. The first client that the parent recognize the importance of following
would be seen last because the child with encopresis has through with the medication regimen prescribed by the
a condition associated with constipation and fecal reten- practitioner. Failure to do so could quickly lead to resist-
tion in which watery colonic contents bypass hard fecal ant HIV and opportunistic infection. Answer 4 is incorrect
masses and pass through the rectum. This is the lowest because immunization is one of the keys to infection preven-
priority because this condition does not have the poten- tion in the child with HIV and should not be delayed.
tial for acute complications, and the childs parent is at TEST-TAKING TIP: Note the time frame given in the stem
the bedside. of the question: during the first meeting. Initial counseling
TEST-TAKING TIP: Children with the potential for airway sessions regarding chronic illnesses should stress those self-care
complications (the third client) and hazards of immobility topics that have the greatest potential for harm, especially in
(the second client) should be the nurses highest priority. the short term.
2164_Ch03_069-114 29/03/12 12:20 PM Page 109

Answers/Rationales/Tips 109
Content Area: Child Health, Infectious Disease; Integrated risk for problems with arterial circulation. It is not necessary to
Process: Teaching and Learning; Cognitive Level: Application; do frequent pulse checks. Answer 2 is correct because, during
Client Need/Subneed: Safe and Effective Care Environment/ breast surgery, lymph nodes are removed to rid the body of
Safety and Infection Control/Standard/Transmission-based/ spreading cancer. This leaves the distal tissue at risk for
Other Precautions swelling and infections. IV catheters, carrying a risk of
infection, should be placed in the opposite arm. Answer 3 is
42. CORRECT ANSWER: 3. Answer 1 is incorrect because a incorrect because lymph nodes may have been removed from
slight elevation in WBC count is common with the normal
the right auxiliary area with the mastectomy; the client is at
inflammation accompanying surgery. No postoperative risks
risk for arm swelling. Raising the arm is permitted and may
are identified in these data and thus no immediate treatment
help minimize swelling. Answer 4 is incorrect because move-
is required. Answer 2 is incorrect because an elderly client
ment of the right arm will help promote venous return in this
who is postoperative with a history of congestive heart
arm with limited lymphatic damage.
failure (CHF) should be evaluated for response to pain
TEST-TAKING TIP: Three of the choices relate to the right
medications and respiratory status, but is not the most
arm, only one the left arm. Choose the option, in this case, that
critical of the clients described. Answer 3 is correct because
is different.
bloodstream infections are of great concern in clients

ANSWERS
Content Area: Adult Health, Respiratory; Integrated Process:
without the normal filter of the spleen. The high temper-
Nursing Process, Implementation; Cognitive Level: Application;
ature indicates this client could be septic and will require
Client Need/Subneed: Physiological Integrity/Reduction of Risk
immediate care with fluids and antibiotics. Answer 4 is
Potential/Potential for Alterations in Body Systems
incorrect because a platelet count of 100,000, although
low, does not put this client at a clinically significant risk 45. CORRECT ANSWER: 3. Answer 1 is incorrect because the
of bleeding. Assessment and care of more critical clients immediate goal is to remove the potentially infectious material.
should take priority. Covering the scab will not accomplish this. Answer 2 is incorrect
TEST-TAKING TIP: Three of the clients have orthopedic because ice will not help remove the blood from the nurses skin.
conditions. Choose the only option that concerns a vital organ Answer 3 is correct because washing is the most effective way
(spleen). Who is at imminent risk? Think about the stability of of removing potentially infectious material. Avoid disrupting
the clients. Select the most unstable. the scab, which is providing a microbial barrier. Answer 4 is
Content Area: Adult Health, Infection Control; Integrated incorrect because scrubbing the scab may remove the barrier that
Process: Nursing Process, Assessment; Cognitive Level: Analysis; prevents blood from entering the nurses body.
Client Need/Subneed: Safe and Effective Care Environment/ TEST-TAKING TIP: Washing the area is the first action for
Management of Care/Establishing Priorities controlling the spread of infection.
Content Area: Adult Health, Infection Control; Integrated
43. CORRECT ANSWER: 4. Answer 1 is incorrect because Process: Nursing Process, Implementation; Cognitive Level:
Betadine (an iodine solution) might be introduced into the IV.
Application; Client Need/Subneed: Safe and Effective Care
If the port requires cleaning, use an alcohol swab. Answer 2 is
Environment/Safety and Infection Control/Standard/
incorrect because indwelling bladder catheters are associated
Transmission-based/Other Precautions
with infections. If there were a perineal wound, a bladder
catheter might help to protect the wound from infection, but 46. CORRECT ANSWER: 1. Answer 1 is correct because the
when perineal tissue is intact it should be maintained that elevated blood pressure may indicate stress or pain, and
way with normal skin cleaning. Answer 3 is incorrect because further assessment is needed to establish why the blood
moisture promotes growth of skin organisms such as yeast. pressure is elevated. Answer 2 is incorrect because a low
Skinfolds should be kept dry. Answer 4 is correct because pulse is common in athletes. Answer 3 is incorrect because
wounds that require dressings should be covered. Frequent this client would not be the first priority. The nurse should
removal for wound assessments increases infection risk and see this client second, as the temperature in the older adult
reduces optimal presence of healing fluids and substances may not be an accurate indicator of the clients condition.
in wound bed. Answer 4 is incorrect because right shoulder pain or pain in
TEST-TAKING TIP: Look for the option that provides the the scapular area are normal and result from the gas used to
greatest protection or reduces risk. insufflate the abdominal cavity during the procedure.
Content Area: Adult Health, Infection Control; Integrated TEST-TAKING TIP: Look at the vital signs; select the elevated
Process: Nursing Process, Implementation; Cognitive Level: BP. Choose the client who is most unstable, or the condition that
Application; Client Need/Subneed: Safe and Effective Care puts the client at greater risk.
Environment/Safety and Infection Control/Standard/ Content Area: Adult Health, Triage; Integrated Process: Nursing
Transmission-based/Other Precautions Process, Analysis; Cognitive Level: Analysis; Client Need/
Subneed: Safe and Effective Care Environment/Management
44. CORRECT ANSWER: 2. Answer 1 is incorrect because of Care/Establishing Priorities
neither the mastectomy nor the pneumonia puts the client at
2164_Ch03_069-114 29/03/12 12:20 PM Page 110

110 chapter 3 Safe, Effective Care Environment

47. CORRECT ANSWERS: 1, 3, 4. Answer 1 is correct because of 10 mm or more is a positive reaction. Answer 3 is incor-
this is the probability for these parents to have a child with rect because redness is not the indicator of a reaction. The
sickle cell anemia. Answer 2 is incorrect because the probability diameter of the induration (raised area) is important. Answer 4
for these parents to have a child with sickle cell trait is 100%. is incorrect because the test is not read before 48 hours regard-
Answer 3 is correct because this is the probability for these less of when redness or induration appears.
parents to have a child with sickle cell trait. Answer 4 is cor- TEST-TAKING TIP: Select the only option that just addresses
rect because this is the probability for these parents to have a induration. The other three all include redness.
child with sickle cell anemia. Answer 5 is incorrect because Content Area: Adult Health, Infectious Disease; Integrated
the probability for these parents to have a child with sickle cell Process: Nursing Process, Assessment; Cognitive Level:
anemia is zero. Application; Client Need/Subneed: Safe and Effective Care
TEST-TAKING TIP: Write out a mendelian square when Environment/Safety and Infection Control/Standard/
considering heredity. See the table below, which uses the Transmission-based/Other Precautions
letter S to represent sickle cell trait and s to represent 50. CORRECT ANSWER: 2. Answer 1 is incorrect because calf
no trait, with SS indicating an individual with sickle cell pain may indicate an embolus, which can be fatal. Answer 2 is
disease. In the example, a parent with sickle cell disease is correct because the oxygen saturation is normal. The asthma
ANSWERS

paired with a parent who carries the trait, with a resulting is being controlled. Answer 3 is incorrect because older adults
50% chance that their offspring will have the disease or have subnormal temperatures, so the cause of the elevated
carry the trait. temperature has not been resolved. Answer 4 is incorrect
because rebound abdominal pain may indicate appendicitis.
S S Rupture may occur if untreated.
S SS SS TEST-TAKING TIP: Look for the client whose condition is most
s Ss Ss stable. Determine if the clinical situation is a normal state and
choose that option.
Content Area: Adult Health, Triage; Integrated Process:
Content Area: Child Health, Hematological; Integrated Process:
Nursing Process, Analysis; Cognitive Level: Analysis;
Nursing Process, Analysis; Cognitive Level: Application;
Client Need/Subneed: Safe and Effective Care Environment/
Client Need/Subneed: Safe and Effective Care Environment/
Management of Care/Establishing Priorities
Management of Care/Case Management
51. CORRECT ANSWER: 2. Answer 1 is incorrect because
48. CORRECT ANSWER: 4. Answer 1 is incorrect because there is no immediate risk to the client. Answer 2 is correct
endometriosis is a condition where tissue similar to the lining
because chemotherapy drugs are toxic and can cause tissue
of the uterus (the endometrial stroma and glands, which
destruction if infiltrated, and maintaining the access port is
should only be located inside the uterus) is found elsewhere in
vital to therapy. Answer 3 is incorrect because there is no
the body. It is not an infection and does not cause fever,
immediate risk to the client. Answer 4 is incorrect because there
abdominal tenderness, or foul-smelling lochia. Answer 2 is
is no immediate risk to the client.
incorrect because, even though this client is at risk for postop-
TEST-TAKING TIP: Look for the client who is most vulnerable
erative wound infection, a postoperative wound infection does
to a serious consequence. Eliminate those answers that focus on
not cause foul-smelling lochia. Answer 3 is incorrect because,
GI feeding (Answer 1), elimination (Answer 3), and psychoso-
although this client is at risk for pelvic thrombophlebitis,
cial need (Answer 4).
which causes fever, it does not have foul-smelling lochia or
Content Area: Adult Health, Triage; Integrated Process:
abdominal tenderness. Answer 4 is correct because the most
Nursing Process, Analysis; Cognitive Level: Analysis;
common reason for postpartum fever is endometritis. Other
Client Need/Subneed: Safe and Effective Care Environment/
symptoms include abdominal tenderness and foul-smelling
Management of Care/Establishing Priorities
lochia. Postpartum endometritis occurs in 15% to 20% of
unscheduled cesarean deliveries after a prolonged labor. 52. CORRECT ANSWER: 2. Answer 1 is incorrect because dia-
TEST-TAKING TIP: Endometriosis and endometritis sound betes is not a contraindication for breastfeeding. Answer 2 is
similar; the ending itis usually indicates infection. correct because HIV can be transmitted through breast milk.
Content Area: Maternity, Intrapartum; Integrated Process: Answer 3 is incorrect because hypertension is not a contraindi-
Nursing Process, Analysis; Cognitive Level: Analysis; Client cation for breastfeeding. Answer 4 is incorrect because thyroid
Need/Subneed: Safe and Effective Care Environment/Safety disease is not a contraindication for breastfeeding.
and Infection Control/Standard/Transmission-based/Other TEST-TAKING TIP: Consider the risk and benefit of breast-
Precautions feeding. The transmission of HIV is a bigger risk to the neonate
than the benefit of breastfeeding.
49. CORRECT ANSWER: 2. Answer 1 is incorrect because Content Area: Maternity, Postpartum; Integrated Process:
redness alone is not an indication of a positive test. Also, the Teaching and Learning; Cognitive Level: Application;
reaction is assessed at 48 hours after testing, not after 24 hours. Client Need/Subneed: Safe and Effective Care Environment/
Answer 2 is correct because the presence of a positive TB
Safety and Infection Control/Standard/Transmission-based/
test is determined by the size of the induration. Induration Other Precautions
2164_Ch03_069-114 29/03/12 12:20 PM Page 111

Answers/Rationales/Tips 111
53. CORRECT ANSWER: 4. Answer 1 is incorrect because the is unsafe for an infant. Answer 4 is incorrect because the
right side would not be weakened. Answer 2 is incorrect bassinette will put the infant at risk for alterations in body tem-
because the right side would not be weakened. Answer 3 is perature, and the phototherapy should be applied through a
incorrect because the unaffected right side should be next to shield such as plastic to minimize ultraviolet radiation exposure.
the bed. Answer 4 is correct because, with a right-sided TEST-TAKING TIP: Think about the possible treatments for
brain attack, the client would have left-sided hemiplegia or a child with hyperbilirubinemia. Since the child will likely
weakness. The clients unaffected side should be closest to receive phototherapy, the equipment must be suited for this
the bed to facilitate the transfer. treatment.
TEST-TAKING TIP: Visualize the client with weakness on the Content Area: Child Health, Hematological; Integrated Process:
opposite side of the brain attack. Nursing Process, Implementation; Cognitive Level: Application;
Content Area: Adult Health, Cardiovascular; Integrated Process: Client Need/Subneed: Safe and Effective Care Environment/
Nursing Process, Implementation; Cognitive Level: Application; Safety and Infection Control/Safe Use of Equipment
Client Need/Subneed: Safe and Effective Care Environment/
57. CORRECT ANSWER: 2. Answer 1 is incorrect because
Safety and Infection Control/Accident Prevention and Injury there is no indication that this client is the most unstable of the
Prevention four clients. Answer 2 is correct because the K+ level is above

ANSWERS
54. CORRECT ANSWER: 1. Answer 1 is correct because the the normal high of 5.5 mEq/L. An elevated K+ can have
client has left visual field blindness. The client will see only serious effects on the heart and affect the stability of the
from the right side. Answer 2 is incorrect because the client client. The physician needs to be called regarding the report
would not see the call light on the left side. Answer 3 is incor- so that changes in treatment can be made to manage the
rect because the client only sees from the nasal portion of the elevated K+. Answer 3 is incorrect because it is the client with
left eye and the temporal area of the right eye. Directly in front the elevated K+ who is at risk for life-threatening arrhythmias,
is better than the left side, but not the best choice. Answer 4 is and the K+ level needs to be reported right away. Answer 4 is
incorrect because the nurse must ensure client safety by placing incorrect because the client sent to ICU is receiving close
the call light where it will be seen. observation. The family of this client definitely needs to be
TEST-TAKING TIP: Requires knowledge of visual fields and contacted after reporting the client with the elevated K+ level.
optic nerve structure. TEST-TAKING TIP: Consider which client is at greatest risk
Content Area: Adult Health, Neurological; Integrated Process: for a life-threatening consequence.
Nursing Process, Implementation; Cognitive Level: Application; Content Area: Adult Health, Triage; Integrated Process:
Client Need/Subneed: Safe and Effective Care Environment/ Nursing Process, Analysis; Cognitive Level: Analysis;
Safety and Infection Control/Accident Prevention and Injury Client Need/Subneed: Safe and Effective Care Environment/
Prevention Management of Care/Establishing Priorities
55. CORRECT ANSWER: 3. Answer 1 is incorrect because the 58. CORRECT ANSWER: 2. Answer 1 is incorrect because
stoma will be 1 to 2 cm above the skin. Because the stoma is swimming is a suggested exercise for pregnancy and is safe unless
swollen after surgery, the size will change. Answer 2 is incorrect the amniotic membranes have ruptured. Answer 2 is correct
because some bleeding is normal. If it bleeds too often, the because women who are pregnant should not sit in hot tubs
physician should be notified. Answer 3 is correct because, for longer than 10 minutes. Becoming overheated in a hot
since there are no nerves in the mucous membranes, there tub is not recommended during pregnancy. The American
should be no pain when touching. Pain would possibly College of Obstetricians and Gynecologists (ACOG) also
indicate a problem internally. Answer 4 is incorrect because recommends that women who are pregnant should never let
this is a normal appearance. If the stoma becomes dusky in their core body temperature rise above 102.2F. Women who
color, this could need immediate action. choose to use a hot tub should monitor their temperature to
TEST-TAKING TIP: Pain is not normal and indicates a avoid overheating. Answer 3 is incorrect because a warm bath,
problem for any client situation. which is not uncomfortable or scalding, is a safer way to relax.
Content Area: Adult Health, Gastrointestinal; Integrated In a bath, much of the upper body will remain out of the water,
Process: Nursing Process, Analysis; Cognitive Level: Application; making it less likely to overheat. Additionally, the water in a
Client Need/Subneed: Safe and Effective Care Environment/ bath begins to cool off, as opposed to a hot tub, which further
Management of Care/Establishing Priorities reduces a risk of overheating. Answer 4 is incorrect because a
sauna, like a hot tub, can lead to overheating and should be
56. CORRECT ANSWER: 1. Answer 1 is correct because the used with caution in short periods of time.
infant will likely be receiving phototherapy, which requires TEST-TAKING TIP: This question is asking which statement is
the baby to be unclothed. The isolette allows the baby to be not a safe action by the client and therefore requires further
in a closed, temperature-regulated environment. Answer 2 is health teaching.
incorrect because the open crib will put the infant at risk for Content Area: Maternity, Antepartum; Integrated Process:
alterations in body temperature, and the phototherapy should be Nursing Process, Evaluation; Cognitive Level: Analysis; Client
applied through a shield such as plastic to minimize ultraviolet Need/Subneed: Safe and Effective Care Environment/Safety and
radiation exposure. Answer 3 is incorrect because a hospital bed Infection Control/Accident Prevention and Injury Prevention
2164_Ch03_069-114 29/03/12 12:20 PM Page 112

112 chapter 3 Safe, Effective Care Environment

59. CORRECT ANSWER: 1. Answer 1 is correct because, require a mask. Answer 3 is correct because standard
before delivery of the neonates body, it is important to (universal) precautions should be used, and washing
check for a cord around the neonates neck. It is the first hands is the most important technique for preventing
action after delivery of the head and before the shoulders disease transmission. Answer 4 is incorrect because reverse
are delivered. Answer 2 is incorrect because antibiotic oint- (protective) isolation would usually be indicated for a client
ment in the infants eyes can wait for up to an hour. It is not a with AIDS with a very low WBC count. There is no infor-
priority after the delivery of the head. Answer 3 is incorrect mation indicating that that is the status.
because the DeLee suction is no longer used to remove secre- TEST-TAKING TIP: Look for hand washing and gloves first.
tions; a bulb syringe should be used to remove secretions prior Content Area: Adult Health, Infectious Disease; Integrated
to delivery of the neonates body. Answer 4 is incorrect because Process: Nursing Process, Implementation; Cognitive Level:
the body needs to be delivered before the heart rate and respi- Application; Client Need/Subneed: Safe and Effective Care
rations can be evaluated. This is not the first action after deliv- Environment/Safety and Infection Control/Standard/
ery of the neonates head. Transmission-based/Other Precautions
TEST-TAKING TIP: Think cord as the first action after deliv- 62. CORRECT ANSWER: 2. Answer 1 is incorrect because this
ery of the head and before the shoulders and body. Picture the is a priority question. Ambulating after the sedation wears off
ANSWERS

cardinal movements and the process of delivery. is not wrong; it just is not the first concern. Answer 2 is correct
Content Area: Maternity, Intrapartum; Integrated Process:
because the throat will have been anesthetized to facilitate
Nursing Process, Implementation; Cognitive Level: Application; passage of the tube. If the gag reflex has not returned before
Client Need/Subneed: Safe and Effective Care Environment/
food or fluid is given, the client may aspirate. Answer 3 is
Safety and Infection Control/Accident Prevention and Injury incorrect because teaching should have been done before the
Prevention procedure. It is also not the first concern. Answer 4 is incorrect
60. CORRECT ANSWER: 3. Answer 1 is incorrect because because no particular positioning is indicated following a
hot water can cause leakage of lead from old pipes. Cold diagnostic bronchoscopy.
water should be used for drinking, cooking, and preparation TEST-TAKING TIP: Priority: Which option, if not done, would
of infant formula. Cold water reduces the leakage of lead cause greatest harm to the client?
from old pipes. Answer 2 is incorrect because hot water can Content Area: Adult Health, Respiratory; Integrated Process:
cause leakage of lead from old pipes. The family should use Nursing Process, Implementation; Cognitive Level: Analysis;
cold water for drinking, cooking, and preparation of infant Client Need/Subneed: Safe and Effective Care Environment/
formula. Cold water reduces the leakage of lead from old Management of Care/Establishing Priorities
pipes. Answer 3 is correct because hot water can cause 63. CORRECT ANSWER: 2. Answer 1 is incorrect because,
leakage of lead from old pipes. The family should use although plausible, gauze is not occlusive and may not be readi-
cold water for drinking, cooking, and preparation of ly available. Answer 2 is correct because the priority is to
infant formula. Cold water reduces the leakage of lead prevent air from entering the chest cavity with the most
from old pipes. Answer 4 is incorrect because buying bot- occlusive method. (Gloves might be available, but time is
tled water may not be financially realistic or possible for this critical.) Answer 3 is incorrect because it would take too
family. The family should instead use the available cold much time to reposition the client in bed. Cover the opening
water in their apartment building for drinking, cooking, immediately, then put the client to bed. Answer 4 is incorrect
and preparation of infant formula. Cold water reduces the because the best answer requires action by the nurse, not the
leakage of lead from old pipes. physician.
TEST-TAKING TIP: Hot causes dilation, whereas cold causes TEST-TAKING TIP: Priority: Do something to reduce
constriction. This is true in pipes as well as in the human immediate risk to the clienta sucking hole. Consider the
body. Dilation will lead to the release of lead particles. two options that include cover and choose the one with the
Content Area: Child Health, Poisoning; Integrated Process:
most available resource.
Teaching and Learning; Cognitive Level: Application; Client Content Area: Adult Health, Respiratory; Integrated Process:
Need/Subneed: Safe and Effective Care Environment/Safety
Nursing Process, Implementation; Cognitive Level: Analysis;
and Infection Control/Handling Hazardous and Infectious Client Need/Subneed: Safe and Effective Care
Materials Environment/Management of Care/Establishing Priorities
61. CORRECT ANSWER: 3. Answer 1 is incorrect because 64. CORRECT ANSWER: 2. Answer 1 is incorrect because,
the nurse is not responsible for double-bagging the soiled unless the person is debilitated, is taking steroids, or is
dressing, which would be placed in the contaminated recep- immunosuppressed, there is no restriction. Answer 2 is correct
tacle. Answer 2 is incorrect because PCP is not transmitted because infants and children under age 5 are susceptible to
through droplets. Masks and eyewear are only indicated TB. Answer 3 is incorrect because, unless the person is debili-
when the procedure may produce splashes of blood or body tated, is taking steroids, or is immunosuppressed, there is no
fluids. A routine dressing change of a leg wound should not restriction. Answer 4 is incorrect because, unless the person is
2164_Ch03_069-114 29/03/12 12:20 PM Page 113

Answers/Rationales/Tips 113
debilitated, is taking steroids, or is immunosuppressed, there is TEST-TAKING TIP: Do not delegate functions of assessment,
no restriction. evaluation, and nursing judgment.
TEST-TAKING TIP: Identify the nursing concept being tested Content Area: Management of Care, Delegation; Integrated
low immunity. Process: Nursing Process, Planning; Cognitive Level:
Content Area: Adult Health, Infectious Disease; Integrated Application; Client Need/Subneed: Safe and Effective Care
Process: Nursing Process, Planning; Cognitive Level: Analysis; Environment/Management of Care/Delegation
Client Need/Subneed: Safe and Effective Care Environment/
68. CORRECT ANSWER: 4. Answer 1 is incorrect because a
Safety and Infection Control/Standard/Transmission-based/
competent client, not a relative, gives consent. There is no
Other Precautions
indication of an emergency. Answer 2 is incorrect because the
65. CORRECT ANSWER: 2. Answer 1 is incorrect because the client has received the preoperative medication, so he or she
client will feel better soon after drug therapy is started, but cannot sign a consent form. Answer 3 is incorrect because
must take the drug long after physical improvement results. the responsibility for a correct consent form is the MDs, not
Answer 2 is correct because total destruction of the infection the nurses. Answer 4 is correct because the responsibility for
takes at least 9 months, if not longer. Answer 3 is incorrect an accurate informed consent is the physicians. An excep-
because a negative culture occurs in 2 months; however, the tion to this answer would be a life-threatening emergency,

ANSWERS
disease process can reactivate if drug therapy is stopped prema- but there are no data to support another response.
turely. Answer 4 is incorrect because, although some pathologic TEST-TAKING TIP: Know the legal responsibility for informed
changes with TB may be difficult to treat, lifelong therapy is consent. There is potential for risk in this situation.
not required. Content Area: Management of Care, Informed Consent;
TEST-TAKING TIP: Choose the only specific time element listed Integrated Process: Nursing Process, Implementation; Cognitive
(9 months). Know the expected outcome of treatment. Level: Analysis; Client Need/Subneed: Safe and Effective Care
Content Area: Adult Health, Infectious Disease; Integrated Environment/Management of Care/Informed Consent
Process: Teaching and Learning; Cognitive Level: Application;
69. CORRECT ANSWER: 3. Answer 1 is incorrect because pre-
Client Need/Subneed: Physiological Integrity/Physiological
venting damage, if the neck was broken, would be a priority
Adaptation/Illness Management
over neurological checks. Answer 2 is incorrect because check-
66. CORRECT ANSWER: 3. Answer 1 is incorrect because ing the airway is not included. Answer 3 is correct because
respiratory isolation guidelines do not require gowns and checking the airway would be a priority, and a neck injury
gloves for visitors or for staff during casual contact. Masks should be suspected. Answer 4 is incorrect because checking
would be worn by the client (if alert) or by family and staff the airway and stabilizing the neck and spine are not included.
if in close contact. TB is transmitted by airborne droplets. TEST-TAKING TIP: When appropriate for the injury, remem-
Answer 2 is incorrect because the door should not be open to ber ABCs as priorities.
the hallway. Ventilation is important, but is handled through Content Area: Adult Health, Trauma; Integrated Process:
the facilitys ventilation system. Answer 3 is correct because Nursing Process, Implementation; Cognitive Level: Analysis;
hand washing is the best method for reducing cross-con- Client Need/Subneed: Safe Effective Care Environment/
tamination. Gowns and gloves are not always required Management of Care/Establishing Priorities
when entering a clients room. Answer 4 is incorrect
70. CORRECT ANSWER: 4. Answer 1 is incorrect because
because it is not part of respiratory isolation requirements.
the nurses aide may not communicate the status of the
(Usually the client will be in a private room, but the isola-
assignment to the RN, or may be hesitant to report uncom-
tion precautions do not stipulate this.)
pleted work. The RN cannot assume. Answer 2 is incorrect
TEST-TAKING TIP: The most important action for infection
because, depending on the condition of the clients, the time
control is hand washing.
interval between communications may vary. Answer 3 is
Content Area: Adult Health, Infectious Disease; Integrated
incorrect because the clients may not be aware of the care
Process: Nursing Process, Evaluation; Cognitive Level:
needed. Answer 4 is correct because setting definite
Application; Client Need/Subneed: Safe and Effective Care
parameters will guide the nurses aide as to the priorities
Environment/Safety and Infection Control/Standard/
and what should be reported to the RN.
Transmission-based/Other Precautions
TEST-TAKING TIP: Three of the options are inappropriate
67. CORRECT ANSWER: 2. Answer 1 is incorrect because the (Answers 1 and 3) or unrealistic (Answer 2). Look at the
nasogastric tube may not be correctly positioned. Answer 2 is verbs to help select the best option for the concept of accounta-
correct because nursing assistants can walk clients (or feed bility; assume (no!); communicate and ask (too similar);
them, if there is no risk of aspiration). If assessment is define (yes!).
required, the task cannot be delegated. Answer 3 is incorrect Content Area: Management of Care, Delegation; Integrated
because nursing assistants cannot administer medications. Process: Nursing Process, Implementation; Cognitive Level:
Answer 4 is incorrect because the clients ability to swallow Analysis; Client Need/Subneed: Safe and Effective Care
must be assessed before the client is fed by a nursing assistant. Environment/Management of Care/Delegation
2164_Ch03_069-114 29/03/12 12:20 PM Page 114

114 chapter 3 Safe, Effective Care Environment

71. CORRECT ANSWER: 3. Answer 1 is incorrect because it Content Area: Adult Health, Cardiovascular; Integrated Process:
will not provide enough data to make a decision. It may Nursing Process, Implementation; Cognitive Level: Analysis;
yield only a yes or no reply. Answer 2 is incorrect because Client Need/Subneed: Safe and Effective Care Environment/
there are insufficient data to warrant this action as a priority Management of Care/Establishing Priorities
for RN intervention. Answer 3 is correct because more 74. CORRECT ANSWER: 3. Answer 1 is incorrect because con-
information is necessary in order to clarify the urgency fronting the RN may escalate the situation and result in con-
of the situation. This information will help delineate and flict. This choice does not resolve the basic issue of client abuse
prioritize nursing interventions. Answer 4 is incorrect by the staff member. Answer 2 is incorrect because documenta-
because, although this is valuable information, it is not the tion may compound the situation, creating legal ramifications.
best initial response by the RN. Answer 3 is correct. Objective reporting allows for client
TEST-TAKING TIP: Key word: initial. Note that two advocacy while maintaining anonymity for the staff. It
options (Answers 1 and 3) focus on asking for more data allows the supervisor to initiate corrective and therapeutic
from the LVN/LPN; choose the one that is likely to provide action with the person who is abusive, while protecting
more information (i.e., describe). clients from such abuse in the future. Appropriate action is
Content Area: Adult Health, Respiratory; Integrated Process:
objective reporting and following the agencys chain of
ANSWERS

Nursing Process, Implementation; Cognitive Level: Analysis; command. Answer 4 is incorrect because it is a temporary
Client Need/Subneed: Safe and Effective Care Environment/
measure and does not address the basic problem.
Management of Care/Establishing Priorities TEST-TAKING TIP: Look for the most comprehensive action
72. CORRECT ANSWER: 1. Answer 1 is correct because the that ensures client safety and protects all clients. The key word
priority is to maintain client safety. With syncope and dizzi- in Answer 3 is objective.
ness, the client is at high risk for falling. Answer 2 is incorrect Content Area: Management of Care, Advocacy; Integrated
because, although a current hemoglobin (Hgb) and hematocrit Process: Communication and Documentation; Cognitive Level:
(Hct) would be important data to assess for bleeding, these tests Analysis; Client Need/Subneed: Safe and Effective Care
are not the highest priority. Answer 3 is incorrect because sucral- Environment/Safety and Infection Control/Reporting of
fate (Carafate) is primarily prescribed for the short-term man- Incident/Event/Irregular Occurrence/Variance
agement of duodenal, not gastric, ulcers. Answer 4 is incorrect 75. CORRECT ANSWER: 3. Answer 1 is incorrect because
because, although vital signs are an important assessment, espe- the tracheostomy should not become dislodged if it is
cially with syncope and dizziness, safety is the priority in this secured with appropriately fitting tracheostomy ties. The
situation, followed by further assessment. obturator should always be at the bedside. Answer 2 is incor-
TEST-TAKING TIP: The key word is initial; safety is the rect because the purpose of deflating a tracheostomy cuff is
priority with a client complaining of syncope and vertigo. to alleviate pressure and wean the client from the tube. This
Content Area: Adult Health, Gastrointestinal; Integrated
assessment would be essential if the tracheostomy tube were
Process: Nursing Process, Implementation; Cognitive Level:
being removed. Answer 3 is correct because secretions may
Analysis; Client Need/Subneed: Safe and Effective Care have pooled above the tracheostomy cuff. If these are not
Environment/Management of Care/Establishing Priorities suctioned before deflation, the secretions may be aspirat-
73. CORRECT ANSWER: 3. Answer 1 is incorrect because ed. Answer 4 is incorrect because, although it is an impor-
increasing the oxygen to 6 L/min is unnecessary. The usual tant intervention for any client with respiratory difficulties,
rate of oxygen is 2 to 4 L/min to maintain the pulse oxime- deep breathing and coughing at this time may precipitate
ter reading at 96% to 98%. Answer 2 is incorrect because, additional movement of the tracheostomy tube and
although these levels are important data, the urgency of the bronchospasm.
clients situation is comfort and pain management while TEST-TAKING TIP: Identify the nursing concept (i.e., to
increasing coronary artery perfusion. Answer 3 is correct prevent aspiration). Oxygenation (Answers 2 and 4) and
because morphine sulfate is given for chest pain at a rate replacement of the tracheostomy tube (Answer 1) do not
of 1 mg/min repeated every 5 minutes as needed to block address this concept. All air passages should be free of secre-
pain perception. However, in myocardial ischemia it tions before deflating a tracheostomy cuff.
decreases anxiety and oxygen demand as it promotes Content Area: Adult Health, Respiratory; Integrated Process:
vasodilation. Answer 4 is incorrect because it is not the pri- Nursing Process, Implementation; Cognitive Level: Application;
ority. However, this test will probably be ordered if the Client Need/Subneed: Safe and Effective Care Environment/
clients symptoms suggest an extension of the infarction. Safety and Infection Control/Safe Use of Equipment
TEST-TAKING TIP: Look at the symptom in the stem (pain);
choose an option (analgesic) that relates to pain management.
2164_Ch04_115-248 29/03/12 12:22 PM Page 115

CHAPTER 4

Health Promotion
and Maintenance
Nursing Care of the Childbearing Family

Janice Lloyd McMillin

115
2164_Ch04_115-248 29/03/12 12:22 PM Page 116

116 chapter 4 Health Promotion and Maintenance

GROWTH C. Obstetric conjugatemeasured by x-ray (not


AND DEVELOPMENT commonly performed).
D. Tuber-ischial diameter9 to 11 cm indicates
Biological Foundations adequate size; evaluated by examiner.
of Reproduction III. FEMALE EXTERNAL ORGANS
A. Mons venerisprotects symphysis.
General overview: This review of the structures, functions, B. Labia majoracovers, protects labia minora.
and important assessment characteristics of the reproduc- C. Labia minoratwo located within labia majora.
tive system provides essential components of the database D. Clitorissmall erectile tissue.
required for accurate nursing judgments. Comparing nor- E. Hymenthin membrane at opening of vagina.
mal characteristics and established patterns with nursing F. Urinary meatusopening of urethra.
assessment findings assists in identifying client needs and G. Bartholin glandsproducers of alkaline secretions
in planning, implementing, and evaluating appropriate that enhance sperm motility, viability.
goal-directed nursing interventions.
IV. FEMALE INTERNAL REPRODUCTIVE
Female Reproductive Anatomy ORGANS (FIG. 4.2)
and Physiology A. Vaginaoutlet for menstrual flow, depository of
semen, lower birth canal.
I. THE FEMALE PELVIS (FIG. 4.1) B. Cervixcone-shaped neck of the uterus that
A. Two hip bones (right and left innominate: sacrum, protrudes into the vagina.
coccyx). C. Uterusmuscular organ that houses fetus during
MATERNAL/INFANT

B. False pelvisupper portion above brim, support- gestation.


ive structure for uterus during last half of D. Fallopian tubestwo tubes stretching from cornua
pregnancy. of uterus to ovaries; transport ovum.
C. True pelvisbelow brim; pelvic inlet, midplane, E. Ovariestwo oval-shaped structures that produce
pelvic outlet. Fetus passes through during birth. ovum and hormones (estrogen and progesterone).
II. PELVIC MEASUREMENTS F. Breaststwo mammary glands capable of secreting
A. Diagonal conjugate12.5 cm or greater is milk for infant nourishment.
adequate size; evaluated by examiner. V. MENSTRUAL CYCLE (FIG. 4.3)
B. Conjugate vera11 cm is adequate size; A. Reproductive hormones
can be measured by x-ray (not commonly 1. Follicle-stimulating hormone (FSH)secreted
performed). during the first half of cycle; stimulates develop-
ment of graafian follicle; secreted by anterior
pituitary.
2. Interstitial cell-stimulating hormone, luteinizing
hormone (ICSH, LH)stimulates ovulation
and development of corpus luteum; secreted by
pituitary.
3. Estrogenassists in ovarian follicle maturation;
stimulates endometrial thickening; responsible
for development of secondary sex characteristics;
maintains endometrium during pregnancy.
Secreted by ovaries and adrenal cortex during
cycle and by placenta during pregnancy.
4. Progesteroneaids in endometrial thickening;
facilitates secretory changes; maintains uterine
lining for implantation and early pregnancy;
relaxes smooth muscle. Secreted by corpus
luteum and placenta.
5. Prostaglandinssubstances produced by various
body organs that act hormonally on the
endometrium to influence the onset and contin-
uation of labor. A medication that may be used
to facilitate onset of second-trimester abortion;
also used to efface the cervix before induction of
Figure 4.1 The female pelvis. labor in term pregnancies.
2164_Ch04_115-248 29/03/12 12:22 PM Page 117

Growth and Development 117

Figure 4.2 Female internal


reproductive organs.

MATERNAL/INFANT
GRAAFIAN
FOLLICLE

SECONDARY CORPUS
FOLLICLE LUTEUM CORPUS
OVARIAN PRIMARY ALBICANS
CYCLE FOLLICLE

ENDOMETRIAL
CHANGES DURING
THE MENSTRUAL
CYCLE

MENSTRUAL FLOW

FUNCTIONAL
LAYER

BASILAR
LAYER Figure 4.3 The menstrual cycle. (From Venes, D
[ed]: Tabers Cyclopedic Medical Dictionary, ed 20. FA Davis,
DAYS 0 5 10 15 20 25 28 Philadelphia, 2005, p 1341.)

B. Ovulationmaturation and release of egg from and increase in the time between periods at end of
ovary; generally occurs 14 days before beginning of fertility cycle; average age is 51 to 52. Early
next menses. menopause rare but may be influenced by hypothy-
C. Menstruationvaginal discharge of blood and roidism, surgical ovarian removal, overexposure to
fragments of the endometrium; cyclic; occurs in radiation. Treatments during menopause for symp-
response to dropping levels of estrogen and tom relief: hormone replacement therapy, isofla-
progesterone. vanoids, vitamins B and E for hot flashes, vaginal
D. Fertilizationimpregnation of ovum by sperm. creams for dyspareunia (painful intercourse), and
E. Implantationfertilized ovum attaches to uterine calcium for osteoporosis. Alternative treatments
wall for growth. include herbal supplements and soy.
F. Menopausenormally occurring cessation of G. Spinnbarkeitstretchable, thin cervical mucus
menses with gradual decrease in amount of flow present at ovulation.
2164_Ch04_115-248 29/03/12 12:22 PM Page 118

118 chapter 4 Health Promotion and Maintenance

VI. ASSESSMENT OF REPRODUCTIVE


TRACT/REPRODUCTIVE HEALTH:
Decision Making Regarding
A. Health history Reproduction
1. Menarche: onset and duration. General overview: During the reproductive years, the
2. Menstrual problems. woman who is sexually active often faces the decision to
3. Contraceptive use. postpone, prevent, or terminate a pregnancy. The nursing
4. Pregnancy history. role focuses on assisting her to make an informed decision
5. Fertility problems. consistent with individual needs.
6. Lifestyle choices that may have an impact on
health and reproductive decision making. I. FAMILY PLANNING
B. Physical examination A. Assessment:
1. External, internal reproductive organs. 1. Determine interest in and present knowledge of
2. Breast examination. methods of family planning.
3. Mammographyevery 1 to 2 years for women 2. Identify factors affecting choice of method: cul-
beginning age 40; annually beginning age 50; tural and religious objections, contraindications
earlier and more frequently if have risk factors for individual methods, motivation/ability to
for breast cancer: follow chosen method successfully, financial
a. Mother, daughter, or sister had breast cancer. considerations, and sexual orientations.
b. Menses before age 11, or menopause before B. Analysis/nursing diagnosis: knowledge deficit
age 45 or after age 55. regarding family planning methods/options.
c. Previous benign needle biopsies. C. Nursing care plan/implementation: Goal: health
MATERNAL/INFANT

d. Birth of first child greater than 30 years old. teaching to facilitate informed decision making,
e. High-fat diet, obesity. selection of option appropriate to individual needs,
f. Obesity after menopause. desires.
g. Alcohol intake of 2 to 5 drinks per day. 1. Describe, explain, discuss options available and
h. Nulliparous. appropriate to the woman. Include information
i. Personal history of cancer. on advantages and disadvantages of each option
j. Defects in certain genes (BRCA1, (Tables 4.1 and 4.2).
BRCA2, HER). 2. Demonstrate, as necessary, method selected.
4. Pap smearFirst Papanicolaou (Pap) smear at 3. Quick health teaching reminders for missed oral
age 18 or earlier if sexually active; then annually hormone preparations:
until three consecutive normal Paps. After three a. Woman should take one pill at the same time
consecutive normal Paps, physician discretion is every day for 21 (or 28) days.
recommended. b. If woman misses one pill, she should take it as
5. Tests for sexually transmitted infections (STIs). soon as she remembers it; she should then
VII. ANALYSIS/NURSING DIAGNOSIS: take the next one at the usual time.
c. If woman misses two or more pills in a row in
A. Health-seeking behaviors related to health the first 2 weeks of her cycle, she should take
promotion. two pills for 2 days and use a backup method
B. Health-seeking behaviors related to menopause. of contraception for the next 7 days.
VIII. NURSING CARE PLAN/IMPLEMENTATION: d. If woman misses two pills in the third week,
A. Discuss anatomy and physiology of reproductive or three or more pills anytime:
tract. (1) A Sunday starter should keep taking pills
B. Review menstruation, ovulation, fertilization. until the next Sunday, then start a new
C. Explain need for periodic Pap smears, annual gyne- pack that Sunday. She should use a
cological examinations, including mammography. backup method of contraception for the
D. Discuss lifestyle choices and sexuality issues that next 7 days.
might affect health. (2) A day 1 starter should throw out the rest of
IX. EVALUATION/OUTCOME CRITERIA: the pack and start a new pack that day.
She should use a backup method of con-
A. Woman displays basic understanding of anatomy traception for the next 7 days.
and physiology. e. 28-day pill pack: If woman misses any of the
B. Woman understands cycle and contraception. seven pills that do not have any hormones,
C. Woman regularly seeks preventive care and per- she should throw out the pills missed and
forms monthly breast self-examination (BSE). (text continues on page 122)
2164_Ch04_115-248 29/03/12 12:22 PM Page 119

Decision Making Regarding Reproduction 119

Table 4.1
Contraception
Method Action/Effectiveness Advantages Disadvantages and Side Effects

Hormonal Contraceptives
Pillcombination of Suppresses ovulation by Convenient; easy to take Absolute contraindications (e.g.,
estrogen and suppressing production of Withdrawal bleeding cycles thromboembolic or coronary
progesterone FSH and LH are predictable artery disease; some cancers or
liver disease)
Oral contraceptives Most efficient form of Not related to sex act Relative contraindications
(daily) contraception (99.7%) if Safe for older nonsmoking (e.g., migraines, hypertension,
used consistently women until menopause immobility 4 weeks or more,
Many noncontraceptive abnormal genital bleeding)
health benefits (oral) Some decrease in glucose
tolerance
Effectiveness decreased if taken
during use of barbiturates,
phenytoin, antibiotics
No protection against STIs
Contraceptive Suppresses ovulation Convenient Absolute contraindications (e.g.,
patchcombination Thickens cervical mucus Patch applies to abdomen, thromboembolic or coronary

MATERNAL/INFANT
of estrogen and Decreases sperm penetration buttocks, upper arm, or artery disease, some cancers or
progesterone 99% effective in women upper torso liver disease)
(change q wk) <198 lb; 92% effective in Changed once/week Relative contraindications
women >198 lb (e.g., migraines, hypertension,
immobility 4 weeks or more,
allergic reaction to patch)
No protection against STIs
Mini-pillprogestin Impairs fertility Convenient, easy to take Ovulation may occur
only (PO daily) Thickens cervical mucus Irregular bleeding, mood
Decreases sperm penetration changes, weight gain
Alters endometrial maturation May change glucose and insulin
Effectiveness: undetermined; values
can reach 100% reliability if No protection against STIs
used exactly as prescribed
Depo-Provera Suppresses ovulation Private Requires injections every
synthetic Thickens cervical mucus Effective after 24 hours 3 months
progesterone Changes the uterine lining, Does not require Delay of return to fertility
(IM q 3 mo) making it harder for sperm to regular attention Possible weight gain and irregular
enter or survive in the uterus Does not interrupt sex play bleeding
Effectiveness: 99.7% Has no estrogen No protection against STIs
May decrease risk for ovar-
ian and uterine cancers

Emergency Postcoital Contraception


Estradiol (100 mcg) Antifertility; taken within Available prn Nausea, headache, dizziness
and levonorgestrel 72 hours of unprotected sex
(0.5 mg) Take as soon as possible;
repeat 12 hours later
Effectiveness: 75%85%
Intrauterine Devices (IUDs)
Small, T-shaped, Prevents fertility Can be used by women Contraindications: history of PID,
medicated device who cannot use hormonal pregnancy, undiagnosed genital
inserted into uterine contraception; no disrup- bleeding, genital malignancy,
cavity tion of ovulation pattern abnormal uterine cavity, severe
cervicitis, HIV/AIDS, history of
ectopic pregnancy, history of
toxic shock
Continued
2164_Ch04_115-248 29/03/12 12:22 PM Page 120

120 chapter 4 Health Promotion and Maintenance

Table 4.1
Contraceptioncontd
Method Action/Effectiveness Advantages Disadvantages and Side Effects
Copper (Paragard) Recommended for women Can be used effectively for Risks: uterine perforation, infec-
who have had at least one 10 years tion (may be followed by PID) in
child the first 3 months of insertion;
Damages sperm in transit to unnoticed expulsion
fallopian tube Side effects (especially with
Paragard): heavy flow, spotting
between periods, and cramping
within first few months of insertion
Must check for string after each
menses and before intercourse
Progesterone Alters cervical mucus and Not necessary to change No protection against STIs
(Mirena) endometrial maturation every year. Can be used up
Effectiveness: 90%99% to 5 years.
Less blood loss during
menses and decreases
primary dysmenorrhea
Mechanical Barriers
Diaphragm: shallow Barrier preventing sperm Does not interrupt sex act, Requires careful cleansing with
MATERNAL/INFANT

rubber device that from entering cervix (if it is except to add spermicide warm water and mild soap;
fits over cervix correct size, undamaged, just before act* powder with cornstarch, and
correctly placed, and used Insert up to 6 hours before store away from heat.
with spermicide) intercourse and leave in Use might be associated with
Effectiveness: 83%90%; place for 6 hours after last TSS
99% in highly motivated intercourse, but not longer Size/fit must be checked after
women than 24 hours birth, second/third-trimester
Safe: no side effects from abortion, weight gain or loss of
well-fitted device if woman 1520 lb or more, or every
is not allergic to diaphragm 2 years
or spermicide Spermicide must be reinserted
Decreased incidence of for additional acts that may follow
vaginitis, cervicitis, PID after initial intercourse
Cervical cap: Physical barrier to sperm Worn for 8 hours but not Need a Pap smear every year:
11/411/2-inch soft Spermicide inside cap adds longer than 48 hours higher rate of conversion from
natural rubber dome a chemical barrier No need to add spermicide class I to class III
with a firm but Effectiveness depends on for repeated acts of If in place over 48 hours, it
pliable rim its fit. intercourse produces an odor and might be
80%91% effective for associated with TSS
nulliparas Cannot be worn during menstrual
60%74% effective in flow (menses) or up to at least
multiparas 6 weeks postpartum
Contraindications: abnormal Pap
smear, hard to fit, history of TSS
or genital infection, allergy
Change after genital surgery,
abortion or birth, major change in
weight
Must be checked each year
Does not protect against STIs
Female condom: Barrier preventing sperm Apply up to 8 hours in Cost
vaginal sheath of from entering vagina advance of intercourse; A new one must be used for
natural latex rubber Effectiveness similar to other spermicide added just every act of intercourse
with flexible mechanical methods used before intercourse
rings at both the with spermicide (79%95%) Heightens sensation for man
closed and the Note: Male and female con- About as satisfying for both
open ends doms should not be used at woman and man as inter-
the same time course without it
Provides protection from
STIs
2164_Ch04_115-248 29/03/12 12:22 PM Page 121

Decision Making Regarding Reproduction 121

Table 4.1
Contraceptioncontd
Method Action/Effectiveness Advantages Disadvantages and Side Effects
Condom: thin, Barrier preventing sperm Safetyno side effects Check expiration date
stretchable latex from entering vagina, applied Provides protection from Requires high motivation to use
sheath to cover penis over erect penis before loss spread of STIs correctly/consistently
of pre-ejaculatory drops and With spermicide (0.5 gm Must be properly applied and
is held in place as penis is of nonoxynol-9) added removed
withdrawn to interior or exterior Sheath may tear during
Spermicidal foam, surface, provides protection intercourse
jelly, or cream is also from STIs, including HIV Can have small undetectable
used* holes
Effectiveness rate: 64%98%
when used with spermicide
Chemical Barriers
Spermicide: Physical barrier to sperm Increases effectiveness of Messy
aerosol foams, penetration mechanical barriers Some people are allergic to
foaming tablets, Chemical action on sperm Ease of application. Aids preparations
suppositories, creams, (kills sperm) lubrication of vagina* Tabs or suppositories take
and films (VCF) Nonoxynol-9 has Requires no medical exam- 1015 minutes to dissolve
a bacteriostatic ination or prescription If it is only method being used,
action May be used during each intercourse should be

MATERNAL/INFANT
Effectiveness rate: 70%98% lactation preceded (by 30 minutes) by a
when used with diaphragm Backup for missed oral fresh application; may be
or condoms contraceptive pills allergenic
Other Methods
Natural family Requires sexual abstinence Physically safe to useno Effectiveness depends on high
planning: basal body during womans fertile period drugs or appliances are level of motivation and diligence
temperature (BBT) (4 days before ovulation), used; meets requirements Requires fairly predictable men-
each morning before and for 3 or 4 days after of most religions strual cycle
any physical activity ovulation
Symptothermal Effectiveness: about 80%
variation: BBT plus
cervical mucus
changes
Calendar method
Predictor test for
ovulation
*Spermicide provides lubrication, but if additional lubrication is needed, use water-based products only (e.g., K-Y Jelly).
TSS: toxic shock syndrome. Although there is no direct link between TSS and use of the diaphragm or cervical cap, a possible association remains (see p. 122).
Class I Pap smear: no abnormal cells; class III Pap smear: suspicious abnormal cells present.

Table 4.2
Sterilization
Method Advantages Disadvantages and Side Effects
Men
Vas deferens is occluded Relatively simple surgical procedure Sterility is not immediate; sperm are cleared from vas
(ligated and severed; after about 15 ejaculations; need to have follow-up
bands; clips) to prevent semen analysis to confirm sterility
passage of sperm Does not affect endocrine function, Antibody produced to own sperm
production of testosterone
Does not alter volume of ejaculate Some men become impotent due to psychological
response to procedure
Tubal reconstruction usually Fertility after tubal reconstruction (30%85%)
possible (90%)
Does not protect against STIs
Continued
2164_Ch04_115-248 29/03/12 12:22 PM Page 122

122 chapter 4 Health Promotion and Maintenance

Table 4.2
Sterilizationcontd
Method Advantages Disadvantages and Side Effects
Women
Both fallopian tubes are Abdominal surgery utilizing 1-inch Major surgery (if done by laparotomy) with possible
ligated and severed, or incision and laparoscopy complications of anesthesia, infection, hemorrhage, and
occluded with bands or trauma to other organs; psychological trauma in some
clips to prevent passage Greater than 99.5% effective Success rate for pregnancy after tubal reversal using
of eggs; fulguration of the microsurgical techniques: 40%75%
tubes at the cornu is
most effective
Hysterectomy or Abdominal or vaginal surgery
oophorectomy or both Absolute sterility

keep taking one pill a day until the pack is (4) Sore throat; severe nausea, vomiting.
empty. She does not need a backup method (5) Copious vaginal discharge.
of contraception. b. Instructions for prevention:
4. Alert woman to discontinue use of oral hormone (1) General
contraceptive preparations and report any of the (a) Avoid use of tampons, cervical caps,
MATERNAL/INFANT

following signs of potential problems to the and diaphragms during the postpartum
physician STAT: ACHES* period (6 weeks).
AAbdominal pain: possible problem with the (b) Do not use any of the above if you have
liver or gallbladder. a history of TSS.
CChest pain or shortness of breath: possible (c) Call physician if you experience
clot within lungs or heart. sudden onset of a high fever, vomit-
HHeadaches (sudden or persistent): possibly ing, diarrhea, or skin rash.
caused by stroke (brain attack) or hypertension. (d) Insert clean tampons and contraceptive
EEye problems: possible vascular incident or devices with clean hands.
hypertension. (e) Remove within prescribed time limits.
SSevere leg pain: possible thromboembolic (2) Tampons
process. (a) Change tampons every 3 to 6 hours.
5. Signs of potential problems related to IUD use: (b) Do not use superabsorbent tampons.
PAINS* (c) For overnight protection, substitute
PPeriod (menstrual) late, abnormal spotting other products such as sanitary napkins
or bleeding. or minipads.
AAbdominal pain, pain during coitus (3) Diaphragm or cervical cap
(dyspareunia). (a) Avoid use during your menstrual
IInfection, abnormal vaginal discharge. period.
NNot feeling well; fever or chills. (b) Remove within 8 hours after inter-
SString missing (nonpalpable on vaginal course (diaphragm must be removed no
self-examination, or not seen on speculum later than 24 hours; the cap, no later
examination). than 48 hours).
6. Toxic Shock Syndrome (TSS) D. Evaluation/outcome criteria:
a. Signs/symptoms: 1. Woman avoids or achieves a pregnancy as
(1) Fever of sudden onsetover 102F desired.
(38.9C). 2. Woman expresses comfort and satisfaction with
(2)Hypotensionsystolic pressure method selected.
90 mm Hg; orthostatic dizziness; II. INFERTILITY
disorientation.
A. Definition: inability to conceive after 1 year of
(3) Rashdiffuse, macular erythroderma
unprotected intercourse.
(resembling sunburn).
B. Pathophysiology: contributing factorshormonal
deficiencies, reproductive system disorders, congen-
ital anomalies, male impotence, sexual knowledge
*Source: Hatcher, R, et al: Contraceptive Technology, ed. 16. Irving, New York. deficit, debilitating disease.
2164_Ch04_115-248 29/03/12 12:22 PM Page 123

Decision Making Regarding Reproduction 123


C. Assessment: F. Evaluation/outcome criteria:
1. Historygeneral health, reproduction, social 1. The couple conceives, or,
history. 2. If the couple does not conceive, they accept
2. Maternal diagnosis referral for help with adoption, other reproduc-
a. Basal body temperature (BBT) (Fig. 4.4). tive alternatives, or childlessness.
b. Endocrine studies. III. INTERRUPTION OF PREGNANCYalso
c. Huhner test (postcoital). known as elective, voluntary, or therapeutic abortion.
d. Rubins test (tubal patency). Once the diagnosis of pregnancy and the length of
e. Hysterosalpingogram (tubal patency). gestation are established, the woman faces the decision
3. Male diagnosishistory, physical examination, to interrupt or to maintain the pregnancy (Table 4.3).
laboratory studies (e.g., semen analysis).
A. Decision-making stage:
D. Analysis/nursing diagnosis: altered sexuality,
1. Assessment:
altered family process related to infertility.
a. Health history:
E. Nursing care plan/implementation:
(1) Determine womans feelings about the
1. Provide emotional support.
pregnancy, reasons for considering abor-
2. Explain testing procedures for diagnosis.
tion, level of maturity; if decision was
3. Assist with referral process.

BASAL BODY TEMPERATURE CHART


MONTH
Date

MATERNAL/INFANT
Day of 1 2 3 4 5 6 7 8 9 10 1112 13 1415 16 17 18 19 20 21 22 2324 2526 27 28 29 30 31 32 33 34 35 36 37 38 39 40 41 42
Cycle
.8
.7
.6
.5
MENSES
.4
.3
.2 IF WOMAN IS NOT
PREGNANT, PERIOD
TEMPERATURE IN FAHRENHEIT

.1
99.0 WOULD HAVE
STARTED BY NOW
.9
AND BASAL
.8
TEMPERATURE
.7 WOULD HAVE FALLEN
.6
.5
.4
.3
.2
.1
98.0 IN THIS WOMAN'S
.9 CHART, CONCEPTION
.8 OCCURRED 28 DAYS
AGO; AND SHE IS
.7 PREGNANT
.6
.5 OVULATION
.4
.3
.2
.1
Figure 4.4 Basal body temperature chart.
97.0 (From Venes, D [ed]: Tabers Cyclopedic Medical Dictionary,
ed 20. FA Davis, Philadelphia, 2005.)

Table 4.3
Interruption of Pregnancy (Elective/Voluntary Abortion)
Method Advantages Disadvantages and Side Effects
First-Trimester Procedures
Early uterine evacuation (EUE)aspiration Performed for women who have not Cervical trauma may occur and may
of endometrium through undilated cervix yet missed a menstrual period lead to incompetent cervix
100% effective if implantation site is Hemorrhage
not missed
RU 486 (mifepristone)a progesterone Prevents implantation of fertilized egg Slight nausea and fatigue during
antagonist; taken up to 7 weeks Most effective in early gestation, dur- period of bleeding
gestation (56 days LMP); taken orally ing luteal phase, within 10 days of first Uterine aspiration may be needed if
missed period RU 486 does not work
Misoprostol (Cytotec)initiates Softens cervix and aids in expelling
uterine contractions; given intravaginally products of conception

Continued
2164_Ch04_115-248 29/03/12 12:22 PM Page 124

124 chapter 4 Health Promotion and Maintenance

Table 4.3
Interruption of Pregnancy (Elective/Voluntary Abortion)contd
Method Advantages Disadvantages and Side Effects
Uterine aspiration (vacuum or suction Relatively few complicationsminimal Performed after one or two missed
curettage)cannula suction under local bleeding, minimal discomfort menstrual periods
anesthesia, following cervical dilatation, Outpatient procedure Cervical or endometrial trauma
usually with laminaria tent possible
Surgical dilation and curettage (D & C) After cervix is dilated, procedure takes Performed after one or two missed
cervix dilated with laminaria tents; about 15 minutes periods
endometrium scraped with metal curette Outpatient procedure
or flexible aspiration tip, under local Relatively few complications (1%): Possible but rare; cervical trauma,
anesthesia (paracervical block) bleeding like a heavy period, some uterine perforation infection,
cramping hemorrhage
Second-Trimester Procedures
D & E (dilation and evacuation)extends D Woman does not experience labor Requires 3 days for laminaria to dilate
& C and vacuum curettage up to 20 weeks cervix; procedure done on third day
gestation
Second- and Third-Trimester Procedures
Hysterotomycesarean delivery Available for gestations more than After: major surgery complications
1416 weeks hemorrhage and infection possible
Preferred method if woman wishes a Fetus may be born alive, causing
MATERNAL/INFANT

tubal ligation or hysterectomy to ethical, moral, religious, and legal


follow problems

already made before she came to clinic, (2) Encourage verbalization of feelings,
how was decision made? Does she have a perceptions, and values.
support system? (3) Support womans decision.
(2) Identify factors influencing/complicating b. Goal: health teaching to facilitate informed
her decisions (religious beliefs, cultural decision making.
mores, peer and family pressures). (1) Explain and discuss available options as
(3) Information needs. applicable (see Table 4.3).
b. Physical examination. (2) Describe procedure selected and what to
c. Laboratory tests: blood type, Rh, hemoglobin, expect after procedure.
hematocrit, urinalysis, pregnancy test, anti- c. Goal: minimize impact on intrafamily rela-
body titer, other tests dependent on her tions, family process. Where applicable,
health status. encourage open communication between
2. Analysis/nursing diagnosis: deciding partners.
a. Ineffective coping related to emotional conflicts 4. Evaluation/outcome criteria:
associated with need for decision to continue/ a. Woman states she understands all information
terminate pregnancy. necessary to give consent.
b. Altered family process related to intrafamily b. Woman expresses comfort and satisfaction
conflict associated with need for/decision to with the decision.
continue/terminate pregnancy. B. Preoperative period
c. Anticipatory grieving related to loss of 1. Assessment:
pregnancy/child. a. Reassess womans emotional and physical
d. Altered self-concept, self-esteem disturbance status and current feelings regarding
related to possible guilt feelings associated decision.
with pregnancy/termination. b. Determine womans current knowledge/
e. Knowledge deficit related to available options. understanding of authorization form,
3. Nursing care plan/implementation: anticipated procedure, and consequences
a. Goal: emotional support to minimize impact (informed consent).
on self-image and self-esteem. c. Monitor womans physiological and
(1) Maintain accepting, nonjudgmental (if awake) psychological response to
attitude. procedure.
2164_Ch04_115-248 29/03/12 12:22 PM Page 125

Childbearing: Pregnancy by Trimester 125


2. Analysis/nursing diagnosis: (b) Administer RhoGAM, as ordered.
a. Anxiety/fear related to procedure, potential (5) Provide and explain perineal care.
complications. b. Goal: health teaching to facilitate active partici-
b. Knowledge deficit related to ongoing pation in own health maintenance, informed
procedure, sights, sounds, and sensations decision making, provide predischarge antici-
experienced. patory guidance (also provide in written form
3. Nursing care plan/implementation: with attention to womans level of reading skill
a. Goal: provide opportunity to reconsider decision and understanding, and in her native language
regarding termination of pregnancy. whenever possible):
(1) Check to ensure all required permission (1) Immediately report any cramping,
(informed consent) forms have been excessive bleeding, signs of infection.
signed/filed. (2) Provide name and telephone number of
(2) Refer to physician if woman is ambivalent person to call if she has questions.
or insecure in decision. (3) Schedule a postabortal checkup.
b. Goal: reduce anxiety/fear related to procedure. (4) Discuss contraception, if woman indicates
(1) Explain all anticipated preoperative, interest; or give her place and name to call
operative, and postoperative care. for information later.
(2) Assist with procedure; if woman is awake, (5) Discuss resumption of tampon use
explain what is happening and what she (3 days to 3 weeks as ordered) and sexual
may be experiencing. intercourse (1 to 3 weeks as ordered).
c. Goal: emotional support to facilitate effective (6) Discuss need to avoid douching.

MATERNAL/INFANT
coping. 4. Evaluation/outcome criteria:
(1) Encourage verbalization of feelings, fears, a. Woman returns for postabortal appointment.
concerns. b. Woman suffers no adverse physical sequelae
(2) Support womans decision. to the procedure.
4. Evaluation/outcome criteria: woman does not c. Woman suffers no adverse psychological
experience physiological or psychological sequelae to the procedure.
problems during procedure. d. Woman is successful in achieving her goal of
C. Postoperative period either contraception or conception at the time
1. Assessment: she desires.
a. Monitor physiological response to procedure 5. Postabortion psychological impact:
(vital signs, blood loss, uterine cramping). a. Majorityrelieved and happy.
b. Determine psychological response (happy, b. Small number (5% to 10%)negative
relieved; guilt feelings, lowered self-esteem). feelings, such as guilt or low self-esteem.
c. Determine desire for family planning
information.
d. Determine need for Rho(D) immune globu-
CHILDBEARING:
lin, rubella vaccination
PREGNANCY BY
2. Analysis/nursing diagnosis:
TRIMESTER
a. Pain related to procedure. General overview: This review of the normal physiological
b. High risk for infection related to lack of and psychosocial changes occurring during each trimester
knowledge of postabortal self-care. of pregnancy provides essential components of the data-
3. Nursing care plan/implementation: base for accurate nursing judgments and anticipatory
a. Goal: provide and explain postoperative care. guidance during the prenatal period. Complications of
(1) Administer intravenous (IV) fluids. pregnancy are correlated with the trimester of common
(2)Administer medications prn for occurrence; relationships with other NCLEX categories
discomfort. of human function are described.
(3) Administer oxytocic medications for uter- I. GENERAL ASPECTS OF NURSING CARE
ine atony, prn.
A. Assessmentbased on nursing knowledge of the
(4) If mother is Rh-negative, 8 or more weeks
following:
of gestation, and laboratory tests indicate
1. Biophysical and psychosocial aspects of concep-
no current sensitization (i.e., she is
tion and gestation.
Coombs negative):
2. Parameters of normal pregnancy.
(a) Explain rationale for postabortion
3. Risk factors, signs, symptoms, and implications
administration of Rho (D antigen)
of deviations from normal patterns of maternal
immune globulin (RhoGAM).
and fetal health.
2164_Ch04_115-248 29/03/12 12:22 PM Page 126

126 chapter 4 Health Promotion and Maintenance

B. Analysis/nursing diagnosis: 7. Fetusperiod from end of embryo stage until


1. Knowledge deficit related to normal pregnancy- birth.
related alterations (physiological and emotional First Trimester
alterations per trimester). Susceptible to teratogens
2. Pain related to normal physiological alterations Heart functions at 34 wk
in pregnancy. Eye formation at 45 wk
3. Altered elimination related to normal Arm and leg buds at 45 wk
physiological changes during pregnancy Recognizable face at 8 wk
(polyuria, constipation). Brain: rapid growth
4. Altered nutrition related to increased metabolic External genitalia at 8 wk
needs due to pregnancy. Placenta formed at 12 wk
5. Impaired adjustment related to altered self-image; Bone ossification at 12 wk
anticipated role change; resurgence of old, Second Trimester
unresolved conflicts. Less danger from teratogens after 12 wk
C. Nursing care plan/implementation: Facial features formed at 16 wk
1. Goal: emotional support. Fetal heartbeat heard by 1820 wk; with a
a. Encourage verbalization of feelings, fears, fetoscope/Doppler at 1012 wk
concerns. Quickening at 1820 wk
b. Validate normalcy of behavioral response to Length: 10 inches, weight: 810 oz
pregnancy. Vernix: present
2. Goal: anticipatory guidance. Third Trimester
MATERNAL/INFANT

a. Facilitate achievement of developmental tasks. Iron stored


b. Strengthen coping techniques for pregnancy, Surfactant production begins in increasing
labor, birth. Suggest appropriate resources amounts
(preparation for childbirth classes). Size: 15 inches, 23 lb
3. Goal: health teaching. Describe, explain, discuss: Calcium stored at 2832 wk
a. Normal physiological alterations during Reflexes present at 2832 wk
pregnancy. Subcutaneous fat deposits at 36 wk
b. Common discomforts of pregnancy, Lanugo shedding at 3840 wk
management. Average size: 1822 inches, 7.58.5 lb at
D. Evaluation/outcome criteria: 3840 wk
1. Woman takes an active, informed part in her B. Anatomical and physiological modifications
pregnancy-related care. 1. Bases of functional alterations
2. Woman copes effectively with common alter- a. HormonalTable 4.4 discusses the effects of
ations associated with pregnancy (physiological, estrogen and progesterone during pregnancy.
psychological, role change). Nursing implications provide the knowledge
3. Woman successfully carries an uneventful base for the following:
pregnancy to term. (1) Anticipatory guidance regarding normal
II. BIOLOGICAL FOUNDATIONS maternal adaptations.
OF PREGNANCY (2) Early identification of deviations from
normal patterns.
A. Conception
b. Mechanicalenlarging uterus displace-
1. Egglife span, approximately 24 hours after
ment and pressure; increased weight of
ovulation.
uterus and breasts changes in posture
2. Spermlife span, approximately 72 hours after
and pressure.
ejaculation into female reproductive tract.
2. Breastsenlarged darkened areolae; secrete
3. Conception (fertilization)usually occurs 12 to
colostrum.
24 hours after ovulation, within fallopian tube.
3. Reproductive organs
4. Implantation (nidation)usually occurs within
a. Uterus
7 to 9 days of conception, or about day 21 to 23
(1) Amenorrhea. Occasional spotting com-
of a 28-day menstrual cycle.
mon, especially at time of first missed
5. Ovumperiod of conception until primary villi
menstrual period.
have appeared; usually about 12 to 14 days.
(2) Increased vascularity adds to increase in
6. Embryoperiod from end of ovum stage until
size and softening of the lower uterine
measurement reaches approximately 3 cm; 54 to
segment (Hegars sign).
56 days.
2164_Ch04_115-248 29/03/12 12:22 PM Page 127

Biological Foundations of Pregnancy 127

Table 4.4
Hormones of Pregnancy
Primary Effects Clinical Implications for Nursing Actions
Estrogen
Level rises in serum and urine Basis of test for maternal/placental/fetal well-being
Uterine enlargement Probable sign of pregnancy
Breast enlargement Probable sign of pregnancy; increased tingling, tenderness
Genital enlargement: increased vascularization, hyperplasia Vaginal growth facilitates vaginal birth
Softens connective tissue Results in backache and leg ache; relaxes joints to increase size of
birth canal and rib cage
Alters nutrient metabolism: Gastrointestinal and metabolic changes:
Decreases HCl and pepsin Digestive upsets
Antagonist to insulinmakes glucose available to fetus Anti-insulin effect challenges maternal pancreas to produce more
insulin; failure of beta cells to respond leads to gestational dia-
betes. For the woman who is insulin dependent, insulin requirements
increase by an average of 67% during the second half of pregnancy
Supports fat deposition Protect source of energy for fetus
Sodium and water retention; edema of lower extremities Meet increased plasma volume needs and maintain fluid

MATERNAL/INFANT
(nonpitting) reserve
Hematological changes:
Increased coagulability Increased tendency to thrombosis
Increased sedimentation rate (SR) SR loses diagnostic value for heart disease
Vasodilation: spider nevi; palmar erythema Resolves spontaneously after birth
Increased production of melanin-stimulating hormone Resolves spontaneously after birth; causes chloasma and linea nigra
Progesterone
Development of decidua High levels result in tiredness, listlessness, and sleepiness
Reduces uterine excitability Protection against abortion/early birth (i.e., maintains pregnancy)
Development of mammary glands Prepares breasts for lactation
Alters nutrient metabolism: Nutritional significance:
Antagonist to insulin Diabetogenic
Favors fat deposition Energy reserve
Decreases gastric motility and relaxes sphincters Favors heartburn and constipation
Increased sensitivity of respiratory center to CO2 Increased depth, some dyspnea, increased sighing
Decreased smooth-muscle tone: Decreased tone can lead to:
Colon Constipation
Bladder, ureters Stasis of urine with chance of infection
Veins Dependent edema; varicosities
Gallbladder Gallbladder disease
Increased basal body temperature (BBT) by 0.5C Discomfort from hot flashes and perspiration
Human Chorionic Gonadotropin
Maintains corpus luteum during early pregnancy Placenta must take over after a few weeks
Stimulates male testes Increased testosterone in male fetuses
May suppress immune response May inhibit response to foreign protein (e.g., fetal portion of placenta)
Diagnostic value:
Basis for pregnancy test
Decreased level with threatened abortion
Increased level with multiple pregnancy
Very high level with hydatidiform mole
Continued
2164_Ch04_115-248 29/03/12 12:22 PM Page 128

128 chapter 4 Health Promotion and Maintenance

Table 4.4
Hormones of Pregnancycontd
Primary Effects Clinical Implications for Nursing Actions
Human Placental Lactogen
Antagonizes insulin Diabetogenic; may gestational diabetes or complicate manage-
ment of existing diabetes
Mobilizes maternal free fatty acids Increased tendency of ketoacidosis in pregnant diabetic
Prolactin
Suppressed by estrogen and progesterone No milk produced before birth
Increased level after placenta is delivered Milk production (lactation) 23 days after birth
Follicle-Stimulating Hormone
Production suppressed during pregnancy; level returns to No ovulation during pregnancy; ovulation usually returns within
prepregnant levels within 3 weeks after birth 6 weeks for 15%, within 12 weeks for 30%
Oxytocin
Causes uterus to contract when the oxytocin levels exceed Labor induction or augmentation; treatment for postpartum uterine
those of estrogen and progesterone atony
MATERNAL/INFANT

(3) Growth is due to hypertrophy and e. Ovaries


hyperplasia of existing muscle cells and (1) Ovum production ceases.
connective tissue. (2) Corpus luteum persists; produces hor-
(4) Fundal height measurement landmarks: mones to weeks 10 to 12 until placenta
takes over.
C. Alterations affecting fluid-gas transport
Uterus Nonpregnant Pregnant (at Term) 1. Cardiovascular system (Table 4.5)
Length 6.5 cm 32 cm a. Physiological changes
Width 4 cm 24 cm (1) Heart displaced upward and to the left.
Depth 2.5 cm 22 cm
(2) Circulation:
(a) Cardiac volume increases by 20%
Weight 50 gm 1000 gm to 30%.
(b) Laborcardiac output increases by
20% to 30%.
b. Cervix (3) Hemoglobin and hematocrit values remain
(1) Increased vascularity softening between 10 and 14 gm and 35% and
(Goodells sign) and deepened blue-purple 42%; normal drop is 10% during second
coloration (Chadwicks sign). trimester.
(2) Edema, hyperplasia, thickening of mucous (4) Hypercoagulabilityincreased levels of
lining, and increased mucus production; blood factors VII, VIII, IX, and X.
formation of mucous plug by end of (5) Nonpathological increased sedimentation
second month. ratedue to 50% increase in fibrinogen
(3) Becomes shorter, thicker, and more elastic. level.
c. Vagina (6) Blood pressure should remain stable with
(1) Hyperemia deepens color (Chadwicks sign). drop in second trimester.
(2) Hypertrophy and hyperplasia thicken (7) Heart rate often increases 10 to 15 beats/min
vaginal mucosa. at term.
(3) Relaxation of connective tissue. (8) Compression of pelvic veins stasis of
(4) pH acidic (4.06.0). blood in lower extremities.
(5) Leukorrheanonirritating. (9) Compression of inferior vena cava when
d. Perineum supine bradycardia reduced cardiac
(1) Increases in sizehypertrophy of muscle output, faintness, sweating, nausea (supine
cells, edema, and relaxation of elastic tissue. hypotension). Fetal response: marked brady-
(2) Deepened colorincreased vascularization/ cardia due to hypoxia secondary to
hyperemia. decreased placental perfusion.
2164_Ch04_115-248 29/03/12 12:22 PM Page 129

Common Discomforts During Pregnancy 129

Table 4.5
Blood Values
Component Prepregnant Pregnant Postpartum*
WBC 400011,000 900016,000 (); 20,00025,000 within 1012 days of birth, then returns to normal
25,000labor
RBC volume 1600 mL 1900 mL Prepregnant level of 1600 mL
Plasma volume 2400 mL 3700 mL Prepregnant level of 2400 mL
Hct (PCV) 37%47% 32%42% () At 72, returns to prepregnant level of 37%47%
Hgb (at sea level) 1216 g/dL 1014 g/dL At 72, returns to prepregnant level of 1216 g/dL
Fibrinogen 250 mg/dL 400 mg/dL At 72, returns to prepregnant level of 250 mg/dL
*Postpartum values depend on factors of amount of blood loss, mobilization, and physiological edema (excretion of extravascular water). Normal blood loss for vagi-
nal birth is 300 to 400 mL.

b. Assessment: 2. Respiratory system


(1) Apical systolic murmur. a. Physiological changes:
(2) Exaggerated splitting of first heart sound. (1) Increased: tidal volume, vital capacity,
(3) Physiological anemia. respiratory reserve, oxygen consumption,
(4) Dependent edema in third trimester production of CO2.

MATERNAL/INFANT
(Table 4.6). (2) Diaphragm elevated, increased substernal
(5) Vena cava syndrome (supine hypotension) angle flaring of rib cage.
drop in systolic blood pressure may occur (3) Uterine enlargement prevents maximum
due to compression of descending aorta lung expansion in third trimester.
and inferior vena cava when supine. b. Assessment:
(6) Varicosities (vulvar, anal, leg). (1)Shortness of breath or dyspnea on
c. Nursing care plan/implementation: Goal: exertion and when lying flat in third
health teaching. trimester.
(1) Elevate lower extremities frequently. (2) Nasal stuffiness due to estrogen-induced
(2) Apply support hose. edema (see Table 4.6).
(3) Avoid excess intake of sodium. (3) Deeper respiratory excursion.
(4) Assume side-lying position at rest. c. Nursing care plan/implementation: Goal:
(5) Learn signs and symptoms of pregnancy- health teaching.
induced hypertension. (1) Sit and stand with good posture.

Table 4.6
Common Discomforts During Pregnancy
Discomfort and Cause Health Teaching
Morning sicknessfirst 3 months; nausea and vomiting; Alternate dry carbohydrates and fluids hourly; take dry
may occur anytime, day or night; cause: hormonal, carbohydrates before rising, stay in bed 15 more minutes;
psychological, and empty stomach avoid empty stomach, offending odors, and food difficult to
digest (e.g., food high in fat); avoid acidic foods (e.g., citrus)
Fatigue (sleep hunger)first 3 months; cause: possibly Iron supplement if anemicfoods high in iron, folic acid,
hormones; often returns in late pregnancy when physical and protein; adequate rest
load is great
Fainting (syncope)early pregnancy, due to slightly Elevate feet; sit down when necessary; when standing, do
decreased arterial blood pressure; late pregnancy, due not lock knees; avoid prolonged standing, fasting
to venous stasis in lower extremities
Urinary frequencyenlarging uterus presses on bladder, Kegel exercises; limit fluids just before bedtime to ensure rest; rule
turgescence of structures from hormone stimulation; out urinary tract infection
relieved somewhat as uterus rises from pelvis; recurs
with lightening
Continued
2164_Ch04_115-248 29/03/12 12:22 PM Page 130

130 chapter 4 Health Promotion and Maintenance

Table 4.6
Common Discomforts During Pregnancycontd
Discomfort and Cause Health Teaching
Vaginal dischargemonths 29; mucus less acidic, and Cleanliness important; treat only if infection sets in; douche con-
increases in amount (leukorrhea) traindicated in pregnancy
Hot flashesheat intolerance, due to increased Alter clothing, bathing, and environmental temperature prn
metabolism diaphoresis
Headachecause unknown; possibly blood pressure If pain relief needed, consult physician (avoid over-the-counter
change, nutritional, tension (unless associated with drugs without prescription); reduce tension
preeclampsia)
Nasal stuffinessdue to increased vascularization; Antihistamines and nasal sprays by prescription only
allergic rhinitis of pregnancy
Heartburnenlarging uterus and hormones slow Physician may prescribe an antacid; avoid use of antacids
digestion; progesterone reverse peristaltic waves containing sodium; instead of leaning over, bend at the
reflux of stomach contents into esophagus knees, keeping torso straight; sit on firm chairs; limit fatty
and fried foods; small, frequent meals
Flatulencealtered digestion from enlarging uterus and Maintain regular bowel habits, avoid gas-forming foods;
hormones antiflatulent may be prescribed
MATERNAL/INFANT

Insomniafetal movements, fears or concerns, and Medication by prescription only; exercise; side-lying
general body discomfort from heavy uterus positions with pillow supports; change position often; back
rubs, ventilate feelings
Shortness of breathenlarging uterus limits expansion Good posture; cut down/stop smoking; positionsupine
of diaphragm and upright

Backacheincreased elasticity of connective tissue, Correct posture, low-heeled, wide-base shoes, and abdominal
increased weight of uterus, and increased lumbar support (binders); do pelvic rock often; avoid fatigue
curvature
Pelvic joint painhormones relax connective tissue and Rest; good posture; will go away after giving birth, in
joints and allow movement within joints 68 weeks; avoid prolonged standing/walking
Leg crampspressure of enlarging uterus on nerve Stretch affected muscle and hold until it subsides; do
supplying legs; possible causes: lack of calcium, fatigue, not rub (may release a blood clot, if present); calcium
chilling, and tension intake
Constipationdecreased motility (hormones, enlarging Dietprunes, fruits, vegetables, roughage, and fluids;
uterus) and increased reabsorption of water; iron regular habits; exercise; sit on toilet with knees up; avoid
therapy (oral) enemas, mineral oil, laxatives
Hemorrhoidsvaricosities around anus; aggravated by As above, avoid constipation; pure Vaseline or Desitin is mild and
pushing with stool and by uterus pressing on blood sometimes soothing; use any other preparation with prescription
vessels supplying lower body only
Ankle edemanormal and nonpitting; gravity Rest legs often during day with legs and hips raised

Varicose veinslower legs, vulva, pelvis; pressure of Progressively worse with subsequent pregnancies and
heavy uterus; relaxation of connective tissue in vein obesity; elevate legs above level of heart; support hose
walls; hereditary may help
Cramp in side or groinround ligament pain; stretching To get out of bed, turn to side, use arm and upper body and push
of round ligament with cramping up to sitting position
2164_Ch04_115-248 29/03/12 12:22 PM Page 131

Biological Foundations of Pregnancy 131


(2) When resting, assume semi-Fowlers (b) Gastric secretion of HCl and pepsin
position. decreases.
(3) Avoid overdistention of stomach. (c) Decreased motility delays emptying;
D. Alterations affecting elimination increased acidity.
1. Urinary system (4) Cardiac sphincter relaxes.
a. Physiological changes: (5) Increasing size of uterus and displace-
(1) Relaxation of smooth muscle results in ment of intra-abdominal organs.
conditions that can persist 4 to 6 weeks (6) Gallbladder: decreased emptying.
after birth: b. Assessment:
(a) Dilatation of ureters. (1) Nausea and vomiting in first trimester.
(b) Decreased bladder tone. (2) Constipation and flatulence.
(c) Increased potential for urinary stasis (3) Hemorrhoids.
and urinary tract infection (UTI). (4) Heartburn, reflux esophagitis, indigestion.
(2) Increased glomerular filtration rate (50%) (5) Hiatal hernia.
during last two trimesters. (6) Epulisedema and bleeding of gums.
(3)Increased renal plasma flow (25%50%) (7) Ptyalismexcessive salivation.
during first two trimesters; returns to (8) Jaundice.
near-normal levels by end of last (9) Gallstones.
trimester. (10) Pruritus due to increased retention of
(4) Increased renal-tubular reabsorption rate bile salts.
compensates for increased glomerular c. Nursing care plan/implementation: Goal:

MATERNAL/INFANT
activity. health teaching: dietary.
(5)Glycosuriareflects kidneys inability to (1) Nausea and vomiting
reabsorb all glucose filtered by glomeruli (a) Avoid fatty food; increase
(may be normal or may indicate gesta- carbohydrates.
tional diabetes; glycosuria always warrants (b) Eat small, frequent meals.
further testing). (c) Eat dry crackers in morning.
(6) Increased renal clearance of urea and creati- (d) Decrease liquids with meals.
nine (creatinine clearance used as test of (e) Avoid odors that predispose to nausea.
renal function during pregnancy). (f ) Avoid acidic foods (e.g., citrus,
(7) Hormone-induced turgescence of bladder tomatoes).
and pressure on bladder from gravid uterus (2) Constipation and flatulence
(see Table 4.5). (a) Increase fluids (68 glasses/day).
b. Assessment: (b) Maintain exercise regimen.
(1) Urinary frequency, first and third (c) Add fiber to diet.
trimesters (see Table 4.6). (d) Avoid mineral oil laxatives.
(2) Nocturia. (e) Avoid gas-producing foods
(3) Stress incontinence in third trimester. (e.g., beans, cabbage).
c. Nursing care plan/implementation: Goal: (3) Heartburn and indigestion
health teaching. (a) Eliminate fatty, spicy, or acidic
(1) Void with urge, to prevent bladder foods.
distention. (b) Eat small, frequent meals (6/day).
(2) Learn signs and symptoms of UTI. (c) Eat slowly.
(3) Decrease fluid intake in late evening. (d) Avoid gastric irritants (e.g., alcohol,
(4) Perform Kegel exercises to reduce coffee).
incontinence. (e) Avoid lying flat.
2. Gastrointestinal system (see Table 4.6) (f ) Take antacids without sodium or
a. Physiological changes: phosphorus.
(1) General decrease in smooth-muscle (g) Try chewing gum to increase the
tone and motility due to actions of secretion of alkaline saliva.
progesterone. (h)Wait 2 to 3 hours after meals before
(2) Intestines: slowed peristalsis, increased lying down.
water reabsorption in bowel. (i) Wear loose-fitting clothes.
(3) Stomach (4) Hemorrhoids
(a) Gastric emptying time is delayed (a) Increase fluid and fiber intake.
(e.g., 3 hours vs. 11/2 hours). (b) Maintain exercise regimen.
2164_Ch04_115-248 29/03/12 12:22 PM Page 132

132 chapter 4 Health Promotion and Maintenance

(c) Avoid constipation and straining to 3. Nursing care plan/implementation: Goal:


defecate. health teaching.
(d) Take warm sitz baths. a. Evaluate diet for adequacy of nutrient and
(e) Apply witch hazel pads. caloric intake.
(f ) Elevate hips and legs frequently. b. Evaluate cultural, religious, and economic
(g) Use hemorrhoidal ointments only with influences on diet.
advice of health-care provider. c. Review dietary recommendation for pregnancy
E. Alterations affecting nutrition with woman.
1. Physiological changes: d. Avoid dieting in pregnancy (even if obese).
a. Gastrointestinal system e. Supplement diet with vitamins, iron, or folic
(1) Gingivae soften and enlarge due to acid on advice of health provider.
increased vascularity. f. Ptyalism:
(2) Increased saliva production. (1) Suck hard candies.
b. Endocrine system (2) Perform frequent oral hygiene.
(1) Increased size and activity of pituitary, (3) Maintain adequate oral intake
parathyroids, adrenals. (68 glasses/day).
(2) Increased vascularity and hyperplasia of (4) Use lip balm to prevent chapping.
thyroid. g. Epulis:
(3) Pancreasincreased insulin production dur- (1) Frequent oral hygiene.
ing second half of pregnancy, needed to (2) Use soft toothbrush.
meet rising maternal needs; human placen- (3) Floss gently.
MATERNAL/INFANT

tal lactogen (HPL) and insulinase deactivate (4) See dentist regularly.
maternal insulin; may precipitate gestational F. Alterations affecting protective functions
diabetes in women who are susceptible. integumentary system
c. Metabolism 1. Physiological changesestrogen-induced
(1)Basal metabolic rate (BMR)increases vascular and pigment changes.
25% as pregnancy progresses, due to 2. Assessment:
increasing oxygen consumption; protein- a. Increased pigmentation (chloasma and linea
bound iodine (PBI) increases to 7 to nigra).
10 mcg/dL; metabolism returns to b. Striae gravidarum (stretch marks).
normal by sixth postpartum week. c. Increased sebaceous and sweat gland activity.
(2) Proteinneed increased for fetal and uterine d. Palmar erythema.
growth, maternal blood formation. e. Angiomasvascular spiders.
(3) Water retentionincreased. 3. Nursing care plan/implementation: Goal:
(4) Carbohydratesneed increases to spare health teaching.
protein stores. a. Bathe or shower daily.
(a) First half of pregnancyglucose rapidly b. Reassure woman that skin changes decrease
and continuously siphoned across pla- after pregnancy.
centa to meet fetal growth needs; may G. Alterations affecting comfort, rest, mobility
lead to hypoglycemia and faintness. musculoskeletal system
(b) Second half of pregnancyplacental 1. Physiological changes
production of anti-insulin hormones; a. Progesterone, estrogen, and relaxin-induced
normal maternal hyperglycemia; affects relaxation of joints, cartilage, and ligaments.
coexisting diabetes. b. Function in childbearingincreases antero-
(5) Fatincreased plasma lipid levels. posterior diameter of rib cage and enlarges
(6) Ironsupplements recommended to birth canal.
meet increased need for red blood cells 2. Assessment:
(RBCs) by maternal/placental/fetal a. Complaint of pelvic looseness.
unit. b. Duck-waddle walk.
2. Assessment: c. Tenderness of symphysis pubis.
a. Weight gain: 20 to 35 lb (11.516 kg); d. Lordosis (exaggerated lumbar curve)
depends on body mass index (BMI) and increased weight of pelvis tilts pelvis forward;
prepregnant nutritional status. to compensate, woman throws head and
b. Normal pattern: first trimester, 2 to 5 lb shoulders backward; complaint of leg and
(12.3 kg); remainder of gestation, approxi- back strain and fatigue (see Table 4.6).
mately 1 lb/wk (0.40.5 kg/wk). e. Feet often increase by half a shoe size or more.
2164_Ch04_115-248 29/03/12 12:22 PM Page 133

Biological Foundations of Pregnancy 133


3. Nursing care plan/implementation: Goal: 3. Seek protection for self and fetus through
health teaching. pregnancy and labor (safe passage).
a. Good body alignmenttuck pelvis under; 4. Prepare realistically for the coming child and
tighten abdominal muscles. for necessary role change: I am going to be a
b. Pelvic-rock exercises. parent.
c. Squat; bend at knees, not at waist. B. Physical bases of changes
d. Wear low-heeled, sturdy shoes. 1. Increased metabolic demands may result in
e. Avoid tight-fitting clothing that interferes anemia and fatigue.
with circulatory return in legs. 2. Increased hormone levels (steroids, estrogen,
III. PSYCHOSOCIAL-CULTURAL ALTERATIONS progesterone)affect mood as well as
physiology.
A. Emotional changesaffected by: age, maturity, sup-
C. Characteristic behaviorsTable 4.7 describes
port system, amount of current stresses, coping
behaviors commonly exhibited in each trimester.
abilities, physical and mental health status.
D. Sexuality and sexual expressionfeelings and
Developmental tasks of pregnancy:
expressions of sexuality may vary during
1. Accept the pregnancy as real: I am pregnant;
pregnancy due to maternal adaptations and
progress from symbiotic relationship with
physiological changes.
the fetus to perceiving the child as an
E. Intrafamily relationships
individual.
1. Pregnancy is a maturational crisis for the
2. Seek and ensure acceptance of child by
family.
others.

MATERNAL/INFANT
Table 4.7
Behavioral Changes in Pregnancy
Assessment/Characteristics Nursing Care Plan/Implementation
First Trimester
Emotional lability (mood swings) Encourage verbalization of feelings, concerns
Displeasure with subjective symptoms of early pregnancy Health teaching: diet, rest, relaxation, diversion
(nausea, fatigue, etc.)
Feelings of ambivalence Validate normalcy of feelings, behaviors
Second Trimester
Accepts pregnancy (usually coincides with awareness of fetal Encourage exploration of feelings of dependency, introspec-
movement [i.e., quickening]) tion, mood swings
Becomes introspective: resolves old conflicts (feelings toward Discuss childbirth preparation and preparation-for-parenthood
mother, sexual intimacy, masturbation) classes; refer, as necessary
Reevaluates self, lifestyle, marriage
Daydreams, fantasizes self as mother
Seeks out other women who are pregnant and new mothers
Third Trimester
Altered body image Encourage verbalization of concerns, discomforts of late
pregnancy
Fears body mutilation (stretching of body tissues, episiotomy, Help meet dependency needs; offer reassurance, as possible
cesarean birth)
Distress over loss of control over body functions (ptyalism, Health teaching: Kegel exercises; preparation for labor.
colostrum leakage, leukorrhea, urinary frequency, constipation, Anticipatory guidance and planning for needs of self, baby,
stress incontinence) and family in early postpartum
Anxiety for baby (deformity, death)
Fears pain, loss of control in labor
Acceptance of impending labor during last 2 weeks (ready to
move on)
2164_Ch04_115-248 29/03/12 12:22 PM Page 134

134 chapter 4 Health Promotion and Maintenance

2. Requires changes in lifestyle and interactions: 1-0-0-0-0. After giving birth to a full-term
a. Increased financial demands. living neonate, she becomes 1-1-0-0-1.
b. Changing family and social relationships. b. If a womans second pregnancy ends in abor-
c. Adapting communication patterns. tion and she has a living child from a previous
d. Adapting sexual patterns. pregnancy, born at term, she is designated as
e. Anticipating new responsibilities and needs. 2-1-0-1-1.
f. Responding to reactions of others. c. A woman who is pregnant for the fourth time
and whose previous pregnancies yielded one
full-term neonate, premature twins, and one
Prenatal Management abortion (spontaneous or induced), and who
now has three living children, may be desig-
I. INITIAL ASSESSMENT: Goal: establish baseline nated as 4-1-1-1-3.
for health supervision, teaching, emotional support, or d. Others record as follows: number gravida/
referral. number para. Applying this system to the
II. OBJECTIVES: examples given above, those mothers
A. Determine womans present health status and would be designated as follows: aG1P1;
validate pregnancy. bG2P1; cG4P2.
B. Identify factors affecting or affected by pregnancy. e. Others include recording of abortions:
C. Determine current gravidity and parity. G1P1 Ab0
D. Identify present length of gestation. G2P1 Ab1
E. Establish an estimated date of delivery (EDD). G4P2 Ab1
MATERNAL/INFANT

Ngeles determination of EDDsubtract 3 months, IV. ASSESSMENT: initial physical aspects:


add 7 days to last menstrual period (LMP). A. Height and weight.
F. Determine relevant knowledge deficit. B. Vital signs.
C. Blood workhematocrit and hemoglobin for
III. ASSESSMENT: history anemia; type and Rh factor; tests for sickle cell
A. Familyinheritable diseases, reproductive trait, syphilis, rubella antibody titer, and hepatitis B
problems. screen.
B. Personalmedical, surgical, gynecological, past D. Urinalysisglucose, protein, ketones, signs of
obstetric, average nonpregnant weight. infection, and pregnancy test (human chorionic
1. Gravidaa pregnant woman. gonadotropin [HCG]).
a. Nulligravidawoman who has never been E. Breast examination.
pregnant. F. Pelvic examination.
b. Primigravidawoman with a first pregnancy. 1. Signs of pregnancy.
c. Multigravidawoman with a second or later 2. Adequacy of pelvis and pelvic structures.
pregnancy. 3. Size and position of uterus.
2. Pararefers to past pregnancies (not number of 4. Papanicolau smear.
babies) that reached viability (2022 weeks 5. Smears for monilial and trichomonal
whether or not born alive). infections.
a. Nulliparawoman who has not carried a 6. Signs of pelvic inflammatory disease.
pregnancy to viability (e.g., may have had 7. Tests for STIs: Gonorrhea (gonococcus [GC]),
one or more abortions). chlamydia.
b. Primiparawoman who has carried one G. Validation of pregnancyphysician or midwife
pregnancy to viability. makes differential diagnosis between presumptive/
c. Multiparawoman who had two or more probable signs/symptoms of early pregnancy and
pregnancies that reached viability. other signs.
d. Grandmultiparawoman who has had six or 1. Presumptive symptomssubjective experiences.
more viable pregnancies. a. Amenorrheamore than 10 days past missed
3. Examples of gravidity/parity. Several methods of menstrual period.
describing gravidity and parity are in common b. Breast tenderness, enlargement.
use. One method (GTPAL) describes number c. Nausea and vomiting.
of Gravida (pregnancies), Term (or full-term) d. Quickening (weeks 1618).
infants, Preterm infants, Abortions, and number e. Urinary frequency.
of Living children. f. Fatigue.
a. A woman who is pregnant for the first time g. Constipation (50% of women).
and is currently undelivered is designated as
2164_Ch04_115-248 29/03/12 12:22 PM Page 135

Prenatal Management 135


2. Presumptive signs VI. ASSESSMENT: psychosocial aspects:
a. Striae gravidarum, linea nigra, chloasma A. Pregnancy: planned or not; desired or not.
(after week 16). B. Present plans:
b. Increased basal body temperature (BBT) 1. Carry pregnancy, keep baby.
3. Probable signsexaminers objective findings. 2. Carry pregnancy, adoption.
a. Positive pregnancy test. 3. Abortion.
b. Enlargement of abdomen/uterus. C. Cultural, ethnic influences on decisions: will
c. Reproductive organ changes (after sixth week): influence range of activities, types of safeguarding
(1) Goodells signcervical softening. actions, diet, and health-promotion behaviors.
(2) Hegars signsoftening of lower uterine D. Parenting potential: actively seeking medical
segment. care and information about pregnancy, childbirth,
(3) Vaginal changes (Chadwicks sign): purple parenthood.
hue in vulvar/vaginal area. E. Family readiness for childbearing and child rearing:
d. Ballottement (after 1620 weeks). 1. Physical maintenance.
e. Braxton Hicks contractions. 2. Allocation of resources: identify support
4. Positive signs of pregnancy: system.
a. Fetal heart tones. 3. Division of labor.
(1) Doptone: weeks 10 to 12. 4. Socialization of family members.
(2) Fetoscope: week 20. 5. Reproduction, recruitment, launching of family
b. Examiner visualizes and feels fetal movements members into society.
(usually after week 24). 6. Maintenance of order (relationships within

MATERNAL/INFANT
c. Sonographic examination (after week 14) family).
when fetal head is sufficiently developed for F. Perceptions of present and projected family
accurate determination of gestational age. relationships.
Pregnancy may be detected as early as fifth G. Review lifestyle for smoking, drugs, alcohol
or sixth week after LMP. (ETOH), attitudes about pregnancy, health-care
V. ASSESSMENT: nutritional status: practices, and risks for hepatitis and human
A. Physical findings suggesting poor nutritional status: immunodeficiency virus (HIV).
1. Skin: rough, dry, scaly. VII. ANALYSIS/NURSING DIAGNOSIS:
2. Lips: lesions in corners. A. Altered role performance related to stress imposed by
3. Hair: dull, brittle. developmental tasks.
4. Mucous membranes: pale. B. Ineffective coping: individual, family related to stress
5. Dental caries. caused by developmental tasks/crises.
B. Height, weight, ageaverage weight gain approxi- C. Altered family process related to developmental tasks.
mately 24 lb. Range 24 to 32 lb is best for mother First baby may precipitate individual or family
and neonate. developmental crisis.
C. Laboratory valuesHemoglobin (Hgb): less than
VIII. NURSING CARE PLAN/IMPLEMENTATION:
10.5/100 mg; hematocrit (Hct): less than 32%
indicates anemia. A. Goal: anticipatory guidance/support.
D. Nutrition history. 1. Discuss mood swings, ambivalent feelings, nega-
E. Analysis/nursing diagnosis: tive feelings.
1. Altered nutrition: less than body requirements 2. Reinforce normalcy of such feelings.
related to anemia, vitamin/mineral deficit. B. Goal: increase individual/family coping skills, reduce
2. Altered nutrition: more than body requirements intrafamily stress.
related to obesity. 1. Reinforce family strengths (both partners), sense
F. Nursing care plan/implementation: Goal: health of family identity.
teaching. a. Encourage open communication between
1. Nutritional counseling for diet in pregnancy partners; share feelings and concerns.
and/or lactation. b. Increase understanding of mutual needs,
G. Evaluation/outcome criteria: encourage mutuality of support.
1. If underweight at conception: should gain 28 to c. Increase tendency of mother to turn to part-
42 lb (12.518 kg). ner as most significant person (as opposed to
2. If overweight at conception: 15 to 25 lb physician).
(711.5 kg). d. Enhance bond, success of childbirth prepara-
3. If obese at conception: 15 lb (7 kg) or more. tion classes.
2164_Ch04_115-248 29/03/12 12:22 PM Page 136

136 chapter 4 Health Promotion and Maintenance

2. Promote understanding/acceptance of role (2) Venous bloodHgb, Hct; blood type


change. and Rh factor; rapid plasma reagin (RPR);
a. Facilitate/support achievement of develop- rubella titer, antibody titer, sickle cell.
mental tasks. HIV and hepatitis antigen recommended
b. Reduce probability of postpartum psycho- for all pregnant clients.
logical problems. (3) Cultures (vaginal discharge; cervical scrap-
c. Promote family bonding. ings, for Chlamydia trachomatis, Neisseria
C. Goal: health teaching. gonorrhoeae).
1. Siblings: (4) Tuberculosis (TB) screening in high-risk
a. Alert parents to sibling needs for security, areas.
love. (5) Multiple marker screen, 16 to 18 weeks
b. Include sibling in pregnancy experience. optimum time.
c. Provide clear, simple explanations of (6) Serum glucose screen, 24 to 28 weeks;
happenings. 1-hour glucose tolerance test.
d. Continue demonstrations of love. f. Follow up on medications (vitamins, iron)
e. Describe increased status (big sister/brother). and nutrition.
f. Discuss possible misbehavior to gain g. If TB positive during pregnancy, isoniazid
attention. (INH) and rifampin given daily. INH is
2. Relatives: alert parents to possible negative feel- associated with increase in fetal malforma-
ings of in-laws. tions, particularly neurotoxicity. Pyridoxine
3. Referral to childbirth preparation/parenting administered simultaneously to prevent their
MATERNAL/INFANT

classes. development.
4. Appropriate community referrals for financial B. Common minor discomforts during pregnancy
relief to decrease stress and provide aid. (for Assessment, see Table 4.6).
IX. EVALUATION/OUTCOME CRITERIA: 1. Etiology: normal maternal physiological/
psychological alterations in pregnancy.
A. Actively participates in pregnancy-related decision
2. Nursing care plan/implementation:
making.
a. Goal: anticipatory guidance. Discuss the
B. Expresses satisfaction with decisions made.
importance of adequate rest, exercise, diet,
C. Demonstrates growth and development in parent-
and hydration in minimizing symptoms.
ing role.
b. Goal: health teaching (see Table 4.6).
D. Prepared for the birth and for early parenthood.
3. Evaluation/outcome criteria: woman avoids,
minimizes, or copes effectively with minor usual
discomforts of pregnancy.
Antepartum C. Danger signs:
I. NURSING CARE AND OBSTETRIC SUPPORT 1. Etiology: Specific disease processes are discussed
A. General aspects of prenatal management under Complications.
1. Scheduled visits: 2. Nursing care plan/implementation: Goal:
a. Once monthlyuntil week 28. health teachingto safeguard status. Signs to
b. Every 2 weeksweeks 28 to 36. report immediately:
c. Weeklyweek 36 until labor. a. Persistent vomiting beyond first trimester or
2. Assessment: severe vomiting at any time. Possible cause:
a. General well-being, signs of deviations, Hyperemesis gravidarum.
concerns, questions. b. Fluid discharge from vaginableeding or
b. Weight gain pattern. amniotic fluid (anything other than leukor-
c. Blood pressure (sitting). rhea). Possible causes: Placental problem,
d. Abdominal palpation: rupture of membranes (ROM).
(1) Fundal height; tenderness, masses, hernia. c. Severe or unusual pain: abdominal. Possible
(2) Fetal heart rate (FHR). cause: Abruptio placentae.
(3) Leopolds maneuver for presentation d. Chills or fever (if lasts over 24 hours, or over
(after week 32). 102F). Possible cause: Infection.
e. Laboratory tests: e. Urinary frequency or burning on urination.
(1) Urinalysisfor protein, sugar, signs of Possible cause: UTI.
asymptomatic infection; drug screen for f. Absence of fetal movements after quicken-
high-risk groups. ing, lasting more than 24 hours. Possible cause:
Intrauterine fetal death.
2164_Ch04_115-248 29/03/12 12:22 PM Page 137

Complications During the Antepartum 137


g. Visual disturbancesblurring, double (2) Irregular/episodic prenatal care visits.
vision, spots before eyes. Possible cause: (3) Noncompliance with medical/nursing
Preeclampsia. recommendations.
h. Swelling of fingers, ankles, hands, feet, or g. Preexisting or coexisting medical disorders:
face. Possible cause: Preeclampsia. (1) Cardiovascular: hypertension, heart
i. Severe, frequent, or continual headache. disease.
Possible cause: Preeclampsia. (2) Diabetes.
j. Muscle irritability or convulsions. Possible (3) Other: renal, respiratory, infections, acquired
cause: Preeclampsia. immunodeficiency syndrome (AIDS).
k. Rapid weight gain not associated with eat- h. Substance abuse.
ing. Possible cause: Preeclampsia. 4. Nursing care plan/implementation:
l. More than four uterine contractions per hour a. Goal: health teaching (discussed under specific
(before 38 weeks). Possible cause: Preterm labor. health problem).
3. Evaluation/outcome criteria: b. Goal: early identification/treatment of emerging
a. Actively participates in own health maintenance/ health problems (if any).
pregnancy management. (1) Monitor status and progress of pregnancy.
b. Identifies early signs of potentially serious (2) Refer for medical management, as
complications during the antepartal period. necessary.
c. Promptly reports and seeks medical attention. c. Goal: emotional support.
II. COMPLICATIONS DURING 5. Evaluation/outcome criteria:
THE ANTEPARTUM a. Understands present health status, interac-

MATERNAL/INFANT
tions of coexisting disorder and pregnancy.
A. General aspects:
b. Accepts responsibility for own health
1. Etiology:
maintenance.
a. Normal alterations and increasing physiological
c. Makes informed decisions regarding
stress of pregnancy affect status of coexisting
pregnancy.
medical disorders.
d. Minimizes potential for complications of
b. Conditions affecting mothers general health
coexisting disorder/pregnancy.
also affect ability to adapt successfully to
(1) Avoids factors predisposing to health
normal physiological stress of pregnancy.
problems.
c. Aberrations of normal pregnancy.
(2) Understands and implements therapeutic
2. Goal: reduce incidence of health problems affecting
management of coexisting disorder/
maternal/fetal health and pregnancy outcome.
pregnancy.
a. Identify presence of risk factors and signs and
(3) Increases compliance with medical/nursing
symptoms of complications early.
recommendations.
b. Treat emerging complications promptly and
e. Carries uneventful pregnancy to term.
effectively.
B. Disorders affecting fluid-gas transport: cardiac
c. Minimize effects of complications on
disease
pregnancy outcome.
1. Pathophysiology: cardiac overload cardiac
3. Assessment: risk factors:
decompensation right-sided heart failure
a. Age:
systemic edema.
(1) Adolescent.
2. Etiology:
(2) Primigravida, age 35 or older.
a. Congenital heart defects.
(3) Multigravida, age 40 or older.
b. Valvular damagedue to rheumatic fever
b. Socioeconomic level: lower.
(most common lesion is mitral stenosis,
c. Ethnic group.
which can lead to pulmonary edema and
d. Previous pregnancy history:
emboli).
(1) Habitual abortion.
c. Increased circulating blood volume and
(2) Multiparity greater than 5.
cardiac outputexceeds cardiac reserve.
(3) Previous stillbirths.
Greatest risk: after 28 weeks of gestation
(4) Previous cesarean birth.
reaches maximum (30%50%) volume
(5) Previous preterm labor or delivery.
increase; postpartumdue to diuresis.
e. Multifetal pregnancy.
d. Secondary to treatment (e.g., tocolysis and
f. Prenatal care:
steroids).
(1) Enters health-care system late in
e. Pregnancy after valve replacement.
pregnancy.
2164_Ch04_115-248 29/03/12 12:22 PM Page 138

138 chapter 4 Health Promotion and Maintenance

3. Normal physiological alterations during preg- (2) Digitalis (dose may need to be higher
nancy that mimic cardiac disorders: because of dilution in the increased blood
a. Systolic murmurs, palpitations, tachycardia, volume of pregnancy).
and hyperventilation with some dyspnea on (3) Antibioticsprophylaxis against rheumatic
normal moderate exertion. fever; treatment of bacterial infections dur-
b. Edema of lower extremities. ing pregnancy.
c. Cardiac enlargement. (4) Anticoagulants. Heparin is preferred
d. Elevated sedimentation rate near term. because its large molecule cannot easily
4. Assessment: cross placenta. Occasionally, sequelae may
a. Medical evaluation of cardiac status. include maternal hemorrhage, preterm
Classification of severity of cardiac birth, stillbirth.
involvement: (5) Oxygen, as needed.
(1) Class Ileast affected; asymptomatic with (6) Mitral valvotomy for mitral stenosis often
ordinary activity. brings dramatic relief.
(2) Class IIactivities somewhat limited; b. Goal: health teaching.
ordinary activities cause fatigue, dyspnea, (1) Need for compliance with therapeutic regi-
angina. men, medical/nursing recommendations.
(3) Class IIImoderate/marked limitation of (2) Drug actions, dosage, necessary actions
activity; common activities result in severe (how to take own pulse, reportable
symptoms of fatigue, etc. signs/symptoms).
(4) Class IVmost affected; symptomatic (3) Methods for decreasing work of heart:
MATERNAL/INFANT

(dyspnea, angina) at rest; should avoid (a) Adequate restminimum 10 hours


pregnancy. sleep each night; 30-minute nap after
b. Cardiac decompensation: each meal.
(1) Subjective symptoms (b) Avoid heavy physical activity (including
(a) Palpitations; feeling that the heart is housework), fatigue, excessive weight
racing. gain, emotional stress, infection.
(b) Increasing fatigue or difficulty breath- (c) Avoid situations of reduced ambient O2,
ing, or both, with the usual activities. such as smoking, exposure to pollu-
(c) Feeling of smothering and/or frequent tants, flight in unpressurized small
cough. planes.
(d) Periorbital edema; edema of face, fingers c. Goal: nutritional counseling.
(e.g., rings do not fit anymore), feet, (1) Well-balanced diet; adequate protein, fresh
legs. fruits and vegetables, water.
(2) Objective signs (2) Avoid junk food, stimulants (caffeine),
(a) Irregular weak, rapid pulse (100 excessive salt intake.
beats/min). d. Goal: anticipatory planning: management of
(b) Rapid respirations (25 breaths/min). labor.
(c) Progressive, generalized edema. (1) Goal: minimize physiological and
(d) Crackles (rales) at base of lungs, after psychological stress.
two inspirations and exhalations. (2) Medical management:
(e) Orthopnea; increasing dyspnea on (a) Reevaluation of cardiac status before
minimal physical activity. EDD and labor.
(f ) Moist, frequent cough. (b) Regional anesthesia for labor/birth.
(g) Cyanosis of lips and nailbeds. (c) Low-outlet forceps or vacuum extrac-
5. Analysis/nursing diagnosis: tion birth; episiotomy.
a. Fluid volume excess related to inability of com- (d) Continuous hemodynamic monitoring.
promised heart to handle increased workload (3) Assessment: continuous.
(decreased cardiac reserve congestive heart (a) Physiological response to labor stimuli
failure). frequent vital signs (pulse rate most sensi-
b. Impaired gas exchange related to pulmonary tive and reliable indicator of impending
edema secondary to congestive heart failure. congestive heart failure).
6. Nursing care plan/implementation: (b) Color, respiratory effort, diaphoresis.
a. Medical management: (c) Contractions, etc.same as for any
(1) Diuretics, electrolyte supplements. mother in labor.
2164_Ch04_115-248 29/03/12 12:22 PM Page 139

Complications During the Antepartum 139


(4) Nursing care plan/implementation: labor. 7. Evaluation/outcome criteria:
(a) Goal: safeguard status. a. Successfully carries uneventful pregnancy
(i) Report promptly: pulse rate over to term.
100; respirations more than 24 b. Experiences no cardiopulmonary decompen-
between contractions. sation during labor, birth, or postpartum.
(ii) Oxygen at 6 liters, as needed. C. Disorders affecting fluid-gas transport in fetus:
(b) Goal: emotional supportto reduce Rh incompatibility
anxiety, facilitate cooperation. 1. Pathophysiologyin a mother who is Rh
(i) Encourage verbalization of feelings, negative: Rh-positive fetal red blood cells enter
fears, concerns. the maternal circulation maternal antibody
(ii) Explain all procedures. formation antibodies cross placenta and
(c) Goal: promote cardiac function. enter fetal bloodstream attack fetal red blood
Positionsemirecumbent; support cells hemolysis anemia, hypoxia.
arms and legs. a. The mother who is pregnant and Rh positive
(d) Goal: promote relaxation/control over labor carries her infant (Rh negative or positive)
discomfort. Encourage Lamaze (or other) without incident.
breathing/relaxation techniques. b. The mother who is pregnant and Rh positive
(e) Goal: reduce stress on cardiopulmonary carries an Rh-negative infant without incident.
system. Discourage bearing-down c. The mother who is pregnant and Rh nega-
efforts. tive usually carries her first Rh-positive
(f ) Goal: relieve stress of pain, eliminate child without problems unless she has been

MATERNAL/INFANT
bearing-down. Prepare for regional sensitized by inadvertent transfusion with
anesthesia. Rh-positive blood. Note: Fetal cells do not
(g) Goal: maintain effective cardiac function. usually enter the maternal bloodstream until
Administer medications, as ordered placental separation (at abortion, abruptio
(e.g., digitalis, diuretics, antibiotics). placentae, amniocentesis, or birth).
e. Goal: anticipatory planning: postpartum 2. Etiology:
management. a. The Rh factor is an antigen on the red blood
(1) Factors increasing risk of cardiac cells of some people (these people are Rh posi-
decompensation: tive); the Rh factor is dominant; a person may
(a) Delivery rapid, decreased intra- be homozygous or heterozygous for Rh factor.
abdominal pressure vasocongestion b. A person who is Rh negative is homozygous
and rapid rise in cardiac output. for this recessive traitdoes not carry the
(b) Loss of placental circulation. antigen; develops antibodies when exposed to
(c) Normal diuresis increases circulating Rh-positive red blood cells (isoimmunization)
blood volume. through transplacental (or other) transfusion.
(2) Assessment: c. Following birth of an infant who is Rh
(a) Observe for tachycardia or respiratory positive, if fetal cells enter the mothers
distress. bloodstream, maternal antibody formation
(b) Monitor blood loss, I&Opotential begins; antibodies remain in the maternal
hypovolemic shock, cardiac overload circulation.
due to diuresis. d. At time of next pregnancy with fetus who
(c) Pain levelpotential neurogenic is Rh positive, antibodies cross placenta
shock. hemolysis. Note: Degree of hemolysis depends
(d) Same as for any woman who is postpar- on amount (titer) of maternal antibodies
tum (fundus, signs of infection, etc.). present.
(3) Nursing care plan/implementation: 3. Possible serious complication (fetal)rare today.
postpartum. Hydrops fetalismost severe hemolytic reaction:
(a) Goal: minimize stress on cardiopul- severe anemia, cardiac decompensation, hypoxia,
monary system. edema, ascites, hydrothorax; may be stillborn.
(i) Rest, dangle, ambulate with 4. Assessment:
assistance. a. Prenataldiagnostic procedures:
(ii) Gradual increase in activityas (1) Maternal blood type and Rh factor.
tolerated without symptoms. (2) Indirect Coombs testto determine pres-
(iii) Position: semi-Fowlers if needed. ence of Rh sensitization (titer indicates
(iv) Extra help with newborn care. amount of maternal antibodies).
2164_Ch04_115-248 29/03/12 12:22 PM Page 140

140 chapter 4 Health Promotion and Maintenance

(3)Amniocentesisas early as 26 weeks dormant infection in hosts with normally func-


of gestationamount of bilirubin by- tioning immune systems. M. tuberculosis is a
products indicates severity of hemolytic slow-growing obligate aerobe and a facultative
activity. intracellular parasite.
b. Intrapartum observation of amniotic fluid 2. Etiology:
(after membrane rupture). a. Symptoms of tuberculosis in pregnancy are
(1) Straw-colored fluidmild disease. vague and nonspecific. Fatigue, shortness of
(2) Golden fluidsevere fetal disease. breath, sweating, and tiredness can all be
c. Postnatal (see III. A. Rh incompatibility, attributed to the pregnancy.
p. 215). b. Reluctance of health-care professionals to
5. Nursing care plan/implementation: perform a chest x-ray on a woman who is
a. Goal: prevent isoimmunization in women who pregnant due to fear of harming the fetus
are Coombs negative delay in diagnosis.
(1)Postabortionif no evidence of Rh 3. Assessment:
sensitization (antibody formation) in the a. Heaf and Mantoux skin tests (as reliable as in
mother who is Rh negative, administer women who are nonpregnant).
RhoGAM. b. Same as for women who are nonpregnant:
(2) Prenatalif no evidence of sensitization, sputum examination, and culture and scans.
administer RhoGAM at 28 weeks of gesta- 4. Nursing care plan/implementation:
tion, as ordered, to all women who are Rh a. Goal: prevent spread of disease.
negative. (1)Initial treatment regimen: isoniazid
MATERNAL/INFANT

(3) Postpartumif no evidence of sensitization, (INH), rifampin (RIF), and ethambutol


administer RhoGAM within 72 hours (EMB).
of birth to women who are Rh negative (2)Pyridoxine (vitamin B6) recommended
and who gave birth to a baby who is Rh for women who are pregnant and
positive. taking INH.
Give RhoGAM to: (3) Routine use of pyrazinamide (PZA)
1. Mother who is Rh negative who should be avoided because of inadequate
gives birth to neonate who is Rh teratogenicity data.
positive. (4) Avoid: streptomycin (which interferes
2. Mother who is Rh negative after with development of the ear; may cause
spontaneous or induced abortion congenital deafness).
(>8 weeks). b. Goal: health teaching.
3. Mother who is Rh negative after (1) Explain, discuss transmission of disease,
amniocentesis or chorionic villus importance of completion of medication
sampling (CVS). regimen.
4. Mother who is Rh negative between 28 (2)Because small concentration of antituber-
and 32 weeks of gestation. culosis drugs in breast milk do not
b. Goal: health teaching. produce toxicity in the newborn who
(1) Explain, discuss that RhoGAM suppresses is nursing, breastfeeding should not be
antibody formation in susceptible woman discouraged for a woman who is HIV
who is Rh negative carrying fetus that is seronegative and is planning to take
Rh positive. Note: Cannot reverse sensiti- (or is taking) INH or other anti-TB
zation if already present. medications.
(2) Required during and after each pregnancy c. Goal: TB treatment for women who are HIV
with fetus who is Rh positive. infected and pregnant.
6. Evaluation/outcome criteria: (1)If have a positive M. tuberculosis culture
a. Successfully carries pregnancy to term. or suspected TB disease, treat without
b. No evidence of Rh isoimmunization. delay.
c. Birth of viable infant. (2) Rifamycin.
D. Disorders affecting fluid-gas transport in fetus: (3) Although routine use of pyrazinamide not
tuberculosis recommended if pregnant (due to inade-
1. Pathophysiology: Mycobacterium tuberculosis quate teratogenicity data), benefits for
primarily is spread as an airborne aerosol from women who are HIV infected and pregnant
infected noninfected individuals, through the outweigh potential pyrazinamide-related
lung. Initial TB infection usually latent or risks to fetus.
2164_Ch04_115-248 29/03/12 12:22 PM Page 141

Complications During the Antepartum 141


E. Disorders affecting fluid-gas transport in fetus: 3. Normal physiological alterations during pregnancy
hepatitis B (HBV) that may affect management of the woman who
1. Pathophysiology: Hepatitis B is one of the is diabetic, or precipitate gestational diabetes in
most highly transmitted forms of hepatitis from women who are susceptible:
mother to child around the world, especially in a. Hormone production:
developing countries. Hepatitis B virus (HBV) (1) Human placental lactogen (HPL).
is highly contagious; the risk that newborn (2) Progesterone.
infant will develop hepatitis B is 10% to 20% (3) Estrogen.
if the mother is positive for the hepatitis B (4) Cortisol.
surface antigen (HBsAg); and as high as 90% b. Effects of hormones:
if she is also positive for the HBeAg (hepatitis (1) Decreased glucose tolerance.
Be antigen). (2) Increased metabolic rate.
2. Etiology: Usually, hepatitis B is passed on dur- (3) Increased production of adrenocortical and
ing delivery with exposure to the blood and flu- pituitary hormones.
ids during the birth process. (4) Decreased effectiveness of insulin (increased
3. Assessment: resistance to insulin by peripheral tissues).
a. Blood: highest concentration. (5) Increased gluconeogenesis.
b. Semen, vaginal secretions, wound exudates: (6) Increased size and number of islets of
lower concentration. Langerhans to meet increased maternal
c. Hepatitis B surface antigen = active infection. needs.
4. Nursing care plan/implementation: (7) Increased mobilization of free fatty acids.

MATERNAL/INFANT
a. Goal: prevent spread of disease. (8) Decreased renal threshold, increased
(1) Hepatitis B immune globulin (HBIG) to glomerular filtration rate; glycosuria
infant at birth. common.
(2) Hepatitis B vaccine at 1 week, 1 month, (9) Decreased CO2-combining power of
6 months after birth. blood; higher metabolic rate increases
b. Goal: health teaching. tendency to acidosis.
(1) Explain, discuss transmission of disease, c. Effect of pregnancy on diabetes:
importance of completion of vaccination (1) Nausea and vomitingpredispose to
regimen. ketoacidosis.
(2) Centers for Disease Control and (2) Insulin requirementsrelatively stable or
Prevention (CDC) has recommended may decrease in first trimester; rapid
that all newborn infants be vaccinated increase during second and third
for hepatitis B. trimesters; rapid decrease after birth to
(3) The risk of HBV infection in children is prepregnant level.
not only from perinatal transmission from (3) Pathophysiological progression (nephropa-
mothers who are HBV infected, but also thy, retinopathy, and arteriosclerotic
from close contact with household mem- changes) may appear; existing pathology
bers and caregivers who have acute or may worsen.
chronic HBV infection. 4. Effect of poorly controlled diabetes on pregnancy
(4) Ensure that all infants born to mothers increased incidence of:
who are HBsAg positive receive timely a. Infertility.
and appropriate immunoprophylaxis with b. UTI.
HBIG and hepatitis B vaccine. c. Vaginal infections (moniliasis).
(5) Discontinue interferon therapy during d. Spontaneous abortion.
pregnancy (effect on fetus is unknown). e. Congenital anomalies (three times as prevalent).
F. Disorders affecting nutrition: diabetes mellitus f. Preeclampsia/eclampsia.
1. Pathophysiology: increased demand for insulin g. Polyhydramnios.
exceeds pancreatic reserve inadequate insulin h. Preterm labor and birth.
production; enzyme (insulinase) activity breaks i. Fetal macrosomiacephalopelvic dispropor-
down circulating insulin further reduction in tion (CPD).
available insulin; increased tissue resistance to j. Stillbirth.
insulin; glycogenolysis/gluconeogenesis ketosis. 5. Assessment: gestational diabetes (mellitus)
2. Etiology: increased metabolic rate; action of a. History:
placental hormones (see following), enzyme (1) Family history.
(insulinase) activity. (2) Previous infant 9 lb or more.
2164_Ch04_115-248 29/03/12 12:22 PM Page 142

142 chapter 4 Health Promotion and Maintenance

(3) Unexplained fetal wastageabortion, still- (2) Skill and accuracy in monitoring serum
birth, or early neonatal death. glucose (dextrometer use).
(4) Obesity with very rapid weight gain. (3) Skill and accuracy in preparing and
(5) Polyhydramnios (excessive amniotic fluid). administering insulin dosage; site rotation;
(6) Previous infant with congenital anomalies. subcutaneous injection in abdomen.
(7) Increased tendency for intense vaginal or (4) Close monitoringprenatal status assess-
urinary tract infections. ment every 2 weeks until 30 weeks, then
(8) Previous history of gestational diabetes. weekly until birth. Alert to signs of emerg-
b. Symptoms: 3 Pspolydipsia, polyphagia, ing problems (need for insulin adjustment,
polyuriaand weight loss. polyhydramnios, macrosomia).
c. Abdominal assessment: (5) Otheras for any woman who is
(1) Fetal heart rate. pregnant.
(2) Excessive fundal height. 6. Analysis/nursing diagnosis:
(a) Polyhydramnios. a. Knowledge deficit related to pathophysiology,
(b) Large-for-gestational-age (LGA) fetus. interactions with pregnancy, management
Note: With vascular pathology, small- (e.g., insulin administration).
for-gestational-age (SGA) fetus. b. Altered nutrition, more or less than body
d. Medical diagnosisprocedures: requirements, related to weight gain.
(1)50-gm oral glucose tolerance test c. High-risk pregnancy: high risk for infection,
(GTT): woman ingests 50 gm oral ketosis, fetal demise, fetal macrosomia,
glucose solution; 1 hour later plasma cephalopelvic disproportion, polyhydram-
MATERNAL/INFANT

glucose obtained. If 140 mg/dL, 3-hour nios, preterm labor and birth, congenital
oral GTT ordered. anomalies.
(2) Abnormal 3-hour GTT: two or more of 7. Nursing care plan/implementation:
the following findings are diagnostic of a. Goal: health teaching.
gestational diabetes: (1) Pathophysiology of diabetes, as necessary;
(a) Fasting blood sugar (FBS) 95 mg/dL. effect of pregnancy on management.
(b) One hour 180 mg/dL. (2) Signs and symptoms of hyperglycemia,
(c) Two hours 155 mg/dL. hypoglycemia; appropriate management of
(d) Three hours 140 mg/dL. symptoms.
(3) Diabetic classification criteria: (3) Hygieneto reduce probability of
(a) Type 1autoimmune disease in which infection.
the bodys immune system destroys (4) Exerciseneeded to control serum glucose
pancreatic beta cells; production of levels, to regulate weight gain, and for
insulin; need additional insulin. About feeling of well-being.
10% with diabetes are type 1. (5) Need for close monitoring during
(b) Type 2 insulin resistance despite pregnancy.
adequate insulin production; treatment (6) Insulin regulation:
may include: diet, exercise, weight loss, (a) Requirements vary through pregnancy:
oral drugs to stimulate release of first trimestermay decrease with some
insulin; or insulin injections. About periods of hypoglycemia due to fetal
90% with diabetes are type 2. drain; second trimesterincreased need
(c) Gestationaloccurs in about 3% of all for insulin; third trimesterneeds may
pregnancies. GTT administered at 24 be triple prepregnant dose; acidosis more
to 28 weeks gestation; two abnormal common in late pregnancy (precipitated
values indicate diagnosis of gestational by emotional stress, infection).
diabetes. About 40% of women with (b) Serum glucose testingdextrometer,
gestational diabetes will develop type 2 blood glucose, or other.
diabetes within 5 years. (c) Preparation and self-administration of
e. Woman with known diabetesall classes. insulin injection, as necessary.
(1) Knowledge and acceptance of disease and (d) Prompt reporting of fluctuating serum
its management: glucose levels.
(a) Signs and symptoms of hyperglycemia/ (7) Diagnostic testing/hospitalization:
hypoglycemia (see Table 6.28). (a) Nonstress test.
(b) Appropriate behaviors (e.g., skim milk (b) Sonography.
for symptoms of hypoglycemia). (c) Amniocentesis.
2164_Ch04_115-248 29/03/12 12:22 PM Page 143

Complications During the Antepartum 143


b. Goal: dietary counseling. 10. Anticipatory planningmanagement of labor
(1) Optimal weight gainabout 24 lb. a. Assessment: continuous.
(2) Needs 25 to 35 calories/kg of ideal body (1) Signs and symptoms of hyperglycemia,
weight (18002600 calories). hypoglycemia (see Table 6.28). Hourly
(3) Protein18% to 25% (2 gm/kg, or about blood sugar measurements.
70 gm daily). (2) Electronic fetal monitoringto identify
(4) Carbohydrates: 50% to 60% in complex signs of fetal distress.
form (milk, bread). (3) Otheras for any woman in labor.
(5) Fats25% to 30% unsaturated. b. Nursing care plan/implementation: Goal:
(6) No fruit juice; no cold cereal; carbohy- safeguard maternal/fetal status.
drates limited. (1) Position: lateral Simsto reduce compres-
c. Medical management: hospitalize woman for: sion of inferior vena cava and aorta due to
(1) Regulation of insulin (oral hypoglycemics polyhydramnios or LGA baby. (Supine
contraindicated in early pregnancy, due to hypotensive syndrome results from compres-
teratogenicity; cross placental barrier). sion; reduced placental perfusion increases
(2) Control of infection. incidence of fetal hypoxia/anoxia.)
(3) Determination of fetal jeopardy or indica- (2) Medical managementvaries widely.
tions for early termination of pregnancy. (a) Timingamniocentesis to determine
8. Evaluation/outcome criteria: PG and phosphatidylinositol levels
a. Understands and accepts diagnosis of (estimate fetal pulmonary surfactant).
diabetes. (b) Insulin added to intravenous infusion

MATERNAL/INFANT
b. Actively participates in effective management of 0.9 NaCl and titrated to maintain
of diabetes and pregnancy. serum glucose approximately 100 mg/dL.
c. Maintains serum glucose levels within accept- 5% to 10% D/W IV needed to pre-
able parameters (e.g., 70120 mg/dL). vent hypoglycemia that may lead to
(1) Monitors serum glucose levels accurately maternal ketoacidosis; hyperglycemia
(dextrometer, blood glucose). may result in newborn hypoglycemia.
(2) Prepares and self-administers insulin (c) Ultrasound to identify macrosomia
appropriately. >4050 gm.
(3) Complies with dietary regimen. 11. Anticipatory planningmanagement of
9. Antepartal hospitalization postpartum
a. Assessment: a. Factors influencing serum glucose levels:
(1) Medical evaluationprocedures: (1) Loss of placental hormones that degrade
(a) Serum glucose levels ( 120 mg/dL). insulin.
(b) Sonography for fetal growth: biophysical (2) Lower metabolic rate. Woman requiring
profile (BPP) evaluates fetal physical large doses of insulin may need to triple
well-being and volume of amniotic fluid. caloric intake and decrease insulin by
(c) Nonstress testing/contraction stress one-half.
testing. b. Assessment:
(d) Amniocentesis for fetal maturity. Note: (1) Observe for:
Lecithin/sphingomyelin (L/S) ratio may (a) Hypoglycemia.
be elevated in women who are diabetic; (b) Infection.
phosphatidylglycerol [PG] more accurate (c) Preeclampsia/eclampsia (higher inci-
for women who are diabetic. dence in women who are diabetic).
(2) Nursing assessment: (d) Hemorrhage (associated with polyhy-
(a) Daily weight, vital signs, FHR q4h, dramnios, macrosomia, induction of
I&O. labor, forceps birth, or cesarean birth).
(b) Fundal height and Leopolds maneuver (2) Monitor healing of episiotomy/abdominal
on admission. incision.
b. Nursing care plan/implementation: Goal: c. Nursing care plan/implementation:
emotional support to reduce anxiety and ten- (1) Medical management: insulin calibration
sion, which contribute to insulin imbalance. requirement may drop to one-half or two-
(1) Explain all procedures. thirds pregnant dosage on first postpartum
(2) Assist with tests for fetal status. day if woman is on full diet (due to loss of
(3) Prepare for possibility of preterm or cesare- human placental lactogen and conversion
an birth. of serum glucose to lactose).
2164_Ch04_115-248 29/03/12 12:22 PM Page 144

144 chapter 4 Health Promotion and Maintenance

(2) Nursing management G. Disorders affecting psychosocial-cultural behav-


(a) Goal: euglycemia. Blood glucose, iors: substance abuse
insulin as ordered. 1. Assessment: woman who is pregnant and abuses
(b) Goal: avoid trauma, reduce risk of substances
UTI. Avoid catheterization, where a. Medical history
possible. (1)Infections: HIV-positive status,
(c) Goal: health teaching. Nipple careto AIDS, STIs, hepatitis, cirrhosis,
prevent fissures and possible mastitis. cellulitis, endocarditis, pancreatitis,
(d) Goal: reduce serum glucose and insulin pneumonia.
needs. Encourage/support breastfeed- (2) Psychiatric illness: depression, paranoia,
ing antidiabetogenic effect. Note: irritability.
If acetonuria occurs, stop breastfeeding (3) Trauma related to violence.
while physician readjusts diet/insulin b. Obstetric history
balance; may pump breasts to maintain (1) Spontaneous abortions.
lactation. If hypoglycemic, adrenalin (2) History of abruptio placentae.
level rises decreased milk supply (3) Preterm labor.
and let-down reflex. (4) Preterm rupture of membranes.
12. Anticipatory guidancedischarge plan/ (5) Fetal death.
implementation (6) Low-birth-weight (LBW) infants.
a. Goal: counseling. Reinforce recommendations (7) Tremors/seizures.
of physicians/genetic counselors. c. Current pregnancy
MATERNAL/INFANT

(1) Risk of infant inheriting gene for diabetes (1) Preterm labor contractions.
is greater if mother has early-onset, (2) Hypoactivity or hyperactivity in fetus.
insulin-dependent disease. (3) Poor or decreased weight gain.
(2) Increased risk of congenital disorders. (4) STI.
b. Goal: family planning. (5) Undiagnosed vaginal bleeding.
(1) Oral contraceptives are controversial (6) Drugs being used and methods of
because they decrease carbohydrate toler- self-administration.
ance; may be cautiously prescribed for d. Psychosocial history
women with no vascular disease and who (1) Attitudes re: pregnancy.
are nonsmokers. Intrauterine device (2) Current support system: lacking.
(IUD) contraindicated because of (3) Current living arrangements; lifestyle.
impaired response to infection. Barrier (4) History of psychiatric illness.
contraceptives (diaphragm or condoms (5) History of physical, sexual abuse.
with spermicides) recommended. (6) Involvement with legal system.
(2) Tubal ligation: if mother has vascular e. Physical examination
involvement (i.e., retinopathy or f. Commonly abused substances
nephropathy), increased risk with later (1) Nicotine.
pregnancies. (2) Alcohol (fetal alcohol syndrome [FAS] or
c. Goal: health teaching. fetal alcohol effects [FAE]).
(1) Self-care measures. (3) Marijuana.
(2) Importance of eating on time, even if (4) Stimulantscocaine, crack, ice,
infant must wait to breastfeed or bottle methamphetamine.
feed. (5) Opiatesheroin, methadone, Darvon,
(3) Importance of adequate rest and exercise codeine, Vicodin, OxyContin.
to maintain insulin/glucose balance. (6) Sedatives, hypnotics.
(4) Organize schedule to care for infant, other (7) Caffeine.
children, and her diabetes. Allow time (8) Ecstasy.
for self. g. Neonatal outcomes
d. Evaluation/outcome criteria: (1) LBW, small heads.
(1) Successfully completes an uneventful (2) Irritable, difficult to console.
pregnancy, labor, and birth of a newborn (3) Disorganized suck-swallow reflex.
who is normal and healthy. (4) Impaired motor development.
(2) Makes informed judgments regarding (5) Congenital anomalies: genitourinary,
parenting, family planning, management gastrointestinal, limb anomalies.
of her diabetes. (6) Cerebral infarctions.
2164_Ch04_115-248 29/03/12 12:22 PM Page 145

Lifestyle Choices and Influences That Impact Health in Pregnancy and Newborn 145
(7) Breastfeeding allowed; thought to ease e. Social problems: mothers who are poorly
infant withdrawal. educated, child abuse, single-parent families,
(8) Poor, slow weight gain; failure to thrive. mothers who are unemployed or working at
2. Analysis/nursing diagnosis: minimum wage or who lack support system.
a. Altered nutrition: less than body requirements 2. Assessment:
poor weight gain related to poor nutrition. a. Present physical/health status.
b. Altered nutrition: less than body requirements b. Feelings toward pregnancy.
slow fetal growth related to slow gain in c. Plans for the future.
weight. d. Factors influencing decisions related to self,
c. Altered placental function related to high risk pregnancy, baby.
for abruptio placentae. e. Signs and symptoms of complications of
d. Noncompliance with health-care protocols pregnancy (see A. 1. d. Associated health
related to persistent drug use. problems).
e. Altered parenting related to psychological f. Need/desire for health maintenance informa-
illness (substance dependence). tion (family planning).
3. Nursing care plan/implementation: 3. Analysis/nursing diagnosis:
a. Early identification of substance abuse. a. Ineffective coping, individual/family, related to
b. Stabilize physiological status. need to alter lifestyle, plans, expectations.
c. Fetal surveillance. b. Altered family processes related to
d. Urge consistent obstetric care. unexpected/unwanted pregnancy.
e. Refer for social services. c. Altered parenting related to intrafamily stress

MATERNAL/INFANT
4. Evaluation/outcome criteria: secondary to unexpected pregnancy, develop-
a. Seeks out and uses social services and drug mental tasks.
treatment program. d. Self-esteem disturbance related to altered
b. Abstains from illicit substances during self-concept, body image, role performance,
pregnancy. personal identity.
c. Successfully completes an uneventful pregnancy, e. Knowledge deficit related to family planning,
labor, and birth of normal healthy infant. health maintenance, risk factors, pregnancy
options.
f. Altered nutrition related to lifestyle.
Lifestyle Choices and 4. Nursing care plan/implementation:
a. Goal: emotional support.
Influences That Impact (1) Ensure confidentiality.
Health in Pregnancy, (2) Establish acceptant, supportive
environment.
Intrapartum, Postpartum, (3) Encourage verbalization of feelings,
concerns, fears, desires, etc.
and Newborn (see also STIs, (4) Maintain continuity of careconsistency
pp. 146148, pp. 153157, and of nursing approach, to establish trust,
confidentiality.
substance abuse, pp. 144145) b. Goal: facilitate informed decision making.
Discuss available options; aid in exploring
I. OTHER HIGH-RISK WOMEN implications of possible decisions.
A. An adolescent who is pregnant c. Goal: nutritional counseling (anemia).
1. General aspects: (1) Needs for own growth and that of fetus.
a. Pregnancy in women between 12 and (2) High-quality dietvalue for character of
17 years old. skin, return to prepregnant figure.
b. Incidence has started to ; approximately (3) Include pizza, hamburgers, milkshakes as
one third of all births are to adolescents. acceptableto minimize anger at being
c. Predisposing factors: early menarche, early different.
experimentation with sex, poor family d. Goal: health teaching.
relationships, poverty, late or no prenatal care, (1) Rest, exercise, hygieneas for other women.
cultural influence. (2) Prevention of infectionSTI, UTI, etc.
d. Associated health problems: preeclampsia, (3) Breast self-examination; Pap smear.
preterm labor, SGA infants, anemia, bleeding (4) Future family planning options
disorders, infections, CPD. (see Table 4.1).
2164_Ch04_115-248 29/03/12 12:22 PM Page 146

146 chapter 4 Health Promotion and Maintenance

e. Goal: assist in achievement of normal develop- 5. Evaluation/outcome criteria:


mental tasks. Encourage exploration of new a. Experiences normal, uncomplicated pregnancy,
role and responsibilities. labor, and birth of a newborn who is normal
f. Goal: referral to appropriate resources. and healthy.
(1) Abortion; adoption resources. b. Expresses satisfaction with decision and
(2) Preparation for childbirth and parenting outcome of this pregnancy.
classes. C. Older mother: multipara over age 40
(3) Family counseling. 1. General aspects
(4) Social services. a. Increased incidence of preexisting and coexist-
g. Goal: assist in facilitating/continuing/completing ing medical disorders (hypertension, diabetes,
basic education. arthritis).
(1) Communicate with school nurse. b. Increased incidence of complications of
(2) Explore other options available in pregnancy (preeclampsia/eclampsia,
community. hemorrhage).
5. Evaluation/outcome criteria: c. Smoking is major risk factor.
a. Makes informed decisions appropriate to 2. Assessment:
individual and family needs, desires. a. Same as for other women who are
b. Actively participates in own health pregnant.
maintenance. b. Reaction to pregnancy (varies from pleasure
(1) Complies with medical/nursing at still being young enough, to despair, if
recommendations. facing decision to abort).
MATERNAL/INFANT

(2) Minimizes potential for complications of c. History, signs and symptoms of coexisting
pregnancy. disorders.
c. Copes effectively with normal physiological d. Indications of reduced physical ability to
and psychosocial alterations of pregnancy. cope with normal physiological alterations
d. Both woman and babys father express of pregnancy.
satisfaction with decision and management e. Family constellation: stage of family develop-
of this pregnancy. If parenthood is chosen mental cycle, responses to this pregnancy
and pregnancy is successful, accepts parent- (especially adolescents reaction to parents
ing role. pregnancy).
B. Older mother: primigravida over age 35 3. Analysis/nursing diagnosis: same as for
1. General aspectshigher incidence of congenital over-35 age group.
anomalies (e.g., Down syndrome), increased 4. Nursing care plan/implementation:
possibility of complications of pregnancy. a. Goal: emotional support. Encourage verbaliza-
However, generally it is a conscious decision to tion of feelings, fears, concerns.
have postponed childbearing. Individuals are b. Goal: referral to appropriate resource.
usually used to making own decisions regarding (1) Genetic counseling.
career and health care. (2) Abortion/support groups.
2. Assessment: (3) Preparation for childbirth and parenthood
a. Same as for other women who are pregnant. classes.
b. Reaction to reality of pregnancy. c. Goal: facilitate/support effective family process.
c. Family response to pregnancy. Involve family in preparation for birth and
3. Analysis/nursing diagnosis: integration of newborn into family unit.
a. Fear related to threat to pregnancy. d. Othersame as for other women who are
b. Knowledge deficit related to aspects of preg- pregnant.
nancy care. 5. Evaluation/outcome criteria:
4. Nursing care plan/implementation: a. Makes informed decisions related to
a. Goal: anticipatory guidance. Preparation for pregnancy.
parenthood, altered lifestyle, potential change b. Expresses satisfaction with decision and
of career. Assist with realistic expectations. outcome of this pregnancy.
Refer to over 30 parents support group. c. Experiences uncomplicated pregnancy, labor,
b. Goal: health teaching. Explain, discuss and birth of a newborn who is normal and
special diagnostic procedures (Fig. 4.5) healthy.
(Amniocentesis). D. AIDS
c. Othersame as for other women who are 1. General aspectsAIDS is a serious condition
pregnant. affecting the immune system. Heterosexual
2164_Ch04_115-248 29/03/12 12:22 PM Page 147

Lifestyle Choices and Influences That Impact Health in Pregnancy and Newborn 147
AMNIOCENTESIS

(GREATLY ENLARGED) CELLS


IN AMNIOTIC CAVITY

SYRINGE

INTRAUTERINE CAVITY

UMBILICAL
CORD
PLACENTA

FETUS

CHORIONIC VILLI

WALL OF
UTERUS

MATERNAL/INFANT
CERVICAL CANAL

CERVIX

SAMPLE OF AMNIOTIC FLUID


IS REMOVED, THEN THE UTERUS
NEEDLE IS REMOVED

Figure 4.5 Amniocentesis. (From Venes, D [ed]:


Tabers Cyclopedic Medical Dictionary, ed 20. FA Davis,
Philadelphia, 2005, p 92.)

women are considered at risk if they or their c. Chronic diarrhea.


sexual partners: d. HIV positive.
a. Are HIV positive. e. Weight loss: 10 lb in 2 months.
b. Use IV drugs (50%). f. Night sweats; lymphadenopathy.
c. Received blood between 1977 and g. Skin lesions; thrush.
1985 (9%). h. Pelvic inflammatory disease (PID); STIs;
d. Are homosexual or bisexual men (39%). vulvovaginitis (usually, yeast [Candidiasis]),
e. Have hemophilia. often refractory and severe.
2. Assessmentgeneral symptoms: i. Cervical cytologic abnormalities; often
a. Malaise. infected with human papillomavirus
b. Chronic cough; possible tuberculosis. (HPV).
2164_Ch04_115-248 29/03/12 12:22 PM Page 148

148 chapter 4 Health Promotion and Maintenance

3. Analysis/nursing diagnosis: who is infected (i.e., breastfeeding). Classic


a. Altered nutrition, less than body requirements, signs evident in adult often not present.
related to general malaise. Common signs: lymphadenopathy,
b. Fatigue, related to altered health status, weight hepatosplenomegaly, oral candidiasis,
loss. bacterial infections, failure to thrive.
c. Fear related to progressively debilitating b. Implement standard precautions for all inva-
disease. sive procedures. Bathe infant immediately
d. Knowledge deficit related to disease progres- after birth to decrease contact with mothers
sion, treatment, life expectancy. blood. Wear gloves for all contact before
e. Ineffective individual coping related to disease first bath.
progression. c. Provide supportive nursing care (thermoregu-
4. Nursing care plan/implementation: lation, respiratory).
a. Identify women at risk. d. Encourage parent-infant contact.
b. Protect confidentiality. e. Provide opportunities for sensory stimuli
c. Implement standard precautions. and touch.
d. Use proper gloves, gown, hand washing. f. Monitor intake and weight gain.
e. Use protective eyewear and mask during g. Observe for signs of infection.
labor, birth. h. Initiate social service consultation.
5. Evaluation/outcome criteria: i. Counsel family about vaccinations (should
a. No further transmission of virus. receive all except oral polio).
b. Womans confidentiality maintained. j. Administer medications as ordered
MATERNAL/INFANT

c. Standard precautions implemented. (zidovudine [AZT]).


d. Emotional support implemented.
e. Supportive groups contacted.
6. Women who are HIV positivepregnancy Common Complications
management:
a. Antepartum of Pregnancy
(1) Increased incidence of other STIs
(gonorrhea, syphilis, herpes, HPV). First-Trimester Complications
(2) Increased incidence of cytomegalovirus I. COMPLICATIONS AFFECTING FLUID-GAS
(CMV). TRANSPORT: HEMORRHAGIC DISORDERS
(3) Differential diagnosis for all pregnancy- A. General aspects (review Table 4.8)
induced complaints. 1. Assessment:
(4) Counsel regarding nutrition. a. Vital signs, output, general status.
(5) Advise about risk to infant. b. Evidence of internal/external bleeding.
(6) Counsel regarding safer sex. c. Pain.
b. Intrapartum d. Emotional response.
(1) Focus on prevention of transmission. e. Perineal pads saturated and number
(2) Mode of birth not based on disease. (pad count).
(3) External electronic fetal monitoring f. Speculum examination.
(EFM) preferred. 2. Analysis/nursing diagnosis:
(4) Avoid use of fetal scalp electrodes or fetal a. Knowledge deficit related to diagnosis, progno-
scalp sampling. sis, treatment, sequelae.
c. Postpartum b. Anxiety/fear related to loss of pregnancy,
(1) No remarkable alteration in disease surgery.
progression. c. Fluid volume deficit, potential/actual, related
(2) Breastfeeding contraindicated. to excessive blood loss.
(3) Implement standard precautions for d. Pain.
mother and infant. e. Ineffective coping, individual/family, related to
(4) Refer to specialists in AIDS care and knowledge deficit and fear.
treatment. f. Anticipatory/dysfunctional grieving, related to
7. Newborn or neonate: loss of pregnancy.
a. General aspects: Neonatal AIDStransmission g. Disturbance in self-esteem, body image, role
may be transplacental, contact with maternal performance, related to threat to self-image as
blood at birth, or postnatal exposure to parent woman and childbearer.
2164_Ch04_115-248 29/03/12 12:22 PM Page 149

Common Complications of Pregnancy 149

Table 4.8
Emergency Conditions
First Trimester
Assessment/Observations Possible Problem Nursing Care Plan/Implementation
Fluid-Gas Transport
a. Crampingwith or without bleeding or passage Abortion (before 24 Bedrest, sedation, avoid coitusif
of tissue weeks): threatened, immi- threatened; bedrest, start IV fluids
nent, incomplete, septic and draw blood for laboratory work:
CBC, type/crossmatch, electrolytes,
platelets, HCG levels
b. Passage of tissue (products of conception; Hydatidiform mole Vital signs q515 min, prn
grapelike vesicles) or brown spotting; fundus too (trophoblastic disease)
high for gestational age; blood pressure elevated.
Often associated with hyperemesis gravidarum
and preeclampsia
c. Severe pain, shock out of proportion to amount Ectopic pregnancy Save all pads or tissue passed
of overt blood; shoulder-strap pain (Kehrs sign), through vagina for physician
a referred pain that indicates intra-abdominal evaluation
bleeding (or rupture of ovarian cyst); amenorrhea No rectal or vaginal examination until
of 612 weeks physician is present
d. Malodorous discharge; hyperthermia and chills; Septic abortion (self- Take complete history, if possible

MATERNAL/INFANT
tender abdomen induced or criminal) Convulsion precautions if hypertensive
e. Ecchymosis or bleedingwith a history that Missed abortion with Emotional support for loss of preg-
includes any or all of the following: had possible DIC (retained nancy (through nurses manner, tone
symptoms of pregnancy, but they subsided; dead fetus syndrome) of voice, touch, use of womans
pregnancy test negative; uterine size name; keep her informed of what is
diminishing; no FHR happening); oxygen, prn
Second Trimester
Assessment/Observations Possible Problem Nursing Care Plan/Implementation
Fluid-Gas Transport
a. Cramping; passage of products of conception Late abortion Same as for first trimester
b. Laborcervical changes, show Incompetent cervical os See physician immediately for
possible cerclage
c. Prolonged nausea and vomiting; unexplained Hydatidiform mole Maintain hydration; assess for
hypertension or preeclampsia; passage of dark dehydration; refer to physician
blood or grapelike vesicles; absent FHRs;
excessive fundal height for gestation
Sensory-Perceptual
a. Preeclampsia/eclampsia With increased severity: Pharmacological management of
renal failure, circulatory hypertension (see Chapter 6)
collapse, stroke, coagula-
tion defects (DIC); abruptio
placentae; convulsions
Assessment: hypertension first noted after Convulsion precautions:
24 weeks; followed by increased proteinuria
Symptoms: blurred or double vision; pain: 1. Emergency tray at bedside
headache, epigastric (late sign) 2. Oxygen/suction
3. Start IV
Signs: BP 160/110; 3+ proteinuria 4. Padded siderails
5. Limit environmental stimulation
Edema: facial, digital; pulmonary 6. Constant observation
7. Deep tendon reflexes
Oliguria 8. Daily weight
9. I&Ostrict
Hyperreflexia 10. Note any complaints and
changes
11. Prepare for lab work (type and
crossmatch, CBC, platelets, BUN
and creatinine, uric acid, SGOT,
SGPT)
Continued
2164_Ch04_115-248 29/03/12 12:22 PM Page 150

150 chapter 4 Health Promotion and Maintenance

Table 4.8
Emergency Conditionscontd
Second Trimester
Assessment/Observations Possible Problem Nursing Care Plan/Implementation
Fluid-Gas Transport
b. Convulsions in absence of hypertension, Stroke, epilepsy, drug Convulsion care:
proteinuria, or facial edema toxicity; intracranial injury; 1. Oxygen/mask; drugs (Valium,
diabetic complications; magnesium sulfate IV)
encephalopathy 2. Observe:
a. Uterine tone, FHR, fetal activity
b. Signs of labor
3. Emotional support for woman and
family
Third Trimester
Assessment/Observations Possible Problem Nursing Care Plan/Implementation
Fluid-Gas Transport
a. Bleeding: painless, bright red, vaginal Placenta previa No vaginal examination
Apply fetal monitor; assess for
labor
Contractions or uterine tone normal Position: semi- to high Fowlers
Ultrasound to verify placental
MATERNAL/INFANT

b. Pain: abdomen rigid and tender to touch Abruptio placentae location


As for placenta previa;
Increased uterine tone; signs of shock position: Sims
disproportionate to visible blood loss; may have Prepare for possible emer-
loss of FHTs; associated with: preeclampsia, gency cesarean delivery
multiparity, precipitous labor, oxytocin induction,
trauma, cocaine use

3. Nursing care plan/implementation: 2. Assessment: types


a. Goal: minimize blood loss, stabilize physiologi- a. Threatenedmild bleeding, spotting, cramp-
cal status. ing; cervix closed.
(1) Facilitate prompt medical management. b. Inevitablemoderate bleeding, painful
(2) Administer IV fluids, blood, as ordered. cramping; cervix dilated, positive nitrazine
(3) Administer analgesics, as needed. test (membranes ruptured).
b. Goal: prevent infection. Strict aseptic c. Imminentprofuse bleeding, severe cramp-
technique. ing, urge to bear down.
c. Goal: emotional support. d. Incompletefetal parts or fetus expelled;
(1) Encourage verbalization of anxiety, fears, placenta and membranes retained.
concerns. e. Completeall products of conception
(2) Supportive care for grief reaction expelled; minimal vaginal bleeding.
(see pp. 730732). f. Habitual/recurrenthistory of
4. Evaluation/outcome criteria: spontaneous loss of three or more
a. Blood loss minimized; physiological status successive pregnancies.
stable. g. Missedfetal death with no spontaneous
b. Copes effectively with loss of pregnancy. expulsion within 4 weeks.
B. Spontaneous abortion: before viable age of 20 to (1) Anorexia, malaise, headache.
22 weeks (2) Fundal heightinconsistent with
1. Etiology: gestational estimate.
a. Defective products of conception. (3)Laboratoryprolonged clotting time,
b. Insufficient production of progesterone. due to resultant concurrent hypofibrino-
c. Acute infections. genemia (disseminated intravascular
d. Reproductive system abnormalities coagulation [DIC], a major threat to
(e.g., incompetent cervical os). mother).
e. Trauma (physical or emotional). h. Elective abortions (intentionally induced loss
f. Rh incompatibility. of pregnancy).
2164_Ch04_115-248 29/03/12 12:22 PM Page 151

Common Complications of Pregnancy 151


3. Analysis/nursing diagnosis: 2. Etiologygenetic base of complete mole (sperm
a. Altered family processes related to pregnancy, enters empty egg and its chromosomes replicate;
circumstances surrounding abortion. 23 pairs of chromosomes are all paternal); rare
b. Sexual dysfunction related to compromised complication; more common in women over
self-image, altered interpersonal relationship, 45 years of age and women who are Asian.
guilt feelings. 3. Assessment:
4. Nursing care plan/implementation: a. Uterusrapid enlargement; fundal height
a. ThreatenedGoal: health teaching. Suggest: inconsistent with gestational estimate.
avoid coitus and orgasm, especially around b. Brownish dischargebeginning about
normal time for menstrual period. week 12; may contain vesicles.
b. Incomplete, inevitable, imminent. c. Signs and symptoms of preeclampsia/
(1) Goal: safeguard status. eclampsia (before third trimester), increased
(a) Save all pads, clots, tissue for expert incidence of hyperemesis gravidarum.
diagnosis. d. Medical evaluationprocedures:
(b) Report immediately any change in (1) Sonography, x-ray, amniographyno fetal
status, excessive bleeding, signs of parts present; snowstorm.
infection, shock. (2) Laboratory testfor elevated human
(c) Prepare for surgery. chorionic gonadotropin (HCG) levels.
(2) Goal: comfort measures. (3) Follow-up surveillance of HCG levels for
(a) Administer analgesics, as necessary. at least 1 year; persistent HCG level is
(b) Bedrest, quiet diversional activities. consistent with choriocarcinoma; x-ray.

MATERNAL/INFANT
(3) Goal: emotional support. 4. Analysis/nursing diagnosis:
(a) Encourage verbalization of fear, concerns. a. Anxiety/fear related to treatment, possible seque-
(b) Reduce anxiety, as possible. lae of hydatidiform mole (choriocarcinoma).
(c) If pregnancy terminates, facilitate griev- b. Potential for injury related to hemorrhage,
ing process; assist in working through perforation of uterine wall, preeclampsia/
guilt feelings (see pp. 183184). eclampsia.
(d) Supportive care for grief reaction c. Fluid volume deficit related to hemorrhage.
(see pp. 730732). 5. Nursing care plan/implementation:
(4) Goal: prevent isoimmunization (see a. Medical management
II. C. 5. Rh incompatibility, p. 140). (1) Monitor for preeclampsia.
(5) Medical management: (2) Evacuate the uterushysterectomy may
(a) Laboratoryblood type and Rh factor, be necessary.
indirect Coombs test, platelets, serum (3) Strict contraception for at least 1 year to
fibrinogen, clotting time. enable accurate assessment of status.
(b) Replace blood loss; maintain fluid levels (4) Choriocarcinomachemotherapy
with IV. (methotrexate plus dactinomycin) or
(c) Dilation and curettage or dilation and radiation therapy, or both.
evacuation. b. Nursing management
(d) Habitualdetermine etiology. (1) Goal: safeguard status. Observe for hemor-
5. Evaluation/outcome criteria: rhage, passage of retained vesicles and
a. Threatenedresponds to medical/nursing abdominal pain, or signs of infection
regimen; abortion avoided, successfully carries (because woman is at risk for perforation
pregnancy to term. of uterine wall).
b. Spontaneous abortionafter uterus emptied. (2) Goal: health teaching.
(1) Bleeding is controlled. (a) Explain, discuss diagnostic tests;
(2) Vital signs are stable. prepare for tests.
(3) Copes effectively with loss of pregnancy. (b) Discuss contraceptive options.
(4) Expresses satisfaction with care. (c) Importance of follow-up.
c. Habitual abortioncause identified and (3) Goal: preoperative and postoperative care.
corrected; carries subsequent pregnancy to (4) Goal: emotional support. Facilitate grieving.
successful termination. 6. Evaluation/outcome criteria:
C. Hydatidiform mole (complete) a. Verbalizes understanding of diagnosis, tests,
1. Pathophysiologychorionic villi degenerate and treatment.
into grapelike cluster of vesicles; may be b. Complies with medical/nursing
antecedent to choriocarcinoma. recommendations.
2164_Ch04_115-248 29/03/12 12:22 PM Page 152

152 chapter 4 Health Promotion and Maintenance

c. Tolerates surgical procedure well. spotting, some symptoms of pregnancy;


(1) Bleeding controlled. possible dull pain on affected side.
(2) Vital signs stable. b. Impending or posttubal rupturesudden,
(3) Urinary output adequate. acute, lower abdominal pain; nausea and
d. Copes effectively with loss of pregnancy. vomiting; signs of shock; referred shoulder
e. Returns for follow-up care/surveillance. pain (Kehrs sign) or neck paindue to
f. Selects and effectively implements method blood in peritoneal cavity; blood in
of contraception; avoids pregnancy for 1 year cul-de-sac may rectal pressure.
or more. c. Sharp, localized pain when cervix is touched
g. Tests for HCG remain negative for 1 year; during vaginal examination; shock and circu-
no evidence of malignancy. latory collapse in some, usually following
h. Achieves a pregnancy when desired. vaginal examination.
i. Successfully carries pregnancy to term; d. Positive pregnancy test in many women.
normal, uncomplicated birth of viable 5. Analysis/nursing diagnosis:
infant. a. Fear related to abdominal pain and pregnancy
D. Ectopic pregnancy (Fig. 4.6) status.
1. Pathophysiologyimplantation outside of b. Grief related to pregnancy loss.
uterine cavity. 6. Nursing care plan/implementation:
2. Types: a. Medical management: Methotrexate, a folic
a. Tubal (most common). acid antagonist, which acts by inhibiting
b. Cervical. cell division (may be used in early ectopic
MATERNAL/INFANT

c. Abdominal. pregnancy).
d. Ovarian. b. Surgical removal/repair.
3. Etiology: c. Nursing management:
a. PIDpelvic salpingitis and endometritis. (1) Goal: preoperative and postoperative care,
b. 43% caused by STI-related factors: 25%, health teaching.
chlamydial; 20%, previous STI. (2) Goal: supportive care for grief reaction;
c. Tubal or uterine anomalies, tubal spasm. encourage verbalization of anxiety and
d. Adhesions from PID or past surgeries. concerns of further pregnancies.
e. Presence of IUD. 7. Evaluation/outcome criteria:
4. Assessment: dependent on implantation site. a. Woman experiences uncomplicated postoper-
a. Early signsabnormal menstrual period ative course.
(usually following a missed menstrual period), b. Woman copes effectively with loss of pregnancy.

UTERUS
ISTHMIC
AMPULLAR
INTRALIGAMENTOUS

INFUNDIBULAR
OVARIAN

FIMBRIAL
INTRAMURAL
CERVICAL

ABDOMINAL

Figure 4.6 Ectopic pregnancy. (From Venes, D


[ed]: Tabers Cyclopedic Medical Dictionary, ed 19. FA Davis,
VARIOUS SITES OF ECTOPIC PREGNANCY
Philadelphia, 2001.)
2164_Ch04_115-248 29/03/12 12:22 PM Page 153

Common Complications of Pregnancy 153


II. COMPLICATIONS AFFECTING NUTRITION/ 2. Goal: minimize environmental stimuli.
ELIMINATION: HYPEREMESIS GRAVIDARUM a. Limit visitors and phone calls.
A. Pathophysiologypernicious vomiting during b. Bedrest with bathroom privileges.
first 14 to 16 weeks (peak incidence around 3. Goal: emotional support.
10 weeks of gestation); excessive vomiting at a. Establish accepting, supportive environment.
any time during pregnancy. Potential hazards b. Encourage verbalization of anxiety, fears,
include the following: concerns.
1. Dehydration with fluid and electrolyte imbalance. c. Support positive self-image.
2. Starvation, with loss of 5% or more of body F. Evaluation/outcome criteria:
weight; protein and vitamin deficiencies. 1. Womans signs and symptoms subside; she takes
3. Metabolic acidosisdue to breakdown of fat oral nourishment and gains weight.
stores to meet metabolic needs. 2. Womans pregnancy continues to term without
4. Hypovolemia and hemoconcentration; increased recurrence of hyperemesis.
blood urea nitrogen (BUN); decreased urinary III. COMPLICATIONS AFFECTING PROTECTIVE
output. FUNCTION: SEXUALLY-TRANSMITTED
5. Embryonic or fetal death may result, and the INFECTIONS (STIS). This category measures
woman may suffer irreversible metabolic changes applications of knowledge about conditions related
or death. to clients capacity to maintain defenses and prevent
B. Etiology: physical and chemical trauma, injury, infection, and
1. Physiologicalsecretion of HCG, decrease in threats to health status.
free gastric HCl, decreased gastrointestinal

MATERNAL/INFANT
A. Vaginitisinflammation of vagina.
motility. Increased incidence in hydatidiform 1. Pathophysiologylocal inflammatory reaction
mole and multifetal pregnancy (due to high (redness, heat, irritation/tenderness, pain). May
levels of HCG). cause preterm labor in pregnancy.
2. Psychologicalthought to be related to rejection 2. Etiology:
of pregnancy or sexual relations. a. Common causative organisms:
C. Assessment: (1)Bacteriastreptococci, Escherichia
1. Intractable vomiting. coli, gonococci, Chlamydia, bacterial
2. Abdominal pain. vaginosis.
3. Hiccups. (2) Virusesherpes simplex virus type 2,
4. Marked weight loss. CMV, HPV
5. Dehydrationthirst, tachycardia, skin turgor. (3) ProtozoaTrichomonas vaginalis.
6. Increased respiratory rate (metabolic acidosis). (4) FungiCandida albicans.
7. Laboratoryelevated BUN. b. Atrophic changesdue to declining hormone
8. Medical evaluation: rule out other causes level (women who are postmenopausal).
(infection, tumors). 3. Assessment: differentiate among common
D. Analysis/nursing diagnosis: vaginal infections:
1. Altered nutrition, less than body requirements, a. Vulvovaginal erythema.
related to inability to retain oral feedings. b. Pruritus, dysuria, dyspareunia.
2. Fluid volume deficit related to dehydration. c. Vaginal dischargecolor, consistency.
3. Ineffective individual coping related to symptoms, 4. Analysis/nursing diagnosis: pain related to
insecurity in role, psychological stress of unwanted inflammation, discharge.
pregnancy. 5. Nursing care plan/implementation:
4. Personal identity disturbance related to symptoms a. Goal: emotional support.
or perception of self as inadequate in role, sick, b. Goal: health teaching. Instruct woman in
socially unpresentable. self-care measures to promote comfort and
E. Nursing care plan/implementation: healing:
1. Goal: physiological stability. (1) Perineal care.
a. Rest GI tract (keep NPO) (e.g., maintain (2) Sitz baths.
IV fluids, parenteral nutrition). (3) Douching (as ordered). Not recommended
b. Progress diet, as ordered; present small during pregnancy.
feedings attractively; carbohydrates, fat, (4) Exposing vulva to air.
acidic foods. (5) Cotton briefs.
c. Weigh daily, assess hydration; note weight (6) Proper insertion of vaginal suppository.
gain. (7) Antibiotic use, as ordered.
d. Antiemetics (IV, suppository).
2164_Ch04_115-248 29/03/12 12:22 PM Page 154

154 chapter 4 Health Promotion and Maintenance

c. Goal: prevent reinfection. (2) Medical diagnosisprocedure: urethral


(1) Suggest sexual partner use condom until discharge Gram stain.
infection is eliminatedor abstain from c. Women:
intercourse. (1)Often asymptomatic; acute infection:
(2) Recommend sexual partner seek examina- severe vulvovaginal inflammation,
tion and treatment. venereal warts, greenish-yellow vaginal
d. Goal: medical consultation/treatment. Refer for discharge.
diagnosis and treatment. (2) Medical diagnosisprocedure: endocervical
6. Evaluation/outcome criteria: culture.
a. Woman is asymptomatic; unable to recover d. Gonococcal urethritis (men and women)
organism from body fluids or tissue. sudden severe dysuria, frequency, burning,
b. Woman avoids reinfection. edema.
c. Woman carries pregnancy to term without e. Salpingitis/oophoritissevere, sudden
complications. abdominal pain, fever (with or without
B. Gonorrhea vaginal discharge).
1. Pathophysiology: 5. Analysis/nursing diagnosis: impaired tissue
a. Menearly infection usually confined to integrity related to tissue inflammation.
urethra, vestibular glands, anus, or pharynx. 6. Nursing care plan/implementation:
Untreated: ascending infection may involve a. Goal: emotional support.
testes, causing sterility. b. Goal: health teaching to prevent transmission,
b. Womenearly infection usually confined to sequelae, reinfection.
MATERNAL/INFANT

vestibular glands, endocervix, urethra, anus (1) Need for accurate diagnosis and effective
(vagina is resistant). May ascend to involve treatment, follow-up examination in 7 to
pelvic structures (e.g., PID: fallopian tubes, 14 days, and culture.
ovaries); scarring may cause sterility. (2) All sexual partners need examination,
c. Women who are pregnantmay result in treatment.
preterm rupture of membranes, amnionitis, (3) Possible sequelae/complications (sterility,
preterm labor, postpartum salpingitis. carrier state).
d. Sequelae (untreated): c. Goal: medical consultation/treatment.
(1) May develop carrier state (asymptomatic; (1) Determine allergy to antibiotics.
organism resident in vestibular glands). (2) Refer for diagnosis and treatment.
(2) Systemic spread may result in gonococcal: (a) Diagnosis: culture.
(a) Arthritis. (b) Treatmentceftriaxone IM, plus doxy-
(b) Endocarditis. cycline PO. May use erythromycin or
(c) Meningitis. spectinomycin in pregnancy.
(d) Septicemia. (c) Follow-up culture before birth.
e. Newbornophthalmia neonatorum (d) Notification of sexual partners.
(gonococcal conjunctivitis). Untreated 7. Evaluation/outcome criteria:
sequela: blindness. a. Verbalizes understanding of mode of
2. Etiology: gram-negative diplococcus (Neisseria transmission, prevention, importance
gonorrhoeae). of examination, treatment of sexual
3. Epidemiology: contacts.
a. Portal of entryoral or genitourinary mucous b. Informs sexual contacts of need for
membranes. examination.
b. Mode of transmissionusually sexual c. Returns for follow-up examinations.
contact. d. Successfully treated; weekly follow-up
c. Incubation period: 2 to 5 days; may be cultures: negative on two successive visits.
asymptomatic. e. Avoids reinfection.
d. Communicable periodas long as organisms C. Chlamydia trachomatis
are present; to 4 days after antibiotic therapy 1. Pathophysiology:
begun. a. Most common sexually transmitted infection
4. Assessment: in United States.
a. History of known (or suspected) contact. b. Initial infection mild in women; inflamma-
b. Men: tion of cervix with discharge.
(1) Complaint of mucoid or mucopurulent c. If untreated, may lead to urethritis, dysuria,
discharge. PID, tubal occlusion, infertility.
2164_Ch04_115-248 29/03/12 12:22 PM Page 155

Common Complications of Pregnancy 155


2. Etiology: f. Woman who is pregnantvaginal bleeding,
a. Chlamydia trachomatis has maternal-fetal spontaneous abortion, fetal death.
effects. g. May shed virus for 7 weeks.
b. Bacteria can exist only within living cells. h. Medical diagnosis: multinucleated giant cells
c. Transmission is by direct contact from one in microscopic examination of lesion exudate;
person to another. culture for herpes simplex virus (HSV).
3. Assessmentmaternal: 5. Analysis/nursing diagnosis:
a. Inflamed cervix (may be asymptomatic). a. Pain related to inflammation process.
b. Cervical congestion, edema. b. Fear related to longevity of disease.
c. Mucopurulent discharge. c. Fear related to no cure for disease.
4. Assessmentfetal-neonatal: d. Knowledge deficit related to transmission to
a. Increased incidence of stillbirth. future partners, suppressive treatment.
b. Preterm birth may result. 6. Nursing care plan/implementation:
c. Contact with infected mucus occurs during a. Goal: emotional support.
birth. b. Goal: health teaching.
d. Newborn may be asymptomatic. (1)Virus remains in body for life (dormant,
e. Conjunctivitis; may lead to scarring. noninfectious) in 25% to 30% of
f. Chlamydial pneumonia. population; small percentage have
5. Analysis/nursing diagnosis: symptoms.
a. Pain related to inflamed reproductive organs. (2) Recurrence probable; usually shorter and
b. Fatigue related to inflammation. milder.

MATERNAL/INFANT
c. Knowledge deficit related to mode of treatment, (3)Need for close surveillance during preg-
disease transmission. nancy; cesarean birth may be indicated
6. Nursing care plan/implementation: if woman has active genital lesions or
a. Treatment with antibiotics, generally doxy- positive culture.
cycline or azithromycin. Erythromycin in c. Goal: promote comfort.
pregnancy. d. Goal: accurate definitive treatment. Refer for
b. Provide pain relief, analgesics. diagnosis and treatment.
c. Counsel regarding use of condoms, spermicidal (1) Diagnosiscervical smears, labial and
agents (containing nonoxynol-9) to prevent vaginal smears.
reinfection. (2) Treatmentacyclovir; used for suppressive
7. Evaluation/outcome criteria: treatment only.
a. Woman understands treatment and shows 7. Evaluation/outcome criteria:
compliance. a. Woman remains asymptomatic.
b. Woman understands portal of entry and risk b. Pregnancy continues to term with no
for reinfection. newborn effects.
D. Herpes genitalis E. Syphilis
1. Pathophysiologyinitial infection: varies in 1. Pathophysiology:
severity of symptoms, may be local or systemic; a. Primary stage: nonreactive RPR.
duration: prolonged; morbidity: severe. (1) Men: 3 to 4 weeks after contact, painless,
2. EtiologyHerpes simplex virus type 2. localized penile/anal ulcer (chancre);
3. Epidemiology: lymph nodesenlarged, regional.
a. Portal of entryskin, mucous membranes. (2) Women: often asymptomatic; labial,
b. Mode of transmissionusually sexual. vaginal, or cervical chancre.
c. Incubation: 3 to 14 days. (3)Medical diagnosisprocedure: dark-
d. Communicable periodwhile organisms are field microscopic examination of lesion
present. exudate.
4. Assessment: b. Secondary stage: reactive Venereal Disease
a. Lesionspainful, red papules; pustular Research Laboratories (VDRL).
vesicles that break and form wet ulcers that (1) 6 to 8 weeks after infection.
later crust; self-limiting (3 weeks). (2) Rashmacular, papular; on trunk, palms,
b. Severe itching, tingling, or pain. soles.
c. Dischargecopious; foul smelling. (3) Malaise, headache, sore throat, weight loss,
d. Dysuria. low-grade temperature.
e. Lymph nodesenlarged, inflammatory, c. Latent stage: reactive serologic test for syphilis
inguinal. (STS). Asymptomatic; noninfectious.
2164_Ch04_115-248 29/03/12 12:22 PM Page 156

156 chapter 4 Health Promotion and Maintenance

d. Tertiary stage: (b) Other stages7.2 million units over


(1) Gumma formation in skin, cardiovascular 3-week period.
system, or central nervous system. (c) Erythromycin or doxycycline for clients
(2) Psychosis. who are allergic to penicillin.
2. Etiology: Treponema pallidum (spirochete). 7. Evaluation/outcome criteria:
3. Epidemiology: a. If treated by 18th week of pregnancy, congen-
a. Portal of entryskin, mucous membranes. ital syphilis is prevented.
b. Mode of transmissionusually sexual. b. Appropriate treatment after 18th week cures
c. Incubation period9 days to 3 months. both mother and fetus; however, any fetal
d. Communicable periodprimary and damage occurring before treatment is irre-
secondary stages. versible.
4. Assessment: c. Follow-up VDRL: nonreactive at 1, 3, 6, 9,
a. Primarychancre, when detectable. and 12 months.
(1) Medical diagnosisprocedure: dark-field d. Tertiarycerebrospinal fluid examination
examination of lesion exudate. negative at 6 months and 1 year following
b. Secondary: treatment.
(1) Malaise, lymphadenopathy, headache, e. Verbalizes understanding of mode of
elevated temperature. transmission, potential sequelae without
(2) Macular, papular rash on palms and soles; treatment, importance of examination/
may be disseminated. treatment of sexual contacts, preventive
(3) Medical diagnosis(see d., following). techniques.
MATERNAL/INFANT

c. Tertiary: f. Informs contacts of need for examination.


(1) Subcutaneous nodules (gumma). g. Returns for follow-up visit.
(2) Note: Gumma formation may affect any h. Avoids reinfection.
body system; symptoms associated with F. Pelvic inflammatory disease (PID)
area of involvement. 1. Pathophysiologyascending pelvic infection;
d. Medical diagnosisprocedures: stages may involve fallopian tubes (salpingitis), ovaries
other than primarySTS: VDRL, RPR, (oophoritis); may develop pelvic abscess (most
T. pallidum immobilization (TPI), fluores- common complication), pelvic cellulitis, pelvic
cent treponemal antibody absorption (FTA). thrombophlebitis, peritonitis.
False-positive STS in: collagen diseases, 2. Etiology:
infectious mononucleosis, malaria, a. Chlamydia trachomatis.
systemic tuberculosis. b. Gonococci.
5. Analysis/nursing diagnosis: c. Streptococci.
a. Pain related to inflammation process. d. Staphylococci.
b. Knowledge deficit related to treatment and 3. Assessment:
transmission of the disease. a. Pain: acute, abdominal.
6. Nursing care plan/implementation: b. Vaginal discharge: foul smelling.
a. Goal: emotional support c. Fever, chills, malaise.
(1) Nonjudgmental. d. Elevated white blood cell (WBC) count.
(2) Caring, supportive manner. 4. Analysis/nursing diagnosis:
b. Goal: health teaching. a. Pain related to occluded tubules.
(1) Need for accurate diagnosis and treatment, b. Infertility related to permanent block of
follow-up examinations. tubes.
(2) All sexual partners need examination and c. Knowledge deficit related to transmission of
treatment. disease.
c. Goal: medical consultation/treatment. d. Altered urinary elimination related to dysuria.
(1)Refer for diagnosis and treatment. Note: 5. Nursing care plan/implementationfor
In pregnancytreatment by 18th gesta- woman who is hospitalized:
tional week prevents congenital syphilis a. Goal: emotional support.
in neonate; however, treat at time of b. Goal: limit extension of infection.
diagnosis. (1) Bedrestposition: semi-Fowlers, to
(2) Treatment: promote drainage.
(a) Primary, secondarybenzathine (2) Force fluids to 3000 mL/day.
penicillin G, 2.4 million units. (3) Administer antibiotics, as ordered.
2164_Ch04_115-248 29/03/12 12:22 PM Page 157

Common Complications of Pregnancy 157


c. Goal: prevent autoinocculation/transmission. b. Goal: health teaching.
(1) Strict aseptic technique (hand washing, (1) Avoid: strenuous physical activity;
perineal care). straining, infection.
(2) Contact-item isolation. (2) Report promptly: signs of labor (vaginal
d. Goal: Health teaching: if untreated: high risk bleeding, cramping).
of tubal scarring, sterility, or ectopic pregnan- (3) Need for continued, close health
cy; pelvic adhesions; transmission of disease. surveillance.
e. Goal: promote comfort. F. Evaluation/outcome criterion: woman car-
(1) Analgesics, as ordered. ries pregnancy to successful termination.
(2) External heat, as ordered. II. COMPLICATIONS AFFECTING SENSORY/
6. Evaluation/outcome criteria: PERCEPTUAL FUNCTIONS: PREGNANCY-
a. Woman responds to therapy; uneventful INDUCED HYPERTENSION (PIH);
recovery. PREECLAMPSIA/ECLAMPSIA
b. Woman avoids reinfection.
A. Pathophysiology:
1. Generalized arteriospasm increased peripheral
Second-Trimester Complications resistance, decreased tissue perfusion, and
(see Table 4.8) hypertension.
2. Kidney:
I. COMPLICATIONS AFFECTING COMFORT, a. Reduced renal perfusion and vasospasm
REST, MOBILITY: INCOMPETENT CERVIX glomerular lesions.
b. Damage to membrane loss of serum

MATERNAL/INFANT
A. Pathophysiologyinability of cervix to support
growing weight of pregnancy; associated with protein (albuminuria). Note: Reduced serum
repeated spontaneous second trimester abortion. albumin/globulin (A/G) ratio alters blood
B. Etiology: osmolarity edema.
1. Unknown. c. Increased tubular reabsorption of sodium
2. Congenital defect in cervical musculature increased water retention (edema).
(exposure to diethylstilbestrol [DES]). d. Release of angiotensin contributes to
3. Cervical trauma during previous birth, abortion; vasospasm and hypertension.
aggressive, deep, or repeated dilation and 3. Brain: decreased oxygenation, cerebral edema,
curettage. and vasospasm visual disturbances and
C. Assessment: hyperirritability, convulsions, and coma.
1. History of habitual, second-trimester abortions. 4. Uterus: decreased placental perfusion
2. Painless, progressive cervical effacement and increased risk of SGA baby, abruptio placentae,
dilation during second trimester. oligohydramnios.
3. Signs of threatened abortion or (early third- B. Etiology: unknown. Risk factors:
trimester) preterm labor. 1. Pregnancyoccurs only when a functioning
D. Analysis/nursing diagnosis: trophoblast is present; more common in first
1. Pain related to early dilation. pregnancies. Onset: develops after week 20 of
2. Fear related to possible pregnancy loss. gestation, through labor, and up to 48 hours
E. Nursing care plan/implementation: postpartum.
1. Medical management 2. Coexisting conditionsdiabetes, multifetal
a. Cerclage surgical procedure (Shirodkar, gestation, polyhydramnios, renal disease.
McDonald). 3. Angiotensin gene T235.
2. Preoperative nursing management C. Assessmenttypes:
a. Goal: reduce physical stress on incompetent 1. Preeclampsiamild
cervix. Bedrest, supportive care. a. Hypertensionsystolic increase of 30 mm
b. Goal: emotional support. Encourage verbaliza- Hg or more over baseline; diastolic rise of
tion of anxiety, fear, concerns. 15 mm Hg or more.
c. Goal: health (preoperative) teaching. Explain b. Proteinuria1 gm/day.
procedurepurse-string suture encircles c. Edemadigital and periorbital; weight gain
cervix and reinforces musculature. over 0.45 kg (1 lb)/wk.
d. Goal: preparation for surgery. 2. Preeclampsiasevere
3. Postoperative nursing management a. Increasing hypertensionsystolic at or above
a. Goal: maximize surgical result. Bedrest, 160 mm Hg or more than 50 mm Hg over
supportive care. baseline; diastolic, 110 mm Hg or more.
2164_Ch04_115-248 29/03/12 12:22 PM Page 158

158 chapter 4 Health Promotion and Maintenance

b. Urine: proteinuria (5 gm or more in 24 hours); 7. Diversional activity deficit related to need for
oliguria (400 mL or less in 24 hours). reduced environmental stimuli, bedrest.
c. Hemoconcentration, hypoproteinemia, 8. Risk for injury related to seizure.
hypernatremia, hypovolemic condition. F. Prognosis:
d. Nausea and vomiting. 1. Goodsymptoms mild, respond to treatment.
e. Epigastric paindue to edema of liver capsule. 2. Poorconvulsions (number and duration);
f. Cerebral or visual disturbances (before persistent coma; hyperthermia, tachycardia
convulsive state): (120 beats/min); cyanosis, and liver damage.
(1) Disorientation and somnolence. 3. Terminalpulmonary edema, congestive heart
(2) Severe frontal headache. failure (CHF), acute renal failure, cerebral
(3) Increased irritability; hyperreflexia. hemorrhage. The earlier the symptoms appear,
(4) Visual disturbance: blurred vision, halo the poorer the outcome for the pregnancy.
vision, dimness, blind spots. G. Nursing care plan/implementation: Goal: health
g. HELLP syndrome (Hemolysis, Elevated Liver teaching.
enzymes, and Low Platelets). 1. Restfrequent naps in lateral Sims position.
3. Eclampsia 2. Immediate report of danger signs:
a. Tonic and clonic convulsions; coma. a. Digital and periorbital edema.
b. Renal shutdownoliguria, anuria. b. Severe headache, irritability.
D. Assessmentwoman who is hospitalized: c. Visual disturbances.
1. Vital signs (blood pressure in side-lying position, d. Epigastric pain.
pulse, respirations)q24h, while awake (if 3. Do roll-over test (blood pressure while on back
MATERNAL/INFANT

mild to moderate preeclampsia) or as necessary. and lateral positions).


Note: Record, report persistent hypertension. 4. Importance of regular prenatal visits.
2. Fetal heart tones at time of vital signs. 5. Monitoring own blood pressure between
3. Deep tendon reflexes (DTRs) and clonusto prenatal visits.
identify/monitor CNS hyperirritability. H. Nursing care plan/implementationwoman
4. I&Oto identify diuresis. (Note: Oliguria who is hospitalized:
indicates pathologic progression.) 1. Goal: reduce environmental stimuli, to minimize
5. Urinalysis (clean-catch specimen) for protein, stimulation of hyperirritable CNS. Limit visitors
daily or after each voiding, as necessary. and phone calls.
6. Signs of pathologic progression (see II. C. 2. Goal: emotional support.
Assessmenttypes). a. Encourage verbalization of anxiety, fears,
7. Signs of labor, abruptio placentae (Note: high concerns.
blood pressure, or a rapid drop, may initiate b. Explain all procedures, seizure precautions.
abruptio), DIC. 3. Goal: supportive care.
8. Emotional status. a. Encourage bedrestto increase tissue
9. Daily weight, amount/distribution of edema perfusion, promote diuresis.
(pitting; pedal, digital, periorbital)to identify b. Position: lateral Simsto reduce risk of
signs of mobilization of tissue fluid, diuresis. supine hypotensive syndrome.
E. Analysis/nursing diagnosis: 4. Goal: health teaching. Rest with reduced stimuli.
1. Fluid volume excess: hemoconcentration, edema 5. Goal: monitor and administer drugs as ordered.
related to altered blood osmolarity and sodium/ a. Anticonvulsants (especially magnesium
water retention. sulfate).
2. Altered nutrition, less than body requirements: b. Antihypertensives.
protein deficiency related to loss through c. Diuretics (used rarely, and only in presence
damaged renal membrane. of CHF).
3. Altered tissue perfusion related to increased d. Blood volume expanders.
peripheral resistance and vasospasm in renal, 6. Goal: seizure precautions. To safeguard maternal/
cardiovascular system. fetal status:
4. Altered urinary elimination: oliguria, anuria a. Observe for signs and symptoms of impending
related to hypovolemia, vasospasm. convulsion:
5. Sensory/perceptual alterations: visual disturbances, (1) Frontal headache.
hyperirritability related to cerebral edema, (2) Epigastric pain.
decreased oxygenation to brain. (3) Sharp cry.
6. Anxiety related to symptoms, implications of (4) Eyes: fixed, unresponsive.
pathophysiology. (5) Facial twitching.
2164_Ch04_115-248 29/03/12 12:22 PM Page 159

Common Complications of Pregnancy 159


b. Emergency items (suction equipment, airway, c. Boggy lower uterine segmentpalpated
drugs, IV fluids) immediately available. on vaginal examination. (Note: If placenta
7. Goal: convulsion care (woman with eclampsia). previa is suspected, internal examinations
a. Maintain patent airway; administer oxygen. are contraindicated.)
b. Safetypadded bed rails. d. Medical diagnosisprocedure: sonographyto
c. Reduce environmental stimuli: dim lights, determine placental site.
quiet. 2. Analysis/nursing diagnosis:
d. Observe, report, and record: a. Anxiety related to bleeding, outcome.
(1) Onset and progression of convulsion. b. Fluid volume deficit related to excessive
(2) If followed by coma or incontinence. blood loss.
e. Prepare for immediate cesarean delivery; c. Altered tissue perfusion related to blood loss.
check FHR. Close observation for d. Altered urinary elimination related to
48 hours postpartum, even if no further hypovolemia.
convulsions. e. Fear related to fetal injury or loss.
I. Evaluation/outcome criteria: 3. Nursing care plan/implementation:
1. Woman complies with medical/nursing plan a. Medical management
of care. (1) Sterile vaginal examination under double
2. Womans symptoms respond to treatment; setup.
progression halted. (2) Vaginal birth possible if bleeding minimal,
3. Woman carries uneventful pregnancy to marginal implantation; if fetal vertex is
successful termination. presenting so that presenting part acts as

MATERNAL/INFANT
tamponade.
Third-Trimester Complications (3) Cesarean birth for complete previa.
b. Nursing management. Goal: safeguard status.
(see Table 4.8) 4. Evaluation/outcome criteria: (Table 4.9)
I. COMPLICATIONS AFFECTING FLUID-GAS (see following section on abruptio placentae).
TRANSPORT B. Abruptio placentaepremature separation of
A. Placenta previaabnormal implantation; near or normally implanted placenta from uterine wall.
over internal cervical os. Increased incidence with 1. Assessment:
multiparas, multiple gestation, previous uterine a. Sudden-onset, severe abdominal pain.
surgery. b. Increased uterine tonemay contract
1. Assessment: unevenly, fails to relax between contractions;
a. Painless, bright red vaginal bleeding (may very tender.
be intermittent); absence of contractions, c. Shock usually more profound than expected
abdomen soft. on basis of external bleeding or internal
b. If in labor, contractions usually normal. bleeding.

Table 4.9
Comparison of Placenta Previa and Abruptio Placenta
Pathology Etiology Assessment Nursing Care Plan/Implementation
Placenta Previa
Types: More common with Painless, bright red No vaginal or rectal examinations
multiparity, advanced vaginal bleeding or enemas
maternal age
Marginallow-lying Fibroid tumors Usually manifests in Bedrest (high Fowlers if
8th month marginal previa)

Partialpartly covers Endometriosis Postpartum: signs of Continuous fetal monitor


internal cervical os hemorrhage, infection
Completecovers Old uterine scars Maternal vital signs q4h, or prn
internal cervical os Multiple gestation Note character and amount of
bleeding
Emotional support
Continued
2164_Ch04_115-248 29/03/12 12:22 PM Page 160

160 chapter 4 Health Promotion and Maintenance

Table 4.9
Comparison of Placenta Previa and Abruptio Placentacontd
Pathology Etiology Assessment Nursing Care Plan/Implementation
Abruptio Placenta
Types: Preeclampsia/eclampsia Pain: sudden, severe Position: supine; elevate
(right) hip

Partialsmall part Before birth of second Abdomen: rigid Monitor: vital signs, blood loss,
separates from twin Uterus: very tender to fetus
uterine wall Traction on cord touch
Completetotal Rupture of membranes Fetal hyperactivity; I&O (anuria, oliguria; hematuria)
placenta separates High parity bradycardia, death
from uterine wall
Retroplacental Chronic renal hypertension Shock: rapid, profound Prepare for surgery
bleeding (concealed) Port-wine amniotic fluid Emotional support
Marginaloccurs at Oxytocin induction/ Signs of DIC Fluid, blood replacement
edges; external augmentation of labor Postpartum: signs
bleeding Cocaine use of atony, infection,
Trauma pulmonary emboli
MATERNAL/INFANT

d. Medical evaluationprocedures: DIC screen- (2) Woman has minimal blood loss.
ing (bleeding time, platelet count, prothrom- (3) Womans assessment findings within
bin time, activated partial thromboplastin normal limits.
time, fibrinogen); sonogram to see placental (4) Woman retains capacity for further
hemoseparation. childbearing.
2. Analysis/nursing diagnosis: b. No evidence of complications (anemia, hypo-
a. Fluid volume deficit related to bleeding. tonia, DIC) during postpartum period.
b. Potential for fetal injury related to uteropla- II. COMPLICATIONS AFFECTING COMFORT,
cental insufficiency. REST, MOBILITY
c. Fear related to unknown outcome.
A. Polyhydramniosamniotic fluid over 2000 mL
3. Potential complications:
(normal volume: 5001200 mL).
a. Afibrinogenemia and DIC.
1. Etiology: unknown. Risk factors:
b. Couvelaire uterusbleeding into uterine
a. Maternal diabetes.
muscle.
b. Multifetal gestation.
c. Amniotic fluid embolus.
c. Erythroblastosis fetalis.
d. Hypovolemic shock.
d. Preeclampsia/eclampsia.
e. Renal failure.
e. Congenital anomalies (e.g., anencephaly,
f. Uterine atony, hemorrhage, infection in
upper-GI anomalies, such as esophageal
postpartum.
atresia).
4. Nursing care plan/implementation:
2. Assessment:
a. Medical management
a. Fundal height: excessive for gestational
(1) Control: hemorrhage, hypovolemic shock;
estimate.
replace blood loss.
b. Fetal parts: difficult to palpate, small in
(2) Cesarean birth.
proportion to uterine size.
(3) Fibrinogen, crystalloids, blood
c. Increased discomfortdue to large, heavy
replacement.
uterus.
(4) IV heparinby infusion pumpto
d. Increased edema in vulva and legs.
reduce coagulation and fibrinolysis.
e. Shortness of breath.
b. Nursing management. Goal: safeguard status.
f. GI discomfortheartburn, constipation.
5. Evaluation/outcome criteria:
g. Susceptibility to supine hypotensive syndrome
a. Experiences successful termination of
due to compression of inferior vena cava
pregnancy.
and descending aorta while in supine
(1) Woman gives birth to viable newborn
position.
(by vaginal or cesarean method).
2164_Ch04_115-248 29/03/12 12:22 PM Page 161

Diagnostic Tests 161


h. Medical diagnosisprocedures: (3) Goal: emotional support for loss of pregnancy
(1) Sonographyto diagnose multifetal (if applicable).
pregnancy, gross fetal anomaly, locate (a) Encourage verbalization of feelings.
placental site. (b) Facilitate grieving: permit parents to
(2) Amniocentesisto diagnose anomalies, see, hold infant; if desired, take photo-
erythroblastosis. graph, footprints for them.
3. Potential complications: 6. Evaluation/outcome criteria:
a. Maternal respiratory impairment. a. Woman complies with medical/nursing
b. Premature rupture of membranes (PROM) management.
with prolapsed cord or amnionitis. b. Womans symptoms of respiratory impair-
c. Preterm labor. ment, etc., reduced; comfort promoted.
d. Postpartum hemorrhagedue to overdisten- c. Woman experiences normal, uncomplicated
tion and uterine atony. pregnancy, labor, birth, and postpartum.
4. Analysis/nursing diagnosis: III. DIAGNOSTIC TESTS TO EVALUATE FETAL
a. Pain related to excessive size of uterus GROWTH AND WELL-BEING
impinging on diaphragm, stomach,
A. Daily fetal movement count (DFMC)
bladder.
1. Assesses fetal activity.
b. Impaired physical mobility related to increased
2. Noninvasive test done by woman who is pregnant.
lordotic curvature of back, increased weight
3. Five to 10 movements per hour: normal activity.
on legs.
4. Five movements or less per hour may indicate
c. Altered tissue perfusion related to decreased

MATERNAL/INFANT
fetal jeopardy or sudden change in movement
venous return from lower extremities, com-
pattern.
pression of body structures by overdistended
5. Assess for fetal sleep patterns.
uterus.
B. Nonstress test (NST)
d. Potential fluid volume deficit related to poten-
1. Correlates fetal movement with FHR. Requires
tial uterine atony in immediate postpartum,
electronic monitoring.
secondary to loss of contractility due to
2. Reactive testthree accelerations of FHR to
overdistention.
15 beats/min above baseline FHR, lasting
e. Sleep pattern disturbance related to respiratory
for 15 seconds or more, over 20-minute time
impairment and discomfort in side-lying
period.
position.
3. Nonreactive testno accelerations or acceleration
f. Anxiety related to discomfort, potential for
less than 15 beats/min above baseline FHR. May
complications associated with congenital
indicate fetal jeopardy. Vibroacoustic simulator
anomalies.
(VAS) to differentiate hypoxia from fetal sleep.
g. Altered urinary elimination (frequency)
4. Unsatisfactory testdata that cannot be inter-
related to pressure of overdistended uterus
preted or inadequate fetal activity; repeat.
on bladder.
C. Contraction stress test (CST); oxytocin challenge
5. Nursing care plan/implementation:
test (OCT)
a. Medical management
1. Correlates fetal heart rate response to sponta-
(1) Amniocentesisremove excess fluid very
neous or induced uterine contractions.
slowly, to prevent abruptio placentae.
2. Requires electronic monitoring.
(2) Termination of pregnancyif fetal abnor-
3. Indicator of uteroplacental sufficiency.
mality present and woman desires.
4. Identifies pregnancies at risk for fetal compro-
b. Nursing management
mise from uteroplacental insufficiency.
(1) Goal: health teaching.
5. Increasing doses of oxytocin are administered
(a) Need for lateral Sims position during
to stimulate uterine contractions until three in
resting; semi-Fowlers may alleviate
10-minute period.
respiratory embarrassment.
6. Interpretation: negative results indicate absence
(b) Explain diagnostic or treatment
of late decelerations with all contractions.
procedures.
7. Positive results indicate late FHR decelerations
(c) Signs and symptoms to be reported
with contractions.
immediately: bleeding, loss of fluid
8. Nipple stimulation (breast self-stimulation test)
through vagina, cramping.
may also release enough systemic oxytocin to
(2) Goal: prepare for diagnostic and/or
contract uterus to obtain CST. Instruct not to
treatment procedures.
do at home.
(a) Permission for amniocentesis.
2164_Ch04_115-248 29/03/12 12:22 PM Page 162

162 chapter 4 Health Promotion and Maintenance

D. Biophysical profile (BPP) 4. Evaluation of phospholipids; aids in determining


1. Observation by ultrasound of four variables lung maturity.
for 30 minutes and results of nonstress 5. Determination of creatinine levels; aids in
testing: determining fetal age. (Greater than 1.8 mg/dL
a. Fetal body movements. indicates fetal maturity and the fetal age.)
b. Fetal tone. 6. Assesses isoimmune disease.
c. Amniotic fluid volume. 7. Presence of meconium may indicate fetal
d. Fetal breathing movements. hypoxia.
2. Variables are scored at 2 for each variable if H. Chorionic villus sampling (CVS)
present, score of 0 if not present; score of less 1. Cervically invasive procedure.
than 6 is associated with perinatal mortality. 2. Advantageresults can be obtained after
E. Ultrasound 10 weeks of gestation due to fast-growing
1. Noninvasive procedure involving passage of fetal cells.
high-frequency sound waves through uterus to 3. Procedureremoval of small piece of tissue
obtain data regarding fetal growth, placental (chorionic villus) from fetal portion of placenta.
positioning, and the uterine cavity. Tissue reflects genetic makeup of fetus.
2. Purpose may include: 4. Determines some genetic aberrations and
a. Pregnancy confirmation. allows for earlier decision for induced abortion
b. Fetal viability. (if desired) from abnormal results. Does not
c. Estimation of fetal age. diagnose neural tube defects; clients who
d. Biparietal diameter (BPD) measurement. have CVS need further diagnoses with
MATERNAL/INFANT

e. Placenta location. ultrasound.


f. Detection of fetal abnormalities. 5. Protects pregnancy privacy because results can
g. Confirmation of fetal death. be obtained before the pregnancy is apparent
h. Identification of multifetal gestations. and decisions can be made regarding abortion
i. Amniotic fluid index. or continuation of gestation.
3. No risk to mother with infrequent use. Fetal risk 6. Risks involve: spontaneous abortion, infection,
not determined on long-term basis. hematoma, intrauterine death, Rh isoimmu-
F. Amniocentesis (see Fig. 4.5) nization, and fetal limb defects, if done before
1. Invasive procedure for amniotic fluid analysis to 9 weeks of gestation.
assess fetal lung maturity or disease; done after
14 weeks of gestation.
2. Needle placed through abdominal-uterine wall;
T H E I N T R A PA R T U M
designated amount of fluid is withdrawn for
EXPERIENCE
examination. General overview: This review of the anatomical and
3. Empty bladder if gestation greater than physiological determinants of successful labor provides
20 weeks. baseline data against which the nurse compares findings of
4. Risk of complications less than 1%. Ultrasound an ongoing assessment of the woman in labor. Nursing
always precedes this procedure. actions are planned and implemented to meet the present
5. Possible complications: onset of contractions; and emerging needs of the woman in labor.
infections (probably amnionitis); placental I. BIOLOGICAL FOUNDATIONS OF LABOR
punctures; cord puncture; bladder or fetal
A. Premonitory signs
puncture.
1. Lighteningprocess in which the fetus drops
6. Advise women to observe and report the
into the pelvic inlet.
following to physician: fetal hypoactivity or
a. Characteristics
hyperactivity, vaginal bleeding, vaginal dis-
(1) Nulliparausually occurs 2 to 3 weeks
charge (clear or colored), signs of labor, signs
before onset of labor.
of infection.
(2) Multiparacommonly occurs with onset
G. Analysis of amniotic fluid
of labor.
1. Chromosomal studies to detect genetic
b. Effects
aberrations.
(1) Relieves pressure on diaphragmbreathing
2. Biochemical analysis of fetal cells to detect
is easier.
inborn errors of metabolism.
(2) Increases pelvic pressure.
3. Determination of fetal lung maturity by assess-
(a) Urinary frequency returns.
ing lecithin/sphingomyelin ratio.
2164_Ch04_115-248 29/03/12 12:22 PM Page 163

The Intrapartum Experience 163


(b) Increased pressure on thighs. and opens (010 cm dilation) (Table 4.10).
(c) Increased tendency to vulvar, vaginal, Voluntary contraction of secondary abdominal
perianal, and leg varicosities. muscles during the second stage (e.g., pushing,
2. Braxton Hicks contractionsmay become more bearing-down) forces fetal descent. Changing
uncomfortable. pelvic dimensions force fetal head to accommo-
B. Etiology: unknown. Theories include: date to the birth canal by molding (cranial bones
1. Uterine overdistention. overlap to decrease head size).
2. Placental agingdeclining estrogen/progesterone Stages of labor:
levels. 1. Firstbegins with establishment of regular,
3. Rising prostaglandin level. rhythmic contractions; ends with complete
4. Fetal cortisol secretion. effacement and dilation (10 cm); divided into
5. Maternal/fetal oxytocin secretion. three phases:
C. Overview of labor processforces of labor a. Latent and early active.
(involuntary uterine contractions) overcome b. Active.
cervical resistance; cervix thins (effacement) c. Transitional.

Table 4.10
First Stage of Labor

MATERNAL/INFANT
Phases of First Stage Assessment: Expected Maternal Behaviors Nursing Care Plan/Implementation
04 cm: Latent Phase and Early Active Phase
1. Time: multipara 56 hours; 1. Usually comfortable, euphoric, excited, 1. Provide encouragement, feedback for
nullipara 810 hours, average talkative, and energetic, but may be relaxation, companionship, hydration,
fearful and withdrawn nutrition
2. Contractions: regular, mild, 2. Relieved or apprehensive that labor has 2. Coach during contractions: signal begin-
510 minutes apart, begun ning of contraction, mark the seconds,
2030 seconds duration signal end of contraction; Follow my
breathing, Watch my lips, etc.
3. Low-back pain and abdominal 3. Alert, usually receptive to teaching, 3. Comfort measures: position change for
discomfort with contractions coaching, diversion, and anticipatory comfort; praise; keep aware of progress;
guidance maintain hydration
4. Cervix thins: some bloody
show
5. Station: Multipara 2 to +1;
nullipara 0.
48 cm: Midactive Phase, Phase of Most Rapid Dilation
1. Average time: nullipara 1. Tired, less talkative, and less energetic 1. Coach during contractions; partner
12 hours; multipara (coach) may need some relief
11/22 hours
2. Contractions: 25 minutes 2. More serious, malar flush between 5 2. Comfort measures (to partner tooas
apart, 3040 seconds and 6 cm, tendency to hyperventilate, needed): position for comfort while pre-
duration, intensity increasing may need analgesia, needs constant venting hypotensive syndrome; encour-
coaching age relaxation, focusing her on areas of
tension; provide counterpressure to
sacrococcygeal area, prn; praise; keep
aware of progress; minimize distractions
from surrounding environment (loud talk-
ing, other noises); offer analgesics and
anesthetics, as appropriate; provide
hygiene: mouth care, ice chips, clean
perineum; warmth, as needed
3. Membranes may rupture now 3. Monitor progress of labor and maternal/
fetal response, color of fluid, time of
rupture of membranes (ROM)
4. Increased bloody show 4. If monitors are in use, attention on
mother; periodically check accuracy of
monitor readouts
5. Station: 1 to 0
Continued
2164_Ch04_115-248 29/03/12 12:22 PM Page 164

164 chapter 4 Health Promotion and Maintenance

Table 4.10
First Stage of Laborcontd
Phases of First Stage Assessment: Expected Maternal Behaviors Nursing Care Plan/Implementation
810 cm: Transition, Deceleration Period of Active Phase
1. Average time: nullipara 1. If not under regional anesthesia, more 1. Stay with woman (couple) and provide
40 minutes1 hour; multipara introverted; may be amnesic between constant support
20 minutes contractions
2. Contractions: 11/22 minutes 2. Feeling she cannot make it; increased 2. Continue to coach with contractions: may
apart, 6090 seconds duration, irritability, crying, nausea, vomiting, and need to remind, reassure, and encourage
strong intensity belching; increased perspiration over her to reestablish breathing techniques
upper lip and between breasts; leg and concentration with each contraction;
tremors; and shaking coach panting or he-he respirations to
prevent pushing
3. Increased vaginal show; rectal 3. May have uncontrollable urge to push at 3. Comfort measures; remind her and part-
pressure with beginning urge this time ner her behavior is normal and OK;
to bear down coach breathing to quell nausea; offer
ice chips
4. Station: +13 to +14 4. Assist with countertension techniques
woman requested
5. Monitor contractions, FHR (after each
contraction), vaginal discharge, perineal
bulging, maternal vital signs; record every
MATERNAL/INFANT

15 minutes
6. Assess for bladder filling
7. Keep mother (couple) aware of progress
8. Prepare partner for birth (scrub, gown, etc.)

2. Secondbegins with complete dilation and ends (b) Durationtime from beginning of
with birth of infant. contraction to its relaxation.
3. Thirdbegins with birth of infant and ends (c) Strength (intensity)resistance to
with expulsion of placenta. indentation.
4. Fourthbegins with expulsion of placenta; ends (d) False/true labordifferentiation
when maternal status is stable (usually 12 hours (Table 4.11).
postpartum). (e) Signs of dystocia (dysfunctional labor)
D. Anatomical/physiological determinants (see pp. 184186).
1. Maternal b. Pelvic structures and configuration:
a. Uterine contractionsinvoluntary; birth; begin (1) False pelvisabove linea terminalis (line
process of involution. travels across top of symphysis pubis
(1)Characteristics: rhythmic; increasing around to sacral promontory); supports
tone (increment), peak (acme), relaxation gravid uterus during pregnancy.
(decrement). (2) True pelvisbelow linea terminalis;
(2) Effects: divided into:
(a) Decreases blood flow to uterus and (a) Inletbrim, demarcated by linea
placenta. terminalis.
(b) Dilates cervix during first stage of labor. (i) Widest diameter: transverse.
(c) Raises maternal blood pressure during (ii) Narrowest diameter: anterior-
contractions. posterior (true conjugate).
(d) With voluntary bearing-down efforts (b) Midplanepelvic cavity.
(abdominal muscles), expels fetus (c) Outlet.
(second stage) and placenta (third (i) Widest diameter: anterior-posterior
stage). (requires internal rotation of fetal
(e) Begins involution. head for entry).
(3) Assessment: (ii) Narrowest diameter: transverse
(a) Frequencytime from beginning of (intertuberous); facilitates birth
one contraction to beginning of the in occipitoanterior (OA)
next. position.
2164_Ch04_115-248 29/03/12 12:22 PM Page 165

The Intrapartum Experience 165

Table 4.11
Assessment: Differentiation of False/True Labor
False Labor True Labor
Contractions: Braxton Hicks intensify (more noticeable at night); Contractions: begin in lower back, radiate to abdomen
short, irregular, little change (girdling), become regular, rhythmic; frequency, duration,
intensity increase
Discomfort: mostly abdominal and groin Discomfort: mostly low back
Relieved by change of position or activity (e.g., walking) Unaffected by change of position, activity, drinking two
glasses of water, or moderate analgesia
Cervical changesnone; no effacement or dilation progress Cervical changesprogressive effacement and dilation

(3) Classifications (b) Posteriortriangular; junction of


(a) Gynecoidnormal female pelvis; sagittal and lambdoid sutures; closes
rounded oval. by 4 months of age.
(b) Androidnormal male pelvis; funnel b. Fetal lierelationship of fetal long axis to
shaped. maternal long axis (spine).
(c) Anthropoidoval. (1) Transverseshoulder presents.
(d) Platypelloidflattened, transverse (2) Longitudinalvertex or breech presents.

MATERNAL/INFANT
oval. c. Presentationfetal part entering inlet first
2. Fetal (Fig. 4.8).
a. Fetal head (Fig. 4.7). (1) Cephalicvertex (most common); face,
(1) Bonesone occipital, two frontal, two brow.
parietals, two temporals. (2) Breech
(2)Sutureline of junction or closure (a) Completefeet and legs flexed on
between bones; sagittal (longitudinal), thighs; buttocks and feet presenting.
coronal (anterior), and lambdoid (b) Franklegs extended on torso, feet up
(posterior, frontal); permit molding to by shoulders; buttocks presenting.
accommodate head to birth canal. (c) Footlingsingle (one foot), double
(3)Fontanelsmembranous space between (both feet) presenting.
cranial bones during fetal life and d. Attituderelationship of fetal parts to one
infancy. another (e.g., head flexed on chest).
(a) Anterior soft spotdiamond e. Positionrelationship of presenting fetal part
shaped; junction of coronal and to quadrants of maternal pelvis; vertex most
sagittal sutures; closes (ossifies) by common, occiput anterior on maternal left
18 months. side (LOA) (see Fig. 4.8).

Figure 4.7 The fetal head.


Bones: two frontal, two
temporal, one occipital, two
parietal. Sutures: sagittal,
frontal, coronal, lambdoid.
Fontanels: anterior, posterior.
(From Clinical Education Aid No. 13.
Used with permission of Ross Products
Division, Abbott Laboratories, Inc.,
Columbus, OH 43216.)
2164_Ch04_115-248 29/03/12 12:22 PM Page 166

166 chapter 4 Health Promotion and Maintenance

A B C D
MATERNAL/INFANT

E F G
Figure 4.8 Categories of fetal presentation. (A) Left occipitoanterior (LOA): fetal occiput is in left anterior quadrant of mater-
nal pelvis. (B) Left occipitoposterior (LOP): fetal occiput is in left posterior quadrant of maternal pelvis. (C) Right occipitoanterior
(ROA): fetal occiput is in right anterior quadrant of maternal pelvis. (D) Right occipitoposterior (ROP): fetal occiput is in right
posterior quadrant of maternal pelvis. (E) Left sacroposterior (LSP): fetal sacrum is in left posterior quadrant of maternal pelvis.
(F) Shoulder presentation with fetus in transverse lie. (G) Prolapse of umbilical cord with fetus in LOA position. (From Clinical
Education Aid No. 18. Used with permission of Ross Products Division, Abbott Laboratories, Inc., Columbus, OH 43216.)

3. Assessment: determine presentation and (4) Factors affecting audibility:


position. (a) Obesity.
a. Leopolds maneuverabdominal palpation. (b) Maternal position.
(1) Firstpalms over fundus, breech feels (c) Polyhydramnios.
softer, not as round as head would be. (d) Maternal gastrointestinal activity.
(2) Secondpalms on either side of abdomen, (e) Loud uterine bruitorigin: hissing of
locates fetal back and small parts. blood through maternal uterine arteries;
(3) Thirdfingers just above pubic symph- synchronous with maternal pulse.
ysis, grasp lower abdomen; if unengaged, (f ) Loud funic souffleorigin: hissing of
presenting part is mobile. blood through umbilical arteries; syn-
(4) Fourthfacing mothers feet, run palms chronous with fetal heart rate (FHR).
down sides of abdomen to symphysis; (g) External noise, faulty equipment.
check for cephalic prominence (usually c. Vaginal examination: palpable sutures,
on right side), and if head is floating or fontanels (triangular-shaped superior, dia-
engaged. mond-shaped inferior = vertex presentation,
b. Location of fetal heart tones (FHTs)heard OA position).
best through fetal back or chest. 4. Cardinal movements of the mechanisms of
(1) Breech presentationusually most audible normal laborvertex presentation, positional
above maternal umbilicus. changes of fetal head accommodate to changing
(2) Vertex presentationusually most audible diameters of maternal pelvis (Fig. 4.9).
below maternal umbilicus. a. Descenthead engages and proceeds down
(3) Changing location of most audible birth canal.
FHTsuseful indicator of fetal b. Flexionhead bent to chest; presents smallest
descent. diameter of vertex (suboccipital-bregmatic).
2164_Ch04_115-248 29/03/12 12:22 PM Page 167

The Intrapartum Experience 167

A B C

MATERNAL/INFANT
D E F

G
Figure 4.9 Cardinal movements in the mechanism of labor with the fetus in vertex presentation. (A) Engagement, descent,
flexion. (B) Internal rotation. (C) Extension beginning (rotation complete). (D) Extension complete. (E) External rotation (restitution).
(F) External rotation (shoulder rotation). (G) Expulsion. (From Clinical Education Aid No. 13. Used with permission of Ross Products Division, Abbott
Laboratories, Inc., Columbus, OH 43216.)

c. Internal rotationduring second stage of b. Stationrelationship of presenting part to


labor, transverse diameter of fetal head enters ischial spines (IS).
pelvis; occiput rotates 90 degrees to bring (1) Floatingpresenting part above inlet, in
back of neck under symphysis (e.g., left false pelvis.
occipitotransverse [LOT] to LOA to OA); (2) Station 5 is at inlet (presenting part
presents smallest diameter (biparietal) to well above IS).
smallest diameter of outlet (intertuberous). (3)Station 0presenting part at IS
d. Extensionback of neck pivots under symph- (engaged).
ysis, allows head to be born by extension. (4) Station +4presenting part at the outlet.
e. Restitutionhead returns to normal align- E. Warning signs during labor
ment with shoulders (with LOA, results in 1. Contractionhypertonic, poor relaxation,
head facing right thigh), presents smallest or tetanic (greater than 90 seconds long and
diameter of shoulders to outlet. 2 minutes apart).
f. Delivery of headshoulders in anterior- 2. Abdominal painsharp, rigid abdomen.
posterior position. 3. Vaginal bleedingprofuse.
g. Expulsionbirth of neonate completed. 4. FHRlate decelerations, prolonged variable
5. Assessment: relationship of fetal head to ischial decelerations, bradycardia, tachycardia
spines (degree of descent). (Fig. 4.10), decreased variability.
a. Engagementwidest diameter of presenting 5. Maternal hypertension.
part has passed through pelvic inlet 6. Meconium-stained amniotic fluid (MSAF).
(e.g., biparietal diameter of fetal head). 7. Prolonged ROM.
2164_Ch04_115-248 29/03/12 12:22 PM Page 168

168 chapter 4 Health Promotion and Maintenance

Sterile vaginal examination (SVE)


to check for dilation, station.
MATERNAL/INFANT

4. Baseline variability.

may

Figure 4.10 Fetal heart rate (FHR) decelerations and nursing interventions.
2164_Ch04_115-248 29/03/12 12:22 PM Page 169

The Intrapartum Experience 169


II. PARTICIPATORY CHILDBIRTH TECHNIQUES c. Preexisting and coexisting medical disorders,
A. PsychoprophylaxisLamaze method allergies.
1. Premiseconditioned responses to stimuli d. Pregnancy-related health problems
occupy nerve pathways, reducing perception (hyperemesis, bleeding, etc.).
of pain. Emphasis is on childbirth as a natural e. Infectious diseases (past and present
event, with a woman who is informed as herpes, etc.).
the active participant. The ability to relax f. Past obstetric history, if any.
effectively reduces the perception of pain, g. Pelvic size estimation.
and the involvement of the coach fosters the h. Height.
family concept. i. Weight gain.
2. Childbirth partners are taught: j. Laboratory results:
a. Anatomy and physiology of labor. (1) Blood type and Rh factor.
b. Psychology of man and woman. (2) Serology.
c. What to expect in the birthing setting. (3) Urinalysis.
d. Conditioned responses to labor stimuli. (4) Hepatitis.
(1) Concentration on focal point. (5) Rubella.
(2) Breathing techniques. k. Prenatal care history.
(3) Need for active coaching to enable l. Use of medications.
woman to: 2. Admission findings
(a) Use techniques appropriate to present a. Emotional status.
stage of labor. b. Vital signs.

MATERNAL/INFANT
(b) Avoid hyperventilation. c. Present weight.
e. Specific stageappropriate techniques: d. Fundal height.
(1) First stage of laborearly: slow, deep chest e. Estimated fetal weight.
breathing; active: patterned breathing. f. Edema.
(2) Transition (810 cm)rapid, shallow g. Urinalysis (for protein and glucose).
breathing pattern, to prevent pushing 3. FHRnormal, 110 to 160 beats/min
prematurely. (see Fig. 4.10).
(a) Panting. a. Check and record every 15 to 30 minutes
(b) Pant-blow. monitor fetal response to physiological stress
(c) He-he pattern. of labor.
(3) Second stage of labor b. Bradycardia (mild, 100110 beats/min, or
(a) Pushing (or bearing-down)aids fetal 30 beats/min lower than baseline reading).
descent through birth canal. c. Tachycardia (moderate, 160179 beats/min,
(b) Pantingaids relaxation between or 30 beats/min above baseline reading
contractions; prevents explosive birth lasting 5 or more minutes).
of head. 4. Contractionsevery 15 to 30 minutes.
f. Effects on labor behaviors/coping: a. Place fingertips over fundus, use gentle
(1) Helps mother cope with and assist pressure; contraction felt as hardening or
contractions. tensing.
(2) Prevents premature bearing-down efforts; b. Time: frequency and duration.
reduces possibility of cervical lacerations, c. Intensity/strength at acme:
edema due to pushing on incompletely (1) Weakeasily indent fundus with fingers.
dilated cervix. (2) Moderatesome tension felt, fundus
(3) When appropriate, improves efficiency of indents slightly with finger pressure.
bearing-down efforts. (3) Strongunable to indent fundus.
B. Other methodsinclude parent classes, classes for 5. Maternal response to laborassess for effective
siblings, multiparas, and those who plan cesarean coping, cooperation, and using effective breath-
birth. ing techniques.
6. Maternal vital signsbetween contractions.
III. NURSING ACTIONS DURING FIRST STAGE
a. Response to pain or use of special breathing
OF LABOR
techniques alters pulse and respirations.
A. Assessment: careful evaluation of: b. BP, P, RRif normotensive: on admission,
1. Antepartal history and then every hour and prn; after regional
a. EDD anesthesia: every 30 minutes (every 5 minutes
b. Genetic and familial problems. first 20 minutes).
2164_Ch04_115-248 29/03/12 12:22 PM Page 170

170 chapter 4 Health Promotion and Maintenance

c. Temperatureif within normal range: 2. Ineffective individual coping related to lack of


on admission, and then every 4 hours preparation for childbirth or poor support from
and prn. Every 2 hours after rupture of coach.
membranes. 3. Altered nutrition: less than body requirements relat-
d. Before and after analgesia/anesthesia. ed to physiological stress of labor.
e. After rupture of membranes (see Amniotic 4. Altered urinary elimination related to pressure of
fluid embolus, pp. 186187). presenting part.
7. Character and amount of bloody show. 5. Altered thought processes related to sleep depriva-
8. Bladder status: encourage voiding every 1 to tion, transition, analgesia.
2 hours, monitor output. 6. Fluid volume deficit related to anemia, excessive
a. Determine bladder distentionpalpate blood loss.
just above symphysis (full bladder may 7. Impaired (fetal) gas exchange related to impaired
impede labor progress or result in trauma placental perfusion.
to bladder). C. Nursing care plan/implementation:
b. Admission urinalysischeck for protein 1. Goal: comfort measures.
and glucose. a. Maintain hydration of oral mucosa.
9. Signs of deviations from normal patterns. Encourage sucking on cool washcloth, ice
10. Status of membranes: chips, lollipops, clear liquids (if ordered).
a. Intact. b. Reduce dryness of lips. Apply lip balm.
b. Ruptured (nitrazine paper turns blue on c. Relieve backache. Apply sacral counterpres-
contact with alkaline amniotic fluid). sure (particularly with occipitoposterior
MATERNAL/INFANT

Fluid may be placed on a glass slide to [OP] presentation).


dry; a fernlike crystallization of sodium d. Encourage significant other to participate.
chloride will appear. Note, record, and e. Encourage ambulation when presenting part
report: engaged.
(1) Timedanger of infection if ruptured 2. Goal: management of physical needs.
more than 24 hours. a. Encourage frequent voiding to prevent full
(2) FHR stat and 10 minutes laterto bladder from impeding oncoming head.
check for prolapsed cord. b. Encourage ambulation throughout labor;
(3) Character and color of fluid lateral Sims position with head elevated to:
(see 11.b. and c.). (1) Encourage relaxation.
11. Amniotic fluid. (2) Allow gravity to assist in anterior rotation
a. Amountpolyhydramnios (>2000 mL) of fetal head.
associated with congenital anomalies/poorly (3) Prevent compression of inferior vena cava
controlled diabetes. and descending aorta (supine hypotensive
b. Characterthick consistency or odor syndrome).
associated with infection. (4) Promote placental perfusion.
c. Colornormally clear with white specks. c. Perineal prep, if ordered to promote
(1) Yellowpresence of bilirubin; Rh or cleanliness.
ABO incompatibility. d. Fleets enema, if orderedto stimulate
(2) Green or meconium stained; if fetus in peristalsis, evacuate lower bowel. Note:
vertex position, indicates recent fetal contraindicated if:
hypoxia secondary to respiratory distress (1) Cervical dilation (4 cm or more) with
in fetus. unengaged headdue to possibility of
(3) Port winemay indicate abruptio cord prolapse.
placentae. (2) Fetal malpresentation/malpositiondue to
12. Labor progress: possible fetal distress.
a. Effacement. (3) Preterm labormay stimulate contractions.
b. Dilation. (4) Painless vaginal bleedingdue to possible
c. Station. placenta previa.
d. Bulging membranes. 3. Goal: management of psychosocial needs.
e. Molding of fetal head. Emotional support:
13. Perineumobserve for bulging. a. Encourage verbalization of feelings, fears,
B. Analysis/nursing diagnosis: concerns.
1. Anxiety, fear related to uncertain outcome, b. Explain all procedures.
pain. c. Reinforce self-concept (Youre doing well!).
2164_Ch04_115-248 29/03/12 12:22 PM Page 171

The Intrapartum Experience 171


4. Goal: management of discomfort. 8. Response to regional anesthesia, if
a. Analgesia or anesthesiamay be required administered.
or desiredto facilitate safe, comfortable a. Signs of hypotensionreduces placental
birth. perfusion, increases risk of fetal hypoxia.
b. Support/enhance/teach childbirth techniques. b. Effect on contractionsnote and report any
(1) Reinforce appropriate breathing techniques slowing of labor progress.
for current labor status. 9. Efforts to bear downincreases expulsive
(a) If woman is hyperventilating, to effects of uterine contractions.
increase PaCO2, minimize fetal acidosis, 10. Perineal bulging with contractionsfetal head
and relieve symptoms of vertigo and distends perineum, crowns; head born by
syncope, suggest: extension.
(i) Breathe into paper bag. B. Analysis/nursing diagnosis:
(ii) Breathe into cupped hands. 1. Pain related to strong uterine contractions,
(b) Demonstrate appropriate breathing for pressure of fetal descent, stretching of
several contractionsto reestablish rate perineum.
and rhythm. 2. High risk for injury:
5. Goal: sustain motivation. a. Infection related to ruptured membranes,
a. Offer support, encouragement, and praise, as repeated vaginal examinations.
appropriate. b. Laceration related to pressure of fetal head
b. Keep informed of status and progress. exceeding perineal elasticity.
c. Reassure that irritability is normal. 3. Impaired skin integrity related to laceration,

MATERNAL/INFANT
d. Serve as surrogate coach when necessary episiotomy.
(if no partner, before partner arrives, while 4. Fluid volume deficit related to hypotension
partner changes clothes, during needed secondary to regional anesthesia.
breaks); assist with effleurage, breathing, 5. Anxiety related to imminent birth of fetus.
focusing. 6. Ineffective individual coping related to prolonged
e. Discourage bearing-down efforts by pant- sensory stimulation (contractions) and anxiety.
blow until complete (10-cm) dilation to 7. Altered urinary elimination related to anesthesia
avoid cervical edema/laceration. and contractions, descent of fetal head.
f. Facilitate informed decision making regarding 8. Sleep pattern disturbance.
medication for relaxation or pain relief. C. Nursing care plan/implementation:
g. Minimize distractions: quiet, relaxed 1. Goal: emotional support.
environment; privacy. a. To sustain motivation/control:
D. Evaluation/outcome criteria: (1) Never leave mother and significant other
1. Woman manages own labor discomfort alone during second stage.
effectively. (2) Keep informed of progress.
2. Woman maintains control over own behavior. (3) Direct bearing-down efforts (pushing)
3. Woman successfully completes first stage of without holding breath* while pushing.
labor without incident. Encourage pushing out through vagina
IV. NURSING ACTIONS DURING SECOND and encourage mother to touch crowning
STAGE OF LABOR head; position mirror so woman can see
perineal bulging with effective efforts;
A. Assessment:
minimize distractions.
1. Maternal (or couples) response to labor.
b. To allay significant others anxiety: reassure
2. FHRcontinuous electronic monitoring, or
regarding mothers behavior if she is not
after each contraction with fetoscope, Doppler.
anesthetized.
3. Vital signs.
c. Support family choices.
4. Time elapsedaverage: 2 minutes to 1 hour;
2. Goal: safeguard status.
prolonged second stage increases risk of: fetal
a. Precautions when putting legs in stirrups:
distress, maternal exhaustion, psychological
(1) If varicosities, do not put legs in stirrups.
stress, intrauterine infection.
5. Contraction patternaverage every 11/2 to
3 minutes, lasting 60 to 90 seconds.
*The woman must be discouraged from using the Valsalva maneuver (holding ones breath
6. Vaginal dischargeincreases. and tightening abdominal muscles) for pushing during the second stage. This activity
7. Nausea, vomiting, disorientation, tremors, increases intrathoracic pressure, reduces venous return, and increases venous pressure.
Cardiac output and blood pressure increase, and pulse slows temporarily. During the
amnesia between contractions, panic. Valsalva maneuver, fetal hypoxia may occur. The process is reversed when the woman
takes a breath.
2164_Ch04_115-248 29/03/12 12:22 PM Page 172

172 chapter 4 Health Promotion and Maintenance

(2) Avoid pressure to popliteal veins; pad C. Nursing care plan/implementation:


stirrups. 1. Goal: prevent uterine atony. Administer oxytocin,
(3) Ensure proper, even alignment by adjust- as ordered.
ing stirrups. 2. Goal: facilitate parent-child bonding.
(4) Move legs simultaneously into or out of a. While protecting neonate from cold stress,
stirrupsto avoid nerve, ligament, and encourage parents to see, hold, touch
muscle strain. neonate.
(5) Provide proper support to woman not b. Comment about neonates individuality,
using stirrups. Do not hold legs (can cause characteristics, and behaviors.
back injury). c. After neonate is assessed for congenital anom-
b. Support woman in whatever position selected alies (e.g., cleft palate, esophageal atresia),
for birth (e.g., side-lying position). encourage breastfeeding, if desired.
c. Cleanse perineum, thighs as ordered. 3. Goal: health teaching.
3. Goal: maintain a comfortable environment. a. Describe, discuss common neonatal behavior
a. Free of unnecessary noise, light. in transitional period (periods of reactivity,
b. Comfortable temperature (warm). sleep, hyperactivity).
4. Medical management: b. Demonstrate removal of mucus by aspiration
a. Episiotomy may be performed to facilitate with bulb syringe.
birth. c. Demonstrate ways of facilitating breastfeeding.
b. Forceps may be applied to exert traction and D. Evaluation/outcome criteria:
expedite birth. 1. Woman has a successful, uneventful completion
MATERNAL/INFANT

c. Vacuum extraction also used to assist birth. of labor.


5. Birthing room birth with alternative positions a. Minimal blood loss.
(squat). b. Vital signs within normal limits.
D. Evaluation/outcome criteria: c. Fundus well contracted at level of umbilicus.
1. Cooperative, actively participates in birth; 2. Parents express satisfaction with outcome,
maintains control over own behavior. demonstrate infant attachment.
2. Successful, uncomplicated birth of viable VI. NURSING ACTIONS DURING THE
infant. FOURTH STAGE OF LABOR1 TO
3. All assessment findings within normal limits 2 HOURS POSTPARTUM
(vital signs, emotional status, response to birth).
A. Assessmentevery 15 minutes four times; then,
4. Presence of significant other.
every 30 minutes two timesor until stableto
V. NURSING ACTIONS DURING THIRD STAGE monitor response to physiological stress of labor/
OF LABOR birth.
A. Assessment: 1. Vital signs:
1. Time elapsedaverage: 5 minutes; prolonged a. Temperature taken once; if elevated, requires
third stage (greater than 25 minutes) may follow-upmay indicate infection, dehydra-
indicate complications (placenta accreta). tion, excessive blood loss. Note, record, report
2. Signs of placental separation: temperature of 100.4F (38C).
a. Increase in bleeding from the vagina. b. Blood pressureevery 15 minutes 4.
b. Cord lengthens. (1) Returns to prelabor leveldue to loss of
c. Uterus rises in abdomen, assumes globular placental circulation and increased circu-
shape. lating blood volume.
3. Assess mothers level of consciousness. (2) Elevation may be in response to use
4. Examine placenta for intactness and number of of oxytocic drugs or preeclampsia (first
vessels in umbilical cord (normal: three. Note: 48 hours).
two vessels onlyassociated with increased (3) Lowered blood pressuremay reflect
incidence of congenital anomalies); condition significant blood loss during labor/birth,
of placenta for calcification, infarcts, etc. or occult bleeding.
B. Analysis/nursing diagnosis: c. Pulseevery 15 minutes 4.
1. Family coping: potential for growth related to bond- (1) Physiological bradycardiadue to normal
ing, beginning achievement of developmental vagal response.
tasks. (2) Tachycardiamay indicate excessive blood
2. Fluid volume deficit related to blood loss during loss during labor/birth, dehydration, exhaus-
third stage. tion, maternal fever, or occult bleeding.
2164_Ch04_115-248 29/03/12 12:22 PM Page 173

The Intrapartum Experience 173


2. Location and tone of fundusevery 15 minutes 6. Knowledge deficit related to self-care procedures,
4 to ensure continuing contraction; prevent infant care.
blood loss due to uterine relaxation. 7. Fatigue related to sleep disturbances and
a. Fundusfirm; at or slightly lower than the anxiety.
umbilicus; in midline. 8. Anxiety regarding status of self and infant.
b. May be displaced by distended bladder 9. Altered nutrition, less than body requirements,
due to normal diuresis; common cause of related to decreased food and fluid intake
bleeding in immediate postpartum, uterine during labor.
atony. C. Nursing care plan/implementation:
3. Character and amount of vaginal flow. 1. Goal: comfort measures.
a. Moderate lochia rubra. a. Position, pad change.
b. Excessive loss: if perineal pad saturated in b. Perineal careto promote healing; to reduce
15 minutes, or blood pools under possibility of infection.
buttocks. c. Ice pack to perineum; as orderedto reduce
c. Bright red bleeding may indicate cervical or edema, discomfort, and pain related to
vaginal laceration. hemorrhoids.
4. Perineum. 2. Goal: nutrition/hydration. Offer fluids, foods as
a. Edema. tolerated.
b. Bruisingdue to trauma. 3. Goal: urinary elimination.
c. Distention/hematoma, rectal pain. a. Encourage voidingto avoid bladder
5. Bladder fullness/voidingto prevent distention.

MATERNAL/INFANT
distention. b. Record: time, amount, character.
6. Rate of IV, if present; response to added c. Anticipatory guidance related to nocturnal
medication, if any. diuresis and increased output.
7. Intake and outputto evaluate hydration. 4. Goal: promote bonding.
8. Recovery from analgesia/anesthesia. a. Provide privacy, quiet; encourage sustained
9. Energy level. contact with newborn.
10. Verbal, nonverbal interaction between woman b. Encourage: touching, holding baby; breast-
and significant other. feeding (also promotes involution).
a. Dialogue. 5. Goal: health teaching.
b. Posture. a. Perineal carefront to back, labia closed
c. Facial expressions. (after each void/bowel movement).
d. Touching. b. Hand washingbefore and after each pad
11. Interactions between parent(s) and newborn; change; after voiding, defecating; before and
signs of bonding (culturally appropriate). after baby care.
a. Eye contact with newborn. c. Signs to report:
b. Calls by name. (1) Uterine cramping/ pain.
c. Explores with fingertips, strokes, cuddles. (2) Increased vaginal bleeding, passage of large
12. Signs of postpartum emergencies: clots.
a. Uterine atony, hemorrhage. (3) Nausea, dizziness.
b. Vaginal hematoma. D. Evaluation/outcome criteria:
B. Analysis/nursing diagnosis: 1. Expresses comfort, satisfaction in fourth
1. Fluid volume deficit related to excessive stage.
intrapartum blood loss, dehydration. 2. Vital signs stable, fundus contracted, moderate
2. Altered urinary elimination related to intra- lochia rubra, perineum undistended.
partum bladder trauma, dehydration, blood 3. Tolerates food and fluids well.
loss. 4. Voids an adequate amount.
3. Impaired skin integrity related to episiotomy, 5. Demonstrates culturally appropriate contact
lacerations, cesarean birth. with infant.
4. Altered family processes related to role 6. Verbalizes abnormal signs to report to
change. physician.
5. Altered parenting related to interruption in 7. Returns demonstration of appropriate perineal
bonding secondary to: care.
a. Compromised maternal status. 8. Ambulates without pain, dizziness, numbness
b. Compromised neonatal status. of legs.
2164_Ch04_115-248 29/03/12 12:22 PM Page 174

174 chapter 4 Health Promotion and Maintenance

VII. NURSING MANAGEMENT OF THE 6. High risk for injury (biochemical, metabolic)
NEWBORN IMMEDIATELY AFTER BIRTH related to impaired thermoregulation.
A. Assessment: 7. Ineffective thermoregulation related to environ-
1. Mucus in nasopharynx, oropharynx. mental conditions/prematurity.
2. Apgar score: note and recordat 1 and C. Nursing care plan/implementation:
5 minutes of age (Table 4.12). 1. Goal: ensure patent airway.
a. Score of 710: good condition. a. Suction mouth first, then nose; when stimu-
b. Score of 46: fair condition; assess for CNS lated, sensitive receptors around entrance to
depression. nares initiate gasp, causing aspiration of
c. Score of 03: poor condition; requires imme- mucus present in mouth.
diate intervention. Asphyxia neonatorumfails b. Suction with bulb syringe.
to breathe spontaneously within 3060 seconds (1) If deeper suctioning necessary, use DeLee
after birth; heart rate (HR) <100. Mucus Trap attached to suction. Oral use
3. Number of vessels in umbilical stump. of DeLee is contraindicated due to risk of
4. Passage of meconium stool, urine. contact with babys secretions (new DeLee
5. General physical appearance/status. now available that has no such risk).
a. Signs of respiratory distress (nasal flaring, (2) Avoid prolonged, vigorous suctioning.
grunting, sternal retraction, cyanosis, (a) Reduces oxygenation.
tachypnea). (b) May traumatize tissue, cause edema,
b. Skin condition (meconium stained, cyanosis, bleeding, laryngospasm, and cardiac
jaundice, lesions). arrhythmia.
MATERNAL/INFANT

c. Crypresence, pitch, quality. c. Assist gravity drainage of fluids. Position: head


d. Signs of birth trauma (lacerations, disloca- dependent (Trendelenburg) and side-lying
tions, fractures). position.
e. Symmetry (absent parts, extra digits, gross 2. Goal: maintain body temperatureto conserve
malformations, ears, palm creases, sacral energy, preserve store of brown fat, decrease
dimples). oxygen needs; prevent acidosis. Prevent chilling:
f. Molding, caput succedaneum, a. Minimize exposure; dry quickly.
cephalohematoma. b. Keep warm; apply hat.
g. Assess gestational age. c. Take temperature hourly until stable.
6. Identify high-risk infant. 3. Goal: identify infant:
B. Analysis/nursing diagnosis: a. Apply Identiband.
1. Ineffective airway clearance related to excessive b. Take infants footprints and maternal
nasopharyngeal mucus. fingerprints.
2. Ineffective breathing pattern related to CNS 4. Goal: prevent eye infection (gonorrheal and chlamy-
depression secondary to intrauterine hypoxia dial ophthalmia neonatorum). Within 1 hour of
narcosis, prematurity, and lack of pulmonary birth, apply antibiotic ointment in each eye.
surfactant. 5. Goal: facilitate prompt identification/vigilance for
3. Impaired gas exchange related to respiratory potential neonatal complications.
distress. a. Record significant data from mothers chart:
4. Fluid volume deficit related to birth trauma; (1) History of: pregnancy, diabetes, hyperten-
hemolytic jaundice. sion, current drug abuse, excessive caffeine,
5. Impaired skin integrity related to cord stump. medications, alcohol, malnutrition.

Table 4.12
Apgar Score
Sign 0 1 2
Heart rate Absent <100 >100
Respiratory effort Absent Slow and irregular Good and strong, loud crying
Activity: muscle tone Flaccid Some flexion or extremities Active motions, general flexion
Reflex irritability No response to stimuli Weak cry or grimace Cry: vigorous
Appearance: color Blue, pale Body pink, extremities blue Completely pink
2164_Ch04_115-248 29/03/12 12:22 PM Page 175

Nurse-Attended Emergency Birth 175


(2) Course of labor, evidence of fetal distress, 3. Goal: prevent infection.
medications received in labor. a. Provide clean field for birth.
(3) Birth history of anesthesia. b. Avoid touching birth canal without gloved
(4) Apgar score; resuscitative efforts. hands.
6. Goal: facilitate prompt identification/intervention c. Support perineum (and advancing head) with
in hemolytic problems of the newborn. sterile (or clean) towel.
a. Collect and send cord blood for appropriate 4. Goal: prevent, or minimize, infant hypoxia and
tests: perineal lacerations.
(1) Blood type and Rh factor. a. If membranes intact as head emerges, tear at
(2) Coombs test. neck to facilitate first breath.
b. Give vitamin K to facilitate clotting. b. Feel for cord around neck (if present, and if
D. Evaluation/outcome criteria: successful transition possible, slip cord over head; if tight, and
to extrauterine life. sterile equipment at hand, clamp cord in
1. Status satisfactory; all assessment findings two places, cut between clamps, unwrap
within normal limits. cord). If unsterile environment, keep fetus
2. Responsive in bonding process with and placenta attacheddo not cut cord.
parents. 5. Goal: facilitate/assist birth.
VIII. NURSE-ATTENDED EMERGENCY BIRTH a. Hold head in both hands.
(PRECIPITATE BIRTH)When woman presents b. Apply gentle downward pressure to
without prenatal care (to emergency department), bring anterior shoulder under pubic
may represent drug abuse. symphysis.

MATERNAL/INFANT
c. Gently lift head to ease birth of posterior
IMMINENT BIRTH
shoulder.
A. Assessment: identify signs of imminent birth:
d. Support infant as body slips free of mothers
1. Strong contractions.
body.
2. Bearing-down efforts.
6. Goal: facilitate drainage of mucus and fluid
3. Perineal bulging; crowning.
patent airway.
4. Mother states, Its coming.
a. Hold infant in head-dependent position.
B. Analysis/nursing diagnosis:
b. Clear mucus with bulb syringe (if available),
1. Pain related to:
or use fingertip, wipe with towel.
a. Strong, sustained contractions.
7. Goal: prevent placental transfusionhold infant
b. Descent of fetal head.
level with placenta until cord stops pulsating;
c. Stretching of perineum.
clamp and cut.
2. Anxiety/fear related to imminent birth.
8. Goal: prevent chilling.
3. Ineffective individual coping related to circum-
a. Wrap infant in towel or other clean
stances surrounding birth; anxiety, fear for self
material.
and infant.
b. Place infant on side, head dependent, on
4. Injury (mother) related to lacerations (vaginal,
mothers abdomen.
perineal).
c. Dry head, cover with cap or material.
5. Fluid volume deficit related to:
9. Goal: stimulate respiration. If neonate fails to
a. Lacerations.
breathe spontaneously:
b. Uterine atony.
a. Maintain body temperaturedry and
c. Retained placental fragments.
cover.
6. Impaired gas exchange (infant) related to intact
b. Clear airway
membranes after birth.
(1) Position: head down.
7. Risk for injury (infant) related to:
(2) Turn head to side.
a. Precipitate birth.
c. Stimulate.
b. Trauma.
(1) Rub back gently.
c. Hypoxia.
(2) Flick soles of feet.
C. Nursing care plan/implementation:
d. If no response to stimulation:
1. Goal: reduce anxiety/fearreassure mother.
(1) Slightly extend neck to sniffing position
2. Goal: delay birth, as possible.
(head tiltchin lift method).
a. Discourage bearing-down efforts.
(2) Place mouth over newborns nose and
b. Encourage panting.
mouth and exhale air in cheeks, saying
c. Side-lying position to slow descent and allow
ho (prevents excessive pressure).
for more controlled birth.
2164_Ch04_115-248 29/03/12 12:22 PM Page 176

176 chapter 4 Health Promotion and Maintenance

10. Goal: begin cardiopulmonary resuscitation (CPR) c. Coexisting medical disorders:


if heart rate <60 beats/min. (1) Uncontrolled diabetes.
a. Place infant on firm, flat surface. (2) Progressive preeclampsia.
b. With two fingers placed 1/2 to 3/4 of an inch (3) Severe renal disease.
above the xyphoid process, depress 1/3 to 1/2 (4) Cardiac disease.
depth of anterior-posterior chest. d. PROMspontaneous rupture of membranes
c. Assist ventilation on upstroke of every third before onset of labor and less than 37 weeks
compression (3:1 ratio). from last menstrual period. Hazards:
d. Go immediately to emergency department. (1) Maternalintrauterine infection
11. Goal: maintain infants body temperature. (chorioamnionitis, endometritis).
a. Wrap placenta with baby, if cord intact. (2) Fetalsepsis; prolapsed cord.
b. Place infant in mothers arms. e. Rh or ABO incompatibility, fetal hemolytic
PLACENTAL SEPARATION disease.
A. Assessmentthird stage: identify signs of placental f. Congenital anomaly (e.g., anencephaly).
separation. g. Postterm pregnancy with nonreactive
B. Nursing care plan/implementation: nonstress test (NST), or oligohydramnios.
1. Goal: avoid/minimize potential for complica- h. Intrauterine fetal demise.
tions (everted uterus, tearing of placenta with 2. Criteria for induction:
fragments remaining, separation of cord from a. Absence of CPD, malpresentation, or
placenta). malposition.
a. Avoid traction (pulling) on cord. b. Engaged vertex of single gestation.
MATERNAL/INFANT

b. Avoid vigorous fundal massage. c. Nearing, or at term.


c. Discourage maternal bearing-down efforts d. Fetal lung maturity.
unless placenta visible at introitus. (1) Survival ratebetter at 32 weeks or
d. With fundus well contracted, and placenta more.
visible at introitus, encourage mother to bear (2)Lecithin/sphingomyelin ratio greater
down to expel placenta. than 2:1.
2. Goal: prevent maternal hemorrhage (uterine (3) Mother who is diabeticPG is present in
atony). amniotic fluid.
a. Encourage breastfeeding, or stimulate nipple. e. Ripe cervixsoftening, partially effaced, or
b. Gently massage fundus, support lower part ready for effacement/dilation (if not already
of uterus, and express clots when uterus is present). Note: Intravaginal or paracervical
contracted. application of prostaglandin gel, or misoprostol
c. Encourage voiding if bladder is full. may be used to prepare cervix for labor.
d. Get to a medical facility. 3. Methods:
3. Goal: encourage bonding/stimulate uterine a. Amniotomyartificial rupture of membranes
contractions. Encourage breastfeeding. with fetal head engaged and dilation of
4. Goal: legal accountability as birth attendant. cervix.
Record date, time, birth events, maternal and b. Intravenous oxytocin infusion.
fetal status. 4. Potential complications:
C. Evaluation/outcome criteria: a. Amniotomyirrevocably committed to birth.
1. Experiences normal spontaneous birth of viable Hazards:
infant over intact perineum. (1) Prolapsed cord.
2. Uncomplicated fourth stagestatus satisfactory (2) Infection.
for both mother and infant. b. IV oxytocin infusion:
3. Expresses satisfaction in management and result. (1) Overstimulation of uterus.
IX. ALTERATIONS AFFECTING PROTECTIVE (2) Decreased placental perfusion/fetal
FUNCTION distress, neonatal jaundice.
(3) Precipitate labor and birth.
A. Induction of labordeliberate initiation of uter-
(4) Cervical/perineal lacerations.
ine contractions.
(5) Uterine rupture.
1. Indications for:
(6) Postpartum hemorrhage.
a. History of rapid or silent labors, precipitate
(7) Water intoxicationif large doses given in
birth.
D/W over prolonged period (antidiuretic
b. Woman resides some distance from hospital
effect increases water reabsorption).
(controversial).
(8) Hypertensive crisis.
2164_Ch04_115-248 29/03/12 12:22 PM Page 177

Induction of Labor 177


BEFORE INDUCTION 8. Evaluation/outcome criterion: woman verbalizes
5. Assessmentbefore induction: understanding of process, rationale, procedures,
a. Estimate of gestation (EDD, fundal height, and alternatives.
cervical status). DURING INDUCTION AND LABOR
b. Bishops score: evaluation of cervical 9. Assessmentduring induction and labor:
inducibility. a. Amniotomysame as for spontaneous
c. General health status: rupture of membranes:
(1) Weight, vital signs, FHR, edema. (1) Observe fluidnote color, amount.
(2) Status of membranes. (2) Monitor FHR; assess for fetal distress.
(3) Vaginal bleeding. (3) Observe for signs of prolapsed cord.
(4) Coexisting disorders. (4) Assess fetal activity.
d. History of previous labors, if any. (a) Excessive activity may indicate distress.
e. Emotional status. (b) Absence of activity may indicate
f. Knowledge/understanding of anticipated distress or demise.
procedures: b. IV oxytocin infusion:
(1) Amniotomy (artificial rupture of (1) Continually assess response to oxytocin
membranes). stimulation/flow rate; always given by
(2) Cervical ripening (prostaglandin gel, controlled infusion.
Cervidil, Cytotec). (a) Uterine contractions.
(3) IV oxytocin infusion. (b) Maternal vital signs, FHR.
(4) Fetal monitoring (2) Identify signs of:

MATERNAL/INFANT
g. Preparation for childbirth (Lamaze method, (a) Deviation from normal patterns:
etc.); coping strategies. Identify support (i) Lack of response to increasing
person. flow rate.
6. Analysis/nursing diagnosis: (ii) Uterine hyperstimulation
a. Knowledge deficit related to process of (contractionsless than
induction. 2 minutes apart).
b. Anxiety/fear related to need for induction of (iii) Lack of adequate uterine relax-
labor. ation between contractions.
c. Ineffective individual coping related to (b) Side effects of oxytocin: diminished
psychological stress. outputpotential water intoxication.
d. Pain related to uterine contractions. (c) Hazards to mother or fetus:
7. Nursing care plan/implementation: (i) Sustained (over 90-second) or
a. Goal: health teaching. tetanic (strong, spasmlike)
(1) Explain rationale for procedures: contractionspotential abruptio
(a) Amniotomy. placentae, uterine rupture, fetal
(i) Induces labor. hypoxia/anoxia/death.
(ii) Relieves uterine overdistention. (ii) Fetal arrhythmias, decelerations.
(iii) Increases efficiency of contractions, (iii) Maternal hypertensionpotential
shortening labor. for hypertensive crisis, cerebral
(b) Oxytocin infusion. hemorrhage.
(i) Induces labor. 10. Nursing care plan/implementation:
(ii) Stimulates uterine contractions. a. Same as for other women in labor.
(c) Internal fetal monitor. b. If indications of deviations from normal
(i) Provides continuous assessment patterns:
of uterine response to oxytocin (1) Change maternal position.
stimulation. (2) Stop oxytocin infusion, maintain IV with
(ii) Provides continuous assessment of Ringers lactate, etc.
fetal response to physiological stress (3) Begin oxygen per mask; up to 8 to
of labor. 10 L/min.
(2) Describe procedureto reduce anxiety and (4) Notify physician promptly.
increase cooperation. (5) Check maternal blood pressure and
(3) Explain advantages/disadvantagesto pulse rate.
ensure informed consent. c. Anticipatory guidance: may or may not
b. Goal: emotional supportencourage verbaliza- have strong contractions soon after
tion of concerns; reassure, as possible. induction starts.
2164_Ch04_115-248 29/03/12 12:22 PM Page 178

178 chapter 4 Health Promotion and Maintenance

11. Evaluation/outcome criteria: (b) Administer analgesics, topical


a. Demonstrates response to oxytocin sprays, ointments, witch hazel pads,
stimulation. hydrocortisone.
(1) Establishes desired contraction pattern, (c) Encourage use of sitz bath or rubber
not hyperstimulated. ring.
(2) Progresses through laborwithin normal (d) Encourage Kegel exercises.
limits: (e) Do health teaching:
(a) Normotensive. (i) Instruct in tightening gluteal
(b) Voids in adequate amounts. muscles before sitting.
(c) No evidence of deviation from normal (ii) Instruct to avoid sitting on
contraction patterns. one hip.
b. No evidence of fetal distress. (2) Goal: minimize potential for infection.
c. Experiences normal vaginal birth of viable (a) Teach/provide perineal care during
infant. fourth stage of labor.
B. Operative obstetricsprocedures used to prevent (b) Health teaching: instruct in self-perineal
trauma/reduce hazard to mother or infant during care after voiding, defecation, and with
the birth process. each pad change.
1. Episiotomyincision of perineum to facilitate f. Evaluation/outcome criteria:
infants birth. (1) Womans incision heals by primary
a. Rationale: intention.
(1) Surgical incision reduces possibility of (2) Woman demonstrates appropriate
MATERNAL/INFANT

laceration. self-perineal care.


(2) Protects infants head from pressure (3) Woman evidences no signs of hematoma,
exerted by resistant perineum. infection, or separation of suture line.
(3) Shortens second stage of labor. (4) Woman experiences minimal discomfort.
b. Types: 2. Forceps-assisted birthuse of instruments to
(1) Midlinechance of extension into anal assist birth of infant.
sphincter greater than with mediolateral. a. Indications:
(2) Mediolateralhealing is more painful (1) Fetal distress.
than midline. (2) Maternal need:
c. Assessment: (a) Exhaustion.
(1) REEDA: (b) Coexisting disease, such as cardiac
(a) Redness disorder.
(b) Edema (c) Poor progress in second stage.
(c) Ecchymosis (d) Persistent fetal occipitotransverse (OT)
(d) Discharge or occipitoposterior (OP) position.
(e) Approximation (suture line intact, b. Criteria for forceps application:
separated) (1) Engaged fetal head.
(2) Healing. (2) Ruptured membranes.
(3) Bruised; hematoma. (3) Full dilation.
(4) Tenderness; pain. Note: Evaluate com- (4) Absence of CPD.
plaints of pain carefully. If intense, and (5) Some anesthesia has been given; usually,
unrelieved by usual measures, report episiotomy has been performed.
promptly. May indicate vulvar, paravagi- (6) Empty bladder.
nal, or ischiorectal abscess or hematoma. c. Types:
d. Analysis/nursing diagnosis: (1) Lowoutlet forceps.
(1) Pain related to labor process. (2) Midapplied after head is engaged
(2) Impaired skin integrity related to surgical (rarely used).
incision. (3) Piper forcepsapplied to after-coming
(3) Fluid volume deficit related to hematoma. head in selected breech births (rarely
(4) Sexual dysfunction related to discomfort. done).
e. Nursing care plan/implementation: d. Potential complications:
(1) Goal: prevent/reduce edema, promote (1) Maternal:
comfort and healing. (a) Lacerations of birth canal, rectum,
(a) Place covered ice pack during bladder.
immediate postpartum. (b) Uterine rupture/hemorrhage.
2164_Ch04_115-248 29/03/12 12:22 PM Page 179

Operative Obstetrics 179


(2) Neonatal: (4) Breech presentation. Note: To reduce
(a) Cephalohematoma. infant morbidity/mortality, elective
(b) Skull fracture. cesarean birth is common method of
(c) Intracranial hemorrhage, brain damage. choice.
(d) Facial paralysis. (5) Neoplasms of cervix, uterus, or birth
(e) Direct tissue trauma (abrasions, canal.
ecchymosis). (6) Maternal diabetes with placental aging;
(f ) Umbilical cord compression. fetal macrosomia (CPD); >4050 gm.
e. Assessment: b. Criterion for elective cesarean birth:
(1) FHR immediately beforeand after L/S ratio greater than 2:1indicates presence
forceps application (forceps blade may of pulmonary surfactant; less risk of respiratory
compress umbilical cord). distress syndrome.
(2) Observe mother/newborn for injury or c. Indications for emergency cesarean birth:
signs of complications. (1) Fetal:
f. Analysis/nursing diagnosis: (a) Fetal distress: prolapsed cord, repetitive
(1)Self-esteem disturbance related to late decelerations, prolonged bradycardia.
inability to give birth without surgical (b)Fetal jeopardy: Rh or ABO
assistance. incompatibility.
(2) Anxiety/fear related to infants appearance (c) Fetal malposition/malpresentation.
(forceps marks) or awareness of potential (2) Maternal:
complications. (a) Uterine dysfunction; rupture.

MATERNAL/INFANT
g. Nursing care plan/implementation: (b) Placental disorders:
(1) Goal: minimize feelings of failure due to (i) Placenta previa.
inability to give birth naturally. (ii) Abruptio placentae, with Couvelaire
(a) Explain, discuss reasons/indications uterus.
for forceps-assisted birth. (c) Severe maternal preeclampsia/eclampsia.
(b) Emphasize no maternal control over (d) Fetopelvic disproportion.
circumstances. (e) Sudden maternal death.
(2) Goal: reduce parental anxiety, maternal (f ) Carcinoma.
guilt over infant bruising/forceps marks. (g) Failed induction.
Explain condition is temporary and has d. Types:
no lasting effects on childs appearance. (1) Low segmentmethod of choice:
h. Evaluation/outcome criteria: (a) Transverse incision through abdominal
(1) Woman verbalizes understanding of wall and lower uterine segment.
reasons for forceps-assisted birth. (b) Transverse incision through abdominal
(2) Woman evidences no interruption in wall, with vertical incision of lower
bonding with infant. uterine segment.
(3) Woman experiences uncomplicated (c) Advantagesfewer complications:
recovery. (i) Less blood loss.
3. Vacuum extraction (soft plastic cup with (ii) More comfortable convalescence.
vacuum from a handheld suction pump). (iii) Less adhesion formation.
Used to assist in rotation or delivery of the (iv) Lower risk of uterine rupture in
fetal head. subsequent pregnancy/labor and
a. Risks may include caput succedaneum and birth.
cephalohematoma. (v) Cosmetically more acceptable.
b. Causes neonatal jaundice; intraventricular (2) Classicalvertical incision through abdomi-
hemorrhage can result in death. nal wall and uterus. May be necessary for
4. Cesarean birthincision through abdominal anterior placenta previa and transverse lie,
wall and uterus to give birth. less than 28 weeks prematurity.
a. Indications for elective cesarean birth: (3) Porroshysterotomy followed by
(1) Known CPD. hysterectomy. Necessary in presence of:
(2)Previous uterine surgery (e.g., myomec- (a) Hemorrhage from uterine atony.
tomy), repeated cesarean births (depends (b) Placenta accreta/percreta.
on type of incision done). (c) Large uterine myomas.
(3) Active maternal genital herpes type 2 (d) Ruptured uterus.
infection; human papillomavirus (HPV). (e) Cancer of uterus or ovary.
2164_Ch04_115-248 29/03/12 12:22 PM Page 180

180 chapter 4 Health Promotion and Maintenance

e. Assessment: (2) Postoperative:


(1) Maternal physical status. (a) Same as for other clients having
(a) Vital signs. abdominal surgery (see Chapter 6).
(b) Labor status, if any. (b) Same as for other women who are
(c) Contractions (if any). postpartum.
(d) Membranes (intact; ruptured). h. Evaluation/outcome criteria:
(e) Bleeding. (1) Verbalizes understanding of reasons for
(2) Fetal status. cesarean birth.
(a) FHR pattern. (2) Successful birth of viable infant.
(b) Color and amount of amniotic fluid. (3) Evidences no surgical/birth complications.
(c) Biophysical profile (BPP), if performed. (4) Evidences no interference with bonding.
(3) Maternal emotional status. (5) Expresses satisfaction with procedure and
(4) Understanding of procedure, indications result.
for, implications. 5. Trial of labor after cesarean (TOLAC)
(5) Otheras for any abdominal surgery a. Candidates for TOLAC.
(see Chapter 6). (1) Previous low transverse cesarean birth.
f. Analysis/nursing diagnosis: (2) Fetal head well engaged in pelvis (vertex
(1) Self-esteem disturbance related to perceived presentation).
failure to give birth vaginally. (3) Soft, anterior cervix.
(2) Anxiety/fear related to impending surgery (4) Preexisting reason for repeat cesarean
and/or reasons for cesarean birth. birth not apparent.
MATERNAL/INFANT

(3) Ineffective individual coping related to b. Assessment:


anxiety and fear for self, infant. (1) Monitor FHR carefully during trial of
(4) Fluid volume deficit related to abdominal labor.
surgery or reason for cesarean birth. (2) Monitor contractions carefully for
(5) Pain related to abdominal surgery. adequate progress of labor.
(6) Constipation related to decreased bowel (3) Observe mother for signs of complications/
activity. uterine rupture.
(7) Altered urinary elimination related to fluid c. Analysis/nursing diagnosis:
volume deficit. (1) Knowledge deficit related to trial of labor.
g. Nursing care plan/implementation: (2) Fear related to outcome for fetus.
(1) Preoperative: (3) Ineffective individual coping related to labor
(a) Goal: safeguard fetal status. progress and outcome.
(i) Monitor fetal heart rate
continually.
(ii) Notify neonatology and neonatal Complications During
intensive care unit (NICU) of
scheduled surgical birth, if suspect the Intrapartum Period
complications. I. GENERAL ASPECTS
(b) Goal: health teaching. A. Pathophysiologyinterference with normal
(i) Describe, discuss anticipated processes and patterns of labor/birth result in
anesthesia. maternal or fetal jeopardy (e.g., preterm labor,
(ii) Explain rationale for preoperative dysfunctional labor patterns; prolonged [over
antacids to minimize effects of 24 hours] labor; hemorrhage: uterine rupture/
aspiration: cimetidine, Bicitra, inversion, amniotic fluid embolus).
histamine blocker to decrease B. Etiology:
production of gastric acid; Reglan 1. Preterm laborunknown.
(metoclopramide), to hasten 2. Dysfunctional labor (dystocia: see p. 184):
gastric emptying. a. Physiological response to anxiety/fear/
(iii) Describe, explain anticipated painresults in release of catecholamines,
proceduresabdominal shave, increasing physical/psychological stress
indwelling catheter, intravenous myometrial dysfunction; painful and
fluidsto woman and support ineffectual labor.
person. b. Iatrogenic factors: premature or excessive
(c) Otheras for any abdominal surgery. analgesia, particularly during latent phase.
(d) Prepare for cesarean birth.
2164_Ch04_115-248 29/03/12 12:22 PM Page 181

Complications During the Intrapartum Period 181


c. Maternal factors: E. Nursing care plan/implementation:
(1) Pelvic contractures. 1. Goal: minimize physical/psychological stress
(2)Uterine tumors (e.g., myomas, during labor/birth. Assist woman in coping
carcinoma). effectively:
(3)Congenital uterine anomalies a. Reinforce relaxation techniques.
(e.g., bicornate uterus). b. Support couples effective coping techniques/
(4) Pathological contraction ring (Bandls mechanisms.
ring). 2. Goal: emotional support.
(5) Rigid cervix, cervical stenosis/stricture. a. Encourage verbalization of anxiety/fear/
(6) Hypertonic/hypotonic contractions. concerns.
(7) Prolonged rupture of membranes. Note: b. Explain all proceduresto minimize anxiety/
Intrauterine infection may have caused fear, encourage cooperation/participation in
rupture of membranes or may follow care.
rupture. c. Provide quiet environment conducive to rest.
(8) Prolonged first or second stage. 3. Goal: continuous monitoring of maternal/fetal
(9)Medical conditions: diabetes, status and progress through laborto identify
hypertension. early signs of dysfunctional labor, fetal distress;
d. Fetal factors: facilitate prompt, effective treatment of emerg-
(1) Macrosomia (LGA). ing complications.
(2) Malposition/malpresentation. 4. Goal: minimize effects of complicated labor on
(3) Congenital anomaly (e.g., hydrocephalus, mother, fetus.

MATERNAL/INFANT
anencephaly). a. Position change: lateral Simsto reduce
(4) Multifetal gestation (e.g., interlocking compression of inferior vena cava.
twins). b. Oxygen per mask, as indicated.
(5) Prolapsed cord. c. Institute interventions appropriate to
(6) Postterm. emerging problems (see specific disorder).
e. Placental factors: F. Evaluation/outcome criteria:
(1) Placenta previa. 1. Woman has successful birth of viable infant.
(2) Inadequate placental function with 2. Maternal/infant status stable, satisfactory.
contractions. II. DISORDERS AFFECTING PROTECTIVE
(3) Abruptio placentae. FUNCTIONS: Preterm laboroccurs after 20 weeks
(4) Placenta accreta. of gestation and before beginning of week 38.
f. Physical restrictions: when confined to bed,
A. Pathophysiologyphysiological events of labor
flat position, etc.
(i.e., contractions, spontaneous rupture of
C. Assessment:
membranes, cervical effacement/dilation) occur
1. Antepartal history.
before completion of normal, term gestation.
2. Emotional status.
B. Etiologycauses may be from maternal, fetal,
3. Vital signs, FHR.
or placental factors.
4. Contraction pattern (frequency, duration,
C. Coexisting disorders:
intensity).
1. Infections that may cause PROM.
5. Vaginal discharge.
2. PROM of unknown etiology.
D. Analysis/nursing diagnosis:
3. PIH (preeclampsia/eclampsia).
1. Anxiety/fear for self and infant related to
4. Uterine overdistention.
implications of prolonged or complicated
a. Polyhydramnios.
labor/birth.
b. Multifetal gestation.
2. Pain related to hypertonic contractions/
5. Maternal diabetes, renal or cardiovascular
dysfunctional labor.
disorder, UTI.
3. Ineffective individual coping related to
6. Severe maternal illness (e.g., pneumonia, acute
physical/psychological stress of complicated
pyelonephritis).
labor/birth, lowered pain threshold secondary
7. Abnormal placentation.
to fatigue.
a. Placenta previa.
4. High risk for injury related to prolonged rupture
b. Abruptio placentae.
of membranes, infection.
8. Iatrogenic: miscalculated EDD for repeat
5. Fluid volume deficit related to excessive blood
cesarean birth (rare).
loss secondary to placenta previa, abruptio
9. Fetal death.
placentae, Couvelaire uterus, DIC.
2164_Ch04_115-248 29/03/12 12:23 PM Page 182

182 chapter 4 Health Promotion and Maintenance

10. Incompetent cervical os (small percentage). 5. Signs of continuing and progressing labor. Note:
11. Uterine anomalies (rare). Vaginal examination only if indicated by other
a. Intrauterine septum. signs of continuing labor progress:
b. Bicornate uterus. a. Effacement.
12. Uterine fibroids. b. Dilation.
13. Positive fetal fibronectin assay (protein found in c. Station.
fetal tissue, membranes, amniotic fluid, and the 6. Status of membranes.
decidua) found in cervical/vaginal fluid first 7. Fetal heart rate, activity (continuous monitoring).
half of pregnancy and normally absent through G. Analysis/nursing diagnosis:
mid to late pregnancy ( risk of preterm labor 1. Anxiety/fear related to possible outcome.
by 20%). 2. Self-esteem disturbance related to feelings of guilt,
D. Prevention: failure.
1. Primaryclose obstetric supervision; education 3. Impaired physical mobility related to imposed
in signs/symptoms of labor. bedrest.
2. Secondaryprompt, effective treatment of 4. Knowledge deficit related to medication side
associated disorders (see II. C., pp. 181182). effects.
3. Tertiarysuppression of preterm labor. 5. Ineffective individual coping related to possible
a. Bedrest. outcome.
b. Position: side-lyingto promote placental 6. Impaired gas exchange related to side effects of
perfusion. medication (circulatory overload; pulmonary
c. Hydration. edema).
MATERNAL/INFANT

d. Pharmacological (may require informed 7. Diversional activity deficit related to imposed


consent; follow hospital protocol). Beta- bedrest, decreased environmental stimuli.
adrenergic agents, MgSO4 (recent studies 8. Altered urinary elimination related to bedrest.
show poor results with MgSO4), Procardia 9. Constipation related to bedrest.
to reduce sensitivity of uterine myometrium H. Nursing care plan/implementation:
to oxytocic and prostaglandin stimulation; 1. Goal: inhibit uterine activity. Administer
increase blood flow to uterus. medications as orderedterbutaline,
e. May be maintained at home with adequate magnesium sulfate, Procardia, Indocin.
follow-up and health teaching. 2. Goal: safeguard status.
E. Contraindications for suppression: Labor is not a. Continuous maternal/fetal monitoring.
suppressed in presence of: b. I&Oto identify early signs of possible
1. Placenta previa or abruptio placentae with circulatory overload.
hemorrhage. c. Position: side-lyingto increase placental
2. Chorioamnionitis. perfusion, prevent supine hypotension.
3. Erythroblastosis fetalis. d. Report promptly to physician:
4. Severe preeclampsia. (1) Maternal pulse of 110 or more.
5. Severe diabetes (e.g., brittle). (2) Diastolic pressure of 60 mm Hg or less.
6. Increasing placental insufficiency. (3) Respirations of 24 or more; crackles (rales).
7. Progressive cervical dilation of 4 cm or more. (4) Complaint of dyspnea.
8. Ruptured membranes (with maternal fever). (5) Contractions: increasing frequency, strength,
F. Assessment: duration, or cessation of contractions.
1. Maternal vital signs. Response to medication: (6) Intermittent back and thigh pain.
a. Hypotension. (7) Rupture of membranes.
b. Tachycardia, arrhythmia. (8) Vaginal bleeding.
c. Dyspnea, chest pain. (9) Fetal distress.
d. Nausea and vomiting. 3. Goal: comfort measures.
2. Signs of infection: a. Basic hygienic carebath, mouth care, cold
a. Increased temperature. washcloth to face, perineal care.
b. Tachycardia. b. Back rub, linen changeto promote relaxation.
c. Diaphoresis. 4. Goal: emotional support.
d. Malaise. a. Encourage verbalization of guilt feelings,
e. Increased baseline fetal heart rate; variability. anxiety, fear, concerns; provide factual
3. Contractions: frequency, duration, strength. information.
4. Emotional statussigns of denial, guilt, b. Support positive self-concept.
anxiety, exhaustion. c. Keep informed of progress.
2164_Ch04_115-248 29/03/12 12:23 PM Page 183

Complications During the Intrapartum Period 183


5. Goal: provide quiet diversion. Television, reading 2. Goal: continuous monitoring of status.
materials, handicrafts (may not be able to focus a. Electronic monitorsto observe respiratory
well if on magnesium therapy). and cardiac functions.
6. Goal: health teaching. b. Blood samplesto monitor blood gases,
a. Explain, discuss proposed management to pH, hypoglycemia.
suppress preterm labor. K. Postpartum care: Goal: emotional support.
b. Describe, discuss side effects of medication. 1. Facilitate attachmentphotos, if baby transferred
c. Explain rationale for bedrest, position. or mother unable to visit.
I. If labor continues to progress: 2. If couple, foster sense of mutual experience and
1. Goal: facilitate infant survival. closeness.
a. Administer betamethasone, as ordered, 24 to 3. Help her/them maintain a positive self-image.
48 hours before birthto increase/stimulate 4. Encourage touching of infant before transport to
production of pulmonary surfactant. nursery or high-risk facility; father/partner may
b. Give antibiotic to mother to chance of accompany infant and report back to mother.
neonatal sepsis. 5. Encourage early contactto facilitate mothers
c. Notify NICUto increase chances for fetal need to ventilate her feelings.
survival; ensure prompt, expert management 6. Assist parent(s) with grieving process, if necessary.
of neonate; and provide information and 7. Refer to support group if necessary.
support to parents. L. Otheras for any woman who is postpartum.
d. Monitor progress of labor to identify signs of M. Evaluation/outcome criteria:
impending birth. Note: May give birth before 1. Woman verbalizes understanding of medical/

MATERNAL/INFANT
complete (10-cm) dilation. nursing recommendations and treatments.
e. Consider transfer to high-risk facility. 2. Woman complies with medical/nursing regimen.
f. Prepare for birth, or cesarean birth if infant 3. Woman experiences no discomfort from side
less than 34 to 36 weeks of gestation. effects of therapy.
2. Goal: emotional support. 4. Woman experiences successful outcomelabor
a. Do not leave woman (or couple) alone. inhibited.
b. Encourage verbalization of anxiety, fear, 5. Woman carries pregnancy to successful
concern. termination.
c. Explain all procedures. 6. If preterm birth occurs, woman copes effectively
3. Goal: comfort measures. Note: Analgesics used with outcome (physiologically compromised
conservativelyto prevent depression of fetus/ neonate, neonatal death).
neonate. III. GRIEF AND CHILDBEARING EXPERIENCE.
4. Goal: support effective coping techniques. Encourage/ The loss of a pregnancy or a newborn, or the birth of
support Lamaze (or other) techniquescoach, as a child who is physiologically compromised (preterm,
necessary; discourage hyperventilation. congenital disorder) is a crisis situation. The unexpected
5. Goal: health teachingfor severe preterm birth. outcome can cause the parent(s) to suffer a sense of
a. Discuss need for episiotomy, possibility of loss of self-esteem, self-concept, positive body image,
outlet forcepsassisted birthto reduce stress feelings of worth (see Postpartum depression/
on fetal head, or psychosis, p. 199).
b. Prepare for cesarean birthto reduce possibility
A. Assessment:
of fetal intraventricular hemorrhage.
1. Response to loss of the fantasy child/real child.
c. Give rationale for avoiding use of medications
a. Behavioralanger, hostility, depression, disin-
to reduce contraction pain.
terest in activities of daily living, withdrawal.
J. Immediate care of neonate:
b. Biophysicalsomatic complaints (stomach
1. Goal: safeguard status.
pain, malaise, anorexia, nausea).
a. Stabilize environmental temperatureto
c. Cognitivefeelings of guilt.
prevent chilling (isolette or other controlled-
2. Knowledge/understanding/perception of situation.
temperature bed).
3. Coping abilities, mechanisms.
b. Suction, oxygen, as needed; may need
4. Support system.
intubation.
B. Analysis/nursing diagnosis:
c. Parenteral fluids, as orderedto support
1. Ineffective family coping: compromised related to
normal acid-base balance, pH; administer
psychological stress due to fear for infant, guilt
antibiotics, as necessary.
feelings, impact on self-image.
d. Arrange transport to high-risk facility, as
2. Ineffective individual coping related to anxiety, stress.
necessary.
2164_Ch04_115-248 29/03/12 12:23 PM Page 184

184 chapter 4 Health Promotion and Maintenance

3. Ineffective family coping: disabling related to dis- (2) Uterine overdistention (multifetal
turbance in intrafamily relations secondary to pregnancy, fetal macrosomia).
individual coping deficits, recriminations. (3) Uterine myomas.
4. Altered parenting related to lack of effective (4) Grandmultipara.
bonding secondary to emotional separation b. PASSAGEWAY: resistance of cervix, pelvic
from infant, feelings of guilt. structures.
5. Dysfunctional grieving related to guilt feelings, (1) Rigid cervix.
impact of loss on self-concept. (2) Distended bladder.
6. Disturbance in body image, self-esteem, role per- (3) Distended rectum.
formance related to perceived failure to complete (4) Dimensions of the bony pelvis: pelvic
gestational task, produce perfect, healthy infant; contractures.
associated with sleep deprivation. c. PASSENGER: accommodation of the
7. Social isolation related to severe coping deficit, presenting part to pelvic diameters.
dysfunctional grieving, disturbance in self-esteem. (1) Fetal malposition/malpresentation.
C. Nursing care plan/implementation: (a) Transverse lie.
1. Goal: emotional support. (b) Face, brow presentation.
a. Provide privacy; encourage open expression/ (c) Breech presentation.
verbalization of feelings, fears, concerns, (d) Persistent occipitoposterior position.
perceptions. (e) CPD.
b. Crisis intervention techniques. (2) Fetal anomalies.
2. Goal: facilitate bonding, effective coping, or (a) Hydrocephalus.
MATERNAL/INFANT

anticipatory grieving processes. (b) Conjoined (Siamese) twins.


a. Encourage contact and participation in care (c) Meningomyelocele.
of premature or compromised infant. (3) Fetal size: macrosomia.
b. Keep informed of infants status. 3. Hazards:
c. Provide realistic data. a. Maternal:
3. Goal: health teaching. (1) Fatigue, exhaustion, dehydrationdue to
a. Clarify misperceptions, as appropriate. prolonged labor.
b. Discuss, demonstrate infant care techniques (2)Lowered pain threshold, loss of control
(e.g., feeding infant who has cleft lip or palate). due to prolonged labor, continued uter-
c. Refer to appropriate community resources. ine contractions, anxiety, fatigue, lack of
D. Evaluation/outcome criteria: sleep.
1. Woman verbalizes recognition and acceptance (3) Intrauterine infectiondue to prolonged
of diagnosis. rupture of membranes and frequent
2. Woman verbalizes understanding of relevant vaginal examinations.
information regarding treatment, prognosis. (4) Uterine rupturedue to obstructed labor,
3. Woman makes informed decision regarding hyperstimulation of uterus.
infant care. (5) Cervical, vaginal, perineal lacerations
4. Woman demonstrates comfort and increasing due to obstetric interventions.
participation in care of neonate. (6) Postpartum hemorrhagedue to uterine
5. Woman shows evidence of culturally appropriate atony or trauma.
bonding (eye contact, cuddles, calls infant by b. Fetal:
name). (1) Hypoxia, anoxia, demisedue to decreased
IV. DISORDERS AFFECTING COMFORT, REST, O2 concentration in cord blood.
MOBILITY: DYSTOCIA (2) Intracranial hemorrhagedue to changing
intracranial pressure.
A. Definitiondifficult labor.
C. Hypertonic dysfunction
B. General aspectsthere are 6 Ps that affect the progress
1. Pathophysiologyincreased resting tone of
of labor: Mother3 Ps: psych, placenta, position
uterine myometrium; diminished refractory
and Fetus3 Ps: power, passageway, passenger
period; prolonged latent phase:
1. Pathophysiologysee specific disorders.
a. Nulliparamore than 20 hours.
2. Etiologydue to effects of three other factors
b. Multiparamore than 14 hours.
that affect the FETUS:
2. Etiologyunknown. Theoryectopic initiation
a. POWER: forces of labor (uterine contractions,
of incoordinate uterine contractions.
use of abdominal muscles).
3. Assessment:
(1) Premature analgesia/anesthesia.
a. Onsetearly labor (latent phase).
2164_Ch04_115-248 29/03/12 12:23 PM Page 185

Complications During the Intrapartum Period 185


b. Contractions: b. Demonstrates no signs of fetal distress.
(1) Continuous fundal tension, incomplete c. Successfully completes uneventful labor.
relaxation. D. Hypotonic dysfunction during labor
(2) Painful. 1. Pathophysiologyafter normal labor at onset,
(3) Ineffectualno effacement or dilation. contractions diminish in frequency, duration
c. Signs of fetal distress: and strength; lowered uterine resting tone;
(1) Meconium-stained amniotic fluid. cervical effacement and dilation slow/cease.
(2) FHR irregularities. 2. Etiology:
d. Maternal vital signs. a. Premature or excessive analgesia/anesthesia
e. Emotional status. (epidural block or spinal block).
f. Medical evaluation: vaginal examination, x-ray b. CPD.
pelvimetry, ultrasonographyto rule out c. Overdistention (polyhydramnios, fetal
CPD (rarely used). macrosomia, multifetal pregnancy).
4. Analysis/nursing diagnosis: d. Fetal malposition/malpresentation.
a. Pain related to hypertonic contractions, e. Maternal fear/anxiety.
incomplete uterine relaxation. 3. Assessment:
b. Anxiety/fear for self and infant related to strong, a. Onsetmay occur in latent phase; most
painful contractions without evidence of common during active phase.
progress. b. Contractions: normal previously,
c. Ineffective individual coping related to fatigue, demonstrate:
exhaustion, anxiety, tension, fear. (1) Decreased frequency.

MATERNAL/INFANT
d. Impaired gas exchange (fetal) related to incom- (2) Shorter duration.
plete relaxation of uterus. (3) Diminished intensity (mild to moderate).
e. Sleep pattern disturbance related to prolonged (4) Less uncomfortable.
ineffectual labor. c. Cervical changesslow or cease.
5. Nursing care plan/implementation: d. Signs of fetal distressrare.
a. Medical management: (1) Usually occur late in labor due to infec-
(1) Short-acting barbituratesto encourage tion secondary to prolonged rupture of
rest, relaxation (see Chapter 8). membranes.
(2) Intravenous fluidsto restore/maintain (2) Tachycardia.
hydration and fluid-electrolyte balance. e. Maternal vital signs may indicate infection
(3) If CPD, cesarean birth. ( temperature).
b. Nursing management: f. Medical diagnosisprocedures: vaginal
(1) Goal: emotional supportassist coping examination.
with fear, pain, discouragement. 4. Analysis/nursing diagnosis:
(a) Encourage verbalization of anxiety, a. Knowledge deficit related to limited exposure
fear, concerns. to information.
(b) Explain all procedures. b. Anxiety/fear related to failure to progress as
(c) Reassure. Keep couple informed of anticipated; fear for fetus.
progress. c. High risk for injury (infection) related to
(2) Goal: comfort measures. prolonged labor or ruptured membranes.
(a) Position: side-lyingto promote 5. Nursing care plan/implementation:
relaxation and placental perfusion. a. Medical management:
(b) Bath, back rub, linen change, clean (1) Amniotomyartificial rupture of
environment. membranes.
(c) Environment: quiet, darkened room (2) Oxytocin augmentation of labor
to minimize stimuli and encourage intravenous infusion of oxytocin to
relaxation, warmth. increase frequency, duration, strength,
(d) Encourage voidingto relieve bladder and efficiency of uterine contractions
distention; to test urine for ketones. (see Induction of labor, p. 176).
(3) Goal: prevent infection. Strict aseptic (3) If CPD, cesarean birth.
technique. b. Nursing management:
(4) Goal: prepare for cesarean birth if necessary. (1) Goals: emotional support, comfort measures,
6. Evaluation/outcome criteria: prevent infectionsame as for Hypertonic
a. Relaxes, sleeps, establishes normal labor dysfunction (see pp. 184185).
pattern. (2) Other (see Induction of labor, p. 176).
2164_Ch04_115-248 29/03/12 12:23 PM Page 186

186 chapter 4 Health Promotion and Maintenance

6. Evaluation/outcome criteria: 6. Nursing care plan/implementation:


a. Reestablishes normal labor pattern. a. Medical management:
b. Experiences successful birth of viable (1) Surgicallaparotomy, hysterectomy.
infant. (2) Replace blood losstransfusion, packed
V. DISORDERS AFFECTING FLUID-GAS cells.
TRANSPORT: maternal (3) Reduce possibility of infection
antibiotics.
A. Uterine rupture
b. Nursing management:
1. Pathophysiologystress on uterine muscle
(1) Goal: safeguard status.
exceeds its ability to stretch.
(a) Report immediately; mobilize staff.
2. Etiology:
(b) Prepare for immediate laparotomy.
a. Overdistentiondue to large baby, multifetal
(c) Oxygen per maskto increase circulat-
gestation.
ing oxygen level.
b. Old scarsdue to previous cesarean births or
(d) Order STAT type and crossmatch for
uterine surgery.
bloodto replace blood loss.
c. Contractions against CPD, fetal
(e) Establish IV lineto infuse fluids,
malpresentation, pathological retraction
blood, medications.
ring (Bandls).
(f ) Insert indwelling catheterto deflate
d. Injudicious obstetricsmalapplication
bladder.
of forceps (or application without full
(g) Abdominal prepto remove hair,
effacement/dilation).
bacteria.
MATERNAL/INFANT

e. Tetanic contractiondue to hypersensitivity


(h)Surgical permit (informed consent) for
to oxytocin (or excessive dosage) during
hysterectomy.
induction/augmentation of labor.
(2) Goal: emotional supportto allay anxiety
3. Assessment:
(woman and family).
a. Identify predisposing factors early.
(a) Encourage verbalization of fears,
b. Complete rupture
anxiety, concerns.
(1) Pain: sudden, sharp, abdominal; followed
(b) Explain all procedures.
by cessation of contractions; tender
(c) Keep family informed of progress.
abdomen.
7. Evaluation/outcome criteria:
(2) Signs of shock; vaginal bleeding.
a. Experiences successful termination of
(3) Fetal heart tonesabsent.
emergency; minimal blood loss.
(4) Presenting partnot palpable on vaginal
b. Postoperative status stable.
examination.
B. Amniotic fluid embolus (anaphylactoid syndrome)
c. Incomplete rupture
1. Pathophysiology: acute cor pulmonaledue to
(1) Contractions: continue, accompanied by
embolus blocking vessels in pulmonary circula-
abdominal pain and failure to dilate; may
tion; massive hemorrhagedue to DIC resulting
become dystonic.
from entrance of thromboplastin-like material
(2) Signs of shock.
into bloodstream.
(3) May demonstrate vaginal bleeding.
2. Etiologyamniotic fluid (with any meconium,
(4) Fetal heart tonesabsent/bradycardia.
lanugo, or vernix) enters maternal circulation
4. Prognosis
through open venous sinuses at placental site;
a. Maternalguarded.
travels to pulmonary arterioles. Triggers cardio-
b. Fetalgrave.
genic shock and anaphylactoid reaction.
5. Analysis/nursing diagnosis:
a. Rare.
a. Pain related to rupture of uterine muscle.
b. Associated with: tumultuous labor, abruptio
b. Fluid volume deficit related to massive blood
placentae, artificial ROM, placement of
loss secondary to uterine rupture.
intrauterine catheter.
c. Anxiety/fear related to concern for self, fetus.
3. Prognosispoor; often fatal to mother.
d. Altered tissue perfusion related to blood loss
4. Assessment:
secondary to uterine rupture.
a. May occur during labor, at time of rupture of
e. Altered urinary elimination related to necessary
membranes, or immediately postpartum.
conservation of intravascular fluid secondary
b. Sudden dyspnea and cyanosis.
to blood loss.
c. Chest pain.
f. Anticipatory grieving related to expected loss of
d. Hypotension, tachycardia.
fetus; inability to have more children.
e. Frothy sputum.
2164_Ch04_115-248 29/03/12 12:23 PM Page 187

Complications During the Intrapartum Period 187


f. Signs of DIC: (3) Hemorrhage; anemia.
(1) Purpuralocal hemorrhage. (4) Placental problem:
(2) Increased vaginal bleedingmassive. (a) Malformation of the placenta/cord.
(3) Rapid onset of shock. (b) Premature aging of placenta.
5. Analysis/nursing diagnosis: (c) Placental infarcts.
a. Impaired gas exchange related to pulmonary (d) Abruptio placentae.
edema. (e) Placenta previa.
b. Risk for fluid volume deficit related to DIC. (5) Postterm gestation.
c. Anxiety/fear for self and fetus related to severity (6) Maternal infection.
of symptoms, perception of jeopardy. (7) Polyhydramnios.
6. Nursing care plan/implementation: (8) Hypertonic uterine contractions.
a. Medical management: f. Premature rupture of membranes (PROM)
(1) IV heparin, whole blood. with chorioamnionitis.
(2) Birth: immediate, by forceps, if possible; g. Dystocia (e.g., from CPD).
or cesarean birth. 3. Assessmentintrapartum:
(3) Digitalize, as necessary. a. Amniotic fluid examinationat or after rup-
b. Nursing management: ture of membranes. Signs of fetal distress: meco-
(1) Goal: assist ventilation. nium stained, vertex presentationdue to
(a) Position: semi-Fowlers. relaxation of fetal anal sphincter secondary to
(b) Oxygen under positive pressure. hypoxia/anoxia. Note: Fetus gasps in utero
(c) Suction prn. may aspirate meconium and amniotic fluid.

MATERNAL/INFANT
(2) Goal: facilitate/expedite administration of b. Fetal activity:
fluids, medications, blood. (1) Hyperactivitydue to hypoxemia,
(a) Establish intravenous line with large- elevated CO2.
bore needle. (2) Cessationpossible fetal death.
(b) Administer heparin, fluids, as ordered. c. Methods of monitoring FHR:
(3) Goal: restore cardiopulmonary functions, if (1) Fetoscope.
needed. Cardiopulmonary resuscitation (2) Phonocardiography with microphone
techniques. application.
(4) Goal: emotional support of woman, family. (3) Internal fetal electrodeattached directly
(a) Allay anxiety, as possible. to fetus through dilated cervix after
(b) Explain all procedures. membranes ruptured.
(c) Keep informed of status. (4) Doppler probe using ultrasound flow.
7. Evaluation/outcome criteria: (5) Cardiotocographtransducer on maternal
a. Dyspnea relieved. abdomen transmits sound.
b. Bleeding controlled. d. Abnormal FHR patterns (see Fig. 4.10).
c. Successful birth of viable infant. (1) Persistent arrhythmia.
d. Uneventful postpartum course. (2) Persistent tachycardia of 160 or more
VI. DISORDERS AFFECTING FLUID-GAS beats/min.
TRANSPORT: fetal (3) Persistent bradycardia of 100 or fewer
beats/min.
A. Fetus in jeopardygeneral aspects:
(4) Early decelerationdue to vagal response
1. Pathophysiologymaternal hypoxemia,
to head compression.
anemia, ketoacidosis, Rh isoimmunization,
(5) Late decelerationdue to uteroplacental
or decreased uteroplacental perfusion.
insufficiency.
2. Etiologymaternal:
(6) Variable decelerationdue to cord
a. Preeclampsia/eclampsia, PIH.
compression.
b. Heart disease.
(7) Decreased or loss of variability in FHR
c. Diabetes.
pattern.
d. Rh or ABO incompatibility.
e. Medical evaluationprocedures: fetal blood
e. Insufficient uteroplacental/cord circulation
gases, pH (rarely performed).
due to:
(1) Purposeto identify fetal acid-base status.
(1) Maternal hypotension/hypertension.
(2) Requirements for:
(2) Cord compression:
(a) Ruptured membranes.
(a) Prolapsed.
(b) Cervical dilation.
(b) Knotted.
(c) Engaged head.
(c) Nuchal.
2164_Ch04_115-248 29/03/12 12:23 PM Page 188

188 chapter 4 Health Promotion and Maintenance

(3)Procedureunder sterile condition, e. Goal: expedite termination of threat to fetus:


sample of fetal scalp blood obtained for prepare for immediate cesarean birth.
analysis. f. Goal: support mother and significant other by
(4) Signs of fetal distress: staying with them and explaining.
(a) pH less than 7.20 (normal range is 6. Evaluation/outcome criteria:
7.37.4). a. FHR returns to normal rate and pattern.
(b) Increased CO2 b. Uncomplicated birth of viable infant.
(c) Decreased PO2 VII. SUMMARY OF DANGER SIGNS DURING
4. Analysis/nursing diagnosis: LABOR
a. Impaired gas exchange, fetal, related to
A. Contractionsstrong, every 2 minutes or less,
decreased placental perfusion/insufficient
lasting 90 seconds or more; poor relaxation
cord circulation.
between contractions.
b. Altered tissue perfusion related to hemolytic
B. Sudden sharp abdominal pain followed by board-
anemia.
like abdomen and shockabruptio placentae or
c. High risk for fetal injury related to hypoxia.
uterine rupture.
B. Prolapsed umbilical cord
C. Marked vaginal bleeding.
1. Pathophysiologycord descent in advance of
D. FHR periodic pattern decelerationslate; variable;
presenting part; compression interrupts blood
absent variability (see Fig. 4.10).
flow, exchange of fetal/maternal gases fetal
E. Baseline.
hypoxia, anoxia, death (if unrelieved).
1. Bradycardia (<100 beats/min).
2. Etiology:
MATERNAL/INFANT

2. Tachycardia (>160 beats/min).


a. Spontaneous or artificial rupture of mem-
F. Amniotic fluid.
branes before presenting part is engaged.
1. Amount: excessive; diminished.
b. Excessive force of escaping fluid, as in
2. Odor.
polyhydramnios.
3. Color: meconium stained; port-wine; yellow.
c. Malpositionbreech, compound presenta-
4. 24 hours or more since rupture of membranes.
tion, transverse lie.
G. Maternal hypotension, or hypertension.
d. Preterm or fetus who is SGAallows space
for cord descent.
3. Assessment: T H E P O S T PA R T U M
a. Visualization of cord outside (or inside) PERIOD
vagina.
General overview: This review of the normal physiological
b. Palpation of pulsating mass on vaginal
and psychological changes occurring during the postpar-
examination.
tum period (birth to 6 weeks after) provides the database
c. Fetal distressvariable deceleration and
necessary for assessing the womans progress through
persistent bradycardia.
involution, planning and implementing care, anticipato-
4. Analysis/nursing diagnosis:
ry guidance, health teaching, and evaluating the results.
a. Impaired gas exchange, fetal, related to inter-
Emerging problems are identified by comparing the
ruption of blood flow from placenta/fetus.
womans status against established standards.
b. Anxiety/fear, maternal, related to knowledge
of fetal jeopardy. I. BIOLOGICAL FOUNDATIONS
5. Nursing care plan/implementation: OF THE POSTPARTUM PERIOD
a. Goal: reduce pressure on cord. A. Uterine involutionintegrated processes by which
(1) Position: knee to chest; lateral modified the uterus returns to nonpregnant size, shape, and
Sims with hips elevated; modified consistency.
Trendelenburg. 1. Assessment:
(2) With gloved hand, support fetal presenting a. Contractions (afterpains)shorten muscles,
part. close venous sinuses, restore normal tone.
b. Goal: increase maternal/fetal oxygenation: (1) Frequency, intensity, and discomfort
oxygen per mask (810 L/min). decrease after first 24 hours.
c. Goal: protect exposed cord: continuous pres- (2) More common in multiparas and after
sure on the presenting part to keep pressure birth of a large baby; primiparous uterus
off cord. remains contracted.
d. Goal: identify fetal response to above measures, (3) Increased by breastfeeding.
reduce threat to fetal survival: monitor FHR b. Autolysisbreakdown and excretion of
continuously. muscle protein (decreasing size of myometrial
2164_Ch04_115-248 29/03/12 12:23 PM Page 189

The Postpartum Period 189


cells). Lochiasloughing of decidua and b. Healing, incisional line clean; no separation.
blood. c. Hematomablood in connective tissue
c. Formation of new endometrium4 to 6 weeks beneath skin; complains of pain, unrelieved
until placental site healed. by mild analgesia or heat; perineal distention;
d. Cervix painful, tense, fluctuant mass.
(1) Immediately following birthbruised, C. Abdominal wall
small tears; admits one hand. 1. Overdistention during pregnancy may
(2) Eighteen hours after birthbecomes shorter, rupture of elastic fibers, persistent striae, and
firmer; regains normal shape. diastasis of the rectus muscles.
(3) One week postpartumadmits two fingers. 2. Usually takes 6 to 8 weeks to retrogress, depend-
(4) Never returns fully to prepregnant state. ing on previous muscle tone, obesity, and
(a) Parous os is wider and not perfectly amount of distention during pregnancy.
round. 3. Strenuous exercises discouraged until 8 weeks
(b) Lacerations heal as scars radiating out postpartum.
from the os. D. Cardiovascular systemcharacteristic changes:
e. Fundal height and consistency 1. Immediately after birthincreased cardiac load,
(1) After birthat umbilicus; size and consis- due to:
tency of firm grapefruit. a. Return of uterine blood flow to general
(2) Day 1 (first 12 hours)one finger above circulation.
umbilicus. b. Diuresis of excess interstitial fluid.
(3) Descends by one fingerbreadth daily 2. Volumereturns to prepregnant state (4 L) in

MATERNAL/INFANT
until day 10. about 3 weeks. Major reductionduring first
(4) Day 10behind symphysis pubis, week, due to diuresis and diaphoresis.
nonpalpable. 3. Blood values (see Table 4.5).
f. Lochia a. High WBC during labor (25,000
(1) Character: 30,000/mm3), drops to normal level in
(a) Days 1 to 3: rubra (red). first few days.
(b) Days 3 to 7: serosa (pink to brown). b. Week 1Hgb, RBC, Hct, elevated fibrinogen
(c) Day 10: alba (creamy white). return to normal.
(2) Amount: 4. Blood coagulation
(a) Moderate: 4 to 8 pads/day (average a. During labor: rapid consumption of clotting
6 pads/day). factors.
(b) Following cesarean birth: less lochia b. During postpartum: increased consumption
due to manipulation during surgery. of clotting factors. Hypercoagulability main-
(3) Odor: normal lochia has characteristic tained during first few days postpartum; pre-
fleshy odor; foul odor is characteristic disposes to thrombophlebitis, pulmonary
of infection. embolism.
(4) Clots: normal: a few small clots, most 5. Assessment: potential complicationsvital
commonly on arisingdue to pooling. signs:
Note: Large clots and heavy bleeding are a. Temperatureelevated in:
associated with uterine atony, retained (1) Excessive blood loss, dehydration, exhaus-
placental fragments. tion, infection.
B. Birth canal (2) Elevation: 100.4F (38C) after first day
1. Vaginanever returns fully to prepregnant postpartum suggests puerperal infection.
state. b. Pulsephysiological bradycardia (5070
a. First few weeks postpartumthin walled, due beats/min) common through second day post-
to lack of estrogen; few rugae. partum; may persist 7 to 10 days; etiology:
b. Week 3: rugae may reappear. unknown. Tachycardiaassociated with:
2. Pelvic floor excessive blood loss, dehydration, exhaustion,
a. Immediately after birthinfiltrated with infection.
blood, stretched, torn. c. Blood pressuregenerally unchanged.
b. Month 6: considerable tone regained. Elevationassociated with: preeclampsia,
3. Perineum essential hypertension.
a. Immediately following birthedematous; E. Urinary tractcharacteristic changes:
may have episiotomy (or repaired lacerations); 1. Outputincreased due to: diuresis (12 hours to
hemorrhoids. 5 days postpartum); daily output to 3000 mL.
2164_Ch04_115-248 29/03/12 12:23 PM Page 190

190 chapter 4 Health Promotion and Maintenance

2. Urine constituents: erectile, intact (no signs of fissure); bilateral


a. Sugarprimarily lactose, usually not detected leakage of colostrum is common.
by conventional dipstick. b. Note: reddened areas, elevations, supernumerary
b. Acetonuriaafter prolonged labor; nipples, inverted nipples, cracks.
dehydration. c. Observe for signs of (normal) engorgement
c. Proteinuriafirst 3 days in response to the (i.e., tenderness, distention, prominent veins).
catalytic process of involution 1+. Transient; normally occurs shortly before
3. Dilation of ureterssubsides in first few weeks. lactation is establisheddue to venous and
4. Assessment: potential complicationsmeasure lymphatic stasis.
first few voidings, palpate bladder to determine d. Palpate for: local heat, edema, tenderness,
emptying. swelling (signs of localized infection).
a. Edema, trauma, or anesthesia may reten- 4. Fundus, lochia, perineum.
tion with overflow. 5. Voiding and bowel function.
b. Overdistended bladdercommon cause 6. Legs (see I. G.).
of excessive bleeding in immediate 7. Signs of complications.
postpartum. B. Analysis/nursing diagnosis (see VI. NURSING
F. Integument (skin)characteristic changes: ACTIONS DURING THE FOURTH STAGE
1. Striaepersist as silvery or brownish lines. OF LABOR, pp. 172173).
2. Diastasis recti abdominissome midline separa- C. Nursing care plan/implementation:
tion may persist. 1. Goal: comfort measures.
3. Diaphoresisexcessive perspiration for first few a. Perineal careto promote healing, prevent
MATERNAL/INFANT

(approximately 5) days. infection.


4. Breast changes (see II. A. 3. Breasts, below). b. Sitz bathsto promote healing.
5. Linea nigra and darkened areolae fade. c. Apply topical anesthetics, witch hazel to
G. Legs episiotomy area, hemorrhoids.
1. Should have no redness, tenderness, local areas d. Administer mild analgesia, as ordered.
of increased skin temperature, or edema. e. Instruct in tensing buttocks on position
2. May have some soreness from birth position. changeto reduce stress on suture line,
3. Homans sign should be negative (no calf pain discomfort.
when knee is extended and gentle pressure f. Breast care: mother who is bottle-feeding
applied to dorsiflex the foot). (1) Wash daily with clear water and mild soap.
H. Weightcharacteristic changes: (2) Support with well-fitting brassiere.
1. Initial weight lossfetus, placenta, amniotic (3) For engorgement:
fluid, excess tissue fluid. (a) Prevent with tight binder.
2. Weighs more than in prepregnant state (weight (b) Treat with ice pack and mild analgesic.
maintained in breasts). (c) Avoid nipple stimulation.
3. Week 6weight loss is individualized. (4) See also Breastfeeding and lactation
I. Menstruation and ovarian functionfirst (pp. 192193).
menstrual cycle may be anovulatory. 2. Goal: encourage normal bowel function. (Normal
1. Nonnursingovulation at 4 to 6 weeks; to take 1 to 3 days for function to resume.)
menstruation at 6 to 8 weeks. a. Administer stool softeners, as ordered.
2. Nursinganovulatory period varies (39 days b. Encourage ambulation.
to 6 months or more); some for duration of c. Increase dietary fiber (salads, fresh fruit,
lactation; contraceptive value: very unreliable. vegetables, bran cereals).
II. NURSING MANAGEMENT DURING d. Provide adequate fluid intake.
THE POSTPARTUM PERIOD 3. Goal: health teaching and discharge planning.
a. Reinforce appropriate perineal self-care.
A. Assessmentminimum of twice daily.
b. Reinforce hand washing (see VI. NURSING
1. Vital signs.
ACTIONS DURING THE FOURTH
2. Emotional status, response to baby.
STAGE OF LABOR, pp. 172173).
3. Breasts
c. Infant care
a. Observe: size, symmetry, placement and
(1) Bathing, cord care, circumcision care,
condition of nipples, leakage of colostrum.
diapering.
Normal: although one breast is usually larg-
(2) Feeding, burping, scheduling.
er than the other, breasts are essentially sym-
(3) Assessmenttemperature, skin color,
metrical in shape; nipples: in breast midline,
newborn rash, jaundice.
2164_Ch04_115-248 29/03/12 12:23 PM Page 191

The Postpartum Period 191


(4) Normal stool cycle and voiding pattern. b. Common maternal emotional/behavior
(5) Common sleep/activity patterns. changes, feelings:
(6) Signs to report immediately: (1) Jealous of infant; guilt feelings.
(a) Fever, vomiting, diarrhea. (2) Baby bluesdue to hormonal fluctua-
(b) Signs of inflammation or infection at tions, fatigue, change of lifestyle.
cord stump. (3) Feelings of inadequacy.
(c) Bleeding from circumcision site. c. Discuss support groups, aid in identifying
(d) Lethargy, irritability. supportive people.
d. Self-care D. Evaluation/outcome criteria:
(1) Adequate rest, nutrition, hydration. 1. Woman experiences normal, uncomplicated
(2) Breast self-examination; wear bra to postpartum period. All assessment findings
support breasts and promote comfort. within normal limits.
(3) Normal process of involution; lochial 2. Woman returns demonstrations of appropriate
patterns. self-care measures/techniques:
e. Resumption of intercourse approximately a. Perineal care, pad change, hand washing.
4 weeks postpartum (wait until lochia stops). b. Breast care, breast self-examination.
(1) Explain that time interval varies as to first 3. Woman verbalizes understanding of:
postpartum ovulation. a. Need for adequate rest and diversion.
(2) Family planning options may resume if b. Appropriate time for resumption of intercourse
desired: and exercise.
(a) If not breastfeeding, oral contraceptives c. Appropriate nutritional intake to meet needs

MATERNAL/INFANT
(estrogen and progesterone); low-dose (own and, if breastfeeding, infants).
progesterone given to mothers who are d. Signs to be reported immediately.
breastfeeding (see Table 4.1). e. Returns demonstration of appropriate infant
(b) Long-acting progestins (subcutaneous care measures.
implants or injectable). Safe to use f. Evidences beginning comfort and increasing
during lactation. confidence in parenting role.
(c) Use of IUD or diaphragm decided at E. Postpartum assessment6 or less weeks after
postpartum checkup. birth:
(d) Emphasize need to recheck size and fit 1. Weight, vital signs, urine for protein, complete
of diaphragm. blood count (CBC).
(e) Other options: condom plus 2. Breast examination lactating or not.
spermicides. 3. Pelvic examinationinvolution and position of
f. Exercisesto restore muscle tone, relieve uterus; perineal healing; tone of pelvic floor.
tension. 4. Desire for selection of method of contraception.
(1) Mild exercise during first few weeks. III. PSYCHOLOGICAL/BEHAVIORAL CHANGES.
(a) Deep abdominal breathing. Achievement of developmental tasksprogress in
(b) Supine head-raising. assuming maternal role.
(c) Stretching from head to toe.
A. Assessment:
(d) Pelvic tilt.
1. Taking-in phase1 to 3 days following birth.
(e) Kegelto regain perineal muscle tone.
a. Talkative; verbally relives labor/birth experience.
(2) Strenuous exercises (sit-ups, leg lifts)
b. Passive, dependent, concerned with own
deferred until later in postpartum.
needs (eating, sleeping, elimination).
g. Maternal signs to report immediately:
2. Taking-hold phaseday 3 to 2 weeks.
(1) Prolonged lochia rubra.
a. Impatient to control own bodily functions,
(2) Cramping.
care for self.
(3) Signs of infection.
b. Expresses interest/concern in learning
(4) Excessive fatigue, depression.
how to care for baby (desire to assume
(5) Dysuria.
mothering role).
4. Goal: anticipatory guidancedischarge planning:
c. Responds to positive reinforcement.
mothers are discharged earlier in their postpar-
3. Letting-go phasemother lets go of former
tum recovery today(648 hours after birth if
self-concept, role, lifestyle; begins to integrate
asymptomatic).
new role and self-concept as mother.
a. Discuss, assist in organizing time schedule.
a. Feelings of insecurity, inadequacy.
Nap, when possible, when infant asleepto
b. Hesitancy in approaching infant care tasks.
minimize fatigue.
2164_Ch04_115-248 29/03/12 12:23 PM Page 192

192 chapter 4 Health Promotion and Maintenance

4. Baby bluesmay appear on day 4 or 5. b. Increased secretion of prolactinstimulates


Note: Often, father/partner experiences same alveolar cells milk.
feelings. c. Sucklingstimulates release of oxytocin
a. Thought to result from fatigue (sleep depri- contraction of ducts milk ejection
vation), realization of need for role change, (let-down reflex).
recognition of new responsibilities, hormonal d. Engorgementdue to venous and lymphatic
change. stasis.
b. Mild depression, cries without provocation. (1) Immediately precedes lactation.
c. Frightenedintimidated by own perceptions (2) Lasts about 24 hours.
of responsibilities, hormonal changes. (3) Frequent feeding reduces engorgement.
5. Lag in experiencing maternal feelingsusually B. Assessment:
resolved within 6 weeks. 1. Colostrum (yellowish fluid)continues for
a. May contribute to baby blues. first 2 to 3 days; has some antibiotic, immuno-
b. Guilt regarding lack of maternal feelings. logic, and nutritive value.
c. Diminished by prompt bonding experience. 2. Milk (bluish-white, thin)secreted on about
B. Analysis/nursing diagnosis: third day.
1. Ineffective family coping: compromised, related to C. Analysis/nursing diagnosis:
achieving developmental tasks. 1. Knowledge deficit related to breastfeeding
2. Situational low self-esteem related to perceived techniques.
inadequacy in acceptance of maternal role. 2. Pain related to engorgement.
3. Ineffective individual coping related to baby 3. Personal identity disturbance related to problems
MATERNAL/INFANT

blues, lag in experiencing maternal feelings. in breastfeeding.


C. Nursing care plan/implementation: 4. Sleep pattern disturbance related to discomfort or
1. Taking-in. Goal: emotional support. infant care needs.
a. Encourage verbalization of labor/birth experi- D. Nursing care plan/implementation:
ences; compliment parents on how well 1. Goal: promote successful breastfeeding.
they did. a. Encourage first feeding within 1 hour after
b. Explore feelings of disappointment, if any. giving birth.
c. Meet dependency needs; comment on b. Encourage emptying both breasts at each
appearance, hair, personal gowns. feeding and before engorgement to stimulate
d. Encourage rooming in. milk production, prevent mastitis.
2. Taking-hold. Goal: health teaching. c. Encourage rest, relaxation, fluids.
a. Discuss self-care, postpartum d. Nutritional counseling (see Chapter 9).
physiological/psychological changes. (1) Additional 500 calories dailymay be
b. Demonstrate infant care; mother returns supplied by one extra pint of milk, one
demonstration. extra egg, and one extra serving of meat,
D. Evaluation/outcome criteria: citrus fruit, and vegetable.
1. Woman demonstrates beginning comfort in (2) Increase fluid intake to 3000 mL daily.
maternal role. 2. Goal: prevent or relieve engorgement.
2. Woman develops confidence and competence in a. Pain: relieved by warm packs, emptying
infant care. breasts.
3. Woman expresses satisfaction with self, infant; b. Wear good, supportive bra.
eager to return home. c. Administer analgesics, as ordered/necessary.
4. Woman succeeds in breastfeeding. (Tension 3. Goal: health teaching.
inhibits let-down reflex; baby nurses poorly.) a. Instruct, demonstrate rooting reflex and
IV. BREASTFEEDING AND LACTATION putting infant to breast. Infant must grasp
nipple and areola over location of milk
A. Biological foundations:
sinuses.
1. Antepartal alterations:
b. Demonstrate burping techniques, what to
a. High estrogen/progesterone levels
do if infant chokes; removing infant from
stimulate proliferation and development
breast.
of breast ducts.
c. Instruct in basic nipple care.
b. High progesterone levelsalso develop-
(1) Teach good hand washing.
ment of mammary lobules and alveoli.
(2)Nurse on each breast, making sure
2. Postpartum alterations:
areola is in mouth, alternating position
a. Rapid drop in estrogen/progesterone levels.
of infant.
2164_Ch04_115-248 29/03/12 12:23 PM Page 193

Complications During the Postpartum Period 193


(3) Alternate beginning breast.
(4) Break suction before removing infant from
Complications During
breast. the Postpartum Period
(5)Air-dry nipples after each feeding and
apply lanolin if abraded. Note: Creams, I. DISORDERS AFFECTING FLUID-GAS
lotions, or ointments block secretion of a TRANSPORT
natural bacteriostatic oil by Montgomery A. Postpartum hemorrhage
glandsand infant may refuse breast 1. Definitionloss of 500 mL of blood or more
until it is washed. Instead: expressed milk during first 24 hours postpartum in vaginal
may be massaged gently around nipple. birth; 1000 mL in cesarean birth.
(6) Teach daily hygiene of breasts. 2. Pathophysiologyexcessive loss of blood
d. Instruct in care of cracked or fissured nipples. secondary to trauma, decreased uterine contrac-
(1) Encourage and support mothers. tility; results in hypovolemia.
(2) Air-dry nipples after each feeding. 3. Etiology (in decreasing order of frequency):
(3)Use nipple shield if nipples extremely a. Uterine atony
sore. (1) Uterine overdistention (multipregnancy,
(4) Discontinue nursing for 48 hours; main- polyhydramnios, fetal macrosomia).
tain milk supply by expressing milk with (2) Multiparity.
pump. (3) Prolonged or precipitous labor.
e. Discuss avoiding use of any drugs except (4) Anesthesiadeep inhalation or regional
under medical supervisionmay affect (particularly saddle block).

MATERNAL/INFANT
infant or suppress lactation. (5) Myomata (fibroids).
f. Discuss possibility of sexual stimulation (6) Oxytocin induction of labor.
during breastfeeding. (7) Overmassage of uterus in postpartum.
(1) Validate normalcy and acceptability. (8) Distended bladder.
(2) Note: During orgasm, milk may squirt b. Lacerationscervix, vagina, perineum.
from nipples. c. Retained placental fragmentsusually
g. Explain that contraceptive value of nursing is delayed postpartum hemorrhage.
unpredictable; the time that ovulation is d. Hematomadeep pelvic, vaginal, or
inhibited varies widely. episiotomy site.
h. Explain contraindications to breastfeeding: 4. Assessment:
(1) Active tuberculosis. a. Uterusboggy, flaccid; excessive vaginal
(2) Severe chronic maternal disease. bleeding (dark; seepage, large clots)due to
(3) Mastitis (temporary interruption may be uterine atony, retained placental fragments.
necessary). b. Late signs of shockair hunger;
(4) Some therapeutic drugs. anxiety/apprehension, tachycardia, tachypnea,
(5) Severe cleft lip or palate in newborn (may hypotension.
pump and give in special bottles). c. Blood values (admission and postpartum)
(6) HIV-positive status; AIDS. hemoglobin (Hgb), hematocrit (Hct),
E. Evaluation/outcome criteria: clotting time.
1. Woman verbalizes understanding of breastfeed- d. Estimated blood loss: during labor/birth; in
ing techniques, nutritional requirements for early postpartum.
successful lactation. e. Pain: vulvar, vaginal, perineal.
2. Woman successfully demonstrates breastfeeding; f. Perineum: distendeddue to edema;
infant nurses well. discolorationdue to hematoma. May
3. Woman demonstrates appropriate burping complain of rectal pressure.
techniques; clears excessive mucus from infants g. Lacerationsbright red vaginal bleeding
mouth without incident. with firm fundus.
4. Woman verbalizes understanding of basic breast 5. Analysis/nursing diagnosis:
care techniques: a. Fluid volume deficit related to excessive
a. Self-examination. blood loss secondary to uterine atony,
b. Clear water bath. retained placental fragments.
c. Drying nipples after bathing, feeding. b. Anxiety/fear related to unexpected complication.
d. Care of cracked or irritated nipples. c. Altered tissue perfusion related to decreased
e. Correct infant positioning for feeding. oxygenation secondary to blood loss.
d. Activity intolerance related to fatigue.
2164_Ch04_115-248 29/03/12 12:23 PM Page 194

194 chapter 4 Health Promotion and Maintenance

6. Nursing care plan/implementation: b. Retained placental fragments.


a. Medical management: c. Oxytocin stimulation or augmentation of
(1) IV oxytocin infusion; IV or oral ergot labor of overdistended uterine muscle may
preparations (ergonovine [Ergotrate interfere with involution.
Maleate]; methylergonovine [Methergine]; 3. Assessment:
carboprost (Prostin/M15), an oxytocic; a. Uterus: large, boggy; lack of uterine tone;
prostaglandin. failure to shrink progressively.
(2) Order blood work: clotting time, platelet b. Discharge: persistent lochia; painless fresh
count, fibrinogen level, Hgb, Hct, CBC. bleeding, hemorrhagic episodes.
(3) Type and crossmatch for blood replacement. 4. Analysis/nursing diagnosis:
(4) Surgical: a. Pain related to tender, inflamed uterus
(a) Repair of lacerations. secondary to endometritis.
(b) Evacuation, ligation of hematoma. b. Anxiety/fear related to change in physical
(c) Curettageretained placental fragments. status.
b. Nursing management: c. Knowledge deficit related to diagnosis,
(1) Goal: minimize blood loss. treatment, prognosis.
(a) Notify physician promptly of abnormal d. High risk for injury related to infection.
assessment findings. e. Fluid volume deficit related to excessive
(b) Order lab work STAT, as directedto bleeding.
determine blood loss and etiology. 5. Nursing care plan/implementation:
(c) Fundal massage. a. Medical management:
MATERNAL/INFANT

(d) Administer medications to stimulate (1) Have woman void or catheterize; massage
uterine tone. For ergot products and fundus.
carboprost, monitor blood pressure (2) Surgical (curettage)to remove placental
(contraindicated in PIH). fragments.
(2) Goal: stabilize status. (3) Antibiotic therapyto treat intrauterine
(a) Establish IV lineto enable adminis- infection.
tration of medications and rapid (4) Oxytocicsto stimulate/enhance uterine
absorption/action. Administer whole contractions.
blood (with larger catheter). b. Nursing management:
(b)Administer medications, as ordered (1) Goal: health teaching.
to control bleeding, combat shock. (a) Explain condition and treatment.
(c) Prepare for surgery, as ordered. (b) Describe, demonstrate perineal care,
(3) Goal: prevent infection. Strict aseptic pad change, hand washing.
technique. (2) Goal: emotional support. Encourage
(4) Goal: continual monitoring. Vital signs, verbalization of anxiety regarding
bleeding (do pad count or weigh pads), recovery, separation from newborn.
fundal status. (3) Goal: promote healing.
(5) Goal: prevent sequelae (Sheehans syndrome). (a) Encourage rest, compliance with
(6) Goal: health teachingafter episode: medical/nursing regimen.
Reinforce appropriate perineal care and (b) Administer oxytocics, antibiotics, as
hand-washing techniques. ordered.
7. Evaluation/outcome criteria: 6. Evaluation/outcome criteria:
a. Maternal vital signs stable. a. Verbalizes understanding of condition and
b. Bleeding diminished or absent. treatment.
c. Assessment findings within normal limits. b. Complies with medical/nursing regimen.
B. Subinvolutiondelayed return of uterus to nor- c. Demonstrates normal involutional
mal size, shape, position. progress.
1. Pathophysiologyinability of inflamed uterus d. All assessment findings (vital signs, fundal
(endometritis) to contract effectively incom- height, consistency, lochial discharge)
plete uterine involution; failure of contractions within normal limits.
to effect closure of vessels in site of placental e. Expresses satisfaction with care.
attachment bleeding. C. Hypofibrinogenemia
2. Etiology: 1. Pathophysiologydecreased clotting
a. PROM with secondary amnionitis, factors, fibrinogen; may be accompanied
endometritis. by DIC.
2164_Ch04_115-248 29/03/12 12:23 PM Page 195

Complications During the Postpartum Period 195


2. Etiology: b. Nursing management:
a. Missed abortion (retained dead fetus syndrome). (1) Goal: continuous monitoring.
b. Fetal death, delayed emptying of uterine (a) Vital signs.
contents. (b) I&O hourly.
c. Abruptio placentae; Couvelaire uterus. (c) Skin: color, emergence of petechiae.
d. Amniotic fluid embolism. (d) Note, measure (as possible), record,
e. Hypertension. and report blood loss.
3. Assessment: (2) Goal: control blood loss.
a. Observe for bleeding from injection sites, (a) Establish IV line, administer fluids or
epistaxis, purpura. blood products as ordered.
b. See DIC assessment, p. 429 in Chapter 6. (b) Position: side-lyingto maintain blood
c. Maternal vital signs, color. supply to vital organs.
d. I&O. (3) Goal: emotional support.
e. Medical evaluationprocedures. (a) Encourage verbalization of anxiety,
(1) Thrombin clot testimportant: size fear, concerns.
and persistence of clot. (b) Explain all procedures.
(2) Prothrombin timeprolonged. (c) Remain with woman continuously.
(3) Bleeding timeprolonged. (d) Keep woman and family informed.
(4) Platelet countdecreased. 6. Evaluation/outcome criteria:
(5) Activated partial thromboplastin time a. Bleeding controlled.
prolonged. b. Laboratory studiesreturning to normal

MATERNAL/INFANT
(6) Fibrinogen (factor I concentration) values.
decreased. c. Status stable.
(7) Fibrin degradation productspresent. II. DISORDERS AFFECTING PROTECTIVE
4. Analysis/nursing diagnosis: FUNCTIONS: postpartum infection (Table 4.13).
a. Fluid volume deficit related to uncontrolled
A. General aspects
bleeding secondary to coagulopathy.
1. Definitionreproductive system infection
b. Anxiety/fear related to unexpected critical
occurring during the postpartum period.
emergency.
2. Pathophysiologybacterial invasion of birth
c. Altered tissue perfusion related to decreased
canal; most common: localized infection of the
oxygenation secondary to blood loss.
lining of the uterus (endometritis).
5. Nursing care plan/implementation:
3. Etiology:
a. Medical management:
a. Anaerobic nonhemolytic streptococci.
(1) Replace platelets.
b. E. coli.
(2) Replace blood loss.
c. C. trachomatis (bacteroides).
(3) IV heparinto inhibit conversion of fib-
d. Staphylococci.
rinogen to fibrin.

Table 4.13
Postpartum Infections
Condition/Etiology Assessment: Signs/Symptoms Nursing Interventions
Postpartum Infection
Traumatic labor and birth and Depends on location and 1. Monitor: Signs and symptoms, drainage (e.g., uterine)
postpartum hemorrhage make severity of infection; usually 2. Obtain culture and sensitivity
woman more vulnerable to include fever, pain, swelling, 3. Administer antimicrobial agents and analgesic
infection by such bacteria as and tenderness agents
nonhemolytic streptococci, Temperature of 100.4F 4. Ensure comfort; encourage rest
Escherichia coli, and (38C) or more after first 5. Use standard precautions
Staphylococcus species 24 hours after birth on two 6. Force fluids and provide high-calorie diet
or more occasions indicates 7. Keep family informed of mothers and newborns
puerperal infection (childbed progress
fever) 8. Promote maternal-infant contact as soon as
possible
9. Plan and implement discharge and follow-up care
Continued
2164_Ch04_115-248 29/03/12 12:23 PM Page 196

196 chapter 4 Health Promotion and Maintenance

Table 4.13
Postpartum Infectionscontd
Condition/Etiology Assessment: Signs/Symptoms Nursing Interventions
Endometritis
Microorganisms invade Temperature, chills, anorexia, 1. Administer antimicrobial agents and analgesic
placental site and may spread malaise, boggy uterus, agents
to entire endometrium foul-smelling lochia, and 2. Encourage Fowlers position to promote
cramps drainage
3. Force fluids
4. Take standard precautions
Pelvic Cellulitis or Parametritis
Microorganisms spread through Fever, chills, lower abdominal 1. Administer antimicrobial agents and analgesic
lymphatics and invade tissues pain, and tenderness agents
surrounding uterus 2. Encourage bedrest
3. Force fluids
Perineal Infection
Trauma to perineum makes Localized pain, fever, swelling, 1. Administer antimicrobial agents and analgesic
woman more vulnerable to redness, and seropurulent agents
infection drainage 2. Provide sitz baths or heat/cold applications
3. Take standard precautions
MATERNAL/INFANT

Mastitis
Lesions or fissure on nipples Marked engorgement, pain, 1. Order culture and sensitivity studies of mothers milk
allow entry of microorganisms chills, fever, tachycardia 2. Administer antimicrobial agents and analgesic
(e.g., Staphylococcus aureus) If untreated, single or multi- agents
from infants nose/mouth or ple breast abscesses may 3. Apply heat or cold therapy
mothers unwashed hands form 4. Assist with incising and draining abscesses
(Breastmilk is a good medium 5. Use standard precautions and perform meticulous hand
for growth of organism) washing
Thrombophlebitis
Infected pelvic or femoral Pain, chills, and fever
thrombi Femoral: stiffness of affected Femoral:
area or part and positive 1. Rest and elevate leg
Increased tendency to clot
Homans sign 2. Administer antimicrobial agents, analgesic agents,
formation during pregnancy,
and anticoagulants
trauma to tissues, and
hemorrhage decrease new
Pelvic: severe chills and wide Pelvic:
mothers resistance to infection
fluctuations in temperature 1. Encourage bedrest
2. Force fluids
3. Administer antimicrobial agents and
anticoagulants

4. Predisposing conditions: 6. Analysis/nursing diagnosis:


a. Anemia. a. Fluid volume deficit related to excessive blood
b. PROM. loss, anemia.
c. Prolonged labor. b. Knowledge deficit related to danger signs of
d. Repeated vaginal examinations during labor. postpartum period.
e. Intrauterine manipulation (e.g., manual c. High risk for injury related to infection.
extraction of placenta). 7. Nursing care plan/implementation: prevention
f. Retained placental fragments. a. Goal: prevent anemia.
g. Postpartum hemorrhage. (1) Minimize blood lossaccurate postpar-
5. Assessment: tum assessment and management of
a. Fever 38C (100.4F) or more on two bleeding.
or more occasions, after first 24 hours (2) Diet: high protein, high vitamin.
postpartum. (3) Vitamins, ironsuggest continuing
b. Other signs of infection: pain, malaise, prenatal pattern until postpartum
dysuria, subinvolution, foul lochial odor. checkup.
2164_Ch04_115-248 29/03/12 12:23 PM Page 197

Complications During the Postpartum Period 197


b. Goal: prevent entrance/transport of e. Altered family processesinterruption of
microorganisms. adjustment to altered life pattern related to
(1) Strict aseptic technique during labor, birth, postpartum infection/hospitalization.
and postpartum (standard precautions). 6. Nursing care plan/implementation:
(2) Minimize vaginal examinations during a. Goal: prevent cross-contamination. Contact-
labor. item isolation.
(3) Perineal care. b. Goal: facilitate drainage. Position: semi-Fowlers.
c. Goal: health teaching. c. Goal: nutrition/hydration.
(1) Hand washingbefore and after each pad (1) Diet: high calorie, high protein, high vitamin.
change, after voiding or defecating. (2) Push fluids to 4000 mL/day (oral or IV,
(2) Perineal carefrom front to back; use clear, or both, as ordered).
warm water or mild antiseptic solution as a (3) I&O.
cascade; do not separate labia. d. Goal: increase uterine tone/facilitate involu-
(3) Maintain sterility of pads; apply from tion. Administer medications, as ordered
front to back. (e.g., oxytocics, antibiotics).
(4) Avoid use of tampons until normal e. Goal: minimize energy expenditure, as possible.
menstrual cycle resumes. (1) Bedrest.
8. Evaluation/outcome criteria: (2) Maximize rest, comfort.
a. Woman has assessment findings within f. Goal: emotional support.
normal limits: (1)Encourage verbalization of anxiety,
(1) Vital signs. concerns.

MATERNAL/INFANT
(2) Rate of involution (fundal height, (2) Keep informed of progress.
consistency). 7. Evaluation/outcome criteria:
(3) Lochia: character, amount, odor. a. Vital signs stable, within normal limits.
b. Woman avoids infection. b. All assessment findings within normal limits.
B. Endometritisinfection of lining of uterus. c. Unable to recover organism from discharge.
1. Pathophysiology (see II. A. General aspects, C. Urinary tract infections
p. 195). 1. Pathophysiologynormal physiological
2. Etiologymost common: invasion by normal changes associated with pregnancy (e.g., ureteral
body flora (e.g., anaerobic streptococci). dilation) and the postpartum period (e.g., diure-
3. Characteristics: sis, increased bladder capacity with diminished
a. Mild, localizedasymptomatic, or low-grade sensitivity of stretch receptors) increased
fever. susceptibility to bacterial invasion and
b. Severemay lead to ascending infection, growth ascending infections (cystitis,
parametritis, pelvic abscess, pelvic pyelonephritis).
thrombophlebitis. 2. Etiology: usually bacterial.
c. If remains localized, self-limiting; usually 3. Predisposing factors:
resolves within 10 days. a. Birth trauma to bladder, urethra, or meatus.
4. Assessment: b. Bladder hypotonia with retention (due to
a. Signs of infection: fever, chills, malaise, intrapartum anesthesia or trauma).
anorexia, headache, backache. c. Repeated or prolonged catheterization, or
b. Uterus: large, boggy, extremely tender. poor technique.
(1) Subinvolution. d. Weakening of immune response secondary to
(2) Lochia: dark brown; foul odor. anemia, hemorrhage.
5. Analysis/nursing diagnosis: 4. Assessment:
a. Anxiety/fear related to effects on self and a. Maternal vital signs (fever, tachycardia).
newborn. b. Dysuria, frequency (flank painwith
b. Self-esteem disturbance and altered role per- pyelonephritis).
formance related to inability to meet own c. Feeling of not emptying bladder.
expectations regarding parenting, secondary d. Cloudy urine; frank pus.
to unexpected hospitalization. 5. Analysis/nursing diagnosis:
c. Pain related to inflammation/infection. a. Altered urinary elimination related to diuresis,
d. Ineffective individual coping related to physical dysuria, inflammation/infection.
discomfort and psychological stress associated b. Pain related to dysuria secondary to cystitis.
with self-concept disturbance; worry, guilt, c. Knowledge deficit related to self-care (perineal
concern regarding newborn at home. care).
2164_Ch04_115-248 29/03/12 12:23 PM Page 198

198 chapter 4 Health Promotion and Maintenance

6. Nursing care plan/implementation: 5. Nursing care plan/implementation:


a. Goal: minimize perineal edema. Perineal a. Goal: prevent infection. Health teaching in
ice pack in fourth stageto limit swelling early postpartum:
secondary to trauma, facilitate voiding. (1) Hand washing.
b. Goal: prevent overdistention of bladder. (2) Breast carewash with warm water only
(1) Monitor level of fundus, lochia, bladder (no soap)to prevent removing protective
distention. (Note: Distended bladder body oils.
displaces uterus, limits its ability to (3) Let breast milk dry on nipples to prevent
contract boggy fundus, increases its drying of tissue.
vaginal bleeding.) (4) Clean bra (with no plastic pads or liners) to
(2) Encourage fluids and voiding; I&O. support breasts, reduce friction, minimize
(3) Aseptic technique for catheterization. exposure to microorganisms.
(4)Slow emptying of bladder on (5) Good breastfeeding techniques
catheterizationto maintain tone. (see pp. 192193).
c. Goal: identification of causative organismto (6) Alternate position of infant for nursing to
facilitate appropriate medication (antibiotics). change pressure areas.
Obtain clean-catch (or catheterized) specimen b. Goal: comfort measures.
for culture and sensitivity. (1) Encourage bra or binderto support
d. Goal: health teaching. See previous discussion of breasts, reduce pain from motion.
fluids, general hygiene, diet, and medications. (2) Local heat or ice packs as orderedto
7. Evaluation/outcome criteria: reduce engorgement, pain.
MATERNAL/INFANT

a. Voiding: quantity sufficient (although small, (3) Administer analgesics, as necessary.


frequent output may mean overflow with c. Goal: emotional support.
retention). (1) Encourage verbalization of feelings, concerns.
b. Urine character: clear, amber, or straw colored. (2)If breastfeeding is discontinued, reassure
c. Vital signs: within normal limits. woman she will be able to resume
d. No complaints of frequency, urgency, burning breastfeeding.
on urination, flank pain. d. Goal: promote healing.
D. Mastitisinflammation of breast tissue: (1) Maintain lactation (if desired) by manual
1. Pathophysiologylocal inflammatory response expression or breast pump, q4h.
to bacterial invasion; suppuration may occur; (2) Administer antibiotics as ordered.
organism can be recovered from breast milk. 6. Evaluation/outcome criteria:
2. Etiologymost common: Staphylococcus aureus; a. Woman promptly responds to medical/
sourcemost common: infants nose, throat. nursing regimen.
3. Assessment: (1) Symptoms subside.
a. Signs of infection (may occur several weeks in (2) Assessment findings within normal limits.
postpartum). b. Woman successfully returns to breastfeeding.
(1) Fever. E. Thrombophlebitis
(2) Chills. 1. Pathophysiologyinflammation of a vein
(3) Tachycardia. secondary to lodging of a clot.
(4) Malaise. 2. Etiology:
(5) Abdominal pain. a. Extension of endometritis with involvement
b. Breast of pelvic and femoral veins.
(1) Reddened area(s). b. Clot formation in pelvic veins following
(2) Localized/generalized swelling. cesarean birth.
(3) Heat, tenderness, palpable mass. c. Clot formation in femoral (or other) veins
4. Analysis/nursing diagnosis: secondary to poor circulation, compression,
a. Impaired skin integrity related to nipple and venous stasis.
fissures, cracks. 3. Assessment:
b. Pain related to tender, inflamed tissue a. Pelvicpain: abdominal or pelvic tenderness.
secondary to infection. b. Calfpain: positive Homans sign (pain elicited
c. Disturbance in body image, self-esteem related by flexion of foot with knee extended).
to association of breastfeeding with female c. Femoral
identity and role. (1) Pain.
d. Anxiety/fear related to sexuality; impact on (2) Malaise, fever, chills.
breastfeeding, if any. (3) Swellingmilk leg.
2164_Ch04_115-248 29/03/12 12:23 PM Page 199

Complications During the Postpartum Period 199


4. Analysis/nursing diagnosis: 4. Depressionmay alternate with manic behavior.
a. Pain in affected region related to local 5. Potential for self-injury or child abuse/neglect.
inflammatory response. D. Analysis/nursing diagnosis:
b. Anxiety/fear related to outcome. 1. Ineffective individual coping related to perceived
c. Ineffective individual coping related to inability to meet role expectations (mother)
unexpected postpartum complications, and ambivalence related to dependence/
hospitalization, separation from newborn. independence.
d. Impaired physical mobility related to imposed 2. Self-esteem disturbance and altered role perform-
bedrest to prevent emboli formation and ance related to femaleness and reaction to
dislodging clot (embolus). responsibility for care of newborn.
5. Nursing care plan/implementation: 3. High risk for violence, self-directed or directed at
a. Goal: prevent clot formation. newborn, related to anger or depression.
(1) Encourage early ambulation. 4. Ineffective family coping related to lack of support
(2) Position: avoid prolonged compression system in early postpartum.
of popliteal space, use of knee gatch. 5. Altered family processes related to psychological
(3) Apply thromboembolic disease (TED) stress, interruption of bonding.
hose, or sequential compression device, as 6. Altered parenting related to hormonal changes
ordered, preoperatively or postoperatively, and stress.
or both, for cesarean birth. E. Nursing care plan/implementation:
b. Goal: reduce threat of emboli. 1. Goal: emotional support.
(1) Bedrest, with cradle to support bedding. a. Encourage verbalization of feelings, fears,

MATERNAL/INFANT
(2) Discourage massaging leg cramps. anxiety, concerns.
c. Goal: prevent further clot formation. b. Support positive self-image, feelings of
Administer anticoagulants, as ordered. adequacy, self-worth.
d. Goal: prevent infection. (1) Reinforce appropriate comments and
(1) Administer antibiotics, as ordered. behaviors.
(2) Push fluids. (2) Encourage active participation in self-care,
e. Goal: facilitate clot resolution. Heat therapy, as comment on accomplishments.
ordered. (3) Reduce threat to self-image, fear of failure.
6. Evaluation/outcome criteria: Maintain support, gradually increase tasks.
a. Symptoms subside; all assessment findings 2. Goal: safeguard status of mother/newborn.
within normal limits. a. Unobtrusive, protective environment.
b. No evidence of further clot formation. b. Stay with woman when she is with infant.
III. DISORDERS AFFECTING PSYCHOSOCIAL- 3. Goal: nutrition/hydration.
CULTURAL FUNCTIONSpostpartum a. Encourage selection of favorite foodsto aid
depression/psychosis security in decision making; counteract
anorexia (refusal to eat) by tempting appetite.
A. General aspects
b. Push fluids (juices, soft drinks, milkshakes)
1. Can occur in both new parents and experienced
to maintain hydration.
parents.
4. Goal: minimize stress, facilitate effective
2. Usually occurs within 2 weeks of birth.
coping. Administer therapeutic medications,
3. Increased incidence among single parents
as ordered.
4. Increased incidence among women with history
a. Schizophreniaphenothiazines.
of clinical depression.
b. Depressionmood elevators.
5. Most common symptomatology: affective
c. Manic behaviorssedatives, tranquilizers.
disorders.
F. Evaluation/outcome criteria:
6. Psychiatric intervention required if prolonged or
1. Woman increases interaction with infant.
severe; if underlying cause unresolved; increased
2. Woman expresses interest in learning how to
risk in subsequent pregnancies.
care for infant.
B. Etiologytheory: birth of child may emphasize:
3. Woman evidences no agitation, depression.
1. Unresolved role conflicts.
4. Woman actively participates in caring for self
2. Unachieved normal development tasks.
and infant.
C. Assessment:
5. Woman demonstrates increasing comfort in
1. Withdrawal.
mothering role.
2. Paranoia.
6. Woman has positive family interactions.
3. Anorexia, sleep disturbance, mood swings.
2164_Ch04_115-248 29/03/12 12:23 PM Page 200

200 chapter 4 Health Promotion and Maintenance

THE NEWBORN 2. Umbilical cordextends from fetus to center of


I N FA N T placenta: usually 50 cm (1822 inches) long and
1 to 2 cm (1/21 inch) in diameter. Contains:
General overview: Effective nursing care of the newborn a. Whartons jellyprotects umbilical vessels
infant is based on: (1) knowledge of the conditions pres- from pressure, cord kinking, and interfer-
ent during fetal life; (2) requirements for independent ence with fetal-placental circulation.
extrauterine life; and (3) alterations needed for successful b. Umbilical veincarries oxygen and nutrients
transition. The first 24 hours are the most hazardous. from placenta to fetus.
I. BIOLOGICAL FOUNDATIONS OF NEONATAL c. Two umbilical arteriescarry deoxygenated
ADAPTATIONGeneral aspects: blood and fetal wastes from fetus to placenta.
A. Fetal anatomy and physiology Note: Absence of one artery indicates need to
1. Fetal circulationfive intrauterine structures rule out intra-abdominal anomalies.
that differ from extrauterine structures 3. Characteristics of fetal blood
(Fig. 4.11): a. Fetal hemoglobin (HbF)
a. Umbilical veincarries oxygen and nutrient- (1) Higher oxygen-carrying capacity than
enriched blood from placenta to ductus adult hemoglobin.
venosus and liver. (2) Releases oxygen easily to fetal tissues.
b. Ductus venosusconnects to inferior vena (3) Ensures high fetal oxygenation.
cava; allows most blood to bypass liver. (4) Normal range at term: 12 to 22 g/dL;
c. Foramen ovaleallows fetal blood to bypass average: 15 to 20 g/dL.
fetal lungs by shunting it from right atrium b. Total blood volume at term: 85 mL/kg body
MATERNAL/INFANT

into left atrium. weight; Hct: 38% to 62%, average 53%;


d. Ductus arteriosusallows fetal blood to bypass RBCs: 3 to 7 million, average 4.9 million/unit.
fetal lungs by shunting it from pulmonary B. Extrauterine adaptation: tasks
artery into aorta. 1. Establish and maintain ventilation, successful gas
e. Umbilical arteries (two)allow return of transferrequires patent airway and adequate
deoxygenated blood to the placenta. pulmonary surfactant.

AORTIC ARCH
DUCTUS ARTERIOSUS
PULMONARY ARTERY

LEFT ATRIUM

FORAMEN
OVALE
AORTA
RIGHT
ATRIUM
DUCTUS VENOSUS

INFERIOR VENA CAVA


UMBILICAL CORD
UMBILICAL ARTERIES
UMBILICAL VEIN
NAVEL OF PLACENTA
FETUS

MATERNAL
BLOOD VESSELS

Figure 4.11 Fetal circulation. (From Venes, D


[ed]: Tabers Cyclopedic Medical Dictionary, ed 20. FA INTERNAL
Davis, Philadelphia, 2005, p 422.) ILIAC ARTERIES
2164_Ch04_115-248 29/03/12 12:23 PM Page 201

The Newborn Infant 201


2. Modify circulatory patternsrequires closure of 5. Vital signs:
fetal structures. a. Axillary temperature.
3. Absorb and utilize fluids and nutrients. b. Respirations (check rate, character, rhythm).
4. Excrete body wastes. c. Apical pulse.
5. Establish and maintain thermal stability. 6. General physical assessment (Table 4.14) and
C. Nursing care plan/implementation: reflexes (Table 4.15).
1. Facilitate successful transition to independent 7. Estimate of gestational age (Table 4.16).
life. B. Analysis/nursing diagnosis:
2. Protect infant from physiological stress and 1. Altered health maintenance related to separation
environmental hazards. from maternal support system.
3. Encourage development of a strong family 2. Impaired skin integrity related to umbilical stump;
unit. incontinence of urine and meconium stool; skin
II. ADMISSION/ASSESSMENT: 1 TO 4 HOURS penetration by scalp electrode, injections, heel
AFTER BIRTH stick, scalpel during cesarean birth; abrasion from
obstetric forceps.
A. Admission assessment of normal, term neonate
3. Ineffective airway clearance related to excessive
1. Color and reactivity.
mucus.
2. General appearance, symmetry.
4. Pain related to environmental stimuli.
3. Length and weight.
5. Ineffective thermoregulation related to immature
4. Head and chest circumferences.
temperature regulation mechanism.
(text continues on page 205)

MATERNAL/INFANT
Table 4.14
Physical Assessment of the Term Neonate
Criterion Average Values and Normal Variations Deviations from Normal
Vital Signs
Heart rate 120140/min, irregular, especially when Faint soundpneumomediastinum; and heart rate
crying, and functional murmur <100 beats/min or >180 beats/min
Respiratory rate 3060/min with short periods of apnea, Distressflaring of nares, retractions, tachypnea,
irregular; vigorous and loud cry grunting, excessive mucus, <30 respirations/min or
>60 respirations/min; cyanosis
Temperature Stabilizes about 810 hours after birth; Unreliable indicator of infection due to environmental
36.537C (97.798.6F) axillary influences
Blood pressure 6080/4050; varies with change in Hypotension: with RDS
activity level Hypertension: coarctation of aorta
Measurements
Weight 3400 gm (71/2 lbs); range: 5 lb 8 oz8 lb Birth weight <2500 gm: preterm or SGA infant;
13 oz >4000 gm: LGA infant, evaluate mother for gesta-
tional diabetes
Length 50 cm (20 inches); range: 1822 inches
Chest circumference 2 cm (3/4 inch) less than head If relationship varies, check for reason
circumference
Head circumference 3335 cm (1314 inches) Check for microcephalus and macrocephalus
General Assessment
Muscle tone Good tone and generalized flexion; full Flaccid, and persistent tremor or twitching; movement
range of motion; spontaneous movement limited; asymmetrical
Skin color Mottling, acrocyanosis, and physiological Pallor, cyanosis, or jaundice within 24 hours of birth
jaundice; petechiae (over presenting Petechiae or ecchymoses elsewhere; all rashes,
part), milia, mongolian spotting, lanugo, except erythema toxicum; pigmented nevi; heman-
and vernix caseosa gioma; and yellow vernix
Head Molding of fontanels and suture spaces; Cephalohematoma, caput succedaneum, sunken or
one fourth of body length bulging fontanels, closed sutures; excessively wide
sutures
Continued
2164_Ch04_115-248 29/03/12 12:23 PM Page 202

202 chapter 4 Health Promotion and Maintenance

Table 4.14
Physical Assessment of the Term Neonatecontd
Criterion Average Values and Normal Variations Deviations from Normal
Hair Silky, single strands, lies flat; grows toward Fine, wooly; unusual swirls, patterns, hairline; coarse
face and neck
Eyes Edematous eyelids, conjunctival hemorrhage; Epicanthal folds (in non-Asians); discharges; agenesis;
grayish-blue to grayish-brown; blink reflex, opaque lenses; lesions; strabismus; dolls eyes
usually no tears; uncoordinated movements beyond 10 days; absence of reflexes
may focus for a few seconds; good place-
ment on face; cornea is bright and shiny;
pupillary reflex equal and reactive to light;
eyebrows distinct
Nose Appears to have no bridge; should have no Discharge and choanal atresia; malformed; flaring of
discharge; preferential nose breathers; nares beyond first few moments of life
sneezes to clear nose
Mouth Epsteins pearls on gum ridges; tongue does Cleft lip or palate; teeth, cyanosis, circumoral pallor;
not protrude and moves freely, symmetrically; asymmetrical lip movement; excessive saliva; thrush;
uvula in midline; reflexes present; sucking, incomplete or absent reflexes
rooting, gag, extrusion
Ears Well formed, firm; notch of ear should be on Low placement, clefts; tags; malformed; lack of
straight line with outer canthus cartilage
MATERNAL/INFANT

Face Symmetrical movements and contours Facial palsy (7th cranial nerve); looks funny
Neck Short, freely movable; some head control Wry neck, webbed neck; restricted movement;
masses; distended veins; absence of head control
Chest Enlarged breasts, witchs milk; barrel Flattened, funnel-chested, asynchronous movement;
shaped; both sides move synchronously; lack of breast tissue; fracture of clavicle(s); supernu-
nipples symmetrical merary or widely spaced nipples; bowel sounds
Abdomen Dome shaped, abdominal respirations, soft; Scaphoid shaped, omphalocele, diastasis recti, and
may have small umbilical hernia; umbilical distention; umbilical cord containing two vessels;
cord well formed, containing three vessels; redness or drainage around base of cord
dry around base; bowel sounds within 2 hours
of birth; voiding; passage of meconium
Genitalia
Girl Large labia; may have pseudomenstruation, Agenesis and imperforate hymen; ambiguous labia
smegma; vaginal orifice open; increased pig- widely separated, fecal discharge per vagina; epispa-
mentation; ecchymosis and edema following dias or hypospadias
breech birth; pink-stained urine (uric acid
crystals)
Boy Pendulous scrotum covered with rugae, and Phimosis, epispadias, or hypospadias; ambiguous;
testes usually descended; voids with ade- scrotum smooth and testes undescended
quate stream; increased pigmentation; edema Hydrocele: collection of fluid in the sac surrounding
and ecchymosis following breech birth the testes
Extremities Synchronized movements, freely movable Fractures, brachial nerve palsy, clubbed foot, pho-
through full range of motion; legs appear comelia or amelia, unusual number or webbing of
bowed, and feet appear flat; attitude of digits, and abnormal palmar creases; poor muscle
general flexion; arms longer than legs; tone; asymmetry; hypertonicity; unusual hip contour
grasp reflex; palmar and sole creases; and click sign (hip dysplasia); hypermobility of joints
normal contour
Back Spine straight, easily movable, and flexible; Fusion of vertebrae: pilonidal dimple with tuft of hair;
may have small pilonidal dimple at base of spina bifida, agenesis of part of vertebral bodies;
spine; may raise head when prone limitation of movement; weak or absent reflexes
Anus Patent, well placed; wink reflex Imperforate, and absence of wink (absence of
sphincter muscle); fistula
2164_Ch04_115-248 29/03/12 12:23 PM Page 203

The Newborn Infant 203

Table 4.14
Physical Assessment of the Term Neonatecontd
Criterion Average Values and Normal Variations Deviations from Normal
Stools Meconium within first 24 hours; transi- Light-colored meconium (dry, hard), or absent with
tionaldays 25; breastfed: loose, distended abdomen (cyctic fibrosis or Hirschsprungs
golden yellow; bottle-fed: formed, light disease); diarrhetic
yellow (see Table 4.17)
Laboratory Values
Hemoglobin (cord) 13.619.6 gm/dL Evaluate for anemia and persistent polycythemia
Serum bilirubin 26 mg/dL Hyperbilirubinemia (term: 15 mg or more; preterm:
10 mg or more)
Blood glucose 40 mg/dL for term; >30 mg/dL for Identify hypoglycemia before overt or asymptomatic
preterm hypoglycemiado Dextrostix on all suspected infants
(LGA or SGA neonates, or neonates of mothers who
are diabetic)
Neurological Examination* Specific to gestational age and state of
wakefulness
1. Behavioral patterns
a. Feeding Variations in interest, hunger; usually Lethargic. Poor suck, poor coordination with swal-
feeds well within 48 hours low, choking, cyanosis

MATERNAL/INFANT
b. Social Crying is lusty, strong, and soon indica- Absent; no focusing on person holding him/her;
tive of hunger, pain, attention seeking. unconsolable
Responds to cuddling, voice by quiet-
ness and increased alertness
c. Sleep-wakefulness Two periods of reactivity: at birth, and Lethargy, drowsiness
68 hours later. Stabilization, with Disorganized pattern
wakeful periods about every 34 hours
d. Elimination Stooling: see Stools See Stools
Urination Diminished number: dehydration
First few days: 34 qd
End of first week: 56 qd
Later: 610 qd, with adequate
hydration
2. Reflex response Bilateral, symmetrical response (see Absent, hyperactive, incomplete, asynchronous
Table 4.15)
3. Sensory capabilities
a. Vision Limited accommodation, with clearest Absence of these responses may be due to absence
vision within 78 inches. Focuses and of or diminished acuity or to sensory deprivation
follows by 15 minutes of age. Prefers
patterns to plain.
b. Hearing By 2 minutes of age, can move in Absence of response: deafness
direction of sound: responds to high
pitch by freezing, followed by agita-
tion; to low pitch (crooning) by relaxing
c. Touch Soothed by massaging, warmth, Unable to be comforted: possible drug dependence
weightlessness (as in water bath)
d. Smell By fifth day, can distinguish between Newborns who are cocaine-addicted avoid eye
mothers breasts and those of another contact
woman
e. Taste Can distinguish between sweet and
sour
f. Motor Coordinates body movement to Absence
parents voice and body movement
*Based on Brazeltons method.
2164_Ch04_115-248 29/03/12 12:23 PM Page 204

204 chapter 4 Health Promotion and Maintenance

Table 4.15
Assessment: Normal Newborn Reflexes*
Reflex Description Implications of Deviations from Normal Patterns
Moro (startle) Symmetrical abduction and extension of arms Asymmetrical reflex may indicate brachial (Erbs)
with fingers extended in response to sudden palsy or fractured clavicle
movement or loud noise
Tonic neck (fencing) When head turned to one side, arm and leg on Asymmetry may indicate cerebral lesion, if
that side extend, and opposite arm and leg flex persistent
Rooting and sucking With stimulus to cheek, turns toward stimulus, Absence of response may indicate prematurity,
opens mouth, sucks neurological problem, or depressed infant (or not
hungry)
Palmar grasp If palm stimulated, fingers curl; holds adult finger Asymmetry may indicate neurological
briefly involvement
Plantar grasp Pressure on sole will elicit curling of toes Absence/asymmetry associated with defects of
lower spinal column
Stepping/dancing If held in upright position with feet in contact Asymmetry may indicate neurological problem
with hard surface, alternately raises feet
Babinski Stroking the sole in a upward fashion elicits Same as for plantar grasp
hyperextension of toes
MATERNAL/INFANT

Crawling When placed in prone position, attempts to Absence may indicate prematurity or depressed
crawl infant
*Reflexes are good indicators of the neurological system in infants who are well, but not in neonates who are sick.
Infants with infections may not show normal reflexes yet have an intact neurological system.

Table 4.16
Estimation of Gestational Age: Common Clinical Parameters
Characteristic Preterm Term
Head Ovalnarrow biparietal; large in propor- Square-shaped biparietal prominences; 1/4 body
tion to body; face looks like old man length
Ears: form, cartilage Soft, flat, shapeless Pinna firm; erect from head
Hair: texture, distribution Fine, fuzzy, or wooly; clumped; appears Silk; single strands apparent
at 20 weeks
Sole creases Starting at ball of foot: 1/3 covered with Entire sole heavily creased
creases by 36 weeks, 2/3 by 38 weeks
Breast nodules 0 mm at 36 weeks; 4 mm at 37 weeks 10 mm or more
Nipples No areolae Formed; raised above skin level
Genitalia
Girl Clitoris large, labia gaping Labia larger, meet in midline
Boy Small scrotum, rugae on inferior surface Scrotum pendulous, covered with rugae; testes
only, and testes undescended usually descended
Skin: texture, opacity Visible abdominal veins; thin, shiny Few indistinct larger veins; thick, dry, cracked,
peeling
Vernix Covers body by 3133 weeks Small amount in creases or absent at term; post-
term: dry, wrinkled
Lanugo Apparent at 20 weeks; by 3336 weeks, Minimal or no lanugo (depends on parental ethnicity)
covers shoulders
Muscle tone Hypotonia; extension of arms and legs Hypertonia; well flexed
Posture Froglike Attitude of general flexion
2164_Ch04_115-248 29/03/12 12:23 PM Page 205

The Newborn Infant 205

Table 4.16
Estimation of Gestational Age: Common Clinical Parameterscontd
Characteristic Preterm Term
Head lag Head lags; arms have little or no flexion Head follows trunk; strong arm flexion
Scarf sign Elbow can extend to opposite axilla Elbow to midline only; infant resists
Square window 90 degrees 0 degrees
Ankle dorsiflexion 90 degrees 0 degrees
Popliteal angle 180 degrees <90 degrees
Heel-to-ear maneuver Touches ear easily 90 degrees
Ventral suspension Hypotonia; rag-doll Good caudal and cephalic tone
Reflexes
Moro Apparent at 28 weeks; good, but no Complete reflex with adduction; disappears
adduction 4 months postterm
Grasp Fair at 28 weeks; arm is involved at Strong enough to sustain weight for a few seconds
32 weeks when pulled up; hand, arm, shoulder involved
Cry 24 weeks: weak; 28 weeks: high- Lusty; can persist for some time
pitched; 32 weeks: good
Length Under 47 cm (181/2 inches), usually 50 cm (20 inches); range: 1822 inches

MATERNAL/INFANT
Weight Under 2500 gm (5 lb 5 oz) 3400 gm (71/2 lb); range: 5 lb 8 oz8 lb 13 oz

C. Nursing care plan/implementation: b. Check blood sugar (Dextrostix or Chem-strip)


1. Goal: promote effective gas transport. at 30 minutes, 1, 2, and 4 hours for infants at
a. Maintain patent airwayto promote effective risk for hypoglycemia (e.g., SGA, LGA, infant
gas exchange and respiratory function. of diabetic mother).
b. Position: right side-lying, head dependent c. First feeding at 1 to 4 hours of age with sterile
(gravity drainage of fluid, mucus). water (or formula) if permissible and if not
c. Suction prn with bulb syringe for mucus. breastfeeding.
2. Goal: establish/maintain thermal stability. d. Note voiding or meconium stool; report if
a. Avoid chillingto prevent metabolic acidosis. failure to void or defecate within 24 hours.
b. Dry, wrap, and apply hat. 7. Goal: promote bonding.
c. Place in heated crib. a. Encourage parent-infant interaction
d. Monitor vital signs hourly until stable. (holding, touching, eye contact, talking
3. Goal: reduce possibility of blood loss. to infant).
a. Check cord clamp for security. b. Encourage breastfeeding within 1 hour of
b. Administer vitamin K injection, as ordered, in birth, if applicable.
anterior or lateral thigh muscleto stimulate c. Encourage parent participation in infant
blood coagulability. careto develop confidence and competence
4. Goal: prevent infection. in caring for newborn.
a. Administer antibiotic treatment to eyes (if (1) Assist with initial efforts at feeding.
not performed in birth room)to prevent (2) Discuss and demonstrate positioning and
ophthalmia neonatorum. burping techniques.
b. Treat cord stump (alcohol), as ordered. (3) Demonstrate/assist with basic care proce-
c. Use standard precautions. dures, as necessary:
5. Goal: promote comfort and cleanliness. Admission (a) Bath.
bath when temperature stable. (b) Cord care.
6. Goal: promote nutrition, hydration, elimination. (c) Diapering.
a. Encourage breastfeeding within 1 hour after (d) Aid parents in distinguishing normal vs.
birth. abnormal newborn characteristics.
2164_Ch04_115-248 29/03/12 12:23 PM Page 206

206 chapter 4 Health Promotion and Maintenance

8. Goal: health teachingto provide anticipatory (c) Maintaining contact/control over


guidance for discharge. infant to prevent falls, drowning in
a. Facilitate sibling bonding. bath.
b. Describe/discuss normal newborn (d) Instruct parents in infant
behavior: cardiopulmonary resuscitation
(1) Sleepingalmost continual (wakes (CPR).
only to feed) or 12 to 16 hours daily. (e) Hepatitis B vaccinationfirst dose
(2) Feedingfrom every 2 to 3 hours to can be given at 24 to 48 hours of
longer intervals; establish own pattern; life; second dose at 1 month; third
babies who are breastfed feed more dose at 6 months. If infant born to
often. mother who is infected, hepatitis B
(3) Weight loss5% to 10% in first few vaccine and hepatitis B immune
days; regained in 7 to 14 days. globulin should be administered
(4) Stools(Table 4.17). within 12 hours of birth.
(5) Cord carecord drops off in 7 to 10 days (11) Describe signs of common health
(a) Keep clean and dry. problems to be reported promptly:
(b) Alcohol may be applied to stump, or (a) Diarrhea, constipation.
it may be allowed to dry naturally. (b) Colic, vomiting.
(c) HIV precautions. (c) Rash, jaundice.
(6) Circumcision care (d) Differentiation from normal
(a) Keep clean and dry; heals rapidly. patterns.
MATERNAL/INFANT

(b) Watch for bleeding. D. Evaluation/outcome criteria:


(c) Petroleum jelly, gauze prn, if 1. Infant demonstrates successful transition to
ordered. independent life:
(d) Do not remove yellowish exudate. a. Breastfeeds well.
(7) Physiological jaundiceoccurs 24 to b. Normal feeding, sleeping, elimination
72 hours after birth. patterns.
(a) Nonpathologic. c. No evidence of infection or abnormality.
(b) Need for hydration. 2. Mother/family evidences bonding.
(8) Identify need for newborn screening test a. Eye contact.
after ingestion of milk (done routinely b. Stroking, cuddling.
at 24 hours of age and later). Includes c. Crooning, calling baby by name, talking to
screen for congenital hypothyroidism infant.
and galactosemia (see Table 11.2, 3. Mother demonstrates comfort and skill in basic
Chapter 11, for other newborn screen- newborn care.
ing procedures). 4. Mother verbalizes understanding of subjects
(9) Describe suggested sensory stimulation discussed:
modalities (mobiles, color, music). a. Safety precautions.
(10) Discuss safety precautions: b. Health maintenance actions.
(a) Position on back for sleep. c. Signs of normal infant behavior and
(b) Infant seat for travel and home safety. health.

Table 4.17
Infant Stool Characteristics
Age Bottle-fed Breastfed Implications of Abnormal Patterns
1 day Meconium Meconium Absence may indicate obstruction, atresia
25 days (transitional) Greenish yellow, loose Greenish yellow, loose, NoteAt any time:
frequent
>5 days Yellow to brown, firm, Bright golden yellow, Diarrheagreenish, mucus or blood tinged,
24 daily, foul odor loose, 610 daily or forceful expulsion, may indicate infection
Constipationdry, hard stools or infrequent
or absent stools may indicate obstruction
2164_Ch04_115-248 29/03/12 12:23 PM Page 207

Complications During the Neonatal Period: The High-Risk Newborn 207

Complications During with swab, after feedings. Note: Before


medicating, feed sterile water to rinse
the Neonatal Period: out milk.
7. Evaluation/outcome criteria:
The High-Risk Newborn a. Oral mucosa intact, lesions healed, no evidence
I. GENERAL OVERVIEWSuccessful newborn adapta- of infection.
tion to the demands of independent extrauterine life b. Feeds well; maintains weight or regains weight
may be complicated by environmental insults during lost, if any.
the prenatal period or those arising in the period imme- C. Neonatal sepsis
diately surrounding birth. The nursing role focuses on 1. Pathophysiologygeneralized infection; may
minimizing the effect of present and emerging health overwhelm infants immature immune system.
problems and on facilitating and supporting a successful 2. Etiology:
transition to extrauterine life. a. Prolonged rupture of membranes.
b. Long, difficult labor.
II. GENERAL ASPECTScommon neonatal risk c. Resuscitation procedures.
factors: d. Maternal infection (e.g., -hemolytic strepto-
A. Gestational age profile (see Tables 4.15 and 4.16): coccus vaginosis).
1. Prematurity. e. Aspirationamniotic fluid, formula, mucus.
2. Dysmaturity. f. Iatrogenic (nosocomial)caused by infected
3. Postmaturity. health personnel or equipment.
B. Congenital disorders. 3. Assessment:

MATERNAL/INFANT
C. Birth trauma. a. Respirationsirregular, periods of apnea.
D. Infections. b. Irritability or lethargy.
III. DISORDERS AFFECTING PROTECTIVE 4. Analysis/nursing diagnosis:
FUNCTIONS: NEONATAL INFECTIONS a. Fatigue related to increased oxygen needs.
A. Assess for intrauterine infections. b. High risk for infection related to septic
B. Oral thrush (mycotic stomatitis). condition.
1. Pathophysiologylocal inflammation of oral 5. Nursing care plan/implementation:
mucosa due to fungal infection. a. Cultures (spinal, urine, blood).
2. Etiology: b. Check vitals.
a. OrganismCandida albicans. c. Monitor respirations.
b. More common in newborn who is vulnerable d. Give medications, as ordered.
(i.e., sick, debilitated; those receiving antibiotic 6. Evaluation/outcome criteria:
therapy). a. Responds to medical/nursing regimen
3. Mode of transmissiondirect contact with: (all assessment findings within normal
a. Maternal birth canal, hands, and linens. limits).
b. Contaminated feeding equipment, staff s b. Parent(s) verbalize understanding of diagno-
hands. sis, treatment; demonstrate appropriate
4. Assessment: techniques in participating in care
a. White patches on oral mucosa, gums, and (as possible).
tongue that bleed when touched. c. Parent(s) demonstrate effective coping with
b. Occasional difficulty swallowing. situation; express satisfaction with care.
5. Analysis/nursing diagnosis: IV. DISORDERS AFFECTING NUTRITION:
a. Pain related to irritation of oral mucous INFANT OF THE DIABETIC MOTHER (IDM)
membrane secondary to oral moniliasis. A. Pathophysiologyhyperplasia of pancreatic beta
b. Altered nutrition, less than body requirements cells increased insulin production excessive
related to irritability and poor feeding. deposition of glycogen in muscles, subcutaneous
6. Nursing care plan/implementation: Goal: fat, and tissue growth. Results in fetal:
prevent cross-contamination. 1. MacrosomiaLGA infant.
a. Aseptic technique; good hand washing. 2. Enlarged internal organscommon.
b. Give medications as ordered: a. Cardiomegaly.
(1) Aqueous gentian violet, 1% to 2%: apply b. Hepatomegaly.
to infected area with swab. c. Splenomegaly.
(2) Nystatin (Mycostatin)instill into mouth 3. Neonatalinadequate carbohydrate reserve to
with medicine dropper, or apply to lesions meet energy needs.
2164_Ch04_115-248 29/03/12 12:23 PM Page 208

208 chapter 4 Health Promotion and Maintenance

4. Associated with increased incidence of: 4. Altered nutrition, less than body requirements,
a. Congenital anomalies (five times average related to hypoglycemia, hypocalcemia.
incidence with pregestational diabetes) 5. Risk for altered endocrine/metabolic processes related
includes cardiac, pelvic, and spinal to hyperbilirubinemia and kernicterus.
anomalies. E. Nursing care plan/implementation:
b. Preterm birth: respiratory distress syndrome 1. Hypoglycemiaadminister formula or IV glucose,
(RDS); increased insulin needs prenatally as ordered (may cause rebound effect).
lead to decreased surfactant production. 2. Preterm/immatureinstitute preterm care prn.
c. Fetal dystociadue to CPD. 3. Hypocalcemiaadminister oral or IV calcium
d. Neonatal metabolic problems: gluconate, as ordered.
(1) Hypoglycemia. 4. Inform pediatrician immediately of signs of:
(2) Hypocalcemic tetany. a. Jaundice.
(3) Metabolic acidosis. b. Hyperirritability.
(4) Hyperbilirubinemia. c. Birth injury.
B. Etiologyhigh circulating maternal glucose d. Increased intracranial pressure/hemorrhage
levels during fetal growth and development; loss F. Evaluation/outcome criteria:
of maternal glucose supply following birth; 1. Infant makes successful transition to
decreased hepatic gluconeogenesis. extrauterine life.
C. Assessment: 2. Infant responds to medical/nursing regimen.
1. Characteristics of IDM. Experiences minimal or no metabolic
2. HypoglycemiaDextrostix or Chem-strip to disturbances (hypoglycemia, hypocalcemia,
MATERNAL/INFANT

heel stick at: hyperbilirubinemia).


a. 30 minutes 2. 3. Infant exhibits normal respiratory function
b. 1, 2, and 4 hours of age; before meals 4 or and gas exchange.
until stable. V. HYPOGLYCEMIA
c. Chem-strip: if less than 20 mg/dL, must
A. Pathophysiologylow serum-glucose level
draw glucose STAT.
altered cellular metabolism cerebral irritability,
d. Hypoglycemia laboratory values for preterm
cardiopulmonary problems.
and term infants: under 45 mg/dL.
B. Etiology:
e. Behavioral signstremors; twitching,
1. Loss of maternal glucose supply.
hypotonia, seizures.
2. Normal physiological activities of respiration,
3. Gestational age, since macrosomia may mask
thermoregulation, muscular activity exceed
prematurity.
carbohydrate reserve.
4. Hypocalcemiausually within first 24 hours
3. Decreased hepatic ability to convert amino acids
a. Irritability.
into glucose.
b. Coarse tremors, twitching, convulsions.
4. More common in:
5. Birth injuries:
a. Infants of diabetic mothers.
a. Fractures: clavicle, humerus, skull.
b. Preterm, postterm infants.
b. Brachial palsy.
c. SGA infants.
c. Intracranial hemorrhage/signs of increased
d. Smaller twin.
intracranial pressure.
e. Infant of mother with preeclampsia.
d. Cephalohematoma.
f. Birth asphyxia.
6. Respiratory distress:
C. Assessment:
a. Nasal flaring.
1. Jitteriness, tremors, convulsions; lethargy and
b. Expiratory grunt.
hypotonia.
c. Sternal retraction.
2. Sweating; unstable temperature.
d. Intercostal retractions.
3. Tachypnea; apneic episodes; cyanosis.
e. Cyanosiscentral.
4. High-pitched, shrill cry.
7. Jaundice.
5. Difficulty feeding.
D. Analysis/nursing diagnosis:
D. Analysis/nursing diagnosis:
1. High risk for injury related to CPD, dystocia.
1. Altered tissue perfusion (fetal) related to placen-
2. Altered cardiopulmonary tissue perfusion related
tal insufficiency associated with maternal
to placental insufficiency, RDS.
diabetes, preeclampsia, renal or cardiac
3. Impaired gas exchange related to RDS.
disorders; erythroblastosis.
2164_Ch04_115-248 29/03/12 12:23 PM Page 209

Complications During the Neonatal Period: The High-Risk Newborn 209


2. Risk for altered endocrine metabolic processes 4. Sweating.
related to high incidence of morbidity associated 5. Hungersucks on fists; feeding problems
with birth asphyxia. regurgitation, vomiting, poor feeding, diarrhea,
3. Impaired gas exchange related to coexisting RDS. and increased mucus production.
4. Altered nutrition, less than body requirements, 6. Convulsions with abnormal eye-rolling and
related to hypoglycemia. chewing motions.
5. High risk for injury related to coexisting infec- 7. Developmental lags/mental retardation.
tion, metabolic acidosis. E. Analysis/nursing diagnosis:
E. Nursing care plan/implementation 1. High risk for injury related to convulsions
(see IV. INFANT OF THE DIABETIC secondary to physiological response to
MOTHER, p. 208). withdrawal, CNS hyperirritability.
F. Evaluation/outcome criteria (see IV. INFANT 2. Impaired gas exchange related to respiratory
OF THE DIABETIC MOTHER, p. 208). distress secondary to inhibition of reflex clearing
VI. DISORDERS AFFECTING PSYCHOSOCIAL- of fluid by the lungs.
CULTURAL FUNCTIONS: NEONATE WHO 3. Altered nutrition, less than body requirements,
IS DRUG-DEPENDENT (HEROIN) related to feeding problems secondary to
respiratory distress and GI hypermotility.
A. General aspects
4. High risk for impaired skin integrity related to
1. Maternal drug addiction has been associated
scratching secondary to withdrawal symptoms.
with:
F. Nursing care plan/implementation:
a. Prenatal malnutrition and vitamin
1. Goal: prevent/minimize respiratory distress.

MATERNAL/INFANT
deficiencies.
a. Position: side-lying, head dependentto
b. Increased risk of antepartal infections.
facilitate mucus drainage.
c. Higher incidence of antepartal and
b. Suction prn with bulb syringe for excess
intrapartum complications.
mucusto maintain patent airway.
2. Infant at risk for:
c. Monitor respirations and apical pulse.
a. Intrauterine growth retardation (IUGR).
2. Goal: minimize possibility of convulsions.
b. Prematurity.
a. Decrease environmental stimuliquiet,
c. Fetal distress.
touch only when necessary, offer pacifier.
d. Perinatal death.
b. Keep warm, swaddle for comfort.
e. Child abuse.
3. Goal: maintain nutrition/hydration.
f. Sudden infant death syndrome (SIDS)
a. Food/fluidsoral or IV, as ordered.
(510 times higher than normal).
b. I&O.
g. Learning and behavior disorders.
c. Daily weight.
h. Poor social adjustment.
4. Goal: assist in diagnosis of drug and drug level.
B. Pathophysiologywithdrawal of accustomed
Collect all urine and meconium during first
drug levels physiological deprivation response.
24 hours for toxicological studies.
C. Etiologyrepeated intrauterine absorption of
5. Goal: maintain/promote skin integrity.
heroin/cocaine/methadone from maternal blood-
a. Mitts over handsto minimize scratching.
stream fetal drug dependency.
b. Keep clean and dry.
D. Assessmentdegree of withdrawal depends on
c. Medicated ointment/powder, as ordered,
type and duration of addiction and maternal drug
q24h, to excoriated areas.
levels at birth.
d. Expose excoriated areas to air.
1. Irritability, hyperactivity, hypertonicity, exagger-
6. Goal: minimize withdrawal symptoms. Administer
ated reflexes, tremors, high-pitched cry, difficult
medications, as ordered.
to comfort:
a. Paregoric elixirto wean from drug.
a. Step reflex (dancing)infant places both
b. Phenobarbitalto reduce CNS hyperirritability,
feet on surface; assumes rigid stancedoes
hyperbilirubinemia.
not step or dance.
c. Chlorpromazine (Thorazine), diazepam
b. Head-righting reflexholds head rigid; fails
(Valium)to tranquilize, reduce hyperirri-
to demonstrate head lag.
tability. Note: Valium is contraindicated
2. Nasal stuffiness and sneezing; respiratory
for the neonate who is jaundiced because it
distress, tachypnea, cyanosis, or apnea.
predisposes to hyperbilirubinemia.
3. Exaggerated acrocyanosis or mottling in the
d. Methadone.
infant who is warm.
2164_Ch04_115-248 29/03/12 12:23 PM Page 210

210 chapter 4 Health Promotion and Maintenance

7. Goal: emotional support to mother. VIII. CLASSIFICATION OF INFANTS BY WEIGHT


a. Encourage verbalization of feelings of guilt, AND GESTATIONAL AGE
anxiety, fear, concerns. A. Terminology
b. Refer to social service. 1. Preterm, or premature37 weeks gestation or
G. Evaluation/outcome criteria: less (usually 2500 gm [5 lb] or less).
1. Infant responds to medical/nursing regimen. 2. Term38 to 42 weeks gestation.
a. Maintains adequate respirations. 3. Posttermover 42 weeks.
b. Feeds well, gains weight. 4. Postmaturegestation greater than 42 weeks.
c. No evidence of CNS hyperirritability, convul- 5. Appropriate for gestational age (AGA)for each
sions; demonstrates normal newborn reflexes. week of gestation, there is a normal range of
2. Infant evidences bonding with parent(s). expected weight (between 10th and 90th
Responsive to mothers voice. percentile).
VII. DISORDERS AFFECTING PSYCHOSOCIAL- a. Term infants weighing 2500 gm or more are
CULTURAL FUNCTION: FETAL ALCOHOL usually mature in physiological functions.
SYNDROME (FAS) b. If respiratory distress occurs, it is usually
A. General aspects: related to meconium aspiration syndrome.
1. Maternal alcohol abuse has been associated with: 6. SGA or dysmatureweight falls below normal
a. Malnutrition, vitamin deficiencies. range for age (<10th percentile).
b. Bone marrow suppression. a. Preeclampsia.
c. Liver disease. b. Malnutrition.
c. Smoking.
MATERNAL/INFANT

d. Child abuse.
2. Infant at risk for: d. Placental insufficiency.
a. Congenital anomalies (FAS). e. Alcohol syndrome.
b. Mental deficiency; learning disabilities. f. Rubella.
c. IUGR. g. Syphilis.
B. Pathophysiologypermanent damage to develop- h. Multifetal gestation (twins, etc.).
ing embryonic/fetal structures; cardiovascular i. Genetic.
anomalies (ventricular septal defects). j. Cocaine abuse.
C. Etiologyhigh circulating alcohol levels are lethal 7. LGAabove expected weight for age (>90th
to the embryo; lower levels cause permanent cell percentile). Note: If preterm, at risk for RDS. If
damage. postterm, at risk for aspiration and sudden
D. Assessment: intrauterine death.
1. Characteristic craniofacial abnormalities: a. Etiology:
a. Short, palpebral fissure. (1) Maternal diabetes or prediabetes.
b. Epicanthal folds. (2) Maternal weight gain over 35 lb.
c. Maxillary hypoplasia. (3) Maternal obesity.
d. Micrognathia. (4) Genetic.
e. Long, thin upper lip. b. Associated problems:
2. Short stature. (1) Hypoglycemia.
3. Irritable, hyperactive, poor feeding. (2) Hypocalcemia.
4. High-pitched cry, difficult to comfort. (3) Hyperbilirubinemia.
E. Nursing care plan/implementation: (4) Birth injury (e.g., fractures, Erb-Duchenne
1. Goal: reduce irritability. paralysis).
a. Reduce environmental stimuli. B. Estimation of gestational ageplanning appropriate
b. Wrap, cuddle. care for the newborn requires accurate assessment
c. Administer sedatives, as ordered. to differentiate between preterm and term infants.
2. Goal: maintain nutrition/hydration. PRETERM INFANTBorn at 37 weeks of
3. Goal: emotional support to mother. gestation or less.
F. Evaluation/outcome criteria (see VI. A. Pathophysiologyanatomical and physiological
NEONATE WHO IS DRUG-DEPENDENT immaturity of body systems compromises ability to
[HEROIN], p. 209): adapt to extrauterine environment and independ-
1. No respiratory distress. ent life.
2. Infant feeding properly. 1. Interference with protective functions
3. Maternal bonding apparent. a. Temperature regulationunstable, due to:
4. Social serviceshome involvement. (1) Lack of subcutaneous fat.
2164_Ch04_115-248 29/03/12 12:23 PM Page 211

Complications During the Neonatal Period: The High-Risk Newborn 211


(2) Large body surface area in proportion to c. Heavy smoking.
body weight. d. High-altitude environment.
(3) Small muscle mass. e. Cocaine use.
(4) Absent sweat or shiver responses. C. Factors influencing survival:
(5) Poor capillary response to changes in 1. Gestational age.
environmental temperature. 2. Lung maturity.
b. Resistance to infectionlow, due to: 3. Anomalies.
(1) Lack of immune bodies from mother 4. Size.
(these cross placenta late in pregnancy). D. Causes of mortality (in order of frequency):
(2) Inability to produce own immune bodies 1. Abnormal pulmonary ventilation.
(immature liver). 2. Infection.
(3) Poor WBC response to infection. a. Pneumonia.
c. Immature liver b. Septicemia.
(1) Inability to conjugate bilirubin liberated by c. Diarrhea.
normal breakdown of RBCs increased d. Meningitis.
susceptibility to hyperbilirubinemia and 3. Intracranial hemorrhage.
kernicterus. 4. Congenital defects.
(2) Immature production of clotting factors E. Disorders affecting fluid-gas transport: RDS
and immune globulins. 1. Pathophysiologyinsufficient pulmonary
(3) Inadequate glucose stores increased surfactant (lecithin) and insufficient number/
susceptibility to hypoglycemia. maturity of alveoli predispose to atelectasis;

MATERNAL/INFANT
2. Interference with elimination: immature renal alveolar ducts and terminal bronchi become
functionunable to concentrate urine lined with fibrous, glossy membrane.
precarious fluid-electrolyte balance. 2. Etiology:
3. Interference with sensory-perceptual func- a. Primarily associated with prematurity.
tions: CNSimmature weak or absent b. Other predisposing factors:
reflexes and fluctuating primitive control of (1) Fetal hypoxiadue to decreased placental
vital functions. perfusion secondary to maternal bleeding
B. Etiology: often unknown; preterm labor. (e.g., abruptio placentae) or hypotension.
1. IatrogenicEDD miscalculated for repeat (2) Birth asphyxia.
cesarean birth (rare). (3) Postnatal hypothermia, metabolic acidosis,
2. Placental factors or hypotension.
a. Placenta previa. 3. Factors protecting neonate from RDS:
b. Abruptio placentae. a. Chronic fetal stressdue to maternal hyper-
c. Placental insufficiency. tension, preeclampsia, or heroin addiction.
3. Uterine factors b. PROM.
a. Incompetent cervix. c. Maternal steroid ingestion
b. Overdistention (multifetal gestation, (e.g., betamethasone).
polyhydramnios). d. Low-grade chorioamnionitis.
c. Anomalies (e.g., myomas). 4. Assessment:
4. Fetal factors a. Usually appears during first or second day
a. Malformations. after birth.
b. Infections (rubella, toxoplasmosis, HIV-positive b. Signs of respiratory distress:
status, AIDS, cytomegalic inclusion disease). (1) Nasal flaring.
c. Multifetal gestations (twins, triplets). (2) Expiratory grunt.
5. Maternal factors (3) Sternal retractions.
a. Severe physical or emotional trauma. (4) Tachypnea (60 breaths/min or more).
b. Coexisting disorders (preeclampsia, hyperten- (5) Cyanosiscentral.
sion, heart disease, diabetes, malnutrition). (6) Increasing number and length of apneic
c. Infections (streptococcus, syphilis, bacterial episodes.
vaginosis, pyelonephritis, pneumonia, (7) Increasing exhaustion.
influenza, leukemia, UTI). c. Respiratory acidosisdue to hypercapnea and
6. Miscellaneous factors rising CO2 level.
a. Close frequency of pregnancies. d. Metabolic acidosisdue to increased lactic
b. Advanced maternal age. acid levels and falling pH.
2164_Ch04_115-248 29/03/12 12:23 PM Page 212

212 chapter 4 Health Promotion and Maintenance

5. Analysis/nursing diagnosis: 7. Evaluation/outcome criteria:


a. Impaired gas exchange related to lack of pul- a. Respiratory distress treated successfully; infant
monary surfactant secondary to preterm birth, breathes without assistance.
intrapartum stress and hypoxia, infection, b. Infant completes successful transition to
postnatal hypothermia, metabolic acidosis, extrauterine life.
or hypotension. F. Iatrogenic (oxygen toxicity) disorders: retinopa-
b. Altered nutrition, less than body requirements, thy of prematurity
related to poor feeding secondary to respirato- 1. Pathophysiologyintraretinal hemorrhage
ry distress, caloric demand. fibrosis retinal detachment loss of vision.
6. Nursing care plan/implementation: 2. Etiologyprolonged exposure to high concen-
a. Goal: reduce metabolic acidosis, increase trations of oxygen.
oxygenation, support respiratory efforts. 3. Assessmentonly perceptible retinal change is
(1) Ensure warmth (isolette at 97.6F). vasoconstriction. Note: Arterial blood gas (PaO2)
(2) Warmed, humidified O2 at lowest concen- readings less than 50 or more than 80 mm Hg.
tration required to relieve cyanosis, through 4. Nursing care plan/implementation: Goal: pre-
hood, nasal prongs, or endotracheal tube. vent disorder. Maintain PaO2 of 50 to 70 mm Hg.
(3) Monitor continuous positive airway pressure 5. Evaluation/outcome criteria:
(CPAP)oxygenair mixture administered a. Successful recovery from respiratory distress.
under pressure during inhalation and exha- b. No evidence of retinopathy.
lation to maintain alveolar patency. G. Iatrogenic (oxygen toxicity) disorders:
(4) Position: side-lying or supine with neck bronchopulmonary dysplasia (BPD)
MATERNAL/INFANT

slightly extended (sniffing position); 1. Pathophysiologydamage to alveolar cells


arms at sides. result in focal emphysema.
(5) Suction prn with bulb syringefor 2. Etiologypositive-pressure ventilation (CPAP
excessive mucus. and positive end-expiratory pressure [PEEP])
b. Goal: modify care for infant with endotracheal and prolonged administration of high concentra-
tube. tions of oxygen.
(1) Disconnect tubing at adapter. 3. Assessmentmonitor for signs of:
(2) Inject 0.5 mL sterile normal saline (may a. Tachypnea.
be omitted). b. Increased respiratory effort.
(3) Insert sterile suction tube, start suction, c. Respiratory distress.
rotate tube, withdraw. 4. Nursing care plan/implementation: Goal:
(4) Suction up to 5 seconds. prevent disorder.
(5) Ventilate with bag and mask during proce- a. Use of positive-pressure devices.
dure. b. Maintain oxygen concentration below 80%.
(6) Reconnect tubing securely to adapter. c. Supportive care.
(7) Auscultate for breath sounds and pulse. d. Wean off ventilator, as possible.
c. Goal: maintain nutrition/hydration. 5. Evaluation/outcome criteria:
(1) Administer fluids, electrolytes, calories, a. Successful recovery from respiratory distress.
vitamins, minerals PO or IV, as ordered. b. No evidence of disorder.
(2) I&O. H. Intraventricular hemorrhage
d. Goal: prevent secondary infections. 1. Pathophysiologyrupture of thin, fragile capil-
(1) Strict aseptic technique. lary walls within ventricles of the brain (more
(2) Hand washing. common in preterm).
e. Goal: emotional support of infant. 2. Etiology:
(1) Gentle touching. a. Hypoxia.
(2) Soft voices. b. Respiratory distress.
(3) Eye contact. c. Birth trauma.
(4) Rocking. d. Birth asphyxia.
f. Goal: emotional support of parents. e. Hypercapnia.
(1) Keep informed of status and progess. 3. Assessment:
(2) Encourage contact with infantto promote a. Hypotonia.
bonding, understanding of treatment. b. Lethargy.
g. Goal: minimize possibility of iatrogenic disor- c. Hypothermia.
ders associated with oxygen therapy (see F. and d. Bradycardia.
G., below). e. Bulging fontanels.
2164_Ch04_115-248 29/03/12 12:23 PM Page 213

Complications During the Neonatal Period: The High-Risk Newborn 213


f. Respiratory distress or apnea. J. Disorders affecting nutrition/elimination:
g. Seizures. necrotizing enterocolitis (NEC)
h. Cry: high-pitched whining. 1. Pathophysiologyintestinal thrombosis,
4. Nursing care plan/implementation: Goal: infarction, autodigestion of mucosal lining, and
supportive care to promote healing. necrotic lesions; incidence increased in preterm.
a. Monitor vital signs. 2. Etiologyintestinal ischemia, due to blood
b. Maintain thermal stability. shunt to brain and heart in response to:
c. Ensure adequate oxygenation (may be placed a. Fetal distress.
on CPAP). b. Fetal/neonatal asphyxia.
5. Evaluation/outcome criteria: c. Neonatal shock.
a. Condition stable, all assessment findings d. After birth, may result from:
within normal limits. (1) Low cardiac output.
b. No evidence of residual damage. (2) Infusion of hyperosmolar solutions.
I. Disorders affecting nutrition e. Complicated by action of enteric bacteria on
1. Pathophysiologyunderdeveloped feeding damaged intestine.
abilities, small stomach capacity, immature 3. Assessmentearly identification is vital.
enzyme system, fat intolerance. a. Abdominal distention or erythema, or both.
2. Etiologyimmature body systems associated b. Poor feeding, vomiting.
with preterm birth. c. Blood in stool.
3. Assessment: d. Systemic signs associated with sepsis that may
a. Weak suck, swallow, gag reflexestendency need temporary colostomy or iliostomy:

MATERNAL/INFANT
to aspiration. (1) Lethargy or irritability.
b. Signs of malabsorption and fat intolerance (2) Hypothermia.
(abdominal distention, diarrhea, weight loss, (3) Labored respirations or apnea.
or failure to gain weight). (4) Cardiovascular collapse.
c. Signs of vitamin E deficiency (edema, anemia). e. Medical diagnosis:
4. Analysis/nursing diagnosis: (1) Increased gastric residual.
a. Altered nutrition, less than body requirements, (2) X-ray shows ileus, air in bowel wall.
related to poor feeding reflexes, reduced 4. Analysis/nursing diagnosis:
stomach capacity, inability to absorb needed a. Altered nutrition, less than body requirements,
nutrients. related to inability to tolerate oral feedings,
b. Impaired gas exchange related to aspiration. and gastrointestinal dysfunction secondary to
5. Nursing care plan/implementation: Goal: ischemia, thrombosis, or necrosis.
maintain/increase nutrition. b. Constipation related to paralytic ileus with
a. Frequent, small feedingsto avoid exceeding stasis; diarrhea related to water loss.
stomach capacity, facilitate digestion. c. High risk for injury related to infection,
b. Frequent burping during feedingto avoid thrombosis, metabolic alterations (acidosis,
regurgitation/aspiration. osmotic diuresis, dehydration, hyperglycemia)
c. Supplement vitamin E (alpha-tocopherol) due to parenteral nutrition.
intake, as ordered, in infants who are formula- d. Altered parenting related to physiological
fed. (Note: intake adequate in infants who are compromise and prolonged hospitalization.
breastfed.) Vitamin E actions: e. Impaired skin integrity when colostomy is
(1) Antioxidant. necessary.
(2) Maintains structure and function of 5. Nursing care plan/implementation:
smooth, skeletal, and cardiac muscle. a. Goal: supportive care.
(3) Maintains structure and function of (1) Rest GI tract: no oral intaketo achieve
vascular tissue, liver, and RBC integrity. gastric decompression.
(4) Coenzyme in tissue respiration. (2) IV fluids, as orderedto maintain
(5) Treatment for malnutrition with hydration.
macrocytic anemia. b. Goal: prevent infection. Administer antibiotics,
d. Encourage parent/family participation. as ordered.
6. Evaluation/outcome criteria: c. Goal: prevent trauma to skin surrounding stoma.
a. Feeds well without regurgitation/aspiration. 6. Evaluation/outcome criteria:
b. Maintains/gains weight. a. Tolerates oral feedings.
c. No evidence of malabsorption, vitamin b. Demonstrates weight gain.
deficiency. c. Normal stool pattern.
2164_Ch04_115-248 29/03/12 12:23 PM Page 214

214 chapter 4 Health Promotion and Maintenance

d. Parents are accepting and knowledgeable extrauterine life may pose a major challenge to infants
about care of infant. compromised by anatomical or physiological disorders.
POSTTERM INFANTOver 42 weeks of gestation. Knowledge regarding the implications of the neonates
A. General aspects structural or metabolic problems enables the nurse to
1. Labor may be hazardous for mother and fetus identify early signs of health problems and to plan,
because: provide, and evaluate appropriate outcome-directed care
a. Large size of infant contributes to cephalopelvic to safeguard the status of the infant with a congenital
disproportion; obtain estimate of fetal weight disorder.
(EFW) by ultrasound.
II. DISORDERS AFFECTING FLUID-GAS
b. Placental insufficiency fetal hypoxia;
TRANSPORT: congenital heart disease
diagnosis by:
(1) Contraction stress test. A. Pathophysiologyaltered hemodynamics,
(2) Nonstress test due to persistent fetal circulation or structural
(3) Amniotic fluid index (AFI). abnormalities.
c. Meconium passage (common physiological 1. Acyanotic defectsno mixing of blood in the
response) increases chance of meconium systemic circulation.
aspiration. a. Patent ductus arteriosus.
B. Assessment: b. Atrial septal defect.
1. If postmature skin: dry, wrinkleddue to c. Ventriclar septal defect.
metabolism of fat and glycogen reserves to d. Coarctation of the aorta.
meet in utero energy needs. 2. Cyanotic defectsunoxygenated blood enters
MATERNAL/INFANT

2. Long limbs, fingernails, and toenailsdue to systemic circulation.


continued growth in utero. a. Tetralogy of Fallot.
3. Lanugo and vernixabsent. b. Transposition of the great vessels.
4. Expression: wide-eyed, alertprobably due to B. Etiologyunknown. Associated with maternal:
chronic hypoxia (oxygen hunger). 1. Prenatal viral disease (e.g., rubella,
5. Placentasigns of aging. coxsackievirus).
C. Analysis/nursing diagnosis: High risk for injury 2. Malnutrition; alcoholism.
related to high incidence of morbidity and 3. Diabetes (poorly controlled).
mortality due to dystocia or hypoxia. 4. Ingestion of lithium salts.
D. Nursing care plan/implementation: C. Assessment:
1. During labor: 1. Patent ductus arteriosus (see Fig. 5.5, p. 278).
a. Goal: emotional support of mothermay require a. Characteristic machine murmur, mid to
cesarean birth due to CPD or fetal distress. upper left sternal border (cardiomegaly);
b. Goal: continuous electronic monitoring of persists throughout systole and most of
FHR. Report late decelerations immediately diastole; associated with a thrill.
(indicate fetal distress). b. Widened pulse pressure.
2. After birth: c. Bounding pulse, tachycardia, gallop rhythm.
a. Goal: if born vaginally, prompt identification 2. Atrial septal defect (see Fig. 5.3, p. 277).
of birth injuries, respiratory distress. Continual a. Characteristic crescendo-decrescendo systolic
observation. ejection murmur.
b. Goal: early identification/treatment of emerging b. Fixed S2 splitting.
signs of complications. c. Dyspnea, fatigue on normal activity.
(1) HypoglycemiaDextrostix readings and d. Medical diagnosiscardiac catheterization,
behavior. x-ray.
(2) Administer oral or intravenous glucose, as 3. Ventricular septal defect (see Fig. 5.4, p. 278).
ordered. a. Loud, harsh, pansystolic murmur; heard
E. Evaluation/outcome criterion: successful transi- best at left lower sternal border; radiates
tion to extrauterine life (all assessment findings throughout precordium. (Note: may be
within normal limits). absentdue to high pulmonary vascular
resistance equalization of interventricular
pressure.)
Congenital Disorders b. Medical diagnosiscardiac catheterization,
ECG, chest x-ray.
I. GENERAL OVERVIEW: Genetic abnormalities and 4. Coarctation of the aorta (see p. 279).
environmental insults often lead to congenital disorders a. Absent femoral pulse.
of the newborn. Successful transition to independent b. Late systolic murmur.
2164_Ch04_115-248 29/03/12 12:23 PM Page 215

Congenital Disorders 215


c. Decreased blood pressure in lower III. DISORDERS AFFECTING FLUID-GAS
extremities. TRANSPORT: hemolytic disease of the newborn
d. Medical diagnosis: x-ray. A. Rh incompatibility
5. Tetralogy of Fallot (blue baby) (see Fig. 5.6, 1. Pathophysiology (see p. 139).
p. 278). 2. Etiology (see Rh isoimmunization, p. 140).
a. Acute hypoxic/cyanotic episodes. 3. Assessment:
b. Limp, sleepy, exhausted; hypotonic extended a. Prenatalmaternal Rh titers, amniocentesis.
positionpostepisode. b. Intrapartumamniotic fluid color:
c. Medical diagnosiscardiac catheterization. (1) Straw-colored: mild disease.
6. Transposition of the great arteries (see Fig. 5.7, (2) Golden: severe fetal disease.
p. 278). c. Direct Coombs blood test; positive test demon-
a. Cyanotic after crying or feeding. strates Rh antibodies in fetal blood.
b. Progressive tachypneaattempt to compen- 4. Nursing care plan/implementationexchange
sate for decreased PaO2, metabolic acidosis. transfusion:
c. Heart sounds vary; consistent with defect. a. Goal: health teaching.
d. Signs of CHF. (1) Explain purpose and process to parents:
e. Medical diagnosiscardiac catheterization, (a) Removes anti-Rh antibodies and fetal
x-ray, ECG. cells that are coated with antibodies.
D. Analysis/nursing diagnosis: (b) Reduces bilirubin levelsindicated
1. Fluid volume excess related to persistent fetal when 20 mg/dL in term neonate and
circulation, structural abnormalities. 15 mg/dL in preterm.

MATERNAL/INFANT
2. Impaired gas exchange related to abnormal (c) Corrects anemiasupplies RBCs that
circulation, secondary to pathology. will not be destroyed by maternal
3. Altered nutrition, less than body requirements, antibodies.
related to exhaustion, dyspnea. (d) Rh-negative type O blood elicits no reac-
E. Nursing care plan/implementation: tion; maximum exchange is 500 mL;
1. Goal: minimize cardiac workload. duration of exchange: 45 to 60 minutes.
a. Minimize cryingsnuggle; pacifierto meet b. Goal: minimize transfusion hazards.
psychological needs. (1) Warm blood to room temperature, since
b. Keep clean and dry. cold blood may precipitate cardiac arrest.
2. Goal: maintain thermal stabilityto reduce body (2) Use only fresh bloodto reduce possibility
need for oxygen. of hypocalcemia, tetany, convulsions.
3. Goal: prevent infection. (3) Give calcium gluconate, as ordered, after
a. Strict aseptic technique; standard each 100 mL of transfusion.
precautions. c. Goal: prepare for transfusion procedure. Ready
b. Hand washing. necessary equipmentmonitor, resuscitation
4. Goal: parental emotional support. equipment, radiant heater, light.
a. Encourage verbalization of anxiety, fears, d. Goal: assist with exchange transfusion.
concerns. (1) Continuous monitoring of vital signs;
b. Keep informed of status. record baseline, and every 15 minutes
5. Goal: health teachingexplain, discuss: during procedure.
a. Diagnostic procedures. (2) Record: time, amount of blood withdrawn;
b. Treatment procedures. time and amount injected; medications
c. Basic care modalities. given.
6. Goal: promote bonding. Encourage parents to (3) Observe for: dyspnea, listlessness, bleeding
participate in infant care, as possible. from transfusion site, cyanosis, cardiovas-
7. Medical-surgical management: surgical cular irregularity or arrest; coolness of
intervention/repair of congenital cardiac lower extremities.
abnormality. e. Goal: posttransfusion care.
F. Evaluation/outcome criteria: (1) Assessment:
1. Experiences no respiratory distress in immediate (a) Observe for: dyspnea, cyanosis,
postnatal period. cardiac arrest or irregularities,
2. Completes transfer to high-risk center without jaundice, hypoglycemia; frequent
incident, if applicable. vital signs.
3. Surgical intervention successful, where (b) Signs of sepsisfever, tachycardia,
applicable. dyspnea, chills, tremors.
2164_Ch04_115-248 29/03/12 12:23 PM Page 216

216 chapter 4 Health Promotion and Maintenance

(2) Nursing care plan/implementation: bilirubin level and neonatal age and
(a) Maintain thermal stabilityto reduce condition; poor fluid-and-caloric balance
physiological stress, possibility of meta- subjects the infant (especially the preterm
bolic acidosis. infant) to kernicterus at low serum bilirubin
(b) Give oxygento relieve cyanosis. levels.
(c) Keep cord moistto facilitate repeat b. Kernicterushigh bilirubin levels result in
transfusion, if necessary. deposition of yellow pigment in basal
(d) Maintain nutrition/hydrationfeed per ganglia of brain irreversible retardation.
schedule. 2. Etiology:
5. Evaluation/outcome criteria: a. Rh or ABO incompatibility, during first
a. Infants hemolytic process ceases; bilirubin 48 hours.
level drops. b. Resolution of an enclosed hemorrhage
b. Infant makes successful transition to (e.g., cephalohematoma).
extrauterine life. c. Infection.
c. Infant experiences no complications of d. Drug inducedvitamin K injection, maternal
therapeutic regimen. ingestion of sulfisoxazole (Gantrisin).
d. Infant shows evidence of bonding. e. Bile duct blockage.
B. ABO incompatibility f. Albumin-binding capacity is exceeded.
1. Pathophysiologyfetal blood carrying antigens g. Breastfeeding jaundice (e.g., pregnanediol
A/B enters maternal type O bloodstream in milk). Breastfeeding is not dangerous and
antibody formation antibodies cross placenta not a cause of physiological jaundice.
MATERNAL/INFANT

hemolyze fetal RBCs. Note: less severe than h. Dehydration.


Rh reaction. i. Immature liver (interferes with conjugation).
2. Etiology: 3. Assessment:
a. Type O mother carries anti-A and anti-B a. Jaundice noted after blanching skin to
antibodies. suppress hemoglobin color; noted in sclera
b. Even first pregnancy is jeopardized if fetal or mucosa in dark-skinned neonates; make
blood enters maternal system. sure light is adequate; spreads from head
c. Reaction possible if fetus is type A, type B, down, with increasing severity.
or type AB and mother is type O. b. Pallor.
3. Assessment: c. Concentrated, dark urine.
a. Jaundice within first 24 hours. d. Blood level determinationhemoglobin or
b. Rising bilirubin levels. indirect bilirubin (unconjugated, unbound
c. Enlarged liver and spleen. bilirubin deposits in CNS).
4. Nursing care plan/implementation: Goal: e. Kernicterussimilar to intracranial
reduce hazard to newborn. hemorrhage.
a. Prepare for exchange transfusion with (1) Poor feeding or sucking.
O-negative blood. (2) Regurgitation, vomiting.
b. Phototherapy may be ordered if bilirubin is (3) High-pitched cry.
10 mg/dL, and anemia is mild or absent. (4) Temperature instability.
c. Close monitoring of status. (5) Hypertonicity/hypotonicity.
d. Supportive care. (6) Progressive lethargy; diminished Moro
5. Evaluation/outcome criteria: reflex.
a. Infant responds to medical/nursing (7) Respiratory distress.
regimen. (8) Cerebral palsy, mental retardation.
b. Infants assessment findings within (9) Death.
normal limits. 4. Analysis/nursing diagnosis:
C. Hyperbilirubinemia a. Fluid volume (RBC) deficit related to hemolysis
1. Pathophysiologybilirubin, a breakdown secondary to blood incompatibility.
product of hemolyzed RBCs, appears at b. High risk for injury (brain damage) related to
increased levels; exceeds 13 to 15 mg/dL. kernicterus.
Bilirubin is safe when bound with albumin and c. Altered thought processes (mental retardation)
conjugated by user for body excretion; danger is related to brain damage secondary to
when unconjugated and deposits in CNS. kernicterus.
a. WARNING: There is no safe serum biliru- d. Knowledge deficit (parental) related to infant
bin level; kernicterus is a function of the condition.
2164_Ch04_115-248 29/03/12 12:23 PM Page 217

General Aspects: Nursing Care of the High-Risk Infant and Family 217
5. Nursing care plan/implementation: B. Treatment for serious physiological compromise
a. Medical management: may result in:
(1) Prenatalamniocentesis. 1. Isolation.
(2) Postnatalexchange transfusion, 2. Sensory deprivation or noxious stimuli.
phototherapy. 3. Emotional stress.
b. Goal: assist bilirubin conjugation through II. ASSESSMENTsigns of neonatal emotional stress:
phototherapy.
A. Does not look at person performing care.
(1) Cover closed eyelids while under light to
B. Does not cry or protest.
protect eyes. (If Biliblanket is used, no
C. Poor weight gain; failure to thrive.
need to cover eyes.) Remove eye pads
when not under light (feeding, cuddling, III. ANALYSIS/NURSING DIAGNOSIS: sensory-
during parental visits). perceptual alterations related to isolation in isolette,
(2) Expose as much skin as possibleto oxygen hood.
maximize exposure of circulating blood IV. NURSING CARE PLAN/IMPLEMENTATION:
to light. Remove for only brief periods. A. Goal: provide consistent parenting contact. Assign
(3) Change position q1hto maximize same nurses whenever possible.
exposure of circulating blood to light. B. Goal: emotional support.
(4) Note: any loose green stools as bile is 1. Comfort when crying.
cleared through gut; watch for skin 2. Provide positive sensory stimulation. Arrange
breakdown on buttocks. time to:
(5) Monitor temperatureto identify a. Stroke skin.

MATERNAL/INFANT
hyperthermia. (Not necessary if using b. Hold hand.
Biliblanket.) c. Hum, sing, talk.
(6) Push fluids (to 25% more than average) d. Hold in en-face position (nurse looking into
between feedingsto counteract dehydra- infants eyes).
tion. Breast milk has natural laxative e. Hold when feeding, if possible.
effects that help clear bile. C. Goal: encourage parents to participate in careto:
c. Goal: health teaching. Explain, discuss 1. Reduce their psychological stress, anxiety, fear.
phototherapy, bilirubin levels, implications. 2. Promote bonding.
d. Goal: emotional support. 3. Reduce possibility of later child abuse (higher
(1) Encourage verbalization of anxiety, fears, incidence of child abuse against children who
concerns. have been high-risk infants).
(2) Encourage contact with infant.
(3) Reassure, as possible. V. EVALUATION/OUTCOME CRITERIA:
6. Evaluation/outcome criteria: A. Infant demonstrates successful resolution of physio-
a. Infants hemolytic process ceases; bilirubin logical problems.
level drops. B. Parents and infant evidence bonding.
b. Infant makes successful transition to C. Parents express satisfaction with care and result.
extrauterine life.
c. Infant experiences no complications of
therapeutic regimen. General Aspects: Nursing
d. Infant shows evidence of effective bonding.
Care of the High-Risk Infant
and Family
Emotional Support I. GENERAL OVERVIEW: The birth of a physiologi-
of the High-Risk Infant cally compromised neonate is psychologically stressful
for both infant and family and physiologically stressful
I. GENERAL ASPECTS
for the neonate. Effective, goal-directed nursing care is
A. The high-risk infant has the same developmental directed toward:
needs as the healthy term infant:
A. Minimizing physiological and psychological stress.
1. Social and tactile stimulation.
B. Facilitating/supporting successful coping or
2. Comfort and removal of discomfort (hunger,
adaptation.
soiling).
C. Encouraging parental attachment/separation/
3. Continuous contact with a consistent, parenting
grieving, as appropriate.
person.
2164_Ch04_115-248 29/03/12 12:23 PM Page 218

218 chapter 4 Health Promotion and Maintenance

II. ASSESSMENTdirected toward determining C. Infant maintains/increases adequacy of adaptation


neonates present and projected status: to extrauterine life.
A. Determine neonates current physical status. D. If relevant, parents demonstrate progress in grieving
B. Identify specific status and diagnosis-related process.
problems and needs.
C. Describe family psychological status, strengths,
and coping mechanisms/skills.
Questions
D. Determine medical-surgical/nursing approach to Select the one answer that is best for each question, unless
problemsand prognosis. otherwise directed.
III. ANALYSIS/NURSING DIAGNOSIS: 1. A client who is 23 years old, gravida 2, para 1 with a twin
A. Parental anxiety/fear related to physiological gestation, is admitted to labor and delivery at 29 weeks
compromise of neonate. gestation with complaints of lower abdominal pain and
B. Self-esteem disturbance related to feelings of guilt or decreased fetal movement. Which interventions should a
anger. nurse perform? Select all that apply.
C. Ineffective individual coping related to severe psy- 1. Place the client on electronic fetal monitor.
chological stress. 2. Administer betamethasone IM.
D. Knowledge deficit related to diagnosis, treatment, 3. Perform nipple stimulation stress test.
prognosis of infant. 4. Limit oral fluid intake.
E. High risk for altered parenting related to concern 5. Place client on strict bedrest.
about infant. 2. A postpartum nurse is caring for a client suspecting of
IV. NURSING CARE PLAN/IMPLEMENTATION: having endometritis. What are the risk factors for develop-
QUESTIONS

A. Goal: preoperative and postoperative care. ing endometritis? Select all that apply.
1. Maintain/improve physiological stability. 1. Protracted active phase of labor.
a. Temperature stabilizationkeep warm. 2. Prolonged rupture of membranes.
b. Oxygenation: 3. Precipitous delivery.
(1) Position. 4. Prolonged latent phase of labor.
(2)Administer oxygen, as ordered or 5. Internal fetal monitoring.
necessary. 3. A client who is a 38-year-old gravida 1, para 0 with type 2
c. Nutrition/hydration: noninsulin-dependent diabetes, is 6 weeks pregnant. She
(1) Administer/monitor IV fluids. asks a clinic nurse how she should manage her diabetes
(2) Oral fluids, as ordered. now that she is pregnant. Which would be the most
(3) Feed, as status permits. appropriate response by the nurse?
2. Assist with diagnostic testing. 1. You can control your blood sugar with oral
B. Goal: emotional support of parents. hypoglycemic agents.
1. Encourage exploring and ventilating feelings. 2. You can control your blood sugar with insulin
2. Involve parents in decision-making process. injections.
C. Goal: health teaching. 3. You can control your blood sugar with dietary
1. Determine knowledge/understanding of changes.
problem. 4. You can control your blood sugar by exercising more.
2. Explain/simplify/clarify, as needed, physicians
discussions with parents. 4. A 26-year-old client, who is a gravida 1, para 1, delivered
3. Describe/explain/discuss neonates present an 8-pound, 12-ounce female infant over an intact
status and any auxiliary equipment; teach perineum 24 hours ago. Since the clients delivery, an
CPR to family. RN has observed that the client does not seem to show a
4. Refer, as needed, to hospital/community lot of attention to her new infant. The client frequently
resources. lets her family take care of changing and holding the
D. Goal: promote bonding. Encourage parental partici- newborn. How should the RN interpret the clients
pation in care of the neonate. behavior?
1. The client seems to doubt her competency as a
V. EVALUATION/OUTCOME CRITERIA:
mother.
A. Parents verbalize understanding of relevant 2. The client is developing an attachment disorder.
information; make informed decisions regarding 3. The client is showing expected behaviors for the
infant care. taking-in period.
B. Parents demonstrate comfort and increasing partici- 4. The client is probably suffering from postpartum
pation in care of neonate. depression.
2164_Ch04_115-248 29/03/12 12:23 PM Page 219

Questions 219
5. A physician orders fentanyl 75 mcg IV for a client in 10. A nurse is caring for a client at 35 weeks gestation who
early labor. The dose of fentanyl on hand is labeled has been admitted to an obstetrics unit with abruptio
100 mcg/2 mL. A nurse should correctly administer placentae. Which symptoms are likely to be a result of
_______ mL of fentanyl. Fill in the blank. the placental abruption? Select all that apply.
1. Painless vaginal bleeding.
6. A client who is a 23-year-old primigravida is admitted 2. Tetanic uterine contractions.
for induction of labor for macrosomia. Upon assess-
3. Premature rupture of membranes.
ment, an RN notes the client is having contractions
4. Severe abdominal pain.
every 2 minutes, lasting 60 seconds in duration. A
5. Rigid, boardlike abdomen.
physician orders misoprostol 25 mcg to be placed in
the posterior fornix of the vagina. Which action should 11. A client delivered a 7-pound, 6-ounce neonate 14 hours
be taken by the RN? ago over a midline episiotomy. Upon assessment, a nurse
1. Call the physician to clarify the dosage of misoprostol. notes that the client has perineal pain of 7/10 on a
2. Perform a vaginal exam to determine dilation and numeric scale. Which actions should the nurse take?
place the misoprostol as ordered by the physician. Select all that apply.
3. Withhold the dosage and notify the physician of con- 1. Perform perineal hygiene after voiding.
traction pattern. 2. Encourage adequate fluid intake.
4. Withhold the dosage of misoprostol due to the risks 3. Apply an ice pack to her perineum.
associated with macrosomia. 4. Suggest taking a sitz bath several times a day.
5. Provide topical anesthetics as ordered by the
7. A client who is a 38-year-old multipara had an epidural physician.
during her last labor for pain control. She and her hus-
band have chosen to decline any medication during this 12. A nurse is providing discharge teaching to a 16-year-old

QUESTIONS
labor and delivery and ask an RN to help them achieve primigravida client. The client is asking questions about
this goal. To best assist them during the second stage of her neonates care. Which methods should be used by the
labor, which action should be taken by the RN? nurse to assist the client in learning to care for her new-
1. Perform vigorous perineal massage to decrease pain born? Select all that apply.
during delivery. 1. Relate stories of the nurses own personal
2. Encourage patterned breathing to decrease pain during experiences.
delivery. 2. Provide written materials to reinforce teaching.
3. Encourage strong pushing between contractions. 3. Demonstrate skills to the client using her own
4. Inform the couple about progress toward delivery and neonate.
all procedures. 4. Show videotapes to the parent to demonstrate skills.
5. Tell her to ask her family members for advice about
8. A client who is a primigravida is admitted in early labor infant care.
at 38 weeks gestation. Her physician orders auscultation
of fetal heart tones every 30 minutes. The RN knows 13. A client had a vaginal delivery 2 days ago. During the
that the most appropriate time to listen to fetal heart morning assessment, the client complains that excessive
tones is: perspiration kept her awake all night. She is worried that
1. For 30 seconds after a contraction. there is a problem. Which response by a nurse would be
2. During a contraction and for 30 seconds after the the most appropriate?
contraction. 1. You may be experiencing signs of infection.
3. Between the contractions. 2. Fluids that were retained during pregnancy are nor-
4. During the contractions. mally lost in this manner.
3. Maybe you drank too much fluid during the day.
9. A client, who is a gravida 7, para 3, delivers a healthy 6- 4. IV fluids administered during labor sometimes cause
pound, 4-ounce infant. When an RN performs a post-
sweating.
partum assessment, the client states, I hope this is our
last baby because we cant afford any more children. 14. A client is admitted in early labor. The client requests to
What is the best response by the nurse? ambulate to enhance the strength of her contractions.
1. You wont have to worry about getting pregnant again Which event should prompt an RN to return the client
until you stop breastfeeding. to bed and evaluate her condition?
2. You should discuss this with your physician at your 1. Intense pressure at the peak of contractions.
postpartum checkup. 2. Warm flushed skin.
3. Perhaps the social worker can help you find a way to 3. Nausea during contractions.
budget your finances. 4. Contractions that are mild and last 60 seconds in
4. We can discuss different birth-control methods you duration.
can use.
2164_Ch04_115-248 29/03/12 12:23 PM Page 220

220 chapter 4 Health Promotion and Maintenance

15. A client, 39 weeks gestation, gravida 2, para 1, is in 3. Pattern 3 because the tracing shows variable decelera-
active labor and reports frequent, painful uterine contrac- tions.
tions. Which fetal monitoring pattern would require an 4. Pattern 3 because the tracing shows late decelerations.
immediate nursing intervention and why? 16. A neonate is flaccid at delivery and is not breathing. A
Pattern 1
200 200 200
pediatrician orders positive-pressure ventilation using a
180 180 180 mask attached to wall oxygen. While administering the
160 160 160 oxygen, in which position should a nurse place the
140 140 140 neonate?
120 120 120
100 100 100
1. On the left side, with the neck slightly flexed.
80 80 80 2. On the back, with the head turned to the left side.
60 60 60 3. On the abdomen, with the head down.
100 HEDS 100 HEDS 100 4. On the back, with the neck slightly extended.
DL DL
80 80 80
17. Which observation of an 8-pound, 4-ounce neonate, if
DRF DRF
STL STL
60 ROM 60 ROM 60
pH pH
40 O2 40 O2 40 made by an RN, would require an intervention?
PULSE PULSE
20 TEMP. 20 TEMP. 20
0 DP
0 DP
0 1. The neonates respirations are 36, shallow, and irregu-
Pattern 2 lar in rate, rhythm, and depth.
240 2. The neonates axillary temperature is 96.2F (35.6C).
3. Rapid pulsations are visible in the fifth intercostal
210
space, left midclavicular line.
4. There is asynchronous spontaneous movement of the
180
infants extremities.
QUESTIONS

150 18. A client presents to a labor and delivery unit at 38 weeks


gestation. Before applying an external fetal monitor, an
120
RN performs Leopolds maneuver and finds a hard, dis-
tinct mass in the upper right quadrant of the clients
90
uterus. Where should the RN anticipate finding the fetal
60 heart tones?
1. Below the umbilicus on the clients left side.
30 2. Above the umbilicus on the clients left side.
Pattern 3 3. Below the umbilicus on the clients right side.
100 4. Above the umbilicus on the clients right side.

75
19. A pregnant client expresses concern about her gestational
diabetes and asks an RN, Does this mean I will be dia-
50 betic for the rest of my life? Which is the most appropri-
ate response by the RN?
25 1. In most cases, a woman with gestational diabetes will
become a diabetic who is insulin-dependent.
0 2. If you follow your diabetic diet, you will probably not
180 180 180 have anything to worry about.
150 150 150
3. As long as you dont become pregnant again you will
not become a diabetic.
120 120 120
4. There is a possibility that you may develop type 2
90 90 90 diabetes at some point in the future.
60 60 60
20. A physician orders oxytocin IV at 2 milliunits/minute for
30 30 30
100 100 100
the induction of labor. The premixed IV has 15 units of
oxytocin in 250 mL of normal saline. A nurse calculates
80 80 80
that _______ mL/hr should be administered to the client
60 60 60 via an infusion pump. Fill in the blank.
40 40 40
21. A client gave birth 40 minutes ago. The placenta has not
20 20 20
yet delivered. What is the most appropriate action for an
0 0 0 RN to take at this time?
1. Pattern 1 because the tracing shows early decelerations. 1. Inform the primary health-care provider.
2. Pattern 2 because the tracing shows late decelerations. 2. Apply traction to the umbilical cord.
2164_Ch04_115-248 29/03/12 12:23 PM Page 221

Questions 221
3. Prepare the client for surgery. 3. Restricted movement in utero.
4. Reassure the client that this is normal. 4. An anomaly during embryonic development.
22. During a routine ultrasound examination, a client, who is 28. A nursing student is preparing to administer an injection
33 weeks pregnant, states that she feels that her heart is of vitamin K, 1 mg IM, to a newborn. The student asks a
pounding. She also reports feeling hot and sweaty. nurse, Where should I give the injection? Which is the
Which intervention by a nurse is the most appropriate? most accurate response by the nurse?
1. Get the client a cool washcloth for her forehead. 1. Gluteus maximus.
2. Administer nasal oxygen at 12 L/min. 2. Gluteus minimus.
3. Place the client in Trendelenburg position. 3. Vastus lateralis.
4. Assist her to a left lateral position. 4. Vastus medialis.
23. A client who is 32 weeks gestation presents to a physi- 29. A client, who is multiparous, delivered a 9-pound,
cians office for a routine prenatal checkup. Leopolds 12-ounce baby girl at 41 3/7 weeks gestation. After
maneuver reveals the fetal position as right occipitoanteri- 14 hours of labor, she had an arrest of descent and a
or (ROA). At which site would an RN expect to find the cesarean section was performed. While performing an
fetal heart tone? initial newborn assessment, what observations should a
1. Below the umbilicus, on the clients left side. nurse expect to find in a neonate of this gestation?
2. Below the umbilicus, on the clients right side. Select all that apply.
3. Above the umbilicus, on the clients left side. 1. Vernix.
4. Above the umbilicus, on the clients right side. 2. Plethora.
3. Milia.
24. An RN is completing a home visit with a client who 4. Crepitus.
delivered a full-term infant 4 days ago. The client states

QUESTIONS
5. Parchment skin.
that she is exclusively bottle feeding her infant. Upon
assessment, the RN notes white, curdlike patches on the 30. A client who is at 38 weeks gestation presents to labor
newborns oral mucous membranes. Which action should and delivery and states, I have water leaking down my
the RN take? legs. Which assessment by a nurse is most appropriate in
1. Explain that the newborn will need to receive some this situation?
medication. 1. Deep tendon reflexes.
2. Suggest that the newborns formula be changed. 2. Fern test.
3. Remind the caregiver not to let the infant sleep with 3. Blood pressure check.
the bottle. 4. Urine test for protein.
4. Determine the babys blood glucose level.
31. A nurse performs a sterile vaginal examination on a
25. During the examination of a newborn infant, an multiparous client at 37 weeks gestation. The client is
RN suspects congenital dislocation of the left hip. 8 cm dilated, completely effaced, and a +1 station. She
Which assessment finding would confirm the nurses tells the nurse, I cant keep myself from pushing when
suspicion? I have a contraction. Which nursing intervention is
1. Lengthening of the limb on the affected side. most appropriate in this situation?
2. Deformities of the foot and ankle. 1. Encourage her to push through the last 2 cm of
3. Plantar flexion of the foot. dilation.
4. Asymmetry of the gluteal and thigh folds. 2. Assist the client to pant-blow at the peak of contractions.
3. Readjust the fetal monitor.
26. A nurse is caring for a client who is hospitalized with 4. Tell the coach that it is not good for the baby if she
severe pregnancy-induced hypertension (PIH). What
pushes right now.
would be an appropriate nursing intervention for this
client? 32. A client, who is gravida 1, para 0 at 40 weeks gestation,
1. Encourage visitors to spend time with her. is in the active phase of labor. The client has had no
2. Keep her in the supine position. identified risk factors during her prenatal care. A nurse
3. Keep her room darkened and limit nursing visits. evaluates the fetal monitor strip at 10:00 hours. The fetal
4. Keep her on modified bedrest in a lateral position. heart rate (FHR) is 130 with moderate variability, and
accelerations are present, but no decelerations. At what
27. The mother of a neonate with a clubfoot feels guilty time should the nurse reevaluate the FHR?
because she believes she did something during her preg-
1. 10:05 a.m.
nancy to cause the deformity. A nurse should explain to
2. 10:15 a.m.
the mother that the cause of clubfoot is:
3. 10:30 a.m.
1. Unknown.
4. 11:00 a.m.
2. Hereditary.
2164_Ch04_115-248 29/03/12 12:23 PM Page 222

222 chapter 4 Health Promotion and Maintenance

33. A client, who is gravida 4, para 3 at 39 weeks gestation, sways slowly while humming softly. At the end of the
has no identified prenatal risk factors and is in early contraction, her husband gently places her on her side and
labor. The client declines electronic fetal monitoring she rests with her eyes closed until the next contraction
(EFM), stating that she delivered her second baby at begins. Which is the most appropriate nursing action for
home without a monitor and everything went well. What this client?
is the most appropriate way for the nurse to handle this 1. Offer her clear liquids.
situation? 2. Offer her an epidural.
1. Explain that she will need to be on the monitor for 3. Offer her pain medication.
about 20 minutes to assess fetal well-being. If the 4. Encourage her to begin pant-blow breathing.
initial tracing is reassuring, the baby may be moni- 37. A client, who is gravida 3, para 1, is 8 cm dilated, com-
tored intermittently. pletely effaced, and a +1 station. She becomes agitated
2. Insist that the fetal monitor be used because there are and snaps at her husband and a nurse. She exclaims, I
not enough staff members to adequately monitor her cant stand this anymore! I want a cesarean section! What
using any other method. is the most appropriate response by the nurse at this time?
3. Request a change in assignment because of the respon- 1. I understand you are in a lot of pain, but you are
sibility if anything were to happen to her baby during doing very well and you are almost ready to have this
labor. baby.
4. Tell her that it is her decision, although continuous 2. I will call the doctor and tell him about your request.
EFM is the only effective way to monitor infant 3. Just breathe; you dont need to have a cesarean section.
well-being during labor. 4. I understand you want a cesarean section, but you are
34. A nurse evaluates a fetal monitor tracing. The nurse in the transition phase of labor and your feelings are
should anticipate that an obstetrician will consider very common in this stage.
QUESTIONS

performing an amnioinfusion if the electronic fetal 38. A nurse gives a report on a client who is 29 years old,
monitor tracing reveals: gravida 2, para 1 at 32 weeks gestation. The fetus is in a
1. Baseline heart rate = 130, moderate variability vertex presentation and the water bag is intact. A vaginal
present, occasional accelerations and consistent examination was done 1 hour previously. At that time,
early decelerations. the clients cervix was 3 cm dilated and 80% effaced. The
2. Baseline heart rate = 145, minimal variability present, station was 2. Continuous EFM is ordered and IV flu-
no accelerations and no decelerations are present. ids are infusing into the clients left arm. Which is the
3. Baseline heart rate = 135, moderate variability most important nursing assessment at this time?
present, no accelerations and occasional mild 1. The frequency and duration of contractions.
variable decelerations. 2. The patency of the IV infusion.
4. Baseline heart rate = 120, moderate variability present, 3. The baseline fetal heart rate.
and deep variable decelerations with every contraction. 4. The maternal vital signs.
35. A client, who is gravida 1, para 0, presents to labor and 39. A client delivers a 7-pound, 4-ounce neonate at 38 weeks
delivery because her contractions have become regular, gestation. What observations, if found on initial assess-
every 5 minutes, and she rates them a 5 on a pain scale ment, should a nurse report to a physician? Select all that
of 1 to 10. She talks excitedly with her husband, who is apply.
to be her coach. She says that she is pleased that the 1. Slight acrocyanosis.
happy day is finally here, but she is afraid that she will 2. Respiratory rate of 20.
start screaming and she will not be able to handle it when 3. Consistent heart rate of 145.
the contractions get stronger. She and her husband have 4. Pilonidal dimple.
attended childbirth preparation classes. Based on her 5. Ecchymosis over the occiput.
contractions pattern and her behavior, what are the most
likely stage and phase of labor? 40. A client, who is gravida 1, para 0, is admitted to labor
1. First stage, active phase. and delivery for induction of labor at 41 weeks gestation.
2. First phase, active stage. Her cervical examination reveals her cervix to be closed
3. First phase, latent stage. and thick and the fetal head is out of the pelvis. Which
4. First stage, latent phase. medications should a nurse anticipate being ordered by a
physician? Select all that apply.
36. Several hours after admission to a birthing center, a client, 1. Nifedipine.
who is gravida 1, para 0, is dilated 5 cm. The cervix is 2. Cervidil.
completely effaced and the fetus is at a +1 station. When a 3. Prostaglandin gel.
contraction begins, the client kneels on the bed, rests her 4. Misoprostol.
upper body against her husband, closes her eyes, and 5. Brethine.
2164_Ch04_115-248 29/03/12 12:23 PM Page 223

Questions 223
41. A 23-year-old client, who is gravida 3, para 2, delivered a 3. The intestines are compressed during pregnancy,
baby by vacuum-assisted vaginal delivery 2 days ago. As which causes stool stasis.
part of the discharge teaching class, for which conditions 4. The muscle movement of the intestines speeds up,
should a nurse instruct the client to call a physician? which causes dry, hard stools.
Select all that apply. 47. A client, who is gravida 6, para 2, is admitted to labor
1. Pain that is not well controlled by prescription pain and delivery in early labor on October 18. A nurse pal-
medication. pates regular uterine contractions every 5 minutes with
2. Pain in her calves when walking. moderate intensity. A sterile vaginal examination reveals a
3. Foul-smelling lochia. soft cervix that is 2 cm dilated, 85% effaced, and a 2
4. Lochia serosa that is soaking less than a pad every station. Which admission information is most important
3 hours. in planning this clients nursing care?
5. Soaking more than a pad an hour with lochia. 1. The clients LMP was January 2.
42. A client, who is a primigravida at 10 weeks gestation, 2. The clients blood type is A and Rh+.
reports mild uterine cramping and slight vaginal spotting 3. The clients hemoglobin is 11 gm/dL.
without passage of tissue. When assessed, no cervical dila- 4. The clients blood pressure is 100/64 mm Hg.
tion is noted. Which is the best action by the nurse? 48. A pregnant client attends a first-trimester class on
1. Anticipate that the woman will be sent home and eating well for pregnancy. Which statement should
placed on bedrest. indicate to a nurse that the client requires additional
2. Prepare the woman for a dilation and curettage. teaching?
3. Notify a grief counselor to assist the woman with the 1. I should gain around 30 pounds by my due date.
imminent loss of her fetus. 2. My strange craving to eat laundry starch is a normal
4. Tell the woman that the doctor will most likely per-

QUESTIONS
part of pregnancy.
form a cerclage to help maintain the pregnancy. 3. I need about an extra 300 calories per day during my
43. A client, who is gravida 3, para 2 at 28 weeks gestation, pregnancy.
exhibits glucosuria during her routine prenatal visit. A 4. Frozen foods are more nutritious than canned
nurse knows that glucosuria during pregnancy indicates: foods.
1. An increased glomerular filtration of glucose. 49. A client, who is 22 years old, gravida 1, para 0, is
2. Gestational diabetes. admitted to an obstetrics unit with contractions every
3. A need for a 3-hour glucose tolerance test. 8 to 10 minutes. A physical assessment reveals that her
4. A need for small doses of insulin. cervix is 3 cm dilated, 100% effaced, and a 0 station.
44. A client, who is gravida 4, para 1 at 28 weeks gestation, During the admission process, a nurse asks the couple
completes a 1-hour glucose tolerance test. Her 1-hour about their plans for pain relief during labor. The client
blood sugar is 150 mg/dL. Which type of follow-up and her support person state they planned to use pre-
should a nurse recommend to this client? pared childbirth techniques. Based on the information
1. No follow-up. provided, which pain relief method should the nurse
2. A 2000-calorie diet. expect the couple to use during this phase of labor?
3. A 3-hour glucose tolerance test. 1. Slow, deep breathing.
4. A 2-hour postprandial blood sugar. 2. Narcotic analgesia.
3. Local anesthesia.
45. A client, who is at 32 weeks gestation, comes to a clinic for 4. Rapid, shallow breathing.
a routine prenatal visit. She is having discomfort related to
constipation. The nurse should teach the client that the 50. A client, who is 37 years old, gravida 4, para 3 at 14
most appropriate measures to alleviate this problem include: weeks gestation, asks a nurse, Do you think Im having
1. Taking a mild laxative daily. a boy? All my other children are girls. If I dont have a
2. Adding more protein and fat to the daily diet. boy this time, my husband will really be disappointed in
3. Drinking hot coffee or tea each morning at breakfast. me. Which explanation by the nurse would be most
4. Drinking 810 cups of water and take a daily walk. accurate?
1. The heartbeat of the baby is fast; that means its a
46. A client, who is at 26 weeks gestation, presents for a rou- boy.
tine appointment. The client asks a nurse why she is hav- 2. Girls probably run in your family; theres nothing you
ing trouble with constipation during her pregnancy. can do about it.
Which explanation by the nurse would be most accurate? 3. The fathers sperm determines if the baby is male or
1. The intestines are expanded during pregnancy, which female.
causes stool stasis. 4. Dont worry; you are carrying this baby low; that
2. The muscle movement of the intestines slows down, means its a boy.
which causes dry, hard stools.
2164_Ch04_115-248 29/03/12 12:23 PM Page 224

224 chapter 4 Health Promotion and Maintenance

51. A client at 12 weeks gestation is at her first prenatal visit. 3. -Fetoprotein (AFP).
The client is given instructions to get prenatal labs and a 4. Partial thromboplastin time (PTT).
multiple marker screen drawn at the laboratory. She asks 56. A client, who is gravida 6, para 4, has a routine ultra-
a nurse about the purpose of the multiple marker screen. sound at 16 weeks gestation. The ultrasound shows that
The nurse should inform the client that the conditions the client is carrying dizygotic twins. She asks a nurse
that may be detected with this test are: what the difference is between monozygotic and dizygot-
Select all that apply. ic twins. Which explanation by the nurse would be most
1. Gestational diabetes. accurate?
2. Edwards syndrome. 1. Monozygotic twins come from two different eggs and
3. Pregnancy-induced hypertension. sperm.
4. Trisomy 21. 2. Monozygotic twins come from one egg and two
5. Neural tube defects. sperm.
52. A 24-hour-old neonate is icteric with a serum bilirubin 3. Dizygotic twins come from one fertilized egg that
level of 14 mg/100 mL. The mothers blood type is O+; splits.
the blood type of the baby is B and has a positive direct 4. Dizygotic twins come from two different eggs and
Coombs test. The infant is being breastfed. Which meas- sperm.
ure should a nurse include in the neonates plan of care? 57. A client, who is 18 years old, presents to a prenatal clinic
1. No special measures are necessary; newborns normally because her menstrual period is 9 days late. She tells a
get a little jaundiced. nurse, Im sure Im pregnant because my period is late
2. Tell the mother to stop breastfeeding and give formula and my breasts are tender. Which response by the nurse
to the baby instead. would be most appropriate?
3. Place the infant under the bililights and prepare for an
QUESTIONS

1. A missed menstrual period and breast tenderness are


exchange transfusion. probable signs of pregnancy.
4. Encourage the mother to increase the frequency of 2. A missed menstrual period and breast tenderness are
breastfeeding sessions. presumptive signs of pregnancy.
53. A client, who is gravida 2, para 0, has a routine ultra- 3. A missed menstrual period and breast tenderness are
sound at 18 weeks gestation to confirm her estimated positive signs of pregnancy.
date of delivery; the ultrasound shows the fetus in a 4. A missed menstrual period and breast tenderness are
breech position. The client asks a nurse Does this mean negative signs of pregnancy.
I will have to have a C-section? Which response by the 58. A client who is gravida 2, para 1 is seen in an emergency
nurse would be most accurate? department at 14 weeks gestation with bright red vaginal
1. If a baby is breech at this gestation, it must always be bleeding and severe lower abdominal cramping. Which
delivered by cesarean section. assessment finding is the most important observation at
2. The baby will have more room to turn as your delivery this time?
date nears. 1. Increased temperature.
3. You can probably deliver normally; most babies are 2. Increased pulse pressure.
born breech. 3. Increasing heart rate.
4. Many babies are breech at this gestation; most turn to 4. Increased blood pressure.
head-down by term.
59. An anesthetist has just placed an epidural catheter dosed
54. A laboring client, who is gravida 1, para 0, has had no with bupivacaine hydrochloride (Marcaine) in a client in
change in her cervix for 2 hours and remains 9 to 10 cm labor who is gravida 2, para 1 at 39 weeks gestation. For
dilated. The fetal head has remained at 0 station. A sterile which side effect should a nurse observe this client?
vaginal examination reveals a position of occiput posterior. 1. Hypotension.
Which action by the nurse would be most appropriate? 2. Hypertension.
1. Prepare the client for a forceps rotation. 3. Hypoventilation.
2. Assist the client to a hands-and-knees position. 4. Hyperventilation.
3. Assist the client to a supine position.
4. Prepare the client for a cesarean delivery. 60. A nurse assesses a client who is 40 years old, gravida 11,
para 9 at 14 weeks gestation, and has a history of throm-
55. A client with preterm contractions at 34 weeks gestation is boembolitic disease. The client is placed on daily heparin
dilated to 3 cm. A physician orders an amniocentesis for therapy. Which laboratory test will be the most important
fetal lung maturity. Which laboratory test will provide the for this client?
most information to a nurse about fetal lung maturity? 1. Prothrombin time.
1. Human chorionic gonadotropin (HCG). 2. Partial thromboplastin time.
2. Phosphatidylglycerol (PG).
2164_Ch04_115-248 29/03/12 12:23 PM Page 225

Questions 225
3. Bleeding time. 3. The client will ambulate in the room without
4. D-dimer. assistance.
4. The client will maintain physiological homeostasis.
61. A client, who is 36 years old, has been trying to get preg-
nant for about 5 months. At her physician appointment, 66. A client, who is 19 years old, gravida 1, para 0, is admitted
she states that she knows she should take folic acid sup- to labor and delivery with HELLP syndrome at 28 weeks
plements but dislikes taking pills. Which foods that are gestation. A nurse knows that management for her condi-
high in folic acid should a nurse suggest? Select all that tion should include:
apply. Select all that apply.
1. Salmon. 1. Administration of corticosteroid.
2. Spinach. 2. Administration of an IV fluid bolus.
3. Orange juice. 3. Liver function tests.
4. Asparagus. 4. Administration of tocolytics.
5. Chicken. 5. Bedrest in the left lateral position.
62. A client has just completed the second stage of labor and 67. A nurse assesses a client who is 15 years old, gravida 1,
has begun the third stage of labor. Which nursing action para 0, and is 11 weeks gestation. Which prenatal labo-
has priority at this time? ratory test would indicate a risk factor for this client?
1. Promotion of the bonding process. 1. White blood cell count = 12,000.
2. Administration of an oxytocic medication. 2. Hematocrit = 32%.
3. Encouraging the client to push. 3. Hemoglobin = 9 gm/dL.
4. Physical assessment of the neonate. 4. Random serum glucose = 105 gm/dL.
63. A client, who delivered a full-term infant 45 minutes ago, 68. A client, who is gravida 3, para 2 at 37 weeks gestation,

QUESTIONS
is carefully examining her newborn during the initial is 5 cm dilated, 80% effaced, and a +1 station. She
breastfeeding. She asks a nurse, My babys head is received epidural anesthesia with bupivacaine hydrochlo-
shaped like a bullet! Will it stay like that? Which ride (Marcaine) for contraction pain 1 hour ago. Which
response by the nurse is most accurate? nursing assessment is most important for this client at
1. That is called a caput. It usually lasts for 3 or 4 days. this time?
2. That is called a hemangioma. It will probably last a 1. Assessing the client hourly for respiratory depression.
few months. 2. Assessing the client hourly for uterine atony.
3. That is called a cephalohematoma. It usually lasts for 3. Assessing the client hourly for bladder distention.
a few weeks. 4. Assessing the client hourly for hypertension.
4. That is called molding. It usually lasts for a few days.
69. A client, who is gravida 1, para 0, arrives in a labor and
64. A client, who is 20 years old, delivered her first baby delivery unit at 38 weeks gestation with mild irregular
2 days ago. She is exclusively breastfeeding and has been contractions. A sterile vaginal examination reveals the fol-
discharged home by her physician. A nurse has completed lowing: cervical dilation of 3 cm, 60% effaced, 1 station,
discharge teaching on newborn care. Which statement by and membranes intact. An external fetal monitor is placed
the client would indicate a need for further teaching? and the fetal heart tracing reveals a baseline of 130 bpm,
1. I should nurse my baby whenever he acts hungry and moderate variability, and accelerations to the 150s during
for as long as he wants to nurse. contractions. Based on this information, which nursing
2. If my baby has at least one wet diaper and one bowel action would be most appropriate?
movement a day, he is getting enough to eat. 1. Turn the client to her left side and administer oxygen.
3. I should dress my baby in clothing I would be com- 2. Prepare the client for an immediate operative delivery.
fortable in, plus a light blanket. 3. Begin an intravenous infusion for hydration.
4. I should clean the cord stump with alcohol every day 4. Send the client home and encourage her to ambulate.
until it falls off.
70. A client, who delivered 8 weeks ago, is exclusively breast-
65. A client, who is gravida 3, para 2, has a spontaneous feeding, and asks for information on birth control meth-
vaginal delivery of a 6-pound baby girl over an intact per- ods that do not interfere with breast milk production.
ineum. She has an epidural for relief of labor pain. She is Which statement should indicate to a nurse that the
currently in the fourth stage of labor. Which nursing goal client needs further instruction?
would be most appropriate for the client at this time? 1. Breastfeeding itself is effective at preventing pregnancy.
1. The clients episiotomy will remain clean, dry, and 2. Using condoms would be a good choice for me.
intact. 3. I can use Depo-Provera and breastfeed without
2. The client will turn, cough, and deep breathe 10 times problems.
an hour. 4. I can use contraceptive foam for birth control.
2164_Ch04_115-248 29/03/12 12:23 PM Page 226

226 chapter 4 Health Promotion and Maintenance

71. A nurse assesses a client who delivered an 8-pound, to be infused at 2 gm/hr. Which action by a nurse
6-ounce infant 2 hours ago. Which assessment findings indicates understanding of the possible side effects of
are considered normal in a client during this time? magnesium sulfate?
Select all that apply. 1. Placing a sign over the bed not to check blood pressure
1. Fundus firm, at the umbilicus. in the right arm.
2. Fundus firm, 2 fingerbreadths below the umbilicus. 2. Placing a padded tongue blade at the bedside.
3. Bluish-white fluid expressed from her breasts. 3. Inserting a Foley catheter.
4. Lochia serosa, moderate amount. 4. Darkening the room.
5. Moderate lochia rubra. 77. A nurse enters the room of a client who is a 22-year-old
72. A client delivered an 8-pound, 9-ounce neonate at primigravida who was 39 weeks gestation. The infant
39 weeks gestation. Which observations, if found during was delivered using vacuum extraction 3 hours ago. The
the initial assessment, are normal findings and do not client is in tears and says When I first delivered, the
need to be reported to a physician? Select all that apply. baby breastfed for almost an hour, now the baby wont
1. Heart rate of 170 at rest. wake up enough to take the nipple! Which intervention
2. Respiratory rate of 46. would be most appropriate in this case?
3. Circumoral cyanosis. 1. Instruct the client to supplement breastfeeding with
4. Caput succedaneum over the occiput. formula; the baby isnt waking up because there isnt
5. Mongolian spots. enough milk.
2. Instruct the client to wake the baby by changing the
73. A client with type 1 insulin-dependent diabetes babys diaper.
presents to a prenatal clinic at 17 weeks gestation for 3. Instruct the client to let the baby sleep, because babies
-fetoprotein testing. She asks a nurse why this test is cluster-feed, and frequently sleep for several hours after
QUESTIONS

being performed. Which explanation by the nurse the initial breastfeeding.


would be most accurate? 4. Instruct the client to submerge the baby into a bath of
1. This test is to determine the sex of your baby. lukewarm water to wake the baby up so that the baby
2. This test is to determine glycemic control. will breastfeed.
3. This test is screening for neural tube defects.
4. This test is to determine fetal lung maturity. 78. A client, who is gravida 3, para 0 at 39 weeks gestation, is
admitted to a labor and delivery unit in active labor. The
74. A client entered the fourth stage of labor 30 minutes ago. physician performs an amniotomy. Which assessment
During an assessment, a nurse notes that there is constant finding should a nurse anticipate after the amniotomy?
trickling of bright red vaginal blood in the presence of a 1. Fetal heart tones 90 bpm.
contracted uterus midline at the umbilicus. Which action 2. A moderate amount of straw-colored fluid.
by the nurse would be most appropriate in this situation? 3. A small amount of greenish fluid.
1. Massage the fundus. 4. A small segment of the umbilical cord.
2. Call the physician.
3. Have the client empty her bladder. 79. A client, who is gravida 1, para 0, is admitted to labor and
4. Increase the oxytocin infusion. delivery with spontaneous rupture of membranes. A vagi-
nal examination reveals clear fluid and the cervix is 2 cm
75. A client, who is a gravida 3, para 1, presents to labor and dilated. Based on the information provided, which state-
delivery at 41 4/7 weeks gestation for induction of labor. ment should a nurse expect this client to make?
Her cervix is 2 cm dilated, 80% effaced, and a 3 station. 1. We have a name picked out for the baby.
A nurse anticipates that, in preparation for labor, a physi- 2. I need to push when I have a contraction.
cian will likely order: 3. I cant concentrate if anyone is touching me.
Select all that apply. 4. When can I get my epidural?
1. Hemabate 250 mcg IM.
2. Pitocin at 2 milliunits/min IV. 80. A client, who is gravida 2, para 0 at 38 weeks gestation,
3. Betamethasone 6 mg IM. is having sharp decreases in fetal heart rate from a base-
4. Cytotec 200 mcg PO. line of 130 bpm to 90 to 110 bpm during the contrac-
5. Cervidil 10 mg vaginally. tions. Which action should a nurse take first?
1. Reposition the monitor.
76. A client, gravida 2, para 0, is admitted to a labor 2. Turn the client to her left side.
and delivery unit at 35 weeks gestation. Her blood 3. Ask the client to ambulate.
pressure is 180/110 mm Hg, pulse 88, respirations 18, 4. Prepare the client for delivery.
and temperature 98.6F. Her urine dipstick shows 3+
proteinuria and a physical assessment reveals 3+ deep 81. A client, who is gravida 1, para 0, is dilated to 6 cm,
tendon reflexes. A physician orders magnesium sulfate 100% effaced, and a 0 station for the past 2 hours
2164_Ch04_115-248 29/03/12 12:23 PM Page 227

Questions 227
without any change in her cervix. A physician orders 3. Reposition the client.
oxytocin (Pitocin) augmentation. When evaluating the 4. Readjust the monitor.
effectiveness of IV oxytocin augmentation, which out-
87. A gravida 4, para 2 client with gestational diabetes is hav-
come should a nurse anticipate?
ing nonstress tests twice a week. A nurse evaluates the
1. A painless delivery.
fetal monitor strip. Which fetal heart rate pattern should
2. Cervical effacement.
the nurse interpret as reassuring?
3. Infrequent contractions.
1. A baseline fetal heart rate of 170180 bpm.
4. Progressive cervical dilation.
2. Baseline variability of 2535 bpm.
82. A client, who is gravida 1, para 0 at 37 weeks gestation, 3. Variable decelerations to 100 bpm.
is admitted to labor and delivery in active labor. A vagi- 4. Acceleration of FHR with fetal movements.
nal examination reveals a footling breech presentation.
88. A client, who is gravida 3, para 2, receives an epidural for
Which action should the nurse take first?
labor pain relief. A nurse knows the client will need to
1. Anticipate the need for a cesarean section.
have a Foley catheter placed if the epidural is in place
2. Apply the fetal heart monitor.
longer than 1 hour. Which is the best rationale for this
3. Place the client in genupectoral position.
nursing action?
4. Perform an ultrasound exam.
1. The bladder fills more rapidly because of the medica-
83. A client, who is gravida 2, para 1, is admitted to a tion used for the epidural.
birthing center. A nurse is using a Doppler to check fetal 2. Her level of consciousness is such that she is in a
heart tones. The nurse finds fetal heart tones of 160 to trancelike state.
170 bpm. A vaginal examination reveals that the cervix 3. The sensation of the bladder filling is diminished or lost.
is 4 cm dilated, with intact membranes and a 1 station. 4. She is embarrassed to ask for the bedpan that frequently.

QUESTIONS
The nurse decides to apply an external fetal monitor rather
89. A woman is practicing natural family planning methods.
than an internal monitor. Which is the best rationale for
She asks a nurse about the most likely time for her to
this decision?
conceive. The nurse explains that conception is most like-
1. The cervix is closed.
ly to occur when:
2. The membranes are still intact.
1. Estrogen levels are low.
3. The fetal heart tones are within normal limits.
2. Luteinizing hormone is high.
4. The contractions are intense enough for insertion of
3. The endometrial lining is thin.
an internal monitor.
4. The progesterone level is low.
84. A client, who is gravida 1, para 0, presents to labor and 90. A client tells a nurse that she plans to use the rhythm
delivery with contractions every 3 minutes. The client
method of birth control. The nurse instructs the client
states that she has not felt the baby move for the last
that the success of this method depends on the:
2 hours. What action should the nurse take first?
1. Age of the client.
1. Call the physician.
2. Frequency of intercourse.
2. Place the client on an external fetal monitor.
3. Regularity of the menses.
3. Start an IV.
4. Range of the clients temperature.
4. Obtain maternal vital signs.
91. A 34-year-old client with type 1 diabetes since age 5 asks
85. A client, who is gravida 3, para 2, is 8 cm dilated, 100% a nurse for advice regarding methods of birth control.
effaced, and a 0 station. When evaluating the fetal moni-
Which method of birth control is the best choice for a
tor tracing, a nurse notes baseline heart rate 170 bpm,
client with type 1 diabetes?
minimal variability, and consistent late decelerations.
1. Intrauterine device.
What is the most likely explanation of this pattern?
2. Oral contraceptives.
1. The fetus is asleep.
3. Diaphragm.
2. The umbilical cord is compressed.
4. Contraceptive patch.
3. There is a head compression.
4. There is uteroplacental insufficiency. 92. A client presents to an emergency department at 8 weeks
gestation. A physician suspects that the client has an
86. A client, who is gravida 4, para 3, is 4 cm dilated, 80% ectopic (tubal) pregnancy. When performing an initial
effaced, and a +1 station. Assessment of the fetal monitor
assessment, which symptom should a nurse recognize as
tracing reveals baseline 140 bpm, moderate variability,
consistent with a diagnosis of ectopic (tubal) pregnancy?
and accelerations and variable decelerations to 100 bpm.
1. Painless vaginal bleeding.
Which action should a nurse take first?
2. Abdominal cramping.
1. Notify her physician.
3. Throbbing pain in the upper quadrant.
2. Give a 500-mL bolus IV.
4. Sudden, stabbing pain in the lower quadrant.
2164_Ch04_115-248 29/03/12 12:23 PM Page 228

228 chapter 4 Health Promotion and Maintenance

93. A client, who is 10 weeks gestation, has been having 4. Hyperbilirubinemia.


severe nausea for the past 3 weeks. She tells a nurse that 5. Gastroschisis.
she cannot eat anything. She has been diagnosed with 99. An 18-year-old client chooses oral contraceptives as
hyperemesis gravidarum and is at risk for developing: her method of birth control. Which instruction
1. Respiratory alkalosis without dehydration. should be included in a nurses teaching regarding
2. Metabolic acidosis with dehydration. oral contraceptives?
3. Respiratory acidosis without dehydration. 1. Weight gain should be reported to the physician.
4. Metabolic alkalosis with dehydration. 2. An alternate method of birth control is needed when
94. A client presents to a prenatal clinic and tells a physician taking antibiotics.
that she thinks she might be pregnant because she has 3. If the client misses one or more pills, two pills
not has a period for about 5 months. Which is the most should be taken per day for 1 week.
definitive sign of pregnancy? 4. Nausea or stomach upset should be reported to the
1. Elevated human chorionic gonadotropin. physician.
2. The presence of fetal heart tones. 100. A 27-year-old client, who is gravida 3, para 1, presents to
3. Uterine enlargement. a labor and delivery unit at 33 weeks gestation. She tells
4. Breast enlargement and tenderness. a nurse that she woke up this morning in a pool of blood
95. A client, who is gravida 3, para 2 at 39 weeks gestation about the size of an orange, but she has no abdominal
with poorly controlled gestational diabetes, has just given cramping or pain. The nurses first action should be to:
birth via cesarean section. A nurse will expect that the 1. Assess the fetal heart tones.
neonate will most likely be: 2. Check for cervical dilation.
1. Hypoglycemic, small for gestational age. 3. Check for firmness of the uterus.
ANSWERS

2. Hyperglycemic, large for gestational age. 4. Obtain maternal vital signs.


3. Hypoglycemic, large for gestational age.
4. Hyperglycemic, small for gestational age. Answers/Rationales/Tips
96. A client, who is 44 years old, gravida 3, para 2, has just 1. CORRECT ANSWERS: 1, 2. Answer 1 is correct because
delivered a newborn suspected of having trisomy 21.
placing the client on an electronic fetal monitor is an
Which characteristics should a nurse observe in an infant
appropriate intervention to evaluate uterine contractions
with this condition? Select all that apply.
and determine fetal well-being. Evaluate of fetal well-
1. Simian creases.
being is very important for this client because of her
2. Increased muscle tone.
complaint of decreased fetal movement. Answer 2 is
3. Flat appearance of the face.
correct because betamethasone is a corticosteroid used
4. Small tongue.
to help stimulate fetal lung maturity in infants who are
5. Upward-slanting eye creases.
preterm. Answer 3 is incorrect because performing a nipple
97. A nursing student is performing an initial newborn stimulation test is inappropriate for a client who is in
assessment. The newborn is observed to have a cephalo- preterm labor; stimulating nipples may stimulate uterine
hematoma. What are the likely causes of this condition? contractions, which is contraindicated in preterm labor.
Select all that apply. Answer 4 is incorrect because adequate hydration (not limit-
1. Scheduled cesarean delivery. ing oral fluids) is important to decrease uterine contraction
2. Prolonged latent phase of labor. in clients who are in preterm labor. Answer 5 is incorrect
3. Prolonged second stage of labor. because placing the client on strict bedrest increases the
4. Vacuum-assisted vaginal delivery. chance of developing deep vein thrombosis. Modified bedrest
5. Breech presentation. is the appropriate management for this client.
TEST-TAKING TIP: Eliminate Answers 3 and 4 because these
98. A client, who is 24 years old, gravida 4, para 3, has had no actions are contraindicated in preterm labor. Since pregnancy
prenatal care and does not know when her last menstrual
increases the chance for developing blood clots, strict bedrest
period was. She presents to labor and delivery completely
also increases the chance of developing blood clots; therefore,
dilated and crowning. She precipitously delivers a 5-pound,
eliminate Answer 5 also.
6-ounce infant. The clients urine toxicology screen shows
Content Area: Maternity, Intrapartum; Integrated Process:
methamphetamines. The client admits that she uses
Nursing Process, Implementation; Cognitive Level: Application;
methamphetamine daily. Which observations should a
Client Need/Subneed: Physiological Integrity/Physiological
nurse expect in the neonate that would be consistent with
Adaptation/Alterations in Body Systems
methamphetamine exposure in utero? Select all that apply.
1. Cleft lip. 2. CORRECT ANSWERS: 1, 2, 5. Answer 1 is correct because a
2. Irritability. risk factor for developing an infection postpartum is a long
3. Clubfoot. active phase of labor. Answer 2 is correct because prolonged
2164_Ch04_115-248 29/03/12 12:23 PM Page 229

Answers/Rationales/Tips 229
rupture of membranes (longer than 24 hours) is a risk Client Need/Subneed: Health Promotion and Maintenance/
factor for developing postpartum endometritis. Answer 3 is Ante/Intra/Postpartum and Newborn Care
incorrect because prolonged labor, not precipitous delivery, is
5. CORRECT ANSWER: 1.5. The calculation
associated with an increased risk of postpartum endometritis.
is completed by dividing 100 mcg/2 mL; there are
Answer 4 is incorrect because a long active phase, not latent
50 mcg/1 mL, 25 mcg/0.5 mL. Therefore, 75 mcg is
phase, of labor is associated with a higher risk of endometritis.
1.5 mL of fentanyl.
Answer 5 is correct because internal fetal monitoring can act
TEST-TAKING TIP: Calculate how many milligrams (mg) are
as a conduit for transmission of bacteria into the uterine
in each milliliter (mL). The calculation is easier when you are
cavity during labor and delivery, increasing the risk of devel-
dealing with 1 mL.
oping postpartum endometritis.
Content Area: Maternity, Intrapartum; Integrated Process:
TEST-TAKING TIP: Endometritis is an infection (not to be
Nursing Process, Analysis; Cognitive Level: Analysis; Client
confused with endometriosis, which is dysfunctional uterine
Need/Subneed: Physiological Integrity/Pharmacological and
bleeding). Focus on the conditions that increase a chance for
Parenteral Therapies/Dosage Calculation
infection: long labor, long ago rupture of membranes, and an
internal conduit. 6. CORRECT ANSWER: 3. Answer 1 is incorrect because this
Content Area: Maternity, Postpartum; Integrated Process: is an appropriate dosage of misoprostol for induction of labor;
Nursing Process, Analysis; Cognitive Level: Application; no clarification is needed for this order. Answer 2 is incorrect
Client Need/Subneed: Physiological Integrity/Physiological because this client is already having very frequent uterine con-
Adaptation/Pathophysiology tractions and placing misoprostol is contraindicated for this
client. Answer 3 is correct because this client is having very
3. CORRECT ANSWER: 2. Answer 1 is incorrect because oral frequent uterine contractions and placing misoprostol is
hypoglycemic agents cross the placenta and can cause fetal anom-
contraindicated for this client. The physician should be
alies during the first trimester of pregnancy. Answer 2 is correct

ANSWERS
notified of the contraction pattern and withholding the
because insulin does not cross the placenta and is safe for use
misoprostol dose. Answer 4 is incorrect because macrosomia
in pregnancy. Answer 3 is incorrect because only gestational dia-
is an indication for induction of labor; however, this client is
betes can be treated with diet during pregnancy. Answer 4 is
not a candidate for placement of misoprostol due to the fre-
incorrect because only gestational diabetes can be treated with
quency of her uterine contractions.
exercise during pregnancy.
TEST-TAKING TIP: Consider that misoprostol cannot be
TEST-TAKING TIP: This client had diabetes before
removed once it is given vaginally and the contractions caused
pregnancy and is asking how to manage her blood sugar.
by the medication can result in uterine hyperstimulation. Focus
Eliminate the options that are only for controlling
on the two options that state: do not give (withhold).
gestational diabetes.
Content Area: Maternity, Intrapartum; Integrated Process:
Content Area: Maternity, Antepartum; Integrated Process:
Nursing Process, Implementation; Cognitive Level: Application;
Teaching and Learning; Cognitive Level: Application; Client
Client Need/Subneed: Physiological Integrity/Pharmacological
Need/Subneed: Physiological Integrity/Reduction of Risk
and Parenteral Therapies/Adverse Effects/Contraindications/
Potential/Potential for Complications from Surgical Procedures
Interactions
and Health Alterations
7. CORRECT ANSWER: 4. Answer 1 is incorrect because vig-
4. CORRECT ANSWER: 3. Answer 1 is incorrect because, orous perineal massage does not decrease pain during deliv-
according to Rubin, maternal role attachment develops over
ery; gentle perineal massage may be used by some practition-
about a 3- to 10-month period, so it is too early to make this
ers to decrease perineal tearing. Answer 2 is incorrect because
determination. Answer 2 is incorrect because attachment is an
the second stage of labor is the pushing stage and coaching
ongoing process that occurs gradually. Answer 3 is correct
with patterned breathing is used to decrease pain during the
because this client is demonstrating normal behavior for the
first stage of labor. Answer 3 is incorrect because pushing
taking-in period, which, according to Rubin, includes
should be performed during the peak of the contractions,
dependence and passivity that may last up to 3 days after
not between, to enhance progress toward delivery. Answer 4
delivery. Answer 4 is incorrect because a client experiencing
is correct because keeping the couple informed decreases
postpartum depression consistently demonstrates anxiety,
anxiety, increases confidence in their ability to cope with
confusion, or other signs and symptoms (i.e., feelings of worth-
an unmedicated labor and delivery, and helps accomplish
lessness or guilt, tearfulness, hopelessness and feeling empty
their goals.
inside, lack of joy in new baby).
TEST-TAKING TIP: Remember the stages of labor and that the
TEST-TAKING TIP: Consider the time that has passed since
correct coaching for a woman who is pushing (second stage of
delivery and what stage of adaptation the new mother is expe-
labor) is different than during the first stage of labor.
riencing. Three of the options focus on problems (e.g., doubting
Content Area: Maternity, Intrapartum; Integrated Process:
competency, attachment disorder, depression). Select the one
Nursing Process, Implementation; Cognitive Level: Application;
option that is normal.
Client Need/Subneed: Health Promotion and Maintenance/
Content Area: Maternity, Postpartum; Integrated Process:
Ante/Intra/Postpartum and Newborn Care
Nursing Process, Analysis; Cognitive Level: Application;
2164_Ch04_115-248 29/03/12 12:23 PM Page 230

230 chapter 4 Health Promotion and Maintenance

8. CORRECT ANSWER: 2. Answer 1 is incorrect because = painless. Both are placenta complications, but are very
auscultation should be performed to hear decelerations of different in manifestation of symptoms.
the fetal heart rate. Correct procedure includes listening for Content Area: Maternity, Antepartum; Integrated Process:
decelerations during a contraction and for 30 seconds after Nursing Process, Analysis; Cognitive Level: Comprehension;
the contraction, to make sure that there are no audible Client Need/Subneed: Physiological Integrity/Physiological
decelerations. Answer 2 is correct because this is the cor- Adaptation/Pathophysiology
rect procedure for fetal heart tone auscultation. Answer 3 11. CORRECT ANSWERS: 2, 3, 4, 5. Answer 1 is incorrect
is incorrect because this answer doesnt include listening because, although perineal hygiene can decrease the chance
during the contraction for decelerations; therefore, it is for infection, perineal hygiene after voiding will not be help-
incomplete. Answer 4 is incorrect because this answer ful in decreasing her perineal pain. Using the peri bottle
doesnt include listening after the contraction. Late decelera- during urination can decrease the pain caused by urination.
tions may not be heard. Answer 2 is correct because staying well hydrated by
TEST-TAKING TIP: The reason for obtaining fetal heart tones drinking plenty of water can be helpful in decreasing
is to ensure that the fetus is not having any fetal distress. pain caused by constipation. Straining with bowel move-
Eliminate the answers that do not provide complete assessment ments will stretch the episiotomy scar and perineum and
for fetal distress. can cause pain. Avoid constipation by eating fiber-rich
Content Area: Maternity, Intrapartum; Integrated Process:
foods such as fresh fruits and vegetables. Answer 3 is cor-
Nursing Process, Assessment; Cognitive Level: Application; rect because applying an ice pack is helpful in decreasing
Client Need/Subneed: Health Promotion and Maintenance/
perineal pain by decreasing perineal edema. Answer 4 is
Ante/Intra/Postpartum and Newborn Care correct because taking a sitz bath several times a day can
9. CORRECT ANSWER: 4. Answer 1 is incorrect because decrease the pain by increasing the circulation to the per-
breastfeeding is not a reliable method of birth control. Answer ineum. Answer 5 is correct because topical anesthetics, as
ANSWERS

2 is incorrect because ovulation may occur before the 6-week ordered by the physician, can decrease the perineal pain
postpartum checkup, and may lead to an unwanted pregnancy. by slowing down the rate at which pain sensors in the
Answer 3 is incorrect because, although a referral to a social skin send pain messages to the brain, and by reducing
worker may assist in providing information about available perineal swelling.
resources, it will not prevent an unwanted pregnancy. Answer 4 TEST-TAKING TIP: The key concept in this question is the
is correct because it is appropriate to discuss birth control treatment and prevention of perineal pain related to the deliv-
at the time when the client is asking questions and is recep- ery. Choose the interventions that decrease pain.
tive to teaching. Content Area: Maternity, Postpartum; Integrated Process:
TEST-TAKING TIP: The client is asking about birth control. Nursing Process, Implementation; Cognitive Level: Application;
Eliminate the answers where the nurse doesnt focus on birth Client Need/Subneed: Health Promotion and Maintenance/
control or that contain incorrect advice (Answer 1). Ante/Intra/Postpartum and Newborn Care
Content Area: Maternity, Postpartum; Integrated Process:
12. CORRECT ANSWERS: 2, 3, 4. Answer 1 is incorrect
Communication and Documentation; Cognitive Level: because relating personal experiences is not an appropriate
Application; Client Need/Subneed: Psychosocial Integrity/ method of teaching or reinforcing learning. Answer 2 is
Therapeutic Communications correct because providing written materials is a
10. CORRECT ANSWERS: 2, 3, 4, 5. Answer 1 is incorrect suitable method for reinforcing teaching. Answer 3 is
because painless vaginal bleeding is a symptom of placenta correct because demonstrating skills to the client using
previa, not abruptio placentae. Answer 2 is correct because her own neonate is an acceptable method that involves
a placental abruption causes tetanic uterine contractions the parent in learning skills using her own infant.
in response to the placenta detaching from the uterine Answer 4 is correct because showing videotapes to the
wall. Answer 3 is correct because premature rupture of parent is a common and appropriate method to demon-
membranes is associated with placental abruption. strate skills. Answer 5 is incorrect because telling her to
Answer 4 is correct because severe abdominal pain is a ask her family members for advice about infant care is
symptom of placental abruption; and uterine contrac- not an appropriate method of teaching neonatal care.
tions, in response to the placenta detaching from the Using standardized materials for instruction, providing
uterine wall, cause severe pain. Answer 5 is correct feedback about neonatal care and answering the clients
because a rigid, boardlike abdomen is a symptom of questions are appropriate methods. Simply telling the client
placental abruption; and uterine contractions, in to ask her family members does not provide instruction for
response to the placenta detaching from the uterine wall, the client.
cause severe pain. The uterus stays contracted, causing TEST-TAKING TIP: Appropriate teaching methods include
the rigid abdomen. visual information such as videotapes and written materials,
TEST-TAKING TIP: Note the difference between abruption the kinesthetic method using the infant, and auditory materials
(pain) and previa (painless). Four options = pain; one option such as a recording of discharge instructions.
2164_Ch04_115-248 29/03/12 12:23 PM Page 231

Answers/Rationales/Tips 231
Content Area:Maternity, Postpartum; Integrated Process: mask. The physician must also be notified immediately.
Teaching and Learning; Cognitive Level: Analysis; Client Answer 3 is incorrect because this pattern is variable deceler-
Need/Subneed: Health Promotion and Maintenance/Principles ations, caused by compression of the umbilical cord. The
of Teaching and Learning fetus demonstrates good oxygen reserve since the baseline
variability is normal and the fetal heart rate rapidly returns
13. CORRECT ANSWER: 2. Answer 1 is incorrect because this to baseline after the deceleration. Immediate intervention in
response could unnecessarily worry the client. Excessive perspi-
this type of pattern is not required, but continued close
ration may be a sign of infection, but there is no additional
observation of the pattern is needed. Answer 4 is incorrect
information in the stem that would lead to that possibility.
because this pattern is variable decelerations, caused by
Answer 2 is correct because this explanation is factually true.
compression of the umbilical cord. The fetus demonstrates
Fluids are retained during pregnancy and lost in this man-
good oxygen reserve since the baseline variability is normal
ner. This is called postpartum diuresis and is commonly
and the fetal heart rate rapidly returns to baseline after the
experienced about 24 to 48 hours after delivery. Answer 3 is
deceleration. Immediate intervention in this type of pattern
incorrect because speculating about the clients intake is not an
is not required, but continued close observation of the pat-
appropriate response to her concerns. Answer 4 is incorrect
tern is needed.
because it assumes that the client had IV fluids administered
TEST-TAKING TIP: To determine the type of deceleration,
during labor and does not address her concerns about having
draw a line from the lowest part of the deceleration down to
a problem.
the uterine contraction to establish the relationship as early,
TEST-TAKING TIP: Consider why the client is asking the
variable, or late between the deceleration and the uterine
questionshe is concerned about developing a problem;
contraction.
eliminate responses that do not address her concern.
Content Area: Maternity, Intrapartum; Integrated Process:
Content Area: Maternity, Postpartum; Integrated Process:
Nursing Process, Analysis; Cognitive Level: Analysis;
Communication and Documentation; Cognitive Level:

ANSWERS
Client Need/Subneed: Physiological Integrity/Physiological
Application; Client Need/Subneed: Health Promotion and
Adaptation/Medical Emergencies
Maintenance/Ante/Intra/Postpartum and Newborn Care
16. CORRECT ANSWER: 4. Answer 1 is incorrect because
14. CORRECT ANSWER: 1. Answer 1 is correct because
the left side, with the neck slightly flexed, will prevent
intense pressure at the peak of contractions may indicate
proper ventilation. Answer 2 is incorrect because the back,
the client is completely dilated and is close to delivery. A
with the head turned to the left side, will prevent proper
return to bed and a vaginal examination are needed to con-
ventilation. Answer 3 is incorrect because placing the
firm this possibility. Answer 2 is incorrect because warm
neonate on the abdomen, with the head down, will prevent
flushed skin is a common finding during labor and is not
ventilation and may cause suffocation. Answer 4 is correct
indicative of a need for reevaluation. Answer 3 is incorrect
because placing the neonate on the back, with the neck
because nausea during contractions is a common finding during
slightly extended, will open the airway and facilitate
labor and is not indicative of a need for reevaluation. Answer 4
ventilation.
is incorrect because contractions that are mild will probably
TEST-TAKING TIP: Picture the positions described and
not result in cervical change; therefore, this finding does not
eliminate the ones that prevent air exchange.
require a reevaluation.
Content Area: Child Health, Newborn; Integrated Process:
TEST-TAKING TIP: Consider the reason for reevaluating the
Nursing Process, Implementation; Cognitive Level: Application;
client. Eliminate responses that are common to all clients in
Client Need/Subneed: Physiological Integrity/Reduction of Risk
labor, such as skin flushing and nausea, and are not specific to
Potential/Potential for Complications of Diagnostic
this clients condition.
Tests/Treatments/Procedures
Content Area: Maternity, Intrapartum; Integrated Process:
Nursing Process, Assessment; Cognitive Level: Analysis; 17. CORRECT ANSWER: 2. Answer 1 is incorrect because
Client Need/Subneed: Physiological Integrity/Reduction of these are normal findings in a neonate. Answer 2 is correct
Risk Potential/Potential for Alterations in Body Systems because this subnormal temperature indicates prematurity,
infection, low environment temperature, inadequate
15. CORRECT ANSWER: 2. Answer 1 is incorrect because
clothing, and/or dehydration and requires an intervention.
the pattern here is early decelerations and is caused by
Answer 3 is incorrect because these are normal findings in a
compressions of the fetal head descending through the
neonate. Answer 4 is incorrect because these are normal
birth canal. No nursing intervention is needed. Answer 2 is
findings in a neonate.
correct because the pattern is late decelerations and is
TEST-TAKING TIP: Identify which answers are within normal
caused by uteroplacental insufficiency. The fetal heart
ranges for a neonate and eliminate these options.
rate is tachycardic, baseline heart rate of 180 bpm
Content Area: Child Health, Newborn; Integrated Process:
with decreased baseline variability. The nurse must
Nursing Process, Assessment; Cognitive Level: Application;
immediately attempt to correct the pattern by turning
Client Need/Subneed: Physiological Integrity/Reduction of Risk
the client on her side and giving a fluid bolus and oxygen
Potential/Potential for Alterations in Body Systems
2164_Ch04_115-248 29/03/12 12:23 PM Page 232

232 chapter 4 Health Promotion and Maintenance

18. CORRECT ANSWER: 4. Answer 1 is incorrect because, Content Area:Maternity, Intrapartum; Integrated Process:
when the fetal head is in the upper right quadrant, the body Nursing Process, Analysis; Cognitive Level: Analysis; Client
will be on the right side with the fetal hands and feet on the Need/Subneed: Physiological Integrity/Pharmacological and
left side. The heart tones would not be audible on the left Parenteral Therapies/Dosage Calculation
side. Answer 2 is incorrect because the fetus is in a breech 21. CORRECT ANSWER: 3. Answer 1 is incorrect
position, so the heart tones would be upper quadrant on the because the primary health-care provider should still be
right side, not the left side, because the hands and feet are present until after the delivery of the placenta. Answer 2 is
on the left side. Answer 3 is incorrect because the fetal head incorrect because the primary health-care provider may
is in the upper right quadrant; the body will be on the right sometimes apply gentle tension to the cord to facilitate
side with the heart tones in the upper quadrant above the delivery of the placenta, but this is not the role of the RN;
umbilicus, not below the umbilicus, which would be a and traction on the cord will increase the risk of complica-
vertex position. Answer 4 is correct because the fetal head tion, such as uterine inversion. Answer 3 is correct
is in the upper right quadrant, the body will be on the because the client has not delivered the placenta within
right side with the heart tones in the upper quadrant 40 minutes; the physician will explore the uterus and
above the umbilicus. may determine that the client has a placenta accreta,
TEST-TAKING TIP: Picture the position of the fetus and the which would require surgical intervention. Answer 4 is
heart on the side where the head is palpable. incorrect because the normal time frame for delivery of the
Content Area: Maternity, Intrapartum; Integrated Process:
placenta is 5 to 30 minutes. Most placentas deliver within
Nursing Process, Assessment; Cognitive Level: Application; 5 minutes, but 40 minutes is longer than the expected time
Client Need/Subneed: Health Promotion and Maintenance/
frame for this stage.
Ante/Intra/Postpartum and Newborn Care TEST-TAKING TIP: Know the stages and phases of labor and
19. CORRECT ANSWER: 4. Answer 1 is incorrect because, the expected time frame for each of these stages.
ANSWERS

in most cases, a woman with gestational diabetes will not Content Area: Maternity, Intrapartum; Integrated Process:
become a diabetic who is insulin dependent, but rather has Nursing Process, Implementation; Cognitive Level: Application;
a high chance of developing type 2 diabetes in the future. Client Need/Subneed: Physiological Integrity/Physiological
Answer 2 is incorrect because, even with following a diabetic Adaptation/Alterations in Body Systems
diet, there is still a high chance of developing type 2 diabetes 22. CORRECT ANSWER: 4. Answer 1 is incorrect because
in the future. Answer 3 is incorrect because, even if there are the clients symptoms are related to vena cava syndrome and
no subsequent pregnancies, there is still a high chance of are a result of low blood pressure. Assisting her to her side
developing type 2 diabetes in the future. Answer 4 is correct will likely resolve the problem. Answer 2 is incorrect because
because there is about a 75% chance that a woman with nasal oxygen is not needed and the flow rate is far too high for
gestational diabetes will develop type 2 diabetes in the a nasal cannula. Assisting the mother to a lateral Sims posi-
future. tion will improve her blood pressure and reduce her symp-
TEST-TAKING TIP: Eliminate the answer that gives false reas- toms. Answer 3 is incorrect because this position is indicated
surance. Telling a client she has nothing to worry about is not for a client in shock. This client should be positioned on her
an appropriate response. side to alleviate symptoms. Answer 4 is correct because
Content Area: Maternity, Antepartum; Integrated Process:
changing to a left lateral position helps to alleviate vena
Communication and Documentation; Cognitive Level: cava compression from the gravid uterus, and will reduce
Application; Client Need/Subneed: Physiological Integrity/ her discomfort.
Reduction of Risk Potential/Potential for Alterations in Body TEST-TAKING TIP: Think about the underlying reason for
Systems this clients symptoms. Focus on the options that relate to posi-
20. CORRECT ANSWER: 2. The answer is calculated as tioning, and eliminate the option that is used for shock.
follows: Content Area: Maternity, Antepartum; Integrated Process:
Nursing Process, Implementation; Cognitive Level: Analysis;
15 units/250 mL or 60 units/1000 mL = 0.06 units/mL Client Need/Subneed: Physiological Integrity/Physiological
(concentration of the drug) Adaptation/Hemodynamics
0.06 units/hr (60 minutes = 0.001 units/min)
0.001 units/min 1000 (for conversion to 23. CORRECT ANSWER: 2. Answer 1 is incorrect because
milliunits) = 1 milliunit/min 2 = 2 the fetal heart tones would be heard below the umbilicus on
milliunits/min infusion rate the right side, not on the left side, which would then be
LOA position. Answer 2 is correct because the fetal head
TEST-TAKING TIP: Pay attention to the concentration of and back are pressing against the right side of the clients
the oxytocin and the final calculation. The concentration is abdomen; the fetal heart tones would be heard below the
15 units/250 mL, but the conversion must be done to milliu- umbilicus on the right side. Answer 3 is incorrect because
nits per minute or the answer will be incorrect. the fetal heart tones would be heard above the umbilicus in
2164_Ch04_115-248 29/03/12 12:23 PM Page 233

Answers/Rationales/Tips 233
breech presentation. Answer 4 is incorrect because the fetal severe PIH can decrease perfusion to the placenta and the
heart tones would be heard above the umbilicus in breech supine position can also decrease perfusion, leading to fetal dis-
presentation. tress. Answer 3 is incorrect because, although the management
TEST-TAKING TIP: Picture the situation described. It may be of a client with severe PIH includes reducing stimulation to
helpful for you to draw this out so that you can imagine where decrease her chance of developing seizures, it should not
the heartbeat would be found. include limiting nursing visits. The client requires frequent
Content Area: Maternity, Antepartum; Integrated Process: observation by a nurse. Answer 4 is correct because modified
Nursing Process, Analysis; Cognitive Level: Application; bedrest in a lateral position can decrease the clients blood
Client Need/Subneed: Health Promotion and Maintenance/ pressure and increase the placental perfusion.
Ante/Intra/Postpartum and Newborn Care TEST-TAKING TIP: Read options carefully. Both parts of the
answer must be true to have the answer be the correct option.
24. CORRECT ANSWER: 1. Answer 1 is correct because
Content Area: Maternity, Antepartum; Integrated Process:
the newborn most likely has thrush (oral candidiasis):
Nursing Process, Implementation; Cognitive Level: Application;
white plaque on the oral mucous membranes, gums, or
Client Need/Subneed: Physiological Integrity/Physiological
tongue. Treatment usually includes good hand washing
Adaptation/Alterations in Body Systems
and nystatin (Mycostatin). Answer 2 is incorrect because
thrush in newborns is caused by poor hand washing or expo- 27. CORRECT ANSWER: 1. Answer 1 is correct because
sure to Candida during vaginal birth. Changing formula is the cause of clubfoot isnt known, but it is not related
unnecessary and will not cure the thrush. Answer 3 is incor- to anything a woman did during pregnancy to cause the
rect because thrush in newborns is not caused by bottle malformation. Answer 2 is incorrect because clubfoot isnt
propping. Tooth decay is related to sleeping with a bottle. hereditary, although it can occur with other congenital
Answer 4 is incorrect because thrush in newborns is caused spinal malformations. Answer 3 is incorrect because evidence
by poor hand washing or exposure to Candida during shows that clubfoot isnt caused by the position of the fetus

ANSWERS
vaginal birth, not glucose. in the uterus. Answer 4 is incorrect because the cause of
TEST-TAKING TIP: Consider the outcome for each option; clubfoot is unknown and not due to a problem during
will the intervention solve the issue? If not, eliminate the embryo development.
answer. TEST-TAKING TIP: The client is concerned about exposure
Content Area: Child Health, Newborn; Integrated Process: to something during pregnancy that resulted in this condition.
Nursing Process, Implementation; Cognitive Level: Analysis; Basically, two of the choices are the same, an anomaly during
Client Need/Subneed: Physiological Integrity/Physiological development and a heredity condition; therefore, eliminate
Adaptation/Alterations in Body Systems these options.
Content Area: Child Health, Newborn; Integrated Process:
25. CORRECT ANSWER: 4. Answer 1 is incorrect because
Nursing Process, Implementation; Cognitive Level: Application;
the femur on the affected side is shortened, not lengthened.
Client Need/Subneed: Physiological Integrity/Physiological
Answer 2 is incorrect because deformities of the foot and
Adaptation/Pathophysiology
ankle are not symptoms of congenital hip dislocation.
Answer 3 is incorrect because plantar flexion of the foot is 28. CORRECT ANSWER: 3. Answer 1 is incorrect because
seen with clubfoot, and is not a symptom of congenital hip the gluteus maximus is never an appropriate injection
dislocation. Answer 4 is correct because asymmetry of the site for a newborn because nerve damage can occur when
gluteal and thigh folds is caused by restricted movement performing an injection in this site. Answer 2 is incorrect
on the affected side, which is a symptom of congenital because the gluteus minimus is never an appropriate
hip dislocation. injection site for a newborn because nerve damage can
TEST-TAKING TIP: Form a mental image of the deformity occur when performing an injection in this site. Answer 3
and read the options carefully. Note the anatomical is correct because the vastus lateralis is the proper injec-
proximity between hip in the stem and gluteal and thigh. tion site; it is a large muscle and avoids the nerve tracts.
Eliminate the other three options that refer to the lower Answer 4 is incorrect because the vastus medialis is not an
extremity: . . . limb (Answer 1); . . . foot and ankle appropriate injection site for a newborn because nerve
(Answer 2); . . . foot (Answer 3). damage can occur when performing an injection in
Content Area: Child Health, Newborn; Integrated Process: this site.
Nursing Process, Assessment; Cognitive Level: Application; TEST-TAKING TIP: Eliminate the gluteal muscles; they are
Client Need/Subneed: Physiological Integrity/Reduction of never a correct injection site for an infant. Then picture the
Risk Potential/System Specific Assessments infants thigh and the muscles that make up the thigh to choose
the correct injection site.
26. CORRECT ANSWER: 4. Answer 1 is incorrect because the Content Area: Child Health, Newborn; Integrated Process:
management of a client with severe pregnancy-induced hyper-
Nursing Process, Implementation; Cognitive Level: Application;
tension (PIH) includes reducing stimulation to decrease her
Client Need/Subneed: Physiological Integrity/Pharmacological
chance of developing seizures. Answer 2 is incorrect because
and Parenteral Therapies/Medication Administration
2164_Ch04_115-248 29/03/12 12:23 PM Page 234

234 chapter 4 Health Promotion and Maintenance

29. CORRECT ANSWERS: 3, 5. Answer 1 is incorrect because TEST-TAKING TIP: The goal of the question is to directly
vernix is usually absorbed in neonates who are past their esti- assist the client in coping with increased pressure. Look for the
mated date of delivery. Answer 2 is incorrect because plethora intervention that will provide direct support.
is an indication of a high hematocrit and is not a normal find- Content Area: Maternity, Intrapartum; Integrated Process:
ing in a neonate of any gestation. Answer 3 is correct because Nursing Process, Implementation; Cognitive Level: Application;
milia are clogged sebaceous glands usually found on the Client Need/Subneed: Health Promotion and Maintenance/
nose and are a normal finding in all gestational ages. Ante/Intra/Postpartum and Newborn Care
Answer 4 is incorrect because crepitus is the crunching, crack-
32. CORRECT ANSWER: 3. Answer 1 is incorrect because this
ling, or popping sounds and sensations that are felt under the client in labor is a low risk with a reassuring fetal heart rate
skin when palpating the clavicles in a neonate who is suspected tracing. Five minutes is too frequent for a client who is a low
of having a fractured clavicle. Answer 5 is correct because risk. Answer 2 is incorrect because this client in labor is a low
parchment skin is a common finding in postdates neonates. risk with a reassuring fetal heart rate tracing. Fifteen minutes is
The neonate who is postdates absorbs the vernix, which too frequent for a low-risk client. It is an appropriate interval
protects the skin. When the vernix is absorbed, the skin for a client who is high risk. Answer 3 is correct because
dries out and begins to peel off in large sheets that resemble reevaluating the fetal heart rate tracing every 30 minutes is
parchment. the recommended interval for a client who is low risk.
TEST-TAKING TIP: The question is focusing on evaluation of Answer 4 is incorrect because 60 minutes is too long an inter-
a neonate who has passed the estimated date of delivery. Think val for a client in active labor.
about what a neonate who is postdates looks like: dry, peel- TEST-TAKING TIP: Two key concepts guide the interval of
ing, and wrinkled. evaluating the fetal heart rate tracing: the risk factors (in
Content Area: Child Health, Newborn; Integrated Process:
this case, none) and the phase of labor (in this case, active
Nursing Process, Assessment; Cognitive Level: Application; labor).
ANSWERS

Client Need/Subneed: Health Promotion and Maintenance/


Content Area: Maternity, Intrapartum; Integrated Process:
Ante/Intra/Postpartum and Newborn Care Nursing Process, Implementation; Cognitive Level: Application;
30. CORRECT ANSWER: 2. Answer 1 is incorrect because Client Need/Subneed: Health Promotion and Maintenance/
deep tendon reflexes are a part of every labor assessment and Ante/Intra/Postpartum and Newborn Care
are not specific to rupture of membranes. Answer 2 is correct 33. CORRECT ANSWER: 1. Answer 1 is correct because
because the fern test is done to check for ruptured mem- studies have shown that intermittent monitoring is a
branes. It is positive if the amniotic sac has broken. safe and effective method for monitoring fetal well-being
Answer 3 is incorrect because a blood pressure check is a
for a low-risk pregnancy. This client fits the criteria
part of every labor assessment and is not specific to rupture of for intermittent monitoring. In addition, the use of
membranes. Answer 4 is incorrect because a urine test for intermittent monitoring may depend on the hospitals
protein is a part of every labor assessment and is not specific policy and procedure. Answer 2 is incorrect because the
to rupture of membranes. client has the right to refuse any medical procedure and it
TEST-TAKING TIP: When you see assessment or is inappropriate to tell clients there are not enough staff
evaluation in the stem of the question, think about members to care for them during labor. Answer 3 is incorrect
an assessment specific to this case and not applicable in because changing assignments does not deal with the under-
general. lying problemthe clients desire to avoid electronic fetal
Content Area: Maternity, Intrapartum; Integrated Process:
monitoring. Answer 4 is incorrect because, although the
Nursing Process, Implementation; Cognitive Level: Application; nurse is acknowledging the clients decision-making rights,
Client Need/Subneed: Physiological Integrity/Reduction of Risk
continuous EFM is not the only effective way to monitor
Potential/System Specific Assessments infant well-being during labor; and the statement could be
31. CORRECT ANSWER: 2. Answer 1 is incorrect perceived by the client as manipulating her into accepting
because pushing before the cervix is completely dilated the electronic fetal monitoring.
can result in cervical tears. Answer 2 is correct because TEST-TAKING TIP: Watch out for the word only. The choice
assisting the client to pant-blow at the peak of contrac- with that absolute word is usually wrong.
tions will help her to avoid pushing before her cervix Content Area: Maternity, Antepartum; Integrated Process:
is completely dilated. Answer 3 is incorrect because Communication and Documentation; Cognitive Level:
readjusting the fetal monitor will not help the client avoid Application; Client Need/Subneed: Health Promotion and
pushing and damaging her cervix. Answer 4 is incorrect Maintenance/Ante/Intra/Postpartum and Newborn Care
because telling the coach that it is not good for the baby 34. CORRECT ANSWER: 4. Answer 1 is incorrect because this
if she pushes right now is not directly helping the client is a normal baseline heart rate, with a benign pattern of head
avoid pushing and it is unnecessarily trying to scare her compressions; no intervention is needed for this pattern.
into compliance. The baby will not be affected by pushing Answer 2 is incorrect because this pattern may be indicative
too early.
2164_Ch04_115-248 29/03/12 12:23 PM Page 235

Answers/Rationales/Tips 235
of hypoxia, maternal drug administration, or fetal sleep. An Content Area: Maternity, Intrapartum; Integrated Process:
amnioinfusion will not relieve this pattern. Answer 3 is incor- Nursing Process, Implementation; Cognitive Level: Analysis;
rect because this is a normal baseline heart rate, with mild Client Need/Subneed: Health Promotion and Maintenance/
variable decelerations. In the presence of normal baseline vari- Ante/Intra/Postpartum and Newborn Care
ability, an amnioinfusion is not needed to relieve mild variable
37. CORRECT ANSWER: 4. Answer 1 is incorrect because
decelerations. Answer 4 is correct because this pattern has
this response does validate her feelings, but it promises the
the potential for prolonged decelerations and fetal distress.
client something that may not be true; she may have a while
An amnioinfusion will help cushion the umbilical cord
before delivery. Answer 2 is incorrect because calling the
compression and relieve or decrease the severity of the
doctor and passing the buck to him or her does nothing
variable decelerations.
for the clients pain and coping. Answer 3 is incorrect
TEST-TAKING TIP: Try drawing the pattern out from the
because this response promises the client something that
description on a piece of paper. Seeing the pattern may help you
may not be true (she may need a cesarean section to deliver),
focus on the physiology of the pattern and the interventions
and it does nothing to validate her feelings. Answer 4 is
needed. Focus on the lowest heart rate and the word deep
correct because it validates the clients feelings, gives an
variable decelerations.
explanation for her condition, and does not promise
Content Area: Maternity, Antepartum; Integrated Process:
anything to the client.
Nursing Process, Planning; Cognitive Level: Application;
TEST-TAKING TIP: Validating a clients feelings is important
Client Need/Subneed: Physiological Integrity/Reduction of
to let her know that you hear her and understand her.
Risk Potential/Vital Signs Changes/Abnormalities
Eliminate the choices that do not validate the clients feelings.
35. CORRECT ANSWER: 4. Answer 1 is incorrect because the Content Area: Maternity, Intrapartum; Integrated Process:
active phase of the first stage of labor usually is associated Communication and Documentation; Cognitive Level:
with behavior that is self-focused and requires concentration Application; Client Need/Subneed: Health Promotion and

ANSWERS
to breathe with the contractions. This client is excited and Maintenance/Ante/Intra/Postpartum and Newborn Care
talkative, suggesting she is in early labor or the latent phase.
38. CORRECT ANSWER: 1. Answer 1 is correct because this
Answer 2 is incorrect because there is no first phase of labor; it
client is at 32 weeks gestation, multipara, and 3 cm dilat-
is the first stage of labor. Answer 3 is incorrect because there
ed. The contractions are the most important assessment in
is no first phase of labor; it is the first stage of labor. Answer 4
a client who is in preterm labor. If the contractions are
is correct because this client is excited and talkative, sug-
close together, tocolytics will be required to decrease the
gesting she is in early labor or the latent phase of the first
contractions. If there are rare or no contractions, the
stage of labor.
nurse can focus on other assessments. Answer 2 is incorrect
TEST-TAKING TIP: Remember stages and phases of labor.
because the client is preterm and her contractions are the
There are four stages of labor, with the first stage having three
most important assessment for this client. If her contractions
phases of labor: latent, active and transition.
are regular or close together, tocolytics will be required to
Content Area: Maternity, Intrapartum; Integrated Process:
decrease the contractions. The patency of the IV infusion is
Nursing Process, Analysis; Cognitive Level: Analysis;
not the most important assessment in a client with preterm
Client Need/Subneed: Health Promotion and Maintenance/
labor. Answer 3 is incorrect because the client is preterm and
Ante/Intra/Postpartum and Newborn Care
her contractions are the most important assessment. The fetal
36. CORRECT ANSWER: 1. Answer 1 is correct because the heart rate is an important assessment for all clients in labor,
client is coping well with her labor and, with the partners but the most important assessment for this woman who is
support, is using prepared childbirth techniques. Offering preterm with a cervix that is dilated and effaced is the fre-
clear liquids would be an appropriate nursing action dur- quency and duration of the contractions. If the client is hav-
ing the active phase of labor. Answer 2 is incorrect because ing regular contractions, tocolytics may be required to stop
offering her an epidural when she is coping well is not an her uterine contractions. Answer 4 is incorrect because the
appropriate nursing action. If the client asked for pain relief, maternal vital signs are a common part of the assessment for
then offering an epidural would then be an appropriate all obstetric clients, but this client is preterm and her con-
option. Answer 3 is incorrect because offering her pain med- tractions are the most important assessment. If the client is
ication when she is coping well is not an appropriate nursing having regular contractions, tocolytics may need to be
action. If the client asked for pain relief, then offering pain ordered to stop the uterine contractions.
medication would be an appropriate option. Answer 4 is TEST-TAKING TIP: Consider the gestational age, and which
incorrect because pant-blow breathing is a technique used assessment is the most important to perform at the beginning of
during transition phase of labor; this client is in the active the shift for a client who is in preterm labor.
phase of labor. Content Area: Maternity, Intrapartum; Integrated Process:
TEST-TAKING TIP: Two of the alternatives are essentially the Nursing Process, Assessment; Cognitive Level: Analysis;
same: offering an epidural and offering pain medication; there- Client Need/Subneed: Physiological Integrity/Reduction of
fore, eliminate both of those choices. Risk Potential/System Specific Assessments
2164_Ch04_115-248 29/03/12 12:23 PM Page 236

236 chapter 4 Health Promotion and Maintenance

39. CORRECT ANSWERS: 2, 4. Answer 1 is incorrect walking may be a sign of thrombophlebitis, and the
because slight acrocyanosis, a bluish color of the extremities, physician should be notified. Answer 3 is correct because
is a benign, normal part of neonatal transition and is foul-smelling lochia is a symptom of endometritis, and
common during the first 24 to 48 hours after birth. It the physician should be notified. Answer 4 is incorrect
does not require any intervention, and does not need to because lochia serosa that is soaking less than a pad every
be reported to the physician. Answer 2 is correct because 3 hours is normal for a client who is postpartum, and does
the normal respiratory rate is 30 to 60 breaths/min. A not need to be reported to the physician. Answer 5 is cor-
rate of 20 is abnormal after birth and may indicate rect because soaking more than a pad an hour may be a
apnea and respiratory distress, and should be reported sign of retained placental fragments, and the physician
to the physician. Answer 3 is incorrect because a heart should be notified.
rate of 145 is in the normal range of 110 to 160, and does TEST-TAKING TIP: Reportable conditions are: pain,
not need to be reported to the physician because it is foul-smelling, and soaking more . . . .
within the normal limits for this infant. Answer 4 is correct Content Area: Maternity, Postpartum; Integrated Process:
because a pilonidal dimple may be a sign of spina bifida Nursing Process, Implementation; Cognitive Level: Analysis;
occulta, and requires further evaluation by a physician. Client Need/Subneed: Physiological Integrity/Reduction of Risk
Answer 5 is incorrect because ecchymosis (bruising) over Potential/Potential for Alterations in Body Systems
the back of the fetal head is a normal finding related to 42. CORRECT ANSWER: 1. Answer 1 is correct because
the pressure of the presenting part on the fetal head this client is experiencing a threatened abortion. At
resulting in bruising, and doesnt need to be reported to this gestation, there is no medical intervention to
the physician. prevent miscarriage. Bedrest is the expected management.
TEST-TAKING TIP: Do not be distracted by the word Answer 2 is incorrect because this client is experiencing a
consistent, which in this case means consistently threatened abortion; a dilation and curettage would be
ANSWERS

normal. expected for a client who had passed some tissue, had a
Content Area: Child Health, Newborn; Integrated Process:
dilated cervix, and had an incomplete abortion. Answer 3 is
Nursing Process, Assessment; Cognitive Level: Application; incorrect because this client is experiencing a threatened
Client Need/Subneed: Physiological Integrity/Reduction of
abortion; notifying a grief counselor to assist the woman
Risk Potential/Potential for Alterations in Body Systems with the imminent loss of her fetus is unnecessary. Answer 4
40. CORRECT ANSWERS: 2, 3, 4. Answer 1 is incorrect is incorrect because this client is experiencing a threatened
because nifedipine is a calcium channel blocker that is used to abortion; a cerclage is usually done after the first trimester for
treat preterm labor, not to induce labor. Answer 2 is correct an incompetent cervix.
because cervidil is a prostaglandin E2 vaginal insert used TEST-TAKING TIP: To remember the different types of abor-
to help ripen the cervix for induction of labor. Answer 3 tions, the cervix and the passage of tissue are the key. If the
is correct because prostaglandin gel is a ripening agent cervix is not dilated and no tissue is passed, it is a threatened
used to help ripen the cervix for induction of labor. abortion. If the cervix is dilated and tissue is passed, then
Answer 4 is correct because misoprostol is a prostaglandin imminent abortion is likely to occur.
E1 analog used to help ripen the cervix for induction of Content Area: Maternity, Antepartum; Integrated Process:
labor. Answer 5 is incorrect because Brethine (terbutaline) Nursing Process, Implementation; Cognitive Level: Analysis;
is an asthma drug, which is a beta-adrenergic that relaxes Client Need/Subneed: Physiological Integrity/Reduction of
the smooth muscles of the uterus. It is not a drug used to Risk Potential/Potential for Alterations in Body Systems
induce labor. 43. CORRECT ANSWER: 1. Answer 1 is correct because
TEST-TAKING TIP: The key information in the stem is that glucosuria is most often secondary to the pregnancy-
the client has an unripe cervix (closed, thick, and fetal head out related increased glomerular filtration of glucose without
of the pelvis). Select drugs that ripen the cervix for induction of an increased tubular reabsorption. Answer 2 is incorrect
labor. Eliminate the calcium channel blocker and the beta- because the finding of glucose in the urine of a woman
adrenergic drugs. who is pregnant does not mean that she has gestational
Content Area: Maternity, Intrapartum; Integrated Process:
diabetes. Answer 3 is incorrect because, although glucosuria
Nursing Process, Analysis; Cognitive Level: Analysis; Client is an indication to perform glucose testing, the initial test
Need/Subneed: Physiological Integrity/Pharmacological and
would be a 1-hour screening test and not the 3-hour test,
Parenteral Therapies/Expected Effects/Outcomes which is only done if the 1-hour glucose tolerance test is
41. CORRECT ANSWERS: 1, 2, 3, 5. Answer 1 is correct abnormal. Answer 4 is incorrect because small doses of
because pain that is not well controlled by pain medica- insulin would be unnecessary in the absence of a diagnosis
tion can be indicative of a postpartum complication, of diabetes.
such as endometritis, and the physician should be noti- TEST-TAKING TIP: Glucosuria is a screening test that may
fied. Answer 2 is correct because pain in her calves while indicate the presence of gestational diabetes, but it is not
2164_Ch04_115-248 29/03/12 12:23 PM Page 237

Answers/Rationales/Tips 237
diagnostic for gestational diabetes. Eliminate the answers more nutrients can be absorbed. Answer 3 is incorrect
that indicate glucosuria is a diagnosis for gestational because compression of the intestines during pregnancy
diabetes. does not contribute to constipation. Answer 4 is incorrect
Content Area: Maternity, Antepartum; Integrated Process: because, during pregnancy, there is a decrease, not an
Nursing Process, Analysis; Cognitive Level: Application; increase, in intestinal motility.
Client Need/Subneed: Health Promotion and Maintenance/ TEST-TAKING TIP: When there are similar answers, read the
Ante/Intra/Postpartum and Newborn Care answer carefully to determine the differences.
Content Area: Maternity, Antepartum; Integrated Process:
44. CORRECT ANSWER: 3. Answer 1 is incorrect
Nursing Process, Implementation; Cognitive Level: Application;
because, since the screening 1-hour glucose tolerance test
Client Need/Subneed: Physiological Integrity/Basic Care and
was abnormal, the 3-hour glucose tolerance test follow-up
Comfort/Elimination
is needed. Answer 2 is incorrect because, even though the
1-hour test result was abnormal, a diagnosis of gestational 47. CORRECT ANSWER: 1. Answer 1 is correct
diabetes cannot be made on that value alone; therefore, because, when using Naegeles rule, this clients EDD is
there is no immediate need for dietary control. Answer 3 October 8th. This makes the client 41 weeks and 3 days
is correct because the abnormal 1-hour test results gestation on admission. This is the most important factor
should be followed up by a 3-hour glucose tolerance in planning her nursing care since this is a postdates
test. Answer 4 is incorrect because, even though the pregnancy. Answer 2 is incorrect because blood type A
1-hour test result was abnormal, a diagnosis of gestational with Rh+ does not present any problems in client care; but,
diabetes cannot be made on that value alone; therefore, maternal blood type O and/or Rh blood types can cause
there is no immediate need for postprandial blood sugar severe jaundice in the newborn due to maternal antibodies
monitoring. that destroy fetal RBCs. Answer 3 is incorrect because
TEST-TAKING TIP: The 1-hour glucose tolerance test is a hemoglobin of 11 gm/dL is physiological anemia, caused

ANSWERS
screening test for gestational diabetes; the 3-hour glucose by the increase in plasma volume in pregnancy. It is not a
tolerance test is the diagnostic test for gestational diabetes. true anemia. Answer 4 is incorrect because this is a normal
Eliminate the answers that are interventions for a diagnosis blood pressure in pregnancy. The maternal blood pressure
of gestational diabetes. normally decreases by 5 to 10 mm Hg in both systolic and
Content Area: Maternity, Antepartum; Integrated Process: diastolic pressures. This decrease is due to peripheral vasodi-
Nursing Process, Planning; Cognitive Level: Application; lation caused by pregnancy hormones.
Client Need/Subneed: Physiological Integrity/Reduction of TEST-TAKING TIP: Think about which of the answers repre-
Risk Potential/Potential for Alterations in Body Systems sents a risk factor for the pregnancy. Does that answer corre-
spond to a risk factor or a normal value for pregnancy? Note
45. CORRECT ANSWER: 4. Answer 1 is incorrect because
that three options focus on blood. Choose the option that is
laxatives stimulate the intestinal tract, but also can initiate
different.
uterine contractions, which can be a cause of preterm labor
Content Area: Maternity, Intrapartum; Integrated Process:
contractions. Answer 2 is incorrect because high-fat and
Nursing Process, Assessment; Cognitive Level: Analysis;
high-protein foods contribute to constipation. Answer 3 is
Client Need/Subneed: Health Promotion and Maintenance/
incorrect because caffeine can cause tachycardia in the fetus
Ante/Intra/Postpartum and Newborn Care
and its use during pregnancy is discouraged. Answer 4 is
correct because pregnancy slows intestinal motility due to 48. CORRECT ANSWER: 2. Answer 1 is incorrect because the
the influence of progesterone. Increasing fluid intake and suggested weight gain for pregnancy is between 25 and 35 lb.
exercise stimulates peristalsis and can relieve the discom- Answer 2 is correct because strange cravings called pica are
fort of constipation. not a normal part of pregnancy and may be associated
TEST-TAKING TIP: Exclude interventions that involve drugs with iron deficiency. Answer 3 is incorrect because about
(laxative, caffeine) to relieve the common complaint of consti- 300 calories is the additional calories needed in pregnancy.
pation. Suggesting natural measures for common pregnancy Answer 4 is incorrect because canned foods lose some nutri-
complaints are safer in pregnancy. ents in processing. Foods that are frozen have less processing
Content Area: Maternity, Antepartum; Integrated Process: and may be more nutritious.
Teaching and Learning; Cognitive Level: Application; TEST-TAKING TIP: Be sure to read the question carefully. By
Client Need/Subneed: Physiological Integrity/Basic Care asking which response requires additional teaching, the ques-
and Comfort/Elimination tion is asking for an incorrect response.
Content Area: Maternity, Antepartum; Integrated Process:
46. CORRECT ANSWER: 2. Answer 1 is incorrect because
Nursing Process, Evaluation; Cognitive Level: Analysis;
the intestines do not increase in diameter due to pregnancy.
Client Need/Subneed: Physiological Integrity/Basic Care
Answer 2 is correct because, during pregnancy, proges-
and Comfort/Nutrition and Oral Hydration
terone causes peristalsis to slow the intestines so that
2164_Ch04_115-248 29/03/12 12:23 PM Page 238

238 chapter 4 Health Promotion and Maintenance

49. CORRECT ANSWER: 1. Answer 1 is correct because that may be detected by the multiple marker screen.
this client is in the latent phase of labor; their plan is to Answer 5 is correct because neural tube defects are
use prepared childbirth techniques. Slow, deep breathing conditions that may be detected by the multiple marker
and relaxation techniques are used to relieve contraction screen.
pain during this phase. Answer 2 is incorrect because mini- TEST-TAKING TIP: Select the three options that are congenital
mizing the use of narcotics is preferred when prepared child- (Answers 2, 4, and 5). Multiple marker screen is usually per-
birth techniques are used. Answer 3 is incorrect because local formed about 16 weeks gestation; gestational diabetes and
anesthesia is used for numbing the perineum immediately pregnancy-induced hypertension are conditions that are usually
before delivery of the fetus during the second stage of labor. found during the third trimester of pregnancy.
Answer 4 is incorrect because rapid, shallow breathing can Content Area: Maternity, Antepartum; Integrated Process:
lead to hyperventilation and is inappropriate for any phase Nursing Process, Analysis; Cognitive Level: Application;
of labor. Client Need/Subneed: Health Promotion and Maintenance/
TEST-TAKING TIP: Eliminate the two options that focus on Ante/Intra/Postpartum and Newborn Care
drugs, which would be the opposite of the couples desire to use 52. CORRECT ANSWER: 4. Answer 1 is incorrect because
prepared childbirth techniques. bilirubin levels in excess of 12 mg/100 mL at 24 hours
Content Area: Maternity, Intrapartum; Integrated Process:
of age may indicate the presence of a pathological process.
Nursing Process, Planning; Cognitive Level: Application; This clients jaundice is most likely due to an ABO incom-
Client Need/Subneed: Health Promotion and Maintenance/
patibility. Answer 2 is incorrect because breastfeeding
Ante/Intra/Postpartum and Newborn Care jaundice occurs around the third day of age. Encouraging
50. CORRECT ANSWER: 3. Answer 1 is incorrect because early and frequent feedings at the breast lowers neonatal
the heart rate of the fetus is unrelated to the gender of the bilirubin levels. This client developed high serum bilirubin
fetus. The range for the fetal heart rate is 110 to 160 bpm at 24 hours of age. Answer 3 is incorrect because bililight
ANSWERS

regardless of gender. Answer 2 is incorrect because the moth- therapy requires an order from the physician. Exchange
er has two X chromosomes; the father has both X and Y transfusions for ABO incompatibilities are seldom necessary.
chromosomes, and his sperm determines the sex of the fetus. Answer 4 is correct because early and frequent breastfeed-
Answer 3 is correct because meiosis results in the X and Y ing tends to lower serum bilirubin levels and avoids
chromosomes of the male splitting so that each sperm dehydration that increases the serum bilirubin level.
carries either an X or a Y chromosome, thus determining TEST-TAKING TIP: The risk factor for this client is the ABO
the gender of the fetus. Answer 4 is incorrect because how incompatibility. Eliminate the answer that refers to the level
the mother carries the fetus is unrelated to the gender. How being normal, or the answer that deals with jaundice related to
a fetus is carried is related to maternal uterine and abdomi- breastfeeding.
nal muscle tone. The gender of the fetus does not determine Content Area: Child Health, Newborn; Integrated Process:
how high or low it is carried. Nursing Process, Implementation; Cognitive Level: Analysis;
TEST-TAKING TIP: Eliminate the options that are myths Client Need/Subneed: Physiological Integrity/Physiological
about pregnancy. Choose the option that is scientifically Adaptation/Alterations in Body Systems
based. 53. CORRECT ANSWER: 4. Answer 1 is incorrect because
Content Area: Maternity, Antepartum; Integrated Process:
the fetus usually turns to vertex by term. If the fetus remains
Teaching and Learning; Cognitive Level: Application; in a breech position, a cesarean delivery is the recommended
Client Need/Subneed: Health Promotion and Maintenance/
method of delivery. Answer 2 is incorrect because the uterus
Ante/Intra/Postpartum and Newborn Care becomes more crowded, not less, as pregnancy progresses.
51. CORRECT ANSWERS: 2, 4, 5. Answer 1 is incorrect Answer 3 is incorrect because few fetuses (3%4%) are in a
because gestational diabetes cannot be detected in the breech position by delivery; a cesarean delivery is the recom-
multiple marker screen; the correct screening test is a 1-hour mended method of delivery for a fetus in breech presenta-
glucose tolerance test. Answer 2 is correct because Edwards tion. Answer 4 is correct because approximately 96% of
syndrome (trisomy 18) is one of the conditions that may fetuses in a breech position will turn to a vertex position
be detected by the multiple marker screen. It is a severe, by term.
usually lethal developmental disorder. It is characterized TEST-TAKING TIP: The key to this question is the gestational
by cranial, neurological, facial, cardiac, and GI malfor- age. At 18 weeks gestation, many fetuses are breech, and most
mations. Answer 3 is incorrect because pregnancy-induced will turn by full term.
hypertension cannot be detected in the multiple marker Content Area: Maternity, Antepartum; Integrated Process:
screen. Serial blood pressure readings, urine protein, and Communication and Documentation; Cognitive Level:
liver and renal function tests are the screening tests for preg- Application; Client Need/Subneed: Health Promotion
nancy-induced hypertension. Answer 4 is correct because and Maintenance/Ante/Intra/Postpartum and
Down syndrome (trisomy 21) is one of the conditions Newborn Care
2164_Ch04_115-248 29/03/12 12:23 PM Page 239

Answers/Rationales/Tips 239
54. CORRECT ANSWER: 2. Answer 1 is incorrect because twins develop from two different eggs fertilized by two differ-
use of forceps when the client is not completely dilated is ent sperm. Answer 4 is correct because dizygotic twins
not acceptable practice according to the American College of develop from two different eggs fertilized by two different
Obstetricians and Gynecologists. Answer 2 is correct sperm.
because maternal position changes such as sitting, kneel- TEST-TAKING TIP: When there are two alternatives that
ing, lateral, or hands and knees can assist fetal head rota- seem similar, read the answers very carefully to determine
tion from an occipitoposterior to an occipitoanterior the correct answer. Look at the root prefix: mono = one, and
position. Answer 3 is incorrect because, in the supine posi- di = two (2 eggs, 2 sperm).
tion, the gravid uterus compresses the pelvic blood vessels Content Area: Maternity, Antepartum; Integrated Process:
and compromises uteroplacental blood flow. This position Nursing Process, Implementation; Cognitive Level: Application;
not only has no effect on rotation of the fetal head, but can Client Need/Subneed: Health Promotion and Maintenance/
cause decreased perfusion to the placenta. Answer 4 is incor- Ante/Intra/Postpartum and Newborn Care
rect because cesarean delivery should be considered only if 57. CORRECT ANSWER: 2. Answer 1 is incorrect because
adequate contractions are documented and there is still no probable signs of pregnancy are: Hegars sign, ballottement,
change in the cervical dilation. positive pregnancy test, and Goodells sign, which are
TEST-TAKING TIP: This question is asking for the interven- objective signs. Answer 2 is correct because presumptive
tion to rotate the fetal head from occipitoposterior to occipi- signs of pregnancy are: amenorrhea, fatigue, breast
toanterior. Which action assists the fetus to rotate? tenderness and enlargement, morning sickness, and
Content Area: Maternity, Intrapartum; Integrated Process:
quickening, which are more subjective signs. Answer 3
Nursing Process, Implementation; Cognitive Level: Application; is incorrect because the only positive signs of pregnancy are:
Client Need/Subneed: Health Promotion and Maintenance/
auscultation of fetal heart tones, visualization of the fetus
Ante/Intra/Postpartum and Newborn Care by ultrasound, and fetal movement felt by the health-care

ANSWERS
55. CORRECT ANSWER: 2. Answer 1 is incorrect because provider. Answer 4 is incorrect because amenorrhea and
human chorionic gonadotropin (HCG) is a hormone breast tenderness are presumptive signs of pregnancy, not
produced by the developing placenta that prevents the cor- negative signs.
pus luteum from deteriorating and secretes estrogen and TEST-TAKING TIP: Missed periods and breast tenderness
progesterone, which maintains the pregnancy for the first are associated with pregnancy. Eliminate Answer 4, which
20 weeks of gestation. It is found in maternal blood and states that they are negative signs of pregnancy. There are
urine. Answer 2 is correct because phosphatidylglycerol three positive signs of pregnancy that are objective and
(PG) is a major phospholipid of surfactant. The presence determined by the health-care provider; thus, Answer 3
of PG in amniotic fluid indicates fetal lung maturity. can also be eliminated.
Answer 3 is incorrect because -fetoprotein (AFP) is a plas- Content Area: Adult Health, Womens Health; Integrated
ma protein that is produced by the fetus. Abnormally high Process: Nursing Process, Implementation; Cognitive
or low levels can indicate fetal anomalies. AFP levels are Level: Application; Client Need/Subneed: Health
drawn from maternal blood. Answer 4 is incorrect because Promotion and Maintenance/Ante/Intra/Postpartum
partial thromboplastin time (PTT) levels are drawn to and Newborn Care
determine if anticoagulant drugs are at a therapeutic level 58. CORRECT ANSWER: 3. Answer 1 is incorrect because an
in women with thromboembolic disease. increased temperature may be a sign of infection; however, this
TEST-TAKING TIP: The key to this question is choosing the clients major symptom is bleeding, not infection. Answer 2 is
proper laboratory test. Eliminate the alternative that is obviously incorrect because the client is at a high risk for excess blood
not specific to pregnancy. loss. The pulse pressure decreases with hemorrhage. Answer 3 is
Content Area: Maternity, Intrapartum; Integrated Process:
correct because an increased pulse in the presence of visible
Nursing Process, Assessment; Cognitive Level: Comprehension; bleeding indicates excessive blood loss. Answer 4 is incorrect
Client Need/Subneed: Physiological Integrity/Reduction of
because increased blood pressure at this stage of pregnancy
Risk Potential/Laboratory Values would be a sign of a hydatidiform mole or chronic hyperten-
56. CORRECT ANSWER: 4. Answer 1 is incorrect because sion; decreased blood pressure is a sign of hemorrhage.
monozygotic twins (identical) develop from one TEST-TAKING TIP: Note that all the answers have increased
fertilized egg that splits into identical halves early in embry- as the first word. Since the clients symptoms are indicative of
onic development. Answer 2 is incorrect because monozygot- hemorrhage, the vital sign that increases with hemorrhage is
ic twins (identical) develop from one fertilized heart rate (pulse).
egg that splits into identical halves early in embryonic Content Area: Maternity, Antepartum; Integrated Process:
development. Once an egg has been penetrated by a single Nursing Process, Analysis; Cognitive Level: Analysis;
sperm, chemical changes take place that prevent multiple Client Need/Subneed: Physiological Integrity/Physiological
sperm fertilization. Answer 3 is incorrect because dizygotic Adaptation/Alterations in Body Systems
2164_Ch04_115-248 29/03/12 12:23 PM Page 240

240 chapter 4 Health Promotion and Maintenance

59. CORRECT ANSWER: 1. Answer 1 is correct because, Content Area: Maternity, Antepartum; Integrated Process:
with epidural anesthesia, the sympathetic nerves are Nursing Process, Implementation; Cognitive Level: Application;
also blocked by the medication, resulting in vasodilation Client Need/Subneed: Physiological Integrity/Basic Care and
and hypotension. Answer 2 is incorrect because hyperten- Comfort/Nutrition and Oral Hydration
sion is not a side effect of epidural anesthesia. Answer 3 is 62. CORRECT ANSWER: 4. Answer 1 is incorrect because
incorrect because hypoventilation does not occur when promoting the bonding process as soon as possible after birth
bupivacaine is used as the anesthetic agent. If epidural is important, but the evaluation of the newborns physical
narcotics are used, hypoventilation can occur. Answer 4 is stability is most important at this time. Answer 2 is incorrect
incorrect because hyperventilation is not a side effect of because oxytocin should not be administered until after the
epidural anesthesia; it is more likely to occur during placenta is delivered, which usually occurs within 30 minutes
painful contractions. Epidural anesthesia helps to relieve after delivery of the infant. Answer 3 is incorrect because the
contraction pain. third stage of labor is the stage of delivery of the placenta.
TEST-TAKING TIP: The question has hypo- and hyper- The placenta will spontaneously separate from the uterine wall
options for respirations and blood pressure. Think about the and be expelled by uterine contractions. Maternal pushing is
physiology behind these options and eliminate the three options not needed. Answer 4 is correct because the client has just
that do not fit with epidural analgesia. delivered; the placenta has not delivered yet. The new-
Content Area: Maternity, Intrapartum; Integrated Process:
borns physical condition and Apgar scoring is a priority
Nursing Process, Assessment; Cognitive Level: Application; at 1 and 5 minutes after delivery.
Client Need/Subneed: Physiological Integrity/Pharmacological
TEST-TAKING TIP: The key to this question is the wording,
and Parenteral Therapies/Expected Effects/Outcomes has begun the third stage of labor, the delivery of the placen-
60. CORRECT ANSWER: 2. Answer 1 is incorrect because ta. Once the placenta has delivered, the client would be in the
prothrombin time is assessed to maintain correct dosages of fourth stage of labor.
ANSWERS

warfarin (Coumadin). Coumadin crosses the placental barri- Content Area: Maternity, Intrapartum; Integrated Process:
er and is contraindicated in pregnancy. Answer 2 is correct Nursing Process, Analysis; Cognitive Level: Analysis;
because partial thromboplastin time is evaluated to deter- Client Need/Subneed: Health Promotion and Maintenance/
mine the effectiveness of heparin therapy. Answer 3 is Ante/Intra/Postpartum and Newborn Care
incorrect because a bleeding time is not routinely performed 63. CORRECT ANSWER: 4. Answer 1 is incorrect because
on women who are pregnant, and it is unnecessary for a caput succedaneum is an area of generalized edema of the scalp
pregnant woman on heparin therapy. Answer 4 is incorrect on the presenting part of the fetal skull, usually the occiput.
because D-dimer tests are ordered, along with other labora- Answer 2 is incorrect because a hemangioma (also known as
tory tests and imaging scans, to help rule out and diagnose stork bites) are pink areas on the upper eyelids, nose, upper
conditions that cause hypercoagulability. Since this client lip, lower occiput, and the nape of the neck that are transient,
already has a history of thromboembolitic disease, the and usually resolve within a few months. Answer 3 is incorrect
D-dimer is unnecessary. because a cephalohematoma is a collection of blood between
TEST-TAKING TIP: The question is asking which test the skull bone and the periosteum. It does not cross suture
should be used for monitoring this clients heparin lines and may last up to several months. Answer 4 is correct
therapy. Choose the test that reflects monitoring heparin because molding is an overlapping of the fetal skull bones
effectiveness. at the occiput of the skull. The infant skull has a bullet-
Content Area: Maternity, Antepartum; Integrated Process:
shaped appearance that quickly resolves.
Nursing Process, Assessment; Cognitive Level: Analysis; TEST-TAKING TIP: Visualize the different conditions
Client Need/Subneed: Physiological Integrity/Reduction of
described in the options. Eliminate the options that do not fit
Risk Potential/Laboratory Values with the shape of the head. In this question, a tip is to choose
61. CORRECT ANSWERS: 2, 3, 4. Answer 1 is incorrect the shortest time period (few days, not weeks or months).
because salmon is not a good source of folic acid. Answer 2 Content Area: Child Health, Newborn; Integrated Process:
is correct because spinach is the richest source of folic Nursing Process, Implementation; Cognitive Level: Application;
acid, with about 130 mcg per serving. The recommended Client Need/Subneed: Health Promotion and Maintenance/
folic acid intake is 600 mcg/day during pregnancy. Ante/Intra/Postpartum and Newborn Care
Answer 3 is correct because orange juice is a good source
64. CORRECT ANSWER: 2. Answer 1 is incorrect because
of folic acid, with about 100 mcg per serving. Answer 4 is breast-fed infants should be fed on demand. Breast milk is more
correct because asparagus is a good source of folic acid, completely and quickly digested than formula. It is important
with about 130 mcg per serving. Answer 5 is incorrect not to limit infant sucking time at the breast because it is
because chicken is not a good source of folic acid. directly related to breast milk production. Answer 2 is correct
TEST-TAKING TIP: Leafy greens and citrus foods are the high- because an infant who is exclusively breastfed should have
est in folic acid content.
2164_Ch04_115-248 29/03/12 12:23 PM Page 241

Answers/Rationales/Tips 241
at least four to six wet diapers daily. Adequate urinary out- this clients condition. Worsening liver function would
put is a reliable indicator of adequate intake of breast milk. be an indicator for early delivery. Answer 4 is incorrect
Answer 3 is incorrect because the infant should be dressed as because this client is admitted for HELLP, not preterm
parents would dress themselves. Overdressing can cause prickly labor. Although she is preterm, tocolytics are not appropriate
heat rash. Wrapping the infant in a light blanket maintains management for HELLP syndrome. Delivery is the only
body temperature and makes the infant feel secure. Answer 4 is treatment for HELLP syndrome. Answer 5 is correct
incorrect because the cord stump should be cleaned with the because bedrest in the left lateral position would be a
solution ordered by the health-care provider daily until it falls part of the management to decrease blood pressure in
off. Some providers may prefer natural drying, without using this client.
any alcohol for treating the cord. TEST-TAKING TIP: The key for this question is HELLP
TEST-TAKING TIP: This question is asking which answer is syndrome. The fact that the client is preterm is important for
not correct. Be careful when reading and answering this type of administration of corticosteroids for fetal lung maturity. Do
question so that you do not pick out the first correct response not get distracted by the early gestation for treatment and
rather than the proper incorrect response. eliminate the answer that refers to tocolytics (Answer 4).
Content Area: Child Health, Newborn; Integrated Process: Content Area: Maternity, Antepartum; Integrated Process:
Nursing Process, Evaluation; Cognitive Level: Analysis; Nursing Process, Implementation; Cognitive Level: Application;
Client Need/Subneed: Health Promotion and Maintenance/ Client Need/Subneed: Physiological Integrity/Physiological
Ante/Intra/Postpartum and Newborn Care Adaptation/Alterations in Body Systems
65. CORRECT ANSWER: 4. Answer 1 is incorrect because 67. CORRECT ANSWER: 3. Answer 1 is incorrect because
the client doesnt have an episiotomy. Answer 2 is incorrect white blood cell counts indicate the presence or absence
because pulmonary hygiene is important in clients with a of infection. The normal range for WBCs in pregnancy is
respiratory condition, or those who have undergone a cesarean 9000 to 15,000. There is no evidence of infection.

ANSWERS
section. Answer 3 is incorrect because epidural anesthesia takes Answer 2 is incorrect because the normal hematocrit in
approximately an hour to wear off. The client will be unable pregnancy ranges from 32% to 45%. Answer 3 is correct
to ambulate during this time. In addition, all clients who are because the normal hemoglobin in pregnancy ranges
postpartum should be assisted with ambulation the first few from 11 to 12 gm/dL. This client is a teenager who is
times out of bed. Answer 4 is correct because maintaining pregnant and is anemic. Answer 4 is incorrect because
physiological homeostasis is the most important goal dur- the normal serum glucose in pregnancy ranges from 65 to
ing the fourth stage of labor, the immediate postpartum 110 gm/dL.
period. The most common complication of the fourth stage TEST-TAKING TIP: Knowing common laboratory values
of labor is uterine atony and hemorrhage. will help to eliminate the three options that are normal, and
TEST-TAKING TIP: The question states the client has an therefore not risk factors (Answers 1, 2, and 4).
intact perineum; therefore, eliminate the obvious answer that Content Area: Maternity, Antepartum; Integrated Process:
discusses an episiotomy (Answer 1). Choose the answer with Nursing Process, Assessment; Cognitive Level: Application;
the most general content (i.e., physiological homeostasis), Client Need/Subneed: Physiological Integrity/Reduction of
rather than specific behaviors (i.e., turn, cough . . . and Risk Potential/Laboratory Values
. . . ambulate without assistance).
68. CORRECT ANSWER: 3. Answer 1 is incorrect because
Content Area: Maternity, Postpartum; Integrated Process:
Marcaine is an anesthetic agent and does not cause
Nursing Process, Implementation; Cognitive Level: Analysis;
respiratory depression. Respiratory depression is associated
Client Need/Subneed: Health Promotion and Maintenance/
with epidural narcotics. Answer 2 is incorrect because uterine
Ante/Intra/Postpartum and Newborn Care
atony is not an effect of epidural anesthesia. Answer 3 is
66. CORRECT ANSWERS: 1, 3, 5. Answer 1 is correct correct because the client may not sense the urge to void
because the client who is 28 weeks gestation with due to decreased sensation to the area. Bladder distention
HELLP syndrome will most likely have to be delivered can interfere with descent of the fetus and cause pain.
before reaching full-term gestation. Administration of Answer 4 is incorrect because maternal hypotension, not
corticosteroid to help mature the fetal lungs would be an hypertension, is a side effect of epidural anesthesia.
appropriate part of management for this client. Answer 2 TEST-TAKING TIP: All the answers involve assessment on
is incorrect because an IV fluid bolus is not an appropriate an hourly basis; therefore, focus on the differences between the
management for a client with increased blood pressure in alternatives. Eliminate the three options that are not an effect
HELLP syndrome. A fluid bolus can lead to fluid overload of epidural anesthesia.
and increase third spacing of fluid, with possible pulmonary Content Area: Maternity, Intrapartum; Integrated Process:
edema. Answer 3 is correct because elevated liver function Nursing Process, Assessment; Cognitive Level: Analysis;
tests are a part of the HELLP syndrome; following serial Client Need/Subneed: Physiological Integrity/Pharmacological
liver function tests will be a part of the management of and Parenteral Therapies/Expected Effects/Outcomes
2164_Ch04_115-248 29/03/12 12:23 PM Page 242

242 chapter 4 Health Promotion and Maintenance

69. CORRECT ANSWER: 4. Answer 1 is incorrect delivery, lasting about 5 days. Answer 5 is correct because
because turning the client to her left side and administering moderate lochia rubra is a normal finding 2 hours after
oxygen is an intervention for fetal distress and there is no delivery.
evidence of fetal distress. Answer 2 is incorrect because this TEST-TAKING TIP: The key to this question is the time frame
is a reassuring fetal heart pattern; no immediate nursing after delivery. Choose the options that apply to a client 2 hours
actions other than comfort measures are necessary. Answer 3 is after delivery.
incorrect because the client is in no need for fluid volume Content Area: Maternity, Postpartum; Integrated Process:
expansion; neither she nor the fetus is in distress. Answer 4 Nursing Process, Evaluation; Cognitive Level: Application;
is correct because this client is in prodromal labor. The Client Need/Subneed: Health Promotion and Maintenance/
fetal heart pattern is reassuring. There is no reason to Ante/Intra/Postpartum and Newborn Care
admit the client to the hospital at this time. Ambulation 72. CORRECT ANSWERS: 2, 4, 5. Answer 1 is incorrect
at home may stimulate labor and descent of the present- because the normal heart rate at rest is 110 to 160. A rate of
ing part, and decrease hospitalization time. 170 is tachycardia, which is not normal after birth, and may
TEST-TAKING TIP: The key to this question is the stage indicate neonatal sepsis. This needs to be reported to the
and phase of labor. Prodromal labor in a primipara can physician. Answer 2 is correct because the normal respira-
last several days. The fetal heart tracing is reassuring; tory rate is 30 to 60 breaths/min. This finding does not
therefore, the client can be safely discharged. Note that require any intervention and does not need to be report-
three of the options are active interventions (give O2, ed to the physician. Answer 3 is incorrect because circum-
immediate operative delivery, start IV). Choose the one oral cyanosis, a bluish color around the mouth, is an abnor-
that is a different option from the others (send the client mal finding and requires further assessment, including pulse
home). oximetry. Findings need to be reported to the physician.
Content Area: Maternity, Intrapartum; Integrated Process:
Answer 4 is correct because caput succedaneum is edema
ANSWERS

Nursing Process, Implementation; Cognitive Level: Analysis; over the back of the fetal head caused by pressure over
Client Need/Subneed: Health Promotion and Maintenance/
the presenting part of the fetal head and it resolves spon-
Ante/Intra/Postpartum and Newborn Care taneously. This finding does not require any intervention
70. CORRECT ANSWER: 1. Answer 1 is correct because and does not need to be reported to the physician.
high prolactin levels with exclusive breastfeeding can Answer 5 is correct because mongolian spots are a normal
delay ovulation for up to 6 months. However, it is a finding in dark-skinned neonates. This finding does not
very unpredictable method of birth control. Answer 2 is require any intervention and does not need to be report-
incorrect because condoms are a mechanical barrier method ed to the physician.
that does not interfere with breastfeeding. Answer 3 is incor- TEST-TAKING TIP: Note the key word in the stem
rect because Depo-Provera is an injectable form of progestin normaland eliminate the two answers that pertain to
that prevents pregnancy for 3 months. It is safe for use dur- abnormal findings.
ing lactation once the milk supply is established. Answer 4 is Content Area: Child Health, Newborn; Integrated Process:
incorrect because contraceptive foam has no hormones to Nursing Process, Assessment; Cognitive Level: Analysis;
affect lactation, and is safe for use during the postpartum Client Need/Subneed: Physiological Integrity/Physiological
period. Adaptation/Alterations in Body Systems
TEST-TAKING TIP: This question is asking for the incorrect 73. CORRECT ANSWER: 3. Answer 1 is incorrect
answer. Be careful when reading the stem so that you do not because analysis of amniotic fluid from amniocentesis
choose the first correct response rather than the incorrect allows determination of fetal gender. Answer 2 is incorrect
response. because glycemic control is determined by hemoglobin A1c,
Content Area: Maternity, Postpartum; Integrated Process:
a maternal blood test. Answer 3 is correct because the
Nursing Process, Evaluation; Cognitive Level: Analysis; -fetoprotein (AFP) is a maternal blood test that screens
Client Need/Subneed: Health Promotion and Maintenance/
for possible neural tube defects (the most common
Lifestyle Choices anomaly) in fetuses of a client with diabetes. It can also
71. CORRECT ANSWERS: 1, 5. Answer 1 is correct because indicate the presence of Down syndrome. Answer 4 is
the fundus is usually at about the level of the umbilicus incorrect because the presence of phosphatidylglycerol and
2 hours after delivery. Answer 2 is incorrect because the fun- the L/S ratio determine fetal lung maturity. It is obtained
dus is not found 2 fingerbreadths below the umbilicus until from amniotic fluid.
about 48 hours after delivery. Answer 3 is incorrect because TEST-TAKING TIP: The most important fact in this question
bluish-white fluid in milk does not come in until 3 to 4 days is that the client with insulin-dependent diabetes has a higher
after delivery, not 2 hours after delivery. Answer 4 is incorrect risk for having a fetus with neural tube defects, and the AFP
because lochia serosa does not occur until about 5 days after test screens for this possibility.
2164_Ch04_115-248 29/03/12 12:23 PM Page 243

Answers/Rationales/Tips 243
Content Area: Maternity, Antepartum; Integrated Process: 76. CORRECT ANSWER: 4. Answer 1 is incorrect because
Nursing Process, Implementation; Cognitive Level: Application; checking the blood pressure in the right arm has no associa-
Client Need/Subneed: Physiological Integrity/Reduction of Risk tion with the side effects of magnesium sulfate. Answer 2
Potential/Diagnostic Tests is incorrect because this intervention is contraindicated in
74. CORRECT ANSWER: 2. Answer 1 is incorrect because
a seizure. It acts to occlude the airway and prevent proper
fundal massage would be an appropriate intervention for ventilation. Answer 3 is incorrect because inserting a Foley
uterine atony. Symptoms of uterine atony would reveal a catheter is not associated with any side effects of magnesium
constant trickle of bright red blood in the presence of a sulfate. It may be used to carefully measure output and
boggy uterus. In this case it is in the presence of a contracted signs of worsening preeclampsia, but not for side effects
uterus. Answer 2 is correct because a constant trickling of magnesium sulfate. Answer 4 is correct because magne-
of bright red vaginal blood in the presence of a contract- sium sulfate side effects include headaches and seizures.
ed uterus may be an unrepaired laceration of the birth Darkening the room can help reduce the occurrence of
canal. The physician must be notified so that the lacera- these side effects.
tion can be repaired. Answer 3 is incorrect because excessive TEST-TAKING TIP: This question is asking about the side
bleeding caused by a full bladder would reveal a uterus that effects of magnesium sulfate and not about treatments for
was above the umbilicus and deviated to the right side. In preeclampsia. Eliminate the options that do not treat side
this case the uterus is midline at the umbilicus. Answer 4 effects.
Content Area: Maternity, Intrapartum; Integrated Process:
is incorrect because increasing the rate of an infusion of
oxytocin would not correct the problem of a lacerated birth Nursing Process, Implementation; Cognitive Level: Application;
Client Need/Subneed: Physiological Integrity/Pharmacological
canal.
TEST-TAKING TIP: When there is a postpartum hemorrhage, and Parenteral Therapies/Adverse Effects/Contraindications/
the first consideration is uterine atony. Do not immediately Interactions

ANSWERS
assume that all postpartum bleeding is uterine atony. This 77. CORRECT ANSWER: 3. Answer 1 is incorrect
client had a contracted uterus, so you must consider other because this infants initial alertness and eagerness are
reasons for bleeding that may require a medical, not a nursing, followed by an increasingly deeper sleep. This infant is
intervention. only 3 hours old, and the client should be instructed to
Content Area: Maternity, Intrapartum; Integrated Process: wait for the infant to wake up. At about 20 to 24 hours
Nursing Process, Implementation; Cognitive Level: Analysis; of age, the baby will be awake more often and be more
Client Need/Subneed: Physiological Integrity/Reduction of interested in nursing. Answer 2 is incorrect because this
Risk Potential/Potential for Complications of Diagnostic infant is in a normal sleep state and does not need to be
Tests/Treatments/Procedures awakened for a feeding. Answer 3 is correct because the
75. CORRECT ANSWERS: 2, 5. Answer 1 is incorrect because infants suckling reflex is greatest from 45 minutes to
Hemabate is used to control postpartum hemorrhage, and is 2 hours after birth. The baby may nurse several times
contraindicated for induction of labor. Answer 2 is correct close together (cluster feedings) and then sleep several
because pitocin at 2 milliunits/min IV is an appropriate hours without nursing. Answer 4 is incorrect because this
dose for induction of labor in a client who is past her esti- infant is in a normal sleep state and does not need to be
mated date of delivery. Answer 3 is incorrect because awakened for a feeding. In addition, the client should be
betamethasone is used to stimulate fetal lung maturity in instructed to sponge bathe, not submerse, the infant until
preterm gestation, and is not used for induction of labor. the cord falls off.
Answer 4 is incorrect because Cytotec is used for induction of
TEST-TAKING TIP: Is this a normal newborn infant
labor, but 200 mcg is too high a dose for induction of labor and behavior? Or, is this a problem? Remember that normal
may cause uterine hyperstimulation and uterine rupture. The newborn infants are demand-fed and the normal feeding
normal dose of Cytotec for induction is 25 mcg vaginally or pattern is cluster feeding 8 to 12 times in a 24-hour period.
50 mcg orally. Answer 5 is correct because cervidil 10 mg The infants initial alertness and eagerness are followed by
vaginally is an appropriate method to induce labor. an increasingly deeper sleep. At about 20 to 24 hours of
TEST-TAKING TIP: Focus on the agents used to induce labor age, the baby will be awake more often and be more
(Answers 2 and 5). Eliminate those medications that are interested in nursing. Eliminate the two options that call
used in postpartum hemorrhage and preterm labor (Answers 1 for waking the baby who is in a normal sleep state
and 3) but are not used to induce labor. (Answers 2 and 4).
Content Area: Maternity, Postpartum; Integrated Process:
Content Area: Maternity, Intrapartum; Integrated Process:
Nursing Process, Analysis; Cognitive Level: Application; Teaching and Learning; Cognitive Level: Application;
Client Need/Subneed: Health Promotion and Maintenance/
Client Need/Subneed: Physiological Integrity/Pharmacological
and Parenteral Therapies/Expected Effects/Outcomes Ante/Intra/Postpartum and Newborn Care
2164_Ch04_115-248 29/03/12 12:23 PM Page 244

244 chapter 4 Health Promotion and Maintenance

78. CORRECT ANSWER: 2. Answer 1 is incorrect because Content Area: Maternity, Intrapartum; Integrated Process:
fetal heart tones less than 110 bpm is bradycardia and not Nursing Process, Implementation; Cognitive Level: Analysis;
a normal finding after amniotomy. Answer 2 is correct Client Need/Subneed: Health Promotion and Maintenance/
because a moderate amount of straw-colored fluid is a Ante/Intra/Postpartum and Newborn Care
normal color and an amount of amniotic fluid that is 81. CORRECT ANSWER: 4. Answer 1 is incorrect
expected after the artificial rupture of membranes. because oxytocin makes the contractions more painful.
Answer 3 is incorrect because a small amount of greenish
Painless delivery would not be a measure of the
fluid is meconium staining, not a normal result after effectiveness of IV oxytocin. Answer 2 is incorrect
amniotomy. Answer 4 is incorrect because a small segment because this client was already 100% effaced before the
of the umbilical cord would be a prolapsed cord, which is oxytocin augmentation was started. Answer 3 is incorrect
an obstetric emergency. because infrequent contractions would mean the oxytocin
TEST-TAKING TIP: This question is asking which finding is was not effective. IV oxytocin increases the frequency of
normal after rupture of membranes. Think about the normal the contractions. Answer 4 is correct because this client
color and consistency of amniotic fluid. has uterine dystocia, and the effectiveness of the
Content Area: Maternity, Intrapartum; Integrated Process:
oxytocin infusion is measured by a change in the
Nursing Process, Planning; Cognitive Level: Application; dilation of the cervix.
Client Need/Subneed: Physiological Integrity/Reduction of
TEST-TAKING TIP: Restate this question to ask, how
Risk Potential/Diagnostic Tests would the nurse know if oxytocin augmentation is
79. CORRECT ANSWER: 1. Answer 1 is correct because working?
this client is a primigravida at 2 cm dilated. This is the Content Area: Maternity, Intrapartum; Integrated Process:
first stage of labor, and latent phase. Typically, clients in Nursing Process, Evaluation; Cognitive Level: Analysis;
this stage of labor are not in much pain, and are excited Client Need/Subneed: Physiological Integrity/Pharmacological
ANSWERS

and talkative about what is happening. Answer 2 is incor- and Parenteral Therapies/Expected Effects/Outcomes
rect because the urge to push during a contraction occurs 82. CORRECT ANSWER: 1. Answer 1 is correct
during the transition phase of labor, not the latent phase. because the client will need a cesarean section as
Answer 3 is incorrect because inability to concentrate during
quickly as possible to avoid a vaginal delivery and
a contraction happens during the active phase of labor, not possible head entrapment. Answer 2 is incorrect because
the latent phase of labor. Answer 4 is incorrect because applying the fetal heart monitor is important, but does not
clients usually ask for an epidural when they are in pain dur- facilitate preparation for the cesarean section. Answer 3 is
ing the active or transition phase of labor. The epidural is incorrect because placing the client in genupectoral position
generally given when the client is in active labor, not latent is appropriate for a prolapsed cord, not for footling breech
phase. presentation. Answer 4 is incorrect because the vaginal
TEST-TAKING TIP: The best answer is based on identifying examination has already revealed a footling breech; getting
the stage and phase of labor based on her cervical dilation, an ultrasound is not a priority at this time.
and the typical response of a woman in the latent phase of TEST-TAKING TIP: Eliminate the interventions that delay
labor. getting the client ready for a cesarean section.
Content Area: Maternity, Intrapartum; Integrated Process:
Content Area: Maternity, Intrapartum; Integrated Process:
Nursing Process, Analysis; Cognitive Level: Analysis; Nursing Process, Implementation; Cognitive Level: Analysis;
Client Need/Subneed: Health Promotion and Maintenance/
Client Need/Subneed: Physiological Integrity/Reduction of
Ante/Intra/Postpartum and Newborn Care Risk Potential/Potential for Complications from Surgical
80. CORRECT ANSWER: 2. Answer 1 is incorrect because Procedures and Health Alterations
this action will do nothing to resolve the decelerations of the 83. CORRECT ANSWER: 2. Answer 1 is incorrect
fetal heart tones. This would be an appropriate action if there because the cervix is 4 cm dilated, not closed. Answer 2 is
was an inconsistent or broken tracing. Answer 2 is correct correct because the nurse cannot artificially rupture
because the first action the nurse should take is to turn the the amniotic membranes in order to apply an internal
client to her left side, which may help resolve the variable fetal monitor. The only choice for continuous fetal
decelerations. Answer 3 is incorrect because asking the client monitoring is an external fetal monitor. Answer 3 is
to ambulate may make the variable decelerations worse, espe- incorrect because the fetal heart tones show tachycardia,
cially if the reason for the decelerations is cord compression. which is an indicator that continuous fetal monitoring is
Answer 4 is incorrect because preparing the client for delivery
needed to ensure fetal well-being. Answer 4 is incorrect
is not the first action the nurse should take, since it will not because, when the contractions are intense enough, there
resolve the decelerations. is no need for insertion of an internal monitor. The purpose
TEST-TAKING TIP: Envision the type of decelerations that of an internal uterine monitor is to ensure the strength of
occur during a contraction and think about the actions that the contractions.
would resolve the pattern of decelerations.
2164_Ch04_115-248 29/03/12 12:23 PM Page 245

Answers/Rationales/Tips 245
TEST-TAKING TIP: Think about the purpose of fetal monitor- intervention to resolve variable decelerations, repositioning
ing and the difference between external and internal monitor- the client is the first action the nurse should take. Answer 3
ing. The membranes must be ruptured to place an internal is correct because repositioning the client will help to
monitor. This client has intact membranes, so only an external alleviate the variable decelerations. This should be the
monitor can be used on this client. first action of the nurse. Answer 4 is incorrect because
Content Area: Maternity, Intrapartum; Integrated Process: readjusting the monitor does nothing to resolve the
Nursing Process, Implementation; Cognitive Level: Analysis; variable decelerations.
Client Need/Subneed: Health Promotion and Maintenance/ TEST-TAKING TIP: Assess the fetal monitor strip in the
Ante/Intra/Postpartum and Newborn Care same way each time: What is the baseline and is it normal?
What is the variability and is it normal? Are there accelera-
84. CORRECT ANSWER: 2. Answer 1 is incorrect
tions? Are there decelerations and what types of decelerations
because an assessment should be completed before calling
are present? What interventions are appropriate for each
the physician. Answer 2 is correct because decreased
type of deceleration?
fetal movement may be a sign of hypoxia in the fetus
Content Area: Maternity, Intrapartum; Integrated Process:
or a fetal demise. Placing the client on a fetal monitor
Nursing Process, Implementation; Cognitive Level: Analysis;
will assist the nurse to evaluate the fetal well-being.
Client Need/Subneed: Physiological Integrity/Reduction of
Answer 3 is incorrect because starting an IV will not demon-
Risk Potential/Potential for Complications from Surgical
strate fetal well-being, or do anything about evaluating the
Procedures and Health Alterations
decreased fetal movement. Answer 4 is incorrect because
maternal vital signs are not as crucial at this time. 87. CORRECT ANSWER: 4. Answer 1 is incorrect because a
Evaluating the fetus is most important when a client baseline fetal heart rate of 170 to 180 bpm is tachycardia and
presents with decreased fetal movement. is nonreassuring. Answer 2 is incorrect because baseline vari-
TEST-TAKING TIP: The key word is fetal. Focus on what ability of 25 to 35 bpm is marked variability, and not a char-

ANSWERS
is the most important evaluation of fetal status. Calling acteristic of a reassuring fetal heart rate pattern. Answer 3 is
the physician is always required, but an assessment needs incorrect because variable decelerations to 100 bpm are a
to be completed so the physician can understand the result of cord compression and are not a characteristic of a reas-
condition. suring fetal heart rate pattern. Answer 4 is correct because
Content Area: Maternity, Intrapartum; Integrated Process: accelerations are correlated with fetal movement and ade-
Nursing Process, Implementation; Cognitive Level: Analysis; quate oxygen reserves in the fetus.
Client Need/Subneed: Health Promotion and Maintenance/ TEST-TAKING TIP: Remember the characteristics of normal
Ante/Intra/Postpartum and Newborn Care fetal heart rate patterns in the same way every time: baseline
heart rate 110 to 160 bpm, baseline variability 6 to 25 bpm,
85. CORRECT ANSWER: 4. Answer 1 is incorrect because
accelerations and no decelerations.
fetal sleep can manifest as minimal variabilitybut this
Content Area: Maternity, Antepartum; Integrated Process:
fetus also has tachycardia and late decelerations, which are
Nursing Process, Evaluation; Cognitive Level: Application;
not associated with fetal sleep. Answer 2 is incorrect because
Client Need/Subneed: Health Promotion and Maintenance/
umbilical cord compression is manifested by variable deceler-
Ante/Intra/Postpartum and Newborn Care
ations, not late decelerations or tachycardia. Answer 3 is incor-
rect because head compressions are early decelerations that 88. CORRECT ANSWER: 3. Answer 1 is incorrect because the
mirror the contractions, not late decelerations or tachycardia. bladder fills more rapidly because of the fluid bolus given
Answer 4 is correct because late decelerations are indica- before the epidural, not the medication used for the epidural.
tive of uteroplacental insufficiency, and tachycardia can Answer 2 is incorrect because the epidural does not affect the
be a sign of hypoxia as well. level of consciousness. Answer 3 is correct because the sensa-
TEST-TAKING TIP: Picture the fetal heart tracing and the tion of the bladder filling is diminished or lost when the
physiology behind each of the patterns. Evaluate a fetal heart client has an epidural. Answer 4 is incorrect because there is
rate tracing the same way every time: baseline, variability, no information in the stem that would indicate that she is
accelerations, and finally, decelerations. embarrassed to ask for the bedpan that frequently. Even if that
Content Area: Maternity, Intrapartum; Integrated Process: were the case, the bladder sensation is diminished when the
Nursing Process, Analysis; Cognitive Level: Analysis; client has an epidural.
Client Need/Subneed: Physiological Integrity/Reduction of TEST-TAKING TIP: Restate this question as What are the
Risk Potential/Potential for Complications from Surgical side effects of epidural anesthetics for a client in labor?
Procedures and Health Alterations Content Area: Maternity, Intrapartum; Integrated Process:
Nursing Process, Analysis; Cognitive Level: Application;
86. CORRECT ANSWER: 3. Answer 1 is incorrect because
Client Need/Subneed: Physiological Integrity/Basic Care and
the nurse should try to resolve the variable decelerations
Comfort/Elimination
before notifying the physician. Answer 2 is incorrect
because, although giving a 500-mL bolus IV is part of the
2164_Ch04_115-248 29/03/12 12:23 PM Page 246

246 chapter 4 Health Promotion and Maintenance

89. CORRECT ANSWER: 2. Answer 1 is incorrect because, TEST-TAKING TIP: Oral contraceptives and contraceptive
during the ovulatory phase, the developing follicles produce patches utilize the same hormones; eliminate both choices
estrogen, which is critical for the buildup of the uterine because they both are basically the same choice.
lining. To conceive, the estrogen levels need to be elevated. Content Area: Adult Health, Family Planning; Integrated
Answer 2 is correct because the luteinizing hormone surge Process: Teaching and Learning; Cognitive Level: Application;
is responsible for triggering ovulation, or the release of Client Need/Subneed: Health Promotion and Maintenance/
the egg from the ovary, and conception. Answer 3 is incor- Lifestyle Choices
rect because, during the ovulatory phase, the developing 92. CORRECT ANSWER: 4. Answer 1 is incorrect because
follicles produce estrogen, which is critical for the buildup painless vaginal bleeding is a symptom of placenta previa.
of the uterine lining. The lining must be thick to nourish the Ectopic pregnancy is a painful condition. Answer 2 is incorrect
fertilized egg. Answer 4 is incorrect because progesterone is because abdominal cramping is related to threatened or immi-
the reproductive hormone that is actually produced by the nent abortion. Ectopic pregnancy is characterized by sharp
corpus luteum (a part of the ovary from which the mature pain. Answer 3 is incorrect because throbbing pain in the
egg bursts during ovulation). The levels need to be elevated upper quadrant is not a characteristic of ectopic pregnancy.
for conception to occur. The fallopian tubes are in the lower quadrant. Answer 4 is
TEST-TAKING TIP: Review the hormones responsible for fer- correct because sudden, stabbing pain in the lower quad-
tility and the role of each in conception. Eliminate the obvious rant is a symptom of ectopic pregnancy. The pain is related
response that does not refer to hormone production because to the stretching of the fallopian tube in the lower abdomi-
the endometrial lining must be conductive to a pregnancy for nal quadrant.
conception to occur. TEST-TAKING TIP: Eliminate Answer 3, which states the
Content Area: Adult Health, Family Planning; Integrated Process:
pain is in the upper quadrant; fallopian tubes are in the lower
Teaching and Learning; Cognitive Level: Application; Client quadrant.
ANSWERS

Need/Subneed: Health Promotion and Maintenance/Self-Care


Content Area: Maternity, Antepartum; Integrated Process:
90. CORRECT ANSWER: 3. Answer 1 is incorrect because Nursing Process, Assessment; Cognitive Level: Application;
the age of the client has nothing to do with the success of Client Need/Subneed: Physiological Integrity/Physiological
the method. Answer 2 is incorrect because the timing of Adaptation/Medical Emergencies
intercourse, not the frequency of intercourse, is the important 93. CORRECT ANSWER: 2. Answer 1 is incorrect because
factor. Answer 3 is correct because the effectiveness of the hyperemesis leads to metabolic acidosis, not respiratory
rhythm method depends on how consistent her cycle is alkalosis. Hyperemesis also includes dehydration. Answer 2
and how accurately the couple tracks when she could be is correct because hyperemesis leads to metabolic
ovulating. Answer 4 is incorrect because, although the range acidosis and dehydration. Answer 3 is incorrect because
of the clients temperature can be important, the regularity hyperemesis leads to metabolic acidosis, not respiratory
of the menstrual cycle and commitment to timing the inter- acidosis. Hyperemesis also includes dehydration. Answer 4
course is more critical. is incorrect because, if the client has prolonged vomiting, it
TEST-TAKING TIP: Remember that the rhythm method is can lead to metabolic alkalosis. This client has only nausea
based on predicting ovulation, and having a regular menstrual and dehydration for a 3-day period.
cycle is the most important factor. TEST-TAKING TIP: Hyperemesis results in metabolic
Content Area: Adult Health, Family Planning; Integrated
acidosis. Eliminate the options with respiratory acidosis
Process: Teaching and Learning; Cognitive Level: Application;
and alkalosis.
Client Need/Subneed: Health Promotion and Maintenance/
Content Area: Maternity, Antepartum; Integrated Process:
Lifestyle Choices Nursing Process, Analysis; Cognitive Level: Analysis;
91. CORRECT ANSWER: 3. Answer 1 is incorrect because Client Need/Subneed: Physiological Integrity/Physiological
intrauterine devices have a higher rate of infection. A client Adaptation/Fluid and Electrolyte Imbalances
with diabetes should be protected from the risk of developing a 94. CORRECT ANSWER: 2. Answer 1 is incorrect because
pelvic infection due to the intrauterine device. Answer 2 is elevated human chorionic gonadotropin can been seen in a
incorrect because oral hormonal contraceptive methods can hydatidiform mole. Answer 2 is correct because the presence
affect the carbohydrate metabolism and insulin utilization in of fetal heart tones is a positive sign of pregnancy. Answer 3
clients with type 1 diabetes. Answer 3 is correct because barri- is incorrect because uterine enlargement can be a result of a
er methods (e.g., diaphragm) are the best choice for a client tumor, a hydatidiform mole, or myomata. Answer 4 is incorrect
with diabetes since they do not have the undesirable side because breast enlargement and tenderness can be caused by
effects that the other methods have. Answer 4 is incorrect hormonal fluctuations of the normal menstrual cycle.
because a contraceptive patch is a hormonal contraceptive TEST-TAKING TIP: Remember that the positive signs of preg-
method that can affect the carbohydrate metabolism and nancy are limited to: fetal heart tones, fetal outline on ultra-
insulin utilization in clients with type 1 diabetes. sound or x-ray, and fetal movement felt by an examiner.
2164_Ch04_115-248 29/03/12 12:23 PM Page 247

Answers/Rationales/Tips 247
Content Area: Maternity, Antepartum; Integrated Process: Answer 4 is correct because vacuum-assisted vaginal delivery
Nursing Process, Assessment; Cognitive Level: Comprehension; may result in a cephalohematoma caused by the application
Client Need/Subneed: Physiological Integrity/Reduction of Risk of a vacuum extractor. Answer 5 is incorrect because breech
Potential/System Specific Assessments presentation does not result in fetal head trauma; therefore,
cephalohematoma does not form on the fetal head.
95. CORRECT ANSWER: 3. Answer 1 is incorrect because, TEST-TAKING TIP: Remember that cephalohematoma is the
with poorly controlled gestational diabetes, the neonate will
collection of blood and is a result of trauma to the fetal head.
likely be hypoglycemic, but large for gestational age, not small
When there is no trauma, such as with cesarean section or
for gestational age. Answer 2 is incorrect because, with poorly
breech presentation, you can eliminate those options.
controlled gestational diabetes, the neonate will likely be
Content Area: Child Health, Newborn; Integrated Process:
hypoglycemic, not hyperglycemic. Answer 3 is correct because,
Nursing Process, Analysis; Cognitive Level: Application;
with poorly controlled gestational diabetes, the neonate
Client Need/Subneed: Health Promotion and Maintenance/
will likely be hypoglycemic and large for gestational age.
Ante/Intra/Postpartum and Newborn Care
Answer 4 is incorrect because, with poorly controlled gesta-
tional diabetes, the neonate will likely be hypoglycemic, not 98. CORRECT ANSWERS: 1, 2, 3, 5. Answer 1 is correct
hyperglycemic. because the developing fetus appears to be vulnerable to
TEST-TAKING TIP: Focus on pairs of responses: hypo- versus DNA damage from methamphetamine exposure because it
hyper- and small versus large. In poorly controlled gestational hasnt yet developed the enzymes that protect it against free
diabetes, the fetus produces excess insulin, which acts as a radicals. Methamphetamines can cause neural tube defects
growth hormone in the fetus; the neonate is likely to be large such as cleft lip and palate. Answer 2 is correct because, when
for gestational age. a neonate is withdrawing from methamphetamine, there is
Content Area: Child Health, Newborn; Integrated Process: an increased incidence of irritability, inability to be consoled,
Nursing Process, Analysis; Cognitive Level: Analysis; and difficulty sleeping. Answer 3 is correct because metham-

ANSWERS
Client Need/Subneed: Physiological Integrity/Physiological phetamine exposure can cause skeletal malformations such as
Adaptation/Alterations in Body Systems clubfoot. Answer 4 is incorrect because hyperbilirubinemia is
not associated with exposure to methamphetamines. Answer 5 is
96. CORRECT ANSWERS: 1, 3, 5. Answer 1 is correct correct because gastroschisiswhen all of the intestines are
because simian creases are a typical finding in a newborn
outside of the bodyis a common birth defect in metham-
with trisomy 21. Answer 2 is incorrect because decreased,
phetamine-exposed infants.
not increased, muscle tone is typical of newborns with tri-
TEST-TAKING TIP: Methamphetamines are manufactured
somy 21. Answer 3 is correct because flat appearance of
with a variety of toxic chemicals that can cause DNA damage
the face is typical of newborns with trisomy 21. Answer 4
in the developing fetus. Many defects can be attributed to
is incorrect because large tongue, not small tongue, is typical
methamphetamine exposure in utero.
of newborns with trisomy 21. Answer 5 is correct because
Content Area: Child Health, Newborn; Integrated Process:
upward-slanting eye creases are typical of newborns with
Nursing Process, Assessment; Cognitive Level: Application;
trisomy 21.
Client Need/Subneed: Physiological Integrity/Pharmacological
TEST-TAKING TIP: Read the answers carefully; in two of the
and Parenteral Therapies/Adverse Effects/Contraindications/
options, the typical characteristics of trisomy 21 are the opposite
Interactions
of the characteristics listed here. If not carefully read, the wrong
choice can be made. 99. CORRECT ANSWER: 2. Answer 1 is incorrect because
Content Area: Child Health, Newborn; Integrated Process: weight gain is a common side effect of oral contraceptives that
Nursing Process, Assessment; Cognitive Level: Application; does not need to be reported to the physician. Answer 2 is cor-
Client Need/Subneed: Physiological Integrity/Physiological rect because an alternate method of birth control is needed
Adaptation/Pathophysiology when taking antibiotics due to the decreased effectiveness
of oral contraceptives when taking antibiotics. Answer 3 is
97. CORRECT ANSWERS: 3, 4. Answer 1 is incorrect because incorrect because, if the client misses one or more pills, two
cephalohematoma is an area of bleeding underneath one of the
pills should be taken on the day that the client remembers; and
cranial bones usually caused by a difficult birth. A scheduled
two pills the next day if two pills are missed, not two pills for a
cesarean delivery does not result in trauma to the fetal head
week. Answer 4 is incorrect because nausea and stomach upset
with a result of cephalohematoma. Answer 2 is incorrect
are common side effects of oral contraceptives that do not
because prolonged latent phase does not necessarily mean a
need to be reported to the physician.
difficult birth. A prolonged latent phase is fairly common in
TEST-TAKING TIP: Eliminate the common side effects of oral
clients who are primigravida. This is the phase of labor when
contraceptives that do not need to be reported to the physician.
the cervix is less than 4 cm dilated. Answer 3 is correct
Content Area: Adult Health, Family Planning; Integrated
because prolonged second stage of labor is a long pushing
Process: Teaching and Learning; Cognitive Level: Application;
stage that may result in a cephalohematoma, caused by the
Client Need/Subneed: Physiological Integrity/Pharmacological
fetal head being in the birth canal for a prolonged period.
and Parenteral Therapies/Expected Effects/Outcomes
2164_Ch04_115-248 29/03/12 12:23 PM Page 248

248 chapter 4 Health Promotion and Maintenance

100. CORRECT ANSWER: 1. Answer 1 is correct because the TEST-TAKING TIP: Know the difference between placenta
client likely has a placenta previa. Assessing the fetal heart previa and placental abruption. A previa has painless bleeding
tones for signs of fetal distress is the most important action. and an abruption has pain with or without bleeding.
Answer 2 is incorrect because the client likely has a placenta Content Area: Maternity, Intrapartum; Integrated Process:
previa, and a cervical examination is contraindicated. Answer 3 Nursing Process, Implementation; Cognitive Level: Analysis;
is incorrect because the client likely has a placenta previa. The Client Need/Subneed: Physiological Integrity/Physiological
uterus is firm with minimal relaxing with a placental abrup- Adaptation/Alterations in Body Systems
tion. Answer 4 is incorrect because, although the client likely
has a placenta previa and obtaining maternal vital signs is
important, assessing the fetal heart tones for signs of fetal dis-
tress is the most important action.
ANSWERS
2164_Ch05_249-346 29/03/12 12:24 PM Page 249

CHAPTER 5

Health Promotion
and Maintenance
Nursing Care of the Pediatric Client

Kathleen E. Snider Denise Wall Parilo

249
2164_Ch05_249-346 29/03/12 12:24 PM Page 250

250 chapter 5 Health Promotion and Maintenance

Management of Care 2. Engaging in activities designed to prevent errors,


such as identifying pediatric clients only by their
of Infants, Children, name bands or by a reliable adult who knows the
and Adolescents child.
The role of the pediatric nurse in management of care 3. Handling hazardous and infectious materials
includes the following: appropriatelyfor example, always disposing of
contaminated waste in a clearly marked biohazard
1. Serving as an advocate for the child and family, container.
such as informing the child and family of all treat- 4. Using medical and surgical asepsis, such as using
ments and procedures. sterile gloves and technique when changing a
2. Coordinating clinical case management to provide surgical incision dressing.
access to high-quality clinical resources appropriate
to the level of care needed, such as referring a child Prevention and Early
to an acute rehabilitation facility following spinal
cord injury. Detection of Disease
3. Coordinating continuity of care across the in Infants, Children,
health-care delivery continuum, such as dis- and Adolescents
cussing the needs of the child and family with The role of the pediatric nurse in the prevention and early
the home-care nurse before the childs discharge detection of disease includes the following:
in a spica cast.
4. Delegating care to and supervising care among 1. Engaging in disease prevention activities (e.g., wash-
various members of the health-care delivery team, ing hands thoroughly before and after providing
such as assigning a float nurse to care for selected client care).
clients on the pediatrics unit. 2. Participating in health promotion programs, such as
5. Completing incident/irregular occurrence/variance teaching a class on accident prevention to parents
reports as needed, including filing, monitoring, of infants and young children.
and analyzing reports of drug reactions to a new 3. Conducting health screening as needed, such as
medication. performing the Denver II developmental assess-
6. Working toward continuous quality improvement, ment on age-appropriate clients (1 month to
such as studying incident reports of accidents on 6 years) in the pediatric clinic as part of routine
PEDIATRICS

the pediatrics unit to determine what changes in screening.


practice are needed to increase client safety. 4. Teaching families about and administering
7. Suggesting organ donation to selected families immunizations as orderedsigns/symptoms
when appropriatefor example, asking the of reactions, providing parents with an immu-
family whose 16-year-old child has been nization record card after administering
declared brain dead about the possibility of immunization(s).
organ donation. 5. Discussing lifestyle choices with older children and
8. Collaborating with other members of the health- adolescents, as appropriate (e.g., discussing risks
care team in consultation and referrals, such as associated with body piercing and tattooing).
requesting a psychiatric consultation for a child
who is depressed. Coping and Adaptation in
9. Coordinating resource management, such as Infants, Children,
reminding the unlicensed assistive personnel and Adolescents
(UAP) to stamp all equipment charge forms used
The role of the pediatric nurse in coping and adaptation
for each child.
includes the following:
Safety and Infection 1. Understanding and respecting religious
Control for Infants, and spiritual influences on childrens health
Children, and Adolescents (e.g., allowing families to engage in dying and
death rituals according to their spiritual beliefs)
The role of the pediatric nurse in safety and infection con-
(see Table 5.1).
trol includes the following:
2. Helping clients engage in problem-solving
1. Engaging in disaster planning activitiesfor exam- related to situational role changes, such as dis-
ple, the nurse reads and thoroughly reviews the cussing transportation and child care issues with
disaster plan for the health-care facility in which the parents of a child who is newly diagnosed
the nurse is practicing. with leukemia.
2164_Ch05_249-346 29/03/12 12:24 PM Page 251

Growth and Development 251

Table 5.1
Stages of Spiritual Development in Childhood
Infancy No concept of right/wrong
(Stage 0: undifferentiated) No convictions to guide behavior
Beginnings of a faith are established with the development of basic trust through relation-
ships with the primary caregiver
Toddler/Preschooler Toddlers:
(Stage 1: intuitive-projective) Imitate the religious gestures and behaviors of others without comprehending the mean-
ing or significance of the activities
Preschoolers:
Assimilate some of the values and beliefs of their parents
Parents attitudes toward moral codes and religious beliefs convey to child what they
consider good and bad
Follow parental beliefs as part of daily lives rather than through an understanding of their
basic concepts
School-age Spiritual development parallels cognitive development
(Stage 2: mythical-literal) Closely related to childs experiences and social interactions
Develop strong interest in religion
Accept existence of a deity
Petitions to an omnipotent being are important and expected to be answered
Good behavior is rewarded/bad behavior is punished
Conscience is bothered when they disobey
Have reverence for many thoughts and matters
Able to articulate own faith
May even question validity of faith
Preadolescent Become increasingly aware of spiritual disappointments
(Stage 3: synthetic-convention) Recognize that prayers are not always answered (at least on their own terms)
Begin to reason, to question some of the established parental religious standards
May drop or modify some religious practices
Adolescent Become more skeptical
(Stage 4: individuating-reflexive) Begin to compare parents religious standards with the standards of others

PEDIATRICS
Attempt to determine which to adopt and incorporate into their own set of values
Begin to compare religious standards with the scientific viewpoint
Time of searching rather than reaching
Uncertain about many religious ideas but will not achieve profound insights until late
adolescence or early adulthood

GROWTH AND c. Fear of strangers is normalindicates attach-


DEVELOPMENT ment between infant and primary caregiver.
d. Child may repeat over and over newly learned
I. INFANT (28 DAYS TO 1 YEAR) behaviors (e.g., sitting or standing).
A. Eriksons theory of personality development e. Weaning can begin around the time child
1. Central task: basic trust vs. mistrust; central begins walking.
person: primary caregiver/maternal person. f. Review Preventive Care Timeline and
2. Behavioral indicators Recommended Health Screenings (Tables 5.2
a. Crying is only means of communicating and 5.3) with parents of infants.
needs. 4. For additional information about behavioral con-
b. Quieting usually means needs are met. cerns for each age group, see Tables 5.4 and 5.5.
c. Fear of strangers at 6 to 8 months. B. Physical growth (by 1 year)
3. Parental guidance/teaching 1. Height (length): 50% increase by first birthday.
a. Must meet infants needs consistentlycannot 2. Weight
spoil infant by holding, comforting. a. Doubles by 4 to 7 months, triples by 1 year.
b. Neonatal reflexes fade between 4 and 6 b. Gains 5 to 7 oz/wk in first 6 months of life.
months, replaced with increase in purposeful c. Gains 3 to 5 oz/wk in second 6 months of life.
behavior (e.g., babbling, reaching). 3. Vital signs: see Table 5.6.
(text continues on page 255)
2164_Ch05_249-346 29/03/12 12:24 PM Page 252

Table 5.2
Child Preventive Care Timeline
Clinical Preventive Services for Normal-Risk Children
Month/
Years of Age B 1m 2m 4m 6m 12m 15m 18m 2y 3y 4y 6y 7-10y 11-12y 13-18y
Hepatitis B (HepB) 1 2 3
Rotavirus (RV) 1 2 3
Polio (IPV)* 1 2 3 4
Haemophilus
influenzae type B 1 2 3 4
(HIB)*
Pneumococcal
IMMUNIZATION

1 2 3 4 High-Risk Groups
Disease (PCV)
Diphtheria, Tetanus,
1 2 3 4 5 Tdap
Pertussis (DTaP, Tdap)
Measles, Mumps,
1 2
Rubella (MMR)
Chickenpox (VZV) 1 2
Hepatitis A (Hep A) 2 Doses High-Risk Groups
Influenza Yearly
Meningococcal
High-Risk Groups Once
(MCV)
Human
3 Doses
Papillomavirus (HPV)
Years of Age B 1y 2y 3y 4y 5y 6y 7y 8y 9y 10y 11y 12y 13y 14-18y
Newborn Screening:
PKU, Sickle cell
hemoglobinopathies,
Hypothyroidism
PEDIATRICS

Hearing
Head Circumference Periodically
SCREENING

Height and Weight Periodically


Lead
Vision Screening
Blood Pressure Periodically
Dental Health Periodically
Alcohol Use Adolescents
Chlamydia Adolescents
Years of Age B 1y 2y 3y 4y 5y 6y 7y 8y 9y 10y 11y 12y 13y 14-18y
Development,
nutrition, physical
activity, safety,
COUNSELING

unintentional injuries
and poisonings,
As appropriate for age
violent behaviors
and firearms, STIs
and HIV, family
planning, tobacco
use, drug use
Recommended by most U.S. authorities. Recommended by CDC for high-risk groups.
This is an example of a typical schedule that indicates the recommended ages for routine administration of currently
licensed childhood vaccines for children through age 18 years. Any dose not administered at the recommended
age should be administered at any subsequent visit when indicated and feasible. Keep current on vaccine recom-
mendations by going to the Centers for Disease Control and Prevention Web site: www.cdc.gov/vaccines.
*Schedules may vary according to vaccine type.
The human papillomavirus vaccine may also be given in three doses to males between the ages of 9 and

19 years to prevent genital warts. On October 25, 2011, CDC recommended that boys get HPV also.
Modified from Child Health Guide: Put Prevention into Practice. U.S. Department of Health and Human Services, Agency
for Healthcare Research and Quality, updated January 2003. Modified from Recommended Immunization Schedule for
Persons Aged 0 Through 6 Years and from Recommended Immunization Schedule for Persons Aged 7 Through
10 Years, U.S. Department of Health and Human Services, Centers for Disease Control and Prevention, 2011.
2164_Ch05_249-346 29/03/12 12:24 PM Page 253

Growth and Development 253

Table 5.3
Recommended Health Screenings for Infants, Children, and Adolescents: Specific Conditions
Area of Concern Screening Method Recommendations
Neonatal metabolic and Blood tests: All are done in immediate neonatal period, within first few days of
genetic screening: life. Testing for PKU must be done after infant has ingested formula
Hypothyroidism Serum T3/T4 levels or breast milk for 4872 hours.
Sickle cell anemia Sickledex
PKU Phenylalanine levels
Lead poisoning (see Blood lead level (BLL) Centers for Disease Control and Prevention (CDC) recommends
Table 5.2) universal or targeted screening for all children.
At a minimum, all children should have BLL drawn between
12 years, or earlier if needed.
BLL should also be done on any child between 36 years who
has never been tested.
Those children at high risk (e.g., live in older home with lead in
paint and plumbing, have sibling or friend with lead poisoning)
should be screened earlier and more frequently.
Hyperlipidemia Serum cholesterol levels Routinely at 24 years, 6 years, 10 years, 1114 years,
1517 years, and 1821 years.
May be done earlier or more frequently with risk factors such as
diabetes, hypertension, parent with high cholesterol level.
Cystic fibrosis Genetic studies, sweat test Screen children who have sibling or other family members with CF.
Screen family members of child with CF.
Tuberculosis Mantoux or PPD First test done at age 1215 months; repeated prn based on risk
and exposure.
Latex allergies Health history During each routine visit, but especially important preoperatively or
before procedures or children with ongoing urinary catheterizations
(e.g., in myelomeningocele).

PEDIATRICS
Table 5.4
Pediatric Behavioral Concerns: Nursing Implications and Parental Guidance
Behavioral Concern Nursing Implications/Parental Guidance
Teething Begins around age 4 monthsinfant may seem unusually fussy and irritable but should not run a fever.
Provide relief with teething rings, acetaminophen, topical preparations.
Thumb sucking Need to suck varies: may be due to hunger, frustration, loneliness.
Do not stop infant from doing thisusually stops by preschool years.
If behavior persists, evaluate need for attention, peer play.
Temper tantrums Normal in the toddleroccurs in response to frustration.
Avoid abrupt end to play or making excessive demands.
Offer only allowable choices.
Once a decision is verbalized, avoid sudden changes of mind.
Provide diversion to achieve cooperation.
If it occurs, best means to handle is to ignore the outburst.
Toilet training Assess child for readiness: awareness of body functions, form of mutual communication, physical
control over sphincters.
Use child-size seat.
No distractions (food, toys, books).
Offer praise for success or efforts (never shame accidents).
Discipline Not for infant.
Can begin with toddler, within limits.
Be consistent and clear.
Avoid excessively strict measures.
Sibling rivalry Fairly common, normal.
Allow older child to help.
Give each child special time, with individual attention.
Continued
2164_Ch05_249-346 29/03/12 12:24 PM Page 254

254 chapter 5 Health Promotion and Maintenance

Table 5.4
Pediatric Behavioral Concerns: Nursing Implications and Parental Guidancecontd
Behavioral Concern Nursing Implications/Parental Guidance
Masturbation Normal, common in preschooler.
Set firm limits.
Avoid overreacting.
Lying In preschooler: not deliberate; child is often unable to differentiate between real and lie, and by
saying something often feels it makes a thing real.
In older child: may indicate problems and need for professional attention if persists.
Serve as role modelno white lies.
Cursing Avoid overreacting.
Defuse use of the word by simply stating not here, not now.
Distract, change subject, substitute activity.
Serve as role model by own language.
Accidents (enuresis) Occasionalcommon and normal through preschool.
If frequentneed complete physical examination to rule out pathology.
Training: after dinneravoid fluids; before bedtoilet (perhaps awaken once during night).
Never put back into diapers or attempt to shame.
Smoking/drinking May begin in older school-age child or adolescent.
Serve as role model with own habits.

Table 5.5
Pediatric Sleep and Rest Norms: Nursing Implications and Parental Guidance
Pediatric Sleep and Rest Norms Nursing Implications/Parental Guidance
INFANT: 1620 hr/day
3 months: nocturnal pattern No set schedule can be predetermined.
PEDIATRICS

6 months: 12 naps, with 12 hr at night If waking at night after age 3 mo, investigate hunger as a probable cause.
12 months: 1 nap, with 12 hr at night Monitor behavior to determine sleep needs: Alert, active? Growing, developing?
Routine fairly well established.
TODDLER: 1214 hr/night
Dawdles at bedtime. Set firm, realistic limits.
Dependency on security object. Place favorite blanket or toy in crib/bed.
May ask to sleep with bottle. Avoid bottle mouth syndrome (caries).
May rebel against going to sleep. Establish bedtime ritual.
PRESCHOOL: 1012 hr/night
Gives up afternoon nap. May regress in behavior when tired; provide quiet time in place of nap.
Difficulty falling asleep/nighttime waking. Avoid overstimulation in evening.
Fear of dark. Leave night-light on, door open.
Enuresis. Occasional accidents are normal.
May begin to have nightmares. Comfort child but leave in own bed.
SCHOOL-AGE: 812 hr/night
Nightmares common. Comfort child but leave in own bed.
Awakens early in morning. Important that child play/relax after school.
May not be aware he or she is tired. Remind about bedtime.
Likes to stay up late. Privilege of later bedtime can be awarded as child gets older.
Slumber parties. Permit, as good opportunity to socialize.
ADOLESCENT: 1014 hr/night
Need for sleep increases greatly. Needs vary greatly among individuals.
May complain of excessive fatigue. Related to rapid growth rate and overall increased activity.
2164_Ch05_249-346 29/03/12 12:24 PM Page 255

Growth and Development 255

Table 5.6
Normal Vital Signs: Measurements and Variations with Age
Age (yr) Heart Rate (beats/min) Respiratory Rate (breaths/min) Blood Pressure (mm Hg)
Newborn 120160 3040 70/55
1 100140 2535 90/55
2 80120 2030 90/56
5 70100 1824 95/56
10 6090 1822 102/62
14 5590 1620 110/65
18 5590 1618 116/68
Adapted from Hockenberry, M, & Wilson, D: Wongs Nursing Care of Infants and Children, ed 8. Mosby, St. Louis,
2007.

4. Cardiac system 11. Teething


a. Heart begins to function effectively. a. Generally begins around 6 months.
b. Decreased heart rate, increased blood b. First two teeth: lower central incisors.
pressure from neonatal values. c. By 1 year: six to eight teeth.
5. Pulmonary system C. Denver II Developmental Assessment: See
a. Predisposed to upper respiratory infections Table 5.7.
due to anatomical differences (e.g., eustachi- 1. Birth to 3 months
an tube is shorter and straighter in infant). a. Personal-social: smiles responsively, then
b. Decreased respiratory rate from neonatal spontaneously.
values. b. Fine motor-adaptive:
6. Gastrointestinal system (1) Follows 180 degrees, past midline.
a. Swallowing improves. (2) Grasps rattle.
b. Stomach enlarges to hold greater volume. (3) Holds hands together.

PEDIATRICS
c. Digests more complex foods as enzymes c. Language: laughs/squeals; vocalizes without
increase (by 46 months). crying.
7. Genitourinary system d. Gross motor: while on stomach, lifts
a. Immature; waste products poorly head 45 to 90 degrees, able to hold head
eliminated. steady and erect; rolls over, from stomach
b. Easily prone to fluid and electrolyte to back.
imbalances. 2. 4 to 6 months
8. Immune system a. Personal-social: works for toy; feeds self
a. Functional by 2 months. (bottle).
b. Produces IgG and IgM antibodies. b. Fine motor-adaptive: palmar grasp, reaches
9. Neurological system for objects.
a. Fontanels: c. Language: turns toward voice, imitates speech.
(1) Anterioropen or patent through first d. Gross motor: some weight-bearing on legs;
1218 months. no head lag when pulled to sitting; sits with
(2) Posteriorclosed by 2 months. support.
b. Head circumference increases as brain grows 3. 7 to 9 months
rapidly. a. Personal-social
c. Neurological reflexes (e.g., Landau, parachute) (1) Indicates wants.
appear; neonatal reflexes (e.g., Moro, rooting) (2) Plays pat-a-cake, waves bye-bye.
disappear. b. Fine motor-adaptive: takes two cubes in
10. Sensory hands and bangs them together; passes cube
a. Hearing improves from quieting to a hand to hand; crude pincer grasp.
sound, to locating a sound easily and c. Language: dada, mama, nonspecific,
turning toward it. jabbers.
b. Vision improves from 8 to 18 inches, to d. Gross motor: creeps on hands and knees; gets
searching for hidden objects and following self up to sitting; pulls self to standing; stands
moving objects. holding on.
2164_Ch05_249-346 29/03/12 12:24 PM Page 256

256 chapter 5 Health Promotion and Maintenance

Table 5.7
Facts About the DENVER II Developmental Assessment
Parents Questions Nurses Best Response
Will this be used as a measure of my childs IQ? No, it is a screening test for your childs development.
What ages can be tested? Infants through preschoolers, or from 1 month to 6 years.
What will they test? There are four areas: personal-social, fine motor-adaptive, language, gross
motor.
Can I stay with my child? Yes, in fact it is preferred you be there.
If my child fails, does it mean he is retarded? No, this is not a diagnostic tool but rather a screening test.
If he fails, what do we do? Repeat the test in a week or two to rule out temporary factors.
Why didnt my child accomplish everything? He is not expected to.
Why did my child score so poorly? Perhaps its a bad day for the child, he isnt feeling up to par, etc.

4. 10 to 12 months (4) No toys with small, removable parts.


a. Personal-social (5) No balloons or plastic bags.
(1) Plays ball. c. Falls
(2) Imitates activities. (1) Raise crib rails.
(3) Drinks from cup. (2)Never place child on high surface
b. Fine motor-adaptive: neat pincer grasp. unsupervised.
c. Language: dada, mama, specific. (3) Use restraining straps in seats, high
d. Gross motor: stands alone well; walks holding chairs, etc.
on; stoops and recovers. d. Poisoning
D. Nursing interventions/parental guidance, (1) Check that paint on toys/furniture is
teaching: lead-free.
1. Play (2) Treat all medications as drugs, never as
PEDIATRICS

a. First yeargenerally solitary. candy.


b. Visual stimulation (3) Store all poisonous substances in locked
(1) Best color: red. cabinet, closet.
(2) Toys: mirrors, brightly colored pictures. (4) Have telephone number of Poison Control
c. Auditory stimulation Center on hand.
(1) Talk and sing to infant. (5) Instruct in use of syrup of ipecac, if
(2) Toys: musical mobiles, rattles, bells. indicated. The use of syrup of ipecac is
d. Tactile stimulation controversial in children (see p. 312).
(1) Hold, pat, touch, cuddle, swaddle/keep e. Burns
warm; rub body with lotion. (1) Use microwave oven to heat refrigerated
(2) Toys: various textures; nesting and stacking; formula only; heat only 4 oz or more for
plastic milk bottle with blocks to dump about 30 seconds. Test formula on top of
in, out. your hand, not inside wrist.
e. Kinetic stimulation (2) Check temperature of bath water; never
(1) Cradle, stroller, carriage, infant seat, car leave infant alone in bath.
rides, furniture strategically placed for (3) Special care with cigarettes, hot liquids.
walking. (4) Do not leave infant in sun.
(2) Toys: cradle gym, push-pull. (5) Cover all electrical sockets.
2. Safety (6) Keep electrical wires out of sight/reach.
a. Refer to Tables 3.7, 3.8, and 3.9 for addi- (7) Avoid tablecloths with overhang.
tional information on safety and infection (8) Put guards around heating devices.
control. f. Motor vehicles
b. Note: Most common accident during first (1) Use only federally approved car seat for
12 months is the aspiration of foreign objects. all car rides; safest position is rear-facing
(1) Keep small objects out of reach. in middle of back seat (from birth to
(2) Use one-piece pacifier only. 20 pounds, and as close to 1 year of age
(3) No nuts, raisins, hot dogs, popcorn. as possible).
2164_Ch05_249-346 29/03/12 12:24 PM Page 257

Growth and Development 257


(2) Never leave stroller behind parked car. 5. Pulmonary system
(3) Do not allow infant to crawl near parked a. Mainly abdominal breathing.
cars or in driveway. b. Lumina of bronchial vessels in size
II. TODDLER (13 YEARS) decrease in lower respiratory system
infections.
A. Eriksons theory of personality development
c. Decreased respiratory rate from infant values.
1. Central task: autonomy vs. shame and doubt;
6. Gastrointestinal system
central person(s): parent(s)
a. Increased capacity three-meals-a-day
2. Behavioral indicators
feeding schedule.
a. Does not separate easily from parents.
b. Gastric juices increase in acidity decrease
b. Negativistic.
in GI infections.
c. Prefers rituals and routine activities.
c. Possible voluntary control of anal sphincter.
d. Active physical explorer of environment.
7. Genitourinary system
e. Begins attempts at self-assertion.
a. Bladder capacity increases; to determine
f. Easily frustrated by limits.
bladder capacity in ounces, add 2 to the
g. Temper tantrums.
childs age (e.g., 2-year-old has a bladder
h. May have favorite security object.
capacity of 4 oz or 120 mL).
i. Uses mine for everythingdoes not under-
b. Ability to hold urine increases.
stand concept of sharing.
c. Possible voluntary control of urethral
3. Parental guidance/teaching
sphincter.
a. Avoid periods of prolonged separation if
8. Immune system
possible.
a. Possible immunity from intrauterine
b. Avoid constantly saying no to toddler.
life/maternal transfer disappears.
c. Avoid yes/no questions.
b. Gradual increase in IgA, IgD, and IgE
d. Stress that child may use no even when he
antibodies.
or she means yes.
9. Neurological system
e. Establish and maintain rituals (e.g., toilet
a. Anterior fontanel closed by 1218 months.
training, going to sleep).
b. Brain increases to 90% of adult size.
f. Offer opportunities for play, with supervision.
c. Progressive increase in intelligence.
g. Allow child to feed self.

PEDIATRICS
d. Myelinization of spinal cord is complete.
h. Offer only allowable choices.
10. Sensory
i. Best method to handle temper tantrums:
a. Hearing evidences basic auditory skills.
ignore them.
b. Vision shows evidence of convergence and
j. Keep security object with child, if so desired.
accommodation; full binocular vision devel-
k. Do not force toddler to share.
oped; visual acuity: 20/40.
l. Review Preventive Care Timeline and
11. Teething
Recommended Health Screenings (see
a. Introduce toothbrushing as a ritual.
Tables 5.2 and 5.3) with parents of
b. By 30 months: all 20 primary teeth present.
toddlers.
c. First dental checkup should be between 12
4. Additional information about behavioral
and 18 months.
concerns for each age group may be found in
12. Musculoskeletal system
Tables 5.4 and 5.5.
a. Lordosis: abdomen protrudes.
B. Physical growth (by 3 years)
b. Walks like a duck: wide-based gait,
1. Height
side-to-side.
a. Slow, steady growth at 2 to 4 inches/yr,
C. Denver II Developmental Assessment
mainly in legs rather than trunk.
(see Table 5.7)
b. Adult height is roughly twice childs height at
1. 12 to 18 months
2 years of age.
a. Personal-social
2. Weight
(1) Imitates housework.
a. Slow, steady growth at 4 to 6 lb/yr.
(2) Uses spoon, spilling little.
b. Birth weight quadruples by 2.5 years of age.
(3) Removes own clothes.
3. Vital signs: refer to Table 5.6.
(4) Drinks from cup.
4. Cardiac system
(5) Feeds doll.
a. Heart begins to function more efficiently.
b. Fine motor-adaptive
b. Decreased heart rate, slight increase in blood
(1) Scribbles spontaneously.
pressure from infant values.
(2) Builds tower with two to four cubes.
2164_Ch05_249-346 29/03/12 12:24 PM Page 258

258 chapter 5 Health Promotion and Maintenance

c. Language 3. Safety
(1) Three to six words other than mama, a. Refer to Tables 3.7, 3.8, and 3.9 for addi-
dada. tional information on safety and infection
(2) Points to at least one named body part. control.
d. Gross motor b. Accidents are the leading cause of death
(1) Kicks ball forward. among toddlers.
(2) Walks up steps. c. Motor vehicles: most accidental deaths in chil-
2. 19 to 24 months dren under age 3 are related to motor vehicles.
a. Personal-social (1) Use only federally approved car seat for all
(1) Puts on clothing. car rides, through age 8 or 60 lb.
(2) Washes and dries hands. (2) Follow manufacturer directions carefully.
(3) Brushes teeth with help. (3) Make car seat part of routine for toddler.
b. Fine motor-adaptive d. Drowning
(1) Builds tower with four to six cubes. (1) Always supervise child near water:
(2) Imitates vertical line. bathtub, pool, hot tub, lake, ocean.
c. Language (2) Keep bathroom locked to prevent
(1) Combines two or three words. drowning in toilet.
(2) Speech partially understandable. e. Burns
(3) Names picture. (1) Turn pot handles in when cooking.
d. Gross motor (2) Do not allow child to play with electrical
(1) Throws ball overhand. appliances.
(2) Jumps in place. (3) Decrease water temperature in house to
3. 2 to 3 years avoid scald burns.
a. Personal-social f. Poisonings: most common in 2-year-olds.
(1) Puts on T-shirt. (1) Consider every nonfood substance a haz-
(2) Can name a friend. ard and place out of childs sight/reach.
b. Fine motor-adaptive (2) Keep all medications, cleaning materials,
(1) Thumb wiggles. etc., in clearly marked containers in locked
(2) Builds tower of eight cubes. cabinets.
PEDIATRICS

c. Language (3) Instruct in use of syrup of ipecac, if


(1) Knows two verbs and two adjectives. indicated by Poison Control Center
(2) Names one color. (see p. 312).
d. Gross motor g. Falls
(1) Balances on one foot briefly. (1) Provide barriers on open windows.
(2) Pedals tricycle. (2) Avoid gates on stairschild can strangle
D. Nursing interventions/parental guidance: on gate.
1. Play: toddler yearsgenerally parallel. (3) Move from crib to bed.
2. Toysstimulate multiple senses simultaneously: h. Choking: avoid food on which child might
a. Push-pull. choke:
b. Riding toys (e.g., straddle horse or car). (1) Fish with bones.
c. Small, low slide or gym. (2) Fruit with seeds, pits, or skin.
d. Balls, in various sizes. (3) Nuts, raisins.
e. Blocksmultiple shapes, sizes, colors. (4) Hot dogs.
f. Dolls, trucks, dress-up clothes. (5) Chewing gum.
g. Drums, horns, cymbals, xylophones, toy (6) Hard candy.
piano. (7) Coin-cut foods.
h. Pounding board and hammer, clay. III. PRESCHOOLER (35 YEARS)
i. Finger paints, chalk and board, thick
A. Eriksons theory of personality development
crayons.
1. Central task: initiative vs. guilt; central person(s):
j. Wooden puzzles with large pieces.
basic family unit.
k. Toy record player with kiddie records.
2. Behavioral indicators
l. Talking toys: dolls, read-along books,
a. Attempts to perform activities of daily living
phones.
(ADLs) independently.
m. Sand, water, soap bubbles.
b. Attempts to make things for self/others.
n. Picture books, photo albums.
c. Tries to help.
o. Nursery rhymes, songs, music.
2164_Ch05_249-346 29/03/12 12:24 PM Page 259

Growth and Development 259


d. Talks constantly: verbal exploration of the 6. Genitourinary system
world (Why?). a. Bladder remains palpable above symphysis
e. Extremely active, highly creative imagination: pubis.
fantasy and magical thinking. b. Needs to void frequently, or accidents may
f. May demonstrate fears: monsters, dark occur despite sphincter control.
rooms, etc. c. Low-grade urinary tract infections common.
g. Able to tolerate short periods of separation. 7. Immune system
3. Parental guidance/teaching a. Growth of lymphoid tissue (especially tonsils).
a. Encourage child to dress self by providing b. Illnesses (especially respiratory) tend to be
simple clothing. more localized.
b. Remind to go to bathroom (tends to forget). c. Continued increase in IgA and IgG.
c. Assign small, simple tasks or errands. 8. Neurological system
d. Answer questions patiently, simply; do not a. Handedness (right or left) is established.
offer child more information than what the 9. Sensory
child is asking for. a. Visionfar-sightedness is improving; visual
e. Normal to have imaginary playmates. acuity: 20/30.
f. Offer realistic support and reassurance with b. Vision and hearing screening should be con-
regard to fears. ducted before kindergarten, and annually
g. Expose to a variety of experiences: zoo, train thereafter.
ride, shopping, sleigh riding, etc. 10. Teeth
h. Enroll in preschool/nursery school program; a. All 20 primary or deciduous teeth (baby
kindergarten at 5 years. teeth) should be present.
i. Review Preventive Care Timeline and Recom- b. Annual dental checkups; continue daily
mended Health Screenings (see Tables 5.2 and brushing.
5.3) with parents of preschoolers. C. Denver II Developmental Assessment/
4. Additional information about behavioral developmental norms: see Table 5.7.
concerns for each age group may be found in 1. 3 years
Tables 5.4 and 5.5. a. Personal-social
B. Physical growth (by 5 years) (1) Dresses without help.

PEDIATRICS
1. Height and weight (2) Plays board/card games.
a. Continued slow, steady growth. b. Fine motor-adaptive
b. Generally grows more in height than weight. (1) Picks longer of two lines.
c. Posture: appears taller and thinner; lordosis (2) Copies circle, intersecting lines.
of toddler gradually disappears. (3) Draws person, three parts.
2. Vital signs: see Table 5.6. c. Language
3. Cardiac system (1) Comprehends cold, tired, hungry.
a. Increased heart size (4 times larger than at (2) Comprehends prepositions: over,
birth); heart function is comparable to a under.
healthy adult (by 5 years). (3) Names four colors.
b. Heart assumes vertical position in thoracic d. Gross motor
cavity. (1) Pedals tricycle; hops, skips on alternating
c. May hear splitting of heart sounds, as well feet.
as innocent murmurs on auscultation. (2) Broad jumps, jumps in place.
d. Decreased heart rate, steady blood pressure (3) Balances on one foot.
from toddler values. 2. 4 years
4. Pulmonary system a. Personal-social
a. Increase in amount of lung tissue. (1) Brushes own teeth, combs own hair.
b. Adult-like lung sounds heard on auscultation. (2) Dresses without supervision.
c. Decreased respiratory rate from toddler values. (3) Knows own age and birthday.
5. Gastrointestinal system (4) Ties own shoes.
a. Continued increase in size. b. Fine motor-adaptive
b. Position is straighter and more upright than (1) Draws person with six body parts.
adult stomach more rapid emptying (2) Copies square.
(defecation, vomiting). c. Language
c. Lining still sensitive to roughage and spices. (1) Knows opposite analogies (two of three).
d. Elimination controlled. (2) Defines seven words.
2164_Ch05_249-346 29/03/12 12:24 PM Page 260

260 chapter 5 Health Promotion and Maintenance

d. Gross motor 2. Behavioral indicators


(1) Balances on each foot for 5 seconds. a. Moving toward complete independence in
(2) Can walk heel-to-toe. ADLs.
3. 5 years b. May be very competitivewants to achieve in
a. Personal-social school, at play.
(1) Interested in money. c. Likes to be alone occasionally, may
(2) Knows days of week, seasons. seem shy.
b. Fine motor-adaptive d. Prefers friends and peers to siblings.
(1) Prints name. 3. Parental guidance/teaching
c. Language a. Be accepting of the child as he or she is.
(1) Counts to 10. b. Offer consistent support and guidance.
(2) Verbalizes number sequences c. Avoid authoritative or excessive demands on
(e.g., telephone number). child.
d. Gross motor d. Respect need for privacy.
(1) Attempts to ride bike. e. Assign household tasks, errands, chores.
(2) Rollerskates, jumps rope, bounces ball. f. Review Preventive Care Timeline and
(3) Backward heel-toe walk. Recommended Health Screenings (see
D. Nursing interventions/parental guidance: Tables 5.2 and 5.3) with parents of
1. Play: preschool yearsassociative and school-age children.
cooperative. 4. Additional information about behavioral
a. Likes to play house, work, school, concerns for each age group may be found in
firehouse. Tables 5.4 and 5.5.
b. Arts and crafts: color, draw, paint dot-to-dot, B. Physical growth (by 12 years)
color by number, cut and paste, simple sewing 1. Height and weight
kits. a. Almost double in weight from 6 to 12 years.
c. Ball, rollerskate, jump rope, jacks. b. Period of slow, steady growth.
d. Swimming. c. 1 to 2 inches/yr.
e. Puzzles, blocks (e.g., Lego blocks). d. 3 to 6 lb/yr.
f. Tricycle, then bicycle (with/without training e. Girls and boys differ in size at end of
PEDIATRICS

wheels). school-age years.


g. Simple card games and board games. 2. Vital signs: refer to Table 5.6.
h. Costumes and dress-up: make-believe. 3. Cardiac system
2. Safety: Emphasis shifts from protective supervi- a. Increased size of left ventricle (to meet
sion to teaching simple safety rules. Preschoolers demands for increased blood to growing
are the great imitators of parents, who now structures).
serve as role models. b. Decreased heart rate, increased blood
a. Refer to Tables 3.7, 3.8, and 3.9 for addi- pressure from preschool values.
tional information on safety and infection 4. Pulmonary system
control. a. Front sinuses develop (by 7 years).
b. Teach child car/street safety rules. b. Lymphatic tissue completes growth
c. Convertible safety seats should be used until (by 9 years).
child weighs at least 40 lb. c. Remains well oxygenated on exertion.
d. Teach child not to go with strangers or accept d. Continued decrease in respiratory rate from
gifts or candy from strangers. preschool values; using intercostal muscles
e. Teach child danger of fire, matches, flame: more effectively for breathing.
drop and roll. 5. Gastrointestinal system
f. Teach child rules of water safety; provide a. Higher metabolic rate requires adequate food
swimming lessons. and fluids to ensure nutrition and hydration.
g. Provide adult supervision, frequent checks on b. Food retained in stomach for longer periods
activity/location. Despite safety teaching, pre- of time.
schooler is still a child and may be unreliable. 6. Genitourinary system
IV. SCHOOL AGE (612 YEARS) a. Bladder capacity increase continues.
b. Kidneys mature.
A. Eriksons theory of personality development
c. Less likely to have fluid and electrolyte (F/E)
1. Central task: industry vs. inferiority; central per-
imbalance as increased conservation of water
son(s): school, neighborhood friend(s).
occurs.
2164_Ch05_249-346 29/03/12 12:24 PM Page 261

Growth and Development 261


7. Immune system D. Nursing interventions/parental guidance:
a. Growth of lymphoid tissue increases, then 1. Play
plateaus, then decreases. a. Wants to win, likes competitive games.
b. Continued improvement noted with bodys b. Prefers to play with same-sex children.
increased ability to localize infections. c. Enjoys group, team play.
8. Neurological system d. Loves to do magic tricks and other show-off
a. Central nervous system matures. activities (e.g., puppet shows, plays, singing).
b. Myelinization continues increase in both e. Likes to collect things: cards, compact discs.
fine motor-adaptive and gross motor skills. f. Does simple scientific experiments, computer
9. Sensory games.
a. Vision: 20/20 vision well established between g. Has hobbies: needlework, woodwork, models.
9 and 11 years. h. Enjoys pop music, musical instruments,
b. Should be screened for vision/hearing annually, videos, posters.
usually in school. 2. Safety (see Table 3.7)
10. Teeth a. As passenger: use specially designed car
a. Begins to lose primary teeth around sixth restraints until age 8 or 60 lbs, then safety
birthday. belts. Teach child not to distract driver.
b. Eruption of permanent teeth, including b. As pedestrian: teach bike, street safety.
molars; 28 permanent teeth (by 12 years). c. Teach how to swim, rules of water safety.
c. Dental screening annually, daily brushing. d. Sports: teach safety rules.
11. Pubescence (preliminary physical changes of e. Adult supervision still necessary; serve as role
adolescence) model for safe activities.
a. Average age of onset: girls at 10, boys at 12. f. Teach about stranger danger and online
b. Beginning of growth spurt. safety.
c. Some sexual changes may start to occur. g. Suggest Red Cross courses on first aid, water
C. Developmental norms safety, babysitting, etc.
1. 6 to 8 years h. Refer to Tables 3.7, 3.8, and 3.9 for additional
a. Dramatic, exuberant, boundless energy. information on safety and infection control.
b. Alternating periods: quiet, private V. ADOLESCENT (1218 YEARS)

PEDIATRICS
behavior.
A. Eriksons theory of personality development
c. Conscientious, punctual.
1. Central task: identity vs. role confusion; central
d. Wants to care for own needs but needs
person(s): peer group.
reminders, supervision.
2. Behavioral indicators
e. Oriented to time and space.
a. Changes in body image related to sexual
f. Learns to read, tell time, follow map.
development.
g. Interested in moneyasks for allowance.
b. Awkward and uncoordinated in the beginning.
h. Eagerly anticipates upcoming events, trips.
c. Much interest in opposite sex: girls become
i. Can bicycle, swim, play ball.
romantic.
2. 9 to 11 years
d. Wants to be exactly like peers.
a. Worries over tasks; takes things seriously, yet
e. Becomes hostile toward parents, adults, family.
also developing sense of humorlikes to tell
f. Concerned with vocation, life after high
jokes.
school.
b. Keeps room, clothes, toys relatively tidy.
3. Parental guidance/teaching
c. Enjoys physical activity, has great stamina.
a. Offer firm but realistic limits on behavior.
d. Very enthusiastic at work and play; has lots
b. Continue to offer guidance, support.
of energymay fidget, drum fingers, tap
c. Allow child to earn own money, control own
foot.
finances.
e. Wants to work to earn money: mow lawn,
d. Assist adolescent to develop positive self-image.
baby-sit, deliver papers.
e. Review Preventive Care Timeline and
f. Loves secrets (secret clubs).
Recommended Health Screenings (see
g. Very well behaved outside own home
Tables 5.2 and 5.3) with parents of
(or with company).
adolescents.
h. Uses tools, equipment; follows directions,
4. Additional information about behavioral
recipes.
concerns for each age group may be found in
i. By 12th birthday: paradoxical stormy
Tables 5.4 and 5.5.
behavior, onset of adolescent conflicts.
2164_Ch05_249-346 29/03/12 12:24 PM Page 262

262 chapter 5 Health Promotion and Maintenance

B. Physical growth (by 18 years) 2. Cognitive


1. Height and weight a. Academic ability and interest vary greatly.
a. Adolescent growth spurt lasts 24 to 36 months. b. Think about thinkingperiod of
b. Growth in height commonly ceases at 16 to introspection.
17 years in girls, 18 to 20 years in boys. 3. Emotional
c. Boys gain more weight than girls, are generally a. Same-sex best friend, leading to strong friend-
taller and heavier. ship bonds.
2. Vital signs approximately those of the adult b. Highly romantic period for boys and girls.
(see Table 5.6). c. May be moody, unpredictable, inconsistent.
3. Cardiac system 4. Social
a. Increased heart size and strength to near-adult a. Periods of highs and lows, sociability and
values. loneliness.
b. Decreased heart rate, increased blood pressure b. Turmoil with parentsrelated to changing
from school-age values. roles, desire for increased independence.
4. Pulmonary system c. Peer group is important socializing agent
a. Lungs increase in size to near-adult levels, conformity increases sense of belonging.
but not as rapidly as other body systems d. Friendships: same-sex best friend advancing
(may explain lack of energy). to heterosexual relationships.
b. System is mature by 12 years. D. Nursing interventions/parental guidance:
c. Continued decreased respiratory rate from 1. Play
school-age values. a. School-related group activities and sports.
5. Gastrointestinal system b. Develops talents, skills, and abilities.
a. Continued need for increased calories. c. Televisionwatches soap operas, romantic
6. Genitourinary system movies, sports.
a. Fully developed; bladder can hold 700 mL. d. Develops interest in art, writing, poetry,
7. Immune system musical instrument.
a. Fully developed; infections become increasingly e. Girls: increased interest in makeup and clothes.
rare in an adolescent who is healthy. f. Boys: increased interest in mechanical and
b. Lymphoid tissue matures and regresses. electronic devices.
PEDIATRICS

8. Neurological system 2. Safety (see Table 3.7)


a. Fully developed. a. Motor vehicles (cars and motorcycles)as
9. Sensory passenger or as driver.
a. Fully developed. b. Encourage driver education; serve as positive
10. Teeth: 32 permanent teeth by 18 to 21 years. role model.
11. Sexual changes c. Teach rules of safety for water sports.
a. Girls d. Teach about stranger danger and online
(1) Changes in nipple and areola; development safety.
of breast buds. e. Wants to earn money but still needs guidance:
(2) Growth of pubic hair. advocate safe job, reasonable hours.
(3) Change in vaginal secretions.
(4) Menstruationaverage onset between 12
and 13 years of age; range 8 to 15 years.
D E V E L O P M E N TA L
(5) Growth of axillary hair.
DISABILITIES
(6) Ovulation. I. DOWN SYNDROME
b. Boys A. Introduction: Down syndrome (trisomy 21) is a
(1) Enlargement of genitalia. chromosomal abnormality involving an extra chro-
(2) Growth of pubic, axillary, facial, and body mosome #21 and resulting in 47 chromosomes
hair. instead of the normal 46 chromosomes. As a con-
(3) Lowering of voice. sequence, the child usually has varying degrees of
(4) Production of sperm; nocturnal emission mental retardation, characteristic facial and physi-
(wet dreams). cal features, and other congenital anomalies. Down
C. Developmental norms syndrome is the most common chromosomal dis-
1. Motor development order, occurring in approximately 1 of 800 to
a. Early (1215 years)awkward, uncoordinated, 1000 live births. Perinatal risk factors include
poor posture, decrease in energy and stamina. advanced maternal age, especially with the first
b. Later (1518 years)increased coordination pregnancy (although average maternal age is now
and better posture; more energy and stamina. 25-28 years for an infant with Downs); paternal
2164_Ch05_249-346 29/03/12 12:24 PM Page 263

Developmental Disabilities 263


age is thought to be a related factor. Multiple 4. Self-care deficit related to Down syndrome.
causality is suspected. 5. Altered family processes related to birth of an
B. Assessment: infant with a congenital defect.
1. Physical characteristics 6. Knowledge deficit related to Down syndrome.
a. Brachycephalic (small, round head) with D. Nursing care plan/implementation:
oblique palpebral fissures (almond-shaped 1. Goal: prevent physical complications.
eyes) and Brushfield spots (speckling of iris) a. Respiratory
depressed nasal bridge (saddle nose) and (1) Use bulb syringe to clear nose, mouth.
small, low-set ears. (2) Vaporizer.
b. Mouth (3) Frequent position changes.
(1) Small oral cavity with protruding tongue (4) Avoid contact with people with upper res-
causes difficulty sucking and swallowing. piratory infections.
(2) Delayed eruption/misalignment of teeth. b. Aspiration
c. Hands (1) Small, more frequent feedings.
(1) Clinodactylyincurved little finger. (2) Burp well during/after infant feedings.
(2) Simian creasetransverse palmar crease. (3) Allow sufficient time to eat.
d. Muscles: hypotonic (floppy baby) with (4) Position after meals: head of bed elevated,
hyperextensible joints. right sideor on stomach, with head to
e. Skin: dry, cracked. side.
2. Genetic studies reveal an extra chromosome #21 c. Observe for signs and symptoms of: heart dis-
(trisomy 21). ease, constipation/GI obstruction, leukemia,
3. Intellectual characteristics thyroid dysfunction.
a. Mental retardationvaries from severely 2. Goal: meet nutritional needs.
retarded to low-average intelligence. a. Suction (before meals) to clear airway.
b. Most fall within trainable range, or IQ of b. Adapt feeding techniques to meet special
36 to 51 (moderate mental retardation). needs of infant/child (e.g., use long, straight-
4. Congenital anomalies/diseases handled spoon).
a. 40% to 45% have congenital heart defects: c. Monitor height and weight.
mortality highest in clients with Down d. As child grows, monitor caloric intake (tends

PEDIATRICS
syndrome and cyanotic heart disease. toward obesity with advancing age).
b. GI: tracheoesophageal fistula (TEF), e. Offer foods high in bulk to prevent constipa-
Hirschsprungs disease. tion related to hypotonia.
c. Thyroid dysfunction, especially 3. Goal: promote optimal growth and development.
hypothyroidism. a. Encourage parents to enroll infant/toddler in
d. Visual defects: cataracts, strabismus. early stimulation program and to follow
e. Hearing loss. through with suggested exercises at home.
f. Increased incidence of leukemia. b. Preschool/school-age: special education classes.
5. Growth and development c. Screen frequently, using Denver II to monitor
a. Slow growth, especially in height. development.
b. Delay in developmental milestones. d. Help parents focus on normal or positive
6. Sexual development aspects of infant/child.
a. Delayed or incomplete. e. Help parents work toward realistic goals with
b. Womensmall number have had offspring their child.
(majority have had abnormality). 4. Goal: health teaching.
c. Meninfertile. a. Explain that tongue-thrust behavior is normal
7. Aging and that child should be re-fed.
a. Premature aging, with shortened life b. Before adolescencecounsel parents and
expectancy. child about delay in sexual development,
b. Deathgenerally related to respiratory com- decreased libido, marriage and family
plications: repeated infections, pneumonia, relations.
lung disease. c. In severe cases, assist parents to deal with
C. Analysis/nursing diagnosis: issue of placement/institutionalization.
1. Risk for aspiration related to hypotonia. E. Evaluation/outcome criteria:
2. Altered nutrition, less than body requirements, 1. Physical complications are prevented.
related to hypotonia or congenital anomalies. 2. Adequate nutrition is maintained.
3. Altered growth and development related to Down 3. Child attains optimal level of growth and
syndrome. development.
2164_Ch05_249-346 29/03/12 12:24 PM Page 264

264 chapter 5 Health Promotion and Maintenance

II. ATTENTION DEFICITHYPERACTIVITY b. Answer questions directly, simply.


DISORDER (ADHD); BEHAVIORAL c. Encourage family to verbalize; offer support.
DISORDER (DSM-IV) 2. Goal: provide therapeutic environment using
A. Introduction: As defined by the American principles of behavior modification and/or
Psychiatric Association (APA), this diagnostic term psychotherapy.
includes a persistent pattern of inattention or a. Reduce extraneous or distracting stimuli.
hyperactivity-impulsivity. The exact cause and b. Reduce stress by decreasing environmental
pathophysiology remain unknown. The major expectations (home, school).
symptoms include a greatly shortened attention c. Provide firm, consistent limits.
span and difficulty in integrating and synthesizing d. Special education programs.
information. This disorder is three times more e. Special attention to safety needs.
common in boys than girls, with onset before age 3. Goal: reduce symptoms by means of prescribed
7; the diagnosis is based on the childs history medication.
rather than on any specific diagnostic test. a. Medications: Ritalin and Cylertboth are
B. Assessment: CNS stimulants but have a paradoxical
1. The behaviors exhibited by children with calming effect on the childs behavior.
ADHD are not unusual behaviors seen in Tofranil and Norpraminboth are tricyclic
children. The behavior of children with ADHD antidepressants that action of norepineph-
differs from the behavior of non-ADHD children rine and serotonin in nerve cells, but also
in both quality and appropriateness: can have paradoxical calming effect on
a. Motor activity is excessive. childs behavior. Must monitor for develop-
b. Developmentally younger than ment of tics and arrhythmias.
chronological age. b. Health teaching (child and parents).
2. Inattention (1) Need to take medication regularly, as
a. Does not pay attention to detail. ordered. Avoid taking medication late in
b. Does not listen when spoken to. the day because it may cause insomnia;
c. Does not do what he or she is told to do. monitor neurological and cardiac status.
3. Hyperactivity Assess for appetite weight; avoid
a. Fidgets and squirms excessively. caffeine.
PEDIATRICS

b. Cannot sit quietly. (2) Need for long-term administration, with


c. Has difficulty playing quietly. probable decreased need as child nears
d. Seems to be constantly in motion, moving or adolescence.
talking; always on. 4. Goal: provide safe outlet for excess energy.
4. Impulsiveness a. Alternate planned periods of outdoor play
a. Blurts out answers before question is with schoolwork or quiet indoor play.
completed. b. Channel energies toward safe, large-muscle
b. Has difficulty awaiting turn. Interrupts activities: running track, swimming, bicycling,
others. hiking.
C. Analysis/nursing diagnosis: E. Evaluation/outcome criteria:
1. Altered thought processes related to inattention 1. Family and child verbalize understanding of
and impulsiveness. attention deficit disorders.
2. Impaired physical mobility related to 2. Therapeutic environment enhances socially
hyperactivity. acceptable behavior.
3. Risk for injury related to impulsivity. 3. Medication taken regularly, with behavioral
4. Self-esteem disturbance related to hyperactivity improvements noted.
and impulsivity. 4. Excess energy directed appropriately.
5. Knowledge deficit related to behavioral
modification program, medications, and PSYCHOSOCIAL-
follow-up care. C U LT U R A L
D. Nursing care plan/implementation: FUNCTIONS
1. Goal: teach family and child about ADHD.
a. Provide complete explanation about Refer to Table 5.8 for information on the nursing care of
disorder, probable course, treatment, and hospitalized infants and children as it relates to key devel-
prognosis. opmental differences.
2164_Ch05_249-346 29/03/12 12:24 PM Page 265

Neurological System 265

Table 5.8
Nursing Care of Hospitalized Infants and Children: Key Developmental Differences
Age Assessment: Reaction to Hospitalization Nursing Care Plan/Implementation: Key Nursing Behaviors
Infant Difficult to assess needs, pain Close observation, must look at behavioral cues
Wants primary caretaker Rooming-in
Toddler Separation anxiety Rooming-in
Frustration, loss of autonomy Punching bag, pounding board, clay
Regression Behavior modification
Fears intrusive procedures Axillary temperatures
Preschooler Fearful Therapeutic play with puppets, dolls
Fantasy about illness/hospitalization (may feel Therapeutic play with puppets, dolls
punished, abandoned)
Peak of body mutilation fear Care with dressings, casts, IMs; invasive procedures
Behavior problems: aggressive, manipulative Clear, consistent limits
Regression Behavior modification
School Age Cooperative Use diagrams, models to teach
Quiet, may withdraw Indirect interview: tell story, draw picture
May complain of being bored Involve in competitive game with peer; encourage peers
to call, send get well cards, and visit
Adolescent Fears loss of control Provide privacy; allow to make some decisions
Competitiveafraid of failing Provide tutor prn; get books and homework
Difficulty with body image Provide own clothes; give realistic feedback
Does not want to be separated from peers Telephone in room; liberal visiting; teen lounge
Rebellious behavior Set clear rules; form teen support group

NEUROLOGICAL 5. Altered nutrition, less than body requirements,


SYSTEM related to fever and poor oral intake.
6. Knowledge deficit regarding diagnostic proce-

PEDIATRICS
I. BACTERIAL MENINGITIS dures, condition, treatment, prognosis.
A. Assessment: C. Nursing care plan/implementation:
1. Abrupt onset: initial sign may be a seizure, 1. Goal: prevent spread of infection.
following an episode of upper respiratory a. Institute standard precautions.
infection (URI)/acute otitis media. b. Enforce strict hand washing.
2. Chills and fever. c. Institute and maintain respiratory isolation
3. Vomiting; may complain of headache, neck pain for minimum of 24 hours after starting
(older children). IV antibiotics, at which time child is no
4. Photophobia. longer considered to be communicable
5. Alterations in level of consciousness: delirium, and can be removed from isolation.
stupor, increased intracranial pressure. d. Supervise all visitors in isolation techniques.
6. Nuchal rigidity (older children). e. Identify family members and others at high
7. Opisthotonos position: head is drawn backward risk: do cultures (Haemophilus influenzae,
into overextension; bulging fontanel (most Escherichia coli, etc.); possibly begin prophy-
significant finding in infants). lactic antibiotics (e.g., rifampin). Lumbar
8. Hyperactive reflexes related to CNS irritability. puncture (LP) is the definitive diagnostic test.
B. Analysis/nursing diagnosis: f. Treat with IV antibiotics (as ordered) as soon
1. Risk for infection related to communicability of as possible after admission (after cultures are
meningitis. obtained); continue 10 to 14 days (until
2. Risk for injury related to CNS irritability and cerebrospinal fluid [CSF] culture is negative
seizures. and child appears clinically improved).
3. Pain related to nuchal rigidity, opisthotonos g. Anticipate large-dose IV medications
position, increased muscle tension. onlyadminister slowly in dilute form to
4. Sensory/perceptual alterations related to seizures prevent phlebitis.
and changes in level of consciousness. h. Restrain as needed to maintain IV.
2164_Ch05_249-346 29/03/12 12:24 PM Page 266

266 chapter 5 Health Promotion and Maintenance

2. Goal: promote safety and prevent injury/seizures. 3. Rapidly progressive neurological deterioration:
a. Maintain seizure precautions. Give anticon- a. Cerebral edema and increased intracranial
vulsants, as ordered (e.g., phenytoin). pressure.
b. Place child near nurses station for maxi- b. Alteration in level of consciousness from
mum observation; provide private room lethargy through coma, decerebrate posturing,
for isolation. and respiratory arrest.
c. Minimize stimuli: quiet, calm environment. 4. Liver biopsy reveals liver dysfunction, necrosis,
d. Restrict visitors to immediate family. and failure:
e. Position: Head of bed (HOB) slightly a. Elevated serum alanine aminotransferase
elevated to decrease intracranial pressure. (ALT) (serum glutamic-pyruvic transaminase
(If opisthotonos: side-lying, for comfort [SGPT]), aspartate aminotransferase (AST)
and safety.) (serum glutamic-oxaloacetic transaminase
3. Goal: maintain adequate nutrition. [SGOT]), lactate dehydrogenase (LDH),
a. NPO or clear liquids initially; supplement serum ammonia levels.
with IVs, because child may be unable to b. Severe hypoglycemia.
coordinate sucking and swallowing. c. Increased prothrombin time, coagulation
b. Offer diet for age, as toleratedchild defects, and bleeding.
may experience anorexia (due to disease) C. Analysis/nursing diagnosis:
or vomiting (due to increased intracranial 1. Altered cerebral tissue perfusion related to cerebral
pressure). edema and increased intracranial pressure.
c. Monitor I&O, daily weights. 2. Altered hepatic tissue perfusion related to fatty
D. Evaluation/outcome criteria: degeneration of the liver.
1. No spread of infection noted; immunize all 3. Risk for injury related to coagulation defects and
children against H. influenzae type B bleeding.
2. Safety maintained. 4. Knowledge deficit related to diagnosis, course of
3. Adequate nutrition and fluid intake maintained. disease, treatment, and prognosis.
4. Child recovers without permanent neurological D. Nursing care plan/implementation:
damage (e.g., seizure disorders, hydrocephalus). 1. Goal: reduce intracranial pressure.
II. REYE SYNDROME a. Child is admitted to pediatric intensive
PEDIATRICS

care unit (PICU) for intensive nursing care,


A. Introduction: Reye syndrome, first described as a
continuous observation, and monitoring.
disease entity in the mid-1960s, is a multisystem
b. Monitor neurological status and vital signs
disorder primarily affecting children between 6
continuously.
and 12 years of age. Although not truly a com-
c. Assist with/prepare for numerous invasive
municable disease, studies have confirmed a rela-
procedures, including endotracheal (ET)
tionship between aspirin administration during a
tube/mechanical ventilation and intracranial
viral illness (e.g., chickenpox, flu) and the onset
pressure (ICP) monitor.
of Reye syndrome. The exact cause remains
d. Monitor closely for the development of
unknown. Reye syndrome is characterized by
seizures; institute seizure precautions.
acute metabolic encephalopathy and fatty degen-
e. Position: elevate HOB 30 to 45 degrees.
eration of the visceral organs, particularly the
f. Administer medications as ordered:
liver. Earlier diagnosis, more sophisticated moni-
(1) Osmotic diuretics (e.g., mannitol) to
toring equipment, and more aggressive treatment
decrease ICP.
have greatly improved the survival rate of chil-
(2) Diuretics (e.g., Lasix) to decrease CSF
dren with Reye syndrome. Recovery is generally
production.
rapid in those children who do survive, though
(3) Anticonvulsants (e.g., Dilantin,
they may suffer certain deficits.
phenobarbital).
B. Assessment:
(4) Vitamin K, fresh frozen plasma, or
1. Onset typically follows a viral illness, just as
platelet transfusions for overt or covert
child appears to be recovering.
bleeding.
2. Early signs and symptoms:
2. Goal: restore and maintain fluid and electrolyte
a. Rapidly progressing behavioral changes:
balance, including perfusion of liver.
irritability, agitation, combativeness,
a. Administer IV fluids per physicians order
hostility, confusion, apathy, lethargy.
usually 10% glucose (or higher).
b. Vomiting, which becomes progressively
b. Strict I&O.
worse.
2164_Ch05_249-346 29/03/12 12:25 PM Page 267

Neurological System 267


c. Prepare for/assist with Foley catheter place- 4. Setting sun sign: sclera visible above pupil;
ment, central venous pressure (CVP) monitor, pupils are sluggish, with unequal response to
ICP monitor, nasogastric (NG) tube, etc. light.
d. Monitor serum electrolyte laboratory values. 5. Cry: shrill, high pitched.
3. Goal: prevent injury and possible bleeding. 6. Developmental milestones: delayed.
a. Observe child for petechiae, unusual bruising, 7. Reflexes: persistence of neonatal reflexes;
oozing from body orifices or tubes, frank hyperactive reflexes.
hemorrhage. 8. Feeds poorly.
b. Check all urine and stool for occult blood. 9. Signs of increased ICP:
c. Monitor laboratory values, including pro- a. Vomiting.
thrombin time (PT), partial thromboplastin b. Irritability.
time (PTT), platelets. c. Seizures.
d. Administer blood products per physicians d. Decreased pulse.
order. e. Decreased respirations.
4. Goal: provide parents with thorough understanding f. Increased blood pressure.
of Reye syndrome. g. Widened pulse pressure.
a. Primary nurse assigned to provide care and 10. History may reveal other CNS defects
follow through with teaching. (e.g., spina bifida), infection (e.g., meningitis),
b. Encourage parents presence, even in PICU trauma, or neoplasm.
explain all equipment and procedures in C. Analysis/nursing diagnosis:
simple, direct terms. 1. Altered cerebral tissue perfusion related to
c. Provide factual, honest, and complete increased intracranial pressure.
information regarding disease, diagnosis, 2. Impaired skin integrity related to enlarged head
and prognosis. size and lack of motor coordination.
E. Evaluation/outcome criteria: 3. Altered nutrition, less than body requirements,
1. Intracranial pressure is reduced and normal related to anorexia and vomiting.
neurological functioning is restored. 4. Anxiety related to diagnosis and uncertain
2. Fluid and electrolyte balance is restored. outcome.
3. No clinical evidence of bleeding is found. 5. Knowledge deficit regarding care of the child

PEDIATRICS
4. Parents express understanding of Reye syndrome. with a shunt and follow-up care.
III. HYDROCEPHALUS D. Nursing care plan/implementation:
1. Goal: monitor neurological status.
A. Introduction: Hydrocephalus, known to the
a. Measure head circumference daily, and note
layperson as water on the brain, is actually a
any abnormal increase.
syndrome resulting from disturbances in the
b. Perform neurological checks at least
dynamics of CSF. The accumulation of this fluid
every 4 hours to monitor for signs of
causes enlargement and dilation of the ventricles
increased ICP.
of the brain and increased ICP. If untreated,
c. Report signs of increased ICP STAT to
severe brain damage will result; treatment is a sur-
physician.
gical shunting procedure that allows CSF to drain
d. Assist with diagnostic procedures/treatments:
from the ventricles of the brain to another, less
ventricular tap, computed tomography (CT)
harmful area within the body: most commonly
scan, etc.
the peritoneal cavity, less often the jugular vein or
2. Goal: health teaching to reduce parental anxiety.
right atrium of the heart. Hydrocephalus can
a. Do preoperative teaching regarding the
develop as the result of a congenital malformation
shunt procedure: stress need to remove
(e.g., Arnold-Chiari malformation); can be associ-
excessive CSF to relieve pressure on brain;
ated with other congenital defects (e.g., spina
done as soon as possible after diagnosis is
bifida); or can be acquired secondary to infection
established.
(e.g., meningitis), trauma, or neoplasm.
b. Stress early diagnosis and prompt shunting
B. Assessment:
procedure to minimize the risk of long-term
1. Head: increased circumferenceearliest
neurological complications.
sign of hydrocephalus in the infant (more
c. Offer realistic information regarding
than 1 inch/mo).
prognosis:
2. Fontanels: tense and bulging without head
(1) Surgically treated, with continued follow-
enlargement.
up care: 80% survival rate.
3. Veins: dilated scalp veins.
2164_Ch05_249-346 29/03/12 12:25 PM Page 268

268 chapter 5 Health Promotion and Maintenance

(2) Of these survivors, 50% are completely girls. There also appears to be an increased suscepti-
normal and 50% have some degree of neu- bility within families, suggesting a possible genetic
rological disability (such as inattentiveness predisposition. Note: Epilepsy is discussed in
or hyperactivity). Chapter 6.
3. Goal: provide postoperative shunt care. B. Assessment:
a. Position: 1. History usually reveals presence of URI or
(1) Flat in bed for 24 hours to prevent gastroenteritis.
subdural hematoma. 2. Occurs with a sudden rise in fever: often spiked
(2) Gradually increase the angle of elevation of and quite high (102F or higher) vs. prolonged
HOB, as ordered by surgeon. temperature elevation.
(3) On the nonoperative side, to prevent C. Analysis/nursing diagnosis:
mechanical pressure and obstruction to 1. Risk for injury related to seizures.
shunt. 2. Knowledge deficit related to prevention of future
b. Monitor head circumference daily to note seizures, care of child having a seizure, and
any abnormal increase that might indicate possible long-term effects.
malfunctioning shunt. D. Nursing care plan/implementation:
c. Monitor vital signs; monitor for signs of 1. Goal: reduce fever/prevent further elevation of
increased ICP. fever.
d. Monitor for possible complications: a. Administer antipyretics, as ordered: acetamin-
(1) Infection. ophen only (not aspirin).
(2) Malfunction of shunt: increased ICP. b. Use cool, loose, cotton clothes to decrease
4. Goal: provide discharge teaching to parents regard- heat retention.
ing home care of the child with a shunt. c. Avoid shivering, which increases metabolic
a. Stress need for long-term follow-up care. rate and temperature.
b. Discuss feeding techniques, care of skin d. Encourage child to drink cool fluids.
(especially scalp), need for stimulation. e. Monitor temperature hourly.
c. Prepare parents for shunt revisions to be done f. Minimize stimulation, frustration for child.
periodically as child grows. 2. Goal: teach parents about care of child who
d. Teach parents signs and symptoms of shunt experiences febrile seizure.
PEDIATRICS

malfunctioning (i.e., of increased ICP or a. Discuss how to prevent seizures from recur-
infection) and to report these promptly to ring: best method is to prevent temperature
physician. from rising over 102F (see Goal 1).
e. Encourage parents to enroll infant in early b. Discuss how to handle seizures if they do
infant stimulation program to maximize recur: prevent injury, maintain airway, etc.
developmental potential. c. Answer questions simply and honestly:
f. Stress need to monitor development at frequent (1) 25% of children with one febrile seizure
intervals, make referrals prn. will experience a recurrence.
E. Evaluation/outcome criteria: (2) 75% of recurrences occur within 1 year.
1. Neurological functioning is maintained or (3) Reassure parents of the benign nature of
improved. febrile seizures; 95% to 98% of children
2. Adequate nutrition is maintained. with febrile seizures do not develop epilepsy
3. No impairment of skin integrity occurs. or neurological damage.
4. Parents anxiety is relieved; they verbalize E. Evaluation/outcome criteria:
understanding of how to care for child after 1. Fever is kept below 102F; additional seizures are
discharge. prevented.
IV. FEBRILE SEIZURES 2. Parents verbalize their understanding of how to
care for child at home.
A. Introduction: Febrile seizures are transient neurologi-
cal disorders of childhood, affecting perhaps as
many as 3% of all children. Although the exact R E S P I R AT O R Y
cause of febrile seizures remains uncertain, they SYSTEM
seem to be a relatively transient problem that occurs
I. ACUTE OTITIS MEDIA
exclusively in the presence of high, spiked fevers.
Children in the infant and toddler stages (6 months A. Introduction: Acute bacterial ear infection (acute
to 3 years) appear to be most susceptible to febrile otitis media) is common in young children, prima-
seizures, and they are twice as common in boys as in rily because their eustachian tube is shorter and
2164_Ch05_249-346 29/03/12 12:25 PM Page 269

Respiratory System 269


straighter than the adults; this allows for ready 7. Hoarseness or aphonic crying.
drainage of infected mucus from URIs directly into 8. Dyspnea or prostration.
the middle ear. In some cases, acute otitis media 9. Dehydrationrelated to increase in insensible
precedes the onset of bacterial meningitis, an fluid loss and poor PO intake.
extremely serious and potentially fatal disease. 10. Color change (pallor, cyanosis)later sign of
Bacterial meningitis is a medical emergency, respiratory distress.
requiring early detection and prompt, aggressive B. Analysis/nursing diagnosis:
therapy to prevent permanent neurological damage 1. Ineffective airway clearance related to infection
or death. Serous otitis (chronic) may result in hear- or obstruction.
ing impairment or loss but is not likely to result in 2. Fluid volume deficit related to excessive losses
meningitis (Refer to Myringotomy, Table 5.22). through normal routes, discomfort and inability
B. Assessment: to swallow.
1. Fever. 3. Anxiety related to hypoxia.
2. Pain in affected ear. An infant who is prelingual 4. Risk for injury related to spread of infection.
may not complain of pain but may tug at ear, 5. Knowledge deficit related to disease process,
cry, shake head, refuse to lie down. infection control, home care, and follow-up.
3. Malaise, irritability, anorexia (possibly vomiting). C. Nursing care plan/implementation:
4. May have symptoms and signs of URI: rhinorrhea, 1. Goal: relieve respiratory distress by reducing
coryza, cough. swelling and edema and liquefying secretions.
5. Diminished response to sound. a. Environment: age- and disease-appropriate
C. Analysis/nursing diagnosis: oxygen delivery system (see Table 5.10).
1. Pain related to pressure of pus/purulent material b. Administer oxygen as ordered.
on eardrum. c. Position: semi-Fowlers or in infant seat to
2. Risk for injury/infection related to complication promote maximum expansion of the lungs;
of meningitis. small blanket or diaper roll under neck to
D. Nursing care plan/implementation: keep airway patent; change position at least
1. Goal: eradicate infection and prevent further q2h to prevent pooling of secretions.
complications (meningitis). Administer antibiotics d. Suction/postural drainage and percussion prn.
as ordered. e. Tape diapers loosely and use only loose-

PEDIATRICS
2. Goal: relieve pain and promote comfort. fitting clothing to avoid pressure on
a. Administer decongestants as ordered. abdominal organs, which could impinge
b. Offer analgesics/antipyretics to provide on diaphragm and impede respirations.
symptomatic relief and to decrease fever. f. Administer medications: antibiotics, bron-
3. Goal: health teaching. chodilators, steroids.
a. Teach parents that the child needs to finish g. Monitor temperature q4h/prn; reduce
all medication, even though child will seem fever with acetaminophen, cool sponges,
clinically better within 24 to 48 hours. hypothermia blanket.
b. Review appropriate measures to control fever: 2. Goal: observe for potential respiratory failure
antipyretics, cool sponges. related to exhaustion or complete airway
E. Evaluation/outcome criteria: obstruction.
1. Infection is eradicated, no complications. a. Place in room near nurses station for
2. Child appears to be comfortable. maximum observation.
II. PEDIATRIC RESPIRATORY INFECTIONS b. Monitor vital signs: q1h during acute phase,
then q4h.
A. Assessment: general assessment of infant/child
c. Place emergency equipment near bedside
with respiratory distress. Note: Additional informa-
prn: endotracheal tube, tracheostomy set.
tion about specific respiratory infections may be
d. Monitor closely for signs of impending
found in Table 5.9.
respiratory failure: increased rapid, shallow
1. Restlessnessearliest sign of hypoxia.
respirations, progressive hoarseness/aphonia,
2. Difficulty sucking/eatingparents may state
deepening cyanosis.
the infant or child has poor appetite.
e. Report adverse changes in condition STAT to
3. Expiratory grunt, nasal flaring, retractions.
physician.
4. Changes in vital signs: fever, tachycardia,
3. Goal: maintain normal fluid balance.
tachypnea.
a. May be NPO initially to prevent aspiration.
5. Cough: productive/nonproductive.
b. IVs until severe distress subsides and child is
6. Wheeze: expiratory/inspiratory.
able to suck and swallow.
PEDIATRICS

270

Table 5.9
Pediatric Respiratory Infections
Assessment: Definitive
Name Definition Age Group Etiology Clinical Signs and Symptoms Plan: Specifics of Treatment Prognosis
Upper Airway Infections
Croup (acute spasmodic Paroxysmal attacks 3 months Viral (possible Most common onset at night Teach parentsturn on hot Excellent (but
laryngitis) (spasms of larynx) 3 years allergy or Inspiratory stridor water in bathroom and likely to recur)
psychogenic) Croupy barking cough close door (steam); warm
Dyspnea temperature will not relieve
Anxiety the congestion
2164_Ch05_249-346 29/03/12 12:25 PM Page 270

Common to treat at home


Epiglottitis Extremely acute, 18 years Bacterial Abrupt onsetrapid progres- Do not visualize epiglottis Very good
severe, and rapid, (H. influenzae sion; medical emergency unless airway support is if detected
progressive swelling type b) Dyspnea immediately available and treated
(due to infection) Dysphagia early
Will need endotracheal
of epiglottis and Sit up/chin thrust/mouth open Prevent
tube or tracheostomy for
surrounding tissue Thick muffled voice via Hib
2448 hr to maintain
Cherry red, swollen epiglottis immu-
patent airway
nization
chapter 5 Health Promotion and Maintenance

IV ampicillin for 1014 days


to treat bacterial infection
IV corticosteroids (e.g.,
Solu-Cortef) to reduce
inflammation
Laryngotracheobronchitis Acute infection of 3 months Viral (possible Inspiratory stridor Hospitalization: Good
(LTB) lower respiratory 8 years secondary bac- High fever Tracheostomy set at
tract: larynx, trachea, terial infection) Signs and symptoms of bedside
and bronchi severe respiratory distress Racemic epinephrine/
Hoarseness, progressing steroids
to aphonia and respiratory Antibiotics if cultures are
arrest without treatment positive
Lower Airway Infection
Bronchiolitis/respiratory Acute viral infection Infants Respiratory Hyperinflation of alveoli Supportive care during acute Excellent (less
syncytial virus (RSV) of lower respiratory 212 months syncytial virus Scattered areas of atelectasis phase: than 1% mor-
tract (small, low (peak at 80% of cases Acute, severe respiratory Hospitalization tality rate)
bronchioles), with 25 months) distress for first 4872 hours, High humidity
resultant trapping followed by rapid recovery Oxygen
of air IV fluids
Clear liquids
Ribavirin (not given
everywhere)
RespiGam or Synagis may
be given as prophylaxis
2164_Ch05_249-346 29/03/12 12:25 PM Page 271

Respiratory System 271

Table 5.10
Comparison of Common Oxygen Delivery Systems
System Advantages Disadvantages
Cannula Provides low-moderate oxygen concentration Difficulty in controlling O2 concentrations if child breathes
(22%40%) through mouth
Child can talk/eat without altering FIO2 Must have patent nasal passages
Possibility of more complete observation of Possibility of causing abdominal
child because nose/mouth remain unobstructed distention/discomfort/vomiting
Relatively comfortable and inexpensive Can cause drying/bleeding of nasal mucosa
Hood Achievement of high O2 concentrations; FIO2 up Moist environment may lead to skin irritation and prevent
to 1.00 quick assessment of color or respiratory effort
Quick recovery time of FIO2 Need to remove infant for feeding and weighing
Free access to infants chest for assessment
Mask Various sizes available Accumulation of moisture on face leading to skin irritation
Delivers higher, more precise FIO2 concentra- Possibility of aspiration of vomitus
tions than cannula Eating disrupts O2 delivery
Comfortable for older children who are quiet Not well tolerated by most children due to fear of
and do not struggle suffocation
Tent Achievement of lower O2 concentrations (FIO2 Necessity for tight fit around bed to prevent leakage of
of 0.30.5) O2 and maintain specific O2 concentrations
Child receives increased inspired O2 concentra- Child is difficult to see/assess
tions even while eating Cool/wet tent environment will decrease body tempera-
Child can move around in bed and play while ture, increasing O2 requirements
receiving O2 and humidity Inspired O2 levels will fall whenever tent is entered for
caregiving purposes

c. Monitor hydration status: I&O, urine specific b. Long term: how to handle recurrences, how to
gravity, weight. check temperature at home, medications for

PEDIATRICS
d. When resuming PO fluidsstart with sips of fever, when to call physician about respiratory
clear liquids, advance slowly as tolerated: problem.
Pedialyte, clear broth, gelatin, popsicles, fruit D. Evaluation/outcome criteria:
juices, ginger ale. 1. No further evidence of respiratory distress.
e. Avoid milk/milk products, which may cause 2. Resumption of normal respiratory pattern.
increased mucus production. 3. Normal fluid balance maintained/restored.
4. Goal: provide calm, secure environment. 4. Parents verbalize their concerns and express con-
a. During acute distress: remain with child/ fidence in their ability to care for their child
family (do not leave unattended). after discharge.
b. Keep crying to a minimum to prevent severe III. LONG-TERM RESPIRATORY DYSFUNCTION:
hypoxia and to reduce the bodys demand for ASTHMA
oxygen.
A. Introduction: Asthma is generally considered a
c. Avoid painful/intrusive procedures if
chronic, lower airway disorder characterized by
possible.
heightened airway reactivity with bronchospasm
d. Organize nursing care to provide planned
and obstruction. The exact cause of asthma is
periods of uninterrupted rest.
unknown; however, it is believed to include an
e. Allow parents to room-in, and encourage
allergic reaction to one or more allergens, or
their participation in care of their child to
triggers, that either precipitate or aggravate
keep the child relatively calm and reduce
asthmatic exacerbation. The child usually exhibits
anxiety.
other symptoms of allergy, such as infantile
f. Allow child to keep favorite toy or security
eczema or hay fever; in addition, 75% of children
object.
with asthma have a positive family history for
5. Goal: provide parents with teaching, as necessary.
asthma. The onset is usually before age 5 and the
a. Short term: discuss equipment, treatments,
disorder remains with the child throughout life,
procedures; offer frequent progress reports,
although some children experience dramatic
answer parents questions.
2164_Ch05_249-346 29/03/12 12:25 PM Page 272

272 chapter 5 Health Promotion and Maintenance

improvement in their asthma with the onset of c. Administer sedation as ordered.


puberty. Most children do not require continuous d. Encourage parents to remain with child.
medication. Early relief of symptoms with a com- 3. Goal: teach principles of prophylaxis.
bination of drugs can reverse bronchospasm. a. Review home medications, including
B. Assessment: cromolyn sodium. See Chapter 8.
1. Expiratory wheeze. b. Review breathing exercises.
2. General signs and symptoms of respiratory c. Discuss precipitating factors (triggers) and
distress, including: anxiety, cough, shortness of offer suggestions on how to avoid them.
breath, crackles, cyanosis due to obstruction d. Teach how to use peak expiratory flowmeter
within the respiratory tract, use of accessory to monitor respiratory status and determine
muscles of respirations. need for treatment.
3. Cough: hacking, paroxysmal, nonproductive; e. Introduce need for child to assume control
especially at night. over own care.
4. Position of comfort for breathing: sitting straight E. Evaluation/outcome criteria:
up, leaning forward, which is the position for 1. Adequate oxygenation provided, as evidenced by
optimal lung expansion. pink color of nailbeds and mucous membranes
5. Peak expiratory flow rate (PEFR) is in the yellow and ease in respiratory effort.
zone (50%80% of personal best) or in the red 2. Anxiety is relieved.
zone (<50% of personal best). 3. Child verbalizes confidence in, and demonstrates
C. Analysis/nursing diagnosis: mastery of, skills needed to care for own asthma.
1. Ineffective airway clearance related to IV. CYSTIC FIBROSIS
bronchospasm.
A. Introduction: Cystic fibrosis is a generalized dys-
2. Anxiety related to breathlessness.
function of the exocrine glands that produces
3. Knowledge deficit, actual or risk for potential,
multisystem involvement. The disorder is inherited
related to disease process, treatment, and
as an autosomal recessive defect. The mutated gene
prevention of future asthmatic attacks.
responsible for CF is located on the long arm of
4. Activity intolerance related to dyspnea and
chromosome 7 (CFTR). The basic problem is one
bronchospasm.
of thick, sticky, tenacious mucous secretions that
D. Nursing care plan/implementation:
obstruct the ducts of the exocrine glands, thus
PEDIATRICS

Treatment is aimed toward improvement of ventila-


affecting their ability to function. Cystic fibrosis
tion, correction of dehydration and acidosis, and
is found in all races and socioeconomic groups,
management of concurrent infection.
although there is a significantly lower incidence
1. Goal: provide patent airway and effective breath-
in Asians and African Americans. It is a chronic
ing patterns.
disease with no known cure and guarded prognosis;
a. Initiate oxygen therapy (by tent, face mask,
median age at death in the United States is 31 years.
or cannula), as ordered, to relieve hypoxia,
Those born in the late 1990s can be expected to
with high humidity (to liquefy secretions).
survive into their 40s with new therapies.
b. Administer bronchodilators, as ordered, to
B. Assessment:
relieve the obstruction: epinephrine (1:1000),
1. Newborn: meconium ileus.
nebulized albuterol, Atrovent. Inhalers may be
2. Frequent, recurrent pulmonary infections: bron-
used with metered-dose inhalers (MDIs) to
chitis, bronchopneumonia, pneumonia, and
ensure proper delivery of the medication.
ultimately chronic obstructive pulmonary disease
c. Administer corticosteroids as ordered (PO
(COPD) due to mechanical obstruction of respi-
or IV) to reduce inflammation, relieve
ratory tract caused by thick, tenacious mucous
edema (prednisone, Decadron) and decrease
gland secretions.
bronchial hyperreactivity.
3. Malabsorption syndrome: failure to gain weight,
d. Administer antibiotics as ordered; infection
distended abdomen, thin arms and legs, lack
is commonly either a trigger or complication
of subcutaneous fat due to disturbed absorption
of asthma.
of nutrients that results from the inability of
e. Note: methylxanthines (theophylline, amino-
pancreatic enzymes to reach intestinal tract.
phylline) are third-line agents that are rarely
4. Steatorrhea: bulky, foul-smelling, frothy, fatty
used to treat asthma.
stools in increased amounts and frequency
2. Goal: relieve anxiety.
(predisposed to rectal prolapse).
a. Provide relief from hypoxia (refer to Goal 1),
5. Parents may note that child tastes salty when
which is the chief source of anxiety.
kissed, due to excessive loss of sodium and
b. Remain with child, offer support.
chloride in sweat.
2164_Ch05_249-346 29/03/12 12:25 PM Page 273

Respiratory System 273


6. Sweat test reveals high sodium and chloride levels checkups, stay away from known sources of
in childs sweat, unique to children with cystic infection.
fibrosis. 3. Goal: maintain adequate nutrition.
7. Sexual development: a. Diet: well balanced, high calorie and protein to
a. Boys/Men: sterile (due to aspermia). prevent malnutrition. Fat content in diet is
b. Girls/Women: difficulty conceiving and bear- controversial and must be individualized.
ing children (due to increased viscosity of b. Administer pancreatic enzyme (Pancrease,
cervical mucus, which acts as a plug in the Ultrase) immediately before every meal and
cervical os and mechanically blocks the every snack to enhance the absorption of vital
entry of sperm). nutrients, especially fats.
C. Analysis/nursing diagnosis: c. If child is unable to swallow capsules, take cap-
1. Ineffective breathing patterns related to thick, sule apart and sprinkle on food at beginning
viscid secretions. of meal or mix with chilled applesauce.
2. Altered nutrition, less than body requirements, d. Administer water-miscible preparations
related to diarrhea and poor intestinal absorption of fat-soluble vitamins (A, D, E, K),
of nutrients. multivitamins, and iron.
3. Decreased cardiac output related to COPD and e. Encourage extra salt intake to compensate
decreased compliance of lungs. for excessive sodium losses in sweat (unless
4. Activity intolerance related to respiratory congestive heart failure [CHF] is present);
compromise. especially important in hot weather, after
5. Self-esteem disturbance related to body image physical exertion, febrile periods.
changes. f. Encourage extra fluid intake (e.g., Gatorade)
6. Knowledge deficit related to disease process, to prevent dehydration/electrolyte imbalance,
treatments, medications, genetics. thickening of mucous secretions.
7. Risk for noncompliance related to complicated g. Daily I&O and weights to monitor nutritional
and prolonged treatment regimen. and hydration status.
D. Nursing care plan/implementation: h. Encourage child to assume gradually increas-
1. Goal: assist child to expectorate sputum. ing responsibility for choosing own foods
a. Perform postural drainage and percussion as within dietary restrictions.

PEDIATRICS
prescribed: first thing in morning, between 4. Goal: teach child and family about cystic
meals, before bedtime, not after meals to fibrosis.
prevent aspiration. a. Discuss diagnostic procedures: sweat test,
b. Administer nebulizer treatments, expecto- stool specimens.
rants, mucolytics, bronchodilators. Avoid or b. Review multiple medications: use, effects,
limit use of medications that suppress cough side/toxic effects.
mechanism. c. Stress need to care for pulmonary system
c. Provide for exercises that promote position (major cause of mortality/morbidity).
changes and keep sputum moving up d. Teach various treatments: postural drainage,
and out. nebulizers, oxygen therapy, breathing
d. Encourage high fluid intake to keep secretions exercise.
liquefied. e. Encourage child to assume as much responsi-
e. Suction, administer oxygen prn. bility for own care as possible: medications,
2. Goal: prevent infection. treatments, diet.
a. Standard precautions to prevent infection. f. Promote development of healthy attitude
b. Evaluate carefully, check continually for toward disease/prognosis (no known cure).
potential infection (especially respiratory); Heart/lung transplantation may be
report to physician promptly. considered as an option.
c. Limit contact with staff or visitors (especially g. Refer to appropriate community agencies for
children) with infection. assistance with home care.
d. Administer antibiotics as ordered, to h. Assist with genetic counseling.
treat respiratory infections and prevent i. Discuss sexual concerns with adolescent.
overwhelming sepsis. 5. Goal: promote compliance with treatment
e. May be placed on prophylactic antibiotic regimen.
therapy between episodes of infection. a. Encourage child to verbalize anger or
f. Teach importance of prevention of infection frustration at being different/body image
at home: adequate nutrition, frequent medical alterations.
2164_Ch05_249-346 29/03/12 12:25 PM Page 274

274 chapter 5 Health Promotion and Maintenance

b. Suggest alternatives to chest physical therapy (3) Suction, oxygen, and resuscitation equip-
(CPT) (e.g., yoga/standing on head). ment readily available if needed.
c. Offer rewards for compliance: going (4) Observe for apnea or bradycardia; note
swimming with friends or other types of duration and associated symptomscolor
peer activities. change, change in muscle tone.
E. Evaluation/outcome criteria: (5) If apnea occurs, use gentle stimulation to
1. Child can clear own airway, expectorate start infant breathing again. If ineffective,
sputum. begin CPR (Figs. 5.1 and 5.2).
2. Child is maintained in infection-free state. (6)If suctioning is needed, do it gently for
3. Adequate nutrition is maintained. the shortest time and least number of
4. Child and family verbalize understanding of the times possible to maintain patent airway.
disease. Note: Repeated, vigorous suctioning is
5. Child complies with rigors of treatment. associated with prolonged periods of
V. APNEA-RELATED DISORDERS apnea.
(7) Medications: respiratory stimulant drugs
A. Apnea of infancy
(such as theophylline or caffeine) may be
1. Introduction: Apnea of infancy is the unexplained
given until 2 to 3 months have passed
cessation of breathing for 20 seconds or longer
without an episode of apnea.
in an apparently healthy, full-term infant who is
(8) Positions: side-lying or supine; never prone,
more than 37 weeks of gestation. It is usually
to prevent SIDS.
diagnosed by the second month of life and is
(9) Feedings: smaller and more frequent; avoid
generally thought to resolve during the first 12
overfeeding, which can lead to reflux and
to 15 months of life. The exact cause is
apnea.
unknown. The association between apnea of
b. Goal: teach parents how to care for their infant
infancy and sudden infant death syndrome
at home (Table 5.11).
(SIDS) is still controversial. However, infants
(1)Thoroughly explain discharge plans
experiencing significant apnea without a known
to parents; encourage questions and
cause are thought to be at increased risk for
discussion.
SIDS and must be treated accordingly. The
(2) Begin teaching use of apnea monitor and
diagnosis of apnea of infancy (AOI) is made
PEDIATRICS

infant CPR techniques several days before


when no identifiable cause for the apparent
discharge; allow parents to handle the
life-threatening event (ALTE) is found.
monitor and become thoroughly familiar
2. Assessment:
with its use.
a. Unexplained cessation of breathing (apnea)
(3) Provide parents with emergency response
for 20 seconds or longer.
numbers and community health nurse
b. Bradycardia.
referral.
c. Color change: cyanosis or pallor.
(4) Stress need for at least 1 year of ongoing
d. Limp, hypotonic.
care with constant use of monitor, or 2 to
e. Diagnostic tests, including cardiopneumo-
3 months without an episode requiring
gram, pneumocardiogram, and
intervention.
polysomnography.
(5) Discuss need for support and refer to local
3. Analysis/nursing diagnosis:
self-help/support group.
a. Ineffective breathing patterns related to
(6) Encourage parents to take time for them-
apnea.
selves if a reliable caregiver is available
b. Anxiety, fear related to apnea and threat of
who is trained in use of monitor and
infants death.
infant CPR.
c. Knowledge deficit regarding home care of
5. Evaluation/outcome criteria:
infant on an apnea monitor and infant car-
a. Effective breathing pattern is established.
diopulmonary resuscitation (CPR).
b. Parents verbalize their concerns and express
4. Nursing care plan/intervention:
confidence in their ability to care for their
a. Goal: maintain effective breathing pattern.
infant at home.
(1) Apnea monitor on infant at all times,
B. Sudden infant death syndrome (SIDS)
including at home.
1. Introduction: SIDS is the sudden, unexpected
(2) Place in room near nurses station for max-
death of an apparently healthy infant under
imum observation with a nurse or parent
1 year of age, which remains unexplained after
present at all times.
a complete postmortem examination. Various
2164_Ch05_249-346 29/03/12 12:25 PM Page 275

Respiratory System 275

ACTIONS
Objectives
Adult Child (1 year to puberty) Infant (up to 1 year)

1. Determine
Tap victim or gently shake shoulder. Say, Are you okay? Speak loudly.
unresponsiveness.

Check for absent breathing


2. Determine
or abnormal breathing Check for absent breathing or only agonal gasps.
breathlessness.
(such as agonal gasps).
A. RECOGNITION
3. Determine Feel for brachial pulse
Feel for carotid pulse (no more than 10 seconds).
pulselessness. (no more than 10 seconds).

4. Recognize emergency Activate EMS. Get Automated External Defibrillator (AED), if available. Deploy AED as
and get help. soon as available but do not delay CPR.

B. CPR 5. Start CPR. Use CAB sequence: Compression, Airway, Breathing.

6. Begin chest
compressions: Imagine a line drawn between the nipples of the chest.
Landmark check.

Single rescuer: Place 2


fingers just between nipple
7. Position hands. Place hand(s) at center of breastbone between nipples. line. Two-person rescuer:
use 2 thumb-encircling
hands technique.

C. COMPRESSION Depress at least 1/3 depth Depress at least 1/3 depth


8. Push hard. Depress at least 2 inches.
of chest, about 2 inches. of chest, about 1.5 inches.

9. Push fast. Perform compressions at a rate of at least 100 per minute.

10. Allow chest recoil. Allow chest to recoil completely between compressions.

11. Minimize Limit interruptions to 10 seconds or less while performing compressions. Two-person

PEDIATRICS
interruptions. rescuers should rotate every 2 minutes.

Perform compressions only if not CPR proficient and continue to step 15.
If proficient, continue to steps 12-15.

D. AIRWAY 12. Open airway. Perform head tiltchin lift (or jaw thrust, if trauma suspected).

13. Provide ventilations Give 2 breaths for every 30 Give 2 breaths for every 30 compressions for single
until advanced airway compressions (1 or 2 rescuer; give 2 breaths for every 15 compressions for 2
established. rescuers). rescuers.
E. BREATHING
14. Provide ventilations
Provide 1 breath every 6-8 seconds during uninterrupted CPR; one second per breath.
with advanced airway
Observe for chest rise.
in place.

Apply and deploy AED as soon as available. Minimize interruptions in compressions


F. DEFIBRILLATION 15. Defibrillate.
before and continue compressions immediately after shock delivered.

Figure 5.1 CPR recommendations for the healthcare provider. If victim is breathing or resumes effective breathing, place in
recovery position; (1) move head, shoulders, and torso simultaneously; (2) turn onto side; (3) leg not in contact with ground
may be bent and knee moved forward to stabilize victim; (4) victim should not be moved in any way if trauma is suspected
and should not be placed in recovery position if rescue breathing or CPR is required. (Modified from CPR Overview: 2010 American Heart
Association guidelines for cardiopulmonary resuscitation and emergency cardiovascular care. Circulation, 122[18 Suppl 3], 2010. Adapted from Hockenberry, M, &
Wilson, D: Wongs Nursing Care of Infants and Children, ed 8. Mosby, St. Louis, 2007.)

theories have been suggested, none proved; between 1 month and 1 year, affecting almost
research is ongoing. It has been suggested that 2500 infants annually.
prone sleeping position, cigarette smoke, and 2. Assessment:
excessive swaddling may be associated with a. Sudden, unexplained death in otherwise nor-
SIDS. It is the third leading cause of death mal infant; occurs exclusively during sleep.
2164_Ch05_249-346 29/03/12 12:25 PM Page 276

276 chapter 5 Health Promotion and Maintenance

Signs of life-threatening obstruction: truly choking child cannot speak, becomes cyanotic, and collapses.

ACTIONS
Objectives
Adult Child (1 year to puberty) Infant (up to 1 year)

Observe breathing difficulty,


1. Determine airway Ask, Are you choking? Determine if victim can cough or
ineffective cough, no strong
obstruction. speak.
cry.
CONSCIOUS
VICTIM 2. Act to relieve Give 5 back blows.
Perform up to 5 subdiaphragmatic abdominal thrusts.
obstruction. Give 5 chest thrusts.

3. Be persistent. Repeat Step 2 until obstruction is relieved or victim becomes unconscious.

4. Position the victim and Turn on back as a unit, supporting head and neck, face up, arms by sides. Call out,
get help. Help! Activate EMS. If second rescuer available, have person activate EMS.

Use CAB sequence: Compression, Airway, Breathing.


5. Start CPR.
Perform a series of 30 chest compressions first. Do not perform pulse check.
VICTIM WHO
6. Open airway. Perform head tiltchin lift (do not tilt too far for infants).
BECOMES
UNCONSCIOUS 7. Look for foreign object. If you see foreign body, remove it. Do not perform a blind finger sweep.

8. Give rescue breaths. Attempt to give 2 rescue breaths.

Continue with chest compressions and attempted ventilations until obstruction is relieved.
9. Continue CPR.
After 2 minutes of CPR, activate EMS if this has not already been done.

Figure 5.2 Foreign body airway obstruction management. (Modified from CPR Overview: 2010 American Heart Association guidelines for
cardiopulmonary resuscitation and emergency cardiovascular care. Circulation, 122[18 Suppl 3], 2010. Adapted from Hockenberry, M, & Wilson, D: Wongs Nursing
Care of Infants and Children, ed 8. Mosby, St. Louis, 2007.)

Table 5.11
PEDIATRICS

Guidelines for Home Care of Infant on Apnea Monitor


1. Show the parents how to connect the monitor leads.
2. Remind parents to remove the leads unless they are connected to the infant.
3. Stress that the infant must be on the monitor whenever respirations are not being directly observed and that a trained person
must be present in the home at all times in case the alarm sounds.
4. Teach parents not to adjust the monitor to eliminate false alarms.
5. Explain that the infant will need direct observation whenever loud noises could obscure the monitor alarm (e.g., dishwasher,
vacuum).
6. Teach parents what to look for when alarm sounds (i.e., loose monitor leads vs. apnea).
7. Teach parents how to assess the infant for an episode of apnea (i.e., lack of respirations, duration, color, muscle tone).
8. Teach the parents to first use gentle physical stimulation if the infant experiences an apnea spell (e.g., touching the face or
stroking the soles of the feet).
9. Demonstrate infant CPR to be used if tactile stimulation is not effective in reestablishing respirations.
10. Encourage parents to keep emergency numbers posted near the telephone.
11. Explain that monitor will not interfere with normal growth and development. Encourage the parents to promote normal growth
and development as much as possible.
Also refer to Table 3.9, Safety Considerations, p. 94.

b. Note overall appearance of infant (differenti- 4. Nursing care plan/implementation:


ate from child abuse). a. Immediate goal: support parents who are
c. Obtain history from parentsnote affect or grieving.
how parents are dealing with grief. (1) Stress that nothing could have been done
3. Analysis/nursing diagnosis: to prevent the death.
a. Dysfunctional grieving related to loss of infant. (2) Allow parents to express grief emotions;
b. Knowledge deficit related to SIDS. provide privacy.
2164_Ch05_249-346 29/03/12 12:25 PM Page 277

Cardiovascular System 277


(3) Offer parents opportunity to see, hold C A R D I O VA S C U L A R
infant. SYSTEM
(4) Explain purpose of autopsy (physician to
obtain consent). I. CONGENITAL HEART DISEASE (CHD)
(5) Contact spiritual advisor: priest, rabbi, A. Introduction: There are more than 35 documented
minister. types of congenital heart defects, which occur in
(6)Assist parents to plan what to tell 5 to 8 per 1000 live births. For the purpose of this
siblings. review, only five major defects are given. These are
b. Ongoing goal: provide factual information presented in Figures 5.3 through 5.7. Note: The
regarding SIDS. (text continues on page 279)
(1)Offer information that is known
about SIDS in simple, direct terms
(Table 5.12).
(2) Answer questions honestly.
(3) Give parents printed literature on SIDS.
(4) Refer to local/national SIDS foundation
group.
c. Long-term goal: assist family to resolve grief.
(1) Track progress of other siblings.
(2) Refer to local perinatal bereavement
group.
(3) Consider subsequent pregnancy to be at
risk for:
(a) Attachment/bonding.
(b) SIDS recurrence.
5. Evaluation/outcome criteria:
a. Parents are able to express their grief and
receive adequate support. Figure 5.3 Atrial septal defect. An opening between the two
b. Parents raise questions about SIDS and can atria, allowing oxygenated blood and unoxygenated blood to

PEDIATRICS
understand answers. mix. Left-to-right shunting of blood occurs due to the higher
c. Familys grief is resolved; in time, normal pressure on the left side of the heart. (From Ashwill, JW & James,
SR: Nursing Care of Children: Principles and Practice. WB Saunders,
family dynamics resume.
Philadelphia, 2007.)

Table 5.12
SIDS: What to Tell Families
Concern Facts
Cause Unknown (possibly related to delayed maturation of cardiorespiratory system)
Incidence Almost 2500 cases annually; leading cause of death between ages 1 month and 1 year
When Occurs during sleep (nap, night)
Age Peak at 24 months; 95% of cases occur by age 6 months
Sex More common in boys
Race More common in African Americans, Native Americans, and Hispanics
Season More common in winter, peaks in January
Siblings May have greater incidence
Perinatal More common in preterm infants, in multiple births, in infants with low Apgar scores, and with maternal
smoking
Socioeconomic More common in lower socioeconomic classes
Feeding habits More common in infants who are bottle-fed, less common in infants who are breastfed.
Means of prevention Supine (on back) sleeping position (mnemonic: back to sleep); avoid soft bedding and overheating
during sleep; avoid cigarette smoking in house.
2164_Ch05_249-346 29/03/12 12:25 PM Page 278

278 chapter 5 Health Promotion and Maintenance

Pulmonic
stenosis
Overriding
aorta

Ventricular
septal
defect

Right
ventricular
hypertrophy

Figure 5.4 Ventricular septal defect. An opening between Figure 5.6 Tetralogy of Fallot. Four cardiac anomalies
the two ventricles, allowing oxygenated and unoxygenated make up tetralogy of Fallot: a ventricular septal defect, pul-
blood to mix. Shunting of blood occurs due to the higher monary stenosis, an overriding aorta, and right ventricular
pressure on the left side of the heart. Ventricular septal hypertrophy. There are other associated defects in certain
defects are classified as membranous or muscular accord- cases. Pulmonary stenosis results in reduction of pulmonary
ing to location in the septum. (From Ashwill, JW & James, SR: blood flow; the ventricular septal defect allows mixing of
Nursing Care of Children: Principles and Practice. WB Saunders, oxygenated and unoxygenated blood. (From Ashwill, JW &
Philadelphia, 2007.) James, SR: Nursing Care of Children: Principles and Practice. WB Saunders,
Philadelphia, 2007.)
PEDIATRICS

Figure 5.5 Patent ductus arteriosus. An artery that con- Figure 5.7 Transposition of the great arteries. The aorta
nects the aorta and the pulmonary artery during fetal life. and the pulmonary artery are reversed; that is, the aorta
It generally closes spontaneously within a few hours to rises from the right instead of the left ventricle, and the
several days after birth. Allows abnormal blood flow from pulmonary artery arises from the left instead of the right ven-
the high-pressure aorta to the low-pressure pulmonary tricle. Systemic venous blood (unoxygenated blood) returns
artery, resulting in a left-to-right shunt. (From Ashwill, JW & to the right side of the heart to pass through the aorta back
James, SR: Nursing Care of Children: Principles and Practice. WB Saunders, to the body without being oxygenated because of bypassing
Philadelphia, 2007.) the lungs. The pulmonary venous blood (oxygenated blood)
enters the left side of the heart, returning to the lungs via
the pulmonary artery. The systemic (unoxygenated) and
pulmonary (oxygenated) circulations are totally separate.
There must be some opening (i.e., patent ductus arteriosus
or septal defect) to allow blood to mix. (From Ashwill, JW &
James, SR: Nursing Care of Children: Principles and Practice. WB Saunders,
Philadelphia, 2007.)
2164_Ch05_249-346 29/03/12 12:25 PM Page 279

Cardiovascular System 279


content has been synthesized for ease in review and c. History of maternal prenatal infection with
recall; for additional study aids, the student may rubella, cytomegalovirus, etc.
wish to refer to Tables 5.13 and 5.14. Chapter 6 d. High-risk maternal factors:
also contains information on congestive heart fail- (1) Age: under 18 years, over 40 years.
ure, and Chapter 8 covers the most commonly used (2) Weight: under 100 lb, over 200 lb.
drugs, including digoxin and furosemide (Lasix). (3) Maternal type 1 (insulin-dependent)
B. Assessment: diabetes.
1. Exact cause unknown, but related factors e. Maternal history of drinking during
include: pregnancy, with resultant fetal alcohol
a. Familial history of CHD, especially in syndrome.
siblings, parents. f. Extracardiac defects, including tracheoe-
b. Presence of other genetic defects in infant sophageal fistula, renal agenesis, and
(e.g., Down syndrome, trisomy 13 or 18). diaphragmatic hernia.

Table 5.13
Comparison of Acyanotic and Cyanotic Heart Disease
Feature Acyanotic Cyanotic
Shunting of blood LR RL
Cyanosis Not usual (unless congestive heart failure) Always; blue babies
Surgery Usually done in one stagetechnically simple Usually done in several stagestechnically complex
Prognosis Very good/excellent Guarded
Major types 1. Atrial septal defect (ASD) 1. Tetralogy of Fallot (ToF)
2. Ventricular septal defect (VSD) 2. Transposition of the great vessels (TGV)
3. Patent ductus arteriosus (PDA)
4. Coarctation of the aorta (COA)

PEDIATRICS
Table 5.14
Overview of the Most Common Types of Congenital Heart Disease
Type of Defect Medical Treatment Surgical Treatment Prognosis
Acyanotic
Atrial septal defect Clinical trials using device closure Open chest/open heart surgery Excellent, with survival
(ASD) during cardiac catheterization with closure through patch (rec- rate greater than 99%
ommended age: preschooler).
Ventricular septal Clinical trials using device closure Palliative treatment: pulmonary Excellent, with 95% or
defect (VSD) during cardiac catheterization banding; definitive repair: same greater survival rate
as for ASD
Patent ductus In newborns, attempt pharmaco- Open chest: surgical division or Excellent, with survival
arteriosus (PDA) logical closure with indomethacin ligation of the patent ductus rate greater than 99%
(prostaglandin inhibitor)
Coarctation of the Infants or children with CHF: Open chest: resection of coarcted Fairless than 5%
aorta (COA) digoxin and diuretics portion of aorta with end-to-end mortality rate
anastomosis within first 2 years
of life
Nonsurgical: balloon angioplasty
Cyanotic
Tetralogy of Fallot Nonesupportive prn Often done in stages with defini- Fairless than 5%
(ToF) tive repair accomplished within mortality rate
first year of life
Transposition of the Nonesupportive prn Arterial switch procedure during Guarded5%10%
great vessels (TGV) first weeks of life mortality rate
2164_Ch05_249-346 29/03/12 12:25 PM Page 280

280 chapter 5 Health Promotion and Maintenance

2. Most frequent parental complaint: difficulty 3. Altered nutrition, less than body requirements,
feeding. related to difficulty in feeding.
a. Infant must be awakened to feed. 4. Risk for infection related to poor nutritional
b. Has weak suck. status.
c. May turn blue when eating, especially with 5. Knowledge deficit related to diagnostic proce-
cyanotic defects. dures, condition, surgical/medical treatments,
d. Infant takes overly long time to feed. prognosis.
e. Falls asleep during feeding, without finishing. D. Nursing care plan/implementation:
3. Nursing observations 1. Goal: promote adequate oxygenation.
a. Most frequent symptomtachycardia, as body a. Administer oxygen per physicians order/prn.
attempts to compensate for lack of oxygen b. Use loose-fitting clothing; tape diapers
(hypoxia): heart rate over 160 beats/min. loosely to avoid pressure on abdominal
b. Tachypnea, corresponding to heart rate: organs, which could impinge on diaphragm
respirations over 60 breaths/min. and impede respiration.
c. Cyanosis due to hypoxia: c. Position: neck slightly hyperextended to keep
(1) Not with acyanotic defects (unless CHF airway patent; place in knee-chest position
is present). to relieve tet spell (choking spell).
(2) Always with cyanotic defects (blue d. Suction prn to clear the airway.
infants). e. Administer digoxin, per physicians order, to
d. Failure to grow at a normal rate: slow weight slow and strengthen hearts pumping action
gain, height and weight below the norm due (refer to Chapter 8 and to Table 5.6 for
to difficulty feeding and hypoxia. pediatric pulse rate norms).
e. Developmental delays related to weakened f. Monitor pulse oximetry, as ordered.
physical condition. 2. Goal: reduce workload of heart to conserve
f. Frequent respiratory infections associated with energy.
increased pulmonary blood flow or aspiration. a. Position: infant seat, semi-Fowlers to promote
g. Dyspnea on exertion due to hypoxia, shunting maximum expansion of the lungs.
of blood. b. Provide pacifier to promote psychological rest.
h. Murmurs may or may not be present (e.g., c. Organize nursing care to provide periods of
PEDIATRICS

patent ductus arteriosus [PDA] machinery uninterrupted rest.


murmur). d. Adjust physical activity according to childs
i. Changes in blood pressure (e.g., coarctation condition, capabilities to conserve energy.
increased blood pressure in arms; decreased e. Provide diversion, as tolerated, to meet devel-
blood pressure in legs). opmental needs yet conserve energy.
j. Possible congestive heart failurerefer to f. Avoid extremes of temperature to avoid the
Chapter 6. Note: Infants may not demon- stress of hypothermia/hyperthermia, which
strate distended neck veins and may have will increase the bodys demand for oxygen.
difficult-to-detect generalized edema if not g. Administer diuretics (Lasix), per physicians
yet walkingcheck for facial, scrotal edema. order, to eliminate excess fluids, which
k. Cyanotic heart defects: increase the hearts workload. Note: Refer to
(1) Tet spellschoking spells with paroxys- Chapter 8.
mal dyspnea: severe hypoxia, deepening 3. Goal: provide for adequate nutrition.
cyanosis; relieved by placing infant in a. May need standard infant formula with
knee-chest position, which alters car- caloric density to minimize fluid retention
diopulmonary dynamics, thus increasing and meet nutritional needs.
the flow of blood to the lungs. b. Discourage foods with high or added sodium
(2) Clubbing of fingers and toesdue to to minimize fluid retention.
chronic hypoxia. c. I&O, daily/weekly weights, and monitor for
(3) Polycythemia (increased red blood cells rate of growth.
[RBCs]) with possible thrombi/emboli d. Limit PO feedings to 20 minutes to avoid
formation. overtiring infant. Supplement PO feeding
C. Analysis/nursing diagnosis: with gavage feeding (prn with physicians
1. Ineffective breathing pattern related to tachypnea order) to meet fluid and caloric needs.
and respiratory infection. e. Encourage foods high in potassium (prevent
2. Activity intolerance related to tachycardia and hypokalemia) and high in iron (prevent
hypoxia. anemia). Note: Refer to Chapter 9.
2164_Ch05_249-346 29/03/12 12:25 PM Page 281

Cardiovascular System 281


4. Goal: prevent infection. 4. The family (and child, when old enough)
a. Standard precautions to prevent infection. verbalize their understanding of the type of
b. Use good hand-washing technique. CHD, its treatment, and prognosis.
c. Limit contact with staff/visitors (especially 5. The family and child demonstrate adequate
children) with infections. coping mechanisms to deal with CHD.
d. Monitor for early symptoms and signs of II. RHEUMATIC FEVER
infection; report STAT.
A. Introduction: Rheumatic fever is an acute, systemic,
5. Goal: meet teaching needs of client, family.
inflammatory disease affecting multiple organs and
a. Explain diagnostic procedures: blood tests,
systems: heart, joints, CNS, collagenous tissue, etc.
x-rays, urine, ECG, echocardiogram, cardiac
Thought to be autoimmune in nature, it most com-
catheterization.
monly follows a streptococcus infection (Fig. 5.8)
b. Explain condition/treatment/prognosis
and occurs primarily in school-age children. In addi-
(see Table 5.14).
tion, it tends to recur, and the risk of permanent
c. Review nutrition and medications.
heart damage increases with each subsequent attack
d. Discuss how to adjust realistically to life
of rheumatic fever.
with congenital heart disease, activity
B. Assessment:
restrictions, etc.
1. Major manifestations (modified Jones criteria)
E. Evaluation/outcome criteria:
a. Carditis: tachycardia, cardiomegaly, murmur,
1. Childs level of oxygenation is maintained, as
congestive heart failure (CHF).
evidenced by pink color in nailbeds and mucous
b. Migratory polyarthritis: swollen, hot, red, and
membranes (for both light- and dark-skinned
excruciatingly painful large joints; migratory
children) and ease in respiratory effort.
and reversible.
2. Energy is conserved, thus reducing the hearts
c. Sydenhams chorea (St. Vitus dance): sudden,
workload as evidenced by vital signs within
aimless, irregular movements of the extremi-
normal limits.
ties; involuntary facial grimaces, speech dis-
3. The childs fluid and caloric requirements are
turbances, emotional lability, muscle weak-
met, allowing for physical growth to occur at
ness; completely reversible.
normal or near-normal rate.

PEDIATRICS

Figure 5.8 Sequelae of strep infections.


2164_Ch05_249-346 29/03/12 12:25 PM Page 282

282 chapter 5 Health Promotion and Maintenance

d. Erythema marginatum: reddish pink rash most e. Administer oxygen, digoxin/Lasix as ordered
commonly found on the trunk; nonpruritic, (if CHF develops). Note: Refer to Chapter 6
macular, clear center, wavy but clearly marked for additional information on CHF.
border; transient. 2. Goal: relieve discomfort.
e. Subcutaneous nodules: small, round, freely a. Use bed cradle to keep linens from resting on
movable, and painless swellings usually found painful joints.
over the extensor surfaces of the hands/feet b. Administer aspirin as ordered to relieve
or bony prominences; resolve without any pain.
permanent damage. c. Move child carefully, minimallysupport
2. Minor manifestations joints.
a. Clinical d. Do not massage; do not perform range-of-
(1) Previous history of rheumatic fever. motion (ROM) exercises; do not apply splints;
(2) Arthralgia. do not apply heat/cold. All these treatments
(3) Fevernormal in morning, rises in will cause increased pain and are not needed,
midafternoon, normal at night. because no permanent deformities will result
b. Laboratory from this type of arthritis.
(1) Increased erythrocyte sedimentation 3. Goal: promote safety and prevent injury related to
rate (ESR). chorea.
(2) Positive C-reactive protein. a. Use side rails: elevated, padded.
(3) Leukocytosis. b. Restrain in bed if necessary.
(4) Anemia. c. No oral temperatureschild may bite
(5) Prolonged P-R/Q-T intervals on ECG. thermometer.
3. Supportive evidence d. Spoon-feedno forks or knives, to prevent
a. Recent history of streptococcus infection: injury to oral cavity.
(1) Strep throat/tonsillitis. e. Assist with all aspects of ADLs until child can
(2) Otitis media. care for own needs.
(3) Impetigo. 4. Goal: provide diversion as tolerated.
(4) Scarlet fever. a. Encourage quiet diversional activities: hobbies,
b. Positive throat culture for streptococcus. reading, puzzles.
PEDIATRICS

c. Increased antistreptolysin-O (ASO) titer: b. Get homework, books; provide tutor as


indicates presence of streptococcus antibodies; condition permits.
begins to rise in 7 days, reaches maximum c. Encourage contact with peers: telephone calls,
level in 4 to 6 weeks. letters, cards.
C. Analysis/nursing diagnosis: 5. Goal: encourage child and family to comply with
1. Decreased cardiac output related to carditis. long-term antibiotic therapy.
2. Pain related to migratory polyarthritis. a. Begin antibiotics immediately, to eradicate
3. Risk for injury related to chorea. any lingering streptococcus infection.
4. Diversional activity deficit related to lengthy b. Duration of prophylaxis varies (5 years
hospitalization and recuperation. lifelong) and depends on cardiac involvement.
5. Knowledge deficit related to preventing cardiac c. Stress need to adhere to prescribed prophylaxis
damage, relieving discomfort, and preventing schedule.
injury. d. Enlist childs cooperation with therapy
6. Ineffective management of therapeutic regimen (e.g., hero badge).
with long-term antibiotic therapy and follow- 6. Goal: health teaching.
up care. a. To encourage compliance with prolonged
D. Nursing care plan/implementation: bedreststress that ultimate prognosis
1. Goal: prevent cardiac damage. depends on amount of cardiac damage.
a. Hospitalization, with strict bedrest. b. Teach necessity for long-term prophylactic
b. Monitor apical pulse for changes in rate, therapy, for example, during dental work,
rhythm, murmurs. childbirth, surgery (to prevent subacute
c. Evaluate tolerance of increased activity by bacterial endocarditis [SBE]). Instruct adoles-
apical rate: if heart rate increases by more cents to avoid body piercing and tattooing
than 20 beats/min over resting rate, child for same rationale.
should return to bed. c. Teach rationale: permanent cardiac damage
d. Offer low-sodium diet to prevent fluid (mitral valve) is more likely to occur with
retention. subsequent attacks of rheumatic fever.
2164_Ch05_249-346 29/03/12 12:25 PM Page 283

Cardiovascular System 283


E. Evaluation/outcome criteria: 6. Generalized erythematous rash on trunk and
1. No permanent cardiac damage occurs. extremities, without vesicles or crusts.
2. Child is free from discomfort or is able to 7. Irritability, anorexia.
tolerate discomfort. 8. Arthralgia and arthritis.
3. Injuries are avoided. 9. Panvasculitis of coronary arteries: formation
4. Childs need for diversional activity is met. of aneurysms and thrombi; CHF, myocarditis,
5. Child/family comply with long-term antibiotic pericardial effusion, arrhythmias, mitral
therapy/prophylactic therapy. insufficiency, myocardial infarction (MI).
III. KAWASAKI DISEASE (MUCOCUTANEOUS 10. Three phases: acute (onset of fever) subacute
LYMPH NODE SYNDROME) (resolution of fever and all outward clinical
signs) convalescent (without clinical signs
A. Introduction: Kawasaki disease (mucocutaneous
but laboratory values remain abnormal).
lymph node syndrome) is an acute, febrile, multi-
11. Laboratory tests:
system disorder believed to be autoimmune in
a. Elevated: ESR.
nature. Affecting primarily the skin and mucous
b. Elevated: white blood cell (WBC) count.
membranes of the respiratory tract, lymph nodes,
c. Elevated: platelet count.
and heart, Kawasaki disease has a low fatality rate
C. Analysis/nursing diagnosis:
(<2%), although vasculitis and cardiac involvement
1. Hyperthermia related to high, unremitting
(coronary artery changes) may result in major com-
fever.
plications in as many as 20% to 25% of children
2. Altered oral mucous membrane and impaired
with this disease. The disease is not believed to be
swallowing related to oropharyngeal
communicable, and the exact cause remains
manifestations.
unknown; geographic (living near fresh water) and
3. Impaired skin integrity related to desquamation.
seasonal (late winter, early spring) outbreaks do
4. Fluid volume deficit related to high fever and
occur. Kawasaki disease occurs in both boys and
poor oral intake.
girls between 1 and 14 years of age; 80% of cases
5. Altered tissue perfusion (cardiovascular,
occur in children under age 5 years. It is more
potential/actual) related to vasculitis or
common among children of Japanese or Korean
thrombi.
descent, although children from any ethnic back-
6. Knowledge deficit related to disease course,
ground may be affected. It may be preceded by

PEDIATRICS
treatment, prognosis.
URI or exposure to a freshly cleaned carpet. A
D. Nursing care plan/implementation:
complete and apparently spontaneous recovery
1. Goal: reduce fever.
occurs within 3 to 4 weeks in the majority of cases.
a. Monitor temperature every 2 hours or prn.
Treatment, which is primarily symptomatic, does
b. Administer aspirin (not acetaminophen
not appear to either enhance recovery or prevent
[Tylenol]) per physicians order. (Note:
complications, although recent research indicates
aspirin is the drug of choice to reduce fever;
that life-threatening complications and long-term
also has anti-inflammatory effect and
disability may be avoided or minimized with early
antiplatelet effect. Dose is 100 mg/kg/day
treatment (i.e., gamma globulin) to reduce cardio-
in divided doses q6h. Monitor for signs of
vascular damage.
salicylate toxicity.)
B. Assessment:
c. Tepid sponge baths or hypothermia blanket
1. Abrupt onset with high fever (102 to 106F)
per physicians order.
lasting more than 5 days that does not remit
d. Offer frequent cool fluids.
with the administration of antibiotics and
e. Apply cool, loose-fitting clothes; use cotton
antipyretics.
bed linens only (no heavy blankets).
2. Conjunctivitisbilateral, nonpurulent.
f. Seizure precautions.
3. Oropharyngeal manifestations:
2. Goal: provide comfort measures to oral cavity to
a. Dry, red, cracked lips.
ease the discomfort of swallowing.
b. Oropharyngeal reddening and a strawberry
a. Good oral hygiene with soft sponge and
tongue.
diluted hydrogen peroxide.
4. Peeling (desquamation) of the palms of the
b. Apply petroleum jelly to lips.
hands and the soles of the feet; begins at the
c. Bland foods in small amounts at frequent
fingertips and the tips of the toes; as peeling
intervals.
progresses, hands and feet become very red,
d. Avoid hot, spicy foods.
sore, and swollen.
e. Offer favorite foods from home or preferred
5. Cervical lymphadenopathy.
foods from hospital selection.
2164_Ch05_249-346 29/03/12 12:25 PM Page 284

284 chapter 5 Health Promotion and Maintenance

3. Goal: prevent infections and promote healing of d. Monitor circulatory status of extremities
skin. check for possible development of
a. Monitor skin for desquamation, edema, thrombi.
rash. e. Stress need for long-term follow-up, including
b. Keep skin clean, dry, well lubricated. ECGs and echocardiograms, possible cardiac
c. Avoid soap to prevent drying. catheterization (if coronary artery abnormali-
d. Gentle handling of skin to minimize ties exist at 1 year after disease).
discomfort. E. Evaluation/outcome criteria:
e. Provide sheepskin to lie on. 1. Fever returns to normal.
f. Prevent scratching and itchingapply cotton 2. Oral cavity heals, and child is able to swallow.
mittens if necessary. 3. Skin heals, and no infection occurs.
g. Bedrest; elevate edematous extremities. 4. Normal fluid balance is restored.
4. Goal: prevent dehydration and restore normal fluid 5. Normal cardiovascular functioning is
balance. reestablished, and no complications occur.
a. Strict I&O. 6. Parents/child verbalize their understanding of
b. Monitor urine specific gravity q8h for Kawasaki disease.
increase (dehydration) or decrease
(hydration).
c. Monitor vital signs for fevers, tachycardia,
GASTROINTESTINAL
arrhythmia.
SYSTEM
d. Monitor skin turgor, mucous membranes,
anterior fontanel for dehydration.
Structural Defects
e. Force fluids. I. CLEFT LIP AND CLEFT PALATE
f. IV fluids per physicians order. A. Introduction: Cleft lip and cleft palate are congeni-
5. Goal: prevent cardiovascular complications. tal facial malformations resulting from faulty
a. ECG monitorreport arrhythmias or embryonic development; there appear to be multi-
tachycardia. ple factors involved in the exact etiology: mutant
b. Administer aspirin (see Goal 1) and genes, chromosomal abnormalities, teratogenic
high-dose IV gamma globulin. agents, etc. The infant may be born with cleft lip
PEDIATRICS

c. Monitor for signs and symptoms of CHF: alone, cleft palate alone, or with both cleft lip and
tachycardia, tachypnea, dyspnea, crackles, cleft palate. Cleft lip and palate may occur unilat-
orthopnea, distended neck veins, dependent erally or bilaterally. Table 5.15 compares these
edema. conditions.

Table 5.15
Comparison of Cleft Lip and Cleft Palate
Dimension Cleft Lip Only Cleft Palate Only Both Cleft Lip and Cleft Palate
Incidence 1/7800 1/2000 Most common facial malformation
More common among boys More common among girls More common among boys
More common among Caucasians
than African Americans
Surgical repair Cheiloplasty Palatoplasty Lip always repaired before palate
Often done in a single stage Often done in staged repairs to enhance parent-infant attach-
Timing: age 612 weeks Timing: age 1218 months ment, bonding
(controversial; usually done prior
to development of faulty speech
habits)
Position Never on abdomen Always on abdomen
postoperatively

Feeding No sucking No sucking


postoperatively Use Breck feeder or Asepto Use wide-bowl spoon or
syringe plastic cup
Nursing care Elbow restraints Elbow restraints OK to show parents pictures of
postoperatively Lessen crying Lessen crying before and after repair
2164_Ch05_249-346 29/03/12 12:25 PM Page 285

Gastrointestinal System: Structural Defects 285

Table 5.15
Comparison of Cleft Lip and Cleft Palatecontd
Dimension Cleft Lip Only Cleft Palate Only Both Cleft Lip and Cleft Palate
Long-term concerns Bonding, attachment Defective speechrefer to
speech therapist
Social adjustmentpotential
threat to self-image Abnormal dentitionrefer to
orthodontist
Hearing lossrefer to pediatric
eye, ear, nose, throat specialist/
physician

B. Assessment: (2) Monitor weight gain carefully, to ensure


1. Cleft lipobvious facial defect, readily detectable adequate rate of growth.
at time of birth. (3) No sucking for either cleft lip or palate
2. Cleft palatemust feel inside infants mouth to repair until incision is healed.
check for presence of palatal defect and to note (4) Avoid stretching or pulling at incision site;
extent of defect: soft palate only or soft palate metal Logan bow may be used as external
and hard palate. brace for cleft lip repair.
3. Bothmajor problems with feeding: difficult to 2. Goal: promote parent-infant attachment.
feed, noisy sucking, swallows excessive amounts a. Show no discomfort handling infant; convey
of air, prone to aspiration. acceptance.
4. Parent-infant attachment (bonding) may be b. Stay with parents the first time they see/hold
adversely affected due to loss of perfect infant, infant.
multiple hospitalizations; note amount and c. Offer positive comments about infant.
quality of parent-infant interaction. d. Give positive reinforcement to parents initial
C. Analysis/nursing diagnosis: attempts at parenting.

PEDIATRICS
1. Altered nutrition, less than body requirements, e. Encourage parents to assume increasing
related to physical defect. independence in care of their infant.
2. Impaired physical mobility (postoperative) related f. Allow rooming-in on subsequent
to postoperative care requirements. hospitalizations.
3. Altered parenting related to birth of child with 3. Goal: teach parents particulars of feeding and need
obvious facial defect. for long-term follow-up care.
4. Knowledge deficit, actual or risk for, potential, a. Teach parents regarding long-term concerns
related to treatment and follow-up. (see Table 5.15).
D. Nursing care plan/implementation: b. Make necessary referrals before discharge:
1. Goal: maintain adequate nutrition. (1) Specialists: speech, dentition, hearing.
a. Preoperative: first encourage parents to watch (2) Home health nurse.
nurse feed infant, then teach parents proper (3) Social service.
feeding techniques: (4) Disabled childrens services for financial
(1) Use Breck feeder or Asepto syringe. assistance.
(2) Deposit formula on back of tongue to facil- (5) Local craniofacial malformations support
itate swallowing and to prevent aspiration. group.
(3) Rinse mouth with sterile water after feed- c. Refer parents to genetic counseling services
ings, to prevent infection. because of mixed genetic/environmental
(4) Feed slowly, with child in sitting position, etiology.
to prevent aspiration. d. Encourage parents to promote self-esteem in
(5) Burp frequently, because infant will swallow infant/child as child grows and develops.
air along with formula due to the defect. E. Evaluation/outcome criteria:
(6) Monitor weight. 1. Adequate nutrition is provided, and infant grows
b. Postoperative at normal rate for age.
(1) Begin with clear liquids when child has 2. Parent-infant attachment is formed.
fully recovered from anesthesia (see 3. Parents verbalize confidence in their ability to
Table 5.15). care for infant.
2164_Ch05_249-346 29/03/12 12:25 PM Page 286

286 chapter 5 Health Promotion and Maintenance

II. TRACHEOESOPHAGEAL FISTULA 3. Second system affected is GI:


A. Introduction: Tracheoesophageal fistula (TEF) is a a. Abdominal distention because excessive
congenital anomaly resulting from faulty embry- air enters stomach with each breath infant
onic development; although there are numerous takes.
types of TEF, the major problem is an anatomi- b. Inability to aspirate stomach contents when
cal defect that results in an abnormal connection attempting to pass NG tube.
between the trachea (respiratory tract) and the c. If all these signs are not correctly interpreted
esophagus (GI system) (Fig. 5.9). No exact cause and feeding is attempted, infant takes two to
has been identified; however, infants born with three mouthfuls, coughs and gags, and force-
TEF are often premature, with a maternal history fully exhales formula through nostrils.
of polyhydramnios. Diagnosis should be made C. Analysis/nursing diagnosis:
immediately, within hours after birth, and prefer- 1. Ineffective breathing pattern/ineffective airway
ably before feeding (to avoid aspiration pneumo- clearance related to excess mucus.
nia). Associated anomalies include: CHD, 2. Altered nutrition, less than body requirements,
anorectal malformations, and genitourinary related to inability to take fluids by mouth.
anomalies. 3. Anxiety related to surgery, condition, preterm
B. Assessment: delivery, and uncertain prognosis.
1. Perinatal history: maternal polyhydramnios, pre- 4. Knowledge deficit regarding discharge care of
mature birth. infant related to gastrostomy tube, feeding.
2. Most important system affected is respiratory: D. Nursing care plan/implementation:
a. Shortly after birth, infant has excessive 1. Goal: prepare neonate for surgery.
amounts of mucus. a. Stress to parents that surgery is only possible
b. Mucus bubbles or froths out of nose and treatment.
mouth as infant literally exhales mucus. b. Allow parents to see neonate before surgery to
c. The 3 Cs: coughing, choking, cyanosis promote bonding and attachment.
because mucus accumulates in respiratory c. Maintain NPOprovide IV fluids, monitor
tract. I&O, gastrostomy tube.
d. Pinks up with suctioning, only to experi- d. Position: elevate HOB 20 to 30 degrees to
ence repeated respiratory distress within a prevent aspiration.
PEDIATRICS

short time as mucus builds up again. e. Administer warmed, humidified oxygen, as


e. Aspiration pneumonia occurs early. ordered, to relieve hypoxia and to prevent
f. Respiratory arrest may occur. cold stress.

A B C
Figure 5.9 Esophageal malformations. EA = esophageal atresia; TEF = tracheoesophageal fistula. (A) The esophagus ends in a
blind pouch with a fistula between the distal esophagus and trachea. (B) Esophageal atresia without fistula. (C) Tracheoesophageal
fistula without esophageal atresia. (From Bowden, VR, Dickey, SB, & Greenberg, CS: Children and Their Families: The Continuum of Care. WB Saunders,
Philadelphia, 1999, p 105.)
2164_Ch05_249-346 29/03/12 12:25 PM Page 287

Gastrointestinal System: Obstructive Disorders 287


2. Goal (postoperative): maintain patent airway. E. Evaluation/outcome criteria:
a. Position: elevate HOB 20 to 30 degrees. 1. Neonate survives immediate surgical repair with-
b. Care of chest tubes (open-chest procedure). out untoward difficulties.
c. Care of endotracheal tube/ventilator (neonate 2. Patent airway is maintained; adequate oxygena-
frequently requires ventilatory assistance for tion is provided.
24 to 48 hours postoperatively). 3. Adequate nutrition is maintained; infant begins
d. Monitor for symptoms and signs of pneumonia to gain weight and grow.
(most common postoperative complication): 4. Parents verbalize confidence in ability to care for
(1) Aspiration. infant on discharge.
(2) Hypostatic, secondary to anesthesia.
e. Monitor for symptoms and signs of respirato-
ry distress syndrome (preterm infant). Obstructive Disorders
f. Use special precautions when suctioning:
suction with marked catheter to avoid I. HYPERTROPHIC PYLORIC STENOSIS
exerting undue pressure on newly sutured A. Introduction: Hypertrophic pyloric stenosis (HPS)
trachea. causes obstruction of the upper GI tract, but the
g. Administer prophylactic/therapeutic antibiotics, infant frequently does not have symptoms until 2 to
as ordered. 4 weeks of age. HPS results in thickening, or hyper-
h. Administer warmed, humidified oxygen, as trophy, of the pyloric sphincter located at the distal
ordered; monitor arterial blood gases (ABGs). end of the stomach; this causes a mechanical intes-
3. Goal: maintain adequate nutrition. tinal obstruction that becomes increasingly evident
a. 48 to 72 hours postoperatively: IV fluids as the infant begins to consume larger amounts of
only. formula during the early weeks of life. Pyloric steno-
b. Maintain NPO for 10 to 14 days, until sis is five times more common in boys than girls and
esophagus is fully healed (offer pacifier). is most often found in full-term Caucasian infants.
c. When condition is stable: begin gastrostomy The exact etiology remains unknown; however, there
tube (G-tube) feedings, as ordered. does seem to be a genetic predisposition.
(1) Start with small amounts of clear liquids. B. Assessment:
(2) Gradually increase to full-strength formula. 1. Classic symptom is vomiting:
a. Begins as nonprojectile at age 2 to 4 weeks.

PEDIATRICS
(3) Postoperative: leave G-tube open and
elevated slightly above level of stomach b. Advances to projectile at age 4 to 6 weeks.
to prevent aspiration if infant vomits. c. Vomitus is nonbile stained (stomach con-
(4) Offer pacifier ad lib. tents only).
d. Between 10th and 14th postoperative day: d. Most often occurs shortly after a feeding.
begin oral feedings. e. Major problem is the mechanical obstruction
(1) Start with clear liquids again. of the flow of stomach contents to the small
(2) Note ability to suck and swallow. intestine due to the anatomical defect of
(3)Offer small amounts at frequent stenosis of the pyloric sphincter.
intervals. f. No apparent nausea or pain, as evidenced by
(4) May need to supplement postop feeding the fact that infant eagerly accepts a second
with G-tube feeding prn. feeding after episode of vomiting.
e. Monitor weight, I&O. g. Metabolic alkalosis develops due to loss of
4. Goal: prepare parents to successfully care for the hydrochloric acid.
infant after discharge. 2. Inspection of abdomen reveals:
a. Teach parents that infant will probably be a. Palpable olive-shaped mass in right upper
discharged with G-tube in place; teach care of quadrant.
G-tube at home. b. Visible peristaltic waves, moving from left to
b. Teach parents symptoms and signs of most right across upper abdomen.
common long-term problems (i.e., esophageal 3. Weight: fails to gain or loses.
reflux, stricture formation). 4. Stools: constipated, diminished in number and
(1) Refusal to eat solids or swallow liquids. sizedue to loss of fluids with vomiting.
(2) Dysphagia. 5. Signs of dehydration may become evident
(3) Increased coughing or choking. (Table 5.16).
c. Stress need for long-term follow-up care. 6. Upper GI series and ultrasonography reveal:
d. Offer realistic encouragement, because prog- a. Delayed gastric emptying.
nosis is generally good. b. Elongated and narrowed pyloric canal.
2164_Ch05_249-346 29/03/12 12:25 PM Page 288

288 chapter 5 Health Promotion and Maintenance

Table 5.16 e. Position after feeding: high Fowlers turned to


right side; minimal handling after feeding to
Signs and Symptoms of Dehydration in Infants prevent vomiting.
and Young Children 3. Goal (preoperative and postoperative): institute
Weight loss (most important to assess): preventive measures to avoid infection or skin
Mild dehydration: <5% weight loss breakdown.
Moderate dehydration: 5%9% weight loss a. Use good hand-washing technique.
Severe dehydration: 10%15% weight loss
Skin: gray, cold to touch, poor skin turgor (check skin
b. Administer good skin care, especially in
across abdomen) diaper area (urine is highly concentrated);
Mucous membranes: dry oral buccal mucosa; salivation give special care to any reddened areas.
absent c. Give mouth care when NPO or after
Eyes: sunken eyeballs; absence of tears when crying vomiting.
Anterior fontanel (in infant): sunken
Shock: increased pulse, increased respirations, decreased
d. Tuck diaper down below suture line to prevent
blood pressure contamination with urine (postoperatively).
Urine: oliguria, increased specific gravity, ammonia odor e. Note condition of suture linereport any
Alterations in level of consciousness: irritability, lethargy, redness or discharge immediately.
stupor, coma, possible seizures f. Screen staff and visitors for any sign of
Metabolic acidosis (with diarrhea)
Metabolic alkalosis (with vomiting)
infection.
4. Goal: do discharge teaching to prepare parents to
care for infant at home.
a. Teach parents that defect is anatomical and
unrelated to their parenting behavior/skill.
C. Analysis/nursing diagnosis:
b. Demonstrate feeding techniques, and remind
1. Fluid volume deficit related to vomiting.
parents that vomiting may still occur.
2. Altered nutrition, less than body requirements,
c. Stress that repair is complete; this condition
related to vomiting.
will never recur.
3. Risk for injury/infection related to altered nutri-
d. Instruct parents in care of the suture line: no
tional state.
baths for 10 days, tuck diaper down, report
4. Impaired skin integrity related to dehydration and
any signs of infection promptly.
altered nutritional state.
PEDIATRICS

e. Offer follow-up referrals as indicated.


5. Knowledge deficit related to cause of disease,
E. Evaluation/outcome criteria:
treatment and surgery, prognosis, and follow-up
1. Infant survives surgical repair without untoward
care.
difficulties (including infection/skin breakdown).
D. Nursing care plan/implementation:
2. Adequate nutrition is maintained, and infant
1. Goal (preoperative): restore fluid and electrolyte
begins to grow and gain weight.
balance.
3. Parents verbalize confidence in their ability to
a. Generally NPO, with IVs preoperatively to
care for their infant on discharge.
provide fluids and electrolytes.
b. Observe and record I&O, including vomiting II. HIRSCHSPRUNGS DISEASE (congenital
and stool. aganglionic megacolon)
c. Weight: check every 8 hours or daily. A. Introduction: Hirschsprungs disease is a congenital
d. Monitor laboratory data. anomaly of the lower GI tract, but the diagnosis
2. Goal (postoperative): provide adequate nutrition. often is not established until the infant is 6 to
a. Maintain NPO with IVs for 4 to 6 hours 12 months old. The major problem is a functional
postoperatively, as ordered (can offer obstruction of the colon caused by the congenital
pacifier). anatomical defect of lack of nerve cells in the walls
b. Follow specific feeding regimen ordered by of the colon, resulting in the absence of peristalsis.
doctorgenerally start with clear fluids in Hirschsprungs disease is four times more common
small amounts hourly, increasing slowly as in boys than girls and is frequently noted in chil-
tolerated. Full feeding schedule reinstated dren with Down syndrome.
within 48 hours. Offer pacifier between B. Assessment:
feedings. 1. In the newborn, failure to pass meconium
c. Fed only by RN for 24 to 48 hours, because (in addition to other signs and symptoms of
vomiting tends to continue in immediate intestinal obstruction).
postoperative period. 2. Obstinate constipationhistory of inability to
d. Burp wellbefore, during, and after feeding. pass stool without stool softeners, laxatives, or
2164_Ch05_249-346 29/03/12 12:25 PM Page 289

Gastrointestinal System: Obstructive Disorders 289


enemas; persists despite all attempts to treat 4. Goal (postoperative): discharge teaching to pre-
medically. pare parents to care at home for infant with a
3. Stools are infrequent and tend to be thin and colostomy.
ribbonlike. a. Home care of colostomy of infant is essentially
4. Vomiting: bile stained, flecked with bits of stool same as for adult (see Chapter 6).
(breath has fecal odor), due to GI obstruction b. Teach parents to keep written records of
and eventual backing up of stools. stools: number, frequency, consistency.
5. Abdominal distention can be severe enough to c. Teach parents to tape diaper below colostomy
impinge on respirations, due to GI obstruction to prevent irritation.
and retention of stools. d. Because colostomy is usually temporary, discuss:
6. Anorexia, nausea, irritability due to severe (1) Second-stage repair (closure and pull-
constipation. through) done when the child weighs
7. Malabsorption results in anemia, hypoproteinemia, approximately 20 lb.
and loss of subcutaneous fat. (2) Possible difficulties in toilet training.
8. Visible peristalsis and palpable fecal masses may e. Stress need for long-term follow-up care.
also be detected. f. Make referral to home care if indicated.
C. Analysis/nursing diagnosis: E. Evaluation/outcome criteria:
1. Constipation related to impaired bowel 1. Infant is prepared for surgery and tolerates
functioning. procedure well.
2. Altered nutrition, less than body requirements, 2. Postoperative recovery is uneventful.
related to poor absorption of nutrients. 3. Parents verbalize confidence in ability to care at
3. Risk for injury/infection related to malnutrition. home for infant with a colostomy and verbalize
4. Pain related to surgery and treatments. their understanding that second surgery will be
5. Knowledge deficit regarding care of the child with needed to close the colostomy.
a colostomy and follow-up care. III. INTUSSUSCEPTION
D. Nursing care plan/implementation:
A. Introduction: Intussusception is the apparently spon-
1. Goal (preoperative): promote optimum nutritional
taneous telescoping of one portion of the intestine
status, fluid and electrolyte balance.
into another, resulting in a mechanical obstruction
a. Monitor for signs and symptoms of progres-
of the lower GI tract. There is no known cause, and

PEDIATRICS
sive intestinal obstruction: measure abdominal
intussusception is three times more common in boys
girth daily.
than girls; the child with intussusception is usually
b. Administer IV fluids, as orderedmay
between 3 and 36 months of age.
include total parenteral nutrition (TPN) or
B. Assessment:
intravenous lipids.
1. Typically presents with sudden onset in child
c. Daily weights, I&O, urine specific gravity.
who is healthy, thriving.
d. Monitor for possible dehydration.
2. Pain: paroxysmal, colicky, abdominal, with
e. Diet: low residue.
intervals when the child appears normal and
2. Goal (preoperative): assist in preparing bowel for
comfortable.
surgery.
3. Stools: currant-jelly, bloody, mixed with mucus.
a. Teach parents what will be done and
4. Vomiting due to intestinal obstruction.
whyenlist their cooperation as much as
5. Abdomen: distended, tender, with palpable,
possible.
sausage-shaped mass in right upper quadrant
b. Insert NG tube, connect to low suction to
(RUQ).
achieve and maintain gastric decompression.
6. Late signs: fever, shock, signs of peritonitis as the
c. Position: semi-Fowlers.
compressed bowel wall becomes necrotic and
d. Bowel is cleansed with a series of isotonic
perforates.
saline (0.9%) enemas.
C. Analysis/nursing diagnosis:
e. Administer oral antibiotics and colonic irriga-
1. Fluid volume deficit related to diarrhea and
tions to decrease bacteria.
vomiting.
f. Take axillary temperatures only.
2. Pain related to bowel-wall ischemia, necrosis,
g. If child can understand, prepare for probable
and death.
colostomy using pictures, dolls (usual age at
3. Risk for injury/infection related to bowel-wall
surgery is 10 to 16 months).
perforation and peritonitis.
3. Postoperative goals: same as for adult having
4. Knowledge deficit regarding the disease, medical
major abdominal surgery or a colostomy (see
or surgical treatment, and prognosis.
Chapter 6).
2164_Ch05_249-346 29/03/12 12:25 PM Page 290

290 chapter 5 Health Promotion and Maintenance

D. Nursing care plan/implementation: 5. Altered tissue perfusion related to dehydration


1. Goal: assist with attempts at medical and hypovolemia.
treatment. 6. Knowledge deficit regarding diagnosis, dietary
a. Explain to parents that a barium enema restrictions, treatment.
will be given to the child in an attempt to D. Nursing care plan/implementation:
reduce the telescoping through hydrostatic 1. Goal: prevent spread of infection.
pressure (succeeds in 75% of cases). a. Standard precautions to prevent infection.
b. Stress that if this treatment is not successful, b. Enforce strict hand washing.
or if perforation of the bowel wall has already c. Institute and maintain enteric precautions
occurred, surgery will be necessary. follow policies regarding linens, excretions,
c. If medical treatment is apparently successful, specimens (double-bag, special tag).
monitor child for 24 to 36 hours for recur- d. Tape diapers snugly; keep hands out of mouth.
rence before discharge. e. Obtain stool culture to identify causative
2. Preoperative and postoperative goals: same as organism; then administer antibiotics as
for adult with major abdominal surgery (see ordered.
Chapter 6). f. Identify family members and others at high
3. Goal: discharge teaching to prepare parents for care risk, obtain cultures.
of the child at home. 2. Goal: restore fluid and electrolyte balance.
a. Stress that recurrence is rare (10%) and most a. Administer IV fluids and electrolytes as
often occurs within the first 24 to 36 hours ordered.
after reduction. b. Monitor for appropriate response to therapy:
b. Other teaching: same as for adult going home decreased specific gravity, good skin turgor,
after bowel surgery (see Chapter 6). normal vital signs.
E. Evaluation/outcome criteria: c. Monitor weight, I&O, specific gravity.
1. Infant tolerates medical-surgical treatment and d. Oral feedingsoral rehydration therapy
completely recovers. (ORT) with Pedialyte or comparable solution;
2. Parents verbalize confidence in ability to care resume normal diet as quickly as possible.
for infant after discharge. e. Ongoing assessment of stools: note Amount,
Color, Consistency, Timing (ACCT).
PEDIATRICS

3. Goal: maintain or restore skin integrity.


Disorders of Motility a. Frequent diaper changes.
b. Keep perineal area clean and dry.
I. ACUTE GASTROENTERITIS (AGE) c. Apply protective ointments (e.g., petroleum
A. Introduction: In infants and young children, jelly, A and D emollient ointment).
gastroenteritis is a common acute illness d. If feasible, expose reddened buttocks to air
that can rapidly progress to dehydration, (but not with explosive diarrhea).
hypovolemic shock, and severe electrolyte 4. Goal: provide discharge teaching to parents.
disturbances. a. Careful review of diet to be followed at home.
B. Assessment: b. Review principles of food preparation and
1. Diarrhea: often watery, green, explosive, contains storage to prevent infection.
mucus and blood. c. Instruct in disposal of stools at home.
2. Abdominal cramping and pain, often accompa- d. Emphasize importance of good hygiene.
nied by bouts of diarrhea. E. Evaluation/outcome criteria:
3. Dehydration: see Table 5.16. 1. No spread of infection noted.
4. Irritability, restlessness, alterations in level of 2. Fluid and electrolyte balance normal.
consciousness. 3. No skin breakdown noted.
5. Electrolyte disturbances: see Chapter 6. 4. Parents verbalize understanding of home care.
C. Analysis/nursing diagnosis:
1. Fluid volume deficit related to vomiting and
diarrhea. GENITOURINARY
2. Altered nutrition, less than body requirements, SYSTEM
related to AGE and its treatment (i.e., dietary
restrictions). Genitourinary Tract Disorder
3. Pain related to abdominal cramping, I. HYPOSPADIAS
diarrhea. A. Introduction: Hypospadias is a congenital anatomi-
4. Impaired skin integrity related to diarrhea. cal defect of the male genitourinary tract, readily
2164_Ch05_249-346 29/03/12 12:25 PM Page 291

Genitourinary System: Kidney Tumor 291


detected at birth through simple visual examina-
tion. In hypospadias, the urethral opening is locat-
Kidney Tumor
ed on the ventral surface of the penile shaft; this I. WILMS TUMOR (NEPHROBLASTOMA)
makes voiding in the standing position virtually A. Introduction: Wilms tumor, a malignant tumor
impossible, which creates potential for serious psy- of the kidney, is the most common form of renal
chological problems. Ideally, staged surgical repair cancer in children. Peak incidence occurs at 3 years
should be completed by 6 to 18 months of age, of age, with a slightly higher incidence in boys than
before body image is developed or castration fears girls. Ninety percent of the cases occur unilaterally;
are evident. the treatment of choice is nephrectomy (and
B. Assessment: adrenalectomy) followed by chemotherapy and
1. Urethral opening is located on ventral surface radiation.
of penis. B. Assessment:
2. May be accompanied by chordeeventral 1. Most common sign: abdominal mass (firm,
curvature of the penis due to a fibrous band nontender).
of tissue. 2. Most often first found by parent changing
3. Rare: ambiguous genitalia, resulting in need for diaper; felt as a mass over the kidney area.
chromosomal studies to determine sex of 3. Intravenous pyelogram (IVP), abdominal
neonate. ultrasound, and CT confirm the diagnosis.
C. Analysis/nursing diagnosis: 4. Metastasis occurs most frequently to the lungs:
1. Altered urinary elimination related to congenital pain in chest, cough, dyspnea.
anatomical defect of penis. C. Analysis/nursing diagnosis: altered urinary
2. Pain related to surgery and treatments. elimination (other diagnoses depend on stage of
3. Self-esteem disturbance related to anatomical tumor and presence of metastasissimilar to
defect in penis and resulting disturbance in adult with cancer).
ability to void standing up. D. Nursing care plan/implementation:
4. Knowledge deficit related to condition, surgeries, 1. Goal: promote normal urinary function.
outcome. a. Inform family that surgery is scheduled as
D. Nursing care plan/implementation: soon as possible after confirmed diagnosis
1. Goal: promote normal urinary function. (within 2448 hours).

PEDIATRICS
a. Teach family that surgery is done in several b. Explain to family that the preferred surgical
stages, beginning in the early months of life approach is nephrectomy (and adrenalectomy).
and finishing by age 18 months. c. Preoperative: do not palpate abdomen
b. Provide age-appropriate information to child because the tumor is highly friable, and
regarding condition, surgery. palpation increases the risk of metastasis.
c. Preoperative teaching with child: simulate d. Postoperative nursing care: similar to care of
anticipated postoperative urinary drainage adult with nephrectomy (see Chapter 6).
apparatus and dressings on dolls; allow child e. Postoperative care also includes long-term
to handle and play with them now, but stress radiation therapy and chemotherapy
need not to touch postoperatively. (actinomycin D, vincristine, Adriamycin;
d. Postoperatively: Monitor urinary drainage see Chapters 6 and 8).
apparatus; note hourly urine output, color, 2. Goal: discharge teaching to prepare parents to care
appearance (should be clear yellow, no for child at home.
blood). a. Teach parents need for long-term follow-up
2. Goal: promote self-esteem. care with specialists: oncologist, urologist.
a. Do not scold child if he exposes penis, dress- b. Answer questions regarding prognosis,
ings, catheters, etc. offering realistic hope.
b. Reassure parents that preoccupation with (1) Children with localized tumor:
penis is normal and will pass. 90% survival rate.
c. Encourage calm, matter-of-fact acceptance of, (2) Children with metastasis: 50% survival
and avoid strict discipline for this behavior, rate.
which could affect the child negatively. E. Evaluation/outcome criteria:
E. Evaluation/outcome criteria: 1. Child is able to maintain normal urinary
1. Child is able to void in normal male pattern. elimination.
2. Child does not experience disturbances in 2. Parents verbalize their understanding of home
self-concept and has normal self-esteem. care for the child.
2164_Ch05_249-346 29/03/12 12:25 PM Page 292

292 chapter 5 Health Promotion and Maintenance

Glomerular Diseases The nursing care plan for the child with nephrosis is
very similar to that for the adult with compromised renal
I. NEPHROSIS functioning. Refer to Chapters 8 and 9 for additional
Nephrosis (idiopathic nephrotic syndrome) is a chronic information about dietary restrictions and medications;
renal disease having no known cause, variable pathology, refer to Table 5.17 for a chart comparing nephrosis and
and no known cure. It is thought that several different nephritis.
pathophysiological processes adversely affect the glomeru- II. ACUTE POSTSTREPTOCOCCAL
lar membranes of the kidneys, resulting in increased per- GLOMERULONEPHRITIS
meability to protein. This leakage of protein into the A. Introduction: Acute poststreptococcal glomeru-
urine results in massive proteinuria, severe hypoproteinemia, lonephritis (APSGN) is a bilateral inflammation
and total body edema. A chronic disease, nephrosis often of the glomeruli of the kidneys and is the most
has its onset during the preschool years but is characterized common noninfectious renal disease of childhood.
by periods of exacerbation and remission throughout the It occurs most frequently in early school-age
childhood years.

Table 5.17
Comparison of Nephrosis and Acute Poststreptococcal Glomerulonephritis
Factor Nephrosis (Nephrotic Syndrome) Acute Poststreptococcal Glomerulonephritis (APSGN)
Illness type Chronic Acute
Illness course Characterized by periods of Predictable, self-limiting, typically lasting 410 days
exacerbations and remissions (acute edematous phase)
during many years
Cause Unknown Group A -hemolytic streptococci
Age at onset 24 years Early school-age children; peaks at 67 years
Sex More common among boys More common among boys
Major signs and symptoms:
PEDIATRICS

General Syndrome with variable pathology: Hematuria, hypertension


massive proteinuria, hypoalbu-
minemia, severe edema,
hyperlipidemia
Blood pressure Normal or decreased Elevated
Edema Generalized and severe Periorbital and peripheral
Proteinuria Massive Moderate
Serum protein level Decreased (6.17.9 g/dL) Slightly decreased
Serum lipid level Elevated Normal
Potassium level Normal (3.55 mEq/L) Increased
Treatment Symptomaticno known cure; Penicillin (EES), hydralazine, furosemide (sources are
prednisone, cyclophosphamide, divided regarding use of prophylactic antimicrobials)
furosemide
Diet Decrease sodium, increase Decrease sodium, decrease potassium, decrease
protein (unless azotemia protein (if azotemia develops)
develops)
Fluid restrictions Seldom necessary Necessary if output is significantly reduced
Specific nursing care Treat at home if possible; good Treat in hospital during acute phase; monitor vital
skin care; prevent infection signs, especially blood pressure; on discharge, stress
need to restrict strenuous activity until microscopic
hematuria is gone
Prognosis Fair; subject to long-term steroid Good; stress that recurrence is rare because specific
treatment and social isolation immunity is conferred
related to frequent hospitaliza-
tions/confinement during relapses;
20% suffer chronic renal failure
2164_Ch05_249-346 29/03/12 12:25 PM Page 293

Genitourinary System: Glomerular Diseases 293


children, with a peak age of onset of 6 to 7 years; it c. Urine: strict I&O; specific gravity and
is twice as common in boys as in girls. Like rheu- dipstick for blood every void.
matic fever, acute glomerulonephritis is thought to d. Note edema: extent, location, progression.
be the result of an antigen-antibody reaction to a e. Adhere to fluid restrictions if ordered.
streptococcus infection (see Fig. 5.8); however, f. Monitor for possible development of hyper-
unlike rheumatic fever, it does not tend to recur, tensive encephalopathy (seizures, increased
because specific immunity is conferred following intracranial pressure); report any changes
the first episode of APSGN. (Further information STAT to physician.
about APSGN is found in Table 5.17.) g. Administer medications as ordered:
B. Assessment: (1) Antibioticseradicate any lingering
1. Typical concerns from family about urine: streptococcus infection; controversial.
change in color/appearance of urine (thick, (2) Antihypertensives (e.g., Apresoline).
reddish brown; decreased amounts). (3) Rarely use diureticslimited value.
2. Acute edematous phaseusually lasts 4 to 10 days. (4) If CHF developsmay use digoxin.
a. Laboratory examination of urine: (5) Refer to Chapter 8 for additional informa-
(1) Severe hematuria. tion on medications.
(2) Mild proteinuria. 2. Goal: provide adequate nutrition.
(3) Increased specific gravity. a. Diet: low sodium, low potassiumto prevent
b. Hypertension fluid retention and hyperkalemia; decrease
(1) Headache. protein (if azotemia develops). Refer to
(2) Potential hypertensive encephalopathy Chapter 9 for additional information on
leading to seizures, increased intracranial diets.
pressure. b. Stimulate appetite: offer small portions,
c. Mild-moderate edema: chiefly periorbital; attractively prepared; meals with family
increased weight due to fluid retention. or other children; offer preferred foods,
d. General: if possible; encourage parents to bring in
(1) Abdominal pain. special foods (e.g., culturally related
(2) Malaise. preferences).
(3) Anorexia. 3. Goal: provide reasonable measure of comfort.

PEDIATRICS
(4) Vomiting. a. Encourage parental visiting.
(5) Pallor. b. Provide for positional changes, give good
(6) Irritability. skin care.
(7) Lethargy. c. Provide appropriate diversion, as tolerated.
(8) Fever. 4. Goal: prevent further infection.
3. Diuresis phase: a. Use good hand washing technique.
a. Copious diuresis. b. Screen staff, other clients, visitors (especially
b. Decreased body weight. children) to limit contact with people who
c. Marked clinical improvement. are infectious.
d. Decrease in gross hematuria, but microscopic c. Administer antibiotics if ordered (usually only
hematuria may persist for weeks/months. for children with positive cultures).
C. Analysis/nursing diagnosis: d. Keep warm and dry, stress good hygiene.
1. Fluid volume excess related to decreased urine e. Note possible sites of infection: increased skin
output. breakdown secondary to edema.
2. Pain related to fluid retention. 5. Goal: teach child and family about APSGN/
3. Altered nutrition, less than body requirements, discharge planning.
related to anorexia and vomiting. a. Teach how to check urine at home: dipstick
4. Impaired skin integrity related to immobility. for protein and blood. (Note: occult hema-
5. Activity intolerance related to fatigue. turia may persist for months.)
6. Knowledge deficit related to disease process, b. Teach activity restriction: no strenuous activi-
treatment, and follow-up care. ty until hematuria is completely resolved.
D. Nursing care plan/implementation: c. Teach family how to prepare low-sodium,
1. Goal: monitor fluid balance, observing carefully for low-potassium diet.
complications. d. Arrange for follow-up care: physician, home
a. Check and record blood pressure at least every health nurse.
4 hours to monitor hypertension. e. Stress: subsequent recurrences are rare because
b. Monitor daily weights. specific immunity is conferred.
2164_Ch05_249-346 29/03/12 12:25 PM Page 294

294 chapter 5 Health Promotion and Maintenance

E. Evaluation/outcome criteria: is typically insidious, and the disease is most


1. No permanent renal damage occurs. common in preschoolers (age 26 years).
2. Normal fluid balance is maintained/restored. B. Assessment:
3. Adequate nutrition is maintained. 1. Major problemleukopenia: decreased
4. No secondary infections occur. WBCs/increased blasts (overproduction of
5. Child/family verbalize their understanding of immature, poorly functioning WBCs).
the disease, its treatment, and its prognosis. 2. Bone marrow dysfunction results in:
a. Neutropenia: multiple prolonged infections.
b. Anemia: pallor, weakness, irritability, shortness
ENDOCRINE SYSTEM of breath.
I. DIABETES MELLITUS TYPE 1 c. Thrombocytopenia: bleeding tendencies
Diabetes mellitus type 1 was formerly called insulin- (petechiae, epistaxis, bruising).
dependent diabetes mellitus (IDDM) and juvenile-onset 3. Infiltration of reticuloendothelial system
diabetes. Diabetes mellitus is fully covered in Chapter 6. (RES): hepatosplenomegaly, abdominal pain,
Only the differences between adults and children are cov- lymphadenopathy.
ered in Table 5.18. 4. Leukemic invasion of CNS: increased intracranial
pressure/leukemic meningitis.
5. Leukemic invasion of bone: pain, pathological
H E M AT O L O G I C A L fractures, hemarthrosis.
SYSTEM C. Analysis/nursing diagnosis:
I. LEUKEMIA 1. Risk for infection related to neutropenia.
A. Introduction: Known as cancer of the blood, 2. Risk for injury related to thrombocytopenia.
leukemia is the most common form of childhood 3. Altered nutrition, less than body requirements,
cancer, with an incidence of 4/100,000. Acute related to loss of appetite, vomiting, mouth
leukemia is basically a malignant proliferation of ulcers.
white blood cell (WBC) precursors triggered by an 4. Pain related to disease process and treatments
unknown cause and affecting all blood-forming (e.g., hemarthrosis, bone pain, bone marrow
organs and systems throughout the body. The onset aspiration).
PEDIATRICS

Table 5.18
Differences Between Type 1 and Type 2 Diabetes Mellitus
Variable Type 1 (Insulin-Dependent) Diabetes Mellitus Type 2 (NonInsulin-Dependent) Diabetes Mellitus
Endogenous (naturally Absolute deficiency of insulin; pancreas Relative insufficiency of insulin
occurring) insulin produces no insulin
Dependence on Total dependence on insulin injections for Varies; disease may be controlled by combination
exogenous insulin remainder of life of: diet, exercise, oral hypoglycemics, and insulin
injections
Typical onset Abrupt Insidious
Age at onset Anytime during childhood years; usually before Most common between age 40 and 60 years,
age 20 but incidence in children with obesity, lack of
exercise, diet high in junk food
Weight at onset Normal or sudden, unexplained loss of weight Overweight or obese
Gender Slightly more common in boys than girls More common in women than men
Ethnicity More common in Caucasians More common in Hispanics and Native Americans
Ketoacidosis Relatively common Relatively rare
Ease of control/stability Unstable, difficult to achieve steady blood sugar Stable, easier to achieve steady blood sugar level
level
Presenting symptoms Polyuria, polyphagia, polydipsia (three Ps) More commonly related to long-term complications
(e.g., changes in vision or kidney function)
Complications Occur more frequently and at relatively Occur less frequently, and often not seen until later
younger age because of younger age at adult years; rate is variable
onset of disease; rate 80% or greater
2164_Ch05_249-346 29/03/12 12:25 PM Page 295

Hematological System 295


5. Activity intolerance related to infection and g. Offer dietary supplements to increase caloric
anemia. intake.
6. Self-esteem disturbance related to disease process h. Encourage local anesthetics such as dex-
and treatments (e.g., loss of hair with tromethorphan throat lozenges (Chloraseptic)
chemotherapy, moon face with prednisone). before meals to allow child to eat without
7. Anticipatory grieving related to life-threatening pain from oral mucous membrane ulcers.
illness. 4. Goal: relieve pain.
8. Knowledge deficit related to diagnosis, treatment, a. Offer supportive alternatives: extra company,
prognosis. back rub. Offer complementary therapies:
D. Nursing care plan/implementation: meditation, visualization (if age appropriate).
1. Goal: maintain infection-free state. b. Administer medications regularly, before pain
a. Standard precautions to prevent infection. becomes excessive.
b. Use good hand washing technique. c. Use beanbag chair for positional changes.
c. Ongoing evaluation of sites for potential d. Avoid excessive stimulation (noise, light),
infection (e.g., gums). which may heighten perception of pain.
d. Provide meticulous oral hygiene. 5. Goal: promote self-esteem.
e. Keep record of vital signs, especially a. Stress what child can still do to keep the child
temperature. as independent as possible.
f. Provide good skin care. b. Encourage performance of ADLs as much as
g. Screen staff and visitors (especially children) possible to foster a sense of independence.
restrict anyone with infection. c. Provide diversion/activity as tolerated.
h. Protective isolation/reverse isolation to mini- d. Give lots of positive reinforcement to enhance
mize exposure to potentially life-threatening a sense of accomplishment.
infection. e. Provide realistic feedback on childs appear-
i. Discharge planning: return to school, but ance; offer suggestions, such as a wig or cap to
isolate from chickenpox or known commu- cover alopecia secondary to chemotherapy.
nicable diseases. f. Encourage early return to peers/school to
2. Goal: prevent injury. avoid social isolation.
a. Avoid IMs/IVs if possible, due to bruising and 6. Goal: prevent complications related to

PEDIATRICS
bleeding tendencies. leukemia/prolonged immobility/treatments.
b. Do not give aspirin or medications contain- a. Inspect skin for breakdown, especially over
ing aspirin, which will interfere with bony prominences, due to poor nutritional
platelet formation, thus increasing the risk intake and limited mobility due to bone pain.
of bleeding. b. Anticipate need for and administer (per
c. Use soft toothbrush to avoid trauma to gums, physicians order) multiple transfusions of
which may cause bleeding and infection. platelets, packed red blood cells (RBCs), etc.
d. Avoid per rectum suppositories, due to c. Check for hemorrhagic cystitis; push fluids
probable rectal ulcers. (especially with Cytoxan).
e. Supervise play/activity carefully to promote d. Check for constipation or peripheral neuropa-
safety and prevent excessive bruising or thy (especially with vincristine). Refer to
bleeding. Chapter 8 for specific information on
3. Goal: promote adequate nutrition. chemotherapy.
a. Diet: high calorie, high protein, high iron. 7. Goal: assist child and parents to cope with life-
b. Encourage extra fluids to prevent constipation threatening illness.
or dehydration. a. Teach rationale for repeated hospitalizations,
c. I&O, daily weights, to monitor fluid and multiple invasive tests/treatments, long-term
nutritional status. follow-up care.
d. Allow child to be involved with food selection/ b. Encourage compliance with all aspects of
preparation; allow child almost any food he or therapy, to increase chances of survival.
she tolerates, to encourage better dietary c. Support family members and their coping
intake. mechanisms.
e. Thoroughly wash/peel fresh fruits and vegeta- d. Offer factual information regarding ultimate
bles if child is neutropenic (or avoid eating prognosis (80% cure for acute lymphocytic
these foods). leukemia [ALL]).
f. Serve frequent, small snacks to increase fluid e. If death appears imminent, assist family to
and caloric consumption. cope with dying and death.
2164_Ch05_249-346 29/03/12 12:25 PM Page 296

296 chapter 5 Health Promotion and Maintenance

E. Evaluation/outcome criteria: (hemoglobin electrophoresis). Sickle cell anemia


1. Child is maintained in infection-free state. has no known cure.
2. Injuries are prevented or kept to a minimum. B. Assessment:
3. Adequate nutrition is maintained. 1. Increased susceptibility to infection (cause:
4. Child is free from pain or can live with minimum unknown; most common cause of death in
level of pain. children under 5 years).
5. Childs self-esteem is maintained; child is treated 2. Inherited as autosomal recessive disorder
as living (not dying). (Fig. 5.10).
6. Complications are prevented or kept to a 3. Precipitated by conditions of low oxygen
minimum. tension, dehydration, vascular obstruction,
7. Child and family use positive coping mecha- increased blood viscosity, infection.
nisms to deal with illness. 4. Signs of anemia:
II. SICKLE CELL ANEMIA a. Pallor (in dark-skinned children, do not rely
on pallor alonecheck hemoglobin [Hgb]
A. Introduction: Sickle cell anemia is a congenital
and hematocrit [Hct]).
hemolytic anemia resulting from a defective hemo-
b. Jaundice, due to excessive hemolysis.
globin (Hgb) molecule (hemoglobin S). It is most
c. Irritability, lethargy, anorexia, malaise.
common in African Americans (8% have sickle cell
5. Vaso-occlusive crisis: severe pain (variable
trait) and in people of Mediterranean, Hispanic,
sites, e.g., chest, back, abdomen), fever,
and Middle Eastern descent. The diagnosis is usual-
swelling of hands and feet, joint pain and
ly made during the toddler or preschool years, dur-
swelling, all related to hypoxia, ischemia, and
ing the first crisis episode following an infection.
necrosis at the cellular level. Most common;
There is also the need to differentiate between sickle
usually nonlife threatening.
cell trait (Sickledex test) and sickle cell anemia
PEDIATRICS

Figure 5.10 Genetic transmission of sickle cell anemia.


2164_Ch05_249-346 29/03/12 12:25 PM Page 297

Hematological System 297


6. Splenic sequestration crisis: blood is sequestered d. Administer oxygen, as ordered, to relieve
(pooled) in spleen; precipitous drop in hemoglo- hypoxia and prevent further sickling.
bin levels and blood pressure, increased pulse e. Administer blood transfusions, as ordered,
rate, shock, and ultimately death from profound to correct severe anemia. Complication: If
anemia and cardiovascular collapse. hypertransfusion programs are implemented,
C. Analysis/nursing diagnosis: child may develop iron overload. Desferal
1. Altered tissue perfusion related to anemia and chelates the iron so that it can be excreted
occlusion of vessels. through the urine or bile to help reduce this
2. Pain related to vaso-occlusion. complication.
3. Impaired physical mobility related to pain, f. Standard medications: hydroxyurea, folic
immobility. acid.
4. Knowledge deficit related to disease process and g. Monitor fluid and electrolyte balance: I&O,
treatment (e.g., prevention of sickling or infec- weight, electrolytes.
tion; genetic counseling). h. Perform ADLs for child if unable to care for
D. Nursing care plan/implementation: own needs; encourage self-care as soon as
1. Goal: prevent sickling. possible to promote independence. Avoid
a. Avoid conditions of low oxygen tension stress and fatigue.
(hypoxia), which causes RBCs to assume a 4. Goal: teach child and family about sickle cell
sickled shape. anemia.
b. Provide continuous extra fluids to prevent a. Provide factual information based on childs
dehydration, which causes sluggish circula- developmental level.
tion. State specific amounts to be consumed b. When asked, offer information regarding
rather than encourage fluids. Avoid tempera- prognosis (no known cure).
ture extremes. c. Encourage child to live as normally as
c. Avoid activities that may result in overheating, possible.
to prevent dehydration; suggest appropriate d. Genetic screening and counseling
clothes; limit time in sun. (see Fig. 5.10).
d. If dehydrated due to acute illness, supplement E. Evaluation/outcome criteria:
with IV fluids and additional oral fluids to 1. Sickling is prevented or kept to a minimum.

PEDIATRICS
reestablish fluid balance. 2. Child is maintained in infection-free state.
2. Goal: maintain infection-free state. 3. Child/family verbalize that they can cope
a. Standard precautions to prevent infection. adequately with crisis.
b. Use good hand washing technique. 4. Child/family verbalize their understanding about
c. Evaluate carefully, check continually for disease, its management, and its prognosis.
potential infection sites, which may either III. HEMOPHILIA
lead to death due to sepsis or precipitate sickle
A. Introduction: Hemophilia is a bleeding disorder
cell crisis.
inherited as a sex-linked (X-linked) recessive
d. Teach importance of prevention of sickle cell
trait; that is, it occurs only in men but is trans-
crisis: adequate fluids and nutrition; frequent
mitted by women carriers who are symptom free
medical checkups; keep away from known
(Fig. 5.11). Hemophilia results in a deficiency
sources of infection.
of one or more clotting factors; it is necessary to
e. Stress need to report early signs of infection
determine which clotting factor is deficient and
(chest crisis) promptly to physician.
to what extent. Classic hemophilia (hemophilia
f. Need to balance prevention of infection with
A), a lack of clotting factor VIII, accounts for
childs need for a normal life.
75% of all cases of hemophilia.
g. Administer yearly influenza vaccine.
B. Assessment:
3. Goal: provide supportive therapy during crisis.
1. Major problem is bleeding.
a. Provide bedrest/hospitalization during crisis to
a. In newborn boy: abnormal bleeding from
decrease the bodys demand for oxygen.
umbilical cord, prolonged bleeding from
b. Relieve pain due to infarction of tissues by
circumcision site.
administering pain medications as ordered;
b. In toddler boy: excessive bruising, possible
handle gently and use proper positioning
intracranial bleeding, prolonged bleeding
techniques.
from cuts or lacerations.
c. Apply heat (never cold) to affected painful
c. General: hemarthrosis, petechiae, epistaxis,
areas to increase blood flow (vasodilation) and
frank hemorrhage anywhere in body, anemia.
oxygen supply.
2164_Ch05_249-346 29/03/12 12:25 PM Page 298

298 chapter 5 Health Promotion and Maintenance

Figure 5.11 Genetic transmission of hemophilia.

2. Need to determine which clotting factor is 2. Goal: control bleeding episodes when they occur.
deficient/missing and extent of deficiency: a. Local measures: RICE (Rest, Ice, Compression,
a. Mild: child has 5% to 50% of normal Elevation); keep immobilized during acute
amount of clotting factor. bleeding episodes only. For epistaxis: child
b. Moderate: child has 1% to 5% of normal should sit up and lean slightly forward.
amount of clotting factor. b. Systemic measures: administer clotting anti-
c. Severe: child has less than 1% of normal hemophilic factor (factor VIII or DDAVP) via
PEDIATRICS

amount of clotting factor. IV infusion. Note: These are blood products,


3. Definitive test: PTT so a transfusion reaction is possible.
C. Analysis/nursing diagnosis: c. Note: cryoprecipitate (a blood product) is no
1. Risk for injury related to bleeding tendencies. longer used due to risk of transmission of
2. Pain related to hemarthrosis. hepatitis and HIV.
3. Impaired physical mobility related to bleeding 3. Goal: prevent long-term disability related to joint
and pain. degeneration.
4. Knowledge deficit related to home care and a. Keep immobilized during period of acute
follow-up. bleeding and for 24 to 48 hours afterward to
D. Nursing care plan/implementation: allow blood to clot and to prevent dislodging
1. Goal: prevent injury and possible bleeding. the clot.
a. Provide an environment that is as safe as b. Administer prescribed pain medications before
possible (e.g., toys with no sharp edges, physical therapy sessions.
childs safety scissors). c. Begin prescribed exercise program, starting
b. Use soft toothbrush to prevent trauma to with passive range of motion (ROM) and
gums. Wear safety equipment in PE; no gradually advancing to active ROM, then
contact sports. full exercise program, as tolerated, to main-
c. When old enough to shave, use only electric tain maximum joint function. Monitor
razor (no straight-edge razors). weight to prevent strain on joints
d. Avoid IMs/IVsbut when absolutely neces- (especially knees).
sary, treat as arterial puncture; that is, apply d. Avoid: prolonged immobility, braces,
direct pressure to the site for at least 5 min splintswhich can lead to permanent
after withdrawing needle. deformities and loss of mobility.
e. Do not use aspirin or medication containing 4. Goal: promote independence in management of
aspirin (prolongs bleeding/clotting time). own care.
Caution with NSAIDs (inhibit platelet a. Encourage child to assume responsibility for
function). choosing safe activities.
2164_Ch05_249-346 29/03/12 12:25 PM Page 299

Integumentary System 299


b. Encourage child to attend regular school as IV. CONTRAINDICATIONS/PRECAUTIONS
much as possible; provide support through TO IMMUNIZATIONS
school nurse. A. Child who has a severe febrile illness (e.g., URI,
c. Advise child to wear Medic Alert bracelet. gastroenteritis, or any fever).
d. Caution parents to avoid overprotecting B. Child with alteration in skin integrity: rash,
child. eczema.
e. Offer child chance to self-limit activities C. Child with alteration in immune system;
within appropriate limits (parents can offer steroids; chemotherapy, radiation therapy;
guidance). human immunodeficiency virus (HIV)/acquired
f. Assist child to cope with life-threatening immunodeficiency syndrome (AIDS) (no live
disorder with no known cure. virus vaccine).
5. Goal: health teaching. D. Child with a known allergic reaction to
a. Between 9 and 12 years of age: child can be previous immunization or substance in the
taught to self-administer clotting factor IV immunization.
(before this, family can perform). E. Recent recipient of blood/blood products.
b. As child enters adolescence: begin to
V. CHILDHOOD COMMUNICABLE DISEASES
discuss issues such as realistic vocations,
(Table 5.19). Basic principles of care:
insurance coverage, genetic transmission
(see Fig. 5.11). A. Standard precautions to prevent communicability/
E. Evaluation/outcome criteria: infection.
1. Serious injuries are prevented; bleeding is kept B. Fever control with acetaminophen.
to a minimum. C. Extra fluids for hydration.
2. Episodes of bleeding are controlled by prompt, D. General home care procedures: comfort
effective intervention. measures/supportive care.
3. There are no long-term disabilities.
4. Child is able to manage own care independently,
with minimum supervision. Streptococcus Infections/Sequelae
Introduction: Group A -hemolytic streptococcus is a com-
IMMUNOLOGICAL mon infectious organism that causes illness in children and

PEDIATRICS
SYSTEM is highly contagious. In themselves, the diseases caused by
streptococcus do not seem very serious (e.g., strep throat,
I. RECOMMENDED SCHEDULE FOR ACTIVE otitis media, impetigo, or scarlet fever). The most common
IMMUNIZATION OF HEALTHY INFANTS treatment for strep is a full course of antibiotic therapy:
AND CHILDREN (see Table 5.2). 10 days of penicillin (or, if allergic, erythromycin). With
II. ASSESSMENT: common side effects of immuniza- adequate therapy, generally no sequelae are seen. If the
tions (occur 24 to 48 hours after immunization except strep is not treated, or is only partially treated, the sequelae
as noted): include serious systemic diseases, with potentially long-
term effects. If the effect is manifested primarily in the
A. Soreness, redness, tenderness, or lump at injection
heart (carditis), it is acute rheumatic fever. If the effect is
site.
manifested primarily in the kidneys, it is acute glomeru-
B. Fever: brief, mild to moderate.
lonephritis (see Fig. 5.10).
C. Crankiness and fussiness; anorexia; drowsiness.
D. Measlescoryza and rash 7 to 10 days after
immunization.
I N T E G U M E N TA R Y
E. Rubellaarthralgia and arthritis-like symptoms
SYSTEM
2 weeks after immunization. I. INFANTILE ECZEMA (ATOPIC DERMATITIS)
III. NURSING INTERVENTIONS/HOME CARE A. Introduction: Eczema is an allergic skin reaction,
for infant or child who has been immunized: most commonly to foods (e.g., cows milk,
A. Explain to parents the reason for each immuniza- eggs). It is most common in infants and young
tion and common side effects. children (under 2 years). Infantile eczema
B. Suggest acetaminophen for fever or discomfort. generally undergoes permanent, spontaneous
C. Extra affection and closenesscuddling, soothing, remission by age 3 years; however, approximately
rocking. 50% of children who have had infantile eczema
D. Teach parents to notify physician of untoward, develop asthma during the preschool or school-
serious, or prolonged side effects. age years.
(text continues on page 306)
2164_Ch05_249-346 29/03/12 12:25 PM Page 300

300 chapter 5 Health Promotion and Maintenance

Table 5.19
Communicable Diseases of Childhood
Disease
Chickenpox (Varicella)
Agent: Varicella-zoster virus (VZV)
Source: Primary secretions of respiratory tract of person who is
infected and to a lesser degree skin lesions (scabs not infectious)
Transmission: Direct contact, droplet (airborne) spread, and
contaminated objects
Incubation period: 23 weeks, usually 1317 days
Period of communicability: Probably 1 day before eruption of
lesions (prodromal period) to 6 days after first crop of vesicles
when crusts have formed
PEDIATRICS

Chickenpox (varicella). (From Habif, TP: Clinical Dermatology: A


Color Guide to Diagnosis and Therapy, ed 3. St. Louis, Mosby, 1996.)
2164_Ch05_249-346 29/03/12 12:25 PM Page 301

Communicable Diseases of Childhood 301

Table 5.19
Communicable Diseases of Childhoodcontd
Clinical Manifestations Therapeutic Management/Complications Nursing Considerations
Prodromal stage: Slight fever, malaise, Specific: Antiviral agent acyclovir Maintain strict isolation in hospital.
and anorexia for first 24 hours; rash (Zovirax), varicella-zoster immune
Isolate child in home until vesicles
highly pruritic; begins as macule, rapidly globulin (VZIG) after exposure in
have dried (usually 1 week after onset
progresses to papule and then vesicle children who are high risk.
of disease), and isolate children who
(surrounded by erythematous base, Supportive: Diphenhydramine
are high risk from children who are
becomes umbilicated and cloudy, hydrochloride or antihistamines to
infected.
breaks easily and forms crusts); all relieve itching; skin care to prevent
three stages (papule, vesicle, crust) secondary bacterial infection Administer skin care: give bath and
present in varying degrees at one time. change clothes and linens daily;
Complications:
administer topical calamine lotion;
Distribution: Centripetal, spreading to Secondary bacterial infections (abscesses,
keep childs fingernails short and
face and proximal extremities but cellulitis, pneumonia, sepsis)
clean; apply mittens if child scratches.
sparse on distal limbs and less on Encephalitis
areas not exposed to heat (i.e., from Varicella pneumonia Keep child cool (may decrease number
clothing or sun) Hemorrhagic varicella (tiny hemorrhages of lesions).
in vesicles and numerous petechiae in
Constitutional signs and symptoms: Lessen pruritus; keep child occupied.
skin)
Elevated temperature from lym-
Chronic or transient thrombocytopenia Remove loose crusts that rub and
phadenopathy, irritability from
irritate skin.
pruritus
Teach child to apply pressure to
pruritic area rather than scratch it.
If older child, reason with child regard-
ing danger of scar formation from
scratching.
Avoid use of aspirin; use of
acetaminophen controversial.

Continued

PEDIATRICS
2164_Ch05_249-346 29/03/12 12:25 PM Page 302

302 chapter 5 Health Promotion and Maintenance

Table 5.19
Communicable Diseases of Childhoodcontd
Disease
Measles (Rubeola)
Agent: Virus
Source: Respiratory tract secretions, blood, and urine of
person who is infected
Transmission: Usually by direct contact with droplets of person
who is infected
Incubation period: 1020 days
Period of communicability: From 4 days before to 5 days after
rash appears but mainly during prodromal (catarrhal) stage
PEDIATRICS

Mumps
Agent: Paramyxovirus
Source: Saliva of people who are infected
Transmission: Direct contact with or droplet spread from a
person who is infected
Incubation period: 1421 days
Period of communicability: Most communicable immediately
before and after swelling begins

Measles (rubeola). (From Seidel, HM and others: Mosbys Guide to


Physical Examination, ed 3. St. Louis, Mosby, 1995.)
2164_Ch05_249-346 29/03/12 12:25 PM Page 303

Communicable Diseases of Childhood 303

Table 5.19
Communicable Diseases of Childhoodcontd
Clinical Manifestations Therapeutic Management/Complications Nursing Considerations
Prodromal (catarrhal) stage: Fever and Vitamin A supplementation Isolation until fifth day of rash; if
malaise, followed in 24 hours by coryza, Supportive: Bedrest during febrile hospitalized, institute respiratory
cough, conjunctivitis, Kopliks spots period precautions.
(small, irregular red spots with a minute, Antipyretics Maintain bedrest during prodromal
bluish-white center first seen on buccal Antibiotics to prevent secondary stage; provide quiet activity.
mucosa opposite molars 2 days before bacterial infection in high-risk Fever: Instruct parents to
rash); symptoms gradually increase in children administer antipyretics; avoid
severity until second day after rash Complications: chilling; if child is prone to seizures,
appears, when they begin to subside Otitis media institute appropriate precautions (fever
Pneumonia spikes to 104F [40C] between fourth
Rash: Appears 34 days after onset of
Bronchiolitis and fifth days).
prodromal stage; begins as erythema-
Obstructive laryngitis and Eye care: Dim lights if photophobia
tous maculopapular eruption on face
laryngotracheitis present; clean eyelids with warm
and gradually spreads downward; more
Encephalitis saline solution to remove secretions
severe in earlier sites (appears conflu-
or crusts; keep child from rubbing
ent) and less intense in later sites
eyes; examine cornea for signs of
(appears discrete); after 34 days
ulceration.
assumes brownish appearance, and fine
Coryza/cough: Use cool mist
desquamation occurs over areas of
vaporizer; protect skin around
extensive involvement
nares with layer of petrolatum;
Constitutional signs and symptoms: encourage fluids and soft, bland
Anorexia, malaise, generalized foods.
lymphadenopathy Skin care: Keep skin clean; use tepid
baths as necessary.

PEDIATRICS
Prodromal stage: Fever, headache, Symptomatic and supportive: Isolation during period of communica-
malaise, and anorexia for 24 hours, fol- Analgesics for pain and antipyretics bility; institute respiratory precautions
lowed by earache that is aggravated for fever during hospitalization.
by chewing Intravenous fluid may be necessary for
Maintain bedrest during prodromal
Parotitis: By third day, parotid gland(s) child who refuses to drink or vomits
phase until swelling subsides.
(either unilateral or bilateral) enlarge(s) because of meningoencephalitis
Give analgesics for pain; if child
and reach(es) maximum size in 13 days;
Complications: is unwilling to chew medication,
accompanied by pain and tenderness
Sensorineural deafness use elixir form.
Other manifestations: Submaxillary
Postinfectious encephalitis Encourage fluids and soft, bland
and sublingual infection, orchitis, and
Myocarditis foods; avoid foods that require
meningoencephalitis
Arthritis chewing.
Hepatitis Apply hot or cold compresses to neck,
Epididymo-orchitis whichever is more comforting.
Sterility (extremely rare in adult men)
To relieve orchitis, provide warmth and
local support with tight-fitting under-
pants (stretch bathing suit works well).
Continued
2164_Ch05_249-346 29/03/12 12:26 PM Page 304

304 chapter 5 Health Promotion and Maintenance

Table 5.19
Communicable Diseases of Childhoodcontd
Disease
Rubella (German Measles)
Agent: Rubella virus
Source: Primarily nasopharyngeal secretions of persons with
apparent or inapparent infection; virus also present in blood,
stool, and urine
Transmission: Direct contact and spread via person who is
infected; indirectly via articles freshly contaminated with
nasopharyngeal secretions, feces, or urine
Incubation period: 1421 days
Period of communicability: 7 days before to approximately
5 days after appearance of rash

Scarlet Fever
Agent: Group A -hemolytic streptococci
Source: Usually from nasopharyngeal secretions of person
who is infected and carriers
Transmission: Direct contact with person who is infected, or
A droplet spread indirectly by contact with contaminated articles,
ingestion of contaminated milk or other food
Incubation period: 24 days, with range of 17 days
Period of communicability: During incubation period and
PEDIATRICS

clinical illness, approximately 10 days; during first 2 weeks of


carrier phase, although may persist for months

B
Rubella (German Measles). (A) Progression of rash.
(B) Appearance of rash. (From Habif, TP: Clinical Dermatology:
A Color Guide to Diagnosis and Therapy, ed 3. St. Louis, Mosby, 1996.)

Scarlet Fever.
2164_Ch05_249-346 29/03/12 12:26 PM Page 305

Communicable Diseases of Childhood 305

Table 5.19
Communicable Diseases of Childhoodcontd
Clinical Manifestations Therapeutic Management/Complications Nursing Considerations
Prodromal stage: Absent in children, No treatment necessary other than Reassure parents of benign nature of
present in adults and adolescents; antipyretics for low-grade fever and illness in child who is affected.
consists of: low-grade fever, headache, analgesics for discomfort
Use comfort measures as necessary.
malaise, anorexia, mild conjunctivitis, Complications:
coryza, sore throat, cough, and lym- Rare (arthritis, encephalitis, or purpura); Isolate child from women who are
phadenopathy; lasts for 15 days, sub- most benign of all childhood communicable pregnant.
sides 1 day after appearance of rash diseases; greatest danger is teratogenic
effect on fetus
Rash: First appears on face and rapidly
spreads downward to neck, arms, trunk,
and legs; by end of first day, body is
covered with a discrete, pinkish red
maculopapular exanthema; disappears
in same order as it began and is usually
gone by third day
Constitutional signs and symptoms:
Occasionally low-grade fever, headache,
malaise, and lymphadenopathy
Prodromal stage: Abrupt high fever, Treatment of choice is a full course Institute respiratory precautions until
pulse increased out of proportion to of penicillin (or erythromycin for 24 hours after initiation of treatment.
fever, vomiting, headache, chills, children who are penicillin-sensitive);
Ensure compliance with oral
malaise, abdominal pain fever should subside 24 hr after
antibiotic therapy (intramuscular
beginning therapy
Enanthema: Tonsils enlarged, edema- benzathine penicillin G [Bicillin]
tous, reddened, and covered with Antibiotic therapy for newly may be given if parents
patches of exudate; in severe cases diagnosed carriers (nose or reliability in giving oral drugs
appearance resembles membrane seen throat cultures positive for is questionable).
in diphtheria; pharynx is edematous and streptococci)
Maintain bedrest during febrile phase;
beefy red; during first 12 days tongue
Supportive measures: Bedrest provide quiet activity during convales-
is coated and papillae become red and

PEDIATRICS
during febrile phase, analgesics cent period.
swollen (white strawberry tongue); by
for sore throat
fourth or fifth day white coat sloughs off, Relieve discomfort of sore throat
leaving prominent papillae (red straw- Complications: with analgesics, gargles,
berry tongue); palate is covered with Otitis media lozenges, antiseptic throat
erythematous punctate lesions Peritonsillar abscess sprays (Chloraseptic), and
Sinusitis inhalation of cool mist.
Exanthema: Rash appears within
Glomerulonephritis
12 hours after prodromal stage; red Encourage fluids during febrile
Carditis, polyarthritis (uncommon)
pinhead-sized punctate lesions rapidly phase; avoid irritating liquids
become generalized but are absent (citrus juices) or rough foods;
on face, which becomes flushed with when child is able to eat, begin
striking circumoral pallor; rash is more with soft diet.
intense in folds of joints; by end of
Advise parents to consult practitioner if
first week desquamation begins (fine,
fever persists after beginning therapy.
sandpaper-like on torso; sheetlike
sloughing on palms and soles), which Discuss procedures for preventing
may be complete by 3 weeks or longer spread of infection.

Strawberry tongue. (From Wong, D: Whaley and Wongs Nursing Care of


Infants and Children, ed 7. Mosby, St. Louis, 2003.)
2164_Ch05_249-346 29/03/12 12:26 PM Page 306

306 chapter 5 Health Promotion and Maintenance

B. Assessment: b. Keep nails trimmed shortmay need mittens


1. Erythematous lesions, beginning on cheeks and (preferable not to use elbow restraints,
spreading to rest of face and scalp. because the antecubital space is a common
2. May spread to rest of body, especially in flexor site for eczema).
surfaces (e.g., antecubital space). c. Use clothes and bed linens that are nonirritat-
3. Lesions may ooze or crust over. ing, that is, pure cotton (no wool or blends).
4. Severe pruritus, which may lead to secondary d. Institute elimination/hypoallergenic diet:
infection. (1) No milk or milk products.
5. Lymphadenopathy near site of rash. (2) Change to lactose-free formula
6. Unaffected skin tends to be dry and rough. (e.g., Isomil).
7. Systemic manifestations are rarebut child (3) Avoid: eggs, wheat, nuts, beans, chocolate.
may be irritable, cranky. e. No stuffed animals or hairy dolls.
C. Analysis/nursing diagnosis: 3. Goal: provide discharge planning/teaching for
1. Impaired tissue integrity related to lesions. parents and child.
2. Pain related to pruritus. a. Include all above information.
3. Risk for (secondary) infection related to breaks in b. Include information on course of disease:
the skin (first line of defense) and itching. characterized by exacerbations and remissions
4. Knowledge deficit related to care of child with throughout early years.
eczema, prognosis, how to prevent exacerbations. c. Include information on prognosis: 50% to
D. Nursing care plan/implementation: 60% will go into spontaneous (and perma-
1. Goal: promote healing of lesions. nent) remission during preschool years; 40%
a. Wet method: frequent tepid baths (up to four to 50% will develop asthma/hay fever during
times a day) followed by immediate applica- school-age years.
tion of a lubricant while the skin is still moist; E. Evaluation/outcome criteria:
no soap or use very mild, nonperfumed soap 1. Lesions heal well, without secondary infection.
(e.g., Dove, Neutrogena); most useful method 2. Adequate relief from itching is achieved.
if child lives in a dry climate. 3. Parents verbalize understanding of eczema,
b. Dry method: infrequent baths; cleanse skin prognosis, and how to prevent exacerbations.
with nonlipid, hydrophilic agent (e.g., II. INFESTATIONS
PEDIATRICS

Cetaphil); most useful method if child


A. Lice (pediculosis)
lives in a humid climate.
1. Introduction: In children, the most common form
c. Can add cornstarch to bath water to relieve
of lice is pediculosis capitis, or head lice. This
itching and promote healing; keep skin well
parasite feeds on the scalp, and its saliva causes
hydrated by applying emollients containing
severe itching. Head lice are frequently associated
petrolatum or lanolin, which are occlusive
with the sharing of combs and brushes, hats, and
and prevent evaporation of moisture.
clothing; thus, they are more common in girls,
d. Apply wet soaks with Burows solution
especially those with long hair. Lice are also asso-
(aluminum acetate solution; topical astringent
ciated with overcrowded conditions and poor
and antiseptic); wet soaks should not be used
hair hygiene.
for more than 3 days at a time
2. Assessment:
e. Protect child from possible sources of
a. Severe itching of scalp.
infection; standard precautions to prevent
b. Visible eggs/nits on shafts of hair.
infection.
3. Analysis/nursing diagnosis:
f. Absolutely no immunizations during acute
a. Impaired skin integrity related to infestation
exacerbations of eczema because of the possi-
of scalp with lice.
bility of an overwhelming dermatitis, allergic
b. Risk for impaired skin integrity related to severe
reaction, shock, or even death.
pruritus of scalp.
g. Apply topical creams/ointments as prescribed:
c. Knowledge deficit related to transmission and
A and D emollient ointment, hydrocortisone
prevention of disease and treatment regimen.
cream to promote healing.
4. Nursing care plan/implementation:
2. Goal: provide relief from itching/keep child from
a. Goal: eradicate lice infestation. Apply perme-
scratching.
thrin (Nix) as drug of choice for infants and
a. Administer systemic oral antihistamines as
childrenrub shampoo in for 4 to 5 minutes,
ordered (e.g., Benadryl or Atarax) to break
then comb with fine-tooth comb to remove
itch-scratch cycle. Most useful at bedtime
dead lice and nits (eggs).
when itching tends to increase.
2164_Ch05_249-346 29/03/12 12:26 PM Page 307

Musculoskeletal and Neuromuscular Systems 307


b. Goal: prevent spread of lice. 3. Analysis/nursing diagnosis:
(1) Wear gloves and cap to protect self. a. Risk for infection/injury related to the anus-
(2) Inspect other family members; treat prn to-mouth contamination cycle of pinworm
with pediculocide. infestation, severe rectal itching.
(3)Wash all clothes and linens to kill any b. Knowledge deficit related to transmission and
lice that may have fallen off the childs prevention of disease and treatment regimen.
hair. 4. Nursing care plan/implementation:
(4) Encourage short hair, if acceptable. a. Goal: eradicate pinworm infestation. Treat all
(5) Teach preventive measures: do not share family members simultaneously with an
comb, brushes, hats. antiparasitic agent (e.g., Vermox, Povan).
5. Evaluation/outcome criteria: lice are eradicated b. Goal: prevent spread of pinworms.
and do not spread. (1) Launder all underwear, bed linens, and
B. Pinworms (enterobiasis) towels in hot soapy water to kill eggs.
1. Introduction: In children, the most common (2) Teach family members the importance of
helminthic infestation is pinworms. Infestation good hygiene, especially hand washing
usually occurs when the child places fingers (and before eating (or preparing food) and after
the pinworm eggs) into the mouth. Breaking the toileting. Stress to children to keep their
anus-to-mouth contamination cycle can best be fingers out of their mouths.
accomplished by good hygiene, especially hand 5. Evaluation/outcome criteria: Pinworms are
washing before eating and after toileting. If one eradicated and do not spread; reinfestation does
family member has pinworms, it is highly likely not occur.
that other family members are also infested;
therefore, treat the entire family to eradicate the
parasite. Pinworms are easily eradicated with
MUSCULOSKELETAL
antiparasitic medications.
AND NEUROMUSCULAR
2. Assessment:
SYSTEMS
a. Intense perianal itching. Orthopedic conditions in infants and children are many
b. Visible pinworms in the stool. and varied, but treatment is based on basic principles of
c. Vague abdominal discomfort. nursing care. Table 5.20 and Figs. 5.12 and 5.13 offer a

PEDIATRICS
d. Anorexia and weight loss. quick review of the major pediatric orthopedic conditions.

Table 5.20
Common Pediatric Orthopedic Conditions
Age at Onset/Sex Nursing
Condition Definition Difference Treatment Considerations
Congenital Musculoskeletal Disorders
Clubfoot Downward, inward Newborn (congenital); Series of casts changed Care of child in
rotation of one or twice as common in weekly followed by cast/brace.
both feet: talipes boys Denis Browne splint Stress need for
equinovarus (95%) and then corrective follow-up.
shoes (severe cases Encourage
surgery) compliance.
Developmental Abnormal develop- Newborn (congenital); NewbornPavlik Early identification.
dysplasia of the hip ment of hip joint more common in girls harness; older infant Care of child in
(see Fig. 5.12) (most frequently or toddlerpossible traction/cast:
unilateral) surgery, spica cast check circulation;
turn q2h while cast
is wet.
Encourage
compliance.
Check for other
anomalies (e.g.,
spina bifida).
Continued
2164_Ch05_249-346 29/03/12 12:26 PM Page 308

308 chapter 5 Health Promotion and Maintenance

Table 5.20
Common Pediatric Orthopedic Conditionscontd
Age at Onset/Sex Nursing
Condition Definition Difference Treatment Considerations
Congenital Musculoskeletal Disorders
Legg-Calv-Perthes Aseptic necrosis Peak: 48 years; Conservative therapy Early identification.
disease of the head of the range: 312 years; lasts 24 years, Care of child in
femur (cause 5 times more common usually begins traction/cast: check
unknown) in boys; 10 times with bedrest and for frayed pulley
more common in traction, followed by ropes.
Caucasians than nonweight-bearing Provide diversion.
non-Caucasians devices such as brace, Assist child and
cast family to cope with
childs prolonged
immobility.
Scoliosis (see Lateral curvature Adolescence; more Braces specific to Care of child in
Fig. 5.13) of the spine (cause common in girls type of curvature; traction/cast/brace.
unknown) halo-pelvic traction; Teach that brace is
Harrington rod; Luque, worn 1623 hr/day,
Cotrel-Dubousset, 7 days/wk for
or Dwyer/Zielke 6 months2 years.
instrumentation Encourage
compliance.
Promote positive
self-image.
Acquired Defect
Osteomyelitis Most frequently 514 years; twice as Blood cultures to Care of child in
occurring bone common in boys diagnose causative splint/cast.
infection among organismsselect Provide diversion.
children appropriate Pain
antibiotic; bedrest, medications/
PEDIATRICS

immobilization with antibiotics


splint or cast per physician
order.
Joint Disorder
Juvenile idiopathic Chronic systemic Peak: 13 years and Prevent joint deformity Care of child in
arthritis (JIA) (formerly inflammatory 810 years; more by exercise, brace/splint.
known as juvenile disease (cause common in girls splints, medica- Provide diversion.
rheumatoid arthritis) unknown) tions (NSAIDs, Encourage
SAARDs, corticos- compliance.
teroids, biological
agents [Etanercept],
cytotoxic agents); relieve
symptoms (as per adult
with arthritis)
Bone Tumor
Osteosarcoma Most frequently Adolescence Limb salvage procedure; Prepare child for
occurring bone (1025 years); more amputation loss of limb
cancer among common in boys and prosthesis; Help cope with
children men chemotherapy prosthesis, life-
threatening illness
Assist with grieving
process

I. CEREBRAL PALSY appears to be fixed and nonprogressive, and


A. Introduction: Cerebral palsy (CP) is the most the cause is unknown. However, although a
common permanent physical disability of child- variety of factors have been implicated in the
hood. It is a neuromuscular disorder of the etiology of CP, it is now known that CP
pyramidal motor system resulting in debilitating results more commonly from prenatal brain
impaired voluntary muscle control. The damage abnormalities.
2164_Ch05_249-346 29/03/12 12:26 PM Page 309

Musculoskeletal and Neuromuscular Systems 309

A C

D E
Figure 5.12 Signs of developmental dysplasia of the hip. (A) Asymmetry of gluteal and thigh folds. (B) Limited hip abduction
as seen in flexion. (C) Apparent shortening of the femur, as indicated by the level of the knees in flexion. (D) Ortolani click (if
infant is under 4 weeks of age). (E) Positive Trendelenburg sign or gait (if child is weight bearing). (From Hockenberry, M, & Wilson, D:

PEDIATRICS
Wongs Nursing Care of Infants and Children, ed 8. Mosby, St. Louis, 2007.)

B. Assessment: 4. Impaired verbal communication related to


1. Most common type of cerebral palsyspastic. difficulty with speech.
a. Delayed developmental milestones. 5. Sensory/perceptual alterations related to potential
b. Tongue thrust with difficulty swallowing and vision and hearing defects.
sucking. Poor weight gain. Aspiration may 6. Risk for injury related to difficulty controlling
occur. voluntary muscles.
c. Increased muscle tone: scissoring (legs 7. Self-esteem disturbance related to disability.
crossed, toes pointed). 8. Note: Because the level of disabilities with
d. Persistent neonatal reflexes. CP can vary, the nurse must select those
e. Associated problems: diagnoses that apply, and clearly specify
(1) Mental retardation in 30% of children the individual childs limitations in any
with cerebral palsy (70% are normal). diagnostic statements.
(2) Sensory impairment: vision, hearing. D. Nursing care plan/implementation:
(3) Orthopedic conditions: congenital 1. Goal: maintain patent airway.
dysplasia of hip, clubfoot. a. Have suction and oxygen readily
(4) Dental problems: malocclusion. available.
(5) Seizures. b. Use feeding and positioning techniques to
C. Analysis/nursing diagnosis: maintain patent airway.
1. Ineffective airway clearance related to hyperactive c. Institute prompt, aggressive therapy for
gag reflex and possible aspiration. URIs, to prevent the possible development
2. Altered nutrition, less than body requirements, of pneumonia.
related to difficulty sucking and swallowing. 2. Goal: promote adequate nutrition.
3. Fluid volume deficit related to difficulty sucking a. Diet: high in calories (to meet extra energy
and swallowing. demands).
2164_Ch05_249-346 29/03/12 12:26 PM Page 310

310 chapter 5 Health Promotion and Maintenance


PEDIATRICS

Figure 5.13 The four major curve patterns in


idiopathic scoliosis. (From Bowden, VR, Dickey, SB,
& Greenberg, CS: Children and Their Families: The
Continuum of Care. WB Saunders, Philadelphia, 1999.)

b. Ensure balanced diet of basic foods that can 4. Goal: prevent injury. Refer to information on
be easily chewed. Refer to dentist for early safety throughout growth and development
dental care. sections, pp. 256257, 258, 260, 261,
c. Provide feeding utensils that promote Table 3.9, p. 94, and Table 3.10, pp. 9495.
independence. a. Use individually designed chairs with
d. Feed in upright position. restraints for positioning and safety.
e. Relaxed mealtimes, decreased emphasis on b. Provide protective helmet to prevent head
manners, cleanliness. trauma.
f. Monitor I&O, weight gain. c. Implement seizure precautions.
3. Goal: facilitate verbal communication. 5. Goal: provide early detection of and correction for
a. Refer to speech therapist. vision and hearing defects.
b. Speak slowly, clearly to child. a. Arrange for screening tests.
c. Use pictures or actual objects to reinforce b. Assist family with obtaining corrective
speech. devices: eyeglasses, hearing aids.
2164_Ch05_249-346 29/03/12 12:26 PM Page 311

Musculoskeletal and Neuromuscular Systems 311


6. Goal: promote locomotion. E. Evaluation/outcome criteria:
a. Encourage infant stimulation program 1. Patent airway and adequate oxygenation
to assist infant in reaching developmental maintained.
milestones. 2. Adequate nutrition maintained, and child begins
b. Refer to physical therapy for exercise program. to grow and gain weight.
c. Incorporate play into exercise routine. 3. Child has an acceptable means of verbal
d. Use devices that promote locomotion: parallel communication.
bars, crutches, and braces. 4. Safety is maintained.
e. Surgical approach may be needed to relieve 5. Vision and hearing within normal limits using
contractures. corrective devices prn.
f. Medications: focus on excessive motion 6. Child is as mobile as possible, given disabilities.
and tension; antianxiety agents, skeletal 7. Child is performing ADLs, within capabilities.
muscle relaxants, Botox injections, baclofen 8. Child has positive self-image/self-esteem.
pump (IT). II. SPINA BIFIDA (MYELODYSPLASIA)
7. Goal: encourage independence in ADLs.
A. Introduction: Three different types of spina
a. Adapt clothing, feeding utensils, etc., to
bifida:
facilitate self-help.
1. Spina bifida occultaa hidden bony defect
b. Encourage child to perform ADLs as much as
without herniation of the meninges or cord;
possible; offer positive reinforcement.
not visible externally, no symptoms are present,
c. Assist parents to have realistic expectations for
and no treatment is needed.
their child; avoid excessively high expectations
2. Spina bifida cysticavisible defect of the spine
that might increase frustration.
with external saclike protrusion.
8. Goal: promote self-esteem.
a. Meningocele (Table 5.21).
a. Praise child for each accomplishment or for
b. Myelomeningocelesee Table 5.21. Most
sincere effort.
serious type of spina bifida cystica and also
b. Help child dress and groom self daily in an
most common.
attractive normal manner for developmental
The remainder of this section deals with
level and age.
myelomeningocele exclusively.
c. Encourage child to form friendships with
B. Assessment:

PEDIATRICS
children with similar problems.
1. Congenital defect.
d. Enroll child in special education classes to
2. Readily detected by visual inspection in delivery
meet the childs educational needs.
room: round, bulging sac filled with fluid, usually
e. Encourage parents to expose child to wide
in lumbosacral area.
variety of experiences.

Table 5.21
Comparison of Two Major Types of Spina Bifida
Dimension Meningocele Myelomeningocele
Contents of sac Meninges and Meninges, cerebrospinal fluid, spinal cord
cerebrospinal fluid
Transillumination Present Absent
Percentage of total cases 25% 75%
Motor function Present Absent
Sensory function Present Absent
Urinary/fecal incontinence Absent Present
Associated orthopedic anomalies Rare Developmental dysplasia of the hip, clubfoot
Other anomalies Rare Hydrocephalus (90%95%)
Treatment Surgery Surgery
Major short-term complication Infection (meningitis) Infection (meningitis)
Major long-term complication None Chronic urinary tract infection leading to renal disease/failure
Prognosis Excellent Guarded
2164_Ch05_249-346 29/03/12 12:26 PM Page 312

312 chapter 5 Health Promotion and Maintenance

3. Sensation and movement: complete lack below the 4. Goal: prevent complications of prolonged immobility
level of the lesion. or associated orthopedic anomalies.
4. Urinary: retention, with overflow incontinence. a. Position: hips abducted.
5. Fecal: constipation, fecal impaction, oozing of b. Use positional devices, rotating pressure
liquid stool around impaction. mattress/flotation mattress.
6. 80% to 85% develop signs and symptoms of c. Refer to physical therapy for ROM exercises.
hydrocephalus. d. Make necessary referrals for care of possible
7. May have associated orthopedic anomalies: clubfoot/developmental hip dysplasia.
clubfoot, developmental hip dysplasia. 5. Goal: monitor for possible development of hydro-
C. Analysis/nursing diagnosis: cephalus. Occurs in 90% to 95% of infants born
1. Risk for injury/infection related to rupture with myelomeningocele.
of the sac. E. Evaluation/outcome criteria:
2. Altered urinary elimination related to urinary 1. Integrity of sac is maintained until surgery is
retention and overflow incontinence. done.
3. Impaired skin integrity related to immobility. 2. No infection occurs.
4. Constipation related to fecal incontinence and 3. Adequate patterns of urinary and bowel
impaired innervation. elimination are achieved with necessary
D. Nursing care plan/implementation: support.
1. Goal: prevent rupture of the sac and possible 4. Complications of immobility, orthopedic
infection (preoperative). anomalies are prevented or treated promptly.
a. Position: no pressure on sac; prone, to prevent
contamination with urine or stool.
b. No clothing or diapers, to avoid pressure on sac.
ACCIDENTS:
c. Place in heated isolette to maintain body
INGESTIONS
temperature. Avoid radiant heat, which can
AND POISONINGS
dry and crack the sac. I. GENERAL PRINCIPLES OF TREATMENT FOR
d. Keep sac covered with sterile, moist, nonad- INGESTIONS AND POISONINGS
herent dressing (sterile normal saline) to pre- A. Prevention: refer to section on toddler safety,
vent drying, cracking, and leakage of CSF; p. 258.
PEDIATRICS

change every 2 to 4 hours; document appear- B. How to induce vomiting:


ance of sac with each dressing change to note 1. Drug of choicesyrup of ipecac (available over
signs and symptoms of infection, leaks, abra- the counter; does not require a physicians
sions, or irritation. order). If families with young children keep this
e. Enforce strict aseptic technique to prevent medication on hand it should be administered
infection (leading cause of morbidity/ only if directed by Poison Control Center.
mortality in neonatal period). Note: Safety of ipecac has been questioned due
f. Avoid repeated latex exposure (e.g., gloves, to esophageal tears (when misused) and anorexia
catheters) to decrease risk of latex allergy. nervosa/bulimia (when abused).
2. Goal: prevent infection in postoperative period. 2. Dose:
a. Position: prone, side-lying, or partial a. 30 mL for adolescents over 12 years; repeat
side-lying. dosage once if vomiting has not occurred
b. Use myelomeningocele apron (specific type within 20 minutes.
of dressing) to prevent urine or stool from b. 15 mL for children 1 to 12 years; repeat
contaminating suture line. dosage once if vomiting has not occurred
c. Administer antibiotics as ordered. within 20 minutes. Note: Do not administer
d. Use strict aseptic techniques in dressing to infants less than 1 year of age without
changes; standard precautions to prevent physicians order.
infection. c. 10 mL for infants 6 to 12 months; do not
3. Goal: prevent urinary retention and UTI. repeat dosage.
a. Monitor I&O, offer extra fluids to flush 3. Follow dose of ipecac with 4 to 8 oz of tap
kidneys. water or as much water as child will drink. In
b. Keep urethral meatus clean of stool to prevent young children, give water first because child
ascending bacterial infection. may refuse to drink anything else after tasting
c. Monitor urinary output for retention. the ipecac.
d. Administer antibiotics/urinary tract antisep- 4. The child must vomit the syrup of ipecac
tics as ordered. to avoid its being absorbed and causing
2164_Ch05_249-346 29/03/12 12:26 PM Page 313

Accidents: Ingestions and Poisonings 313


potentially fatal cardiotoxicity (cardiac arrhyth- d. Administer IV fluids, as ordered.
mias, atrial fibrillation, severe heart block). e. Assist with hemodialysis, as ordered, to
If child does not vomit within 20 minutes promote excretion of salicylates and fluids.
of second dose, summon paramedics; gastric 2. Goal: restore fluid and electrolyte balance.
lavage may be indicated upon arrival in a. Monitor I&O, urinalysis, specific gravity.
emergency department. Do not manually b. Prepare sodium bicarbonate, administer as
stimulate gagging because gagging may ordered to correct metabolic acidosis.
vagal response significant bradycardia. c. Monitor IV fluids and electrolytes.
C. When not to induce vomiting: d. NPO initially (NG tube).
1. Child is stuporous or comatose. 3. Goal: reduce temperature.
2. Poison ingested is a corrosive substance or a. No aspirin or acetaminophen, which might
petroleum distillate. further complicate bleeding tendencies or lead
3. Child is having seizures. to liver or kidney damage.
4. Child is in severe shock. b. Supportive measures: cool soaks, ice packs to
5. Child has lost the gag reflex. armpits/groin, hypothermia blanket.
II. SALICYLATE POISONING 4. Goal: prevent bleeding and possible hemorrhage.
a. Monitor urine and stools for occult blood.
A. Assessment:
b. Insert NG tube to detect gastric bleeding.
1. Determine how much aspirin was ingested,
c. Observe for petechiae, bruising; monitor
when, which type.
laboratory values for Hct and Hgb.
2. Evaluate salicylate levels: normal, 0; therapeutic
d. Administer vitamin K as ordered to correct
range = 15 to 30 mg/dL; toxic, >30 mg/dL.
bleeding tendencies.
3. Early identification of mild toxicity:
5. Goal: health education to prevent another
a. Tinnitus (ringing in the ears).
accidental poisoning.
b. Changes in vision, dizziness.
a. Teach principles of poison prevention.
c. Sweating.
b. Stress need to avoid accidental overdose
d. Nausea, vomiting, abdominal pain.
with over-the-counter medications or dosage
4. Immediate recognition of salicylate poisoning:
mix-ups.
a. Hyperventilation (earliest sign).
c. Allow child/parents to verbalize guilt, but
b. Fevermay be extremely high (105 to

PEDIATRICS
avoid blaming or scapegoating.
106F).
D. Evaluation/outcome criteria:
c. Respiratory alkalosis or metabolic acidosis.
1. Aspirin is successfully removed from childs body
d. Late signs: bleeding tendencies, severe
without permanent damage.
electrolyte disturbances, liver or kidney
2. Fluid and electrolyte balance is restored and
failure.
maintained.
B. Analysis/nursing diagnosis:
3. Child is afebrile.
1. Ineffective breathing patterns related to
4. Bleeding is controlled, no hemorrhage occurs.
hyperventilation/respiratory alkalosis.
5. No further episodes of poisoning occur.
2. Fluid volume deficit (dehydration) related to
increased insensible loss of fluids through III. ACETAMINOPHEN POISONING
hyperventilation, increased loss of fluids A. Assessment:
through vomiting, and increased need for 1. Determine how much acetaminophen was
fluids due to hyperpyrexia (fever). ingested, when, and which type.
3. Risk for injury related to bleeding. 2. Evaluate acetaminophen levels: normal = 0;
4. Anxiety related to parental/child feelings of guilt, therapeutic range = 15 to 30 mcg/mL; toxic =
uncertainty as to outcome, invasive nature of 150 mcg/mL 4 hours after ingestion.
treatments. 3. Initial period (24 hours after ingestion):
5. Knowledge deficit regarding accident malaise, nausea, vomiting, anorexia, diaphoresis,
prevention. pallor.
C. Nursing care plan/implementation: 4. Latent period (13 days after ingestion): clinical
1. Goal: promote excretion of salicylates. improvement with asymptomatic rise in liver
a. If possible, induce vomiting using syrup of enzymes.
ipecac (save, bring to emergency department). 5. Hepatic involvement (may last 7 days or may be
b. Assist with gastric lavage, if appropriate. permanent): pain in RUQ, jaundice, confusion,
c. Administer activated charcoal as early as hepatic encephalopathy, clotting abnormalities.
possible. 6. Gradual recuperation.
2164_Ch05_249-346 29/03/12 12:26 PM Page 314

314 chapter 5 Health Promotion and Maintenance

B. Analysis/nursing diagnosis: IV. LEAD POISONING (PLUMBISM)


1. Altered tissue perfusion (liver) related to hepatic A. Introduction: Lead poisoning is a heavy-metal
necrosis. poisoning that occurs from ingestion or inhalation
2. Fluid volume deficit related to increased loss of of lead. In children, this is most common in the
fluids secondary to vomiting and diaphoresis. toddler age group (13 years) and is usually a chronic
3. Risk for injury related to bleeding and clotting type of poisoning that occurs as the result of
disorders. repeated ingestions of lead. Older plumbing is one
4. Anxiety related to parental/child feelings of guilt, source of lead. Children who engage in the practice
uncertainty as to outcome, and invasive nature of pica, the ingestion of nonnutritive substances,
of treatments. often ingest lead in flecks of lead-based paint
5. Knowledge deficit regarding accident prevention. from plumbing, walls, furniture, or toys. In addi-
C. Nursing care plan/implementation: tion, research demonstrates that the parent-child
1. Goal: promote excretion of acetaminophen. relationship is a significant variable in lead poison-
a. If possible, induce vomiting; save, bring to ing; typically, there is a lack of adequate parental
emergency department. supervision that enables the child to engage in pica
b. Assist with gastric lavage, if appropriate. repeatedly over a fairly long time, until symptoms
c. Administer activated charcoal. of lead poisoning become evident. (Fig. 5.14 shows
d. Assist with obtaining acetaminophen level the pathophysiological effects of lead poisoning.)
4 hours after ingestion. B. Assessment:
2. Goal: prevent permanent liver damage. 1. Investigate history of pica.
a. Treatment must begin as soon as possible; 2. Evaluate parent-child relationship.
therapy begun later than 10 hours after 3. Chronic lead poisoning: vague, crampy
ingestion has no value. abdominal pain; constipation; anorexia and
b. Administer the antidote (acetylcysteine vomiting; listlessness.
[Mucomyst]) per physicians order. Usually 4. Neurological, renal, hematological effects: see
administered in cola or through NG tube Fig. 5.14.
because of offensive odor. Given as one 5. Blood lead linebluish-black line seen
loading dose and 17 maintenance doses. in gums.
c. Monitor hepatic functioningassist with 6. X-rays: lead lines in long bones and flecks of
PEDIATRICS

obtaining specimens and check results lead in GI tract.


frequently; be aware that liver enzymes will 7. Elevated serum blood lead levels: 20 mcg/dL
rise and peak within 3 days and then should requires clinical management; 45 mcg/dL
rapidly return to normal. requires parenteral chelating therapy.
3. Goal: restore fluid and electrolyte balance. C. Analysis/nursing diagnosis:
a. Monitor vital signs and perform neurological 1. Altered thought processes related to neurotoxicity.
checks every 2 to 4 hours and prn. 2. Activity intolerance (and risk for infection) related
b. Monitor I&O; urine analysis, including to anemia.
specific gravity; and weight. 3. Altered urinary elimination related to excretion
c. Monitor IV fluids as ordered. of lead by kidneys.
4. Goal: prevent bleeding. 4. Pain related to lead poisoning and its treatment.
a. Assist in monitoring childs PT; notify 5. Knowledge deficit related to etiology of lead
physician of significant changes. poisoning.
b. Monitor urine and stool for occult blood. D. Nursing care plan/implementation:
c. Observe for and report any petechiae or 1. Goal: promote excretion of lead.
unusual bruising. a. Administer chelating agents (EDTA [IM or
5. Goal: health education to prevent another IV], BAL [IM only]) as ordered. Chelation
accidental poisoning. (See Goal 5, Nursing therapy typically continues over several days,
care plan/implementation, Salicylate with multiple treatments daily.
poisoning.) b. Monitor kidney function carefully: the treat-
D. Evaluation/outcome criteria: ment itself is potentially nephrotoxic.
1. Acetaminophen is successfully removed from Maintain adequate oral intake of fluids.
childs body. c. Institute seizure precautions.
2. Normal liver function is reestablished. 2. Goal: prevent reingestion of lead.
3. Fluid and electrolyte balance is restored and a. Determine primary source of poisoning.
maintained. b. Eliminate source from childs environment
4. No further episodes of poisoning occur. before discharge.
2164_Ch05_249-346 29/03/12 12:26 PM Page 315

Pediatric Surgeries 315

Ingestion of lead by child

Absorption of lead into body tissues

Central nervous system Hematologic system Renal system

Intracranial pressure Blocks normal RBC formation Damages proximal tubules

Permanent, irreversible Anemia (reversible) Glycosuria (reversible)


neurologic damage: Proteinuria (reversible)
Seizures Ketonuria (reversible)
Learning disabilities
Hyperactivity
Mental retardation
Blindness
Paralysis
Coma, death

Figure 5.14 Pathophysiological effects of lead poisoning.

PEDIATRICS
c. Follow up with home care referral. b. Teach that the chelating agent binds with the
(1) Screen other siblings prn. lead and promotes its excretion through the
(2) Monitor blood lead level of all children in kidneys.
the home. E. Evaluation/outcome criteria:
3. Goal: assist child to cope with multiple painful 1. Lead is successfully removed from childs body
injections when treated with IM chelation therapy. without permanent damage.
a. Prepare child for treatment regimen. 2. No further episodes of lead poisoning.
b. Stress that this is not a punishment. 3. Child copes successfully with the disease and its
c. Rotate sites as much as possible. treatment.
d. May use a local anesthetic (e.g., procaine,
injected simultaneously with chelating agent
to decrease pain of injections).
P E D I AT R I C
e. Apply warm soaks to injection sites: may
SURGERIES:
help lessen pain.
NURSING
f. Encourage child to self-limit gross muscle
C O N S I D E R AT I O N S
activity (which increases pain). I. In general, basic care principles for children are the
g. Offer child safe outlets for anger, fear, same as for adults having surgery.
frustrationpunching bag, pounding
II. EXCEPTIONS:
board, clay.
h. Offer opportunity for medical play with A. Children should be prepared according to their
empty syringes, etc. developmental level and learning ability.
4. Goal: health teaching. B. Children cannot sign own surgical consent form; to
a. Stress (to child and parents) that removing the be done by parent or legal guardian.
lead is the only way to prevent permanent, C. Parents should be actively involved in the childs care.
irreversible neurological damage (irreversible III. See Table 5.22, which reviews specific nursing care
damage may have already occurred). for the most common pediatric surgical procedures.
2164_Ch05_249-346 29/03/12 12:26 PM Page 316

316 chapter 5 Health Promotion and Maintenance

Table 5.22
Pediatric Surgeries: Nursing Considerations
Surgical Procedure Specific Nursing Care
TONSILLECTOMY Preoperative: check bleeding and clotting times.
Postoperative:
Positionplace on abdomen or semiprone with head turned to side to prevent aspiration.
Observe for most frequent complicationhemorrhage (frequent swallowing, emesis of bright
red blood, shock).
Prevent bleeding:
Do not suctionmay cause bleeding.
Do not encourage coughing, clearing throat, or blowing nosemay aggravate operative site and
cause bleeding.
Minimize crying.
Decrease pain:
Offer ice collar to decrease pain and for vasoconstriction, but do not force.
Acetaminophen for pain (no aspirin, ibuprofen).
Nutrition:
NPO initially, then cool, clear fluids such as cool water, crushed ice, flavored icepops, dilute
(noncitrus) fruit juice.
No red or brown fluids (punch, Jell-O, icepops, colas), citrus juices, warm fluids (tea, broth), toast,
milk/ice cream/pudding, carbonated sodas; progress to soft, bland foods; milk products can
increase production of mucus.
Teach parents/discharge planning:
Signs and symptoms of infection; call physician promptly.
510 days postoperatively, expect slight bleeding.
Continue soft, bland diet as tolerated.
MYRINGOTOMY (tubes) Postoperative:
Positionplace with operated ear down, to allow for drainage. Expect moderate amount of
purulent drainage initially.
Keep external ear canal clean and dry.
Teach parents/discharge planning:
Need to keep water out of earuse special ear plugs when bathing or swimming.
PEDIATRICS

Tubes will remain in place 37 months and then fall out spontaneously (with healing of eardrum).
APPENDECTOMY (Observe same principles of preoperative and postoperative care as for adult GI surgery.)
NPO until bowel sounds return (2448 hours).
If appendix ruptured preoperatively or intraoperatively, position in semi-Fowlers and
implement wound precautions; administer antibiotics as ordered.
Monitor for signs and symptoms of peritonitis.
Typical course: speedy recovery, with discharge in about 23 days and excellent prognosis.
HERNIORRHAPHY Umbilical: increased incidence in infants who are of African descent.
(umbilical/inguinal) Inguinal: increased incidence in boys.
Preoperative: monitor for possible complications of strangulation.
Routine postoperative GI surgery care.
Prognosisexcellent, with discharge 2448 hours postoperatively.

SELECTED Febrile seizures ______________________________ 268


P E D I AT R I C Intussusception _________________________ 289290
EMERGENCIES Pediatric respiratory infections: epiglottitis, bronchiolitis,
croup, LTB ____________________ Table 5.9, p. 270
For a quick review, use this index to locate content on Poisoning:
19 pediatric emergencies that are covered in this book. Acetaminophen poisoning ________________ 313314
Emergency _______________________________ Page Salicylate poisoning _________________________ 313
AGE: dehydration _______________________ 288, 290 Reye syndrome _________________________ 266267
Apnea of infancy ________________________ 274, 276 Sickle cell anemia:
Appendicitis _______________________________ 316 Vaso-occlusive crisis _________________________ 296
Asthma _______________________________ 271272 Splenic sequestration crisis ____________________ 297
Bacterial meningitis __________________________ 265 SIDS _________________________________ 274277
Diabetes: hypoglycemia, ketoacidosis ____ 294, 469470 Tracheoesophageal fistula _________________ 286287
2164_Ch05_249-346 29/03/12 12:26 PM Page 317

Questions 317

Questions 7. A hospitalized child is experiencing sickle cell vaso-


occlusive crisis. The child is currently receiving an intra-
Select the one answer that is best for each question, unless venous (IV) fluid bolus, pain medication every 4 hours,
otherwise directed. and warm compresses to the extremities per physician
1. A nurse is caring for a child diagnosed with syndrome of orders. During the midday assessment, the child reports
inappropriate antidiuretic hormone (SIADH). Which labo- no pain. Which action should a nurse take?
ratory test would the nurse be least likely to obtain? 1. Continue to apply warm compresses per physician
1. Urine specific gravity. order.
2. Blood glucose. 2. Hold the next dosage of pain medication.
3. Serum sodium. 3. Hold the next round of warm compresses.
4. Urine osmolality. 4. Contact the physician for a change in orders.

2. A physician prescribes digoxin (Lanoxin) for a toddler with 8. What should be the expected weight of an infant at
congestive heart failure (CHF). Before administering the 12 months of age whose birth weight was 3600 grams?
medication, it is most important for the nurse to: 1. 5600 grams.
1. First obtain an apical heart rate. 2. 7200 grams.
2. Determine the serum potassium. 3. 11 kilograms.
3. Review the childs admission electrocardiogram (ECG). 4. 15 kilograms.
4. Mix the medication with a pleasant-tasting food. 9. An infant in a newborn nursery is identified as having
3. A nurse prepares to administer spironolactone (Aldactone) phenylketonuria (PKU). What is the best initial source of
to an infant with congenital heart disease. The nurse nutrients for an infant with this diagnosis?
understands that the main purpose of this medication is to: 1. Maternal breast milk.
1. Preserve the patent ductus arteriosus. 2. Pregestimil.
2. Cause vasodilation of the blood vessels. 3. Lofenalac.
3. Prevent the secretion of potassium. 4. Isomil.
4. Block aldosterone, which leads to diuresis. 10. In assessing the reflexes of a 15-month-old child, which
4. Which symptom(s), if present in a child, should a nurse finding would indicate that the child is experiencing
recognize as being characteristic of Kawasaki disease? Select normal development?
1. Positive Babinski reflex.

QUESTIONS
all that apply.
1. Strawberry tongue. 2. Asymmetric tonic neck reflex.
2. High fever. 3. Positive patellar reflex.
3. Irritability. 4. Presence of dolls eye reflex.
4. Cough. 11. A child with type 1 diabetes is receiving insulin based on
5. Desquamation of the extremities. carbohydrate intake. The childs insulin-to-carbohydrate
6. Elevated ESR. ratio is 15:1. Of the items listed on the childs lunch
5. Which orders should a nurse question for a 5-month-old menu shown below, the child ate 2 slices of bread, a slice
infant with hypoplastic left heart syndrome who is hospital- of cheese, a glass of milk, a cup of soup, and half of a
ized awaiting the second stage of surgical repair? Select all banana. How many units of insulin should the nurse
that apply. administer based on the clients carbohydrate count?
1. Call physician for oxygen saturations below 85%. Round to the nearest whole number.
2. Daily weights.
Food Item: Carbohydrates:
3. Hold digoxin (Lanoxin) for heart rate less than 80 beats
per minute. Banana 22 g
Glass of low-fat milk 10 g
4. Strict I&O.
Bread slice 15 g
5. Enfamil formula ad lib. Cheese slice Free
Cup of soup 10 g
6. A nurse would be most correct in withholding digoxin
(Lanoxin) prescribed to an infant if the heart rate falls
1. 2 units.
below which parameter?
2. 3 units.
1. Below 100 beats per minute.
3. 4 units.
2. Below 120 beats per minute.
4. 5 units.
3. Below 140 beats per minute.
4. Below 160 beats per minute.
2164_Ch05_249-346 29/03/12 12:26 PM Page 318

318 chapter 5 Health Promotion and Maintenance

12. A nurse is caring for a child with acute glomeru- 18. A child arrives in an emergency department with a
lonephritis. Which nursing assessment should be the chief complaint of asthma exacerbation. Which assess-
nurses first priority when caring for this child? ment information is most important for the nurse to
1. Obtaining a daily weight. obtain first?
2. Palpating extremities frequently for edema. 1. Whether the child has been taking asthma medications
3. Assessing urine for hematuria. as prescribed.
4. Obtaining the childs blood pressure every shift. 2. When the child began having symptoms.
3. Whether the child is able to speak in full sentences.
13. In developing a plan of care for a hospitalized preschool- 4. The childs ABG levels.
er, a nurse recognizes that it is most essential to consider:
1. That the child may believe the hospitalization is a 19. A child is seen in an emergency department following the
punishment. ingestion of lighter fluid. Which nursing action is of the
2. Ways to provide visitation from peers. highest priority at this time?
3. How to incorporate play activities with other children. 1. Induce vomiting.
4. Ways to promote privacy and independence. 2. Determine the amount of poison ingested.
3. Assess the respiratory system.
14. A nurse assesses a child who is 12 hours status post 4. Administer Mucomyst as ordered.
tonsillectomy and adenoidectomy. The child reports
feeling nauseated and shows the nurse a moderate 20. A nurse prepares to administer a chelating agent to a
amount of red-tinged vomitus in the emesis basin. child with lead poisoning. Which laboratory tests should
Which action should the nurse take first? be obtained prior to the administration of this agent?
1. Administer an antiemetic as ordered. 1. BUN and creatinine.
2. Offer the child ice chips as tolerated. 2. PT, PTT.
3. Report the findings to the physician. 3. Urine specific gravity.
4. Apply bilateral pressure to the childs neck. 4. CBC.
15. The parents of a 2-year-old child ask a nurse how to best 21. A nurse is caring for a child with meningococcemia who
assist the child to accomplish developmental tasks at this is on a ventilator. This morning, the nurse finds the
age. What is the best response by the nurse? childs mother sitting at the bedside, crying. The mother
1. Make sure that the childs siblings insist that the child tells the nurse, I thought it was the flu. This is my fault
share toys at playtime. because I should have come to the hospital earlier. What
QUESTIONS

2. Since the child understands the word no, use this is the best action by the nurse in response to the mothers
word frequently to establish house rules. statements?
3. Ask grandparents and other child care providers to 1. Tell the mother not to worry since many parents and
follow your home schedule as much as possible. even physicians frequently mistake meningitis symp-
4. Attend to the child quickly during temper tantrums toms for other infectious conditions.
by hugging and offering reassurance. 2. Make a referral to social services.
3. Call the childs father and explain that the mother
16. An infant is hospitalized following a febrile seizure. needs emotional support from him.
When a nurse teaches the infants family about the pre-
4. Remind the mother that she did seek proper treatment
vention of future seizures, what would be the nurses
as soon as she became concerned, and review the spe-
best recommendation?
cial care the child is receiving now.
1. Place the child in a tepid bath during the next febrile
illness. 22. An infant is admitted for probable pyloric stenosis. A
2. Administer antipyretics around the clock the next time physician orders IV fluids and makes the infant NPO
the child has a fever. pending a surgical consult. The infant is crying vigorous-
3. Contact the physician for antibiotics if the child ly and the parents express frustration that they cannot
becomes feverish again. feed their baby even though the surgery is not yet defi-
4. Take the childs temperature frequently during the next nite. Which is the best action for the nurse to take now?
illness. 1. Explain to the parents that feeding an infant with
pyloric stenosis can lead to electrolyte imbalances from
17. A toddler with Kawasaki disease is being evaluated by a possible vomiting.
primary care clinic nurse 1 week following discharge. The
2. Offer the parents a pacifier for the infant.
nurse understands that it is a priority to instruct the par-
3. Place a call to the surgeon to find out how long it will
ents to contact the clinic immediately if the child:
be before the consult.
1. Throws frequent temper tantrums.
4. Feed the infant a small amount of Pedialyte since the
2. Is exposed to someone with chickenpox.
surgical repair for this condition will most likely not
3. Experiences night terrors.
occur until the following day.
4. Develops a low-grade fever.
2164_Ch05_249-346 29/03/12 12:26 PM Page 319

Questions 319
23. A 1-day-old infant, born at 39 weeks gestation, weighs 3. Improved general appetite.
4 pounds, 7 ounces at birth. A pediatrician diagnoses the 4. Hemoglobin and hematocrit within normal limits.
neonate with intrauterine growth restriction (IGR). An 29. A nurse is performing discharge teaching with the parents
RN observes the newborn to be irritable, difficult to con- of a preschooler diagnosed with cystic fibrosis. What part
sole, restless, fist-sucking, and demonstrating a high- of the teaching plan will best assist the parents to prevent
pitched, shrill cry. Based on these assessment data, the future pulmonary infections in this child?
RN should: 1. Teaching the parents proper administration of pancre-
1. Increase stimulation of the baby by handling the infant atic enzymes.
as much as possible. 2. Emphasizing the need for regular and consistent chest
2. Schedule routine feeding times every 3 to 4 hours. physiotherapy.
3. Encourage stimulation by rubbing the infants back 3. Stressing the need to seek prompt medical attention
and head. for increased work of breathing.
4. Tightly swaddle the infant in a flexed position. 4. Instructing the parents to monitor the childs daily
24. A nurse performs a scoliosis screening at a local school. fluid intake for adequacy.
Which assessment finding by the nurse would least likely 30. The mother of a child asks a clinic nurse how to safety-
result in a scoliosis referral? proof the home. What should the nurse recognize as the
1. Unilateral rib hump noted when the child is bent most effective means to prevent accidental poisoning?
forward. 1. Keep the Poison Control Center phone number near
2. Asymmetrical hip height noted when the child is the phone.
standing erect. 2. Store poisons in the garage rather than in the home.
3. Uneven wear noted on the bottom of the childs pant 3. Scan the home from the childs eye level and remove
legs. accessible toxins.
4. Rounded shoulders noted when the child is standing 4. Tell children where toxic substances are kept and
erect. instruct them not to go there.
25. Which conditions in children and/or adolescents should 31. A nurse visits the home of a toddler. With what aspect
a nurse identify as being associated with metabolic of the home environment would the nurse be most
alkalosis? Select all that apply. concerned?
1. Pyloric stenosis. 1. Power cords plugged into capped electrical outlets.

QUESTIONS
2. Diabetes. 2. Presence of a television in the childs bedroom.
3. Renal failure. 3. A swimming pool located in the backyard.
4. Bulimia nervosa. 4. Cooking pot handle turned toward the front of the
5. Aspirin ingestion. stove.
26. A child is admitted for treatment of lead poisoning. A 32. An infant is admitted to a pediatric unit with labored
nurse recognizes that the priority nursing diagnosis for breathing and moderate amounts of thick nasal secre-
this child is: tions. What nursing intervention is most likely to
1. Alteration in comfort related to abdominal pain. improve the infants oxygenation?
2. Alteration in nutrition related to pica. 1. Frequent suctioning of the nares with a nasal olive.
3. Pain related to chelation therapy. 2. Providing supplemental oxygen via nasal cannula.
4. Alteration in neurologic functioning. 3. Strict monitoring of oxygen saturation levels.
27. A newborn arrives in a neonatal intensive care unit with a 4. Placing the child in an infant seat.
myelomeningocele. A physician writes orders to keep the 33. A client is attending a newborn discharge class and asks a
infant in the prone position. A nurse should know that nurse about the bump on the infants head. Upon assess-
the most important rationale behind this order is to: ment, the neonate has a large, diffuse swelling on the left
1. Prevent infection. occiput that crosses the sagittal suture line. The nurse
2. Promote circulation in the lower extremities. should explain to the mother that:
3. Prevent trauma to the meningeal sac. Select all that apply.
4. Promote comfort. 1. This is a collection of blood under the skull bone of
28. A child, hospitalized with nephrotic syndrome, has been the infant.
receiving corticosteroids for a week. What should the 2. It is edematous swelling that overlies the periosteum.
nurse recognize as early evidence that the child is 3. It leads to hyperbilirubinemia in the infant.
responding well to treatment? 4. It will require no treatment to resolve.
1. Decreased general edema. 5. It is caused by pressure on the fetal head before
2. Increased urinary output. delivery.
2164_Ch05_249-346 29/03/12 12:26 PM Page 320

320 chapter 5 Health Promotion and Maintenance

34. A child with status postHarrington rod placement for 40. A nurse prepares to administer digoxin to an infant.
the correction of scoliosis is being cared for on the pedi- Where is the most appropriate location for the nurse to
atric unit. The child suddenly experiences facial sweating assess the infants heart rate?
and complains of a headache. A nurse notes also a slower
heart rate on the monitor. What action should the nurse
take first?
1. Call the surgeon immediately.
2. Assess patency of the urinary catheter.
3. Administer pain medication as ordered.
4. Complete a neurological assessment.
35. What assessment findings should a nurse expect in a
child with acute post-streptococcal glomerulonephritis?
Select all that apply. D
B
1. Severe hematuria.
2. Pallor.
3. Decreased urine specific gravity.
4. Weight gain. A
5. Headache.
C
6. Massive proteinuria.
36. In doing a childs admission assessment, which signs and
symptoms should a nurse recognize as most likely related
to rheumatic fever?
1. Vomiting and diarrhea. 1. A.
2. Arthralgia and muscle weakness. 2. B.
3. Conjunctivitis and red, cracked lips. 3. C.
4. Bradycardia and hypotension. 4. D.
37. When preparing an intramuscular injection for a 41. Which statement made by the mother of a child with
1-week-old infant, which needle would be the most cystic fibrosis should indicate to a nurse that the mother
QUESTIONS

appropriate for the nurse to select? is in need of further teaching regarding the administra-
1. 18 G, 7/8 inch. tion of pancreatic enzymes?
2. 21 G, 1 inch. 1. Ill crush the capsules and mix with my childs food.
3. 25 G, 5/8 inch. 2. The capsule can be broken and its contents sprinkled
4. 25 G, 11/2 inch. onto food.
3. I may need to give more enzymes with larger meals.
38. A young child diagnosed with iron-deficiency anemia is 4. I will administer the enzymes 30 minutes after the
prescribed a liquid iron supplement. A nurse provides the
meal.
parents with instructions on administration and should
be certain to advise them that: 42. A nurse is working with a nursing student in the care of a
1. The medication should be given along with the childs young child status post-appendectomy. The student
morning cereal breakfast. checks the current order of IV gentamicin and discovers
2. The child may experience some pale-colored stools. the ordered dose is above the safe dose range based on
3. The child should be permitted to sip the medication the childs weight. What should be the nurses first action?
from a medicine cup. 1. Check the childs recent lab work.
4. The medication can be mixed with a small amount of 2. Contact the physician.
fruit juice. 3. Order a hearing test.
4. Obtain an order for BUN and creatinine.
39. A child is receiving chemotherapy for the treatment of
osteosarcoma. Which morning laboratory result must a 43. A child with type 1 diabetes is being prepared for dis-
nurse report immediately to the physician? charge from a hospital. What should a nurse include as
1. Absolute neutrophil count of 1200. part of the teaching regarding diabetes care?
2. Platelet count of 150,000. 1. Expect hypoglycemic episodes to always occur after
3. Urine dipstick positive for heme. meals.
4. WBC count of 4500. 2. Insulin dosage may need to be decreased during sports
activities.
2164_Ch05_249-346 29/03/12 12:26 PM Page 321

Questions 321
3. The child should not self-administer injections until 3. C.
the teen years. 4. D.
4. Insulin should never be administered during febrile
48. A nurse is planning to teach a child safety class to a
illnesses.
group of new parents. When preparing a lesson regarding
44. An LVN/LPN from an orthopedic unit is floated to a car seats, what should the nurse recommend?
child health unit. In creating assignments, which child 1. Children should be seated in the rear of the car until
should the charge nurse avoid assigning to the 6 years of age.
LVN/LPN? 2. Infants should face forward in an infant seat until
1. A 10-year-old in traction for a fractured femur. 20 pounds.
2. An 8-year-old child with Legg-Calv-Perthes disease. 3. Children should face the rear of the car until as close
3. A 4-year-old with osteogenesis imperfecta. to 1 year of age as possible.
4. A teenager receiving chemotherapy for osteosarcoma. 4. Make sure to use the automobile air bags as these
enhance the safety of car seats.
45. A nurse performs a head-to-toe assessment on a newborn.
Which finding should be of greatest concern to the nurse? 49. Which would be an abnormal finding when doing a
1. Capillary refill time of 2 seconds. well-child checkup on a 1-week-old infant?
2. Transient mottling of the skin. 1. An audible clunk during the Ortolani test.
3. Irregular respirations. 2. Symmetrical gluteal folds when the infant is held
4. Negative Babinski reflex. upright.
3. Negative Barlow test.
46. The parent of a young child phones an advice nurse to 4. Symmetrical knee height when the infant is supine.
report that the child is ill. The child has a reddish pin-
point rash most concentrated in the axilla and groin 50. A physician orders penicillin 200,000 units/kg/day IV
areas, a high fever, flushed cheeks, and abdominal pain. q6h for a child weighing 16 kg. The penicillin on hand
The parent also reports that the childs tongue is dark red comes prepared in a concentration of 250,000 units/mL.
with white spots. A nurse should recognize these symp- In order to administer the correct dose, a nurse calculates
toms as indicative of which infection? that _______ mL of penicillin should be administered to
1. Mumps. the child. Fill in the blank.
2. Measles.
51. Which nursing intervention should a nurse perform on a
3. Scarlet fever.

QUESTIONS
young child suspected of having a diagnosis of acute
4. Varicella.
epiglottitis whose oxygen saturation measures 93% on
47. Where should the nurse place the stethoscope on an room air?
infant to best auscultate bronchial breath sounds? 1. Allow the child to sit in a position of comfort.
2. Provide small amounts of liquid orally via a syringe.
3. Inspect the childs nares to assess degree of swelling.
4. Apply 100% oxygen via mask.
52. A nurse teaches a child with spina bifida how to perform
urinary self-catheterization. Which steps should the nurse
include in the teaching? Place each correct step in sequen-
tial order.
1. Wash hands.
B 2. Open latex catheter package.
3. Lubricate tip of catheter.
A 4. Wash catheter with soap and water.
C
5. Cleanse perineum with Betadine swabs.
53. A child recovering from abdominal surgery removes the
D nasogastric tube accidentally. A nurse replaces the naso-
gastric tube and places it to low wall suction. Two hours
later, the nurse discovers that there is no drainage from
the tube. What should be the nurses first action?
1. Ask the child to change position.
2. Place an urgent call to the surgeon.
1. A. 3. Flush the tube with 10 mL of sterile water.
2. B. 4. Check the suction mechanism and settings.
2164_Ch05_249-346 29/03/12 12:26 PM Page 322

322 chapter 5 Health Promotion and Maintenance

54. A charge nurse is seated in front of a bank of cardiac 60. A school nurse is preparing to teach a group of teenagers
monitors on a pediatric unit. There are four children how to prevent meningitis. What aspect of meningitis
receiving cardiac monitoring. Which finding should the prevention should the nurse be certain to include in the
charge nurse communicate at once to the childs nurse? presentation?
1. A heart rate of 50 in a 15-year-old adolescent who is 1. Getting a meningitis vaccine is the only way to
sleeping. guarantee prevention.
2. A heart rate of 190 in a 1-month-old infant who is 2. Refraining from sharing food and drinks is a good way
crying. to prevent meningitis infection.
3. A heart rate of 160 in a 2-year-old child who is walk- 3. Avoiding team sports is one way to stop the spread of
ing in the hallway. meningitis infection.
4. A heart rate of 75 in a 5-year-old child who is watch- 4. Meningitis prevention methods should be employed
ing television. whenever children are in crowds.
55. A school-age child visits a school nurse and states that a 61. When providing anticipatory guidance to the parents of a
family member has been behaving inappropriately by child with hemophilia, a nurse should stress that:
touching the child near the groin area. What should be 1. Active range-of-motion exercise should be used to treat
the school nurses priority action? sore joints.
1. Make a report to the proper child protective authori- 2. Aspirin should be given for minor bumps and bruises.
ties as mandated by law. 3. Warm compresses should be applied to wounds to
2. Contact the childs parents to share what the child has promote circulation.
reported. 4. A soft toothbrush should be used to promote oral
3. Question the child to determine all of the details of health.
the inappropriate touching.
62. When providing client teaching to the caregivers of a young
4. Provide the child with a safe and calm environment in
child with sickle cell disease, a nurse should stress that:
which to continue the discussion.
1. The childs diet should include whole grains and leafy
56. Which child would be the best roommate for a 9-year- green vegetables.
old child with myelodysplasia who is hospitalized for a 2. Immunizations should be delayed until the child
foot infection? enters school.
1. A 13-year-old with juvenile idiopathic arthritis. 3. There is a 50% chance that the childs future offspring
QUESTIONS

2. A 10-year-old with a fractured femur. will have sickle cell anemia.


3. An 8-year-old status post-appendectomy. 4. The parents should request IV Demerol if the child is
4. A 6-year-old with bacterial meningitis. hospitalized with pain crisis.
57. A 7-year-old child is hospitalized for a tonsillectomy. 63. A school nurse advises the dietary staff that a special
What are priority nursing actions when caring for this lunch tray must be created for a student who has celiac
child after surgery? Select all that apply. disease. What recommendation should the nurse provide
1. Advancing diet as tolerated. to the dietary staff?
2. Encouraging coughing to clear the throat. 1. Make sure the student has a whole-grain bread roll
3. Monitoring PT and PTT. each day.
4. Administering pain medication around the clock. 2. The child may have cake if the staff is celebrating
5. Suctioning mouth and throat frequently. someones birthday.
3. The childs pizza should be topped with a variety of
58. A nurse is caring for a child newly diagnosed with congen- vegetables.
ital heart disease. The nurse should monitor the child with
4. Beans and rice are suitable side dishes for this student.
the understanding that the earliest sign of heart failure is:
1. Audible lung crackles. 64. An infant is brought to an emergency department with a
2. Increased heart rate. chief complaint of nausea and vomiting. Which nursing
3. Weight gain. assessment finding should indicate to a nurse that the
4. Generalized edema. infants dehydration is severe?
1. The infant is lethargic with a urinary output of less
59. When teaching a class on home safety to new parents, than 1 mL/kg/hr.
on which type of exposure should a nurse focus as the
2. The infant has weak pulses, poor skin turgor, and cool,
primary cause of lead poisoning in children?
mottled skin.
1. Ingesting paint dust or chips from an old home.
3. The infant has warm skin, increased pulse, and capil-
2. Having a parent who works near lead products.
lary refill of 2 seconds.
3. Riding in a car that uses leaded gasoline.
4. The infant is irritable, with dry mucous membranes
4. Chewing on pencils with lead tips.
and increased respirations.
2164_Ch05_249-346 29/03/12 12:26 PM Page 323

Questions 323
65. Where is the best location for a nurse to auscultate a 69. A nurse admits a teenager in sickle cell crisis to a pedi-
murmur created by pulmonic stenosis? atric unit. The child has an elevated heart rate but nor-
mal blood pressure, respiratory rate, and temperature.
The child has an oxygen saturation of 98% on room air
and rates pain in the extremities at an 8 on a 1-to-10
numeric pain rating scale. Which actions should the
nurse perform at this time? Prioritize the nurses actions
by placing each correct intervention in priority order.
1. Administer oxygen.
C 2. Obtain the childs weight.
3. Administer IV fluids as ordered.
D
4. Monitor I&O.
5. Obtain an order for pain medication via PCA.
B 6. Apply cool, moist compresses to extremities.
70. A school nurse is creating an informational brochure for
parents regarding the treatment of head lice. What form
of treatment should the nurse caution against?
1. Applying repeated doses of permethrin for as long as it
A takes until the infestation clears.
2. Washing all clothing and linens in hot water followed
by drying them in a hot dryer.
3. Wearing gloves when washing the childs hair or
1. A.
inspecting for nits.
2. B.
4. Removing nits daily from the childs hair with a fine-
3. C.
tooth comb.
4. D.
71. The parents of a newborn infant ask a nurse how to pre-
66. When visiting the home of a school-age child who is vent future ear infections. What is the best advice the
dying, what would be the best action by a hospice nurse?
nurse should provide these parents?
1. Speak softly (whisper) when speaking in the childs

QUESTIONS
1. Avoid crowds during the winter months.
presence.
2. Allow the baby to bottle-feed in the supine position.
2. Provide as little interaction with the child as possible.
3. Make sure the baby receives the DTaP vaccine as
3. Avoid correcting the child who is in denial about dying.
scheduled.
4. Rely on the parents for pain assessment.
4. Continue breastfeeding as close to the babys first
67. A nurse is preparing to administer an unpleasant-tasting birthday as possible.
liquid medication to a toddler. What is the best method
72. A nurse enters the room of a child following the place-
for administering this medication?
ment of a ventriculoperitoneal shunt. The child is sitting
1. Mix the medication with a cup of ice cream to mask
up in bed, crying, and has vomited a small amount on
the taste.
the bed linens. What are the priority nursing actions?
2. Ask the child to choose between two types of fluids as
Select all that apply.
a chaser.
1. Complete a neurological assessment.
3. Request the parents hold the child firmly so the nurse
2. Place the child in the supine position.
can place the medication into the mouth.
3. Administer the antiemetic as ordered.
4. Offer the child a toy out of the toy box as a reward if
4. Complete a pain assessment.
the child agrees to take the medication.
5. Increase the childs IV rate.
68. A nursing student prepares to administer eyedrops to a 73. A charge nurse is creating nursing assignments for a pedi-
young child. What action by the nursing student should
atric unit when one of the oncoming nurses calls to say,
cause a registered nurse to intervene?
Sorry, Ill be a few minutes late since I have a child
1. The student positions the child supine with head
home ill with the chickenpox. What type of assignment
extended.
would be most acceptable for the nurse who will be late?
2. After administration, the student asks the child to
1. Any assignment is fine as long as the nurse wears a mask.
close eyes and move them around.
2. The nurse needs an assignment that does not include
3. The student schedules medication administration to
children with neutropenia.
occur just before lunchtime.
3. The nurse should not be given an assignment and
4. Prior to administration, the student pulls the lower lid
should be called off.
down, forming a sac.
4. Any care assignment is acceptable, without restrictions.
2164_Ch05_249-346 29/03/12 12:26 PM Page 324

324 chapter 5 Health Promotion and Maintenance

74. A nurse assesses the respiratory status of an infant. Which 80. A nurse prepares to insert a nasogastric tube in a
finding should be of most concern to the nurse? 10-month-old child. Which actions should the nurse
1. Tachypnea. take to complete this procedure? Prioritize the nurses
2. Scattered rhonchi. actions by placing each correct step in sequential order.
3. Expiratory grunt. 1. Aspirate gastric contents.
4. Abdominal breathing. 2. Have the child begin a bottle feeding.
3. Place child supine with head and neck
75. While suctioning a child with a tracheostomy tube in elevated.
place, a nurse discovers that the suction catheter will not
4. Inject 10 mL of air into the tube while
advance inside the tracheostomy tube and the child is
auscultating the stomach.
becoming pale and anxious, with noticeable suprasternal
5. Tape tube securely to infants cheek.
retractions. What should be the nurses priority action?
6. Measure from the infants earlobe to the area
1. Change the tracheostomy tube at once.
of the stomach.
2. Instill normal saline into the tracheostomy tube and
attempt suctioning again. 81. What is the priority nursing diagnosis for an infant
3. Obtain a pulse oximetry reading. receiving treatment for hyperbilirubinemia?
4. Auscultate lung sounds. 1. Imbalanced body temperature.
2. Alteration in elimination.
76. While preparing for an admission, a nurse hears the 3. Deficient fluid volume.
alarm sound on the cardiac monitor of a child in the next
4. Interrupted family processes.
bed. The nurse views the screen and sees what appears to
be ventricular fibrillation. What is the best initial action 82. Which assessment findings would cause a nurse to
by the nurse? withhold scheduled immunizations in a child? Select
1. Call out for help. all that apply.
2. Assess the child. 1. Current cold symptoms (e.g., runny nose, cough).
3. Begin chest compressions. 2. History of recent blood transfusion.
4. Press the Code Blue button. 3. Currently taking corticosteroids.
4. Mild diarrhea without symptoms of dehydration.
77. Which response to hospitalization is a nurse most likely 5. Family history of penicillin allergy.
to observe in a 4-year-old child?
6. Positive for HIV.
1. Fearfulness of loud noises and sudden movements.
QUESTIONS

2. Frequent crying outbursts and agitation. 83. A nurse and nursing student are caring for a child who
3. Urinary frequency and fear of mutilation. sustained a head injury as a result of a fall from a play
4. Boredom or loneliness. structure. The nurse knows the nursing student is pre-
pared to care for the child when the student states:
78. A 13-year-old client diagnosed with beta-thalassemia is 1. I will be sure to let you know if the childs pupils
hospitalized for blood transfusion. What are the priority
become fixed and dilated.
nursing diagnoses related to this childs care? Select all
2. I will keep the child straight in the supine position.
that apply.
3. I will look for any changes in the childs respirations,
1. Risk for infection.
pulse, or blood pressure.
2. Impaired elimination.
4. I will notify the physician if the child becomes sleepy.
3. Risk for injury.
4. Disturbed body image. 84. A child diagnosed with hypopituitarism is to begin receiv-
5. Chronic pain. ing daily injections. At what time should a nurse instruct
6. Activity intolerance. the childs parents to administer the injection each day?
1. Before breakfast.
79. A 3-year-old child is hospitalized with multiple fractures 2. At bedtime.
as a result of a car accident. What is the best way for a
3. With lunch.
nurse to assess this childs pain level?
4. Any time the child prefers.
1. Ask the child to rate pain using a numeric pain rating
scale. 85. An infant is hospitalized for congenital adrenal hyperpla-
2. Rely on vital sign measurements as a way to verify pain sia (CAH). Which medication should a nurse anticipate
ratings. to be part of the childs treatment plan?
3. Employ the FACES pain scale with every nursing 1. Insulin.
assessment. 2. Cortisone.
4. Try to have the child describe the pains intensity and 3. Growth hormone.
quality. 4. Thyroid hormone.
2164_Ch05_249-346 29/03/12 12:26 PM Page 325

Questions 325
86. A child hospitalized with hydrocephalus is being treated 2. Treat the spasms by sitting in the bathroom while a
with an externalized ventricular drain (EVD). A nurse hot shower runs.
begins the afternoon assessment and discovers that the 3. Bring the child back to the emergency department as
drain is positioned several inches below the childs ear soon as possible.
level. What should be the nurses priority action? 4. Elevate the childs head at bedtime using pillows.
1. Raise the drain to the childs ear level. 91. A 12-month-old child with infantile eczema is seen at the
2. Leave the drain as is and monitor the CSF drainage clinic for several open lesions on the arms and legs. What
hourly. should a nurse caution the childs parents against?
3. Quickly elevate the head of the bed. 1. Initiating a diet free of milk products.
4. Clamp the drain and complete a neurological 2. The use of topical hydrocortisone cream.
assessment. 3. Adding cornstarch to bath water.
87. A nurse is planning to teach a group of 10-year-old 4. Immunization during eczema exacerbations.
children about drug and alcohol prevention. Which 92. A nurse is working with a nursing student in caring for
characteristics of this age group are important for the an infant who has just returned from the surgical recov-
nurse to consider when developing the teaching plan? ery area following a cleft lip repair. Which action by the
Select all that apply. nursing student should cause the nurse to intervene?
1. These children are achievement-oriented. 1. Placement of elbow restraints on the infant.
2. They expect good behavior to be rewarded. 2. Offering the parents a regular bottle with which to
3. Their problem-solving approach tends to be concrete feed the infant.
and systematic. 3. Positioning the infant in the semi-Fowlers position.
4. The central persons in their lives tend to be friends. 4. Advising the parents of a plan to administer pain
5. These children are nearing puberty. medication around the clock.
88. A nurse visits the home of a young child to administer 93. A school-age child visits a school nurse with complaints
the Denver II developmental assessment. The child is of dizziness and shaking. The nurse confirms that the
unable to perform several required items, and the parent child has a history of type 1 diabetes mellitus when the
expresses concern regarding the childs performance. child becomes diaphoretic and begins to faint. What
What is the best way for the nurse to respond to the should be the nurses first action?
parents concerns? 1. Administer an injection of glucagon.

QUESTIONS
1. Reassure the parent that the Denver II is not a 2. Activate EMS.
measure of the childs IQ. 3. Squeeze glucose gel into the cheek.
2. Offer the parent some skill-building activities and 4. Test the childs blood sugar.
explain that the child will be reassessed in 2 weeks.
3. Advise the parent that the childs primary physician 94. A nurse should suspect Hirschsprungs disease in a child
will be notified and will make any necessary referrals. who has which type of stooling pattern?
4. Tell the parent that it is not unusual for children to 1. Pale gray stools.
fail the Denver II. 2. Currant-jelly stools.
3. Loose, yellow stools.
89. A nurse is caring for a child with tetralogy of Fallot. 4. Thin, ribbon-like stools.
Which assessment findings should the nurse expect?
Select all that apply. 95. A nurse attempts to give a newborn infant the first bottle
1. Ventricular septal defect (VSD). feeding. While sucking, the infant becomes cyanotic and
2. Atrial septal defect (ASD). coughs, and formula is seen coming out of the infants
3. Overriding aorta. nose. What should be the nurses first action?
4. Pulmonic stenosis. 1. Auscultate the lungs.
5. Right ventricular hypertrophy. 2. Suction the childs airway.
6. Patent ductus arteriosus (PDA). 3. Obtain an order for an x-ray.
7. Left-to-right shunting of blood. 4. Contact the physician.
8. Aortic stenosis 96. A nurse is caring for a newborn infant diagnosed with
90. The parents of a child recently discharged with acute hypospadias. The parents ask when the surgical repair
spasmodic laryngitis contact a nurse to report that will be complete. The nurse knows that the most likely
the child continues to have croupy coughing spells at time for completion of the surgical repair will be:
nighttime but is otherwise fine. What should the nurse 1. Within the first month of life.
recommend? 2. Not until the child reaches puberty.
1. Contact the childs physician for another round of 3. Nearer the childs first birthday.
antibiotics. 4. Before the child begins school.
2164_Ch05_249-346 29/03/12 12:26 PM Page 326

326 chapter 5 Health Promotion and Maintenance

97. A clinic nurse has a follow-up appointment with an TEST-TAKING TIP: Note how three of the answer options are
adolescent with juvenile idiopathic arthritis (JIA). What useful in assessing fluid shifts while one is not. The stem of the
topic should be the nurses top priority? question calls for the least likely laboratory test. Select the
1. Sleep patterns. answer that does not belong to the group.
2. Participation in daily exercise. Content Area: Child Health, Endocrine; Integrated Process:
3. Information regarding JIA support groups. Nursing Process, Analysis; Cognitive Level: Analysis;
4. Avoidance of alcohol use. Client Need/Subneed: Physiological Integrity/Reduction of
Risk Potential/Laboratory Values
98. An RN and LVN/LPN are working as a team on a
pediatric unit. Which task should the RN perform 2. CORRECT ANSWER: 1. Answer 1 is correct because the
rather than delegating to the LVN/LPN? apical heart rate must always be obtained by the nurse
1. Obtain a 12-lead ECG on a 10-year-old. prior to the administration of digoxin (Lanoxin) to a child.
2. Change the dressing and examine the decubitus ulcer Unless otherwise prescribed, the medication is typically
of a preschooler. withheld for a heart rate below 90 to 110 bpm in young
3. Administer a gavage feeding to an infant with failure children. Answer 2 is incorrect because, although the potassi-
to thrive. um level may be affected by the combined administration of
4. Check the blood sugar of a teen in DKA. digoxin (Lanoxin) with diuretics, the most important nursing
action is to determine whether the apical heart rate meets
99. A clinic nurse prepares to perform a physical assessment the parameter for safe digoxin (Lanoxin) administration.
on a preschool child. What are the appropriate actions Answer 3 is incorrect because the heart rate and not the
for the nurse to take when preparing for and perform- ECG waveform should be assessed prior to digoxin (Lanoxin)
ing the examination? Prioritize the nurses actions by administration. Answer 4 is incorrect since mixing the medica-
placing each correct step in sequential order. tion with a food or fluid is generally unnecessary due to the
1. Allow child to keep underpants on. medications sweet taste. Additionally, mixing with foods/fluids
2. Allow child to undress in private. poses the risk that the child will refuse the mixture and the
3. Ask childs preference for parental nurse may be unable to determine how much medication the
involvement. child ingested.
4. Inspect ears, eyes, and mouth. TEST-TAKING TIP: Since the main action of the drug is to
5. Proceed in head-to-toe direction. slow the heart rate, the nurse should monitor the heart rate for
6. Gain cooperation with bright objects as a signs of toxicity. The best answer is based upon the key safety
ANSWERS

distraction. parameter for this medication.


100. A child is admitted with acute exacerbation of asthma. Content Area: Child Health, Cardiovascular; Integrated Process:
A physician orders 100% oxygen via mask. Which Nursing Process, Implementation; Cognitive Level: Application;
physician order should be a nurses next priority? Client Need/Subneed: Physiological Integrity/Pharmacological
1. Continuous inhaled albuterol. and Parenteral Therapies/Medication Administration
2. IV Solu-Medrol 2 mg/kg loading dose. 3. CORRECT ANSWER: 4. Answer 1 is incorrect since
3. IV fluids at maintenance rate. spironolactone (Aldactone) does not affect the ductus arterio-
4. Chest x-ray. sus. Answer 2 is incorrect since spironolactone (Aldactone)
does not cause vasodilation. Answer 3 is incorrect because,
Answers/Rationales/Tips although spironolactone (Aldactone) is a potassium-sparing
diuretic, the main purpose of the medication is to remove
1. CORRECT ANSWER: 2. Answer 1 is incorrect because excess fluid through diuresis. Answer 4 is correct because
this is a measure of urine concentration, which provides spironolactone (Aldactone) is a diuretic that blocks
the practitioner information regarding the childs degree aldosterone. Use of this medication is common among
of fluid retention from excessive ADH production. children with congenital heart disease for the prevention
Answer 2 is correct because obtaining a blood glucose and treatment of congestive heart failure.
level is not expected for a child diagnosed with SIADH TEST-TAKING TIP: When unsure about medications and their
since the priority measures involve blood and urine actions, it is sometimes helpful to examine the root of the med-
osmolality. Blood glucose is a likely laboratory test in a ication name. Spironolactone (Aldactone) begins with A-L-D
child with diabetes insipidus. Answer 3 is incorrect as does the hormone aldosterone. The mechanism of action of
because a child with SIADH should be monitored closely spironolactone (Aldactone) is to inhibit aldosterone, thereby
for hyponatremia due to abnormal fluid retention. preventing water and sodium retention.
Answer 4 is incorrect because this is a measure of urine Content Area: Child Health, Cardiovascular; Integrated Process:
concentration, which provides the practitioner information Nursing Process, Analysis; Cognitive Level: Application;
regarding the childs degree of fluid retention from excessive Client Need/Subneed: Physiological Integrity/Pharmacological
ADH production. and Parenteral Therapies/Expected Effects/Outcomes
2164_Ch05_249-346 29/03/12 12:26 PM Page 327

Answers/Rationales/Tips 327
4. CORRECT ANSWERS: 1, 2, 3, 5, 6. Answer 1 is correct the medication to an infant with an apical heart rate below
because strawberry tongue is a symptom of Kawasaki 140 bpm if the infants heart rate was at least 90 bpm.
disease (mucocutaneous lymph node syndrome), occur- Answer 4 is incorrect because the nurse would administer
ring as the skin of the tongue sloughs off, leaving a the medication to an infant with an apical heart rate below
bright red tongue with white spots. Answer 2 is correct 160 bpm if the infants heart rate was at least 90 bpm.
because a symptom of Kawasaki disease is high fever TEST-TAKING TIP: Recall that the minimum apical heart rate
lasting more than 5 days. Answer 3 is correct because for an infant to receive digoxin (Lanoxin) is 90 to 110 bpm.
irritability is a symptom of Kawasaki disease. Answer 4 is The nurse would administer the medication if the infants
incorrect since a cough is not a symptom of Kawasaki dis- apical heart rate was 120, 140, or 160 bpm.
ease. Answer 5 is correct because the child with Kawasaki Content Area: Child Health, Cardiovascular; Integrated Process:
disease may experience peeling of the hands (on palms Nursing Process, Assessment; Cognitive Level: Application;
and fingertips) and feet (on soles and tips of toes) Client Need/Subneed: Physiological Integrity/Pharmacological
following the initial inflammatory rash on these areas. and Parenteral Therapies/Medication Administration
Answer 6 is correct because an elevated erythrocyte
7. CORRECT ANSWER: 1. Answer 1 is the correct response
sedimentation rate (ESR) is a symptom of Kawasaki since the child is currently receiving a fluid bolus, which
disease. Elevated ESR is indicative of an inflammatory may be providing temporary improvement of pain symp-
process, which would include Kawasaki disease. toms. Ongoing application of warm compresses will con-
TEST-TAKING TIP: When given the option to select more than tinue to promote circulation in the extremities, thereby
one answer, review each option carefully since each may be cor- preventing pain. Once the fluid bolus is completed, the
rect. Do not assume that only a few in the list must be correct. nurse should reassess the childs pain and circulation.
Content Area: Child Health, Infectious Disease; Integrated
Answer 2 is incorrect because around-the-clock dosing of
Process: Nursing Process, Assessment; Cognitive Level: Analysis;
pain medication is the best way to prevent pain in a child
Client Need/Subneed: Physiological Integrity/Physiological
with a chronic painful condition such as sickle cell crisis.
Adaptation/Pathophysiology Answer 3 is incorrect because cessation of the warm com-
5. CORRECT ANSWERS: 1, 3, 5. Answer 1 is correct because presses may lead to decreased circulation in the extremities,
the nurse should question an order that requires the resulting in a recurrence of the pain. Answer 4 is incorrect
nurse to call the physician for an oxygen saturation that because both warm compresses and pain medication should
is appropriate for an infant waiting surgical repair for a be continued to prevent recurrence of pain. A change in
severe cyanotic heart defect. Answer 2 is incorrect because orders is not necessary at this time, and the nurse should

ANSWERS
this is an appropriate order; the infant with a heart defect reassess the child following the IV fluid bolus.
should be weighed daily to assist in monitoring fluid status. TEST-TAKING TIP: Whenever a client is in the middle of
Answer 3 is correct because this parameter is too low. The receiving a prescribed treatment, such as a fluid bolus, the
heart rate should be at least 100 when administering nurse should continue supportive measures until the treatment
digoxin (Lanoxin) to an infant. Answer 4 is incorrect is complete and the nurse is able to reassess. Choose the one
because this is an appropriate order; the infant with a heart option that is differentcontinue.
defect should have strict monitoring of intake and output Content Area: Child Health, Cardiovascular; Integrated Process:
(I&O). Answer 5 is correct because an infant with a con- Nursing Process, Implementation; Cognitive Level: Analysis;
genital heart defect frequently requires formula with Client Need/Subneed: Physiological Integrity/Physiological
extra calories per ounce. Regular Enfamil formula would Adaptation/Alterations in Body Systems
not provide sufficient calories. 8. CORRECT ANSWER: 3. Answer 1 is incorrect because
TEST-TAKING TIP: Read the stem carefully. The question asks 5600 gm would be too little weight gain. Answer 2 is incor-
for the identification of orders that should not be implemented. rect because the infants birth weight should double by 4
Content Area: Child Health, Cardiovascular; Integrated Process:
to 7 months of age. Answer 3 is correct because an infant
Nursing Process, Analysis; Cognitive Level: Analysis; is expected to triple its birth weight in the first year of
Client Need/Subneed: Physiological Integrity/Reduction of
life; therefore, 11 kilograms (11,000 grams) is the best
Risk Potential/Potential for Alterations in Body Systems answer of the options given. Answer 4 is incorrect because
6. CORRECT ANSWER: 1. Answer 1 is correct because 15 kg would be too much weight gain compared to what is
digoxin (Lanoxin) should be withheld if the heart rate of expected.
the infant falls below 90 to 110 beats per minute (bpm). TEST-TAKING TIP: Note that the answer options may only
If digoxin (Lanoxin) is given when the infants heart rate approximate the answer that is calculated. Choose the answer
is 100, the resulting cardiac output may not be adequate. that most closely approximates your calculation.
Answer 2 is incorrect because the nurse would cautiously Content Area: Child Health, Growth and Development;
administer digoxin to an infant with an apical heart rate Integrated Process: Nursing Process, Analysis; Cognitive
below 120 bpm if the infants heart rate was at least 90 bpm. Level: Analysis; Client Need/Subneed: Health Promotion
Answer 3 is incorrect because the nurse would administer and Maintenance/Developmental Stages and Transitions
2164_Ch05_249-346 29/03/12 12:26 PM Page 328

328 chapter 5 Health Promotion and Maintenance

9. CORRECT ANSWER: 1. Answer 1 is correct because the because 3 units is an insufficient amount of insulin. Answer 3 is
child with PKU is missing the enzyme needed to digest correct because 4 units of insulin would be required to metab-
the amino acid phenylalanine. Maternal breast milk has olize 61 carbohydrates based on an insulin-to-carbohydrate
many beneficial properties and it contains low levels of ratio of 15:1. Answer 4 is incorrect because 5 units is an exces-
phenylalanine. Therefore, breast milk should be given sive amount of insulin.
until laboratory findings demonstrate the child is not TEST-TAKING TIP: When utilizing carbohydrate counts, be
tolerating the breast milk. Answer 2 is incorrect because sure to account for partial ingestion of foods as well as extra
Pregestimil is an example of a formula containing excessive servings of food items.
amounts of phenylalanine and is contraindicated for an Content Area: Child Health, Endocrine; Integrated Process:
infant with PKU. Answer 3 is incorrect. Although Lofenalac Nursing Process, Analysis; Cognitive Level: Analysis;
is a formula specially made for infants with PKU and con- Client Need/Subneed: Physiological Integrity/Pharmacological
tains very low amounts of phenylalanine, maternal breast and Parenteral Therapies/Medication Administration
milk is the best initial source of nutrients. Answer 4 is incor- 12. CORRECT ANSWER: 1. Answer 1 is correct because of
rect because Isomil is an example of a formula containing primary concern in the child with glomerulonephritis is
excessive amounts of phenylalanine and is contraindicated the monitoring of fluid balance. The nurse should obtain a
for an infant with PKU. weight for this child at the same time and on the same
TEST-TAKING TIP: Note that the stem of the question asks for scale daily to monitor for changes. Answer 2 is incorrect
the best initial source of nutrients. Breastfeeding should be because edema is expected in a child with glomerulonephritis
started first, and then special formulas may be considered if and is part of the diagnostic assessment. Although the nurse
necessary. should assess edema regularly, increased frequency of this
Content Area: Child Health, Gastrointestinal; Integrated
assessment is not needed and is not the highest priority of
Process: Nursing Process, Analysis; Cognitive Level:
care. Answer 3 is incorrect because gross hematuria is expected
Application; Client Need/Subneed: Physiological Integrity/Basic in a child with glomerulonephritis and is part of the diagnos-
Care and Comfort/Nutrition and Oral Hydration tic assessment. The presence of blood in the childs urine may
10. CORRECT ANSWER: 3. Answer 1 is incorrect because the last for some months; therefore, this is not the highest priori-
Babinski reflex is a normal newborn reflex. This reflex should dis- ty. Answer 4 is incorrect because a major complication of
appear well before the child reaches 15 months of age, and its glomerulonephritis is hypertensive encephalopathy. The nurse
persistence should be considered abnormal. Answer 2 is incorrect should obtain blood pressure at least every 4 hours; once per
because the tonic neck reflex is a normal newborn reflex. This shift is insufficient monitoring.
ANSWERS

reflex should disappear well before the child reaches 15 months TEST-TAKING TIP: Since glomerulonephritis involves improp-
of age, and its persistence should be considered abnormal. er kidney functioning, focus on responses dealing with fluid sta-
Answer 3 is correct because a positive patellar reflex is part of tus. The best answer calls for monitoring complications of fluid
a normal assessment. The reflex is obtained when the practi- imbalance with appropriate frequency.
tioner strikes the patellar tendon, causing the leg to kick. Content Area: Child Health, Genitourinary; Integrated Process:
Answer 4 is incorrect because the dolls eye reflex is a normal Nursing Process, Assessment; Cognitive Level: Application;
newborn reflex. This reflex should disappear well before the child Client Need/Subneed: Physiological Integrity/Reduction of Risk
reaches 15 months of age, and its persistence should be consid- Potential/System Specific Assessments
ered abnormal. 13. CORRECT ANSWER: 1. Answer 1 is correct because
TEST-TAKING TIP: Make sure to note when the stem of a preschoolers may perceive hospitalization as a punishment.
question asks you to identify the normal finding or the abnor- The nurse should create a plan of care that reassures the
mal finding. In this question, three options would be abnormal child and helps the child understand the reasons for hospi-
since they are newborn reflexes. Recall that a 15-month-old talization. Answer 2 is incorrect because providing peer visita-
should have the same reflexes as an adult, including a positive tion is a high priority for adolescents. Preschoolers are most
patellar reflex. concerned with having parents present during hospitalization.
Content Area: Child Health, Neurological; Integrated Process:
Answer 3 is incorrect because, although preschoolers should be
Nursing Process, Assessment; Cognitive Level: Application; provided play opportunities, group play is most essential for
Client Need/Subneed: Health Promotion and Maintenance/
the school-age child who craves interaction with children of the
Developmental Stages and Transitions same age. Answer 4 is incorrect because privacy and independ-
11. CORRECT ANSWER: 3. To solve this calculation, first add ence are most important to adolescents.
up the total amount of carbohydrates consumed by using the TEST-TAKING TIP: Recall that preschoolers are magical
menu as a reference. The total carbohydrates consumed are thinkers and may create their own explanations for events.
61. Next, divide the 61 total carbohydrates by 15 to arrive at Only one option reflects this developmental theory, making it
4.06. Rounded to the nearest whole number, the correct the best answer.
response equals 4 units of insulin. Answer 1 is incorrect because Content Area: Child Health, Hospitalization; Integrated
2 units is an insufficient amount of insulin. Answer 2 is incorrect Process: Nursing Process, Analysis; Cognitive Level: Analysis;
2164_Ch05_249-346 29/03/12 12:26 PM Page 329

Answers/Rationales/Tips 329
Client Need/Subneed: Health Promotion and Maintenance/ parents should monitor the childs temperature, the adminis-
Developmental Stages and Transitions tration of an antipyretic is the best prevention measure for
febrile seizures.
14. CORRECT ANSWER: 3. Answer 1 is incorrect because TEST-TAKING TIP: The question asks how to prevent febrile
administering an antiemetic fails to recognize the seriousness
seizures. The responses that assess temperature (Answer 4) and
of the childs symptoms, thereby placing the child in danger.
treat infection (Answer 3) should therefore be eliminated. Of
Answer 2 is incorrect because offering ice chips may induce
the remaining two options, one is contraindicated and cannot
further vomiting and is contraindicated in the client who is
be the correct response.
nauseated. Answer 3 is correct because the appearance of
Content Area: Child Health, Neurological; Integrated Process:
moderate red-tinged vomitus could indicate hemorrhage
Nursing Process, Implementation; Cognitive Level: Application;
in the surgical area. The physician should be notified
Client Need/Subneed: Physiological Integrity/Reduction of Risk
immediately of this potentially harmful complication.
Potential/Potential for Complications from Surgical Procedures
Answer 4 is incorrect because the application of pressure to
and Health Alterations
the childs neck is contraindicated and would not resolve
potential incisional bleeding and might block the carotid 17. CORRECT ANSWER: 2. Answer 1 is incorrect because tem-
arteries, thereby causing harm to the child. per tantrums are normal in this age group, and children with
TEST-TAKING TIP: When the stem of a question appears to Kawasaki disease may experience the symptom of irritability for
indicate active bleeding in a client, it is likely to be a medical some time after treatment. Answer 2 is correct because chil-
emergency. The proper initial response is to stop the bleeding dren with Kawasaki disease are placed on aspirin therapy, so
if possible. In this case, the bleeding is in the airway so only the exposure to chickenpox puts the child at risk for Reye syn-
physician is capable of stopping the bleeding. drome. Answer 3 is incorrect because night terrors can occur in
Content Area: Child Health, Respiratory; Integrated Process: toddlers and are a normal part of development. Answer 4 is
Nursing Process, Implementation; Cognitive Level: Application; incorrect because a low-grade fever can be related to minor
Client Need/Subneed: Physiological Integrity/Reduction of Risk infections, unlike the high, unremitting fever present in
Potential/Potential for Complications from Surgical Procedures Kawasaki disease.
and Health Alterations TEST-TAKING TIP: The word immediately in the stem of
the question implies urgency, so you should look for a concern
15. CORRECT ANSWER: 3. Answer 1 is incorrect because that must be addressed urgently. The behavioral issues listed
toddlers have not mastered the concept of sharing, so insist-
would not require such attention (even if they were abnormal
ing that the child share toys will likely result in frustration.
for the age group) and can be readily eliminated.
Answer 2 is incorrect because children at this age are increasing-

ANSWERS
Content Area: Child Health, Infectious Disease; Integrated
ly negative. Their negativity is reinforced when parents use the
Process: Nursing Process, Implementation; Cognitive Level:
word no frequently; avoidance of the word is suggested
Application; Client Need/Subneed: Physiological Integrity/
whenever possible Answer 3 is correct because toddlers prefer
Reduction of Risk Potential/Potential for Complications from
predictable schedules and routines. The child will feel more
Surgical Procedures and Health Alterations
comfortable if the home schedule is implemented when
away from the parents. Answer 4 is incorrect because temper 18. CORRECT ANSWER: 3. Answer 1 is incorrect because
tantrums will extinguish more rapidly if the child is ignored knowing whether the child has been taking medications is not
rather than given positive reinforcement. part of the initial physical assessment. Answer 2 is incorrect
TEST-TAKING TIP: Toddlers are in the stage of autonomy and because knowing when the child began having symptoms is
have a need to control their environment; therefore, choose the not part of the initial physical assessment. Answer 3 is correct
option that assists in this goal. because the nurse should first assess the childs airway to
Content Area: Child Health, Growth and Development; determine the severity of respiratory symptoms. One way to
Integrated Process: Nursing Process, Implementation; Cognitive assess shortness of breath is to determine whether the child
Level: Application; Client Need/Subneed: Health Promotion speaks in full sentences, short phrases, or barely at all.
and Maintenance/Developmental Stages and Transitions Answer 4 is incorrect because the nurse should first assess the
airway. ABGs (arterial blood gases) may be obtained later if
16. CORRECT ANSWER: 2. Answer 1 is incorrect because ordered by the practitioner.
tepid baths are contraindicated in the treatment of fevers as
TEST-TAKING TIP: The question asks what the nurse should
they cause shivering, which leads to a further elevation in tem-
do first. Remember the nursing processfirst assess the ABCs
perature. Answer 2 is correct because febrile seizures are
(airway, breathing, circulation). The other three responses
thought to occur when a child who is ill has a sudden high
contain information that could be obtained subsequent to the
fever. To prevent this situation, the parents should be
initial physical assessment.
instructed to administer an antipyretic around the clock
Content Area: Child Health, Respiratory; Integrated Process:
during the next febrile illness. Answer 3 is incorrect because
Nursing Process, Implementation; Cognitive Level: Application;
antibiotics are not indicated for every childhood illness and
Client Need/Subneed: Physiological Integrity/Reduction of Risk
antipyretic administration is the best way to prevent a spike in
Potential/System Specific Assessments
body temperature. Answer 4 is incorrect because, although the
2164_Ch05_249-346 29/03/12 12:26 PM Page 330

330 chapter 5 Health Promotion and Maintenance

19. CORRECT ANSWER: 3. Answer 1 is incorrect because Content Area: Child Health, Infectious Disease; Integrated
inducing vomiting is contraindicated for the ingestion of a Process: Communication and Documentation; Cognitive Level:
hydrocarbon. Answer 2 is incorrect because, although it is Application; Client Need/Subneed: Psychosocial Integrity/
helpful to know the approximate amount of poison ingested, Therapeutic Communications
the highest priority upon admission is to assess the child for 22. CORRECT ANSWER: 1. Answer 1 is correct because the
symptoms of aspiration. Answer 3 is correct because, when best action for the nurse is to help the parents under-
a child ingests a hydrocarbon such as lighter fluid, there stand that the NPO status is to avoid vomiting. This
is an immediate danger of aspiration. Therefore, the message should be delivered while expressing empathy for
nurses first priority is to assess the lungs. Answer 4 is the situation. Answer 2 is incorrect because simply offering
incorrect because Mucomyst (N-acetylcysteine) is the treat- a pacifier does not address the parents concern about feed-
ment for an overdose of acetaminophen and is not indicated ing the infant. Answer 3 is incorrect because knowing how
in the treatment of hydrocarbon ingestion. long it will be until the surgeon arrives does not address the
TEST-TAKING TIP: Any time a child ingests a poison, the parents concern about feeding the infant, and may actually
nurse should first assess for aspiration. increase their frustration if the surgeon is not able to arrive
Content Area: Child Health, Poisoning; Integrated Process:
until many hours later. Answer 4 is incorrect because this
Nursing Process, Implementation; Cognitive Level: Application; will likely result in more vomiting, which can lead to
Client Need/Subneed: Physiological Integrity/Reduction of Risk
electrolyte imbalances. This action also directly violates a
Potential/Potential for Alterations in Body Systems written order.
20. CORRECT ANSWER: 1. Answer 1 is correct because TEST-TAKING TIP: Helping families understand the treat-
the chelating agent binds with lead and is excreted by ment plan is always important in child health and is best done
the kidneys; therefore, normal kidney function should in combination with an empathetic and problem-solving
be established beforehand; blood urea nitrogen (BUN) approach.
and creatinine provide the best evidence of function. Content Area: Child Health, Gastrointestinal; Integrated
Answer 2 is incorrect because these laboratory tests would Process: Nursing Process, Implementation; Cognitive Level:
provide evidence of liver and not kidney function. Answer 3 Analysis; Client Need/Subneed: Psychosocial Integrity/
is incorrect because urine specific gravity is not a specific Therapeutic Communications
test of kidney function. Answer 4 is incorrect because a 23. CORRECT ANSWER: 4. Answer 1 is incorrect because
complete blood count (CBC) is not a specific test of kidney this infant needs comfort and security with decreased stimula-
function. tion. Increasing stimulation will not meet this infants needs.
ANSWERS

TEST-TAKING TIP: First consider the organ system best repre- Answer 2 is incorrect because this infant will benefit from
sented by the laboratory tests. If kidney function is of primary small, frequent feedings on demand rather than a schedule
importance, the best response is the test that best demonstrates that may increase stimulation by arousing the infant to eat on
normal functioning. a defined schedule. Answer 3 is incorrect because this infant
Content Area: Child Health, Poisoning; Integrated Process:
needs comfort and security with decreased stimulation.
Nursing Process, Analysis; Cognitive Level: Analysis; Increasing stimulation by rubbing will increase the infants
Client Need/Subneed: Physiological Integrity/Reduction of
irritability. Answer 4 is correct because tightly swaddling
Risk Potential/Laboratory Values the baby promotes the infants comfort and security and
21. CORRECT ANSWER: 4. Answer 1 is incorrect because, decreases the stimulation that may contribute to the
although meningitis symptoms are sometimes mistaken for infants irritability.
other conditions, this action does little to refocus the mothers TEST-TAKING TIP: The focus is on decreasing the irritability
attention away from her guilt. Answer 2 is incorrect because it in the infant. Choose the interventions that will decrease
is appropriate for a parent to express guilty feelings in this situ- stimulation and enhance security for the infant.
ation, and a referral to social services is not necessary based Content Area: Child Health, Newborn; Integrated Process:
solely on the mothers statements. Answer 3 is incorrect Nursing Process, Implementation; Cognitive Level: Application;
because contacting the father would violate the trusting rela- Client Need/Subneed: Health Promotion and Maintenance/
tionship that the mother has built with the nurse. Answer 4 is Ante/Intra/Postpartum and Newborn Care
correct because the mothers statement expresses guilt feel- 24. CORRECT ANSWER: 4. Answer 1 is incorrect because a
ings about the childs condition. A nurse needs to validate unilateral rib hump is indicative of scoliosis. Answer 2 is
that the mother did seek treatment appropriately, and assist incorrect because asymmetrical hip height is indicative of
the mother to focus on what is happening now to help her scoliosis. Answer 3 is incorrect because uneven pant lengths
child recover. may be indicative of scoliosis. Answer 4 is correct because
TEST-TAKING TIP: An essential component of the nurse-client the nurse is least likely to refer a child for scoliosis
relationship is trust. The best response incorporates both valida- follow-up based on an assessment finding of rounded
tion of the mothers efforts to help her child and reassurance shoulders. This finding may simply reflect the childs
that the child is receiving appropriate care.
2164_Ch05_249-346 29/03/12 12:26 PM Page 331

Answers/Rationales/Tips 331
poor posture or in severe cases may indicate the condi- important rationale for the prone position is to prevent
tion of kyphosis, not scoliosis. damage to the meningeal sac, which could result in
TEST-TAKING TIP: Screening for scoliosis does involve assess- damage to the nerves and infection. Answer 4 is
ing for differences between the right and left side of the childs incorrect because, although a newborn may prefer the
body when standing erect (Answer 2) and when bent forward prone position, the rationale for this intervention is not to
(Answer 1) at 90 degrees. The stem asks for least likely as the promote comfort.
best response. TEST-TAKING TIP: Two of the main priorities of care for an
Content Area: Child Health, Musculoskeletal; Integrated infant with myelomeningocele are to prevent infection and
Process: Nursing Process, Assessment; Cognitive Level: Analysis; prevent damage to the meningeal sac. To identify the most
Client Need/Subneed: Health Promotion and Maintenance/ important, think about which complication is more apt to
Health Screening cause irreversible damage.
Content Area: Child Health, Neurological; Integrated Process:
25. CORRECT ANSWERS: 1, 4. Answer 1 is correct because Nursing Process, Planning; Cognitive Level: Analysis;
children with pyloric stenosis experience loss of stomach
Client Need/Subneed: Physiological Integrity/Reduction of
acid from excessive vomiting. Answer 2 is incorrect because
Risk Potential/Potential for Complications from Surgical
a complication of diabetes is diabetic ketoacidosis, not meta-
Procedures and Health Alterations
bolic alkalosis. Answer 3 is incorrect because this condition
results in metabolic acidosis due to build-up of uric acid. 28. CORRECT ANSWER: 2. Answer 1 is incorrect because,
Answer 4 is correct because children with bulimia nervosa although decreasing edema is a sign of improvement, this
vomit frequently, resulting in a loss of stomach acid. is not the earliest sign. Answer 2 is correct because the
Answer 5 is incorrect because aspirin (acetylsalicylic acid) earliest sign that a child with nephrotic syndrome is
ingestion results in metabolic acidosis due to the large intake improving is an increase in urine output. Answer 3 is
of acid. incorrect because a childs appetite should improve as the
TEST-TAKING TIP: Recall that metabolic alkalosis is caused nephrotic syndrome resolves, but childrens appetites can
by excess loss of acid or excess intake of base. vary for a number of reasons. This sign would not necessari-
Content Area: Child Health, Acid-Base; Integrated Process: ly signify that the child is or is not getting better. Answer 4
Nursing Process, Analysis; Cognitive Level: Analysis; is incorrect because resolution of anemia would be a later
Client Need/Subneed: Physiological Integrity/Physiological sign of improvement.
Adaptation/Fluid and Electrolyte Imbalances TEST-TAKING TIP: Note the word earliest in the stem of the
question. To arrive at the best answer, order the possible correct
26. CORRECT ANSWER: 4. Answer 1 is incorrect because,

ANSWERS
responses from early to late signs of improvement.
although the child may experience discomfort, the effects of
Content Area: Child Health, Genitourinary; Integrated Process:
lead on the central nervous system have a higher priority.
Nursing Process, Evaluation; Cognitive Level: Analysis; Client
Answer 2 is incorrect because, although the child may exhibit
Need/Subneed: Physiological Integrity/Pharmacological and
the symptom of pica, nutritional effects of lead poisoning are
Parenteral Therapies/Expected Effects/Outcomes
of a lesser priority than neurologic effects. Answer 3 is incor-
rect because, although the child may experience pain with 29. CORRECT ANSWER: 2. Answer 1 is incorrect because,
injections of the chelating agent, the effects of lead on the although proper nutrition does help prevent infection, regular
central nervous system have a higher priority. Answer 4 is cor- and consistent pulmonary toilet has a greater preventive effect.
rect because the priority nursing diagnosis for this child is Answer 2 is correct because chest physiotherapy (CPT) is
alteration in neurologic functioning due to the effects of essential to help loosen sticky respiratory secretions and
lead on the central nervous system. facilitate sputum removal in the child with cystic fibrosis.
TEST-TAKING TIP: The question asks for the highest priority Failure to implement this treatment would create a ready
nursing diagnosis. Only one diagnosis leads to death, so it ranks environment for pulmonary infection. Answer 3 is incorrect
as the top priority. because increased work of breathing would most likely occur
Content Area: Child Health, Poisoning; Integrated Process: after an infection has already begun. Answer 4 is incorrect
Nursing Process, Assessment; Cognitive Level: Application; because, although proper hydration helps loosen respiratory
Client Need/Subneed: Safe and Effective Care Environment/ secretions, regular and consistent pulmonary toilet has a greater
Management of Care/Establishing Priorities preventive effect.
TEST-TAKING TIP: Recall that, in a child with cystic fibrosis,
27. CORRECT ANSWER: 3. Answer 1 is incorrect because, the respiratory tract is filled with thick, tenacious secretions.
although prevention of infection is a top priority, trauma
The best answer focuses on the loosening and removal of the
to the sac could permanently affect the function of the
secretions through CPT.
lower extremities while infection may be treated with
Content Area: Child Health, Respiratory; Integrated Process:
antibiotics. Answer 2 is incorrect because the rationale for
Nursing Process, Implementation; Cognitive Level: Analysis;
placing the infant in the prone position is not to promote
Client Need/Subneed: Physiological Integrity/Reduction of
increased circulation. Answer 3 is correct because the most
Risk Potential/System Specific Assessments
2164_Ch05_249-346 29/03/12 12:26 PM Page 332

332 chapter 5 Health Promotion and Maintenance

30. CORRECT ANSWER: 3. Answer 1 is incorrect because, expansion through upright positioning, but the effect will be
although this is the proper place to post the Poison Control minimal if the childs nares are blocked with thick secretions.
Center phone number, having the number does not prevent TEST-TAKING TIP: Pay attention to both the childs symptoms
ingestion from taking place. Answer 2 is incorrect because and physical development in order to select the correct response.
children may come into contact with poisonous substances The symptom of thick nasal secretions must be addressed in
in the garage just as easily as they do in the home. Answer 3 terms of the infants unique respiratory pattern of nose breath-
is correct because the parents should bend down and view ing. Only one response focuses on clearing the airway in order
the home from the childs eye level to better examine for the other interventions to be more successful.
potential access to poisonous substances. This includes Content Area: Child Health, Respiratory; Integrated Process:
checking all storage areas inside and outside the home Nursing Process, Implementation; Cognitive Level: Application;
that are easily accessible and those that may be reached Client Need/Subneed: Physiological Integrity/Reduction of Risk
by children when climbing. Answer 4 is incorrect because Potential/Potential for Alterations in Body Systems
simply instructing children to stay away from poisonous 33. CORRECT ANSWERS: 2, 4, 5. Answer 1 is incorrect because
substances will not ensure they follow these instructions and this neonate has caput succedaneum, a collection of fluid and not
might in fact increase the curiosity of children. blood. Cephalohematoma is a collection of blood under the skull
TEST-TAKING TIP: The best way to prevent ingestions is to bones and does not cross the suture lines. Answer 2 is correct
keep harmful substances out of the reach of children. Choose the because caput succedaneum is an edematous swelling that
option that has the greatest likelihood of accomplishing this goal. overlies the periosteum. Answer 3 is incorrect because caput suc-
Content Area: Child Health, Poisoning; Integrated Process:
cedaneum is a collection of fluid, not blood; therefore, there is no
Nursing Process, Implementation; Cognitive Level: Application; increased chance of hyperbilirubinemia that is associated with
Client Need/Subneed: Safe and Effective Care Environment/
cephalohematoma. Answer 4 is correct because the only man-
Safety and Infection Control/Home Safety agement is observation. No treatment is needed for caput suc-
31. CORRECT ANSWER: 4. Answer 1 is incorrect because cedaneum. Answer 5 is correct because caput succedaneum is
capped electrical outlets are less accessible to children and the result of pressure on the fetal head before delivery.
provide some degree of electrocution prevention. Answer 2 is TEST-TAKING TIP: Remember that caput succedaneum cross-
incorrect because the presence of a television in the childs es the suture lines and cephalohematoma does not. Answer 1
bedroom is not a safety hazard. Answer 3 is incorrect because, relates to Answer 3; therefore, you can eliminate both, as they
although having a swimming pool introduces the risk of both relate to collection of blood in cephalohematoma. This
drowning, the risk of burn injury from the pot handle is a infant has caput succedaneum.
ANSWERS

more immediate danger. Answer 4 is correct because toddlers Content Area: Child Health, Newborn; Integrated Process:
like to reach for objects. Having pot handles turned toward Nursing Process, Implementation; Cognitive Level: Application;
the front of the stove creates the potential for the child to Client Need/Subneed: Health Promotion and Maintenance/
pull the pot and its contents onto the child, causing a Ante/Intra/Postpartum and Newborn Care
severe burn injury. The parents should be instructed to turn 34. CORRECT ANSWER: 2. Answer 1 is incorrect because the
handles toward the back of the stove and consider placing a nurse should first assess the likely causes of the symptoms
safety guard at the front of the stove. before calling the surgeon. Answer 2 is correct because the
TEST-TAKING TIP: Look for the option in closest proximity to child is experiencing symptoms of autonomic dysreflexia,
the child, then consider the potential severity of the injury. The an excessive stimulation of the sympathetic nervous system
pot handle is readily accessible to the child and can lead to that is a potential complication of spinal cord surgery.
severe burns. Since bladder distention can lead to this problem, the nurse
Content Area: Child Health, Growth and Development;
should first assess the urinary catheter for obstruction or
Integrated Process: Nursing Process, Assessment; Cognitive
malfunction. Answer 3 is incorrect because the headache is a
Level: Analysis; Client Need/Subneed: Safe and Effective Care
symptom of the high blood pressure caused by the autonomic
Environment/Safety and Infection Control/Home Safety dysreflexia, and administering pain medication will not resolve
32. CORRECT ANSWER: 1. Answer 1 is correct because the headache. Answer 4 is incorrect because, although a neuro-
infants are obligatory nose breathers. A nurse should logical assessment is an important priority of care, the nurse
attempt to keep nasal passages open through frequent should first investigate the potential cause of the symptoms.
suctioning with a nasal olive. Answer 2 is incorrect because, TEST-TAKING TIP: When deciding between the option to
although supplemental oxygen may be necessary, its delivery phone the physician or perform additional assessments, think
via nasal cannula will be ineffective in an infant whose nares about what the nurse might find with the assessment that has
are blocked with thick secretions. Answer 3 is incorrect because the potential to resolve the problem or provide the physician
monitoring oxygen saturations does nothing to improve with additional, necessary information. In this case, the nurse
oxygenation. Rather, the nurse must intervene by suctioning should suspect bladder distention, investigate urinary catheter
in order to improve saturation levels. Answer 4 is incorrect functioning, and fix any malfunction as a way to resolve the
because placing the child in an infant seat does promote lung problem without first contacting the physician.
2164_Ch05_249-346 29/03/12 12:26 PM Page 333

Answers/Rationales/Tips 333
Content Area: Child Health, Musculoskeletal; Integrated Process: TEST-TAKING TIP: Recall that the gauge of the needle gets
Nursing Process, Implementation; Cognitive Level: Application; larger as the number gets smaller (i.e., an 18 G is larger than a
Client Need/Subneed: Physiological Integrity/Reduction of Risk 25 G). The best answer contains both the smallest gauge and
Potential/Potential for Complications from Surgical Procedures the shortest length.
and Health Alterations Content Area: Child Health, Medication Administration;
Integrated Process: Nursing Process, Implementation; Cognitive
35. CORRECT ANSWERS: 1, 2, 4, 5. Answer 1 is correct Level: Application; Client Need/Subneed: Physiological Integrity/
because the child with acute glomerulonephritis has
Pharmacological and Parenteral Therapies/Medication
large amounts of red blood cells in the urine due to
Administration
ruptured glomerular capillaries. Answer 2 is correct
because pallor is a symptom of acute glomerulonephritis 38. CORRECT ANSWER: 4. Answer 1 is incorrect because
as a result of anemia. Answer 3 is incorrect because urine the iron supplement is best absorbed between meals.
specific gravity increases with the fluid retention of acute Additionally, a bowl of cereal is likely to contain milk,
glomerulonephritis. Answer 4 is correct because the child which would impair the absorption of iron. Answer 2 is
with acute glomerulonephritis gains weight due to fluid incorrect because iron supplementation is more likely to
retention. Answer 5 is correct because the child with acute cause dark-colored stools. Answer 3 is incorrect because the
glomerulonephritis may have a headache as a result of medication is best administered at the back of the mouth to
hypertension caused by hypervolemia. Answer 6 is incor- avoid staining the childs teeth. Answer 4 is correct since
rect because proteinuria is mild, not massive for the child iron is best absorbed in the presence of vitamin C. The
with acute glomerulonephritis. Massive proteinuria is a liquid iron supplement may be mixed with fruit juices
symptom of nephrotic syndrome. such as orange juice to make the medication more palat-
TEST-TAKING TIP: Look for answers that go together. able while also increasing the absorption.
Hematuria (Answer 1) results in pallor (Answer 2); weight TEST-TAKING TIP: Remember that a frequent recommenda-
gain (Answer 4) from fluid retention goes with headache from tion is to administer iron supplements with orange juice. Fruit
hypertension (Answer 5). juices contain vitamin C, or ascorbic acid, which makes
Content Area: Child Health, Genitourinary; Integrated Process: Answer 4 the best answer.
Nursing Process, Assessment; Cognitive Level: Analysis; Content Area: Child Health, Medication Administration;
Client Need/Subneed: Physiological Integrity/Physiological Integrated Process: Teaching and Learning; Cognitive Level:
Adaptation/Alterations in Body Systems Application; Client Need/Subneed: Physiological Integrity/
Pharmacological and Parenteral Therapies/Medication
36. CORRECT ANSWER: 2. Answer 1 is incorrect because

ANSWERS
Administration
vomiting and diarrhea are not part of the diagnostic criteria
for rheumatic fever and are symptoms common to many 39. CORRECT ANSWER: 3. Answer 1 is incorrect because
other illnesses. Answer 2 is correct because symptoms of an absolute neutrophil count (ANC) of 1200 is just below
rheumatic fever include muscle weakness and arthralgia. the normal range of 1500 or greater. This slightly low value
Answer 3 is incorrect because together these are symptoms of is not unexpected for a child with cancer who is receiving
Kawasaki disease, not rheumatic fever. Answer 4 is incorrect chemotherapy. An ANC of less than 500 should be reported
because children with rheumatic fever are likely to experience since this would meet the criteria for neutropenia. Answer 2 is
tachycardia. In addition, hypotension is not part of the diag- incorrect because this is within the range of normal (150,000
nostic criteria for rheumatic fever. to 400,000) for the platelet count. If the platelet count fell
TEST-TAKING TIP: The question contains the word below 100,000, the physician should be notified. Answer 3 is
rheumatic, which is related to joints. Only one answer correct because a positive urine dipstick for the presence of
option contains a symptom associated with joints (arthralgia), red blood cells could indicate hemorrhagic cystitis, a com-
which should help you narrow your choice. plication of chemotherapy agents, including cyclophos-
Content Area: Child Health, Infectious Disease; Integrated phamide and ifosfamide. This finding should be communi-
Process: Nursing Process, Assessment; Cognitive Level: cated immediately to the physician. Answer 4 is incorrect
Application; Client Need/Subneed: Physiological Integrity/ because a white blood cell (WBC) count of 4500 is just below
Physiological Adaptation/Pathophysiology the range of normal (5000 to 10,000), which is expected for a
child receiving chemotherapy.
37. CORRECT ANSWER: 3. Answer 1 is incorrect since an TEST-TAKING TIP: When given laboratory values, consider both
18-gauge (G) needle would be too large a diameter for a
the range of normal values as well as expected values for given
newborn infant. Answer 2 is incorrect since a 21-gauge,
medical conditions. Pay attention to those values that are abnor-
1-inch needle would be too large for a newborn infant.
mal for the child even with the stated diagnosis and treatment.
Answer 3 is correct because the most appropriate needle
Content Area: Child Health, Oncology; Integrated Process:
to select for use in administering IM injection to a
Nursing Process, Evaluation; Cognitive Level: Analysis;
1-week-old infant would be a 25 gauge, 5/8 inch long.
Client Need/Subneed: Physiological Integrity/Reduction of
Answer 4 is incorrect since a 11/2-inch needle would be far
Risk Potential/Laboratory Values
too long for a newborn infant and also for most children.
2164_Ch05_249-346 29/03/12 12:26 PM Page 334

334 chapter 5 Health Promotion and Maintenance

40. CORRECT ANSWER: 1. Answer 1 is correct because, dose was changed to achieve a therapeutic blood level.
prior to digoxin administration, all children should Answer 4 is incorrect because, although gentamicin may be
have their apical pulse assessed by auscultation at the nephrotoxic, the nurse should first discover if the dose was
apex of the heart. The heartbeats are best heard at the changed to achieve a therapeutic blood level.
apex of the heart, making this the best location for TEST-TAKING TIP: Note that two of the options (Answers 3
assessing the heart rate. Answer 2 is incorrect because and 4) require consultation with the physician, so they are
this location is best for assessing the brachial pulse. This is subsumed under the contact the physician response. Then, to
not the best location for assessing heart rate because the decide between the first two answers, consider that the nurse
brachial artery may be difficult to palpate, leading to inaccu- should review the assessment data first to determine whether a
rate assessment. Answer 3 is incorrect because this location call to the physician is even necessary.
is best for assessing the radial pulse. This is not the best Content Area: Child Health, Medication Administration;
location for assessing heart rate because the radial artery Integrated Process: Nursing Process, Implementation; Cognitive
may be difficult to palpate, leading to inaccurate assessment. Level: Application; Client Need/Subneed: Physiological Integrity/
Answer 4 is incorrect because heart beats are best heard at Pharmacological and Parenteral Therapies/Medication
the apex of the heart, not the base of the heart, making the Administration
apex the best location for auscultation and assessment of 43. CORRECT ANSWER: 2. Answer 1 is incorrect
heart rate. because hypoglycemia is more likely to occur just prior
TEST-TAKING TIP: The correct response does not vary with to meals. Answer 2 is correct since the body becomes
the childs age. A nurse must assess the apical heart rate, not more sensitive to insulin with physical activity, and it
simply the pulse rate, prior to digoxin administration. may be necessary to reduce the childs insulin dosage
Content Area: Child Health, Cardiovascular; Integrated Process:
with sports participation. Answer 3 is incorrect because
Nursing Process, Assessment; Cognitive Level: Application; self-administration of insulin depends on developmental
Client Need/Subneed: Physiological Integrity/Pharmacological
readiness rather than age. Children as young as early school
and Parenteral Therapies/Medication Administration age may be capable of giving injections with caregiver
41. CORRECT ANSWER: 1. Answer 1 is correct supervision. Answer 4 is incorrect because insulin is not
because this statement by the mother indicates more contraindicated during febrile illnesses. The dosage of
teaching is needed, since crushing the capsule would insulin may increase, decrease, or stay the same depending
destroy the enteric coating on the enzyme beads, on the childs needs.
leading to their destruction in the acid environment TEST-TAKING TIP: Be wary of answer options that contain
ANSWERS

of the stomach. Answer 2 is incorrect because this extreme terms such as never or always.
statement demonstrates appropriate knowledge. Answer 3 Content Area: Child Health, Endocrine; Integrated Process:
is incorrect because this statement demonstrates appropriate Teaching and Learning; Cognitive Level: Application;
knowledge. Answer 4 is incorrect because this statement Client Need/Subneed: Physiological Integrity/Physiological
demonstrates appropriate knowledge. Pancreatic enzymes Adaptation/Illness Management
must be administered within 30 minutes of the ingestion 44. CORRECT ANSWER: 4. Answer 1 is incorrect because
of food. the LVN/LPN should have experience caring for clients in
TEST-TAKING TIP: Make sure to look for an incorrect state- traction for bone fractures. Answer 2 is incorrect because
ment when the stem says further teaching is needed. the LVN/LPN has experience caring for clients with alter-
Content Area: Child Health, Respiratory; Integrated Process:
ations in bone health. Legg-Calv-Perthes disease involves
Nursing Process, Evaluation; Cognitive Level: Analysis; necrosis of the femoral head. Answer 3 is incorrect because
Client Need/Subneed: Physiological Integrity/Pharmacological
the LVN/LPN floated from the orthopedic unit may have
and Parenteral Therapies/Medication Administration experience caring for clients with osteogenesis imperfecta
42. CORRECT ANSWER: 1. Answer 1 is correct because since this is a chronic condition. Answer 4 is correct
the nurse should first check the childs recent laboratory because the child with osteosarcoma is receiving
work to see if a gentamicin level has been done. The chemotherapy, which requires continuous monitoring
physician may have increased the dose of IV gentamicin for complications. This child would not be an appropri-
above the safe dose range if the childs gentamicin level ate client for the LVN/LPN due to the need for frequent
fell below that which is effective. The safe dose range is assessment by a registered nurse.
the starting point for this medication, but the dose is TEST-TAKING TIP: Note the key word avoid and look for
then increased or decreased to achieve therapeutic blood the incorrect answer as the best choice. When determining the
levels. Answer 2 is incorrect because the nurse should first proper assignment for this LPN/LVN, consider the scope of
discover if the dose was changed to achieve a therapeutic practice. In this case, only the child receiving chemotherapy
blood level. Answer 3 is incorrect because, although gentam- would require too much expertise and too high a level of
icin may be ototoxic, the nurse should first discover if the assessment.
2164_Ch05_249-346 29/03/12 12:26 PM Page 335

Answers/Rationales/Tips 335
Content Area: Child Health, Musculoskeletal; Integrated trachea above the suprasternal notch. Answer 3 is incorrect
Process: Nursing Process, Analysis; Cognitive Level: Analysis; because placing the stethoscope beneath the scapula will
Client Need/Subneed: Safe and Effective Care Environment/ produce bronchovesicular breath sounds. Answer 4 is incor-
Management of Care/Delegation rect because placing the stethoscope at the base of the lungs
will produce vesicular breath sounds.
45. CORRECT ANSWER: 4. Answer 1 is incorrect because TEST-TAKING TIP: The word bronchial is derived from the
a capillary refill time of 2 seconds is a normal finding.
word bronchus. The best place to auscultate breath sounds is
Extended refill time would be a concern. Answer 2 is
over the main bronchus, which is located directly underneath
incorrect because transient mottling of the skin is likely
the sternum.
caused by a decrease in environmental temperature or as a
Content Area: Child Health, Respiratory; Integrated Process:
result of stress. Sustained mottling of the skin would be
Nursing Process, Assessment; Cognitive Level: Application;
cause for concern. Answer 3 is incorrect because irregular,
Client Need/Subneed: Health Promotion and Maintenance/
periodic breaths are normal findings in an infant. Apneic
Techniques of Physical Assessment
spells or respirations that never sustain any pattern would
be a concern. Answer 4 is correct because the newborn 48. CORRECT ANSWER: 3. Answer 1 is incorrect because
should have a positive Babinski, or plantar, reflex. This children should ride in the back seat until they are 12 years
reflex occurs when the toes extend in response to the of age or older. Answer 2 is incorrect because infants should
stroking of the sole of the foot. A negative finding be rear facing rather than forward facing. Answer 3 is cor-
should occur in older infants and adults and is noted rect because infants should face the rear of the vehicle
when the toes demonstrate a flexor response. until they weigh 20 pounds, from birth to as close to the
TEST-TAKING TIP: The key word in the stem of the question first birthday as possible. Answer 4 is incorrect because air
is newborn. Look for the assessment finding that is abnormal bags should be turned off where children are sitting, as air
for a newborn. bag deployment could strike a child in the face, causing
Content Area: Child Health, Newborn; Integrated Process: serious injury.
Nursing Process, Assessment; Cognitive Level: Analysis; Client TEST-TAKING TIP: When deciding whether a child should
Need/Subneed: Health Promotion and Maintenance/Health face the front or the rear of the car, recall that an infants neck
Screening muscles are weak. The safest place for the infant is facing the
rear since this would prevent the childs head from flexing for-
46. CORRECT ANSWER: 3. Answer 1 is incorrect ward in a head-on collision.
because, although children with mumps have a fever,
Content Area: Child Health, Safety; Integrated Process:
the other symptoms are not present. Answer 2 is incorrect

ANSWERS
Nursing Process, Implementation; Cognitive Level: Application;
because children with measles have a fever and rash, but
Client Need/Subneed: Health Promotion and Maintenance/
the rash spreads from the face to the rest of the body,
Health Promotion Programs
becoming more prominent in the upper body. The other
symptoms are not present. Answer 3 is correct because 49. CORRECT ANSWER: 1. Answer 1 is correct because
these symptoms are classic for scarlet fever. The child an audible, low-pitched, clunk during the Ortolani
develops a high fever, abdominal pain, flushed cheeks, test is caused by the sound of the femur head exiting
and strawberry tongue, as well as a generalized pinpoint or entering the acetabulum, indicating hip dislocation.
red rash that is more concentrated in the axillae and Answer 2 is incorrect because symmetrical gluteal and
groin. Answer 4 is incorrect because varicella begins with a thigh folds are negative signs of hip dislocation. Asymmetry
macular rash, progressing to papular and then vesicular would indicate dislocation of the hip. Answer 3 is incorrect
lesions. The child also has a fever but the other symptoms because a positive Barlow test would indicate hip dislocation.
are not present. Answer 4 is incorrect because asymmetry of the knees while
TEST-TAKING TIP: Recall that two pediatric infections have the infant is in the supine position would indicate hip
the symptom of a strawberry-like tongue: Kawasaki disease and dislocation.
scarlet fever. TEST-TAKING TIP: Note the key words abnormal findings.
Content Area: Child Health, Infectious Disease; Integrated Recall that, when investigating hip dislocation, symmetrical
Process: Nursing Process, Assessment; Cognitive Level: findings are normal, which eliminates Answers 2 and 4. Next,
Application; Client Need/Subneed: Physiological Integrity/ recall that positive Ortolani and Barlow tests are pathologic for
Physiological Adaptation/Pathophysiology developmental dysplasia of the hip, which makes Answer 1 the
only correct response.
47. CORRECT ANSWER: 2. Answer 1 is incorrect because Content Area: Child Health, Musculoskeletal; Integrated
placing the stethoscope over the sternum will produce
Process: Nursing Process, Assessment; Cognitive Level:
bronchovesicular breath sounds. Answer 2 is correct because
Analysis; Client Need/Subneed: Physiological Integrity/
the nurse is best able to auscultate bronchial breath
Reduction of Risk Potential/System Specific Assessments
sounds in an infant by placing the stethoscope over the
2164_Ch05_249-346 29/03/12 12:26 PM Page 336

336 chapter 5 Health Promotion and Maintenance

50. CORRECT ANSWER: 3.2. To solve this problem, first Content Area: Child Health, Neurological; Integrated Process:
multiply the ordered 200,000 units per kilogram by the Nursing Process, Implementation; Cognitive Level: Application;
childs weight of 16 kg to get a total of 3,200,000 units per Client Need/Subneed: Physiological Integrity/Basic Care and
day. Next, divide the total daily units of 3,200,000 by 4 Comfort/Elimination
(since the drug is administered every 6 hours) to get a sin- 53. CORRECT ANSWER: 4. Answer 1 is incorrect because the
gle dose of 800,000 units. Finally, take the single dose of nurse should first assess the suction. Although changing posi-
800,000 units and divide by the 250,000 units per mL to tion may help move the tip of the tube from a potential block-
get a total of 3.2 mL. age in the stomach, it is more effective to aspirate and irrigate
TEST-TAKING TIP: Approach pharmacology math questions in a the tube in this situation. Answer 2 is incorrect because the
stepwise fashion, paying attention to the number of doses given in nurse should first assess the situation and attempt to resolve the
a 24-hour day rather than the hours between doses (in this exam- issue before consulting the surgeon. Answer 3 is incorrect
ple, divide by 4 doses per day, not by 6 hours between doses). because the nurse should first assess the suction. If the suction
Content Area: Child Health, Medication Administration;
is working properly, the nurse should check placement before
Integrated Process: Nursing Process, Implementation; Cognitive
instilling anything into the tube. Answer 4 is correct because
Level: Application; Client Need/Subneed: Physiological Integrity/
the most likely cause of poor drainage is ineffective suction.
Pharmacological and Parenteral Therapies/Dosage Calculation The suction tubing may have become dislodged or the set-
51. CORRECT ANSWER: 1. Answer 1 is correct because the tings may have been altered. A nurse should inspect this
child with acute epiglottitis is likely to be restless and first, then continue to problem-solve if needed.
agitated due to the progressive airway obstruction. The TEST-TAKING TIP: Look for nursing actions that are the least
child should be allowed to maintain a position of com- invasive. Checking the suction settings can be done quickly and
fort (e.g., sitting upright) to avoid further agitation and easily without disturbing the child.
impaired oxygenation. Answer 2 is incorrect because plac- Content Area: Child Health, Gastrointestinal; Integrated
ing any item in the childs mouth is contraindicated with a Process: Nursing Process, Evaluation; Cognitive Level:
diagnosis of acute epiglottitis because this may lead to com- Application; Client Need/Subneed: Physiological Integrity/
plete airway obstruction. Answer 3 is incorrect because the Reduction of Risk Potential/Potential for Complications of
inspection of the childs nares is likely to agitate the child, Diagnostic Tests/Treatments/Procedures
and is also unnecessary since the primary area of swelling is 54. CORRECT ANSWER: 3. Answer 1 is incorrect because
in the epiglottis and not the nares. Answer 4 is incorrect teenagers have heart rates that generally range from 55 to
because the child is maintaining oxygenation at this time 90. However, children who are athletic may have even lower
ANSWERS

and the application of a mask on a young child is likely to heart rates, especially at rest. Answer 2 is incorrect because,
create anxiety, which will decrease oxygenation. although an infants heart rate ranges from 120 to 160, the
TEST-TAKING TIP: Children with epiglottitis are at risk for rate can near 200 with vigorous crying. If the infant were at
complete airway obstruction. Note that all but one of the rest, a rate of 190 would be suspicious. Answer 3 is correct
answer options requires a nursing intervention near the childs because the normal heart rate in a 2-year-old child is
nose or mouth, which may cause distress. Select the option that from 80 to 120 beats per minute. Even though the child
will not agitate the child further. is active, this heart rate is quite high and should be inves-
Content Area: Child Health, Respiratory; Integrated Process:
tigated further. Answer 4 is incorrect because this is within
Nursing Process, Implementation; Cognitive Level: Application; the range of normal (70 to 100) for a 5-year-old child.
Client Need/Subneed: Physiological Integrity/Reduction of Risk
TEST-TAKING TIP: Pay particular attention to the childs
Potential/Potential for Alterations in Body Systems activity level when evaluating heart rates. An elevated heart
52. CORRECT ANSWERS: 1, 3, 4. Answer 1 is correct as the rate during normal or restful activity is more of a concern than
first step because the child should wash hands prior to the a decreased heart rate at rest or an elevated heart rate during
procedure to prevent infection. Answer 2 is incorrect because vigorous activity.
children with spina bifida are at increased risk for latex allergy Content Area: Child Health, Cardiovascular; Integrated Process:
and should not utilize latex catheters. Answer 3 is correct as Nursing Process, Assessment; Cognitive Level: Analysis;
the second step because lubricating jelly should be applied. Client Need/Subneed: Physiological Integrity/Physiological
Answer 4 is correct because, after insertion and removal of Adaptation/Alterations in Body Systems
the catheter, the third (and last) step is to cleanse the 55. CORRECT ANSWER: 1. Answer 1 is correct because the
catheter for storage. Answer 5 is incorrect because the child nurses priority is to fulfill the legal duties of a mandated
should cleanse the perineum with soap and water, not reporter by contacting the agency responsible for taking
Betadine. reports of suspected child abuse. Answer 2 is incorrect
TEST-TAKING TIP: Since self-catheterization is done at home, because the nurse should not discuss the allegations with the
remember that the procedure is clean, not sterile. Start and end childs family members as this may interfere with an investiga-
with washing (Answers 1 and 4). The child may reuse tion. Answer 3 is incorrect because further questioning by the
catheters as long as they are properly cleaned and stored.
2164_Ch05_249-346 29/03/12 12:26 PM Page 337

Answers/Rationales/Tips 337
nurse may harm the investigation. Questioning victims of sus- achieved with around-the-clock dosing. Without adequate
pected child abuse should be left to professionals skilled in this pain control, the child may cry, putting stress on the surgi-
procedure. Answer 4 is incorrect because, although the nurse cal site. Answer 5 is incorrect because frequent oral suctioning
should provide a safe environment for the child, the nurse puts stress on the tonsillectomy site and causes bleeding.
should not continue the discussion without the presence of the TEST-TAKING TIP: Consider that the surgical area for tonsillec-
proper authorities. tomy is in the childs throat. Potential hemorrhage in this area
TEST-TAKING TIP: Look to eliminate answers that contain could lead to serious respiratory compromise; therefore, priority
only partially correct responses. Answer 4 is only half right and nursing actions should be to monitor and protect the airway.
therefore can be eliminated. Of the remaining options, only Content Area: Child Health, Respiratory; Integrated Process:
Answer 1 contains an action that the nurse must always do Nursing Process, Planning; Cognitive Level: Analysis; Client
when suspecting child abuse. Need/Subneed: Physiological Integrity/Reduction of Risk
Content Area: Child Health, Abuse; Integrated Process: Potential/Potential for Complications from Surgical Procedures
Nursing Process, Implementation; Cognitive Level: Application; and Health Alterations
Client Need/Subneed: Safe and Effective Care Environment/
58. CORRECT ANSWER: 2. Answer 1 is incorrect because
Management of Care/Legal Rights and Responsibilities
lung crackles indicate fluid in the lungs, which occurs after the
56. CORRECT ANSWER: 2. Answer 1 is incorrect because, increase in heart rate is no longer effective in compensating for
although this child is likely to have mobility problems, pairing poor cardiac functioning. Answer 2 is correct because the body
a teenager with a 9-year-old may be problematic as they may tries to compensate for a failing heart by first increasing the
not share similar interests. Answer 2 is correct because this heart rate as a way to increase circulating blood volume.
child is close in age and development and is likely to be Answer 3 is incorrect because weight gain occurs more slowly
immobilized in the injured leg due to a cast and/or trac- over time as the kidneys attempt to retain fluid in response to
tion. Since the child with myelodysplasia is likely to have the poor cardiac output. Answer 4 is incorrect because general-
impaired mobility in the infected foot or even complete ized edema occurs more slowly over time as the kidneys attempt
paralysis of both lower extremities, these children share to retain fluid in response to the poor cardiac output.
similar limitations and the nursing staff can encourage TEST-TAKING TIP: Note how three of the answers are similar
them to play video games or participate in suitable activi- in that they describe fluid retained in the wrong area. Only one
ties. Answer 3 is incorrect because, although the child is in the answer is an effort to prevent fluid retention, and is therefore
same age range, the physical needs are not alike. The child sta- the earliest sign of failure.
tus post-appendectomy may be NPO and have a nasogastric Content Area: Child Health, Cardiovascular; Integrated Process:

ANSWERS
tube for a few days but will then be prodded to ambulate and Nursing Process, Assessment; Cognitive Level: Application;
increase activity. Neither of these situations matches well with Client Need/Subneed: Physiological Integrity/Physiological
the child with myelodysplasia. Answer 4 is incorrect because a Adaptation/Alterations in Body Systems
child with bacterial meningitis is contagious until after 24
59. CORRECT ANSWER: 1. Answer 1 is correct because the
hours of antibiotic therapy. Also, a 6-year-old child is not the
primary means of lead exposure in children results from
best playmate for a 9-year-old child.
ingestion. The presence of lead-based paint should be suspect-
TEST-TAKING TIP: To select the best roommate for a pediatric
ed in homes built prior to the late 1970s when these paints
client, first eliminate the children who do not share develop-
were discontinued. Children can ingest paint chips or dust by
mental needs. Next, eliminate those who are potentially infec-
chewing on contaminated surfaces such as windowsills, and
tious to the child in question. Finally, match the child with a
they can become exposed by playing in contaminated soil.
roommate who has similar care needs.
Answer 2 is incorrect because this is a means of exposure, but not
Content Area: Child Health, Hospitalization; Integrated
the most common means. Answer 3 is incorrect because leaded
Process: Nursing Process, Planning; Cognitive Level: Analysis;
gasoline is not permitted for on-road vehicles. Answer 4 is incor-
Client Need/Subneed: Safe and Effective Care Environment/
rect since pencil lead does not contain lead, but graphite, and is
Management of Care/Concepts of Management
not a source of lead poisoning in children.
57. CORRECT ANSWERS: 1, 3, 4. Answer 1 is correct TEST-TAKING TIP: To determine the most likely source of
because, following tonsillectomy, the child may begin oral exposure, consider that an ingested poison is absorbed most eas-
intake after surgery, beginning with ice chips and progress- ily. Two answers describe ingestion, but one answer can be gen-
ing as tolerated to avoid vomiting, which could injure the eralized to a greater population of children (many children live
surgical site. Answer 2 is incorrect because coughing should be in old homes, while only some would chew on pencils). Even if
discouraged as this puts stress on the tonsillectomy site and lead is possibly contained in pencils, not enough children would
causes bleeding. Answer 3 is correct because increased bleed- be exposed to make that the primary route of exposure.
ing times put the child at risk for hemorrhage at the tonsil- Content Area: Child Health, Poisoning; Integrated Process:
lectomy site, which could compromise the airway. Answer 4 Nursing Process, Analysis; Cognitive Level: Analysis; Client
is correct because the nurse should expect that the child Need/Subneed: Safe and Effective Care Environment/Safety
will have pain from the tonsillectomy. Pain control is best and Infection Control/Home Safety
2164_Ch05_249-346 29/03/12 12:26 PM Page 338

338 chapter 5 Health Promotion and Maintenance

60. CORRECT ANSWER: 2. Answer 1 is incorrect because the Demerol in children with sickle cell anemia is contraindicated
meningitis vaccine is only effective against some strains, and the since it carries an increased risk of seizure activity.
vaccine is not a guarantee that the child will never become infect- TEST-TAKING TIP: When a question has only one correct
ed. Answer 2 is correct because meningitis is primarily spread response (instead of several good responses and one best
through contact with droplets that arise from the nasophar- response), look for the response that seems reasonable. A diet
ynx of a person who is infected. Teenagers should be taught to with healthy foods is reasonable and correct for this child.
not share food, drinks, or any other item that touches the Content Area: Child Health, Hematological; Integrated Process:
nose or mouth of another person. Answer 3 is incorrect because Nursing Process, Planning; Cognitive Level: Application;
children should not avoid group activities for fear of infection. Client Need/Subneed: Physiological Integrity/Reduction of
Team sports should not be avoided, but participants must have Risk Potential/Potential for Complications from Surgical
their own water bottles. Answer 4 is incorrect because prevention Procedures and Health Alterations
methods should be employed whether children are in crowds or 63. CORRECT ANSWER: 4. Answer 1 is incorrect because
in small groups. It is never recommended that children share whole grains contain gluten. Answer 2 is incorrect because cake
items touching the nose or mouth of other children. is made with wheat flour, which contains gluten. Answer 3 is
TEST-TAKING TIP: The primary focus of infection prevention incorrect because pizza dough is made with wheat flour, which
is to encourage behavior change. However, prevention methods contains gluten. Answer 4 is correct because beans and rice
should not expect individuals to make extreme and unrealistic are acceptable foods for a child with celiac disease, who
behavioral changes. Teenagers especially are not going to avoid requires a gluten-free diet.
group activities (team sports, crowds) since peer groups are TEST-TAKING TIP: Recall that any food item made with
important. The easiest behavior to change is to avoid sharing wheat flour is unacceptable for a child with celiac disease.
drinks and food. Choose the foods that do not contain flour.
Content Area: Child Health, Infectious Disease; Integrated
Content Area: Child Health, Gastrointestinal; Integrated
Process: Nursing Process, Planning; Cognitive Level:
Process: Nursing Process, Implementation; Cognitive Level:
Application; Client Need/Subneed: Safe and Effective Care Application; Client Need/Subneed: Physiological Integrity/Basic
Environment/Safety and Infection Control/Standard/ Care and Comfort/Nutrition and Oral Hydration
Transmission-based/Other Precautions
64. CORRECT ANSWER: 2. Answer 1 is incorrect because,
61. CORRECT ANSWER: 4. Answer 1 is incorrect because although the infant with severe dehydration may have
joint pain is often related to hemarthrosis, which should first decreased or absent urinary output, lethargy and decreased
be treated with immobilization and then later with passive, urine output alone may also be indicative of moderate
ANSWERS

not active, range of motion. Answer 2 is incorrect because dehydration. Answer 2 is correct because these symptoms
aspirin is contraindicated for the child with hemophilia as it describe a child with significantly diminished circulation
increases the risk of bleeding. Answer 3 is incorrect because as a result of dehydration. An infant with severe dehydra-
ice should be applied to injuries to promote vasoconstriction. tion has weak to absent pulses, poor skin turgor, and cool,
Warm compresses are not recommended because these lead discolored skin. Answer 3 is incorrect because these are
to vasodilation and potential bleeding. Answer 4 is correct symptoms of an infant with mild to moderate dehydration.
because a soft toothbrush will prevent trauma to the Answer 4 is incorrect because these are symptoms of an infant
childs gums (i.e., bleeding) while keeping the teeth clean. with moderate dehydration. The infant with severe dehydra-
TEST-TAKING TIP: Since the key concern in hemophilia is tion may be lethargic, may have parched mucous membranes,
bleeding, look for the answer that is least likely to cause harm by and may experience an abnormally irregular respiratory
increasing bleeding tendencies. Only Answer 4 prevents trauma. pattern.
Content Area: Child Health, Hematological; Integrated Process:
TEST-TAKING TIP: The stem of the question asks you to identify
Nursing Process, Implementation; Cognitive Level: Application; the most severe symptoms. It is helpful to rank the answer options
Client Need/Subneed: Physiological Integrity/Reduction of Risk
from best to worst in order to identify the most severe description.
Potential/Potential for Complications from Surgical Procedures Content Area: Child Health, Gastrointestinal; Integrated
and Health Alterations Process: Nursing Process, Analysis; Cognitive Level: Analysis;
62. CORRECT ANSWER: 1. Answer 1 is correct because Client Need/Subneed: Physiological Integrity/Physiological
these foods are high in fiber and folic acid. Fiber prevents Adaptation/Alterations in Body Systems
constipation, a potential side effect of pain medication, and 65. CORRECT ANSWER: 3. Answer 1 is incorrect because
folic acid is needed for healthy red blood cell production. the apical area is a better location to listen for mitral valve
Answer 2 is incorrect because children with sickle cell anemia
clicks. Answer 2 is incorrect because the lower left sternal
are at high risk for infection and should be properly immu- border is a better location to listen for the murmur of a
nized. Answer 3 is incorrect because the odds of the childs off- ventricular septal defect. Answer 3 is correct because the
spring having sickle cell anemia cannot be determined without pulmonic area, located at the upper left sternal border, is
knowing the genetic status of the person with whom the client directly over the pulmonary artery, making the murmur
conceives a child. Answer 4 is incorrect because the use of best heard at this location. Answer 4 is incorrect because the
2164_Ch05_249-346 29/03/12 12:26 PM Page 339

Answers/Rationales/Tips 339
upper right sternal border is a better location to hear aortic Cognitive Level:Comprehension; Client Need/Subneed:
valve clicks. Physiological Integrity/Pharmacological and Parenteral
TEST-TAKING TIP: Since the defect involves the pulmonary Therapies/Medication Administration
artery, visualize the hearts anatomy as it sits in the chest and
68. CORRECT ANSWER: 3. Answer 1 is incorrect because this
place the stethoscope closest to the pulmonary artery.
is proper positioning for the child receiving an optic medica-
Content Area: Child Health, Cardiovascular; Integrated Process:
tion. Answer 2 is incorrect because the child should be instruct-
Nursing Process, Assessment; Cognitive Level: Analysis; Client
ed to close the eyelids while moving the eyes around so as to
Need/Subneed: Physiological Integrity/Reduction of Risk
better distribute the medication. Answer 3 is correct because
Potential/System Specific Assessments
eyedrops should be administered when they are least likely
66. CORRECT ANSWER: 3. Answer 1 is incorrect because the to interfere with an activity that requires effective vision. A
nurse should speak in a normal tone of voice rather than a nurse should intervene and advise the student that the child
whisper. Children may interpret whispers as an indication that should eat lunch first. Answer 4 is incorrect because this is
death should not be talked about. Answer 2 is incorrect because proper technique for the administration of eyedrops.
the nurse should always interact with the child, even if the child TEST-TAKING TIP: Look for the action that is not okay and
is comatose or not verbally responsive, because children may be therefore needs an intervention.
able to hear the nurse even if they do not respond. Answer 3 is Content Area: Child Health, Medication Administration;
correct because many children use denial as a defense mech- Integrated Process: Nursing Process, Evaluation; Cognitive
anism in the face of their own death. A nurse should not Level: Application; Client Need/Subneed: Physiological
take away the childs defenses; rather, the nurse should be Integrity/Pharmacological and Parenteral Therapies/
honest when answering the childs questions while allowing Medication Administration
the child to accept death when ready. Answer 4 is incorrect
69. CORRECT ANSWERS: 2, 3, 5, 4. Answer 1 is incorrect
because a school-age child should be able to report pain to the
because the childs respiratory assessment did not reveal a
nurse. The parents may assist in the pain assessment since they
need for supplemental oxygen at this time. Routine use of
know their child best; deferring to the parents denies the child a
oxygen is not recommended for a child with sickle cell
voice in the given treatment.
disease because it will not reverse sickling or treat pain.
TEST-TAKING TIP: Look for the answer that supports a trust-
Answer 2 is correct because the nurse must first obtain
ing nurse-client relationship. Answers 1, 2, and 4 describe
an accurate weight before determining safe dosages of
interventions that prevent interaction with the child. Answer 3
ordered medications and IV fluids. Additionally, the
allows the child to express feelings, building trust between the
childs weight will be used to determine whether I&O

ANSWERS
nurse and child.
are meeting appropriate targets. Answer 3 is correct
Content Area: Child Health, End-of-Life Care; Integrated
because IV fluid administration is a priority treatment
Process: Nursing Process, Planning; Cognitive Level: Application;
for the child in sickle cell crisis. After the childs weight
Client Need/Subneed: Psychosocial Integrity/End of Life Care
is obtained, the nurse may safely administer the ordered
67. CORRECT ANSWER: 2. Answer 1 is incorrect because IV fluids, being sure to calculate that the amount and
mixing medications with food may not improve the taste. rate is appropriate. Answer 4 is correct because the nurse
Also, once the medication is mixed with food, there is typi- should monitor intake and output for adequacy since
cally more volume for the child to swallow since the child hydration status is an important part of this childs
must take all the food to get all the medication. Answer 2 is assessment. I&O monitoring should take place after the
correct because the child should be given a choice of child has been weighed and is started on IV fluids and
fluid chaser to wash the unpleasant taste out of the medications. Answer 5 is correct because the child needs
mouth following ingestion of the medication. The child effective pain management, yet this is a lower priority
is not given a choice of whether or not to take the med- than fluid administration. A teenager with a chronic
ication. Answer 3 is incorrect because it is best for the nurse painful condition is an excellent candidate for PCA
to elicit the cooperation of the child rather than forcing the (patient-controlled analgesia). Answer 6 is incorrect
medication into the mouth. Answer 4 is incorrect because because warm compresses, not cool compresses, should be
offering the child a toy does not guarantee that the child used to facilitate circulation in affected extremities.
will agree to take the medication. Additionally, if the nurse TEST-TAKING TIP: When placing options in sequential order,
gives a toy this time, the child will expect a similar reward consider whether assessments should be done before or after
with each dose of this medication. interventions. In child health, IV fluids and medications are
TEST-TAKING TIP: Toddlers need to be provided with real, based upon the childs weight; therefore, the weight should be
not false, choices. A nurse should calmly tell the child that the assessed before these treatments can be safely administered.
medication must be swallowed, then work with the child to Content Area: Child Health, Hematology; Integrated Process:
find the least offensive way to accomplish this task. Nursing Process, Analysis; Cognitive Level: Analysis;
Content Area: Child Health, Medication Administration; Client Need/Subneed: Physiological Integrity/Reduction of
Integrated Process: Nursing Process, Implementation; Risk Potential/Potential for Alterations in Body Systems
2164_Ch05_249-346 29/03/12 12:26 PM Page 340

340 chapter 5 Health Promotion and Maintenance

70. CORRECT ANSWER: 1. Answer 1 is correct because, neurosurgery. The antiemetic should be administered
although permethrin is an over-the-counter medication because vomiting needs to be prevented since it increases
for the treatment of pediculosis (head lice), repeated intracranial pressure (ICP). Answer 4 is correct because
doses may become toxic over time; parents should be determining the childs pain level should be part of the
cautioned against such treatments. A nurse should stress physical assessment. It is expected that the child may
that nonpharmacologic treatments such as nit removal have pain from this surgery. Answer 5 is incorrect because
may be more effective and pose less risk to the child. increasing the childs IV rate could lead to increased ICP
Answer 2 is incorrect because this is the recommended and should not be done without an order.
method for killing nits or lice contained in linen and cloth- TEST-TAKING TIP: The priority of care for children recover-
ing. Answer 3 is incorrect because this is recommended as a ing from neurosurgery is monitoring and preventing increased
way to prevent nits or lice from being transferred to the ICP. Select the actions that will not increase ICP.
parents during treatment of pediculosis. Answer 4 is incor- Content Area: Child Health, Neurological; Integrated Process:
rect because this is recommended treatment for children Nursing Process, Implementation; Cognitive Level: Analysis;
with pediculosis. This treatment should be continued until Client Need/Subneed: Physiological Integrity/Reduction of Risk
no nits remain. Potential/Potential for Alterations in Body Systems
TEST-TAKING TIP: Note the words in the stem caution 73. CORRECT ANSWER: 3. Answer 1 is incorrect since
against, and choose the option that is different from the others wearing a mask will not guarantee that the nurse, if
(e.g., pharmacologic treatments, which pose a greater risk to the infected, will not transmit the virus to hospitalized children.
child than nonpharmacologic treatments). Answer 2 is incorrect since all pediatric clients could be at
Content Area: Child Health, Infectious Disease; Integrated
risk if the oncoming nurse is not immune to varicella.
Process: Nursing Process, Planning; Cognitive Level:
The charge nurse cannot assume that the nurse is immune.
Application; Client Need/Subneed: Safe and Effective Care Answer 3 is correct because the nurse has been exposed to
Environment/Safety and Infection Control/Standard/ someone with varicella (chickenpox). The charge nurse
Transmission-based/Other Precautions must first determine the nurses varicella immune status
71. CORRECT ANSWER: 4. Answer 1 is incorrect because, before permitting the nurse to provide care. Answer 4 is
although avoiding crowds during winter months may incorrect since all pediatric clients could be at risk if the
decrease the incidence of respiratory infections in general, oncoming nurse is not immune to varicella. The charge
this is not the best way to prevent otitis media. Answer 2 is nurse cannot assume that the nurse is immune.
incorrect because feeding the infant in a supine rather than TEST-TAKING TIP: Choose the one answer that states not be
ANSWERS

upright position increases the risk of otitis media since the given an assignment. Varicella is easily transmitted among
bottles contents may leak into the pharyngeal cavity, creat- close contacts such as family members. Even though most adults
ing a medium for bacterial growth. Answer 3 is incorrect have immunity to chickenpox, this can only be verified with a
because pneumococcal vaccine, not the DTaP (diphtheria, varicella titer.
tetanus, and acellular pertussis) may prevent otitis media. Content Area: Child Health, Infectious Disease; Integrated
The pneumococcal vaccine inoculates against infection by Process: Nursing Process, Implementation; Cognitive Level:
Streptococcus pneumoniae, one of the common causes of ear Analysis; Client Need/Subneed: Safe and Effective Care
infection in children. Answer 4 is correct because infants Environment/Safety and Infection Control/Standard/
who are exclusively breastfed have a decreased incidence Transmission-based/Other Precautions
of otitis media (ear infections) compared to those who are 74. CORRECT ANSWER: 3. Answer 1 is incorrect because,
formula-fed. although tachypnea does occur with respiratory distress,
TEST-TAKING TIP: Select the response that is most specific for tachypnea also occurs during well states. Answer 2 is incor-
prevention of otitis media, not respiratory infections in general. rect because, although scattered rhonchi are indicative of
Content Area: Child Health, Infectious Disease; Integrated
respiratory infection, the child may have rhonchi and not
Process: Nursing Process, Implementation; Cognitive Level:
experience respiratory distress. Answer 3 is correct because
Application; Client Need/Subneed: Safe and Effective Care grunting respirations indicate that the infant is attempt-
Environment/Safety and Infection Control/Standard/ ing to increase positive airway pressure to prevent airway
Transmission-based/Other Precautions collapse. Answer 4 is incorrect because infants are abdomi-
72. CORRECT ANSWERS: 1, 3, 4. Answer 1 is correct nal breathers. This is an expected finding.
because the nurse should assess the child thoroughly TEST-TAKING TIP: Consider that grunting noises are usually
to determine whether the childs neurological status has heard when an individual exerts extra effort at something.
changed since the last assessment. Answer 2 is incorrect Grunting during respiration is an indication that the child is
because this child should be placed with the head of bed working hard to breathe.
elevated to prevent increase of ICP. Answer 3 is correct Content Area: Child Health, Respiratory; Integrated Process:
because nausea and vomiting are common following Nursing Process, Assessment; Cognitive Level: Analysis; Client
2164_Ch05_249-346 29/03/12 12:26 PM Page 341

Answers/Rationales/Tips 341
Need/Subneed: Physiological Integrity/Reduction of Risk Application; Client Need/Subneed: Health Promotion and
Potential/System Specific Assessments Maintenance/Developmental Stages and Transitions
75. CORRECT ANSWER: 1. Answer 1 is correct because 78. CORRECT ANSWERS: 1, 3, 4, 6. Answer 1 is correct
the child is displaying symptoms of respiratory distress because children with beta-thalassemia are at increased
due to tracheostomy occlusion since the nurse is unable risk for infection due to the impaired oxygen-carrying
to pass the catheter through the tracheostomy tube. This capacity of their blood. Answer 2 is incorrect because elimi-
is an emergency requiring the nurse to promptly change nation is not a problem with beta-thalassemia. Answer 3 is
the tracheostomy tube. Answer 2 is incorrect because correct because these children are at risk for injury from
this action puts the child at risk by delaying insertion of a an increased destruction of red blood cells. As red blood
new tracheostomy tube. Answer 3 is incorrect because this cells die, iron is released with deposits in the liver and
action puts the child at risk by delaying insertion of a new spleen, enlarging these organs and impairing their func-
tracheostomy tube. Answer 4 is incorrect because this tion while also causing vomiting from abdominal pres-
action puts the child at risk by delaying insertion of a new sure. Answer 4 is correct because these children may expe-
tracheostomy tube. rience bone deformities, growth retardation, and delayed
TEST-TAKING TIP: If there is enough evidence in the stem maturation of the sexual organs. These symptoms (e.g.,
of a question that a client is experiencing a respiratory broad forehead, short stature, immature appearance) may
emergency, the best answer is to provide necessary action to be troubling for an adolescent, whose main concern is to
resolve the emergency rather than taking time to gather fit in with the peer group. Answer 5 is incorrect because
additional assessment data. these children do not experience chronic pain. Answer 6 is
Content Area: Child Health, Respiratory; Integrated Process: correct because chronic hypoxia results from the produc-
Nursing Process, Implementation; Cognitive Level: Analysis; tion of abnormal red blood cells. If the body does not
Client Need/Subneed: Physiological Integrity/Physiological have sufficient red blood cell production, oxygen is not
Adaptation/Medical Emergencies supplied to the tissues adequately, leading to activity
intolerance and fatigue.
76. CORRECT ANSWER: 2. Answer 1 is incorrect TEST-TAKING TIP: Risk for infection (Answer 1) and injury
because the nurse is taking action before first assessing
(Answer 3) and circulation (Answer 6) are high priorities on
the child. Answer 2 is correct because the nurse should
Maslows hierarchy of needsfollowed by self-esteem related to
first assess the childs physical condition before assuming
body image (Answer 4).
that the monitor is accurate. The monitor could be
Content Area: Child Health, Hematology; Integrated Process:
displaying an artifact as a result of the childs activity.

ANSWERS
Nursing Process, Analysis; Cognitive Level: Analysis;
Answer 3 is incorrect because the nurse is taking action
Client Need/Subneed: Physiological Integrity/Physiological
before first assessing the child. Answer 4 is incorrect
Adaptation/Hemodynamics
because the nurse is taking action before first assessing
the child. 79. CORRECT ANSWER: 3. Answer 1 is incorrect
TEST-TAKING TIP: In this question the priority is assessment because the child is too young to provide a numeric
before action. pain rating. Answer 2 is incorrect because vital signs are
Content Area: Child Health, Cardiovascular; Integrated Process: transient and should not be used to verify pain ratings. A
Nursing Process, Implementation; Cognitive Level: Application; nurse should assume that a child with multiple fractures is
Client Need/Subneed: Physiological Integrity/Reduction of Risk telling the truth about pain. Answer 3 is correct because
Potential/Potential for Alterations in Body Systems the FACES pain rating scale can be used with children as
young as 3 years of age, and pain should be investigated
77. CORRECT ANSWER: 3. Answer 1 is incorrect since with every nursing assessment. Answer 4 is incorrect
these behaviors would be expected in an infant, not a
because young children have minimal language skills and do
preschooler. Answer 2 is incorrect since these behaviors
not understand the difference between descriptive qualities
would be expected in a toddler, not a preschooler. Answer 3
such as dull and sharp.
is correct since preschoolers have great concerns over
TEST-TAKING TIP: Recall that the FACES pain scale uses
body mutilation and may demonstrate somatic symptoms
drawings of smiling and frowning faces to elicit childrens pain
as a response to the stress of hospitalization. Answer 4 is
ratings. Simple assessment tools are best suited for young chil-
incorrect since these behaviors would be expected in a
dren, while more descriptive and finite tools are better suited
school-age child or adolescent, not a preschooler.
for children in later stages of cognitive development.
TEST-TAKING TIP: Focus on the age of the child. Preschoolers
Content Area: Child Health, Pain; Integrated Process: Nursing
are vulnerable to threats of bodily harm. Look for the best
Process, Assessment; Cognitive Level: Comprehension; Client
answer that describes this fear.
Need/Subneed: Health Promotion and Maintenance/
Content Area: Child Health, Hospitalization; Integrated
Developmental Stages and Transitions
Process: Nursing Process, Assessment; Cognitive Level:
2164_Ch05_249-346 29/03/12 12:26 PM Page 342

342 chapter 5 Health Promotion and Maintenance

80. CORRECT ANSWERS: 3, 1, 5. Answer 1 is correct 82. CORRECT ANSWERS: 2, 3, 6. Answer 1 is incorrect
because, once the tube is inserted, placement must be because the common cold is not a contraindication for
verified by both auscultation and aspiration of gastric receiving immunizations; it does not cause a decreased
contents. Answer 2 is incorrect since feeding the infant response to vaccines. Young children have many mild cold
prior to, during, or immediately after the procedure may viruses each year; avoiding immunizations during these
cause vomiting and would contradict the gavage feeding times may cause a child to miss many immunizations.
order. Answer 3 is correct because the child should first Answer 2 is correct because antibodies present in the
be placed on the back with the chest elevated prior to transfused blood can inhibit the immune response to
NG tube placement, to prevent aspiration if the child the immunization. Answer 3 is correct because corticos-
gags during the procedure. Answer 4 is incorrect because teroids can suppress the immune response, limiting
10 mL of air is too much. No more than 5 mL should be the effectiveness of immunization. Answer 4 is incorrect
used in young children because the extra air can cause dis- because mild diarrheal illness is not a contraindication to
comfort and lead to abdominal distention. Answer 5 is immunization. However, moderate to severe diarrhea is a
correct because taping the tube to the cheek after it has contraindication since the childs immune response may
been inserted prevents the child from being able to get not be sufficient if immunizations are administered.
fingers around the tube and pull it out. Answer 6 is Answer 5 is incorrect because penicillin allergy is not a
incorrect because the tube should be measured first from contraindication to immunization. Allergy to streptomycin
the tip of the nose, extended to the earlobe, and then extend- is a contraindication for IPV (inactivated polio vaccine),
ed to the area past the xiphoid process. This measurement and allergy to neomycin is a contraindication for MMR
technique provides the best estimation of the length of tube (measles, mumps, and rubella), IPV, and varicella.
necessary for stomach placement. Answer 6 is correct because a child with HIV should
TEST-TAKING TIP: Note that the nursing tasks related to not receive immunizations that contain live viruses
NG tube insertion are similar for adults and childrenthe (e.g., varicella), as these may lead to infection.
differences pertain to tube sizes and volume of air/fluid. TEST-TAKING TIP: Mild to moderate illness is not a
Content Area: Child Health, Gastrointestinal; Integrated contraindication for immunization. The child must be
Process: Nursing Process, Implementation; Cognitive Level: significantly immunocompromised or at risk for diminished
Application; Client Need/Subneed: Physiological Integrity/Basic immune response for the nurse to withhold immunization.
Care and Comfort/Nutrition and Oral Hydration Content Area: Child Health, Infectious Disease; Integrated
Process: Nursing Process, Analysis; Cognitive Level: Analysis;
81. CORRECT ANSWER: 3. Answer 1 is incorrect
ANSWERS

Client Need/Subneed: Health Promotion and


because, although elevated body temperature is a concern
Maintenance/Immunizations
during phototherapy treatment, the main complication of
elevated temperature is dehydration, making fluid volume 83. CORRECT ANSWER: 3. Answer 1 is incorrect since
deficiency the highest priority. Answer 2 is incorrect because the student should recognize and alert the nurse to more
a change in stooling patterns is expected as the body elimi- subtle ocular changes, such as sluggishness or unequal reac-
nates the bilirubin and is not of concern. Answer 3 is cor- tions, prior to the time that the pupils become fixed and
rect because an infant with hyperbilirubinemia will have dilated and the child is in a potentially irreversible state.
increased fluid needs due to increased insensible fluid Answer 2 is incorrect because the child should be straight,
losses from phototherapy treatment and increased fluid but with the head of bed slightly elevated. Answer 3 is
losses resulting from loose stools as the bilirubin is correct because this statement is evidence that the
eliminated through the bowels. Failure to monitor and student understands that alterations in any of these vital
treat potential fluid volume imbalances can quickly signs could be an indication of worsening condition and
put the infant at risk for dehydration. Answer 4 is should be promptly noted. Answer 4 is incorrect because
incorrect because a diagnosis of interrupted family sleeping is a normal behavior for children, especially if they
processes is not the highest priority for maintaining have been examined frequently throughout the day. The
physiological integrity. student should assess the child frequently, waking the
TEST-TAKING TIP: Consider which nursing diagnosis is the child as necessary without assuming that a sleeping child
most life-threatening to identify the nurses top priority. Infants warrants a call to the practitioner.
are particularly susceptible to fluid imbalances and can become TEST-TAKING TIP: Think: vital signs! Nurses should monitor
seriously compromised because of health conditions and treat- clients with head injury for changes in vital signs because these
ments that affect fluid status. may indicate problems with neurological functioning.
Content Area: Child Health, Hematological; Integrated Process: Content Area: Child Health, Neurological; Integrated Process:
Nursing Process, Analysis; Cognitive Level: Analysis; Client Nursing Process, Evaluation; Cognitive Level: Analysis;
Need/Subneed: Physiological Integrity/Reduction of Risk Client Need/Subneed: Physiological Integrity/Reduction of
Potential/Potential for Alterations in Body Systems Risk Potential/System Specific Assessments
2164_Ch05_249-346 29/03/12 12:26 PM Page 343

Answers/Rationales/Tips 343
84. CORRECT ANSWER: 2. Answer 1 is incorrect because Client Need/Subneed: Physiological Integrity/Reduction of Risk
this is not the optimal time for growth hormone injections. Potential/Potential for Alterations in Body Systems
Answer 2 is correct because the child will be receiving
87. CORRECT ANSWERS: 1, 2, 3, 4, 5. Answer 1 is correct
growth hormone injections and these should be timed to because this is a developmental characteristic of the
simulate the bodys normal growth hormone peak that school-age child. The teaching plan should include activi-
occurs within the first 2 hours of sleep. Answer 3 is incor- ties that allow the children to succeed, such as games with
rect because this is not the optimal time for growth hormone a drug-free focus. Answer 2 is correct because this is a
injections. Answer 4 is incorrect because allowing the child to developmental characteristic of the school-age child. The
choose the injection time may not meet the goal of bedtime teaching plan should provide for rewards (e.g., giving
injections and may result in varying times each day. children pencils with fun slogans in exchange for signing
TEST-TAKING TIP: Eliminate the two answers that relate to a no-drug pledge form). Answer 3 is correct because this is
eating times and eliminate anytime. The administration of a developmental characteristic of the school-age child.
hormone replacement therapy should always attempt to mimic The teaching plan should include basic steps for avoiding
the bodys normal secretory patterns (2 hours of sleep). substance abuse, such as ways to refuse substances when
Content Area: Child Health, Endocrine; Integrated Process:
offered by peers. Answer 4 is correct because this is a
Nursing Process, Implementation; Cognitive Level: Application; developmental characteristic of the school-age child. The
Client Need/Subneed: Physiological Integrity/Pharmacological
teaching plan should remind children that the majority of
and Parenteral Therapies/Medication Administration their peers do not abuse illicit substances. Answer 5 is
85. CORRECT ANSWER: 2. Answer 1 is incorrect because correct because this is a developmental characteristic of
insulin is not part of the treatment plan for a child the school-age child. The teaching plan should include
with CAH. Answer 2 is correct since the child with discussions regarding physical and emotional consequences
congenital adrenal hyperplasia (CAH) is given cortisone of substance abuse in boys and girls.
to stop the increased production of adrenocorticotropic TEST-TAKING TIP: Every answer in a select all that apply
hormone (ACTH), thereby inhibiting adrenocorticoid question may be correct. Select all of the answers if they are all
secretion and virilization of girls/early genital develop- applicable.
ment in boys. Answer 3 is incorrect because growth Content Area: Child Health, Growth and Development;
hormone is used to treat hypopituitarism, not CAH. Integrated Process: Nursing Process, Planning; Cognitive Level:
Answer 4 is incorrect because thyroid hormone is used to Application; Client Need/Subneed: Health Promotion and
treat hypothyroidism. Maintenance/Developmental Stages and Transitions

ANSWERS
TEST-TAKING TIP: The adrenal glands synthesize corticos- 88. CORRECT ANSWER: 2. Answer 1 is incorrect because,
teroids. Only one answer option is a corticosteroidcortisone. although the Denver II is not an IQ test, it does provide
Content Area: Child Health, Endocrine; Integrated Process:
information regarding the childs developmental level in the
Nursing Process, Planning; Cognitive Level: Application; Client areas of language, personal-social, fine motor-adaptive, and
Need/Subneed: Physiological Integrity/Pharmacological and
gross motor. Answer 2 is correct because a suspect
Parenteral Therapies/Expected Effects/Outcomes Denver II should be repeated in 1 to 2 weeks to rule out
86. CORRECT ANSWER: 4. Answer 1 is incorrect because sim- factors such as fatigue or illness that may influence the
ply raising the EVD to the proper level does not assess if the childs performance. The parent is also provided some
child is currently experiencing symptoms from improper EVD skill-building activities to enjoy with the child to encour-
positioning. Answer 2 is incorrect because the EVD should not age development. Answer 3 is incorrect because the nurse
remain as is at a level below the ventricles. Answer 3 is incor- should not refer the child to the physician until a second
rect because elevating the head of the bed will exacerbate the Denver II is suspect. In addition, the nurse is able to make
cerebrospinal fluid (CSF) drainage, since the EVD will become necessary referrals to developmental specialists without first
even lower than the level of the ventricles. Answer 4 is correct notifying the physician. Answer 4 is incorrect because this is
because the external ventricular drain (EVD) should be at not accurate and provides false hope in the event the child
the level of the ventricles, or at the childs ear level. When does have developmental delay.
the EVD is too low, CSF can drain quickly and lead to neu- TEST-TAKING TIP: When parents express concern regarding
rologic complications. A nurse should prevent the CSF their childs health or development, the nurse should discuss the
from draining any further and assess the child. next step in plan of care, while also providing the parents with
TEST-TAKING TIP: Since tubes and drains work by pressure, tools they can use now.
the lower the drain, the less pressure required to induce Content Area: Child Health, Growth and Development;
drainage. Integrated Process: Nursing Process, Implementation; Cognitive
Content Area: Child Health, Neurological; Integrated Process: Level: Application; Client Need/Subneed: Health Promotion
Nursing Process, Implementation; Cognitive Level: Application; and Maintenance/Developmental Stages and Transitions
2164_Ch05_249-346 29/03/12 12:26 PM Page 344

344 chapter 5 Health Promotion and Maintenance

89. CORRECT ANSWERS: 1, 3, 4, 5, 7. Answer 1 is correct bathwater may relieve itching and promote the healing
because a VSD is one of the components of tetralogy of of the lesions. Answer 4 is correct because the child should
Fallot. Answer 2 is incorrect because an ASD is not a not receive immunizations during an acute exacerbation
component of tetralogy of Fallot. Answer 3 is correct of eczema (atopic dermatitis), as this may lead to compli-
because an overriding aorta is one of the components of cations such as allergic reaction. The child with atopic
tetralogy of Fallot. Answer 4 is correct because pulmonic dermatitis is experiencing an inflammatory response.
stenosis is one of the components of tetralogy of Fallot. Care should be directed at relieving inflammation and
Answer 5 is correct because right ventricular hypertrophy avoiding exposure to substances thought to trigger an
is one of the components of tetralogy of Fallot. Answer 6 immune response.
is incorrect because a PDA is not a component of tetralogy TEST-TAKING TIP: Note that the question calls for what
of Fallot. Answer 7 is correct because the blood flows from not to do.
left to right in a child with tetralogy of Fallot through Content Area: Child Health, Integumentary; Integrated
the VSD. Answer 8 is incorrect because aortic stenosis is not Process: Nursing Process, Implementation; Cognitive Level:
a component of tetralogy of Fallot. Application; Client Need/Subneed: Physiological Integrity/
TEST-TAKING TIP: The tetra in tetralogy signifies the Reduction of Risk Potential/Potential for Alterations in
number 4. Recall that there are four defects in the heart of a Body Systems
child with tetralogy of Fallot: VSD, pulmonic stenosis, an 92. CORRECT ANSWER: 2. Answer 1 is incorrect because
overriding aorta, and right ventricular hypertrophy. restraints are indicated in the care of a child who has
Content Area: Child Health, Cardiovascular; Integrated
had a cleft lip repair, to prevent the child from poking or
Process: Nursing Process, Assessment; Cognitive Level:
pulling at the suture line. Answer 2 is correct because
Application; Client Need/Subneed: Physiological the infant should not be fed using a regular bottle.
Integrity/Physiological Adaptation/Alterations in Body Postoperative feedings for the child with a cleft lip
Systems should be administered through special feeders to
90. CORRECT ANSWER: 2. Answer 1 is incorrect since minimize trauma to the suture line. Since a cleft lip
croup is caused by a virus and is not treatable with repair involves only the childs upper lip, the nurse
antibiotics. Answer 2 is correct because the humidity should perform interventions that reduce the risk of
of the shower will create an environment that is damage or infection at the operative location. Answer 3 is
soothing to the childs airway. Cool-mist humidifiers incorrect because this is the correct position for a child
are also recommended for the childs room to relieve following cleft lip repair, to promote drainage of mucus
ANSWERS

the symptoms of spasmodic croup. Any of the croup and decrease risk of aspiration. Answer 4 is incorrect because
syndromes may be treated with humidified air. In the the child should be given pain medication on a schedule, to
case of acute spasmodic laryngitis, both warm mist minimize pain-induced crying episodes that can put undue
and cool mist are acceptable interventions since the tension on the suture line.
problem is airway spasm rather than severe inflamma- TEST-TAKING TIP: The question calls for an answer about
tion. Answer 3 is incorrect because paroxysmal attacks what is not okay.
may last for some time; the child is best treated with sup- Content Area: Child Health, Gastrointestinal; Integrated
portive care at home, and the presence of attacks is not an Process: Nursing Process, Implementation; Cognitive Level:
indication for further hospitalization. Answer 4 is incorrect Application; Client Need/Subneed: Safe and Effective Care
because this intervention is not likely to prevent laryngeal Environment/Management of Care/Supervision
spasms and may cause the child to be more anxious, which 93. CORRECT ANSWER: 1. Answer 1 is correct because
could induce an attack. the child is demonstrating symptoms of severe hypo-
TEST-TAKING TIP: Note that the word spasmodic in the glycemia and the nurse must administer an emergency
stem of the question is also in the correct answer (spasms). dose of glucagon to prevent the child from going into
Content Area: Child Health, Respiratory; Integrated
shock. Answer 2 is incorrect because the nurse should first
Process: Nursing Process, Implementation; Cognitive Level:
administer the emergency dose of glucagon, then contact
Application; Client Need/Subneed: Physiological Integrity/ emergency medical services (EMS). Answer 3 is incorrect
Reduction of Risk Potential/Potential for Alterations in because the child is becoming unresponsive, and placing
Body Systems anything into the mouth would be contraindicated due to
91. CORRECT ANSWER: 4. Answer 1 is incorrect because the risk of aspiration. Answer 4 is incorrect because the
a hypoallergenic diet may alleviate the severity of eczema child has sufficient symptoms of hypoglycemia that need
symptoms. Answer 2 is incorrect because hydrocortisone immediate treatment.
creams are recommended in the treatment of eczema. TEST-TAKING TIP: The key piece of information in the
Answer 3 is incorrect because adding cornstarch to stem of the question is that the child is beginning to lose
2164_Ch05_249-346 29/03/12 12:26 PM Page 345

Answers/Rationales/Tips 345
consciousness. A nurse must act quickly in this situation much earlier. Answer 3 is correct because the surgical
and assume that it is due to low blood sugar, because waiting repair of hypospadias generally begins within the first
to differentiate between hyper- and hypoglycemia could be few months of life and continues in stages, finishing
life-threatening. between 6 and 18 months of age, before the child begins
Content Area: Child Health, Endocrine; Integrated Process: toilet training. Answer 4 is incorrect because the surgical
Nursing Process, Implementation; Cognitive Level: repair is completed much earlier.
Application; Client Need/Subneed: Physiological Integrity/ TEST-TAKING TIP: Since toilet training and the
Physiological Adaptation/Alterations in Body Systems development of body image begins in toddlerhood,
hypospadias repair is best completed prior to this state of
94. CORRECT ANSWER: 4. Answer 1 is incorrect because development.
pale gray stools are associated with conditions related to
Content Area: Child Health, Genitourinary; Integrated
insufficient bile output (e.g., cirrhosis). Answer 2 is
Process: Nursing Process, Implementation; Cognitive Level:
incorrect because currant-jelly stools are associated with a
Comprehension; Client Need/Subneed: Physiological
diagnosis of intussusception. Answer 3 is incorrect because
Integrity/Reduction of Risk Potential/Potential for
loose, yellow stools are normal for a breastfed newborn.
Alterations in Body Systems
Answer 4 is correct because the child with Hirschsprungs
disease will have infrequent stools that appear thin and 97. CORRECT ANSWER: 4. Answer 1 is incorrect because,
ribbon-like. although children with JIA frequently have sleep distur-
TEST-TAKING TIP: Since Hirschsprungs disease results in bances, this is not the top priority. Answer 2 is incorrect
obstruction of the lower GI tract, the expected stooling because, although children with JIA should participate in
pattern will reflect stools that have become compressed by regular exercise, this is not the top priority. Answer 3 is
the obstruction. incorrect because the child should be informed of support
Content Area: Child Health, Gastrointestinal; Integrated groups, but this is not the top priority. Answer 4 is correct
Process: Nursing Process, Assessment; Cognitive Level: because adolescents with JIA are frequently prescribed
Comprehension; Client Need/Subneed: Physiological medications that are taxing to the liver, including
Integrity/Basic Care and Comfort/Elimination NSAIDs, such as naproxen sodium, and SAARDs (slower-
acting antirheumatic drugs), such as methotrexate.
95. CORRECT ANSWER: 2. Answer 1 is incorrect because, Alcohol abuse could cause serious complications when
although the nurse should assess the childs lungs for
taking these medications. A nurses top priority takes
possible aspiration, the first action is to clear the airway.
into consideration that the adolescent is facing
Answer 2 is correct because the nurses first action

ANSWERS
increasing peer pressure to drink alcohol, which could
should be to clear the childs airway of formula. Since
lead to hepatotoxicity.
this is the infants first feeding, the nurse should suspect
TEST-TAKING TIP: The best answer is what not to do
a tracheoesophageal fistula (TEF) and should not
(i.e., what to avoid).
attempt to feed the child again. Answer 3 is incorrect
Content Area: Child Health, Musculoskeletal; Integrated
because an x-ray may be obtained after the childs airway is
Process: Nursing Process, Implementation; Cognitive Level:
stabilized. Answer 4 is incorrect because the nurse should
Analysis; Client Need/Subneed: Physiological Integrity/
first stabilize the airway and assess the child before contact-
Reduction of Risk Potential/Potential for Alterations in
ing the physician.
Body Systems
TEST-TAKING TIP: It is helpful to remember the 3 Cs of
TEF: coughing, choking, and cyanosis. A nurses top priority 98. CORRECT ANSWER: 2. Answer 1 is incorrect
should be the airway of an infant with these symptoms. because the RN could delegate this task to the LVN/LPN.
Content Area: Child Health, Respiratory; Integrated Answer 2 is correct because the RN should change the
Process: Nursing Process, Implementation; Cognitive Level: wound dressing and assess the condition of the
Application; Client Need/Subneed: Physiological Integrity/ decubitus. Answer 3 is incorrect because the RN could
Reduction of Risk Potential/Potential for Alterations in delegate this task to the LVN/LPN. Answer 4 is
Body Systems incorrect because the RN could delegate this task to the
LVN/LPN.
96. CORRECT ANSWER: 3. Answer 1 is incorrect because TEST-TAKING TIP: When the task involves assessment and
this is too early for completion of the surgical repair.
analysis of assessment findings, the RN should not delegate it
Delaying surgery for several months to a year allows time
to the LVN/LPN.
to treat the infant with male hormones to increase the
Content Area: Child Health, Hospitalization; Integrated
amount of penile tissue prior to surgery. Additionally,
Process: Nursing Process, Implementation; Cognitive Level:
surgery on newborns carries an increased risk of complica-
Application; Client Need/Subneed: Safe and Effective Care
tions and should be avoided unless medically necessary.
Environment/Management of Care/Delegation
Answer 2 is incorrect because the surgical repair is completed
2164_Ch05_249-346 29/03/12 12:26 PM Page 346

346 chapter 5 Health Promotion and Maintenance

99. CORRECT ANSWERS: 3, 1, 5, 4. Answer 1 is correct Content Area: Child Health, Growth and Development;
because the preschool child may feel more comfortable Integrated Process: Nursing Process, Implementation; Cognitive
keeping underpants on during the assessment because a Level: Application; Client Need/Subneed: Health Promotion
common fear of preschoolers is genital mutilation. Answer 2 and Maintenance/Developmental Stages and Transitions
is incorrect since it would be unsafe to leave a preschool child 100. CORRECT ANSWER: 1. Answer 1 is correct because
alone in an examination room to undress. This intervention the nurses priority is to alleviate airway inflammation,
would be appropriate for an adolescent. Answer 3 is correct and administration of a beta agonist such as albuterol is
because the nurse should first ask the child if the parents recommended. Answer 2 is incorrect because the loading
should participate in the procedure. The child should be dose of Solu-Medrol (methylprednisolone) should be adminis-
given options for parent participation, such as whether par- tered after starting albuterol treatments. Answer 3 is incorrect
ents should be present, if the child would like help undress- because IV fluids can be given after starting albuterol treat-
ing, and if the child would prefer to sit on the parents lap ments. Answer 4 is incorrect because a chest x-ray can be
or sit alone on the examination table. Answer 4 is correct obtained after starting albuterol treatments.
because, although the nurse proceeds in a head-to-toe TEST-TAKING TIP: Airway is the first priority, especially in
direction, inspecting eyes, ears, and mouth is invasive and is children with asthma. Begin treatments that have quick effects
best performed at the end of the assessment in order to not on oxygen exchange, then administer long-acting medications
disrupt the rest of the examination. Answer 5 is correct and gather assessment data.
because the nurse should proceed in a head-to-toe direction Content Area: Child Health, Respiratory; Integrated Process:
while keeping the most invasive assessments for the end. Nursing Process, Implementation; Cognitive Level: Analysis;
Answer 6 is incorrect because this technique is appropriate for
Client Need/Subneed: Physiological Integrity/Reduction of Risk
infants, not preschoolers. Potential/Potential for Alterations in Body Systems
TEST-TAKING TIP: Because preschool childrens central task is
initiative vs. guilt, they benefit from interventions that offer
independence in decision making.
ANSWERS
2164_Ch06_347-578 29/03/12 12:29 PM Page 347

CHAPTER 6

Physiological Integrity
Nursing Care of the Adult Client

Robyn Marchal Nelson

347
2164_Ch06_347-578 29/03/12 12:29 PM Page 348

348 chapter 6 Physiological Integrity

ASSESSMENT OF VI. INFORMATION RELATIVE TO PROTECTIVE


T H E A D U LT C L I E N T FUNCTIONS: skin problemsrash, itch; current
treatment; unusual hair loss.
Assessment is the process of gathering a comprehensive data-
VII. INFORMATION RELATIVE TO COMFORT,
base about the clients present, past, and potential health
REST, ACTIVITY, MOBILITY: usual activity (activi-
problems, as well as a description of the client as a whole in
ties of daily living [ADLs]); present ability and restric-
his or her environment. It includes a comprehensive nurs-
tions; rest and sleep pattern; weakness; joint or muscle
ing history, a physical examination, and laboratory/x-ray
stiffness, pain, or swelling; occupation; interests.
data, and it concludes with the formulation of nursing
diagnoses. VIII. INFORMATION RELATIVE TO
The Health Insurance Portability and Accountability ELIMINATION: bowel habits; changesconstipation,
Act of 1996 (HIPAA) strengthened the privacy protec- diarrhea; ostomy; emesis; nausea; voidingretention,
tions for consumers. Communications are confidential, frequency, dysuria, incontinence.
and the client needs to give permission for other family IX. INFORMATION RELATIVE TO SENSORY/
members to remain in the roomparticularly when ask- PERCEPTUAL FUNCTIONS: painverbal report,
ing sensitive questions on pregnancies, abortions, drug acute/chronic, treatment, quality, location, precipitat-
use, or multiple sex partners (see Chapter 3, pp. 8182). ing factors, duration; limitations in vision (glasses),
hearing, touch, smell; orientation to person, place,
time; confusion; headaches; fainting; dizziness;
Subjective Data convulsions.
Nursing History
The nursing history obtains data for planning and imple-
menting nursing actions.
Objective Data
I. GENERAL HEALTH INFORMATION: reason for I. GENERAL provides information on the client as
admission; duration of present illness; previous hospi- a whole.
talization; history of illnesses; diagnostic procedures A. Race, sex, apparent age in relation to stated age.
before admission; allergiestype and severity of reac- B. Nutritional statuswell hydrated and developed or
tions; medications taken at homeover-the-counter obesity, cachexiainclude weight.
(OTC) and prescription medications, and C. Apparent health statusgeneral good health or
alternative/complementary therapies. mild, moderate, severe debilitation.
D. Posture and motor activityerect, symmetrical,
II. INFORMATION RELATIVE TO GROWTH
balanced gait and muscle development, or ataxic,
AND DEVELOPMENT: age; menarcheage at
circumducted, scissor, or spastic gait; slumped or
onset; heavy menses; dysmenorrhea; vaginal discharge;
bent-over posture; scoliosis, lordosis, kyphosis;
date of last Pap smear; pregnancies; abortions; miscar-
mild, moderate, or hyperactive motor responses.
riages; last menstrual period; history of sexually trans-
E. Behavioralert; oriented to person, time, place;
mitted infections (STIs).
hears and comprehends instructions, or tense,
ADULT

III. INFORMATION RELATIVE TO anxious, angry; uses abusive language; slightly or


PSYCHOSOCIAL FUNCTIONS: feelings (anger, largely unresponsive; delusions, hallucinations.
denial, fear, anxiety, guilt, lifestyle changes); language F. Odorsnoncontributory, or acetone, alcohol, fetid
barriers; cultural needs; family support; spiritual breath, incontinent of urine or feces.
needs; religious preference; history of trauma/rape;
II. PHYSICAL ASSESSMENTrequires knowledge
job status; current stressors.
of normal findings, organization, and keen senses
IV. INFORMATION RELATIVE TO NUTRITION: (i.e., visual, auditory, touch, smell). For abnormal
appetitenormal, changes; dietary habits; food findings, refer to the Assessment section of each health
preferences or intolerances; difficulty swallowing or problem discussed.
chewing; dentures; use of caffeine/alcohol; weight A. Components
changes; excessive thirst, hunger, sweating. 1. Inspectionuses observations to detect
V. INFORMATION RELATIVE TO FLUID AND deviations from normal.
GAS TRANSPORT: difficulty breathing; shortness 2. Auscultationused to perceive and interpret
of breath; home O2 use; history of cough/smoking; sounds arising from various organs, particularly
colds; sputum; swelling of extremities; chest pain; heart, lungs, and bowel.
palpitations; varicosities; excessive bruising; blood 3. Palpationused to assess for discomfort, temper-
transfusions; excessive bleeding. ature, pulsations, size, consistency, and texture.
2164_Ch06_347-578 29/03/12 12:29 PM Page 349

Assessment of the Adult Client 349


4. Percussiontechnique used to elicit vibrations 5. Necksuppleness. Trachea, larynx,
produced by underlying organ structures; used thyroid, blood vessels (jugular veins,
less frequently in nursing practice. carotid arteries).
a. Flatnormal percussion; note over muscle or 6. Nodesany cervical, supraclavicular, axillary,
bone. epitrochlear, inguinal lymphadenopathy? If so,
b. Dullnormal percussion; note over organs size of nodes (in centimeters), consistency
such as liver. (firm, rubbery, tender), mobile or fixed.
c. Resonancenormal percussion; note over 7. Eyes:
lungs. a. External eye. Conjunctivae, sclerae, lids,
d. Tympanynormal percussion; note over cornea, pupils (including reflexes), visual
stomach or bowel. fields, extraocular motions.
B. Approachhead to toe b. Fundus. Disk, blood vessels, pigmentation.
1. General appearancewell or poorly developed or 8. Earsshape of pinnae, external canal,
nourished. Color (black, white, jaundiced, pale). discharge, tympanic membrane, acuity, air
In distress (acutely or chronically)? conduction versus bone conduction (Rinne test),
2. Vital signsblood pressure (which arm or both, lateralization (Weber test).
orthostatic change); pulse (regular or irregular, 9. Nosenares (symmetry), septum, mucosa,
orthostatic change); respirations (labored or polyps, discharge, flaring.
unlabored, wheeze); temperature (axillary, rectal, 10. Mouth and throatlips, teeth (loose, dental
temporal [forehead], tympanic membrane, or hygiene, odor), tongue (size, papillation,
oral); weight; height (Table 6.1). position), buccal mucosa, palate, tonsils,
3. Skin, hair, and nailspigmentation, scars, oropharynx.
lesions, bruises, turgor. Describe or draw rashes. 11. Chest
a. Skin color: a. Inspection. Contour, symmetry, expansion,
(1) Redfever, allergic reaction, carbon retractions.
monoxide (CO) poisoning, burn. b. Palpation. Expansion, rib tenderness, tactile
(2) White (pallor)excessive blood loss, fremitus.
fright. c. Percussion. Diaphragmatic excursion,
(3) Blue (cyanosis)hypoxemia, peripheral dullness.
vasoconstriction, shock. d. Auscultation. Crackles, rubs, wheezes,
(4) Mottledcardiovascular embarrassment, egophony, pectoriloquy.
shock. (1) Use diaphragm or bell. Normal sounds
b. Skin temperature: over alveolivesicular. Large airway or
(1) Hot, dryexcessive body heat abnormal soundsbronchial or bron-
(heatstroke). chovesicular. Adventitious sounds
(2) Hot, wetreaction to increased internal crackles or wheezes.
or external temperature. (2) Cracklesdiscontinuous noises heard on

ADULT
(3) Cool, dryexposure to cold. auscultation; caused by popping open of
(4) Cool, clammyshock. air spaces; usually associated with
4. Headscalp, skull (configuration), scars, increased fluid in the lungs; formerly
tenderness, bruits. called rales and rhonchi.

Table 6.1
Factors Affecting Vital Signs
H&H BS BS/DKA Narcotic K+ K+
Infection (Hypovo- (Insulin (Hypergly- (CNS Anxiety Pain Acute (Hyperkal- (Hypokal-
Factor (Fever) lemia) Shock) cemia) Depression) (Fear) (Acute) MI emia) emia) Exercise
T Normal Normal Normal Normal Normal
HR Normal/
RR Normal Shallow Shallow
BP Normal Normal/ Normal/
From Myers, Ehren: RNotes. FA Davis, Philadelphia, 2009.
2164_Ch06_347-578 29/03/12 12:29 PM Page 350

350 chapter 6 Physiological Integrity

(3) Wheezeshigh-pitched, whistling sounds d. Motor. Muscle mass, strength; deep


made by air flowing through narrowed tendon reflexes. Pathological or primitive
airways. reflexes.
(4) Stridorharsh, high-pitched, heard e. Sensory. Touch, pain, vibration. Heat and
during inspiration and expiration; life cold as indicated.
threatening. III. ROUTINE LABORATORY STUDIESsee
12. Breastssymmetry, retraction, lesions, Appendix A for normal ranges.
nipples (inverted, everted), masses, tender-
A. Hematology:
ness, discharge.
1. Complete blood countdetects presence of
13. Heart:
anemia, infection, allergy, and leukemia.
a. Inspection. Point of maximal impulse
2. Prothrombin timeincrease may indicate liver
(PMI), chest contour.
disease or cancer.
b. Palpation. Point of maximal impulse (PMI),
3. Serology (Venereal Disease Research
thrills, lifts, thrusts.
Laboratories [VDRL])determines presence
c. Auscultation. Heart sounds, gallops,
of syphilis; false-positive result may indicate
murmurs, rubs. Use diaphragm for high-
collagen dysfunction.
pitched sounds of normal heart sounds
B. Urinalysis:
(S1 and S2) and bell for abnormal sounds
1. Specific gravitymeasures ability of kidney to
(S3 and S4).
concentrate urine. Fixed specific gravity indicates
14. Abdomen:
renal tubular dysfunction.
a. Inspection. Scars (draw these), contour,
2. Proteinindicates glomerular dysfunction.
masses, vein pattern.
3. Albumin, white blood cells (WBCs), and
b. Auscultation. Bowel sounds, rubs, bruits.
pusindicate renal infection.
Use diaphragm. Auscultate after inspection
4. Sugar and acetonepresence indicates metabolic
and before palpation and percussion. Listen
disorder.
to each quadrant for at least 1 minute. If
C. Chest x-raydetects tuberculosis or other pul-
bowel sounds are present, they will be heard
monary dysfunctions, as well as changes in size or
in lower right quadrant (area of ileocecal
configuration of heart.
valve). Hypoactivityevery minute; normal
D. Electrocardiogram (ECG or EKG)detects rhythm
every 15 to 20 seconds; hyperactivityabout
and conduction disturbances, presence of myocardial
every 3 seconds.
ischemia or necrosis, and ventricular hypertrophy.
c. Percussionorganomegaly, hepatic dullness.
E. Blood chemistriesdetect deviation in electrolyte
d. Palpationtenderness, masses, rigidity, liver,
balance, presence of tissue damage, and adequacy
spleen, kidneys.
of glomerular filtration.
e. Herniafemoral, inguinal, ventral,
umbilical. IV. PREVENTIVE CARE
15. Genitalia: A. Checkup visits recommended every 1 to 3 years
ADULT

a. Male. Penile lesions, discharge, scrotum, until age 65 and then yearly thereafter. Table 6.2
testes. Circumcised? lists suggested timelines.
b. Female. Labia, discharge, odor, Bartholins B. Individuals with special risk factors may need more
and Skenes glands, vagina, cervix. Bimanual frequent and additional types of preventive care.
examination of internal genitalia. 1. Diabeteseye, foot examinations; urine, blood
16. Rectumperianal lesions, sphincter tone, sugar tests.
tenderness, masses, prostate, stool color, 2. Drug abuseAIDS, tuberculosis (TB) tests;
occult blood. hepatitis immunization.
17. Extremitiespulses (symmetry, bruits, perfu- 3. Alcoholisminfluenza, pneumococcal immuniza-
sion). Joints (mobility, deformity). Cyanosis, tions; TB test.
edema. Varicosities. Muscle mass. Grips equal. 4. Overweightblood sugar test, triglycerides,
18. Backcontour of spine, tenderness. Sacral blood pressure.
edema. 5. Homeless, recent refugee or immigrantTB test.
19. Neurological: 6. High-risk sexual behaviorAIDS, syphilis,
a. Mental status. Alertness, memory, judgment, gonorrhea, chlamydia (every year for women
mood. who are sexually active), hepatitis tests.
b. Cranial nerves (IXII) (Fig. 6.1). 7. Pregnancyrubella blood test (prior to first
c. Cerebellum. Gait, finger-nose, heel-shin, pregnancy).
tremors. 8. Cancercolonoscopy, mammography, x-ray.
2164_Ch06_347-578 29/03/12 12:29 PM Page 351

Growth and Development 351


OLFACTORY 1

OCULOMOTOR 3
OPTIC 2
TROCHLEAR 4
ABDUCENS 6

TRIGEMINAL 5

FACIAL 7

GLOSSOPHARYNGEAL 9

ACOUSTIC 8

HYPOGLOSSAL 12

VAGUS 10 Figure 6.1 Cranial nerves and their distribu-


ACCESSORY 11 tions. (From Venes, D [ed]: Tabers Cyclopedic Medical
Dictionary, ed 21. FA Davis, Philadelphia, 2009.)

C. Adult immunizationsprevention of disease and III. COGNITIVE DEVELOPMENT

ADULT
reduction in the severity of disease (Table 6.3). A. Close to peak of intelligence, memory, and abstract
Assessment is followed by analysis of data and formula- thought.
tion of a nursing diagnosis. Possible nursing diagnoses are B. Maximum ability to solve problems and learn new
given in the following sections. skills.
IV. SOCIALIZATION
GROWTH AND A. Has a vision of the future and imagines various
DEVELOPMENT possibilities for self.
B. Defines and tests out what can be accomplished.
Young Adulthood (20 to C. Seeks out a mentor to emulate as a guiding, though
30 Years of Age) transitional, figure; the mentor is usually a mixture
of parent, teacher, and friend who serves as a role
I. STAGE OF DEVELOPMENTPSYCHOSOCIAL model to support and facilitate the developing
STAGE: intimacy versus isolation. vision of self.
II. PHYSICAL DEVELOPMENT (see also Chapter 7) D. Grows from a beginning to a fuller understanding
A. At the height of bodily vigor. of own authority and autonomy.
B. Maximum level of strength, muscular development, E. Transfers an interest into an occupation or profes-
height, and cardiac and respiratory capacity; also, sion; crucial work choice may be made after one
period of peak sexual capacity for men. has knowledge, judgment, and self-understanding,
(text continues on page 355)
ADULT

352
Table 6.2
Preventive Care Timelines
Years of Age
18 25 30 35 40 45 50 55 60 65 70 75 <
Tests
Blood Pressure Every 2 Years
Height & Weight Periodically
Cholesterol Men Every 23 Years (Men & Women)
2164_Ch06_347-578 29/03/12 12:29 PM Page 352

Hearing Periodically
Mammography Baseline Yearly (Women)
Pap Smear (Cervical Cancer) Every 13 Years (Women)
chapter 6 Physiological Integrity

Sigmoidoscopy/Colonoscopy Every 35 Years


and/or and/or
Stool Occult Blood (FOBT) Yearly
Blood At Least Every 34 Years

Examinations
Dental, Oral Health Yearly
Vision/Glaucoma Every 24 Years Every 12 Years
Breast (by Doctor) Every 13 Years (Women) Every Year (Women)
Exam for Cancer: thyroid, mouth, skin, ovaries,
testicles (monthly between ages 1940), lymph Every 3 Years Yearly
nodes, rectum (40+), prostate (men 50+)
Bone Density (Osteoporosis) Every 2 Years Yearly
Health Guidance
Smoking, Alcohol & Drugs
Sexual Behavior, AIDS
Eating Disorders, Nutrition, Physical Activity,
Weight Management
Periodically
Violence & Guns, Injuries
Family Planning
Occupational Health
Folate (Women 1245), Aspirin (Men 40+)
Upper age limits should be individualized for each person
Recommended by most major authorities
Recommended by some major authorities
Source: Office of Disease Prevention and Health Promotion, in cooperation with the agencies of the Public Health Service, U.S. Department of Health and Human
Services.
2164_Ch06_347-578 29/03/12 12:29 PM Page 353

Table 6.3
2011 Recommended Adult Immunization Schedule
Recommended adult immunization schedule, by vaccine and age group United States, 2011

VACCINE AGE GROUP 1926 years 2749 years 5059 years 6064 years 65 years
1,* 1 dose annually
Influenza
2,* Substitute 1-time dose of Tdap for Td booster; then boost with Td every 10 years Td booster
Tetanus, diphtheria, pertussis (Td/Tdap) every 10 years

Varicella3,* 2 doses

Human papillomavirus (HPV)4,* 3 doses (females)

Zoster5 1 dose

Measles, mumps, rubella (MMR)6,* 1 or 2 doses 1 dose

Pneumococcal (polysaccharide)7,8 1 or 2 doses 1 dose

Meningococcal9,* 1 or more doses

Hepatitis A10,* 2 doses

11,*
Hepatitis B 3 doses

* Covered by the Vaccine For all persons in this category who meet the age Recommended if some other risk No recommendation
Injury Compensation requirements and who lack evidence of immunity factor is present (e.g., based on
Program (e.g., lack documentation of vaccination or have medical, occupational, lifestyle,
no evidence of previous infection) or other indications)

Vaccines that might be indicated for adults, based on medical and other indications United States, 2011

INDICATION Immunocompro- HIV infection3,6,12,13 Asplenia12 (including


mising conditions CD4 T Diabetes, elective splenectomy) Kidney failure,
(excluding hu- lymphocyte count heart disease, and persistent end-stage
man immuno- chronic lung complement Chronic renal disease,
deficiency virus 200 200 disease, chron- component liver receipt of Health-care
VACCINE Pregnancy [HIV])3,5,6,13 cells/L cells/L ic alcoholism deficiencies disease hemodialysis personnel
1 dose TIV or
Influenza1,* 1 dose TIV annually
LAIV annually

Tetanus, diphtheria, per- Td Substitute 1-time dose of Tdap for Td booster; then boost with Td every 10 years
tussis (Td/Tdap)2,*

Varicella3,* Contraindicated 2 doses

Human 3 doses through age 26 years


papillomavirus (HPV)4,*

Zoster5 Contraindicated 11 dose


dose

Measles, mumps, Contraindicated 11or


or 22 doses
doses
rubella6,*

ADULT
Pneumococcal 11 or
of 2 doses
doses
(polysaccharide)7,8

Meningococcal9,* 1 or more doses


10,* 2 doses
Hepatitis A

Hepatitis B11,* 3 doses

* Covered by the Vaccine For all persons in this category who meet the age Recommended if some other risk No recommendation
Injury Compensation requirements and who lack evidence of immunity factor is present (e.g., on the basis
Program (e.g., lack documentation of vaccination or have of medical, occupational, lifestyle,
no evidence of previous infection) or other indications)

NOTE: The above recommendations must be read along with the following footnotes.

1. Influenza vaccination Adults with uncertain or incomplete history of completing a 3-dose primary vaccination series with
Annual vaccination against influenza is recommended for all persons aged 6 months and older, Td-containing vaccines should begin or complete a primary vaccination series. For unvaccinated adults,
including all adults. Healthy, nonpregnant adults aged less than 50 years without high-risk medical administer the first 2 doses at least 4 weeks apart and the third dose 612 months after the second. If
conditions can receive either intranasally administered live, attenuated influenza vaccine (FluMist), or incompletely vaccinated (i.e., less than 3 doses), administer remaining doses. Substitute a one-time dose
inactivated vaccine. Other persons should receive the inactivated vaccine. Adults aged 65 years and older of Tdap for one of the doses of Td, either in the primary series or for the routine booster, whichever comes
can receive the standard influenza vaccine or the high-dose (Fluzone) influenza vaccine. Additional first.
information about influenza vaccination is available at http://www.cdc.gov/vaccines/vpd-vac/flu/default.htm. If a woman is pregnant and received the most recent Td vaccination 10 or more years previously,
2. Tetanus, diphtheria, and acellular pertussis (Td/Tdap) vaccination administer Td during the second or third trimester. If the woman received the most recent Td vaccination
Administer a one-time dose of Tdap to adults aged less than 65 years who have not received Tdap less than 10 years previously, administer Tdap during the immediate postpartum period. At the clinicians
previously or for whom vaccine status is unknown to replace one of the 10-year Td boosters, and as soon discretion, Td may be deferred during pregnancy and Tdap substituted in the immediate postpartum
as feasible to all 1) postpartum women, 2) close contacts of infants younger than age 12 months (e.g., period, or Tdap may be administered instead of Td to a pregnant woman after an informed discussion with
grandparents and child-care providers), and 3) health-care personnel with direct patient contact. Adults the woman.
aged 65 years and older who have not previously received Tdap and who have close contact with an infant The ACIP statement for recommendations for administering Td as prophylaxis in wound management is
aged less than 12 months also should be vaccinated. Other adults aged 65 years and older may receive available at http://www.cdc.gov/vaccines/pubs/acip-list.htm.
Tdap. Tdap can be administered regardless of interval since the most recent tetanus or diphtheria-
containing vaccine.

Continued
2164_Ch06_347-578 29/03/12 12:29 PM Page 354

354 chapter 6 Physiological Integrity

Table 6.3
2011 Recommended Adult Immunization Schedulecontd
3. Varicella vaccination Other : Residents of nursing homes or long-term care facilities and persons who smoke cigarettes.
All adults without evidence of immunity to varicella should receive 2 doses of single-antigen varicella Routine use of PPSV is not recommended for American Indians/Alaska Natives or persons aged less
vaccine if not previously vaccinated or a second dose if they have received only 1 dose, unless they have than 65 years unless they have underlying medical conditions that are PPSV indications. However, public
a medical contraindication. Special consideration should be given to those who 1) have close contact with health authorities may consider recommending PPSV for American Indians/Alaska Natives and persons
persons at high risk for severe disease (e.g., health-care personnel and family contacts of persons with aged 50 through 64 years who are living in areas where the risk for invasive pneumococcal disease is
immunocompromising conditions) or 2) are at high risk for exposure or transmission (e.g., teachers; increased.
child-care employees; residents and staff members of institutional settings, including correctional 8. Revaccination with PPSV
institutions; college students; military personnel; adolescents and adults living in households with children; One-time revaccination after 5 years is recommended for persons aged 19 through 64 years with chronic
nonpregnant women of childbearing age; and international travelers). renal failure or nephrotic syndrome; functional or anatomic asplenia (e.g., sickle cell disease or
Evidence of immunity to varicella in adults includes any of the following: 1) documentation of 2 doses of splenectomy); and for persons with immunocompromising conditions. For persons aged 65 years and
varicella vaccine at least 4 weeks apart; 2) U.S.-born before 1980 (although for health-care personnel and older, one-time revaccination is recommended if they were vaccinated 5 or more years previously and were
pregnant women, birth before 1980 should not be considered evidence of immunity); 3) history of varicella aged less than 65 years at the time of primary vaccination.
based on diagnosis or verification of varicella by a health-care provider (for a patient reporting a history of 9. Meningococcal vaccination
or having an atypical case, a mild case, or both, health-care providers should seek either an epidemiologic Meningococcal vaccine should be administered to persons with the following indications:
link with a typical varicella case or to a laboratory-confirmed case or evidence of laboratory confirmation, if Medical: A 2-dose series of meningococcal conjugate vaccine is recommended for adults with
it was performed at the time of acute disease); 4) history of herpes zoster based on diagnosis or verification anatomic or functional asplenia, or persistent complement component deficiencies. Adults with HIV
of herpes zoster by a health-care provider; or 5) laboratory evidence of immunity or laboratory confirmation infection who are vaccinated should also receive a routine 2-dose series. The 2 doses should be
of disease. administered at 0 and 2 months.
Pregnant women should be assessed for evidence of varicella immunity. Women who do not have Other: A single dose of meningococcal vaccine is recommended for unvaccinated first-year college
evidence of immunity should receive the first dose of varicella vaccine upon completion or termination of students living in dormitories; microbiologists routinely exposed to isolates of Neisseria meningitidis;
pregnancy and before discharge from the health-care facility. The second dose should be administered 48 military recruits; and persons who travel to or live in countries in which meningococcal disease is
weeks after the first dose. hyperendemic or epidemic (e.g., the meningitis belt of sub-Saharan Africa during the dry season
4. Human papillomavirus (HPV) vaccination [December through June]), particularly if their contact with local populations will be prolonged.
HPV vaccination with either quadrivalent (HPV4) vaccine or bivalent vaccine (HPV2) is recommended for Vaccination is required by the government of Saudi Arabia for all travelers to Mecca during the annual
females at age 11 or 12 years and catch-up vaccination for females aged 13 through 26 years. Hajj.
Ideally, vaccine should be administered before potential exposure to HPV through sexual activity; Meningococcal conjugate vaccine, quadrivalent (MCV4) is preferred for adults with any of the preceding
however, females who are sexually active should still be vaccinated consistent with age-based recommen- indications who are aged 55 years and younger; meningococcal polysaccharide vaccine (MPSV4) is
dations. Sexually active females who have not been infected with any of the four HPV vaccine types (types preferred for adults aged 56 years and older. Revaccination with MCV4 every 5 years is recommended for
6, 11, 16, and 18, all of which HPV4 prevents) or any of the two HPV vaccine types (types 16 and 18, both adults previously vaccinated with MCV4 or MPSV4 who remain at increased risk for infection (e.g., adults
of which HPV2 prevents) receive the full benefit of the vaccination. Vaccination is less beneficial for females with anatomic or functional asplenia, or persistent complement component deficiencies).
who have already been infected with one or more of the HPV vaccine types. HPV4 or HPV2 can be 10. Hepatitis A vaccination
administered to persons with a history of genital warts, abnormal Papanicolaou test, or positive HPV DNA Vaccinate persons with any of the following indications and any person seeking protection from hepatitis A
test, because these conditions are not evidence of previous infection with all vaccine HPV types. virus (HAV) infection:
HPV4 may be administered to males aged 9 through 26 years to reduce their likelihood of genital warts. Behavioral: Men who have sex with men and persons who use injection drugs.
HPV4 would be most effective when administered before exposure to HPV through sexual contact. Occupational: Persons working with HAV-infected primates or with HAV in a research laboratory
A complete series for either HPV4 or HPV2 consists of 3 doses. The second dose should be setting.
administered 12 months after the first dose; the third dose should be administered 6 months after the first Medical: Persons with chronic liver disease and persons who receive clotting factor concentrates.
dose. Other: Persons traveling to or working in countries that have high or intermediate endemicity of
Although HPV vaccination is not specifically recommended for persons with the medical indications hepatitis A (a list of countries is available at http://wwwn.cdc.gov/travel/ contentdiseases.aspx).
described in Figure 2, Vaccines that might be indicated for adults based on medical and other indications, Unvaccinated persons who anticipate close personal contact (e.g., household or regular babysitting) with
it may be administered to these persons because the HPV vaccine is not a live-virus vaccine. However, the an international adoptee during the first 60 days after arrival in the United States from a country with high
immune response and vaccine efficacy might be less for persons with the medical indications described in or intermediate endemicity should be vaccinated. The first dose of the 2-dose hepatitis A vaccine series
Figure 2 than in persons who do not have the medical indications described or who are immunocompetent. should be administered as soon as adoption is planned, ideally 2 or more weeks before the arrival of the
5. Herpes zoster vaccination adoptee.
A single dose of zoster vaccine is recommended for adults aged 60 years and older regardless of Single-antigen vaccine formulations should be administered in a 2-dose schedule at either 0 and 612
whether they report a previous episode of herpes zoster. Persons with chronic medical conditions may be months (Havrix), or 0 and 618 months (Vaqta). If the combined hepatitis A and hepatitis B vaccine
vaccinated unless their condition constitutes a contraindication. (Twinrix) is used, administer 3 doses at 0, 1, and 6 months; alternatively, a 4-dose schedule may be used,
6. Measles, mumps, rubella (MMR) vaccination administered on days 0, 7, and 2130, followed by a booster dose at month 12.
Adults born before 1957 generally are considered immune to measles and mumps. All adults born in 11. Hepatitis B vaccination
1957 or later should have documentation of 1 or more doses of MMR vaccine unless they have a medical Vaccinate persons with any of the following indications and any person seeking protection from hepatitis B
contraindication to the vaccine, laboratory evidence of immunity to each of the three diseases, or virus (HBV) infection:
documentation of provider-diagnosed measles or mumps disease. For rubella, documentation of Behavioral: Sexually active persons who are not in a long-term, mutually monogamous relationship
provider-diagnosed disease is not considered acceptable evidence of immunity. (e.g., persons with more than one sex partner during the previous 6 months); persons seeking evaluation
Measles component: A second dose of MMR vaccine, administered a minimum of 28 days after the first or treatment for a sexually transmitted disease (STD); current or recent injection-drug users; and men
dose, is recommended for adults who 1) have been recently exposed to measles or are in an outbreak who have sex with men.
setting; 2) are students in postsecondary educational institutions; 3) work in a health-care facility; or 4) plan Occupational: Health-care personnel and public-safety workers who are exposed to blood or other
to travel internationally. Persons who received inactivated (killed) measles vaccine or measles vaccine of potentially infectious body fluids.
unknown type during 19631967 should be revaccinated with 2 doses of MMR vaccine. Medical: Persons with end-stage renal disease, including patients receiving hemodialysis; persons
Mumps component: A second dose of MMR vaccine, administered a minimum of 28 days after the first with HIV infection; and persons with chronic liver disease.
dose, is recommended for adults who 1) live in a community experiencing a mumps outbreak and are in Other: Household contacts and sex partners of persons with chronic HBV infection; clients and staff
an affected age group; 2) are students in postsecondary educational institutions; 3) work in a health-care members of institutions for persons with developmental disabilities; and international travelers to
facility; or 4) plan to travel internationally. Persons vaccinated before 1979 with either killed mumps vaccine countries with high or intermediate prevalence of chronic HBV infection (a list of countries is available at
or mumps vaccine of unknown type who are at high risk for mumps infection (e.g. persons who are working http://wwwn.cdc.gov/travel/contentdiseases.aspx).
in a health-care facility) should be revaccinated with 2 doses of MMR vaccine. Hepatitis B vaccination is recommended for all adults in the following settings: STD treatment facilities;
Rubella component: For women of childbearing age, regardless of birth year, rubella immunity should be HIV testing and treatment facilities; facilities providing drug-abuse treatment and prevention services;
determined. If there is no evidence of immunity, women who are not pregnant should be vaccinated. health-care settings targeting services to injection-drug users or men who have sex with men; correctional
Pregnant women who do not have evidence of immunity should receive MMR vaccine upon completion or facilities; end-stage renal disease programs and facilities for chronic hemodialysis patients; and institutions
termination of pregnancy and before discharge from the health-care facility. and nonresidential day-care facilities for persons with developmental disabilities.
ADULT

Health-care personnel born before 1957: For unvaccinated health-care personnel born before 1957 who Administer missing doses to complete a 3-dose series of hepatitis B vaccine to those persons not
lack laboratory evidence of measles, mumps, and/or rubella immunity or laboratory confirmation of vaccinated or not completely vaccinated. The second dose should be administered 1 month after the first
disease, health-care facilities should 1) consider routinely vaccinating personnel with 2 doses of MMR dose; the third dose should be given at least 2 months after the second dose (and at least 4 months after
vaccine at the appropriate interval (for measles and mumps) and 1 dose of MMR vaccine (for rubella), and the first dose). If the combined hepatitis A and hepatitis B vaccine (Twinrix) is used, administer 3 doses at
2) recommend 2 doses of MMR vaccine at the appropriate interval during an outbreak of measles or 0, 1, and 6 months; alternatively, a 4-dose Twinrix schedule, administered on days 0, 7, and 21 to 30,
mumps, and 1 dose during an outbreak of rubella. Complete information about evidence of immunity is followed by a booster dose at month 12 may be used.
available at http://www.cdc.gov/vaccines/recs/provisional/default.htm. Adult patients receiving hemodialysis or with other immunocompromising conditions should receive 1
7. Pneumococcal polysaccharide (PPSV) vaccination dose of 40 g/mL (Recombivax HB) administered on a 3-dose schedule or 2 doses of 20 g/mL
Vaccinate all persons with the following indications: (Engerix-B) administered simultaneously on a 4-dose schedule at 0, 1, 2, and 6 months.
Medical: Chronic lung disease (including asthma); chronic cardiovascular diseases; diabetes mellitus; 12. Selected conditions for which Haemophilus influenzae type b (Hib) vaccine may be used
chronic liver diseases; cirrhosis; chronic alcoholism; functional or anatomic asplenia (e.g., sickle cell 1 dose of Hib vaccine should be considered for persons who have sickle cell disease, leukemia, or HIV
disease or splenectomy [if elective splenectomy is planned, vaccinate at least 2 weeks before surgery]); infection, or who have had a splenectomy, if they have not previously received Hib vaccine.
immunocompromising conditions (including chronic renal failure or nephrotic syndrome); and cochlear 13. Immunocompromising conditions
implants and cerebrospinal fluid leaks. Vaccinate as close to HIV diagnosis as possible. Inactivated vaccines generally are acceptable (e.g., pneumococcal, meningococcal, influenza
[inactivated influenza vaccine]) and live vaccines generally are avoided in persons with immune
deficiencies or immunocompromising conditions. Information on specific conditions is available at
http://www.cdc.gov/vaccines/pubs/acip-list.htm.

These schedules indicate the recommended age groups and medical indications for which administration of currently licensed vaccines is commonly indicated for adults ages 19 years and older, as of
January 1, 2011. For all vaccines being recommended on the adult immunization schedule: a vaccine series does not need to be restarted, regardless of the time that has elapsed between doses. Licensed
combination vaccines may be used whenever any components of the combination are indicated and when the vaccines other components are not contraindicated. For detailed recommendations on all
vaccines, including those used primarily for travelers or that are issued during the year, consult the manufacturers package inserts and the complete statements from the Advisory Committee on Immuniza-
tion Practices (http:// www.cdc.gov/vaccines/pubs/acip-list.htm).

Report all clinically significant postvaccination reactions to the Vaccine Adverse Event Reporting System (VAERS). Reporting forms and instructions on filing a VAERS report are available at
http://www.vaers.hhs.gov or by telephone, 800-822-7967.

Information on how to file a Vaccine Injury Compensation Program claim is available at http://www.hrsa.gov/vaccinecompensation or by telephone, 800-338-2382. Information about filing a claim for vaccine
injury is available through the U.S. Court of Federal Claims, 717 Madison Place, N.W., Washington, D.C. 20005; telephone, 202-357-6400.

Additional information about the vaccines in this schedule, extent of available data, and contraindications for vaccination also is available at http://www.cdc.gov/vaccines or from the CDC-INFO Contact
Center at 800-CDC-INFO (800-232-4636) in English and Spanish, 24 hours a day, 7 days a week.
Use of trade names and commercial sources is for identification only and does not imply endorsement by the U.S. Department of Health and Human Services.

U.S. Department of Health and Human Services Centers for Disease Control and Prevention

From http://www.cdc.gov/vaccines/recs/schedules/downloads/adult/2009/adult-schedule-6x4-5-pss.pdf
2164_Ch06_347-578 29/03/12 12:29 PM Page 355

Middle Life 355


usually at the end of young adulthood; when the
choice is deferred beyond these years, valuable time
Middle Life (46 to 64 Years
is lost. of Age)
F. Experiments with and chooses a lifestyle.
G. Forms mature peer relationships with the opposite I. STAGE OF DEVELOPMENTPSYCHOSOCIAL
sex. STAGE: continuation of generativity versus
H. Overcomes guilt and anxiety about the opposite self-absorption.
sex and learns to understand the masculine and II. PHYSICAL DEVELOPMENT
feminine aspects of self, as well as the adult concept A. Failing eyesight, especially for close vision, may be
of roles. one of the first symptoms of aging.
I. Learns to take the opposite sex seriously and may B. Hearing loss is very gradual, especially for low
choose someone for a long-term relationship. sounds; hearing for high-pitched sounds is impaired
J. Accepts the responsibilities and pleasures of more readily.
parenthood. C. There is a gradual loss of taste buds in the 50s and
gradual loss of sense of smell in the 60s, causing the
individual to have a diminished sense of taste.
Adulthood (31 to 45 Years D. Muscle strength declines because of decreased levels
of estrogen and testosterone; it takes more time to
of Age) accomplish the same physical task.
I. STAGE OF DEVELOPMENTPSYCHOSOCIAL E. Lung capacity is impaired, which adds to decreased
STAGE: generativity versus self-absorption. endurance.
F. The skin begins to wrinkle, and hair begins
II. PHYSICAL DEVELOPMENT
graying.
A. Gradual decline in biological functioning, although G. Postural changes take place because of loss of
in the late 30s the individual is still near peak. calcium and reduced activity.
B. Period of peak sexual capacity for women occurs
during the mid-30s. III. COGNITIVE DEVELOPMENT
C. Distinct sense of bodily decline occurs around A. Memory begins to decline slowly around age
40 years of age. 50 years.
D. Circulatory system begins to slow somewhat after B. It takes longer to learn new tasks, and old tasks
40 years of age. take longer to perform.
C. Practical judgment is increased due to experiential
III. COGNITIVE DEVELOPMENT
background.
A. Takes longer to memorize. D. May tend to withdraw from mental activity or
B. Still at peak in abstract thinking and overcompensate by trying the impossible.
problem-solving.
C. Generates new levels of awareness. IV. SOCIALIZATION
D. Gives more meaning to complex tasks. A. The middle years can be very rewarding if previous

ADULT
stages have been fulfilled.
IV. SOCIALIZATION
B. The years of responsibility for raising children are
A. Achieves a realistic self-identity. over.
B. Perceptions are based on reality. C. Husbands and wives usually find a closer bond.
C. Acts on decisions and assumes responsibility for D. There is less financial strain for those with steady
actions. employment.
D. Accepts limitations while developing assets. E. Individuals are usually at the height of their careers;
E. Delays immediate gratification in favor of future the majority of leaders in their field are in this age
satisfaction. group.
F. Evaluates mistakes, determines reasons and causes, F. Self-realization is achieved.
and learns new behavior. 1. There is more inner direction.
G. Struggles to establish a place in society. 2. There is no longer a need to please everyone.
1. Begins to settle down. 3. Individual is less likely to compare self with
2. Pursues long-range plans and goals. others.
3. Has a stronger need to be responsible. 4. Individual approves of self without being
4. Invests self as fully as possible in social structure, dependent on standards of others.
including work, family, and community. 5. There is less fear of failure in life because past
H. Seeks advancement by improving and using skills, failures have been met and dealt with.
becoming more creative, and pursuing ambitions.
2164_Ch06_347-578 29/03/12 12:29 PM Page 356

356 chapter 6 Physiological Integrity

Early Late Years (65 to B. Increase in loss of muscle tone occurs, including
sphincter (urinary and anal) control.
79 Years of Age) C. Individual is insecure and unsure about orientation
to space and sense of balance, which may result in
I. STAGE OF DEVELOPMENTPSYCHOSOCIAL falls and injury.
STAGE: ego integrity and acceptance versus despair
and disgust. III. COGNITIVE DEVELOPMENT
II. PHYSICAL DEVELOPMENT (see also Chapter 7) A. Has better memory for the past than the present.
B. Repetition of memories occurs.
A. Continues to decrease in vigor and capacity. C. Individual may use confabulation to fill in
B. Has more frequent aches and pains. memory gaps.
C. Likely to have at least one major illness. D. Forgetfulness may lead to serious safety problems,
III. COGNITIVE DEVELOPMENT and individual may require constant supervision.
A. Mental acuity continues to slow down. E. Increased arteriosclerosis may lead to mental illness
B. Judgment and problem-solving remain intact, but (dementia and other cognitive disorders).
the processes may take longer. IV. SOCIALIZATION
C. May have problems in remembering names and A. Few significant relationships are maintained;
dates. deaths of friends, family, and associates cause
IV. SOCIALIZATION isolation.
A. Individual is faced with the reality of the experience B. Individual may be preoccupied with immediate
of physical decline. bodily needs and personal comforts; the gastroin-
B. Physical and mental changes intensify the feelings testinal tract frequently becomes the major focus.
of aging and mortality. C. Individuals see that they can provide others with an
C. Increasing frequency of death and serious illness example of wisdom and courage.
among friends, relatives, and associates further D. Individuals come to terms with themselves.
reinforces the concept of mortality. E. Individuals are concerned with own mortality.
D. Constant reception of medical warnings to follow F. Individuals come to terms with the process of
certain precautions or run serious risks adds to dying and prepare for own death.
general feeling of decline.
E. Individual is less interested in obtaining the FLUID AND
rewards of society and is more interested in using E L E C T R O LY T E
own inner resources. IMBALANCES
F. Individuals feel that they have earned the right to
do what is important for self-satisfaction. Imbalances in fluid and electrolytes may be due to changes
G. Retirement allows time for expression of own in the total quantity of either substance (deficit or excess),
creative energies. protein deficiencies, or extracellular fluid volume shifts.
H. Overcomes the splitting of youth and age; gets Clients who are older and very young are particularly
ADULT

along well with adolescents. susceptible.


I. Learns to deal with the reality that only old age I. FLUID VOLUME DEFICIT (dehydration): mecha-
remains. nism that influences fluid balance and sodium levels;
J. Provides moral support to grandchildren; more tol- decreased quantities of fluid and electrolytes may be
erant of grandchildren than was of own children. caused by deficient intake (poor dietary habits,
K. Tends to release major authority of family to chil- anorexia, and nausea), excessive output (vomiting,
dren while holding self in the role of consultant. nasogastric suction, and prolonged diarrhea), or
failure of regulatory mechanism that influences fluid
balance and sodium levels.
Later Years (80 Years of Age A. Pathophysiology: water moves out of the cells to
and Older) replace a significant water loss; cells eventually
become unable to compensate for the lost fluid,
I. STAGE OF DEVELOPMENTPSYCHOSOCIAL and cellular dehydration begins, leading to circula-
STAGE: continuation of ego integrity and acceptance tory collapse.
versus despair and disgust. B. Risk factors:
II. PHYSICAL DEVELOPMENT (see also Chapter 7) 1. No fluids available.
A. Additional sensory problems occur, including 2. Available fluids not drinkable.
diminished sensation to touch and pain. 3. Inability to take fluids independently.
2164_Ch06_347-578 29/03/12 12:29 PM Page 357

Fluid and Electrolyte Imbalances 357


4. No response to thirst; does not recognize the d. Avoid hypertonic solutions (may cause fluid
need for fluids. shift when compensatory mechanisms begin
5. Inability to communicate need; does not speak to function).
same language. 2. Goal: promote comfort.
6. Aphasia. a. Frequent skin care (lack of hydration causes
7. Weakness, comatose. dry skin, which may increase risk for skin
8. Inability to swallow. breakdown).
9. Psychological alterations. b. Position: change every hour to relieve pressure.
10. Overuse of diuretics. c. Medications as ordered: antiemetics,
11. Increased vomiting. antidiarrheal.
12. Fever. 3. Goal: prevent physical injury.
13. Wounds, burns. a. Frequent mouth care (mucous membrane dries
14. Blood loss. due to dehydration; therefore, client is at risk
15. Endocrine abnormalities (e.g., diabetes for breaks in mucous membrane, halitosis).
insipidus). b. Monitor IV flow rateobserve for circulatory
16. Diarrhea. overload, pulmonary edema related to poten-
C. Assessment: tial fluid shift when compensatory mecha-
1. Subjective data: nisms begin or client is unable to tolerate rate
a. Thirst. of fluid replacement.
b. Behavioral changes: apprehension, apathy, c. Monitor vitals, including level of conscious-
lethargy, confusion, restlessness. ness (decreasing BP and level of consciousness
c. Dizziness. indicate continuation of fluid loss).
d. Numbness and tingling of hands and feet. d. Prepare for surgery if hemorrhage present
e. Anorexia and nausea. (internal bleeding can only be relieved by
f. Abdominal cramps. surgical intervention).
2. Objective data: F. Evaluation/outcome criteria:
a. Sudden weight loss of 5%. 1. Mentally alert.
b. Vital signs: 2. Moist, intact mucous membranes.
(1) Decreased BP; postural changes. 3. Urinary output approximately equal to intake.
(2) Increased temperature. 4. No further weight loss.
(3) Irregular, weak, rapid pulse. 5. Gradual weight gain.
(4) Increased rate and depth of respirations. II. FLUID VOLUME EXCESS (fluid overload): most
c. Skin: cool and pale in absence of infection; common cause is an increase in sodium; excessive
decreased turgor. quantities of fluid and electrolytes may be due to
d. Urine: oliguria to anuria, high specific increased ingestion, tube feedings, intravenous infu-
gravity. sions, multiple tap-water enemas, or a failure of regula-
e. Eyes: soft, sunken. tory systems, resulting in inability to excrete excesses.

ADULT
f. Tongue: furrows.
A. Pathophysiology: hypo-osmolar water excess in
g. Laboratory data:
extracellular compartment leads to intracellular
(1) Bloodincreased hematocrit and blood
water excess because the concentration of solutes
urea nitrogen (BUN).
in the intracellular fluid is greater than that in the
(2) Urinedecreased 17-ketosteroids;
extracellular fluid. Water moves to equalize concen-
increased specific gravity, dark-colored
tration, causing swelling of the cells. The most
urine.
common cause is an increase in sodium.
D. Analysis/nursing diagnosis:
B. Risk factors:
1. Fluid volume deficit related to inadequate fluid
1. Excessive intake of electrolyte-free fluids.
intake.
2. Increased secretion of antidiuretic hormone
E. Nursing care plan/implementation:
(ADH) in response to stress, drugs, anesthetics
1. Goal: restore fluid and electrolyte balance
(Table 6.4).
increase fluid intake to hydrate client.
3. Decreased or inadequate output of urine.
a. IVs and blood products as ordered; small,
4. Psychogenic polydipsia.
frequent drinks by mouth.
5. Certain medical conditions: tuberculosis;
b. Daily weights (same time of day) to monitor
encephalitis; meningitis; endocrine disturbances;
progress of fluid replacement.
tumors of lung, pancreas, duodenum; heart failure.
c. I&O, hourly outputs (when in acute
6. Inadequate kidney function or kidney failure.
state).
2164_Ch06_347-578 29/03/12 12:29 PM Page 358

358 chapter 6 Physiological Integrity

Table 6.4
Diabetes Insipidus (DI) Versus Syndrome of Inappropriate Antidiuretic Hormone (SIADH)
DI (Fluid Deficit) SIADH (Fluid Excess)
Pathophysiology: Deficiency of ADH inability to Excessive ADH secreted water retention,
conserve H2O hyponatremia, and hypo-osmolality
Large volumes of hypotonic fluid
excreted
Risk factors: Head injury Vasopressin overuse (Rx of DI) or stimulation
Brain infection (chemotherapy)
Posterior pituitary tumors Small cell carcinoma
Drugs that inhibit vasopressin Adrenal insufficiency
(e.g., glucocorticoids, phenytoin, Myxedema
lithium) Anterior pituitary insufficiency
Assessment: Polyuria (up to 18 L/day) Signs of Na+fatigue, headache, DTR,
Urine specific gravity nausea, anorexia, mental status
Signs of fluid volume deficit: dry, Signs of fluid volume excess: weight gain with-
cool skin; polydipsia; weight out edema; tachycardia; tachypnea; crackles
Laboratory: serum Na+ initially, then
; serum osmolality initially, then
Nursing care Fluids: IV; I/O Fluids: hypertonic IV ( Na+); I/O
plan/implementation: Medications: ADH replace- Medications: diuretics; demeclocycline
ment; Pitressin (tetracycline)
Education: prepare for Daily weight
hypophysectomy

C. Assessment: b. Vital signs: prn, minimum q4h.


1. Subjective data: c. Fluid restriction.
a. Behavioral changes: irritability, apathy, 2. Goal: promote excretion of excess fluid.
confusion, disorientation. a. Medications as ordered: diuretics.
b. Headache. b. Monitor electrolytes, especially Mg++, K+.
c. Anorexia, nausea, cramping. c. If in kidney failure: may need dialysis; explain
d. Fatigue. procedure.
e. Dyspnea. d. Assist client during paracentesis, thoracentesis,
2. Objective data: phlebotomy.
a. Vital signs: elevated blood pressure. (1) Monitor vital signs to detect shock.
b. Skin: warm, moist; edemaeyelids, facial, (2) Prevent injury by monitoring sterile
dependent, pitting. technique.
ADULT

c. Sudden weight gain of 5 lb. (3) Prevent falling by stabilizing appropriate


d. Pink, frothy sputum; productive. position during procedure.
e. Constant, irritating cough. (4) Support client psychologically.
f. Crackles in lungs. 3. Goal: obtain/maintain fluid balance.
g. Pulse: bounding. a. Daily weights; 1 kg = 1,000 mL fluid.
h. Engorgement of neck veins in sitting b. Measure: all edematous parts, abdominal
position. girth, I&O.
i. Urine: polyuria, nocturia, pale color. c. Limit: fluids by mouth, IVs, sodium.
j. Laboratory data: d. Strict monitoring of IV fluids.
(1) Blooddecreasing hematocrit, BUN. 4. Goal: prevent tissue injury.
(2) Urinedecreasing specific gravity. a. Skin and mouth care as needed.
D. Analysis/nursing diagnosis: b. Evaluate feet for edema and discoloration
1. Fluid volume excess related to excessive fluid when client is out of bed.
intake or decreased fluid output. c. Observe suture line on surgical clients
E. Nursing care plan/implementation: (potential for evisceration due to excess
1. Goal: maintain oxygen to all cells. fluid retention).
a. Position: semi-Fowlers or Fowlers to facilitate d. IV route preferred for parenteral medications;
improved gas exchange. Z track if medications are to be given IM
2164_Ch06_347-578 29/03/12 12:29 PM Page 359

Common Electrolyte Imbalances 359


(otherwise injected liquid will escape through of oral or IV potassium. Serum potassium level
injection site). above 5 mEq/L; symptoms usually occur when
5. Goal: health teaching. above 6.5 mEq/L.
a. Improve nutritional status with low-sodium C. Calcium (Ca++): normal 4.5 to 5.5 mEq/L.
diet. Essential to muscle metabolism, cardiac function,
b. Identify cause that put client at risk for and bone health. Controlled by parathyroid
imbalance, methods to avoid this situation hormone; reciprocal relationship between calcium
in the future. and phosphorus.
c. Desired and side effects of diuretics and other 1. Hypocalcemialoss of calcium related to
prescribed medications. inadequate intake, vitamin D deficiency,
d. Monitor urinary output, ankle edema; report hypoparathyroidism, damage to the parathyroid
to health-care manager when fluid retention is gland, decreased absorption in the GI tract,
noticed. excess loss through kidneys. Serum calcium
e. Limit fluid intake when kidney/cardiac level below 4.5 mEq/L.
function impaired. 2. Hypercalcemiacalcium excess related to
F. Evaluation/outcome criteria: hyperparathyroidism, immobility, bone tumors,
1. Fluid balance obtained. renal failure, excess intake of Ca++ or vitamin D.
2. No respiratory, cardiac complications. Serum calcium level above 5.5 mEq/L.
3. Vital signs within normal limits. D. Magnesium (Mg++): normal 1.5 to 2.5 mEq/L.
4. Urinary output improved, no evidence of edema. Essential to cellular metabolism of carbohydrates
III. COMMON ELECTROLYTE IMBALANCES: and proteins.
Electrolytes are taken into the body in foods and 1. Hypomagnesemiamagnesium deficit related to
fluids; normally lost through sweat and urine. impaired absorption from GI tract, excessive loss
May also be lost through hemorrhage, vomiting, through kidneys, and prolonged periods of poor
and diarrhea. Electrolytes have major influences on: nutritional intake. Hypomagnesemia leads to
body water regulation and osmolality, acid-base neuromuscular irritability. Serum magnesium
regulation, enzyme reactions, and neuromuscular level below 1.5 mEq/L.
activity. Clinically important electrolytes are: 2. Hypermagnesemiamagnesium excess related to
renal insufficiency, overdose during replacement
A. Sodium (Na+): normal 135 to 145 mEq/L. Most
therapy, severe dehydration, repeated enemas
prevalent cation in extracellular fluid. Controls
with Mg++ sulfate (epsom salts). Serum magne-
osmotic pressure; essential for neuromuscular func-
sium level above 2.5 mEq/L.
tioning and intracellular chemical reactions. Aids in
E. Table 6.5 provides assessment, analysis/nursing
maintenance of acid-base balance. Necessary for
diagnosis, nursing care plan/implementation, and
glucose to be transported into cells.
evaluation/outcome criteria of the various elec-
1. Hyponatremiasodium deficit, resulting from
trolyte imbalances.
either a sodium loss or water excess. Serum
sodium level below 135 mEq/L; symptoms IV. ACID-BASE BALANCE: Concentration of hydro-

ADULT
usually do not occur until below 120 mEq/L gen ions in extracellular fluid is determined by the
unless rapid drop. ratio of bicarbonate to carbonic acid. The normal
2. Hypernatremiaexcess sodium in the blood, ratio is 20:1. Even when arterial blood gases are
resulting from either high sodium intake, water abnormal, if the ratio remains at 20:1, no imbalance
loss, or low water intake. Serum sodium level will occur. Table 6.6 shows blood gas variations with
above 145 mEq/L. acid-base imbalances.
B. Potassium (K+): normal 3.5 to 5.0 mEq/L. A. Causes of blood gas abnormalities: see
Direct effect on excitability of nerves and muscles. Table 6.7.
Contributes to intracellular osmotic pressure and B. Types of acid-base imbalance:
influences acid-base balance. Major cation of the cell. 1. Acidosis: hydrogen ion concentration increases
Required for storage of nitrogen as muscle protein. and pH decreases.
1. Hypokalemiapotassium deficit related to 2. Alkalosis: hydrogen ion concentration decreases
dehydration, starvation, vomiting, diarrhea, and pH increases.
diuretics. Serum potassium level below 3. Metabolic imbalances: bicarbonate is the
3.5 mEq/L; symptoms may not occur until problem. In primary conditions, the level of
below 2.5 mEq/L. bicarbonate is directly proportional to pH.
2. Hyperkalemiapotassium excess related to severe a. Metabolic acidosis: excessive acid is produced
tissue damage, renal disease, excess administration or added to the body, bicarbonate is lost, or
(text continues on page 363)
ADULT

Table 6.5 360


Electrolyte Imbalances
Assessment
Disorder and Analysis/Nursing Nursing Care Evaluation/
Related Condition Subjective Data Objective Data Diagnosis Plan/Implementation Outcome Criteria
Hyponatremia
Addisons disease Apathy, apprehen- Pulse: rapid and weak Diarrhea Obtain normal sodium level: Na+ 135145 mEq/L
Starvation sion, mental confu- BP: postural hypotension Fluid volume excess identify cause of deficit, No complications of
GI suction sion, delirium Shock, coma Altered nutrition, less increase sodium intake PO (salty shock present
Thiazide diuretics Fatigue GI: weight loss, diarrhea, than body requirements foods), IVshypertonic Return of muscle
Excess water intake, Vertigo, headache loss through NG tubes Sensory/perceptual solutions strength
enemas Anorexia, nausea alteration (kinesthetic) Prevent further sodium loss: irri- Alert, oriented
Muscle weakness
Fever Abdominal and gate NG tubes with saline; hourly Limits intake of plain
2164_Ch06_347-578 29/03/12 12:29 PM Page 360

Fluid shifts muscle cramps I&O to monitor kidney output water


Ascites Prevent injury related to shock,
Burns dizziness, decreased sensorium;
Small bowel dangle before ambulation
chapter 6 Physiological Integrity

obstruction Skin care


Profuse perspiration
Hypernatremia
High sodium intake Lethargy BP and temperature: Fluid volume deficit Obtain normal sodium level: Na+ 135145 mEq/L
Low water intake Restlessness, elevated Fluid volume excess decrease sodium intake No complaint of thirst
Diarrhea agitation Neuromuscular: dimin- Altered nutrition, less I&O to recognize signs and symp- Alert, oriented
High fever with rapid Confusion ished reflexes than body requirements toms of complications (e.g., heart Relaxed in appearance
respirations Skin: flushed; firm turgor Sensory/perceptual failure, pulmonary edema) Identifies high-sodium
Impaired renal GI: mucous membrane alteration (kinesthetic) foods to avoid
functions dry, sticky K+ 3.55.0 mEq/L
Acute GU: decreased output
tracheobronchitis
Hypokalemia
Decreased intake: Apathy, lethargy, Muscles: weakness, Decreased cardiac output Replace lost potassium: Identifies cause of
Poor potassium food fatigue, weakness paralysis, paresthesia, Fatigue increase potassium in diet imbalance
intake Irritability, mental hyporeflexia Altered cardiopulmonary (see Chapter 9); liquid PO Lists foods to include
Excessive dieting confusion Respirations: shallow to tissue perfusion potassium medications in diet
Nausea Anorexia, nausea respiratory arrest Ineffective breathing dilute in juice to aid taste; Lists signs and symp-
Alcoholism Leg cramps Cardiac: decreased BP; patterns give potassium only if kidneys toms of imbalance
IV fluids without elevated, weak, irregular Constipation functioning Return of muscle
added potassium pulse; arrhythmias Bathing/hygiene self- Prevent injury to tissues: prevent strength
ECG: low, flat T waves; care deficit infiltration, pain, tissue damage No cardiac arrhythmias
Increased loss:
prolonged ST segment; Impaired home mainte- Prevent potassium loss:
GI suctioning,
elevated U wave; poten- nance management Irrigate NG tubes with saline, not
vomiting, diarrhea
tial arrest Sensory/perceptual water
Ulcerative colitis
GI: vomiting, flatulence, alteration (gustatory)
Drainage: ostomy,
constipation; decreased
fistulas
motility distention
Medications:
paralytic ileus
potassium-losing
GU: urine not
diuretics, digoxin,
concentrated; polyuria,
cathartics
nocturia; kidney damage
Increased aldosterone
Speech: slow
production
Renal disorders
Hyperkalemia
Burns Irritability Muscles: paresthesia, flac- Decreased cardiac output Decrease amount of K+ 3.55.0 mEq/L
Crushing injuries Weakness, muscle cid muscle paralysis (later) Altered urinary elimination potassium in body; identify No complications (e.g.,
Kidney disease cramps Cardiac; irregular pulse; Activity intolerance and treat cause of imbalance; arrhythmias, acidosis,
Excessive infusion Nausea, intestinal arrhythmias; bradycardia Ineffective breathing give foods low in K+; avoid drugs respiratory failure)
or ingestion of K+ cramps asystole patterns or IV fluids containing K+
Adrenal insufficiency ECG: high T waves; Diarrhea If kidney failure present, may need
Mercurial poisoning depressed ST segment; Impaired home mainte- to prepare for dialysis
widened QRS complex; nance management
diminished or absent P
waves; ventricular
fibrillation
GI: explosive diarrhea;
hyperactive bowel sounds
2164_Ch06_347-578 29/03/12 12:29 PM Page 361

Kidney: scanty to no urine


Hypocalcemia
Acute pancreatitis Fatigue Muscle spasms: tonic Pain Prevent tetany (medical Ca++ 4.55.5 mEq/L
Diarrhea Tingling/numbness; muscles, carpopedal, Diarrhea emergency): calcium No signs of tetany
Peritonitis fingers and laryngeal Altered nutrition, less gluconate IV, 2.55.0 mL Absent Trousseaus
Damage to circumoral Neuromuscular: grimac- than body requirements 10% solution; repeated q10 min and Chvosteks signs
parathyroid during Abdominal cramps ing, hyperirritable facial Risk for injury to maximum dose of 30 mL Lists foods high in
thyroidectomy Palpitations nerves Sensory/perceptual Prevent tissue injury due to vitamin D and calcium
Hypothyroidism Dyspnea Tetany convulsions alteration (gustatory) hypoxia and sloughing; administer
Burns Orthopedic: osteoporosis slowly; avoid infiltration
Pregnancy and fractures Prevent injury related to
lactation Cardiac: arrhythmias medication administration.
Low vitamin D arrest Caution: drug interaction
intake GI: diarrhea with carbonate, phosphate,
Multiple blood digitalis; avoid hypercalcemia
transfusions In less acute condition:
Renal disorders increase calcium intake
Massive infection calcium gluconate or lactate
Hypercalcemia
Parathyroid glands: Pain: flank, deep Muscles: relaxed Decreased cardiac output Reduce calcium intake: Ca++ 4.55.5 mEq/L
overactive, tumor bone, shin splints GU: kidney stones Constipation decrease foods high in No pain reported
Increased immobility Muscle weakness, GI: increased milk intake, Activity intolerance calcium; identify cause of No fractures/calculi
Decreased renal fatigue constipation, dehydration Altered urinary elimination imbalance; give steroids, seen on x-ray
function Anorexia, nausea Neurological: stupor Pain diuretics as ordered; examination
Bone cancer Headache coma isotonic saline IV
Increased vitamin D Thirst polyuria Prevent injury: prevent pathological
and calcium intake fractures (e.g., advanced cancer);
Milk-alkali prevent renal calculi by increasing
syndrome fluid intake
self-administration
of antacids;
increased milk in
diet to relieve GI
Electrolyte Imbalances

symptoms
Continued
361

ADULT
ADULT

362

Table 6.5
Electrolyte Imbalancescontd
Assessment
2164_Ch06_347-578 29/03/12 12:29 PM Page 362

Disorder and Analysis/Nursing Nursing Care Evaluation/


Related Condition Subjective Data Objective Data Diagnosis Plan/Implementation Outcome Criteria
Hypomagnesemia
Impaired GI Agitation Muscles: irritable, Risk for injury related to Provide safety: prevent injury Mg++ 1.52.5 mEq/L
chapter 6 Physiological Integrity

absorption Depression tremors, spasticity, seizure activity to client who is disoriented;


Prolonged Confusion tetany Decreased cardiac administer magnesium salts
malnutrition or Paresthesia convulsions output PO or IV
starvation Cardiac: arrhythmias, Health teaching: prevention;
Alcoholism tachycardia diethigh-magnesium foods:
Excess loss of fruits, green vegetables,
magnesium through whole-grain cereals, milk,
kidneys, related meats, nuts
to increased
aldosterone
production
Prolonged diarrhea
Draining GI fistulas
Hypermagnesemia
Renal failure Drowsiness, lethargy Neuromuscular: loss of Ineffective breathing Obtain normal magnesium Mg++ 1.52.5 mEq/L
Diabetic deep tendon reflexes pattern level: IV calcium, fluids; No complications
ketoacidosis Respiratory: depression Decreased cardiac output possible dialysis (e.g., respiratory
Severe dehydration Cardiac: arrest, Fluid volume deficit depression,
Antacid therapy hypotension Fluid volume excess arrhythmias)
Altered cardiopulmonary Identifies magnesium-
tissue perfusion based antacids
(e.g., Gelusil)
Deep tendon reflexes 2+
2164_Ch06_347-578 29/03/12 12:29 PM Page 363

Blood Gas Abnormalities 363

Table 6.6
Blood Gas Variations with Acid-Base Imbalances
Value with:
Blood Gas Feature Normal Value Respiratory Acidosis Respiratory Alkalosis Metabolic Acidosis Metabolic Alkalosis
HCO3 (bicarbonate) 2226 mm Hg Normal or Normal or
PCO2 3545 mm Hg Normal or Normal or
(Carbonic acid*) (1.051.35) Normal or Normal or
pH (hydrogen ion 7.357.45
concentration)
= increased; = decreased.
*To obtain carbonic acid level, multiply PCO2 value by 0.03.

Table 6.7 acid is retained due to poorly functioning


kidneys. Deficit of bicarbonate.
Blood Gas Abnormalities: Causes b. Metabolic alkalosis: excessive acid is lost or
Decreased PO2 Collapsed alveoli (atelectasis) bicarbonate or alkali is retained. Excess of
1. Airway obstruction bicarbonate.
a. By the tongue
b. By a foreign body
c. As compensatory mechanism, PCO2 will be
2. Failure to take deep breaths low in metabolic acidosis, as the body attempts
a. Pain (rib fracture, pleurisy) to eliminate excess carbonic acid and elevate
b. Paralysis of respiratory muscles pH. PCO2 will become elevated in metabolic
(spinal cord injury, polio) alkalosis.
c. Depression of the respiratory
center (head injury, drug
4. Respiratory imbalances: carbonic acid is the prob-
overdose) lem. In primary conditions, PCO2 is inversely
3. Collapse of the whole lung proportional to the pH.
(pneumothorax) a. Respiratory acidosis: pulmonary ventilation
Fluid in the alveoli decreases, causing an elevation in the level
1. Pulmonary edema of carbon dioxide or carbonic acid. Excess
2. Pneumonia of PCO2.
3. Near-drowning
4. Chest trauma
b. Respiratory alkalosis: pulmonary ventilation
increases, causing a decrease in the level
Other gases in the alveoli of carbon dioxide or carbonic acid. Deficit
1. Smoke inhalation
2. Inhalation of toxic chemicals
of PCO2.

ADULT
3. Carbon monoxide poisoning c. As a compensatory mechanism, the level
Respiratory arrest
of bicarbonate will increase in respiratory
acidosis and decrease in respiratory alkalosis.
Elevated PCO2 Decreased CO2 elimination C. Assessment: see Table 6.8.
(hypoventilation)
1. Decreased tidal volume
D. Analysis/nursing diagnosis:
a. Pain (rib fractures, pleurisy) 1. Impaired gas exchange related to
b. Weakness (myasthenia hyperventilation.
gravis) 2. Ineffective breathing pattern related to
c. Paralysis (spinal cord injury, decreased thoracic movements.
polio)
2. Decreased respiratory rate
3. Ineffective airway clearance related to retained
a. Head injury secretions.
b. Depressant drugs 4. Risk for injury related to poorly functioning
c. Stroke kidneys.
Increased CO2 production 5. Altered renal tissue perfusion related to
1. Fever dehydration.
2. Muscular exertion 6. Altered urinary elimination related to renal
3. Anaerobic metabolism
failure.
2164_Ch06_347-578 29/03/12 12:29 PM Page 364

364 chapter 6 Physiological Integrity

Table 6.8
Acid-Base Imbalances
Assessment
Disorder and Nursing Care Evaluation/Outcome
Related Conditions Subjective Data Objective Data Plan/Implementation Criteria
Respiratory Acidosis
Acute bronchitis Headache Hypoventilation: rate Assist with normal Normal acid-base
Emphysema Irritability or rapid and shallow breathing: encourage balance obtained
Respiratory obstruction Disorientation Cyanosis coughing; suction airway; Respiratory rate: 1620
Atelectasis Weakness Tachycardia postural drainage; No signs of pulmonary
Damage to respiratory Dyspnea on Diaphoresis pursed-lip breathing; infection (e.g., sputum
center exertion Dehydration raise HOB colorless, breath
Pneumonia Nausea Coma (CO2 narcosis) Protect from injury: sounds clear)
Asthmatic attack Hyperventilation to oxygen as needed; Demonstrates
Drug overdose compensate if no encourage fluids; breathing exercises
pulmonary pathology avoid sedation; (e.g., diaphragmatic
present medications as breathing)
HCO3, normal orderedantibiotics,
PaCO2, elevated bronchial dilators
pH <7.35 Health teaching: identify
cause, prevent future
episodes; increase
awareness regarding risk
factors and early signs of
impending imbalance;
encourage compliance
Metabolic Acidosis
Diabetic ketoacidosis Headache Kussmauls respirations: Restore normal Normal acid-base
Hyperthyroidism Restlessness deep, rapid air hunger metabolism: balance obtained
Severe infections Apathy, Temperature correct underlying No rebound respiratory
Lactic acidosis in shock weakness Vomiting, diarrhea problem; sodium alkalosis following
Renal failure uremia Disorientation Dehydration bicarbonate PO/IV; sodium therapy
Prolonged starvation Thirst Stupor convulsions lactate; fluid replacement, No tetany following
diet; low-protein diet Nausea, coma Ringers solution; diet: return of normal pH
Diarrhea, dehydration abdominal HCO3 below normal high calorie Alert, oriented
Hepatitis pain PaCO2 normal Prevent No signs of K+ excess
Burns K+ >5 complications:
pH <7.35 regular insulin for
ketoacidosis;
hourly outputs; prepare
for dialysis if in kidney
failure
ADULT

Health teaching: identify


signs and symptoms of
primary illness, prevent
complications, cardiac
arrest; diet instructions
Respiratory Alkalosis
HyperventilationCO2 Circumoral Increased respirations Increase carbon dioxide Normal acid-base
loss paresthesia Increased neuromuscu- level: rebreathing into a balance obtained
Hypoxia, high altitudes Weakness lar irritability; hypere- paper bag; adjusting res- Recognizes psycho-
Fever Apprehension flexia, muscle twitching, pirator for CO2 retention logical and environ-
Metabolic acidosis tetany, positive and oxygen inspired; mental factors
Increased ICP, Chvosteks sign correct hypoxia causing condition
encephalitis Convulsions Prevent injury: safety Respiratory rate
Salicylate poisoning Unconsciousness measures for those returns to normal
After intensive exercise Hypokalemia who are unconscious; limits
HCO3 normal hypothermia for elevated No cardiac
PaCO2 decreased temperature arrhythmias
pH >7.45 Health teaching: recog- Alert, oriented
nize stressful events;
counseling if problem is
hysteria
2164_Ch06_347-578 29/03/12 12:29 PM Page 365

The Perioperative Experience 365

Table 6.8
Acid-Base Imbalancescontd
Assessment
Disorder and Nursing Care Evaluation/Outcome
Related Conditions Subjective Data Objective Data Plan/Implementation Criteria
Metabolic Alkalosis
Potassium deficiencies Lethargy Respirations: shallow; Obtain, maintain acid- Normal acid-base
Vomiting Irritability apnea, decreased base balance: irrigate balance obtained
GI suctioning Disorientation thoracic movement; NG tubes with No signs of potassium
Intestinal fistulas Nausea cyanosis saline; monitor deficit
Inadequate electrolyte Pulse: irregular I&O; IV saline, Respiratory rate:
replacement cardiac arrest potassium added; 1620
Increased use of antacids Muscles: twitching isotonic solutions PO; No arrhythmias
Diuretic therapy, steroids tetany, convulsions monitor vital signs pulse regular
Increased ingestion/ GI: vomiting, diarrhea, Prevent physical injury: Lists food sources
injection of bicarbonates paralytic ileus monitor for potassium high in potassium
HCO3 elevated above 26 loss, side effects of
PaCO2 normal, medications
K+ <3.5 Health teaching:
pH >7.45 increase sodium
when loss expected;
instructions regarding
self-administration
of medications
(e.g., baking soda)

7. Fluid volume excess related to altered kidney (2) Nature of anesthesia (e.g., fears of going
function. to sleep and not waking up, saying or
8. Fluid volume deficit related to diarrhea or revealing things of a personal nature).
dehydration. (3) Degree of pain (e.g., may be
9. Knowledge deficit (learning need) related incapacitating).
to self-administration of antacid (4) Misunderstandings regarding prognosis.
medications. h. Identification of significant others as a
E. Nursing care plan/implementation: see source of client support or care responsibilities
Table 6.8. postdischarge.
F. Evaluation/outcome criteria: see Table 6.8. 2. Objective data:
a. Speech patterns indicating anxiety
repetition, changing topics, avoiding
T H E P E R I O P E R AT I V E

ADULT
talking about feelings.
EXPERIENCE b. Interactions with otherswithdrawn or
I. PREOPERATIVE PREPARATION involved.
A. Assessment: c. Physical signs of anxiety (i.e., increased
1. Subjective data: pulse, respirations; clammy palms,
a. Understanding of proposed surgerysite, restlessness).
type, extent of hospitalization. d. Baseline physiological status: vital signs;
b. Previous experiences with hospitalization. breath sounds; peripheral circulation; weight;
c. Age-related factors. hydration status (hematocrit, skin turgor,
d. Allergiesiodine, latex, adhesive tape, urine output); degree of mobility; muscle
cleansing solutions, medications. strength.
e. Medication/substance useprescribed, B. Analysis/nursing diagnosis:
OTC, smoking, alcohol, recreational 1. Anxiety related to proposed surgery.
drugs. 2. Knowledge deficit (learning need) related to
f. Cultural and religious background. incomplete teaching or lack of understanding.
g. Concerns or feelings about surgery: 3. Fear related to threat of death or disfigurement.
(1) Exaggerated ideas of surgical risk 4. Risk for injury related to surgical complications.
(e.g., fear of colostomy when none is 5. Ineffective individual coping related to
being considered). anticipatory stress.
2164_Ch06_347-578 29/03/12 12:29 PM Page 366

366 chapter 6 Physiological Integrity

C. Nursing care plan/implementation: b. Coughinghelps clear chest of secretions


1. Goal: reduce preoperative and intraoperative and, although uncomfortable, will not harm
anxiety and prevent postoperative complications. incision site.
a. Preoperative teaching: (1) Have client lean forward slightly from a
(1) Provide information about hospital sitting position, and place clients hands
nursing routines and preoperative over incisional site; this acts as a splint
procedures to reduce fear of unknown. during coughing.
(2) Explain purpose of diagnostic procedures (2) Have client inhale and exhale slowly
to enhance ability to cooperate and several times.
tolerate procedure. (3) Have client inhale deeply, hold breath
(3) Inform about what will occur and what 3 seconds, and cough sharply three times
will be expected in the postoperative while exhalingclients mouth should be
period: slightly open.
(a) Will return to room, postanesthesia (4) Tell client to inhale again and to cough
care unit, or intensive care unit deeply once or twice. If unable to cough
(ICU). deeply, client should huff cough to
(b) Special equipmentmonitors, tubes, stimulate cough.
suction equipment. c. Turning and leg exerciseshelp prevent
(c) Pain control methods. circulatory stasis, which may lead to
b. Management of latex allergy if present. Three thrombus formation, and postoperative
forms: immediate reaction (most serious, life flatus or gas pains, as well as respiratory
threatening)flushing, diaphoresis, pruritus, problems.
nausea, vomiting, cramping, hypotension, (1) Tell client to turn on one side with upper-
dyspnea; delayed response (most common, most leg flexed; use side rails to facilitate
discomfort)localized symptoms 18 to the movement.
24 hours after contact; and contact dermati- (2) In a supine position, have client do five
tis. Exposure to latex through skin, mucous repetitions every hour of: ankle pumps,
membranes, inhalation, internal tissue, quadriceps-setting exercises, gluteal tight-
and intravascular; sources include: gloves, enings, and straight-leg raises.
anesthesia masks, tourniquets, ECG elec- (3) Apply intermittent pulsatile compression
trodes, adhesive tape, warming blankets, device or sequential compression device to
elastic bandages, tubes/catheters, irrigation promote venous return.
syringes. Nursing goal: provide latex-free 3. Goal: reduce the number of bacteria on the skin
environment. to eliminate incision contamination. Skin
2. Goal: instruct in exercises to reduce preparation:
complications. a. Prepare area of skin wider and longer than
a. Diaphragmatic breathingrefers to flattening proposed incision in case a larger incision is
ADULT

of diaphragm during inspiration, which necessary.


results in enlargement of upper abdomen; b. Gently scrub with an antimicrobial agent.
during expiration the abdominal muscles are (1) Note possibility of allergy to iodine.
contracted, along with the diaphragm. (2) Hexachlorophene should be left on the
(1) The client should be in a flat, semi- skin for 5 to 10 minutes.
Fowlers, or side position, with knees flexed (3) If benzalkonium Cl (Zephiran) solution is
and hands on the midabdomen. ordered, do not soap skin before use; soap
(2) Have the client take a deep breath through reduces effectiveness of benzalkonium by
nose and mouth, letting the abdomen rise. causing it to precipitate.
Hold breath 3 to 5 seconds. c. Hair should remain unless it interferes with
(3) Have client exhale through nose and surgical procedure.
mouth, squeezing out all air by (1) Note any nicks, cuts, or irritations,
contracting the abdominal muscles. potential infection sites.
(4) Repeat 10 to 15 times, with a short rest (2) Depilatory creams or clipping of hair
after each 5 to prevent hyperventilation. is preferred to shaving with a razor;
(5) Inform client that this exercise will be nick may result in cancellation of
repeated 5 to 10 times every hour surgery.
postoperatively. (3) Skin prep may be done in surgery.
2164_Ch06_347-578 29/03/12 12:29 PM Page 367

The Perioperative Experience 367


4. Goal: reduce the risk of vomiting and aspiration care); give valuable personal items to
during anesthesia; prevent contamination of family; chart disposition of items.
abdominal operative sites by fecal material. c. Proper identificationcheck band for secure-
Gastrointestinal tract preparation: ness and legibility; surgical site (limb) may be
a. No food or fluid at least 6 to 8 hours before marked to prevent error.
surgery. d. Vital signsbaseline data.
b. Remove food and water from bedside. e. Void, to prevent distention and possible
c. Place NPO signs on bed or door. injury to bladder.
d. Inform kitchen and oncoming nursing staff f. Give preoperative medication to ensure
that client is NPO for surgery. smooth induction and maintenance of
e. Give IV infusions up to time of surgery if anesthesiaantianxiety (e.g., midazolam,
dehydrated or malnourished. diazepam, lorazepam); narcotics (e.g., meperi-
f. Enemas: two or three may be given the dine, morphine, fentanyl); anticholinergics
evening before surgery with intestinal, colon, (e.g., atropine, glycopyrrolate).
or pelvic surgeries; 3 days of cleansing with (1) Administered 45 to 75 minutes before
large intestine procedures. anesthetic induction.
g. Possible antibiotic therapy to reduce colonic (2) Side rails up (client will begin to feel
flora with large bowel surgery. drowsy and light-headed).
h. Gastric or intestinal intubation may be (3) Expect complaint of dry mouth if
inserted the evening before major anticholinergics given.
abdominal surgery. (4)Observe for side effectsnarcotics
(1) Types of tubes: may cause nausea and vomiting,
(a) Levin: single lumen; sufficient to hypotension, arrhythmias, and/or
remove fluids and gas from stomach; respiratory depression.
suction may damage mucosa. (5) Quiet environment until transported to
(b) Salem sump: large lumen; prevents operating room (OR).
tissue-wall adherence. (6) Anticipate antibiotics to start on call
(c) Miller-Abbott: long single or double to OR.
lumen; required to remove the contents g. Note completeness of chart:
of jejunum or ileum. (1) Surgical checklist completed.
(2) Pressures: low setting with Levin and intes- (2) Vital signs recorded.
tinal tubes; high setting with Salem sump; (3) Routine laboratory reports present.
excessive pressures will result in injury to (4) Preoperative medications given.
mucosal lining of intestine or stomach. (5) Significant client observations.
5. Goal: promote rest and facilitate reduction of h. Assist clients family in finding proper
apprehension. waiting room.
a. Medications as ordered: on evening before (1) Inform family members that the

ADULT
surgery may give barbituratepentobarbital surgeon will contact them after the
(Nembutal), secobarbital (Seconal). procedure is over.
b. Quiet environment: eliminate noises, (2) Explain length of time client is expected
distractions. to be in recovery room.
c. Position: reduce muscle tension. (3)Prepare family for any special equipment
d. Back rub. or devices that may be needed to care
6. Goal: protect from injury; ensure final preparation for client postoperatively (e.g., oxygen,
for surgery. Day of surgery: monitoring equipment, ventilator,
a. Operative permit signed and on chart; blood transfusions).
physician responsible for obtaining informed II. INTRAOPERATIVE PREPARATIONanesthesia:
consent. Possible blood products consent. blocks transmission of nerve impulses, suppresses
b. Shower or bathe. reflexes, promotes muscle relaxation, and in some
(1) Dress: hospital pajamas. instances achieves reversible unconsciousness.
(2) Remove: hairpins (cover hair); nail polish,
A. Intravenous conscious sedationproduces
to facilitate observation of peripheral
sedation and amnesia in ambulatory procedures,
circulation; jewelry (or tape wedding
short surgical or diagnostic procedures. Allays
bands securely); pierced earrings; contact
fear and anxiety, elevates pain threshold,
lenses; dentures (store and give mouth
2164_Ch06_347-578 29/03/12 12:29 PM Page 368

368 chapter 6 Physiological Integrity

maintains consciousness and protective reflexes, (2) Client awake during surgical procedure
and returns client quickly to normal activities. avoid light or upsetting conversations.
Commonly used agents are benzodiazepines (3) Leakage of spinal fluid from puncture
(midazolam [Versed], diazepam) and narcotics sitekeep flat in bed for 24 to 48 hours
(fentanyl [Sublimaze], meperidine, morphine). to prevent headache. Keep well hydrated
B. Regional anesthesiapurpose is to block pain to aid in spinal fluid replacement.
reception and transmission in a specified area. (4) Depression of vasomotor responses
Commonly used drugs are lidocaine HCl, frequent checks of vital signs.
tetracaine HCl, cocaine HCl, and procaine HCl. 7. Intravenous regional anesthesiaused in an
Types of regional anesthetics: extremity whose circulation has been interrupted
1. Topicalapplied to mucous membranes or skin; by a tourniquet; the anesthetic is injected into
drug anesthetizes the nerves immediately below vein, and blockage is presumed to be achieved
the area. May be used for bronchoscopic or from extravascular leakage of anesthetic near a
laryngoscopic examinations. Side effects: rare major nerve trunk. Precautions as for peripheral
anaphylaxis. nerve block.
2. Local infiltrationused for minor procedures; C. General anesthesiaa reversible state in which the
anesthetic drug is injected directly into the area client loses consciousness due to the inhibition of
to be incised, manipulated, or sutured. Side neuronal impulses in the brain by a variety of
effects: rare anaphylaxis. chemical agents; may be given intravenously, by
3. Peripheral nerve blockregional anesthesia is inhalation, or rectally.
achieved by injecting drug into or around a 1. Side effects:
nerve after it passes from vertebral column; a. Respiratory depression.
procedure is named for nerve involved, such b. Nausea, vomiting.
as brachial plexus block. Requires a high degree c. Excitement.
of anatomical knowledge. Side effects: may be d. Restlessness.
absorbed into bloodstream. Observe for: signs e. Laryngospasm.
of excitability, twitching, changes in vital signs, f. Hypotension.
or respiratory difficulties. 2. Nursing care plan/implementationGoal:
4. Field blocka group of nerves is injected prevent hazardous drug interactions.
with anesthetic as the nerves branch from a a. Notify anesthesiologist if client is taking any
major or main nerve trunk. May be used for of the following drugs:
dental procedures, plastic surgery. Side effects: (1) Antidepressants, such as Prozaclong
rare. half-life; monitor renal and liver function.
5. Epidural anesthesiaanesthetizing drug is (2) Antihypertensives, such as reserpine,
injected into the epidural space of vertebral hydralazine, and methyldopapotentiate
canal; produces a bandlike anesthesia around the hypotensive effects of anesthetic
body. Frequently used in obstetrics. Rare com- agents.
ADULT

plications. Slower onset than spinal anesthesia; (3) Anticoagulants, such as heparin, warfarin
not dependent on client position for level of (Coumadin)increase bleeding times,
anesthesia; no postoperative headaches. which may result in excessive blood loss or
6. Spinal anesthesiaanesthetizing drug is injected hemorrhage.
into the subarachnoid space and mixes with (4) Aspirin and NSAIDsdecrease platelet
spinal fluid; drug acts on the nerves as they aggregation and may result in increased
emerge from the spinal cord, thereby inhibiting bleeding.
conduction in the autonomic, sensory, and (5) Steroids, such as cortisoneanti-
motor systems. inflammatory effect may delay wound
a. Advantages: rapid onset; produces excellent healing.
muscle relaxation. b. Stages of inhalation anesthesia and nursing
b. Utilization: surgery on lower limbs, inguinal goals:
region, perineum, and lower abdomen. (1) Stage Iextends from beginning of induc-
c. Disadvantages: tion to loss of consciousness. Nursing
(1) Loss of sensation below point of injection goal: reduce external stimuli, because all
for 2 to 8 hourswatch for signs of blad- movement and noises are exaggerated for
der distention; prevent injuries by main- the client and can be highly distressing.
taining alignment, keeping bedclothes (2) Stage IIextends from loss of conscious-
straightened. ness to relaxation; stage of delirium and
2164_Ch06_347-578 29/03/12 12:29 PM Page 369

Postoperative Experience 369


excitement. Nursing goal: prevent injury F. Evaluation/outcome criteria: complete reversal of
by assisting anesthesiologist to restrain anesthetic effects (e.g., spontaneous respirations,
client if necessary; maintain a quiet, non- pupils react to light). No indication of intraopera-
stimulating environment. tive complicationscardiac arrest, laryngospasm,
(3) Stage IIIextends from loss of lid reflex aspiration, hypotension, malignant hyperthermia.
to cessation of voluntary respirations. III. POSTOPERATIVE EXPERIENCE
Nursing goal: reduce risk of untoward
A. Assessment:
effects by preparing the operative site,
1. Subjective data:
assisting with procedures, and observing
a. Pain: location, onset, intensity.
for signs of complications.
b. Nausea.
(4) Stage IVindicates overdose and consists
2. Objective data:
of respiratory arrest and vasomotor
a. Operative summary:
collapse due to medullary paralysis.
(1) Type of operation performed.
Nursing goal: promote restoration of
(2) Pathological findings if known.
ventilation and vasomotor tone by assisting
(3) Anesthesia and medications received.
with cardiac arrest procedures and by
(4) Problems during surgery that will affect
administering cardiac stimulants or
recovery (e.g., arrhythmias, bleeding
narcotic antagonists as ordered.
[estimated blood loss]).
D. Intravenous agentsrapid and pleasant induction
(5) Fluids received: type, amount.
of anesthesia. Three categories: barbiturates
(6) Need for drainage or suction apparatus.
(thiopental), narcotics, and neuromuscular blocking
b. Observations:
agents (succinylcholine, curare, pancuronium).
(1) Patency of airway.
Ketamine also usedmay produce emergence
(2) Vital signs.
delirium. IV drugs require liver metabolism and
(3) Skin color and dryness.
renal excretion.
(4) Level of consciousness.
E. Hypothermiaa specialized procedure in which
(5) Status of reflexes.
the clients body temperature is lowered to 28 to
(6) Dressings.
30C (82 to 86F).
(7) Type and rate of IV infusion and blood
1. Reduces tissue metabolism and oxygen
transfusion.
requirements.
(8) Tubes/drains: urinary, chest, Penrose,
2. Used in heart surgery, brain surgery, and surgery
Hemovac; note color and amount of
on major blood vessels.
drainage.
3. Nursing care plan/implementation:
B. Analysis/nursing diagnosis:
a. Goal: prevent complications.
1. Ineffective breathing pattern related to general
(1) Monitor vital signs for shock.
anesthesia.
(2) Note level of consciousness.
2. Ineffective airway clearance related to absent or
(3) Record intake and output accurately.
weak cough.

ADULT
(4) Maintain good body alignment; reposition
3. Risk for aspiration related to vomiting.
to prevent edema, pressure, or discoloration
4. Pain related to surgical incision.
of skin.
5. Altered tissue perfusion related to shock.
(5) Maintain patent IV.
6. Risk for fluid volume deficit related to blood loss.
b. Goal: promote comfort.
7. Risk for injury related to disorientation.
(1) Apply blankets to rewarm and prevent
8. Risk for infection related to disruption of skin
shivering.
integrity.
(2) Mouth care.
9. Urinary retention related to anesthetic effects.
c. Goal: observe for indications of malignant
10. Constipation related to decreased peristalsis.
hyperthermiacommon during induction;
C. Nursing care plan/implementationimmediate
may occur 24 to 72 hours postoperatively.
postanesthesia nursing care: refers to time following
Genetic defect of muscle metabolism; early
surgery that is usually spent in the recovery room
sign is unexplained ventricular dysrhythmia,
(1 to 2 hours).
tachypnea, cyanosis, skin mottling; elevated
1. Goal: promote a safe, quiet, nonstressful
temperature is not reliable indicator.
environment.
(1) Administer 100% O2.
a. Side rails up at all times.
(2) Cool with ice packs or cooling blankets.
b. Nurse in constant attendance.
(3) Give dantrolene (muscle relaxant) per
2. Goal: promote lung expansion and gas exchange.
order.
2164_Ch06_347-578 29/03/12 12:29 PM Page 370

370 chapter 6 Physiological Integrity

3. Goal: Prevent aspiration and atelectasis. b. Observe for other alterations in circulatory
a. Position: side or back, head of bed (HOB) functionpallor; thready pulse; cold, moist
30 degrees, head turned to side to prevent skin; decreased urine output, restlessness.
obstruction of airway by tongue; allows for (1) Immediately report to physician.
drainage from mouth. (2) Initiate oxygen therapy.
b. Airway: leave the oropharyngeal or nasopha- (3) Place client in shock position unless
ryngeal airway in place until client awakens contraindicatedfeet elevated, legs
and begins to eject; gagging and vomiting straight, head slightly elevated to
may occur if not removed before pharyngeal increase venous return.
reflex returns. c. Intravenous infusions: time, rate, orders for
c. After removal of airway: turn on side in a added medications.
lateral position; support upper arm with d. Monitor blood transfusions if ordered:
pillow. observe for signs of reaction (chills, elevated
d. Suction: remove excessive secretions from temperature, urticaria, laryngeal edema, and
mouth and pharynx. wheezing). Table 6.9 illustrates the nursing
e. Encourage coughing and deep breathing: care plan/implementation.
aids in upward movement of secretions. e. If reaction occurs, immediately stop transfu-
f. Give humidified oxygen as necessary: reduces sion and notify physician. Send STAT urine
respiratory irritation and keeps bronchotra- to laboratory.
cheal secretions soft and moist. 5. Goal: promote psychological equilibrium.
g. Mechanical ventilation: respirators if a. Reassure on awakeningorient frequently.
needed (see Ventilators in Chapter 11, b. Explain procedures even though client does
p. 836). not appear alert.
4. Goal: promote and maintain cardiovascular c. Answer clients questions briefly and accurately.
function. d. Maintain quiet, restful environment.
a. Vital signs, as ordered: usually q515 min e. Comfort measures:
until stable; continuous pulse oximetry. (1) Good body alignment.
(1) Compare with preoperative vital signs. (2) Support dependent extremities to avoid
(2) Immediately report: systolic blood pressure pressure areas and possible nerve damage.
that drops 20 mm Hg or more, a pressure (3) Check for constriction: dressings, clothing,
below 80 mm Hg, or a pressure that contin- bedding.
ually drops 5 to 10 mm Hg over several (4) Check IV sites frequently for patency and
readings; pulse rates under 60 or over signs of infiltration (swelling, blanching,
110 beats/min, or irregularities; respirations cool to touch).
over 30/min; becoming shallow, quiet, 6. Goal: maintain proper function of tubes and
slow; use of neck and diaphragm muscles apparatus (see Table 11.5, pp. 839842).
(symptoms of respiratory depression); D. Nursing care plan/implementationgeneral post-
ADULT

stridorous breath sounds. operative nursing care: refers to period of time from
(text continues on page 374)

Table 6.9
Postoperative Complications
Condition and Etiology Assessment: Signs and Symptoms Nursing Care Plan/Implementation
Respiratory ComplicationsMost Common Are Atelectasis, Pneumonias (Lobar, Bronchial, and Hypostatic), and Pleuritis; Other
Complications Are Hemothorax and Pneumothorax
Atelectasisundetected preoperative Dyspnea; temperature; absent or 1. Position: unaffected side
upper respiratory infections, aspiration of diminished breath sounds over affected 2. Turn, cough, and deep
vomitus: irritation of the tracheobronchial area, asymmetrical chest expansion, breathe; encourage use of
tree with increased secretions of mucus respirations and pulse rate; tracheal inspirometer hourly while awake
due to intubation and inhalation anesthe- shift to affected side when severe; 3. Postural drainage
sia, a history of heavy smoking, or chronic anxiety and restlessness 4. Nebulization
obstructive pulmonary disease; severe 5. Force fluids if not contraindicated
postoperative pain, or high abdominal or
thoracic surgery, which inhibits deep
breathing; and debilitation or old age,
which lowers the clients resistance
2164_Ch06_347-578 29/03/12 12:29 PM Page 371

Postoperative Complications 371

Table 6.9
Postoperative Complicationscontd
Condition and Etiology Assessment: Signs and Symptoms Nursing Care Plan/Implementation
Pneumoniasee Atelectasis for etiology Rapid, shallow, painful respirations; crack- 1. Position of comfort
les; diminished or absent breath sounds; semi-Fowlers to high
asymmetrical lung expansion; chills and Fowlers
fever, productive cough, rust-colored 2. Force fluids to 3,000 mL/day
sputum; and circumoral and nailbed 3. Provide humidification of air and
cyanosis oxygen therapy
4. Oropharyngeal suction prn
5. Assist during coughing
6. Administer antibiotics and
analgesics as ordered
7. Diet: high calorie, as
tolerated
8. Cautious disposal of secre-
tions; proper oral hygiene
9. Respiratory treatments as ordered
Pleuritissee Atelectasis for etiology Knifelike chest pain on inspiration; inter- 1. Position: affected side to
costal tenderness; splinting of chest by splint the chest
client; rapid, shallow respirations; pleural 2. Manually splint clients
friction rub; temperature; malaise chest during cough
3. Administer analgesics
as ordered
Hemothoraxchest surgery, gunshot or Chest pain; increased respiratory rate; 1. Observe vital signs closely for
knife wounds, and multiple fractures of dyspnea, decreased or absent breath signs of shock and respiratory
chest wall sounds; decreased blood pressure; distress
tachycardia, and mediastinal shift may 2. Assist with thoracentesis (needle
occur (heart, trachea and esophagus, aspiration of fluid)
great vessels are pushed toward 3. Assist with insertion of chest
unaffected side) tube to closed-chest drainage
(see care of water-seal drainage
system, Table 11.5 and Fig.11.2,
pp. 840, 843)
Pneumothorax, closed or tension Marked dyspnea, sudden sharp chest 1. Remain with clientkeep as calm
thoracentesis (needle nicks the lung), pain, subcutaneous emphysema (air in and quiet as possible; STAT chest
rupture of alveoli or bronchi due to acci- chest wall tissue); cyanosis; tracheal shift x-ray
dental injury, and chronic obstructive to unaffected side; hyperresonance on 2. Position: high Fowlers
lung disease percussion, decreased or absent breath (sitting)
sounds; increased respiratory rate, tachy- 3. Notify physician through
cardia; asymmetrical chest expansion, another nurse, and have thoracen-

ADULT
feeling of pressure within chest; mediasti- tesis equipment brought to bedside
nal shiftsevere dyspnea and cyanosis, 4. Administer oxygen as necessary
deviation of larynx and trachea toward 5. Take vital signs to evaluate
unaffected side, deviation either medially respiratory and cardiac function
or laterally of apex of heart, decreased 6. Assist with thoracentesis
blood pressure; distended neck veins; 7. Assist with chest tube insertion
increased pulse and respirations and maintenance of closed-chest
drainage
Circulatory ComplicationsShock, Thrombophlebitis, Pulmonary Embolism, and Disseminated Intravascular Coagulation (DIC)
Shockhemorrhage, sepsis, decreased Dizziness; fainting; restlessness; anxiety; 1. Position: foot of bed raised
cardiac contractility (myocardial infarction, LOC 20 degrees, knees straight,
cardiac failure, tamponade), drug sensitivi- BP: or falling trunk horizontal, head slightly
ties, transfusion reactions, pulmonary Pulse: weak, thready elevated; avoid Trendelenburgs
embolism, and emotional reaction to pain Respirations: , shallow position
or deep fear Skin: pale, cool, clammy, cyanotic, 2. Administer blood transfu-
temperature; sions, plasma expanders,
Oliguria and intravenous infusions
CVP <5 cm H2O; as ordered; medications specific
Thirst to type of shock.
3. Check: vital signs, CVP, temperature
4. Insert urinary catheter to monitor
hourly urine output
5. Administer oxygen as ordered
Continued
2164_Ch06_347-578 29/03/12 12:29 PM Page 372

372 chapter 6 Physiological Integrity

Table 6.9
Postoperative Complicationscontd
Condition and Etiology Assessment: Signs and Symptoms Nursing Care Plan/Implementation
Deep vein thrombosis (thrombophlebitis) Calf pain or cramping in affected 1. Maintain complete bedrest, avoid-
injury to vein wall by tight leg straps or extremity, redness and swelling (the left ing positions that restrict venous
leg holders during gynecological surgery; leg is affected more frequently than the return
hemoconcentration due to dehydration right); slight fever, chills; Homans sign 2. Apply elastic stockings to prevent
or fluid loss; stasis of blood in extremities and tenderness over the anteromedial swelling and pooling of venous
due to postoperative circulatory depres- surface of thigh; decreased pulse in blood
sion; prolonged immobility; placement affected extremity due to swelling and 3. Apply warm, moist soaks to
of catheters (peripherally inserted central venous congestion area as ordered
catheter [PICC] line, femoral line) that 4. Administer anticoagulants
impede venous flow as ordered
5. Use bed cradle over
affected limb
6. Provide active and passive ROM
exercises in unaffected limb
Pulmonary embolismobstruction of a Sudden, severe stabbing chest pain; 1. Administer oxygen and
pulmonary artery by a foreign body in severe dyspnea; cyanosis; rapid pulse; inhalants while client is
bloodstream, usually a blood clot that anxiety and apprehension; pupillary sitting upright
has been dislodged from its original site dilation; profuse diaphoresis; loss of 2. Maintain bedrest and frequent
consciousness reassurance
3. Administer heparin sodium,
as ordered
4. Administer analgesics, such
as morphine SO4, to reduce
pain and apprehension
Disseminated Intravascular Coagulation
(DIC) (see p. 429).
Wound ComplicationsInfection, Dehiscence, and Evisceration
Wound infectionobesity or undernutri- Redness, tenderness, and heat in area of 1. Assist in cleansing and irrigation
tion, particularly protein and vitamin incision; wound drainage; temperature; of wound and insertion of a drain
deficiencies; decreased antibody produc- pulse rate. 2. Give antibiotics as ordered;
tion in aged; decreased phagocytosis in observe responses
newborn; metabolic disorder, such as
diabetes mellitus, Cushings syndrome,
malignancies, and shock; breakdown in
aseptic technique
Wound dehiscence and evisceration Slow parting of wound edges with a 1. Position: bedrest, low
obesity and undernutrition, particularly gush of pinkish serous drainage; or rapid Fowlers or horizontal
ADULT

protein and vitamin C deficiencies; parting with coils of intestines escaping position
immunosuppression; metabolic disorders, onto the abdominal wall; the latter accom- 2. Notify physician STAT
cancer; liver disease; common site is mid- panied by pain and often by vomiting. 3. Cover exposed coils of intestines
line abdominal incision, frequently about Client reports giving sensation. with sterile towels or dressing and
7 days postoperatively; precipitating keep moist with sterile normal
factors include: abdominal distention, saline
vomiting, coughing, hiccups, and uncon- 4. Monitor vital signs frequently
trolled motor activity 5. Remain with client; reassure that
physician is coming
6. Prepare for physicians arrival; set
up IV, suction equipment, and
nasogastric tube; obtain sterile
gown, mask, gloves, towels, and
warmed normal saline.
7. Notify surgery that client will be
returning to operating room
Urinary ComplicationsRetention and Infections
Urinary retentionobstruction in bladder or Inability to void within 8 hours post- 1. Assist client to stand, or use
urethra; neurological disease; mechanical surgery, despite adequate fluid replace- bedside commode if not
trauma as in childbirth or gynecological sur- ment; palpable bladder, frequent voiding contraindicated
gery; psychological conditioning that inhibits of small amounts of urine or dribbling; 2. Provide privacy
voiding in bed; prolonged bedrest; pain with suprapubic pain 3. Reduce tension, provide support
lower abdominal surgery; epidural narcotics 4. Use warm bedpan
2164_Ch06_347-578 29/03/12 12:29 PM Page 373

Postoperative Complications 373

Table 6.9
Postoperative Complicationscontd
Condition and Etiology Assessment: Signs and Symptoms Nursing Care Plan/Implementation
5. Run tap water
6. Place clients feet in warm water
7. Pour warm water over perineum
8. Catheterize if conservative
measures fail
Urinary tract infectionsurinary retention, Urinary: burning and frequency 1. Push fluids to 3,000 mL daily,
bladder distention, repeated or prolonged Pain: low back or flank unless contraindicated
catheterization Pyuria, hematuria; temperature, chills; 2. Avoid stimulants such as
anorexia; positive urine culture caffeine
3. Give antibiotics, sulfon-
amides, or acidifying agents
as ordered
4. Give perianal care after each
bowel movement
Gastrointestinal ComplicationsGastric Distention, Paralytic Ileus, and Intestinal Obstruction
Gastric distentiondepressed gastric Feeling of fullness, hiccups, overflow 1. Report signs to physician
motility due to sympathoadrenal stress vomiting of dark, foul-smelling liquid; immediately
response; idiosyncrasy to drugs: emo- severe retention leads to decreased blood 2. Insert or assist in insertion of
tions, pain, shock; fluid and electrolyte pressure (due to pressure on vagus nerve) NG tube; attach to intermittent
imbalances and other symptoms of shock syndrome suction
3. Irrigate NG tube with saline (water
will deplete electrolytes and result
in metabolic alkalosis)
4. Administer IV infusions with
electrolytes as ordered
Paralytic ileussee Gastric distention Greatly decreased or absent bowel 1. Notify physician
sounds, failure of either gas or feces to be 2. Insert or assist with insertion
passed by rectum; nausea and vomiting; of NG tube; attach to low,
abdominal tenderness and distention; intermittent suction
fever; dehydration 3. Insert rectal tube
4. Administer IV infusion with
electrolytes as ordered
5. Irrigate nasogastric tube
with saline
6. Assist with insertion of Miller-
Abbott tube if indicated
7. Administer medications to

ADULT
increase peristalsis as
ordered
Intestinal obstructiondue to poorly func- Severe, colicky abdominal pains, mild to 1. Assist with insertion of nasoen-
tioning anastomosis, hernia, adhesions, severe abdominal distention, nausea and teric tube and attach to intermit-
fecal impaction vomiting, anorexia and malaise; fever; lack tent suction
of bowel movement; electrolyte imbal- 2. Maintain IV infusions with
ance; high-pitched tinkling bowel sounds electrolytes
3. Encourage nasal breathing
to avoid air swallowing
4. Check abdomen for distention
and bowel sounds every 2 hours
5. Encourage verbalization
6. Plan rest periods for client
7. Administer oral hygiene frequently
Transfusion ReactionsAllergic, Febrile, and Hemolytic
Allergic and febrile reactionsunidentified Fever to 103F, may have sudden onset; 1. Stop transfusion and notify
antigen or antigens in donor blood or chills; itching; erythema; urticaria; nausea, physician
transfusion equipment; previous reaction vomiting; dyspnea and wheezing, 2. Administer antihistamines,
to transfusions; small thrombi; bacteria; occasionally as ordered
lysed red blood cells 3. Send STAT urine to labora-
tory for analysis
Continued
2164_Ch06_347-578 29/03/12 12:29 PM Page 374

374 chapter 6 Physiological Integrity

Table 6.9
Postoperative Complicationscontd
Condition and Etiology Assessment: Signs and Symptoms Nursing Care Plan/Implementation
4. Institute cooling measures if
indicated
5. Maintain strict input and output
records
6. Send remaining blood to laboratory
for analysis, and order recipient
blood sample for analysis
Hemolytic reactioninfusion of incom- Early: chills and fever; throbbing 1. Stop infusion immediately; take
patible blood (less common, more headache, feeling of burning in face; vital signs and notify physician
serious) hypotension; tachycardia; chest, back, 2. Send client blood sample and
or flank pain; nausea, vomiting; feeling unused blood to laboratory for
of doom; later: spontaneous and diffuse analysis
bleeding; icterus; oliguria; anuria; 3. Send STAT urine to laboratory
hemoglobinuria 4. Save all urine for observation of
discoloration
5. Administer parenteral infusions to
combat shock, as ordered
6. Administer medications
as ordereddiuretics,
sodium bicarbonate,
hydrocortisone, and vasopressors
Emotional Complications
Emotional disturbancesgrief associated Restlessness, insomnia, depression, hallu- 1. Report symptoms to physician
with loss of body part or loss of body cinations, delusions, agitation, suicidal 2. Encourage verbalization of feel-
image; previous emotional problems; thoughts ings; give realistic assurance
decreased sensory and perceptual 3. Orient to time and place as
input; sensory overload; fear and pain; necessary
decreased resistance to stress as a result 4. Provide safety measures, such as
of age, exhaustion, or debilitation side rails
5. Keep room lit, to reduce incidence
of visual hallucinations
6. Administer tranquilizers as
ordered.
7. Use restraints as a last
resort
ADULT

admission to the general nursing unit until antici- b. Observe and evaluate reaction to discomfort.
pated recovery and discharge from the hospital Use scale: 1 to 10 numerical or pictorial.
(see Table 6.9 for a review of postoperative c. Use comfort measures, such as back rubs and
complications). proper ventilation, staying with client and
1. Goal: promote lung expansion, gaseous encouraging verbalization.
exchange, and elimination of bronchotracheal d. Reduce incidence of pain: change position
secretions. frequently; support dependent extremities
a. Turn, cough, and deep breathe q2h. with pillows, sandbags, and footboards; keep
b. Use incentive spirometer as ordered to enable bedding dry and straight.
client to observe depth of ventilation. e. Give analgesics or tranquilizers as ordered;
c. Administer nebulization as ordered to help assure client that they will help.
mobilize secretions. f. Observe for desired and untoward effects of
d. Encourage hydration to thin mucous medication.
secretions. 3. Goal: promote adequate nutrition and fluid and
e. Assist in ambulation as soon as allowed. electrolyte balance.
2. Goal: provide relief of pain. a. Parenteral fluids, as ordered.
a. Assess type, location, intensity, and duration; b. Monitor blood pressure, I&O to assess
possible causative factors, such as poor body adequate, deficient, or excessive extracellular
alignment or restrictive bandages. fluid volume.
2164_Ch06_347-578 29/03/12 12:29 PM Page 375

Pain Management 375


c. Diet: liquid when nausea and vomiting stop e. Assist with standing or use of commode if
and bowel sounds are established; progress as allowed.
ordered. f. Encourage resumption of personal care as
4. Goal: assist client with elimination. soon as possible.
a. Encourage voiding within 8 to 10 hours after g. Assist with ambulation in room as soon as
surgery. allowed. Avoid prolonged chair sitting because
(1) Allow client to stand or use commode, if it enhances venous pooling and may predis-
not contraindicated. pose to thrombophlebitis. Elevate legs when
(2) Run tap water or soak feet in warm water chair sitting.
to promote micturition. E. Evaluation/outcome criteria:
(3) Catheterization if bladder is distended and 1. Incision heals without infection.
conservative treatments have failed. 2. No complications (e.g., atelectasis, pneumonia,
b. Maintain accurate I&O records. thrombophlebitis).
c. Expect bowel function to return in 2 to 3 days. 3. Normal bowel and bladder functions resume.
5. Goal: facilitate wound healing and prevent infection. 4. Carries out activities of daily living, self-care.
a. Incision care: avoid pressure to enhance 5. Accepts possible limitations: dietary, activity,
venous drainage and prevent edema. body image (e.g., no depression, complies with
b. Elevate injured extremities to reduce swelling treatment regimen).
and promote venous return.
c. Support or splint incision when coughing.
d. Check dressings q2h for drainage.
PA I N M A N A G E M E N T
e. Change dressings on draining wounds prn; I. PAINthe fifth vital sign in the care of clients; a
aseptic technique; protective ointments to complex subjective sensation; unpleasant sensory and
reduce skin irritation may be ordered. emotional experience associated with real or potential
f. Carefully observe wound suction (e.g., Jackson tissue damage. Pain is considered to be whatever the
Pratt), if applied, for kinking or twisting of person experiencing it says it is, existing whenever he
the tubes. or she says it does.
6. Goal: promote comfort and rest. A. Classifications:
a. Recognize factors that may cause restlessness 1. Acute pain: lasts typically less than 1 month;
fear, anxiety, pain, oxygen lack, wet dressings. characterized by: tachycardia, tachypnea,
b. Comfort measures: analgesics or barbiturates; increased BP, diaphoresis, dilated pupils.
apply oxygen as indicated; change positions; Responsive to analgesics.
encourage deep breathing; massage back to 2. Chronic pain: persists or is recurring for longer
reduce restlessness. than 3 months; often characterized by: lassitude,
c. Allow rest periods between caregroup sleep disturbance, decreased appetite, weight
activities. loss, diminished libido, constipation, depression.
d. Give antiemetic for relief of nausea and Rarely responsive to analgesics.

ADULT
vomiting, as ordered. 3. Somatogenic (organic/physiological):
e. Vigorous oral hygiene (brushing) to prevent a. Nociceptive: somatic or visceral pain
surgical mumps or parotitis from preopera- sensations, normal pain transmission, such
tive atropine or general anesthesia. as aching or pressure (e.g., cancer pain,
7. Goal: encourage early movement and ambulation chronic joint and bone pain).
to prevent complications of immobilization. b. Neuropathic: aberrant processes in peripheral
a. Turn or reposition q2h. and/or central nervous system; part of a
b. Range of motion (ROM): passive and active defined neurological problem; sensations such
exercises. as sharp, burning, shooting pain (e.g., nerve
c. Encourage leg exercises. compression, polyneuropathy, central pain of
d. Use preventive treatmentsantiembolic stroke, phantom pain after amputation).
stockings, graduated compression stockings, 4. Psychogenic (without organic pathology sufficient
or external pneumatic compression sleeves: to explain pain).
(1) With compression stockings, highest pres- B. Components of pain experiencepain
sure (100%) at ankles, lowest pressure related to:
(40%) at midthigh. 1. Stimulisources: chemical, ischemic, mechanical
(2) Compression sleeves, three chambers trauma, extremes of heat/cold.
sequentially inflated-deflated to stimulate 2. Perceptionviewed with fear by children, can be
venous return. altered by level of consciousness, interpreted and
2164_Ch06_347-578 29/03/12 12:29 PM Page 376

376 chapter 6 Physiological Integrity

influenced by previous and current experience, is D. Analysis/nursing diagnosis:


more severe when alone at night or immobilized. 1. Pain, acute or chronic, related to specific client
3. Responsevariations in physiological, cultural, condition.
and learned responses; anxiety is created; pain 2. Activity intolerance related to discomfort.
seen as justified punishment; pain used as means 3. Sleep pattern disturbance related to pain.
for attention-getting. 4. Fatigue related to state of discomfort or
C. Assessment: emotional stress.
1. Subjective data (Table 6.10): 5. Ineffective individual coping related to chronic
a. Sitemedial, lateral, proximal, distal. pain.
b. Strength: E. Nursing care plan/implementation:
(1) Certain tissues are more sensitive. 1. Goal: provide relief of pain.
(2) Change in intensity. a. Assess level of pain; ask client to rate on scale
(3) Based on expectations. of 0 to 10 (0 = no pain; 10 = worst pain) or
(4) Affected by distraction or concentration, smiling/sad faces. Use age-, condition-, and
state of consciousness. language-appropriate scale (Table 6.11).
(5) Described as: slight, medium, severe, b. Determine cause and try nursing comfort
excruciating. measures before giving drugs:
c. Qualityaching, burning, crushing, dull, (1) Environmental factors: noise, light, odors,
piercing, shifting, throbbing, tingling. motion.
d. Antecedent factorsphysical exertion, eating, (2) Physiological needs: elimination, hunger,
extreme temperatures, physical and emotional thirst, fatigue, circulatory impairment,
stressors (e.g., fear). muscle tension, ventilation, pressure on
e. Previous experienceinfluences reaction to nerves.
pain. (3) Emotional: fear of unknown, helplessness,
f. Behavioral cluesdemanding, worried, irrita- loneliness (especially at night).
ble, restless, difficult to distract, sleepless. c. Relieve: anger, anxiety, boredom, loneliness.
2. Objective data: d. Report sudden, severe, new pain; pain not
a. Verbal cluesmoaning, groaning, crying. relieved by medications or comfort measures;
b. Nonverbal cluesclenching teeth, grimacing; pain associated with casts or traction.
splinting of body parts, body position, knees e. Remove pain stimulus:
drawn up, involuntary reflex movements; (1) Administer pain medicationnonopioids:
tossing/turning, rhythmic rubbing move- NSAIDsketorolac (Toradol); opioids:
ments; voice pitch and speed; eyes shut. first-line analgesics include morphine,
c. Physical cluesbreathing irregularities, hydromorphone, fentanyl, oxycodone,
abdominal distention; skin color changes, hydrocodone (see Table 8.13); adjuvants:
skin temperature changes; excessive salivation, local anesthetics, sedatives, muscle relax-
perspiration. ants; give at appropriate time intervals; do
ADULT

d. Time/durationonset, duration, recurrence, not withhold due to overestimated danger


interval, last occurrence. of addiction.

Table 6.10
Required Pain Assessment on Admission for Clients Who are Hospitalized
Ask the client: Do you have pain now? Have you had pain in the recent past?
If yes to either question, the following data are obtained:
Pain intensity (use a scale appropriate for the client population)
Location (ask client to mark on diagram or point to the site)
Quality, patterns of radiation, character (use clients own words)
Onset, duration, variations, and patterns
Alleviating and aggravating factors
Present pain management regimen and effectiveness
Pain management history (medication history, barriers to reporting pain and using analgesics, manner of expressing pain)
Effects of pain (impact on daily life, function, sleep, appetite, relationships with others, emotions, concentration)
Clients pain goal (pain intensity and goals related to function, activities, quality of life)
Physical examination/observation of site(s) of pain
Source: Joint Commission on Accreditation of Healthcare Organizations, Oakbrook Terrace, IL, 1999.
(www.jcaho.org)
2164_Ch06_347-578 29/03/12 12:29 PM Page 377

Pain Management 377

Table 6.11
Format for Assessing Pain
FIRST, Assess the Pain:
Frequency Is it intermittent or constant?
Intensity What is the quality of the pain? Sharp or dull? Throbbing? Squeezing?
Radiation Does the pain move to other parts of the body?
Severity On a scale of 1 to 10, how bad is the pain?
Timing When did the pain begin? How long does it last? Does anything make it worse or take the pain away? What
precedes the pain?

(2) Avoid cold (to reduce immediate tissue 2. Goal: use nonpharmacological methods to
reaction to trauma). reduce pain.
(3) Apply heat (to relieve ischemia). a. Distraction, such as TV (cerebral cortical
(4) Change activity (e.g., restrict activity in activity blocks impulses from thalamus).
cardiac pain). b. Aromatherapyassists in relaxation.
(5) Change, loosen dressing. c. Hypnosisassess appropriateness for use for
(6) Comfort (e.g., smooth wrinkled sheets, psychogenic pain and for anesthesia; client
change wet dressing). needs to be open to suggestion.
(7) Give food (e.g., for ulcer). d. Acupunctureassess emotional readiness and
(8) Decrease stimulation (e.g., bright lights, belief in it.
noise, temperature). 3. Goal: alter interpretation and response to pain.
f. Reduce pain receptor reaction. a. Administer narcoticsresult: no longer sees
(1) Ointment (use as coating). pain as disturbing.
(2) Local anesthetics. b. Administer hypnoticsresult: changes percep-
(3) Padding (of bony prominences). tion and decreases reaction.
g. Assist with medical-surgical interventions to c. Help client obtain interpersonal satisfaction
block pain impulse transmission: from ways other than attention received when
(1) Injection of local anesthetic into nerve in pain.
(e.g., dental). 4. Goal: promote client control of pain and analgesia:
(2) Cordotomysever anterolateral spinal patient-controlled analgesia (PCA), an analgesia
cord nerve tracts. administration system designed to maintain
(3) Electrical stimulationtranscutaneous optimal serum analgesia levels; safely delivers
(skin surface), percutaneous (peripheral intermittent bolus doses of a narcotic analgesic;
nerve). preset to maximum hourly dose.

ADULT
(4) Peripheral nerve implantelectrode to a. Advantages: decreased client anxiety; improved
major sensory nerve. pulmonary function; fewer side effects.
(5) Dorsal column stimulatorelectrode to b. Limitations: requires an indwelling intravenous
dorsal column. line; analgesia targets central pain, may not
h. Minimize barriers to effective pain relieve peripheral discomfort; cost of PCA unit.
management: 5. Goal: health teaching.
(1)Achieve balanced analgesia; around- a. Explain causes of pain and how to describe pain.
the-clock administration of NSAIDs or b. Explain that it is acceptable to admit existence
acetaminophen if possible, continuous of pain.
infusion, patient-controlled analgesia c. Relaxation exercises.
(PCA); combination therapy (opioids, d. Biofeedback methods of pain perception and
nonopioids, adjuvants). control.
(2) Accept client and family report of pain. e. Proper medication administration (PCA,
(3) Discuss fear of addiction with client family continuous around-the-clock dosing), when
(incidence <1% when opioids taken for necessary, for self-care.
pain relief ). F. Evaluation/outcome criteria:
(4) Discuss fear of respiratory depression with 1. Verbalizes comfort; awareness of pain decreased.
staff; preventable; related to sedation. 2. Knows source of pain; how to reduce stimulus
i. Document response to pain relief measures. and perception.
2164_Ch06_347-578 29/03/12 12:29 PM Page 378

378 chapter 6 Physiological Integrity

3. Uses alternative measures for pain relief. C. Mechanisms of injury:


4. Able to cope with pain (e.g., remains active, 1. Deformation (blow to the head).
relaxed appearance; verbal and nonverbal clues 2. Acceleration-deceleration (coup-contracoup)
of pain absent). forward and rebounding motion.
3. Rotation (tension, stretching, shearing
force).
NEUROLOGICAL D. Pathophysiology of impaired CNS functioning:
SYSTEM 1. Depressed neuronal activity in reticular
I. TRAUMATIC INJURIES TO THE BRAIN activating system depressed consciousness
A. Primary trauma: (Table 6.12).
1. Concussiontransient disorder due to injury in 2. Depressed neuronal functioning in lower brain-
which there is brief loss of consciousness due to stem and spinal cord depression of reflex
paralysis of neuronal function; recovery is usually activity decreased eye movements, unequal
total. pupils decreased response to light stimuli
2. Contusionstructural alteration of brain tissue widely dilated and fixed pupils.
characterized by extravasation of blood cells 3. Depression of respiratory center altered respi-
(bruising); injury may occur on side of impact ratory pattern decreased rate respiratory
or on opposite side (when cranial contents shift arrest.
forcibly within the skull with impact). E. Risk factors: accidentsautomobile, industrial
3. Lacerationtearing of brain tissue or blood and home, motorcycle, military.
vessels due to a sharp bone fragment or object F. Assessment:
or tearing force. 1. Subjective data:
4. Fracturelinear (may result in epidural bleed); a. Headache.
comminuted or depressed (may tear dura and b. Dizziness, loss of balance.
result in cerebrospinal fluid [CSF] leak); basilar c. Double vision.
(most serious). Basilar skull fracture may result d. Nausea.
in meningitis or brain abscess; bleeding from 2. Objective data:
nose or ears; CSF present in drainage; bruising a. Laceration or abrasion around face or head;
over mastoid process (Battle sign) and periorbital profuse bleeding from scalp (highly vascular,
ecchymosis (raccoon eyes). poor vasoconstrictive abilities).
B. Secondary trauma (response to primary trauma): b. Drainage from ears or nose (serosanguineous).
1. Hematomas: c. Projectile vomiting, hematemesis.
a. Subduralblood from ruptured or torn vein d. Vital signs indicating increased intracranial
collects between arachnoid and dura; may be pressure (see II. INCREASED INTRACRA-
acute, subacute, or chronic. NIAL PRESSURE, pp. 379380).
b. Extradural (epidural)blood clot located e. Neurological examination:
between dura mater and inner surface of (1) Altered level of consciousness; a numerical
ADULT

skull; most often from tearing of middle assessment, such as the Glasgow Coma
meningeal artery; emergency condition. Scale (Table 6.13), may be used. The
2. Increased intracranial pressure (see II. INCREASED lower the score, the poorer the prognosis,
INTRACRANIAL PRESSURE, pp. 379380). generally.

Table 6.12
Levels of Consciousness
Stage Characteristics
Alertness Aware of time and place
Automatism Aware of time and place but demonstrates abnormality of mood (euphoria to irritability)
Confusion Inability to think and speak in coherent manner; responds to verbal requests but is unaware of time and place
Delirium Restlessness and violent activity; may not comply with verbal instructions
Stupor Quiet and uncommunicative; may appear conscioussits or lies with glazed look; unable to respond to verbal
instructions; bladder and rectal incontinence may occur
Semicoma Unresponsive to verbal instructions but responds to vigorous or painful stimuli
Coma Unresponsive to vigorous or painful stimuli
2164_Ch06_347-578 29/03/12 12:29 PM Page 379

Neurological System 379

Table 6.13 g. Medications as ordered:


(1) Steroids (dexamethasone [Decadron]).
Glasgow Coma Scale (2) Anticonvulsants (phenytoin [Dilantin],
Best eye-opening response Purposeful and spontaneous: phenobarbital).
To voice (3) Analgesics (morphine contraindicated).
To pain h. Cooling measures or hypothermia to reduce
No response
Untestable
elevated temperature.
i. Assist with diagnostic tests:
Best verbal response Oriented
Disoriented
(1) Lumbar puncture (contraindicated with
Inappropriate words increased intracranial pressure).
Incomprehensible sounds (2) Electroencephalogram (EEG).
No response j. Diet: NPO for 24 hours, progressing to clear
Untestable liquids if awake.
Best motor response Obeys commands k. Fluids: IVs; nasogastric tube feedings; I&O.
Localizes pain l. Monitor blood chemistries: sodium imbalance
Withdraws to pain common with head injuries.
Flexion to pain
Extension to pain
2. Goal: provide emotional support and use comfort
No response measures.
Untestable a. Comfort: skin care, oral hygiene; sheepskins;
wrinkle-free linen.
b. Eyes: lubricate q4h with artificial tears if
(2) Pupilsequal, round, react to light; or periocular edema present.
unequal, dilated, unresponsive to light. c. ROMpassive, active; physical therapy as
(3) Extremitiesparesis or paralysis. tolerated.
(4) Reflexeshypotonia or hypertonia; d. Avoid restraints.
Babinski present (flaring of great toe e. Encourage verbalization of concerns about
when sole is stroked). changes in body image, limitations.
G. Analysis/nursing diagnosis: f. Encourage family communication.
1. Altered thought processes related to brain I. Evaluation/outcome criteria:
trauma. 1. Alert, orientedno residual effects (e.g., cogni-
2. Sensory/perceptual alteration related to tive processes intact).
depressed neuronal activity. 2. No signs of increased intracranial pressure
3. Risk for injury related to impaired CNS (e.g., decreased respirations, increased systolic
functioning. pressure with widening pulse pressure,
4. Risk for aspiration related to respiratory bradycardia).
depression. 3. No paralysisregains motor/sensory function.
5. Self-care deficit related to altered level of 4. Resumes self-care activities.

ADULT
consciousness. II. INCREASED INTRACRANIAL PRESSURE (ICP):
6. Ineffective breathing pattern related to CNS intracranial hypertension associated with altered states
trauma. of consciousness.
H. Nursing care plan/implementation: A. Pathophysiology: increases in intracranial blood
1. Goal: sustain vital functions and minimize or volume, cerebrospinal fluid, or brain tissue mass
prevent complications. increased intracranial pressure impaired neural
a. Patent airway: endotracheal tube or impulse transmission cellular anoxia, atrophy.
tracheostomy may be ordered. B. Risk factors:
b. Oxygen: as ordered (hypoxia increases 1. Congenital anomalies (hydrocephalus).
cerebral edema). 2. Space-occupying lesions (abscesses or tumors).
c. Position: semiprone or prone (coma position) 3. Trauma (hematomas or skull fractures).
with head level to prevent aspiration (keep 4. Circulatory problems (aneurysms, emboli).
off back); turn side to side to prevent stasis 5. Inflammation (meningitis, encephalitis).
in lungs. C. Assessment:
d. Vital signs as ordered. 1. Subjective data:
e. Neurological check: pupils, level of conscious- a. Headache (early, but nonspecific symptom).
ness, muscle strength; report changes. b. Nausea.
f. Seizure precautions: padded side rails. c. Visual disturbance (diplopia).
2164_Ch06_347-578 29/03/12 12:29 PM Page 380

380 chapter 6 Physiological Integrity

2. Objective data: hematomas or excess fluid, control hemorrhage, repair


a. Changes in level of consciousness (early sign). skull fractures, remove scar tissue, repair or excise
b. Pupillary changesunequal (emergency aneurysms, and drain abscesses; produces minimal
notify physician, indicates herniation), neurological deficit.
dilated, and unresponsive to light (late sign). A. Analysis/nursing diagnosis:
c. Vital signschanges are variable. 1. Altered cerebral tissue perfusion related to edema.
(1) Blood pressuregradual or rapid elevation, 2. Altered thought processes related to disorientation.
widened pulse pressure. 3. Self-care deficit related to continued neurological
(2) Pulsebradycardia, tachycardia; significant impairment.
sign is slowing of pulse as blood pressure rises. 4. Also see nursing diagnosis for I. TRAUMATIC
(3) Respirationspattern changes (Cheyne- INJURIES TO THE BRAIN, p. 378; II. D.
Stokes, apneusis, Biots), deep and sonorous; INCREASED INTRACRANIAL PRESSURE,
hiccups. p. 379; and THE PERIOPERATIVE EXPERI-
(4) Temperaturemoderate elevation. ENCE, p. 365.
d. Projectile vomiting (more common in B. Nursing care plan/implementation:
children). 1. Preoperative:
e. Diagnostic test: head computed tomography a. Goal: obtain baseline measures.
(CT)structural changes. (1) Vital signs.
D. Analysis/nursing diagnosis: (2) Level of consciousness.
1. Altered cerebral tissue perfusion related to (3) Mental, emotional status.
increased intracranial pressure. (4) Pupillary reactions.
2. Altered thought processes related to cerebral anoxia. (5) Motor strength and functioning.
3. Ineffective breathing pattern related to compres- b. Goal: provide psychological support: listen; give
sion of respiratory center. accurate, brief explanations.
4. Risk for aspiration related to unconsciousness. c. Goal: prepare for surgery.
5. Self-care deficit related to altered level of (1) Cut hair; shave scalp (usually done in
consciousness. surgery); save hair if client/family desire.
6. Impaired physical mobility related to abnormal (2) Cover scalp with clean towel.
motor responses. (3) Insert indwelling Foley catheter as ordered.
E. Nursing care plan/implementation: Goal: promote 2. Postoperative:
adequate oxygenation and limit further impairment. a. Goal: prevent complications and limit further
1. Vital signs: report changes immediately. impairment.
2. Patent airway; keep alkalotic, to prevent (1) Vital signs (indications of complications):
increased intracranial pressure from elevated (a) Decreased blood pressureshock.
CO2; hyperventilate if necessary. (b) Widened pulse pressureincreased ICP.
3. Give medications as ordered: (c) Respiratory failurecompression of
a. Hyperosmolar diuretics (mannitol, urea) to medullary respiratory centers.
ADULT

reduce brain swelling. (d) Hyperthermiadisturbance of


b. Steroids (dexamethasone [Decadron]) for heat-regulating mechanism; infection.
anti-inflammatory action. (2) Neurological:
c. Antacids or histamine2 (H2) antagonist to (a) Pupilsipsilateral dilation (increased
prevent stress ulcer. ICP), visual disturbances.
4. Position: head of bed elevated 30 degrees. (b) Altered level of consciousness.
5. Fluids: restrict; strict I&O. (c) Altered cognitive or emotional status
6. Cooling measures to reduce temperature, disorientation common.
because fever increases ICP. (d) Motor function and strength
7. Prepare for surgical intervention (see III. hypertonia, hypotonia, seizures.
CRANIOTOMY, following). (3) Blood gases, to monitor adequacy of
F. Evaluation/outcome criteria: ventilation.
1. No irreversible brain damageregains (4) Dressings: check frequently; aseptic
consciousness. technique; reinforce as necessary.
2. Resumes self-care activities. (5) Observe for:
III. CRANIOTOMY: excision of a part of the skull (a) CSF leakage (glucose-positive drainage
(burr hole to several centimeters) for exploratory from nose, mouth, ears)report
purpose and biopsy; to remove neoplasms, evacuate immediately.
2164_Ch06_347-578 29/03/12 12:29 PM Page 381

Neurological System 381


(b) Periorbital edemaapply light ice 3. Water and electrolyte disturbances.
compresses as necessaryremove 4. Hypoglycemia.
crusts from eyelids; instill lubricant 5. Tumors.
eyedrops. 6. Vascular disorders (hypoxia or hypocapnia).
(6) Check integrity of seventh cranial nerve C. Generalized seizures:
(facial)incomplete closure of eyelids. 1. Tonic-clonic (grand mal) seizures:
(7) Position: a. Pathophysiology: increased excitability of a
(a) Supratentorial surgery (cerebrum) neuron possible activation of adjacent neu-
semi-Fowlers (30-degree elevation); rons synchronous discharge of impulses
may not lie on operative side. vigorous involuntary sustained muscle spasms
(b) Infratentorial (brainstem, cerebellum) (tonic contractions). Onset of neuronal fatigue
flat in bed (prone); may turn to either intermittent muscle spasms (clonic contrac-
side but not onto back. tions) cessation of muscle spasms
(8) Fluids and food: NPO initially; tube fatigue.
feeding until alert and intact gag, b. Assessment:
swallow, and cough reflexes present. (1) Subjective dataaura: flash of light; pecu-
Aspiration risk. liar smell, sound; feelings of fear; euphoria.
(9) Medications as ordered: (2) Objective data:
(a) Osmotic diuretics (mannitol). (a) Convulsive stagetonic and clonic
(b) Corticosteroids (dexamethasone muscle spasms, loss of consciousness,
[Decadron]). breath-holding, frothing at mouth,
(c) Mild analgesics (do not mask neuro- biting of tongue, urinary or fecal
logical or respiratory depression). incontinence; lasts 2 to 5 minutes.
(d) Stool softeners to prevent constipation (b) Postconvulsionheadache, fatigue
and straining. (postictal sleep), malaise, vomiting,
(10) Orient frequently to person, time, sore muscles, choking on secretions,
placeto reduce restlessness, confusion. aspiration.
(11) Side rails up for safety. 2. Absence (petit mal) seizures:
(12) Avoid restraints (may increase agitation a. Pathophysiology: unknown etiology,
and ICP). momentary loss of consciousness (10 to
(13) Ice bags to head to reduce headache. 20 seconds); usually no recollection of
(14) Activity: assist with ambulation. seizure; resumes previously performed action.
b. Goal: provide optimal supportive care. b. Assessmentobjective data:
(1) Cover scalp once dressings are removed (1) Fixation of gaze; blank facial expression.
(scarves, wigs). (2) Flickering of eyelids.
(2) Deal realistically with neurological (3) Jerking of facial muscle or arm.
deficitsfacilitate acceptance, adjust- 3. Minor motor seizures:

ADULT
ment, independence. a. Myoclonicinvoluntary lightning-like jerk-
c. Goal: health teaching. ing contraction of major muscles; may throw
(1) Prepare for physical, occupational, or person to the floor; no loss of consciousness.
speech therapy, as needed. b. Atonicbrief, total loss of muscle tone; per-
(2) Activities of daily living. son falls to the floor; loss of consciousness
C. Evaluation/outcome criteria: (common in children).
1. Regains consciousnessis alert, oriented. D. Partial (focal) seizures:
2. Resumes self-care activities within limits of 1. Partial motor: arises from region in motor cortex
neurological deficits. (posterior frontal lobe); most commonly begins
IV. EPILEPSY: seizure disorder characterized by sudden in upper extremities, spreading to face and lower
transient aberration of brain function; associated with extremity (jacksonian march); noting progression
motor, sensory, autonomic, or psychic disturbances. is important in identifying area of cortex
involved.
A. Seizure: involuntary muscular contraction and dis-
2. Partial sensory: sensory symptoms occur with
turbances of consciousness from abnormal electrical
partial seizure activity; varies with region in
activity.
brain; transient.
B. Risk factors:
3. Partial complex (psychomotor): arises out of
1. Brain injury.
anterior temporal lobe; frequently begins with
2. Infection (meningitis, encephalitis).
2164_Ch06_347-578 29/03/12 12:29 PM Page 382

382 chapter 6 Physiological Integrity

an aura; characteristic feature is automatism G. Evaluation outcome criteria:


(e.g., lip smacking, chewing, patting body, 1. Avoids precipitating stimuliachieves seizure
picking at clothes); lasts from 2 to 3 minutes control.
to 15 minutes; do not restrain. 2. Complies with medication regimen.
E. Analysis/nursing diagnosis: 3. Retains independence.
1. Risk for injury related to convulsive disorder. V. TRANSIENT ISCHEMIC ATTACKS (TIAs):
2. Anxiety related to sudden loss of consciousness. temporary, complete, or relatively complete cessation
3. Self-esteem disturbance related to chronic illness. of cerebral blood flow to a localized area of brain,
4. Impaired social interaction related to producing symptoms (2 to 30 minutes) ranging from
self-consciousness. weakness (drop attacks) and numbness to monocular
F. Nursing care plan/implementation (generalized blindness; an important precursor to stroke. Surgical
seizures): intervention includes carotid endarterectomy; most
1. Goal: prevent injury during seizure. common postoperative cranial nerve damage causes
a. Do not force jaw open during convulsion. vocal cord paralysis or difficulty managing saliva
b. Do not restrict limbsprotect from injury; and tongue deviation (cranial nerves VII, X, XI, XII);
place something soft under head (towel, usually temporary; stroke may also occur.
jacket, hands).
c. Loosen constrictive clothing. VI. STROKE (cerebrovascular accident [CVA],
d. Note: time, level of consciousness, type and brain attack): neurologic changes caused by
duration of seizure. interruption of blood supply to a part of the brain.
2. Goal: postseizure care: Ischemic strokecommonly due to thrombosis
a. Turn on side to drain saliva and facilitate or embolism; thrombotic strokes more common.
breathing. Hemorrhagic strokerupture of cerebral vessel,
b. Suction as necessary. causing bleeding into the brain tissue; most com-
c. Orient to time and place. mon after age 50.
d. Oral hygiene if tongue or cheek injured. A. Pathophysiology: reduced or interrupted blood
e. Check vital signs, pupils, level of consciousness. flow interruption of nerve impulses down corti-
f. Notify physician; medication may need cospinal tract decreased or absent voluntary
adjusting. movement on one side of the body (fine move-
3. Goal: prevent or reduce recurrences of seizure activity. ments are more affected than coarse movements);
a. Encourage client to identify precipitating later, autonomous reflex activity spasticity and
factors. rigidity of muscles.
b. Moderation in diet and exercise. B. Risk factors:
c. Medications as ordered: phenytoin (Dilantin); 1. Hypertension (modifiable risk factor).
phenobarbital; carbamazepine (Tegretol); 2. Prior ischemic episodes (TIAs).
primidone (Mysoline); valproate (Depacon). 3. Cardiovascular disease; atrial fibrillation.
4. Goal: health teaching. 4. Oral contraceptives.
5. Emotional stress.
ADULT

a. Medications:
(1) Actions, side effects (apathy, ataxia, 6. Family history.
hyperplasia of gums). 7. Advancing age.
(2) Complications with sudden withdrawal 8. Diabetes mellitus.
(status epilepticuscontinuous seizure C. Assessment:
activity; give diazepam per order, O2). 1. Subjective data:
b. Attitude toward life and treatment; adhere to a. Weakness: sudden or gradual loss of move-
medication program. ment of extremities on one side.
c. Clarify misconceptions, fearsespecially b. Difficulty forming words.
about insanity, bad genes. c. Difficulty swallowing (dysphagia).
d. Maintain activities, interestsexpect no driving d. Nausea, vomiting.
until seizure free for period of time specified by e. History of TIAs.
state Department of Motor Vehicles. 2. Objective data:
e. Avoid: stress; lack of sleep; emotional upset; a. Vital signs:
alcohol. (1) BPelevated with thrombosis, normal
f. Relaxation techniques; stress management with embolism. Widened pulse pressure
techniques. with large ischemic strokes or hemorrhage.
g. Use Medic Alert band or tag. (2) Temperatureelevated.
h. Refer to appropriate community resources. (3) Pulsenormal, slow.
2164_Ch06_347-578 29/03/12 12:29 PM Page 383

Neurological System 383


(4) Respirationstachypnea, altered pattern; cardiogenic emboli; antiplatelet agents
deep; sonorous. (aspirin, ticlopidine, clopidogrel) to
(5) CT scan of headnegative if no hemor- decrease risk for thrombus formation.
rhage, indicates ischemic stroke. (2) Hemorrhagic strokeantihypertensives
b. Neurological (vary by type and location of for systolic pressure >160 mm Hg; never
stroke): treat with r-tPA; mannitol to decrease ICP.
(1) Altered level of consciousness; progression c. Fluids: IVs to prevent hemoconcentration;
to coma with hemorrhage. I&O; weigh daily. Nutritional support as
(2) Pupilsunequal; visionhomonymous indicated.
hemianopia. d. ROM exercises to prevent contractures, muscle
(3) Ptosis of eyelid, drooping mouth. atrophy; deep vein thrombosis prophylaxis;
(4) Paresis or paralysis (hemiplegia). early referral to physical therapy (PT).
(5) Loss of sensation and reflexes. e. Skin care and position changes to prevent
(6) Incontinence of urine or feces. decubiti.
(7) Aphasia (see pp. 383384). 3. Goal: provide for emotional relaxation.
D. Analysis/nursing diagnosis: a. Identify grief reaction to changes in body
1. Impaired physical mobility related to hemiplegia. image. Early referral to occupational therapy
2. Impaired swallowing related to paralysis. (OT) if indicated.
3. Impaired verbal communication related to aphasia. b. Encourage expression of feelings, concerns.
4. Risk for aspiration related to unconsciousness. Early referral to speech therapy if indicated.
5. Sensory/perceptual alterations related to altered 4. Goal: client safety.
cerebral blood flow, visual field blindness. a. Identify existence of homonymous hemianopia
6. Altered thought processes related to cerebral (visual field blindness) and agnosia (distur-
edema. bance in sensory information).
7. Self-care deficit related to paresis or paralysis. b. Use side rails and assist as needed.
8. Body image disturbance related to hemiplegia. c. Remind to walk slowly, take adequate rest
9. Total incontinence related to interruption of periods, ensure good lighting, look where
normal nerve transmission. client is going.
10. Impaired social interaction related to aphasia or 5. Goal: health teaching.
neurological deficit. a. Exercise routines.
11. Risk for impaired skin integrity related to b. Diet: self-feeding, but assist as needed.
immobility. c. Resumption of self-care activities.
12. Unilateral neglect related to cerebral damage. d. Use of supportive devices; transfer techniques.
E. Nursing care plan/implementation e. Involvement of family in rehabilitation
1. Goal: reduce cerebral anoxia. activities.
a. Patent airway: F. Evaluation/outcome criteria:
(1) Oxygen therapy as ordered; suctioning to 1. No complications (e.g., pneumonia).

ADULT
prevent aspiration. 2. Regains functional independenceresumes
(2) Turn, cough, deep breathe q2h due to self-care activities.
high incidence of aspiration pneumonia. 3. Return of control over body functions
b. Activity: bedrest, progressing to out of bed (e.g., bowel, bladder, speech).
as tolerated. VII. APHASIA: impaired ability to understand or use
c. Position: commonly accepted words or symbols; interferes with
(1) Maximize ventilation. ability to speak, write, or read; language center
(2) Support with pillows when on side; use usually left hemisphere (85% of population). Dysarthria
hand rolls and arm slings as ordered. is motor impairmentinability to articulate words.
2. Goal: promote cerebrovascular function and
A. Types and pathophysiology:
maintain cerebral perfusion.
1. Receptive (sensory)lesion usually in Wernickes
a. Vital signs; neurological checks.
area of temporal lobe; difficulty understanding
b. Medications as ordered:
spoken word (auditory aphasia) or written word
(1) Ischemic strokethrombolytic agents
(visual aphasia).
(recombinant tissue plasminogen activator
2. Expressive (motor)difficulty expressing
[r-tPA]) within 3 hours of onset of stroke;
thoughts in speech or writing (motor aphasia);
antihypertensives only if BP >185 mm Hg
understands written and spoken words. Three
systolic or 105 mm Hg diastolic; mannitol
types: anomicfluent speech, but unable to
to decrease ICP; heparin only if risk for
2164_Ch06_347-578 29/03/12 12:29 PM Page 384

384 chapter 6 Physiological Integrity

name objects, qualities, and conditions; fluent C. Assessment:


articulate and grammatically correct, but no 1. Subjective data:
content or meaning; nonfluentunable to select, a. Headachesevere.
organize, and initiate speech (involves Brocas b. Feversudden.
area of brain); may affect writing. c. Nausea, vomiting.
B. Risk factors: d. Sensitivity to light (photophobia).
1. Vascular disease of the brain (brain attack). e. Difficulty concentrating.
2. Alzheimers disease (degeneration). 2. Objective data:
3. Tumor. a. Altered level of consciousness.
4. Trauma. b. Nuchal rigidity.
C. Analysis/nursing diagnosis: c. Tremors; facial weakness.
1. Impaired verbal communication related to cerebral d. Nystagmus.
cortex disorder. e. Elevated temperature.
2. Powerlessness related to inability to express f. Diagnostic test: lumbar puncturefluid
needs/concerns. cloudy; increased neutrophils, protein.
3. Impaired social interaction related to difficulty g. Laboratory data: bloodslight to moderate
communicating. leukocytosis (about 14,000).
D. Nursing care plan/implementation: D. Analysis/nursing diagnosis:
1. Goal: assist with communication: client does best 1. Self-care deficit related to altered level of
when rested; small improvements will occur up consciousness.
to 1 year after injury (age is a factor). 2. Risk for injury related to coma.
2. Strategies: 3. Sensory/perceptual alteration related to brain tissue
a. Nonfluentallow time to respond; support injury.
efforts to speak; acknowledge frustration of 4. Altered thought processes related to increased
clientanticipate needs when appropriate; intracranial pressure.
use picture board or flash cards, pointing, to E. Nursing care plan/implementation:
encourage communication; assess efforts to 1. Goal: support physical and emotional relaxation.
communicate with open-ended questions. a. Vital signs; neurological signs as ordered.
b. Fluentface the client, speak slowly and b. Seizure precautions.
distinctly, not loudly; use gestures, repeat c. Position: to maintain patent airway; prevent
instructions as needed; involve family in contractures; ROM.
techniques to improve communication; d. Medications as ordered:
acknowledge frustration. (1) Analgesics for headache and neck pain.
E. Evaluation/outcome criteria: (2) Antipyretics for fever.
1. Communication reestablished. (3) Antivirals.
2. Minimal frustration exhibited. (4) Anticonvulsants for seizures.
3. Participates in speech therapy. (5) Antibiotics for infection in aseptic
ADULT

VIII. BACTERIAL MENINGITIS meningitis.


(see Chapter 5, pp. 265266). (6) Osmotic diuretics (mannitol) to reduce
cerebral edema.
IX. ENCEPHALITIS (also includes aseptic meningitis): (7) Corticosteroids for inflammation.
inflammation of the brain and its coverings due e. No isolation.
to direct viral invasion, which usually results in a 2. Goal: health teaching: self-care activities with
lengthy coma. residual motor and speech deficits; physical
A. Pathophysiology: brain tissue injury release of therapy.
enzymes that increase vascular dilation, capillary F. Evaluation/outcome criteria:
permeability edema formation increased 1. Regains consciousness; is alert, oriented.
intracranial pressure depression of CNS 2. Performs self-care activities with minimal
function. assistance.
B. Risk factors:
1. Arboviruses.
2. Enteroviruses (poliovirus, echovirus). EYES, EARS, NOSE
3. Herpesvirus. A N D T H R O AT
4. Varicella-zoster (chickenpox). I. LARYNGECTOMY with radical neck dissection:
5. Postinfection complication (measles, mumps, removal of entire larynx, lymph nodes, submandibular
smallpox). salivary gland, sternomastoid muscle, spinal accessory
2164_Ch06_347-578 29/03/12 12:29 PM Page 385

Eyes, Ears, Nose and Throat 385


nerves, and jugular vein for cancer of the larynx (3) Laryngectomy tube care:
that extends beyond the vocal cords. Permanent (a) Observe for stridor (coarse, high-
tracheostomy; new methods of speech will have to pitched inspiratory sound)report
be learned. immediately.
Partial laryngectomy: removal of lesion on larynx. (b) Have extra laryngectomy tube at
Client will be able to speak after operation, but bedside.
quality of voice may be altered. (c) Suction with sterile equipment; 2 to
A. Assessment: 3 mL of sterile saline into stoma may
1. Subjective data: be used to loosen secretions.
a. Feeling of lump in throat. b. Goal: promote optimal physical and psychological
b. Pain: Adams apple; may radiate to ear. function.
c. Dysphagia. (1) Frequent mouth care.
2. Objective data: (2) Wound: exposed site; note color and
a. Hoarseness: persistent (>2 weeks), progressive. amount of drainage.
b. Lymphadenopathy: cervical. (3)Tubes: closed drainage system (Hemovac,
c. Breath odor: foul. Jackson Pratt) (Fig. 6.2); expect less
B. Analysis/nursing diagnosis: than 100 up to 300 mL of serosan-
1. Impaired verbal communication related to guineous drainage first postoperative
removal of larynx. day; drainage should decrease daily;
2. Body image disturbance related to radical neck observe patency.
dissection. (4)Pain managementconsider impact
3. Ineffective airway clearance related to copious of impaired communication on
amounts of mucus. assessment.
4. Fear related to diagnosis of cancer. (5) Postdrainage system removalobserve:
5. Impaired swallowing related to edema. skin flaps down, adherent to underlying
6. Impaired social interaction related to altered tissue.
speech. (6) Use surgical asepsis.
C. Nursing care plan/implementation: (7) Answer call bell immediately; use preestab-
1. Preoperative care: lished means of communication.
a. Goal: provide emotional support and optimal (8)Reexplain all procedures while giving
physical preparation. care.
(1) Encourage verbalization of fears; answer (9) Support head when lifting.
all questions honestly, particularly about
having no voice after surgery.
(2)Referral: visit from person with laryngec-
tomy (contact New Voice Club, Lost
Chord, or International Association of

ADULT
Laryngectomees).
b. Goal: health teaching.
(1) Prepare for tracheostomy.
(2) Other means to speak (esophageal burp
speech, tracheoesophageal prosthesis or
electronic artificial larynx).
2. Postoperative care:
a. Goal: maintain patent airway and prevent
aspiration.
(1)Position: semi-Fowlers (elevate 30 to
45 degrees), preventing forward flexion
of neck to reduce edema and keep airway
open.
(2) Observe for hypoxia:
(a) Early signs: increased respiratory and Figure 6.2 Closed drainage system for constant suction.
pulse rates, apprehension, restlessness. Suction is maintained by a plastic container. The container
serves as both suction source and receptacle for blood. It is
(b) Late signs: dyspnea, cyanosis; swallow- emptied as required, and drainage tubes are left in the neck
ing difficultiesclient should chew for approximately 3 days. (From De Weese, DD: Saunders Textbook
food well and swallow with water. of Otolaryngology. Mosby, St. Louis, 1987. Out of Print.)
2164_Ch06_347-578 29/03/12 12:29 PM Page 386

386 chapter 6 Physiological Integrity

c. Goal: health teaching. C. Assessment:


(1) Referral: speech rehabilitation as soon as 1. Subjective data:
esophageal suture is healed. a. Tinnitus (constant or intermittent).
(a) Information on laryngeal speech b. Headache; feeling of fullness or pressure in
(International Association of affected ear.
Laryngectomees, American Cancer c. True vertigo: sudden attacks; room appears
Society, American Speech and to spin.
Hearing Association). d. Depression; irritability; withdrawal.
(b) Esophageal speech best learned in speech e. Nausea with sudden head motion.
cliniclearn to burp column of air 2. Objective data:
needed for speech; new voice sounds a. Impaired hearing, especially low tones.
are natural but hoarse. b. Change in gait; lack of coordination.
(2) Stoma care: c. Vomiting with sudden head motion.
(a) Cover with scarf or shirt made of a d. Nystagmusduring attacks.
porous material (material substitutes e. Diagnostic test: caloric (cold water in
for nasal passagewarms and filters ear canal)may precipitate attack;
out particles). audiometryloss of hearing.
(b) Use source of humidification (mister D. Analysis/nursing diagnosis:
or commercial humidifier). 1. Risk for injury related to vertigo, lack of
(c) Caution while bathing or showering, to coordination.
decrease likelihood of aspiration. 2. Auditory sensory/perceptual alteration related to
(d) Swimming not recommended. progressive hearing loss.
(e) Procedure for suctioning if cough 3. Anxiety related to uncertainty of treatment.
ineffective. 4. Risk for activity intolerance related to sudden
(3) Simple ROM of neck; how to support onset of vertigo.
head. 5. Sleep pattern disturbance related to tinnitus.
(4) Possible contraindications: use of talcum 6. Ineffective individual coping related to chronic
powder, tissues. disorder.
D. Evaluation/outcome criteria: E. Nursing care plan/implementation:
1. No surgical complications (e.g., no airway 1. Goal: provide safety and comfort during
obstruction, infection, hemorrhage). attacks.
2. Learns alternative speech 30 to 60 days after a. Activity: bedrest during attack; side rails up;
surgery. lower to chair or floor if attack occurs while
3. Demonstrates proper stoma care. standing; assist with ambulation (sudden
4. Resumes productive lifestyle (work, family). dizziness common).
5. Normal response to change in body image b. Position: recumbent; affected ear uppermost
(e.g., anger, grief, denial). usually.
ADULT

II. MNIRES DISEASE: chronic, recurrent disorder c. Identify prodromal symptoms (aura, ear
of inner ear; attacks of vertigo, tinnitus, and vestibular pressure, increased tinnitus).
dysfunction; lasts 30 minutes to full day; usually no 2. Goal: minimize occurrence of attacks.
pain or loss of consciousness. a. Give medications as ordered:
(1) Anticholinergics (oral or transdermal
A. Pathophysiology: associated with excessive
scopolamine, atropine, glycopyrrolate
dilation of cochlear duct (unilateral) from
[Robinul]) to minimize GI symptoms.
overproduction or decreased absorption of
(2) Antihistamines (dimenhydrinate
endolymph (endolymphatic hydrops)
[Dramamine], diphenhydramine HCl
progressive sensorineural loss.
[Benadryl]) to sedate vestibular system.
B. Risk factors:
(3) Antiemetics and antivertigo agents
1. Emotional or endocrine disturbance (diabetes
(diazepam [Valium], meclizine HCl
mellitus).
[Antivert]).
2. Spasms of internal auditory artery.
(4) Diuretics may help (hydrochlorothiazide)
3. Head trauma.
to decrease endolymphatic fluid.
4. Allergic reaction.
b. Diet: low sodium (<2 gm/day).
5. High salt intake.
c. Avoid precipitating stimuli: bright, glaring
6. Smoking.
lights; noise; sudden jarring; turning head or
7. Ear infections.
eyes (stand in front of client when talking).
2164_Ch06_347-578 29/03/12 12:29 PM Page 387

Eyes, Ears, Nose and Throat 387


3. Goal: health teaching. 2. See THE PERIOPERATIVE EXPERIENCE,
a. No smoking (causes vasospasm) or alcoholic p. 365, for diagnoses relating to surgery.
beverages (fluid retention, contraindicated B. Nursing care plan/implementation:
with medications). 1. Preoperative health teaching.
b. Management of symptoms: play radio to a. Important to keep head in position ordered
mask tinnitus, particularly at night. by physician postoperatively.
c. Keep medication available at all times. b. Caution: sneezing, blowing nose (keep mouth
d. Prepare for surgery if indicated (labyrinthectomy open), vomiting, coughingall of which
if hearing gone; or vestibular neurectomy to increase pressure in eustachian tubes (blow
relieve vertigo and preserve hearing). one side gently).
F. Evaluation/outcome criteria: c. Breathing exercises.
1. Decreased frequency of attacks. 2. Postoperative:
2. Complies with treatment regimen and restric- a. Goal: promote physical and psychological
tions (e.g., low-sodium diet, no smoking). equilibrium.
3. Hearing preserved. (1) Position: as ordered by physicianvaries
III. OTOSCLEROSIS: disease of the bone of otic according to preference.
capsule; insidious, progressive deafness; most common (2) Activity: assist with ambulation; avoid
cause of conductive deafness; cause unknown. rapid turning, which might increase
vertigo.
A. Pathophysiology: formation of new spongy bone
(3) Dressings: check frequently; may change
in labyrinth fixation of stapes prevention
cotton pledget in outer ear.
of sound transmission through ossicles to inner
(4) Give medications as ordered:
ear fluids.
(a) Antiemetics.
B. Risk factors:
(b) Analgesics.
1. Heredity.
(c) Antibiotics.
2. Women, puberty to 45 years.
(5) Reassurance: reduction in hearing is
3. Pregnancy.
normal; hearing may not immediately
C. Assessment:
improve after surgery.
1. Subjective data:
b. Goal: health teaching.
a. Difficulty hearinggradual loss in both ears.
(1) Ear care: keep covered outdoors; keep
2. Diagnostic tests:
outer ear plug clean, dry, and changed.
a. Rinne (tuning fork placed over mastoid
(2) Avoid:
bone)reduced sound conduction by air and
(a) Water in ear for 6 weeks:
intensified by bone.
(i) Use barrier when washing hair.
b. Weber (tuning fork placed on top of head)
(ii) Use two pieces of cotton; saturate
increased sound conduction to affected ear.
outer piece with petroleum jelly.
c. Audiometrydiminished hearing ability.
(b) Pressure or vibration from loud noise,
D. Analysis/nursing diagnosis:

ADULT
flying, or heavy lifting until advised by
1. Auditory sensory/perceptual alteration related to
physician.
hearing loss.
C. Evaluation/outcome criteria:
2. Body image disturbance related to hearing aid.
1. Hearing improvesevaluate 1 month postopera-
3. Ineffective individual coping related to grief
tively (may require hearing aid).
reaction to loss.
2. Returns to work (usually 2 weeks after surgery).
4. Impaired social interaction related to hearing loss.
3. Continues medical supervision.
E. Nursing care plan/implementation, evaluation
outcome criteria (see IV. STAPEDECTOMY, V. DEAFNESS: (1) Hard of hearingslight or moderate
following). hearing loss that is serviceable for activities of daily
living. (2) Deafhearing is nonfunctional for activi-
IV. STAPEDECTOMY: removal of the stapes and
ties of daily living.
replacing it with a prosthesis (steel wire, Teflon piston,
or polyethylene); treatment for deafness due to oto- A. Risk factors:
sclerosis, which fixes the stapes, preventing it from 1. Conductive hearing losses (transmission
oscillating and transmitting vibrations to the fluids in deafness):
the inner ear. a. Impacted cerumen (wax).
b. Foreign body in external auditory canal.
A. Analysis/nursing diagnosis:
c. Defects (thickening, scarring) of eardrum.
1. Sensory/perceptual alteration related to edema and
d. Otosclerosis of ossicles.
ear packing.
2164_Ch06_347-578 29/03/12 12:29 PM Page 388

388 chapter 6 Physiological Integrity

2. Sensorineural hearing losses (perceptive or nerve 2. Goal: health teaching.


deafness): a. Prepare for evaluative studiesaudiogram.
a. Arteriosclerosis. b. Referral: appropriate community resources:
b. Infectious diseases (mumps, measles, National Association of Hearing and Speech
meningitis). Agencies for counseling services; National
c. Drug toxicities (quinine, streptomycin, Association for the Deaf to assist with employ-
neomycin SO4). ment, education, legislation; Alexander Graham
d. Tumors. Bell Association for the Deaf, Inc., serves as
e. Head traumas. information center for those working with
f. High-intensity noises. the client with hearing aid impairments;
3. Central deafness: American Hearing Society provides educa-
a. Tumors. tional information, employment services,
b. Stroke (brain attack). social clubs.
4. Noise-induced or occupational noise hearing loss: c. See Table 11.10 for care of hearing aids.
a. Blast injury. d. Safety precautions: when crossing street,
b. Firearms. driving.
c. Loud music. E. Evaluation/outcome criteria:
5. Aging (presbycusis). 1. Method of communication established.
B. Assessmentobjective data: 2. Achieves independence (use of Dogs for Deaf,
1. Facial expression: inattentive or strained. special telephones, visual signals).
2. Speech: excessive loudness or softness. 3. Copes with lifestyle changes (minimal depres-
3. Frequent need to clarify content of conversation sion, anger, hostility).
or inappropriate responses. VI. GLAUCOMA (acute and chronic): increased
4. Tilting of head while listening. intraocular pressure; second most common cause
5. Lack of response when others speak. of blindness.
6. Audiological examinations:
A. Pathophysiology:
a. Pure tone air conduction test.
1. Acute (closed-angle)impaired passage of aque-
b. Bone conduction test.
ous humor into the circular canal of Schlemm
c. Speech reception threshold.
due to closure of the angle between the cornea
d. Word recognition.
and the iris. Medical emergency; requires surgery.
C. Analysis/nursing diagnosis:
2. Chronic (open-angle)degenerative changes in
1. Auditory/sensory/perceptual alteration related to
trabecular meshwork; local obstruction of aque-
loss of hearing.
ous humor between the anterior chamber and
2. Impaired social interaction related to deafness.
the canal. Most common; treated with medications
D. Nursing care plan/implementation:
(miotics, carbonic anhydrase inhibitors).
1. Goal: maximize hearing ability and provide
3. Secondaryin some cases neovascularization
emotional support.
(new vessels) may form; blocks passage of
ADULT

a. Gain persons attention before speaking; avoid


aqueous humor (uveitis, trauma, drugs,
startling.
diabetes, retinal vein occlusion).
b. Provide adequate lighting so person can see
4. Untreated: imbalance between rate of secretion
who is speaking.
of intraocular fluids and rate of absorption
c. Look at the person when speaking.
of aqueous humor increased intraocular
d. Use nonverbal cues to enhance communica-
pressures decreased peripheral vision
tion (e.g., writing, hand gestures, pointing).
corneal edema halos and blurring of
e. Speak slowly, distinctly; do not shout
vision blindness.
(excessive loudness distorts voice).
B. Risk factorsunknown, but associated with:
f. If person does not understand, use different
1. Emotional disturbances.
words; write it down.
2. Hereditary factors.
g. Use alternative communication system:
3. Allergies.
(1) Speech (lip) reading.
4. Vasomotor disturbances.
(2) Sign language.
C. Assessment:
(3) Hearing aid.
1. Subjective data:
(4) Paper and pencil.
a. Acute (closed-angle):
(5) Flash cards.
(1) Pain: severe, in and around eyes.
h. Supportive, nonstressful environment; alert
(2) Rainbow halos around lights.
staff to clients hearing impairment.
2164_Ch06_347-578 29/03/12 12:29 PM Page 389

Eyes, Ears, Nose and Throat 389


(3) Blurring of vision. d. Medications: purpose, dosage, frequency;
(4) Nausea, vomiting. eyedrop instillation(1) wash hands; (2) head
b. Chronic (open-angle): back, expose conjunctiva of lower lid, instill
(1) Eyes tire easily. in center without touching eyelashes or eye;
(2) Loss of peripheral vision. (3) close eyes gently, apply slight pressure to
(3) Dull, morning headache. corner of eye to decrease systemic absorption;
2. Objective data: (4) wait at least 2 minutes before instilling a
a. Corneal edema. second eyedrop medication; have extra bottle
b. Decreased peripheral vision. in case of breakage or loss.
c. Increased cupping of optic disk. e. Activity: moderate exercisewalking.
d. Tonometrypressures greater than 22 mm Hg. f. Safety measures: eye protection (shield or
e. Pupils: dilated. glasses); Medic Alert band or tag; avoid
f. Redness of eye. driving 1 to 2 hours after instilling miotics.
D. Analysis/nursing diagnosis: g. Community resources as necessary.
1. Visual sensory/perceptual alterations related to F. Evaluation/outcome criteria:
increased intraocular pressure. 1. Eyesight preserved if possible.
2. Pain related to sudden increase in intraocular 2. Intraocular pressure lowered (<22 mm Hg).
pressure. 3. Continues medical supervision for lifereports
3. Risk for injury related to blindness. reappearance of symptoms immediately.
4. Impaired physical mobility related to impaired VII. CATARACT: developmental or degenerative
vision. opacification of the crystalline lens.
E. Nursing care plan/implementation:
A. Risk factors:
1. Goal: reduce intraocular pressure.
1. Aging (most common).
a. Activity: bedrest.
2. Trauma (x-rays, infrared or possibly ultraviolet
b. Position: semi-Fowlers.
exposure).
c. Medications as ordered:
3. Systemic disease (diabetes).
(1) Miotics (pilocarpine, carbachol); may not
4. Congenital defect.
be effective with intraocular pressure
5. Drug effects (corticosteroids).
(IOP) greater than 40 mm Hg.
B. Assessment:
(2) Carbonic anhydrase inhibitors (acetazo-
1. Subjective datavision: blurring, loss of acuity
lamide [Diamox]).
(sees best in low-light conditions); distortion;
(3) Anticholinesterase (demecarium bromide
diplopia; photophobia.
[Humorsol]) to facilitate outflow of aque-
2. Objective data:
ous humor.
a. Blindness: unilateral or bilateral (particularly
(4) Ophthalmic beta blockers (timolol) to
in congenital cataracts).
decrease IOP.
b. Loss of red reflex; gray or cloudy white
2. Goal: provide emotional support.
opacity of lens.

ADULT
a. Place personal objects within field of vision.
C. Analysis/nursing diagnosis:
b. Assist with activities.
1. Visual sensory/perceptual alterations related to
c. Encourage verbalization of concerns, fears of
opacity of lens.
blindness, loss of independence.
2. Risk for injury related to accidents.
3. Goal: health teaching.
3. Social isolation related to impaired vision.
a. Prevent increased IOP by avoiding:
(1) Anger, excitement, worry. VIII. CATARACT REMOVAL: removal of opacified
(2) Constrictive clothing. lens because of loss of vision; extracapsular cataract
(3) Heavy lifting. extraction followed by intraocular lens (IOL) insertion
(4) Excessive fluid intake. is procedure of choice.
(5) Atropine or other mydriatics that cause A. Nursing care plan/implementation:
dilation. 1. Preoperative:
(6) Straining at stool. a. Goal: prepare for surgery (ambulatory center).
(7) Eye strain. Antibiotic drops or ointment, mydriatic
b. Relaxation techniques; stress management if eyedrops as ordered; note dilation of pupils;
indicated. avoid glaring lights; usually done under local
c. Prepare for surgical intervention, if ordered: anesthetic with sedation.
laser trabeculoplasty, trabeculectomy (filter- b. Goal: health teaching. Postoperative expecta-
ing), laser peripheral iridotomy. tions: do not rub, touch, or squeeze eyes shut
2164_Ch06_347-578 29/03/12 12:29 PM Page 390

390 chapter 6 Physiological Integrity

after surgery; eye patches will be on; assistance IX. RETINAL DETACHMENT: separation of neural
will be given for needs; overnight hospitaliza- retina from underlying retinal pigment epithelium.
tion not required unless complications occur; A. Risk factors:
mild iritis usually occurs. 1. Trauma.
2. Postoperative: 2. Degeneration.
a. Goal: reduce stress on the sutures and prevent B. Assessment:
hemorrhage. 1. Subjective data:
(1) Activity: ambulate as ordered, usually soon a. Flashes of light before eyes.
after surgery; generally discharged few b. Vision: blurred, sooty (sudden onset);
hours after surgery. sensation of floating particles; blank areas
(2) Position: flat or low Fowlers; on back or of vision.
turn to nonoperative side 3 to 4 weeks, 2. Objective dataophthalmic examination: retina
because turning to operative side increases is grayish in area of tear; bright red, horseshoe-
pressure. shaped tear; B-mode ultrasonography.
(3) Avoid activities that increase IOP: straining C. Analysis/nursing diagnosis:
at stool, vomiting, coughing, brushing 1. Visual sensory/perceptual alteration related to
teeth, brushing hair, shaving, lifting blurred vision.
objects over 20 lb, bending, or stooping; 2. Anxiety related to potential loss of vision.
wear glasses or shaded lens during day, 3. Risk for injury related to blindness.
eye shield at night. D. Nursing care plan/implementation:
(4) Provide mouthwash, hair care, personal 1. Preoperative:
items within easy reach, step-in slippers a. Goal: reduce anxiety and prevent further
to avoid bending over. detachment.
b. Goal: promote psychological well-being. With (1) Encourage verbalization of feelings; answer
elderly, frequent contacts to prevent sensory all questions; reinforce physicians explana-
deprivation. tion of surgical procedures.
c. Goal: health teaching. (2) Activity: bedrest; eyes usually covered
(1) If intraocular lens not inserted, prescriptive to promote rest and maintain normal
glasses may be used (cataract glasses); position of retina; side rails up.
explain about magnification, perceptual (3) Position: according to location of retinal
distortion, blind areas in peripheral vision; tear; involved area of eye should be in a
guide through activities with glasses; need dependent position.
to look through central portion of lens and (4) Give medications as ordered: cycloplegics
turn head to side when looking to the side or mydriatics to dilate pupils widely and
to decrease distortion. decrease intraocular movement.
(2) Eye care: instillation of eyedrops (mydriatics (5) Relaxing diversion: conversation, music.
and carbonic anhydrase inhibitors) to pre- b. Goal: health teaching. Prepare for surgical
ADULT

vent glaucoma and adhesions if IOL not intervention (often combination used):
inserted; with IOL, steroid-antibiotic use (1) Cryopexy or cryotherapysupercooled
(see VI. GLAUCOMA, pp. 388389, for probe is applied to the sclera, causing a
correct technique); eye shield at night to scar, which pulls the choroid and retina
prevent injury for 1 month. together.
(3) Signs/symptoms of infection (redness, pain, (2) Laser photocoagulationa beam of intense
edema, drainage); iris prolapse (bulging or light from a carbon arc is directed through
pear-shaped pupil); hemorrhage (sharp eye the dilated pupil onto the retina; seals hole
pain, half-moon of blood). if retina not detached.
(4) Avoid: heavy lifting; potential eye trauma. (3) Scleral bucklingthe sclera is resected
B. Evaluation/outcome criteria: or shortened to enhance the contact
1. Vision restored. between the choroid and retina;
2. No complications (e.g., severe eye pain, frequently combined with cryopexy.
hemorrhage). (4) Banding or encirclementsilicone band
3. Performs self-care activities (e.g., instills or strap is placed under the extraocular
eyedrops). muscles around the globe.
4. Returns for follow-up ophthalmology care (5)Pneumatic retinopexyinstillation
recognizes symptoms requiring immediate of expandable gas or oil to tamponade
attention. tear.
2164_Ch06_347-578 29/03/12 12:29 PM Page 391

Eyes, Ears, Nose and Throat 391


2. Postoperative: 4. Diabetic retinopathy.
a. Goal: reduce intraocular stress and prevent 5. Atherosclerosis.
hemorrhage. 6. Trauma.
(1) Position: flat or low Fowlers; sandbags may B. Analysis/nursing diagnosis:
be used to position head; turn to nonoper- 1. Visual sensory/perceptual alteration related to
ative side if allowed, retinal tear dependent; blindness.
special positions may be: prone, side-lying, 2. Impaired social interaction related to loss of
or sitting with face down on table if gas or sight.
oil bubble injected; position may be 3. Risk for injury related to visual impairment.
restricted 4 to 8 days. 4. Self-care deficit related to visual loss.
(2) Activity: bedrest; decrease intraocular C. Nursing care plan/implementation:
pressure by not stooping or bending and 1. Goal: promote independence and provide emotional
avoiding prone position. support.
(3) Give medications as ordered: a. Familiarize with surroundings; encourage use
(a) Cycloplegics (atropine). of touch.
(b) Antibiotics. b. Establish communication lines; answer
(c) Corticosteroids to reduce eye move- questions.
ments and inflammation and prevent c. Deal with feelings of loss, overprotectiveness
infection. by family members.
(4)ROMisometric, passive; elastic d. Provide diversional activities: radio, CDs,
stockings to avoid thrombus related talking books, tapes.
to immobility. e. Encourage self-care activities; allow voicing
b. Goal: support coping mechanisms. of frustrations when activity is not done to
(1) Plan all care with client. satisfaction (spilling or misplacing some-
(2) Encourage verbalization of feelings, fears. thing), to decrease anger and discouragement.
(3) Encourage family interaction. 2. Goal: facilitate activities of daily living.
(4) Diversional activities. a. Eating:
c. Goal: health teaching. (1) Establish routine placement for tableware
(1) Eye care: eye patch or shield at night for (e.g., plate, glass).
about 2 weeks to prevent touching eye (2) Help person mentally visualize the plate
while asleep; dark glasses; avoid rubbing, as a clock or compass (e.g., 3 oclock
squeezing eyes. or east).
(2) Limitations: no reading, needlework for (3) Take persons hand and guide the finger-
3 weeks, no physical exertion for 6 weeks; tips to establish spatial relationship.
OK to watch TV, walk, except with bubble b. Walking:
restrictions. (1) Have person hold your forearm: walk a
(3)Medications: dosage, frequency, purpose, half-step in front.

ADULT
side effects: avoid nonprescription (2) Tell the person when approaching stairs,
medications. curb, incline.
(4) Signs of redetachment: flashes of light, c. Talking:
increase in floaters, blurred vision, acute (1) Speak when approaching person; tell
eye pain. person before you touch him or her.
E. Evaluation/outcome criteria: (2) Tell the person who you are and what
1. Vision restored. you will be doing.
2. No further detachmentrecognizes signs and (3) Do not avoid words such as see or
symptoms. discussing the appearance of things.
3. No injury occursaccepts limitations. 3. Goal: health teaching.
X. BLINDNESS: legally defined as vision less than a. Accident prevention in the home.
20/200 with the use of corrective lenses, or a visual b. Referral: community resources:
field of no greater than 20 degrees; greatest incidence (1) Voluntary agencies:
after 65 years. (a) American Foundation for the Blind
provides catalogs of devices for visually
A. Risk factors:
handicapped.
1. Glaucoma.
(b) National Society for the Prevention of
2. Cataracts.
Blindnesscomprehensive educational
3. Macular degeneration.
programs and research.
2164_Ch06_347-578 29/03/12 12:29 PM Page 392

392 chapter 6 Physiological Integrity

(c) Recording for the Blind, Inc.provides b. Bacterial aspiration pneumonia: related to poor
recorded educational books on free cough mechanisms due to anesthesia, coma
loan. (mixed flora of upper respiratory tract cause
(d) Lions Club. pneumonia).
(e) Catholic charities. 4. Hematogenous pneumonia bacterial infections:
(f ) Salvation Army. related to spread of bacteria from the
(2) Government agencies: bloodstream.
(a) Social and Rehabilitation Service B. Pathophysiology: caused by infectious or
counseling and placement services. noninfectious agents, clotting of an exudate rich
(b) Veterans Administrationscreening in fibrinogen, consolidated lung tissue.
and pensions. C. Assessment:
(c) State Welfare Department, Division for 1. Subjective data:
the Blindvocational. a. Pain location: chest (affected side), referred
D. Evaluation/outcome criteria: to abdomen, shoulder, flank.
1. Acceptance of disabilityparticipates in self-care b. Irritability, restlessness.
activities, remains socially involved. c. Apprehensiveness.
2. Regains independence with rehabilitation. d. Nausea, anorexia.
e. History of exposure.
2. Objective data:
R E S P I R AT O R Y a. Cough:
SYSTEM (1) Productive, rust-colored (blood) or
I. PNEUMONIA: acute inflammation of lungs with yellowish sputum (greenish with atypical
exudate accumulation in alveoli and other respiratory pneumonia).
passages that interferes with ventilation process. (2) Splinting of affected side when coughing.
A. Types: b. Sudden increased fever, chills.
1. Typical/classic pneumonia: pneumococcal; related c. Nasal flaring, circumoral cyanosis.
to diminished defense mechanisms, immuno- d. Respiratory distress: tachypnea.
compromised, critically ill, history of smoking, e. Auscultation:
general anesthesia/abdominal surgery, exposure (1) Decreased breath sounds on affected side.
to airborne pathogens, hospitalization, recent (2) Exaggerated breath sounds on unaffected
respiratory tract infection, viral influenza, side.
increased age, and chronic obstructive pul- (3) Crackles, bronchial breath sounds.
monary disease (COPD). (4) Dullness on percussion over consolidated
a. Lobar pneumoniaoccurs abruptly when an area.
acute bacterial infection affects a large portion (5) Possible pleural friction rub.
of a lobe; causes pleuritic pain, heavy sputum f. Chest retraction (air hunger in infants).
production. g. Vomiting.
ADULT

b. Bronchopneumoniainvolves patchy infiltra- h. Facial herpes simplex.


tion over a general area. i. Diagnostic studies:
c. Alveolar pneumoniacaused by virus; diffuse (1) Chest x-ray: haziness to consolidation.
bilateral infection without patchy infiltrates. (2)Sputum culture: Gram stain and
2. Atypical pneumonia: related to contact with culture; specific organisms, usually
specific organisms. pneumococcus.
a. Mycoplasma pneumoniae or Legionella pneu- (3) Bronchoscopy if sputum results are
mophila, if untreated, can lead to serious com- inconclusive.
plications such as acute respiratory distress j. Laboratory data:
syndrome (ARDS), disseminated intravascular (1) Blood culture: organism specific except
coagulation (DIC), thrombocytopenic purpura, when viral.
renal failure, inflammation of the heart, neu- (2) WBC count: leukocytosis.
rological disorders, or possible death. (3) Sedimentation rate: elevated.
b. Pneumocystis pneumonia in conjunction with 3. Factors contributing to the severity of
AIDS. pneumonia:
3. Aspiration pneumonia: a. Demographics:
a. Noninfectious: aspiration of fluids (gastric (1) Ageseverity increased with age.
secretions, foods, liquids, tube feedings) into (2) Gender.
the airways. (3) Nursing home resident.
2164_Ch06_347-578 29/03/12 12:29 PM Page 393

Respiratory System 393


b. Comorbidities: c. Diet: high carbohydrate, high protein to meet
(1) Congestive heart failure (CHF). energy demands and assist in the healing
(2) Active cancer. process.
(3) Liver disease. d. Mild analgesics for painno opioids.
(4) Renal insufficiency. 4. Goal: prevent potential complications.
(5) Stroke with residual symptoms. a. Cross infection: use good hand-washing
c. Physical examination: technique.
(1) Systolic BP less than 90; mean arterial b. Sterile technique when tracheobronchial suc-
pressure (MAP) less than 60. tioning to reduce risk of possible infection.
(2) Heart rate (HR) 125. c. Hyperthermia: tepid baths, hypothermia
(3) Respiratory rate 30. blanket.
(4) Temperature (PO) 104F or less than d. Respiratory insufficiency and acidosis: clear
95F. airway, promote expectoration of secretions.
(5) Altered level of consciousness (LOC). e. Assess cardiac and respiratory function.
d. Laboratory results: f. Keep ambulatory whenever possible.
(1) Hematocrit (Hct) less than 30. 5. Goal: health teaching.
(2) Na+ less than 130. a. Proper disposal of tissues, cover mouth when
(3) BUN 30. coughing.
(4) Arterial pH less than 7.35. b. Expected side effects of prescribed medications.
(5) Pleural effusion on chest x-ray. c. Need for rest, limited interactions, increased
(6) Glucose greater than 250 mg/dL. caloric intake.
D. Analysis/nursing diagnosis: d. Need to avoid future respiratory infections.
1. Ineffective airway clearance related to retained Immunization: influenza each year for those
secretions. at risk. Vaccine for pneumococcal pneumonia
2. Activity intolerance related to inflammatory every 5 years.
process. e. Correct dosage of antibiotics and the impor-
3. Pain related to continued coughing. tance of taking entire prescription at pre-
4. Knowledge deficit (learning need) related to scribed times (times evenly distributed
proper management of symptoms. throughout the 24-hour period to maintain
5. Risk for fluid volume deficit related to blood level of antibiotic) for increased
tachypnea. effectiveness.
E. Nursing care plan/implementation: F. Evaluation/outcome criteria:
1. Goal: promote adequate ventilation. 1. Adheres to medication regimen.
a. Deep breathe, cough. Small-volume nebulizer 2. Has improved gas exchange as shown by
treatment. improved pulmonary function tests.
b. Remove respiratory secretions, suction prn. 3. No acid-base or fluid imbalance: normal pH.
c. High humidity with or without oxygen 4. Energy level: increased.

ADULT
therapy. 5. Sputum production: decreased, normal color.
d. Intermittent positive-pressure breathing 6. Vital signs: stable.
(IPPB); incentive spirometry, chest physio- 7. Breath sounds: clear.
therapy, as ordered and needed to loosen 8. Cultures: negative.
secretions. 9. Reports comfort level increased.
e. Use of expectorants as ordered. II. SEVERE ACUTE RESPIRATORY SYNDROME
f. Change position frequently. (SARS): viral respiratory illness caused by a coron-
g. Percussion with postural drainage. avirus. Incubation period: 2 to 7 days, maybe as long
2. Goal: control infection. as 10 to 14 days. Recommend limiting contact after
a. Monitor vital signs; hypothermia for elevated infection until 10 days after fever has gone.
temperature.
A. Pathophysiology: little information is known
b. Administer antibiotics as ordered to control
about the SARS-associated coronavirus. May sur-
infectionbroad spectrum (e.g. penicillin,
vive in the environment for several daysdepends
quinolones, aminoglycosides). Note: need
on temperature or humidity, and type of material
cultures before starting on antibiotics.
or body fluid. Spread generally by respiratory
3. Goal: provide rest and comfort.
dropletsup to 3 feet. May spread through air;
a. Planned rest periods.
other ways not known. Progresses to hypoxia
b. Adequate hydration by mouth, I&O; IVs as
pneumonia respiratory distress syndrome.
needed.
2164_Ch06_347-578 29/03/12 12:29 PM Page 394

394 chapter 6 Physiological Integrity

B. Risk factors: influenza virus (novel H1N1 or 2009 H1N1 flu).


1. Weakened immune system. Described initially as a possible pandemic, a
2. Close contact (within 3 feet)kissing, hugging, technical term that refers to the geographical
sharing utensils. (global) spread of the disease, not severity. Called
C. Assessment: swine flu because it resembled a strain that
1. Subjective data: circulates in pigs.
a. Headache. A. Pathophysiology:
b. Feeling of discomfort; body aches; chills. 1. Influenza A virus with two proteins, hemagglu-
c. Dyspnea. tinin (H1) and neuraminidase (N1); human-
2. Objective data: to-human transmission through respiratory
a. Temperature greater than 100.4F (38.0C), secretions (droplets) from coughing or sneezing;
unless on antipyretics. touching something with the virus on the
b. Mild respiratory symptoms; dry cough. surface and then touching mouth, eyes, or
c. Diarrhea in 20%. nose. Not spread by food.
d. Laboratory: reverse transcriptasepolymerase 2. Incubation period: 1 day before symptoms
chain reaction (RT-PCR) of blood, stool, occur and up to 7 days following illness
nasal secretions; serologic test for antibodies; onset.
viral culture (showing antibodies to virus 3. Infectious (viral shedding): as long as sympto-
more than 21 days after onset of illness). matic; a minimum of 7 days after onset up to
e. Additional laboratory findings: 13 days. Young children may be contagious for
(1) Leukopenia. longer periods.
(2) Lymphopenia 4. Respiratory symptoms may progress to
(3) Thrombocytopenia. pneumonia respiratory failure death.
(4) lactose dehydrogenase. B. Risk factors:
(5) aspartate aminotransferase. 1. Ages 6 months to 24 years; children
(6) creatine kinase. younger than 5 years and especially younger
f. Chest x-ray: focal interstitial infiltrates than 2 years.
generalized patchy infiltrates areas of 2. Pregnant women and women who are 2 weeks
consolidation. postpartum.
D. Analysis/nursing diagnosis: 3. Persons 25 to 64 years with chronic
1. Ineffective breathing pattern related to hypoxia conditions (e.g., asthma, diabetes, heart
and pneumonia. disease, kidney disease, and weakened
2. Risk for spread of infection related to droplet or immune system).
airborne transmission. 4. Residents of nursing homes and chronic-care
3. Impaired gas exchange related to pneumonia. facilities.
E. Nursing care plan/implementation, evaluation/ C. Assessment:
outcome criteria (also see I. PNEUMONIA, 1. Subjective data: Classic flu:
ADULT

pp. 392393): a. Fatigue


1. Goal: infection control. b. Body aches; chills
a. Standard precautions (e.g., strict hand hygiene). c. Sore throat
b. Contact precautions (e.g., gown and gloves) d. Headache
with eye protection. e. Emergency signs with swine flu:
c. Airborne precautions (e.g., isolation room (1) Shortness of breath
with negative pressure; use of an N95 filtering (2) Chest pain or abdominal pain
disposable respirator for those entering room, (3) Sudden dizziness; confusion
or surgical mask). 2. Objective data: Classic flu:
d. Identify/isolate suspected SARS cases a. Fever above 100.4F
(quarantine). b. Cough
2. Goal: supportive care. c. Diarrhea; vomiting
a. Empirical antibiotic therapy with broad d. Runny nose
coverage. e. Emergency signs with swine flu:
b. Isolation (i.e., those without symptoms) for (1) Severe or persistent vomiting
10 days after becoming afebrile. (2)Difficulty breathing; rapid respiratory
III. H1N1 INFLUENZA VIRUS (swine flu): respi- rate
ratory disease caused by a new strain of the type A (3) Bluish or gray skin color (particularly in
children)
2164_Ch06_347-578 29/03/12 12:29 PM Page 395

Respiratory System 395

Emergency Warning Signs in Emergency Warning Signs most effective within 48 hours; 5 days of
Children with Swine Flu: in Adults with Swine Flu: treatment.
Fast breathing or trouble Difficulty breathing or (3) Fluids, lozenges; cough suppressant for dry
breathing shortness of breath cough.
Bluish or gray skin color Pain or pressure in the b. Fluids: clear liquids (water, broth, sports
Not drinking enough fluids chest or abdomen drinks, ice chips, frozen popsicles); avoid
Being so irritable that the child Sudden dizziness
does not want to be held Confusion
alcohol and caffeine.
Not waking up or not Severe or persistent c. Monitor for dehydration: urine color (dark
interacting vomiting yellow with dehydration); frequency of trips
Severe or persistent vomiting Flu-like symptoms to bathroom. Look for tears with infants/
Flu-like symptoms improve improve but then return toddlers; wet diapers.
but then return with fever with fever and worse
and worse cough cough
3. Goal: prevent occupational exposure.
a. Eliminate potential exposures: keep distance
Source: www.flu.gov. of 6 feet or more in community setting;
restrict visitors who are ill; keep sick workers
at home.
f. Diagnostic tests: viral culture, polymerase b. Respiratory hygiene and cough etiquette.
chain reaction (PCR), rapid antigen testing, Encourage ill person to use tissues or wear a
and immunofluorescence. Rapid influenza disposable face mask (use only once) or
diagnostic testdetects in 30 minutes, may health-care provider should wear mask.
not be conclusive. c. Respiratory protection (personal protective
D. Analysis/nursing diagnosis: equipment) in occupational health-care
1. Ineffective breathing pattern related to hypoxia settingN95 or higher filtering facepiece
and pneumonia. respirator; fit-tested (not recommended for
2. Risk for spread of infection related to droplet or children or people with facial hair); dispos-
airborne transmission. able, single use, do not share. Remove gloves
3. Risk for fluid volume deficit related to persistent and wash hands before and after touching
vomiting. respirator. Extended use of N95 increases
E. Nursing care plan/implementation: risk of contact transmission. Usually
1. Goal: prevent spread of infection. reserved if assisting with an aerosol-
a. Stay home from work or school at least generating procedureintubation,
24 hours after fever is gone without the use extubation, bronchoscopy, nebulization.
of a fever-reducing medicine; limit contact d. Isolation; standard (droplet) precautions
with others. private room preferred; if not available at
b. Cover nose and mouth with tissue when least 3-foot separation between patient beds
coughing or sneezing. with curtain between; share room with like
c. Wash hands after every sneeze or cough. diagnosis (cohorting). Caregiver wears mask

ADULT
d. Disinfect home surfaces to kill germs respirator not routinely necessary. Eye protec-
virus can survive 2 to 8 hours; use hydrogen tion if within 6 feet of patient as treatments
peroxide, iodophors, detergents, alcohol; may cause splashing of respiratory secretions.
heat. e. Limit transport of patient; client wears mask
2. Goal: reduce risk of complications, alleviate outside of room.
symptoms. f. Vaccination of staffmay be required;
a. Medications exempt: with severe egg allergy; history of
(1) Antipyretic: for fever 100F or higher; Guillain-Barr syndrome; religion prohibits
acetaminophen or ibuprofen; aspirin if vaccinations. May be required to wear mask
over 18 years of age. if not vaccinated.
(2) Antiviral: treatment or prevention; 4. Goal: health teachingmethods to prevent
stop spread of virus: oseltamivir (oral, reinfection.
Tamiflu)take with food, nausea a. Hand-washing techniquesoap, water, and
and vomiting may occur; zanamivir friction for at least 20 seconds; alcohol-based
(inhaled)do not take with history (antibacterial) hand rub may be used.
of asthma or lung disease; may cause b. Avoid touching eyes, nose, or mouth.
dizziness, stuffy nose, sinusitis; peramivir c. Vaccination: one dose of swine flu and seasonal
(IV, reserved for critically ill); neuraminidase flu vaccine for those 10 years of age and
inhibitors; suspected or confirmed cases; oldermay be given on same day; two doses
2164_Ch06_347-578 29/03/12 12:29 PM Page 396

396 chapter 6 Physiological Integrity

of swine flu vaccine (separated 4 weeks, c. Oxygen as ordered.


21 days minimum) for children younger d. Monitor effects of respiratory therapy, ventila-
than 10 years. tors, breathing assistance measures to ensure
d. Pneumococcal vaccination if over 65 years, proper gas exchange.
smoker, or history of chronic health problem. e. Position: on unaffected side to allow for lung
F. Evaluation/outcome criteria: expansion.
1. No complications. f. Prepare for possible needle decompression or
2. Temperature normal for 24 hours without chest tube insertion.
medication. 2. Goal: prevent complications.
3. Breath sounds clear; respirations unlabored. a. Antibiotics as ordered.
4. Covers nose/mouth when coughing or sneezing. b. Turn, cough, and deep breathe. Out of bed,
5. Performs hand hygiene frequently. ambulation.
IV. ATELECTASIS: collapsed alveoli in part or all of c. Increase fluid intake to liquefy secretions.
the lung. 3. Goal: health teaching.
a. Need to report signs and symptoms listed
A. Pathophysiology: due to compression (tumor),
in assessment data for early recognition of
airway obstruction, decreased surfactant produc-
problem.
tion, or progressive regional hypoventilation.
b. Importance of coughing and deep breathing
B. Risk factors:
to improve present condition and prevent
1. Shallow breathing due to pain, abdominal
further problems.
distention, narcotics, or sedatives.
F. Evaluation/outcome criteria:
2. Decreased ciliary action due to anesthesia,
1. Lung expanded on x-ray.
smoking.
2. Acid-base balance obtained and maintained.
3. Thickened secretions due to immobility,
3. No pain on respiration.
dehydration.
4. Activity level increased.
4. Aspiration of foreign substances.
5. Bronchospasms. V. PULMONARY EMBOLISM: undissolved mass that
6. Barotrauma; positive end-expiratory pressure travels in bloodstream and occludes a blood vessel; can
(PEEP). be thromboembolus, fat, air, or catheter. Constitutes a
C. Assessment: critical medical emergency.
1. Subjective data: A. Pathophysiology: obstructs blood flow to lung
a. Restlessness. increased pressure on pulmonary artery and
b. Pain. reflex constriction of pulmonary blood vessels
2. Objective data: poor pulmonary circulation pulmonary
a. Tachypnea. infarction.
b. Tachycardia. B. Risk factors:
c. Dullness on percussion. 1. Thrombophlebitis.
d. Absent bronchial breathing. 2. Recent surgery.
ADULT

e. Crackles at bases as alveoli pop open on 3. Invasive procedures.


inspiration. 4. Immobility.
f. Tactile fremitus in affected area. 5. Obesity
g. O2 saturation. 6. Myocardial infarction, heart failure.
h. X-ray: 7. Smoking.
(1) Patches of consolidation. 8. Varicose veins.
(2) Elevated diaphragm. 9. Hormone replacement therapy.
(3) Mediastinal shift. C. Assessment:
D. Analysis/nursing diagnosis: 1. Subjective data:
1. Impaired gas exchange related to shallow breathing. a. Chest pain: substernal, localized; type
2. Pain, acute, related to collapse of lung. crushing, sharp, stabbing with respirations.
3. Fear related to altered respiratory status. b. Sudden onset of profound dyspnea.
E. Nursing care plan/implementation: c. Restless, irritable, anxious.
1. Goal: relieve hypoxia. d. Sense of impending doom.
a. Frequent respiratory assessment. 2. Objective data:
b. Respiratory hygiene measures, cough, deep a. Respirations: either rapid, shallow or deep,
breathe. Use bedside inspirometer q1h when gasping.
awake. b. Elevated temperature.
2164_Ch06_347-578 29/03/12 12:29 PM Page 397

Respiratory System 397


c. Auscultation: friction rub, crackles; 3. Coagulation studies within normal limits (aPTT
diminished breath sounds. 25 to 41 seconds, ABGs within normal limits).
d. Shock: 4. Reports comfort achieved.
(1) Tachycardia. VI. HISTOPLASMOSIS: infection found mostly in
(2) Hypotension. central United States. Not transmitted from human to
(3) Skin: cold, clammy. human but from dust and contaminated soil. Progressive
e. Cough: hemoptysis. histoplasmosis, seen most frequently in middle-aged
f. X-ray: area of density. white men who have COPD, is characterized by
g. ECG changes that reflect right-sided failure. cavity formation, fibrosis, and emphysema.
h. Echocardiogram shows increased pulmonary
A. Pathophysiology: spores of Histoplasma capsulatum
dynamics.
(from droppings of infected birds and bats) are
i. Lung scan; pulmonary angiography.
inhaled, multiply, and cause fungal infections of
j. Laboratory data:
respiratory tract. Leads to necrosis and healing by
(1) Decreased PaCO2.
encapsulation.
(2) Elevated WBC count.
B. Assessment:
D. Analysis/nursing diagnosis:
1. Subjective data:
1. Ineffective breathing pattern related to shallow
a. Malaise.
respirations.
b. Chest pain, dyspnea.
2. Impaired gas exchange related to dyspnea.
2. Objective data:
3. Pain related to decreased tissue perfusion.
a. Weight loss.
4. Altered peripheral tissue perfusion related to
b. Nonproductive cough.
occlusion of blood vessel.
c. Fever.
5. Fear related to emergency condition.
d. Positive skin test for histoplasmosis.
6. Anxiety related to sense of impending doom.
e. Benign acute pneumonitis.
E. Nursing care plan/implementation:
f. Chest x-ray: nodular infiltrate.
1. Goal: monitor for signs of respiratory distress.
g. Sputum culture shows Histoplasma capsulatum.
a. Auscultate lungs for areas of decreased/absent
h. Hepatomegaly, splenomegaly.
breath sounds.
C. Analysis/nursing diagnosis:
b. Elevate head of bed.
1. Ineffective airway clearance related to
c. Monitor arterial blood gases (ABGs).
pneumonitis.
d. Monitor pulse oximetry; administer oxygen,
2. Ineffective breathing pattern related to dyspnea.
supplemental humidification as indicated.
3. Pain, acute, related to infectious process.
e. Monitor blood coagulation studies (e.g., acti-
4. Risk for infection related to repeated exposure to
vated partial thromboplastin time [aPTT]).
fungal spores.
f. Administer anticoagulation therapy, throm-
5. Impaired gas exchange related to chronic
bolytic agents, morphine for pain, vasopressor
pulmonary disease.
medications.
6. Knowledge deficit (learning need) related to

ADULT
g. Fluids: IV/PO as indicated.
prevention of disease and potential side effects
h. Monitor signs: Homans, acidosis.
of medications.
i. Ambulate as tolerated/indicated; change
D. Nursing care plan/implementation:
position.
1. Goal: relieve symptoms of the disease.
j. Prepare for surgery if peripheral embolectomy
a. Administer medications as ordered.
is indicated.
(1) Amphotericin B (IV) and ketoconazole.
2. Goal: health teaching.
(a) Monitor for drug side effects: local
a. Prevent further occurrence; importance of
phlebitis, renal toxicity, hypokalemia,
antiembolism stockings, intermittent pneu-
anemia, anaphylaxis, bone marrow
matic compression devices.
depression.
b. Decrease stasis.
(b)Azotemia (presence of nitrogen-
c. If history of thrombophlebitis, avoid birth
containing compounds in blood) is
control pills.
monitored by biweekly BUN or
d. Need to continue medication.
creatinine levels.
e. Follow-up care.
BUN greater than 40 mg/dL or creatinine
F. Evaluation/outcome criteria:
greater than 3.0 mg/dL necessitates
1. No complications; no further incidence of
stopping amphotericin B until values
emboli.
return to within normal limits.
2. Respiratory rate returns to normal.
2164_Ch06_347-578 29/03/12 12:29 PM Page 398

398 chapter 6 Physiological Integrity

(2) Aspirin, diphenhydramine HCl (Benadryl), C. Assessment:


promethazine HCl (Phenergan), prochlor- 1. Subjective data:
perazine (Compazine): used to decrease a. Loss of appetite, weight loss.
systemic toxicity of chills, fever, aching, b. Weakness, loss of energy.
nausea, and vomiting. c. Pain: knifelike, chest.
2. Goal: health teaching. d. Though client may be symptom free, the
a. Desired effects and side effects of prescribed disease is found on screening.
medications; importance of taking medications 2. Objective data:
for entire course of therapy (usually from a. Night sweats, chills.
2 weeks to 3 months). b. Fever: low grade, late afternoon.
b. Importance of follow-up laboratory tests to c. Pulse: increased.
monitor toxic effects of drug. d. Respiratory assessment:
c. Identify source of contamination if possible (1) Productive persistent cough, hemoptysis.
and avoid future contact if possible. (2) Respirations: normal, increased depth.
d. Importance of deep breathing, pursed-lip (3) Asymmetrical lung expansion.
breathing, coughing (see VIII. EMPHYSEMA, (4) Increased tactile fremitus.
p. 401, for specific care). (5) Dullness to percussion.
e. Signs and symptoms of chronic histoplasmosis, (6) Crackles following short cough.
COPD, drug toxicity, and drug side effects. e. Hoarseness.
E. Evaluation/outcome criteria: f. Unexplained weight loss.
1. Complies with treatment plan. g. Diagnostic tests:
2. Respiratory complications avoided. (1) Positive tuberculin test (Mantoux)reaction
3. Symptoms of illness decreased. to test begins approximately 12 hours after
4. No further spread of disease. administration with area of redness and a
5. Source of contamination identified and central area of induration. The peak time
removed. is 48 hours. Determination of positive or
VII. TUBERCULOSIS: inflammatory, communicable negative is made. A reaction is positive
disease that commonly attacks the lungs, although when it measures 10 mm. Contacts react-
may occur in other body parts. ing from 5 to 10 mm may need to be
treated prophylactically. This test is
A. Pathophysiology: exposure to causative organism
referred to as purified protein derivative
(Mycobacterium tuberculosis) in the alveoli in
(PPD) or intradermal skin test.
susceptible individual leads to inflammation.
(2) Sputum: three specimens tested positive for
Infection spreads by lymphatics to hilus; antibodies
acid-fast bacilli (AFB) (smear and culture).
are released, leading to fibrosis, calcification, or
Positive equals greater than 10 AFB per field.
inflammation. Exudate formation leads to caseous
(3) X-ray: infiltration cavitation.
necrosis, then liquefication of caseous material
h. Laboratory data: blooddecreased red blood
leads to cavitation.
ADULT

cell (RBC) count, increased sedimentation rate.


B. Risk factors:
i. Classification of tuberculosis:
1. Persons who have inhaled the tubercle bacillus
infectious particles called droplet nuclei. Class Description
2. Persons who have diseases or therapies known to 0 No TB exposure, not infected
suppress the immune system.
3. Immigrants from Latin America, Africa, Asia, 1 TB exposure, no evidence of infection
and Oceania living in the United States for less 2 TB infection, no disease
than 1 year. 3 TB: current disease (persons with com-
4. Americans living in those regions for a pleted diagnostic evidence of TBboth a
prolonged time. significant reaction to tuberculin skin test
5. Residents of U.S. metropolitan cities such as and clinical or x-ray evidence of disease)
New York, Miami; those who live in poverty 4 TB: no current disease (persons with
and are in overcrowded, poorly ventilated previous history of TB or with abnormal
x-ray films but no significant tuberculin
living conditions. skin test reaction or clinical evidence)
6. Men older than 65 years.
7. Women between ages 26 and 44 years and 5 TB: suspected (diagnosis pending)
(used during diagnostic testing period
older than 65 years. of suspected persons, for no longer than
8. Children younger than 5 years. 3 months)
2164_Ch06_347-578 29/03/12 12:29 PM Page 399

Respiratory System 399


D. Analysis/nursing diagnosis: c. Encourage client to maintain role in family
1. Ineffective airway clearance related to productive while home treatment is ongoing and to
cough. return to work and social contacts as soon
2. Impaired gas exchange related to asymmetrical as it is determined safe for progress of
lung expansion. treatment plan.
3. Pain related to unresolved disease process. 4. Goal: health teaching.
4. Body image disturbance related to feelings about a. Desired effects and side effects of medications:
tuberculosis. (1) INH may affect memory and ability to
5. Social isolation related to fear of spreading concentrate. May result in peripheral
infection. neuritis, hepatitis, rash, or fever.
6. Knowledge deficit (learning need) related to (2) Streptomycin may cause eighth cranial
medication regimen. nerve damage and vestibular ototoxity,
E. Nursing care plan/implementation: causing hearing loss; may cause labyrinth
1. Goal: reduce spread of disease. damage, manifested by vertigo and stag-
a. Administer medications: isoniazid (INH), gering; also may cause skin rashes, itching,
rifampin, pyrazinamide, ethambutol, or and fever.
streptomycin; or drug combinations such as (3) Important for client to know that medica-
Rifadin or Rifamate. Client will be treated as tion regimen must be adhered to for entire
an outpatient; may need to go to clinic for course of treatment.
directly observed therapy (DOT) to ensure (4) Discontinuation of therapy may allow
compliance with this long-term medication organism to flourish and make the disease
regimen. more difficult to treat.
b. The following may need to take 300 mg of b. Need for follow-up, long-term care, and
INH daily for 1 year as prophylactic measure: contact identification.
positive skin test reactors, including contacts; c. Importance of nutritious diet, rest, avoidance
persons who have diseases or are receiving of respiratory infections.
therapies that affect the immune system; d. Identify community agencies for support and
persons who have leukemia, lymphoma, follow-up.
or uncontrolled diabetes or who have had a e. Inform that this communicable disease must
gastrectomy. be reported.
c. Avoid direct contact with sputum. F. Evaluation/outcome criteria:
(1)Use good hand-washing technique after 1. Complies with medication regimen.
contact with client, personal articles. 2. Lists desired effects and side effects of medications
(2) Have client cover mouth and nose when prescribed.
coughing and sneezing, and use disposable 3. Gains weight, eats food high in protein and
tissues to collect sputum. carbohydrates.
d. Provide good circulation of fresh air. (Changes 4. Sputum culture becomes negative.

ADULT
of air dilute the number of organisms. This 5. Retains role in family.
plus chemotherapy provide protection needed 6. No complications (i.e., no hemorrhage, bacillus
to prevent spread of disease.) not spread to others).
e. Implement airborne or droplet precautions VIII. EMPHYSEMA: chronic disease with excessive
(see Chapter 3, pp. 8990, Table 3.5, p. 91, inflation of the air spaces distal to the terminal
Tables 3.7 and 3.8). bronchioles, alveolar ducts, and alveoli; character-
2. Goal: promote nutrition. ized by increased airway resistance and decreased
a. Increased protein, calories to aid in tissue repair diffusing capacity. Emphysema and chronic bronchitis
and healing. together constitute chronic obstructive pulmonary
b. Small, frequent feedings. disease (COPD).
c. Increased fluids, to liquefy secretions so they
A. Pathophysiology: imbalance between proteases,
can be expectorated.
which break down lung tissue, and 1-antitrypsin,
3. Goal: promote increased self-esteem.
which inhibits the breakdown. Increased airway
a. Encourage client and family to express
resistance during expiration results in air trapping
concerns regarding long-term illness and
and hyperinflation increased residual volumes.
treatment protocol.
Increased dead space unequal ventilation
b. Explain methods of disease prevention, and
perfusion of poorly ventilated alveoli hypoxia
encourage contacts to be tested and treated
and carbon dioxide retention (hypercapnia).
if necessary.
2164_Ch06_347-578 29/03/12 12:29 PM Page 400

400 chapter 6 Physiological Integrity

Chronic hypercapnia reduces sensitivity of i. Position: sits up, leans forward to compress
respiratory center; chemoreception in aortic arch abdomen and push up diaphragm, increasing
and carotid sinus become principal regulators of intrathoracic pressure, producing more
respiratory drive (respond to hypoxia). efficient expiration.
B. Risk factors: j. Pursed lips for greater expiratory breathing
1. Smoking. phase (pink puffer).
2. Air pollution: long-term exposure to environ- k. Weight loss due to hypoxia.
mental irritants, fumes, dust. l. Skin: ruddy color, nail clubbing; when
3. Antienzymes and 1-antitrypsin deficiencies. combined with bronchitis: cyanosis
4. Destruction of lung parenchyma. (blue bloater).
5. Family history and increased age. m. Respiratory: early diseasealkalosis; late
C. Assessment: diseaseacidosis, respiratory failure.
1. Subjective data: n. Spontaneous pneumothorax.
a. Weakness, lethargy. o. Cor pulmonale (emergency cardiac condi-
b. History of repeated respiratory infections, tion involving right ventricular failure due to
shortness of breath. increased pressure within pulmonary artery).
c. Long-term smoking. p. X-ray: hyperinflation of lung, flattened
d. Irritability. diaphragm; lung scan differentiates between
e. Inability to accept medical diagnosis and ventilation and perfusion.
treatment plan. q. Pulmonary function tests:
f. Refusal to stop smoking. (1) Prolonged rapid, forced exhalation.
g. Dyspnea on exertion, dyspnea at rest (2) Decreased: vital capacity (<4,000 mL);
(Table 6.14). forced expiratory volume.
2. Objective data: (3) Increased: residual volume (may be 200%);
a. Increased BP. total lung capacity.
b. Increased pulse. Decreased O2 saturation. r. Laboratory data:
c. Nostrils: flaring. (1) PaO2 less than 80 mm Hg, pH less than
d. Cough: chronic, productive. 7.35.
e. Episodes of wheezing, crackles. (2) PaCO2 greater than 45 mm Hg.
f. Increased anterior-posterior diameter of Note: In clients whose compensatory mecha-
chest (barrel chest). nisms are functioning, laboratory values may
g. Use of accessory respiratory muscles, be out of the normal range, but if a 20:1 ratio
abdominal and neck. of bicarbonate to carbonic acid is maintained,
h. Asymmetrical thoracic movements, decreased then appropriate acid-base balance also will
diaphragmatic excursion. be maintained. (Carbonic acid value can be

Table 6.14
ADULT

Differentiating Between Causes of Dyspnea


Chronic Obstructive
Asthma Pulmonary Disease Pneumothorax Pulmonary Edema Pulmonary Emboli
Characteristics Episodic, acute History of Chest pain: History of myocar- Sudden dyspnea
History of emphysema, sudden, sharp dial infarction Sharp chest pain
allergies bronchitis Sudden onset; Rapid weight gain Recent immobi-
Recent cold Heavy smoker associated with Cough lization, surgery,
or flu Recent cold or coughing, air Taking diuretics or fracture of
respiratory infection travel, strenuous Increasing need for lower extremities
exertion pillows for sleep History of: throm-
bophlebitis, use of
birth control pills,
sickle cell anemia
Assessment Wheezing Use of pursed-lip Tracheal deviation Crackles Tachycardia
Findings Hyperresonance breathing Asymmetrical Pink, frothy sputum Hypotension
Chest may be Wheezing, crackles chest motion Gallop (S3) Tachypnea
silent with bron- Barrel chest Diminished breath Air hunger Pleural rub
chospasms Uses accessory sounds
muscles to breathe
2164_Ch06_347-578 29/03/12 12:29 PM Page 401

Respiratory System 401


obtained by multiplying the PCO2 value succinate (Solu-Medrol); inhaled:
by 0.003.) triamcinolone acetonide (Azmacort),
D. Analysis/nursing diagnosis: beclomethasone (Beclovent, Vanceril),
1. Impaired gas exchange related to thick pulmonary flunisolide (AeroBid).
secretions. (4) Expectorants (increase water intake to
2. Ineffective breathing pattern related to hyperin- achieve desired effect): glyceryl guaiacolate
flated alveoli. (Robitussin).
3. Ineffective airway clearance related to pulmonary (5) Bronchial detergents/liquefying agents
secretions. (Mucomyst).
4. Altered nutrition, less than body requirements, h. Immunotherapy: helps ward off life-threaten-
related to weight loss due to hypoxia. ing influenza and pneumonia. Flu vaccination
5. Infection related to chronic disease process and every October or November. Pneumococcal
decreased ciliary action. vaccination routinely one dose; revaccinate
6. Activity intolerance related to increased energy 5 years later if high risk.
demands used for breathing. 2. Goal: employ comfort measures and support other
7. Sleep pattern disturbance related to changes in body systems.
body positions necessary for breathing. a. Oral hygiene prn; frequently client is mouth
8. Anxiety related to disease progression. breather.
9. Knowledge deficit (learning need) related to dis- b. Skin care: waterbed, air mattress, foam pads
ease, treatment, and self-care needs. to prevent skin breakdown.
E. Nursing care plan/implementation: c. Active and passive ROM exercises to prevent
1. Goal: promote optimal ventilation. thrombus formation; antiembolic stocking
a. Institute measures designed to decrease airway or woven elastic (Ace) bandages may be
resistance and enhance gas exchange. applied.
b. Position: Fowlers or leaning forward to d. Increase activities to tolerance.
encourage expiratory phase. e. Adequate rest and sleep periods to prevent
c. Oxygen with humidification, as orderedno mental disturbances due to sleep deprivation
more than 2 L/min to prevent depression and to reduce metabolic rate.
of hypoxic respiratory drive (see Oxygen 3. Goal: improve nutritional intake.
Therapy in Chapter 11, p. 835). May need a. High-protein, high-calorie diet to prevent
long-term oxygen therapy as disease progress- negative nitrogen balance.
es, to improve quality of life and reduce risk b. Give small, frequent meals.
of complications. c. Supplement diet with high-calorie drinks.
d. Intermittent positive-pressure breathing d. Push fluids to 3,000 mL/day, unless
(IPPB) with nebulization as ordered. contraindicatedhelps moisten
Small-volume nebulizer treatment. secretions.
e. Assisted ventilation. 4. Goal: provide emotional support for client and

ADULT
f. Postural drainage, chest physiotherapy. family.
g. Medications, as ordered: a. Identify factors that increase anxiety:
(1) Bronchodilators to increase airflow through (1) Fears related to mechanical equipment.
bronchial tree: inhaled: beta2-adrenergic (2) Loss of body image.
agonists (albuterol, metaproterenol); anti- (3) Fear of dying.
cholinergic agent: ipratropium (Atrovent); b. Assist family coping:
aminophylline, theophylline, terbutaline, (1) Do not reinforce denial or encourage
isoproterenol (Isuprel). overconcern.
(2) Antimicrobials to treat infection (deter- (2) Give accurate, up-to-date information on
mined by sputum cultures and sensitivity): clients condition.
trimethoprim and sulfamethoxazole (3) Be open to questioning.
(Bactrim, Septra); doxycycline, erythromy- (4) Encourage clientfamily communication.
cin, amoxicillin, cephalosporins, and (5) Provide appropriate diversional activities.
macrolides (condition deteriorates with 5. Goal: health teaching.
respiratory infections). a. Breathing exercises, such as pursed-lip
(3)Corticosteroids to decrease inflammation, breathing and diaphragmatic breathing.
mucosal edema, improve pulmonary b. Stress-management techniques.
function during exacerbation; systemic: c. Methods to stop smoking.
prednisone, methylprednisolone sodium d. Importance of avoiding respiratory infections.
2164_Ch06_347-578 29/03/12 12:29 PM Page 402

402 chapter 6 Physiological Integrity

e. Desired effects and side effects of prescribed 2. Nonimmunologic, or nonallergic asthma in per-
medications, possible interactions with over- sons who have a history of repeated respiratory
the-counter drugs. tract infections; age usually more than 35 years.
f. Purposes and techniques for effective 3. Mixed, combined immunologic and nonimmuno-
bronchial hygiene therapy. logic; any age, allergen or nonspecific stimuli.
g. Rest/activity schedule that increases with B. Risk factors:
ability. 1. History of allergies to identified or unidentified
h. Food selection for high-protein, high-calorie irritants; seasonal and environmental inhalants.
diet. 2. Recurrent respiratory infection.
i. Importance of taking 2,500 to 3,000 mL fluid C. Assessment:
per day (unless contraindicated by another 1. Subjective data:
medical problem). a. History: upper respiratory infection (URI),
j. Importance of medical follow-up. rhinitis, allergies, family history of asthma.
F. Evaluation/outcome criteria: b. Increasing tightness of the chest dyspnea
1. Takes prescribed medication. (see Table 6.14).
2. Participates in rest/activity schedule. c. Anxiety, restlessness.
3. Improves nutritional intake, gains appropriate d. Attack history:
weight for body size. (1) Immunologic: contact with allergen to
4. No complications of respiratory failure, cor which person is sensitive; seen most often
pulmonale. in children and young adults.
5. No respiratory infections. (2) Nonimmunologic: develops in adults older
6. Acid-base balance maintained through compen- than 35 years; aggravated by infections of
satory mechanisms, acidosis prevented. the sinuses and respiratory tract.
IX. ASTHMA: sometimes called reactive airway 2. Objective data:
disease (RAD) or reversible obstructive airway disease a. Peak flowmeter level drops.
(ROAD); a complex inflammatory process that causes b. Respiratory assessment: increased rate, audible
increased airway resistance and, over time, airway tis- expiratory wheeze (also inspiratory when
sue damage. Characterized by airway inflammation severe) on auscultation, hyperresonance on
and hyperresponsiveness to a variety of stimuli such as percussion, rib retractions, use of accessory
allergens, cold air, dust, smoke, exercise, medications muscles on inspiration.
(e.g., aspirin), some food additives, and viral infec- c. Tachycardia, tachypnea.
tions. Immunologic asthma occurs in childhood and d. Cough: dry, hacking, persistent.
follows other allergic disease. Nonimmunologic asthma e. General appearance: pallor, cyanosis,
occurs in adulthood and is associated with a history of diaphoresis, chronic barrel chest, elevated
recurrent respiratory tract infections. shoulders, flattened malar bones, narrow nose,
prominent upper teeth, dark circles under
A. Pathophysiology: triggers initiate the release of
eyes, distended neck veins, orthopnea.
inflammatory mediators such as histamine, which
ADULT

f. Expectoration of tenacious mucoid sputum.


produce airway obstruction through smooth
g. Diagnostic tests:
muscle constriction, microvascular leakage,
(1) Forced vital capacity (FVC): decreased.
mucus plugging, and swelling. This process
(2) Forced expiratory volume in 1 second
involves six sequential steps: (1) triggeringthe
(FEV1): decreased.
allergic or antigenic stimuli activate the inflam-
(3) Peak expiratory flow rate: decreased.
matory (mast) cells; (2) these mast cells signal
(4) Residual volume: increased.
the systemic immune system to release proinflam-
h. Laboratory data: blood gaseselevated PCO2;
matory substances; (3) migration of circulating
decreased PO2, pH.
inflammatory cells to regions of inflammation in
Emergency note: Persons severely affected may
the respiratory tract; (4) migrating cells are acti-
develop status asthmaticus, a life-threatening
vated by the proinflammatory mediators; (5) this
asthmatic attack in which symptoms of asthma
results in tissue damage; and (6) resolution. The
continue and do not respond to usual treat-
airways of many clients with asthma are chroni-
ment. Could lead to respiratory failure and
cally inflamed.
hypoxemia.
1. Immunologic, or allergic asthma in persons who
D. Analysis/nursing diagnosis:
are atopic (hypersensitivity state that is subject to
1. Ineffective airway clearance related to tachypnea.
hereditary influences); immunoglobulin E (IgE)
2. Impaired gas exchange related to constricted
usually elevated.
bronchioles.
2164_Ch06_347-578 29/03/12 12:29 PM Page 403

Respiratory System 403


3. Anxiety related to breathlessness. (6) Wait 1 to 2 minutes between puffs.
4. Activity intolerance related to persistent cough. Depending on the particular medication,
5. Knowledge deficit (learning need) related to may need to rinse mouth with water or
causal factors and self-care measures. mouthwash after each treatment.
E. Nursing care plan/implementation: e. Methods to facilitate expectorationincrease
1. Goal: promote pulmonary ventilation. humidity, postural drainage when appropriate,
a. Position: high-Fowlers for comfort. percussion techniques.
b. Medications as ordered: f. Breathing techniques to increase expiratory
(1) Rescue medications: corticosteroids (e.g., phase.
prednisone, methylprednisolone [Medrol]; g. Stress-management techniques.
beta-adrenergic agonists, such as albuterol h. Importance of recognizing early signs of
[Proventil, Ventolin], metaproterenol asthma attack and beginning treatment
[Alupent, Metaprel]). immediately.
(2) Maintenance medications: nonsteroidal i. Steps to take during an attack.
anti-inflammatory drugs: cromolyn F. Evaluation/outcome criteria:
(Intal), nedocromil (Tilade); corticosteroids: 1. No complications.
beclomethasone (Vanceril), triamcinolone 2. Has fewer attacks.
(Azmacort); leukotriene inhibitors/receptor 3. Takes prescribed medications, avoids infections.
antagonists: zafirlukast (Accolate), zileuton 4. Adjusts lifestyle.
(Zyflo); theophylline (Theo-Dur, Slo-Bid, 5. Pulmonary function tests return to normal.
Uni-Dur, theophylline ethylenediamino X. BRONCHITIS: acute or chronic inflammation of
[Aminophylline]); anticholinergic: bronchus resulting as a complication from colds and
ipratropium (Atrovent); beta agonists: flu. Acute bronchitis is caused by an extension of upper
salmeterol (Serevent); mast cell stabilizers: respiratory infection, such as a cold, and can be given
nedocromil sodium (Tilade). to others. It can also result from an irritation from
(3) Antibiotics to control infection. physical or chemical agents. Chronic bronchitis is
c. Oxygen therapy with increased humidity as characterized by hypersecretion of mucus and chronic
ordered. cough for 3 months per year for 2 consecutive years.
d. Frequent monitoring for respiratory distress.
A. Pathophysiology: bronchial walls are infiltrated
e. Rest periods and gradual increase in activity.
with lymphocytes and macrophages; lumen
2. Goal: facilitate expectoration.
becomes obstructed due to decreased ciliary action
a. High humidity.
and repeated bronchospasms. Hyperventilation of
b. Increase fluid intake.
alveolar sacs occurs. Long-term condition results
c. Monitor for dehydration.
in respiratory acidosis, recurrent pneumonitis,
d. Respiratory therapy: IPPB.
emphysema, or cor pulmonale.
3. Goal: health teaching to prevent further attacks.
B. Risk factors:
a. Identify and avoid all asthma triggers.
1. Smoking.

ADULT
b. Teach importance of peak flowmeter readings.
2. Repeated respiratory infections.
c. Medicationswhen to use, how to use, side
3. History of living in area where there is much
effects, withdrawals.
air pollution.
d. Using a metered-dose inhaler:
C. Assessment:
(1) Shake vigorously.
1. Subjective data:
(2) Position inhaler about 1 inch in front
a. History: recurrent, chronic cough, especially
of mouth. Use 2 fingers to measure the
when arising in the morning.
1-inch distance.
b. Anorexia.
(3) A spacer is recommended: connect spacer
2. Objective data:
device, shake, press down the canister;
a. Respiratory:
place lips on the mouthpiece and take a
(1) Shortness of breath.
slow deep breath through mouth. Without
(2) Use of accessory muscles.
spacer: breathe out all the way, open
(3) Cyanosis, dusky complexion: blue bloater.
mouth wide, and take a slow deep breath
(4) Sputum: excessive, nonpurulent.
through mouth. Press down the canister
(5) Vesicular and bronchovesicular breath
and continue to breathe in.
sounds; wheezing.
(4) Once lungs are full, hold breath for
b. Weight loss.
10 seconds if possible.
c. Fever.
(5) Exhale normally through pursed lips.
2164_Ch06_347-578 29/03/12 12:29 PM Page 404

404 chapter 6 Physiological Integrity

d. Pulmonary function tests: A. Pathophysiology: damage to alveolar capillary


(1) Decreased forced expiratory volume. membrane, increased vascular permeability creating
(2) PaO2 less than 90 mm Hg; PaCO2 greater noncardiac pulmonary edema, and impaired gas
than 40 mm Hg. exchange; decreased surfactant production
e. Laboratory data: atelectasis; severe hypoxia; refractory to FiO2
(1) RBC count: elevated to compensate for possible death.
hypoxia (polycythemia). B. Risk factors:
(2) WBC count: elevated to fight infection. 1. Primary:
D. Analysis/nursing diagnosis: a. Shock, multiple trauma.
1. Ineffective airway clearance related to excessive b. Infections.
sputum. c. Aspiration, inhalation of chemical toxins.
2. Ineffective breathing pattern related to need to d. Drug overdose.
use accessory muscles for breathing. e. Disseminated intravascular coagulation
3. Impaired gas exchange related to shortness of (DIC).
breath. f. Emboli, especially fat emboli.
4. Activity intolerance related to increased energy 2. Secondary:
used for breathing. a. Overaggressive fluid administration.
E. Nursing care plan/implementation: b. Oxygen toxicity.
1. Goal: assist in optimal respirations. c. Mechanical ventilation.
a. Increase fluid intake. C. Assessment:
b. IPPB, chest physiotherapy. 1. Subjective data:
c. Administer medications as ordered: a. Restlessness, anxiety.
(1) Bronchodilators. b. History of risk factors.
(2) Antibiotics. c. Severe dyspnea (see Table 6.14).
(3) Bronchial detergents, liquefying agents. 2. Objective data:
2. Goal: minimize bronchial irritation. a. Cyanosis.
a. Avoid respiratory irritants (e.g., smoke, dust, b. Tachycardia.
cold air, allergens). c. Hypotension.
b. Environment: air-conditioned, increased d. Hypoxemia, acidosis.
humidity. e. Crackles.
c. Encourage nostril breathing rather than f. X-raybilateral patchy infiltrates.
mouth breathing. g. Death if untreated.
3. Goal: improve nutritional status. D. Analysis/nursing diagnosis:
a. Diet: soft, high calorie. 1. Anxiety related to serious physical condition.
b. Small, frequent feedings. 2. Ineffective breathing pattern related to severe
4. Goal: prevent secondary infections. dyspnea.
a. Administer antibiotics as ordered. 3. Impaired gas exchange related to alveolar
ADULT

b. Avoid exposure to infections, crowds. damage.


5. Goal: health teaching. 4. Altered tissue perfusion related to hypoxia.
a. Avoid respiratory infections. E. Nursing care plan/implementation:
b. Medications: desired effects and side 1. Goal: assist in respirations.
effects. a. May require mechanical ventilatory support
c. Methods to stop smoking. to maintain respirations.
d. Rest and activity balance. b. May need to be transferred to ICU.
e. Stress management. c. May need oxygen to combat hypoxia.
F. Evaluation/outcome criteria: d. Suction prn.
1. Stops smoking. e. Monitor blood gas results to detect early
2. Acid-base balance maintained. signs of acidosis/alkalosis.
3. Respiratory infections less frequent. f. If not on ventilator, assess vital signs and
XI. ACUTE ADULT RESPIRATORY DISTRESS respiratory status every 15 minutes.
SYNDROME (ARDS) (formerly called by other g. Cough, deep breathe every hour.
names, including shock lung): noncardiogenic pul- h. May need:
monary infiltrations resulting in stiff, wet lungs and (1) Rotation therapy and/or prone
refractory hypoxemia in an adult who was previously position.
healthy. Acute hypoxemic respiratory failure without (2) Postural drainage, suction.
hypercapnea. (3) Bronchodilator medications.
2164_Ch06_347-578 29/03/12 12:29 PM Page 405

Respiratory System 405


2. Goal: prevent complications. 4. Laboratory values and pressures within normal
a. Decrease anxiety and provide psychological limits.
care: 5. Urinary output greater than 30 mL/hr.
(1) Maintain a calm atmosphere. XII. PNEUMOTHORAX: presence of air within the
(2) Encourage rest to conserve energy. pleural cavity; occurs spontaneously or as a result of
(3) Emotional support. trauma (Fig. 6.3).
b. Obtain fluid balance:
A. Types:
(1) Slow IV flow rate.
1. Closed (spontaneous): rupture of a subpleural
(2) Diuretics: rapid acting, low dose.
bulla, tuberculous focus, carcinoma, lung
c. Monitor:
abscess, pulmonary infarction, severe coughing
(1) Pulmonary artery and capillary wedge
attack, or blunt trauma.
pressure cardiac output.
2. Open (traumatic): communication between
(2) Central venous pressure (CVP), peripheral
atmosphere and pleural space because of opening
perfusion, arterial line BP.
in chest wall.
(3) I&O.
3. Tension: one-way leak; may occur during
(4) Assess for bleeding tendencies, potential
mechanical ventilation or cardiopulmonary
for disseminated intravascular coagulation.
resuscitation (CPR), or as a complication of any
d. Protect from infection:
type of spontaneous or traumatic pneumothorax.
(1) Strict aseptic technique.
Positive pressure within chest cavity resulting
(2) Antibiotic therapy.
from accumulated air that cannot escape during
(3) Deep vein thrombosis prophylaxis.
expiration. Leads to collapse of lung, mediastinal
e. Provide physiological support:
shift, and compression of the heart and great
(1) Maintain nutrition.
vessels.
(2) Skin care.
B. Pathophysiology: pressure builds up in the pleural
3. Goal: health teaching.
space, lung on the affected side collapses, and the
a. Briefly explain procedures as they are happen-
heart and mediastinum shift toward the unaffected
ing (emergency situation can frighten client).
lung.
b. Give rationale for follow-up care.
C. Assessment:
c. Identify risk factors as appropriate for preven-
1. Subjective data:
tion of recurrence.
a. Pain:
F. Evaluation/outcome criteria:
(1) Sharp, aggravated by activity.
1. Client survives and is alert.
(2) Locationchest; may be referred to shoul-
2. Skin warm to touch.
der, arm on affected side.
3. Respiratory rate within normal limits.

ADULT
PNEUMOTHORAX
(OPEN THE CHEST WALL INJURY PERMITS
AIR TO FLOW IN AND OUT OF THE PLEURAL
SPACE ON THE AFFECTED SIDE)

TRACHEA AND MEDIASTINUM


SHIFTED AWAY FROM
PNEUMOTHORAX
HEART AND VESSELS IN
AIR HAS ENTERED THE THE MEDIASTINUM
TRAUMATIC PLEURAL SPACE AND
RUPTURE OF COLLAPSED THE LUNG NORMAL
THE CHEST WALL PLEURAL
SPACE

PLEURAL SPACE
FILLED WITH AIR

INHALATION: AIR ENTERS THE INJURED EXHALATION: THE AIR IS PARTIALLY Figure 6.3 Pneumothorax. (From Venes, D [ed]:
SIDE, CAUSING COLLAPSE OF THE LUNG FORCED FROM THE AFFECTED SIDE
AND SHIFT OF THE MEDIASTINUM AND PLEURAL SPACE AND THE MEDIASTINUM Tabers Cyclopedic Medical Dictionary, ed 21. FA Davis,
HEART TOWARD THE UNAFFECTED SIDE SHIFT TOWARD THE AFFECTED SIDE Philadelphia, 2009.)
2164_Ch06_347-578 29/03/12 12:29 PM Page 406

406 chapter 6 Physiological Integrity

b. Restlessness, anxiety. 5. Goal: prevent complications with chest tubes.


c. Dyspnea (see Table 6.14). a. Observe for and immediately report: crepita-
2. Objective data: tions (air under skin, also called subcutaneous
a. Cough. emphysema), labored or shallow breathing,
b. Cessation of normal movements on affected tachypnea, cyanosis, tracheal deviation, or
side. signs of hemorrhage.
c. Absence of breath sounds on affected side. b. Monitor for signs of infection.
d. Pallor, cyanosis. c. Ensure that tubing stays intact.
e. Shock. d. Monitor proper tube function. Attach chest
f. Tracheal deviation to unaffected side. tube to a water-seal drainage apparatus
g. X-ray: air in pleural space. and use wall suction for negative pressure.
D. Analysis/nursing diagnosis: Monitor amount and color of tube drainage
1. Ineffective breathing pattern related to collapse every 2 hours. Notify physician if bloody
of lung. drainage exceeds 100 mL/hr. Chest drainage
2. Impaired gas exchange related to abnormal system should be at least 1 foot (30 cm)
thoracic movement. below the chest tube insertion site. Change
3. Pain related to trauma to chest area. dressing at tube insertion site every 48 hours.
4. Fear related to emergency situation. Air bubbles will continue in the water-seal
E. Nursing care plan/implementation: chamber for 24 to 48 hours after insertion.
1. Goal: prevent damage until medical intervention Persistent air bubbles indicate an air leak
available. between alveoli and pleural space. Fluctuation
a. Place sterile occlusive gauze dressing over of fluid level is expected (when suction is
wound. off ) because respiration changes the pleural
b. Tape dressing on three sides to allow air to pressure. If a clot forms in the tube, gently
escape during expiration. squeeze the tube, without occluding it, to
c. Place client on affected side to diminish move the clot, or follow specific orders as
possibility of tension pneumothorax. written. Make sure tube is free of kinks.
2. Goal: protect against injury during thoracentesis. When moving client, do not clamp tube;
a. Provide sterile equipment. disconnect it from the wall suction. See key
b. Explain procedure. points for nursing intervention with chest
c. Monitor vital signs for shock. tubes in Table 11.5.
d. Monitor for respiratory distress, mediastinal e. Change position every 1 to 2 hours.
shift. f. Arm and shoulder ROM.
3. Goal: promote respirations. 6. Goal: health teaching.
a. Position: Fowlers. a. How to prevent recurrence by avoiding
b. Oxygen therapy as ordered. overexertion; avoid holding breath.
c. Encourage slow breathing to improve gas b. Signs and symptoms of condition.
ADULT

exchange. c. Methods to stop smoking.


d. Careful administration of narcotics to d. Encourage follow-up care.
prevent respiratory depression (avoid F. Evaluation/outcome criteria:
morphine). 1. No complications noted.
4. Goal: prepare client for closed chest drainage, 2. Closed system remains intact until chest tubes
physically and psychologically. are removed.
a. Explain purpose of the procedureto 3. Lung reexpands, breath sounds heard, pain
provide means for evacuation of air and diminished, symmetrical thoracic movements.
fluid from pleural cavity; to reestablish XIII. HEMOTHORAX: presence of blood in pleural cavity
negative pressure in pleural space; to pro- related to trauma or ruptured aortic aneurysm (see XII.
mote lung reexpansion. PNEUMOTHORAX for assessment, analysis/nursing
b. Explain procedure and apparatus (see Chest diagnosis, nursing care plan/implementation, and
tubes section in Table 11.5). evaluation/outcome criteria). Table 6.15 compares
c. Cleanse skin at tube insertion site; place client pneumothorax and hemothorax.
in sitting position, ensuring safety by having
locked over-the-bed table for client to lean XIV. CHEST TRAUMA
on, or have a nurse stay with client so appro- Flail chest: multiple rib fractures resulting in insta-
priate position is maintained throughout the bility of the chest wall, with subsequent paradoxical
procedure. breathing (portion of lung under injured chest wall
2164_Ch06_347-578 29/03/12 12:29 PM Page 407

Respiratory System 407

Table 6.15
Comparison of Pneumothorax and Hemothorax
Pneumothorax (Free Air Between Visceral and Parietal Pleurae) Hemothorax (Blood in Pleural Space)
Cause Spontaneousair enters without trauma; rupture of bleb (vesicle) Blunt trauma (assaults, falls)
on pleura; nonlife threatening
Traumaticopen chest wound: life threatening; closed chest Penetrating trauma (knives, gunshot
woundusually nonlife threatening unless untreated wounds)
Tensionair cannot escape via route of entry; causes atelectasis
Assessment Findings Minimal to sudden onset; sharp chest pain; dyspnea; cough; Asymptomatic or pleuritic pain and
hypotension; tachypnea; tachycardia; mediastinal shift (tracheal dyspnea; decreased or absent breath
deviation) sounds; often discovered during
chest x-ray; in severe cases: shock

moves in on inspiration while remaining lung 2. Impaired gas exchange related to asymmetrical
expands; on expiration, the injured portion of chest expansion.
the chest wall expands while unaffected lung tissue 3. Pain related to chest trauma.
contracts). 4. Fear related to emergency situation.
Sucking chest wound: penetrating wound of chest 5. Risk for trauma related to fractured ribs.
wall with hemothorax and pneumothorax, resulting in 6. Risk for infection related to open chest wound.
lung collapse and mediastinal shift toward unaffected C. Nursing care plan/implementation:
lung. 1. Goal: restore adequate ventilation and prevent fur-
A. Assessment: ther air from entering pleural cavity: MEDICAL
1. Subjective data: EMERGENCY.
a. Severe, sudden, sharp pain. a. In emergency situation: place air-occlusive
b. Dyspnea. dressing or hand over open wound as client
c. Anxiety, restlessness, fear, weakness. exhales forcefully against glottis (Valsalva
2. Objective data: maneuver helps expand collapsed lung by
a. Vital signs: creating positive intrapulmonary pressures);
(1) Pulse: tachycardia, weak. or place clients weight onto affected side.
(2) BP: hypotension. Administer oxygen.
(3) Respirations: shallow, decreased expiratory b. Assist with endotracheal tube insertion; client
force, tachypnea, stridor, accessory muscle will be placed on volume-controlled ventila-
breathing. tor. (See discussion of ventilators under
b. Skin color: cyanosis, pallor. Oxygen Therapy in Chapter 11.)
c. Chest: c. Assist with thoracentesis and insertion of

ADULT
(1) Asymmetrical chest expansion (paradoxical chest tubes with connection to water-seal
movement). drainage as ordered (see Chest tubes section
(2) Chest wound, rush of air through in Table 11.5).
trauma site. d. Monitor vital signs to determine early shock.
(3) Crepitus over trauma site (from air e. Monitor blood gases to determine early
escaping into surrounding tissues). acid-base imbalances.
(4) Lateral deviation of trachea, mediastinal f. Pain medications given with caution, so as
shift. not to depress respiratory center.
d. Pneumothorax: documented by absence of D. Evaluation/outcome criteria:
breath sounds, x-ray examination. 1. Respiratory status stabilizes, lung reexpands.
e. Hemothorax: documented by needle aspira- 2. Shock and hemorrhage are prevented.
tion by physician, x-ray examination. 3. No further damage done to surrounding
f. Shock; blood and fluid loss. tissues.
g. Hemoptysis. 4. Pain is controlled.
h. Distended neck veins. XV. THORACIC SURGERY: used for bronchogenic
B. Analysis/nursing diagnosis: and lung carcinomas, lung abscesses, tuberculosis,
1. Ineffective airway clearance related to shallow bronchiectasis, emphysematous blebs, and benign
respirations. tumors.
2164_Ch06_347-578 29/03/12 12:29 PM Page 408

408 chapter 6 Physiological Integrity

A. Types: (2) Position: high-Fowlers.


1. Thoracotomyincision in the chest wall, pleura (a) Turn client who has had a pneumonec-
is entered, lung tissue examined, biopsy secured. tomy to operative side (avoid extreme
Chest tube is needed postoperatively. lateral positioning and mediastinal
2. Lobectomyremoval of a lobe of the lung. shift) to allow unaffected lung expan-
Chest tube is needed postoperatively. sion and drainage of secretions; can
3. Pneumonectomyremoval of an entire lung. also be turned onto back.
No chest tube is needed postoperatively. (b) Client who has had a lobectomy or
B. Analysis/nursing diagnosis: thoracotomy can be turned on either
1. Risk for injury related to chest wound. side or back because chest tubes will
2. Impaired gas exchange related to pain from be in place.
surgical procedure. c. Goal: reduce incisional stress and discomfort
3. Ineffective airway clearance related to decreased pad area around chest tube when turning on
willingness to cough due to pain. operative side to maintain tube patency and
4. Pain related to surgical incision. promote comfort.
5. Impaired physical mobility related to large surgical d. Goal: prevent complications related to
incision and chest tube drainage apparatus. respiratory function.
6. Knowledge deficit (learning need) related to (1) Maintain chest tubes to water-seal
importance of coughing and deep breathing to drainage system.
prevent complications. (2) See Chest tubes section in Table 11.5.
C. Nursing care plan/implementation: (3) Observe for mediastinal shift (trachea
1. Preoperative care: should always be midline; movement
a. Goal: minimize pulmonary secretions. toward either side indicates shift).
(1) Humidify air to moisten secretions. (a) Move client onto back or toward
(2) Use IPPB, as ordered, to improve ventilation. opposite side.
(3) Administer bronchodilators, expectorants, (b) MEDICAL EMERGENCY: Notify
and antibiotics as ordered. physician immediately.
(4) Use postural drainage, cupping, and e. Goal: maintain fluid and electrolyte balance.
vibration to mobilize secretions. (1) Administer parenteral infusion slowly
b. Goal: preoperative teaching. (risk of pulmonary edema due to
(1) Teach client to cough against a closed decrease in pulmonary vasculature
glottis to increase intrapulmonary pressure with removal of lung lobe or whole
for improved expiratory phase. lung).
(2) Instruct in diaphragmatic breathing and f. Goal: postoperative teaching.
coughing. (1) Prevent ankylosis of shoulderteach
(3) Encourage to stop smoking. passive and active ROM exercises of
(4) Instruct and supervise practice of postop- operative arm.
ADULT

erative arm exercisesflexion, abduction, (2) Importance of early ambulation, as


and rotation of shoulderto prevent condition permits.
ankylosis. (3) Importance of stopping smoking.
(5) Explain postoperative use of chest tubes, (4) Dietary instructionsnutritious diet to
IV, and oxygen therapy. aid in healing process.
2. Postoperative care: (5) Importance of deep breathing, coughing
a. Goal: maintain patent airway. exercises, to prevent stasis of respiratory
(1) Auscultate chest for breath sounds; report secretions.
diminished or absent breath sounds on (6) Importance of increased fluids in diet to
unaffected side (indicates decreased venti- liquefy secretions.
lation respiratory embarrassment). (7) Desired and side effects of prescribed
(2) Turn, cough, and deep breathe, every medications.
15 minutes to 1 hour for first 24 hours (8) Importance of rest, avoidance of
and prn according to pulmonary conges- heavy lifting and work during healing
tion heard on auscultation. process.
b. Goal: promote gas exchange. (9) Importance of follow-up care; give
(1) Splint chest during coughingsupport names of referral agencies where client
incision to help cough up sputum (most and family can obtain assistance.
important activity postoperatively). (10) Signs and symptoms of complications.
2164_Ch06_347-578 29/03/12 12:29 PM Page 409

Respiratory System 409


D. Evaluation/outcome criteria: 4. Fear related to need for specialized equipment
1. Client or significant other or both will be able to: to breathe.
a. Give rationale for activity restriction and B. Nursing care plan/implementation:
demonstrate prescribed exercises. 1. Preoperative care:
b. Identify name, dosage, side effects, and a. Goal: relieve anxiety and fear.
schedule of prescribed medications. (1) Explain purpose of procedure and
c. State plans for necessary modifications in equipment.
lifestyle, home. (2) Demonstrate suctioning procedure.
d. Identify support systems. (3) Establish means of postoperative commu-
2. Wound heals without complications. nication (e.g., paper and pencil, magic
3. Obtains ROM in affected shoulder. slate, picture cards, and call bell).
4. No complications of thoracotomy, such as: Specialized tubes such as a fenestrated
a. Respiratorypulmonary insufficiency, tracheostomy tube or a tracheostomy
respiratory acidosis, pneumonitis, button allow the individual to talk when
atelectasis, pulmonary edema. the external opening is plugged.
b. Circulatoryhemorrhage, hypovolemia, (4)Remain with client as much as
shock, myocardial infarction. possible.
c. Mediastinal shift. 2. Postoperative care:
d. Renal failure. a. Goal: maintain patent airway (Table 6.16).
e. Gastric distention. b. Goal: alleviate apprehension.
XVI. TRACHEOSTOMY: opening into trachea, tem- (1) Remain with client as much as possible.
porary or permanent. Rationale: airway obstruction (2)Encourage client to communicate feel-
due to foreign body, edema, tumor, excessive tracheo- ings using preestablished communication
bronchial secretions, respiratory depression, decreased system.
gaseous diffusion at alveolar membrane, increased c. Goal: improve nutritional status.
dead space (e.g., severe emphysema), or failure to (1) Provide nutritious foods/liquids the
wean from mechanical ventilator. client can swallow.
(2) Give supplemental drinks to maintain
A. Analysis/nursing diagnosis:
necessary calories.
1. Ineffective airway clearance related to increased
d. Goal: health teaching.
secretions and decreased ability to cough
(1) Explain all procedures.
effectively.
(2) Teach alternative methods of communica-
2. Ineffective breathing pattern related to physical
tion (best if done before the tracheostomy
condition that necessitated tracheostomy.
if it is not an emergency situation).
3. Impaired verbal communication related to
(3) Teach self-care of tracheostomy as soon as
inability to speak when tracheostomy tube
possible.
cuff inflated.

ADULT
Table 6.16
Tracheostomy Suctioning Procedure
1. Suction as necessary to facilitate respirations. Explain to client what to expect.
2. Position: semi-Fowlers to prevent forward flexion of neck, to facilitate respiration, to promote drainage, and to minimize
edema.
3. Administer mist to tracheostomy because natural humidifying of oropharynx pathways has been eliminated.
4. Auscultate for moist, noisy respirations because nonproductive coughing may indicate need for suctioning.
5. Prevent hypoxia by administering 100% oxygen before suctioning (unless contraindicated).
6. Use strict aseptic technique and sterile suctioning catheters with each aspiration; use sterile saline to clear catheter of secre-
tions. Keep dominant hand gloved with sterile glove, nondominant hand with nonsterile glove to control thumb control of suction.
Suction tracheostomy before nose or mouth.
7. Do not apply suction when inserting suction catheter to prevent injury to respiratory tract and prevent loss of oxygen. Insert
catheter about 5 inches (12.5 cm).
8. If client coughs during suctioning, gently remove catheter to permit ejection and suction of mucus.
9. Apply suction intermittently for no longer than 510 seconds because prolonged suction decreases arterial oxygen concentra-
tion. Allow 23 minutes between attempts.
10. Cuff deflation: if high-volume, low-pressure cuffed tube is used, deflation not necessary. If other tracheostomy cuffed tube is
used, deflate for 5 minutes every hour to prevent damage to trachea.
11. Use caution not to dislodge tube when changing dressing or ties that secure tube.
2164_Ch06_347-578 29/03/12 12:29 PM Page 410

410 chapter 6 Physiological Integrity

C. Evaluation/outcome criteria: 7. Lack of activity.


1. Airway patent. 8. Genetics, heredity.
2. Acid-base balance maintained. C. Classifications:
3. No respiratory infection/obstruction. 1. Essential (primary or idiopathic): occurs in 90% to
95% of clients; etiology unknown; diastolic pres-
sure is 90 mm Hg, and other causes of hyperten-
C A R D I O VA S C U L A R sion are absent. Benign hypertension (diastolic
SYSTEM pressure 120 mm Hg) considered controllable;
I. HYPERTENSION: sustained, elevated, systemic asymptomatic until complications develop.
arterial blood pressure; diastolic elevation more seri- 2. Secondary: occurs in remaining 5% to 10%;
ous, reflecting pressure on arterial wall during resting usually renal, endocrine, neurogenic, or cardiac
phase of cardiac cycle (Table 6.17). in origin.
A. Pathophysiology: increased peripheral resistance 3. Malignant hypertension (diastolic > 140 to
leading to thickened arterial walls and left ventricu- 150 mm Hg); uncontrollable. May arise from
lar hypertrophy. both types.
B. Risk factors: 4. Labile (prehypertensive): a fluctuating blood
1. Black race (2:1). pressure; increases during stress, otherwise
2. Use of birth control pills. normal or near normal.
3. Overweight. D. Assessment:
4. Smoking. 1. Subjective data:
5. Stress. a. Early-morning headache, usually occipital.
6. Excessive sodium intake or saturated fat. b. Light-headedness, tinnitus.

Table 6.17
Imbalances in Blood Pressure: Comparative Assessment of Hypotension and Hypertension
Hypotension Hypertension
Common Causes
Angina pectoris Essential hypertension
Myocardial infarction Iron-deficiency anemia
Acute and chronic pericarditis Pernicious anemia
Valvular defects Arteriosclerosis obliterans
Heart failure Polycythemia vera
Assessment
Behavior Anxiety, apprehension, decreasing mentation, Nervousness, mood swings, irritability, difficulty with
ADULT

confusion memory, depression, confusion


Neurological Essentially noncontributory Decreased vibratory sensations, increased/decreased
reflexes, Babinski reflex, changes in coordination
Head/neck Distended neck veins, worried expression Bruits over carotids, distended neck veins, epistaxis,
diplopia, ringing in ears, dull occipital headaches on
arising
Skin Pale, cool, moist Dry, pale, glossy, flaky, cold; decreased or absent hair
GI Anorexia, nausea, vomiting, constipation Anorexia, flatulence, diarrhea, constipation
Respiratory Dyspnea, orthopnea, paroxysmal nocturnal Dyspnea, orthopnea, crackles
dyspnea, tachypnea, moist crackles, cough
Cardiovascular Tires easily Decreased exercise tolerance, weakness, palpitations
Blood pressuredecreased systolic, decreased Blood pressureincreased systolic and/or diastolic
systolic/diastolic
Pulseincreased/decreased/weak, thready, Decreased or absent pedal pulses
irregular, arrhythmias
Renal Oliguria Oliguria, nocturia, proteinuria
Extremities Dependent edema Tingling, numbness, or cold hands and feet, dependent
edema, ulcers of legs or feet
2164_Ch06_347-578 29/03/12 12:29 PM Page 411

Cardiovascular System 411


c. Palpitations. (faintness, nausea, vomiting, hypotension,
d. Fatigue, insomnia. sexual dysfunction) (see Chapter 8 for
e. Forgetfulness, irritability. specific pharmacological actions).
f. Altered vision: white spots, blurring, or loss. c. Weight control to reduce arterial pressure.
2. Objective data: d. Restrictions: stimulants (tea, coffee, tobacco),
a. Epistaxis (nosebleeds). sodium, calories, fat.
b. Elevated blood pressure: systolic greater than e. Lifestyle adjustments: daily exercise needed;
120 mm Hg, diastolic greater than 90 mm Hg; reduce occupational and environmental
narrowed pulse pressure. Rise in diastolic from stress; importance of rest.
sitting to standing with essential; fall in BP f. Blood pressure measurement: daily, same
from sitting to standing with secondary. conditions, both arms, position preference of
c. Retinal changes; papilledema. physician; use of self-monitoring cuff; check
d. Shortness of breath on slight exertion. at least twice per week.
e. Cardiac, cerebral, and renal changes. g. Signs, symptoms, complications of disease
f. Laboratory data: urinalysis, ECG, chest x-ray (headache, confusion, visual changes,
to rule out complications of hypertension. nausea/vomiting, convulsions).
E. Analysis/nursing diagnosis: h. Causes of intermittent hypotension: alcohol,
1. Knowledge deficit (learning need) regarding hot weather, exercise, febrile illness, hot bath.
condition, treatment plan, and self-care and G. Evaluation/outcome criteria:
discharge needs. 1. Blood pressure within normal range for age
2. Risk for decreased cardiac output related to (diastolic less than 90 mm Hg)stable.
ventricular hypertrophy, vasoconstriction, or 2. Minimal or no pathophysiological or therapeutic
myocardial ischemia. complications (e.g., visual changes, stroke, drug
3. Risk for injury related to complications of side effects).
hypertension. 3. Reduces weight to reasonable level for height,
4. Impaired adjustment related to required lifestyle bone structure.
changes. 4. Takes prescribed medications regularly, even after
5. Activity intolerance related to weakness, fatigue. symptoms have resolved.
F. Nursing care plan/implementation: 5. Complies with restrictions: no smoking; restricted
1. Goal: provide for physical and emotional rest. sodium, fat.
a. Rest periods before/after eating, visiting 6. Exercises regularlyprogram compatible with
hours; avoid upsetting situations. personal and health-care goals.
b. Give tranquilizers, sedatives, as ordered. II. CARDIAC ARRHYTHMIAS (DYSRHYTHMIAS):
2. Goal: provide for special safety needs. any variations in normal rate, rhythm, or configura-
a. Monitor blood pressure: both arms; standing, tion of waves on ECG (Fig. 6.4).
sitting, lying positions.
A. Pathophysiology:
b. Limit/prevent activities that increase pressure
1. Dysfunction of sinoatrial (SA) node, atria,

ADULT
(anxiety, anger, frustration, upsetting visitors,
atrioventricular (AV) node, or ventricular
fatigue).
conduction.
c. Assist with ambulation; change position
2. Primary heart problem or secondary systemic
gradually to prevent dizziness and light-
problem.
headedness (postural hypotension).
B. Risk factors:
d. Monitor for electrolyte imbalance when on
1. Myocardial infarction.
low-sodium diet, diuretic therapy; I&O to
2. Drug toxicity.
prevent fluid depletion and arrhythmias from
3. Stress.
potassium loss.
4. Cardiac surgery.
e. Observe for signs of hemorrhage, shock,
5. Hypoxia.
stroke, which may occur following surgery.
6. Congenital.
3. Goal: health teaching (client and family).
C. Assessment: see Table 6.18 for specific
a. Procedures to decrease anxiety; relaxation
dysrhythmias.
techniques, stress management.
D. Analysis/nursing diagnosis:
b. Side effects of hypotensive drugs: initial
1. Decreased cardiac output related to abnormal
therapy includes diuretics and beta blockers;
ventricular function.
if response inadequate may use angiotensin-
2. Altered tissue perfusion related to inadequate
converting enzyme (ACE) inhibitors, adrenergic
cardiac functioning.
blockers, vasodilators, calcium channel blockers (text continues on page 415)
2164_Ch06_347-578 29/03/12 12:29 PM Page 412

412 chapter 6 Physiological Integrity

P wave Depolarization of atrial


R
5 mm muscle
0.2 SECOND 1 mm
QRS complex Depolarization of ventricular
0.04 muscle
SECOND
T wave Ventricular repolarization
T
PR interval Time from start of atrial
depolarization to start of
0.1 mv
1 mm

P-R S-T
SEGMENT SEGMENT ventricular depolarization
0.5 mv

(1220 sec)
QRS interval Total time for ventricular
P
depolarization (610 sec)
U
QT interval Total time for ventricular
depolarization and
repolarization
S-T
INTERVAL Rate/rhythm 60100, regular
Q
P-R P-QRS ratio 1:1
INTERVAL

QRS S
INTERVAL
Q-T
INTERVAL

Figure 6.4 Interpretation of normal cardiac cycle. (From Venes, D [ed]: Tabers Cyclopedic Medical Dictionary, ed 21. FA Davis, Philadelphia,
2009.)

Table 6.18
Comparison of Selected Cardiac Dysrhythmias
Dysrhythmia Description Etiology Symptoms/Consequences Treatment
Dysrhythmias of Sinus Node
ADULT

Sinus dysrhythmia Phasic shortening, Respiratory variation Usually none Usually none
then lengthening in impulse initiation Atropine if rate below
of P-P and R-R by SA node 40 beats/min
intervals
Sinus tachycardia P waves present Increased metabolic May produce palpitations Treat underlying cause
followed by QRS demands Prolonged episodes may Occasionally sedatives
complex Decreased oxygen lead to decreased cardiac
Rhythm regular delivery output
Heart rate Heart failure
100150 Shock
beats/min Hemorrhage
Anemia
Sinus bradycardia P waves present Physical fitness Very low rates may cause Atropine if cardiac
followed by QRS Parasympathetic decreased cardiac output: output is decreased
complex stimulation (sleep) light-headedness, faintness, Pacemaker
Rhythm regular Brain lesions chest pain Treat underlying cause if
Heart rate Sinus dysfunction necessary
<60 beats/min Digitalis excess
2164_Ch06_347-578 29/03/12 12:29 PM Page 413

Cardiovascular System 413

Table 6.18
Comparison of Selected Cardiac Dysrhythmiascontd
Dysrhythmia Description Etiology Symptoms/Consequences Treatment
Atrial Dysrhythmias
Premature atrial Early P wave Stress May produce palpitations Sedation
beats QRS complex Ischemia Frequent episodes may Eliminate nicotine and caffeine
may or may not Atrial enlargement decrease cardiac output May require no other treatment
be normal Caffeine Is sign of chamber
Rhythm irregular Nicotine irritability
Atrial tachycardia P wave present Sympathetic Palpitations Usually none if short burst
(may merge into stimulation Possible anxiety (<1 min)
previous T wave), Chemical stimuli Hypotension Prolonged episodes may
QRS complex (caffeine, nicotine) require carotid artery
usually normal; Drug toxicity pressure, vagal
rapid heart rate Fluid-electrolyte stimulation, verapamil,
(usually >150 imbalance digitalis, beta blockers,
beats/min) Thoracic surgery calcium channel blockers
Atrial fibrillation Rapid, irregular P Rheumatic heart Hypotension Digitalis
waves (>350/min) disease Palpitations
Ventricular rhythm Mitral stenosis Pulse deficit Cardizem
irregularly irregular Atrial infarction Decreased cardiac output
Ventricular rate Coronary atheroscle- if rate is rapid Amiodarone
varies, may rotic heart disease Promotes thrombus forma- Anticoagulation
increase to Hypertensive heart tion in atria Cardioversion
120150/min if disease
untreated Thyrotoxicosis
Atrial flutter Sawtooth or Heart failure Occasional palpitations Cardioversion
picket fence Mitral valve disease Chest pain Anticoagulation
P waves (220 Pulmonary embolus medications if cardiover-
350 beats/min) sion unsuccessful
Ratio of atrial to
ventricular rate
constant (3:1,
4:1, etc.)
Ventricular Dysrhythmias
Premature Early, wide, bizarre Stress Same as for premature atrial Check Mg++, K+ levels
ventricular beats QRS complex, not Acidosis beats MEDICATIONS:
(PVBs) associated with a Ventricular Procainamide
P wave enlargement Disopyramide (Norpace)
Rhythm irregular Electrolyte imbalance Lidocaine

ADULT
Myocardial infarction Mexiletine
Digitalis toxicity Sodium bicarbonate
Hypoxemia Potassium
Hypercapnia Oxygen
Treat heart failure
Ventricular No P wave before PVBs striking during Decreased cardiac output: MEDICATIONS:
tachycardia QRS complex; vulnerable period hypotension, loss of Lidocaine
QRS complex Hypoxemia consciousness, respiratory Procainamide
wide and bizarre; Drug toxicity arrest Amiodarone
ventricular rate Electrolyte imbalance Cardioversion
>100, usually Bradycardia Electrolytes
140240
Ventricular Chaotic electrical Myocardial infarction No cardiac output Defibrillation
fibrillation activity Electrocution Absent pulse or respiration MEDICATIONS:
No recognizable Freshwater drowning Cardiac arrest Epinephrine
QRS complex Drug toxicity Lidocaine
Sodium bicarbonate
Bretylium
Magnesium sulfate
CPR
Continued
2164_Ch06_347-578 29/03/12 12:29 PM Page 414

414 chapter 6 Physiological Integrity

Table 6.18
Comparison of Selected Cardiac Dysrhythmiascontd
Dysrhythmia Description Etiology Symptoms/Consequences Treatment
Pulseless electrical Organized ECG Electromechanical Pulseless CPR
activity (PEA) rhythm dissociation Minimal or no perfusion Epinephrine
Escape rhythms Fluid challenge
Ventricular standstill Can be distin- Myocardial Same as for ventricular CPR
guished from infarction fibrillation Pacemaker
ventricular fibrilla- Chronic diseases Intracardiac
tion only by ECG of conducting epinephrine
P waves may be system
present
No QRS complex
Straight line
Impulse Conduction Deficits
First-degree P-R interval Rheumatic fever Warns of impaired Usually none as long as it
atrioventricular prolonged, Digitalis toxicity conduction occurs as an isolated deficit
[AV] block >0.20 sec Degenerative Atropine if P-R interval
changes of coronary >0.26 sec or
atherosclerotic heart bradycardia
disease
Infections (e.g.,
Lyme carditis)
Decreased oxygen
in AV node
Bundle branch Same as normal Hypoxia Same as first-degree Usually none unless severe
block sinus rhythm Acute myocardial AV block blockage of left posterior
(NSR) except infarction division (see text)
QRS complex Heart failure
duration Coronary atheroscle-
>0.10 sec rotic heart disease
Pulmonary embolus
Hypertension
Second-degree AV P waves usually Acute myocardial Serious dysrhythmia that May require temporary
blocks occur regularly at infarction may lead to decreased pacemaker
rates consistent Same as first- heart rate and cardiac If symptomatic
with SA node ini- degree AV block output, hypotension (e.g., hypotension,
tiation (not all P dizziness),
waves followed atropine 1 mg
by QRS complex;
ADULT

P-R interval may


lengthen before
nonconducted P
wave or may be
consistent; QRS
complex may be
widened)
Complete Atria and Digitalis toxicity Very low rates may cause Pacemaker
third-degree AV ventricles beat Infectious disease decreased cardiac output: Isoproterenol to
block independently Coronary artery light-headedness, fainting, increase heart rate
P waves have no disease chest pain Epinephrine if isopro-
relation to QRS Myocardial terenol ineffective
complex infarction
Ventricular rate
may be as low as
2040 beats/min
From Phipps, W, Long, B, Woods, N, and Cassmeyer, V (eds): Medical-Surgical Nursing, ed. 5. Mosby, St. Louis,
2007.
2164_Ch06_347-578 29/03/12 12:29 PM Page 415

Cardiovascular System 415


3. Knowledge deficit (learning need) regarding subclavian insertion increases catheter
cause/treatment of condition, self-care, and stability.
discharge needs. (2) Secure wires to chest to prevent tension
4. Anxiety related to dependence, fear of death. on catheter.
E. Nursing care plan/implementation: (3) Do not defibrillate over insertion site, to
1. Goal: provide for emotional and safety needs. avoid electrical hazards.
a. Document ECG tracing for presence of (4) Electrical safety (grounding; disconnect
life-threatening arrhythmia. electric beds/call lights; use battery-operated
b. Encourage discussion of fears, feelings (client equipment).
and significant other). (5) Check settings.
c. Bedrest: restricted activities; quiet environ- c. Permanent pacemaker:
ment; limit visitors. (1) Limit activity of shoulder for 48 to 72 hours
d. Oxygen, if ordered. after insertion of transvenous catheter to
e. Check vital signs frequently for shock, heart prevent dislodgement; avoid extending arms
failure (HF), drug toxicity. over head for 8 weeks.
f. Prepare for cardiac emergency: CPR. (2) Postinsertion ROM (passive) at least once
g. Give cardiac medications; check laboratory per shift after 48 hours to prevent frozen
tests for digitalis and potassium levels, to shoulder.
prevent drug toxicity. (3) If defibrillation is required, place paddles
2. Goal: prevent thromboemboli. at least 4 inches from pulse generator.
a. Apply antiembolic stockings (TED hose); (4) Check site.
segmental compression device. d. Health teaching following permanent pacemaker:
b. Give anticoagulants as ordered. (Check for (1) Explain procedure: duration, equipment,
bleedinggums, urine; monitor laboratory purpose, type of pacemaker.
testsLee-White clotting time and activated (2) Medic Alert bracelet; pacemaker informa-
partial thromboplastin time with heparin; tion card.
prothrombin time or international normalized (3) Daily pulse taking on arising (report varia-
ratio [INR] with warfarin [Coumadin].) tion of 5 beats).
c. Encourage flexion and extension of feet. (4) Signs, symptoms of malfunction (vertigo,
3. Goal: prepare for cardioversion with atrial syncope, dyspnea, slowed speech, confu-
fibrillation if indicated (usually if pulse greater sion, fluid retention); infection (fever, heat,
than 140 beats/min, symptomatic, or no pain, skin breakdown at insertion site).
conversion after 3 days of drug therapy and (5) Restrictions: limit vigorous arm and
anticoagulated). shoulder motion for 6 to 8 weeks; contact
a. Give Cardizem or amiodarone as ordered sports; electromagnetic interferences
at least 24 hours before. (few)TV/radio transmitters, improperly
b. NPO 8 hours before. functioning microwave oven (maintain dis-

ADULT
c. Hold digoxin morning of cardioversion tance of 3 feet), certain cautery machines;
per order. may trigger airport metal-detector alarm.
d. Give conscious sedation medications as F. Evaluation/outcome criteria:
ordered. 1. Regular cardiac rhythm, monitors own radial
4. Goal: provide for physical and emotional needs pulse.
with pacemaker insertion. 2. No complications (e.g., pacemaker malfunction).
a. General concerns: 3. Returns for regular follow-up of pacemaker
(1) Report excessive bleeding/infection at function.
insertion sitehematoma may contribute 4. Tolerates physical or sexual activity.
to wound infection. 5. Wears identification bracelet; carries pacemaker
(2) Encourage verbalization of feelings. identification card.
(3) Report prolonged hiccups, which may 6. Reports anxiety is reduced to manageable level.
indicate pacemaker failure. III. CARDIAC ARREST: sudden unexpected cessation
(4) Know pacing mode: fixed-rate or demand of heartbeat and effective circulation leading to inade-
(most common); type of insertion (tempo- quate perfusion and sudden death.
rary or permanent), sensitivity.
A. Risk factors:
b. Temporary pacemaker:
1. Myocardial infarction.
(1) Limit excessive activity of extremity if ante-
2. Multiple traumas.
cubital insertion, to prevent displacement;
2164_Ch06_347-578 29/03/12 12:29 PM Page 416

416 chapter 6 Physiological Integrity

3. Respiratory arrest. A. Pathophysiology:


4. Drowning. 1. Insufficient blood flow through coronary
5. Electrical shock. arteries. Oxygen demand exceeds supply.
6. Drug reactions. 2. Temporary myocardial ischemia.
7. Multisystem failure. B. Risk factors:
B. Assessmentobjective data: 1. Cardiovascular:
1. Unresponsive to stimuli (i.e., verbal, painful). a. Atherosclerosis.
2. Absence of breathing, carotid pulse. b. Thromboangiitis obliterans.
3. Pale or bluish: lips, fingernails, skin. c. Aortic regurgitation.
4. Pupils: dilated. d. Hypertension.
C. Analysis/nursing diagnosis: 2. Hormonal:
1. Decreased cardiac output related to heart failure. a. Hypothyroidism.
2. Impaired gas exchange related to breathlessness. b. Diabetes mellitus.
3. Altered tissue perfusion related to pulselessness. 3. Blood disorders:
D. Nursing care plan/implementation: a. Anemia.
1. Goal: prevent irreversible cerebral anoxic damage: b. Polycythemia vera.
initiate CPR within 4 to 6 minutes; continue 4. Lifestyle choices:
until relieved; document assessment factors, a. Smoking.
effectiveness of actions; presence or absence of b. Obesity.
pulse at 1 minute and every 4 to 5 minutes. c. Cocaine use.
2. Goal: establish effective circulation, respiration d. Inactivity.
(see Cardiovascular Emergencies, Table 6.44, C. Assessment:
for complete protocols). 1. Subjective data:
E. Evaluation/outcome criteria: a. Paintypical (Table 6.19).
1. Carotid pulse present; check after 1 minute and (1) Type: squeezing, pressing, burning.
every few minutes thereafter. (2)Location: retrosternal, substernal,
2. Responds to verbal stimuli. left of sternum, radiates to left arm
3. Pupils constrict in response to light. (Fig. 6.5).
4. Return of spontaneous respiration; adequate (3) Duration: short, usually 3 to 5 minutes,
ventilation. less than 30 minutes.
IV. ARTERIOSCLEROSIS: loss of elasticity, thickening, (4) Cause: emotional stress, overeating,
hardening of arterial walls; symptoms depend on physical exertion, exposure to cold; may
organ system involved; common typeatherosclerosis. occur at rest.
Atherosclerosis (coronary heart disease [CHD]) pre- (5) Relief: rest, nitroglycerin.
cedes angina pectoris and myocardial infarction. b. Note: Atypical complaints by women include
jaw and upper back pain and persistent gastric
A. Pathophysiology:
upset.
1. Atherosclerotic plaque, discrete lumpy thicken-
ADULT

c. Dyspnea.
ing of arterial wall; cholesterol-lipid-calcium
d. Palpitations.
deposits in lining.
e. Dizziness; faintness.
2. Narrows lumen, can occlude vessel.
f. Epigastric distress; indigestion; belching.
B. Risk factors:
2. Objective data:
1. Increased serum cholesterol (low-density
a. Tachycardia.
lipids 160 mg/dL).
b. Pallor.
2. Hypertension.
c. Diaphoresis.
3. Cigarette smoking.
d. ECG changes during attack.
4. Diabetes mellitus.
D. Analysis/nursing diagnosis:
5. Family history of premature CHD.
1. Altered cardiopulmonary tissue perfusion related to
See following sections V. ANGINA PECTORIS
insufficient blood flow.
and VI. MYOCARDIAL INFARCTION for
2. Pain related to myocardial ischemia.
nursing implications.
3. Activity intolerance related to onset of pain.
V. ANGINA PECTORIS: transient paroxysmal episodes E. Nursing care plan/implementation:
of substernal or precordial pain. Types: stable (follows 1. Goal: provide relief from pain.
an event, same severity); unstable (at rest or minimal a. Rest until pain subsides.
exertion, recent onset, increasing severity); Prinzmetals b. Nitroglycerin or amyl nitrite, beta-adrenergic
variant (at rest, caused by coronary spasms). blockers, as ordered.
2164_Ch06_347-578 29/03/12 12:29 PM Page 417

Cardiovascular System 417

Table 6.19
Comparison of Physical Causes of Chest Pain
Myocardial Gastric Angina Dissecting Pulmonary
Characteristic Infarction Pericarditis Disorders Pectoris Aneurysm Embolism
Onset Gradual or sudden Sudden Gradual or Gradual or Abrupt, without Gradual or
sudden sudden prodromal sudden
symptoms
Precipitating Can occur at rest Breathing deeply, Inflammation of Usually after: Hypertension Immobility or
factors or after exercise rotating trunk, stomach or physical exer- prolonged
or emotional recumbency, esophagus; tion, emotional bedrest follow-
stress swallowing or hypersecretion stress, eating, ing: surgery,
yawning of gastric exposure to trauma, hip
juices; some cold, or defe- fracture, HF,
medications cation; unsta- malignancy, oral
ble angina contraceptives
occurs at rest
Location Substernal, Precordial; radiates Xiphoid to Substernal, Correlates with Pleural area,
anterior chest, to neck or left umbilicus anterior site of intimal retrosternal area
or midline; rarely shoulder and arm chest; poorly rupture; anterior
back; radiates to localized chest or back;
jaw or neck between shoul-
der blades
Quality Crushing, burning, Pleuritic, sharp Aching, burning, Squeezing, Sharp, tearing or Sharp, stabbing
stabbing, squeez- cramplike, feeling of ripping sensation
ing or vicelike gnawing heavy pres-
sure; burning
Intensity Asymptomatic to Mild to severe Mild to severe Mild to Severe and Aggravated by
severe; increases moderate unbearable; breathing
with time maximal from
onset
Duration 30 min to 12 hr; Continuous Periodic Usually 210 Continuous; Variable
may wax and min; average does not abate
wane 35 min once started
Relief Narcotics Sitting up, Physical and Nitro- Large, O2;
leaning emotional glycerin, repeated sitting up;
forward rest, food, rest doses of morphine
antacids, narcotics
H2-receptor
antagonists

ADULT
Associated Nausea, fatigue, Fever, dyspnea, Nausea, vomit- Belching, indi- Syncope, loss of Dyspnea,
symptoms heartburn; periph- nausea, anorexia, ing, dysphagia, gestion, dizzi- sensations or tachypnea,
eral pulses equal anxiety anorexia, weight ness pulses, oliguria; diaphoresis,
loss discrepancy hemoptysis,
between BP in cough, appre-
arms; decrease hension
in femoral or
carotid pulse

c. Identify precipitating factors: large meals, 3. Goal: health teaching.


heavy exercise, stimulants (coffee, smoking), a. Pain: alleviation, differentiation of angina
sex when fatigued, cold air. from myocardial infarction, precipitating
d. Vital signs: hypotension. factors (see Table 6.19).
e. Assist with ambulation; dizziness, flushing b. Medication: frequency, expected effects
occurs with nitroglycerin. (headache, flushing); carry fresh nitroglyc-
2. Goal: provide emotional support. erin; loses potency after 6 months
a. Encourage verbalization of feelings, fears. (stings under tongue when potent);
b. Reassurance; positive self-concept. may use nitroglycerin pasteinstruct
c. Acceptance of limitations. how to apply.
2164_Ch06_347-578 29/03/12 12:29 PM Page 418

418 chapter 6 Physiological Integrity

5. High levels of low-density lipoproteins, and high


serum triglyceride levels.
6. Chronic illness (diabetes, hypertension).
C. Assessment:
1. Subjective data:
a. Pain (see Table 6.19).
(1) Type: sudden, severe, crushing, heavy
tightness. May be absent in elderly or
those who have diabetes.
(2) Location: substernal; radiates to one or
both arms, jaw, neck. May be confused
with indigestion.
(3) Duration: greater than 30 minutes.
(4) Cause: unrelated to exercise; frequently
occurs when sleeping (rapid-eye-movement
Figure 6.5 Angina pectoris. (From Venes, D [ed]: Tabers
[REM] stage).
Cyclopedic Medical Dictionary, ed 21. FA Davis, Philadelphia, 2009.) (5) Relief: oxygen, narcotics; not relieved by rest
or nitroglycerin.
b. Nausea.
c. Diet: restricted calories if weight loss indicated; c. Shortness of breath.
restricted fat, cholesterol, gas-producing foods; d. Apprehension, fear of impending death.
small, frequent meals. e. History of cardiac disease (family); occupa-
d. Diagnostic tests if ordered (e.g., thallium stress tional stress.
test, cardiac catheterization; interventional 2. Objective data:
[stents]; see pp. 421422 and Chapter 11, a. Vital signs: shock; rapid (>100), thready pulse;
pp. 829830). fall in blood pressure; S3 gallop; tachypnea,
e. Exercise: regular, graded, to promote coronary shallow respirations; elevated temperature
circulation. within 24 hours (100 to 103F).
f. Prepare for coronary artery bypass graft b. Skin: ashen or clammy; diaphoretic.
(CABG) surgery, if necessary. c. Emotional: restless.
g. Behavior modification to assist with lifestyle d. Laboratory data: increasedWBC count
changes (e.g., stress reduction, stop smoking). (12,000 to 15,000/microL), troponin T and
F. Evaluation/outcome criteria: I levels, serum enzymes (creatine kinase-MB
1. Relief from pain. [CK-MB]; lactate dehydrogenase (LDH):
2. Fewer attacks. LDH1 > LDH2flipped LDH); changes
3. No myocardial infarction. ECG (elevated ST segment, inverted T wave,
4. Alters lifestyle; stress management; complies arrhythmia).
ADULT

with limitations. D. Analysis/nursing diagnosis:


5. No smoking. 1. Decreased cardiac output related to myocardial
damage.
VI. MYOCARDIAL INFARCTION (MI, HEART
2. Impaired gas exchange related to poor perfusion,
ATTACK): localized area of necrotic tissue in
shock.
myocardium from cessation of blood flow; leading
3. Pain related to myocardial ischemia.
cause of death in North America.
4. Activity intolerance related to pain or inadequate
A. Pathophysiology: oxygenation.
1. Coronary occlusion due to thrombosis, 5. Fear related to possibility of death.
embolism, or hemorrhage adjacent to E. Nursing care plan/implementation:
atherosclerotic plaque. 1. Goal: reduce pain, discomfort.
2. Insufficient blood flow from cardiac hypertrophy, a. Narcoticsmorphine; note response. Avoid IM.
hemorrhage, shock, or severe dehydration. b. Humidified oxygen 2 to 4 L/min; mouth
B. Risk factors: careoxygen is drying.
1. Age (35 to 70 years). c. Position: semi-Fowlers to improve ventilation.
2. Men more than women until menopause. 2. Goal: maintain adequate circulation, stabilize
3. Lifestyle: obesity, smoking, sedentary, heart rhythm.
amphetamine or cocaine use. a. Monitor vital signs and urine output; observe
4. Stress or type A personality. for cardiogenic shock.
2164_Ch06_347-578 29/03/12 12:29 PM Page 419

Cardiovascular System 419


b. Monitor ECG for arrhythmias. 5. Goal: facilitate fecal elimination.
c. Give medications as ordered: antiarrhythmics a. Medications: stool softeners to prevent Valsalva
lidocaine HCl, amiodarone, atropine, beta maneuver (straining); mouth breathing during
blockers, procainamide (Pronestyl), bretylium bowel movement; recognize complications of
(Bretylol); propranolol (Inderal); verapamil; Valsalva maneuverchest pain, cyanosis,
anticoagulantsheparin sodium, bishydroxy- diaphoresis, arrhythmias.
coumarin or dicoumarin; thrombolytic agents b. Bedside commode if possible.
streptokinase (tPA), APSAC/anistreplase 6. Goal: provide emotional support.
(Eminase), reteplase followed by IV heparin or a. Recognize fear of dying: denial, anger,
a glycoprotein IIB/IIIA inhibitor (Integrilin). withdrawal.
d. Diagnostic testsechocardiogram, prepare for b. Encourage expression of feelings, fears,
cardiac catheterization, possible interventional concerns.
cardiology (stents), possible CABG surgery. c. Discuss rehabilitation, lifestyle changes: pre-
e. Recognize heart failure: edema, cyanosis, vent cardiac invalid syndrome by promoting
dyspnea, cough, crackles. self-care activities, independence.
f. Check laboratory datanormal; troponin; 7. Goal: promote sexual functioning.
serum enzymes (CK 20 to 220 IU/L depend- a. Encourage discussion of concerns regarding
ing on gender; CK-MB 0 to 12 IU/L; LDH activity, inadequacy, limitations, expectations,
<115 IU/L; LDH1 < LDH2); blood gases use of drugs for erectile dysfunctioninclude
(pH 7.35 to 7.45; PCO2 35 to 45 mEq/L; partner (usually resume activity 5 to 8 weeks
PO2 80 to 100 mm Hg; HCO3 22 to 26); after uncomplicated MI or when client can
+
electrolytes (K 3.5 to 5.0 mEq/L; Mg++ 1.3 climb two flights of stairs).
to 2.1 mg/dL); clotting time (aPTT 25 to b. Identify need for referral for sexual counseling.
41 seconds; prothrombin time [PT] 11 to 8. Goal: health teaching.
15 seconds). a. Diagnosis and treatment regimen.
g. CVPzero level at right atrium; fluctuates b. Caution about when to avoid sexual activity:
with respiration; normal range 5 to 15 cm after heavy meal, alcohol ingestion; when
H2O; note trend; increases with heart failure. fatigued, tense, under stress; with unfamiliar
h. ROM of lower extremities; TED partners; in extreme temperatures.
hose/antiembolic stockings. c. Information about sexual activity: less
3. Goal: decrease oxygen demand/promote oxygenation, fatiguing positions (side to side; noncardiac
reduce cardiac workload. partner on top); vasodilators, if ordered,
a. O2 as ordered. before intercourse; select comfortable,
b. Activity: bedrest (24 to 48 hours) with familiar environment.
bedside commode; planned rest periods; d. Referral to available community resources for
control visitors. information, support groups (e.g., American
c. Position: semi-Fowlers to facilitate lung Heart Association, Stop Smoking Clinics).

ADULT
expansion and decrease venous return. e. Medications: administration, importance,
d. Anticipate needs of client: call light, water. untoward effects, pulse taking.
e. Assist with feeding, turning. f. Control risk factors: rest, diet, exercise, no
f. Environment: quiet, comfortable. smoking, weight control, stress-reduction
g. Reassurance; stay with client who is anxious. techniques.
h. Give medications as ordered: cardiotonics, g. Need for follow-up care for regulation of
calcium channel blockers, vasodilators, medications, evaluating risk factors.
vasopressors. h. Prepare for angioplasty or coronary bypass
4. Goal: maintain fluid, electrolyte, nutritional if planned.
status. F. Evaluation/outcome criteria:
a. IV (keep vein open); CVP; vital signs. 1. No complications: stable vital signs; relief of
b. Urine output30 mL/hr. pain.
c. Laboratory data within normal limits (Na+ 2. Adheres to prescribed medication regimen,
+
135 to 145 mEq/L; K 3.5 to 5.0 mEq/L; demonstrates knowledge about medications.
Mg++ 1.3 to 2.1 mg/dL). 3. Activity tolerance is increased, participates in
d. Monitor ECGhyperkalemia: peaked T wave; program of progressive activity.
hypokalemia: depressed T wave. 4. Reduction or modification of risk factors.
e. Diet: progressive low calorie, low sodium, low Plans to alter lifestyle (e.g., loses weight,
cholesterol, low fat, without caffeine. quits smoking).
2164_Ch06_347-578 29/03/12 12:29 PM Page 420

420 chapter 6 Physiological Integrity

VII. CARDIAC VALVULAR DEFECTS: alteration 6. Aortic insufficiency:


in the structure of a valve; impede flow of blood or a. Incomplete closure of valve between left
permit regurgitation. ventricle and aorta (regurgitation).
A. Pathophysiology: b. Left ventricular failure leading to right
1. Stenosisnarrowing of valvular opening due to ventricular heart failure.
adherence, thickening, and rigidity of valve cusp B. Risk factors:
from fibrosis, scarring, and calcification. 1. Congenital abnormality.
2. Insufficiency (incompetence)incomplete closure 2. History of rheumatic fever.
of valve due to contraction of chordae tendineae, 3. Atherosclerosis.
papillary muscles; or to calcification, scarring of C. Assessment: see Table 6.20.
leaflets. Results in regurgitation. D. Analysis/nursing diagnosis:
3. Mitral stenosis: 1. Decreased cardiac output related to inadequate
a. Most common residual cardiac lesion of ventricular filling.
rheumatic fever. 2. Fluid volume excess related to compensatory
b. Affects women younger than 45 years more response to decreased cardiac output.
often than men. 3. Impaired gas exchange related to pulmonary
c. Narrowing of mitral valve. congestion.
d. Interferes with filling of left ventricle. 4. Activity intolerance related to impaired cardiac
e. Produces pulmonary hypertension, right function.
ventricular failure. 5. Fatigue related to poor oxygenation.
4. Mitral insufficiency (incompetence): E. Nursing care plan/implementation:
a. Leaking/regurgitation of blood back into 1. Goal: reduce cardiac workload.
left atrium. 2. Goal: promote physical comfort and psychological
b. Results from rheumatic fever, bacterial support.
endocarditis; less common. 3. Goal: prevent complications.
c. Affects men more often. 4. Goal: prepare for surgery (commissurotomy,
d. Produces pulmonary congestion, right valvuloplasty [valvotomy], or valvular replace-
ventricular failure. ment, depending on defect and severity of
5. Aortic stenosis: condition).
a. Fusion of valve flaps between left ventricle 5. See section X. CARDIAC SURGERY,
and aorta. pp. 422425, for specific nursing actions.
b. Congenital or acquired from atherosclerosis F. Evaluation/outcome criteria:
or from rheumatic fever and bacterial endo- 1. Relief of symptoms.
carditis; seen in men more often; pulmonary 2. Increase in activity level.
circulation congested, cardiac output 3. No complications following surgery.
decreased.
ADULT

Table 6.20
Comparison of Symptomatology for Valvular Defects
Assessment Mitral Stenosis Mitral Insufficiency Aortic Stenosis Aortic Insufficiency
Subjective Data
Fatigue
Shortness of breath
Orthopnea
Paroxysmal nocturnal dyspnea
Cough
Dyspnea on exertion
Palpitations
Syncope on exertion
Angina
Weight loss
2164_Ch06_347-578 29/03/12 12:29 PM Page 421

Cardiovascular System 421

Table 6.20
Comparison of Symptomatology for Valvular Defectscontd
Assessment Mitral Stenosis Mitral Insufficiency Aortic Stenosis Aortic Insufficiency
Objective Data
Vital signs
Blood pressure:
Low or normal
Normal or elevated
Pulse:
Weak, irregular
Rapid, waterhammer
Respirations:
Increased, shallow
Cyanosis
Jugular vein distention
Enlarged liver
Dependent edema
Murmur

VIII. CARDIAC CATHETERIZATION: a diagnostic 2. Analysis/nursing diagnosis:


procedure to evaluate cardiac status. Introduces a a. Anxiety related to fear of unknown.
catheter into the heart, blood vessels; analyzes blood b. Knowledge deficit (learning need) related to
samples for oxygen content, ejection fraction, cardiac limited exposure to information or sudden
output, pulmonary artery blood flow; done before need for procedure.
heart surgery; frequently combined with angiography 3. Nursing care plan/implementation:
to visualize coronary arteries; also provides access for a. Goal: provide for safety, comfort.
specialized cardiac techniques (e.g., internal pacing (1) Signed informed consent.
and percutaneous transluminal coronary angioplasty (2) NPO (except for medications 6 to 8 hours
[PTCA]). before).
A. Approaches (3) Have client urinate before going to lab.
1. Right-heart catheterizationvenous approach (4) Give sedatives, as ordered, 30 minutes
(antecubital or femoral) right atrium right before procedure (e.g., midazolam HCl

ADULT
ventricle pulmonary artery. [Versed] IV, diazepam [Valium] PO).
2. Left-heart catheterizationretrograde approach: (5) Possible shaving of insertion site.
right brachial artery or percutaneous puncture of b. Goal: health teaching.
femoral artery ascending aorta left ventricle. (1) Procedure: length (1 to 3 hours).
a. Transseptal: femoral vein right atrium (2) Expectations (strapped to table for safety,
septum left atrium left ventricle. must lie still, awake but mildly sedated).
b. Angiography/arteriography: done during left- (3) Sensations (hot, flushed feeling in head
heart catheterization. with dye injection; thudding in chest
B. Precatheterization from premature beats during catheter
1. Assessment: manipulation; desire to cough, particularly
a. Subjective data: with right-heart angiography and contrast-
(1) Allergies: iodine, seafood. medium injection).
(2) Anxiety. (4) Alert physician to unusual sensations
(3) Comfort. (coolness, numbness, paresthesia).
b. Objective data: C. Postcatheterization
(1) Vital signs: baseline data. 1. Assessment (potential complications):
(2) Distal pulses: mark for reference after a. Subjective data:
catheterization. (1) Puncture site: increasing pain, tenderness.
2164_Ch06_347-578 29/03/12 12:29 PM Page 422

422 chapter 6 Physiological Integrity

(2) Palpitations, chest pain. IX. PERCUTANEOUS TRANSLUMINAL


(3)Affected extremity: tingling, numbness, CORONARY ANGIOPLASTY (PTCA): A balloon-
pain from hematoma or nerve damage. tipped catheter is threaded to site of coronary
b. Objective data: occlusion and inflated repeatedly until blood flow
(1) Vital signs: shock, respiratory distress increases distal to the obstruction. It is a nonsurgical
(related to pulmonary emboli, allergic alternative to bypass surgery for coronary artery
reaction). occlusion (Fig. 6.6); recommended for clients with
(2) Puncture site: bleeding (hematoma). poorly controlled angina, mild or no symptoms,
(3) ECG: arrhythmias, signs of MI. multiple- or single-vessel disease with a noncalcified,
(4) Affected extremity: color, temperature, discrete, and proximal lesion that can be reached
peripheral pulses. by the catheter; costs less and requires shorter
2. Analysis/nursing diagnosis: hospitalization and rehabilitation period; successful
a. Decreased cardiac output related to arrhyth- in 90% of clients; approximately 30% restenose by
mias or MI. 3 months (see VIII. CARDIAC CATHETERIZA-
b. Altered tissue perfusion related to bleeding TION, p. 421, for nursing process). Rotational
following procedure. atherectomy may also be done; a high-speed drill
c. Pain related to puncture site tenderness. pulverizes plaque into small particles. An intravas-
3. Nursing care plan/implementation: cular stent, steel mesh or coiled spring, may be
a. Goal: prevent complications. placed in the coronary artery; the stent acts as a
(1) Bedrest: depends on size of catheter and mechanical scaffold to reopen the blocked artery.
closure procedurePerclose dissolvable The client receives low-molecular-weight heparin
suture, 30 minutes; Angioseal (collagen and/or platelet therapy following the procedure
plug), 2 hours; compression pump or and after discharge.
15-minute manual compression followed X. CARDIAC SURGERY: done to alter the structure
by sandbag, 4 to 5 hours; 12 to 24 hours of the heart or vessels when congenital or acquired
with sheath or antiplatelet drip (abciximab disorders interfere with cardiac functioning: septal
[ReoPro]). defects; transposition of great vessels; tetralogy of
(2) Vital signs: record q15 min for 1 hour, Fallot; pulmonary/aortic stenosis; coronary artery
q30 min for 3 hours or until stable; check bypass; valve replacement.
BP on opposite extremity.
Cardiopulmonary bypass (open-heart surgery):
(3) Puncture site: observe for bleeding,
blood from cardiac chambers and great vessels is
swelling, or tenderness; check pulse distal
diverted into a pump oxygenator; allows full visualiza-
to insertion site to determine patency of
tion of heart during surgery; maintains perfusion and
artery; report complaints of coolness,
body functioning.
numbness, or paresthesia in extremity.
(4) ECG: monitor, document rhythm. A. Preoperative
(5) Give medications as ordered: sedatives; 1. Assessment: see specific conditions for
preoperative signs and symptoms (i.e., valvular
ADULT

mild narcotics; antiarrhythmics; antiplatelet


(Plavix, aspirin) or low-molecular-weight defects, angina, MI); (see also THE PERIOP-
heparin (enoxaparin [Lovenox]) with stent ERATIVE EXPERIENCE, pp. 365367).
insertion. Establish complete baseline: daily weight; vital
b. Goal: provide emotional support. signsintegrity of all pulses, BP both arms;
(1) Explanations: brief, accurate; client CVP or pulmonary artery pressures; neurologi-
anxious to learn results of test. cal status; emotional status; nutritional and
(2) Counseling: refer as indicated. elimination patterns; laboratory values (urine,
c. Goal: health teaching. electrolytes, enzymes, coagulation studies,
(1) Late complications: infection. cholesterol); pulmonary function studies;
(2) Prepare for surgery if indicated. echocardiogram, chest x-ray.
(3) Follow-up medical care. 2. Analysis/nursing diagnosis (see also VI.
(4) Limitations following PTCA procedure MYOCARDIAL INFARCTION, pp. 418, 419):
(see section IX below): no lifting greater a. Decreased cardiac output related to myocardial
than 10 lb and no vigorous exertion for damage.
1 to 2 weeks; return to normal work and b. Activity intolerance related to poor cardiac
sexual activity in 2 to 3 days. function.
4. Evaluation/outcome criteria: no complications c. Knowledge deficit (learning need) related to
(e.g., cardiac arrest, hematoma at insertion site). insufficient time for teaching.
2164_Ch06_347-578 29/03/12 12:29 PM Page 423

Cardiovascular System 423


d. Anxiety related to fear of unknown. (3)Encourage verbalization/questions:
e. Fear related to possible death. fear, depression, despair frequently
f. Risk for spiritual distress related to possible occur.
death. (4) Involve family during explanations.
3. Nursing care plan/implementation: b. Goal: health teaching.
a. Goal: provide emotional and spiritual support. (1) Diagnostic procedures, treatments,
(1) Arrange for religious consultation if specifics for surgery (i.e., internal mam-
desired. marian artery or leg incision with use of
(2) Provide opportunity for family visit morn- saphenous vein in coronary artery bypass
ing of surgery. graft surgery) (Fig. 6.7).

1. UNINFLATED BALLOON 2. TOTAL INFLATION OF


CATHETER IN ARTERY BALLOON CATHETER

DILATED VESSEL

PLAQUE

3. CATHETER IS DEFLATED 4. PROCEDURE COMPLETED.


AND REMOVED. DIAMETER OF LUMEN
HAS BEEN INCREASED.

Figure 6.6 Arterial balloon


angioplasty. (From Venes, D [ed]:
Tabers Cyclopedic Medical Dictionary,
ed 21. FA Davis, Philadelphia, 2009.)

ADULT
LEFT SUBCLAVIAN ARTERY
OFF AORTIC ARCH
VEIN IS REMOVED
INTERNAL MAMMARY FROM LEG AND
ARTERY TRANSPORTED
TO HEART

LEFT ANTERIOR
CORONARY EXPOSED
ARTERY SAPHENOUS
VEIN

BYPASS GRAFTS

MARGINAL BRANCH
OF CIRCUMFLEX ARTERY

LEFT ANTERIOR Figure 6.7 Bypass. Anterior view of heart with


DESCENDING
RIGHT ANTERIOR CORONARY
sites of bypass graft. (From Venes, D [ed]: Tabers
CORONARY ARTERY Cyclopedic Medical Dictionary, ed 21. FA Davis, Philadelphia,
ARTERY ANTERIOR VIEW OF HEART WITH SITES OF BYPASS GRAFTS 2009.)
2164_Ch06_347-578 29/03/12 12:29 PM Page 424

424 chapter 6 Physiological Integrity

(2) Postoperative regimen: turn, cough, deep d. Altered tissue perfusion related to postoperative
breathe, ROM, equipment used, medica- bleeding or thromboemboli.
tion for pain. e. Fluid volume deficit related to blood loss.
(3) Tour ICU; meet personnel. f. Risk for infection related to wound
(4) Alternative method of communication contamination.
while intubated. g. Altered thought processes related to anesthesia
4. Evaluation/outcome criteria: or stress.
a. Displays moderate anxiety level. h. Altered role performance related to uncertainty
b. Verbalizes/demonstrates postoperative about future.
expectations. 3. Nursing care plan/implementation:
c. Quits smoking before surgery. a. Goal: provide constant monitoring to prevent
B. Postoperative complications.
1. Assessment: (1) Respiratory:
a. Subjective data: (a) Observe for respiratory distress: restless-
(1) Pain. ness, nasal flaring, Cheyne-Stokes respi-
(2) Fatiguesleep deprivation. ration, dusky/cyanotic skin; assisted or
b. Objective data: controlled ventilation via endotracheal
(1) Neurological: level of consciousness; tube common 6 to 24 hours; supple-
pupillary reactions; movement of limbs mental O2 after extubation.
(purposeful, spontaneous). (b) Suctioning; cough, deep breathe.
(2) Respiratory: rate changes (increases occur (c) Elevate head of bed at least 30 degrees.
with obstruction, pain; decreases occur (d) Position chest tube to facilitate
with CO2 retention); depth (shallow with drainage; suction maintains patency
pain, atelectasis); symmetry; skin color; do not milk chest tube. (See also
patency/drainage from chest tubes, sputum Chest tube section in Table 11.5.)
(amount, color); endotracheal tube place- (2) Cardiovascular:
ment (bilateral breath sounds); arterial (a) Vital signs: BP greater than 80 to
blood gases, O2 saturation. 90 mm Hg systolic; CVP: range 5 to
(3) Cardiovascular: 15 cm H2O unless otherwise ordered;
(a) BPhypotension may indicate: heart pulmonary artery line (PA catheter):
failure, tamponade, hemorrhage, mean pressure 4 to 12 mm Hg;
arrhythmias, or thrombosis; I&O: report less than 30 mL/hr
hypertension may indicate: anxiety, of urine from indwelling urinary
hypervolemia. catheter.
(b) Pulse: radial, apical, pedal; rate (>100 (b) ECG; premature ventricular contrac-
may indicate: shock, fever, hypoxia, tions (PVCs) occur most frequently
arrhythmias); rhythm, quality. Check following aortic valve replacement and
ADULT

pacing wires. bypass surgery.


(c) CVP and PA catheter (elevated in (c) Peripheral pulses if leg veins used for
cardiac failure); temperature (normal grafting.
postoperative: 98.6 to 101.6F oral). (d) Activity: turn q2h; ROM; progressive,
(4) GI: nausea, vomiting, distention. early ambulation.
(5) Renal: urineminimum output (3) Inspect dressing for bleeding.
(30 mL/hr); color (4) Medications according to therapeutic
Specific gravity (<1.010 occurs with over- directivescardiotonics (digoxin);
hydration, renal tubular damage; >1.020 coronary vasodilators (nitrates); antibiotics
present with dehydration, oliguria, blood in (penicillin); analgesics; anticoagulants
urine). (with valve replacements); antiarrhythmics
2. Analysis/nursing diagnosis: (amiodarone, procainamide HCl
a. Decreased cardiac output related to decreased [Pronestyl]); dobutamine [Dobutrex].
myocardial contractility or postoperative b. Goal: promote comfort, pain relief.
hypothermia. (1) Medicate: morphine sulfatesevere pain
b. Pain (acute) related to incision. lasts 2 to 3 days.
c. Ineffective airway clearance related to effects (2) Splint incision when moving or
of general anesthesia. coughing.
2164_Ch06_347-578 29/03/12 12:29 PM Page 425

Cardiovascular System 425


(3) Mouth care: frequent, especially if intubat- incision without bypass. Small incision and minimal
ed; keep lips moist. recovery time.
(4) Position: use pillows to prevent tension on XII. HEART FAILURE (HF): inability of the heart to
chest tubes, incision. meet the peripheral circulatory demands of the body;
c. Goal: maintain fluid, electrolyte, nutritional cardiac decompensation; combined right and left
balance. ventricular heart failure.
(1) I&O; urine specific gravity.
A. Pathophysiology: increased cardiac workload or
(2) Measure chest drainageshould not
decreased effective myocardial contractility
exceed 200 mL/hr for first 4 to 6 hours.
decreased cardiac output (forward effects). Left
(3) Give fluids as ordered; maintain IV
ventricular failure pulmonary congestion; right
patency, central line care.
atrial and right ventricular failure systemic con-
(4) Diet: clear fluids solid food if no nausea,
gestion peripheral edema (backward effects).
GI distention; sodium intake restricted, low
Compensatory mechanisms in HF include tachy-
fat; give H2 blocker as ordered.
cardia, ventricular dilation, and hypertrophy of the
d. Goal: promote emotional adjustment.
myocardium; develops in 50% to 60% of clients
(1) Anticipate behavior disturbances (depres-
with heart disease.
sion, disorientation often occur 3 days
B. Risk factors:
postoperatively) related to medications,
1. Decreased myocardial contractility:
fear, sleep deprivation.
a. Myocarditis.
(2) Calm, oriented, supportive environment,
b. MI.
as personalized as possible.
c. Tachyarrhythmias.
(3) Encourage verbalization of feelings (family
d. Bacterial endocarditis.
and client).
e. Acute rheumatic fever.
(4) Encourage independence to avoid cardiac
2. Increased cardiac workload:
cripple role.
a. Elevated temperature.
e. Goal: promote early mobilization.
b. Physical/emotional stress.
(1) Out of bed within 24 hours postoperative
c. Anemia.
to prevent deep vein thrombosis (DVT).
d. Hyperthyroidism (thyrotoxicosis).
(2) In chair three times daily by postoperative
e. Valvular defects.
day 2.
f. Uncontrolled hypertension.
f. Goal: health teaching.
C. Assessment:
(1) Alterations in lifestyle; activity, diet, work;
1. Subjective data:
resumption of sexual activity usually when
a. Shortness of breath.
client can climb two flights of stairs.
(1) Orthopnea (sleeps on two or more pillows).
(2) Refer to available community resources
(2) Paroxysmal nocturnal dyspnea (sudden
for cardiac rehabilitation (e.g., American
breathlessness during sleep).
Heart Association, Mended Hearts).
(3) Dyspnea on exertion (climbing stairs).

ADULT
(3) Drug regimen: purpose, side effects.
b. Apprehension; anxiety; irritability.
(4) Potential complications: dyspnea, pain,
c. Fatigue; weakness.
palpitations common postoperatively.
d. Reported weight gain; feeling of puffiness.
4. Evaluation/outcome criteria:
2. Objective data (Table 6.21):
a. No complications; incision heals; no
a. Vital signs:
dysrhythmias; pacing wires discontinued.
(1) BP: decreasing systolic; narrowing pulse
b. Activity level increasesno signs of overexer-
pressure.
tion (e.g., fatigue, dyspnea, pain).
(2)Pulse: pulsus alternans (alternating
c. Relief of symptoms.
strong-weak-strong cardiac contraction),
d. Returns for follow-up medical care.
increased.
e. Takes prescribed medications; knows purposes
(3) Respirations: crackles, Cheyne-Stokes.
and side effects.
b. Edema: dependent, pitting (1+ to 4+ mm).
XI. MINIMALLY INVASIVE DIRECT CORONARY c. Liver: enlarged, tender.
ARTERY BYPASS (MIDCAB): a variation of CABG d. Neck veins: distended.
for clients for whom sternotomy and cardiopulmonary e. Chest x-ray:
bypass is contraindicated or unnecessary. The left (1) Cardiac enlargement.
internal mammary artery is anastomosed to the left (2) Dilated pulmonary vessels.
anterior descending coronary artery through a thoracic (3) Diffuse interstitial lung edema.
2164_Ch06_347-578 29/03/12 12:29 PM Page 426

426 chapter 6 Physiological Integrity

Table 6.21 (5) Tranquilizersphenobarbital, diazepam


(Valium), chlordiazepoxide HCl (Librium).
Left Ventricular Compared with Right (6) Vasodilatorshydralazine, isosorbide.
Ventricular Heart Failure 3. Goal: provide for special safety needs.
Left Ventricular Failure Right Ventricular Failure a. Skin care:
Pulmonary crackles Jugular venous distention
(1) Inspect, massage, lubricate bony
Tachypnea Peripheral edema prominences.
S3 gallop Perioral and peripheral cyanosis (2) Use foot cradle, heel protectors;
Cardiac murmurs Congestive hepatomegaly sheepskin.
Paradoxical splitting of S2 Ascites b. Side rails up if hypoxic (disoriented).
Hepatojugular reflux
c. Vital signs: monitor for signs of fatigue,
pulmonary emboli.
d. ROM: active, passive; elastic stockings, DVT
D. Analysis/nursing diagnosis: prophylaxis.
1. Decreased cardiac output related to decreased 4. Goal: maintain fluid and electrolyte balance,
myocardial contractility. nutritional status.
2. Activity intolerance related to generalized a. Urine output: 30 mL/hr minimum;
weakness and inadequate oxygenation. estimate insensible loss in client who is
3. Fatigue related to edema and poor oxygenation. diaphoretic. Monitor: BUN, serum creati-
4. Altered tissue perfusion related to peripheral nine, and electrolytes, B-type natriuretic
edema and inadequate blood flow. peptide (BNP).
5. Fluid volume excess related to compensatory b. Daily weight; same time, clothes, scale.
mechanisms. c. IV: IV infusion pump to avoid circulatory
6. Impaired gas exchange related to pulmonary overloading; strict I&O.
congestion. d. Diet:
7. Anxiety related to shortness of breath. (1) Low sodium as ordered.
8. Sleep pattern disturbance related to paroxysmal (2) Small, frequent feedings.
nocturnal dyspnea. (3) Discuss food preferences with client.
E. Nursing care plan/implementation: 5. Goal: health teaching.
1. Goal: provide physical rest/reduce emotional a. Diet restrictions; meal preparation.
stimuli. b. Activity restrictions, if any; planned rest
a. Position: sitting or semi-Fowlers until tachy- periods.
cardia, dyspnea, edema resolved; change c. Medications: schedule (e.g., diuretic in early
position frequently; pillows for support. morning to limit interruption of sleep),
b. Rest: planned periods; limit visitors, activity, purpose, dosage, side effects (importance
noise. Chair and commode privileges. of daily pulse taking, daily weights, intake
c. Support: stay with client who is anxious; of potassium-containing foods).
ADULT

have family member who is supportive d. Refer to available community resources for
present; administer sedatives/tranquilizers as dietary assistance, weight reduction, exercise
ordered. program.
d. Warm fluids if appropriate. F. Evaluation/outcome criteria:
2. Goal: provide for relief of respiratory distress; 1. Increase in activity level tolerancefatigue
reduce cardiac workload. decreased.
a. Oxygen: low flow rate; encourage deep 2. No complicationspulmonary edema,
breathing (5 to 10 minutes q2h); auscultate respiratory distress.
breath sounds for congestion, pulmonary 3. Reduction in dependent edema.
edema. XIII. PULMONARY EDEMA: sudden transudation
b. Position: elevating head of bed 20 to of fluid from pulmonary capillaries into alveoli.
25 cm (8 to 10 inches) alleviates Life-threatening condition.
pulmonary congestion.
A. Pathophysiology: increased pulmonary capillary
c. Medications as ordered:
permeability; increased hydrostatic pressure (pul-
(1) Digitalis preparations.
monary hypertension); decreased blood colloidal
(2) ACE inhibitorscaptopril, enalapril.
osmotic pressure; fluid accumulation in alveoli
(3) Inotropic agentdobutamine, dopamine.
decreased compliance decreased diffusion of
(4) Diureticsthiazides, furosemide,
gas hypoxia, hypercapnia.
metolazone.
2164_Ch06_347-578 29/03/12 12:29 PM Page 427

Cardiovascular System 427


B. Risk factors: 100% non-rebreather mask, or intubation,
1. Left ventricular failure. depending on O2 need. Possibly PEEP. Small-
2. Pulmonary embolism. volume nebulizer treatment with ipratropium
3. Drug overdose. (Atrovent) or albuterol (Proventil).
4. Smoke inhalation. b. Give aminophylline, as ordered, to lower
5. CNS damage. venous pressure and increase cardiac output.
6. Fluid overload. c. IV: D5W.
7. Valvular disorders. d. Position: high Fowlers, extremities in dependent
C. Assessment: position, to reduce venous return and facilitate
1. Subjective data: breathing.
a. Anxiety. e. Medications as ordered: digitalis; diuretics
b. Restlessness at onset, progressing to agitation. furosemide (Lasix); inotropic agents
c. Stark fear. dobutamine (Dobutrex), dopamine;
d. Intense dyspnea, orthopnea, fatigue. nitroglycerin, nitroprusside.
2. Objective data: f. Vital signs; auscultate breath sounds.
a. Vital signs: g. Diet: low sodium; fluid restriction as ordered.
(1) Pulse: tachycardia; gallop rhythm. 3. Goal: health teaching (include family or
(2) Respirations: tachypnea, moist, bubbling, significant other).
wheezing, labored, O2 saturation. a. Medications.
(3) Temperature: normal to subnormal. (1) Side effects.
b. Skin: pale, cool, diaphoretic, cyanotic. (2) Potassium supplements if indicated.
c. Auscultation: crackles, wheezes. (3) Pulse taking.
d. Cough: productive of large quantities of pink, b. Exercise; rest.
frothy sputum. c. Diet: low sodium.
e. Right ventricular heart failure: distended d. Signs of complications: edema; weight gain
(bulging) neck veins, peripheral edema, of 2 to 3 lb (0.9 to 1.4 kg) in a few days;
hepatomegaly, ascites. dyspnea.
f. Mental status: restless, confused, stuporous. F. Evaluation/outcome criteria:
g. Arterial blood gases: hypoxia; pulse oximetry: 1. No complications; vital signs stable; clear breath
decreased O2 saturation. sounds.
h. Chest x-ray: haziness of lung fields, 2. No weight gain; weight loss if indicated.
cardiomegaly. Echocardiogram. 3. Alert, oriented, calm.
D. Analysis/nursing diagnosis: XIV. SHOCK: a critically severe deficiency in nutrients,
1. Decreased cardiac output related to decreased oxygen, and electrolytes delivered to body tissues, plus
myocardial contractility. deficiency in removal of cellular wastes; results from:
2. Impaired gas exchange related to pulmonary cardiac failure, insufficient blood volume, or increased
congestion. vascular bed size.

ADULT
3. Altered tissue perfusion related to inadequate
A. Types, pathophysiology, and risk factors:
blood flow.
1. Hypovolemic (hemorrhagic, hematogenic)
4. Anxiety, severe, related to difficulty breathing.
markedly decreased volume of blood (hemor-
5. Fear related to life-threatening situation.
rhage or plasma loss from intestinal obstruction,
E. Nursing care plan/implementation:
burns, physical trauma, or dehydration)
1. Goal: promote physical, psychological relaxation
decreased venous return, cardiac output
measures to relieve anxiety.
decreased tissue perfusion.
a. Slow respirations: morphine sulfate 3 to 10 mg
2. Cardiogenicfailure of cardiac muscle pump
IV/SQ/IM, as ordered, to reduce respiratory
(myocardial infarction) generally decreased
rate, sedate, and produce vasodilation.
cardiac output pulmonary congestion,
b. Remain with client.
hypoxia inadequate circulation; high
c. Encourage slow, deep breathing; assist with
mortality rate.
coughing.
3. Distributive:
d. Work calmly, confidently, unhurriedly.
a. Neurogenicmassive vasodilation from
e. Frequent rest periods.
reduced vasomotor, vasoconstrictor tone (e.g.,
2. Goal: improve cardiac function, reduce venous
spinal shock, head injuries, anesthesia, pain);
return, relieve hypoxia.
interruption of sympathetic nervous system;
a. O2: slow respiratory rate, provide uniform
blood volume is normal but inadequate for
ventilation via nasal cannula, ventimask,
2164_Ch06_347-578 29/03/12 12:29 PM Page 428

428 chapter 6 Physiological Integrity

vessels decreased venous return tissue d. CVP:


hypoxia. (1) Below 5 cm H2O with hypovolemic shock,
b. Vasogenic (anaphylactic, septic, systemic also anaphylactic or septic shock.
inflammatory response syndrome [SIRS], (2) Above 15 cm H2O with cardiogenic shock.
endotoxic)severe reaction to foreign protein e. Urine output: decreased (<30 mL/hr).
(insect bites, drugs, toxic substances, aerobic, f. Capillary refill: slowed; normally nailbed
gram-negative organisms) histamine release pinks up within 2 seconds after blanching
vasodilation, venous stasis diminished (nailbed pressure).
venous return. C. Analysis/nursing diagnosis:
B. Assessment: varies, depending on degree of shock 1. Altered tissue perfusion related to vasodilation or
(Table 6.22). decreased myocardial contractility.
1. Subjective data: 2. Impaired gas exchange related to ventilation-
a. Anxiety; restlessness. perfusion imbalance.
b. Dizziness; fainting. 3. Decreased cardiac output related to loss of
c. Thirst. circulating blood volume or diminished cardiac
d. Nausea. contractility; peripheral vasodilation.
2. Objective data: 4. Altered urinary elimination related to decreased
a. Vital signs: renal perfusion.
(1) BPhypotension (postural changes in 5. Fluid volume deficit related to blood loss.
early shock; systolic less than 70 mm Hg 6. Anxiety related to severity of condition.
in late shock). D. Nursing care plan/implementation: Goal:
(2) Pulsetachycardia, thready; irregular promote venous return, circulatory perfusion.
(cardiogenic shock); could be slow if 1. Position: foot of bed elevated 20 degrees
conduction system of heart damaged. (12 to 16 inches), knees straight, trunk
(3) Respirationsincreased depth, rate; horizontal, head slightly elevated; avoid
wheezing (anaphylactic shock), O2 satura- Trendelenburg position.
tion 90%. 2. Ventilation: monitor respiratory effort, loosen
(4) Temperaturedecreased (elevated in septic restrictive clothing; O2 as ordered.
shock). 3. Fluids:
b. Skin: a. Maintain IV infusionswith sepsis, may
(1) Pale (or mottled), cool, clammy (warm receive 2 to 6 L to keep CVP greater than
to touch in septic shock). 12 mm Hg to prevent end organ hypoxia
(2) Urticaria (anaphylactic shock). and organ failure. Mean arterial pressure
c. Level of consciousness: alert, oriented greater than 60 mm Hg.
acute alteration in mental status b. Give blood, plasma expanders as ordered
unresponsive. (exceptionstop blood immediately in
anaphylactic shock).
ADULT

4. Vital signs:
a. CVP (decreased with hypovolemia) arterial
Table 6.22 line, PA catheter (increased pulmonary artery
Signs of Hypovolemic Shock wedge pressure indicating cardiac failure).
Check central venous O2 (scvO2).
Blood Loss (% of Total) Assessment Data b. Urine output (insert catheter for hourly
8001500 mL (15%30%) Restlessness output).
Pulse >100 c. Monitor ECG (increased rate,
Systolic pressure unchanged dysrhythmias).
Diastolic pressure
Urine output 5. Medications (depending on type of shock) as
ordered:
2000 mL (30%40%) Mental status
Pulse >120
a. Adrenergicsdobutamine, norepinephrine
Respirations > 30 (Levophed), isoproterenol (Isuprel),
Systolic pressure dopamine (Intropin) (cardiogenic, neuro-
>2500 mL (>40%) Cold, clammy skin genic, septic shock).
Pulse >120 b. Antiarrhythmics (cardiogenic shock).
Respirations >30 c. Cardiac glycosides (cardiogenic shock).
Narrowed pulse pressure d. Adrenocorticoids (anaphylactic shock).
(= systolic minus diastolic) e. Antibiotics (septic shock).
2164_Ch06_347-578 29/03/12 12:29 PM Page 429

Cardiovascular System 429


f. Vasodilatorsnitroprusside (cardiogenic E. Nursing care plan/implementation: Goal: prevent
shock). and detect further bleeding.
g. Antihistaminesepinephrine, vasopressin, 1. Carry out nursing measures designed to alleviate
Benadryl IV, methlyprednisone. underlying problem (e.g., shock, birth of fetus,
6. Mechanical support: military (or medical) anti- surgery/irradiation for cancer, antibiotics for
shock trousers (MAST) or pneumatic antishock infection).
garment (PASG); used to promote internal auto- 2. Medications: heparin sodium IV, 1,000 units/hr,
transfusion of blood from legs and abdomen to if ordered, to reverse abnormal clotting (contro-
central circulation; at lower pressures may control versial). Possible human recombinant activated
bleeding and promote hemostasis; do not remove protein C.
(deflate) suddenly to examine underlying areas or 3. IVs: blood to lessen shock; platelets, cryopre-
BP will drop precipitously; compartment syndrome cipitate, fresh plasma to restore clotting
may result with prolonged use and high pressure; factors, fibrinogen.
controversial. 4. Observe: vital signs, CVP (normal 5 to 15 mm
E. Evaluation/outcome criteria: Hg), PA pressure (normal 20 to 30 systolic and
1. Vital signs stable, within normal limits. 8 to 12 diastolic), and I&O for signs of shock
2. Alert, oriented. or fluid overload from frequent infusions; speci-
3. Urine output greater than 30 mL/hr. mens for occult blood (urine, stool).
XV. DISSEMINATED INTRAVASCULAR 5. Precautions: avoid IM injections if possible;
COAGULATION (DIC): diffuse or widespread pressure 5 minutes to venipuncture sites; no
coagulation initially within arterioles and capillaries rectal temperatures.
leading to hemorrhage. F. Evaluation/outcome criteria:
1. Clotting mechanism restored (increased platelets,
A. Pathophysiology: activation of coagulation system
normal PT).
from tissue injury fibrin microthrombi form
2. Renal function restored (urine output
throughout the vascular system microinfarcts,
>30 mL/hr).
tissue necrosis red blood cells, platelets, pro-
3. Circulation to fingers, toes; no cyanosis.
thrombin, other clotting factors trapped in capillar-
4. No irreversible damage from renal, cerebral,
ies, destroyed in process excessive clotting
cardiac, or adrenal hemorrhage.
release of fibrin split products inhibition of
platelet clotting profuse bleeding. XVI. PERICARDITIS: inflammation of parietal or vis-
B. Risk factors: ceral pericardium or both; acute or chronic condition;
1. Obstetric complications (50% of cases). may occur with or without effusion. Cardiac tampon-
2. Neoplastic disease. ade may result.
3. Low perfusion states (e.g., burns, hypothermia); A. Pathophysiology: fibrosis or accumulation of fluid
hypovolemia. in pericardium compression of cardiac pumping
4. Infections, sepsis. decreased cardiac output increased systemic,
C. Assessmentobjective data: pulmonic venous pressure.

ADULT
1. Skin, mucous membranes: petechiae, ecchymosis. B. Risk factors:
2. Extremities (fingers, toes): cyanosis. 1. Bacterial, viral, fungal infections.
3. Bleeding: venipuncture sites, wound, oral, rectal, 2. Tuberculosis.
vaginal. 3. Collagen diseases.
4. Urine output: oliguria anuria. 4. Uremia.
5. Level of consciousness: LOC progressing to 5. Transmural MI.
coma. 6. Trauma.
6. Laboratory data: prolongedprothrombin time C. Assessment:
(PT) greater than 15 seconds; decreased 1. Subjective data:
platelets, fibrinogen level. a. Pain:
D. Analysis/nursing diagnosis: (1) Typesharp, moderate to severe.
1. Altered tissue perfusion related to peripheral (2) Locationwide area of pericardium, may
microthrombi. radiate: right arm, jaw/teeth.
2. Risk for injury (death) related to bleeding. (3) Precipitating factorsmovement, deep
3. Risk for impaired skin integrity related to inspiration, swallowing.
ischemia. b. Chills; sweating.
4. Altered urinary elimination related to renal c. Apprehension; anxiety.
tubular necrosis. d. Fatigue.
2164_Ch06_347-578 29/03/12 12:29 PM Page 430

430 chapter 6 Physiological Integrity

e. Abdominal pain. i. Monitor for:


f. Shortness of breath. Signs of cardiac tamponade: tachycardia;
2. Objective data: tachypnea; hypotension; pallor; narrowed
a. Vital signs: pulse pressure; pulsus paradoxus; distended
(1)BP: decreased pulse pressure; pulsus neck veins; ECG changes.
paradoxusabnormal drop in systolic 2. Goal: maintain fluid, electrolyte balance.
BP of greater than 8 to 10 mm Hg a. Parenteral fluids as ordered;
during inspiration. strict I&O.
(2) Pulse: tachycardia. b. Assist with feedings; low-sodium diet may
(3) Temperature: elevated; erratic course; low be ordered.
grade. F. Evaluation/outcome criteria:
b. Pericardial friction rub. 1. Relief of pain, dyspnea.
c. Increased CVP; distended neck veins; dependent 2. No complications (e.g., cardiac tamponade).
pitting edema; liver engorgement. 3. Return of normal cardiac functioning.
d. Restlessness. XVII. CHRONIC ARTERIAL OCCLUSIVE
e. Laboratory data: elevated aspartate aminotrans- DISEASE: arteriosclerosis obliterans most common
ferase (AST, or serum glutamic-oxaloacetic occlusive disorder of the arterial system (aorta, large
transaminase [SGOT]), WBC count; CT and medium-size arteries); frequently involves the
or magnetic resonance imaging (MRI) femoral, iliac, and popliteal arteries (Buergers disease).
pericardial thickening; troponin, LDH.
A. Pathophysiology: fatty deposits in intimal, medial
f. Serial ECGs: increased ST segment; echocar-
layer of arterial walls; plaque formation narrowed
diogram: pericardial fluid.
arterial lumens; decreased distensibility decreased
D. Analysis/nursing diagnosis:
blood flow; ischemic changes in tissues.
1. Decreased cardiac output related to impaired
B. Risk factors:
cardiac muscle contraction.
1. Age (>50).
2. Pain related to pericardial inflammation.
2. Sex (men).
3. Anxiety related to unknown outcome.
3. Diabetes mellitus.
4. Fatigue related to inadequate oxygenation.
4. Hyperlipidemiaobesity.
E. Nursing care plan/implementation:
5. Cigarette smoking.
1. Goal: promote physical and emotional comfort.
6. Hypertension.
a. Position: semi-Fowlers (upright or sitting);
7. Family history.
bedrest.
C. Assessment:
b. Vital signs: q24h and prn; apical and
1. Subjective data:
radial pulse; notify physician if heart sounds
a. Pain:
decrease in amplitude or if pulse pressure
(1) Typecramplike.
narrows, indicating cardiac tamponade;
(2) Locationfoot, calf, thigh, buttocks.
see i below.
(3) Durationvariable, may be relieved
ADULT

c. O2 as ordered.
by rest.
d. Medications as ordered:
(4) Precipitating causesexercise (intermittent
(1) Analgesicsaspirin, morphine sulfate,
claudication), but occasionally may occur
meperidine or codeine.
when at rest.
(2) Nonsteroidal anti-inflammatory agents
b. Tingling, numbness in toes, feet.
indomethacin, ketorolac [Toradol].
c. Persistent coldness of one or both lower
(3) Antimicrobial.
extremities.
(4)Digitalis and diuretics, if heart failure
2. Objective data:
present.
a. Lower extremities:
e. Assist with aspiration of pericardial sac
(1) Pedal pulsesabsent or diminished.
(pericardiocentesis) if needed: medicate as
(2) Skinshiny, glossy; dry, cold, chalky white,
ordered; elevate head 60 degrees; monitor
decreased/absent hair, ulcers, gangrene.
ECG; have defibrillator and pacemaker
b. Laboratory data: increased serum cholesterol,
available.
triglycerides, complete blood count (CBC),
f. Prepare for pericardiectomy (excision of
platelets.
constricting pericardium) as ordered.
c. Arteriographyindicates location, nature of
g. Continual emotional support.
occlusion. Noninvasive: ultrasound, segmental
h. Enhance effects of analgesics: positioning;
limb pressure, exercise testing.
turning; NPO.
2164_Ch06_347-578 29/03/12 12:29 PM Page 431

Cardiovascular System 431


D. Analysis/nursing diagnosis: than 6 cm in diameter, 50% will rupture; resected
1. Altered tissue perfusion related to peripheral surgically, reconstructed with synthetic or vascular
vascular disease. graft.
2. Risk for activity intolerance related to pain and A. Risk factors:
sensory changes. 1. Atherosclerosis.
3. Pain related to ischemia. 2. Trauma.
4. Risk for impaired skin integrity related to poor 3. Syphilis.
circulation. 4. Congenital weakness.
5. Risk for injury related to numbness of extremities. 5. Local infection.
E. Nursing care plan/implementation: 6. Cigarette smoking.
1. Goal: promote circulation; decrease discomfort. 7. Uncontrolled hypertension.
a. Position: elevate head of bed on blocks (3 to B. Assessment:
6 inches), because gravity aids perfusion to 1. Subjective data:
thighs, legs; elevating legs increases pain. a. Pain:
b. Comfort: keep warm: avoid chilling or use (1) Constant, boring, neuralgic, intermittent
of heating pads, which may burn skin; apply low back, abdominal.
bed socks. (2) Anginasudden onset may mean rupture
c. Circulation: check pedal pulses, skin color, or dissection, which is an emergency
temperature four times daily. condition.
d. Medications: b. Dyspnea; orthopneapressure on trachea or
(1) Vasodilators. bronchus.
(2) Antiplateletacetylsalicylic acid (ASA), 2. Objective data:
ticlopidine, dipyridamole. a. Vital signs:
(3) Dihydropyridinesnifedipine, amlodipine. (1) Radial pulses differ.
(4) Xanthine derivativespentoxifylline. (2) Tachycardia.
2. Goal: prevent infection, injury. (3) Hypotension following rupture leading to
a. Skin care: use bed cradle, sheepskin, heel shock.
pads; mild soap; dry thoroughly; lotion; do b. Pulsating mass: abdominal, chest wall pulsa-
not massage, to prevent release of thrombus. tion; edema of chest wall (thoracic aneurysm);
b. Foot care: wear properly fitting shoes, slippers periumbilical (abdominal aneurysm); audible
when out of bed; inspect for injury or pres- bruit over aorta.
sure areas; nail care by podiatrist. c. Skin: cyanosis, mottled below level of
3. Goal: health teaching. aneurysm.
a. Skin care; inspect daily. d. Veins: dilated, superficialneck, chest, arms.
b. Activity: balance exercise, rest to increase col- e. Cough: paroxysmal, brassy.
lateral circulation; walk only until painful. f. Diaphoresis, pallor, fainting following rupture.
c. Exercises: walking, Buerger-Allen exercises g. Peripheral pulses:

ADULT
(gravity alternately fills and empties blood (1) Femoral present.
vessels). (2) Pedal weak or absent.
d. Diet: low-fat, heart-healthy diet to slow h. Stool: bloody from irritation.
disease progression. C. Analysis/nursing diagnosis:
e. Lifestyle choices: avoid smoking. 1. Risk for injury related to possible aneurysm
f. Recognize and report signs of occlusion rupture.
(e.g., pain, cramping, numbness in extremities; 2. Pain related to pressure on lumbar nerves.
color changeswhite or blue; temperature 3. Anxiety related to risk of rupture.
changescool to cold). D. Nursing care plan/implementation:
F. Evaluation/outcome criteria: 1. Goal: provide emergency care before surgery for
1. Decreased pain. dissection or rupture.
2. Skin integrity preserved; no loss of limb. a. Vital signs: frequent, depending on severity
3. Quits smoking. (systolic BP <100 mm Hg and pulse >100
4. Does exercises to increase collateral circulation. with rupture).
XVIII. ANEURYSMS (thoracic or abdominal aortic): b. IVs: may have 2 to 4 sites; lactated Ringers
localized or diffuse dilations/outpouching of a vessel solution may be ordered.
wall, usually an artery; exerts pressure on adjacent c. Urine output: monitored every 15 to
structures; affects primarily men over age 60; greater 30 minutes.
2164_Ch06_347-578 29/03/12 12:29 PM Page 432

432 chapter 6 Physiological Integrity

d. O2: usually via nasal prongs. 5. Autoimmune conditions:


e. Medications as ordered: antihypertensives to a. Systemic lupus erythematosus (SLE).
prevent extension of dissection. b. Rheumatoid arthritis (RA).
f. Transport to operating room quickly. c. Scleroderma.
g. See THE PERIOPERATIVE EXPERIENCE, 6. Women between teenage years and age 40.
pp. 365368, for general preoperative care. C. Assessment:
2. Goal: prevent complications postoperatively. 1. Subjective data:
a. Position: initially flat in bed; avoid sharp a. Numbness and sensations of cold. Pain.
flexion of hip and knee, which places pressure b. During red phase: throbbing, paresthesia,
on femoral and popliteal arteries; turn gently tingling in one or more digits.
side to side; note erythema on back from 2. Objective data:
pooled blood. a. Intermittent episodes of classic color changes,
b. Vital signs: CVP; hourly peripheral pulses occurring in sequence in digits: pallor (arterial
distal to graft site, including neurovascular spasm starts) bluish (cyanosis from hypoxia)
check of lower extremities; absent pulses for redness (hyperthermia, as arterial spasm
6 to 12 hours indicates occlusion; check with stops); capillary refill.
Doppler blood flow detector. b. Skin and subcutaneous tissue: atrophy.
c. Urine output: hourly from indwelling catheter. c. Nails: brittle.
(1) Immediately report anuria or oliguria D. Analysis/nursing diagnosis:
(<30 mL/hr). 1. Pain (acute/chronic) related to vasospasm/
(2) Check color for hematuria. altered perfusion of affected tissues and ischemia
(3) Monitor daily blood urea nitrogen (BUN) of tissues.
and creatinine. 2. Altered peripheral tissue perfusion related to
d. Observe for signs of atheroembolization vasospastic disease.
(patchy areas of ischemia); report change in 3. Risk for injury related to numbness.
color, motor ability, or sensation of lower E. Nursing care plan/implementation:
extremities. 1. Goal: maintain warmth in extremities.
e. Observe for signs of bowel ischemia (decreased/ a. Use wool gloves (when handling cold objects
absent bowel sounds, pain, guaiac-positive or touching refrigerator/freezer), wool socks
diarrhea, abdominal distention); may have and insulated shoes in cold weather.
nasogastric tube. b. Avoid prolonged exposure to cold material,
f. Measure abdominal girth; increase seen with environment.
graft leakage. 2. Goal: increase hydrostatic pressure, and therefore
3. Goal: promote comfort. circulation.
a. Position: alignment, comfort; prevent heel a. Vigorous exercise of arms.
ulcers. b. Meds: vasodilators, including calcium channel
b. Medication: narcotics. blockers, nitrates.
ADULT

4. Goal: health teaching. 3. Goal: health teaching:


a. Minimize recurrence: avoid trauma, infection, a. Avoid smoking.
smoking, high-cholesterol diet, obesity. b. Biofeedback for stress management.
b. Regular medical supervision. c. Identify and avoid precipitating factors
E. Evaluation/outcome criteria: (e.g., cold, stress).
1. Surgical intervention before rupture. F. Evaluation/outcome criteria:
2. No loss of renal function. 1. Severity and frequency of attacks are
XIX. RAYNAUDS PHENOMENON: a primary reduced.
vasospastic disease that affects digits of both hands 2. Tissue perfusion is maintained.
(rarely feet). 3. Verbalization of less numbness and tingling.
Relief from discomfort.
A. Pathophysiology: constriction of small arteries and
arterioles from vasospasm or obstruction spasm XX. VARICOSE VEINS: abnormally lengthened,
hypoxia hyperthermia as spasm stops. tortuous, dilated superficial veins (saphenous); result
B. Risk factors: of incompetent valves, especially in lower extremities;
1. Cigarette smoking. process is irreversible.
2. Caffeine. A. Pathophysiology: dilated vein venous stasis
3. Cold temperature. edema, fibrotic changes, pigmentation of skin,
4. Emotional upsets (stress reaction). lowered resistance to trauma.
2164_Ch06_347-578 29/03/12 12:29 PM Page 433

Cardiovascular System 433


B. Risk factors: (5) Normal response is slow filling from below
1. Heredity. in 20 to 30 seconds, with no change in
2. Obesity. rate when tourniquet is removed.
3. Pregnancy. (6) Incompetent veins distend very quickly
4. Chronic disease (heart, liver). with backflow.
5. Occupations requiring long periods of standing. d. Prepare for sclerotherapy or vein ligation and
C. Assessment: stripping.
1. Subjective data: F. Evaluation/outcome criteria:
a. Dull aches; heaviness in legs. 1. Relief or control of symptoms.
b. Pain; muscle cramping. 2. Activity without pain.
c. Fatigue in lower extremities, increased with XXI. VEIN LIGATION AND STRIPPING: surgical
hot weather, high altitude, history of risk intervention for advancing varicosities, stasis ulcera-
factors. tions, and cosmetic needs of client. Procedure involves
2. Objective data: ligation of the saphenous vein at the groin, where it
a. Nodular protrusions along veins. joins the femoral vein; saphenous stripping from the
b. Edema. groin to the ankle; legs are wrapped with a pressure
c. Diagnostic tests: Trendelenburg test; phlebog- bandage. Frequently done as outpatient surgery.
raphy; Doppler flowmeter.
A. See preceding section on varicose veins for
D. Analysis/nursing diagnosis:
assessment data and nursing diagnosis of the client
1. Altered tissue perfusion related to venous valve
requiring surgery.
incompetence.
B. Nursing care plan/implementation:
2. Pain related to edema and muscle cramping.
1. Goal: prevent complications after discharge.
3. Risk for activity intolerance related to leg
a. Position: elevate legs as instructed.
discomfort.
b. Activity: No chair sitting to prevent venous
4. Body image disturbance related to disfigurement
pooling, thrombus formation. Avoid standing
of leg.
in one place.
E. Nursing care plan/implementation:
c. Bleeding: report to physician.
1. Goal: promote venous return from lower extremities.
2. Goal: health teaching to prevent recurrence.
a. Activity: walk every hour.
a. Weight reduction.
b. Discourage prolonged sitting, standing, sitting
b. Avoid constricting garments.
with crossed legs.
c. Change positions frequently.
c. Position: elevate legs q23h; elastic stockings
d. Wear support hose/stockings to enhance
or Ace wraps. Compression stockings.
venous return.
2. Goal: provide for safety.
e. No crossing legs at knees.
a. Assist with early ambulation.
C. Evaluation/outcome criteria:
b. Surgical asepsis with wounds, leg ulcers.
1. No complicationshemorrhage, infection,
c. Observe for hemorrhageif occurs: elevate
nerve damage, deep vein thrombosis.

ADULT
leg, apply pressure, notify physician.
2. No recurrence of varicosities.
d. Observe for allergic reactions if sclerosing
3. Adequate circulation to legs: strong pedal pulses.
drugs used; have antihistamine available.
4. Resumes daily activities; free of pain.
3. Goal: health teaching.
a. Weight-reducing techniques, dietary XXII. DEEP VEIN THROMBOSIS
approaches if indicated. (THROMBOPHLEBITIS): formation of a blood
b. Preventive measures: leg elevation; avoiding clot in an inflamed vein, secondary to phlebitis or
prolonged standing, sitting, high chairs, partial obstruction; may lead to venous insufficiency
tight girdles, constrictive clothing; wear and pulmonary embolism. Deep vein thrombosis
support hose. (DVT) is most serious form.
c. Expectations for Trendelenburg test: A. Pathophysiology: endothelial inflammation
(1) While client is lying down, elevate leg formation of platelet plug (blood clot) slowing
65 degrees for approximately 1 minute to of blood flow increase in procoagulants in local
empty veins. area initiation of clotting mechanisms.
(2) Apply tourniquet high on upper thigh B. Risk factors:
(do not constrict deep veins). 1. Immobility/stasisprolonged sitting, bedrest,
(3) Client stands with tourniquet in place. obesity, pregnancy.
(4) Filling of veins is observed. 2. Venous disease; history DVT.
2164_Ch06_347-578 29/03/12 12:29 PM Page 434

434 chapter 6 Physiological Integrity

3. Ageincreased incidence in elderly. 2. Goal: prevent complications.


4. Gendermore often women. a. Observe for signs of embolism (pain at site
5. Hypercoagulability of blood. of embolism); allergic reaction (anaphylactic
6. Intimal damageIVs, drug abuse. shock) with streptokinase.
7. Fractures. b. Precautions: no rubbing or massage of limb.
8. Oral contraceptives (related to estrogen content). c. Medications: anticoagulants (sodium heparin,
C. Assessment: enoxaparin, warfarin [Coumadin]); streptoki-
1. Subjective data: nase (Varidase), tissue plasminogen activator
a. Calf stiffness, soreness. (Table 6.23).
b. Severe pain: walking, dorsiflexion of foot d. Bleeding: hematuria, epistaxis, ecchymosis.
(Homans signmay be unreliable). Check INR levels.
2. Objective data: e. Skin care, to relieve increased redness/
a. Vein: redness, heat, hardness, threadiness. maceration from hot or cold applications.
b. Limb: swollen, pale, cold. f. ROM: unaffected limb.
c. Vital signs: low-grade fever. 3. Goal: health teaching.
d. Diagnostic tests: venogram, impedance plethys- a. Precautions: tight garters, girdles; sitting with
mography (electrical resistance to blood flow), legs crossed; oral contraceptives.
ultrasonography. b. Preventive measures: walking daily, swimming
D. Analysis/nursing diagnosis: several times weekly if possible, wading, rest
1. Altered peripheral tissue perfusion related to periods with legs elevated, elastic stockings
venous stasis. (may remove at bedtime).
2. Pain related to inflammation. c. Medication side effects: anticoagulantspink
3. Activity intolerance related to leg pain. toothbrush, hematuria, easily bruised.
4. Risk for injury related to potential pulmonary (1) Carry Medic Alert card/bracelet.
emboli. (2) Contraindicated drugsaspirin,
E. Nursing care plan/implementation: glutethimide (Doriden), chloramphenicol
1. Goal: provide rest, comfort, and relief from pain. (Chloromycetin), neomycin, phenylbuta-
a. Bedrest until therapeutic level of heparin zone (Butazolidin), barbiturates.
reached (5 to 7 days with traditional heparin; d. Prepare for surgery (thrombectomy, vein
after 24 hours with low-molecular-weight ligation).
heparin). F. Evaluation/outcome criteria:
b. Position: as ordered; usually extremity elevated; 1. No complications (e.g., embolism).
watch for pressure points. 2. No recurrence of symptoms.
c. Apply warm, moist heat to affected area as 3. Free of painambulates without discomfort.
prescribed (cold may also be ordered). XXIII. PERIPHERAL EMBOLISM: fragments of
d. Assess progress of affected area: swelling, pain, thrombi, globules of fat, clumps of tissue, calcified
soreness, temperature, color. plaques, or air moves in the circulation and lodges in
ADULT

e. Administer analgesics as ordered.

Table 6.23
Nursing Responsibilities with Anticoagulant Therapy
Heparin Warfarin (Coumadin) Low-Molecular-Weight Heparin (LMWH)
Monitor PTT (2538 sec) (23 times PT (1115 sec) Antifactor Xa, CBC, platelets
baseline) (11/221/2 times baseline)
INR: 23.5
Inspect Ecchymosis, bleeding gums, Bleeding, ecchymosis Bleeding, hemorrhage, unusual bruising
petechiae, hematuria
Administer With an infusion pump; never mix Same time every day; PO Subcutaneous twice daily
with other drugs; never aspirate;
avoid massaging site
Avoid Salicylates and other anticoagu- Same as heparin Warfarin, platelet aggregation inhibitors
lants, e.g., antacids, corticosteroids, (e.g., ASA, NSAIDs, dextran)
penicillin, phenytoin
Antidote Protamine sulfate Vitamin K Protamine sulfate
2164_Ch06_347-578 29/03/12 12:29 PM Page 435

Gastrointestinal System 435


vessel, obstructing blood flow; thrombic emboli most 5. Ineffective family coping, disabling, related to
common; may be venous or arterial. inadequate resources or knowledge to provide
appropriate family nutrition.
C. Nursing care plan/implementation:
GASTROINTESTINAL 1. Goal: prevent complications of specific deficiency.
SYSTEM a. Identify etiology of nutritional deficiency.
I. GENERAL NUTRITIONAL DEFICIENCIES b. Recognize signs of nutritional deficiencies
A. Assessment: (see Chapter 9, Table 9.6).
1. Subjective data: c. Identify foods high in deficient nutrient
a. Mental irritability or confusion. (see Chapter 9).
b. History of poor dietary intake. d. Evaluate economic resources to purchase
c. History of lack of adequate resources to appropriate foods.
provide adequate nutrition. e. Identify community resources for assistance.
d. Lack of knowledge about proper diet, food f. Monitor progress for potential additional
selection, or preparation. illnesses.
e. History of eating disorders. 2. Goal: health teaching.
f. Paresthesia (burning and tingling): hands a. Effects of nutritional deficiencies on health.
and feet. b. Foods to include in diet to avoid deficits.
2. Objective data: D. Evaluation/outcome criteria:
a. Appearance: listless; posture: sagging shoulders, 1. Complications do not occur.
sunken chest, poor gait. 2. Client gains weight.
b. Muscle: weakness, fatigue, wasted 3. Client selects appropriate foods to alleviate
appearance. deficiency.
c. GI: indigestion, vomiting, enlarged liver, II. CELIAC DISEASE (gluten enteropathy, nontropical
spleen. sprue): immune to gluten, causing impaired absorp-
d. Cardiovascular: tachycardia on minimal tion and digestion of nutrients through the small
exertion; bradycardia at rest; enlarged heart, bowel. Affects adults and children and is characterized
elevated BP. by inability to digest and use sugars, starches, and fats.
e. Hair: brittle, dry, thin, sparse; lack of natural A. Pathophysiology: intolerance to the gliadin frac-
shine; color changes; can be easily plucked tion of grains causing degeneration of the epithelial
out. surface of the intestine, atrophy of the intestinal
f. Skin: dryness (xerosis), scaly, dyspigmentation, villi, and impaired absorption of essential nutrients.
petechiae, lack of fat under skin. B. Risk factors:
g. Mouth: 1. Possible genetic or familial factors.
(1) Teeth: missing, abnormally placed, caries. 2. Hypersensitivity response.
(2) Gums: bleed easily, receding. 3. History of childhood celiac disease.
(3) Tongue: swollen, sore. C. Assessment:

ADULT
(4) Lips: red, swollen, angular fissures at 1. Subjective data: family history.
corners. 2. Objective data:
h. Eyes: pale conjunctivae, corneal changes. a. Loss: weight, fat deposits, musculature.
i. Nails: brittle, ridged. b. Anemia.
j. Nervous system: abnormal reflexes. c. Vitamin deficiencies.
k. Laboratory data: blooddecreased albumin, d. Abdomen distended with flatus.
iron-binding capacity, lymphocyte, hemoglo- e. Stools: diarrhea, foul smelling, bulky, fatty,
bin, and hematocrit. float in commode.
l. Anthropometric measurements document f. Skin condition known as dermatitis
nutritional deficiencies. herpetiformis.
B. Analysis/nursing diagnosis: g. History of acute attacks of fluid and
1. Altered nutrition, less than body requirements, electrolyte imbalances.
related to poor dietary intake. h. Diagnostic tests: stool for fat; barium enema;
2. Knowledge deficit (learning need) related to antibody tests, including endomysial antibody
nutritional requirements. (EMA); blood studies of: iron, folate, pro-
3. Altered health maintenance related to inability teins, minerals, and clotting factors; small
to provide own nutritional care. bowel biopsy.
4. Ineffective individual coping related to eating i. Gluten-free diet leads to remission of symptoms.
disorders.
2164_Ch06_347-578 29/03/12 12:29 PM Page 436

436 chapter 6 Physiological Integrity

D. Analysis/nursing diagnosis: b. Malaise, dull ache in right upper quadrant,


1. Altered nutrition, less than body requirements, abdominal pain.
related to inability to digest and use sugars, c. Repugnance to: food, cigarette smoke, strong
starches, and fats. odors, alcohol.
2. Diarrhea related to intestinal response to gluten d. Headache.
in diet. 2. Objective data:
3. Fluid volume deficit related to loss through a. Fever.
excessive diarrhea. b. Liver: enlarged (hepatomegaly), tender,
4. Knowledge deficit (learning need) related to smooth.
dietary restrictions to control symptoms. c. Skin: icterus in sclerae of eyes, jaundice;
E. Nursing care plan/implementation: rash; pruritus; petechiae, bruises.
1. Goal: prevent weight loss. d. Urine: normal, dark.
a. Diet: high in calories, protein, vitamins, and e. Stool: normal, clay colored, loose.
minerals, and gluten free. f. Vomiting, weight loss.
(1) Avoid: wheat, rye, oats, barley. g. Lymph nodes: enlarged.
(2) All other foods permitted. h. Laboratory data:
b. Daily weights to monitor weight changes. (1) Bloodleukocytosis.
2. Goal: health teaching. (2) Increased AST (SGOT), alanine amino-
a. Nature of disease. transferase (ALT, or serum glutamic-pyruvic
b. Dietary restrictions and allowances. transaminase [SGPT]), and bilirubin levels,
c. Complications of noncompliance. alkaline phosphatase.
F. Evaluation/outcome criteria: (3) Urineincreased urobilinogen.
1. No further weight loss. i. See Table 6.24 for comparison of hepatitis A,
2. Normal stools. B, C, and D.
3. Fluid/electrolyte balance obtained and maintained. D. Analysis/nursing diagnosis:
III. HEPATITIS: inflammation of the liver. 1. Pain related to inflammation of liver.
2. Impaired skin integrity related to pruritus.
A. Pathophysiology:
3. Activity intolerance related to fatigue.
1. Infection with hepatitis A (formerly called infec-
4. Risk for infection to others related to
tious hepatitis), hepatitis B (formerly called
incubation/infectious period.
serum hepatitis), hepatitis C (single-stranded
5. Altered nutrition, less than body requirements,
RNA virus of the Flaviviridae family; usually
related to repugnance of food.
asymptomatic), delta hepatitis (infection caused
6. Social isolation related to isolation precautions.
by a defective RNA virus that requires hepatitis B
E. Nursing care plan/implementation:
virus to multiply), or hepatitis E (major etiologi-
1. Goal: prevent spread of infection to others.
cal agent of the enterically transmitted non-A,
a. Isolation according to type
non-B hepatitis worldwide) inflammation,
(1) Hepatitis A:
necrosis, and regeneration of liver parenchyma.
ADULT

(a) Contact precautions (see Chapter 3,


Hepatocellular injury impairs clearance of uro-
pp. 90, 92, 93, Tables 3.3, 3.7,
bilinogen elevated urinary urobilinogen; and,
and 3.8).
as injury increases conjugated bilirubin not
(b) Private room preferred.
reaching the intestines decreased urine and
(c) Gown/gloves for direct contact with
fecal urobilinogen increased serum bilirubin
feces.
jaundice.
(d) Hand washing when in direct contact
2. Failure of liver to detoxify products increased
with feces.
toxic products of protein metabolism gastritis
(2) Hepatitis B: blood and body fluid precautions.
and duodenitis.
(a) Needle/dressing precautions.
B. Risk factors:
(b) Private room not necessary.
1. Exposure to virus.
(c) Gown: only if enteric precautions also
2. Exposure to carriers of virus.
necessary.
3. Exposure to hepatotoxins such as dry-cleaning
(d) Hand washing: use gloves when in
agents.
direct contact with blood.
4. Nonimmunized.
(3)Hepatitis C: blood and body fluid
C. Assessment:
precautionssame as hepatitis B,
1. Subjective data:
except when in countries with
a. Anorexia, nausea.
2164_Ch06_347-578 29/03/12 12:29 PM Page 437

Gastrointestinal System 437

Table 6.24
Etiology, Incidence, and Epidemiological and Clinical Comparison of Hepatitis A, Hepatitis B,
Hepatitis C, and Delta Hepatitis
Hepatitis A Hepatitis B Hepatitis C Delta Hepatitis
Incubation 26 wk 4 wk to 6 mo Variable: 14160 days; Same as hepatitis B
average, 50 days
Communicable Until 79 days after Several monthsas long as As long as virus As long as virus
jaundice occurs virus present in blood present in blood present in blood
Transmission Fecal-oral; blood; sexual Parenteral; sexual; perinatal Percutaneous, via Parenteral; blood
contaminated blood,
parenteral drug abuse;
some fecal-oral forms;
sexual; perinatal
Sources Crowding; contaminated: Contaminated: needles, Persons who have Contaminated
food, milk, or water syringes, surgical received 15 or more needles, syringes
instruments blood transfusions;
IV drug users; persons
traveling to contami-
nated areas
Portal of entry GI tract; asymptomatic Integumentary: blood Blood Integumentary:
carriers plasma or transfusions blood
HB antigen Not present Present Not present Present as with
hepatitis B
Incidence Sporadic epidemics; Increased in ages 1529, All age groups; higher Same as hepatitis B
increased in children <15 particularly in heroin addic- in adults because
tion; occupational hazard of exposure to risk
for: laboratory workers, factors
nurses, physicians
Immunity Preexposure: immune Preexposure: hepatitis B None; immune None
globulin, 0.02 mL/kg vaccine globulin may be
given
Postexposure: within Postexposure: immune
2 wk of exposure, as globulin with high amounts
above of anti-HBs (HBIG);
hepatitis B vaccine
Prevention Hand washing, Care when handling prod Same as hepatitis B Same as hepatitis B
use of gloves; ucts contaminated by
hepatitis A vaccine blood, use of gloves;

ADULT
hepatitis B vaccine
Severity Mild Mild to moderate Mild to moderate Moderate to severe
Fever Common Uncommon Uncommon Uncommon
Nausea/vomiting Common Common Common Common

fecal-oral form, then use hepatitis A (3) Hepatitis C: prophylaxis not as effective;
precautions also. ISG may be given.
(4) Delta: same as hepatitis B. (4) Delta: same as for hepatitis B.
b. Passive immunity for contacts. 2. Goal: promote healing.
(1) Hepatitis A: hepatitis A vaccine (Havrix, a. Diet as tolerated:
VAQTA), immune serum globulin (ISG), (1) NPO with parenteral infusions, when in
administered before and after exposure. acute stage.
(2) Hepatitis B: hepatitis B immune globulin (2) High protein, high carbohydrate, low fat,
(HBIG) (Recombivax HB, Energix-B) or offered in frequent small meals.
ISG. (3) Push fluids, if not contraindicated; I&O.
2164_Ch06_347-578 29/03/12 12:29 PM Page 438

438 chapter 6 Physiological Integrity

b. Medications: IV. PANCREATITIS: inflammatory disease of the pan-


(1) Antiviral for clients with persistently creas that may result in autodigestion of the pancreas
elevated ALT levels. by its own enzymes.
(2) Interferon for initial treatment of A. Pathophysiology: proteolytic enzymes within the
hepatitis C. pancreas are activated by endotoxins, exotoxins,
3. Goal: monitor for worsening of disease process, ischemia, anoxia, or trauma. Pancreatic enzymes
failure to respond to prescribed treatment. begin process of autodigestion of pancreas and sur-
a. Observe urinedark due to presence of rounding tissues; also activate other enzymes that
bile and stool, clay colored. digest cellular membranes. Autodigestion leads to:
b. Observe sclerae, laboratory tests for increasing edema, hemorrhage, vascular damage, coagulation
jaundice. necrosis, and fat necrosis.
c. Mental confusion, unusual somnolence may B. Risk factors:
indicate decreased liver function. 1. Obesity.
d. Weigh dailyincrease indicates fluid retention 2. Alcoholism, alcohol consumption.
and possible ascites. 3. Biliary tract disease.
4. Goal: health teaching. 4. Abdominal trauma.
a. Diet and fluid intake to promote liver 5. Surgery.
regeneration. 6. Drugs.
b. Importance of rest and limited activity to 7. Metabolic disease.
reduce metabolic workload of liver. 8. Intestinal disease.
c. Personal hygiene practices to prevent 9. Obstruction of the pancreatic ducts.
contamination. 10. Infections.
d. Avoid: alcohol, blood donations, and 11. Carcinoma.
contact with communicable infections. 12. Adenoma.
e. Follow-up case referral; may take 13. Hypercalcemia.
6 months for full recovery. C. Assessment:
f. Teach contacts about available 1. Subjective data:
immunizations. a. Pain:
5. Goal: promote comfort. (1) Sudden onset; severe, widespread,
a. Bedrest to combat fatigue and reduce constant, and incapacitating.
metabolic needs until hepatomegaly subsides; (2) Locationepigastrium, right upper
semi-Fowlers or supine positioning. quadrant (RUQ) and left upper quadrant
b. Oral hygiene q12h to decrease nausea. (LUQ) of abdomen; radiates to back,
c. ROM exercises to maintain muscle strength. flanks, and substernal area.
d. Measures to reduce pruritus: b. Nausea.
(1) Mild, oil-based lotion to reduce itching. c. History of risk factors.
(2) Nails cut short, cotton gloves, long-sleeved d. Dyspnea.
ADULT

clothing to prevent skin injury from 2. Objective data:


scratching. a. Elevated: temperature, pulse, respirations,
(3) Environment: cool and dry. BP (unless in shock).
(4) Cool wet soaks to skin. b. Decreased breath sounds related to
(5) Diversional activities. atelectasis/pleural effusion.
(6) Medications as ordered: c. Increased crackles, cyanosis.
(a) Emollients to relieve dry skin. d. Hemorrhage, shock.
(b) Topical corticosteroids to reduce e. Vomiting.
inflammation. f. Fluid and electrolyte imbalances,
(c) Antihistamines to reduce itch. dehydration.
(d) Tranquilizers and sedatives to allow rest g. Decreased bowel sounds; abdominal
and prevent exhaustion. tenderness with guarding.
F. Evaluation/outcome criteria: h. Stools: bulky, pale, foul smelling,
1. Tolerates food; nausea and vomiting steatorrhea (excessive fat in stools).
decreased. i. Skin: pale, moist, cold; may be
2. Signs of infection/inflammation absent. jaundiced.
3. No complications, hemorrhage, liver damage, j. Muscle rigidity.
ascites. k. Supine position leads to increased pain.
4. No jaundice noted. l. Fluid accumulation in the abdomen.
2164_Ch06_347-578 29/03/12 12:29 PM Page 439

Gastrointestinal System 439


m. Laboratory data: c. Vitamin supplements.
(1) Elevated: d. Avoid alcohol.
(a) Amylase, serum, and urine. 6. Goal: prevent complications.
(b) Serum lipase, AST (SGOT). a. Monitor for signs of:
(c) Alkaline phosphatase. (1) Peritonitis.
(d) Bilirubin, glucose; serum and urine. (2) Perforation.
(e) Urine protein, WBC count. (3) Respiratory complications.
(f ) Leukocytes. (4) Hypotension, shock.
(g) BUN. (5) DIC.
(h)LDH (liver function). (6) ARDS.
(2) Decreased: (7) Hemorrhage from ulcers, varices.
(a) Serum calcium. (8) Anemia.
(b) Protein. (9) Encephalopathy.
n. Ultrasound for gallstones, CT scan. 7. Goal: health teaching.
D. Analysis/nursing diagnosis: a. Food selections for low-fat, bland diet.
1. Altered nutrition, less than body requirements, b. Necessity of vitamin therapy.
related to nausea and vomiting. c. Importance of avoiding alcohol.
2. Pain related to inflammatory and autodigestive d. Signs and symptoms of recurrence.
processes of pancreas. e. Importance of rest, to prevent relapse.
3. Fluid volume deficit related to inflammation, f. Desired effects and side effects of prescribed
decreased intake, and vomiting. medications:
4. Ineffective breathing pattern related to pain and (1) Narcotics for pain.
pleural effusion. (2) Antiemetics for nausea and vomiting.
5. Knowledge deficit (learning need) related to risk (3) Pancreatic hormone and enzymes to replace
factors and disease management. enzymes not reaching duodenum.
E. Nursing care plan/implementation: F. Evaluation/outcome criteria:
1. Goal: control pain. 1. Pain is relieved.
a. Medications: analgesicsmeperidine (not 2. No complications (e.g., peritonitis,
morphine or codeine due to spasmodic respiratory).
effect). 3. States dietary allowances and restrictions.
b. Position: sitting with knees flexed. 4. Takes medications as ordered; states purposes,
2. Goal: rest injured pancreas. side effects.
a. NPO. V. CIRRHOSIS: chronic inflammation and fibrosis
b. Nasogastric (NG) tube to low suction. (irreversible scarring) of the liver in which some
c. Medications: liver cells (hepatocytes) undergo necrosis and others
(1) Antiulcers. undergo proliferative regeneration.
(2) Antibiotics.
A. Pathophysiology: progressive destruction of hepatic

ADULT
(3) Antiemetics.
cells loss of normal metabolic function of the
(4) Antispasmodics.
liver and formation of scar tissue. Regeneration and
(5) Anticholinergics.
proliferation of fibrous tissue obstruction of the
(6) Histamine2 receptors (cimetidine).
portal vein increased portal hypertension, ascites,
3. Goal: prevent fluid and electrolyte imbalance.
liver failure, and eventual death.
a. Monitor: vitals, CVP.
B. Risk factors:
b. IVs, fluids, blood, albumin, plasma.
1. Alcohol abuse most common cause.
4. Goal: prevent respiratory and metabolic
2. Nutritional deficiency with decreased protein
complications.
intake.
a. Cough, deep breathe, change position.
3. Hepatotoxins.
b. Monitor: blood sugar as ordered.
4. Virus.
c. Monitor calcium levels: Chvosteks and
5. Hepatitis B and C.
Trousseaus signs positive when calcium
C. Assessment:
deficit exists (see ENDOCRINE SYSTEM,
1. Subjective data:
IV. THYROIDECTOMY, p. 473, for
a. Chronic feeling of malaise.
description of tests).
b. Anorexia, nausea.
5. Goal: provide adequate nutrition.
c. Abdominal pain.
a. Low-fat diet.
d. Pruritus.
b. Bland, small, frequent meals.
2164_Ch06_347-578 29/03/12 12:29 PM Page 440

440 chapter 6 Physiological Integrity

2. Objective data: b. Deep breathing q2h to prevent respiratory


a. GI: complications.
(1) Malnutrition, weight loss. c. Skin care, topical medications to relieve pruri-
(2) Vomiting. tus; nail care to decrease possibility of further
(3) Flatulence. skin injury.
(4) Ascites. d. Frequent oral hygiene related to nausea,
(5) Enlarged liver and spleen. vomiting, and fetid breath.
(6) Glossitis. 3. Goal: improve fluid and electrolyte balance.
(7) Fetid breath (sweet, musty odor). a. IV fluids and vitamins.
b. Bloodcoagulation defects, possible b. I&O, hourly urines during acute attacks.
esophageal varices, portal hypertension, c. Daily: girths, weights to monitor fluid
bleeding from gums and injection sites. balance.
c. Skin and hairedema, jaundice, spider d. Diuretics as ordered to decrease edema.
angioma (telangiectasias); palmar erythema, e. May receive serum albumin to promote
decreased pubic and axillary hair. adequate vascular volume, prevent azotemia
d. Reproductivemenstrual abnormalities, and encephalopathy, and promote diuresis
gynecomastia, testicular atrophy, impotence. (observe carefully, because albumin could
e. Neurological deficitsmemory loss, hepatic escape quickly through cell walls and cause
coma, decreased level of consciousness: increase in ascites).
flapping tremor, grimacing. 4. Goal: promote optimum nutrition within dietary
f. Laboratory data: restrictions.
(1) Decreased: albumin, potassium, magne- a. NPO during acute episodes.
sium, blood urea nitrogen (BUN). b. Small, frequent meals when able to eat.
(2) Elevated: prothrombin time, globulins, c. Low protein (to decrease the amount of
ammonia, AST (SGOT), bromsulphalein nitrogenous materials in the intestines) and
(BSP), alkaline phosphate, uric acid, blood sodium (to decrease fluid retention).
sugar. d. Moderate carbohydrate (to meet energy
g. Diagnostic tests: demands) and fat (to make diet more
(1) Celiac angiography, hepatoportography. palatable to clients who are anorexic).
(2) Liver biopsy. 5. Goal: provide emotional support.
(3) Paracentesis. a. Quiet environment during acute episodes
D. Analysis/nursing diagnosis: to decrease external stimuli.
1. Altered nutrition, less than body requirements, related b. Refer to community agencies for assistance
to decreased intake, nausea, and vomiting. for client (e.g., Alcoholics Anonymous; for
2. Risk for injury related to decreased prothrombin family, Al-Anon/Alateen).
production. 6. Goal: health teaching.
3. Activity intolerance related to fatigue. a. Avoid alcohol, exposure to infections.
ADULT

4. Fatigue related to anorexia and nutritional b. Dietary allowances, restrictions (see Chapter 9,
deficiencies. Sodium-restricted diet, p. 694, and purine-
5. Self-esteem disturbance related to physical body restricted diet, p. 699).
changes. c. Drugs: names, purposes.
6. Risk for impaired skin integrity related to pruritus. d. Signs, symptoms of disease; complications.
E. Nursing care plan/implementation: e. Stress-management techniques.
1. Goal: provide for special safety needs. F. Evaluation/outcome criteria:
a. Monitor vitals (including neurological) fre- 1. No complications.
quently for hemorrhage from esophageal 2. Nutritional status improves; lists dietary
varices (may have Sengstaken-Blakemore or restrictions.
Linton tube inserted). 3. No alcohol consumption.
b. Prepare client for LeVeen shunt surgery for 4. Lists signs and symptoms of progression of
portal hypertension as needed. disease and complications.
c. Assist with paracentesis performed for ascites; 5. Complies with discharge plan, becomes involved
monitor vitals to prevent shock during with an alcohol treatment program.
procedure. VI. ESOPHAGEAL VARICES: life-threatening
2. Goal: relieve discomfort caused by complications. hemorrhage from tortuous dilated, thin-walled veins
a. Position: semi-Fowlers or Fowlers to decrease in submucosa of lower esophagus. May rupture when
pressure on diaphragm due to ascites. chemically or mechanically irritated, or when pressure
2164_Ch06_347-578 29/03/12 12:29 PM Page 441

Gastrointestinal System 441


is increased because of sneezing, coughing, use of the b. Antibiotics as ordered to prevent
Valsalva maneuver, or excessive exercise. infection.
A. Pathophysiology: portal hypertension related to c. Reduce portal hypertension; give
cirrhosis of the liver distended branches of the propranolol (Inderal), vasopressin
azygos vein and inferior vena cava where they join (Pitressin).
the smaller vessels of the esophagus. 4. Goal: provide emotional support.
B. Risk factors for hemorrhage: a. Stay with client.
1. Exertion that increases abdominal pressure. b. Calm atmosphere.
2. Trauma from ingestion of coarse foods. 5. Goal: health teaching.
3. Acid pepsin erosion. a. Explain use of tube to client and family.
C. Assessment: b. Bland diet instructions.
1. Subjective data: c. Recognize signs of bleeding.
a. Fear. d. Avoid straining at stool.
b. Dysphagia. e. Avoid aspirin because of increased bleeding
c. History: alcohol ingestion, liver tendency.
dysfunction. F. Evaluation/outcome criteria:
2. Objective data: 1. Survives acute bleeding episode.
a. Hematemesis. 2. Further episodes prevented by avoiding irritants,
b. Hemorrhage: sudden, often fatal. especially alcohol.
c. Decreased BP; increased pulse, respirations. 3. Improves nutritional status.
d. Melena (occult blood in stool). 4. Recognizes symptoms of complications
e. Diagnostic endoscopy. (e.g., bleeding).
D. Analysis/nursing diagnosis: 5. Demonstrates knowledge of medications by
1. Fluid volume deficit related to blood loss. avoiding aspirin.
2. Risk for injury related to hemorrhage. VII. DIAPHRAGMATIC (HIATAL) HERNIA: pro-
3. Fear related to massive blood loss. trusion of part of stomach through diaphragm and
4. Ineffective individual coping related to into thoracic cavity (Fig. 6.8). Types: sliding (most
complications of cirrhosis. common); paraesophageal rolling.
E. Nursing care plan/implementation: A. Pathophysiology: weakening of the musculature
1. Goal: provide safety measures related to of the diaphragm, aggravated by increased intra-
hemorrhage. abdominal pressure protrusion of the abdominal
a. Recognize signs of shock; vitals q15 min. organs through the esophageal hiatus reflux of
b. Assist with insertion of Sengstaken-Blakemore gastric contents esophagitis.
(or Minnesota) or Linton tube (tube is B. Risk factors:
large and uncomfortable for client during 1. Congenital abnormality.
insertion); explain procedure briefly to 2. Penetrating wound.
decrease fear and attempt to gain clients 3. Age (middle-aged or elderly).

ADULT
cooperation. 4. Women more than men.
c. While tube is in place, observe for 5. Obesity.
respiratory distress; if present, deflate 6. Ascites.
the balloon by releasing pressure; do not 7. Pregnancy.
cut the tube. 8. History of constipation.
d. Deflate the balloon as ordered to prevent C. Assessment:
necrosis. 1. Subjective data:
e. NG tube to low gastric suction; monitor a. Pressure: substernal.
for amount of bright-red blood; irrigate b. Pain: epigastric, burning.
only as ordered using tepid, not iced, c. Eructation, heartburn after eating.
solutions. d. Dysphagia.
f. Vitamin K as ordered to control bleeding. e. Symptoms aggravated when recumbent.
2. Goal: promote fluid balance. 2. Objective data:
a. IV fluids, expanders. a. Cough, dyspnea.
b. Fresh blood as ordered to avoid increased b. Tachycardia, palpitations.
ammonia; aids in coagulation. c. Bleeding: hematemesis, melena, signs of
3. Goal: prevent complications of hepatic coma. anemia due to gastroesophageal irritation,
a. Saline cathartics as ordered to remove old ulceration, and bleeding.
blood from GI tract.
2164_Ch06_347-578 29/03/12 12:29 PM Page 442

442 chapter 6 Physiological Integrity

HIATAL
HERNIA

ESOPHAGUS

GASTRIC
RUGAE

DIAPHRAGM

STOMACH

Figure 6.8 Hiatal hernia. SMALL LARGE INTESTINE


(From Venes, D [ed]: Tabers INTESTINES
Cyclopedic Medical Dictionary, ed 21.
FA Davis, Philadelphia, 2009.)

d. Diagnostic tests: (c) Avoid eating 2 hours before bedtime.


(1) Chest x-rays, showing protrusion of (d) High-protein, low-fat foods to decrease
abdominal organs into thoracic cavity. heartburn.
(2) Barium swallow (upper GI series) to show (2) Positioning: head elevated to increase
presence of hernia. movement of food into stomach.
(3) Endoscopy. Symptoms may decrease if head of bed
e. Symptoms parallel those of gastroesophageal at home is elevated on 8-inch blocks.
reflux disease (GERD). (3) Weight reduction to decrease abdominal
D. Analysis/nursing diagnosis: pressure.
ADULT

1. Pain related to irritation of lining of GI tract. (4) Medications as ordered:


2. Altered nutrition, less than body requirements, (a) 30 mL antacid 1 hour after meals
related to dysphagia. and at bedtime.
3. Sleep pattern disturbance related to increase in (b) Avoid anticholinergic drugs, which
symptoms when recumbent. decrease gastric emptying.
4. Risk for aspiration related to reflux of gastric 2. Postsurgical:
contents. a. Goal: provide for postoperative safety needs.
5. Activity intolerance related to dyspnea. (1) Respiratory: deep breathing, coughing,
6. Anxiety related to palpitations. splint incision area.
E. Nursing care plan/implementation: (2) Nasogastric (NG) tube: check patency.
1. Presurgical: (a) Drainage: should be small amount.
a. Goal: promote relief of symptoms. (b)Color: dark brown 6 to 12 hours after
(1) Diet: surgery, changing to greenish-yellow.
(a) Small, frequent feedings of soft, bland (c) Do not disturb tube placement to avoid
foods, to reduce abdominal pressure traction on suture line.
and reflux. (3) Position: initially head of bed elevated
(b) Fluid when swallowing solids may slightly, then semi-Fowlers; turn side to
push food into stomach; hot fluid may side frequently, to prevent pressure on
work best. diaphragm.
2164_Ch06_347-578 29/03/12 12:29 PM Page 443

Gastrointestinal System 443


(4) Maintain closed chest drainage if indicated 4. Drugs (calcium channel blockers, NSAIDs,
(see Table 11.5). theophylline).
(5) Check for return of bowel sounds. 5. Elevated intra-abdominal pressure (obesity,
b. Goal: promote comfort and maintain pregnancy, heavy lifting).
nutrition. C. Assessment:
(1) IVs for hydration and electrolytes. 1. Subjective data:
(2) Initiate feeding through gastrostomy tube a. Heartburn (pyrosis)substernal or retrosternal;
if present. may mimic angina; 20 minutes to 2 hours after
(a) Usually attached to intermittent, low eating.
suction after surgery. b. Regurgitationsour or bitter taste not
(b) Aspirate gastric contents before associated with belching or nausea.
feedingdelay if 75 mL or more is c. Dysphagia or odynophagia (difficult or
present; report these findings to painful swallowing)severe cases.
physician. d. Belching, feeling bloated.
(c) Feed in high Fowlers or sitting position; e. Nocturnal cough.
keep head elevated for 30 minutes after 2. Objective data:
eating. a. Hoarseness, wheezing.
(d) Warm feeding to room temperature; b. Diagnostic tests: 24-hour pH monitoring;
dilute with H2O if too thick. barium swallow with fluoroscopy; endoscopy.
(e) Give 50 mL H2O before feeding; D. Analysis/nursing diagnosis:
200 to 500 mL feeding by gravity 1. Pain related to acid reflux and esophageal
over 10 to 15 minutes; follow with inflammation.
50 mL H2O. 2. Knowledge deficit (learning need) related to mod-
(f ) Give frequent mouth care. ifications needed to control reflux.
c. Goal: health teaching. E. Nursing care plan/implementation:
(1) Avoid constricting clothing and activities 1. Goal: promote comfort and reduce reflux episodes.
that increase intra-abdominal pressure a. Medications as ordered:
(e.g., lifting, bending, straining at stool). (1) Antacids to neutralize gastric acid.
(2) Weight reduction. (2) Histamine2 (H2) receptor antagonists
(3) Dietary needs: small, frequent, soft, (cimetidine, ranitidine, famotidine) to
bland meals. reduce gastric acid secretion and support
(4) Chew thoroughly. tissue healing.
(5) Upright position for at least 1 hour after (3) Proton pump inhibitor (omeprazole
meals. [Prilosec]) to inhibit gastric enzymes
F. Evaluation/outcome criteria: and suppress gastric acid secretion.
1. Obtains relief from symptoms; is comfortable. b. Diet: avoid strong stimulants of acid secretion
2. Receives adequate, balanced nutrition. (caffeine, alcohol); avoid foods that reduce

ADULT
3. Describes dietary changes, recommended LES competence (fatty foods, onions,
positioning, and activity limitations to prevent tomato-based foods); increase protein; restrict
recurrence. spicy, acidic foods until healing occurs.
VIII. GASTROESOPHAGEAL REFLUX DISEASE c. Activity: avoid heavy lifting, straining,
(GERD): inappropriate relaxation of the lower constrictive clothing, bending over.
esophageal sphincter (LES) in response to unknown d. Position: elevate head of bed 6 to 12 inches for
stimulus. sleeping. Reflux more likely on right side.
2. Goal: health teaching.
A. Pathophysiology: gastric volume or intra-
a. Weight reduction if indicated.
abdominal pressure elevated, or LES tone
b. Smoking cessation.
decreased frequent episodes of acid reflux
c. Diet modification: avoid overeatingeat 4
breakdown of mucosal barrier esophageal
to 6 small meals.
inflammation, hyperemia, and erosion fibrotic
d. Medication administration.
tissue formation esophageal stricture
e. Potential complications if uncontrolled
impaired swallowing.
(hemorrhage, aspiration).
B. Risk factors:
F. Evaluation/outcome criteria:
1. Hiatal hernia.
1. No heartburn reported.
2. Dietfoods that lower pressure of LES (fatty
2. Changes diet as instructed.
foods, chocolate, cola, coffee, tea).
3. No complications of continued reflux.
3. Smoking.
2164_Ch06_347-578 29/03/12 12:29 PM Page 444

444 chapter 6 Physiological Integrity

IX. PEPTIC ULCER DISEASE: circumscribed loss of g. Alcohol and caffeine use (aggravates preexist-
mucosa, submucosa, or muscle layer of the gastroin- ing condition).
testinal tract caused by a decreased resistance of gastric h. Difficulty coping with stressful situations.
mucosa to acid-pepsin injury. Peptic ulcer disease is a 3. Stress ulcers.
chronic disease and may occur in the distal esophagus, a. Severe trauma or major illness.
stomach, upper duodenum, or jejunum. Gastric ulcers, b. Severe burns (Curlings ulcer); develop in
located on the lesser curvature of the stomach, are 72 hours with majority of persons with burns
larger and deeper than duodenal ulcers and tend to over more than 35% of the body surface.
become malignant. Duodenal ulcers are located on c. Head injuries or intracranial disease
the first part of the duodenum and are more common (Cushings ulcer).
than gastric ulcers. Esophageal ulcers occur in the d. Medications in large doses: corticosteroids,
esophagus. Stress ulcers, an acute problem, occur salicylates, ibuprofen, indomethacin,
after a major insult to the body. phenylbutazone (Butazolidin).
A. Pathology: failure of the body to regenerate e. Shock.
mucous epithelium at a sufficient rate to counter- f. Sepsis.
balance the damage to tissue during the break- C. Assessment:
down of protein; decrease in the quantity and 1. Subjective data:
quality of the mucus; poor local mucosal blood a. Gastric ulcers.
flow, along with individual susceptibility to (1) Pain:
ulceration. A peptic ulcer is a hole in the lining (a) Type: gnawing, aching, burning.
of the stomach, duodenum, or esophagus. This (b) Location: epigastric, left of midline,
hole occurs when the lining of these organs is localized.
corroded by the acidic digestive juices secreted by (c) Occurrence: periodic pain, often 2 hours
the stomach cells. Excess acid is still considered after eating.
to be significant in ulcer formation. The leading (d) Relief: antacids; may be aggravated, not
cause of ulcer disease is currently believed to be relieved, by food.
infection of the stomach by Helicobacter pylori (e) Some clients report no discomfort at all.
(H. pylori). Another major cause of ulcers is (2) Weakness.
chronic use of nonsteroidal anti-inflammatory (3) History of risk factors as above.
drugs (NSAIDs). Cigarette smoking is also an b. Duodenal ulcers.
important cause of ulcers. (1) Pain:
B. Risk factors: (a) Type: gnawing, aching, burning,
1. Gastric ulcers. hungerlike, boring.
a. Infection with H. pylori. (b) Location: right epigastric, localized;
b. Decreased resistance to acid-pepsin injury. steady pain near midline of back may
c. Increased histamine release inflammatory indicate perforation.
reaction. (c) Occurrence: 1 to 3 hours after eating,
ADULT

d. Ulcerogenic drugs (aggravate preexisting worse at end of day or during the night;
conditions). initial attack occurs spring or fall; history
e. Cigarette smoking. of remissions and exacerbations.
f. Increased alcohol and caffeine use (aggravates (d) Relief: food, antacids, or both.
preexisting conditions). (e) Some clients report no discomfort
g. Gastric ulcer is thought to be a risk for gastric at all.
cancer. (2) Nausea.
h. Difficulty coping with stressful situations. (3) History of risk factors (see IX. B.).
2. Duodenal ulcers. c. Stress ulcers.
a. Infection with H. pylori. (1) Pain: often painless until serious complica-
b. Elevated gastric acid secretory rate. tion (hemorrhage, perforation) occurs.
c. Elevated gastric acid levels postprandially (2) History of risk factors as above.
(after eating). 2. Objective data:
d. Increased rate of gastric emptying increased a. Gastric ulcer.
amount of acid in duodenum irritation and (1) Vomiting blood (hematemesis).
breakdown of duodenal mucosa. (2) Melena (tarry stools).
e. Ulcerogenic medication use (aggravates (3) Weight loss.
preexisting conditions). (4) X-ray (upper GI series) confirms crater
f. Cigarette smoking. (punched-out appearance, clean base).
2164_Ch06_347-578 29/03/12 12:29 PM Page 445

Gastrointestinal System 445


(5) Endoscopy confirms presence of ulcer; 4. Goal: health teaching.
biopsy for cytology. a. Medications:
(6) Monitor for blood loss: CBC, stool for (1)Antibiotics. Sometimes antibiotics
occult blood. work best if given in combination with
(7) Orthostatic hypotension. omeprazole (Prilosec), H2 blockers, or
(8) Laboratory data: positive for H. pylori. bismuth (Pepto-Bismol). Caution: use of
b. Duodenal ulcer. antibiotic treatment can cause allergic
(1) Eructation. reactions, diarrhea, and severe antibiotic-
(2) Vomiting blood (hematemesis). induced colitis.
(3) Regurgitation of sour liquid into back of (a) Tetracycline.
mouth. (b) Amoxicillin.
(4) Constipation. (c) Metronidazole (Flagyl).
(5) X-ray (upper GI series) confirms ulcer (d) Clarithromycin (Biaxin).
craters and niches, as well as outlet defor- (2) Histamine antagonists: given with
mities: round or oval funnel-like lesion meals/bedtime to block the action of
extending into musculature. histamine-stimulated gastric secretions
(6) Endoscopy for direct visualization. (basal and stimulated); inhibit pepsin
(7) Monitor for blood loss: CBC, stool for secretion and reduce the volume of
occult blood. gastric secretions.
(8) Orthostatic hypotension. (a) Cimetidine (Tagamet) inhibits gastrin
(9) Laboratory data: positive for H. pylori. release; can be given PO, IV, or IM;
c. Stress ulcer. cannot be given within 1 hour of
(1) GI bleeding. antacid therapy.
(2) Multiple, superficial erosions affecting (b) Ranitidine (Zantac) has greater reduc-
large area of gastric mucosa. tion of acid secretion, longer duration,
D. Analysis/nursing diagnosis (all types): less frequent administration (twice daily
1. Pain related to erosion of gastric lining. versus 4 times a day), and fewer side
2. Ineffective individual coping related to inability effects than cimetidine.
to change lifestyle. (c) Nizatidine (Axid).
3. Altered nutrition, less than body requirements, (d) Famotidine (Pepcid).
related to inadequate intake. (3) Proton pump inhibitors (gastric acid
4. Knowledge deficit (learning need) regarding inhibitors): superior in treating esophageal
preventive measures. ulcers; equal to other H2 receptors for
5. Risk for injury related to possible hemorrhage or gastric and duodenal ulcers.
perforation. (a) Omeprazole (Prilosec).
E. Nursing care plan/implementation (all types): (b) Lansoprazole (Prevacid).
1. Goal: promote comfort. (c) Pantoprazole.

ADULT
a. Medications as ordered to decrease pain (4) Antiulcers: give 1 to 3 hours after meals and
(see E. 4. Goal: health teaching); sedatives to at bedtime to decrease pain by lowering
decrease anxiety. acidity; monitor for:
b. Prepare for diagnostic tests. (a) Diarrhea (seen most often with magne-
(1) X-rays; upper GI series (barium swallow); sium carbonate and magnesium oxide
lower GI (barium enema). [Maalox, Mylanta]).
(2) Endoscopy. (b) Constipation (seen most often with cal-
(3) Gastric analysis, to determine amount of cium carbonate [Tums] or aluminum
hydrochloric acid in GI tract. hydroxide [Amphojel]).
2. Goal: prevent/recognize signs of complications. (c) Electrolyte imbalance (seen with sys-
a. Monitor vitals for shock. temic antacid, soda bicarbonate).
b. Check stool for occult blood/ (d) Best 1 to 3 hours after meals.
hemorrhage. (e) Liquids more effective than tablets; if
c. Palpate abdomen for perforation taking tablets, chew slowly.
(rigid, boardlike); arterial bleeding. (5) Sucralfate (Carafate) and misoprostol
3. Goal: provide emotional support. (Cytotec): given 1 hour before meals and at
a. Stress-management techniques. bedtime.
b. Restful environment. (a) Locally active topical agent that forms a
c. Prepare for surgery, if necessary. protective coat on mucosa, prevents
2164_Ch06_347-578 29/03/12 12:30 PM Page 446

446 chapter 6 Physiological Integrity

further digestive action of both acid gastric secretions and decreased physical activity
and pepsin. of the stomach (being done less often).
(b) Must not be given within 30 minutes of 6. Combination of vagotomy and gastrectomy.
antacids. B. Analysis/nursing diagnosis:
(6) Anticholinergicwhen used, given before 1. Pain related to surgical incision.
meals to decrease gastric acid secretion and 2. Ineffective breathing pattern related to high
delay gastric emptying. surgical incision.
(7) Important: avoid aspirin (could increase 3. Risk for trauma related to possible complications
bleeding possibility). postgastrectomy.
b. Diet: 4. Knowledge deficit (learning need) regarding
(1) Change diet only to relieve symptoms; diet medication regimen and factors that aggravate
may not influence ulcer formation. condition.
(2) Avoid foods that increase aciditycaffeine 5. Fear related to possible precancerous lesion.
and alcohol in moderation. 6. Ineffective individual coping related to adjust-
(3) Plan: ments in lifestyle needed to lessen symptoms.
(a) Small, frequent meals (to prevent C. Nursing care plan/implementation:
exacerbations of symptoms related to 1. Goal: promote comfort in the postoperative period.
an empty stomach). a. Analgesics: to relieve pain and allow client to
(b) Weight control. cough, deep breathe to prevent pulmonary
c. Complicationssigns and symptoms: complications.
(1) Gastric ulcers may be premalignant. b. Position: semi-Fowlers to aid in breathing.
(2) Perforation. 2. Goal: promote wound healing.
(3) Hemorrhage. a. Keep dressings dry.
(4) Obstruction. b. NG tube to low intermittent suction (Levin)
d. Lifestyle changes: or low continuous suction (Salem sump).
(1) Decrease: (1) Check drainage from NG tube; normally
(a) Smoking. bloody first 2 to 3 hours postsurgery, then
(b) Noise. brown to dark green.
(c) Rush. (2) Excessive bright-red blood drainage: take
(d) Confusion. vital signs; report vital signs, color and vol-
(2) Increase: ume of drainage to physician immediately.
(a) Communication. (3) Irrigate gently with saline in amount
(b) Mental/physical rest. ordered; do not irrigate against resistance;
(c) Compliance with medical regimen. may not be done in early postoperative
F. Evaluation/outcome criteria: period.
1. Avoids foods/liquids that cause irritation. (4) Tape tube securely to face, but prevent
2. Takes prescribed medications. obstructed vision.
ADULT

3. Pain decreases. (5) Frequent mouth and nostril care.


4. No complications. 3. Goal: promote adequate nutrition and hydration.
5. States signs and symptoms of complications. a. Administer parenteral fluids as ordered.
6. Participates in stress-reduction activities. b. Accurate I&O.
7. Stops smoking. c. Check bowel sounds, at least q4h; NPO
X. GASTRIC SURGERY: peformed when ulcer medical 1 to 3 days; bowel sounds normally return
regimen is unsuccessful, ulcer is determined to be in 21 to 36 hours; oral fluids as ordered
precancerous, or complications are present. when bowel sounds presentusually
30 mL, then small feedings, then bland
A. Types:
liquids to soft diet.
1. Subtotal gastrectomy: removal of a portion of the
d. Observe for nausea and vomiting due to
stomach.
suture line edema, food intake (too much,
2. Total gastrectomy: removal of the entire
too fast).
stomach.
4. Goal: prevent complications.
3. Antrectomy: removal of entire antrum (lower)
a. Check dressing q4h for bleeding.
portion of the stomach.
b. Vitamin B12 and iron replacement as
4. Pyloroplasty: repair of the pyloric opening of the
indicated to avoid pernicious anemia and
stomach.
iron-deficiency anemia.
5. Vagotomy: interruption of the impulses carried
c. Avoid dumping syndrome.
by the vagus nerve, which results in reduction of
2164_Ch06_347-578 29/03/12 12:30 PM Page 447

Gastrointestinal System 447


D. Evaluation/outcome criteria: XII. CHOLECYSTITIS/CHOLELITHIASIS: inflam-
1. Hemorrhage, dumping syndrome avoided. mation of gallbladder due to bacterial infection, pres-
2. Healing begins. ence of cholelithiasis (stones, cholesterol, calcium, or
3. Adjust lifestyle to prevent recurrence/marginal bile in the gallbladder), or choledocholithiasis (stone
ulcer. in the common bile duct) and/or obstruction. Acute
XI. DUMPING SYNDROME: hypoglycemic-type cholecystitis is abrupt in onset, but the client usually
episode; occurs postoperatively after gastric resection has a history of several attacks of fatty food intoler-
(may also occur after vagotomy, antrectomy, or ance. The client with chronic cholecystitis has a
gastroenterostomy), when food and fluids that are history of several attacks of moderate severity and
more hyperosmolar than the jejunal secretions pass has usually learned to avoid fatty foods to decrease
quickly into jejunum, producing fluid shifts from symptoms.
bloodstream to jejunum. This is a mild problem for A. Pathophysiology: calculi from increased concen-
about 20% of clients and will disappear in a few tration of bile salts, pigments, or cholesterol due to
months to a year. Symptoms cause serious problem metabolic or hemolytic disorders, biliary stasis
for about 7% of clients. This discomfort may occur precipitation of salts into stones, or inflammation
during a meal or up to 30 minutes after the meal and causing bile constituents to become altered.
last from 20 to 60 minutes. The reaction is greatest B. Risk factors:
after the ingestion of sugar. 1. Adult women.
A. Assessment: 2. Obesity.
1. Subjective data: 3. Pregnancy or previous pregnancies.
a. Feeling of fullness, weakness, faintness. 4. Use of birth control pills or hormone
b. Palpitations. replacement therapy.
c. Nausea. 5. High-fat, low-fiber diets.
d. Discomfort during or after eating. 6. Rapid weight loss.
2. Objective data: 7. History of Crohns disease.
a. Diaphoresis. 8. Genetics.
b. Diarrhea. 9. Age: increased risk over 40.
c. Fainting. 10. Certain drugs to lower lipids; clofibrate
d. Symptoms of hypoglycemia. (Atromid-S).
B. Analysis/nursing diagnosis: 11. Other diseases: cirrhosis of liver.
1. Altered nutrition, more than body requirements, C. Assessment:
related to bodys inability to properly digest 1. Subjective data:
high-carbohydrate, high-sodium foods. a. Pain:
2. Diarrhea related to food passing into jejunum (1) Typesevere colic, radiating to the
too quickly. back under the scapula and to the right
3. Risk for injury related to hypoglycemia. shoulder.
4. Knowledge deficit (learning need) related to (2) Positive Murphys signa sign of gallblad-

ADULT
dietary restrictions. der disease consisting of pain on taking a
C. Nursing care plan/implementation: deep breath when pressure is placed over
1. Goal: health teaching. the location of the gallbladder.
a. Include: (3) Locationright upper quadrant,
(1) Increased fat, protein to delay emptying. epigastric area, flank (Fig. 6.9).
(2) Rest after meals. (4) Durationspasm of duct attempting to
(3) Small, frequent meals. dislodge stone lasts until dislodged or
(4) Fluids between meals. relieved by medication, or sometimes by
b. Avoid: vomiting.
(1) Foods high in salt, carbohydrate. b. GIanorexia, nausea, feeling of fullness,
(2) Large meals. indigestion, intolerance of fatty foods.
(3) Stress at mealtime. 2. Objective data:
(4) Fluids at mealtime. a. GIbelching, vomiting, clay-colored stools.
D. Evaluation/outcome criteria: b. Vital signsincreased pulse, fever.
1. No complications. c. Skinchills, jaundice.
2. Client heals. d. Urinedark amber.
3. No further ulcers. e. Laboratory dataelevated:
4. Incorporates health teaching into lifestyle and (1) WBC count.
prevents syndrome. (2) Alkaline phosphatase.
2164_Ch06_347-578 29/03/12 12:30 PM Page 448

448 chapter 6 Physiological Integrity

Epigastric Region
Cardiovascular:
Aortic aneurysm
Gastrointestinal:
Bowel obstruction
Early appendicitis
Gallbladder
Gastroenteritis
Intestinal ischemia
Pancreatitis
Peritonitis
Right Upper Quadrant Hematologic: Left Upper Quadrant
Cardiovascular: Sickle cell crisis Cardiovascular:
Myocardial infarction Metabolic: Myocardial infarction
Gastrointestinal: Diabetic ketoacidosis Gastrointestinal:
Appendicitis Diverticulitis
Gallbladder Duodenal ulcer
Hepatitis Gastric ulcer
Liver abscess Gastritis
Pancreatitis Pancreatitis
Perforated ulcer Ruptured spleen
Genitourinary:
Genitourinary:
Right kidney infection
Right kidney stone Left kidney infection
Left kidney stone
Respiratory:
Respiratory:
Right lower lobe pneumonia
Pulmonary embolism Pneumonia
Viral:
Viral:
Herpes zoster (shingles)
Herpes zoster (shingles) RIGHT LEFT
UPPER UPPER
QUADRANT QUADRANT

RIGHT LEFT
LOWER LOWER
Right Lower Quadrant QUADRANT QUADRANT Left Lower Quadrant
Gastrointestinal: Gastrointestinal:
Appendicitis Diverticulitis
Cecal perforation Incarcerated hernia
Cecal volvulus Regional ileitis
Diverticulitis Genitourinary:
Incarcerated hernia
ADULT

Kidney stone
Regional ileitis
Gynecologic:
Genitourinary:
Ectopic pregnancy
Kidney stone
Ovarian cyst
Gynecologic: Pelvic inflammatory disease
Ectopic pregnancy Salpingitis
Ovarian cyst
Pelvic inflammatory disease
Salpingitis
Figure 6.9 Abdominal pain by location. (Modified from Caroline, NL: Emergency Care in the Streets, ed 5. Boston, Little, Brown, 1995, p 641; and
from Venes, D [ed]: Tabers Cyclopedic Medical Dictionary, ed 20. FA Davis, Philadelphia, 2005.)

(3) Serum amylase, lipase. (3) Computed tomography (CT) scan.


(4) AST (SGOT). (4) Endoscopic retrograde cholangiopancre-
(5) Bilirubin. atography (ERCP).
f. Diagnostic studies: D. Analysis/nursing diagnosis:
(1) Ultrasound. 1. Pain related to obstruction of bile duct due to
(2) Cholangiography. cholelithiasis.
2164_Ch06_347-578 29/03/12 12:30 PM Page 449

Gastrointestinal System 449


2. Altered nutrition, more than body requirements, (iii) Provide enough tubing to allow
related to ingestion of fatty foods. turning without tension.
3. Altered nutrition, less than body requirements, (iv) Empty and record bile drainage q8h.
related to hesitancy to eat due to anorexia and (b) Position: low Fowlers to semi-Fowlers to
nausea. facilitate T-tube drainage.
4. Risk for fluid volume deficit related to episodes (c) Dressing: dry to protect skin (because
of vomiting. bile excoriates skin).
5. Knowledge deficit (learning need) related to (d) Clamp T-tube as ordered.
dietary restrictions. (i) Observe for: abdominal distention,
E. Nursing care plan/implementation: pain, nausea, chills, or fever.
1. Nonsurgical interventions: (ii) Unclamp tube and notify physician
a. Goal: promote comfort. if symptoms appear.
(1) Medications as ordered: meperidine, c. Goal: prevent complications.
antibiotics, antispasmodics, electrolytes. (1) IV fluids with vitamins.
(2) Avoid morphine due to spasmodic effect. (2) Cough, turn, and deep breathe with
(3) NG tube to low suction. open incision, particularly with removal
(4) Diet: fat free when able to tolerate food. of gallbladder (prone to respiratory com-
(5) Lithotripsy: gallstones fragmented by plication because of high incision).
shock waves. (3) Early ambulation to prevent vascular
(6) Oral dissolution therapy: ursodeoxycholic complications and aid in expelling flatus.
acid (ursodiol [Actigall]). (4) Monitor for jaundice: skin, sclerae, urine,
b. Goal: health teaching: stools.
(1) Signs, symptoms, and complications of (5) Monitor for signs of hemorrhage,
disease. infection.
(2) Fat-free diet. d. Goal: health teaching.
(3) Desired effects and side effects of (1) Diet: fat free for 6 weeks.
prescribed medications. (2) Signs of complications of food intolerance,
(4) Prepare for possible removal of gallbladder pain, infection, hemorrhage.
(cholecystectomy) if conservative treatment F. Evaluation/outcome criteria:
unsuccessful. 1. No complications.
2. Surgical interventions: 2. Able to tolerate food.
a. PreoperativeGoal: prevent injury (see 3. Plans follow-up care.
I. PREOPERATIVE PREPARATION, 4. Possible weight reduction.
p. 365). XIII. OBESITYmore calories consumed than
b. PostoperativeGoal: promote comfort (see expended leading to fat accumulation. Most common
III. POSTOPERATIVE EXPERIENCE, nutritional/metabolic disease in the United States.
p. 369).
A. Definition: Womenmore than 45% above ideal

ADULT
(1) Laparoscopic laser cholecystectomytiny
body weight. Menmore than 35% above ideal
incisions/puncture wounds; gallbladder is
body weight. Determined by body mass index
removed using a video-guided system with
(BMI) formula: divide weight in kilograms by
a camera; client is discharged that day or
height in meters squared (or divide weight in
next day, able to resume normal diet and
pounds by height in inches squared) and multiply
work activities in a few days.
by 703:
(2) Endoscopic retrograde cholangiopancreatogra-
phy (ERCP) with papillotomyremoves Wt (lb) Wt (kg)
703 or: 703
stones from bile duct. No incision; done Ht (in2) Ht (m2)
under sedation, not anesthesia.
(3) Open-incision cholecystectomy. B. Risk factors:
(a) Promote tube drainage: 1. Genetics (e.g., BMI), hormonal.
(i) NG tube to low suction. 2. Environmental (e.g., physical activity).
(ii) T-tube to closed-gravity drainage, 3. Diet (e.g., fat, calories).
to preserve patency of edematous 4. Some medications (e.g., tricyclic antidepressants
common duct and ensure bile [TCAs], insulin, sulfonylurea agents).
drainage; usual amount 500 to C. Comorbidity:
1,000 mL/24 hr; dark brown 1. Cardiovascular disease; hypertension.
drainage. 2. Type 2 diabetes.
2164_Ch06_347-578 29/03/12 12:30 PM Page 450

450 chapter 6 Physiological Integrity

3. Gallbladder disease. B. Risk factors:


4. Arthritis. 1. Men more than women.
5. Cancer (colorectal, breast, prostate). 2. Most frequently seen between ages 10 and
6. Stroke. 30 years.
7. Emotional distress. C. Assessment:
8. Surgical risk. 1. Subjective data:
D. Assessment: a. Pain: generalized, then right lower quadrant at
1. OverweightBMI 25.0 to 29.9 kg/m2. McBurneys point, with rebound tenderness.
2. ObeseBMI 30.0 kg/m2. b. Anorexia, nausea.
3. Morbid obesityBMI greater than 40 kg/m2. 2. Objective data:
E. Analysis/nursing diagnosis: a. Vital signs: elevated temperature, shallow
1. Altered nutrition, more than body requirements, respirations.
related to genetics, environmental, or dietary b. Either diarrhea or constipation.
factors. c. Vomiting, fetid breath odor.
2. Chronic low self-esteem/body image disturbance d. Splinting of abdominal muscles, flexion of
related to view of self in contrast to societal knees onto abdomen.
values; control, sex, and love issues (see also e. Laboratory data:
Chapter 10, Body Image Disturbance, (1) WBC count elevated (>10,000).
p. 720). (2) Neutrophil count elevated (>75%).
3. Activity intolerance related to imbalance between f. Diagnostic studies:
oxygen supply and demand, and to sedentary (1) Ultrasound.
lifestyle. (2) CT scan.
F. Nursing care plan/implementationGoal: D. Analysis/nursing diagnosis:
decrease weight, initially 10% from baseline. 1. Pain related to inflammation of appendix.
1. Modify eating pattern (quality vs. quantity); 2. Risk for trauma related to ruptured appendix.
portion size; modify composition: calories, 3. Knowledge deficit (learning need) related to
fat, and cholesterol; daily kcal by 500 to possible surgery.
1,000. E. Nursing care plan/implementation:
2. Increase activitymoderate activity 1. Goal: promote comfort.
30 minutes daily. a. Preoperative:
3. Behavioral therapychange eating behaviors; (1) Explain procedures.
motivation and readiness to lose weight. (2) Assist with diagnostic work-up.
4. Weight-loss drugs in combination with lifestyle b. Postoperative:
changes. (1) Relieve pain related to surgical incision.
5. Surgerygastric stapling, bariatric (Roux-en-Y). (2) Prevent infection: wound care, dressing
G. Evaluation/outcome criteria: technique.
1. Weight loss of 1 to 2 lb/wk for 6 months. (3) Prevent dehydration: IVs, I&O, fluids to
ADULT

2. Reduction in kcal by 500 to 1,000/day. solids by mouth as tolerated.


3. Increased daily physical activity. (4) Promote ambulation to prevent postopera-
4. Expressed commitment to lose weight. tive complications.
XIV. APPENDICITIS: obstruction of appendiceal F. Evaluation/outcome criteria:
lumen and subsequent bacterial invasion of appen- 1. No infection.
diceal wall; acute emergency. 2. Tolerates fluid; bowel sounds return.
3. Heals with no complications.
A. Pathophysiology: when obstruction is partial or
mild, inflammation begins in mucosa with slight XV. HERNIA: protrusion of the intestine through a
appendiceal swelling, accompanied by periumbilical weak portion of the abdominal wall.
pain. As the inflammatory process escalates and/or A. Types:
obstruction becomes more complete, the appendix 1. Reducible: visceral contents return to their
becomes more swollen, the lumen fills with pus, normal position, either spontaneously or by
and mucosal ulceration begins. When inflamma- manipulation.
tion extends to the peritoneal surface, pain is 2. Irreducible, or incarcerated: contents cannot be
referred to the right lower abdominal quadrant. returned to normal position.
Danger: rigidity over the entire abdomen is usually 3. Strangulated: blood supply to the structure
indicative of ruptured appendix; the client is then within the hernia sac becomes occluded (usually
prone to peritonitis. a loop of bowel).
2164_Ch06_347-578 29/03/12 12:30 PM Page 451

Gastrointestinal System 451


4. Most common hernias: umbilical, femoral, C. Assessment:
inguinal, incisional, and hiatal. 1. Subjective data: pain: cramplike; left lower
B. Pathophysiology: weakness in the wall may be quadrant of abdomen.
either congenital or acquired. Herniation occurs 2. Objective data:
when there is an increase in intra-abdominal pres- a. Constipation or diarrhea, flatulence.
sure from: coughing, lifting, crying, straining, b. Fever.
obesity, or pregnancy. c. Rectal bleeding.
C. Assessment: d. Diagnostic procedures:
1. Subjective data: (1) Palpation reveals tender colonic mass.
a. Pain, discomfort. (2) Barium enema (done only in absence
b. History of feeling a lump. of inflammation) reveals presence of
2. Objective data: diverticula.
a. Soft lump, especially when straining or (3) Sigmoidoscopy/colonoscopy.
coughing. D. Analysis/nursing diagnosis:
b. Sometimes alteration in normal bowel 1. Constipation related to dietary intake.
pattern. 2. Pain related to inflammatory process of
c. Swelling. intestines.
D. Analysis/nursing diagnosis: 3. Risk for fluid volume deficit related to episodes
1. Activity intolerance related to pain and of diarrhea or bleeding.
discomfort. 4. Risk for injury related to bleeding.
2. Risk for trauma related to lack of circulation to 5. Knowledge deficit (learning need) related to
affected area of bowel. prevention of constipation.
3. Pain related to protrusion of intestine into E. Nursing care plan/implementation:
hernia sac. 1. Goal: bowel rest during acute episodes.
E. Nursing care plan/implementation: a. Diet: soft, liquid.
1. Goal: prevent postoperative complications. b. Fluids, IVs if oral intake not adequate.
a. Monitor bowel sounds. c. Medications:
b. Prevent postoperative scrotal swelling with (1) Antibiotics: ciprofloxacin (Cipro),
inguinal hernia by applying ice and support metronidazole (Flagyl), cephalexin
to scrotum. (Keflex), doxycycline (Vibramycin).
2. Goal: health teaching. (2) Antispasmodics: chlordiazepoxide (Librax),
a. Prevent recurrence with correct body dicyclomine (Bentyl), Donnatal,
mechanics. hyoscyamine (Levsin).
b. Gradual increase in exercise. d. Monitor stools for signs of bleeding.
F. Evaluation/outcome criteria: healing occurs with 2. Goal: promote normal bowel elimination.
no further hernia recurrence. a. Diet: bland, high in vegetable fiber if no
XVI. DIVERTICULOSIS: A diverticulum is a small inflammation.

ADULT
pouch or sac composed of mucous membrane that (1) Include: fruits, vegetables, whole-grain
has protruded through the muscular wall of the cereal, unprocessed bran.
intestine. The presence of several of these is called (2) Avoid: foods difficult to digest (corn, nuts).
diverticulosis. Inflammation of the diverticula is b. Bulk-forming agents as ordered: methylcellu-
called diverticulitis. lose, psyllium.
c. Monitor: abdominal distention, acute bowel
A. Pathophysiology: weakening in a localized area of
symptoms.
muscular wall of the colon (especially the sigmoid
3. Goal: health teaching.
colon), accompanied by increased intraluminal
a. Methods to avoid constipation.
pressure.
b. Foods to include/avoid in diet.
B. Risk factors:
c. Relaxation techniques.
1. Diverticulosis:
d. Signs and symptoms of complications of
a. Age: seldom before 35 years; 60% incidence
chronic inflammation: abscess, obstruction,
in older adults.
fistulas, perforation, or hemorrhage.
b. History of constipation.
F. Evaluation/outcome criteria:
c. Diet history: low in vegetable fiber, high in
1. Inflammation decreases.
carbohydrate.
2. Bowel movements return to normal.
2. Diverticulitis: highest incidence between ages 50
3. Pain decreases.
and 60 years.
4. No perforation, fistulas, or abscesses noted.
2164_Ch06_347-578 29/03/12 12:30 PM Page 452

452 chapter 6 Physiological Integrity

XVII. ULCERATIVE COLITIS: inflammation of mucosa 4. Fluid volume deficit related to frequent episodes
and submucosa of the large intestine. Inflammation of diarrhea.
leads to ulceration with bleeding. Involved areas are 5. Knowledge deficit (learning need) related to
continuous. Disease is characterized by remissions and methods to control symptoms.
exacerbations. 6. Social isolation related to continual diarrhea
A. Pathophysiology: Currently believed to be an episodes.
autoimmune disease. The bodys immune system is E. Nursing care plan/implementation:
called on to attack the inner lining of the large 1. Goal: prevent disease progression and complications.
intestine, causing inflammation and ulceration. a. Administer medications:
Edema and hyperemia of colonic mucous mem- (1) Salicylates: sulfasalazine (Azulfidine),
brane superficial bleeding with increased peri- olsalazine (Dipentum), mesalamine
stalsis, shallow ulcerations, abscesses; bowel wall (Asacol, Pentasa). All given PO in high
thins and shortens and becomes at risk for perfora- doses. Mesalamine (Rowasa) is given in
tion. Increased rate of flow of liquid ileal contents enema or suppository form.
decreased water absorption and diarrhea. (2) Corticosteroids: prednisone, PO or
B. Risk factors: IV. Hydrocortisone (Cortenema) is
1. Highest occurrence in young adults (ages 20 to given by enema.
40 years). (3) Immunosuppressants: azathioprine
2. Genetic predisposition: higher in whites, Jews. (Imuran), and 6-mercaptopurine
3. Autoimmune response. (Purinthol), cyclosporine (Sandimmune),
4. Infections. and methotrexate (Rheumatrex).
5. More common in urban areas (upper-middle (4) Nicotine.
incomes and higher educational levels). (5) Sedatives and tranquilizers to produce rest
6. Nonsmokers/ex-smokers. and comfort.
7. Genetic, inherited, or familial tendencies. (6) Absorbents: kaolin/pectin (Kaopectate).
8. Chronic ulcerative colitis is a risk factor for (7) Anticholinergics and antispasmotics to
colon cancer. relieve cramping and diarrhea: atropine
C. Assessment: sulfate, phenobarbital, diphenoxylate/
1. Subjective data: atropine sulfate (Lomotil).
a. Urgency to defecate, particularly when (8) Anti-infective agents to relieve bacterial
standing. overgrowth in bowel and limit secondary
b. Loss of appetite, nausea. infections: metronidazole (Flagyl).
c. Colic-like abdominal pain. (9) Potassium supplements to relieve deficiencies
d. History of intolerance to dairy products. related to excessive diarrhea.
e. Emotional depression. (10) Calcium folate and vitamin B12 when
2. Objective data: malabsorption is present.
a. Diarrhea: 10 to 20 stools/day; can be chronic 2. Goal: reduce psychological stress.
ADULT

or intermittent, episodic or continual; stools a. Provide quiet environment.


contain blood, mucus, and pus. b. Encourage verbalization of concerns.
b. Weight loss and malnutrition, dehydration. 3. Goal: health teaching.
c. Fever. a. Diet:
d. Rectal bleeding. (1) Avoid: coarse-residue, high-fiber foods
e. Laboratory data: decreased: RBC count, potas- (e.g., raw fruits and vegetables), whole
sium, sodium, calcium, bicarbonate related to milk, cold beverages (because of
excessive diarrhea. inflammation).
f. Lymphadenitis. (2) Include: bland, high-protein, high-vitamin,
g. Diagnostic tests: high-mineral, high calorie foods.
(1) Sigmoidoscopy/colonoscopy for visualiza- (3) Parenteral hyperalimentation for
tion of lesions. severely ill.
(2) Barium enema. (4) Force fluids by mouth.
D. Analysis/nursing diagnosis: b. Monitor for colon cancer, especially 8 to
1. Diarrhea related to increased flow rate of ileal 10 years after incidence.
contents. 4. Goal: prepare for surgery if medical regimen
2. Self-esteem disturbance related to progression of unsuccessful.
disease and increased number and odor of stools. a. Possible surgical procedures:
3. Pain (acute) related to inflammatory process. (1) Permanent ileostomy (J pouch).
2164_Ch06_347-578 29/03/12 12:30 PM Page 453

Gastrointestinal System 453


(2) Continent ileostomy (Kock pouch). D. Analysis/nursing diagnosis, Nursing care plan/
(3) Total colectomy, anastomosis with rectum. implementation, Evaluation/outcome criteria
(4) Total colectomy, anastomosis with anal (see XVII. ULCERATIVE COLITIS, p. 452).
sphincter. XIX. INTESTINAL OBSTRUCTION: blockage in
F. Evaluation/outcome criteria: movement of intestinal contents through small or
1. Fluid balance is obtained and maintained. large intestine.
2. Alterations in lifestyle are managed.
A. Pathophysiology:
3. Stress-management techniques are successful.
1. Mechanical causesphysical impediments to pas-
4. Complications such as fistulas, obstruction,
sage of intestinal contents (e.g., adhesions, hernias,
perforation, and peritonitis are avoided.
neoplasms, inflammatory bowel disease, foreign
5. Client is prepared for surgery if medical regimen
bodies, fecal impactions, congenital or radiational
is unsuccessful or complications develop.
strictures, intussusception, or volvulus).
XVIII. CROHNS DISEASE: a chronic inflammatory 2. Paralytic causespassageway remains open, but
disease causing ulcerations in the small and large peristalsis ceases (e.g., after abdominal surgery,
intestines. The immune system seems to react to a abdominal trauma, hypokalemia, myocardial
variety of substances and/or bacteria in the intes- infarction, pneumonia, spinal injuries, peritoni-
tines, causing inflammation, ulceration, and bowel tis, or vascular insufficiency).
injury. Called Crohns colitis when only large B. Assessment:
intestine is involved; Crohns enteritis when only 1. Subjective data: pain related to:
small intestine is involved; terminal ileitis when a. Proximal loop obstruction: upper abdominal,
lowest part of small intestine is involved; Crohns sharp, cramping, intermittent pain.
enterocolitis or ileocolitis when both small and large b. Distal loop obstruction: poorly localized,
intestines are involved. cramping pain.
A. Pathophysiology: one of two conditions called 2. Objective data:
inflammatory bowel disease (ulcerative colitis is the a. Bowel sounds: initially loud, high pitched;
other) that affects all layers of the ileum, the colon, then when smooth muscle atony occurs,
or both, causing patchy, shallow, longitudinal bowel sound absent.
mucosal ulcers; possible correlation with autoim- b. Increased peristalsis above level of obstruction
mune disease and adenocarcinoma of the bowel. in attempt to move intestinal contents
Small, scattered, shallow crater-like areas cause scar- through the obstructed area.
ring and stiffness of the bowel bowel becomes c. Obstipation (no passage of gas or stool
narrow obstruction, then pain, nausea, and through obstructed portion of bowel; no
vomiting. reabsorption of fluids).
B. Risk factors: d. Distention.
1. Age: 15 to 20, 55 to 60 years. e. Vomiting:
2. Whites, especially Jews. (1) Proximal loop obstruction: profuse nonfecal
3. Familial predisposition. vomiting.

ADULT
4. Possible virus involvement. (2) Distal loop obstruction: less frequent
5. Possible psychosomatic involvement. fecal-type vomiting.
6. Possible hormonal or dietary influences. f. Urinary output: decreased.
C. Assessment: g. Temperature: elevated; pulse: tachycardia; BP:
1. Subjective data: hypotension shock if untreated.
a. Abdominal pain. h. Dehydration, hemoconcentration,
b. Anorexia. hypovolemia.
c. Nausea. i. Laboratory data:
d. Malaise. (1) Leukocytosis.
e. History of isolated, intermittent, or recurrent (2) Decreased: sodium (<138 mEq/L), potassi-
attacks. um (<3.5 mEq/L).
2. Objective data: (3) Increased: bicarbonate (>26 mEq/L), BUN
a. Diarrhea. (>18 mg/dL).
b. Weight loss, vomiting. (4) pH: If obstruction is at gastric outlet, pH
c. Fever, signs of infection. will be elevated, indicating metabolic alka-
d. Fluid/electrolyte imbalances. losis; if obstruction is distal duodenal or
e. Malnutrition, malabsorption. proximal jejunal, the pH will drop and
f. Occult blood in feces. metabolic acidosis occurs.
2164_Ch06_347-578 29/03/12 12:30 PM Page 454

454 chapter 6 Physiological Integrity

C. Analysis/nursing diagnosis: b. Observe for bowel sounds, flatus (tape intes-


1. Fluid volume deficit related to vomiting. tinal tube to face once peristalsis begins).
2. Pain related to increased peristalsis above the c. Skin and frequent mouth care.
level of obstruction. 3. Goal: prevent respiratory complications.
3. Altered nutrition, less than body requirements, a. Encourage coughing and deep breathing.
related to vomiting. b. Semi-Fowlers or position of comfort.
4. Risk for trauma related to potential perforation. 4. Goal: postoperative nursing care (if treated
D. Nursing care plan/implementation: surgically) (see III. POSTOPERATIVE
1. Goal: obtain and maintain fluid balance. EXPERIENCE, p. 369).
a. Nursing care of client with nasogastric tube E. Evaluation/outcome criteria:
(see Table 11.5, pp. 841842). 1. Fluid balance obtained and maintained.
(1) Miller-Abbott tube: dual lumen, balloon 2. Shock prevented.
inflated with air after insertion. Caution: 3. Obstruction resolved.
do not tape tube to face until tube reaches 4. Pain decreased.
point of obstruction. 5. Fluids tolerated by mouth.
(2)Cantor tube: has mercury in distal sac, 6. Complications such as perforation and peritoni-
which helps move tube to point of tis avoided.
obstruction. Caution: do not tape tube XX. FECAL DIVERSIONstomas: performed because
to face until tube reaches point of of disease or trauma; may be temporary or permanent.
obstruction.
A. Types (Table 6.25).
b. Nothing by mouth, IV therapy, strict I&O.
1. Temporaryfecal stream rerouted to allow GI
c. Take daily weights (early morning), monitor
tract to heal or to provide outlet for stool when
CVP for hydration status.
obstructed.
d. Monitor abdominal girth for signs of
2. Permanentintestine cannot be reconnected.
distention and urinary output for signs of
Rectum and anal sphincter removed (abdominal
retention or shock.
perineal resection). Often performed for cancer
2. Goal: relieve pain and nausea.
of the colon and/or rectum.
a. Medications as ordered:
3. Continent ileostomypouch is created inside the
(1) Analgesics, antiemetics.
wall of the intestine. The pouch serves as a reser-
(2)If problem is paralytic: medical
voir similar to a rectum. The pouch is emptied
treatment includes neostigmine to
on a regular basis with a small tube.
stimulate peristalsis.

Table 6.25
Comparison of Ileostomy and Colostomy
ADULT

Ileostomy Colostomy
Procedure Surgical formation of a fistula, or Surgical formation of an artificial opening between the surface
stoma, between the abdominal wall of the abdominal wall and colon
and ileum: continent ileostomy Single barrelonly one loop of bowel is opened to the
(Kock pouch) may be constructed abdominal surface
Double barreltwo loops of bowel, a proximal and distal por-
tion, are open to the abdominal wall; feces will be expelled
from the proximal loop, mucus will be expelled from the distal
loop; client may expel some excreta from rectum as well
Reasons performed Unresponsive ulcerative colitis: Single barrel: colon or rectal cancer
complications of ulcerative colitis Double barrel: relieve obstruction
(e.g., hemorrhage, carcinoma
[suspected])
Results Permanent stoma Single barrel: permanent stoma
Double barrel: temporary stoma
Discharge Green liquid, nonodorous Consistency of feces dependent on diet and portion of the
bowel used as the stoma; from brown odorous liquid to
normal stool consistency
Nursing care See FECAL DIVERSION, See Tables 11.6, Emptying Colostomy Appliance, and 11.7,
pp. 454456 Changing Colostomy Appliance
2164_Ch06_347-578 29/03/12 12:30 PM Page 455

Gastrointestinal System 455


4. Ileoanal anastomosis (J pouch, S reservoir, or c. Goal: health teaching.
ileoperistaltic reservoir)the large intestine is (1) Determine knowledge of surgery and
removed and the small intestine is inserted into potential impact.
the rectum and attached just above the anus. (2) Begin teaching regarding ostomy.
The muscles of the rectum remain intact and the 2. Postoperative period:
normal route of stool elimination is maintained. a. Goal: maintain fluid balance.
B. Analysis/nursing diagnosis: (1) Monitor I&O because large volume of
1. Bowel incontinence related to lack of sphincter in fluid is lost through stoma.
newly formed stoma. (2) Administer IV fluids as ordered.
2. Altered health maintenance related to knowledge (3) Monitor losses through NG tube.
of ostomy care. b. Goal: prevent other postoperative complications.
3. Body image disturbance related to stoma. (1) Monitor for signs of intestinal obstruction.
4. Fear related to medical condition requiring stoma. (2) Maintain sterility when changing dress-
5. Fluid volume deficit related to increased output ings; avoid fecal contamination of incision.
through stoma. (3) Observe appearance of stoma: rosy pink,
C. Nursing care plan/implementation: raised (Fig. 6.10).
1. Preoperative period: c. Goal: initiate ostomy care.
a. Goal: prepare bowel for surgery. (1) Protect skin around stoma: use commercial
(1) Administer neomycin as ordered to reduce preparation to toughen skin and use pro-
colonic bacteria. tective barrier wafer (Stomahesive) or paste
(2) Administer cathartics, enemas as ordered (Karaya or substitute) to keep drainage
to cleanse the bowel of feces. (which can cause excoriation) off the skin.
(3) Administer low-residue or liquid diet as (2) Keep skin around stoma clean and dry;
ordered. empty appliance frequently. Check for
b. Goal: relieve anxiety and assist in adjustment to drainage in appliance at least twice during
surgery. each shift. If drainage present (diarrhea-
(1) Provide accurate, brief, and reassuring type stool):
explanations of procedures; allow time for (a) Unclip bottom of bag.
questions. (b) Drain into bedpan.
(2) Referral: have enterostomal nurse visit to (c) Use a squeeze-type bottle filled with
discuss ostomy management and place- warm water to rinse inside of appliance.
ment of stoma appliance. (d) Clean clamp, if soiled.
(3) Referral: offer opportunity for a visit with (e) Put a few drops of deodorant in appli-
an Ostomy Association Visitor. ance if not odor-proof.

ADULT
SEMIMUSHY
SEMIFLUID FECES
MUSHY FECES
FECES

COLOSTOMY IN
TRANSVERSE COLON
COLOSTOMY IN
ASCENDING COLON
COLOSTOMY IN
FLUID DESCENDING COLON
FECES

SOLID
FECES

SOLID FECES
Figure 6.10 Colostomy sites. (From Venes, D [ed]:
Tabers Cyclopedic Medical Dictionary, ed 21. FA Davis,
Philadelphia, 2009.)
2164_Ch06_347-578 29/03/12 12:30 PM Page 456

456 chapter 6 Physiological Integrity

(f ) Fasten bottom of appliance securely (4) Use deodorizing drops in appliance and
(fold bag over clamp two or three times provide adequate room ventilation to
before closing). decrease odors. Caution: deodorizing drops
(g) Check for leakage under appliance must be safe for mucous membranes. No
every 2 to 4 hours. pinholes in pouch.
(3) Change appliance when drainage leaks (5) If continent ileostomy (a Kock pouch) has
around seal, or approximately every 2 to been constructed, the client does not have
3 days. Initially, size of stoma will be large to wear an external pouch. The stool is
due to edema. Pouch opening should be stored intra-abdominally. The client drains
slightly larger than stoma so it will not the pouch several times daily, when there
constrict. Stoma will need to be measured is a feeling of fullness, using a catheter.
for each change until swelling subsides to The stoma is flat and on the right side of
ensure appropriate fit. the abdomen.
(a) Gather equipment: gloves, skin prep d. Goal: promote psychological comfort.
packet, colostomy appliance measured (1) Support client and familyaccept feelings
to fit stoma properly (use stoma meas- and behavior.
uring guide), skin barrier, warm water (2) Recognize that such a procedure may initi-
and soap, face cloth/towel, plastic bag ate the grieving process.
for disposal of old equipment. e. Goal: health teaching.
(b) Remove old appliance carefully, pulling (1) Self-care management skills related to
from area with least drainage to area ostomy appliance, skin care, and irrigation,
with most drainage. if indicated (Table 6.26).
(c) Wash skin area (not stoma) with soap (2) Diet: adjustments to control character of
and water. Be careful not to: irritate skin, feces; avoid foods that increase flatulence.
put soap on stoma, irritate stoma; do (3) Signs of complications of infection,
not put anything dry onto stoma. obstruction, or electrolyte imbalance.
Remember: bowel is very fragile; working (4) Community referral for follow-up care.
near bowel increases peristalsis so that D. Evaluation/outcome criteria:
feces and flatulence may be expelled. 1. Demonstrates self-care skill for independent
(d) Observe skin area for potential living.
breakdown. 2. Makes dietary adjustments.
(e) Use packet of skin prep on the skin 3. Ostomy functions well.
around the stoma. Do not put this solu- 4. Adjusts to alteration in bowel elimination pattern.
tion onto stoma, because it will cause XXI. HEMORRHOIDS: enlarged vein/veins in mucous
irritation. Allow skin prep solution to membrane of rectum. Hemorrhoids can be internal or
dry on skin before applying colostomy external. Bleeding internal hemorrhoids can be painful
appliance. and are best treated by: rubber band ligation, injection
ADULT

(f ) Apply skin barrier you have measured


and cut to size.
(g) Put appliance on so that bottom of Table 6.26
appliance is easily accessible for empty-
ing (e.g., if client is out of bed most of Colostomy Irrigation*
the time, put the bottom facing the 1. Assemble all equipment for irrigation and appliance
feet; if client is in bed most of the change.
time, have bottom face the side). 2. Remove and discard old pouch.
Picture-frame the adhesive portion 3. Clean the peristomal skin.
of the appliance with 1-inch tape. 4. Apply the irrigating sleeve; place in toilet or bedpan.
5. Fill container with 5001,000 mL of warm water, never
(h)Put a few drops of deodorant in more than 1,000 mL. Clear air from tubing. Insert lubricated
appliance if not odor-proof. tubing 24 inches into stoma. Do not force. Hold container
(i) Use clamp to fasten bottom of about 18 inches above stoma. Infuse gently over 710 min.
appliance. 6. Allow stool to empty into toilet. Evacuation usually occurs
(j) Talk to client (or communicate in in 2025 min.
7. If no return after irrigation, ambulate, gently massage
best way possible during and after abdomen, or give client a warm drink.
procedure). This is a very difficult 8. Once complete, remove the sleeve, and follow guidelines
alteration in body image. for applying appliance.
(k) Use good hand-washing technique. *Colostomy irrigation is seldom used with newer colostomies.
2164_Ch06_347-578 29/03/12 12:30 PM Page 457

Genitourinary System 457


sclerotherapy, infrared coagulation, or surgery (scalpel, e. Monitor for: syncope/vertigo during first
cautery, or laser). Laser surgery is usually done on postoperative bowel movement.
an outpatient basis and causes minimal discomfort. f. Diet:
High-fiber diets can minimize constipation and pre- (1) Low residue (postoperative)until healing
vent hemorrhoids. has begun.
A. Pathophysiology: venous congestion and interfer- (2) High fiber to prevent constipation after
ence with venous return from hemorrhoidal veins healing.
increase in pelvic pressure, swelling, and distortion. g. Increase fluid intake.
B. Risk factors: 3. Goal: health teachingmethods to avoid
1. Straining to expel constipated stool. constipation.
2. Pregnancy. F. Evaluation/outcome criteria:
3. Intra-abdominal or pelvic masses. 1. No complications.
4. Interference with portal circulation. 2. Client has bowel movement.
5. Prolonged standing or sitting. 3. Incorporates knowledge of correct foods into
6. History of low-fiber, high-carbohydrate diet, lifestyle.
which contributes to constipation.
7. Family history of hemorrhoids. GENITOURINARY
8. Enlarged prostate. SYSTEM
C. Assessment:
1. Subjective data: discomfort, anal pruritus, pain. I. PYELONEPHRITIS (PN): acute or chronic inflam-
2. Objective data: mation due to bacterial infection of the parenchyma
a. Bleeding, especially on defecation. and renal pelvis; 95% of cases caused by gram-negative
b. Narrowing of stool. enteric bacilli (Escherichia coli); occurs more frequently
c. Grapelike clusters around anus (pink, red, in women.
or blue). A. Pathophysiology: inflammation of renal medulla
d. Diagnostic test: or lining of the renal pelvis nephron destruc-
(1) Visualization for external hemorrhoids. tion; hypertrophy of nephrons needed to maintain
(2) Digital examination or proctoscopy for urine output impaired sodium reabsorption (salt
internal hemorrhoids. wasting); inability to concentrate urine; progressive
D. Analysis/nursing diagnosis: renal failure; hypertension (two thirds of all cases).
1. Pain related to defecation. B. Risk factors:
2. Constipation related to dietary habits and pain at 1. Urinary obstruction (tumors, prostate).
time of defecation. 2. Cystitis.
3. Knowledge deficit (learning need) related to foods 3. Neurogenic bladder.
to prevent constipation. 4. Pregnancy.
E. Nursing care plan/implementation: 5. Catheterization, cystoscopy.
1. Goal: reduce anal discomfort. C. Assessment:

ADULT
a. Sitz baths, as ordered; perineal care to prevent 1. Subjective data:
infection. a. Pain: flankone or both sides; back; dysuria;
b. Hot or cold compresses as ordered to reduce headache.
inflammation and pruritus. b. Loss of appetite; weight loss.
c. Topical medications as ordered: c. Night sweats; chills.
(1) Anti-inflammatory: hydrocortisone cream d. Urination: frequency, urgency.
(Anusol). 2. Objective data:
(2) Astringents: witch hazelimpregnated pads. a. Fever; shaking chills.
(3) Topical anesthetics: pramoxine (Procto- b. Laboratory data:
Foam); dibucaine (Nupercainal). (1) Bloodpolymorphonuclear leukocytosis
d. Bulk laxatives: psyllium (Metamucil), Konsyl, greater than 11,000/mm3.
polycarbophil (FiberCon). (2) Urineleukocytosis, hematuria, white
2. Goal: prevent complications related to surgery. blood cell casts, proteinuria (<3 gm in
a. Encourage postoperative ambulation. 24 hours), positive cultures; specific
b. Pain relief until packing removed. gravitynormal or increased with acute
c. Monitor for: bleeding, infection, pulmonary PN, decreased with chronic PN; cloudy;
emboli, phlebitis. foul smelling.
d. Facilitate bowel evacuation: stool softeners, c. Intravenous pyelogram (IVP)may manifest
laxatives, suppositories, oil enemas as ordered. structural changes.
2164_Ch06_347-578 29/03/12 12:30 PM Page 458

458 chapter 6 Physiological Integrity

D. Analysis/nursing diagnosis: II. ACUTE GLOMERULONEPHRITIS (see


1. Altered urinary elimination related to kidney Chapter 5, pp. 292294).
disease. III. ACUTE RENAL FAILURE (ARF): broadly defined
2. Pain related to dysuria and kidney damage. as rapid onset of oliguria accompanied by a rising
3. Hyperthermia related to inflammation. BUN and serum creatinine; usually reversible.
4. Risk for fluid volume excess related to renal
A. Pathophysiology: acute renal ischemia tubular
failure.
necrosis decreased urine output. Oliguric phase
E. Nursing care plan/implementation:
(<400 mL/24 hr)waste products are retained
1. Goal: combat infection, prevent recurrence,
metabolic acidosis water and electrolyte imbal-
alleviate symptoms.
ances anemia. Recovery phasediuresis dilute
a. Medications:
urine rapid depletion of sodium, chloride, and
(1) Antibiotics, urinary antiseptics, and/or
water dehydration.
sulfonamides appropriate for causative
B. Types and risk factors:
organism; also reduce pain.
1. Prerenaldue to factors outside of kidney; usually
(2) Analgesics for painphenazopyridine
circulatory collapsecardiovascular disorders,
(Pyridium); stronger if calculi present.
hypovolemia, peripheral vasodilation, renovascu-
(3) Antipyretics for feveracetaminophen
lar obstruction, severe vasoconstriction.
(Tylenol).
2. Intrarenalparenchymal disease from ischemia
b. Fluids: 1,500 to 2,000 mL/day to flush
or nephrotoxic damage; nephrotoxic agents
kidneys, relieve dysuria, reduce fever,
poisons, such as lead (carbon tetrachloride);
prevent dehydration.
heavy metals (mercury); antibiotics (gentamicin);
c. Observe hydration status: I&O (output
incompatible blood transfusion; alcohol
minimum 1,500 mL/24 hr); daily weight;
myopathies; obstetric complications; acute
urinecheck each voiding for protein, blood,
rena1 diseaseacute glomerulonephritis, acute
specific gravity; vital signs q4h to monitor for
pyelonephritis.
hypertension, tachycardia; skin turgor.
3. Postrenalobstruction in collecting system: renal
d. Hygiene: meticulous perineal care; cleanse
or bladder calculi; tumors of bladder, prostate, or
with soap and water; antibiotic ointment may
renal pelvis; gynecological or urological surgery
be used around urinary meatus with retention
in which ureters are accidentally ligated.
catheter.
C. Assessment:
e. Cooling measures: tepid sponging.
1. Subjective data:
f. Diet: sufficient calories and protein to prevent
a. Sudden decrease or cessation of urine output
malnutrition; sodium supplement as ordered.
(<400 mL/24 hr).
Acid-ash to prevent renal calculus.
b. Anorexia, nausea, vomiting from azotemia.
2. Goal: promote physical and emotional rest.
c. Sudden weight gain from fluid accumulation.
a. Activity: bedrest or as tolerateddepends on
d. Headache.
whether anemia or fever is present; encourage
2. Objective data:
ADULT

activities of daily living as tolerated.


a. Vital signs (vary according to cause and
b. Emotional support: encourage expression of
severity):
fears (possible renal failure, dialysis); provide
(1) BPusually elevated.
diversional activities; include family in care;
(2) Pulsetachycardia, irregularities.
answer questions.
(3) Respirationsincreased rate, depth, crackles.
3. Goal: health teaching.
b. Neurological: decreasing mentation, unrespon-
a. Medications: take regularly to maintain blood
sive to verbal or painful stimuli, psychoses,
level; side effects.
convulsions.
b. Personal care: perineal hygiene; avoid urethral
c. Halitosis; cracked mucous membranes; uremic
contamination (by wiping perineum front to
odor.
back); avoid tub baths.
d. Skin: dry, rashes, purpura, itchy, pale.
c. Possible recurrence with pregnancy.
e. Laboratory data:
d. Monitoring daily weight.
(1) Blood: increasedpotassium, BUN,
F. Evaluation/outcome criteria:
creatinine, WBC count; decreasedpH,
1. Normal renal function (minimum 1,500 mL
bicarbonate, hematocrit, hemoglobin.
urine/24 hr).
(2) Urine: oliguric renal failuredecreased
2. Blood pressure within normal range.
volume; specific gravity fixed or ;
3. No recurrence of symptoms.
increasedprotein, casts, red and white
4. Laboratory findings within normal limits.
2164_Ch06_347-578 29/03/12 12:30 PM Page 459

Genitourinary System 459


blood cells, sodium. Non-oliguric renal 4. Goal: health teaching.
failureup to 2 L/day, specific gravity, a. Preparation for dialysis (indications: uremia,
dilute, isomolar. uncontrolled hyperkalemia, or acidosis).
D. Analysis/nursing diagnosis: Continuous renal replacement therapy
1. Altered urinary elimination related to kidney (CRRT) may be used in ARF with fluid
malfunction. overload or rapidly developing azotemia and
2. Fluid volume excess related to decreased urine metabolic acidosis. Continuous ultrafiltration
output. (8 to 24 hours) of extracellular fluid and
3. Altered nutrition, less than body requirements, uremic toxins. Clients BP powers system.
related to anorexia. Arterial and venous access required.
4. Altered oral mucous membrane related to stomatitis. b. Dietary restrictions: low sodium, fluid restriction.
5. Altered thought processes related to uremia. c. Disease process; treatment regimen.
E. Nursing care plan/implementation: F. Evaluation/outcome criteria:
1. Goal: maintain fluid and electrolyte balance and 1. Return of kidney functionnormal creatinine
nutrition. level (<1.5 mg/dL), urine output.
a. Monitor: daily weight (should not vary more 2. Resumes normal life pattern (about 3 months
than 1 lb); vital signsinclude CVP; blood after onset).
chemistries (BUN 6 to 20 mg/dL; creatinine IV. CHRONIC RENAL FAILURE: as a result of
0.6 to 1.5 mg/dL). progressive destruction of kidney tissue, the kidneys
b. Fluids: IV as ordered; blood: plasma, packed are no longer able to maintain their homeostatic
cells, electrolyte solutions to replace losses; functions; considered irreversible.
restricted to 400 mL/24 hr if hypertension
A. Pathophysiology: destruction of glomeruli
present or during oliguric phase to prevent
reduced glomerular filtration rate retention of
fluid overload.
metabolic waste products; decreased urine output;
c. Diet, as tolerated: high carbohydrate, low pro-
severe fluid, electrolyte, acid-base imbalances
tein, may be low potassium and low sodium;
uremia. Clinical picture includes:
hypertonic glucose (total parenteral nutrition
1. Ammonia in skin and alimentary tract by
[TPN]) if oral feedings not tolerated; intra-
bacterial interaction with urea inflamma-
venous L-amino acids and glucose.
tion of mucous membranes.
d. Control hyperkalemia: infusions of hypertonic
2. Retention of phosphate decreased serum
glucose and insulin to force potassium into
calcium muscle spasms, tetany, and increased
cells; calcium gluconate (IV) to reduce
+ parathormone release demineralization of
myocardial irritability from K ; sodium bicar-
bone.
bonate (IV) to correct acidosis; polystyrene
3. Failure of tubular mechanisms to regulate
sodium sulfonate (Kayexalate) or other
blood bicarbonate metabolic acidosis
exchange resins, orally or rectally (enema), to
+ hyperventilation.
remove excess K ; continuous renal replace-
4. Urea osmotic diuresis flushing effect on

ADULT
ment therapy, peritoneal or hemodialysis.
tubules decreased reabsorption of sodium
e. Medicationsdiuretics (mannitol, furosemide
sodium depletion.
[Lasix]) to increase renal blood flow and
5. Waste product retention depressed bone
diuresis.
marrow function decreased circulating RBCs
2. Goal: use assessment and comfort measures to
renal tissue hypoxia decreased erythropoi-
reduce occurrence of complications.
etin production further depression of bone
a. Respiratory: monitor rate, depth, breath
marrow anemia.
sounds, arterial blood gases; encourage deep
B. Risk factors:
breathing, coughing, turning; use incentive
1. Diabetic retinopathy.
spirometer or nebulizer as indicated.
2. Chronic glomerulonephritis.
b. Frequent oral care to prevent stomatitis.
3. Chronic urinary obstruction, ureteral stricture,
c. Observe for signs of:
calculi, neoplasms.
(1) Infectionelevated temperature, localized
4. Chronic pyelonephritis.
redness, swelling, heat, or drainage.
5. Hypertensive nephrosclerosis.
(2) Bleedingstools, gums, venipuncture sites.
6. Congenital or acquired renal artery stenosis.
3. Goal: maintain continual emotional support.
7. Systemic lupus erythematosus.
a. Same caregivers, consistency in procedures.
C. Assessment:
b. Give opportunities to express concerns, fears.
1. Subjective data: excessive fatigue, weakness.
c. Allow family interactions.
2164_Ch06_347-578 29/03/12 12:30 PM Page 460

460 chapter 6 Physiological Integrity

2. Objective data: b. Hygiene: mouth care to prevent stomatitis


a. Skin: bronze colored, uremic frost. and reduce discomfort from mouth ulcers;
b. Ammonia breath. perineal care.
c. See also III. ACUTE RENAL FAILURE, c. Skin care: soothing lotions to reduce pruritus.
p. 458; symptoms gradual in onset. d. Encourage communication of concerns.
D. Analysis/nursing diagnosis: 3. Goal: health teaching.
1. In addition to the following, see III. ACUTE a. Dietary restrictions: no added salt when
RENAL FAILURE, p. 459. cooking; change cooking water in vegetables
2. Fatigue related to severe anemia. during process to decrease potassium; read
+
3. Risk for impaired skin integrity related to pruritus. food labels to avoid Na+ and K ; protein
4. Ineffective individual coping related to chronic illness. restriction according to BUN/creatinine
5. Body image disturbance related to need for dialysis. ratio (10:1).
6. Noncompliance related to denial of illness. b. Importance of daily weight: same scale, time,
E. Nursing care plan/implementation: clothing.
1. Goal: maintain fluid, electrolyte balance and c. Prepare for dialysis; transplantation.
nutrition (see also III. ACUTE RENAL F. Evaluation/outcome criteria:
FAILURE, p. 459). 1. Acceptance of chronic illness (no indication
a. Diet: low sodium; foods high in: calcium, of indiscretions, destructive behavior, suicidal
vitamin B complex, vitamins C and D, and tendency).
iron (to reduce edema, replace deficits, and 2. Compliance with dietary restrictionno signs
promote absorption of nutrients). of protein excess (e.g., nausea, vomiting) or fluid
b. Medications: given to control BP, regulate sodium excess (e.g., edema, weight gain).
electrolytes, control fluid volume; supplemen- V. DIALYSIS: diffusion of solute through a semiperme-
tal vitamins if deficient; electrolyte modifier able membrane that separates two solutions; direction
(aluminum hydroxide [Alu-Cap, Amphojel]), of diffusion depends on concentration of solute in
calcium carbonate to bind phosphate. each solution; rate and efficiency depend on concen-
c. I&O; intake should be equivalent to previous tration gradient, temperature of solution, pore size of
daily output to prevent fluid retention. membrane, and molecular size; two methods available
2. Goal: employ comfort measures that reduce distress (Table 6.27).
and support physical function.
A. Indications: acute poisonings; acute or chronic renal
a. Activity: bedrest; facilitate ventilation; turn,
failure; hepatic coma; metabolic acidosis; extensive
cough, deep breathe q2h; ROMactive and
burns with azotemia.
passive, to prevent thrombi.

Table 6.27
Comparison of Hemodialysis and Peritoneal Dialysis
ADULT

Hemodialysis Peritoneal Dialysis


Process Rapiduses either external AV Intermittentup to 36 hr for hospitalized clients; outpatient
shunt (acute renal failure) or internal 48 hr, 56 times per week; dwell time 3045 min for manual
AV fistula (chronic renal failure); typi- dialysis or 1020 min for automatic cycler; either rigid stylet
cal treatment is 34 hr, 3 days/wk; catheter or surgically inserted soft catheter; advantage for
also used for barbiturate overdoses clients who cannot tolerate rapid fluid and electrolyte changes
to remove toxic agent quickly Continuoustwo methods: (1) four cycles in 24 hr; dwell
time is 45 hr during the day and 812 hr overnight; no need
for machinery, electricity, or water source; surgically inserted
soft catheter; closely resembles normal renal function;
(2) automated cycler infuses and removes dialysate; generally
done while client sleeps; built-in alarms for client safety.
Vascular access Required Not necessary; therefore suitable for clients with vascular
problems
Heparinization Required: systemic or regional Little or no heparin necessary; therefore suitable for clients
with bleeding problems
Complications (other Dialysis dysequilibrium syndrome Peritonitis
than fluid and electrolyte (preventable) Hypoalbuminemia
imbalances, which are Mechanical dysfunctions of dialyzer Bowel or bladder perforation
common to all) Plugged or dislodged catheter
2164_Ch06_347-578 29/03/12 12:30 PM Page 461

Genitourinary System 461


B. Goals: b. Arteriovenous fistulas or graft (Fig. 6.12).
1. Reduce level of nitrogenous waste. (1)Large artery and vein are sewn together
2. Correct acidosis, reverse electrolyte imbalances, (anastomosed) below the surface of the
remove excess fluid. skin (fistula) or subcutaneous graft using
C. Hemodialysis: circulation of clients blood through a the saphenous vein, synthetic prosthesis,
compartment formed of a semipermeable mem- or bovine xenograft to connect artery
brane (polysulfone, polyacrylonitrile) surrounded and vein.
by dialysate fluid. (2) Purpose is to create one blood vessel for
1. Types of venous access for hemodialysis: withdrawing and returning blood.
a. External shunt (Fig. 6.11). (3) Advantages: greater activity range than
(1) Cannula is placed in a large vein and a AV shunt and no protective asepsis.
large artery that approximate each other. (4) Disadvantage: necessity of two venipunc-
(2) External shunts, which provide easy and tures with each dialysis.
painless access to bloodstream, are prone c. Vein catheterization.
to infection and clotting and cause erosion (1) Femoral or subclavian vein access is
of the skin around the insertion area. immediate.
(a) Daily cleansing and application of a (2) May be short- or long-term duration.
sterile dressing. 2. Complications during hemodialysis:
(b) Prevention of physical trauma and a. Dysequilibrium syndromerapid removal
avoidance of some activities, such as of urea from blood reverse osmosis, with
swimming. water moving into brain cells cerebral
edema possible headache, nausea, vomit-
ing, confusion, and convulsions; usually
occurs with initial dialysis treatments; shorter
dialysis time and slower rate minimizes.
b. Hypotensionresults from excessive ultrafiltra-
tion or excessive antihypertensive medications.
c. Hypertensionresults from volume overload
(water and/or sodium), causing dysequilibrium
syndrome or anxiety.
d. Transfusion reactions (see Chapter 8, Table 8.2).
e. Arrhythmiasdue to hypotension, fluid over-
load, or rapid removal of potassium.
f. Psychological problems:
(1) Clients react in varying ways to depend-
ence on hemodialysis.
(2) Nurse needs to identify client reactions

ADULT
A and defense mechanisms and to employ

B
Figure 6.11 AV shunt (cannulae). Figure 6.12 AV fistula.
2164_Ch06_347-578 29/03/12 12:30 PM Page 462

462 chapter 6 Physiological Integrity

supportive behaviors (e.g., include client in peritoneum; concealed in cloth pouch, strapped
care; continual repetition and reinforce- to the body during dwell time; client can move
ment); do not interpret clients behavior about doing usual activities.
for example, do not say, Youre being F. Continuous cycling peritoneal dialysis (CCPD): same
hostile or Youre acting like a child; principles as CAPD, except uses an automated
answer questions honestly regarding system to infuse and remove dialysate; reduces
quality and length of life with dialysis nursing care needs with clients who are hospital-
and/or transplantation; encourage inde- ized. Cumbersome equipment inhibits nighttime
pendence as much as possible. mobility. Procedure:
3. Commonly used medications: 1. Long exchanges without an automated cycler
a. Antihypertensives (ACE inhibitors, beta during the day, and short exchanges with a cycler
blockers, diuretics). at night.
b. Phosphorus binders (calcium acetate, calcium 2. Dwell time6 to 8 hours while sleeping.
carbonate, Renagel). 3. Automatic alarms to prevent malfunction with
c. B-complex vitamins with vitamin C and home use.
folic acid. VI. KIDNEY TRANSPLANTATION: placement of a
d. Synthetic erythropoietin (Aranesp, Epogen). donor kidney (from sibling, parent, cadaver) into the
e. Iron. iliac fossa of a recipient and the anastomosis of its
f. Activated vitamin D. ureter to the bladder of the recipient; indicated in
D. Intermittent peritoneal dialysis: involves introduction end-stage renal disease.
of a dialysate solution into the abdomen, where
A. Criteria for recipient: irreversible kidney function;
the peritoneum acts as the semipermeable mem-
under 70 years of age; patent and functional lower
brane between the solution and blood in abdomi-
urinary tract; and good surgical risk, free of serious
nal vessels. Procedure:
cardiovascular complications. Contraindicated in
1. Area around umbilicus is prepared and anes-
those with another life-threatening condition.
thetized with local anesthetic, and a catheter is
B. Donor selection:
inserted into the peritoneal cavity through a
1. Sibling or parentsurvival rate of kidney is
trocar; the catheter is then sutured into place
greater; preferred for transplantation.
to prevent displacement.
2. Cadavergreater rate of rejection following
2. Warmed dialysate is then allowed to flow into the
transplantation, although majority of transplan-
peritoneal cavity. Inflow time: 5 to 10 minutes;
tations are with cadaver kidneys.
2 L of solution are used in each cycle in the adult;
+ C. Bilateral nephrectomy: necessary for clients with
solutions contain glucose, Na+, Ca++, Mg++, K ,
rapidly progressive glomerulonephritis, malignant
Cl , and lactate or acetate.
hypertension, or chronic kidney infections;
3. When solution bottle is empty, dwell time
prevents complications in transplanted kidney
(exchange time) begins. Dwell time: 10 to
(see VII. NEPHRECTOMY for nursing care,
20 minutes; processes of diffusion, osmosis,
p. 464).
ADULT

and filtration begin to move waste products


D. Analysis/nursing diagnosis:
from bloodstream into peritoneal cavity.
1. Altered urinary elimination related to kidney
4. Draining of the dialysate begins with the
failure.
unclamping of the outflow clamp. Outflow time:
2. Fear related to potential transplant rejection.
usually 10 minutes; returns less than 2 L usually
3. Risk for infection related to immunosuppression.
result from incomplete peritoneal emptying;
4. Body image disturbance related to
turn side to side to increase return; multiple
immunosuppression.
exchanges in 24 hours depending on client
E. Nursing care plan/implementation:
need.
1. Preoperative:
E. Continuous ambulatory peritoneal dialysis (CAPD):
a. Goal: promote physical and emotional
functions on the same principles as peritoneal
adjustment.
dialysis, yet allows greater freedom and independ-
(1) Informed consent.
ence for clients on dialysis. Procedure:
(2) Laboratory work completed
1. Dialysis solution is infused into peritoneum
histocompatibility, CBC, urinalysis,
three times daily and once before bedtime.
blood type and crossmatch.
2. Dwell time4 hours for each daily exchange,
(3) Skin preparation.
and overnight for the fourth (8 to 12 hours).
b. Goal: encourage expression of feelings: origin of
3. Indwelling peritoneal catheter is connected to
donor, fear of complications, rejection.
solution bag at all timesserves to fill and drain
2164_Ch06_347-578 29/03/12 12:30 PM Page 463

Genitourinary System 463


c. Goal: minimize risk of organ rejection: give (b) Objective data:
medications: begin immunosuppression (i) Laboratory data: Urine:
(azathioprine, corticosteroids, cyclosporine); decreasedoutput, creatinine
antibiotics if ordered. clearance, sodium; increased
d. Goal: health teaching. protein. Blood: increasedBUN,
(1) Nature of surgery; placement of kidney. creatinine.
(2) Postoperative expectations: deep breathing, (ii) Rapid weight gain; more than
coughing, turning, early ambulation; 3 lb/day.
reverse isolation. (iii) Vital signs: BP, temperature
(3) Medications: immunosuppressive therapy: elevated.
purpose, effect. c. Goal: maintain immunosuppressive therapy.
2. Postoperative: (1) Azathioprine (Imuran)an antimetabolite
a. Goal: promote uncomplicated recovery of recipient. that interferes with cellular division. Side
(1) Vital signs; CVP; I&Ourine output effects:
usually immediate with living donor; with (a) Gastrointestinal bleeding (give PO
cadaver kidney may not work for a week or form with food).
more and dialysis will be needed within 24 (b) Bone marrow depression; leukopenia;
to 48 hours. Report less than 100 mL/hr anemia.
immediately. (c) Development of malignant neoplasms.
(2) Isolation: strict reverse isolation with (d) Infection.
immunosuppression; wear face mask (e) Liver damage.
when out of room. (2)Prednisonebelieved to affect lympho-
(3) Position: back to nonoperative side; semi- cyte production by inhibiting nucleic
Fowlers to promote gas exchange. acid synthesis: anti-inflammatory action
(4) Indwelling catheter care: strict asepsis; helps prevent tissue damage if rejection
characteristics of urinereport gross occurs. Side effects:
hematuria, heavy sediment; clots; perineal (a) Stress ulcer with bleeding (give with
care; bladder spasms may occur after food).
removal of catheter. (b) Decreased glucose tolerance
(5) Activity: ambulate 24 hours after surgery; (hyperglycemia).
avoid prolonged sitting. (c) Muscle weakness.
(6) Weigh daily. (d) Osteoporosis.
(7) Medications: immunosuppressives; analgesics (e) Moon facies.
as ordered (pain decreases significantly (f ) Acne and striae.
after 24 hours). (g) Depression and hallucinations.
(8) Drains: irrigate only on physician order; (3) Cyclosporine (Neoral, Sandimmune)
meticulous catheter care. polypeptide antibiotic used to prevent

ADULT
(9) Diet: regular after return of bowel sounds; rejection of kidney, liver, or heart allografts;
liberal amounts of protein; restrict fluids, PO dose given with room-temperature
sodium, potassium only if oliguric. chocolate milk or orange juice in a glass
b. Goal: observe for signs of rejectionmost dispenser. Side effects:
dangerous complication. (a) Nephrotoxicity (increased BUN,
(1) Three classifications: creatinine).
(a) Hyperacuteoccurs within 5 to (b) Hypertension.
10 minutes up to 48 hours after (c) Tremor.
transplantation (rare). (d) Hirsutism, gingival hyperplasia.
(b) Acutemost common 7 to 14 days; (e) GInausea, vomiting, anorexia,
varies depending on living (1 week to diarrhea, abdominal pain.
6 months) or cadaver (1 week to (f ) Infectionspneumonia, septicemia,
2 years) donor. abscesses, wound.
(c) Chronicoccurs several months to (4) Additional drugs may include cyclophos-
years. phamide (Cytoxan), antithymocyte
(2) Assessment: globulin (ATG), antilymphocyte globulin,
(a) Subjective data: muromonab-CD3 (OKT3), tacrolimus
(i) Lethargy, anorexia. (Prograf ), and mycophenolate mofetil
(ii) Tenderness over graft site. (CellCept).
2164_Ch06_347-578 29/03/12 12:30 PM Page 464

464 chapter 6 Physiological Integrity

d. Goal: health teaching. backward pressure in kidney hydronephrosis


(1) Signs of rejection (see Goal b.). atrophy, fibrosis of renal tubules.
(2) Drugs: side effects of immunosuppression B. Risk factors (in over 50% of cases, cause is
(see Goal c.). idiopathic):
(3) Self-care activities: temperature, blood 1. Urinary tract infection.
pressure, I&O, urine specimen collection. 2. Urinary stasisobstruction.
(4) Avoidance of infection. 3. Metabolic factorsexcessive intake of vitamin D
(5) See also goals of care, under III. POST- or C, calcium carbonate.
OPERATIVE EXPERIENCE, pp. 370, C. Assessment (depends on size, shape, location of
374375. stone):
F. Evaluation/outcome criteria: 1. Subjective data:
1. No signs of rejection (e.g., no weight gain, a. Pain: occasional, dull, in loin or back when
oliguria). stones are in calyces or renal pelvis; excruciat-
2. No depression. ing in flank area (renal colic), radiating to
3. Client resumes role responsibilities. groin when stones are ureteral.
VII. NEPHRECTOMY (radical or partial): removal b. Nausea associated with pain.
of kidney through flank, retroperitoneal, abdominal, 2. Objective data:
thoracic, or thoracic-abdominal approach; indicated a. Pallor, sweating, syncope, shock, and
with malignant tumors, severe trauma, or under vomiting due to pain.
certain conditions before renal transplantation (see b. Palpable kidney mass with hydronephrosis.
VI. KIDNEY TRANSPLANTATION, pp. 462464). c. Fever and pyuria with infection.
d. Laboratory data:
A. Analysis/nursing diagnosis:
(1) Urinalysis: abnormalpH (acidic or alka-
1. Pain related to surgical incision.
line); RBCs (injury); WBCs (infection);
2. Risk for infection related to wound contamination.
increasedspecific gravity; casts; crystals;
3. Risk for fluid volume excess.
other organic substances, depending on
4. Risk for aspiration related to vomiting.
type of stone (i.e., calcium, struvite, uric
5. Constipation related to paralytic ileus.
acid, or cystine); positive culture.
6. Anxiety related to possible loss of function in
(2) Blood: increased calcium, phosphorus, total
remaining kidney.
protein, alkaline phosphatase, creatinine,
7. Dysfunctional grieving related to perceived loss.
uric acid, BUN.
B. Nursing care plan/implementation:
e. Diagnostic tests:
1. Preoperative: Goal: optimize physical and psycho-
(1) IVP: reveals nonopaque stones, degree of
logical functioning (see I. PREOPERATIVE
obstruction.
PREPARATION, pp. 365367).
(2) X-ray; radiopaque stones seen.
2. Postoperative: Goal: promote comfort and prevent
(3) Ultrasound may also be used.
complications.
D. Analysis/nursing diagnosis:
a. Observe for signs of:
ADULT

1. Pain related to passage of stone.


(1) Paralytic ileusabdominal distention,
2. Altered urinary elimination related to potential
absent bowel sounds, vomiting (common
obstruction.
complication following renal surgery).
3. Urinary retention related to obstruction of urethra.
(2) Hemorrhage.
E. Nursing care plan/implementation:
b. Fluid balance: daily weightmaintain within
1. Goal: reduce pain and prevent complications.
2% of preoperative level.
a. Medication: narcotics, antiemetics, antibiotics.
C. Evaluation/outcome criteria:
b. Fluids: 3 to 4 L/day; IVs if nauseated,
1. No complications (e.g., hemorrhage, paralytic
vomiting.
ileus, wound infection).
c. Activity: ambulate to promote passage of
2. Acceptance of loss of kidney.
stone, except bedrest during acute attack
VIII. RENAL CALCULI (UROLITHIASIS): formation (colic).
of calculi (stones) in renal calyces or pelvis that pass to d. Reduce spasms: warm soaks to affected flank.
lower regions of urinary tractureters, bladder, or e. Observe for signs of:
urethra; occurs after age 30, with greatest incidence in (1) Obstructiondecreased urinary output,
men, particularly over age 50. increased flank pain.
A. Pathophysiology: organic crystals form (most consist (2) Passage of stone (90% will pass <5-mm
of calcium salts or magnesium and ammonium phos- size)cessation of pain; filter urine with
phate (struvite) obstruction, infection; increased gauze.
2164_Ch06_347-578 29/03/12 12:30 PM Page 465

Genitourinary System 465


f. Monitor: hydration statusI&O, daily changes to prevent infection and prevent skin break-
weight; vital signsparticularly temperature down, and administration of antibiotics and narcotics
for sign of infection; urinecolor, odor. for pain.
2. Goal: health teaching. X. BENIGN PROSTATIC HYPERPLASIA (BPH):
a. Importance of fluids: minimum 3,000 mL/day; bladder outlet obstruction resulting from an enlarge-
2 glasses during night. ment of the prostate gland.
b. Diet: modify according to stone type (see
A. Pathophysiology: prostate enlarges, bulges
Chapter 9).
upward, blocks flow of urine from bladder into
(1) Calcium oxalate and calcium phosphate
urethra obstruction hydroureter,
stoneslow calcium, phosphorus, and
hydronephrosis.
oxalate (e.g., avoid tea, cocoa, cola, beans,
B. Risk factors:
spinach, acidic fruits).
1. Changes in estrogen and androgen levels.
(2) Magnesium and ammonium phosphatelow
2. Men older than 50.
phosphorus.
C. Assessment:
(3) Uric acid stoneslow purine.
1. Subjective dataurination:
(4) Cystine stoneslow protein.
a. Difficulty starting stream.
c. Acid-ash diet with calcium oxalate and calcium
b. Smaller, less forceful.
phosphate stones, magnesium and ammonium
c. Dribbling.
phosphate (struvite) stones.
d. Frequency.
d. Alkaline-ash diet with uric acid and cystine
e. Urgency.
stones (see Common Therapeutic Diets in
f. Nocturia.
Chapter 9, pp. 699701).
g. Retention (incomplete emptying).
e. Signs of urinary infection: dysuria, frequency,
h. Inability to void after ingestion of alcohol or
hematuria; seek immediate treatment.
exposure to cold.
f. Prepare for removal if indicated; 20% of
2. Objective data:
stones require additional treatment: cytoscopy
a. Catheterization for residual urine: 25 to
for small stones; cystolitholapaxy for soft
50 mL after voiding.
stones; lithotripsy or surgical removal (nephro-
b. Enlarged prostate on rectal examination.
scopic, pyelolithotomy, or nephrolithotomy).
c. Laboratory data:
F. Evaluation/outcome criteria:
(1) Urineincreased RBC, WBC counts.
1. Relief from pain.
(2) Bloodincreased creatinine, prostate-
2. No signs of urinary obstruction (e.g., increased
specific antigen (PSA).
flank pain, decreased urine output).
D. Analysis/nursing diagnosis:
3. No recurrence of lithiasis (adheres to diet and
1. Urinary retention related to incomplete
fluid regimen).
emptying.
IX. LITHOTRIPSY. Laser lithotripsynewer treatment 2. Altered urinary elimination related to obstruction.
using laser and a ureteroscope; constant water irriga- 3. Urinary incontinence related to urgency, pressure.

ADULT
tion because of heat. Extracorporeal shock wavea 4. Anxiety related to potential surgery.
noninvasive mechanical procedure used to break up 5. Body image disturbance related to threat to
renal calculi so they can pass spontaneously, in most masculine identity.
cases. The trunk of the client is submerged in distilled E. Nursing care plan/implementation:
water. In addition to being strapped to a frame, the 1. Goal: relieve urinary retention.
client may also receive sedation and analgesia for a. Catheterization: release maximum of 1,000 mL
pain from sound waves. The procedure takes 30 to initially; avoid bladder decompression, which
45 minutes, and remaining still is important. An results in hypotension, bladder spasms, rup-
underwater electrode generates shock waves that tured blood vessels in bladder; empty 200 mL
fragment the stone so it can be excreted in the urine every 5 minutes.
a few days after the procedure. A degree of renal b. Patency: irrigate intermittently or continually,
colic may occur, requiring narcotics for up to 3 days. as ordered.
Nursing measures should encourage ambulation and c. Fluids: minimum 2,000 mL/24 hr.
promote diuresis through forcing fluids. Percutaneous 2. Goal: health teaching.
lithotripsynephrostomy tract above kidney region; a. Preparation for surgery (cystostomy,
nephroscope used to retrieve calculi. Urinary drainage prostatectomy):
from incision for 3 to 4 days is normal. May be (1) Expectationsindwelling catheter (will
required for large fragments remaining after extracor- feel urge to void).
poreal lithotripsy. Nursing measures include: dressing
2164_Ch06_347-578 29/03/12 12:30 PM Page 466

466 chapter 6 Physiological Integrity

(2) Avoid pulling on catheter (this increases dressing if urinary drainage); notify
bleeding and clots). physician of excessive bleeding.
(3) Bladder spasms common 24 to 48 hours (7) Observe for signs of:
after surgery, particularly with (a) Bladder distentiondistinct mound
transurethral resection and suprapubic over pubis, slow drip in collecting
approaches. bottle; irrigate catheter as ordered.
(4) Threatening nature of procedure (possibility (b)Increased bleedingexpect and report
of impotence with perineal prostatectomy). frank bleeding (if venous bleeding,
b. See also I. PREOPERATIVE PREPARA- increase traction on catheter); if
TION, pp. 365367. bright-red drainage, and clots (arterial
XI. PROSTATECTOMY: surgical procedure to relieve bleeding), may need surgical control;
urinary retention and frequency caused by benign cool, clammy, pale skin and increased
prostatic hyperplasia or cancer of the prostate. pulse rate indicate shock.
b. Goal: assist in rehabilitation. Emotional sup-
A. Types
port: fears of incontinence, loss of masculine
1. Transurethral resection (TUR)removal of
identity, impotence.
obstructive prostatic tissue surrounding urethra
c. Goal: health teaching.
by an electrical wire (resectoscope) introduced
(1) Expectations: mild incontinence, dribbling
through the urethra; hypertrophy may recur, and
for a while (several months) after surgery;
TUR repeated; little risk of impotence. Laser
need to void as soon as urge is felt; push
also being used.
fluids.
2. Suprapubiclow midline incision is made
(2) Exercises: perineal (Kegel) 1 to 2 days after
directly over the bladder; bladder is opened and
surgerybuttocks are tightened for a
large mass of prostatic tissue is removed through
count of ten, 20 to 50 times daily.
incision in urethral mucosa.
(3) Avoid:
3. Retropubicremoval of hypertrophied prostatic
(a) Long auto trips, vigorous exercise, heavy
tissue high in pelvic area through a low abdomi-
lifting (anything heavier than 10 lb), and
nal incision; bladder is not opened; client may
sexual intercourse for about 3 weeks or
remain potent.
until medical permission, because they
4. Perinealremoval of prostatic tissue low in
may increase tendency to bleed.
pelvic area is accomplished through an incision
(b) Alcoholic beverages for 1 month, because
made between the scrotum and the rectum;
this may cause burning on urination;
usually results in impotency and incontinence.
caffeine, because it causes diuresis.
B. Nursing care plan/implementation:
(c) Tub baths, because of increase chance
1. Preoperative (see X. BENIGN PROSTATIC
of infection.
HYPERPLASIA, p. 465).
(4) Medications: stool softeners or mild
2. Postoperative:
cathartics to decrease straining.
a. Goal: promote optimal bladder function and
ADULT

C. Evaluation/outcome criteria:
comfort.
1. Relief of symptoms.
(1) Urinary drainage: sterile closed-gravity
2. No complications (e.g., hemorrhage,
systemmaintain external traction as
impotence).
ordered.
(2) Reinforce purposes, sensations to expect. XII. URINARY DIVERSION: Incontinentileal con-
(3) Bladder irrigation to control bleeding, duit: anastomosis of ureters to a small portion of the
keep clots from forming. ileum; stoma is called urostomy; urine flow is constant;
(4) Suprapubic catheter care (suprapubic requires external collection device. ContinentKock
prostatectomy)closed-gravity drainage pouch, Indiana pouch: segment of small bowel or colon
system; observe character, amount, flow of is used to create a pouch; holds urine without leakage;
drainage. requires self-catheterization.
(5) After removal: A. Indications:
(a) Observe for urinary drainage q4h for 1. Congenital anomalies of bladder.
24 hours. 2. Neurogenic bladder.
(b) Skin care. 3. Mechanical obstruction to urine flow (e.g.,
(c) Report excessive drainage to physician. bladder cancer).
(6) Dressings: keep dry, clean; reinforce if 4. Chronic progressive pyelonephritis.
necessary (may need to change suprapubic 5. Trauma to lower urinary tract.
2164_Ch06_347-578 29/03/12 12:30 PM Page 467

Endocrine System 467


B. Analysis/nursing diagnosis: D. Evaluation/outcome criteria:
1. Altered urinary elimination related to surgical 1. Acceptance of new body image.
diversion. 2. Regains independence.
2. Risk for impaired skin integrity related to leakage 3. Demonstrates confidence in management of
of urine. self-care activities.
3. Risk for infection related to contamination of
stoma.
4. Constipation related to absence of peristalsis.
ENDOCRINE SYSTEM
5. Body image disturbance related to stoma. I. DIABETES: heterogeneous group of diseases involv-
C. Nursing care plan/implementation: ing the disruption of the metabolism of carbohydrates,
1. Preoperative: optimal bowel and stoma site fats, and protein. If uncontrolled, serious vascular and
preparation. neurological changes occur.
a. Diet: low residue 2 days followed by clear A. Types:
liquids for 24 hours. 1. Type 1: formerly called insulin-dependent dia-
b. Medications: betes mellitus (IDDM); and also formerly called
(1) Neomycin (for bowel sterilization). juvenile-onset diabetes. Insulin is needed to
(2) Cathartics (GoLYTELY), enemas. prevent ketosis; onset usually in youth but may
c. Site selection: appliance faceplate (incontinent occur in adulthood; prone to ketosis, unstable
diversion) must bond securely; avoid areas of diabetes.
pressure from clothing (waistline); usual site is 2. Type 2: formerly called noninsulin-dependent
right or left lower abdominal quadrant. diabetes mellitus (NIDDM); and also formerly
d. See also I. PREOPERATIVE called maturity-onset diabetes or adult-onset
PREPARATION, pp. 365367. diabetes. May be controlled with diet and oral
2. Postoperative: hypoglycemics or insulin; client less apt to have
a. Goal: prevent complications and promote ketosis, except in presence of infection. May be
comfort. further classified as obese type 2 or nonobese type 2.
(1) Observe for signs of: 3. Type 3: gestational diabetes mellitus (GDM):
(a) Paralytic ileus (common complication) glucose intolerance during pregnancy in
keep NG tube patent. women who were not known to have diabetes
(b) Stoma necrosisdusky or cyanotic color before pregnancy; will be reclassified after
(emergency situation). birth; may need to be treated or may return to
(2) Skin care: check for leakage around prepregnancy state and need no treatment.
ostomy bag. 4. Type 4: diabetes secondary to another condition,
(3) Urinary drainagestents or catheter in such as pancreatic disease, other hormonal
stoma; blood in urine in immediate imbalances, or drug therapy such as receiving
postoperative period; mucus normal. glucocorticoids.
(4) See III. POSTOPERATIVE B. Pathophysiology:

ADULT
EXPERIENCE, pp. 370, 374375. 1. Type 1absolute deficiency of insulin due to
b. Goal: health teaching. destruction of pancreatic beta cells by the inter-
(1) Self-care activities: action of genetic, immunological, hereditary, or
(a) Peristomal skin careprevent irrita- environmental factors.
tion, breakdown; proper cleansing 2. Type 2relative deficiency of insulin due to:
soap and water; adhesive remover, if a. An islet cell defect resulting in a slowed or
needed. delayed response in the release of insulin to a
(b) Appliance application and emptying; glucose load; or
pouch opening 2 to 3 mm larger than b. Reduction in the number of insulin receptors
stoma; do not remove each day; change from continuously elevated insulin levels; or
appliance every 3 to 5 days or when c. A postreceptor defect; or
leaking. d. A major peripheral resistance to insulin
(c) Odor controldilute urine, hygiene, induced by hyperglycemia. These factors lead
acid-ash diet; avoid asparagus, tomatoes. to deprivation of insulin-dependent cells a
(d) Use of night drainage system if necessary marked decrease in the cellular rate of glucose
for uninterrupted sleep. uptake, and therefore elevated blood glucose.
(2) Signs of complications: change in urine C. Risk factors:
color, clarity, quantity, smell; stomal color 1. Obesity.
change (normal is bright pink or red). 2. Family history of diabetes.
2164_Ch06_347-578 29/03/12 12:30 PM Page 468

468 chapter 6 Physiological Integrity

3. Age 45 or older. (3) Neuropathy: due to reduced tissue perfu-


4. Women whose babies at birth weighed more sion; affecting motor, sensory, voluntary,
than 9 lb. and autonomic functions.
5. History of autoimmune disease. (4) Arteriosclerosis: due to lesions of the
6. Members of high-risk ethnic group (African intimal wall.
American, Latino, or Native American). (5) Cardiac: angina, coronary insufficiency,
7. History of gestational diabetes mellitus. myocardial infarction.
8. Hypertension. (6) Vascular changes: occlusions, intermittent
9. Elevated high-density lipoprotein (HDL). claudication, loss of peripheral pulses,
D. Assessment: arteriosclerosis.
1. Subjective data: E. Analysis/nursing diagnosis:
a. Eyes: blurry vision. 1. Altered nutrition, less than body requirements,
b. Skin: pruritus vulvae. related to inability to metabolize nutrients and
c. Neuromuscular: paresthesia, peripheral neu- weight loss.
ropathy, lethargy, weakness, fatigue, increased 2. Altered nutrition, more than body requirements,
irritability. related to excessive glucose intake.
d. GI: polydipsia (increased thirst). 3. Risk for injury related to complications of
e. Reproductive: impotence. uncontrolled diabetes.
2. Objective data: 4. Body image disturbance related to long-term
a. Genitourinary: polyuria, glycosuria, nocturia illness.
(nocturnal enuresis in children). 5. Knowledge deficit (learning need) related to
b. Vital signs: management of long-term illness and potential
(1) Pulse and temperature: normal or elevated. complications.
(2) BP: normal or decreased, unless complica- 6. Ineffective individual coping related to inability
tions present. to follow diet/medication regimen.
(3) Respirations: increased rate and depth 7. Sexual dysfunction related to impotence from
(Kussmauls respirations). diabetes and treatment.
c. GI: F. Nursing care plan/implementation:
(1) Polyphagia, dehydration. 1. Goal: obtain and maintain normal sugar balance.
(2) Weight loss, failure to gain weight. a. Monitor: vital signs; blood glucose before
(3) Acetone breath. meals, at bedtime, and as symptoms demand
d. Skin: cuts heal slowly; frequent infections, (urine testing for glucose levels is not as
foot ulcers, vaginitis. accurate as capillary blood testing).
e. Neuromuscular: loss of strength, peripheral b. Medications:
neuropathy. (1) Oral hypoglycemics:
f. Laboratory data: (a) Sulfonylureas; tolbutamide (Orinase),
(1) Elevated: chlorpropamide (Diabenese), tolaza-
ADULT

(a) Blood sugar greater than 126 mg/dL mide (Tolinase), acetohexamide
fasting or 200 mg/dL 1 to 2 hours after (Dymelor), glimepiride (Amaryl),
eating. glyburide (DiaBeta and Micronase),
(b) Glucose tolerance test. and glipizide (Glucocotrol).
(c) Glycosuria (>170 mg/100 mL). (b) Others: metformin (Glucophage)
(d) Potassium (>5 mg/dL) and chloride increases bodys sensitivity to insulin.
(>145 mg/dL). Acarbose (Precose) inhibits cells of the
(e) Hemoglobin A1c greater than 7%. small intestine from absorbing complex
(2) Decreased: carbohydrates.
(a) pH (<7.4). (2) Insulin (biosynthetic human insulin): bolus
(b) PaCO2 (<32). insulinreleased in response to meals;
g. Long-term pathological considerations: basal insulinreleased between meals, at
(1) Cataract formation and retinopathy: thick- nighttime.
ened capillary basement membrane, (a) Rapid-acting bolus analogue of human
changes in vascularization and hemor- insulin (Lispro, Aspart).
rhage, due to chronic hyperglycemia. (b) Short-acting bolus (crystalline, regular)
(2) Nephropathy: due to glomerulosclerosis, (Humulin R).
arteriosclerosis of renal artery and (c) Intermediate-acting basal (Lente and
pyelonephritis, progressive uremia. NPH).
2164_Ch06_347-578 29/03/12 12:30 PM Page 469

Endocrine System 469


(d) Slow acting (protamine zinc). compromise, neuropathy, nephropathy,
(e) Long, extended-acting basal retinopathy) complications.
(Ultralente). 2. Goal: health teaching:
(f ) Long-acting basal analogue (Glargine). a. Diet: foods allowed, restricted, substitutions.
(3) Methods of administration: b. Medications: administration techniques,
(a) Subcutaneous injection. importance of using room-temperature
(b) Prefilled injectable insulin pens insulin and rotating injection sites to prevent
(Novopen and Novolin). tissue damage.
(c) Continuous subcutaneous insulin c. Desired and side effects of prescribed insulin
infusion therapy (insulin pumps). type; onset, peak, and duration of action of
c. Diet, as ordered. prescribed insulin.
(1) Carbohydrate, 50% to 60%; protein, d. Blood glucose testing techniques.
20%; fats, 30% (saturated fats limited to e. Signs of complications (Table 6.28).
10%, unsaturated fats, 90%). f. Importance of health maintenance:
(2) Calorie reduction in adults who are obese; (1) Infection prevention, especially foot and
enough calories to promote normal growth nail care.
and development for children or adults (2) Routine checkups.
who are not obese. (3) Maintain stable balance of glucose by care-
(3) Limit refined sugars. fully monitoring glucose level and making
(4) Add vitamins, minerals as needed for necessary adjustments in diet and activity
well-balanced diet. level; seeking medical attention when
d. Monitor for signs of acute (hypoglycemia, unable to maintain balance; regular exer-
ketoacidosis) or chronic (circulatory cise program.

Table 6.28
Comparison of Diabetic Complications
Hypoglycemia Ketoacidosis
Pathophysiology Major metabolic complication when too Major metabolic complication in which there is insufficient
little food or too-large dose of insulin or insulin for metabolism of carbohydrates, fats, and proteins;
hypoglycemic agents administered; inter- seen most frequently with clients who are insulin dependent;
feres with oxygen consumption of nerv- precipitated in the person with known diabetes by stressors
ous tissue (such as infection, trauma, major illness) that increase insulin
needs
Risk factors Too little food Insufficient insulin or oral hypoglycemics
Emotional or added stress Noncompliance with dietary instructions
Vomiting or diarrhea Major illness/infections

ADULT
Added exercise Therapy with steroid administration
Trauma, surgery
Elevated blood sugar: >200 mg/dL
Assessment Behavioral change: Behavioral change:
Subjective datanervous, irritable, Subjective datairritable, confused
anxious, confused, disoriented
Objective dataabrupt mood changes, Objective datadrowsy
psychosis
Visual: Visual:
Subjective datablurred vision, diplopia
Objective datadilated pupils Objective dataeyeballs: soft, sunken
Skin: Objective datadiaphoresis, Skin: Objective dataloss of turgor, flushed face, pruritus
pale, cool, clammy, goosebumps vulvae
(piloerection), tenting
Vitals: Subjective datapalpitations Vitals: Objective data tachycardia; thready; respirations:
Kussmauls; BP: hypovolemic shock
Gastrointestinal: Gastrointestinal:
Subjective datahunger, nausea Subjective dataincreased thirst and hunger, abdominal pain,
nausea
Objective datadiarrhea, vomiting Objective datavomiting, diarrhea, dry mucous membrane;
lips, tongue: red, parched; breath: fruity
Continued
2164_Ch06_347-578 29/03/12 12:30 PM Page 470

470 chapter 6 Physiological Integrity

Table 6.28
Comparison of Diabetic Complicationscontd
Hypoglycemia Ketoacidosis
Neurological: Neurological:
Subjective dataheadache; lips/tongue: Subjective dataheadache; irritability; confusion; lethargy,
tingling, numbness weakness
Objective datafainting, yawning;
speech: incoherent; convulsions; coma
Musculoskeletal: Musculoskeletal
Subjective dataweak, fatigue Subjective datafatigue; general malaise
Objective datatrembling
Renal: Objective datapolyuria
Blood sugar: <80 mg/dL Blood sugar: >130 mg/dL
Analysis/nursing Risk for injury related to deficit of Risk for injury related to glucose imbalance
diagnosis needed glucose
Knowledge deficit (learning need) Knowledge deficit (learning need) related to proper balance of
related to proper dietary intake or proper diet and insulin dosage
insulin dosage
Altered nutrition, less than body require-
ments, related to glucose deficiency
Nursing care plan/ Goal: provide adequate glucose to Goal: promote normal balance of food and insulin:
implementation reverse hypoglycemia: administer simple regular insulin as ordered; IV saline, as ordered;
sugar STAT, PO or IV, glucose paste bicarbonate and electrolyte replacements, as ordered;
absorbed in mucous membrane; monitor potassium replacements once therapy begins and urine output
blood sugar levels; identify events lead- is adequate
ing to complication
Goal: health teaching: how to prevent Goal: health teaching: diet instructions, desired effects and
further episodes (see I. DIABETES, side effects of prescribed insulin or hypoglycemic agent (onset,
Health Teaching, p. 469); importance peak, and duration of action); importance of recognizing signs
of careful monitoring of balance between of imbalance
glucose levels and insulin dosage
Evaluation/outcome Adheres to diet and correct insulin Accepts prescribed diet
criteria dosage Takes medication (correct dose and time)
Adjusts dosage when activity is Serious complications avoided
increased
Glucose level: 80120 mg/dL Glucose level: 80120 mg/dL

G. Evaluation/outcome criteria: of extracellular fluid, and hyperosmolarity related


ADULT

1. Optimal blood-glucose levels achieved. to infection or another stressor as the precipitating


2. Ideal weight maintained. factor. Client unable to replace fluid deficits with
3. Adequate hydration. oral intake.
4. Carries out self-care activities: blood testing, foot B. Risk factors:
care, exchange diets, medication administration, 1. Old age.
exercise. 2. History of noninsulin-dependent diabetes.
5. Recognizes and treats hyperglycemic or 3. Infections: pneumonia, pyelonephritis,
hypoglycemic reactions. pancreatitis, gram-negative infections.
6. Seeks medical assistance appropriately. 4. Kidney failure: uremia and peritoneal dialysis
II. NONKETOTIC HYPERGLYCEMIC or hemodialysis.
HYPEROSMOLAR COMA (NKHHC): profound 5. Shock:
hyperglycemia and dehydration without ketosis or a. Lactic acidosis related to bicarbonate
ketoacidosis; seen in type 2 diabetes; brought on deficit.
by infection or illness. This condition may lead to b. Myocardial infarction.
impaired consciousness and seizures. The client is 6. Hemorrhage:
critically ill. Mortality rate is over 50%. a. GI.
b. Subdural.
A. Pathophysiology: hyperglycemia greater than
c. Arterial thrombosis.
1,000 mg/dL causes osmotic diuresis, depletion
2164_Ch06_347-578 29/03/12 12:30 PM Page 471

Endocrine System 471


7. Medications: metabolism caused by excessive amounts of circulating
a. Diuretics. thyroid hormone. Graves disease is most common
b. Glucocorticoids. cause of hyperthyroidism. The three components of
8. Tube feedings. Graves disease are: (1) hyperthyroidism, (2) ophthal-
C. Assessment: mopathy (protrusion of the eyes), and (3) skin lesions
1. Subjective data: (dermopathy). Graves disease is triggered by: stress,
a. Confusion. smoking, radiation of the neck, some medications
b. Lethargy. (interleukin-2), and infections. Treatment of hyperthy-
2. Objective data: roidism is accomplished by antithyroid medications,
a. Nystagmus. radioactive iodine administration (capsule given once),
b. Dehydration. or surgery. Clients may need to take supplemental
c. Aphasia. thyroid hormone (levothyroxine) after treatment.
d. Nuchal rigidity. A. Pathophysiology: diffuse hyperplasia of thyroid
e. Hyperreflexia. gland overproduction of thyroid hormone and
f. Laboratory data: increased blood serum levels. Hormone stimulates
(1) Blood glucose level greater than mitochondria to increase energy for cellular activities
1,000 mg/dL. and heat production. As metabolic rate increases,
(2) Serum sodium and chloridenormal to fat reserves are utilized, despite increased appetite
elevated. and food intake. Cardiac output is increased to
(3) BUN greater than 60 mg/dL (higher than meet increased tissue metabolic needs, and periph-
in ketoacidosis because of more severe eral vasodilation occurs in response to increased
gluconeogenesis and dehydration). heat production. Neuromuscular hyperactivity
(4) Arterial pHslightly depressed. accentuation of reflexes, anxiety, and increased
D. Analysis/nursing diagnosis: alimentary tract mobility. Graves disease is caused
1. Risk for injury related to hyperglycemia. by stimulation of the gland by immunoglobulins
2. Altered renal peripheral tissue perfusion related to of the IgG class.
vascular collapse. B. Risk factors:
3. Ineffective airway clearance related to coma. 1. Possible autoimmune response resulting in
E. Nursing care plan/implementation: increase of a gamma globulin called long-acting
1. Goal: promote fluid and electrolyte balance. thyroid stimulator (LATS).
a. IVs: fluids and electrolytes, sodium chloride 2. Occurs in third and fourth decade.
solution used initially to combat dehydra- 3. Affects women more than men.
tion. Rate of infusion will be determined by: 4. Emotional trauma, infection, increased
BP assessment, cardiovascular status, balance stress.
between fluid input and output, and labora- 5. Overdose of medications used to treat
tory values. hypothyroidism.
b. Monitor I&O because of the high volume 6. Use of certain weight-loss products.

ADULT
of fluid replaced in the critical stage of this 7. Radiation of neck.
condition. C. Assessment:
c. Administer nursing care for the problem that 1. Subjective data:
precipitated this serious condition. a. Nervousness, mood swings.
d. Diet: Food by mouth when client is able. b. Palpitations.
2. Goal: prevent complications. c. Heat intolerance.
a. Administer regular insulin (initial dose usually d. Dyspnea.
5 to 15 units) and food, as ordered. e. Weakness.
b. Uncontrolled condition leads to: cardiovascular 2. Objective data:
disease, renal failure, blindness, and diabetic a. Eyes: exophthalmos, characteristic stare,
gangrene. lid lag.
F. Evaluation/outcome criteria: b. Skin:
1. Blood sugar returns to normal level of 80 to (1) Warm, moist, velvety.
120 mg/dL. (2) Increased sweating; increased melanin
2. Client is alert to time, place, and person. pigmentation.
3. Primary medical problem resolved. (3) Pretibial edema with thickened skin and
4. Client recognizes and reports signs of imbalance. hyperpigmentation.
III. HYPERTHYROIDISM (also called thyrotoxicosis; c. Weight: loss of weight despite increased
Graves disease): spectrum of symptoms of accelerated appetite.
2164_Ch06_347-578 29/03/12 12:30 PM Page 472

472 chapter 6 Physiological Integrity

d. Muscle: weakness, tremors, hyperkinesia. (3) Iodine preparations: used in combination


e. Vital signs: BPincreased systolic pressure, with other medications when hyperthy-
widened pulse pressure; tachycardia. roidism not well controlled; saturated solu-
f. Goiter: thyroid gland noticeable and tion of potassium iodide (SSKI) or Lugols
palpable. solution; more palatable if diluted with
g. Gynecological: abnormal menstruation. water, milk, or juice; give through a straw to
h. GI: frequent bowel movements. prevent staining teeth. Takes 2 to 4 weeks
i. Activity pattern: overactivity leads to fatigue, before results are evident.
which leads to depression, which stimulates (4) Propranolol (Inderal), atenolol
client into overactivity, and pattern continues. (Tenormin), metoprolol (Lopressor) given
Danger: total exhaustion. to counteract the increased metabolic
j. Laboratory data: effect of thyroid hormones, but do not
(1) Elevated: serum thyroxine (T4) alter their levels. Relieve the symptoms of
(>11 mcg/100 mL), free T4 or free T4 tachycardia, tremors, and anxiety.
index, triiodothyronine (T3) level (>35%) b. Monitor for thyroid storm (crisis)medical
and free T3 level. emergency: acute episode of thyroid overac-
(2) Elevated: radioactive iodine uptake tivity caused when increased amounts of
(RAIU) by thyroid. thyroid hormone are released into the
(3) Elevated: basal metabolic rate (BMR). bloodstream and metabolism is markedly
(4) Decreased: WBC count caused by decreased increased.
granulocytosis (<4,500). (1) Risk factors for thyroid storm: client with
D. Analysis/nursing diagnosis: uncontrolled hyperthyroidism (usually
1. Altered nutrition, less than body requirements, Graves disease) who undergoes severe
related to elevated basal metabolic rate. sudden stress, such as:
2. Risk for injury related to exophthalmos and (a) Infection.
tremors. (b) Surgery.
3. Activity intolerance related to fatigue from (c) Beginning labor to give birth.
overactivity. (d) Taking inadequate antithyroid medica-
4. Fatigue related to overactivity. tions before thyroidectomy.
5. Anxiety related to tachycardia. (2) Assessment:
6. Sleep pattern disturbance related to excessive (a) Subjective datathyroid storm:
amounts of circulating thyroid hormone. (i) Apprehension.
E. Nursing care plan/implementation: (ii) Restlessness.
1. Goal: protect from stress: private room, restrict (b) Objective datathyroid storm:
visitors, quiet environment. (i) Vital signs: elevated temperature
2. Goal: promote physical and emotional (106F), hypotension, extreme
equilibrium. tachycardia.
ADULT

a. Environment: quiet, cool, well ventilated. (ii) Marked respiratory distress,


b. Eye care: pulmonary edema.
(1) Sunglasses to protect from photophobia, (iii) Weakness and delirium.
dust, wind. (iv) If untreated, client could die
(2) Protective drops (methylcellulose) to of heart failure.
soothe exposed cornea. (3) Medicationsthyroid storm:
c. Diet: (a) Propylthiouracil or methimazole
(1) High: calorie, protein, vitamin B. (Tapazole) to decrease synthesis of
(2) 6 meals/day, as needed. thyroid hormone.
(3) Weigh daily. (b) Sodium iodide IV; Lugols solution orally
(4) Avoid stimulants (coffee, tea, colas, to facilitate thyroid hormone synthesis.
tobacco). (c) Propranolol (Inderal) to slow heart rate.
3. Goal: prevent complications. (d) Aspirin to decrease temperature.
a. Medications as ordered: (e) Steroids to combat crisis.
(1) Propylthiouracil to block thyroid synthesis; (f ) Diuretics, digitalis to treat heart failure.
hyperthyroidism returns when therapy is 4. Goal: health teaching.
stopped. a. Stress-reduction techniques.
(2) Methimazole (Tapazole) to inhibit synthe- b. Importance of medications, their desired
sis of thyroid hormone. effects and side effects.
2164_Ch06_347-578 29/03/12 12:30 PM Page 473

Endocrine System 473


c. Methods to protect eyes from environmental 2. Goal: prevent complications of hypocalcemia and
damage. tetany, due to accidental trauma to parathyroid
d. Signs and symptoms of thyroid storm gland during surgery; signs of tetany indicate
(see E. 3. b.). necessity of calcium gluconate IV.
5. Goal: prepare for additional treatment as needed. a. Check Chvosteks signtapping face in front
a. Radioactive iodine therapy: 131I, a radioactive of ear produces spasm of facial muscles.
isotope of iodine to decrease thyroid b. Check Trousseaus signcompression of upper
activity. arm (usually with BP cuff ) elicits carpal
(1) 131I dissolved in water and given by (wrist) spasm.
mouth. c. Monitor for respiratory distress (due to
(2) Hospitalization necessary only when large laryngeal nerve injury, edema, bleeding);
dose is administered. keep tracheostomy set/suction equipment
(3)Minimal precautions needed for usual at bedside.
dose. d. Monitor for elevated temperature, indicative
(a) Sleep alone for several nights. of thyroid storm (see Objective Assessment
(b) Flush toilet several times after use. Data for Thyroid Storm, p. 472).
(4)Effectiveness of therapy seen in 2 to e. Monitor vital signs, check dressing and
3 weeks; single dose controls 90% of beneath head, shoulders for bleeding q1h
clients. and prn for 24 hours; hemorrhage is possible
(5) Monitor for signs of hypothyroidism. complication; if swallowing is difficult, loosen
b. Surgery (see IV. THYROIDECTOMY, dressing. If client still complains of tightness
following). when dressing is loosened, look for further
F. Evaluation/outcome criteria: signs of hemorrhage.
1. Complications avoided. f. Check voice postoperatively as soon as
2. Compliance with medical regimen. responsive after anesthesia and every hour
3. No further weight loss. (assess for possible laryngeal nerve damage);
4. Able to obtain adequate sleep. crowing voice sound indicates laryngeal
IV. THYROIDECTOMY: partial removal of thyroid nerves on both sides have been injured;
gland (for hyperthyroidism) or total removal (for respiratory distress possible from swelling.
malignancy of thyroid). (1) Avoid unnecessary talking to lessen
hoarseness.
A. Risk factor: unsuccessful medical treatment of
(2)Provide alternative means of
hyperthyroidism.
communication.
B. Analysis/nursing diagnosis:
3. Goal: promote comfort measures.
1. Risk for injury related to possible trauma to
a. Narcotics as ordered.
parathyroid gland during surgery.
b. Offer iced fluids.
2. Ineffective breathing pattern related to neck
c. Ambulation and soft diet, as
incision.

ADULT
tolerated.
3. Pain related to surgical incision.
4. Goal: health teaching.
4. Altered nutrition, less than body requirements,
a. How to support neck to prevent pressure on
related to difficulty in swallowing because of
suture line: place both hands behind neck
neck incision.
when moving head or coughing.
5. Impaired verbal communication related to
b. Signs of hypothyroidism; needs supplemental
possible trauma to nerve during surgery.
thyroid hormone if total thyroidectomy.
6. Risk for altered body temperature related to
c. Signs and symptoms of hemorrhage and
thyroid storm.
respiratory distress.
C. Nursing care plan/implementation: Prepare
d. Importance of adequate rest and nutritious diet.
for surgery (see I. THE PREOPERATIVE
e. Importance of voice rest in early recuperative
PREPARATION, pp. 365367). Postoperative:
period.
1. Goal: promote physical and emotional
D. Evaluation/outcome criteria:
equilibrium.
1. No respiratory distress, hemorrhage, laryngeal
a. Position: semi-Fowlers to reduce edema.
damage, tetany.
b. Immobilize head with pillows/sandbags.
2. Preoperative symptoms relieved.
c. Support head during position changes to
3. Normal range of neck motion obtained.
avoid stress on sutures, prevent flexion or
4. States signs and symptoms of possible
hyperextension of neck.
complications.
2164_Ch06_347-578 29/03/12 12:30 PM Page 474

474 chapter 6 Physiological Integrity

V. HYPOTHYROIDISM (MYXEDEMA): deficiency 3. Activity intolerance related to fatigue.


of circulating thyroid hormone; often a final conse- 4. Constipation related to decreased peristalsis.
quence of Hashimotos thyroiditis and Graves disease. 5. Decreased cardiac output related to hypotension
A. Pathophysiology: atrophy, destruction of gland and bradycardia.
by endogenous antibodies or inadequate pituitary 6. Risk for impaired skin integrity related to dry
thyrotropin production insidious slowing of skin and edema.
body processes, personality changes, and general- 7. Social isolation related to lethargy.
ized, interstitial nonpitting (mucinous) edema E. Nursing care plan/implementation:
myxedema; pronounced involvement in systems 1. Goal: provide for comfort and safety.
with high protein turnover (e.g., cardiac, GI, a. Monitor for infection or trauma; may precipi-
reproductive, hematopoietic). tate myxedema coma, which is manifested by:
B. Risk factors: unresponsiveness, bradycardia, hypoventilation,
1. Total thyroidectomy; inadequate replacement hypothermia, and hypotension.
therapy. b. Provide warmth; prevent heat loss and
2. Inherited autosomal recessive gene coding for vascular collapse.
disorder. c. Administer thyroid medications as ordered:
3. Hypophyseal failure. levothyroxine (Synthroid)most common
4. Dietary iodine deficiencies. drug used; liothyronine sodium (Cytomel);
5. Irradiation of thyroid gland. dosage adjusted according to symptoms.
6. Overtreatment of hyperthyroidism. 2. Goal: health teaching.
7. Chronic lymphocytic thyroiditis. a. Diet: low calorie, high protein.
8. Postpartum thyroiditis. b. Signs and symptoms of hypothyroidism and
9. Viral thyroiditis. hyperthyroidism.
10. Medications, such as amiodarone HCl c. Lifelong medications, dosage, desired effects
(Cordarone), used to treat abnormal heart and side effects.
rhythms. d. Medication dosage adjustment: take one
C. Assessment: third to one half the usual dose of narcotics
1. Subjective data: and barbiturates.
a. Weakness, fatigue, lethargy. e. Stress-management techniques.
b. Headache. f. Exercise program.
c. Slow memory, psychotic behavior. F. Evaluation/outcome criteria:
d. Loss of interest in sexual activity. 1. No complications noted. Most common compli-
2. Objective data: cations: atherosclerotic coronary heart disease,
a. Depressed basal metabolic rate (BMR). acute organic psychosis, and myxedema coma.
b. Cardiomegaly, bradycardia, hypotension, 2. Dietary instructions followed.
anemia. 3. Medication regimen followed.
c. Menorrhagia, amenorrhea, infertility. 4. Thyroid hormone balance obtained and
ADULT

d. Dry skin, brittle nails, coarse hair, hair loss. maintained.


e. Slow speech, hoarseness, thickened tongue. VI. CUSHINGS DISEASE: an endogenous overpro-
f. Weight gain: edema, generalized interstitial; duction of adrenocorticotropic hormone (ACTH)
peripheral nonpitting; periorbital puffiness. that can be caused by pituitary-dependent adenomas.
g. Intolerance to cold. Cushings syndrome: condition marked by chronic
h. Hypersensitive to narcotics and barbiturates. excessive circulating cortisol with or without pituitary
i. Laboratory data: involvement. One of the most common causes
(1) Elevated: thyroid-releasing hormone of Cushings syndrome is the administration of
(TRH), thyroid-stimulating hormone cortisone-like medications for treatment of a variety
(TSH), cholesterol (>220 mg/dL), lipids of conditions.
(>850 mg/dL), protein (>8 gm/dL). A. Pathophysiology:
(2) Normal-low: serum thyroxine (T4), serum 1. Excess glucocorticoid production, leading to:
triiodothyronine (T3). a. Increased gluconeogenesis raised serum
(3) Decreased: radioactive iodine uptake glucose levels glucose in urine, increased
(RAIU). fat deposits in face and trunk.
D. Analysis/nursing diagnosis: b. Decreased amino acids protein deficiencies,
1. Risk for injury related to hypersensitivity to drugs. muscle wasting, poor antibody response, and
2. Altered nutrition, more than body requirements, lack of collagen.
related to decreased BMR.
2164_Ch06_347-578 29/03/12 12:30 PM Page 475

Endocrine System 475


B. Risk factors: d. Infectionskin care, urinary tract; check
1. Adrenal hyperplasia. temperature.
2. Excessive hypothalamic stimulation. e. Mood swingsobserve behavior.
3. Tumors: adrenal, hypophyseal, pituitary, 3. Goal: health teaching.
bronchogenic, or gallbladder. a. Diet: increased protein, potassium; decreased
4. Excessive steroid therapy. calories, sodium.
C. Assessment: b. Medications:
1. Subjective data: (1) Cytotoxic agents: aminoglutethimide
a. Headache, backache. (Cytadren), trilostane (Modrastane),
b. Weakness, decreased work capacity. mitotane (Lysodren)to decrease cortisol
c. Mood swings. production.
2. Objective data: (2) Replacement hormones as needed.
a. Hypertension, weight gain, pitting edema. c. Signs and symptoms of progression of disease
b. Characteristic fat deposits, truncal and as noted in assessment.
cervical obesity (buffalo hump). d. Preparation for adrenalectomy if medical
c. Pendulous abdomen, purple striae, easy regimen unsuccessful.
bruising. F. Evaluation/outcome criteria:
d. Moon facies, acne. 1. Symptoms controlled by medication.
e. Hyperpigmentation. 2. No complicationsadrenal steroids within
f. Impotence. normal limits.
g. Virilization in women: hirsutism, breast 3. If adrenalectomy necessary, see VIII.
atrophy, and amenorrhea. ADRENALECTOMY.
h. Pathological fractures, reduced height. VII. PHEOCHROMOCYTOMA: a rare, typically
i. Slow wound healing. benign neuroendocrine tumor of the adrenal medulla.
j. Laboratory data: Appears to have a familial basis; common in middle
(1)Urine: elevated 17-ketosteroids agerare after 60 years.
(>12 mg/24 hr) and glucose
A. Pathophysiology: catecholamine-secreting tumor
(>120 mg/dL).
increased epinephrine and norepinephrine
(2) Plasma: elevated 17-hydroxycorticosteroids,
(paroxysm) hypertensive retinopathy and
cortisol (>10 mcg/dL). Cortisol does not
nephropathy, myocarditis cerebral hemorrhage
decrease during the day as it should.
and cardiac failure.
(3) Serum: elevatedglucose, RBC, WBC
B. Risk factors for paroxysm:
counts; diminishedpotassium, chlorides,
1. Voiding.
eosinophils, lymphocytes.
2. Smoking.
k. X-rays and scans to determine tumors
3. Drugs (i.e., histamine, anesthesia, atropine,
metastasis.
steroids, fentanyl).
D. Analysis/nursing diagnosis:
4. Bending, straining, exercising (displacing

ADULT
1. Body image disturbance related to changes in
abdominal organs) increased abdominal
physical appearance.
pressure.
2. Activity intolerance related to backache and
C. Assessment:
weakness.
1. Subjective data:
3. Risk for injury related to infection and
a. Apprehension.
bleeding.
b. Pounding headache.
4. Knowledge deficit (learning need) related to
c. Nausea.
management of disease.
d. Pain with vomiting.
5. Pain related to headache.
e. Visual disturbances.
E. Nursing care plan/implementation:
f. Palpitations.
1. Goal: promote comfort.
g. Heat intolerance.
a. Assist with preparation of diagnostic work-up.
2. Objective data:
b. Explain procedures.
a. Hypertension: rapid onset, abrupt cessation;
c. Protect from trauma.
postural hypotension.
2. Goal: prevent complications; monitor for:
b. Profuse diaphoresis with acute attack.
a. Fluid balanceI&O, daily weights.
c. Pulse: rapid, dysrhythmia.
b. Glucose metabolismblood, urine for sugar
d. Pupils: dilated.
and acetone.
e. Extremities: cold, tremors.
c. Hypertensionvital signs.
2164_Ch06_347-578 29/03/12 12:30 PM Page 476

476 chapter 6 Physiological Integrity

f. Laboratory data: c. Bilateral excision for metastasis of breast and


(1) Hyperglycemia, glycosuria. sometimes metastasis of prostate cancer.
(2) Urinary catecholamines: single-voided, C. Analysis/nursing diagnosis:
2- to 4-hour specimen and 24-hour urine, 1. Knowledge deficit (learning need) related to
greater than 14 mg/100 mL. planned surgery.
(3) Direct assay of catecholamines 2. Risk for physical injury related to hormone
epinephrine greater than 0.2 mg/L; imbalance.
norepinephrine 0.5 mg/L. 3. Risk for decreased cardiac output related to possi-
(4) BMR. ble hypotensive state resulting from surgery.
g. X-ray, CT, and MRIused to localize tumor 4. Risk for infection related to decreased normal
before surgery. resistance.
D. Analysis/nursing diagnosis: 5. Altered health maintenance related to need for
1. Anxiety related to excessive physiological self-administration of steroid medications, orally
stimulation of sympathetic nervous system. or by injection.
2. Fluid volume deficit related to excessive gastric D. Nursing care plan/implementation:
losses, hypermetabolic state, diaphoresis. 1. Goal: preoperative: reduce risk of postoperative
3. Risk for decreased cardiac output related to complications.
excessive secretion of catecholamines as a. Prescribed steroid therapy, given 1 week before
evidenced by hypertension. surgery, is gradually decreased; will be given
4. Risk for injury related to excessive release of again postoperatively.
epinephrine and norepinephrine. b. Antihypertensive drugs are discontinued because
5. Altered nutrition, greater than body requirements, surgery may result in severe hypotension.
related to elevated glucose. c. Sedation as ordered.
E. Nursing care plan/implementation: d. General preoperative measures (see I.
1. Goal: prevent paroxysmal hypertension. PREOPERATIVE PREPARATION,
a. Rest: reduce stress, environmental pp. 365367).
stimulation. 2. Goal: postoperative: promote hormonal balance.
b. Give sedatives, alpha-adrenergic blocker a. Administer hydrocortisone parenteral therapy
(phenoxybenzamine) for hypertension and as ordered; rate indicated by fluid and
antidysrhythmics as ordered. electrolyte balance, blood sugar, and blood
c. Diet: high vitamin, high calorie, mineral, pressure.
calcium; restrict caffeine. b. Monitor for signs of addisonian (adrenal)
d. Monitor vital signs (especially BP in sitting crisis (see IX. ADDISONS DISEASE,
and supine positions). p. 477).
2. Goal: prepare for surgical removal of tumor 3. Goal: prevent postoperative complications.
(see VIII. ADRENALECTOMY, following). a. Monitor vital signs until stability is regained;
F. Evaluation/outcome criteria: if on vasopressor drugs such as metaraminol
ADULT

1. No paroxysmal hypertension. (Aramine):


2. See VIII. ADRENALECTOMY. (1) Maintain flow rate as ordered.
VIII. ADRENALECTOMY: surgical removal of adrenal (2)Monitor BP q515 min, notify physician
glands because of tumors or uncontrolled overactivity; of significant elevations in BP (dose
also bilateral adrenalectomy may be performed to con- needs to be decreased) or drop in BP
trol metastatic breast or prostate cancer. (dose needs to be increased). Note: read-
ings that are normotensive for some may
A. Risk factors:
be hypotensive for clients who have been
1. Pheochromocytoma.
hypertensive.
2. Adrenal hyperplasia.
b. NPOattach nasogastric tube to intermittent
3. Cushings syndrome.
suction; abdominal distention is common side
4. Metastasis of prostate or breast cancer.
effect of this surgery.
5. Adrenal cortex or medulla tumors.
c. Respiratory care:
B. Assessment:
(1) Turn, cough, and deep breathe.
1. Objective data: validated evidence of:
(2) Splint flank incision when coughing.
a. Benign lesion (unilateral adrenalectomy) or
(3) Administer narcotics to reduce pain and
malignant tumor (bilateral adrenalectomy).
allow client to cough; flank incision is
b. Adrenal hyperfunction that cannot be
close to diaphragm, making coughing very
managed medically.
painful.
2164_Ch06_347-578 29/03/12 12:30 PM Page 477

Endocrine System 477


(4) Auscultate breath sounds q2h; decreased or dehydration hypotension decreased
absent sounds could indicate pneumothorax. cardiac output and resulting decrease in
(5)Sudden chest pain and dyspnea should heart size.
be reported immediately (spontaneous b. Deficiency in glucocorticoid secretion
pneumothorax). (cortisol) decrease in gluconeogenesis
d. Position: flat or semi-Fowlers. hypoglycemia and liver glycogen deficiency,
e. Mouth care. emotional disturbances, diminished resist-
f. Monitor dressings for bleeding; reinforce prn. ance to stress. Cortisol deficiency failure
g. Ambulation, as ordered. to inhibit anterior pituitary secretion of
(1) Check BP q15 min when ambulation is ACTH and melanocyte-stimulating hor-
first attempted. mone increased levels of ACTH and
(2) Place elastic stockings on lower extremities hyperpigmentation.
to enhance stability of vascular system. c. Deficiency in androgen hormone less
h. Dietonce NG tube removed, diet as axillary and pubic hair in women (testes
tolerated. supply adequate sex hormone in men, so no
4. Goal: health teaching. symptoms are produced).
a. Signs and symptoms of adrenal crisis: B. Risk factors:
(1) Pulse: rapid, weak, or thready. 1. Autoimmune processes.
(2) Temperature: elevated. 2. Infection.
(3) Severe weakness and hypotension. 3. Malignancy.
(4) Headache. 4. Vascular obstruction.
(5) Convulsions, coma. 5. Bleeding.
b. Importance of maintaining steroid therapy 6. Environmental hazards.
schedule to ensure therapeutic serum level. 7. Congenital defects.
c. Weigh daily. 8. Bilateral adrenalectomy.
d. Monitor blood glucose levels daily. 9. Tuberculosis.
e. Report undesirable side effects of steroid C. Assessment:
therapy or adrenal crisis to physician. 1. Subjective data:
f. Avoid persons with infections, due to a. Muscle weakness, fatigue, lethargy.
decreased resistance. b. Dizziness, fainting.
g. Daily schedule: include adequate rest, c. Nausea, food idiosyncrasies, anorexia.
moderate exercise, good nutrition. d. Abdominal pain/cramps.
E. Evaluation/outcome criteria: 2. Objective data:
1. Adrenal crisis avoided. a. Vital signs: decreased BP, orthostatic
a. Vital signs within normal limits. hypotension, widened pulse pressure.
b. No neurological deficits noted. b. Pulseincreased, collapsing, irregular.
2. Healing progresses: no signs of infection or c. Temperaturesubnormal.

ADULT
wound complications. d. Vomiting, diarrhea, and weight loss.
3. Adjusts to alterations in physical status. e. Tremors.
a. Complies with medication regimen. f. Skin: poor turgor, excessive pigmentation
b. Avoids infections. (bronze tone).
c. Incorporates good nutrition, periods of rest g. Laboratory data:
and activity into daily schedule. (1) Blood:
IX. ADDISONS DISEASE: chronic primary adrenal (a) Decreased: sodium (<135 mEq/L);
corticotropic insufficiency. A hormonal (endocrine) glucose (<60 mg/dL), chloride
disorder involving destruction of the adrenal glands, (<98 mEq/L), bicarbonate
which then are unable to produce sufficient adrenal (<23 mEq/L).
hormones (cortisol) necessary for the normal body (b) Increased: hematocrit, potassium
functions. (>5 mEq/L).
(2) Urine: decreased (or absent) 17-ketosteroids,
A. Pathophysiology:
17-hydroxycorticosteroids (<5 mg/24 hr).
1. Atrophy of adrenal gland is most common cause
h. Diagnostic tests:
of adrenal insufficiency; manifested by decreased
(1) CT scan, MRI.
adrenal cortical secretions.
(2) ACTH stimulation test (cortisol levels are
a. Deficiency in mineralocorticoid secretion
measured before and after administration of
(aldosterone) increased sodium excretion
synthetic ACTH).
2164_Ch06_347-578 29/03/12 12:30 PM Page 478

478 chapter 6 Physiological Integrity

D. Analysis/nursing diagnosis: 2. Medication regimen followed, is adequate for


1. Fluid volume deficit related to decreased clients needs.
sodium. 3. Adequate nutrition and fluid balance obtained.
2. Altered renal tissue perfusion related to
hypotension.
3. Decreased cardiac output related to aldosterone
H E M AT O L O G I C A L
deficiency.
SYSTEM
4. Risk for infection related to cortisol deficiency. I. IRON-DEFICIENCY ANEMIA (hypochromic
5. Activity intolerance related to muscle weakness microcytic anemia): inadequate production of red
and fatigue. blood cells due to lack of heme (iron); common in
6. Altered nutrition, less than body requirements, infants, women who are pregnant and premenopausal.
related to nausea, anorexia, and vomiting. A. Pathophysiology: decreased dietary intake,
E. Nursing care plan/implementation: impaired absorption, or increased utilization of iron
1. Goal: decrease stress. decreases the amount of iron bound to plasma
a. Environment: quiet, nondemanding transferrin and transported to bone marrow for
schedule. hemoglobin synthesis; decreased hemoglobin in
b. Anticipate events for which extra resources erythrocytes decreases amount of oxygen delivered
will be necessary. to tissues.
2. Goal: promote adequate nutrition. B. Risk factors:
a. Diet: acute phasehigh sodium, low potassium; 1. Excessive menstruation.
nonacute phaseincrease carbohydrates and 2. Gastrointestinal bleedingpeptic ulcer,
protein. hookworm, tumors.
b. Fluids: force, to balance fluid losses; monitor 3. Inadequate dietanorexia, fad diets, cultural
I&O, daily weights. practices.
c. Administer lifelong exogenous replacement 4. Poor absorptionstomach, small intestine disease.
therapy as ordered: C. Assessment:
(1)Glucocorticoidsprednisone, 1. Subjective data:
hydrocortisone. a. Fatigue: increasing.
(2) Mineralocorticoidsfludrocortisone b. Headache.
(Florinef ). c. Change in appetite; difficulty swallowing due
3. Goal: health teaching. to pharyngeal edema/ulceration; heartburn.
a. Take medications with food or milk. d. Shortness of breath on exercise.
b. May need antacid therapy to prevent GI e. Extremities: numbness, tingling.
disturbances. f. Flatulence.
c. Side effects of steroid therapy. g. Menorrhagia.
d. Avoid stress; may need adjustment in 2. Objective data:
medication dosage when stress is a. Vital signs:
ADULT

increased. (1) BPincreased systolic, widened pulse


e. Signs and symptoms of addisonian (adrenal) pressure.
crisis: very serious condition characterized (2) Pulsetachycardia.
by severe hypotension, shock, coma, and (3) Respirationstachypnea.
vasomotor collapse related to strenuous (4) Temperaturenormal or subnormal.
activity, infection, stress, omission of pre- b. Skin/mucous membranes: pale, dry; tongue
scribed medications. If untreated, could smooth, shiny, bright red; cheilosis (cracked,
quickly lead to death. painful corners of mouth).
4. Goal: prevent serious complications if addisonian c. Sclerae: pearly white.
crisis evident. d. Nails: brittle, spoon shaped, flattened.
a. Complete bedrest; avoid stimuli. e. Laboratory data: decreasedhemoglobin
b. High dose of hydrocortisone IV or (<10 g/dL blood), serum iron (<65 mcg/dL
cortisone IM. blood); increased total iron-binding capacity.
c. Treat shockIV saline. D. Analysis/nursing diagnosis:
d. I&O, vital signs q15 min to 1 hour or prn 1. Altered nutrition, less than body requirements,
until crisis passes. related to inadequate iron absorption.
F. Evaluation/outcome criteria: 2. Altered tissue perfusion related to reduction in red
1. No complications occur. cells.
2164_Ch06_347-578 29/03/12 12:30 PM Page 479

Hematological System 479


3. Risk for activity intolerance related to profound c. Shortness of breath.
weakness. d. Diaphoresis on slight exertion.
4. Impaired gas exchange related to decreased 2. Objective data:
oxygen-carrying capacity. a. Skin: pallor, jaundice.
E. Nursing care plan/implementation: b. Posture: drooping.
1. Goal: promote physical and mental equilibrium. c. Laboratory data:
a. Position: optimal for respiratory excursion; (1) Decreased hematocrit.
deep breathing; turn frequently to prevent (2) Increased reticulocyte count; bilirubin.
skin breakdown. (3) Direct Coombs test positive.
b. Rest: balance with activity, as tolerated; assist C. See I. IRON-DEFICIENCY ANEMIA for
with ambulation. Analysis/nursing diagnosis, Nursing care plan/
c. Medication (hematinics): implementation, and Evaluation/outcome criteria.
(1) Oral iron therapy (ferrous sulfate)give III. PERNICIOUS ANEMIA (megaloblastic macrocytic
with meals. anemia) lack of intrinsic factor found in gastric mucosa,
(2) Intramuscular therapy (iron dextran)use which is necessary for vitamin B12 (extrinsic factor)
second needle for injection after with- absorption; slow developing, usually after age 50; may
drawal from ampule; use Z-track method: be an autoimmune disorder.
inject 0.5 mL of air before withdrawing
A. Pathophysiology: atrophy or surgical removal of
needle, to prevent tissue necrosis; use
glandular mucosa in fundus of stomach degen-
2- to 3-inch needle; rotate sites; do not
erative changes in brain, spinal cord, and peripheral
rub site or allow wearing of constricting
nerves from lack of vitamin B12.
garments after injection.
B. Risk factors:
d. Keep warm: no hot water bottles, heating
1. Partial or complete gastric resection.
pads, due to decreased sensitivity.
2. Prolonged iron deficiency; veganism.
e. Diet: high in protein, iron, vitamins (see
3. Heredity.
Chapter 9); assistance with feeding, if
C. Assessment:
needed; nonirritating foods with mouth or
1. Subjective data:
tongue soreness.
a. Hands, feet: tingling, numbness.
2. Goal: health teaching.
b. Weakness, fatigue.
a. Dietary regimen.
c. Sore tongue, anorexia.
b. Iron therapy: explain purpose, dosage, side
d. Difficulties with memory, balance.
effects (black or green stools, constipation,
e. Irritability, mild depression.
diarrhea); take with meals.
f. Shortness of breath.
c. Activity: exercise to tolerance, with planned
g. Palpitations.
rest periods.
2. Objective data:
F. Evaluation/outcome criteria:
a. Skin: pale, flabby, jaundiced.
1. Hemoglobin and hematocrit levels return to
b. Sclerae: icterus (yellow).

ADULT
normal range.
c. Tongue: smooth, glossy, red, swollen.
2. Tolerates activity without fatigue.
d. Vital signs:
3. Selects foods appropriate for dietary regimen.
(1) BPnormal or elevated.
II. HEMOLYTIC ANEMIA (normocytic normochromic (2) Pulsetachycardia.
anemia): premature destruction (hemolysis) of erythro- e. Nervous system:
cytes; occurs extravascularly (autoimmune) or intravas- (1) Decreased vibratory sense in lower
cularly (dialysis, heart valves). extremities.
A. Risk factorsautoimmune hemolytic anemia: (2) Loss of coordination.
1. Warm reacting (idiopathic): women, lupus, (3) Babinski reflex present (flaring of toes with
rheumatoid arthritis, myeloma. stimulation of sole of foot).
2. Cold reacting (e.g., Raynauds): older women, (4) Positive Rombergs sign (loses balance when
Epstein-Barr virus. eyes closed).
3. Drug induced: methyldopa, penicillin, quinine. (5) Increased or diminished reflexes.
B. Assessment: f. Laboratory data: decreasedhemoglobin,
1. Subjective data: RBCs, platelets, gastric secretions (achlorhy-
a. Fatigue; physical weakness. dria); Schilling test (radioactive vitamin B12
b. Dizziness. urine test).
2164_Ch06_347-578 29/03/12 12:30 PM Page 480

480 chapter 6 Physiological Integrity

D. Analysis/nursing diagnosis: c. Feelings of abdominal fullness.


1. Altered nutrition, less than body requirements, d. Shortness of breath; orthopnea.
related to B12 deficiency. e. Pruritus, especially after bathing.
2. Impaired physical mobility related to numbness f. Pain: gouty-arthritic.
of extremities. 2. Objective data:
3. Fatigue related to decreased oxygen-carrying a. Skin: mucosal erythema, ruddy complexion
capacity. (reddish purple).
4. Altered oral mucous membrane related to changes b. Ecchymosis; gingival (gum) bleeding.
in gastric mucosa. c. Enlarged liver, spleen.
5. Altered thought processes related to progressive d. Hypertension.
neurological degeneration. e. Laboratory data:
E. Nursing care plan/implementation: (1) Increasedhemoglobin, hematocrit,
1. Goal: promote physical and emotional comfort. RBCs, leukocytes, platelets, uric acid.
a. Activity: bedrest or activity as tolerated (2) Decreased bone marrow iron.
restrictions depend on neurological or cardiac C. Analysis/nursing diagnosis:
involvement. 1. Altered tissue perfusion related to capillary
b. Comfort: keep extremities warmlight congestion.
blankets, loose-fitting socks. 2. Risk for injury related to dizziness, weakness.
c. Medication: vitamin B12 therapy as ordered. 3. Fluid volume excess related to mass production
d. Diet: of red blood cells.
(1) Six small feedings. 4. Risk for impaired skin integrity related to
(2) Soft or pureed. pruritus.
(3) Organ meats, fish, eggs. 5. Ineffective breathing pattern related to shortness
e. Mouth care: before and after meals, to increase of breath, orthopnea.
appetite and relieve mouth discomfort. D. Nursing care plan/implementation:
2. Goal: health teaching. 1. Goal: promote comfort and prevent complications.
a. Medication: a. Observe for signs of bleeding, thrombosis
(1) Lifelong therapy. stools, urine, gums, skin, ecchymosis.
(2) Injection techniques; rotation of sites. b. Reduce occurrence: avoid prolonged sitting,
b. Diet. knee gatch.
c. Rest; exercise to tolerance. c. Assist with ambulation.
F. Evaluation/outcome criteria: d. Position: elevate head of bed.
1. No irreversible neurological or cardiac e. Skin care: cool-water baths to decrease
complications. pruritus; may add bicarbonate of soda to
2. Takes vitamin B12 for the rest of lifeuses safe water.
injection technique. f. Fluids: force, to reduce blood viscosity
3. Returns for follow-up care. and promote urine excretion; 1,500 to
ADULT

IV. POLYCYTHEMIA VERA: abnormal increase in cir- 2,500 mL/24 hr.


culating red blood cells (myeloproliferative disorder); g. Diet: avoid foods high in iron, to reduce
considered to be a form of malignancy; occurs more RBC production.
frequently among middle-aged Jewish men. h. Assist with venesection (phlebotomy), as
ordered; 350 to 500 mL blood every other
A. Pathophysiology: unknown causes massive
day until Hct low-normal.
increases of erythrocytes, myelocytes (bone marrow
2. Goal: health teaching.
leukocytes), and thrombocytes increased blood
a. Diet: foods to avoid (e.g., liver, egg yolks);
viscosity/volume and tissue/organ congestion;
fluids to be increased.
increased peripheral vascular resistance; intravascular
b. Signs/symptoms of complications: infections,
thrombosis usually develops in middle age, particu-
hemorrhage.
larly in Jewish men; in contrast, secondary poly-
c. Avoid: falls, bumps; hot baths/showers
cythemia occurs as a compensatory response to tissue
(worsens pruritus).
hypoxia associated with prolonged exposure to high
d. Drugs: myelosuppressive agents (busulfan
altitude, chronic lung disease, and heart disease.
[Myleran], cyclophosphamide [Cytoxan],
B. Assessment:
chlorambucil, radioactive phosphorus);
1. Subjective data:
purpose; side effects.
a. Headache; dizziness; ringing in ears.
e. Procedures: venesection (phlebotomy) if
b. Weakness; loss of interest.
ordered.
2164_Ch06_347-578 29/03/12 12:30 PM Page 481

Hematological System 481


E. Evaluation/outcome criteria: (>200,000/mm3); differential is
1. Acceptance of chronic disease. important.
2. Reports at prescribed intervals for (2) RBC countnormal to severely decreased.
follow-up. (3) Hgblow or normal.
3. Remission: reduction of bone marrow activity, (4) Plateletsusually low.
blood volume and viscosity (RBC count E. Analysis/nursing diagnosis:
<6,500,000/mm3; hemoglobin (Hgb) <18 g/dL; 1. Risk for infection related to immature or
Hct <45%; WBC <10,000/mm3). abnormal leukocytes.
4. No complications (e.g., thrombi, hemorrhage, 2. Activity intolerance related to hypoxia and
gout, CHF, leukemia). weakness.
V. LEUKEMIA (ACUTE AND CHRONIC): a neo- 3. Fatigue related to anemia.
plastic disease involving the leukopoietic tissue in 4. Altered tissue perfusion related to anemia.
either the bone marrow or lymphoid areas; acute 5. Anxiety related to diagnosis and treatment.
leukemia occurs in children, young adults; chronic 6. Altered oral mucous membrane related to
forms occur in later adult life. susceptibility to infection.
7. Fear related to diagnosis.
A. Types:
8. Ineffective individual or family coping related to
1. Acute nonlymphocytic (ANLL)also known as
potentially fatal disease.
acute myelogenous leukemia (AML); seen
F. Nursing care plan/implementation:
generally in older age (>60 years).
1. Goal: prevent, control, and treat infection.
2. Acute lymphocytic (ALL)common in children
a. Protective isolation if indicated.
2 to 10 years.
b. Observe for early signs of infection:
3. Chronic lymphocytic (CLL)generally affects the
(1) Inflammation at injection sites.
elderly.
(2) Vital sign changes.
4. Chronic myelogenous (CML)also known as
(3) Cough.
chronic granulocytic leukemia (CGL); more
(4) Obtain cultures.
likely to occur between 25 and 60 years.
c. Give antibiotics as ordered.
B. Pathophysiology: displacement of normal marrow
d. Mouth care: clean q2h, examine for new
cells by proliferating leukemic cells (abnormal,
lesions, avoid trauma.
immature leukocytes) normochromic anemia,
2. Goal: assess and control bleeding, anemia.
thrombocytopenia.
a. Activity: restrict, to prevent trauma.
C. Risk factors:
b. Observe for hemorrhage: vital signs; body
1. Viruses.
orifices, stool, urine.
2. Genetic abnormalities.
c. Control localized bleeding: ice, pressure
3. Exposure to chemicals.
at least 3 to 4 minutes after needle sticks,
4. Radiation.
positioning.
5. Treatment for other types of cancer (e.g., alkylat-
d. Use soft-bristle or foam-rubber toothbrush to
ing agents).

ADULT
prevent gingival bleeding.
D. Assessment:
e. Give blood/blood components as ordered;
1. Subjective data:
observe for transfusion reactions.
a. Fatigue, weakness.
3. Goal: provide rest, comfort, nutrition.
b. Anorexia, nausea.
a. Activity: 8 hours sleep or rest; daily nap.
c. Pain: joints, bones (acute leukemia).
b. Comfort measures: flotation mattress, bed
d. Night sweats, weight loss, malaise.
cradle, sheepskin.
2. Objective data:
c. Analgesics: without delay.
a. Skin: pallor due to anemia; jaundice.
(1) Mild pain (acetaminophen [Tylenol],
b. Fever: frequent infections; mouth ulcers.
tramadol 50 mg without aspirin).
c. Bleeding: petechiae, purpura, ecchymosis,
(2) Severe pain (codeine, meperidine HCl
epistaxis, gingiva.
[Demerol]).
d. Organ enlargement: spleen, liver.
d. Diet: bland.
e. Enlarged lymph nodes; tenderness.
(1) High in protein, minerals, vitamins.
f. Bone marrow aspiration: increased presence
(2) Low roughage.
of blasts.
(3) Small, frequent feedings.
g. Laboratory data:
(4) Favorite foods.
(1)WBC countabnormally low
e. Fluids: 3,000 to 4,000 mL/day.
(<1,000/mm3) or extremely high
2164_Ch06_347-578 29/03/12 12:30 PM Page 482

482 chapter 6 Physiological Integrity

4. Goal: reduce side effects from therapeutic regimen. C. Nursing care plan/implementation:
a. Nausea: antiemetics, usually half-hour before 1. Goal: prevent complications from bleeding tendencies.
chemotherapy. a. Precautions:
b. Increased uric acid level: force fluids. (1) Injectionsuse small-bore needles; rotate
c. Stomatitis: antiseptic anesthetic mouthwashes. sites; apply direct pressure.
d. Rectal irritation: meticulous toileting, sitz (2) Avoid bumping, trauma.
baths, topical relief (e.g., Tucks). (3) Use swabs for mouth care.
5. Goal: provide emotional/spiritual support. b. Observe for signs of bleeding, petechiae fol-
a. Contact clergy if client desires. lowing blood pressure reading, ecchymosis,
b. Allow, encourage client-initiated discussion of purpura.
death (developmentally appropriate). c. Administer steroids (e.g., prednisone) with
c. Allow family to be involved in care. ITP to increase platelet count; give platelets
d. If death occurs, provide privacy for family, for count below 20,000 to 30,000/mm3 with
listening, sharing of grief. STP; high-dose immunoglobulins.
6. Goal: health teaching. 2. Goal: health teaching.
a. Prevent infection. a. Avoid traumatic activities:
b. Limit activity. (1) Contact sports.
c. Control bleeding. (2) Violent sneezing, coughing, nose blowing.
d. Reduce nausea. (3) Straining at stool.
e. Mouth care. (4) Heavy lifting.
f. Chemotherapy: regimen; side effects. b. Signs of decreased plateletspetechiae,
G. Evaluation/outcome criteria: ecchymosis, gingival bleeding, hematuria,
1. Alleviate symptoms; obtain remission. menorrhagia.
2. Prevent complications (e.g., infection). c. Use Medic Alert tag/card.
3. Ventilates emotionsaccepts and deals with anger. d. Precautions: self-medication; particularly avoid
4. Experiences peaceful death (e.g., pain free). aspirin-containing drugs.
VI. IDIOPATHIC THROMBOCYTOPENIC e. Prepare for splenectomy if drug therapy
PURPURA (ITP): potentially fatal disorder character- unsuccessful (prednisone, cyclophosphamide,
ized by spontaneous increase in platelet destruction; azathioprine [Imuran]).
possible autoimmune response; seen predominantly D. Evaluation/outcome criteria:
in 2- to 4-year-olds and girls/women 10 years old. 1. Returns for follow-up.
Remissions occur spontaneously or following splenec- 2. No complications (e.g., intracranial hemorrhage).
tomy; in contrast, secondary thrombocytopenia (STP) 3. Platelet count greater than 200,000/mm3.
is caused by viral infections, drug hypersensitivity 4. Skin remains intact.
(i.e., quinidine, sulfonamides), lupus, or bone marrow 5. Resumes self-care activities.
failure; treat cause. VII. SPLENECTOMY: removal of spleen following
A. Assessment: rupture due to acquired hemolytic anemia, trauma,
ADULT

1. Subjective data: tumor, or idiopathic thrombocytopenic purpura.


a. Spontaneous skin hemorrhageslower A. Analysis/nursing diagnosis:
extremities. 1. Risk for fluid volume deficit related to hemorrhage.
b. Menorrhagia. 2. Risk for infection related to impaired immune
c. Epistaxis. response.
2. Objective data: 3. Pain related to abdominal distention.
a. Bleeding: GI, urinary, nasal; following minor 4. Ineffective breathing pattern related to high
trauma, dental extractions. abdominal incision.
b. Petechiae; ecchymosis. B. Nursing care plan/implementation:
c. Tourniquet testpositive, demonstrating 1. Goal: prepare for surgery.
increased capillary fragility. a. Give whole blood, as ordered.
d. Laboratory data: b. Insert nasogastric tube to decrease postopera-
(1) Decreased platelets (<100,000/mm3). tive abdominal distention, as ordered.
(2) Increased bleeding time. 2. Goal: prevent postoperative complications.
B. Analysis/nursing diagnosis: a. Observe for:
1. Risk for injury related to hemorrhage. (1) Hemorrhagebleeding tendency with
2. Altered tissue perfusion related to fragile capillaries. thrombocytopenia due to decreased
3. Impaired skin integrity related to skin hemorrhages. platelet count.
2164_Ch06_347-578 29/03/12 12:30 PM Page 483

Immunological System 483


(2) Gastrointestinal distentionremoval of 2. Objective data:
enlarged spleen may result in distended a. Rasherythema migrans (stage I); at least
stomach and intestines, to fill void. 5 cm/lesion.
b. Recognize 101F temperature as normal for b. Dysrhythmias; heart block (stage II).
10 days. c. Facial paralysis (stage II).
c. Incision: splint when coughing, to prevent d. Conjunctivitis, iritis, optic neuritis (stage II).
high incidence of atelectasis (common com- e. Laboratory data: Lyme titerelevated (stages
plication), pneumonia with upper abdominal II and III). Often inconclusive.
incision. f. Diagnostic tests: isolation of Borrelia burgdorferi
3. Goal: health teaching. in tissue or body fluid; diagnostic levels of
a. Increased risk of infection postsplenectomy. IgM or IgG antibodies in serum or CSF.
b. Report signs of infection immediately. C. Analysis/nursing diagnosis:
C. Evaluation/outcome criteria: 1. Anxiety related to diagnosis.
1. No complications (e.g., respiratory, subphrenic 2. Pain related to joint inflammation.
abscess or hematoma, thromboemboli, infection). 3. Fatigue related to viral illness.
2. Complete and permanent remissionoccurs in 4. Impaired physical mobility related to joint pain.
60% to 80% of clients. 5. Altered thought processes related to neurological
deficit.
6. Decreased cardiac output related to
IMMUNOLOGICAL dysrhythmias.
SYSTEM 7. Knowledge deficit (learning need) related to
I. LYME DISEASE: a spirochetal illness (syndrome); treatment and course of disease.
most common tick-borne infectious disease in United D. Nursing care plan/implementation:
States; prevalent Northeast, upper Midwest, and 1. Goal: minimize irreversible tissue damage and
coastal northern California. Reporting is mandatory. complications.
With early treatment, recovery is usually quick and a. Medications according to presenting symp-
complete. toms: stage Ioral antibiotics for 21 days
A. Stages: (doxycycline, amoxicillin, cefotaxime); stages
1. Stage I. Rash (erythema migrans) at site of tick II and IIIoral (see stage I) or intravenous
bite; bulls-eye or target pattern; may appear as antibiotics for 21 to 28 days (ceftriaxone).
hives or cellulitis; common in moist areas (groin, b. If hospitalized, monitor vital signs q4h for
armpit, behind knees). Flu-like symptoms may increased temperature, signs of heart failure;
occur (joint pain, chills, fever). check level of consciousness and cranial nerve
2. Stage II. If untreated, may progress to cardiac functioning.
problems (10% of clients) or neurological c. Note treatment response: worsening of
disturbancesBells palsy (10% of clients); symptoms during first 24 hours: redder rash,
occasionally meningitis, encephalitis, and eye higher fever, greater pain (Jarisch-Herxheimer

ADULT
damage may result. reaction).
3. Stage III. From 4 weeks to 1 year after the tick 2. Goal: alleviate pain, promote comfort.
bite, arthritis, primarily large joint, develops a. Medications: salicylates, nonsteroidal
in half the clients. If untreated, chronic neuro- anti-inflammatory agents, or other analgesic,
logical problems may develop. as ordered; observe for side effects (GI
B. Assessment (depends on stage): History is irritation).
importantwhere do they live or work? Recent b. Rest: give instructions on relaxation tech-
travel? Outdoor activities (gardening, hiking, niques; create a quiet environment.
camping, clearing brush)? Knowledge of tick bite 3. Goal: maintain physical and psychological
and how removed? Pets? well-being.
1. Subjective data: a. Activity: ROM at regular intervals; medicate
a. Malaise (stage I). for pain before exercise; encourage proper
b. Headache (stage I). posture to reduce joint stress; rest periods
c. Joint, neck, or back pain (stages I and III). between activities and treatments.
d. Weakness (stages II and III). b. Referral: occupational or physical therapy as
e. Chest pain (stage II). appropriate.
f. Light-headedness (stage II). c. Reassurance: give psychological support;
g. Numbness, pain in arms or legs (stage III). encourage discussion of feelings.
2164_Ch06_347-578 29/03/12 12:30 PM Page 484

484 chapter 6 Physiological Integrity

4. Goal: health teaching. C. Assessment:


a. Information on disease. Transmission from 1. Subjective data:
tick not likely if removed before 48 hours of a. Fatigue: prolonged; associated with headache
attachment. or light-headedness.
b. Instructions for home IV antibiotics with b. Unexplained weight loss: greater than 10%.
heparin lock, if ordered. 2. Objective data:
c. Side effects of antibiotics (drug specific); a. Fever: prolonged or night sweats longer than
importance of completing therapy. 2 weeks.
d. Signs of disease recurrence (later stages of b. Lymphadenopathy.
disease: less severe attacks). c. Skin or mucous membrane lesions:
e. Preventing subsequent infections: wear proper purplish-red, nodules (Kaposis sarcoma).
clothing and tick repellent on clothing (20% d. Cough: persistent, heavy, dry.
to 30% DEET); conduct tick checks of self, e. Diarrhea: persistent.
children, and pets; proper tick removal (use f. Tongue/mouth thrush; oral hairy
tweezers, steady, gentle traction). leukoplakia.
f. Start vaccination series (LYMErix); three g. Diagnostic tests (with permission of client):
injections at 0, 1, and 12 months. enzyme-linked immunosorbent assay
E. Evaluation/outcome criteria: (ELISA); Western blot test.
1. Achieves reasonable comfort. h. Laboratory data: decreasedCD4 T
2. Regains normal physiological and psychological lymphocytes, hematocrit, WBCs, platelets.
functioningno irreversible complications; vital Seropositivesyphilis, hepatitis B; ELISA
signs within normal limits. positive; Western blot testpositive (mean
3. Resumes previous activity level; returns to time for seroconversion is 6 weeks after
work. infection).
4. Adheres to follow-up care recommendations. D. Analysis/nursing diagnosis:
5. Knows ways to minimize risk of reinfection. 1. Risk for infection related to immunocompromised
II. ACQUIRED IMMUNODEFICIENCY state.
SYNDROME (AIDS): the terminal stage of the 2. Fatigue related to anemia.
disease continuum caused by human immunodefi- 3. Altered nutrition, less than body requirements,
ciency virus (HIV), a retrovirus; typically progresses related to anorexia.
from asymptomatic seronegative status to asympto- 4. Impaired skin integrity related to nonhealing
matic seropositive status to subclinical immune viral lesions, Kaposis sarcoma.
deficiency to lymphadenopathy (early AIDS) to 5. Diarrhea related to infection or parasites.
AIDS-related complex (middle stage with combina- 6. Risk for activity intolerance related to shortness
tion of symptoms) to AIDS; hallmarks of HIV of breath.
infection include opportunistic infections: 7. Ineffective airway clearance related to pneumonia.
Pneumocystis jiroveci (carinii) pneumonia (PCP); 8. Visual sensory/perception alteration related to
ADULT

cytomegalovirus (CMV); Mycobacterium tuberculosis; retinitis.


hepatitis B; herpes simplex or zoster; candidiasis; 9. Risk for altered body temperature (fever) related
may take 7 to 10 years before signs and symptoms to opportunistic infections.
occur. 10. Social isolation related to stigma attached to
AIDS.
A. High-risk populations:
11. Powerlessness related to inability to control
1. Men, homosexual or bisexual (71%).
disease progression.
2. Injection drug users (IDU)/heterosexual (10%).
12. Altered thought processes related to dementia.
3. IDU/homosexual (9%).
13. Ineffective individual coping related to poor
4. People who have hemophilia and are recipients
prognosis.
of multiple transfusion (1%).
14. Risk for violence, self-directed, related to anger,
5. Heterosexual (5%).
panic, or depression.
6. Undetermined/other (4%).
E. Nursing care plan/implementation:
B. Pathophysiology: abnormal response to foreign
1. Goal: reduce risk of infection; slow disease
antigen stimulation (acquired immunity)
progression.
deficiency in cell-mediated immunityT
a. Observe signs of opportunistic infections:
lymphocytes, specifically helper T cells (CD4
weight loss, diarrhea, skin lesions, sore throat.
cells) and hyperactivity of the humoral system
b. Monitor vital signs (including temperature).
(B cells).
2164_Ch06_347-578 29/03/12 12:30 PM Page 485

Immunological System 485


c. Note secretions and excretions: changes in 4. Goal: health teaching.
color, consistency, or odor indicating infection. a. Avoidance of environmental sources of infec-
d. Diet: monitor fluid and electrolytes; strict tion (kitty litter, bird cages, tub bathing).
measurement; encourage adequate dietary b. Precautions following discharge: risk-reducing
intake (high calorie, high protein, low behaviors; condoms (latex), limit number of
bulk); 5 to 10 times recommended dietary sexual partners, avoid exposure to blood or
allowance (RDA) for water-soluble vitamins semen during intercourse.
(B complex, C); favorite foods from home; c. Family counseling; availability of community
enteral feedings. Six small meals/day. resources.
e. Protective isolation, if indicated, for severe d. Information on disease progression and life
immunocompromise. span.
f. Antiviral medications, as ordered: nucleoside e. Stress-reduction techniques: visualization,
reverse transcriptase inhibitors (e.g., zidovudine guided imagery, meditation.
[Retrovir]); nonnucleoside reverse transcriptase f. Expected side effects with drug therapy;
inhibitors (e.g., nevirapine [Viramune]); importance of compliance.
protease inhibitors (e.g., indinavir sulfate F. Evaluation/outcome criteria:
[Crixivan]); drug toxicity and numerous side 1. Relief of symptoms (e.g., afebrile, gains weight).
effects likely (rash, GI upset); large number of 2. Resumes self-care activities; returns to work;
pills and tight administration schedule; costly; improved quality of life.
potential for drug resistance. 3. Accepts diagnosis; participates in support
2. Goal: prevent the spread of disease. group.
a. Frequent hand washing, even after wearing 4. Progression of disease slows; improved survival
gloves. probability.
b. Avoid exposure to blood, body fluids of client; 5. Retains autonomy, self-worth.
wear gloves, gowns; proper disposal of needles, 6. Permitted to die with dignity.
IV catheters (see Chapter 3, Table 3.4, p. 91). III. ANIMAL-BORNE DISEASES (Table 6.29).
3. Goal: provide physical and psychological support.
a. Oral care: frequent. IV. BIOTERRORISM (Table 6.30).
b. Cooling bath: 1:10 concentration of isopropyl
alcohol with tepid water; avoid plastic-backed I N T E G U M E N TA R Y
pads if client has night sweats. SYSTEM
c. Encourage verbalization of fears, concerns
without condemnation; may suffer loss of job, I. BURNS: wounds caused by exposure to excessive
lifestyle, significant other. heat, chemicals, fire, steam, radiation, or electricity;
d. Determine status of support network: arrange most often related to carelessness or ignorance; 10,000
contact with support group. to 12,000 deaths annually; survival best at ages 15 to
e. Observe for severe emotional symptoms (sui- 30 years and in burns covering less than 20% of total
body surface.

ADULT
cidal tendencies).
f. Address issues surrounding death to ensure A. Pathophysiology:
quality of life: advance directive prepared and 1. Emergent phase (injury to 72 hours): shock due
on file; code blue status; reassurance of to pain, fright, or terror fatigue, failure of
comfort and pain control. vasoconstrictor mechanisms hypotension.

Table 6.29
Infectious Diseases: Animal-Borne
Disease Caused by Onset Symptoms Treatment
Avian influenza (bird flu) Poultrybirds excrete 2472 hr Eye infection Antiviral drug
Highly infectious virus the virus people Fever, sore throat, (Tamiflu, Relenza)
affecting millions of birds inhale fecal dust cough No vaccine
across Asia (especially Human-to-human In fatal cases:
Hong Kong, Thailand, transmission rare viral pneumonia,
Vietnam). severe respiratory
problems
Continued
2164_Ch06_347-578 29/03/12 12:30 PM Page 486

486 chapter 6 Physiological Integrity

Table 6.29
Infectious Diseases: Animal-Bornecontd
Disease Caused by Onset Symptoms Treatment
Lyme disease (see pp. 483484)
Mad cow disease Eating meat from cattle 5 or more Memory None
(bovine spongiform (brain, spinal cord, years Speech Fatal
encephalopathy [BSE]) eyes, bone marrow, abnormalities Lasts for several months
spleen), or eating con- Hallucinations
Causes variant Creutzfeldt-
taminated tissue from Incontinence
Jakob disease (VCJD): a
cattle that ate these Difficulty in dressing
CNS degenerative brain
parts (have been fed
disorder that has killed
offal)
>100 in past decade,
mostly in Britain.
Monkey pox Animal bites by or 718 days Lymphadenopathy None
Related to smallpox. bodily fluids from rats, Fever
Fatal in 10% of cases. mice, squirrels, prairie Headache
Found in midwestern U.S. dogs, or imported west Fatigue
(Illinois, Indiana, Missouri, African rodent pets Rash blisters
Wisconsin). Can be transmitted over entire body
human-to-human
Rabies Bite from infected cat, Within 24 hr Fever Postexposure
Viral disease. 7,000 cases/yr dog, or wild animal Hypersalivation prophylaxis (PEP):
in U.S. (raccoon, skunk, fox) Dysphagia immune globulin for
Partial paralysis 1 mo; rabies vaccine
Hallucinations No effective therapy after
Excitation symptoms appear
Salmonella Drinking contaminated Hours to Severe diarrhea No treatment, as resolved
Bacterial disease; kills water or eating con- 7 days Fever in 57 days
approx. 1,000/yr; taminated chicken or Abdominal pain Antibiotics if infec-
40,000 cases/yr. eggs tion spreads from
Can be transmitted intestines
by reptiles (snakes,
turtles, lizards), chicks,
ducklings
Tularemia (see Table 6.30 Bioterrorism)
West Nile virus Mosquitoes 314 days Flu-like (fever, IV fluids
Fatal in 10% of cases; first headache, body Prevention of pneumonia
found in Uganda. aches) as secondary infection
ADULT

For more information, go to the Centers for Disease Control and Prevention (CDC) Web site: www.cdc.gov.

Capillary dilation, increased permeability b. Locationfacial, perineal, and hand and foot
plasma loss to blisters, edema hemoconcen- burns have potentially more complications
tration hypovolemia hypotension because of poor vascularization.
decreased renal perfusion potential shutdown. c. Depth of burn (Table 6.31):
2. Acute phase (3 to 5 days): interstitial-to-plasma (1) First degree (superficial)epidermal tissue
fluid shift hemodilution hypervolemia only; not serious unless large areas
diuresis. involved.
B. Assessment: (2) Second degree (shallow or deep partial
1. Subjective data: how the burn occurred. thickness)epidermal and dermal tissue,
2. Objective data: hospitalization required if more than 10%
a. Extent of body surface involved: rule of of body surface involved (major burn).
nineshead and both upper extremities, (3) Third degree (full thickness)destruction of
9% each; front and back of trunk, 18% each; all skin layers; requires immediate hospital-
lower extremities, 18% each; and perineum, ization; involvement of 10% of body sur-
1%. Requires adjustment for variation in face considered major burn.
size of head and lower extremities according (4) Fourth degree (deep penetrating)muscles
to age. (fascia), bone.
(text continues on page 489)
Table 6.30
Recognizing Bioterrorism Agents and Associated Syndromes
Disease Pathophysiology Transmission and Incubation Signs/Symptoms Treatment
AnthraxGreek word Caused by Bacillus Contact with spores through Cutaneous: Skin sores that turn Ciprofloxacin or doxycycline
for coal anthracis, a bacterium break in skin black after a few days plus one or two additional
Cutaneous that is commonly found Eating contaminated meat GI: Nausea, loss of appetite, bloody antimicrobials (e.g., rifampin,
GI in grazing animals Inhaling spores diarrhea, fever, followed by bad vancomycin, penicillin, ampicillin,
Inhaled (most lethal) Forms spores that live Not spread from person to stomach pain clindamycin [60-day treatment])
in soil for years person Inhaled: Cold or flu-like symptoms; Isolation not required
Man-made form more Incubation: 16 days (inhala- cough, fatigue, chest discomfort Standard precautions
potent and resistant to tion can take up to 42 days) shortness of breath (SOB) severe
2164_Ch06_347-578 29/03/12 12:30 PM Page 487

treatment pneumonia severe respiratory dis-


tress: diaphoresis, stridor, cyanosis,
shock; death in 2436 hours
Botulisma A potent neurotoxin Foodborne; ingestion of Vision: double, blurred Botulinum antitoxin from
neuroparalytic illness produced from toxin produced in food Eyelids: drooping CDC
Clostridium botulinum, (home-canned is most Speech: slurred If survive, have SOB and
an anaerobic, spore- frequent) Mouth: dry fatigue for years
forming bacterium Fatal in 5% of cases Muscles: weakness, symmetrical
Characterized by: sym- flaccid paralysis
metrical, descending Swallowing: dysphagia
flaccid paralysis of If untreated, descending paraly-
motor and autonomic sis of respiratory muscles, arms and
nerves legs; death from respiratory failure
Always begins with
cranial nerves
Plague Caused by bacterium Pneumonic: Pneumonic: Give antibiotics within 24 hr
Pneumonic Yersinia pestis (Y. pestis); Infects lungs Fever of symptomsstreptomycin,
Bubonic found in rodents and Airborne transmission Headache gentamicin, tetracyclines,
Septicemic their fleas Spreads person to person Weakness chloramphenicol
Easily destroyed by Signs of pneumonia for 24 days Prophylactic antibiotics for
sunlight and drying (SOB, chest pain, cough, bloody or 7 days
Will survive up to 1 hr watery sputum) respiratory failure Close-fitting surgical mask to
shock protect against infection
Bubonic: Bubonic:
Most common form Swollen lymph glands (buboes)
Infected flea bites the skin Fever
Does not spread person to Headache
person Chills
Weakness
Septicemic: Septicemic:
Often complication of Fever
pneumonic or bubonic Chills
Does not spread from Prostration
Immunological System

person to person Abdominal pain


Shock
Bleeding into skin and organs
487

Continued

ADULT
ADULT

Table 6.30 488


Recognizing Bioterrorism Agents and Associated Syndromescontd
Disease Pathophysiology Transmission and Incubation Signs/Symptoms Treatment
SmallpoxLatin word An acute contagious Aerosolized; droplets Prodrome 24 days (sometimes No specific treatment
for spotted disease caused by vari- Direct face-to-face, pro- contagious): flu-likemalaise, fever Smallpox vaccine effective if
Variola major (severe ola virus (a member of longed contact (101104F), headache, body given within 3 days of
and most common) the orthopoxvirus family) Incubation: 721 daysnot aches, sometimes vomiting exposure
Variola minor contagious at this time Early rash about 4 days (most con- Cidofovir possibly effective
Lethal in 20%40% of cases tagious): small red spots on tongue Ribavirin
if unvaccinated and mouth; erythema spreads from Victim of attack should be
face/arms to legs, then centrally undressed, shower with soap
Pustular rash: macules (bellybutton For visible contamination, use 0.5%
2164_Ch06_347-578 29/03/12 12:30 PM Page 488

center) papules pustular diluted household bleach


vesicles scabs
Tularemia (rabbit A bacterial zoonosis Infects humans through: Skin and oral ulcers Medications:
fever) Caused by Francisella skin, mucous membranes, Fever (sudden) Streptomycin (drug of choice)
chapter 6 Physiological Integrity

tularensis, one of the GI tract, and lungs Chills Gentamicin (an alternative)
most infectious patho- Abrupt onset Headache Doxycycline and ciprofloxacin in
genic bacteria Lower fatality rate than Rigor mass casualty
Survives for weeks at plague or anthrax Generalized body aches (low back) Isolation not recommended
low temperatures in: Aerosol release would be Coryza In hospitals, standard precautions
water, moist soil, hay, likely in terrorist attack Sore throat, dry or slightly are recommended
straw, and decaying Incubation: 114 days productive cough
animal carcasses Not transmitted person to Substernal pain or tightness
person Atypical pneumonia
Pleuritis
Hilar lymphadenopathy
Viral hemorrhagic Four distinct RNA Infected host or vector ini- VHF: marked fever, fatigue, dizzi-
fever (VHF) (multisystem viruses: arenaviruses, tially transmits to human; ness, muscle aches, loss of
syndrome) filoviruses, bunyaviruses, then human to human with strength, exhaustion. Bleeding:
Ebola (severe, often and flaviviruses Ebola or Lassa. under skin, organs, mouth, eyes,
fatal) Often animal or insect ears (rarely fatal)
Lassa host, except for Ebola Ebola: Ebola: fever, headache, red eyes, hic- Ebola:
Hantavirus (host unknown) 221 days; abrupt onset cups, joint/muscle aches, sore throat Supportive therapyfluids,
pulmonary syndrome Damaged vascular diarrhea, vomiting, stomach pain electrolytes, O2
system bleeding Lassa: Lassa: 80% may have no symptoms Lassa:
Spread through contact with or mild. May be varied; fever, ret- Ribavirin
body fluids rosternal pain, sore throat, back pain, Supportive care
13 wk cough, abdominal pain, vomiting,
diarrhea, conjunctivitis, facial bleeding
Hantavirus: Hantavirus: Hantavirus:
Carried by rodents (mice) Earlyfatigue, fever, muscle aches, No specific cure
Aerosolization of virus headache, nausea, vomiting, Early admission to ICUintubation
shed in urine, droppings abdominal pain and ventilator support
and saliva Late410 days; cough, SOB, chest
No person-to-person tightness, feeling of suffocation
transmission
15 wk after exposure
For more information, see www.bioterrorism.uab.edu
2164_Ch06_347-578 29/03/12 12:30 PM Page 489

Integumentary System 489

Table 6.31
Burn Characteristics According to Depth of Injury
Classification Tissue Damage Appearance Pain Clinical Course
Superficial (first degree) Epidermis Mild to fiery red ery- Very painful Ordinarily heals in
thema; no blisters 37 days
Partial thickness; superficial Epidermis and dermis Superficial: mottled, Superficial: Healing takes
or deep (second degree) moist, pink or red; extreme pain and 1018 days; if infec-
may or may not hypersensitivity to tion develops in deep
blanch with pres- touch burn, it converts to
sure; usually blisters full-thickness burn
Deep: dry-white or Deep: may or may Deep: often grafted
deep red not be painful
Full thickness (third degree) All layers of skin and Charred, leathery or Usually absent Heals only with graft-
subcutaneous tissue pale and dry ing or scarring
*Fourth degree All layers of skin, Black Same as full Same as full thickness
subcutaneous tissue, thickness
muscle, and bone
*Used in some classification systems. May require several days after a severe burn to determine fourth degree.

d. Indications of airway burns (e.g., singed nasal (3) Strict intake and output; hourly for
hair, progressive hoarseness, sooty expectora- 36 hours with large burns.
tion); edema may occur in 1 hour; increased (4) Check hematocrit: normal: men greater than
mortality rate. 40%; women greater than 37%. Increased
e. Poorer prognosisinfants, due to immature Hct with intravascular fluid depletion.
immune system and effects of fluid loss; elderly, (5) Weigh daily.
due to degenerative diseases and poor healing. 2. Goal: prevent physical complications.
f. Medical historypresence of hypertension, a. Vital signs: hourly; central venous pressure
diabetes, alcohol abuse, or chronic obstructive (CVP) for signs of shock or fluid overload
pulmonary disease increases complication rate. with clients who are at-risk.
C. Analysis/nursing diagnosis: b. Assess respiratory function (particularly with
1. Impaired skin integrity related to thermal injury. head, neck burns); patent airway; breath
2. Pain (depending on type of burn) related to sounds.
exposure of sensory receptors. c. Give medications as orderedtetanus booster;
3. Fluid volume excess or deficit related to hemody- antibiotics to treat documented infection;
namic changes. sedatives and analgesics; antipyreticsavoid

ADULT
4. Risk for infection related to destruction of protec- aspirin; H2 blockers.
tive skin. d. Isolation: protective; contact isolation (hand
5. Impaired gas exchange related to airway injury. washing, protective clothing).
6. Body image disturbance related to scarring, e. Positioning: turn q2h; prevent contractures.
disfigurement. (1) Head and neck burnsuse pillows under
7. Ineffective individual or family coping related to shoulders only for hyperextension of neck.
traumatic experience. (2) Hand burnssplints.
D. Nursing care plan/implementation: (3) Arm and hand burnskeep arms at
1. Goal: alleviate pain, relieve shock, and 90-degree angle from body and slightly
maintain fluid and electrolyte balance. above shoulders.
a. Medications: give opioid analgesic and (4) Ankle and foot burnssplints; elevate to
anxiolytics incrementally. prevent edema.
b. Fluids: IV therapy (see Chapter 8); colloids, (5) Splints to maintain functional positions.
crystalloids, or 5% dextrose according to burn (6) ROM exercises according to therapy
formula. guidelines; usually several times per day;
c. Monitor hydration status: active exercises most beneficial.
(1) Insert indwelling catheter. f. Diet: begin oral fluids at once; food as
(2) Note: color, odor, and amount of urine toleratedhigh protein, high calorie for
hourly. energy and tissue repair (promote positive
2164_Ch06_347-578 29/03/12 12:30 PM Page 490

490 chapter 6 Physiological Integrity

nitrogen balance); enteral feedings if protein (2) Intermediate periodreactions associated


and calorie goals not met. with pain, dependency, depression, anger;
g. Observe for: constriction (circumferential or give medications to decrease pain; explain
chest wall burns); check peak inspiratory procedures; have open, nonjudgmental
pressure in client who is intubatedreport attitude; use consistent approaches to care;
increased pressure; check pulses in burned contract with client regarding division of
extremities every 1 to 2 hours for 24 hours responsibilities; encourage self-care.
report loss of pulses. (3) Recuperative periodgrief process reacti-
3. Goal: promote emotional adjustment and provide vated. Anxiety, depression, anger, bargain-
supportive therapy. ing, as client tries to cope with altered
a. Care by same personnel as much as possible, body image, leaving security of hospital,
to develop rapport and trust. finances. Encourage verbalization; refer to
b. Involve client in care plans. support group to assist adaptation.
c. Answer questions clearly, accurately. 4. Goal: promote wound healingwound care:
d. Encourage family involvement and a. Open methodexposure of burns to air;
participation. useful in burns of face; thin layer (2 to 4 mm)
e. Provide diversional activities and change fur- topical antimicrobial ointment applied.
nishings or room adornments when possible, b. Closed methoddressings applied to burned
to prevent perceptual deprivation related to areas, changed 1 to 3 times/day; give PO pain
immobility. medication 30 minutes before change; IV pain
f. Point out signs of progress (e.g., decreased medication during dressing change; tubbing
edema, healing) because client and family facilitates removal.
tend to become discouraged and cannot see c. Multiple dressing changecommon approach;
progress. dressings changed twice daily to q4h depend-
g. Encourage self-care to highest level tolerated. ing on wound condition.
h. Anticipate psychological changes: d. Topical antimicrobial therapies (Table 6.32).
(1) Acute periodsevere anxiety: medicate e. Tubbing and dbridement.
with anxiolytics as ordered; maintain eye (1) Hydrotherapybody-temperature bath
contact; explain procedures. water; loosens dressings so some float off;

Table 6.32
Topical Antimicrobials Used in Burn Care
Agent Advantages Disadvantages Nursing Implications
Silver sulfadiazine 1% Broad-spectrum antimicrobial Less eschar penetration than Check for allergy to sulfa
(Silvadene)
ADULT

Antifungal mafenide acetate (Sulfamylon)


Nonstaining Decreased granulocyte forma-
Relatively painless tion; transient leukopenia
No systemic metabolic abnormalities Macular rash
Mafenide acetate Eschar penetration Severe pain and burning Administer pretreatment
(Sulfamylon) Effective with Pseudomonas sensation (lasts 30 min) analgesic
cream or solution Topical of choice for electrical burns Metabolic acidosis Monitor for metabolic aci-
Suitable for open or closed method Carbonic anhydrase inhibitor dosis and hyperventilation
of treatment (cream) Ineffective against fungi Check for allergy to
Used for gram-negative organisms May cause hypersensitivity rash sulfa; observe for rash
Silver nitrate Low cost Continuous wet soaks Check serum
Broad spectrum Superficial penetration electrolytes daily
Effective with Candida Black staining of sheets Rewet dressing to keep
Stinging moist
Electrolyte imbalances (low
sodium, low chloride, low cal-
cium, low potassium), alkalosis
Petroleum- and mineral Bactericidal, gram-positive and Limited ability to penetrate Apply in thin layer (1 mm)
oilbased (e.g., Bacitracin, gram-negative organisms eschar Reapply as needed.
Neosporin, Polysporin) Painless
Prevents drying of wound
2164_Ch06_347-578 29/03/12 12:30 PM Page 491

Integumentary System 491


soak 20 to 30 minutes; encourage limb to granulation bed; face, hands, and
exercises; do not leave unattended; loss of arms grafted first.
body heat may occur, with chilling and (c) Postskin-graft care:
poor perfusion resulting. (i) Sheet grafts: roll cotton-tipped
(2) Removal of eschar (dbridement)done applicator over graft to remove
with forceps and curved scissors; medicate excess exudate; maintain dressings;
for pain before; use sterile technique; aseptic technique; mesh grafts:
only loose eschar removed, to prevent irrigate as ordered.
bleeding; examine wound for: infection, (ii) Third to fifth daygraft takes on
color change, decreased granulation pink appearance if it has taken.
report changes immediately. Chemical (iii)Padding, then splints applied to
dbridement also done; agent digests immobilize grafted extremities.
necrotic tissue. (iv)Pressure garments worn up to
f. Wound coverage, to decrease chances of 18 months to decrease hyper-
infection: trophic scarring.
(1) Temporary and semipermanent wound 5. Goal: health teaching.
coverings (Table 6.33). a. Mobility needs: exercise; physical therapy;
(2) Autograftclient donates skin for wound occupational therapy; splints, braces.
coverage. b. Community resources: mental health practi-
(a) Typesfree grafts (unattached to donor tioner or psychotherapist if needed for
site) and pedicle grafts (attached to problems with self-image or sexual role;
donor site). referrals as needed.
(b) Proceduregeneral anesthesia; donor c. Techniques to camouflage appearance: slacks,
sites shaved and prepared; graft applied turtlenecks, long sleeves, wigs, makeup.

Table 6.33
Wound Coverings
Example Advantages Disadvantages Nursing Implication
Biological
Xenograft Promotes healing of clean Easily digested by Overlap edges slightly; trim away
wound; relieves pain; readily wound collagenase; when skin underneath has healed
available; reduces water and allergic reactions
heat loss
Homograftcadaver skin Reduces water and heat loss; May harbor disease Observe for signs of infection
relieves pain; used with antimi-

ADULT
crobial mesh; may be left in
place until rejection occurs
Amnion Relieves pain; reduces Limited shelf life; Change cover dressing q48h; leave
water and heat loss; has requires special prepara- on wound until it sloughs
bacteriostatic properties tion for use; may harbor
diseases
Biosynthetic (Temporary)
Biobrane Protects from microbial Not effective for preparing Must be secured to skin with
penetration; decreases pain; a granulation bed sutures, closure straps, tape, or
promotes healing in partial- staples; wrap with gauze; after
thickness burn 48 hr, check for adherence; once
adherence has occurred, may be
left open to air; check for signs of
infection
Biosynthetic (Semipermanent)
Integra Artificial Skin Postexcisional treatment of Infection rate lower than Maintain integrity of covering;
life-threatening full or deep autograft 1820 days before thin autograft
partial-thickness burns when applied.
autograft not available Avoid hydrotherapy; observe for
infection.
2164_Ch06_347-578 29/03/12 12:30 PM Page 492

492 chapter 6 Physiological Integrity

E. Evaluation/outcome criteria: 2. Objective data: physical effects of immobility:


1. Return of vital signs to preburn levels. a. Cardiovascular.
2. Minimal to no hypertrophic scarring. (1) Orthostatic hypotension.
3. Free of infection; demonstrates wound care. (2) Increased cardiac load.
4. Maintains functional mobility of limbs; no (3) Thrombus formation.
contractures. b. Gastrointestinal.
5. Adjusts to changes in body image; no (1) Anorexia.
depression. (2) Diarrhea.
6. Regains independence; returns to work, social (3) Constipation.
activities. c. Metabolic.
(1) Tissue atrophy and protein catabolism.
(2) BMR reduced.
M U S C U L O S K E L E TA L (3) Fluid/electrolyte imbalances.
SYSTEM d. Musculoskeletal.
I. IMMOBILITY: Impaired physical mobility or limita- (1) Demineralization (osteoporosis).
tion of physical movement may be accompanied by a (2) Contractures and atrophy.
number of complications that can involve any or all of (3) Skin breakdown.
the major systems of the body. Regardless of the cause e. Respiratory.
of immobilization, there are a number of conditions (1) Decreased respiratory movement.
that arise primarily as a complication of immobility. (2) Accumulation of secretions in respiratory
These are discussed in Table 6.34. tract.
A. Types of immobility: (3) O2/CO2 level imbalance.
1. Physicalphysical restriction due to limitation f. Urinary.
in movement or physiological processes (1) Calculi.
(e.g., breathing). (2) Bladder distention, stasis.
2. Intellectuallack of action due to lack of knowl- (3) Infection.
edge (e.g., mental retardation, brain damage). (4) Frequency.
3. Emotionalimmobilized when highly stressed D. Analysis/nursing diagnosis:
(e.g., after loss of loved person or diagnosis of 1. Impaired physical mobility related to specific
terminal illness). client condition.
4. Socialdecreased social interaction due to sepa- 2. Impaired skin integrity related to physical
ration from family when hospitalized or when immobilization.
alone, as in old age. 3. Urinary retention related to incomplete emptying
B. Risk factors: of bladder.
1. Pain, trauma, injury. 4. Constipation related to inactivity.
2. Loss of body function or body part. 5. Risk for disuse syndrome related to lack of range
3. Chronic disease. of motion.
ADULT

4. Emotional, mental illness; neglect. 6. Bathing/hygiene self-care deficit related to


5. Malnutrition. musculoskeletal impairment.
6. Bedrest, traction, surgery, medications. 7. Sensory/perceptual alteration related to complica-
C. Assessment: tions of immobility.
1. Subjective data: psychological/social effects of 8. Body image disturbance related to physical
immobility: limitations.
a. Decreased motivation to learn; decreased E. Nursing care plan/implementation:
retention. 1. Goal: prevent physical, psychological hazards.
b. Decreased problem-solving abilities. a. Apply nursing measures to promote venous
c. Diminished drives; decreased hunger. flow, muscle strength, endurance, joint
d. Changes in body image, self-concept. mobility, skin integrity.
e. Exaggerated emotional reactions, inappropri- b. Assess and counteract psychological impact of
ate to situation or person; aggression, apathy, immobility (e.g., feelings of helplessness,
withdrawal. hopelessness, powerlessness).
f. Deterioration of time perception. c. Help maintain accurate sensory processing
g. Fear, anxiety, feelings of worthlessness related to prevent and lessen sensory disturbances.
to change in role activities (e.g., when no d. Help adapt to altered body image due to
longer employed). increased dependency, sensory deprivation,
(text continues on page 499)
Table 6.34
Complications of Immobilization
Analysis/Nursing Nursing Care Plan/ Evaluation/Outcome
Disorder Pathophysiology Assessment Diagnosis Implementation Criteria
Orthostatic A decrease in BP Subjective data: Decreased cardiac Prevent trauma resulting from Client tolerates increased
hypotension >30/15 caused by weakness; dizziness output related to sudden decrease in BP activity
failure of vasomotor Objective data: orthostatic 1. Change position gradually No trauma occurs
responses to compen- decreased BP >30/15 hypotension 2. Elastic stockings BP remains within normal
sate for change from a measured 2 min after Risk for injury related 3. Leg exercises limits
2164_Ch06_347-578 29/03/12 12:30 PM Page 493

recumbent to an upright moving from a supine to to vertigo 4. Dangle before getting up


position a sitting or standing Activity intolerance 5. Tilt table
position; loss of muscle related to dizziness 6. Sitting and lying BP
tone and strength; client 7. Monitor side effects of drugs
may faint Health teaching
1. Explain signs and symptoms to
client
2. Encourage client to dangle
before standing
3. Encourage slow movement from
sitting to standing
4. Exercises to maintain muscle
tone
Cardiac When the body is Subjective data: fear; Risk for injury related Prevent injury and further ischemic No complications noted
overload recumbent, some of apprehension to increased workload damage to cardiac tissue by Client tolerates increased
the total blood volume Objective data: Valsalva of heart decreasing workload of heart activity
that would be in the maneuver (pressure Activity intolerance 1. Out of bed in chair when Heart rate within normal
legs as a result of gravity against the closed glot- related to increased possible limit
is redistributed to other tis when breath is held) workload of heart 2. Semirecumbent position
parts of the body, 1020 times/hr, when Fear related to when in bed; pillows
thereby increasing the trying to move in bed; tachycardia between legs when
circulating volume and tachycardia; decreased side-lying
increasing the workload exercise tolerance 3. Exercises: passive and active
of the heart; heart rate, ROM, isometric
which is decreased 4. Encourage participation in
because blood is pre- self-care
vented from entering 5. Turn every 2 hr, dangle
the thoracic vessels by 6. Avoid Valsalva, fatigue
pressure from the 7. Minimize constipation
Valsalva maneuver, 8. Encourage slow, deep breathing
increases when normal when moving in bed
breathing resumes Health teaching
1. Exhale while turning; do not hold
breath
Musculoskeletal System

2. Measures to conserve energy


Continued
493

ADULT
ADULT

494

Table 6.34
Complications of Immobilizationcontd
Analysis/Nursing Nursing Care Plan/ Evaluation/Outcome
Disorder Pathophysiology Assessment Diagnosis Implementation Criteria
Thrombus Mass of blood con- Subjective data: discom- Altered peripheral tis- Prevent injury by reducing risk No thromboemboli
formation stituents formed in the fort over involved vessel sue perfusion related factors and venous stasis Note: If Homans sign pres-
heart or blood vessels Objective data: increased to obstructed vessel 1. Position: change q12h ent (discomfort behind knee
due to pooling of blood RBC count; venous Risk for injury related 2. Do not gatch bed (causes on forced dorsiflexion of the
from lack of activity; stasis; hypercoagulability to emboli pressure against leg vessels) foot) see Nursing care for
increased viscosity 3. Increase fluid intake client with thromboemboli,
2164_Ch06_347-578 29/03/12 12:30 PM Page 494

related to dehydration or 4. Monitor coagulation laboratory pp. 372, 434


possible external values
pressure 5. Medications: anticoagulation
therapy, as prescribed for
clients at risk (immobilized,
chapter 6 Physiological Integrity

trauma, low pelvic surgery)


6. Ambulate as soon as possible
Health teaching
1. How to recognize signs of
thrombophlebitis/thromboemboli
2. Leg exercise program to strength-
en muscles for improved tone,
to prevent pooling of blood in
vessels
3. Precautions necessary when on
anticoagulation therapy
4. Side effects of anticoagulation
therapy (bleeding from gums,
body fluids, obvious bleeding)
Respiratory Decreased thoracic Subjective data: Ineffective breathing Prevent complications related to No respiratory complications
congestion movement due to: dyspnea; pain pattern related to respiratory status or excess secretions noted
related to restriction against bed or Objective data: trauma; splinting to reduce 1. Maintain a clear airway, assist
decreased chair, lack of position immobilization of thorax pain with ventilation prn
respiratory change, restrictive cloth- or abdomen, due to posi- Ineffective airway 2. Remove or minimize causes of
movements ing or binders/bandages, tion in bed; inability to clearance related to dyspnea
or abdominal distention cough or deep breathe; retained secretions 3. Conserve clients energy (periods
abdominal distention Impaired physical of rest and activityclient able
mobility related to to cough more effectively when
trauma rested)
4. Incentive spirometry
Promote comfort
1. Maintain hydration and nutrition
2. Position: change q2h; out
of bed in chair when
possible (chest expansion
greater when sitting in chair)
Health teaching
1. Methods to allay anxieties
precipitated by dyspnea
2. Effective breathing and coughing
exercises
Respiratory Inability of cilia to move Subjective data: Ineffective airway Prevent atelectasis, infection, stasis No respiratory
congestion normal secretions out of dyspnea; pain clearance related to of air, and secretions in lungs complications
related to bronchial tree due to: Objective data: pooled secretions 1. Maintain patent airway; cough; Client coughs and removes
pooled ineffective coughing, lack dehydration; Impaired gas suction; change position secretions
secretions of thoracic expansion, or drugs exchange related to 2. See nursing care plan for
effects of medications anticholinergic, ineffective coughing Respiratory congestion related to
CNS depressants, decreased respiratory move-
anesthesia; inade- ments (above)
quate coughing; Health teaching
stationary position
2164_Ch06_347-578 29/03/12 12:30 PM Page 495

1. Effective coughing techniques


2. Importance of adequate hydration
Oxygen/carbon Imbalance in oxygen and Subjective data: confu- Impaired gas Promote improved respirations No respiratory
dioxide carbon dioxide levels sion, irritable, restless, exchange related to 1. Change position frequently complications
imbalance related to pulmonary dyspnea immobilization 2. Increase humidification Respiratory rate and depth
congestion, ineffective Objective data: hypoxia, 3. Monitor side effects of are adequate for maintain-
breathing patterns, hypercapnia, cyanosis administered medication, ing balance of oxygen and
trauma, or effects of especially narcotics, barbiturates carbon dioxide
medications 4. See nursing care plan for
Respiratory congestion related to
decreased respiratory
movements (p. 494)
Malnutrition of Lack of adequate dietary Subjective data: anorexia, Altered nutrition, less Improved nutritional intake to main- No complications
adult who is intake to maintain nausea; diet history vali- than body require- tain basal metabolism requirements Client obtains/maintains
immobilized healthy tissue related to dating lack of adequate ments, related to and replace losses from catabolism normal weight
lack of food; lack of nutritional intake; mental decreased appetite 1. Provide balanced or No tissue breakdown
knowledge about food; irritability Knowledge deficit prescribed diet, soft or
problems with ingestion, Objective data: (learning need) related ground food if cannot chew
digestion, or absorption; 1. Recent weight loss of to nutrition or is edentulous
or psychosocial factors >10% requirements 2. Increase fluid intake
that influence clients 2. Decreased: healing 3. Attain/maintain normal weight
motivation to eat ability, GI motility, 4. Feed, assist with feeding, or
absorption, secretion place foods within clients reach
of digestive enzymes Promote comfort
3. Appearance: listless- 1. Mouth care; to facilitate
ness, muscle weak- mastication of food improved
ness; posturesagging digestion and absorption
shoulders, sunken chest 2. Relieve constipation (see
4. Anthropometric data nursing care plan for
(measurement of size, Constipation, p. 496)
weight, and body 3. Observe for stomatitis, bleeding,
proportions): <85% of changes in skin texture, color
standard 4. Medications: monitor
Musculoskeletal System

5. Cardiovascular: nausea and vomiting side


tachycardia (>100 beats/ effects of prescribed med-
min) on minimal ications; administer antiemetics
exertion; bradycardia as ordered to control nausea and
495

at rest vomiting
Continued
ADULT
ADULT

Table 6.34
496

Complications of Immobilizationcontd
Analysis/Nursing Nursing Care Plan/ Evaluation/Outcome
Disorder Pathophysiology Assessment Diagnosis Implementation Criteria
6. Hair: brittle, dry, thin 5. Ambulate to alleviate flatulence
7. Skin: dry, scaly and distention
8. Lack of financial 6. Alleviate pain and discomfort by:
resources: sociocultu- distractions, increased social
ral influences interactions, pleasant environ-
9. Decreased blood ment, back rubs, and administra-
values: serum albu- tion of prn pain medications, as
2164_Ch06_347-578 29/03/12 12:30 PM Page 496

min, iron-binding ordered


capacity, lymphocyte Health teaching
levels, hematocrit, 1. Diet and elimination
and hemoglobin 2. See Chapter 9 for foods
high in protein and
chapter 6 Physiological Integrity

carbohydrate
Constipation Waste material in the Subjective data: discom- Constipation related Promote normal pattern of bowel Client has normal bowel
bowel is too hard to pass fort, pain, distress, and to decreased water elimination elimination pattern
easily; or bowel move- pressure in the rectum; and fiber intake 1. Administer: stool softeners No impactions
ments are so infrequent reported decrease in Knowledge deficit or bulk cathartics as Increases fluid and fiber
that client has discomfort normal elimination (learning need) related ordered; oil retention, soap in diet
pattern to dietary and exer- suds enemas as ordered
Objective data: immobi- cise requirements to 2. Encourage change of position
lization; hard formed prevent constipation and activity as tolerated
stool, possible palpable 3. Provide high-bulk diet
impaction; decreased 4. Increase fluid intake
bowel sounds; bowel 5. Provide for privacy
elimination less frequent 6. Encourage regular time for
than usual evacuation
Health teaching
1. Dietary instructions
regarding increased fiber
2. Exercise program as
tolerated
3. Increase fluids
Osteoporosis Metabolic bone disorder Subjective data: Pain related to bone Prevent injury related to decreased No fractures
in which there is a backache fractures or body struc- bone strength No renal calculi
generalized loss of bone Objective data: deminer- tural changes 1. Position: correct body Incorporates dietary
density due to an alization of bone seen alignment, firm mattress improvements in daily
imbalance between on x-ray; kyphosis; 2. Encourage self-care menu selection
bone formation and bone spontaneous fracture activities: plan maximum activity Participates in exercise
resorption; immobiliza- of bone (hip, spine, allowed by physical condition; program on a regular basis
tion can cause wrist); collapsed verte- muscle exercises against Regular bone density tests
calcium losses of brae; loss of height; resistance as tolerated (12 years for ages
200300 mg/day stooped posture 3. Rest/activity pattern: encourage 4065+)
ROM exercise; avoid fatigue
Risk factors: women, 4. Weight-bearing positions,
family history; post- tilt table
menopause, thin and/or 5. Diet: high protein, high
small frame, anorexia or vitamin D, calcium rich
bulimia, diet low in calci- 6. Increase fluids to prevent
um; use of corticos- renal calculi (calcium from bones
teroids and anticonvul- could cause kidney stones)
sants; inactive lifestyle; Health teaching
cigarette smoking, 1. Dietary instructions: foods to
excessive use of alcohol include for high-protein, high
vitamin D, high-calcium diet
2164_Ch06_347-578 29/03/12 12:30 PM Page 497

2. Exercise program
3. Sign and symptoms of renal
calculi
4. Avoid smoking, alcohol
Contractures Abnormal shortening of Subjective data: pain Impaired physical Prevent deformities ROM maintained
muscle tissue, rendering Objective data: mus- mobility related to 1. Active or passive ROM No deformities noted
the muscle highly resist- clesfixed, shortened, muscle weakness and 2. Positioning: functional,
ant to stretching; related decreased tone; resist- contractures correct alignment
to lack of active or pas- ance of muscles to Pain related to injury 3. Footboard to prevent
sive ROM, or improper stretch; decreased ROM Self-care deficit relat- footdrop
support and positioning in affected limb ed to immobility 4. Avoid knee gatch
of joints affected by Health teaching
arthritis or injury 1. Importance of ROM
2. Correct anatomical
positions
Skin breakdown Presence of risk factors Subjective data: fatigue; Impaired skin integrity Prevent skin breakdown No skin breakdown
that could lead to skin pain; inability to turn related to lack of fre- 1. Change position q12h
breakdown, such as: on own quent position change and prn, out of bed when
immobility, inadequate Objective data: interrup- possible
nutrition, lack of position tion of skin integrity, 2. Protect from infection
changes especially over: ears, 3. Increase dietary intake:
occiput, heels, sacrum, protein, carbohydrates
scrotum, elbows, 4. Increase fluids
trochanter, ischium, Assess for/reduce contributing
scapula; immobilization; factors known to cause decubitus
malnutrition ulcers: incontinence, stationary
position, malnutrition, obesity,
sensory deficits, emotional distur-
bances, paralysis
Promote healing
1. Wash gently, pat dryto avoid
skin abrasion
2. Clean, dry, wrinkle-free bed
Musculoskeletal System

linens and pads


Continued
497

ADULT
ADULT

498

Table 6.34
Complications of Immobilizationcontd
2164_Ch06_347-578 29/03/12 12:30 PM Page 498

Analysis/Nursing Nursing Care Plan/ Evaluation/Outcome


Disorder Pathophysiology Assessment Diagnosis Implementation Criteria
3. Massage skin with lotion that
does not contain alcohol (alcohol
chapter 6 Physiological Integrity

dries skin)
4. Protect with: wafer barrier, alter-
nating mattress, sheepskin pads,
protectors, flotation devices
5. No doughnuts or rubber rings
(interfere with circulation of tis-
sue within center of ring)
Urinary stasis Immobility leads to Subjective data: pain, Altered urinary elimina- Prevent urinary infections, stasis, No urinary infections or
inability to completely due to infection or renal tion related to inability and renal calculi evidence of renal calculi;
empty the bladder, which calculi to empty bladder 1. Increase activity as allowed bladder emptied, no urinary
increases risk for urinary Objective data: difficulty 2. Check for distended bladder stasis
tract infection and renal in urinating due to posi- 3. Increase fluids, I&O
calculi tion or lack of privacy; 4. Diet: acid-ash to increase
infection related to acidity, thereby preventing
catheter insertion or infection
stasis of urine; 5. Avoid catheterization; use inter-
hematuria mittent catheterization instead of
Foley whenever possible or
Creds maneuver to empty blad-
der (manual exertion of pressure
on the bladder to force urine out)
6. Bladder training
2164_Ch06_347-578 29/03/12 12:30 PM Page 499

Musculoskeletal System 499


and changes in status and power that 2. Interest in self and environment; positive
accompany immobility. self-image.
e. Offer counseling when sexual expression is 3. Returns to optimal level of physical activity.
impaired. II. FRACTURES: disruptions in the continuity of bone
2. Goal: health teaching: how to prevent physical as the result of trauma or various disease processes,
problems related to immobility (e.g., anticonsti- such as Cushings syndrome or osteoporosis, that
pation diet, range of motion, skin care); teach weaken the bone structure.
activities while immobile that encourage inde-
A. Types (Fig. 6.13):
pendence and provide sensory stimulation.
1. Open or compoundfractured bone extends
F. Evaluation/outcome criteria:
through skin and mucous membranes; increased
1. Minimal contractures, skin breakdown, muscle
potential for infection.
atrophy, or loss of strength.

FRAGMENTS FRAGMENTS SEPARATED FRAGMENTS SEPARATED


UNDISPLACED DUE TO BREAK FROM BY EXTERNAL FORCE
WITHIN SUCH AS BULLET
(COMPOUND FRACTURE) (COMPOUND FRACTURE)

PROXIMAL
PORTION OF
BONE

MIDDLE
PORTION OF
BONE

DISTAL
PORTION
OF BONE

ADULT
DISPLACED INCOMPLETE COMPLETE
GREENSTICK

HAIR-
LINE
COMMINUTED SEGMENTAL BUTTERFLY SPIRAL

Figure 6.13 Types of fractures and terminology.


(From Venes, D [ed]: Tabers Cyclopedic Medical Dictionary,
TYPES OF FRACTURES AND TERMINOLOGY ed 21. FA Davis, Philadelphia, 2009.)
2164_Ch06_347-578 29/03/12 12:30 PM Page 500

500 chapter 6 Physiological Integrity

2. Closed or simplefractured bone does not pro- protrudes through skin on both sides of
trude through skin. the extremity. Skeletal traction for frac-
3. Completefracture extends through entire bone, tured vertebrae accomplished with tongs
disrupting the periosteum on both sides of the (e.g., Crutchfield tongs, Gardner-Wells
bone, producing two or more fragments. tongs) (see Fig. 6.14, parts F through H).
4. Incompletefracture extends only partway d. Specific types of traction:
through bone; bone continuity is not totally (1) Cervicaldirect traction applied to
interrupted. cervical vertebrae using a head halter or
5. Greenstick or willow-hickory stickfracture of Crutchfield, Gardner-Wells, or Vinke tongs
one side of bone; other side merely bends; usually that are inserted into the skull (see
seen only in children. Fig. 6.14, parts C and G). Traction is
6. Impacted or telescopedfracture in which bone increased with weights until vertebrae
fragments are forcibly driven into other or adja- move into position and alignment is
cent bone structures. regained. After reduction is obtained,
7. Comminutedfracture having more than one weights are decreased to the amount
fracture line and with bone fragment broken needed to maintain reduction. Weight
into several pieces. amount is prescribed by physician.
8. Depressedfracture in which bone or bone frag- (2) Balanced suspensioncountertraction pro-
ments are driven inward, as in skull or facial duced by a force other than clients body
fractures. weight; extremity is suspended in a trac-
9. See also VI. TOTAL HIP REPLACEMENT, tion apparatus that maintains the line of
p. 510. traction despite changes in the clients
B. Methods used to reduce/immobilize fractures: position (e.g., Russells leg traction, Thomas
reduction or setting of the bonerestores bone splint with Pearsons attachment) (see
alignment as nearly as possible. Fig. 6.14, parts E and F).
1. Closed reductionmanual traction or manipula- (3) Runningtraction that exerts a pull in one
tion. Usually done under anesthesia to reduce plane; countertraction is supplied by the
pain and muscle spasm. Maintenance of reduc- weight of the clients body or can be
tion and immobilization is accomplished by increased through use of weights and
casting (fiberglass or plaster of Paris). pulleys in the opposite direction
2. Open reductionoperative procedure utilized (e.g., Bucks extension, Russells traction)
to achieve bone alignment; pins, wires, nails, (see Fig. 6.14, parts B through E).
or rods may be used to secure bone fragments (4) Haloan apparatus that employs both
in position; prosthetic implants may also a plastic and metal frame; molded frame
be used. extends from axilla to iliac crest and houses
3. Traction reductionforce is applied in two a metal frame. The struts of the frame
directions: to obtain alignment, and to reduce extend to skull and attach to round metal
ADULT

or eliminate muscle spasm. Used for fractures (halo) device. The halo is attached to skull
of long bones. May be: by four pinstwo located anterolaterally
a. Continuousused with fractures or disloca- and two located posterolaterally. They
tions of bones or joints. are inserted into external cortex of the
b. Intermittentused to reduce flexion contrac- cranium (see Fig. 6.14, part H). Used to
tures or lessen pain and muscle spasm. immobilize the cervical spine following
c. Applied as follows: spinal fusion, give some correction to
(1) Skintraction applied to skin by using a scoliosis before spinal fusion, and immobi-
commercial foam-rubber Bucks traction lize nondisplaced fracture of spine.
splint or by using adhesive, plastic, or a 4. Immobilizationmaintains reduction and
moleskin strip bound to the extremity promotes healing of bone fragments.
by elastic bandage; exerts indirect traction Achieved by:
on bone or muscles (e.g., Bryants, a. External fixation:
Bucks extension, head, pelvic, Russells) (1) Caststypes:
(Fig. 6.14, parts A through E). (a) Spicaapplied to immobilize hip or
(2) Skeletaldirect traction applied to bone shoulder joints.
using pins (Steinmann), wires (Kirchner). (b) Body castapplied to trunk.
Pin is inserted through the bone in or (c) Arm or leg castjoints above and
close to the involved area and usually below site included in cast.
2164_Ch06_347-578 29/03/12 12:30 PM Page 501

Musculoskeletal System 501

SKIN

A. Bryant's traction B. Buck's traction C. Head halter traction

D. Pelvic traction E. Russell's traction

ADULT
SKELETAL

F. Balanced suspension traction G. Crutchfield tongs H. Halo vest


Figure 6.14 Types of skin and skeletal traction.
2164_Ch06_347-578 29/03/12 12:30 PM Page 502

502 chapter 6 Physiological Integrity

(2) Splints, continuous traction. b. Provide activities to reduce perceptual


(3) External fixation devices (Charnley) deprivationreading, handcrafts, music,
multiple pins/rods through limb above special interests/hobbies that can be done
and below fracture site, attached to exter- while maintaining correct position for healing.
nal metal supports. Client able to become 2. Goal: prevent injury or trauma in relation to:
ambulatory. a. Fracture care:
b. Internal fixationpins, wires, nails, rods (see (1) Maintain affected part in optimum
VI. TOTAL HIP REPLACEMENT, p. 510, alignment.
and VII. TOTAL KNEE REPLACEMENT, (2) Maintain skin integrity; check all bony
p. 512). prominences for evidence of pressure q4h
C. Assessment: and prn, depending on amount of pressure.
1. Subjective data: (3) Monitor: circulation in, sensation of, and
a. Pain, tenderness. motion of (CSM) affected part q15 min
b. Tingling, numbness. for first 4 hours; q1h until 24 hours; q4h
c. Nausea. and prn, depending on amount of edema
d. History of traumatic event. (Table 6.36).
e. Muscle spasm. (4) Maintain mobility in unaffected limb and
2. Objective data: unaffected joints of affected limb by active
a. Function: abnormal or lost. and passive ROM; prevent footdrop by
b. Deformities. using ankle-top sneakers.
c. Ecchymosis, increased heat over injured b. Skin traction:
part. (1) Maintain correct alignment:
d. Localized edema. (a) If tape or moleskin is used, shave
e. Crepitation (grating sensations heard or felt extremity and apply benzoin to improve
as bone fragments rub against each other). adherence of strip and reduce itching.
f. Signs of shock. (b) Check apparatus for slippage, bunch-
g. Indicators of anxiety. ing; replace prn.
h. X-ray: fracturepositive interruption of bone; (2) Prevent tissue injury:
dislocationabnormal position of bone. (a) Check all bony prominences for evi-
D. Analysis/nursing diagnosis: dence of pressure: q15 min for first
1. Pain related to interruption in bone. 4 hours; q1h until 24 hours, q4h and
2. Impaired physical mobility related to fracture prn, depending on amount of edema.
treatment modality. (b) Nonadhesive (e.g., Bryants) traction
3. Risk for injury related to complications of may be removed q8h to check skin.
fractures. c. Skeletal traction:
4. Knowledge deficit (learning need) regarding cast (1) Maintain affected part in optimum
care, crutch walking, traction. alignment:
ADULT

5. Constipation related to immobilization. (a) Ropes on pulleys.


6. Risk for impaired skin integrity related to (b) Weights hang free.
immobility or friction from materials used to (c) Elevate head of bed as prescribed.
immobilize the fracture during healing. (d) Check knots routinely.
E. Nursing care plan/implementation: (2) Maintain skin integrity:
1. Goal: promote healing and prevent complications (a) Frequent skin care.
of fractures (Table 6.35). (b) Keep bed linens free of crumbs and
a. Diet: high protein, iron, vitamins to improve wrinkles.
tissue repair; moderate carbohydrates to pre- (3) Prevent infection: special skin care to pin
vent weight gain; no increase in calcium to insertion sites three times daily. Keep area
prevent kidney stones (decalcification and around pins clean and dry. Use prescribed
demineralization occur when client is solution for cleansing.
immobilized). (4) Monitor circulation in, sensation of,
(1) Encourage increased fluid intake, to prevent and motion of affected part (see E. 2. a.
kidney stones. Fracture care (see above), and Table 6.36,
(2) Prevent or correct constipation through p. 506).
increasing bulk foods, fruits, and fruit juices, (5) Maintain mobility in unaffected limb and
or using prescribed stool softeners, laxatives, unaffected joints; prevent footdrop of
or cathartics as necessary. affected limb.
(text continues on page 506)
Table 6.35
Complications of Fractures
Analysis/Nursing Evaluation/
Complication Assessment Diagnosis Nursing Care Plan/Implementation Outcome Criteria
Shock (see pp. 427428)
Thrombophlebitis (see pp. 433434)
Fat emboli: serious, potentially Subjective data: dyspnea, severe Risk for injury related to 1. Position: high Fowlers to relieve Client alert
life-threatening complication in chest pain; confusion, agitation; fat emboli respiratory symptoms Pain relieved
which pressure changes in decrease in level of consciousness; Altered tissue perfusion 2. Administer oxygen STAT, to relieve Respiratory,
interior of fracture force mole- numbness; feeling faint; history of related to fat emboli anoxia and reduce surface tension of fat cardiac, and
2164_Ch06_347-578 29/03/12 12:30 PM Page 503

cules of fat from marrow into diabetes, obesity globules neurological


systemic circulation; may cause Objective data: cyanosis; pupillary 3. Institute respiratory support measures, systems have no
problems in respiratory or changes; muscle twitching; as orderedIPPB, respiratory assistive permanent
nervous system; seen most petechiaechest, buccal cavity, devices: be prepared for CPR in event damage
frequently on third day after axilla, conjunctiva, soft palate; of respiratory failure
multiple fractures, fractures of extremitiespallor, cold; shock; 4. Monitor vital signs, cardiac monitor,
long bones, or comminuted vomiting q15 min during acute episode and prn
fracture (shock/cardiac failure possible)
5. Obtain baseline data and monitor level of
consciousness, neurological signs q15 min
during acute episode and prn (neurological
involvement possible)
6. Administer parenteral fluids, as
ordered: IV alcohol, blood and fluid
replacements
7. Administer medications as ordered:
corticosteroids; digitalis; amino-
phylline; heparin sodium
8. DO NOT RUB ANY LEG CRAMPS, BUT
REPORT IMMEDIATELY
Nerve compression: pressure Subjective data: discomfort, pain, Pain related to pressure 1. Monitor for potential signs q1h for first Sensation, motor
on nerve in affected area from referred pain; burning, tingling, on nerve 48 hr; neurovascular assessment q12h function are
edema, dislocation of bone, or stinging sensation; numbness, Potential for physical and prn as condition indicates (circulation, normal
immobilization apparatus; if altered sensation, inability to injury related to pres- sensation, and motion [CSM]) No complications
pressure not relieved, perma- distinguish touch sure on nerve 2. Elevate affected limb; flex hand or noted
nent paralysis can result Objective data: limited movement; Impaired tissue foot of affected extremity; passive
muscle weakness; paralysis; reflex- perfusion related to and active ROM exercises
esdiminished, irritable, or absent; impaired circulation 3. Be prepared to cut cast or remove con-
color changes related to impaired Impaired physical strictions if signs of impairment exist
circulation mobility related to joint 4. Begin active ROM exercises to unaffected
contracture, numbness extremities
5. Use footboard to prevent footdrop
6. Encourage use of trapeze if applicable
Musculoskeletal System

7. Isometric exercises, as ordered


8. Ambulation, weight bearing as ordered,
support casts
503

Continued

ADULT
ADULT

504

Table 6.35
Complications of Fracturescontd
Analysis/Nursing Evaluation/
Complication Assessment Diagnosis Nursing Care Plan/Implementation Outcome Criteria
Avascular necrosis/ Subjective data: tenderness; pain, Risk for altered 1. Monitor for potential signs q1h for first Circulation
circulatory impairment: especially on passive motion peripheral tissue 48 hr; blanching, coolness, edema; palpate adequate to limb,
2164_Ch06_347-578 29/03/12 12:30 PM Page 504

interference with normal circu- Objective data: edema, swelling in perfusion related to pulse above and below injury, report absent to prevent tissue
lation to affected area due to affected area; decreased color, vessel damage pulse or major discrepancies STAT damage
interruption of blood vessel, temperature, mobility; bleeding 2. Elevate affected limb to decrease
pressure on the vessel from from wound edema
chapter 6 Physiological Integrity

dislocation, edema, or immobi- 3. Report to physician if signs persist


lization devices; results of 4. Be prepared to assist with bivalving of
impaired circulation lead to dis- casts, or cut cast to relieve pressure
comfort and, if not corrected, 5. Monitor size of drainage stains on casts;
necrosis of tissue and bone measure accurately and report if size
due to lack of oxygen supply increases
Infection Subjective data: pain Risk for injury related to 1. Monitor vital signs, drainage No infection or
Objective data: elevated temperature tissue destruction 2. Ensure client has had prophylactic tetanus heals with no seri-
and pulse: erythemadiscoloration Altered peripheral tis- toxoid ous complications
of surrounding skin; edemasudden, sue perfusion related to 3. May have prophylactic antibiotics
local induration; drainagethin, swelling ordered if wound was contaminated
watery, foul-smelling exudate; at time of injury
crepitus (may be indicative of gas 4. Instruct client not to touch open wound or
gangrene); with castwarm area, pin sites or put anything inside cast (could
foul smell interrupt skin integrity and become
potential source of infection)
Delayed union/nonunion: Subjective data: pain Risk for injury related to 1. Maintain immobilization and alignment of Bone heals
failure of bone to heal within Objective data: lack of callus poor healing of bone affected limb No complications
normal time related to lack of formation on x-ray: poor alignment fracture 2. Maintain adequate nutrition noted
use, inadequate circulation, Impaired physical mobil- 3. Avoid trauma to affected limb Pain decreased
other complicating medical ity related to lower-limb 4. Monitor for circulatory or infection Ambulation and
conditions such as diabetes fractures complication self-care return to
or poor nutrition Dressing/grooming 5. Dietary instructions regarding foods preinjury status
bathing/hygiene, containing calcium and protein
self-care deficit related necessary for bone healing
to upper-limb fracture
Skin breakdown (related to Subjective data: pain Impaired skin integrity 1. If open wound: verify tetanus administra- No skin
cast) Objective data: temperature and related to cast trauma tion; monitor site through cast window, breakdown
pulse elevated; erythema; edema change dressing daily and prn
cast edges, exposed distal portion of 2. Apply lotion or cornstarch to exposed skin
limb, limb area within cast; drainage (no powder)
and foul odor from break in skin 3. Petal-tape edges of cast to reduce
(may be under cast and stain through irritation
2164_Ch06_347-578 29/03/12 12:30 PM Page 505

or exit at ends of cast); crepitus 4. Inspect skin for irritation, edema, odor,
(crackling sound could indicate gas drainageq2h initially, then q3h
gangrene); hyperactive reflexes 5. Instruct client not to place any object
under cast because skin abrasions may
lead to decubitus ulcers
6. Promote drying of cast by leaving it
uncovered and exposed to air for 48 hr;
use no plastic
7. Prevent indenting casts with fingertips or
hard surface: place on pillows; use palms
of hands when positioning affected limb
8. Avoid excessive padding of Thomas
splint in groin areapadding traps
moisture, may lead to skin breakdown
Duodenal distress (with spica Subjective data: anorexia, nausea, Ineffective breathing 1. Place on firm mattress; use bedboards if Complications
cast): spica cast incorporates abdominal pain related to pressure necessary to reduce muscle spasm avoided or
the trunk and affected limb and Objective data: duodenal distress, from cast 2. Maintain warmth by covering uncasted detected early
can cause respiratory or abdom- vomiting, distention, cast too tight Pain related to areas enough to
inal distress when edema is abdominal distress 3. Avoid turning for first 8 hr; when turning: prevent serious
present under the cast or cast from pressure use enough personnel to logroll; do not damage
is too tight to allow for normal Fear related to cast use bar between legs as turning device;
body functions constriction support chest with pillows
4. Monitor for signs of respiratory distress:
increased respirations, apprehension
5. Monitor for signs of duodenal
distress: vomiting, distention; if
these signs occur: place in prone
position; have cast bivalved; may need NG
tube; monitor for fluid imbalance
6. Protect cast with nonabsorbent material
during elimination
Musculoskeletal System
505

ADULT
2164_Ch06_347-578 29/03/12 12:30 PM Page 506

506 chapter 6 Physiological Integrity

Table 6.36 (2) Position: maintain body alignment.


(3) Keep tongs free from bed, and keep
Assessing Injured Limb: CSM weights hanging freely to allow traction to
C Circulation function properly.
Is it warm to touch? g. Halo traction:
Are both limbs equal in size? (1) Several times a day, check screws to the
Are peripheral pulses present?
Is there adequate capillary refill?
head and screws that hold the upper
portion of the frame, to determine correct
S Sensation
Does the client feel pain?
position.
Can the client distinguish different sensations (2)Pin sites cleansed three times daily
(e.g., painful stimuli vs. soft touch)? with bacteriostatic solution to prevent
Is the client aware of the position of the limb? infection.
M Motion (3) Monitor for signs of infection.
Can the client move extremities that are not (4) Position as any other client in body cast,
immobilized? except no pressure to rest on halopillows
Can the client do this independently? may be placed under abdomen and chest
Can the client do this on command?
when client is prone.
(5) Institute ROM exercises to prevent
contractures.
d. Running traction (see Fig. 6.14, p. 501): (6) Turn frequently to prevent development
(1) Keep well centered in bed. of pressure areas.
(2) Elevate head of bed only to point of (7) Allow client to verbalize about having
countertraction. screws placed in skull.
(3) No turning from side to sidewill cause (8) Postapplication nursing care same as pin
rubbing of bony fragments. insertion for other traction.
(4) Check distal circulation frequently. h. External fixation devices:
(5) Frequent back care to prevent skin (1) Pin care same as for skeletal traction.
breakdown. (2) Teach clothing adjustment.
(6) Fracture bedpan for toileting. (3) Teach to adjust for size of apparatus.
(7) Avoid excessive padding of splints in groin i. Internal fixation devices:
area to prevent tissue trauma. (1) Monitor for signs of infection/allergic
e. Balanced suspension traction (see Fig. 6.14, reaction to materials used for maintenance
part F, p. 501): of reduction (drainage, pain, increased
(1) Maintain alignment and countertraction: temperature).
(a) Ropes on pulleys. (2)Position as ordered to prevent
(b) Weights hang free. dislocation.
(c) Elevate head of bed as prescribed. j. Casts:
ADULT

(d) Check knots routinely. (1) Support drying cast on firm pillow; avoid
(2) May move client, but turn only slightly finger imprints on cast.
(no more than 30 degrees to unaffected side). (2) Elevate limb to reduce edema.
(3) Heel of affected leg must remain free of (3)Prevent complications of fractures as
the bed. listed.
(4) 20-degree angle between thigh and bed. (4) Closely monitor circulation (blanching,
(5) Check for pressure from sling to popliteal swelling, decreased temperature); sensation
area. (absence of feeling; pain or burning); and
(6) Provide foot support to prevent footdrop. motion (inability to move digits of affected
(7) Maintain abduction of extremity. limb).
(8) Check for signs of infection at pin (5) Be prepared to notify physician or cut
insertion sites; cleanse three times daily cast if circulatory impairment occurs.
as ordered. (6) Protect skin integrity: avoid pressure of
(9) If tape or moleskin is used, shave extremity edges of cast; petal prn.
and apply benzoin to improve adherence (7) Monitor for signs of infection if skin
of strip and reduce itching. integrity impaired.
f. Cervical traction (see Fig. 6.14, p. 501): 3. Goal: provide care related to ambulation with
(1) May be placed on specialized bed crutches.
(e.g., Stryker frame). a. Teach appropriate gait (Table 6.37).
2164_Ch06_347-578 29/03/12 12:30 PM Page 507

Musculoskeletal System 507


b. Measure crutches correctly (Table 6.38). (3) Handgrip should be adjusted so that
(1) Subtract 16 inches from total height; top of complete wrist extension is possible.
crutch should be 2 inches below the axilla. (4) Instruct in correct body alignment:
(2) Complete extension of the elbows should (a) Head erect.
be possible without pressure of axilla bar (b) Back straight.
into the axilla. (c) Chest forward.
(d) Feet 6 to 8 inches apart, wide base for
support.
Table 6.37 4. Goal: provide safety measures related to possible
Teaching Crutch Walking complications following fracture (see Table 6.35).
A. When only one leg can bear weight:
5. Goal: health teaching.
1. Swing-to gait: crutches forward; swing body to crutches. a. Explain and show apparatus before applica-
a. Move both crutches forward. tion, if possible.
b. Move both legs to meet the crutches. b. Pin care at least once daily to prevent granula-
c. Continue pattern. tion and cellulitis.
2. Swing-through gait: crutches forward; swing body
through crutches.
c. Correct position for rest/sleep and prevention
a. Move both crutches forward. of injury with halo tractionno pressure on
b. Move both legs farther ahead than crutches. halo.
c. Continue pattern. d. Purpose of cast: to immobilize, to support
3. Three-point gait: crutches and affected extremity body tissues, to prevent or correct deformities.
forward; swing forward, placing nonaffected foot ahead
of or between crutches.
e. Teach signs and symptoms of complications to
a. Both crutches and affected limb move at same time. report related to cast care (i.e., numbness, odor,
b. Move both crutches and affected leg (e.g., left) ahead crack/break in cast; extremity cold, bluish).
6 inches. f. Isometric exercises for use with affected joint.
c. Move unaffected leg (e.g., right) to same place as left g. Safety measures with crutches:
and crutches.
d. Continue pattern.
(1) Weight-bearing on hands, not axilla.
B. When both legs can move separately and bear some weight: (2) Position crutches 4 inches to side and
1. Four-point gait: right crutch forward, left foot forward; swing 4 inches to front.
weight to right side while bringing left crutch forward, then (3) Use short strides, looking ahead, not at feet.
right foot forward; gait simulates normal walking. (4) Prevent injury: if client begins to fall,
a. Move right crutch forward 46 inches.
b. Move left foot forward same distance as right crutch.
throw crutches to side to prevent falling
c. Move left crutch forward ahead of left foot. on them; body should be relaxed.
d. Move right foot forward to meet right crutch. (5) Check for environmental hazards: rugs,
e. Continue pattern. water spills.
2. Two-point gait: same as four-point gait but faster; one F. Evaluation/outcome criteria:
crutch and opposite leg moving forward at same time.
a. Opposite crutch and limb move together.
1. No injury or complications related to apparatus
b. Move right crutch and left leg ahead 6 inches. or immobilization (e.g., infection, tissue injury,

ADULT
c. Move left crutch and right leg ahead. altered circulation/sensation, dislocation).
d. Continue pattern. 2. Bone remains in correct alignment and begins
C. When client is unable to walk: tripod gait: crutches forward to heal.
at a wide distance; drag legs to point just behind crutches,
balance, and repeat.
3. Demonstrates elevated limb position to relieve
edema with casted extremity.
4. Lists complications related to circulation or
neurological impairment and infection.
Table 6.38 5. Begins to use affected part.
Measuring Crutches Correctly 6. Demonstrates correct technique for ambulation
1. Have client lie on a flat surface. Measure from anterior fold
with crutchesno pressure on axilla, uses
of axilla to 4 inches lateral to heel. strength of arms and wrists.
2. Have client stand. Measure from 12 inches below axilla to 7. No falls while using crutches.
2 inches in front of and 6 inches to the side of the foot.
3. Hand placement on bar of crutch: have client stand upright,
III. COMPARTMENT SYNDROME: an accumulation
support body weight with hand on bar (not of fluid in the muscle compartment, resulting in an
putting weight on axilla). Elbow flexion should be increase in pressure that reduces blood flow to the tis-
30 degrees. sues. Can lead to neuromuscular deficit, amputation,
4. Slightly pad the shoulder rests of the crutches for general and death.
comfort.
5. Make sure there are nonskid rubber tips on the crutches. A. Pathophysiology: inability of the fascia surround-
ing the muscle group to expand to accommodate
2164_Ch06_347-578 29/03/12 12:30 PM Page 508

508 chapter 6 Physiological Integrity

the increased volume of fluid compartment 2. Goal: prevent complications.


pressure increases venous flow impaired a. Elevate injured extremity initially; if ischemia
arterial flow continues, increasing capillary pressure suspected, keep extremity at heart level to
fluid pushed into the extravascular space prevent compensatory increase in blood flow.
intracompartment pressure further increased b. Avoid tight bandages, splints, or casts.
prolonged or severe ischemia muscle and nerve c. Monitor intravenous infusion for signs of
cells destroyed, contracture, loss of function, infiltration.
necrotic tissue, infection, release of potassium, d. Prepare client for fasciotomy (incision of skin
hydrogen, and myoglobin into bloodstream. and fascia to release tight compartment).
B. Risk factors: F. Evaluation/outcome criteria:
1. Fractures. 1. Relief from pain; normal perfusion restored.
2. Burns. 2. Neurovascular status within normal limits.
3. Crushing injuries. 3. Retains function of limb; no contractures or
4. Restrictive bandages. infection.
5. Cast. 4. Compartment pressure returns to normal
6. Prolonged lithotomy positioning. (<20 mm Hg).
7. Ischemic injury (arterial or venous injury). 5. No systemic complications (e.g., normal cardiac
C. Assessment: and renal function, acid-base balance within
1. Subjective data: normal limits).
a. Severe, unrelenting pain, unrelieved by IV. OSTEOARTHRITIS: joint disorder characterized
narcotics and associated with passive by degeneration of articular cartilage and formation
stretching of muscle. of bony outgrowths at edges of weight-bearing joints.
b. Paresthesias.
A. Pathophysiology: excessive friction combined
2. Objective data:
with risk factors thinning of articular cartilage,
a. Edema; tense skin over limb.
narrowing of joint space, and loss of joint stability;
b. Paralysis.
cartilage erodes, producing shallow pits on articular
c. Decreased or absent peripheral pulses.
surface and exposing bone in joint space. Bone
d. Poor capillary refill.
responds by becoming denser and harder.
e. Limb temperature change (colder).
B. Risk factors:
f. Ankle-arm pressure index (API) decreased;
1. Aging (>50).
0.4 indicates ischemia (see Chapter 11,
2. Rheumatoid arthritis.
Doppler ultrasonography, p. 828).
3. Arteriosclerosis.
g. Urine outputdecreased (developing
4. Obesity.
acute tubular necrosis); reddish-brown
5. Trauma.
color.
6. Family history.
D. Analysis/nursing diagnosis:
C. Assessment:
1. Pain related to tissue swelling and ischemia.
1. Subjective data:
ADULT

2. Risk for injury related to neuromuscular


a. Pain; tender joints.
deficits.
b. Fatigability, malaise.
3. Impaired physical mobility related to contracture
c. Anorexia.
and loss of function.
d. Cold intolerance.
4. Risk for infection related to tissue necrosis.
e. Extremities: numbness, tingling.
5. Altered urinary elimination related to acute
2. Objective data:
tubular necrosis from myoglobin accumulation.
a. Joints:
6. Body image disturbance related to limb
(1) Enlarged.
disfigurement.
(2) Stiff, limited movement.
E. Nursing care plan/implementation:
(3) Swelling, redness, and heat around affected
1. Goal: recognize early indications of ischemia.
joint.
a. Assess neurovascular status frequently (q1h):
(4) Shiny stretched skin over and around
skin temperature, capillary refill, peripheral
joint.
pulses, mobility, and sensation.
(5) Subcutaneous nodules.
b. Listen to client complaints; report suspected
b. Weight loss.
complications.
c. Fever.
c. Report nonrelief of pain with narcotics.
d. Crepitation (creaking or grating of joints).
d. Recognize unrelenting pain with passive
e. Deformities, contractures.
muscle stretching.
2164_Ch06_347-578 29/03/12 12:30 PM Page 509

Musculoskeletal System 509


f. Cold, clammy extremities. f. Rest.
g. Laboratory data: decreased Hgb, elevated g. Assistive devices to decrease weight-bearing
WBC count. of affected joints (canes, walkers).
h. Diagnostic tests: x-ray, thermography, 2. Goal: health teaching to promote independence.
arthroscopy. a. Encourage self-care with assistive devices for
D. Analysis/nursing diagnosis: activities of daily living (ADLs).
1. Pain related to friction of bones in joints. b. Activity, as tolerated, with ambulation-assistive
2. Bathing/hygiene self-care deficit related to devices.
decreased mobility of involved joints. c. Scheduled rest periods.
3. Risk for injury related to fatigability. d. Correct body posture and body mechanics.
4. Impaired physical mobility related to stiff, limited 3. Goal: provide for emotional needs.
movement. a. Accept feelings of frustration regarding
5. Impaired home maintenance management related long-term debilitating disorder.
to contractures. b. Provide diversional activities appropriate
E. Nursing care plan/implementation: for age and physical condition to promote
1. Goal: promote comfort: reduce pain, spasms, comfort and satisfaction.
inflammation, swelling. F. Evaluation/outcome criteria:
a. Medications as prescribed. 1. Remains independent as long as possible.
(1) Nonsteroidal antiinflammatory agents: 2. No contractures.
aspirin (Ecotrin), acetaminophen (Tylenol), 3. States comfort has increased.
ibuprofen (Motrin), indomethacin 4. Uses methods that are successful in pain
(Indocin), corticosteroids, nabumetone control.
(Relafen), naproxen (Naprosyn). V. RHEUMATOID ARTHRITIS: chronic, systemic,
(2) Antimalarials: chloroquine (Aralen), collagen inflammatory disease; etiology unknown;
hydroxychloroquine (Plaquenil), to relieve may be autoimmune, viral, or genetic; affects prima-
symptoms. rily women ages 20 to 40 years; present in 2% to
b. Heat to reduce muscle spasms, stiffness. 3% of total population; follows a course of exacer-
c. Cold to reduce swelling and pain. bations and remissions.
d. Prevent contractures:
A. Pathophysiology: synovitis with edema prolif-
(1) Exercise.
eration of various blood material (formation of
(2)Bedrest on firm mattress during
pannus) destruction and fibrosis of cartilage
attacks.
(fibrous ankylosis); calcification of fibrous tissue
(3) Splints to maintain proper alignment.
(osseous ankylosis) (Fig. 6.15).
e. Elevate extremity to reduce swelling.

ADULT
WRIST BONE
INVOLVEMENT

METACARPO-
PHALANGEAL
JOINT

PROXIMAL
INTERPHALANGEAL
JOINTS OF HAND MOST JOINTS
SEVERE FORM WITH
COMMONLY AFFECTED
IN RHEUMATOID ARTHRITIS
ULNAR DEVIATION AND Figure 6.15 Rheumatoid arthritis. (From Venes, D
PRESENCE OF SUBCUTANEOUS
(HIPS AND KNEES ARE NODULES [ed]: Tabers Cyclopedic Medical Dictionary, ed 21. FA Davis,
ALSO COMMONLY AFFECTED) Philadelphia, 2009.)
2164_Ch06_347-578 29/03/12 12:30 PM Page 510

510 chapter 6 Physiological Integrity

B. Assessment: 2. No side effects from drug therapy


1. Subjective data: (e.g., GI bleeding).
a. Joints: pain; morning stiffness; swelling. 3. Copes with necessary lifestyle changes;
b. Easily fatigues; malaise. complies with treatment regimen.
c. Anorexia; weight loss. VI. TOTAL HIP REPLACEMENT: femoral head and
2. Objective data: acetabulum are replaced by a prosthesis, which is
a. Subcutaneous nodules over bony prominences. cemented into the bone with plastic cement. Performed
b. Bilateral symmetrical involvement of joints: to replace a joint with limited and painful function due
crepitation, creaking, grating. to bony alkalosis and deformity, caused by degenerative
c. Deformities: contractures, muscle atrophy. joint disease or when vascular supply to femoral head is
d. Laboratory data: blood: decreased compromised from a fracture. Goal of the surgery: restore
hemoglobin/hematocrit; RBCs; increased or improve mobilization of hip joint and prevent com-
WBCs (12,000 to 15,000), sedimentation plications of extended immobilization.
rate (>20 mm/hr), rheumatoid factor.
A. Risk factors:
Positive antinuclear antibody titer.
1. Rheumatoid arthritis.
C. Analysis/nursing diagnosis:
2. Osteoarthritis.
1. Pain related to joint destruction.
3. Complications of femoral neck fractures
2. Impaired physical mobility related to joint
avascular necrosis and malunion (Table 6.39).
contractures.
4. Congenital hip disease.
3. Risk for injury related to the inflammatory
B. Analysis/nursing diagnosis:
process.
1. Risk for injury related to implant surgery.
4. Body image disturbance related to joint
2. Knowledge deficit (learning need) regarding joint
deformity.
replacement surgery.
5. Self-care deficit related to musculoskeletal
3. Impaired physical mobility related to major hip
impairment.
surgery.
6. Risk for activity intolerance related to fatigue and
4. Pain related to surgical incision.
stiffness.
5. Risk for impaired skin integrity related to immobility.
7. Altered nutrition, less than body requirements,
C. Nursing care plan/implementation:
related to anorexia and weight loss.
1. Preoperative:
8. Self-esteem disturbance related to chronic illness.
a. Goal: prevent deep vein thrombosis or pulmonary
D. Nursing care plan/implementation:
emboli.
1. Goal: prevent or correct deformities.
(1) Antiembolic stockings.
a. Activity:
(2) Increase fluid intake.
(1) Bedrest during exacerbations.
b. Goal: prevent infection: antibiotics as ordered,
(2) Daily ROMactive and passive exercises
given prophylactically (Cefazolin).
even in acute phase, 5- to 10-minute peri-
c. Goal: health teaching.
ods; avoid fatigue and persistent pain.
(1) Isometric exercisesgluteal, abdominal,
ADULT

(3) Heat or pain medication before exercise.


and quadriceps-setting; dorsiflexion and
b. Medications: aspirin (high dosages); nons-
plantar flexion of the feet.
teroidals; steroids; antacids given for possible
(2) Use of trapeze.
GI upset with ASA, steroids; disease-modifying
(3) Explain position of operative leg and
antirheumatics (methotrexate, hydroxychloro-
hip postoperatively to prevent adduction
quine, sulfasalazine).
and flexion.
c. Fluids: at least 1,500 mL liquid daily to avoid
(4) Transfer techniquesbed to chair and
renal calculi; milk for GI upset.
chair to crutches; dangle at bedside first
2. Goal: health teaching.
time out of bed.
a. Side effects of medications: tarry stools
(5) Assist client with skin scrubs with
(GI bleeding); tinnitus (ASA).
antibacterial soap.
b. Psychosocial aspects: possible need for early
2. Postoperative:
retirement; financial hardship; loss of libido;
a. Goal: prevent respiratory complications.
unsatisfactory sexual relations.
(1) Turn, cough, and deep breathe.
c. Prepare for joint repair or replacement if
(2) Incentive spirometry.
indicated.
b. Goal: prevent complications of shock or infection.
E. Evaluation/outcome criteria:
(1) Check dressings for drainage q1h for first
1. Remains as active as possible; limited loss of
4 hours; then q4h and prn; may have
mobility; performs self-care activities.
2164_Ch06_347-578 29/03/12 12:30 PM Page 511

Musculoskeletal System 511

Table 6.39
Fractures of the Hip
Risk Types Assessment Surgical Intervention
Osteoporosis Intracapsular (femoral neck; Slight trauma Total hip replacement
Age >60 within hip joint and capsule) Pain: groin and hip
Women (white) Lateral rotation, shortening
While postmenopausal of leg, minimal deformity
Immobility/sedentary
lifestyle
Extracapsular (outside hip joint; Direct trauma Internal fixation with nail
also called intertrochanteric) Severe pain (open reduction)
External rotation, shortening Possible total hip replacement
of leg, obvious deformity
Subtrochanteric (below lesser Direct trauma Internal fixation with nail
trochanter) Leg pain near fracture site (open reduction)
External rotation, shortening Closed reduction
of leg, deformity
Presence of hematoma

Hemovac or other drainage tubes inserted (3) Plaster booties with an abduction bar may
in wound to keep dressing dry. be used.
(2) Monitor I&O and vital signs hourly for (4) Wedge Charnley (triangle-shaped) pillow to
4 hours, then q4h and prn. maintain abduction between knees and
c. Goal: prevent contractures, muscle atrophy: lower legs.
initiate exercises as soon as allowed: isometric (5) Provide periods throughout day when
quadriceps, dorsiflexion and plantar flexion client lies flat in bed to prevent hip flexion
of foot, and flexion and extension of the and strengthen hip muscles.
anklesequential compression device while (6) Report signs of dislocation: anteriorly
in bed. knee flexes, leg turns outward, leg looks
d. Goal: promote early ambulation and movement. longer than other, femur head may be felt
(1) Use trapeze. in groin area; posteriorlyleg turns inward,
(2) Transfer technique (pivot on unaffected appears shorter than other, greater
leg); crutches/walker. trochanter elevated.
(3) Initiate progressive ambulation as ordered; g. Goal: promote comfort.
ensure maximum extension of leg when (1) Initiate skin care; monitor pressure points

ADULT
walking. for redness; back care q2h.
(4) Administer anticoagulation therapy as (2) Alternating pressure mattress; sheepskin
ordered (warfarin immediately postopera- when sitting in chair.
tively) to prevent deep vein thrombosis h. Goal: health teaching.
and pulmonary emboli. (1) Exercise program with written list of activ-
(5) Recognize early side effects of medications ity restrictions.
and report appropriately. (2) Methods to prevent hip adduction.
e. Goal: prevent constipation. (3) Avoid sitting for more than 1 hour: stand,
(1) Increase fluid intake. stretch, and walk frequently to prevent hip
(2) Use fracture bedpan. flexion contractures.
f. Goal: prevent dislocation of prosthesis. (4)Advise not to exceed 90 degrees of hip flex-
(1) Maintain abduction of the affected joint ion (dislocation can occur, particularly
(prevent external rotation); elevate head of with posterior incisions); avoid low
bed, turn according to physicians order. chairs.
When turning to unaffected side, turn (5) Teach alternative methods of usual self-
with abduction pillow between legs to care activities to prevent hip dislocation
maintain abduction. (e.g., avoid: bending from waist to tie
(2) Bucks extension or Russells traction may be shoes, sitting up straight in a low chair,
applied (temporary skin traction). using a low toilet seat).
2164_Ch06_347-578 29/03/12 12:30 PM Page 512

512 chapter 6 Physiological Integrity

(6) Avoid crossing legs, driving a car for d. Skin color changes, pallor cyanosis
6 weeks. gangrene.
(7)Wear support hose for 6 weeks to e. Infection, hemorrhage, or shock.
enhance venous return and avoid C. Analysis/nursing diagnosis:
thrombus formation. 1. Impaired physical mobility related to lower-limb
D. Evaluation/outcome criteria: amputation.
1. Participates in postoperative nursing care plan 2. Body image disturbance related to loss of body
to prevent complications. part.
2. Reports pain has decreased. 3. Pain related to interruption of nerve pathways.
3. Ambulates with assistive devices. 4. Anxiety related to potential change in lifestyle.
4. Complications of immobility avoided. 5. Knowledge deficit (learning need) related to reha-
5. Able to resume self-care activities. bilitation goals.
VII. TOTAL KNEE REPLACEMENT: both sides of D. Nursing care plan/implementation:
the joint are replaced by metal or plastic implants. 1. Goal: prepare for surgery, physically and emotionally.
a. Validate that client and family are aware that
A. Analysis/nursing diagnosis (see VI. TOTAL HIP
amputation of body part is planned.
REPLACEMENT, p. 510).
b. Validate that informed consent is signed.
B. Nursing plan/implementation:
c. Allow time for grieving.
1. See VI. TOTAL HIP REPLACEMENT,
d. If time allows, prepare client for postoperative
pp. 510512.
phase (e.g., teach arm-strengthening exercises
2. Goal: achieve active flexion beyond 70 degrees.
if lower limb is to be amputated; teach alter-
a. Immediately postoperatively: may have
native methods of ambulation).
continuous passive motion (CPM) device
e. Provide time to discuss feelings.
for flexion-extension of affected knee.
f. Prepare surgical site to decrease possibility of
Maximum flexion 110 degrees.
infection (e.g., shave, scrub as ordered).
b. Monitor drainage in Hemovac (q15 min for
g. Discuss postoperative expectations.
first 4 hours, q1h until 24 hours; q4h and
2. Goal: promote healing postoperatively.
prn while Hemovac in place).
a. Monitor respiratory status q14h and prn:
c. Analgesics as ordered for pain.
rate, depth of respiration; auscultate for signs
d. While dressings are still on: quadriceps-
of congestion; and question client about
setting exercises for approximately 5 days
chest pain (pulmonary emboli common
(consult with physical therapist for specific
complication).
instructions).
b. Monitor for hemorrhage; keep tourniquet at
e. After dressings removed: active flexion
bedside.
exercises.
c. Medicate for pain as orderedclient may
f. Avoid pressure on heel.
have phantom limb pain.
C. Evaluation/outcome criteria:
d. Support stump on pillow for first 24 hours;
1. No complications of infection, hemorrhage noted.
ADULT

remove pillow after 24 hours to prevent


2. ROM of knee increases with exercises.
contracture.
VIII. AMPUTATION: surgical removal of a limb as a e. Position: turn client on to stomach to prevent
result of trauma or circulatory impairment (gangrene). hip contracture.
The amount of tissue amputated is determined by the f. ROM exercises for joint above amputation
severity of disease or trauma and the ability of the to prevent joint immobilization; strengthen-
remaining tissue to heal. ing exercises for arms, nonaffected limbs,
A. Risk factors: abdominal muscles.
1. Atherosclerosis obliterans. g. Stump care:
2. Uncontrolled diabetes mellitus. (1) Early postoperative dressings changed prn.
3. Malignancy. (2) As incision heals, bandage is applied in
4. Extensive and intractable infection. cone shape to prepare stump for prosthesis.
5. Result of severe trauma. (3) Inspect for blisters, redness, abrasions.
B. Assessment (preoperative): (4) Remove stump sock daily and prn.
1. Subjective data: pain in affected part. h. Assist in rehabilitation program.
2. Objective data: E. Evaluation/outcome criteria:
a. Soft tissue damage. 1. Begins rehabilitation program.
b. Partial or complete severance of a body part. 2. No hemorrhage, infection.
c. Lack of peripheral pulses. 3. Adjusts to altered body image.
2164_Ch06_347-578 29/03/12 12:30 PM Page 513

Musculoskeletal System 513


IX. GOUT: disorder of purine metabolism; genetic E. Nursing care plan/implementation:
disease believed to be transmitted by a dominant 1. Goal: decrease discomfort.
gene, characterized by recurrent attacks of acute pain a. Administer antigout medications as ordered:
and swelling of one joint (usually the great toe). (1) Treatment of acute attacks: colchicine,
A. Pathophysiology: urate crystals and infiltrating phenylbutazone (Butazolidin), indomethacin
leukocytes appear to damage the intracellular (Indocin), allopurinol (Zyloprim), naproxen
phagolysosomes, resulting in leakage of lysomal (Naprosyn), corticosteroids (prednisone).
enzymes into the synovial fluid, causing tissue (2) Preventive therapy: probenecid (Benemid),
damage and joint inflammation. sulfinpyrazone (Anturane). These drugs
B. Risk factors: are not used during acute attacks.
1. Men. b. Absolute rest of affected joint gradual
2. Age (>50). increase in activities, to prevent complications
3. Genetic/familial tendency. of immobilization; at the same time, rest for
4. Prolonged hyperuricemia (elevated serum uric comfort.
acid). 2. Goal: prevent kidney damage.
5. Obesity. a. Increase fluid intake to 2,000 to 3,000 mL/day.
6. Moderate to heavy alcohol intake. b. Monitor urinary output.
7. Hypertension. 3. Goal: health teaching.
8. Abnormal kidney function. a. Need for low-purine diet during acute attack
C. Assessment: (see Purine-restricted diet, Common
1. Subjective data: Therapeutic Diets, Chapter 9, p. 699).
a. Pain: excruciating b. Importance of increased fluid in diet.
b. Fatigue c. Signs and symptoms of increased progression
c. Anorexia. of disease.
2. Objective data: d. Dosage and side effects of prescribed
a. Joint: erythema (redness), hot, swollen, medications.
difficult to move; skin stretched and shiny F. Evaluation/outcome criteria:
over joint. 1. Swelling decreased.
b. Subcutaneous nodules, tophi (deposits of 2. Discomfort alleviated.
urate) on hands and feet. 3. Mobility returned to status before attack.
c. Weight loss. 4. Laboratory values return to normal.
d. Fever. X. LUPUS ERYTHEMATOSUS (LE): chronic inflam-
e. Sensory changes, with cold intolerance. matory disease of connective tissue; may affect or
f. Laboratory data: involve any organ; vague etiology, but genetic factors,
(1) Serum uric acid: increased significantly viruses, hormones, and drugs are being investigated;
(6.5 mg/100 mL in women, 7.5 mg/100 mL occurs primarily in women ages 18 to 35 years. Two
in men) in chronic gout; only slightly forms: discoid lupus erythematosus (DLE) affects skin

ADULT
increased in acute gout. only, and systemic lupus erythematosus (SLE) affects
(2) WBC count: 12,000 to 15,000/mm3. multiple organs.
(3)Erythrocyte sedimentation rate: A. Pathophysiology: possible toxic effects from
>20 mm/hr. immune complexes deposited in tissue (antibody-
(4)24-hour urinary uric acid: slightly antigen trapping in organ capillaries)fibrinoid
elevated. necrosis of collagen in connective issue, small
(5) Proteinuria (chronic gout). arterial walls (kidneys and heart particularly)
(6) Azotemia (presence of nitrogen-containing cellular death, obstructed blood flow.
compounds in blood) in chronic gout. B. Assessment:
g. Diagnostic tests: arthrocentesis, x-rays. 1. Subjective data:
D. Analysis/nursing diagnosis: a. Pain: joints.
1. Pain related to inflammation and swelling of b. Anorexia; weight loss.
affected joint. c. Photophobia; sensitivity to sun.
2. Impaired physical mobility related to pain. d. Weakness.
3. Knowledge deficit (learning need) related e. Nausea, vomiting.
to diet restrictions and increased fluid 2. Objective data:
needs. a. Fever.
4. Altered urinary elimination related to kidney b. Rash: butterfly distribution across nose, cheeks.
damage.
2164_Ch06_347-578 29/03/12 12:30 PM Page 514

514 chapter 6 Physiological Integrity

c. Lesions: raised, red, scaling plaquescoinlike (2) Altering dosage of medications.


(discoid). (3) Pregnancy (requires medical clearance).
d. Ulceration: oral or nasopharyngeal. (4) Fatigue, stress.
e. Laboratory data: (5) Infections.
(1) Blood: increased LE cells; decreased c. Medications: side effects of immunosuppres-
RBCs, WBCs, thrombocytes. Positive sives and corticosteroids.
antinuclear antibody (ANA) titer. d. Regular exercise: walking, swimming; but
(2) Urinehematuria, proteinuria (nephritis). avoid fatigue.
C. Analysis/nursing diagnosis: e. Wear Medic Alert bracelet.
1. Risk for injury related to possible autoimmune E. Evaluation/outcome criteria:
disorder. 1. Attains a state of remission.
2. Pain related to joint inflammation. 2. No organ involvement (e.g., no cardiac, renal
3. Risk for activity intolerance related to extreme complications).
fatigue, anemia. 3. Keeps active within limitations.
4. Body image disturbance related to chronic skin 4. Continues follow-up medical carerecognizes
eruptions. symptoms requiring immediate attention.
5. Altered nutrition, less than body requirements,
related to anorexia, nausea, vomiting.
6. Altered oral mucous membrane related to
NEUROMUSCULAR
ulcerations.
SYSTEM
D. Nursing care plan/implementation: I. MULTIPLE SCLEROSIS: progressive neurological
1. Goal: minimize or limit immune response and disease, common in northern climates, characterized
complications. by demyelination of brain and spinal cord leading to
a. Activity: rest; 8 to 10 hours sleep; unhurried degenerative neurological function; chronic remitting
environment; assist with stressful activities; and relapsing disease; cause unknown. Visual prob-
ROM to prevent joint immobility and stiffness. lems are often the first indication of multiple sclerosis.
b. Skin care: hygiene; topical steroid cream as Classifications of multiple sclerosis: relapsing-remitting,
ordered for inflammation, pruritus, scaling. primary progressive, secondary progressive, and
c. Mouth care: several times daily if stomatitis progressive-relapsing. Exacerbations aggravated by
present; soft, bland, or liquid diet to prevent fatigue, chilling, and emotional distress.
irritation. A. Pathophysiology: multiple foci (patches) of nerve
d. Diet: low sodium if edematous; low protein degeneration throughout brain, spinal cord, optic
with renal involvement. nerve, and cerebrum cause nerve impulses to
e. Observe for signs of complications: be interrupted (blocked) or distorted (slowed).
(1) Cardiac/respiratory (tachycardia, tachypnea, Researchers suggest that, in genetically susceptible
dyspnea, orthopnea). people, the disease results from an abnormal
(2) GI (diarrhea, abdominal pain, distention). autoimmune response to some agent, perhaps a
ADULT

(3) Renal (increased weight, oliguria, decreased virus or environmental trigger.


specific gravity). B. Risk factors:
(4) Neurological (ptosis, ataxia). 1. Northern climate.
(5) Hematological (malaise, weakness, chills, 2. Onset age: 20 to 40 years.
epistaxis); report immediately. 3. Two to three times more common in women
f. Medications, as ordered: than men.
(1) Analgesics. C. Assessment:
(2) Anti-inflammatory agents (aspirin, 1. Subjective data:
prednisone) and immunosuppressive drugs a. Extremities: weak, numb, decreased
(azathioprine [Imuran], cyclophosphamide sensation.
[Cytoxan]) to control inflammation. b. Emotional: instability, apathy, irritability,
(3) Antimalarials for skin and joint mood swings, fatigue.
manifestations. c. Eyes: diplopia (double vision), spots before
2. Goal: health teaching. eyes (scotomas), potential blindness.
a. Disease process: diagnosis, prognosis, effects d. Difficulty in swallowing.
of treatment. e. Pain.
b. Avoid precipitating factors: 2. Objective data:
(1) Sun (aggravates skin lesions; thus, cover a. Nystagmus (involuntary rhythmic movements
body as much as possible). of eyeball) and decreased visual acuity.
2164_Ch06_347-578 29/03/12 12:30 PM Page 515

Neuromuscular System 515


b. Inappropriate outbursts of laughing or crying c. Methotrexate (Rheumatrex) (low dose) used
(sometimes related to ingestion of hot food). to delay progression of impairment.
c. Disorders of speech. d. Interferon beta-1b (Betaseron) reduces
d. Susceptible to infections. number of lesions and frequency of relapses.
e. Tremors to severe muscle spasms and e. Glatiramer (Copaxone) to reduce number of
contractures. relapses.
f. Changes in muscular coordination; gait: f. Baclofen (Lioresal) for alleviating spasticity
ataxic, spastic. 5 mg three times daily, increased by 5 mg
g. Changes in bowel habits (e.g., constipation). every 3 days; not to exceed 80 mg/day
h. Urinary frequency and urgency. (20 mg four times daily). Optimal effect
i. Incontinence (urine and feces). between 40 and 80 mg; sudden withdrawal
j. Sexual dysfunction. of medication may cause hallucinations and
k. Cognitive changes/depression. rebound spasticity.
l. Laboratory tests: cerebrospinal fluid has pres- g. Diazepam (Valium), dantrolene (Dantrium)
ence of gamma globulin IgG. to relieve muscle spasm.
m. Diagnostic tests: h. Carbamazepine (Tegretol) or amitriptyline
(1) Neurological examination. HCl (Elavil) for dysesthesias or neuralgia.
(2) Positive Lhermittes sign (electric shocklike i. Ciprofloxacin (Cipro) to treat bladder
sensation along spine on flexion of the dysfunction.
head). j. Psyllium hydrophilic mucilloid (Metamucil)
(3) Positive Babinski reflex. to treat bowel dysfunction.
(4) MRI detects plaques. k. Phenytoin (Dilantin) to treat sensory symptoms.
D. Analysis/nursing diagnosis: l. Analgesics to treat pain.
1. Impaired physical mobility related to changes in 3. Goal: health teaching to prevent complications.
muscular coordination. a. Signs and symptoms of disease; measures to
2. Self-esteem disturbance related to chronic, prevent exacerbations.
debilitating disease. b. Teach to monitor respiratory status to prevent
3. Altered health maintenance related to spasms infections.
and contractures. c. Referral: importance of physical therapy to
4. Risk for impaired skin integrity related to prevent contractures.
contractures. d. Referral: possible counseling or community
5. Constipation related to immobility. support group for assistance in accepting
6. Impaired swallowing related to tremors. long-term condition.
7. Visual sensory/perceptual alteration related to e. Teach special skin care to prevent decubitus
nystagmus and decreased visual acuity. ulcers.
E. Nursing care plan/implementation: f. Teach use of assistive devices to maintain
1. Goal: maintain normal routine as long as possible. independence.

ADULT
a. Maintain mobilityencourage walking as 4. Goal: provide psychosocial support.
tolerated; active and passive ROM; splints to a. Answer questions of client and family members.
decrease spasticity. b. Provide referrals to appropriate agencies.
b. Avoid fatigue, infections. c. Monitor for signs of suicide (higher incidence
c. Frequent position changes to prevent skin of suicide in clients with MS than the general
breakdown and contractures; position at population).
night: prone, to minimize flexor spasms of d. Encourage communication.
knees and hips. F. Evaluation/outcome criteria:
d. Bowel/bladder training program to minimize 1. Establishes daily routine; adjusts to altered
incontinence. lifestyle.
e. Avoid stressful situations. 2. Injuries prevented; no falls.
2. Goal: decrease symptomsmedications as 3. Urinary and bowel routines established; inconti-
ordered: nence decreased.
a. Methylprednisolone (high dose, IV) plus 4. Infections avoided.
sodium succinate, followed by gradual taper- 5. Symptoms minimized by medications.
ing of steroids to treat at initial diagnosis. II. MYASTHENIA GRAVIS: chronic neuromuscular
b. Methylprednisolone (Solu-Medrol), prednisone disease characterized by weakness and easy fatigability
(Deltasone), and dexamethasone (Decadron) of facial, oculomotor, pharyngeal, and respiratory
used to treat acute relapses.
2164_Ch06_347-578 29/03/12 12:30 PM Page 516

516 chapter 6 Physiological Integrity

muscles. The muscle weakness increases during e. Eye care: remove crusts; patch affected
periods of activity and improves after periods of rest. eye prn.
A. Pathophysiology: inadequate acetylcholine f. Monitor respiratory statussuction
or excessive or altered cholinesterase, leading to airway prn.
impaired transmission of nerve impulses to muscles 2. Goal: decrease symptoms.
at myoneural junction. a. Administer medications as ordered:
B. Risk factors: (1) Anticholinesterase (neostigmine
1. Possible autoimmune reaction. [Prostigmin], pyridostigmine [Mestinon])
2. Thymus tumor. to elevate concentration of acetylcholine at
3. Young women and older men, but can occur at myoneural junction.
any age. (2) Give before meals to aid in chewing, with
C. Assessment: milk or food to decrease GI symptoms; may
1. Subjective data: be given parenterally.
a. Diplopia (double vision). 3. Goal: prevent complications.
b. Severe generalized fatigue. a. Respiratory assistance if breathing pattern not
2. Objective data: adequate.
a. Muscle weakness: hands and arms affected b. Monitor for choking/increased oral secretion.
first. c. Avoid: narcotics, barbiturates, tranquilizers.
b. Ptosis (drooping of eyelids), expressionless 4. Goal: promote increased self-concept.
facies. a. Encourage independence when appropriate.
c. Hypersensitivity to narcotics, barbiturates, b. Encourage communication; provide alterna-
tranquilizers. tive methods when speech pattern impaired.
d. Abnormal speech pattern, with high-pitched 5. Goal: health teaching.
nasal voice. a. Medication information:
e. Difficulty chewing/swallowing food. (1) Adjust dosage to maintain muscle
f. Decreased ability to cough and deep breathe, strength.
vital capacity. (2) Medication must be taken at prescribed
g. Positive Tensilon test (administration of time to avoid:
edrophonium chloride, 10 mg IV, produces (a) Myasthenic crisis (too little medication).
relief of symptoms within 30 seconds). (b) Cholinergic crisis (too much medication).
h. Positive Prostigmin test (1.5 mg subcutaneous b. Signs and symptoms of crisis: dyspnea,
neostigmine methylsulfate, produces relief severe muscle weakness, respiratory distress,
of symptoms within 15 minutes; increased difficulty in swallowing.
muscle strength within 30 minutes). c. Importance of avoiding upper respiratory
D. Analysis/nursing diagnosis: infections.
1. Ineffective breathing pattern related to d. Determine methods to conserve energy, to
weakness. maintain independence as long as possible,
ADULT

2. Risk for injury related to muscle weakness. while avoiding overexertion.


3. Activity intolerance related to severe fatigue. e. Refer to Myasthenia Gravis Foundation and
4. Bathing/dressing self-care deficit related to other community agencies for assistance in
progressive disease. reintegration into the community and plans
5. Impaired physical mobility related to decrease in for follow-up care.
strength. F. Evaluation/outcome criteria:
6. Anxiety related to physical symptoms and disease 1. Independence maintained as long as possible.
progression. 2. Respiratory arrest avoided.
7. Knowledge deficit (learning need) related to 3. Infection avoided.
medication administration and expected 4. Medication regimen followed and crisis
effectiveness. avoided.
E. Nursing care plan/implementation: III. PARKINSONS DISEASE: progressive disease of
1. Goal: promote comfort. the brain occurring generally in later life; characterized
a. Passive and active ROM, as tolerated, to by stiffness of muscles and by tremors.
increase strength. A. Pathophysiology: depigmentation of the sub-
b. Mouth care: before and after meals. stantia nigra of basal ganglia decreased
c. Diet: as tolerated, soft, pureed, or tube dopamine (neurotransmitter necessary for proper
feedings. muscle movement) decreased and slowed
d. Skin care to prevent decubiti.
2164_Ch06_347-578 29/03/12 12:30 PM Page 517

Neuromuscular System 517


voluntary movement, masklike facies, and 8. Altered nutrition, less than body requirements,
difficulty initiating ambulation. Decreased inhibi- related to lack of appetite.
tions of alpha-motoneurons increased muscle 9. Impaired swallowing related to excessive drooling.
tone rigidity of both flexor and extensor 10. Constipation related to dietary changes.
muscles and tremors at rest. E. Nursing care plan/implementation:
B. Risk factors: 1. Goal: promote maintenance of daily activities.
1. Age older than 40; most often 50 to 60 years. a. ROM exercises, skin care, physical therapy.
2. Affects men and women equally. b. Encourage ambulation; discourage sitting for
3. Cause unknown; possibly connected to arterioscle- long periods.
rosis or viral infection. c. Assist with mealshigh-protein, high-calorie;
4. Drug-induced parkinsonian syndromes have soft diet; small, frequent feedings; encourage
been linked to: increased fluids.
a. Phenothiazines. d. Encourage compliance with medication
b. Reserpine (Serpasil). regimen:
c. Butyrophenones (haloperidol). (1) Dopamine agonists:
C. Assessment: (a) Levodopa: given in increasing doses
1. Subjective data: until symptoms are relieved; given
a. Insomnia. with food to decrease GI symptoms.
b. Depression. Side effects: nausea, vomiting,
c. Defects in judgment related to emotional anorexia, postural hypotension,
instability; dementia (memory loss). mental changes, cardiac arrhythmias.
2. Objective data: Levodopa (L-Dopa) helps restore
a. Limbs, shoulders: stiff, offer resistance to dopamine deficiency in striated mus-
passive ROM. cles: 500 to 1,000 mg/day in divided
b. Loss of coordination, muscular weakness with doses; increase by 100 to 750 mg/day
rigidity. every 3 to 7 days until response
c. Shuffling gait: difficulty in initiating gait, reached; usual maintenance dose
then propulsive, trunk bent forward. should not exceed 18 gm/day.
d. Tremors: pill-rolling of fingers, to-and-fro (b) Carbidopa-levodopa (Sinemet):
head movements. 25/250 mg/day in 3 to 4 divided
e. Loss of postural reflexes. doses. Limits the metabolism of
f. Weight loss, constipation. levodopa peripherally and provides
g. Difficulty in maintaining social interactions more levodopa for the brain.
because of impaired speech, lack of facial (2) Anticholinergics: effective in lessening
affect, drooling. muscle rigidity.
h. Facies: wide-eyed, decreased eye blinking, (a) Benztropine mesylate (Cogentin): 0.5
decreased facial expression. to 6 mg/day in 1 to 2 divided doses.

ADULT
i. Akinesia (abnormal absence of movement). (b) Biperiden (Akineton): 2 mg 3 to
j. Excessive salivation, drooling. 4 times daily, not to exceed 16 mg/day.
k. Speech: slowed, slurred. (c) Trihexyphenidyl (Artane): 1 to 2 mg
l. Judgment defective (e.g., poor decision day; increased by 2 mg every 3 to
making); intelligence intact. 5 days; usual maintenance dose: 5 to
m. Heat intolerance. 15 mg/day in 3 to 4 divided doses.
D. Analysis/nursing diagnosis: (3) Antihistamines: have mild central
1. Impaired physical mobility related to loss of anticholinergic properties.
coordination. 2. Goal: protect from injury.
2. Altered health maintenance related to defective a. Monitor BP, side effects of medications
judgment. (e.g., orthostatic hypotension).
3. Risk for injury related to altered gait. b. Monitor for GI disturbances.
4. Dressing/grooming self-care deficit related to c. Avoid pyridoxine (vitamin B6): it cancels
muscular rigidity. effect of levodopa.
5. Sleep pattern disturbance related to insomnia. d. Levodopa contraindicated with:
6. Body image disturbance related to tremors and (1) Glaucoma (causes increased intraocular
drooling. pressure).
7. Social isolation related to altered physical (2) Monoamine oxidase (MAO) inhibitors
appearance. (causes possible hypertensive crisis).
2164_Ch06_347-578 29/03/12 12:30 PM Page 518

518 chapter 6 Physiological Integrity

3. Goal: health teaching. g. Fasciculations (muscle twitching visible


a. Teach client and family about medications: under skin).
dosage range, side effects, not discontinuing h. Diagnostic tests:
medications abruptly. (1) Electromyography (EMG).
b. Exercise program to maintain ROM and (2) Nerve conduction velocities (NCVs).
normal body posture; also to get adequate D. Analysis/nursing diagnosis:
rest to prevent fatigue. 1. Ineffective airway clearance related to difficulty
c. Dietary adjustment and precautions regarding in coughing.
cutting food in small pieces to prevent 2. Ineffective breathing pattern related to progressive
choking, taking fluid with food for easier respiratory difficulties and eventually respiratory
swallowing. paralysis.
d. Importance of adding roughage to diet to 3. Altered health maintenance related to inability to
prevent constipation. perform self-care activities.
e. Assist client and family to adjust to this 4. Impaired physical mobility related to progressive
chronic debilitating illness. muscular weakness.
F. Evaluation/outcome criteria: 5. Self-care deficits: bathing/hygiene and
1. Activity level maintained. dressing/grooming.
2. Symptoms relieved by medications; no drug 6. Powerlessness related to lifestyle of progressive
interactions. physical helplessness.
3. Complications avoided. 7. Impaired swallowing related to disease
IV. AMYOTROPHIC LATERAL SCLEROSIS progression.
(ALS; Lou Gehrigs disease): progressive degenera- E. Nursing care plan/implementation:
tion of motor neurons within the brain, spinal cord, 1. Goal: maintain independence as long as possible.
or both, leading to death within 2 to 7 years, usually a. Assistance with ADLs; splints, prosthetic
from infection (pneumonia) or consequences of devices to support weak limbs and maintain
respiratory or bulbar paralysis. There are three types mobility.
of ALS: classic sporadic, familial, and Mariana b. Skin care to prevent decubiti.
Islands (Guam). c. Soft/liquid diet to aid in swallowing, prevent
choking; suction prn; head of bed elevated
A. Pathophysiology: myelin sheaths destroyed,
when eating.
replaced by scar tissue; involves lateral tracts of
d. Respiratory assistance as needed; ventilators
spinal cord, eventually medulla and ventral tracts.
as disease progresses and diaphragm becomes
B. Risk factors:
involved.
1. Affects men more than women.
e. Arrange long-term care arrangements if home
2. Usually in middle age.
maintenance no longer feasible.
3. Viral infection possible causal agent.
f. Emotional support, when client is alert;
4. Possible familial or genetic component.
continue involving client in decisions
C. Assessment:
ADULT

regarding care.
1. Subjective data:
g. Medications: riluzole (Rilutek) has some
a. Early symptoms: fatigue, awkwardness.
effect in reducing disease progression; other
b. Dysphagia, dysarthria; speech slurredmay
drugs offer temporary relief of symptoms.
sound drunk.
2. Goal: health teaching.
c. Alert, no sensory loss.
a. Skin care to prevent decubitus ulcer.
2. Objective data:
b. Explain ramifications of disease so client and
a. Symptoms depend on which motor neurons
family can make decisions regarding future care,
affected.
whether client will remain at home as disease
b. Decreased fine finger movement.
progresses or enter a long-term care facility.
c. Progressive muscular weakness, atrophy,
c. How to use suction apparatus to clear airway.
especially lower limbs at onset of disease;
d. Care of nasogastric or gastrostomy feeding
later arms, hands, and shoulders.
tube.
d. Spasticity of flexor muscles; one side of body
F. Evaluation/outcome criteria:
becomes more involved than other.
1. Obtains physical and emotional support.
e. Progressive respiratory difficulties
2. Complications avoided in early stage of disease.
diaphragmatic paralysis.
3. Remains in control of ADLs as long as possible.
f. Progressive disability of upper and lower
4. Skin breakdown avoided.
extremities.
5. Peaceful death.
2164_Ch06_347-578 29/03/12 12:30 PM Page 519

Neuromuscular System 519


V. GUILLAIN-BARR SYNDROME: an uncommon, (2) Prepare for mechanical ventilation if
acquired autoimmune disease resulting in demyelina- needed.
tion of the cranial and peripheral nerves and a (3) Tracheostomy care and ventilator weaning
progressive ascending paralysis that is usually reversible; as indicated.
develops in hours or up to 10 days. Also known as b. Prepare for IV plasma exchanges.
acute inflammatory polyradiculoneuropathy; (1) 2 to 3 hours daily over 4 to 5 days; 200 to
incidence 1.6/100,000. Client is often completely 250 mL/kg body weight of albumin
paralyzed, yet sensation and mentation remain exchanged.
intact. (2) Monitor for: hypotension, arrhythmias,
A. Pathophysiology: macrophages attack normal vascular access problems.
myelin of the peripheral nerves demyelination c. IV immune globulin also may be given.
and blocked conduction of impulses to muscles Complications include: hypotension, dyspnea,
progressive symmetrical and bilateral muscle weak- fever, transient hematuria.
ness from distal lower extremities to proximal 2. Goal: monitor for signs of autoimmune
upper extremities, trunk, and neck. dysfunction.
B. Risk factors: a. Acute periods of BP alternating with BP.
1. Viral infection 1 to 3 weeks before paralysis in b. Arrhythmias: give antiarrhythmic
50% of cases. medications.
2. Gastroenteritis. 3. Goal: prevent tachycardia.
3. Immunizations. a. Medication: propranolol.
4. May be linked to cytomegalovirus and b. ECG monitoring (continuous) to detect
Epstein-Barr virus. alteration in cardiac rate and pattern.
C. Assessment: 4. Goal: assess cranial nerve function.
1. Subjective data: a. Check gag and swallowing reflex.
a. Pain, tingling in legs and back (paresthesias). b. Check ability to clear secretions.
b. Reports falling. c. Check voice.
c. Dyspnea. 5. Goal: maintain adequate ventilation.
2. Objective data: a. Monitor respiratory rate and depth.
a. Footdrop; unable to walk. b. Perform serial vital capacities and ABGs.
b. Gradual, progressive facial weakness; c. Observe for ventilatory insufficiency.
dysphagia; dysarthria. d. Prevent pneumonia, atelectasis.
c. Flaccid paralysis; absent superficial and deep 6. Goal (in acute phase): check for progression of
tendon reflexes (DTRs). muscular weaknesscheck individual muscle
d. Respiratory muscle paralysis respiratory groups q2h.
failure. 7. Goal: maintain nutrition.
e. Postural hypotension; arterial hypertension, a. NG tubebalanced liquid diet; mouth care.
heart block, tachycardia. b. Check gag and swallowing reflex before

ADULT
f. Laboratory data: CSF protein. starting soft, pureed foods.
g. Diagnostic tests: lumbar puncture; electrophysio- 8. Goal: prevent injury and complications.
logic studies (EEG: abnormal; EMG: slowed a. Eye care: artificial tears or ointment, due to
neural conduction). lack of blinking or poor eyelid closure.
D. Analysis/nursing diagnosis: b. Mobility:
1. Impaired physical mobility related to progressive (1) ROM.
muscular weakness. (2) Prevent DVT, contractures: use splints,
2. Ineffective breathing pattern related to progressive high-top sneakers, footboards, heel and
respiratory difficulties and eventual respiratory elbow protectors.
paralysis. (3) Antiembolic stockings.
3. Anxiety related to disease progression. c. Elimination:
4. Self-care deficits: bathing/hygiene and (1) Observe for urinary retention and
dressing/grooming. constipation.
E. Nursing care plan/implementation: (2) Avoid enemas if possible (prevent further
1. Goal: prevent complications during recovery from autonomic response).
paralysis. 9. Goal: support communication.
a. Respiratory: a. Develop two methods:
(1) Observe for signs of failure: forced vital (1) To indicate immediate needs.
capacity, oximetry. (2) For basic conversation.
2164_Ch06_347-578 29/03/12 12:30 PM Page 520

520 chapter 6 Physiological Integrity

F. Evaluation/outcome criteria: E. Analysis/nursing diagnosis:


1. Complete reversal of paralysis. 1. Ineffective breathing pattern related to high-level
2. Complications avoided: maintains respiratory injury.
function, able to swallow, no complications from 2. Impaired physical mobility related to injuries
immobility. affecting lower limbs.
3. After onset and plateau period, recovery may 3. Fear related to uncertain future health status.
take 3 to 12 months. 4. Anxiety related to loss of control over own
VI. SPINAL CORD INJURIES: trauma from hyperex- activities of daily living.
tension, hyperflexion, axial compression, lateral 5. Bathing/hygiene self-care deficit related to injuries
flexion, or shearing of the spine. above T1.
6. Impaired home maintenance management related
A. Types:
to possible quadriplegia and paraplegia.
1. C1 and C2 injury levelresulting deficit:
7. Risk for altered body temperature related to
a. Phrenic nerve involvement.
absence of sweating below level of injury.
b. Diaphragmatic paralysis.
8. Risk for injury related to equipment necessary
c. Respiratory difficulties (require permanent
for daily activities.
ventilatory support).
9. Tactile sensory/perceptual alterations related to
d. Possible quadriplegia.
injury level.
e. Possible death.
10. Body image disturbance related to permanent
2. C4 through T1 injury levelresulting deficit:
change in physical status.
possible quadriplegia.
F. Nursing care plan/implementation:
3. Thoracic-lumbar injury levelresulting deficit:
1. Goal: maintain patent airway.
possible paraplegia.
a. Suction, cough, tracheostomy care, prn.
B. Pathophysiology: trauma vertebral dislocation
b. Oxygen, ventilator care.
or fractures cord trauma, compression, or
c. Monitor blood gas levels.
severance of the cord.
2. Goal: prevent further damage.
C. Risk factors:
a. Immobilize spine.
1. Motor vehicle accidents.
b. Firm mattress, Stryker frame, Foster frame,
2. Diving, surfing, contact sports.
CircO-electric bed, traction, braces (see II.
3. Falls.
FRACTURES, pp. 499, 500, 502, 506).
4. Gunshot wounds.
c. Skeletal traction via tongs: Crutchfield,
D. Assessment:
Gardner-Wells (see II. FRACTURES,
1. Subjective data:
pp. 499, 500, 501).
a. Pain at the level of injury.
d. Halo traction (see II. FRACTURES,
b. Numbness/weakness, loss of sensation below
pp. 499, 500, 501, 506).
level of injury.
3. Goal: relieve edema: anti-inflammatory
c. Psychological distress related to severity of
medications, corticosteroids.
injury and its effects.
ADULT

4. Goal: relieve discomfort: analgesics, sedatives,


2. Objective data:
muscle relaxants.
a. Symptoms depend on extent of injury to
5. Goal: promote comfort:
spinal cord/spinal nerves.
a. Maintain fluid intake: PO/IV, I&O.
b. Paralysis: motor, sphincter.
b. Increase nutritional intake.
(1) Initially a period of flaccid paralysis and
c. Prevent contractures and decubiti.
loss of reflexes, called spinal or neural
d. Assist client to deal with psychosocial issues
shock.
(e.g., role changes).
(2) Incomplete injuries may lead to loss of
e. Begin rehabilitation plan.
voluntary movement and sensory deficits
6. Goal: prevent complications.
below injury level (symptoms vary
a. Monitor for spinal shock during initial phase
depending on injury).
of injury (see VII. SPINAL SHOCK, p. 521).
(3) Complete injury leads to loss of function
b. Monitor for hyperreflexia with severed spinal
and all voluntary movement below level
cord injuries (see VIII. AUTONOMIC
of injury.
DYSREFLEXIA, pp. 521522).
c. Respiratory distress.
7. Goal: health teaching.
d. Alterations in temperature control.
a. Self-care techniques for highest level of
e. Alterations in bowel and bladder function.
independence; include significant others in
f. Involved muscles become spastic and hyper-
teaching.
reflexic within days or weeks.
2164_Ch06_347-578 29/03/12 12:30 PM Page 521

Neuromuscular System 521


b. How to use ambulation assistive devices d. Urinary retention.
(battery-operated wheelchair controlled by e. Abdominal distention: retention of feces,
mouthpiece or hand controls, depending on paralytic ileus.
level of paralysis). f. Skin: cold, clammy.
c. Identify community resources for follow-up D. Analysis/nursing diagnosis:
care and career counseling. 1. Decreased cardiac output related to loss of
d. Signs and symptoms of autonomic hyper- vasomotor tone below level of injury.
reflexia (see VIII. AUTONOMIC 2. Ineffective breathing pattern related to injuries
DYSREFLEXIA, pp. 521522). involving diaphragm.
e. Methods to prevent skin breakdown, 3. Impaired physical mobility related to loss of
infections of respiratory, urinary tract. voluntary movement of limbs.
f. Bowel, bladder program. 4. Urinary retention related to loss of stretch
G. Evaluation/outcome criteria: reflexes.
1. Complications avoided. 5. Fear related to serious physical condition.
2. Accomplishes self-care to greatest level for injury. 6. Risk for injury related to potential organ damage
3. Participates in rehabilitation plan. if shock continues.
4. Grieves over loss and begins to integrate self into E. Nursing care plan/implementation:
society. 1. Goal: prevent injury related to shock.
VII. SPINAL SHOCK: temporary flaccid paralysis and a. Maintain patent airway: intubation and
areflexia following a severe injury to the spinal cord. mechanical ventilation may be necessary with
cervical spinal injuries due to involvement of
A. Pathophysiology: squeezing or shearing of the
diaphragm.
spinal cord due to fractures or dislocation of verte-
b. Monitor vital signs; profound hypotension and
brae; interruption of sensory tracts; loss of con-
bradycardia are most dangerous aspects of
scious sensation; interruption of motor tracts; loss
spinal shock.
of voluntary movement; loss of facilitation; loss of
c. Administer blood/IV fluids as ordered.
reflex activity; loss of muscle tone; loss of stretch
d. Nutrition and hydration:
reflexes, leading to bowel and bladder retention. If
(1) NPO in acute stage: maintain nutrition
injury between T1 and L2, leads to loss of sympa-
by IV infusions as ordered.
thetic tone and decrease in blood pressure. Afferent
(2) When allowed to eat: high-protein,
impulses are unable to ascend from below the
high-calorie, high-vitamin diet.
injured site to the brain, and efferent impulses are
e. Maintain proper position to prevent further
unable to descend to points below the site.
injury.
B. Risk factors:
(1) Backboard is necessary to transport from
1. Automobile/motorcycle accidents.
place of injury.
2. Athletic accidents (e.g., diving in shallow water).
(2) Support head in neutral alignment and
3. Gunshot wounds.
prevent flexion.
C. Assessment:

ADULT
(3) Skeletal traction will be applied once
1. Subjective data:
diagnosis is made.
a. Loss of sensation below level of injury.
f. Monitor urinary output q1h; may have Foley
b. Inability to move extremities.
catheter while in shock; later, intermittent
c. Pain at level of injury.
catheterization will be used as needed.
2. Objective data:
g. Relieve bowel distention; use lubricant con-
a. Neurological examination:
taining anesthetic, as necessary, when checking
(1) Absent: pinprick, pressure, and vibratory
for or removing impaction.
sensations below level of injury; reflexes
F. Evaluation/outcome criteria:
below level of injury.
1. Complications are avoided.
(2) Muscles: flaccid.
2. Body functions are maintained.
b. Vital signs:
(1) BP decreased (loss of vasomotor tone VIII. AUTONOMIC DYSREFLEXIA (autonomic
below level of injury). hyperreflexia): a group of symptoms in which many
(2) Bradycardia. spinal cord autonomic responses are activated simulta-
(3) Elevated temperature. neously. This may occur when cord lesions are above
(4) Respirations: may be depressed; possible the sixth thoracic vertebra; it is most commonly seen
respiratory failure if diaphragm involved. with cervical and high thoracic cord injuries; may occur
c. Absence of sweating below level of injury. up to 6 years after injury.
2164_Ch06_347-578 29/03/12 12:30 PM Page 522

522 chapter 6 Physiological Integrity

A. Pathophysiology: pathological reflex condition, Do not use Creds maneuver (see Table 6.34,
which is an acute medical emergency characterized p. 498) or tap the bladder; it could exacerbate
by extreme hypertension and exaggerated autonom- the response.
ic responses to stimuli. d. Culture if infection suspected.
B. Risk factors: 3. Goal: promote regular bowel elimination.
1. Distention of bladder or rectum. a. Bowel training program.
2. Stimulation of skin (e.g., decubitus ulcers, wrin- b. Administer suppository/enemas/laxatives as
kled clothing). ordered and prn.
3. Stimulation of pain receptors. c. When checking for and removing impaction,
C. Assessment: first use anesthetic ointment (e.g., dibucaine
1. Subjective data: [Nupercainal] ointment) to decrease irritation.
a. Severe, pounding headache. 4. Goal: prevent decubitus ulcers.
b. Blurred vision; sees spots in front of eyes. a. Meticulous skin care.
c. Nausea. b. Position change q12h.
d. Restlessness. c. Flotation pads, alternating pressure mattress
e. Feels flushed. on bed and wheelchair.
2. Objective data: 5. Goal: health teaching.
a. Severe hypertension (systolic BP may reach a. How to recognize risk factors that could
300 mm Hg). initiate this condition.
b. Bradycardia (30 to 40 beats/min); fever. b. Methods to prevent situations that increase
c. Profuse diaphoresis above level of injury. risk (e.g., bowel program, bladder program,
d. Flushing of skin above level of injury. skin care, position change schedule).
e. Pale skin below level of injury. F. Evaluation/outcome criteria:
f. Pilomotor spasm (gooseflesh); chills. 1. BP remains within normal limits.
g. Nasal congestion. 2. No complications occur.
h. Distended bladder, bowel. IX. HERNIATED/RUPTURED DISK (ruptured
i. Skin breakdown. nucleus pulposus): strain or injury to a weakened
j. Seizures. cartilage between vertebrae can result in herniation of
D. Analysis/nursing diagnosis: the nucleus, causing pressure on nerve roots in spinal
1. Dysreflexia related to high spinal cord injury. canal, pain, and disability.
2. Risk for injury related to complications of
A. Pathophysiology: pulpy substance of disk interior
hypertension, stroke.
(nucleus pulposus) bulges or ruptures through the
3. Visual sensory/perceptual alteration related to
outer annulus fibrosus irritation and pressure on
blurred vision.
nerve endings in the spinal ligaments muscle
4. Urinary retention related to inability to empty
spasm and distortion of the joints of vertebral arches.
bladder due to spinal injury.
B. Risk factors:
5. Constipation related to inability to establish
1. Strain as result of poor body mechanics.
ADULT

successful bowel training program.


2. Trauma.
6. Impaired skin integrity related to immobility.
3. History of back injuries.
E. Nursing plan/implementation:
4. Degenerative disk (spondylosis).
1. Goal: decrease symptoms to prevent serious side
C. Assessment:
effects.
1. Lumbar injuries (90% of herniations):
a. Elevate head of bed; this lowers BP in persons
a. Subjective data:
with high spinal cord injuries.
(1) Pain: low back, radiating to buttocks,
b. Identify and correct source of stimulation if
posterior thigh, and calf; relieved by
possible; notify physician.
recumbency; aggravated by sneezing,
c. Monitor vital signs (BP) q15 min and prn;
coughing, and flexion; sciatic pain
uncontrolled hypertension can lead to stroke,
continues even when back pain subsides.
blindness, death.
(2) Numbness, tingling.
d. Give medications as ordered: nitrates, nifedip-
b. Objective data:
ine (Procardia), or hydralazine (Apresoline).
(1) Muscle weaknessleg and foot.
2. Goal: maintain patency of catheter.
(2) Inability to flex leg.
a. Monitor output; palpate for distended
(3) Sensory loss, leg and foot.
bladder.
(4) Alterations in posture: leans to side,
b. Check for tubing kinks; irrigate catheter prn.
unable to stand up straight.
c. Insert new catheter immediately if blocked.
2164_Ch06_347-578 29/03/12 12:30 PM Page 523

Neuromuscular System 523


(5) Edema: leg and foot. h. Traction application prn for comfort.
(6) Positive Lasgues sign: straight leg raising i. Prepare for surgery if medical regimen unsuc-
with hip flexed and knee extended will cessful.
produce sciatic pain. 2. Goal: health teaching.
2. Cervical injuries (10% of herniations): a. Correct body mechanics, keep back straight.
a. Subjective data: b. Exercise program as symptoms decrease.
(1) Painupper extremities, radiating to F. Evaluation/outcome criteria:
hands and fingers; aggravated by coughing, 1. Reports pain decreased.
sneezing, and straining. 2. Mobility increased, normal body posture
(2) Tingling, burning sensation in upper attained.
extremities and back of neck. X. LAMINECTOMY: excision of dorsal arch of verte-
b. Objective data: brae with or without spinal fusion of two or more
(1)Upper extremities: weakness and vertebrae with a bone graft from iliac crest, to stabilize
atrophy. spine.
(2) Neck: restricted movement.
A. Analysis/nursing diagnosis:
(3) Diagnostic tests for both lumbar and
1. Pain related to edema of surgical procedure.
cervical injuries.
2. Impaired physical mobility related to pain and
(a) Spine x-rays.
discomfort resulting from surgery.
(b) CT scan.
B. Nursing care plan/implementation:
(c) MRI.
1. Goal: relieve anxiety.
(d) Myelography (less preferred than
a. Answer questions, explain routines.
CT scan or MRI).
b. See THE PERIOPERATIVE EXPERI-
(e) Electromyography.
ENCE, p. 365.
(f ) Neurological examination: special atten-
2. Goal: prevent injury postoperatively.
tion to sensory status, including pain,
a. Monitor vital signs:
touch, and temperature identification;
(1) Neurological signs (e.g., check sensation
and to motor status, including strength,
and motor strength of limbs).
gait, and reflexes.
(2) Respiratory status (risk for respiratory
D. Analysis/nursing diagnosis:
depression with cervical laminectomy).
1. Pain related to pressure on nerve roots.
b. Monitor I&O (urinary retention common,
2. Fear related to disease progression and/or
especially with cervical laminectomy); may
potential surgery.
need catheterization. Encourage fluids.
3. Knowledge deficit (learning need) related to
c. Monitor bowel sounds (paralytic ileus
correct body mechanics.
common with lumbar laminectomy).
4. Impaired physical mobility related to continued
d. Monitor dressing for possible bleeding.
pain.
e. Bed position as ordered:
5. Sleep pattern disturbance related to difficulty
(1) For lumbar laminectomy: head of bed flat;

ADULT
finding comfortable position.
supine with slight flexion of legs; with
E. Nursing care plan/implementation:
pillow between knees for turning and
1. Goal: relieve pain and promote comfort.
side-lying position.
a. Bedrest with bedboard.
(2) For cervical laminectomy: head of bed elevated,
b. Positionavoid twisting.
neck immobilized with collar or sandbags.
(1) Lumbar disk: Williams (head elevated
f. Encourage deep breathing to prevent respira-
30 degrees, knee gatch elevated to flatten
tory complications. Use of inspirometer q1h
the lumbosacral curve). Can be duplicated
when awake.
at home with pillow placement.
g. Prevent strain or flexion at surgical site:
(2) Cervical: low Fowlers.
logrolling with spinal fusion.
c. Medications as ordered:
h. Some surgical interventions that require small
(1) Analgesics.
incisions (microsurgery) have no specific post-
(2) Muscle relaxants.
operative positions.
(3) Anti-inflammatory.
3. Goal: promote comfort.
(4) Stool softeners.
a. Administer analgesics if sciatic-type pain con-
d. Moist heat.
tinues after lumbar surgery (arm pain after
e. Fracture bedpan.
cervical surgery), due to edema from trauma
f. Gradual increase in activity.
of surgery.
g. Brace application for support.
2164_Ch06_347-578 29/03/12 12:30 PM Page 524

524 chapter 6 Physiological Integrity

4. Goal: prepare for early discharge. 6. Age.


a. Clients having microsurgery for repair of her- 7. Air pollution.
niated disk will usually be discharged from 8. Diet (e.g., high animal protein).
the hospital 1 day postoperative; teaching 9. Chronic irritation.
regarding allowed and restricted activities 10. Precancerous lesions (e.g., gastric ulcers).
must be done early. 11. Stress.
5. Goal: health teaching. C. Assessment:
a. How to turn and move from side to side in 1. Specific symptoms depend on the anatomical
one motion, sit up, and get out of bed without and functional characteristics of the organ or
twisting spine; to get out of bed: raise head of structure involved.
bed while in side-lying position, then put feet 2. Mechanical effects:
over edge of bed, and stand. a. Pressuretumors growing in confined areas
b. Proper positioning and ambulation techniques. such as bone produce pain early, whereas
c. Correct posture, body mechanics, activities to tumors growing in expandable areas such as
prevent further injury; increase activities the abdomen may be undetected for some
according to tolerance. time.
d. Referral: physiotherapy; encourage compli- b. Obstructiontumors that compress tubular
ance for full rehabilitation. structures such as the esophagus, bronchi, or
C. Evaluation/outcome criteria: lymph channels may cause symptoms such as
1. No respiratory, bowel, or bladder complications swallowing difficulties, shortness of breath,
noted. edema. Symptoms depend on location of
a. Lung sounds clear. tumor and on the particular organ or struc-
b. Bowel sounds present; able to pass gas and feces. ture receiving pressure.
c. Urinary output adequate. c. Interruptions of blood supplycompression
2. Regains mobility. of blood vessels or diversion of blood supply
3. Comfort level increases: reports leg and back may cause necrosis or ulceration or may pre-
pain decreased. cipitate hemorrhage.
4. Demonstrates protective positioning and 3. Systemic effects:
mbulation techniques. a. Anorexia, weakness, weight loss.
b. Metabolic disturbancesmalabsorption
syndrome.
ONCOLOGICAL c. Fluid and electrolyte imbalances.
DISORDERS d. Hormonal imbalancesincreased antidiuretic
I. THE CLIENT WITH CANCER: Cancer is a multi- hormone (ADH), adrenocorticotropic
system stressor. Regardless of the specific type of can- hormone (ACTH), thyrotropin (TSH), or
cer, certain aspects of the disease and of nursing care parathyroid hormone (PTH).
are the same. The following principles apply universal- e. Diagnostic tests:
ADULT

ly and should be referred to when studying individual (1) Biopsyexcision of part of tumor
kinds of cancer. mass.
A. Pathophysiology: result of altered cellular mecha- (2) Needle biopsyaspiration of cells from
nisms. Several theories about causation, but current subcutaneous masses or organs such as
thinking is multiple causation. Alterations result in liver.
a progressive, uncontrolled multiplication of cells, (3) Exfoliative cytologyscraping of any
with selective ability to invade and metastasize. endothelium (cervix, mucous membranes,
B. Risk factors: skin) and applying to slide.
1. Heredity (e.g., retinoblastoma). (4) X-raysdetect tumor growth in GI,
2. Familial susceptibility (e.g., breast). respiratory, and renal systems.
3. Acquired diseases (e.g., ulcerative colitis). (5) Endoscopyvisualization of body cavity
4. Virus (e.g., Burkitts lymphoma). through endoscope.
5. Environmental factors: (6) Computed tomography (CT)visualization
a. Tobacco. of a body part whereby layers of tissue can
b. Alcohol. be seen utilizing the very narrow beams of
c. Radiation. this type of x-ray equipment.
d. Occupational hazards. (7) Magnetic resonance imaging (MRI)a
e. Drugs (e.g., immunosuppressive, cytotoxic). scanning device using a magnetic field for
f. Asbestos. visualization.
2164_Ch06_347-578 29/03/12 12:30 PM Page 525

Oncological Disorders 525


(8) Positron emission tomography (PET)a E. Nursing care plan/implementationgeneral care
scanning device in which radioactive of the client with cancer:
glucose is injected prior to scanning. 1. Goal: promote psychosocial comfort (see also
Areas of high glucose uptake, such as Chapter 10).
rapidly dividing cancer cells, are dramati- a. Assist with diagnostic work-up by providing
cally displayed in the scan imagery; useful psychological support and information about
in detecting early cancers. specific disease, diagnostic tests, diagnosis,
f. Laboratory data: and treatment options.
(1) Blood and urine testsrefer to Appendix A b. Reduce anxiety by listening, making referrals
for normal values. for special problems (peer support groups,
(2) Alkaline phosphatasegreatly increased in self-help groups such as Reach to Recovery),
osteogenic carcinoma (>92 units/L). supplying information, or correcting misin-
(3) Calciumelevated in multiple myeloma formation, as appropriate.
bone metastases (>10.5 mg/dL). c. Stress-management techniques (see
(4) Sodiumdecreased in bronchogenic Orientation, Chapter 1, pp. 1112).
carcinoma (<135 mEq/L). d. Nursing management related to client who is
(5) Potassiumdecreased in extensive liver depressed (see Chapter 10).
carcinoma (<3.5 mEq/L). 2. Goal: minimize effects of complications.
(6) Serum gastrinmeasures gastric secretions. a. Anorexia/anemia:
Decreased in gastric carcinoma. Normal (1) Decrease anemia by:
value 0 to 180 ng/L. (a) Providing well-balanced, iron-rich,
(7) Neutrophilic leukocytosistumors. small, frequent meals.
(8) Eosinophilic leukocytosisbrain tumors, (b) Administering supplemental vitamins
Hodgkins disease. and iron as ordered.
(9) Lymphocytosischronic lymphocytic (c) Administering packed red blood cells as
anemia. ordered.
D. Analysis/nursing diagnosis: (d) Maintaining hyperalimentation as
1. Pain related to diagnostic procedures, ordered.
pressure, obstruction, interruption of (e) Monitoring red blood cell count.
blood supply, or potential side effects of (2) Enhance nutrition by providing nutritional
drugs. supplements and a diet high in protein;
2. Anxiety related to fear of diagnosis or necessary because of increased metabolism
disease progression, treatment, and its known related to metastatic process. Consult with
or expected side effects. dietitian for suggestions of best food for
3. Altered nutrition, less than body requirements, individual client.
related to anorexia. b. Hemorrhage: monitor platelet count and
4. Risk for injury related to radioactive contamina- maintain platelet infusions as ordered.

ADULT
tion of excreta. Teach client to monitor for any signs of
5. Body image disturbance related to loss of bleeding.
body parts, change in appearance as a result c. Infection: observe for signs of sepsis (changes
of therapy. in vital signs, temperature of skin, mentation,
6. Powerlessness related to diagnosis and own urinary output or pain); monitor laboratory
perception of its meaning. values (WBCs); administer antibiotics as
7. Self-esteem disturbance related to impact of ordered.
cancer diagnosis. d. Pain and discomfort: alleviate by frequent
8. Risk for infection related to immunosuppression position changes, diversions, conversations,
from radiation and chemotherapy. guided imagery, relaxation, back rubs, and
9. Altered urinary elimination related to narcotics as ordered.
dehydration. e. Assist in adjusting to altered body
10. Risk for injury related to normal tissue damage image by encouraging expression of
from radiation source. fears and concerns. Do not ignore clients
11. Fluid volume deficit related to nausea and questions, and give honest answers; be
vomiting. available.
12. Diarrhea related to radiation of bowel. f. Fatigue. Encourage periods of rest and a
13. Constipation related to dehydration. decrease in daily exertion.
2164_Ch06_347-578 29/03/12 12:30 PM Page 526

526 chapter 6 Physiological Integrity

3. Goal: general health teaching. (2) Administer prescribed medications as


a. Self-care skills to maintain independence needed.
(e.g., client who has a colostomy should (3) Administer stool softeners and enemas as
know how to manage the colostomy before ordered.
going home). c. Goal: health teaching.
b. Importance of follow-up care and routine (1) Involve client, significant others, and
physical examinations to monitor for general family members in rehabilitation program.
health and possible signs of further disease. (2) Prepare for further therapies, such as
c. Dietary instructions, adjustments necessary to radiation or chemotherapy.
maintain nutrition during and after treatment. (3) Referral: support groups, as appropriate:
d. Health maintenance programs: teach Reach to Recovery, Ostomy Associates,
hazards of the use of tobacco and alcohol. Laryngectomy Association.
Avoid high-fat, low-roughage diet. (4) Develop skills to deal with disease
e. Risk factors: family history, stress, age, diet, progression if cure not realistic or
occupation, environment. metastasis evident.
f. Access to information: clients should have G. Chemotherapy: used as single treatment or in
telephone numbers for facility where ques- combination with surgery and radiation, for early
tions can be answered and symptoms reported or advanced diseases. Used to cure, increase survival
24 hours a day. time, or decrease specific life-threatening complica-
F. General surgical intervention: surgery may be tions. Antineoplastic agents primary mode of
diagnostic, curative (when the lesion is localized or action involves interfering with the supply and
with minimal metastases to the lymph nodes), utilization of building blocks of nucleic acids, as
palliative (to decrease symptomatology), or well as interfering with intact molecules of DNA or
reconstructive. (See also THE PERIOPERATIVE RNA, which are needed for replication and cell
EXPERIENCE, pp. 365367, and specific types growth. Bone marrow, hair follicles, and the gastroin-
of cancer, following.) testinal tract are three areas of the body in which cells
1. Nursing care plan/implementation are actively dividing; this is why most side effects
preoperative: are related to these areas of the body. Most often
a. Goal: prevent respiratory complications. antineoplastic agents are used in combination.
(1)Coughing and deep-breathing 1. Types: alkylating agents, antimetabolites, antitu-
techniques. mor antibiotics, antimiotic agents, plant alkaloids,
(2) No smoking for 1 week before surgery. enzymes, hormones, and biotherapy (e.g., bacille
b. Goal: counteract nutritional deficiencies. Calmette-Gurin [BCG], interferon) (see
(1) Diet: Chapter 8, Table 8.11, p. 625).
(a) High protein, high carbohydrate for 2. Major problem: lacks specificity, thus affecting
tissue repair. normal as well as malignant cells.
(b) Vitamin and mineral supplements. 3. Major side effects: bone marrow depression,
ADULT

(c) Hyperalimentation as ordered. stomatitis, nausea and vomiting, gastrointesti-


(2) Blood transfusions may be needed if nal ulcerations, diarrhea, and alopecia (see
counts are low. Chapter 8, Table 8.12, p. 626).
c. Goal: reduce apprehension. 4. Routes of administration: oral, intramuscular,
(1) Clarify postoperative expectations. intravenous (Hickman or Groshong catheter),
(2) Explain care of ostomies or tubes. subclavian lines, portacaths, peripheral, intra-
(3) Answer clients questions honestly. arterial (may have infusion pump for continuous
2. Postoperative: or intermittent flow rate), intracavitary (e.g.,
a. Goal: prevent complications. bladder through cystoscopy). (See 6. Nursing
(1) Monitor respiratory status and precautions with chemotherapy following
hemodynamic status. for information about administration of
(2) Wound care; active and passive exercises IV chemotherapeutic agents.)
as allowed; respiratory hygiene; coughing, 5. Nursing care plan/implementation:
deep breathing, and turning; fluids (see a. Goal: assist with treatment of specific side effects.
III. POSTOPERATIVE EXPERIENCE, (1) Nausea and vomitingantiemetic drugs
pp. 369375). (e.g., prochloroperazine, ondansetron
b. Goal: alleviate pain and discomfort. [Zofran]) as ordered and scheduled; small,
(1) Encourage early ambulation, depending on frequent, high-calorie, high-potassium,
surgical procedure. high-protein meals; chopped or blended
2164_Ch06_347-578 29/03/12 12:30 PM Page 527

Oncological Disorders 527


foods for ease in swallowing; include (3) Advise client/family on increased risk for
milk and milk products when tolerated infection (avoid uncontrolled crowds and
for increased calcium; carbonated drinks; individuals with upper respiratory tract
frequent mouth care; antacid therapy as infections or childhood diseases).
ordered; rest after meals; avoid food odors (4) Monitor injection site for signs of extrava-
during preparation of meals; pleasant sation (infiltration); site must be changed
environment during meals; appropriate if leakage suspected, and guidelines to
distractions; IV therapy; nasogastric tube neutralize must be followed according to
for control of severe nausea or as route drug protocol.
for tube feeding if unable to take food by 6. Nursing precautions with chemotherapy:
mouth; hyperalimentation. a. Nurse should wear gloves and mask when
(2)Diarrhealow-residue diet; increased preparing chemotherapy drugs for administra-
potassium; increased fluids; atropine tion. Mixing of drug into IV bag done under
SO4diphenoxylate HCl (Lomotil) or laminar flow hood.
kaolin-pectin (Kaopectate) as ordered; b. Drugs are toxic substances, and nurses must
avoid hot or cold foods/liquids. take every precaution to handle them with
(3) Stomatitis (painful mouth)soft tooth- care.
brushes or sponges (toothettes); mouth c. When expelling air bubbles from syringes,
care q2q4; viscous lidocaine HCl care must be taken that the drugs are not
(Xylocaine) as ordered before meals. Oral sprayed into the atmosphere.
salt-and-soda mouth rinses; avoid commer- d. Contaminated needles and syringes should be
cial mouthwashes that contain high level disposed of intact (to prevent aerosol genera-
of alcohol, which could be very irritating tion) in plastic-lined box according to envi-
to mucous membranes. Avoid hot ronmental standards. Disposable equipment
foods/liquids; include bland foods at cool should be used whenever possible.
temperatures; remove dentures if sores e. If skin becomes contaminated with a drug,
are under dentures; moisten lips with wash under running water.
lubricant. f. Nurses should know the half-life and excre-
(4) Skin caremonitor: wounds that do not tion route of the drugs being administered
heal, infections (client receives frequent and take the special precautions necessary.
sticks for blood tests and therapy); avoid For example, while the drug is actively being
sunlight; use sunblock, especially if receiv- excreted, use gloves when touching client,
ing doxorubicin (Adriamycin). stool, urine, dressings, vomitus, etc.
(5) Alopeciabe gentle when combing or g. If the nurse is in the early phase of pregnancy,
lightly brushing hair; use wigs, nightcaps, she should seek specific information about
scarves; provide frequent linen changes. risks to her unborn child before caring for the
Advise client to have hair cut short before client receiving chemotherapeutic agents.

ADULT
treatment with drugs known to cause H. Radiation therapy: used in high doses to kill can-
alopecia (bleomycin, cyclosphosphamide, cer cells, or palliatively for pain relief. Side effects of
dactinomycin, daunorubicin hydrochloride, radiation therapy depend on site of therapy (side
doxorubicin hydrochloride, 5-fluorouracil, effects are also variable in each individual): nausea,
ICRF-159, hydroxyurea, methotrexate, vomiting, stomatitis, esophagitis (Candida), dry
mitomycin, VP 16213, vincristine). Other mouth, diarrhea, depression of bone marrow, sup-
techniques may be used, depending on pression of immune response, decreased life span,
clients age and protocol in clinical agency. and sterility.
(6) Extravasationinfiltration of chemothera- 1. External radiation: cobalt or linear accelerator
peutic agents into surrounding tissues. machine.
Document and treat according to agency a. Procedure: daily treatments, Monday through
protocols for administered drug. Friday, for prescribed number of times accord-
b. Goal: health teaching. ing to size and location of tumor (length of
(1) Orient client and family to purpose of treatment schedule is usually 4 to 6 weeks).
proposed drug regimen and anticipated Client remains alone in room during treat-
side effects. ment. (Nurse, therapist, family members
(2) Advise that frequent checks on hematolog- cannot stay in room with client due to radia-
ical status will be necessary (client will tion exposure during treatment.) Client
receive frequent IV sticks, laboratory tests). instructed to lie still so exactly same area is
2164_Ch06_347-578 29/03/12 12:30 PM Page 528

528 chapter 6 Physiological Integrity

irradiated each treatment. Marks (tattoos or nausea; give small, frequent meals
via permanent-ink markers) are made on skin high in protein and carbohydrates
to delineate area of treatment; marks must not and low in residue.
be removed during entire treatment course. (v) Strong alcohol-based mouthwash;
b. Nursing care plan/implementation: use daily salt-and-soda mouthwash.
(1) Goal: prevent tissue breakdown. (vi) Fatigue, an overwhelming problem.
(a) Do not wash off site identification Need to pace themselves, nap; may
marks (tattoos cannot be removed); need someone to drive them to
dosage area is carefully calculated and therapy; can continue with usual
must be exact for each treatment. activities as tolerated.
(b) Assess skin daily and teach client to do (vii)Crowds and persons with upper
same (most radiation therapy is done respiratory infections or any other
on outpatient basis, so client needs infections.
skills to manage independently). (b) Provide appropriate birth control infor-
(c) Keep skin dry; cornstarch usually the mation for clients of childbearing age.
only topical application allowed; 2. Internal radiation: sealed (radium, iridium,
100% aloe (no alcohol) for redness. cesium):
(d) Contraindications: a. Used for localized masses (e.g., mouth, cervix,
(i) Talcum powders, due to potential breast, testes). Due to exposure from radiation
radiation dosage alteration. source, precautions must be taken while it is
(ii) Lotions, due to increased moistening in place. Health-care personnel and family
of skin. must adhere to principles of time, distance, and
(iii) Products containing alcohol, due to shielding to decrease exposure (shortest amount
increased dryness. of time possible, stay as far away as possible
(e) Reduce skin friction by avoiding con- from the source of radiation, and wear protec-
stricting bedclothes or clothing, and by tive lead apron, gloves). If source of radiation
using electric shaver. accidentally falls out, it should be picked up
(f ) Dress areas of skin breakdown with only with forceps. Radiation officer should be
nonadherent dressing and paper tape. notified immediately. Client should be in
(2) Goal: decrease side effects of therapy. private room, and bed should be in the center
(a) Provide meticulous oral hygiene. of the room, if possible, to protect others.
(b) If diarrhea occurs, may need IV infu- Unless the walls are lead lined, radiation will
sions, antidiarrheal medications; monitor penetrate them; placing the bed in center of
bowel movements (possible adhesions room will decrease exposure. Once the source
from surgery and radiation treatments). of radiation has been removed, there is no
(c) Monitor vital signs, particularly respira- exposure from client, excretions, or linens.
tory function, and BP (sloughing of b. Nursing care plan/implementation:
ADULT

tissue puts client at risk for hemorrhage). (1) Goal: assist with cervical radium implantation
(d) Monitor hematological statusbone (cervical radium is used here as the most
marrow depression can cause fatal common example of internal radiation
toxicosis and sepsis. source).
(e) Institute reverse isolation as necessary (a) Before insertiongive douche, enema,
to prevent infections (reverse perineal prep; insert Foley catheter, as
isolation usually instituted if less ordered.
than 50% neutrophils). (b) After implantationcheck position of
(3) Goal: health teaching. applicator q24h.
(a) Instruct client to avoid: (i) Keep client on bedrest in flat posi-
(i) Strong sunlight; must wear sun- tion to avoid displacing applicator
block lotion, protective clothing (may turn to side for eating).
over radiation site. (ii) Notify physician if temperature
(ii) Extremes in temperature to the area elevates, nausea and/or vomiting
(hot-water bottles, ice caps, spas). occur (indicates radiation reaction
(iii) Synthetic, nonporous clothes or or infection).
tight constrictive clothing over area. (iii) After removal of implant (48 to
(iv) Eating 2 to 3 hours before treat- 144 hours)bathe, douche, and
ment and 2 hours after, to decrease remove catheter as ordered.
2164_Ch06_347-578 29/03/12 12:30 PM Page 529

Oncological Disorders 529


(2) Goal: health teaching. (b) Collect paper plates, eating utensils,
(a) Explain that nursing care will be limit- dressings, and linen in impermeable
ed to essential activities in postinsertion bags; label and dispose according to
period. agency protocol.
(b) Signs and symptoms of complications (c) Collect excreta in shielded container
so client can notify staff if something and send to laboratory daily to monitor
unusual happens (bleeding, radiation excretion rate and disposal.
source falls out, fever, etc.). (2) Radioactive phosphorus (32P): half-life
c. Nursing precautions for sealed internal 14 days; injected into cavity or given IV or
radiation: orally.
(1) Never handle radium directlyif applica- (a) If injected into cavity, turn client
tors should accidentally be removed, pick q1015 min for 2 hours to ensure
up applicator by strings with long-handled distribution.
forceps and notify radiation officer. (b) No radiation hazard unless leakage
(2) Linen must remain in clients room and from instillation site or from clients
not be sent to laundry until source of radi- excreta, which are collected in lead-lined
ation has been accounted for and returned containers and brought to the radioiso-
to its container. tope laboratory for disposal. Linen is
(3)Time, distance, and shielding are factors collected in container, marked radioac-
that increase or decrease potential effects tive, and brought to the radioisotope
on personnel. Need to minimize exposure laboratory for special handling.
of nursing staff, clients family, and other (c) Seepage will stain linens blue; wear
health professionals. Nurses who may be gloves when handling contaminated
pregnant should not care for clients with linens, dressings. Excreta disposed of
internal radiation because of possible as in (b).
damage to the fetus from radiation expo- (3) Radioactive gold (198Au): half-life 2.7 days;
sure. Children under 16 should not be usually injected into pleural or abdominal
allowed to visit while internal radiation is cavity.
in use. (a) May seep from instillation site or
3. Internal radiation: unsealed (radioisotope/ drainage tubes in cavity; stains purple.
radionuclide): (b) Turn client q15 min for 2 hours, as
a. Source of radiation is given orally or intra- in (2)(a).
venously or instilled into a cavity as a liquid. (c) Same precautions regarding handling
b. Nursing care plan/implementation: Goal: excreta as in (1)(a) and (2)(b).
reduce radiation exposure of others. 4. Precautions for nurses:
(1) Isolate client and tag room with radioactivity a. Use principles of time, distance, and shielding
symbol. when caring for clients who are having active

ADULT
(2) Rotate personnel to avoid overexposure radiation therapy treatments.
(principles of time, distance, and shielding). b. Nurses who may be pregnant should not
Staff should use good hand-washing tech- accept an assignment caring for clients who
nique. Client should be in a room with have active radiation in place.
running water. (Nurse who may be c. Always use gloves, gowns to protect skin and
pregnant should not care for client while clothing.
radiation source still active.) d. Wear detection badge to determine exposure
(3) Encourage family to maintain telephone to energy source.
contact or use intercom, to decrease I. Immunotherapy: it has been hypothesized that
exposure to others. clinical malignancy may occur as a result of failure
(4) Plan independent diversional activities. of the immunological surveillance system of the
c. Specific nursing precautions (post in chart, body to fight off cancer cells as they develop. The
on clients door): goal of immunotherapy is to immunize clients
(1) Radioactive iodine (131I): half-life 8.1 days; against their own tumors.
excreted in urine, saliva, perspiration, vom- 1. Nonspecific immunotherapyencourages a host
itus, feces. immune response by use of an unrelated agent.
(a) Wear gloves and isolation gowns when Bacille Calmette-Gurin (BCG) vaccine and
handling client, excreta, or dressings Corynebacterium parvum are the two agents used
directly. for this type of immunotherapy.
2164_Ch06_347-578 29/03/12 12:30 PM Page 530

530 chapter 6 Physiological Integrity

2. Specific immunotherapyuses substances that 2. Nursing care plan/implementation:


are antigenically related to the tumor that stimu- a. Goal: make client as comfortable as possible;
late a specific host immune response. involve nursing staff, family, support person-
3. Side effectsmalaise, chills, nausea, vomiting, nel, clergy, volunteers, support groups.
diarrhea; local reaction at site of injection, such Hospice is very valuable program.
as pruritus, scabbing. (1) Nutrition: obtain nutritional consultation;
4. Nursing care plan/implementation: high-calorie, high-protein diet; small,
a. Goal: decrease discomfort associated with side frequent meals; blenderized or strained;
effects of therapy. commercial nutritional supplements
(1) Identify measures to lessen symptoms of (Ensure, Vivonex, Sustacal).
side effects (see E. Nursing care plan/ (2) Prevent tissue breakdown and vascular
implementationgeneral care of the complications: frequent turning, massage,
client with cancer, pp. 525526). air mattress, active and passive ROM
(2) Know type of immunotherapy being used, exercises.
adverse and desirable effects of therapy. (3)GI tract disturbances: observe for toxic
(3) Administer fluids, encourage rest. reactions to therapy, particularly vomit-
(4) Administer acetaminophen as ordered to ing and diarrhea; administer medica-
decrease flu-like symptoms. tions: antiemetics, antidiarrheal agents
(5) Administer antiemetics as ordered for as ordered.
nausea. (4) Relieve pain.
(6) Monitor for respiratory distress. (a) Use supportive measures such as
(7) Administer analgesics as ordered for pain. massage, relaxation techniques,
b. Goal: health teaching. guided imagery; and drugs for pain
(1) Comfort measures to decrease side effects relief: administer codeine, fentanyl,
of therapy. aspirinoxycodone HCl (Percodan),
(2) Expected and side effects of therapy. pentazocine (Talwin), morphine,
(3) Investigational nature of therapy. methadone, as ordered.
(4) Care of site of administration. (b) Methods of administration: oral,
(5) Answer questions honestly. injected, rectal, analgesic patches,
J. Palliative care: when treatment has been ineffective or pumps (IV or SQ).
in control of the disease, the nurse must plan pal- (c) Monitor for side effects of narcotics:
liative, terminal care. Cure is not possible for such depressed respiratory status, constipa-
clients in an advanced phase of malignancy. tion, anorexia.
Symptoms increase in severity; clients and family b. Goal: assist client to maintain self-esteem and
have many special problems. identity.
1. General problems of clients with terminal (1) Encourage self-care.
cancer: (2) Spend time with client; isolation is a great
ADULT

a. Cachexia: progressive weakness, wasting, and fear for the client who is dying.
weight loss. c. Goal: assist client with psychological adjustment
b. Anemia: leukopenia, thrombocytopenia, see nursing care for clients who are grieving,
hemorrhage. clients who are dying (see Chapter 10).
c. Gastrointestinal disturbances: anorexia, K. Evaluation/outcome criteria:
constipation. 1. Tolerates treatment modalitycomplications
d. Tissue breakdown leading to decubiti, seeping of surgery are avoided or minimized;
wounds. tolerates chemotherapy; completes radiation
e. Urine: retention, incontinence, renal calculi, therapy.
tumor obstruction of ureters. 2. Side effects of treatment are managed by effec-
f. Hypercalcemia occurs in 10% to 30% of tive nursing care and health teaching.
clients. 3. Maintains good nutritional status.
g. Pain due to: tumor growth, obstruction, 4. Uses effective coping mechanisms or seeks
vertebral compression, or secondary to appropriate assistance to deal with psychosocial
complications (e.g., decubiti, stiffened joints, concerns.
stomatitis). Also neuropathy, due to pro- 5. Makes choices for follow-up care based on
longed use of neurotoxic chemotherapeutic accurate information.
agents such as vincristine. 6. Finds methods to control pain and minimize
h. Fatigue: major and debilitating problem. discomfort.
2164_Ch06_347-578 29/03/12 12:30 PM Page 531

Oncological Disorders 531


7. Participates in decisions regarding continuation 2. Goal: prevent complications related to surgery for
of therapy (living will, health-care proxy, client who is diagnosed early and for whom sur-
do-not-resuscitate [DNR] decisions). gery is an option: wedge or segmental resection,
8. Dignity maintained until death and during laser therapy, lobectomy, or pneumonectomy are
dying. usual procedures.
II. LUNG CANCER a. See Nursing care plan/implementation for
the client having thoracic surgery, p. 408.
A. Pathophysiology: squamous cell carcinoma: undiffer-
b. Monitor vital signs, including accurate respi-
entiated, pleomorphic in appearance; accounts for
ratory assessment for respiratory congestion,
45% to 60% of all lung cancer; small cell (oat cell)
blood loss, infection.
carcinoma: small, dark cells located between cells of
c. Assist client to deep breathe, cough, change
mucosal surfaces; characterized by early metastasis
position.
and poor prognosis; large cell (giant cell) carcinoma:
3. Goal: assist client to cope with alternative therapies
located in the peripheral areas of the lung, has poor
when surgery is deemed not possible.
prognosis; adenocarcinoma: found in men and
a. Radiation: megavoltage x-ray, cobaltusual
women; not necessarily related to smoking.
form of radiation (see Nursing care plan/
B. Risk factors:
implementation for the client having
1. Heavy cigarette smoking, 20-year smoking
radiation therapy, p. 528).
history.
b. Chemotherapy:
2. Exposure to certain industrial substances, such
(1) Cisplatin and VP-16 with irradiation
as asbestos.
has become standard form of induction
3. Increased incidence in women during the last
chemotherapy. Cyclophosphamide
decade of life.
(Cytoxan), doxorubicin (Adriamycin),
C. Assessment:
CCNU, methotrexate, vincristine sulfate
1. Subjective data:
(Oncovin) are the other drugs given for
a. Dyspnea.
lung cancer.
b. Pain: on swallowing; dull and poorly localized
(2) See Nursing care plan/implementation
chest pain, referred to shoulders.
for the client having chemotherapy,
c. Anorexia.
pp. 526527.
d. History of cigarette smoking over a period of
4. Goal: health teaching.
years; recurrent respiratory infections with
a. Encourage client to stop smoking to offer best
chills and fever, especially pneumonia or
possible air exchange.
bronchitis.
b. Encourage high-protein, high-calorie diet to
2. Objective data:
counteract weight loss.
a. Wheezing; dry to productive persistent cough;
c. Force fluids, to liquefy secretions so they can
hemoptysis, hoarseness.
be expectorated.
b. Weight loss.
d. Encourage adequate rest and activity to pre-
c. Positive diagnosis: cytology report of cells

ADULT
vent problems of immobility.
from bronchoscopy.
e. Desired effects and side effects of medications
d. Chest pain.
prescribed for therapy and pain relief.
e. Signs of metastasis.
f. Coping mechanisms for maximal comfort and
D. Analysis/nursing diagnosis:
advanced disease (see Palliative care, p. 530).
1. Ineffective breathing pattern related to pain.
F. Evaluation/outcome criteria:
2. Impaired gas exchange related to tumor growth.
1. Copes with disease and treatment.
3. Pain related to disease progression.
2. Side effects of treatment are minimized by
4. Fear related to uncertain future.
proper nursing management.
5. Powerlessness related to inability to control
3. Acid-base balance is maintained by careful
symptoms.
management of respiratory problems.
6. Knowledge deficit (learning need) related to dis-
4. Client is aware of the seriousness of the disease.
ease and treatment.
E. Nursing care plan/implementation: III. COLON AND RECTAL CANCER
1. Goal: make client aware of diagnosis and treat- A. Risk factors:
ment options. 1. Men, middle age, personal or family history
a. Allow time to talk and to discuss diagnosis. of colon and rectal cancer, personal or family
b. Client makes informed decision regarding history of polyps in the rectum or colon,
treatment. ulcerative colitis.
2164_Ch06_347-578 29/03/12 12:30 PM Page 532

532 chapter 6 Physiological Integrity

2. Diet high in beef and low in fiber. E. Evaluation/outcome criteria:


3. Gardners syndrome (multiple colonic adenoma- 1. Return of peristalsis and formed stool following
tous polyps, osteomas of the mandible or skull, resection and anastomosis.
multiple epidermoid cysts, or soft tissue tumors 2. Adjusts to alteration in bowel elimination route
of the skin). following abdominoperineal resection (e.g., no
B. Assessment: depression, resumes lifestyle).
1. Subjective data: 3. Demonstrates self-care skills with colostomy.
a. Change in bowel habits. 4. Makes dietary adjustments that affect elimina-
b. Anorexia. tion as indicated.
c. Weakness. 5. Identifies alternative methods of expressing
d. Abdominal cramping or vague discomfort sexuality, if needed.
with or without pain. IV. BREAST CANCER
e. Chills.
A. Risk factors:
2. Objective data:
1. Women older than age 50.
a. Diarrhea (pencil-like or ribbon-shaped feces)
2. Family history of breast cancer.
or constipation.
3. Never bore children, or bore first child after
b. Weight loss.
age 30.
c. Rectal bleeding; anemia.
4. Had breast cancer in other breast.
d. Fever.
5. Menarche before age 11.
e. Digital examination reveals palpable mass if
6. Menopause after age 50.
lesion is in ascending or descending colon.
7. Exposure to endogenous estrogens.
f. Signs of intestinal obstruction: constipation,
8. Exposure to ionizing radiation.
distention, pain, vomiting, fecal oozing.
9. High alcohol and fat intake may increase
g. Diagnostic tests:
risk.
(1) Digital examination.
B. Assessment:
(2) Slides of stool specimen, for occult blood.
1. Subjective data:
(3) Proctoscopy.
a. Burning, itching of nipple.
(4) Sigmoidoscopy, colonoscopy.
b. Reported painless lump.
(5) Barium enema.
2. Objective data:
h. Laboratory data: occult blood, blood sero-
a. Firm, nontender lump or mass.
tonin increased, carcinoembryonic antigen
b. Asymmetry of breast.
(CEA); positive radioimmunoassay of serum
c. Nippleretraction, discharge.
or plasma indicates presence of carcinoma or
d. Alteration in breast skinredness, dimpling,
adenocarcinoma of colon; positive result after
ulceration.
resection indicates return of tumor.
e. Palpation reveals lump.
C. Analysis/nursing diagnosis:
f. Diagnostic tests: mammography, needle
1. Constipation or diarrhea related to presence of
biopsy, core biopsy, excisional biopsylevel
ADULT

mass.
of estrogen-receptor protein predicts response
2. Altered health maintenance related to care of
to hormonal manipulation of metastatic dis-
stoma.
ease and may represent a prognostic indicator
3. Sexual dysfunction related to possible nerve
for primary cancer; carcinoembryonic antigen
damage during radical surgery.
(CEA) useful with metastatic disease of the
4. Body image disturbance related to colostomy.
breast.
D. Nursing care plan/implementation (see also
C. Analysis/nursing diagnosis:
Nursing care plan/implementationgeneral care
1. Risk for injury related to surgical intervention.
of the client with cancer, pp. 525526):
2. Body image disturbance related to effects of sur-
1. Radiation: to reduce tumor or for palliation.
gery, radiation, or chemotherapy.
2. Chemotherapy: to reduce tumor mass and
3. Altered sexuality patterns related to loss of breast.
metastatic lesions.
D. Nursing care plan/implementation (see also
a. Antitumor antibioticsmitomycin C,
Nursing care plan/implementationgeneral care
doxorubicin HCl (Adriamycin).
of the client with cancer, pp. 525526):
b. Alkylating agentsmethyl-CCNU.
1. Goal: assist through treatment protocol.
c. Antimetabolites5-fluorouracil (5-FU).
a. Radiationprimary treatment modality;
d. Steroids and analgesics for symptomatic relief.
adjunctive, external, or implantation to
3. Prepare client for surgery (colostomy) if
primary lesion site or nodes.
necessary.
2164_Ch06_347-578 29/03/12 12:30 PM Page 533

Oncological Disorders 533


b. Chemotherapy usually given in combinations. (2) Postoperative:
(1) Cytotoxic agents to destroy tumor and (a) Goal: facilitate healing.
control metastasis. (i) Observe pressure dressings for
(2) Alkylating agents: cyclophosphamide bleeding; will appear under
(Cytoxan). axilla and toward the back.
(3) Antitumor antibiotics: doxorubicin (ii) Report if dressing becomes
(Adriamycin). saturated; reinforce dressing as
(4) Antimetabolites: fluorouracil (5-FU); need; monitor drainage from
methotrexate (Amethopterine, MTX). Hemovac or suction pump.
(5) Plant alkaloids: vincristine sulfate (iii) Position: semi-Fowlers to facilitate
(Oncovin). venous and lymphatic drainage; use
(6) Hormones to control metastasis, provide pillows to elevate affected arm above
palliation: androgens, fluoxymesterone right atrium, to prevent edema if
(Halotestin), testosterone (Teslac), nodes removed.
diethylstilbesterol (estrogen). (b) Goal: prevent complications.
(7) Antiestrogens: tamoxifen (Nolvadex) may (i) Monitor vital signs for shock.
be used after initial treatment. (ii) Use gloves when emptying
(8) Cortisols: cortisone, prednisolone drainage.
(Delta-Cortef ), prednisolone acetate (iii) Maintain joint mobilityflexion
(Meticortelone), prednisone (Deltasone, and extension of fingers, elbow,
Deltra). shoulder.
c. Surgery. (iv) ROM as ordered to prevent
(1) Preoperative: ankylosis.
(a) Goal: prepare for surgerytypes: (v) If skin graft done, check donor
(i) Lumpectomy (with or without site and limit exercises.
radiation)used when lesion is (c) Goal: facilitate rehabilitation.
small; section of breast is removed (i) Encourage client, significant others,
with clear margin around lesion and family to look at incision.
(often accompanied by radiation (ii) Involve client in incisional care, as
therapy and then radium interstitial tolerated.
implant). (iii) Refer to Reach to Recovery program
(ii) Simple mastectomybreast of the American Cancer Society
removed, no alteration in nodes. (iv) Exercise program, hydrotherapy for
(iii) Modified radical mastectomy clients who are postmastectomy, to
breast, some axillary nodes, reduce lymphedema.
subcutaneous tissue removed; (d) Goal: health teaching.
pectoralis minor muscle removed. (i) How to avoid injury to affected

ADULT
(iv) Radical mastectomybreast, axillary area; how to prevent lymphedema.
nodes, and pectoralis major and (ii) Exercises to gain full ROM.
minor muscles removed. (iii) Availability of prosthesis, recon-
(v) Reconstructive surgerydone at structive surgery.
time of initial mastectomy or (iv) Correct breast self-examination
(most often) later, when other (BSE) technique (client is at risk
adjuvant therapy has been for breast cancer in remaining
completed. breast) (Fig. 6.16). Best time for
(b) Goal: promote comfort. examination: women who are pre-
(i) Allow client and family to express menopausal, seventh day of cycle;
fears, feelings. women who are postmenopausal,
(ii) Provide correct information about same day each month.
diagnostic tests, operative proce- E. Evaluation/outcome criteria:
dure, postoperative expectations. 1. Identifies feelings regarding loss.
(iii) Client may be hospitalized for 2. Demonstrates postmastectomy exercises.
24 hours or less. Have telephone 3. Gives rationale for avoiding fatigue and avoiding
number available for questions. constricting garments on affected arm; necessity
Make appropriate community for avoiding injury (cuts, bruises, burns) while
referrals. carrying out activities of daily living.
2164_Ch06_347-578 29/03/12 12:30 PM Page 534

534 chapter 6 Physiological Integrity

A B C

D E F
Figure 6.16 Breast self-examination. (A) Examine breasts during bath or shower because flat fingers glide easily over wet
skin. Use right hand to examine left breast and left hand to examine the right breast. (B) Sit or stand before a mirror. Inspect
breasts with hands at sides, then raised overhead. Look for changes in contour or dimpling of skin. (C) Place hands on hips
and press down firmly to flex chest muscles. (D) Lie down with one hand under head and pillow or folded towel under that
scapula. (E) Palpate that breast with the other hand using concentric circle method. It usually takes three circles to cover all
breast tissue. Include the tail of the breast and the axilla. Repeat with other breast. (F) End in a sitting position. Palpate the
ADULT

areola areas of both breasts, and inspect and squeeze nipples to check for discharge.

4. Describes signs and symptoms of infection. 4. Obesity.


5. Demonstrates correct BSE technique. 5. Menopause after age 52.
V. UTERINE CANCER (endometrial): originates from 6. Diabetes.
epithelial tissues of the endometrium; second only to B. Assessment:
cervical cancer as cause of pelvic cancer. Slow grow- 1. Subjective data:
ing; metastasizes late; responsive to therapy with early a. History of risk factor(s).
diagnosis; Papanicolaou (Pap) test not as effective b. Pain (late symptom).
more effective to have endometrial tissue sample 2. Objective data:
(Table 6.40 and Table 6.41). Table 6.42 discusses a. Obese.
cervical cancer. b. Abnormal cells obtained from aspiration of
endocervix or endometrial washings.
A. Risk factors:
c. Postmenopausal uterine bleeding.
1. History of infertility (nulliparity).
d. Abnormal menses; intermenstrual or unusual
2. Failure of ovulation.
discharge.
3. Prolonged estrogen therapy.
2164_Ch06_347-578 29/03/12 12:30 PM Page 535

Oncological Disorders 535


C. Analysis/nursing diagnosis: D. Nursing care plan/implementation (see also
1. Pain related to surgery. Nursing care plan/implementationgeneral care
2. Risk for injury related to surgery. of the client with cancer, pp. 525526):
3. Body image disturbance related to loss of uterus. 1. Goal: assist client through treatment protocol.
a. Radiationexternal, internal, or both with
client who is a poor surgical risk.
Table 6.40 b. Chemotherapyto reduce tumors and produce
Papanicolaou (Pap) Smear Classes remission of metastasis. Antineoplastic drugs:
dacarbazine (DTIC), doxorubicin
Class Recommended Actions
(Adriamycin), medroxyprogesterone acetate
I Normal (Provera), megestrol acetate (Megace).
II Atypical cells, nonmalignant Treat vaginal infections; 2. Goal: prepare client for surgerytypes:
repeat Pap smears a. Subtotal hysterectomy: removal of the uterus;
III Suspicious cells Biopsy; dilation and cervical stump remains.
curettage b. Total hysterectomy: removal of entire uterus,
IV Abnormal cells; suspicious Biopsy: dilation and including cervix (abdominally [approximately
of malignancy curettage, conization 70%] or vaginally).
V Malignant cells present See Table 6.41 c. Total hysterectomy with bilateral salpingo-
oophorectomy: removal of entire uterus,
fallopian tubes, and ovaries.
3. Goal: reduce anxiety and depression: allow for
Table 6.41 expression of feelings, concerns about femininity,
Uterine Cancer: Recommended Treatment, role, relationships.
by Stage of Invasion 4. Goal: prevent postoperative complications.
a. Catheter caretemporary bladder atony
Stage of Invasion Recommended Treatment
may be present as a result of edema or nerve
0 (In situ) Atypical Cryosurgery, conization trauma, especially when vaginal approach is
hyperplasia
used.
I Uterus is of normal size Hysterectomy b. Observe for abdominal distention and
II Uterus slightly enlarged, Radiation implant, x-ray; hemorrhage:
but tumor is hysterectomy 46 wk (1) Auscultate for bowel sounds.
undifferentiated postradiation (2) Measure abdominal girth.
III Uterus enlarged, tumor Radiation implant, total (3) Use rectal tube to decrease flatus.
extends outside uterus hysterectomy 46 wk c. Decrease pelvic congestion and prevent venous
postradiation stasis.
IV Advanced metastatic Radiation, chemotherapy; (1) Avoid high Fowlers position.
disease progestin therapy to (2) Antiembolic stockings as ordered.
reduce pulmonary lesions

ADULT
Table 6.42
International System of Staging for Cervical Carcinoma
Stage Location Prognosis Treatment
0 In situ Highly curable Conization
I Cervix Cure rate decreases Radiation
as stage progresses
II Cervix to upper vagina Radiation
III Cervix to pelvic wall or lower Surgeries:
third of vagina 1. Panhysterectomy, wide vaginal excision with removal of
lymph nodes; ileal conduit
IV Cervix to true pelvis, bladder, 2. Pelvic exenteration:
or rectum a. Anterior: removal of vagina and bladder; ileal conduit
b. Posterior: removal of rectum and vagina; colostomy
c. Total: both anterior and posterior
3. Chemotherapy
2164_Ch06_347-578 29/03/12 12:30 PM Page 536

536 chapter 6 Physiological Integrity

(3) Institute passive leg exercises. (2) Urine RBCs (hematuria).


(4) Apply abdominal support as ordered. (3) Gleason score for prostate cancer grading
(5) Encourage early ambulation. system (range: 2 to 10).
5. Goal: support coping mechanisms to prevent C. Analysis/nursing diagnosis:
psychosocial response of depression: allow for 1. Altered urinary elimination related to
verbalization of feelings. incontinence.
6. Goal: health teaching to prevent complications 2. Altered sexuality pattern related to nerve
of hemorrhage, infection, thromboemboli. damage and erectile dysfunction.
a. Avoid: 3. Anxiety related to diagnosis.
(1) Douching or coitus until advised by 4. Pain related to metastasis to bone.
physician. D. Nursing care plan/implementation (see also
(2) Strenuous activity and work for 2 months. Nursing care plan/implementationgeneral
(3) Sitting for long time and wearing constric- care of the client with cancer, pp. 525526):
tive clothing, which tend to increase pelvic 1. Goal: assist client through decisions about
congestion. treatment protocol (varies by stage: 0 to IV).
b. Explain hormonal replacement if applicable; a. Radiationalone or in conjunction with
correct dosage, desired and side effects of surgery. Types: external beam radiation,
prescribed medications. 3-D conformal (focal), radioactive seed
c. Explain: implants (brachytherapy).
(1) Menstruation will no longer occur. b. Surgerycryosurgery; radical retropubic
(2) Importance of reporting symptoms prostatectomy (see XI. PROSTATECTOMY,
(e.g., fever, increased or bloody vaginal p. 466).
discharge, and hot flashes). c. Other options: hormones (luteinizing
E. Evaluation/outcome criteria: hormonereleasing hormone agonists [Lupron,
1. Adjusts to altered body image. Zoladex, Casodex, Nilandron]), antiandrogen
2. No complicationshemorrhage, shock, (flutamide [Eulexin]); drugs in conjunction
infection, thrombophlebitis. with orchiectomy, to limit production of
VI. PROSTATE CANCER: malignant neoplasm, androgens (androgen deprivation therapy).
usually adenocarcinoma; most common cause of d. Watchful waitingrecommended with small
cancer in men. contained tumor; older men; where surgery
is contraindicated for other serious health
A. Risk factors:
problems.
1. Men older than age 50.
E. Evaluation/outcome criteria (see XI.
2. Familial history.
PROSTATECTOMY, p. 466).
3. Geographic distribution, environmental
(e.g., industrial exposure to cadmium). VII. BLADDER CANCER: bladder is most common
4. Hormonal factors (testosterone). site of urinary tract cancer.
5. Diet (high fat). A. Risk factors:
ADULT

B. Assessment: 1. Contact with certain dyes.


1. Subjective data: 2. Cigarette smoking.
a. Difficulty in starting urinary stream (hesitancy); 3. Excessive coffee intake.
urgency. 4. Prolonged use of analgesics with phenacetin.
b. Pain due to metastasis in lower back, hip, 5. Three times more common in men.
legs; perianal or rectal discomfort. B. Assessment:
c. Symptoms of cystitis; frequency, urgency. 1. Subjective data:
2. Objective data: a. Frequency, urgency.
a. Urinary: smaller, less forceful stream; terminal b. Pain: flank, pelvic; dysuria.
dribbling; frequency, nocturia; retention 2. Objective data:
(inability to void after ingestion of alcohol or a. Painless hematuria (initially).
exposure to cold). b. Diagnostic tests:
b. Diagnostic tests: digital rectal examination (1) Cystoscopy, intravenous pyelogram
(DRE); transrectal ultrasonography (TRUS). (IVP)mass or obstruction.
Needle biopsy or tissue specimen reveals (2) Bladder biopsy, urine cytologymalignant
positive cancer cells. cells.
c. Laboratory data: increased: c. Laboratory data: urinalysisincreased RBCs
(1) Prostate-specific antigen (PSA)over (>4.8 1012/Lmen, >4.3 1012/L
4 ng/mL. women).
2164_Ch06_347-578 29/03/12 12:30 PM Page 537

Oncological Disorders 537


C. Analysis/nursing diagnosis: VIII. LARYNGEAL CANCER
1. Risk for injury related to surgical intervention. A. Risk factors:
2. Altered urinary elimination related to surgery. 1. Eight times more common in men.
D. Nursing care plan/implementation (see also 2. Occurs most often after age 60.
Nursing care plan/implementationgeneral care 3. Cigarette smoking.
of the client with cancer, pp. 525526): 4. Alcohol.
1. Goal: assist client through treatment protocol. 5. Chronic laryngitis, vocal abuse.
a. Radiation: cobalt, radioisotopes, radon seeds; 6. Family predisposition to cancer.
often before surgery to slow tumor growth. B. Assessment:
b. Chemotherapy: 1. Subjective data:
(1) Antitumor antibiotics: doxorubicin HCl a. Dysphagiapain in areas of Adams apple;
(Adriamycin), mitomycin. radiates to ear.
(2) Antimetabolites: 5-fluorouracil (5-FU). b. Dyspnea.
(3) Alkylating agents: thiotepa. 2. Objective data:
(4) Sedatives, antispasmodics. a. Persistent hoarseness.
2. Goal: prepare client for surgerytypes: b. Cough and hemoptysis.
a. Transurethral fulguration or excision: used c. Enlarged cervical nodes.
for small tumors with minimal tissue d. General debility and weight loss.
involvement. e. Foul breath.
b. Segmental resection: up to half the bladder f. Diagnosis made by history, laryngoscopy with
may be resected. biopsy and microscopic study of cells.
c. Cystectomy with urinary diversion: complete C. Analysis/nursing diagnosis:
removal of the bladder; performed when 1. Impaired verbal communication related to
disease appears curable. removal of larynx.
3. Goal: assist with acceptance of diagnosis and 2. Body image disturbance related to radical
treatment. surgery.
4. Goal: prevent complication during postoperative 3. Ineffective airway clearance related to increased
period. secretions through tracheostomy.
a. Transurethral fulguration or excision: D. Nursing care plan/implementation (see also
(1) Monitor for clots, bleeding, spasms. Nursing care plan/implementationgeneral
(2) Maintain patency of Foley catheter. care of the client with cancer, pp. 525526):
b. Urinary diversion with stoma: treatment primarily surgical (see Laryngectomy,
(1) Protect skin, ensure proper fit of pp. 384386); radiation therapy may also be
appliancebecause constantly wet indicated.
with urine (see also Ileal conduit, E. Evaluation/outcome criteria: see Laryngectomy,
pp. 466467 and Ostomies and stoma p. 386.
care, pp. 454456).
IX. ADDITIONAL TYPES OF CANCER

ADULT
(2) Prevent infection by increasing acidity of
(Table 6.43).
urine and increasing fluid intake.
(3) Health teaching.
(a) Self-care of stoma and appliance. EMERGENCY
(b) Expected and side effects of NURSING
medications. PROCEDURES
(c) Importance of follow-up visits for
I. PURPOSEto initiate assessment and intervention
early detection of metastasis.
procedures that will speed total care of the client
E. Evaluation/outcome criteria:
toward a successful outcome.
1. Accepts treatment plan.
2. Uses prescribed measures to decrease side effects II. EMERGENCY NURSING PROCEDURES for
of surgery, radiation, chemotherapy. adults are detailed in Table 6.44, pp. 542552.
3. Plans follow-up visits for further evaluation. III. LEGAL ISSUES (see Chapter 3).
4. Maintains dignity.
2164_Ch06_347-578 29/03/12 12:30 PM Page 538

538 chapter 6 Physiological Integrity

Table 6.43
Selected Cancer Problems
Assessment
Subjective Data Objective Data Risk Factors Specific Treatment
GASTROINTESTINAL TRACT
Oral cancer Difficulty chewing, Sore that bleeds and Heavy smoking and Surgery, with reconstructive
swallowing, moving does not heal; persist- drinking, user of surgery useful for cure and
tongue or jaws; history ent red or white patch; chewing tobacco, palliatively (see F. General surgical
of heavy smoking, diagnosis by biopsy; men >age 40 (affects intervention, p. 526)
alcohol use, or early detection: dental twice as many men Radiation using simulated computer
chewing tobacco checks as women) localization to avoid destruction of
normal tissue (see care of client hav-
ing radiation therapy, pp. 527529)
Esophageal Dysphagiadifficulty Weight loss; regurgita- Over age 50, alco- Surgery: resection with
cancer in swallowing; dis- tion, vomiting; holism, use of tobac- anastomosis, or removal
comfort described as: diagnostic tests co; increasing risk in with gastrostomy
lump in throat, pres- barium swallow, nonwhite women, in Radiation: best form of therapy
sure in chest, pain; esophagoscopy, people with achala- Chemotherapy: antineoplastic
fatigue, lethargy, biopsy sia (inability to relax drugs ineffective; medications
apathy, depression; lower esophagus to reduce symptoms of pain,
anorexia with swallowing) or discomfort, and anxiety (see nursing
hiatal hernias care plan/implementation for client
with cancer [p. 525]; having radia-
tion therapy [pp. 527529]; having
chemotherapy [pp. 526527]; having
surgery [p. 526]
Stomach Vague feeling of Eructation, regurgita- Men, lower socioe- Surgery: gastrectomy (see gastric
cancer fullness, pressure, or tion, vomiting; melena, conomic classes, surgery, p. 446, care)
epigastric pain hematemesis, anemia; colder climates, early Radiation: not as useful because
following ingestion of jaundice, diarrhea, exposure to dietary dosage needed would cause side
food; anorexia, nau- ascites; big belly, carcinogens, blood effects unlikely to be tolerated by
sea, meat intolerance; upper gastric area; group A, pernicious client
malaise often palpable mass anemia, atrophic Chemotherapy alone or in
achlorhydric gastritis conjunction with surgery:
antitumor antibiotics, anti-
metabolites, nitrosoureas, hematinics
(see nursing care plan for client
having chemotherapy, pp. 526527)
Pancreatic Anorexia, nausea; Jaundice, vomiting, Excessive use of Surgery: removal (must
cancer pain in upper weight loss; determi- alcohol; exposure to then have supplemental
ADULT

abdomen, radiating to nation of solid mass in dry-cleaning chemi- pancreatic enzymes; some
back; dyspnea area of pancreas by cals, gasoline; coffee clients become diabetic, type 1) or
computed tomography and decaffeinated bypass to relieve obstruction (see
and ultrasound; tissue coffee; possibly nursing care plan/implementation for
identification by thin- diabetes and chronic client with diabetes, pp. 468469)
needle percutaneous pancreatitis Chemotherapy: pain relief,
biopsy antiemetics, insulin,
pancrelipase, 5-fluorouracil
cyclophosphamide (5-FU), Cytoxan,
carmustine (BCNU), methotrexate,
vincristine, mitomycin-C (see nursing
care plan/implementation for client
having chemotherapy, pp. 526527)
Radiation: intraoperative high dose
to pancreatic tumors with external
high beam; palliative radiation therapy
for pain (see nursing care plan/
implementation for client having
radiation therapy, pp. 527529; see
nursing care plan/implementation for
client with cancer, p. 525)
2164_Ch06_347-578 29/03/12 12:30 PM Page 539

Oncological Disorders 539

Table 6.43
Selected Cancer Problemscontd
Assessment
Subjective Data Objective Data Risk Factors Specific Treatment
SKIN
Skin cancer: Reported painless Scaly plaques, papules Exposure to: sun, Surgery: electrodesiccation (dehy-
basal cell lesion that ulcerate; pale, coal tar, pitch, dration of tissue by use of needle
waxy, pearly nodule or arsenic compounds, electrode); cryosurgery (destruction
red, sharply outlined creosote, radium; fair of tissue by application of extreme
patch; unusual skin complexion cold; see care of cancer client,
condition, change in p. 525).
size or color, or other Radiation therapy (see care of
darkly pigmented client having radiation therapy,
growth or mole pp. 527529)
Prevention: avoid sun from
10 a.m. to 3 p.m.; use protective
clothing, sunblock lotion
NERVOUS SYSTEM
Brain Headache: steady, Vomiting, may be pro- None known for pri- Surgery: craniotomy with excision of
intermittent, severe jectile; sight loss, audi- mary tumors; brain is lesion; ventricular shunt to allow for
(may be intensified by tory changes; signs of common site for drainage of fluid (see nursing care
physical activity); nau- increased intracranial metastasis plan/implementation for craniotomy,
sea; lethargy, easy pressure; seizures; pp. 380381)
fatigability; forgetful- paresthesia; behavior Radiation: cobalt (local or entire
ness, disorientation, changes. CNS); total brain radiation causes
impaired judgment; Diagnostic alopecia, which may be permanent
visual disturbances; studiesCT scan, (see nursing care plan/implementation
blackouts arteriography, for client having radiation therapy,
cytology of pp. 527529); could be used
cerebrospinal fluid alone, with surgery, or with
chemotherapy
Chemotherapy: antineoplastic
alkylating agents: nitrosoureas
(cross blood-brain barrier to
reduce tumor)carmustine (BCNU),
lomustine (CCNU), semustine
(methyl-CCNU); cerebral diuretics to
reduce edema; anticonvulsants; anal-
gesics; sedatives (see nursing care
plan/implementation for client having
chemotherapy, pp. 526527)

ADULT
ENDOCRINE
Thyroid Painless nodule; Enlarged thyroid, Radiation exposure Surgery: total thyroidectomy,
cancer dysphagia; difficulty thyroid nodule; palpa- in childhood possible radical neck dissection
breathing ble thyroid, lymph (see IV. THYROIDECTOMY, p. 473)
nodes; hoarseness; Radiation: external, or with radioac-
hypofunctional nodule tive iodine (131I) (see care of
seen on isotopic imag- client having radiation therapy,
ing scanning; needle pp. 527529)
biopsy for cytology Chemotherapy: chlorambucil,
studies doxorubicin, vincristine
(Leukeran, Adriamycin, Oncovin,
respectively) (see care of client having
chemotherapy, pp. 526527)
BLOOD AND LYMPH TISSUES
Painless enlargement
Hodgkins Fatigue; generalized of lymph nodes, espe- For young adults from Staging and treatment:
disease pruritus; anorexia cially in cervical area; 15 to 35 yr old, not Stage Iinvolvement of a single
fever, night sweats; clearly defined, some node or a single node region;
hepatosplenomegaly; relationship to socioe- excision of lesion, and total nodal
anemia; peak age of conomic status; radiation (see care of client having
men-women ratio radiation therapy, pp. 527529)
Continued
2164_Ch06_347-578 29/03/12 12:30 PM Page 540

540 chapter 6 Physiological Integrity

Table 6.43
Selected Cancer Problemscontd
Assessment
Subjective Data Objective Data Risk Factors Specific Treatment
incidence, 1535; is 1.5:1; increased fre- Stage IIinvolvement of two or
quency among White more lymph node regions on same
Diagnostic tests
persons side of diaphragm; excision of lesion
biopsy shows
and radiation (see care of client hav-
presence of Reed-
ing radiation therapy, pp. 527529)
Sternberg cells;
Stage IIIinvolvement of lymph
x-rays, scans,
node regions on both sides of the
laparotomy
diaphragm, which may include the
spleen; combination of radiation
and chemotherapy
Stage IVinvolvement of one or more
extralymphatic organs or tissues, with
or without lymphatic involvement;
treated with chemotherapy alone,
radiation therapy alone, or both
Presence or absence of symptoms
of night sweats, significant
fever, and weight loss; treated
with chemotherapy, MOPP
Mustargen (mechlorethamine HCl,
alkylating agent), Oncovin (vin-
cristine, plant alkaloid), procarbazine
(antineoplastic); prednisone (corti-
costeroid) (see care of client having
chemotherapy, pp. 526527)
Non- Night sweats Nontender lym- Ages 5060 yr Stage Irarely observed, but remis-
Hodgkins phadenopathy, sion possible with radiation therapy
lymphoma enlarged liver and Stage IIradiation therapy
spleen, weight loss, Stage III and IV combination
fever chemotherapy, with or without
radiation therapy
Multiple Weakness; history of Fractures of long Exposure to ionizing Surgery: relieve spinal cord compres-
myeloma frequent infections, bones; deformity radiation; middle- sion; orthopedic procedures to relieve
especially pneumo- of: sternum, ribs, aged or older women or support bone problems (see nursing
nias; severe bone vertebrae, pelvis; care plan/implementation for clients
pain on motion; neu- hepatosplenomegaly; with internal fixation for fractures,
rological symptoms, anemia and bleeding p. 506; client with spinal cord injuries
ADULT

paralysis tendencies when paralysis occurs, pp. 520521)


Radiation: for some lesions (see
Elevated uric acid
nursing care plan/implementation
for clients having radiation therapy,
pp. 527529)
Chemotherapy: alkylating
agents; antitumor antibiotics;
plant alkaloids; hormonesmelphalan
and prednisone (see nursing care
plan/implementation for clients having
chemotherapy, pp. 526527)
URINARY ORGANS
Kidney Fatigue; abdominal or Painless, gross hema- More common Surgery: stereotactic surgery;
cancer flank pain; night turia; firm, nontender, among men than gamma knife (for brain metastasis);
sweats palpable kidney; women, Whites than nephrectomy (see Nursing care plan/
weight loss; fever of Blacks; lymphoma; implementation for pre/postoperative
unknown origin (FUO); smoking; exposure care, pp. 365367, 369375)
testicular enlargement. to chemicals used Radiation: irradiation of metastatic
Complications: hyper- in leather manufac- sites when tumor is radiosensitive
tension; nephrotic syn- turing; radiation (see Nursing care plan/implementation
drome; metastasis to exposure; possible for client having radiation therapy,
lung, brain, liver, bones familial influence; pp. 527529)
2164_Ch06_347-578 29/03/12 12:30 PM Page 541

Oncological Disorders 541

Table 6.43
Selected Cancer Problemscontd
Assessment
Subjective Data Objective Data Risk Factors Specific Treatment
(e.g., scapula or common site of Chemotherapy: plant
pelvis); vena cava metastasis from lung alkaloidsvincristine
involvement (Oncovin); antitumor antibiotics
Laboratory and diag- dactinomycin (Actinomycin D),
nostic tests: CT scans doxorubicin (Adriamycin), gemci-
(chest, abdomen, tabine (Gemzar); alkylating agents
pelvis) with contrast; cyclophosphamide (Cytoxan); IL-2;
MRI; bone scan; interferon alpha (see Nursing care
urinalysispresence plan/implementation for client having
of red cells and albu- chemotherapy, pp. 526527)
min; CBCdecrease Clinical trials: Avastin, bone marrow
in red cells and leuko- transplant, Bay 43
cytes, reduction in
serum albumin, eleva-
tion of alpha globulin
and calcium
GENITAL ORGANS
Testicular Aching or dragging Gynecomastia; Second most com- Surgery: orchiectomy (see Nursing
cancer sensation in groin, enlargement, swelling, mon malignancy care plan/implementation for the pre-
usually painless lump, hardening of among men ages operative and postoperative client,
testes; young adult 2540 yr; possibly pp. 365367, 368369, 369375).
men early diagnosis exposure to chemi- Radiation: see nursing care plan/
monthly testicular cal carcinogens; implementation for client having
self-examination trauma, orchitis; radiation therapy, pp. 527529)
(see Fig. 6.17) gonadal dysgenesis; Chemotherapy: cisplatin,
cryptorchidism bleomycin, etoposide, steroids
(undescended (see nursing care plan/
testicles) implementation for client having
chemotherapy, pp. 526527)
Cervical Vague pelvic or low Intermenstrual, post- Early age at first Staging (see Table 6.42)
cancer back discomfort, coital, or post- intercourse; multiple Stage 0carcinoma in situ; no dis-
pressure, or pain menopausal bleeding, sex partners; low tinct tumor observable; stage may
vaginal discharge socioeconomic last for 810 yr; cure rate 100% fol-
serosanguineous status; exposure to lowing treatment of wedge or cone
and malodorous herpes simplex virus resection of cervix during childbear-
hypermenorrhea; type 2 ing years, or simple hysterectomy
abdominal distention Stage Imalignant cells infiltrate

ADULT
with urinary frequency; cervical mucosa; lesion bleeds easily;
abnormal Pap test cure rate 80% with treatment of
(see Table 6.40); rec- hysterectomy
ommended guidelines Stage IIneoplasm spreads through
by American Cancer cervical muscular layers, involves
Society: Pap test upper third of vaginal mucosa; cure
annually; after three rate 50% with treatment of radical
consecutive normal hysterectomy
tests, physician may Stage IIIneoplasm involves lower
recommend less third of vagina; cure rate 25% with
frequent testing; pelvic exenteration
pelvic/uterine exami- Stage IVinvolves metastasis to
nation every 3 yr bladder, rectum, and surrounding
tissues; considered incurable
Radiation: external or internal or both,
in conjunction with surgery or alone,
depending on stage of disease or
condition of client (see nursing care
plan/implementation for client having
radiation therapy, pp. 527529)
Chemotherapy: progestin,
antineoplastics, megestrol
(Megace), medroxyproges-
terone (Curretab, Provera); alkylating
agentsdacarbazine (DTIC)
2164_Ch06_347-578 29/03/12 12:30 PM Page 542

542 chapter 6 Physiological Integrity

Figure 6.17 Testicular self-examination. (From Morton, PG: Health Assessment in Nursing. FA Davis, Philadelphia, 1993, p 451.)
ADULT

Table 6.44
Nursing Care of the Adult in Medical and Surgical Emergencies
Assessment: Signs
Condition and Symptoms Prehospitalization Nursing Care In-hospital Nursing Care
Cardiovascular Emergencies
Myocardial infarction Prehospital: 1. If coronary suspected, call 1. Rapidly assess hemodynamic and
ischemia and necrosis Chest pain: viselike, physician, paramedic service, respiratory status
of cardiac muscle sec- choking, unrelieved by or emergency ambulance 2. Place on cardiac monitor to deter-
ondary to insufficient or rest or nitroglycerin 2. Calm and reassure client that mine treatment
obstructed coronary Skin: ashen, cold, help is coming 3. Place on O2
blood flow clammy 3. Place in semi-Fowlers 4. Start IV as orderedusually
Vital signs: pulserapid, position D5W per microdrip to
weak, thready; increased 4. Keep client warm but establish lifeline for emer-
rate and depth of respi- not hot gency drug treatment
rations; dyspnea 5. Relieve painmorphine SO4
Behavior: restless, IV as needed, aspirin,
anxious nitroglycerin
2164_Ch06_347-578 29/03/12 12:31 PM Page 543

Medical-Surgical Emergencies 543

Table 6.44
Nursing Care of the Adult in Medical and Surgical Emergenciescontd
Assessment: Signs
Condition and Symptoms Prehospitalization Nursing Care In-hospital Nursing Care
In hospital: 6. Draw blood for electrolytes,
C/V: blood pressure enzymes, as ordered
and pulse pressure 7. Take 12-lead ECG
decreased 8. Once client is stable, transfer to
Heart sounds: soft; S3 coronary care unit (CCU)
may be present
Respirations: fine
basilar crackles
Laboratory: ECG
consistent with tissue
necrosis (Q waves)
and injury
(ST-segment
elevation); serum
enzymes elevated
(CK-MB, troponin)
Cardiac arrestcardiac Cyanosis, gasping CPR 1. If monitored, note rhythm; call for
standstill or ventricular Respirations: rapid, 1. Position: flat on back help and note time
fibrillation secondary to shallow, absent 2. Check for no breathing 2. Immediate countershock if rhythm
rapid administration or Pulse: weak, thready, or no normal breathing is ventricular fibrillation or ventricu-
overdose of anesthetics >120 beats/min, (only gasping) while checking lar tachycardia
or narcotic drugs, absent for unresponsiveness 3. If countershock unsuccessful,
obstruction of the res- Muscle: twitching 3. Activate the emergency begin CPR, as in prehospital care
piratory tract (mucus, Pupils: dilated response system: call 9-1-1.
vomitus, foreign body), Skin: cold, clammy Get automated external defib-
acute anxiety, cardiac Loss of consciousness rillator (AED) or send someone
disease, dehydration, else to get; use within 5 min
shock, electric shock, 4. Take 10 sec to check pulse
or emboli 5. If no pulse, begin chest com-
pressions30 compressions
at a rate of 100/min. Depress
sternum at least 2 inches
6. After 30 compressions, open
airway and give 2 breaths
(CABcompressions, airway,
breathing) within 10 sec
7. Use AED when available

ADULT
No heartbeat:
One- or two-person lay rescuer Two-person rescue
1. CPR: 100 chest compressions First person: begins CPR as
per minute, with two rescue described in prehospital care
breaths between every 30 Second person:
compressions (no difference 1. Pages arrest team
in ratio for one or two 2. Brings defibrillator to bedside and
persons); countershocks, if indicated by
2. Check pulse at neck after rhythm; defibrillate within 3 min of
5 cycles or 2 min arrest if indicated
3. Provide early defibrillation with 3. Brings emergency cart to
AED within 5 min of arrest if bedside
indicated 4. Suctions airway, if indicated due to
4. If heartbeat returns: assist vomitus or secretions
respiration and monitor pulse; 5. Bags client with 100% O2
continue CPR until help 6. Assists with intubation when arrest
arrives team arrives
7. Establishes intravenous line if one
is not available

Continued
2164_Ch06_347-578 29/03/12 12:31 PM Page 544

544 chapter 6 Physiological Integrity

Table 6.44
Nursing Care of the Adult in Medical and Surgical Emergenciescontd
Assessment: Signs
Condition and Symptoms Prehospitalization Nursing Care In-hospital Nursing Care
Shockcellular hypoxia Early shock 1. Check breathingclear airway 1. Check vital signs rapidlypulse,
and impairment of Sensorium: conscious, if necessary; if no breathing, pupils, respirations
cellular function apprehensive, and give artificial respirations; if 2. Check airway; clear if necessary;
secondary to: trauma, restless; some slurring breathing is irregular or draw ABGs; PO2 should be main-
hemorrhage, fright, of speech labored, raise head and tained above 60 mm Hg; elevated
dehydration, cardiac Pupils: dull but reactive shoulders PCO2 indicates need for intubation
insufficiency, allergic to light 2. Control bleeding by placing and ventilatory assistance
reactions, septicemia, Pulse: rate <140/min; pressure on the wound or at 3. Control gross bleeding
impairment of nervous amplitude full to mildly pressure points (proximal 4. Prepare for insertion of intravenous
system, poisons decreased artery) line and central linesif abdominal
Blood pressure: normal 3. Make comfortable and injuries present
to slightly decreased reassure 5. Peripheral line should be placed in
Neck veins: normal to 4. Cover lightly to prevent heat upper extremity if fluids being lost
slightly flat in supine loss, but do not bundle up in abdomen
position; may be full in 5. If neck or spine injury is
septic shock or grossly suspecteddo not move,
distended in cardio- unless victim in danger of
genic shock more injury
Skin: cool, clammy,
pale
Respirations: rapid,
shallow
GI: nausea, vomiting,
thirst
Renal: urine output
2040 mL/hr
If client unconscious or has 1. Draw blood for specimens: Hgb,
wounds of the lower face Hct, CBC, glucose, CO2, sodium
and jawplace on amylase, BUN, K+; type and cross-
side to promote drainage match, blood gases, CPK-MB,
of fluids; position client on troponin, prothrombin times
back unless otherwise indicated 2. Prepare infusion of
crystalloid replacement
NS usual choice; may
include Ringers lactate or
half-normal saline
Severe or late shock 1. Raise feet 68 inches 1. Assess and intervene as for early
Sensorium: confused, unless client has head or shock; then obtain information as
ADULT

disoriented, apathetic, chest injuries; if victim to onset and past history. Treat
unresponsive; slow, becomes less comfortable, underlying cause STAT.
slurred speech, often lower feet 2. Catheterize and monitor client
incoherent 2. If client complains of thirst, do urine output as ordered
Pupils: dilating, dilated, not give fluids unless client is 3. Take 12-lead ECG
slow or nonreactive to more than 6 hr away from pro- 4. Insert nasogastric tube and assess
light fessional medical help; under aspirate for volume, color, and
Pulse: rate no conditions give water to blood; save specimen if poison or
>150 beats/min, clients: who are unconscious, drug overdose suspected
thready, weak having seizures or vomiting, 5. If CVP low (<12)infuse
Blood pressure: appearing to need general 200300 mL over 510 min. If
80 mm Hg or anesthetic, or with a stomach, CVP rises sharplyfluid restriction
unobtainable chest, or skull injury necessary; if remains low
Neck veins: flat in a 3. Be calm and confident; reas- hypovolemia present
supine positionno sure client help is on the way 6. If client febrileblood cultures and
filling; full to wound cultures will be ordered
distended in septic 7. If urine output scanty or
or cardiogenic absentgive mannitol as
shock ordered
2164_Ch06_347-578 29/03/12 12:31 PM Page 545

Medical-Surgical Emergencies 545

Table 6.44
Nursing Care of the Adult in Medical and Surgical Emergenciescontd
Assessment: Signs
Condition and Symptoms Prehospitalization Nursing Care In-hospital Nursing Care
Skin: cold, clammy, mot- 8. If systolic BP <90 mm Hg
tled; circumoral cyanosis, give vasopressors
dusky, cyanotic (Levophed, vasopressin)
Eyes: sunkenvacant
expression
Renal: urine output
<20 mL/hr
Respiratory Emergencies
Chokingobstruction Gasping, wheezing; Do not interfere if coughing; do
of airway secondary to looks panicky, but can not slap on back; watch closely;
aspiration of a foreign still breathe, talk, call for assistance
object cough Victim standing/sitting and
Cough: weak, ineffec- conscious:
tive; breathing sounds Perform obstructive airway
like high-pitched maneuver (formerly called
crowing; Heimlich maneuver): stand behind
Colorwhite, gray, victim, wrap arms around waist,
blue place fist against abdomen, and
Difficulty speaking; with your other hand, press it into
clutches throat the victims abdomen with a quick
upward thrust until the obstruc-
tion is relieved or the victim
becomes unconscious
Victim lying down: Roll the
victim onto his or her back;
straddle the victims thighs; place
heel of hand in the middle of
abdomen; place other hand on
top of the first; stiffen arms and
deliver 610 abdominal thrusts
Unconscious victim: As in prehospital care; when
Lay rescuersproceed to CPR. probing mouth for foreign
No abdominal thrusts or blind object, turn head to side,
finger sweeps unless client has neck injury; in
EMS respondersTry to venti- event of neck injury, raise the arm
late; if unsuccessful, deliver opposite you and roll the head and
abdominal thrusts using tech- shoulders as a unit, so that head

ADULT
nique described for obstructive ends up supported on the arm
airway; probe mouth for foreign
objects; keep repeating above
procedure until ventilation
occurs; as victim becomes
more deprived of air, muscles
will relax and maneuvers that
were previously unsuccessful
will begin to work; when suc-
cessful in removing obstruction,
give two breaths; check pulse;
start CPR if indicated
On obese or pregnant
victimsuse chest thrusts
instead of abdominal thrusts
You are victim and alone:
Place your two fists for abdomi-
nal thrusts; bend over back of
chair, sink, etc. and exert hard,
repeated pressure on abdomen
to force object up; push fingers
down your throat to encourage
regurgitation
Continued
2164_Ch06_347-578 29/03/12 12:31 PM Page 546

546 chapter 6 Physiological Integrity

Table 6.44
Nursing Care of the Adult in Medical and Surgical Emergenciescontd
Assessment: Signs
Condition and Symptoms Prehospitalization Nursing Care In-hospital Nursing Care
Acute respiratory Hypoxia If you suspect respiratory Check clients ability to speak;
failuresudden onset Sensorium: acute distress, call physician; maintain airway by placing in
of an abnormally low apprehension calm and reassure client; high Fowlers position; check
PaO2 (<60 mm Hg) or Respiration: dyspnea; place in a chair or semisitting vital signs: BP, pulse rate and
high PCO2 (>60 mm Hg) shallow, rapid position; keep warm but not rhythm, temperature, skin color,
secondary to: lung respirations hot; phone for ambulance; if rate and depth of respirations;
disease or trauma, Skin: circumoral respirations cease or client place on O2, pulse oximetry,
peripheral or central cyanosis; pale, dusky becomes unconscious: clear STAT ABG
nervous system skin and nailbeds airway and commence respira- Prepare for intubation if:
depression, cardiac C/V: slight hyperten- tory resuscitation; check pulse: 1. Client has flail chest
failure, severe obesity, sion and tachycardia, initiate CPR if necessary; 2. Client is comatose without gag
airway obstruction, or hypotension and continue resuscitation until reflex
environmental bradycardia help arrives 3. Client has respiratory arrest; open
abnormality Hypercapnia airway, ambu bag with 100% O2
Sensorium: decreasing via face mask until intubation
mentation; headache 4. PaCO2 >55 mm Hg
Skin: flushed, warm, 5. PaO2 <60 mm Hg
moist 6. FiO2 >50% using nasal cannula,
C/V: hypertension; catheter, or mask
tachycardia 7. Respiratory rate >36
After intubation:
1. Check bilateral lung sounds
2. Observe for symmetrical lung
expansion
3. Maintain humidified oxygen at
lowest FiO2 possible to achieve
PaO2 of 60 mm Hg
4. Monitor exhaled CO2 with
caprometer.
Improve ventilation (decrease
PCO2) by:
1. Frequent suctioningoral and
above cuff of ET tube
2. IPPB indicated if tidal volume
decreases
3. Administer drugs as ordered:
sympathomimetics, xanthines,
antibiotics, and steroids
ADULT

4. Monitor: arterial blood gases,


electrolytes, Hct, Hgb, and WBCs
5. Bronchoscopy may be indicated
for thick, tenacious secretions
Do not:
1. Administer sedatives
2. Correct acid-base problems
without monitoring electrolytes
3. Overcorrect PaCO2
4. Leave client alone while oxygen
therapy is initiated. Once client is
stable, transfer to ICU
Near-drowning Conscious victim: Conscious victim: Nonsymptomatic near-drowning
asphyxiation or partial Acute anxiety, panic; 1. Try to talk victim out of panic victim:
asphyxiation due increased rate of so can find footing and way to 1. Draw blood for arterial blood gases
to immersion or sub- respirations shore with client breathing room air
mersion in a fluid or Pale, dusky skin 2. Utilize devices such as poles, 2. PA and lateral chest x-ray
liquid medium rings, clothing to extend to 3. Auscultate lungs
victim; do not let victim who 4. Admit to hospital for further evalu-
is panicked grab you; do not ation if:
attempt swimming rescue a. PaO2 <80 mm Hg
unless specially trained b. pH <7.35
2164_Ch06_347-578 29/03/12 12:31 PM Page 547

Medical-Surgical Emergencies 547

Table 6.44
Nursing Care of the Adult in Medical and Surgical Emergenciescontd
Assessment: Signs
Condition and Symptoms Prehospitalization Nursing Care In-hospital Nursing Care
3. If you suspect head or neck c. Pulmonary infiltrates present,
injuryhandle carefully, or auscultation reveals
floating victim back to crackles
shore with body and d. Victim inhaled fluids containing:
head as straight as possible; choline, hydrocarbons,
do not turn head or bend sewage, or hypotonic or
back freshwater
Unconscious victim: Unconscious victim:
Shallow or no 1. If victim not breathing: as
respirations soon as you have firm sup-
Weak or no pulse port, begin resuscitation
2. Tilt head back, bring jaw
forward, pinch nostril shut,
give two quick breaths
On shore: Symptomatic near-drowning victim:
1. Check breathing 1. Provide basic or advanced cardiac
2. Lay victim flat on back; life support
cover and keep warm 2. Provide clear airway and adequate
3. Calm and reassure victim ventilation by:
4. Do not give food or water a. Suctioning airway
5. Get to medical assistance as b. Inserting artificial airway and
soon as possible attaching it to ventilator as
6. If unconscious and not indicated
breathing: begin sequence for c. Inserting nasogastric tube to
CPR; compress water from suction to minimize aspiration
abdomen only if interfering of vomitus
with ventilation attempts 3. Monitor ECG continuously
7. If airway obstructed: reposi- 4. Start IV infusion D5W at
tion head; attempt to keep-open rate for
ventilate; if EMS responder: freshwater near-drowning;
perform 610 abdominal D5NS in saltwater near-drowning
thrusts; sweep mouth deeply; 5. Assist with insertion of CVP and
attempt to ventilate; repeat pulmonary artery (PA) catheter to
until successful guide subsequent infusion rates
8. Once ventilation established, 6. Administer drugs as ordered:
check pulse; if absent, begin anticonvulsants; steroids,
chest compressions as in antibiotics, stimulants,
CPR, one-person or two- antiarrhythmics

ADULT
person rescue 7. Provide rewarming if hypothermia
9. Continue CPR until victim present
revives or help arrives 8. Insert Foley to assess kidney
10. If victim revives, cover and function because freshwater
keep warm; reassure victim near-drowning causes renal tubular
help is on the way necrosis due to RBC hemolysis
11. Rescue personnel can further 9. Transfer to ICU when stabilized
assist emergency department
personnel by:
a. Documenting prehospital
resuscitation methods
used
b. Immobilizing victims sus-
pected of cervical spine
injuries
c. Using a sterile container to
take a sample of immer-
sion fluid
d. Taking on-scene arterial
blood gas sample for later
analysis
Continued
2164_Ch06_347-578 29/03/12 12:31 PM Page 548

548 chapter 6 Physiological Integrity

Table 6.44
Nursing Care of the Adult in Medical and Surgical Emergenciescontd
Assessment: Signs
Condition and Symptoms Prehospitalization Nursing Care In-hospital Nursing Care
Systemic Injuries
Multiple traumas Sensorium: alert; 1. Do not move client unless 1. Assess vital functions; ECG
disoriented, stuporous, you must, to prevent further monitoring, continuous pulse
comatose injury; send for help oximetry
Respirations: 2. Check breathinggive 2. Establish airway; ventilate with
increased rate, depth; mouth-to-mouth resuscitation ambu bag, ventilator
shallow; asymmetrical; if indicated 3. Draw arterial blood gases
paradoxical breathing; 3. Check for bleeding 4. Control bleeding
mediastinal shift; 4. Control bleeding by applying 5. Prepare infusions of blood,
gasping, blowing pressure on wound or on crystalloids
C/V: signs of shock pressure points (artery proxi- 6. Assess for other injuries:
(see pp. 427429, mal to wound) head injuriessuspect cervical
544545) 5. Use tourniquet only if above neck injury with all head injuries
Abdomen: contusions; pressure techniques fail to 7. Place sandbags to
pain; abrasions; open stop severe bleeding immobilize head and neck
wounds; rigidity; 6. Check for shock (pulse, 8. Do mini-neurological exami-
increasing distention pupils, skin color) and other nation: level of consciousness,
Skeletal system: pain; injuries pupils, bilateral movement, and
swelling; deformity; 7. Fractures: keep open-fracture sensation
inappropriate or no area clean 9. Get historytime of injury; any
movement 8. Do not try to set bone loss of consciousness; any drug
Neurological: pupils 9. If client must be moved ingestion
round, equal, react to splint broken bones with 10. Stop bleeding on or about head
light; ipsilateral dilation splints that extend past the 11. Apply ice to contusions and
and unresponsive; limb joints; tie splints on hematomas
fixed and dilated snugly but not so tight as to 12. Check for bleeding from nose,
bilaterally cut off circulation pharynx, ears
Bilateral movement 10. Check peripheral pulses 13. Check for cerebrospinal fluid from
and sensation in all 11. If head or back injury ears or nose
extremities suspectedkeep body 14. Assist with spinal tap if ordered
Progressive contralat- straight; move only 15. Keep accurate I&O
eral weakness with help 16. Protect from injury if restless,
Loss of voluntary 12. Reassure client that help is seizures; orient to time, place,
motor function on the way person
See Sensorium 17. Administer steroids,
(above) for level of diuretics, as ordered
consciousness 18. Check for signs of increas-
ing intracranial pressure: slowing
ADULT

pulse and respiration, widened


pulse pressure, decreasing
mentation
Spinal injuries 1. Assess and support vital func-
tions as above
2. Immobilizeno flexion or
extension allowed
3. If in respiratory distress
nasotracheal intubation or
tracheostomy to avoid hyperex-
tending neck
4. Check for level of injury and
function, asking client to:
a. Lift elbow to shoulder
height (C5)
b. Bend elbow (C6)
c. Straighten elbow (C7)
d. Grip your hand (C8-T1)
e. Lift leg (L3)
f. Straighten knee (L4, L5)
g. Wiggle toes (L5)
h. Push toes down (S1)
2164_Ch06_347-578 29/03/12 12:31 PM Page 549

Medical-Surgical Emergencies 549

Table 6.44
Nursing Care of the Adult in Medical and Surgical Emergenciescontd
Assessment: Signs
Condition and Symptoms Prehospitalization Nursing Care In-hospital Nursing Care
5. If client is comatose:
a. Rub sternum with knuckles
b. If all extremities move, severe
injury unlikely
c. If one side moves and other
does not, potential hemiplegia
d. If arms move and legs do not,
lower spinal cord injury
6. Administer steroids as
ordered
7. Assist with application of
skull tongsVinke or Crutchfield
8. Maintain IV infusions
9. Insert Foley as indicated
10. Assist with dressing of open
wounds
Chest injuries 1. Note color and pattern of respira-
tions, position of trachea
2. Auscultate lungs and palpate
chest for: crepitus, pain, tender-
ness, and position of trachea
3. Place gauze soaked in petroleum
jelly, if available, over open
pneumothorax (sucking chest
wound) to seal hole and decrease
respiratory distress
4. Assist with tracheostomy if
indicated
5. Prepare for insertion of chest
tubes if pneumothorax or
hemothorax present
Abdominal injuries 1. Observe for rigidity
2. Check for hematuria
3. Auscultate for bowel sounds
4. Assist with paracentesis to
confirm bleeding in abdominal
cavity

ADULT
5. Prepare for exploratory
laparotomy
6. Insert nasogastric tubeto
detect presence of upper GI
bleeding
7. Monitor vital signs
If organs protruding:
1. Flex clients knees
2. Cover intestines with sterile
towel soaked in saline
3. Do not attempt to replace organs
Fractures 1. Administer tetanus toxoid
as ordered
2. Observe for pain, peripheral
pulses, pallor, loss of sensation
and/or movement
3. Assist with wound cleansing,
casting, x-rays, reduction
4. Prepare for surgery if indicated
5. Monitor vital signs
Continued
2164_Ch06_347-578 29/03/12 12:31 PM Page 550

550 chapter 6 Physiological Integrity

Table 6.44
Nursing Care of the Adult in Medical and Surgical Emergenciescontd
Assessment: Signs
Condition and Symptoms Prehospitalization Nursing Care In-hospital Nursing Care
Burnstissue trauma Superficial (first Relieve pain by applying cold, 1. Cleanse thoroughly with mild
secondary to scalding degree): wet towel or cold water (not iced) detergent and water
fluid or flame, chemi- Erythema and 2. Apply gauze or sterile towel
cals, or electricity tenderness 3. Administer sedatives and
Usually sunburn narcotics as ordered
4. Arrange for follow-up care,
or prepare for admission if burn
ambulatory care impractical
Partial thickness 1. Douse with cold water until 1. Check tetanus immunization
(second degree): pain relieved status
Swelling, blisters; 2. Blot skin dry and cover with 2. Administer sedatives or
moisture due to clean towel narcotics as ordered
escaping plasma 3. Do not: break blisters, 3. Assess respiratory and
remove pieces of skin, or hemodynamic status; oxygen or
apply antiseptic ointments ventilatory assist as indicated,
4. If arm or leg burned, intravenous infusions as ordered
keep elevated to combat shock
5. Seek medical attention if 4. Remove all clothing from burn
second-degree burns: area
a. Cover 10% of body 5. Using aseptic technique, cleanse
surface burns as indicated
b. Involve hands, feet, or face
Full thickness (third 1. Do not remove charred 1. Do not break blebs or attempt
degree): clothing dbridement
White, charred areas 2. Cover burned area with clean 2. Assist with application of dressings
towel, sheet as ordered
3. Elevate burned 3. Maintain frequent checks of vital
extremities signs, urine output
4. Apply cold pack to 4. Provide psychological support
hand, face, or feet explain procedures, orient, etc.
5. Sit client up with face or 5. Assist with application of splints as
chest wound to assist ordered
respirations 6. Administer tetanus immune
6. Maintain airway globulin or toxoid as ordered
7. Observe for shock 7. Assist with transfer to
8. Do not: hospital unit
a. Put ice water on burns or
ADULT

immerse wounds in ice


watermay increase shock
b. Apply ointments
9. Calm and reassure victim
10. Get medical help promptly
11. If client conscious, not vomit-
ing, and medical assistance
is more than 6 hr away: may
give sips of weak solution of
salt, soda, and water
Fourth degree: Black Same as full thickness.
Chemical burns 1. Flush with copious amounts of 1. Flush with copious amounts of
water water
2. Get rid of clothing over burned 2. Administer sedation or
area narcotics as ordered
Burns of the eye: 1. Flush eye with water for at 1. Irrigate with water: never use
Acid least 15 min neutralizing solution
2. Pour water from inside to 2. Instill 0.5% tetracaine as
outside of eye to avoid con- ordered
taminating unaffected eye 3. Apply patch
3. Coverseek medical attention
at once
2164_Ch06_347-578 29/03/12 12:31 PM Page 551

Medical-Surgical Emergencies 551

Table 6.44
Nursing Care of the Adult in Medical and Surgical Emergenciescontd
Assessment: Signs
Condition and Symptoms Prehospitalization Nursing Care In-hospital Nursing Care
Alkali (laundry detergent 1. Do not allow client to rub eye As above for acid
or cleaning solvent) 2. Flush eye with water for at
least 30 min
3. Coverseek medical attention
at once
Abdominal Emergencies
Aortic aneurysm Primarily men >age 60 1. Notify physician 1. Assess respiratory and
rupture or dissection Sudden onset of excru- 2. Lay client flat, or raise hemodynamic status
ciating pain: abdominal, head if in respiratory 2. Institute shock measures
lumbosacral, groin, or distress (see pp. 544545) if indicated
rectal 3. Cover to keep warm but 3. Evaluate and compare peripheral
Orthopnea, dyspnea not hot pulses
Fainting, hypotension; 4. Institute shock measures (see 4. Assist with x-rays
if dissecting, marked pp. 544545) 5. Assist with emergency preopera-
hypertension may be 5. Calm; reassure that help is on tive treatment
present the way.
Palpable, tender,
pulsating mass in
umbilical area
Femoral pulse present;
dorsalis pedisweak
or absent
Blunt injuriesspleen Left upper quadrant 1. Lay client flat 1. Assess respiratory and hemody-
pain, tenderness and 2. Institute shock measures namic status
moderate rigidity; left (see pp. 544545). a. Maintain airway and ventilation
shoulder pain (Kehrs as indicated
sign) b. Institute infusions of
Hypotension; weak, colloid or crystalloids as
thready pulse; ordered
increased respirations c. Insert both CVP and arterial
(shock) monitoring lines
d. Insert Foley catheter
2. Prepare for splenectomy
Eye and Ear Emergencies
Chemical burns See Burns, p. 550 See Burns See Burns

ADULT
Blunt injuries secondary Decreased visual acuity, 1. Prevent victim from rubbing 1. Test visual acuity of each eye using
to flying missiles (e.g., diplopia, blood in eye Snellen or Jaeger chart
balls, striking face anterior chamber 2. Cover with patch to protect 2. Assist with fluorescein
against car dashboard) Pain, conjunctiva red- eye administrationto facilitate
dened, edema of eyelids 3. Seek medical help immediately identifying breaks in cornea
Sharp ocular trauma Reports feeling as if 1. Keep victim from rubbing eye 1. Check visual acuity in both eyes
secondary to small something were hitting 2. Cover very lightlydo not 2. Check pupils
or larger foreign bodies eye apply pressure 3. Instill 1% tetracaine HCl
Pain, tearing, reddened as ordered to relieve pain
conjunctiva 4. Administer antibiotic drops
Blurring of vision or ointment as ordered
Foreign object may be 5. Apply eye patch
visible 6. Provide instructions for subsequent
care and follow-up
Foreign bodies in Decreased hearing; 1. Do not attempt to remove 1. Inspect ear canal
earsbeans, peas, pulling, poking at ear object 2. Assist with sedating (especially
candy, foxtails, insects and ear canal; buzzing, 2. Seek medical assistance children)restraint may be
discomfort necessary
Continued
2164_Ch06_347-578 29/03/12 12:31 PM Page 552

552 chapter 6 Physiological Integrity

Table 6.44
Nursing Care of the Adult in Medical and Surgical Emergenciescontd
Assessment: Signs
Condition and Symptoms Prehospitalization Nursing Care In-hospital Nursing Care
3. Assist with procedures to remove
object:
a. Forceps or curved probe for fox-
tails, irregularly shaped objects
b. 10F or 12F catheter with tip cut
squarely off and attached to
suction to remove round object
4. Irrigate external auditory canal to
flush out insects, materials that do
not absorb water; do not irrigate if
danger of perforation

Questions 6. Following an auto accident, a clients vital signs and hema-


tocrit will be monitored for 24 hours for signs of internal
Select the one answer that is best for each question, unless bleeding. During this time, the client should receive:
otherwise directed. 1. High-carbohydrate clear liquids orally.
1. Which action by the nurse is correct for droplet precautions? 2. High-protein liquids via nasogastric tube.
1. Tests N95 respirator for fit prior to use in client room. 3. Intravenous total parenteral nutrition.
2. Wears a surgical mask when within 3 feet of client. 4. Nothing by mouth.
3. Wears eye protection upon entering the clients room. 7. Postoperatively, a clients fingers are cold and pale. Which
4. Uses sterile gloves when bathing the client. action should a nurse take?
2. What health teaching should be included for a client being 1. Apply warm blankets.
discharged home from an emergency department with an 2. Check oxygen saturation on finger.
infection? 3. Encourage deep breathing.
1. Take an extra antibiotic tablet as needed if the tempera- 4. Check blood sugar.
ture is over 38.5C. 8. A client with cancer has anorexia and loss of weight.
2. If chills occur after 24 hours, check temperature and call Which intervention should a nurse perform first?
provider if over 38.5C. 1. Giving TPN through a central line.
3. Do not take acetaminophen for 24 hours after starting 2. Starting liquid nutrition through a gastric port.
antibiotic. 3. Starting liquid nutrition through a duodenal port.
4. Use cooling measures for temperature over 37C. 4. Giving megestrol and a diet of choice with nutritional
QUESTIONS

3. Following a head injury, a client has no cough or gag reflex. supplements.


The correct nursing action for feeding this client is to: 9. A client, who is nondiabetic and receiving 5% dextrose in
1. Position the head of bed 90 degrees for meals. one-half normal saline (D5 1/2 NS) running at 125 mL/hr,
2. Give only solid foods. has a blood sugar of 130 mg/dL on the morning
3. Give only thick liquids. chemistry panel. The client is concerned. It is most
4. Use a feeding tube. important for the nurse to:
4. Which technique is correct for reducing swelling after a 1. Provide diabetic teaching to the client who is newly
traumatic injury to the ankle? diagnosed.
1. Apply intermittent ice during the first 24 hours. 2. Check another blood sugar and ask the physician
2. Apply continuous heat during the first 48 hours. about insulin.
3. Perform range of motion every 4 hours on the ankle. 3. Explain the consequences of stress and IV fluids on
4. Position the foot below the level of the heart. blood sugar level.
4. Keep the client NPO.
5. A nurse tells a client that the most effective treatment for a
rash from lupus erythematosus is: 10. A nurse is planning care for a client admitted with chest
1. Getting sun exposure 15 minutes each day. pain after myocardial ischemia. Which outcome should
2. Washing with soap and water. the nurse document for this problem?
3. Taking an antimalarial drug. 1. Client states that pain is decreased to a tolerable level.
4. Getting a varicella vaccine booster. 2. Client agrees to rest and take pain medication.
2164_Ch06_347-578 29/03/12 12:31 PM Page 553

Questions 553
3. Client rates pain 0 on scale of 1 (least) to 10 (worst). 17. A nurse is caring for a client with a left hemisphere
4. Pain medication is administered within 5 minutes. stroke. The appropriate nursing actions for this client are:
11. A woman who has received radiation therapy says that Select all that apply.
she feels like she is voiding through her vagina. This 1. Place food and the television on the left side of the
client may be experiencing: bedside table.
1. Extreme stress due to a diagnosis of cancer. 2. Assist the client out of bed on the right side.
2. Altered perineal sensations as a side effect of radiation 3. Raise the left side rail and place the television on the
therapy. right side.
3. The development of a vesicovaginal fistula. 4. Talk into the clients right ear and place food on the
4. Rupture of the bladder. right side.
5. Teach the client to pivot on the left leg in and out of bed.
12. The correct nursing action for a client who has a
nephrostomy tube would include: 18. A nurse notes that a client, who experienced a head
1. Attaching tube to suction prn with low urine injury 24 hours ago has returned to the emergency
output. department with slurred speech and is disoriented to
2. Changing the bandage and drainage bag daily. time and place. The first nursing action should be to:
3. Irrigating, if ordered, with no more than 10 mL 1. Continue to assess hourly as ordered.
sterile NS. 2. Report the change to the physician.
4. Clamping and unclamping the tube at hourly 3. Repeat a neurologic assessment in 15 minutes.
intervals. 4. Notify the operating room of the need for surgery.

13. Following perineal surgery, a client is at risk for a wound 19. Which assessment finding should indicate to a nurse that
infection related to incontinence. The correct manage- a client has progression of intermittent claudication?
ment of this problem is to: 1. The distance a client can walk before leg pain starts.
1. Insert a continuous indwelling catheter per order. 2. Presence of pedal edema in the legs after sitting
2. Assist to the toilet and protect the skin with cream. 20 minutes.
3. Limit oral fluid intake. 3. Changes in strength of peripheral pulses in the
4. Give a loop diuretic, such as furosemide, as ordered. affected leg.
4. Changes in skin temperature and color of the feet.
14. A client comes to an emergency department with com-
plaints of low abdominal pain and hematuria. The client 20. Which assessment findings should alert a nurse to early
is afebrile. The nurse should first ask the client if: alcohol withdrawal in a client 2 days after surgery? Select
1. There is a family history of bladder cancer. all that apply.
2. There had been recent trauma to the bladder or lower 1. Auditory hallucinations.
abdomen. 2. Decreased blood pressure.
3. This could be recurrence of glomerulonephritis. 3. Depression.
4. The client had ever had pyelonephritis. 4. Diaphoresis.
5. Tachycardia

QUESTIONS
15. A client with diabetes has a blood sugar of 300 mg/dL 6. Dilated pupils
and an Na+ of 133. What nursing intervention is
indicated to manage the sodium with this client? 21. An older adult attending a community health fair asks
1. Pad the side rails to protect from injury during a about receiving the necessary vaccines for the swine flu.
possible seizure. The nurse tells the client that, to prevent the spread of
2. Notify dietary department to send salt tablets. swine flu, the client should receive:
3. Encourage the client to drink water. 1. Just the seasonal flu and the pneumonia vaccines.
4. Monitor Na+ return to normal with lowering of blood 2. The novel H1N1 vaccine instead of the seasonal vaccine.
sugar. 3. The seasonal and novel H1N1 vaccines the same day.
4. The novel H1N1 and seasonal vaccines a week apart.
16. The nursing priorities for the care of a client with acute
glomerulonephritis are: 22. A client, who had been playing golf in 110F outside
Select all that apply. temperature, is admitted to an emergency department
1. Performing range of motion. with hyperthermia. A nursing assessment would reveal:
2. Encouraging a diet high in protein. Select all that apply.
3. Maintaining strict I&O. 1. Absence of sweating.
4. Teaching intermittent self-catheterization for urine 2. Decrease in body temperature.
retention. 3. Increase in sweating.
5. Monitoring daily weight. 4. Increased blood pressure.
6. Recording BP at least q4h. 5. Tachycardia.
6. Flushed appearance.
2164_Ch06_347-578 29/03/12 12:31 PM Page 554

554 chapter 6 Physiological Integrity

23. A client comes to an emergency department reporting 2. A 61-year-old with osteoarthritis who lives in a
chest pain; vital signs are: BP 110/68 mm Hg, P 110 and nursing home.
irregular, R 24, and pulse oximetry 88%. Which inter- 3. A client who is 48 hours post-MI, with a WBC of
ventions should a nurse perform first? Prioritize the 9,000/mm3 and a CK-MB of 25 ng/mL.
nurses actions by placing each intervention in the correct 4. A 70-year-old man with anemia and a Hct of 39%
sequence. and O2 saturation of 90%.
Give morphine 10 mg as ordered. 30. What should a nurse monitor first when caring for a client
Start O2 6 L per mask. who is undergoing treatment for pheochromocytoma?
Give nitroglycerin (NTG) sublingual gr 1/150. 1. Pulse rate.
Run an ECG strip. 2. Blood sugar.
24. A nurse should recognize the signs of deep vein 3. Blood pressure.
thrombosis (DVT) if a client reports: 4. ECG changes
Select all that apply. 31. Which aspect of care is most important for a client with
1. Leg feeling cool with no pain. diabetic neuropathy?
2. Numbness of legs with diaphoresis. 1. Teach the client to inspect the feet using a mirror.
3. Sudden swelling of one leg with dependent edema. 2. Teach the client to wash feet, then pat dry.
4. Dizziness and blurred vision. 3. Have client moisturize feet with lotion to prevent
5. Pain behind the knee with dorsiflexion of foot. dryness.
25. What nursing action is appropriate if a client has a K+ of 4. Teach the client to cut toenails straight across.
8 mEq/L? 32. A client with diabetic ketoacidosis has been treated with
1. No change is required in treatment. an insulin drip for the past 3 hours. For which imbalance
2. Restrict intake of K+ and/or give sodium polystyrene is this client at greatest risk?
sulfonate (Kayexalate). 1. Hypovolemia.
3. Restrict fluids to reduce K+. 2. Hypokalemia.
4. Give insulin, glucose, calcium, and/or bicarbonate 3. Hyponatremia.
STAT, as ordered. 4. Hypoglycemia.
26. Which client has the greatest need for K+ replacement? 33. Which symptom would a nurse most likely observe in a
1. A client in renal failure with a postdialysis serum K+ client with cholecystitis?
of 3.4. 1. Black stools.
2. A client with a large NG output who is receiving 2. Nausea after a high-fat meal.
Kayexalate with a serum K+ of 5.5. 3. Temperature of 104F.
3. A client with cardiac disease who is about to receive 4. Colicky left upper quadrant pain.
furosemide with a K+ of 3.5.
4. A client with cardiac disease who is about to receive 34. If a clients bowel sounds are absent, a nurse should listen
spironolactone with a K+ of 3.5. for at least:
QUESTIONS

1. 30 seconds.
27. A nurse knows that the choice of a topical antimicrobial 2. 60 seconds.
for a client with burns is most influenced by: 3. 2 minutes.
1. The bactericidal and fungicidal effectiveness of the agent. 4. 3 minutes.
2. The form of the agent, whether it is a liquid or cream.
3. The presence and extent of eschar formation. 35. What should be the first nursing action if an NG tube is
4. The ability of the agent to deliver uniform absorption. not draining?
1. Irrigate the tube.
28. Which condition should a community health nurse 2. Reposition the client.
report to the health department? 3. Determine tube placement.
1. Confirmation of acid-fast bacilli. 4. Remove the tube and reinsert.
2. Pruritic eruptions from Sarcoptes scabiei (scabies).
3. Borrelia burgdorferi (Lyme disease). 36. A nurse is observing a client for possible complications of
4. Microsporum species (ringworm) in a child who is in postoperative peritonitis. Which manifestations are most
preschool. indicative of peritonitis? Select all that apply.
1. Hyperactive bowel sounds.
29. Which client can be safely discharged to make room for 2. Localized or diffuse pain.
clients suffering from a salmonella outbreak from a local 3. Abdominal rigidity.
food chain? 4. Shallow respirations.
1. An 18-year-old diagnosed 1 day ago with type 1 5. Temperature over 102F.
diabetes, who lives in the college dormitory.
2164_Ch06_347-578 29/03/12 12:31 PM Page 555

Questions 555
37. A nurse would know that a client was experiencing 44. Leakage of spinal fluid is a potential neurosurgical compli-
chronic renal failure (CRF) if which signs or symptoms cation. How should a nurse assess for this complication?
were present? 1. Check for urine specific gravity greater than 1.030.
1. Nausea and vomiting. 2. Monitor for urine output greater than 510 L/day.
2. Asterixis and ascites. 3. Test all nasal and ear drainage for glucose.
3. Kussmauls breathing and drowsiness. 4. Test all spinal fluid from lumbar punctures for glucose.
4. Pruritus and anemia.
45. A client is admitted to an ICU with a possible brain
38. A client has returned to the unit from surgery after hav- attack (stroke). Assessment findings consistent with a
ing an arteriovenous (A-V) fistula created in the left arm. brain attack include:
The clients teaching should include: Select all that apply.
Select all that apply. 1. Pronator drift.
1. Protecting site by wearing an elastic sleeve. 2. Facial droop.
2. Squeezing a ball to increase vessel size. 3. Slurred speech.
3. Expecting fistula to be used for the next dialysis 4. Weakness of affected extremity.
treatment. 5. Crackles in lungs.
4. Avoiding BP or drawing of blood samples from 6. Decreased urine output.
left arm.
46. Which is the most common treatment for a client in
5. Ensuring clothing fits loosely over left arm.
addisonian crisis?
39. After a cystectomy and construction of an ileal conduit, 1. IV normal saline and glucocorticoids.
which complication should a nurse instruct a client to 2. IV lactated Ringers and packed cells.
take special precautions to prevent? 3. IV 5% dextrose in normal saline and dopamine.
1. Paralytic ileus. 4. IV total parenteral nutrition (TPN) and insulin
2. Urinary calculi. coverage.
3. Pyelonephritis.
47. When a client is on chemotherapy, for which manifes-
4. Mucus in the urine.
tations of bone marrow depression should a nurse
40. A nurse should teach a client with a Kock pouch urinary continuously assess?
diversion to prevent urinary tract infection by: 1. Night sweats and fatigue.
1. Avoiding people with upper respiratory infections 2. Loss of skin turgor and weight loss.
(URI). 3. Low urine output and elevated BUN levels.
2. Maintaining a daily fluid intake of 2 liters. 4. Ecchymosis, weakness, and fatigue.
3. Using sterile technique to change the appliance.
48. A client is admitted to a hospital in the terminal stage of
4. Irrigating the stoma daily.
illness. At this time, a nurse, who is planning end-of-life
41. A client who is scheduled for surgery to correct a retinal care, should recognize that the client is most likely to fear:
detachment should be positioned: 1. The terminal diagnosis.

QUESTIONS
1. On the side opposite of the retinal detachment. 2. Further chemotherapy.
2. Upright with face down. 3. Being socially inadequate.
3. On the side of the retinal detachment. 4. Dying alone and in pain.
4. Prone with face supported by pillow ring.
49. A complication of Bucks extension traction would be
42. A client has been admitted with mild symptoms of novel noted by a nurse if:
H1N1. Which family member is at greatest risk for 1. Redness and purulent drainage appeared at the
developing the infection? pin site.
1. The elderly grandmother. 2. Toes of affected leg became dusky in color.
2. A 6-year-old daughter. 3. Skin over the fracture site was flushed.
3. The healthy 45-year-old spouse. 4. Dorsiflexion developed in the affected foot.
4. The 13-year-old son with asthma.
50. Due to the extent of a clients fracture, a hip prosthesis is
43. Which respiratory change would a nurse see most often inserted. A nurse knows that the most serious complica-
in a client with increased intracranial pressure? tion of implant surgery is:
1. Nasal flaring and retractions. 1. Infection.
2. Slow, irregular respirations. 2. Phlebitis.
3. Rapid, deep respirations. 3. Urinary retention.
4. Paradoxical chest movements. 4. Narcotic addiction.
2164_Ch06_347-578 29/03/12 12:31 PM Page 556

556 chapter 6 Physiological Integrity

51. A client returns from an operating room. The clients 58. A client with COPD is scheduled for abdominal sur-
IV is running at 150 mL/hr; pulse is 90 and full; respi- gery. Arterial blood gases before surgery were: pH 7.36,
rations are moist and wheezy. A nurses initial action PCO2 54, PO2 70. After surgery the ABGs were pH
should be to: 7.35, PCO2 60, PO2 65 on 2 L of O2. Which action
1. Speed up the IV and call the physician. should a nurse take?
2. Check the electrolyte levels. 1. Suction the client.
3. Report the IV rate to the charge nurse. 2. Have the client cough and deep breathe.
4. Slow down the IV and call the physician. 3. Administer sodium bicarbonate, per order.
4. Position the client in high Fowlers.
52. During the first 24 hours after an above-the-knee
amputation for vascular disease, nursing priority for 59. Which symptom(s) would be expected if a pacemaker
stump care would be: suddenly malfunctioned?
1. Inspecting for redness and pressure points. 1. Pulse rate higher than set.
2. Elevating to reduce edema. 2. Shortness of breath, dizziness.
3. Cleansing with soap and water. 3. Atrial fibrillation, hypotension.
4. Initiating fitting for prosthesis. 4. Premature ventricular contractions (PVCs).
53. What are the emergency nursing actions for a client with a 60. What signs will a nurse observe in a client who is experi-
head injury due to a fall from a third-floor roof? encing right-sided heart failure? Select all that apply.
1. Assess respirations, assess circulation, and assess level of 1. Bilateral ankle edema.
consciousness. 2. Jugular venous distention (JVD).
2. Stabilize C-spine, determine responsiveness, and begin 3. Crackles on auscultation.
chest compressions. 4. Enlarged liver.
3. Stabilize C-spine, assess airway, and assess respirations. 5. Sacral edema.
4. Assess airway, assess respirations, and assess circulation.
61. When assessing a client for signs of early septic shock,
54. As a nurse is assisting a client out of bed, which objective a nurse should observe for:
sign should indicate to the RN that the client is experi- 1. Cool, clammy skin.
encing hypotension? 2. Warm, flushed skin.
1. Pupil response is sluggish to light. 3. Decreased systolic blood pressure.
2. Orientation level changes. 4. Disseminated intravascular coagulation (DIC).
3. Pulse rate increases 20 beats per minute.
62. An indicator of a worsening hypovolemic shock related
4. Increased muscle rigidity in legs.
to GI bleeding would be:
55. Which group is considered high risk for acquiring the 1. Decreased level of consciousness (LOC).
human immunodeficiency virus (HIV)? 2. Complaints of abdominal pain with hematemesis.
1. Women in menopause. 3. Deepening, rapid respirations.
2. Men who are 65 plus years of age. 4. Increasingly rapid, thready pulse.
QUESTIONS

3. People who engage in oral sex.


63. A 50-year-old client is being seen in the outpatient clinic
4. Children of women with multiple partners.
with complaints of a fever, cough, muscle soreness, and
56. Victims of a disastrous earthquake have been transported fatigue. The nurse would be concerned that the client
to an emergency department. Which victim should the had novel H1N1 flu if the complaints included:
nurse assess first? Select all that apply.
1. A victim with second-degree burns on 25% of the 1. Persistent cough.
body. 2. Shortness of breath.
2. A victim with facial lacerations. 3. Persistent vomiting.
3. A victim with a Glasgow Coma Scale score of 15 who 4. Temperature of 104F.
is hyperventilating. 5. Diarrhea.
4. A victim with no heartbeat.
64. A clients vital signs are: BP 80/60 mm Hg, P 120, R 30,
57. A client with a tracheotomy just pulled out the tracheal T 100.4F. A nurse should know that these findings most
tube. Which action by a RN is correct? likely indicate:
1. Call the respiratory therapist to reinsert the tracheal 1. Neurogenic shock from increased intracranial
tube. pressure.
2. Get a hemostat to open the tracheotomy, and then try 2. Hypovolemic shock from fluid volume deficits.
to reinsert the tube. 3. Septic shock from gram-negative sepsis.
3. Give 100% O2 by mask over the stomal opening. 4. Cardiogenic shock from MI.
4. Place mouth to stoma and ventilate every 5 seconds.
2164_Ch06_347-578 29/03/12 12:31 PM Page 557

Questions 557
65. A nurse should expect that a client with a severe loss of 71. A nurse should evaluate the effects of coumadin, used in
potassium (hypokalemia) from diarrhea will have: the treatment of deep vein thrombosis, by looking at the
1. Fatigue, tetany, cardiac standstill. results of which laboratory tests?
2. Kussmauls respirations, thirst, furrowed tongue. 1. Prothrombin time (PT).
3. Muscle weakness, cramps, cardiac irritability. 2. Lee-White clotting time.
4. Confusion, pitting edema, irregular pulse. 3. Partial thromboplastin time (PTT).
4. Fibrinogen clotting time (FCT).
66. Which finding should indicate to a nurse that a client has
recovered from respiratory acidosis? 72. Which client should be the first priority for a telephone
1. Increasing respiratory rate. advice RN? A client who reports:
2. Increasing serum creatinine. 1. Headache, vomiting, and fever.
3. Decreasing respiratory rate. 2. Ankle swelling and shortness of breath.
4. Increasing serum bicarbonate. 3. Productive cough.
4. Palpitations.
67. The goal of care for a client with liver failure is to lower
the blood ammonia level. Which actions would prevent 73. To promote maximum ventilation in a client who is
increased ammonia? Select all that apply. postoperative, a nurse should:
1. Prevent gastrointestinal bleeding. 1. Auscultate breath sounds bilaterally.
2. Reduce dietary protein intake. 2. Give humidified oxygen via cannula.
3. Avoid diarrhea and vomiting. 3. Maintain placement of airway until the client is awake.
4. Decrease bacterial flora in the intestines. 4. Position the client on the side, with the neck extended.
5. Prevent ascites.
74. A nurse gets a report that a client will be admitted with
68. If a bowel obstruction occurs from inflammatory bowel an ejection fraction of 58%. Which history and physical
disease at the transverse colon, a nurse will initially hear finding should the nurse anticipate?
bowel sounds that are: 1. Inspiratory wheezing in upper lobes, bilaterally.
1. Increased RLQ; decreased LLQ. 2. O2 saturation that increases from 90% to 99% when
2. Decreased RLQ; decreased LLQ. head of bed is lowered to a flat position.
3. Absent RLQ; absent LLQ. 3. Crackles in the lung bases that clear with deep
4. Decreased RLQ; increased LLQ. breathing.
4. A client is unable to walk up two flights of stairs
69. Identify the appropriate actions in the prevention of without dyspnea.
thrombophlebitis in a client on bedrest. Select all that
apply. 75. Management of a BUN of 71 and a creatinine of 2.7 in a
1. Elevate the knee gatch of the bed. client with diabetic ketoacidosis (DKA) is correctly
2. Encourage exercises that dorsiflex and plantar flex the accomplished with:
ankle. 1. Rehydration.
3. Apply sequential compression devices bilaterally. 2. Fluid restriction.

QUESTIONS
4. Prevent dehydration. 3. Dialysis.
5. Periodically elevate the feet and lower legs above level 4. Bladder catheterization.
of heart.
76. A nurse notes that a client has a total bilirubin of 1.0
6. Apply warm, moist packs bilaterally to lower legs.
mg/dL. The nurse should:
70. Which nursing actions should be completed before 1. Record this normal finding.
a physician performs a thoracentesis? Select all 2. Check urine for blood.
that apply. 3. Check the stool for guaiac.
1. Assessing the client for any allergy to local 4. Assess the sclerae for yellow coloration.
anesthetics.
77. Ten hours after beginning an insulin drip on a client with
2. Teaching the client to do pursed-lip breathing during
diabetic ketoacidosis, the following laboratory results are
the procedure.
returned: Na+ 130, K+ 4.4, Cl 100, bicarb 15, BUN 60,
3. Placing the client in an upright sitting position leaning
creatinine 2.5, glucose 100. Which action should a nurse
forward, if able.
take?
4. Placing the client in a prone position with the affected
1. Administer potassium.
lung on a pillow.
2. Continue the insulin.
5. Making sure that the consent form is signed and in the
3. Administer salt tablets.
chart.
4. Restrict fluids.
6. Telling the client not to expect discomfort.
2164_Ch06_347-578 29/03/12 12:31 PM Page 558

558 chapter 6 Physiological Integrity

78. A client is admitted with diabetic ketoacidosis (DKA). 3. Administer hormone replacement as prescribed.
Five months ago, the hemoglobin A1c (HgbA1c) was 4. IV administration of fluid, glucose, and
9.4%; currently it is 10.3%. Based on this information, a electrolytes.
nurse should: 86. A nurse knows that the serum sodium level of a client
1. Record this expected finding. with a head trauma should be above 140 to:
2. Recheck the HgbA1c after the DKA is resolved. 1. Keep cerebral perfusion pressure up.
3. Provide diabetic teaching. 2. Prevent cerebral vasodilation.
4. Ask the physician about longer-acting insulin. 3. Lower the seizure threshold.
79. Seizure precautions should be applied to clients who have: 4. Prevent cerebral edema.
1. A serum sodium of less than 125 mg/dL. 87. A client is admitted with diabetic ketoacidosis. A nurses
2. An HgbA1c over 8%. first priority should be to give:
3. An anion gap less than 12. 1. IV bicarbonate.
4. Chronic peripheral neuropathies. 2. IV normal saline.
80. Which techniques are effective in communicating with a 3. IV albumin.
client who has complete loss of vision? Select all that apply. 4. Insulin subcutaneously.
1. Raise the voice when talking. 88. A client with diabetic ketoacidosis (DKA) has been
2. Avoid using the words see and look. receiving IV insulin for 6 hours. Laboratory findings are:
3. Talk very softly, since hearing is overly sensitive. Na+ 131, K+ 3.7, Cl 102, HCO3 22, and glucose 170.
4. Face the client when talking. Which action should a nurse take?
5. Touch the clients hand before speaking. 1. Administer 3% sodium chloride at 200 mL/hr.
6. Avoid using phrases such as over there. 2. Expect the insulin drip to be discontinued.
81. A client with mild hypertension asks a nurse for sugges- 3. Give potassium IV.
tions to control blood pressure. What should a nurse 4. Administer D5 1/4 NS with the insulin drip.
recommend? 89. A client, who is malnourished, is admitted with a serum
1. Follow a regular exercise program. calcium of 7.5. A nurse should:
2. Attend a stress-reduction support group. 1. Administer glucocorticoids, as ordered.
3. Avoid use of tobacco and limit alcohol intake. 2. Check urine for ketones.
4. Increase intake of fruits and vegetables. 3. Check the serum albumin lab result.
82. A nurse is checking a clients third cranial nerve. How 4. Administer Neutra-Phos, as ordered.
should physical assessment be performed? 90. Following a thyroidectomy, a nurse assesses for complica-
1. Sweep a piece of cotton briskly across the cornea. tions related to damage or removal of the parathyroid.
2. Ask the client to follow the examiners finger with The nurse should assess for:
the eyes. 1. Hypertension.
3. Use a Snellen chart to check visual acuity. 2. Numbness around the mouth.
QUESTIONS

4. Check pupillary reaction using a penlight. 3. Polyuria.


83. A nurse should assess for hypercalcemia by checking a 4. Muscle weakness.
client for: 91. A 19-year-old victim of trauma is admitted to an
1. Chvosteks sign. emergency department with a blood pressure of
2. Trousseaus sign. 80/50 mm Hg, and a heart rate of 130 bpm. Which
3. Deep tendon reflex. action should a nurse take first?
4. Babinski reflex. 1. Give 1 liter of D5W IV as fast as possible.
84. The mother of a 2-year-old child is concerned her child 2. Give 1 liter NS IV as fast as possible.
may have novel H1N1. The nurse tells the mother that 3. Start an additional large-bore IV as saline lock.
she should immediately report: 4. Calculate the anion gap before selecting IV fluid.
1. Loss of appetite. 92. A client with CHF has: 2+ pedal edema, jugular venous
2. Fever for 2 days. distention (JVD), bilateral basilar crackles, urine output
3. Vomiting of 1.2 L/24 hr, blood pressure of 145/88 mm Hg, Na+
4. Lack of alertness. of 129. A nurse should plan to:
85. Which nursing intervention should a nurse complete first 1. Restrict fluids.
for a client experiencing addisonian crisis? 2. Give salt tablets PO.
1. Maintain a quiet, nonstressful environment. 3. Change IV to NS.
2. Take measures to avoid exertion by the client. 4. Give 3% sodium chloride IV.
2164_Ch06_347-578 29/03/12 12:31 PM Page 559

Questions 559
93. A client is scheduled for outpatient bariatric surgery. 3. Increase intravenous fluids.
Deep vein thrombosis (DVT) is best prevented by: 4. Administer pain medication.
1. Early ambulation. 100. One day after surgery for intestinal resection, a client
2. Postoperative low-dose heparin. has no bowel sounds. Which action should a nurse
3. Alternating compression leg wraps. take?
4. Range-of-motion exercises to wrist. 1. Take the vital signs and notify the physician.
94. Following general anesthesia for a hip replacement, an 2. Record this expected finding.
elderly clients vital signs are: P 80, R 14, blood pressure 3. Check rectally for impacted stool.
110/78 mm Hg; O2 Sat 100% on 40% mask; pain 2/10. 4. Perform abdominal massage.
The client is shivering and complains of being cold. The 101. A client with gallstones asks a nurse what might have
first nursing priority should be to: caused the condition. The nurse knows the risk of
1. Remove the oxygen. developing this condition is greater in:
2. Check the temperature. 1. African Americans.
3. Apply warm blankets. 2. Men.
4. Give pain medication. 3. Significant recent weight loss.
95. Which finding is consistent with a client developing fluid 4. Drinking a lot of coffee.
overload? 102. A client is in a rehabilitation unit 2 weeks after a right
1. Pulse over 100. middle cerebral artery infarct. Which is the best activity
2. Pulmonary crackles that clear with deep breathing. plan?
3. Concentrated urine. 1. Passive range of motion should be performed on the
4. Oxygen saturation less than 92% on 40% O2 via right arm and leg several times a day.
facemask. 2. The client should know how to get up from the
96. What is the best nursing action to prevent pressure sores right side of the bed.
during a long surgical procedure? 3. Maintain bed rest with all rails up until the client
1. Pad bony prominences before surgery begins. can lift both legs off the mattress.
2. Turn the client every 2 hours. 4. Immobilize joints on the left side of the body into a
3. Perform range of motion on the affected joints. position of flexion contracture.
4. Provide an alternating pressure mattress. 103. A client with a vertebrobasilar stroke is being man-
97. A client with diabetes and hypertension has returned aged for dysphagia. Which intervention will be most
from surgery. Which nursing intervention will most likely helpful?
reduce the risk of wound infection? 1. Eliminate distractions when giving directions.
1. Monitor blood sugar and keep it under 200 2. Keep the head of the bed flat after meals.
postoperatively. 3. Provide a thin liquid diet.
2. Place the client in contact isolation postoperatively. 4. Provide nutrition through a feeding tube.

QUESTIONS
3. Administer prophylactic antibiotics 48 hours 104. Which intervention will be most helpful to a client with
preoperatively, as ordered. aphasia?
4. Administer glucocorticoid stress hormone replace- 1. Encourage use of gestures in communication.
ments postoperatively, as ordered. 2. Use many adjectives and adverbs when describing
98. A client asks a nurse what to expect with a condition of desired activity.
uncomplicated gallstones. The best response would be: 3. Speak loudly when giving directions.
1. There may be blood in the stools, with increased 4. Chew food thoroughly before swallowing.
mucus. 105. The nursing assistant asks the nurse if a N95 respirator
2. There may be RLQ cramping, with pain relieved should be worn when caring for the client with H1N1.
after eating a fatty meal. The correct response by the nurse is:
3. You may feel fatigue because of low hemoglobin 1. Droplet precautions require a direct caregiver to wear
level. a respirator.
4. Most clients do not notice any effects. 2. There is an N95 in the clients room for use by the
99. A nurse is caring for a 32-year-old client who is 1 day nursing staff.
postgastric bypass. Vital signs are: T 37.9C, P 100, 3. The N95 is usually reserved if treatment produces an
R 18, BP 130/80 mm Hg, pain 2/10, O2 Sat 92% on aerosol spray of sputum.
room air. The nurse should first: 4. The respirator should be worn if the nurse has cold
1. Notify the physician for antibiotic orders. symptoms.
2. Have the client use the incentive spirometer.
2164_Ch06_347-578 29/03/12 12:31 PM Page 560

560 chapter 6 Physiological Integrity

Answers/Rationales/Tips Content Area: Adult Health, Neurological; Integrated Process:


Nursing Process, Implementation; Cognitive Level: Application;
1. CORRECT ANSWER: 2. Answer 1 is incorrect because Client Need/Subneed: Physiological Integrity/Reduction of
the N95 respirator is not required whenever the nurse enters Risk Potential/Potential for Complications of Diagnostic
the clients room. It is important if a respiratory is used that Tests/Treatments/Procedures
it properly fit, not be shared or reused. Answer 2 is correct 4. CORRECT ANSWER: 1. Answer 1 is correct because
because a medical-surgical mask is indicated if the nurse intermittent ice causes vasoconstriction and reduces
will be within 3 feet of the client. Answer 3 is incorrect swelling. Answer 2 is incorrect because heat would cause
because eye protection is needed when there is a potential for vasodilation and make swelling worse. Answer 3 is incorrect
sputum to be sprayed during a procedure such as suctioning. because an injured joint needs to rest. Range of motion
Answer 4 is incorrect because the gloves do not have to be would be painful and disruptive to initial healing. Answer 4
sterile. is incorrect because positioning the foot lower than the heart
TEST-TAKING TIP: Think: infection is spread by droplets; a would increase swelling.
very basic precaution is a surgical mask. TEST-TAKING TIP: Think of the effect of each option on blood
Content Area: Adult Health, Infectious Disease; Integrated flow, which affects swelling the first 24 hours.
Process: Nursing Process, Implementation; Cognitive Level: Content Area: Adult Health, Musculoskeletal; Integrated
Application; Client Need/Subneed: Safe, Effective Care Process: Nursing Process, Implementation; Cognitive Level:
Environment/Safety and Infection Control, Preventive Comprehension; Client Need/Subneed: Physiological
Measures Integrity/Reduction of Risk Potential/Potential for
2. CORRECT ANSWER: 2. Answer 1 is incorrect because Complications of Diagnostic Tests/Treatments/Procedures
antibiotics are taken on a routine schedule, not prn for 5. CORRECT ANSWER: 3. Answer 1 is incorrect because
symptom management. Answer 2 is correct because chilling lupus rashes worsen with sunlight exposure. Answer 2 is
is an indication that the hypothalamus thermostat has incorrect because soap and water washing does not help man-
been reset at a higher level, and the body feels cold until age the rash. Answer 3 is correct because antimalarial
metabolism increases the temperature to the new fever drugs are one of the first lines of treatment for lupus.
level. Twenty-four hours after antibiotics have been start- Topical steroids are also often used for skin rashes.
ed, the client should be feeling better. Chilling and fever Answer 4 is incorrect because a varicella vaccine booster is
may indicate that the antibiotic is not effective against not related to the management of lupus rashes.
the organism. Answer 3 is incorrect because it is generally TEST-TAKING TIP: Eliminate the obvious (sunlight and soap,
safe to take acetaminophen with an antibiotic, even in the which is drying). Only one option appears to do something to
first 24 hours. Answer 4 is incorrect because fever does treat the rash.
not usually require cooling until over 38.3C (101F). Content Area: Adult Health, Integumentary; Integrated Process:
Antipyretics are the preferred treatment. A cool cloth on Nursing Process, Implementation; Cognitive Level: Application;
the forehead will not cool internal tissues. Client Need/Subneed: Physiological Integrity/Pharmacological
TEST-TAKING TIP: Only one option indicates a worsening of and Parenteral Therapies/Expected Effects/Outcomes
the clients condition.
ANSWERS

Content Area: Adult Health, Infection Control; Integrated 6. CORRECT ANSWER: 4. Answer 1 is incorrect because the
Process: Teaching and Learning; Cognitive Level: Application; client should not have any foods or fluid externally to avoid
Client Need/Subneed: Physiological Integrity/Reduction of Risk surgical delays, aspiration risk, or leakage of ingested sub-
Potential/Vital Signs Changes/Abnormalities stances out of damaged viscera. Answer 2 is incorrect because
the client should not have any food or fluid internally to avoid
3. CORRECT ANSWER: 4. Answer 1 is incorrect because surgical delays, aspiration risk, or leakage of ingested sub-
having the elevated head reduces the risk of aspiration, but stances out of damaged viscera. Answer 3 is incorrect because
this client is not able to swallow and should not receive the risks of TPN are not warranted in this client who may only
meals. Answer 2 is incorrect because solid food is easier to be NPO for a day or two. Answer 4 is correct because, if
swallow than thin liquids, but this client is not able to swal- internal bleeding is present, the client will need to go to
low and should not have either. Answer 3 is incorrect surgery. Keep the client NPO to avoid surgical delays,
because thick liquids are easier to swallow than thin liquids, aspiration risk, or leakage of ingested substances out of
but this client cannot swallow and should not have any damaged viscera.
liquids. Answer 4 is correct because this client is at great TEST-TAKING TIP: Three options involve ingestion of food or
risk for aspiration. The feeding tube bypasses the junc- fluid. Choose the option that is different.
tion of the esophagus and trachea, where the aspiration Content Area: Adult Health, Hematological; Integrated Process:
could most likely occur. Nursing Process, Implementation; Cognitive Level: Application;
TEST-TAKING TIP: Three options require oral intake. Only Client Need/Subneed: Physiological Integrity/Reduction of Risk
one option is different. Choose the option that is different. Potential/Potential for Alterations in Body Systems
2164_Ch06_347-578 29/03/12 12:31 PM Page 561

Answers/Rationales/Tips 561
7. CORRECT ANSWER: 1. Answer 1 is correct because 10. CORRECT ANSWER: 3. Answer 1 is incorrect because
decreased blood flow can cause fingers to be pale and often tolerable pain is not a reasonable outcome. This pain is
cool, such as one might see with Raynauds phenomenon. a result of ischemia, so any continued pain may mean ongoing
Symptoms lessen when covered. Answer 2 is incorrect damage to the heart. Answer 2 is incorrect because the expect-
because oxygen saturation readings would be inaccurate if ed outcome needs to reflect resolution of the problem. A client
checked on a digit with poor blood flow. Answer 3 is incor- could agree to rest and even take pain medicine, yet the
rect because the problem is related to hypothermia, not ischemia could be ongoing. Answer 3 is correct because con-
hypoventilation. Answer 4 is incorrect because the fingers are tinuing chest pain indicates continued ischemia. Treatment
cool and pale because of poor local blood flow. This is unre- should be given until the pain is gone. Answer 4 is incorrect
lated to blood sugar. because the expected outcome needs to reflect resolution of
TEST-TAKING TIP: Only one intervention will directly affect the problem, not completion of an intervention. Giving pain
cold fingers. medications may not resolve the problem, and additional inter-
Content Area: Adult Health, Circulation; Integrated Process: ventions would be required.
Nursing Process, Implementation; Cognitive Level: Application; TEST-TAKING TIP: Look for the option that indicates the
Client Need/Subneed: Physiological Integrity/Basic Care and problem is resolvedno pain.
Comfort/Non-pharmacological Comfort Interventions Content Area: Adult Health, Cardiology; Integrated Process:
Nursing Process, Planning; Cognitive Level: Analysis;
8. CORRECT ANSWER: 4. Answer 1 is incorrect because Client Need/Subneed: Physiological Integrity/Physiological
TPN may be necessary, but it is not the first strategy to try.
Adaptation/Alterations in Body Systems
The gastrointestinal system should be used for nutrition
when functional. Answer 2 is incorrect because light nutri- 11. CORRECT ANSWER: 3. Answer 1 is incorrect because
tion via a feeding tube may be necessary, but is not the first stress would not be associated with this sensation. Answer 2 is
strategy to try in a client who is able to swallow. Answer 3 is incorrect because, while sensations might be affected, her
incorrect because, although administration of enteral feed- reported symptoms could be a real problem that should be
ings via a tube is a reasonable strategy to manage malnutri- investigated. Answer 3 is correct because radiation damages
tion, less invasive strategies should be tried first. Answer 4 is tissue, and the healing process may not effectively keep the
correct because malnutrition in cancer clients is compli- vagina and bladder separate. Bladder contraction could
cated and often difficult to manage. While megestrol send urine through a fistula, to come out the vagina.
stimulates appetite, nutritional supplements provide Answer 4 is incorrect because rupture of the bladder would
nutrients. cause signs associated with sepsis.
TEST-TAKING TIP: Only one option focuses on oral intake. TEST-TAKING TIP: Look for an option that best explains the
The other three are more invasive or aggressive. description of voiding through. A fistula is an abnormal opening.
Content Area: Adult Health, Oncological; Integrated Process: Content Area: Adult Health, Oncological; Integrated Process:
Nursing Process, Implementation; Cognitive Level: Application; Nursing Process, Analysis; Cognitive Level: Application;
Client Need: Physiological Integrity/Basic Care and Comfort/ Client Need/Subneed: Physiological Integrity/Physiological
Nutrition and Oral Hydration Adaptation/Alterations in Body Systems
9. CORRECT ANSWER: 3. Answer 1 is incorrect because 12. CORRECT ANSWER: 3. Answer 1 is incorrect because

ANSWERS
this slight elevation in blood sugar may reflect a neuroen- urine production is dependent on volume status and kidney
docrine response to stress or may be a result of the intra- function. Suction cannot be used to create more urine out-
venous sugar. Additional testing would be required to put. Answer 2 is incorrect because the dressing is changed
determine if the client is prediabetic. Answer 2 is incorrect weekly unless wet, and the bag is emptied when half to
because insulin is not required for this slight elevation in two thirds full. Answer 3 is correct because a nephrostomy
blood glucose. Answer 3 is correct because this slight tube is inserted into the ureter, directly draining the
elevation in blood sugar may reflect a neuroendocrine kidney. The pelvis of the kidney cannot accommodate
response to stress or may be a result of the intravenous volumes greater than 10 mL. The procedure must be ster-
sugar. Additional testing would be required to determine ile. Answer 4 is incorrect because, unless there was ureteral
if the client is prediabetic. Answer 4 is incorrect because flow around the nephrostomy tube, it would be dangerous
NPO status in not indicated. to clamp the tube. Urine backing up in the kidneys could
TEST-TAKING TIP: Choose the option that best addresses the cause hydronephrosis and infection.
clients concern: information. TEST-TAKING TIP: Think of an action that keeps a tube patent.
Content Area: Adult Health, Endocrine; Integrated Process: Content Area: Adult Health, Renal; Integrated Process:
Nursing Process, Implementation; Cognitive Level: Application; Nursing Process, Implementation; Cognitive Level: Application;
Client Need/Subneed: Physiological Integrity/Reduction of Risk Client Need: Physiological Integrity/Physiological Adaptation/
Potential/Potential for Alterations in Body Systems Alterations in Body Systems
2164_Ch06_347-578 29/03/12 12:31 PM Page 562

562 chapter 6 Physiological Integrity

13. CORRECT ANSWER: 1. Answer 1 is correct because Client Need/Subneed: Physiological Integrity/Physiological
an indwelling catheter diverts urine away from the Adaptation/Fluid and Electrolyte Imbalances
operative site, reducing risk of wound infection. 16. CORRECT ANSWERS: 3, 5, 6. Answer 1 is incorrect
Answer 2 is incorrect because, with a fresh postoperative
because the client is not necessarily on prolonged bedrest.
wound, urine needs to be diverted from the area. Answer 3 Immobility is not the priority. Bedrest is indicated until
is incorrect because fluid intake should match the needs of hypertension is controlled. Answer 2 is incorrect because
the body. It is not appropriate to restrict fluids to reduce protein needs to be limited due to proteinuria, which is
the risk of incontinence. Answer 4 is incorrect because present with the disease. Carbohydrates are the source of
diuretics facilitate removal of excess fluid from the body. energy. Answer 3 is correct because acute glomeru-
They are not used to manage incontinence. lonephritis (AGN) decreases glomerular function.
TEST-TAKING TIP: The question is asking for a way to Fluid is restricted during the edematous phase. There
prevent wound contamination by urine. Although a catheter is a risk of renal failure as a result of the inflammation
increases the risk of urinary tract infections, it is the only (itis). Answer 4 is incorrect because the problem is not
option that fully protects the wound. retention but the potential for renal failure and inadequate
Content Area: Adult Health, Gastrointestinal; Integrated
urine proteinuria. The disease progresses from edema to
Process: Nursing Process, Implementation; Cognitive Level:
diuresis. Answer 5 is correct because the edematous phase
Comprehension; Client Need/Subneed: Physiological of AGN lasts from 4 to 10 days. Daily weight is the best
Integrity/Reduction of Risk Potential/Potential for indicator of fluid changes. Answer 6 is correct because
Alterations in Body Systems hypertension occurs with AGN. Headache is common.
14. CORRECT ANSWER: 2. Answer 1 is incorrect because Antihypertensives will be ordered and BP needs to be
pain would not be typically seen if bladder cancer was sus- recorded q4h.
pected. Answer 2 is correct because pain and blood in the TEST-TAKING TIP: Choose the answers that most directly
urine without a fever would be more consistent with a relate to monitoring renal function.
blow or trauma to the abdomen or bladder. Answer 3 is Content Area: Adult Health, Renal; Integrated Process:
incorrect because glomerulonephritis would present with a Nursing Process, Planning; Cognitive Level: Application;
fever. Answer 4 is incorrect because symptoms of Client Need/Subneed: Physiological Integrity/Reduction of
pyelonephritis would include a fever. Risk Potential/Potential for Alterations in Body Systems
TEST-TAKING TIP: Without a fever, eliminate the 17. CORRECT ANSWERS: 1, 5. Answer 1 is correct
options with itis. Choose an option that would best because loss of half of the visual field (hemianopsia)
explain the pain. will be on the same side as the paralyzed side. The left
Content Area: Adult Health, Renal; Integrated Process: Nursing
visual field will be intact. Answer 2 is incorrect because
Process, Assessment; Cognitive Level: Application; the right side will be weak or paralyzed. The left side
Client Need/Subneed: Physiological Integrity/Reduction of
will be stronger. Answer 3 is incorrect because the client
Risk Potential/System Specific Assessments likely has right visual field blindness. Answer 4 is incorrect
15. CORRECT ANSWER: 4. Answer 1 is incorrect because because there is no expectation of deafness. Food should
seizures can result from very low blood sugar or Na+, be placed on the left side. Answer 5 is correct because
ANSWERS

but these values are not low enough to be associated with the right side will be weakened or paralyzed. The
seizure risk. Answer 2 is incorrect because this Na+ level client should be taught to put weight on the stronger
is only mildly low and does not require treatment with left side when transferring from bed to chair and
sodium. Answer 3 is incorrect because ingestion of free back.
water would further dilute the sodium. Answer 4 is correct TEST-TAKING TIP: Select the option that is different from
because, as blood sugar rises, serum osmolarity increases others. Note that three options relate to the right. Only two
and the pituitary secretes antidiuretic hormone. This options relate to the left side.
causes the kidneys to retain free water, diluting down Content Area: Adult Health, Cardiovascular; Integrated Process:
serum sodium. Direct treatment of the hyponatremia is Nursing Process, Implementation; Cognitive Level: Application;
not needed. Client Need/Subneed: Physiological Integrity/Physiological
TEST-TAKING TIP: Ask: Is there a problem or not at this Adaptation/Alterations in Body Systems
time? Understand the physiologic changes when blood sugar is 18. CORRECT ANSWER: 2. Answer 1 is incorrect because
lowered. The low sodium is occurring because of hemodilution. a delay in treatment could result in irreversible damage.
Select the answer that monitors, rather than an action (pads, Answer 2 is correct because a medical or surgical interven-
salt tablets, or water). tion is needed for an apparent increase in intracranial
Content Area: Adult Health, Endocrine; Integrated Process:
pressure. Answer 3 is incorrect because a change in level
Nursing Process, Implementation; Cognitive Level: Application; of consciousness is an early sign of increased intracranial
2164_Ch06_347-578 29/03/12 12:31 PM Page 563

Answers/Rationales/Tips 563
pressure that requires an intervention. Answer 4 is incorrect should receive both vaccines particularly if they also
because this is not the RNs role. have pre-existing chronic conditions. The vaccines can
TEST-TAKING TIP: A change in level of consciousness requires be taken the same day. Answer 4 is incorrect because the
immediate notification of the physician. vaccines can be given the same day in two different syringes.
Content Area: Adult Health, Neurological; Integrated Process: Only the nasal forms of the vaccine need to be given a
Nursing Process, Implementation; Cognitive Level: Analysis; month apart and the H1N1 nasal vaccine is reserved for
Client Need/Subneed: Physiological Integrity/Physiological ages 2 to 49 years.
Adaptation/Medical Emergencies TEST-TAKING TIP: Choose the answer that focuses on both
vaccines and the same day.
19. CORRECT ANSWER: 1. Answer 1 is correct because Content Area: Adult Health, Infectious Disease;
intermittent claudication occurs with peripheral vascular
Integrated Process: Teaching and Learning; Cognitive
disease and inadequate blood flow to the tissues. As the
Level: Application; Client Need/Subneed: Safe, Effective
disease progresses, the pain will occur sooner. Answer 2 is
Care Environment/Safety and Infection Control,
incorrect because pedal edema is swelling from venous sta-
Preventive Measures
sisnot pain associated with activity. Answer 3 is incorrect
because peripheral vascular disease will affect peripheral 22. CORRECT ANSWERS: 1, 5, 6. Answer 1 is correct
pulses, but the question is looking for the development of because sweating decreases as heat exhaustion progresses
pain. Answer 4 is incorrect because inadequate arterial blood to heat stroke. Answer 2 is incorrect because the body
flow will affect the color and temperature, but the more temperature is usually above 104F. Answer 3 is incorrect
rapid development of pain with walking is the accurate because the skin may first be damp if the client has been
response. active, then no perspiration. Answer 4 is incorrect because
TEST-TAKING TIP: The term intermittent is a hint that blood pressure decreases, and there is tachycardia and rapid
the problem starts and stopsstart walking, pain develops, stop respiratory ratethe symptoms of shock. Answer 5 is
walking. correct because the client is experiencing severe
Content Area: Adult Health, Vascular; Integrated Process: dehydration and will present with symptoms of shock.
Nursing Process, Analysis; Cognitive Level: Application; Answer 6 is correct because the client has lost the
Client Need/Subneed: Physiological Integrity/Physiological normal cooling mechanism of perspiration. The skin
Adaptation/Pathophysiology will be flushed and dry from the high internal body
temperature.
20. CORRECT ANSWERS: 4, 5, 6. Answer 1 is incorrect TEST-TAKING TIP: Think heat strokea failure of the bodys
because hallucinations do occur but not initially. Answer 2
regulatory mechanism.
is incorrect because the vital signs are initially elevated.
Content Area: Adult Health, Thermoregulatory; Integrated
Answer 3 is incorrect because the client is agitated.
Process: Nursing Process, Assessment; Cognitive Level:
Answer 4 is correct because the client will experience
Application; Client Need/Subneed: Physiological Integrity/
autonomic overactivity causing tachycardia, dilated
Physiological Adaptation/Medical Emergencies
pupils, and profuse perspiration. Answer 5 is correct
because the autonomic overactivity will cause tachycar- 23. CORRECT ANSWERS: 2, 4, 3, 1. Supplemental
dia. Answer 6 is correct because the pupils will be dilated oxygen is the first action to prevent any cardiac

ANSWERS
from the autonomic overactivity. damage from hypoxia. The oxygen can be started while
TEST-TAKING TIP: Look for an early autonomic response. continuing the client assessment and applying the ECG
Content Area: Adult Health, Cardiovascular; Integrated Process: electrodes. The ECG leads should be applied second, to
Nursing Process, Analysis; Cognitive Level: Application; assess for any life-threatening arrhythmias. NTG is
Client Need/Subneed: Physiological Integrity/Physiological often given with an antacid for unexplained chest pain.
Adaptation/Alterations in Body Systems It would be the third action after determining the
cardiac rhythm. NTG can change the ECG pattern.
21. CORRECT ANSWER: 3. Answer 1 is incorrect because Morphine may eventually be given if the pain is not
the best protection is to receive both seasonal and novel
relieved by the antacid or NTG. This would be the
H1N1 vaccines. The pneumonia vaccine should be given
fourth action.
once to adults 65 years or older. If given at 60 years of age
TEST-TAKING TIP: When all of the options are correct, think
or earlier a booster may be required 5 to 10 years later.
of the prioritiesprevent damage from hypoxia, gather data,
Children older than 2 years with chronic respiratory or
and treat.
cardiac conditions may receive the pneumonia vaccine.
Content Area: Adult Health, Cardiovascular; Integrated
Answer 2 is incorrect because healthy adults over 65 were
Process: Nursing Process, Implementation; Cognitive Level:
not as susceptible to the novel H1N1 (swine flu) as originally
Analysis; Client Need/Subneed: Physiological Integrity/
feared. Answer 3 is correct because the older adult, while
Physiological Adaptation/Alterations in Body Systems
less susceptible to the swine flu than younger clients,
2164_Ch06_347-578 29/03/12 12:31 PM Page 564

564 chapter 6 Physiological Integrity

24. CORRECT ANSWERS: 3, 5. Answer 1 is incorrect Content Area: Adult Health, Fluid and Electrolyte Imbalances;
because the problem in DVT causes phlebitis, which is Integrated Process: Nursing Process, Analysis; Cognitive Level:
characterized by warmth, redness, and pain. Answer 2 is Analysis; Client Need/Subneed: Physiological Integrity/
incorrect because the problem in DVT is circulatory, not Reduction of Risk Potential/Laboratory Values
neurological. Answer 3 is correct because a clot in a leg 27. CORRECT ANSWER: 3. Answer 1 is incorrect because
vein will affect venous return and cause swelling and all of the topical therapies must be effective against gram-
edema. Answer 4 is incorrect because the problem in negative organisms. The agent does not sterilize the burn
DVT is in the leg, not the eyes. Answer 5 is correct because but reduces the number of bacteria so that the bodys
pain with dorsiflexion (positive Homans sign) may be defense mechanisms are able to overcome the infection.
present with DVT but should not be the only indicator Answer 2 is incorrect because the form of the topical agent
of DVT. is not important. Treatment may alternate between a liquid
TEST-TAKING TIP: Visualize the effect a clot in the leg will and a cream. Answer 3 is correct because, since eschar is
have on venous circulation. devitalized tissue, the topical agent must be able to
Content Area: Adult Health, Vascular; Integrated Process: penetrate eschar and not be systemically toxic. Answer 4
Nursing Process, Analysis; Cognitive Level: Application; is incorrect because burn injuries are not uniform. Variations
Client Need/Subneed: Physiological Integrity/Physiological in burn depth and the presence of eschar affect the absorp-
Adaptation/Pathophysiology tion of a topical agent.
25. CORRECT ANSWER: 4. Answer 1 is incorrect because TEST-TAKING TIP: Look for an option that would inhibit
this is dangerously high potassium. It requires immediate the absorption ability of the topical agent (e.g., eschar), and
treatment to avoid a lethal arrhythmia. Answer 2 is incorrect therefore render the antimicrobial less effective.
because restricting intake or using an ion exchange agent Content Area: Adult Health, Integumentary; Integrated Process:
would reduce potassium level too slowly. Answer 3 is incor- Nursing Process, Analysis; Cognitive Level: Application;
rect because fluid restriction does not reduce potassium. Client Need/Subneed: Physiological Integrity/Pharmacological
Answer 4 is correct because this is dangerously high and Parenteral Therapies/Expected Effects/Outcomes
potassium. Glucose, insulin, and bicarbonate help 28. CORRECT ANSWER: 1. Answer 1 is correct because
move the potassium intracellularly. Calcium helps TB is a bacterial infectious disease that is communicable
protect the heart from hyperkalemia-induced to others and is associated with significant mortality
arrhythmias. and morbidity. Answer 2 is incorrect because scabies is a
TEST-TAKING TIP: Know the normal ranges for common skin infection associated with an infestation of an itch
electrolytes. Recognize the dangerously high level and look for mite. The mites can be transferred to family members and
an option that will produce a change rapidlySTAT. Look at close contacts, but it is not an infectious disease. Answer 3 is
the action words: no change required, restrict, and give. incorrect because Lyme disease is the result of a tick bite
Choose give. and is not infectious. Answer 4 is incorrect because ring-
Content Area: Adult Health, Fluid and Electrolyte Imbalances; worm, while contagious, is a fungal infection.
Integrated Process: Nursing Process, Implementation; Cognitive TEST-TAKING TIP: Determine the potential for a community
Level: Application; Client Need/Subneed: Physiological outbreak from each option.
ANSWERS

Integrity/Physiological Adaptation/Medical Emergencies Content Area: Adult Health, Communicable Diseases;


26. CORRECT ANSWER: 3. Answer 1 is incorrect because gen- Integrated Process: Nursing Process, Implementation;
erally clients with renal failure have trouble excreting potassi- Cognitive Level: Application; Client Need/Subneed: Safe and
um. In the days before the next dialysis treatment, the potassi- Effective Care Environment/Safety and Infection Control/
um will be increasing. It is not appropriate to give potassium to Standard/Transmission-based/Other Precautions
this client. Answer 2 is incorrect because this clients potassium 29. CORRECT ANSWER: 2. Answer 1 is incorrect because
is already too high. More potassium is not needed. Answer 3 is the stability of the clients blood sugar is a concern. Also
correct because clients with cardiac disease are vulnerable to the client needs observation as insulin is being regulated.
arrhythmia when serum potassium is low. Furosemide will Answer 2 is correct because the older adult client has a
cause potassium loss. It is important to replace potassium chronic problem and would be supervised in the long-
before the furosemide is given. Answer 4 is incorrect because term care facility. Answer 3 is incorrect because a client
clients with cardiac disease are vulnerable to arrhythmia when who is post-MI is at risk for possible arrhythmias and
serum potassium is low. Spironolactone is a potassium-sparing requires continuous monitoring. Answer 4 is incorrect
diuretic that would not be expected to drop the potassium level because the clients laboratory values are abnormal, and
as much as furosemide would. the client may be experiencing fatigue and weakness from
TEST-TAKING TIP: Focus on cardiac disease and the drug, if the anemia.
given, that will further affect the K + level (i.e., cause loss of TEST-TAKING TIP: Choose the client who is more stable and
K +), so the client will need K + replacement. does not require acute care.
2164_Ch06_347-578 29/03/12 12:31 PM Page 565

Answers/Rationales/Tips 565
Content Area: Adult Health, Triage; Integrated Process: incorrect because the most immediate risk is the rapid drop
Nursing Process, Analysis; Cognitive Level: Analysis; of potassium as the electrolyte moves back into the cell as a
Client Need/Subneed: Safe and Effective Care Environment/ result of rehydration and the insulin drip.
Management of Care/Establishing Priorities TEST-TAKING TIP: The question asks for the greatest risk.
Hypoglycemia is too obvious. Think about the effects of
30. CORRECT ANSWER: 3. Answer 1 is incorrect because insulin on K +.
tachycardia would be the next concern after the blood
Content Area: Adult Health, Endocrine; Integrated Process:
pressure was stabilized or controlled. Answer 2 is incorrect
Nursing Process, Analysis; Cognitive Level: Analysis;
because blood pressure is the first concern. Hyperglycemia
Client Need/Subneed: Physiological Integrity/Physiological
may be present, and insulin may be required. Answer 3 is
Adaptation/Fluid and Electrolyte Imbalances
correct because treatment initially is directed at reducing
the marked elevation of blood pressure quickly. Answer 4 33. CORRECT ANSWER: 2. Answer 1 is incorrect
is incorrect because dysrhythmias occur as a result of the because inflammation of the gallbladder interferes with
elevated blood pressure. Controlling the blood pressure the normal color of feces, and the stool color is characteristi-
will reduce the likelihood that the client will develop cally grayish, like putty. Black stools would signal old
ECG changes. blood consistent with GI bleeding. Answer 2 is correct
TEST-TAKING TIP: Choose an option that, if treated, will because biliary colic is common in cholecystitis and is
likely correct other findingsthat is, lower the BP, and the associated with nausea and vomiting several hours after
pulse and ECG will likely stabilize. a heavy, high-fat meal. Answer 3 is incorrect because this
Content Area: Adult Health, Hematological; Integrated is a dangerously high temperature for an adult, and not a
Process: Nursing Process, Assessment; Cognitive Level: common sign. A temperature may be present, but one this
Analysis; Client Need/Subneed: Physiological Integrity/ high might result from gangrene of the gallbladder and
Reduction of Risk Potential/System Specific Assessments rupture, which is not common. Answer 4 is incorrect
because the pain from an acute inflammation of the
31. CORRECT ANSWER: 1. Answer 1 is correct because gallbladder would be in the upper right abdomen, with
neuropathies reduce the sensation (pain) in the clients
radiation possible to the midsternal or right shoulder
feet. Consequently, the client is unaware of injuries or
area.
trauma to the feet that could lead to ulceration and
TEST-TAKING TIP: Think of food-related problems. Know
cellulitis, which are more difficult to treat in the person
where the gallbladder is located and a likely symptom.
who has diabetes. Answer 2 is incorrect because inspection
Often the initial warning of a gallbladder problem comes
of the feet with mirrors is most important. Foot care is
with eating.
important; however, any breakdown would be noted using
Content Area: Adult Health, Gastrointestinal; Integrated
a mirror. Answer 3 is incorrect because inspection is most
Process: Nursing Process, Assessment; Cognitive Level:
important. Proper foot care will prevent cracks, which
Application; Client Need/Subneed: Physiological Integrity/
would be noted by using a mirror. Answer 4 is incorrect
Physiological Adaptation/Pathophysiology
because, although this technique is correct and should
prevent foot complications, inspection of feet is most 34. CORRECT ANSWER: 4. Answer 1 is incorrect because
important. all four quadrants should be assessed and normal bowel

ANSWERS
TEST-TAKING TIP: All options are correct. Choose the sounds may not occur more often than every 20 seconds,
one that has the greatest potential for preventing a serious requiring a minimum of 1 minute and 20 seconds.
problem. Answer 2 is incorrect because all four quadrants should be
Content Area: Adult Health, Endocrine; Integrated Process: assessed and normal bowel sounds may not occur more often
Nursing Process, Implementation; Cognitive Level: Analysis; than every 20 seconds requiring, a minimum of 1 minute
Client Need/Subneed: Physiological Integrity/Basic Care and and 20 seconds. Answer 3 is incorrect because, to determine
Comfort/Personal Hygiene if bowel sounds are absent, the nurse must listen for a mini-
mum of 3 minutes and for as long as 5 minutes. Answer 4 is
32. CORRECT ANSWER: 2. Answer 1 is incorrect because correct because, to determine if bowel sounds are absent,
treatment will include as much as 6 to 10 L of normal
the nurse must listen for a minimum of 3 minutes and
saline for 2 to 3 hours. The infusion rate will be slowed to
for as long as 5 minutes.
prevent hypervolemia. Answer 2 is correct because rehydra-
TEST-TAKING TIP: Is it seconds or minutes? An accurate
tion and the administration of insulin will cause the
assessment takes time.
potassium to move back into the cells, resulting in a
Content Area: Adult Health, Gastrointestinal; Integrated
potentially even lower K+ level. Potassium replacement
Process: Nursing Process, Assessment; Cognitive Level:
begins when the K+ level drops to normal to prevent
Comprehension; Client Need/Subneed: Physiological Integrity/
hypokalemia. Answer 3 is incorrect because normal saline
Reduction of Risk Potential/System Specific Assessments
is the solution of choice during rehydration. Answer 4 is
2164_Ch06_347-578 29/03/12 12:31 PM Page 566

566 chapter 6 Physiological Integrity

35. CORRECT ANSWER: 3. Answer 1 is incorrect because 38. CORRECT ANSWERS: 2, 4, 5. Answer 1 is incorrect
placement should be determined by instilling air rapidly, because this would impair blood flow through the
not liquid. Once placement is confirmed, irrigation may be fistula and potentially damage the fistula. Answer 2 is
done. Answer 2 is incorrect because the concern is the place- correct because exercise increases the vessel size to
ment of the tube first. Then, the position of the client may better accommodate the large-bore needles used in
need to be assessed to determine if the position caused the hemodialysis. Answer 3 is incorrect because the fistula
tube to be dislodged. Answer 3 is correct because the lack does not mature sufficiently to use for at least 14 days.
of drainage may mean the tube is no longer in the correct Answer 4 is correct because the arm should be reserved
location. Answer 4 is incorrect because placement should be only for dialysis. Measuring BP or drawing blood
determined before removal. Reinsertion may be distressful to may lead to fistula damage. Answer 5 is correct
the client. because tight or constricting clothing will impair
TEST-TAKING TIP: This is a priority question. More than one blood flow through the fistula and cause loss of
answer may be correct, but what does the nurse do first? Ask patency.
yourself, where is the tube tip? TEST-TAKING TIP: Since Answer 5 is correct, then Answer 1
Content Area: Adult Health, Gastrointestinal; Integrated is incorrect, because it is important to select an option that pre-
Process: Nursing Process, Implementation; Cognitive Level: vents constriction of the fistula and promotes development of
Comprehension; Client Need/Subneed: Physiological Integrity/ the site.
Reduction of Risk Potential/System Specific Assessments Content Area: Adult Health, Vascular; Integrated Process:
Nursing Process, Planning; Cognitive Level: Application;
36. CORRECT ANSWERS: 2, 3, 4. Answer 1 is incorrect Client Need/Subneed: Physiological Integrity/Reduction of
because peristalsis is diminished and paralytic ileus may
Risk Potential/Potential for Complications from Surgical
develop. Answer 2 is correct because the pain is often
Procedures and Health Alterations
diffuse initially and then localized. Answer 3 is correct
because the abdominal muscles are rigid and the 39. CORRECT ANSWER: 3. Answer 1 is incorrect because
abdomen is tender and distended. Answer 4 is correct paralytic ileus is a surgical complication that the client
because, with the abdominal tenderness and distention, cannot prevent. Answer 2 is incorrect because urinary
the client will not breathe as deeply. Answer 5 is calculi are not a surgical complication. A client can prevent
incorrect because the temperature is usually 100 to calculi through dietary and fluid management. Answer 3
101F. is correct because an ileal conduit is a urinary diversion
TEST-TAKING TIP: Think about how inflammation affects that allows direct access of organisms to the kidneys.
the body system as each option is considered. Think pain, as Proper management of the appliance, good skin care,
in Answers 2, 3, and 4. proper hand hygiene, and adequate fluids will reduce
Content Area: Adult Health, Gastrointestinal; Integrated the chance of developing an infection. Answer 4 is
Process: Nursing Process, Assessment; Cognitive Level: incorrect because mucus in the urine is normal since a
Analysis; Client Need/Subneed: Physiological Integrity/ segment of the GI tract was used to create the
Reduction of Risk Potential/Potential for Complications diversion.
from Surgical Procedures and Health Alterations TEST-TAKING TIP: Choose an option that can be controlled
ANSWERS

by the client directlypreventing infection (e.g., an itis).


37. CORRECT ANSWER: 4. Answer 1 is incorrect Content Area: Adult Health, Renal; Integrated Process:
because nausea and vomiting are not specific to CRF. A
Teaching and Learning; Cognitive Level: Application;
client with CRF may have nausea and vomiting along
Client Need/Subneed: Physiological Integrity/Reduction of
with anorexia and hiccups. Answer 2 is incorrect because
Risk Potential/Potential for Complications from Surgical
these signs are consistent with hepatic failure. Answer 3 is
Procedures and Health Alterations
incorrect because these signs would occur during acute
renal failure or with any developing metabolic acidosis. 40. CORRECT ANSWER: 2. Answer 1 is incorrect
Answer 4 is correct because the irritating toxins are because the risk of infection with a Kock pouch would
accumulating and result in skin irritation. Anemia be related to renal deterioration from reflux or fecal
results from the inadequate erythropoietin contamination, not from others with URI. Answer 2 is
production. correct because adequate fluids will maintain the urine
TEST-TAKING TIP: Look for an option that would develop output and optimal renal function. Answer 3 is incorrect
over timeweeks to months (i.e., chronic)and would occur if because a Kock pouch is a continent urinary diversion, and
the kidneys have failed. urine is drained by inserting a catheter into the stoma.
Content Area: Adult Health, Renal; Integrated Process: There is no appliance to change. Answer 4 is incorrect
Nursing Process, Assessment; Cognitive Level: Application; because irrigation is not done with urinary diversion.
Client Need/Subneed: Physiological Integrity/Physiological The Kock pouch collects urine, and a catheter is inserted
Adaptation/Pathophysiology into the stoma to remove the urine.
2164_Ch06_347-578 29/03/12 12:31 PM Page 567

Answers/Rationales/Tips 567
TEST-TAKING TIP: Keep It Simple (K.I.S.) and look for the may occur with pressure in the midbrain. Answer 4 is incor-
best approach to prevent a urinary tract infection for any rect because paradoxical chest movements are characteristic
clientadequate fluid intake. of flail chest from chest trauma.
Content Area: Adult Health, Renal; Integrated Process: TEST-TAKING TIP: Eliminate Answer 3 as it is the
Teaching and Learning; Cognitive Level: Application; opposite of what occurs (Answer 2). Eliminate Answers 1
Client Need/Subneed: Physiological Integrity/Reduction of and 4, because the conditions are not characteristic of
Risk Potential/Potential for Alterations in Body Systems increased ICP, but of pulmonary edema and chest
trauma. Remember that the pressure in the brain will
41. CORRECT ANSWER: 2. Answer 1 is incorrect interfere with the regulatory center. Respiratory response
because the desired outcome is to reduce intraocular
will be irregular.
pressure. Upright and possibly face down would be pre-
Content Area: Adult Health, Neurological; Integrated Process:
ferred. Answer 2 is correct because detachments or
Nursing Process, Assessment; Cognitive Level: Application;
tears are toward the back of the eye. A gas bubble is
Client Need/Subneed: Physiological Integrity/
frequently used to flatten the retina (pneumatic
Physiological Adaptation/Pathophysiology
retinopexy), and the client must be positioned to
make the bubble float into position. Answer 3 is incor- 44. CORRECT ANSWER: 3. Answer 1 is incorrect because
rect because the desired outcome is to reduce intraocular spinal fluid does not affect the concentration of the urine.
pressure. Upright and possible face down would be Head trauma may cause a disorder of the posterior pituitary,
preferred. Answer 4 is incorrect because the clients head but the urine specific gravity would be low. Answer 2 is
should be higher than the heart. Upright and face down incorrect because an excessive output would be more likely
would be desired. to result from trauma to the posterior pituitary, not from a
TEST-TAKING TIP: Visualize each position and how it cerebrospinal fluid (CSF) leak. Answer 3 is correct because
would contribute to reattaching the retina to the back of spinal fluid contains glucose. The RN should also look
the eye. for the halo sign of blood drainage with a yellow ring
Content Area: Adult Health, Sensory; Integrated Process: of spinal fluid. In addition, nose and ears should be kept
Nursing Process, Implementation; Cognitive Level: Application; clean to reduce the chance of infection. Answer 4 is incor-
Client Need/Subneed: Physiological Integrity/Reduction of rect because the concern is leakage of CSF as a result of a
Risk Potential/Therapeutic Procedures tear that would allow fluid to leak from the nose or ears, not
a result of a lumbar puncture.
42. CORRECT ANSWER: 4. Answer 1 is incorrect because TEST-TAKING TIP: Remember: cerebrospinal fluid is high in
older adults appeared to have some immunity to the
glucose.
virus from previous exposure to a similar strain in the
Content Area: Adult Health, Neurological; Integrated Process:
1970s. Chronic conditions increased the risk. Answer 2 is
Nursing Process, Assessment; Cognitive Level: Application;
incorrect because the child was presumed to be healthy.
Client Need/Subneed: Physiological Integrity/Reduction of
The chronic asthma increased the risk for the 13-year-old.
Risk Potential/Potential for Complications from Surgical
Answer 3 is incorrect because the spouse was healthy and
Procedures and Health Alterations
did not suffer from a chronic condition which would
increase the risk. Answer 4 is correct because the risk 45. CORRECT ANSWERS: 1, 2, 3, 4. Answer 1 is correct

ANSWERS
increases in a young person with a chronic condition. because pronator drift reflects ischemic consequences
Ages 6 months to 24 years were effected more often than of a stroke. Answer 2 is correct because facial droop
healthy older adults. reflects ischemic consequences of a stroke. Answer 3 is
TEST-TAKING TIP: Chronic condition is more of a risk factor. correct because slurred or garbled speech reflects
Select the one answer that is different from the other three ischemic consequences of a stroke. Answer 4 is correct
(asthma). because extremity weakness on one side reflects ischemic
Content Area: Adult Health, Infectious Disease; Integrated consequences of a stroke. Answer 5 is incorrect because
Process: Nursing Process, Analysis; Cognitive Level: Analysis; crackles in the lungs would indicate CHF or pneumonia.
Client Need/Subneed: Safe, Effective Care Environment/Safety Answer 6 is incorrect because decreased urine output would
and Infection Control, Prevention and Early Detection result from hypotension or impaired kidney function.
TEST-TAKING TIP: Select choices that are directly influenced
43. CORRECT ANSWER: 2. Answer 1 is incorrect because by the sensorimotor system, not respiratory (Answer 5) or renal
nasal flaring and retractions would be seen with pulmonary
(Answer 6).
edema and left-sided heart failure. Answer 2 is correct
Content Area: Adult Health, Cardiovascular; Integrated Process:
because increased intracranial pressure (ICP) presses on
Nursing Process, Assessment; Cognitive Level: Comprehension;
the structures of the brain, causing irregularities such as
Client Need/Subneed: Physiological Integrity/Physiological
Cheyne-Stokes respirations. Answer 3 is incorrect because
Adaptation/Pathophysiology
slow, irregular breathing is more common. Hyperventilation
2164_Ch06_347-578 29/03/12 12:31 PM Page 568

568 chapter 6 Physiological Integrity

46. CORRECT ANSWER: 1. Answer 1 is correct because 49. CORRECT ANSWER: 2. Answer 1 is incorrect because
immediate treatment is directed toward reversing circula- Bucks is skin traction, not skeletal traction, which uses
tory shock of addisonian crisis. Replacement of sodium pins. Answer 2 is correct because Bucks is skin traction to
and the missing steroid hormones is critical. Answer 2 is the lower leg. Circulatory disturbances and skin abra-
incorrect because the client needs sodium and an increased sions are the most important nursing concerns. Answer 3
volume. Answer 3 is incorrect because initial treatment is is incorrect because Bucks is skin traction using an elastic
normal saline. D5NS may be given with the glucocorticoids; bandage. The fracture site would not be visible. Answer 4 is
however, a vasopressor such as dopamine would not be the incorrect because Bucks uses a foam boot to support the
first line of treatment. Answer 4 is incorrect because the foot of the affected extremity in a dorsiflexed position. This
primary problem is circulatory shock. position is, therefore, not a complication.
TEST-TAKING TIP: Addisons disease is a deficiency of TEST-TAKING TIP: Choose the option that describes a prob-
glucocorticoidsnote that the correct answer has the word lem from traction that is applied externally. If too tight, a limb
glucocorticoid. becomes dusky, not flushed.
Content Area: Adult Health, Endocrine; Integrated Process: Content Area: Adult Health, Musculoskeletal; Integrated
Nursing Process, Implementation; Cognitive Level: Application; Process: Nursing Process, Assessment; Cognitive Level:
Client Need/Subneed: Physiological Integrity/Physiological Application; Client Need/Subneed: Physiological Integrity/
Adaptation/Fluid and Electrolyte Imbalances Basic Care and Comfort/Mobility/Immobility
47. CORRECT ANSWER: 4. Answer 1 is incorrect because 50. CORRECT ANSWER: 1. Answer 1 is correct because
night sweats would not be a symptom of bone marrow there is always a risk of osteomyelitis (bone infection) with
depression. Answer 2 is incorrect because these signs are orthopedic surgery. Answer 2 is incorrect because phlebitis,
consistent with dehydration, which may result from vomiting an inflammation of a vein, is not as serious as an infection in
or diarrhea from chemotherapy if untreated. Answer 3 is the bone. Answer 3 is incorrect because urinary retention can
incorrect because this would indicate nephrotoxicity and result following hip surgery because it is difficult to relax the
renal dysfunction as a result of chemotherapy. Answer 4 is sphincter muscles when in pain. Retention is not usually
correct because myelosuppression decreases WBCs, life-threatening. Answer 4 is incorrect because effective pain
RBCs, and platelets. The anemia reduces oxygen- management following implant surgery does not normally
carrying capacity, leading to fatigue and weakness. lead to addiction.
The reduced platelets increases bruising and bleeding TEST-TAKING TIP: With any orthopedic surgery, the greatest
tendencies. risk is infection.
TEST-TAKING TIP: Think: blood-related symptoms. Content Area: Adult Health, Musculoskeletal; Integrated
Review the role of the bone marrow in the production of Process: Nursing Process, Assessment; Cognitive Level:
red blood cells. Application; Client Need/Subneed: Physiological Integrity/
Content Area: Adult Health, Oncological; Integrated Reduction of Risk Potential/Potential for Complications from
Process: Nursing Process, Assessment; Cognitive Level: Surgical Procedures and Health Alterations
Comprehension; Client Need/Subneed: Physiological
51. CORRECT ANSWER: 4. Answer 1 is incorrect because
Integrity/Pharmacological and Parenteral Therapies/
the pulse rate and moist respirations may be due to the
ANSWERS

Adverse Effects/Contraindications/Interactions
rapid IV rate. Answer 2 is incorrect because the nurse needs
48. CORRECT ANSWER: 4. Answer 1 is incorrect because to do something immediately. Answer 3 is incorrect because
the stem says that this is a terminal admission. Dealing only the physician can provide orders to correct the possible
with the diagnosis would not be the most common fear. volume overload. Answer 4 is correct because the assess-
Answer 2 is incorrect because a terminal admission ment may indicate volume overload. Slowing the IV
generally means that the care will be palliative, not curative. rate keeps the IV access open while waiting for further
Answer 3 is incorrect because the needs of the client at orders.
the time of a terminal admission are for palliation, not TEST-TAKING TIP: Think about what happens when the
continuation of a normal lifestyle. Answer 4 is correct heart cannot handle the circulating volume overload, requiring
because the client who is terminally ill most often wishes IV rate and calling the physician.
to be pain-free and to know that someone will be there Content Area: Adult Health, Fluid and Electrolyte Imbalances;
in the end. Integrated Process: Nursing Process, Implementation; Cognitive
TEST-TAKING TIP: Only one option relates to dying, which is Level: Application; Client Need/Subneed: Physiological
the outcome of a terminal admission. Integrity/Physiological Adaptation/Medical Emergencies
Content Area: Adult Health, End-of-Life; Integrated Process:
52. CORRECT ANSWER: 2. Answer 1 is incorrect because,
Nursing Process, Analysis; Cognitive Level: Comprehension;
during the early postoperative period, pressure would not
Client Need/Subneed: Psychosocial Integrity/End of Life Care
be the first concern. If edema develops, breakdown will be
2164_Ch06_347-578 29/03/12 12:31 PM Page 569

Answers/Rationales/Tips 569
more likely. Answer 2 is correct because reducing edema Content Area: Adult Health, Communicable Diseases;
will promote healing and prevent complications. Integrated Process: Nursing Process, Analysis; Cognitive Level:
Answer 3 is incorrect because the wound will not be cleansed Comprehension; Client Need/Subneed: Health Promotion and
within the first 24 hours. The stump is wrapped securely Maintenance/High Risk Behaviors
after surgery, and the dressing would be reinforced if needed.
56. CORRECT ANSWER: 1. Answer 1 is correct because
Answer 4 is incorrect because a prosthesis will not be indi-
this client will likely survive the injury with pain
cated until there has been sufficient healing.
medication and fluids. Answer 2 is incorrect because this
TEST-TAKING TIP: With any trauma such as surgery,
client is not a priority as the injuries are not life-threatening.
prevent swellingin this situation look for elevation.
Answer 3 is incorrect because this victim has a normal coma
Content Area: Adult Health, Musculoskeletal; Integrated
scale score, and should be seen after the victim with burns.
Process: Nursing Process, Implementation; Cognitive Level:
Answer 4 is incorrect because this client in a triage situation
Application; Client Need/Subneed: Physiological Integrity/Basic
is not considered to be a priority.
Care and Comfort/Mobility/Immobility
TEST-TAKING TIP: The priority in a disaster is the most
53. CORRECT ANSWER: 3. Answer 1 is incorrect because critical with the greatest potential for survival if treated.
this choice does not immobilize the clients neck. Answer 2 is Content Area: Adult Health, Triage; Integrated Process:
incorrect because, after immobilizing the spine, determining Nursing Process, Assessment; Cognitive Level: Application;
responsiveness would be the next priority. Answer 3 is correct Client Need/Subneed: Safe and Effective Care Environment/
because, before assessing the CABs (compression, airway, Management of Care/Establishing Priorities
and breathing) in this client, it is important to prevent
57. CORRECT ANSWER: 2. Answer 1 is incorrect because this
further injury to the spinal cord. Answer 4 is incorrect
delays reestablishing the airway. Answer 2 is correct because
because this choice does not stabilize the spine.
keeping the airway open is most important. Concern for
TEST-TAKING TIP: Whenever there is a possibility of neck
contamination of the tube can be addressed after a patent
injury, stabilize the neck before checking the ABCs.
airway is established. Answer 3 is incorrect because keeping
Content Area: Adult Health, Neurological; Integrated Process:
the airway open is most important. Answer 4 is incorrect because
Nursing Process, Implementation; Cognitive Level: Application;
reinserting the tracheostomy tube is a more effective way to
Client Need/Subneed: Physiological Integrity/Physiological
ventilate the client.
Adaptation/Medical Emergencies
TEST-TAKING TIP: See Answer 2 with the word open in the
54. CORRECT ANSWER: 3. Answer 1 is incorrect because stem. Look for a nursing action that will best keep the airway
pupillary changes related to cerebral hypoxia and increased open and ensure adequate ventilation.
ICP are very late signs. Answer 2 is incorrect because the Content Area: Adult Health, Respiratory; Integrated
body would try to compensate for the drop in BP before a Process: Nursing Process, Implementation; Cognitive Level:
change in LOC. Answer 3 is correct because an increase in Application; Client Need/Subneed: Physiological Integrity/
pulse would be an attempt by the body to increase the Physiological Adaptation/Medical Emergencies
BP. Answer 4 is incorrect because the client would be more
58. CORRECT ANSWER: 2. Answer 1 is incorrect
likely to become weak (i.e., flaccid extremities) if hypoten-
because the ABGs support hypoventilation and atelectasis.
sion occurs.

ANSWERS
Suctioning will not reexpand the alveoli as effectively as
TEST-TAKING TIP: Think how the body would respond to
coughing and deep breathing. Answer 2 is correct because
maintain adequate blood flow.
atelectasis is likely as a result of general anesthesia.
Content Area: Adult Health, Cardiovascular; Integrated
The client also appears to be hypoventilating most
Process: Nursing Process, Analysis; Cognitive Level:
likely because of abdominal pain. The client should be
Application; Client Need/Subneed: Physiological Integrity/
medicated for pain before coughing and deep breathing.
Physiological Adaptation/Pathophysiology
Answer 3 is incorrect because the pH is within normal limits,
55. CORRECT ANSWER: 2. Answer 1 is incorrect because indicating compensation. Answer 4 is incorrect because
menopause does not change the risk. (Women who engage in coughing and deep breathing, not just a position change, is
high-risk sexual activities are at risk at any age.) Answer 2 is needed. Any degree of head elevation will improve deep
correct because older men tend to not use condoms and do breathing.
not consider themselves to be at risk for HIV. Answer 3 is TEST-TAKING TIP: With general anesthesia and abdominal
incorrect because the risk is less (but not risk-free). Answer 4 is surgery, hypoventilation is likely. An independent nursing
incorrect because, if the mother with HIV uses safe sex prac- action is encouraging coughing and deep breathing.
tices, HIV can be prevented. The risk to the child of a mother Content Area: Adult Health, Respiratory; Integrated
with HIV is at birth or if being breastfed. Process: Nursing Process, Implementation; Cognitive Level:
TEST-TAKING TIP: The best answer is the least obvious one Analysis; Client Need/Subneed: Physiological Integrity/
here: older-age men. Reduction of Risk Potential/Laboratory Values
2164_Ch06_347-578 29/03/12 12:31 PM Page 570

570 chapter 6 Physiological Integrity

59. CORRECT ANSWER: 2. Answer 1 is incorrect because metabolic acidosis. Answer 4 is correct because the
the heart is not beating independently, so the pulse rate heart would attempt to increase cardiac output by
would decrease. Answer 2 is correct because ventricular increasing the pulse rate as the volume continued to
function would be affected, causing signs of heart decrease.
failure and inadequate perfusion. Answer 3 is incorrect TEST-TAKING TIP: The key word is worsening. Choose the
because a pacemaker is not correcting atrial fibrillation but option that best describes the ongoing cardiovascular response to
a conduction defect to the ventricles. Answer 4 is incorrect a decrease in circulating volume.
because initially the client would experience symptoms Content Area: Adult Health, Cardiovascular; Integrated
consistent with heart failure from inadequate cardiac Process: Nursing Process, Analysis; Cognitive Level:
pumping. Application; Client Need/Subneed: Physiological Integrity/
TEST-TAKING TIP: Choose the option that is different from Physiological Adaptation/Pathophysiology
the others. Only one describes the effects of poor perfusion. 63. CORRECT ANSWERS: 1, 2, 3, 5. Answer 1 is correct
Content Area: Adult Health, Cardiology; Integrated Process:
because there is frequently a worsening of a cough that
Nursing Process, Assessment; Cognitive Level: Application; may have appeared to be improving. Answer 2 is correct
Client Need/Subneed: Physiological Integrity/Physiological
because shortness of breath is an emergency warning sign
Adaptation/Pathophysiology and requires urgent care. Answer 3 is correct because
60. CORRECT ANSWERS: 1, 2, 4, 5. Answer 1 is correct swine flu may present with vomiting not typically seen
because the heart is unable to handle the venous return, with the seasonal flu. Answer 4 is incorrect because the
and it will back up into the venous system, causing the client with swine flu does not have a high fever and may have
ankle edema. Answer 2 is correct because the heart is a normal temperature. Answer 5 is correct because clients
unable to adequately pump the blood through the right with swine flu may present with diarrhea, non common
side of the heart, so there is a back-pressure and JVD. in adults with seasonal flu, although diarrhea is seen in
Answer 3 is incorrect because crackles would indicate children with seasonal flu.
left-sided heart failure. Answer 4 is correct because organ TEST-TAKING TIP: Recognize the one symptom (high fever)
enlargement will occur from the back-pressure with the that is not typical.
failing heart. Answer 5 is correct because the right side of Content Area: Adult Health, Infectious Disease; Integrated
the heart cannot handle the venous return, and the pres- Process: Nursing Process, Assessment; Cognitive Level:
sure will cause systemic edema. Application; Client Need/Subneed: Safe, Effective Care
TEST-TAKING TIP: Visualize: if the right side of the heart Environment/Safety and Infection Control
cannot pump the blood, where would the effects of the back- 64. CORRECT ANSWER: 2. Answer 1 is incorrect because
pressure be noted? the overriding parasympathetic stimulation causes
Content Area: Adult Health, Cardiology; Integrated
bradycardia, not tachycardia (pulse = 120 here). Answer 2
Process: Nursing Process, Assessment; Cognitive Level:
is correct because the hypotension and tachycardia are
Comprehension; Client Need/Subneed: Physiological Integrity/ classic signs of hypovolemic shock. Answer 3 is incorrect
Physiological Adaptation/Pathophysiology because the temperature would be higher in septic shock,
61. CORRECT ANSWER: 2. Answer 1 is incorrect because usually greater than 38C. Answer 4 is incorrect because in
ANSWERS

cool, clammy skin occurs with hypovolemia and decreased cardiogenic shock the pulse would not be increased because
perfusion. Answer 2 is correct because there is an increase the contractility of the heart is ineffective.
in the body temperature, and vasodilation would occur. TEST-TAKING TIP: Each type of shock has a unique charac-
Answer 3 is incorrect because a drop in the systolic pressure teristic. Know the clinical indications.
occurs with hypovolemic shock, not septic shock. Answer 4 is Content Area: Adult Health, Cardiovascular; Integrated
incorrect because DIC is a hypercoagulable condition that Process: Nursing Process, Analysis; Cognitive Level: Analysis;
would be a late complication of poor perfusion. Client Need/Subneed: Physiological Integrity/Reduction of Risk
TEST-TAKING TIP: Know the classic difference with septic Potential/Vital Signs Changes/Abnormalities
shock: fevertherefore, the client is warm and flushed. 65. CORRECT ANSWER: 3. Answer 1 is incorrect because
Content Area: Adult Health, Cardiovascular; Integrated
these findings would be consistent with hypocalcemia
Process: Nursing Process, Assessment; Cognitive Level:
(low calcium). Answer 2 is incorrect because these findings
Application; Client Need/Subneed: Physiological Integrity/ would be consistent with dehydration and metabolic acidosis.
Physiological Adaptation/Pathophysiology Answer 3 is correct because potassium affects normal
62. CORRECT ANSWER: 4. Answer 1 is incorrect because a muscle function. Answer 4 is incorrect because these find-
change in LOC from hypovolemia would have occurred ings would be consistent with hypoxia and poor perfusion
earlier. Answer 2 is incorrect because this may have been an possibly related to cardiac failure.
early indication of a GI problem. Answer 3 is incorrect TEST-TAKING TIP: Look for the effects of low potassium on
because the respiratory change may occur as a result of muscles, including the heart.
2164_Ch06_347-578 29/03/12 12:31 PM Page 571

Answers/Rationales/Tips 571
Content Area: Adult Health, Fluid and Electrolyte Imbalances; Content Area: Adult Health, Gastrointestinal; Integrated
Integrated Process: Nursing Process, Analysis; Cognitive Process: Nursing Process, Assessment; Cognitive Level:
Level: Application; Client Need/Subneed: Physiological Application; Client Need/Subneed: Health Promotion and
Integrity/Physiological Adaptation/Fluid and Electrolyte Maintenance/Techniques of Physical Assessment
Imbalances
69. CORRECT ANSWERS: 2, 3, 4, 5. Answer 1 is incorrect
66. CORRECT ANSWER: 1. Answer 1 is correct because the because gatching the bed in the area of the knees reduces
respiratory rate would increase as respiratory function venous return. Answer 2 is correct because these exercises
improved. Respiratory acidosis occurs from slow and prevent venous pooling. Answer 3 is correct because use of
inadequate ventilation. Answer 2 is incorrect because an such devices prevents venous pooling. Answer 4 is correct
increase in creatinine and poor renal function would lead because dehydration increases RBC concentration, resulting
to metabolic acidosis. Answer 3 is incorrect because a slow in sluggish circulation. Answer 5 is correct because this will
respiratory rate causes CO2 to be retained, which will lead increase venous return and reduce venous stasis from
to respiratory acidosis. Answer 4 is incorrect because bicar- dependent positioning. Answer 6 is incorrect because heat
bonate increases with respiratory acidosis. will cause vasodilation and venous pooling.
TEST-TAKING TIP: Recall that a compensatory response to a TEST-TAKING TIP: Look for strategies to prevent venous stasis
metabolic problem is a change in respiratory rate. The rate and improve venous return.
should slow as CO2 returns to normal. Content Area: Adult Health, Vascular; Integrated Process:
Content Area: Adult Health, Fluid and Electrolyte Imbalances; Nursing Process, Implementation; Cognitive Level: Application;
Integrated Process: Nursing Process, Analysis; Cognitive Level: Client Need/Subneed: Physiological Integrity/Reduction of Risk
Application; Client Need/Subneed: Physiological Integrity/ Potential/Therapeutic Procedures
Physiological Adaptation/Pathophysiology
70. CORRECT ANSWERS: 1, 3, 5. Answer 1 is correct
67. CORRECT ANSWERS: 1, 2, 4. Answer 1 is correct because a local anesthetic will be used to minimize pain
because ammonia is a by-product of protein breakdown to the puncture site. Answer 2 is incorrect because the rea-
by the intestinal flora. Blood is a source of protein. son for the thoracentesis is fluid in the pleural space, not in
Answer 2 is correct because ammonia is a by-product the lung. Answer 3 is correct because this position drops
of protein breakdown. Reducing dietary protein will the diaphragm and allows for easier access to the pleural
decrease the ammonia level. Answer 3 is incorrect because space. Answer 4 is incorrect because upright and leaning
diarrhea actually decreases ammonia and may be a desired forward is the optimal position. Answer 5 is correct because
therapeutic effect. Answer 4 is correct because protein is the client needs to have been informed of the risks
broken down to form ammonia by intestinal bacteria. before the procedure is started. Answer 6 is incorrect
Sterilizing the bowel with an antibiotic will decrease because there is some discomfort, although use of the local
the ammonia level. Answer 5 is incorrect because the anesthetic reduces the pain.
ammonia is produced in the GI system by the breakdown TEST-TAKING TIP: Think client safety before, during, and
of protein. Ascites is in the peritoneum, not in the after the procedure.
intestine. Content Area: Adult Health; Integrated Process: Nursing
TEST-TAKING TIP: Review how ammonia forms in the Process, Implementation; Cognitive Level: Application;

ANSWERS
presence of blood and bacteria. Client Need/Subneed: Physiological Integrity/Reduction of
Content Area: Adult Health, Gastrointestinal; Integrated Risk Potential/Therapeutic Procedures
Process: Nursing Process, Implementation; Cognitive Level:
71. CORRECT ANSWER: 1. Answer 1 is correct because
Application; Client Need/Subneed: Physiological Integrity/
the PT is the specific test to determine the effectiveness
Physiological Adaptation/Illness Management
of Coumadin therapy. Answer 2 is incorrect because the
68. CORRECT ANSWER: 1. Answer 1 is correct because Lee-White clotting time monitors heparin therapy, but has
initially peristalsis will increase in the ascending been replaced by other tests. Answer 3 is incorrect because
colon (right lower quadrant [RLQ]) in an attempt to the PTT is the appropriate test to determine the effective-
clear the blockage. There will be no peristalsis distal to ness of heparin therapy. Answer 4 is incorrect because
the obstruction. Answer 2 is incorrect because peristalsis FCT is a measure of thrombin activity, and heparin prolongs
will increase proximal (RLQ) to the obstruction. Answer 3 the time.
is incorrect because initially peristalsis will increase proximal TEST-TAKING TIP: Know the differences between coumadin
(RLQ) to the obstruction. Answer 4 is incorrect because and heparin with regard to indications of effectiveness and
peristalsis will be present initially proximal (left lower monitoring of therapeutic level.
quadrant [LLQ]) to the obstruction. Content Area: Adult Health, Vascular; Integrated Process:
TEST-TAKING TIP: Visualize the colon. Initially bowel sounds Nursing Process, Analysis; Cognitive Level: Application;
increase proximal to an obstruction and are absent distal to the Client Need/Subneed: Physiological Integrity/Reduction of
obstruction. Risk Potential/Laboratory Values
2164_Ch06_347-578 29/03/12 12:31 PM Page 572

572 chapter 6 Physiological Integrity

72. CORRECT ANSWER: 2. Answer 1 is incorrect because a decrease renal perfusion and cause buildup of BUN
possible cause, such as dehydration or influenza, while and creatinine. First, the client needs rehydration to
potentially a concern, is not as serious as the potential of correct prerenal failure. Then, an assessment can be
heart failure. Answer 2 is correct because these are signs made of renal function (which can be decreased in dia-
and symptoms associated with heart failure. Answer 3 is betes). Answer 2 is incorrect because fluid restriction would
incorrect because a productive cough alone without a high exacerbate the dehydration. Answer 3 is incorrect because
fever is not the first priority. Answer 4 is incorrect because dialysis is not required for creatinine levels of 2.7. Answer 4
palpitations without chest pain would not be a priority. is incorrect because a bladder catheter would not help man-
TEST-TAKING TIP: Think ABCs as priority and select B age the problem, only help assess the level of dehydration
breathing. Look for the client who is most unstable, or the and the response to treatment with hourly urine output
clinical indications that signal a potentially urgent need. measurements.
Content Area: Adult Health, Triage; Integrated Process: Nursing TEST-TAKING TIP: Remember that the client with DKA
Process, Analysis; Cognitive Level: Analysis; Client Need/ is severely dehydratedflushed and warm. When two
Subneed: Safe and Effective Care Environment/Management options are contradictory (Answers 1 and 2), often one of
of Care/Establishing Priorities them is correct.
Content Area: Adult Health, Endocrine; Integrated Process:
73. CORRECT ANSWER: 3. Answer 1 is incorrect because Nursing Process, Planning; Cognitive Level: Application;
keeping the airway open is more important than listening Client Need/Subneed: Physiological Integrity/Physiological
to breath sounds. Answer 2 is incorrect because keeping the Adaptation/Illness Management
airway open is the priority, not giving oxygen. Answer 3 is
correct because, until fully awake, the tongue may 76. CORRECT ANSWER: 1. Answer 1 is correct because
obstruct the airway. Keeping the airway in place until total bilirubin comes from normal turnover of RBCs in
the client pushes it out prevents obstruction. Answer 4 is the body. Excess destruction or inability of the liver to
incorrect because keeping the airway open is the priority. process waste products may cause an increase in biliru-
Side-lying may prevent aspiration in the client who may bin, which may turn the clients skin yellow. This is a
vomit, but it may not be possible, depending on type of normal total bilirubin. Answer 2 is incorrect because this is
surgery. a normal bilirubin level, unrelated to presence of blood in
TEST-TAKING TIP: Think: ABCs and select airway as the urine. Answer 3 is incorrect because this is a normal
the action that is most important. The best choice will increase bilirubin level, unrelated to blood in the stool. Answer 4 is
the effectiveness of the other options. incorrect because high bilirubin levels may cause a yellow
Content Area: Adult Health, Respiratory; Integrated Process: coloration to the skin, correctly seen in the sclera of the
Nursing Process, Implementation; Cognitive Level: Application; eye. This total bilirubin is normal.
Client Need/Subneed: Physiological Integrity/Reduction of Risk TEST-TAKING TIP: Three of the options imply that the
Potential/Potential for Complications from Surgical Procedures bilirubin level is not normal. Select the one option that states
and Health Alterations it is a normal value. Memorize the ranges of common diag-
nostic laboratory tests.
74. CORRECT ANSWER: 1. Answer 1 is correct because Content Area: Adult Health, Hematological; Integrated Process:
bilateral adventitious sounds are heard early in left-sided
ANSWERS

Nursing Process, Implementation; Cognitive Level: Application;


failure. The ejection fraction is below the normal for a Client Need/Subneed: Physiological Integrity/Reduction of Risk
health heart (60% to 70%). Answer 2 is incorrect because Potential/Laboratory Values
the low O2 saturation is related to pulmonary congestion and
heart failure. A flat position may aggravate the clients condi- 77. CORRECT ANSWER: 2. Answer 1 is incorrect because
tion. Answer 3 is incorrect because crackles due to heart clients on insulin drips often require supplemental potassium
failure would not clear with deep breathing and coughing. to replace that carried into the cells by insulin. A K+ of 4.4 is
Answer 4 is incorrect because the ejection fraction is on the on the high end of the normal and would not need to be
low side of normal (60% to 70% for a healthy heart). replaced. Answer 2 is correct because this client is still aci-
Fatigue would be more common. This degree of activity dotic, which means the cells are burning fats and releasing
intolerance would increase as heart failure progressed. ketones. (Anion gap is 130 100 15 = 15, which is high;
TEST-TAKING TIP: Know the average ejection fraction bicarbonate is low at 15). Insulin is still required. With
(60% to 70%). Select a finding that would be consistent with blood sugar at 100, you will also need to give sugar. Answer
mild heart failure. 3 is incorrect because sodium level needs to correct slowly as
Content Area: Adult Health, Cardiology; Integrated Process: blood sugar is lowered from the high admission level. At 130, a
Nursing Process, Analysis; Cognitive Level: Analysis; correction has almost occurred. Giving sodium tablets to
Client Need/Subneed: Physiological Integrity/Physiological increase sodium too quickly will pull fluid out of the cells, fur-
Adaptation/Pathophysiology ther dehydrating the cells. Salt may be given to correct hyper-
tension, but is not required in this client. Answer 4 is incorrect
75. CORRECT ANSWER: 1. Answer 1 is correct because because clients with DKA need fluids, not fluid restriction. The
severe dehydration is often seen in DKA. This can
2164_Ch06_347-578 29/03/12 12:31 PM Page 573

Answers/Rationales/Tips 573
high blood sugar creates an osmotic loss of intracellular fluids, startling. Answer 6 is correct because the client has no
and when sugar is spilled into the urine, there is osmotic fluid idea where over there is. Describe the location.
loss from the body. TEST-TAKING TIP: Choose options that optimize the reliance
TEST-TAKING TIP: Know the effects of high blood sugar levels on other senses of the client who is visually impaired, such as
on fluid and electrolyte levels, as well as the shifts that occur auditory (Answer 4), and avoid using vague phrases about
with the administration of insulin. locations that describe what the client cannot see (Answer 6).
Content Area: Adult Health, Endocrine; Integrated Process: Content Area: Adult Health, Sensory; Integrated Process:
Nursing Process, Implementation; Cognitive Level: Analysis; Communication and Documentation; Cognitive Level:
Client Need/Subneed: Physiological Integrity/Reduction of Application; Client Need: Psychosocial Integrity/Sensory/
Risk Potential/Laboratory Values Perceptual Alterations
78. CORRECT ANSWER: 3. Answer 1 is incorrect because 81. CORRECT ANSWER: 1. Answer 1 is correct because
clients with diabetes should manage blood sugar at a level exercise in the absence of weight reduction is a significant
that would result in an A1c of less than 7%. An intervention factor in reducing blood pressure. Answer 2 is incorrect
is required to help this client better manage diabetes. because support groups may not be beneficial to all clients.
Answer 2 is incorrect because the A1c would not change Answer 3 is incorrect because avoiding tobacco doesnt reduce
acutely with resolution of DKA. Answer 3 is correct because blood pressure but reduces the risk of a cardiac event. Answer 4
the A1c gives a number that reflects the average blood is incorrect because increasing fruits and vegetables alone will
sugar over the last 3 months. Clients with diabetes are not control blood pressure.
told to try to keep the A1c less than 7% (sometimes less TEST-TAKING TIP: Look for an option that is known to
than 6.5%). This high level, which is getting worse, manage weight and stressexercise.
means that the client needs better management of the Content Area: Adult Health, Cardiovascular; Integrated
diabetes. Client teaching is the correct way to help a Process: Teaching and Learning; Cognitive Level: Application;
client manage this chronic disease. Answer 4 is incorrect Client Need/Subneed: Health Promotion and Maintenance/
because this clients diabetes is out of control. First, an Health Promotion Programs
assessment must be done to determine why the blood sugars
82. CORRECT ANSWER: 4. Answer 1 is incorrect because
have been chronically high. Longer-acting insulin may or
this tests the corneal reflex (blink). Answer 2 is incorrect
may not be part of a reasonable intervention for this client.
because this action tests extraocular movements (cranial nerve
TEST-TAKING TIP: Know that the HgbA1c level is too high,
[CN] IV). Answer 3 is incorrect because this action tests
needing teaching.
the optic nerve (CN II). Answer 4 is correct because the nor-
Content Area: Adult Health, Endocrine; Integrated Process:
mal response of the pupil to light is constriction, if CN III
Nursing Process, Analysis; Cognitive Level: Analysis;
is normal.
Client Need/Subneed: Physiological Integrity/Reduction of
TEST-TAKING TIP: Review the correct assessment technique
Risk Potential/Laboratory Values
for each cranial nerve. Assessing CN III is part of routine
79. CORRECT ANSWER: 1. Answer 1 is correct because neurological checks.
hyponatremia can precipitate seizures. Answer 2 is incorrect Content Area: Adult Health, Sensory; Integrated Process:
because the moderately high blood sugar associated with an Nursing Process, Implementation; Cognitive Level: Application;

ANSWERS
A1c of 8% would not cause seizures. Answer 3 is incorrect Client Need/Subneed: Health Promotion and Maintenance/
because anion gaps less than 12 are normal. Answer 4 is incor- Techniques of Physical Assessment
rect because neuropathies of peripheral neurons would not pre-
83. CORRECT ANSWER: 3. Answer 1 is incorrect because this
cipitate seizures.
is seen with tetany (hypocalcemia). Answer 2 is incorrect
TEST-TAKING TIP: Look for the most abnormal conditiona
because this occurs from ischemia-induced carpal spasm from
low serum sodium.
hypocalcemia. Answer 3 is correct because hypercalcemia
Content Area: Adult Health, Neurological; Integrated
reduces neuromuscular excitability, resulting in muscle
Process: Nursing Process, Analysis; Cognitive Level: Analysis;
weakness and hyporeflexia. Answer 4 is incorrect because the
Client Need/Subneed: Physiological Integrity/Reduction of
Babinski reflex is a test of upper motor neuron function.
Risk Potential/Laboratory Values
TEST-TAKING TIP: Eliminate the options that are clinical
80. CORRECT ANSWERS: 4, 6. Answer 1 is incorrect because manifestations of hypocalcemia (Answers 1 and 2) and the
the client is not deaf. Speak normally. Answer 2 is incorrect option that is not manifested by the electrolyte imbalance
because these words are appropriate. Answer 3 is incorrect (Answer 4) noted in the stem of this question.
because, although senses of a client with visual impairment Content Area: Adult Health, Fluid and Electrolyte Imbalances;
may be extrasensitive, no voice adjustment is needed. Speak Integrated Process: Nursing Process, Assessment; Cognitive
normally. Answer 4 is correct because the client hears bet- Level: Comprehension; Client Need/Subneed: Physiological
ter when spoken to directly. Answer 5 is incorrect because Integrity/Physiological Adaptation/Fluid and Electrolyte
touching a client before speaking to the client will be Imbalances
2164_Ch06_347-578 29/03/12 12:31 PM Page 574

574 chapter 6 Physiological Integrity

84. CORRECT ANSWER: 4. Answer 1 is incorrect because bicarbonate might temporarily fix pH, but it does not fix
loss of appetite may occur in children without severe illness. the problem of dehydration. Answer 2 is correct because
Answer 2 is incorrect because swine flu may present without clients who are admitted with high blood sugar levels
a fever. Children also frequently have fevers without any are very dehydrated and need IV normal saline. High
underlying pathology. Answer 3 is incorrect because vomit- blood sugar levels dehydrate cells, spilling sugar in the
ing would need to be persistent and lead to dehydration to be kidneys and causing obligate water loss. Answer 3 is
a concern. Answer 4 is correct because a change in alert- incorrect because IV albumin is an expensive and unneces-
ness or responsiveness is an emergency warning sign in sary way to correct dehydration. This client needs sodium
children and needs urgent attention. and water. Normal saline IV is the correct action. Answer 4
TEST-TAKING TIP: Choose a neurological sign, rather than is incorrect because, although the client needs insulin,
gastrointestinal (Answers 1 and 3). it is inappropriate to give it subcutaneously. Clients who
Content Area: Child Health, Infectious Disease; Integrated are dehydrated have poor absorption from this tissue.
Process: Teaching and Learning; Cognitive Level: Analysis; These clients need a continuous intravenous drip of
Client Need/Subneed: Safe, Effective Care Environment/ insulin.
Safety and Infection Control TEST-TAKING TIP: This is a priority question. To decide
between two options (fluids or bicarbonate), remember that
85. CORRECT ANSWER: 4. Answer 1 is incorrect because the correcting dehydration (with fluids) will improve acidosis.
first priority is maintaining circulation, not a quiet environ- Content Area: Adult Health, Endocrine; Integrated Process:
ment. Answer 2 is incorrect because the first priority is main- Nursing Process, Analysis; Cognitive Level: Analysis;
taining adequate perfusion, not avoiding exertion. Answer 3 is Client Need/Subneed: Physiological Integrity/Pharmacological
incorrect because the inadequate perfusion must be corrected and Parenteral Therapies/Medication Administration
first. Answer 4 is correct because the client is at risk for
shock. 88. CORRECT ANSWER: 2. Answer 1 is incorrect because
TEST-TAKING TIP: When all options are correct, look for the sodium will rise another 1 or 2 points as the blood sugar
the choice that responds to a crisis and will produce an returns to normal. This will not require aggressive treatment;
immediate responsefluid. 3% NaC1 at 200 mL/hr would raise sodium levels very
Content Area: Adult Health, Endocrine; Integrated Process: rapidly, and could cause seizures. Answer 2 is correct
Nursing Process, Implementation; Cognitive Level: Analysis; because the anion gap is closed and the glucose is close
Client Need/Subneed: Physiological Integrity/Physiological to normal. The client can be switched to subcutaneous
Adaptation/Alterations in Body Systems insulin and the insulin drip discontinued. An anion gap
occurs in metabolic acidosis when there is an excessive
86. CORRECT ANSWER: 4. Answer 1 is incorrect because accumulation of fixed acid. Anion gap is determined by
cerebral perfusion is determined by the pressure of the the balance between the anions (Na+ and K+) and the
blood flowing into the brain and the resistance to inflow cre- cations (Cl and HCO3). Answer 3 is incorrect because
ated by intracerebral pressure. Answer 2 is incorrect because the potassium level might be treated if the client were to
cerebral vasodilation is largely determined by PaCO2. It is remain on an insulin drip. The blood sugar of 170 without
important to ensure good ventilation in clients who have an elevated anion gap indicates that the insulin drip can be
had trauma, but this is unrelated to serum sodium. Answer 3
ANSWERS

discontinued. Answer 4 is incorrect because, when the dex-


is incorrect because lowering the seizure threshold would trose is metabolized, 5% dextrose in one-quarter normal
make the client more at risk for seizures, which is never saline (D5 1/4NS) becomes just 1/4NS. This hypotonic solu-
desirable. Low serum sodium levels can increase seizure risk. tion will further dilute the sodium.
Answer 4 is correct because osmolarity is largely deter-
TEST-TAKING TIP: First ask: Are the values within normal
mined by serum sodium. If the sodium level is low, water limits? Then, look at the impact of each option on the labo-
will move from the bloodstream into the cells. The client ratory values. Select the option that is different from the
is already at risk for cerebral edema because of the injury. three others (discontinue versus three gives).
The swelling could result in a dangerous increase in Content Area: Adult Health, Endocrine; Integrated Process:
intracerebral pressure. Nursing Process, Implementation; Cognitive Level: Analysis;
TEST-TAKING TIP: Remember: H2O follows sodium. The Client Need/Subneed: Physiological Integrity/Reduction of
desired clinical outcome is to reduce or prevent cerebral edema. Risk Potential/Laboratory Values
Content Area: Adult Health, Fluid and Electrolyte Imbalances;
Integrated Process: Nursing Process, Analysis; Cognitive Level: 89. CORRECT ANSWER: 3. Answer 1 is incorrect because
Application; Client Need/Subneed: Physiological Integrity/ glucocorticoids facilitate calcium excretion. Answer 2 is
Reduction of Risk Potential/Laboratory Values incorrect because ketones in the urine indicate fat is being
burned. This is not related to calcium level. Answer 3 is cor-
87. CORRECT ANSWER: 2. Answer 1 is incorrect because rect because most calcium is bound to plasma proteins
the acidosis will correct with insulin once glucose is available such as albumin. As long as the physiologically important
to the cell so that it does not need to burn fats. Giving
2164_Ch06_347-578 29/03/12 12:31 PM Page 575

Answers/Rationales/Tips 575
portion (the ionized, nonbound calcium) is normal, fluid levels. Fluid restriction will help treat the hypona-
low total calcium does not need to be treated. Answer 4 tremia. Answer 2 is incorrect because generally low serum
is incorrect because calcium levels would fall with adminis- sodium results from excessive water intake or retention.
tration of phosphate. It is correctly treated with water restriction. Answer 3 is
TEST-TAKING TIP: Total serum calcium alone does not pro- incorrect because this client does not need fluids. Answer 4
vide sufficient data. Look for an option (check) that collects is incorrect because generally low serum sodium results
more data before treatment. Eliminate the option with check indicate excessive water intake or retention. It is correctly
that is not related to calcium (Answer 2). treated with water restriction, not fluids by IV.
Content Area: Adult Health, Fluid and Electrolyte Imbalances; TEST-TAKING TIP: Interpret the data. Recognize the signs of
Integrated Process: Nursing Process, Implementation; fluid excess. Three of the options add sodium; only one limits
Cognitive Level: Application; Client Need/Subneed: Physiological fluid.
Integrity/Physiological Adaptation/Fluid and Electrolyte Content Area: Adult Health, Fluid and Electrolyte Imbalances;
Imbalances Integrated Process: Nursing Process, Implementation;
Cognitive Level: Analysis; Client Need/Subneed: Physiological
90. CORRECT ANSWER: 2. Answer 1 is incorrect because the Integrity/Physiological Adaptation/Fluid and Electrolyte
parathyroid is responsible for calcium absorption. High
Imbalances
calcium levels raise BP, but a postoperative complication is
hypocalcemia. Answer 2 is correct because the parathyroid is 93. CORRECT ANSWER: 1. Answer 1 is correct because
responsible for calcium absorption. Numbness around the early ambulation is the most effective way to reduce the
mouth is an early sign of hypocalcemia. Answer 3 is incorrect risk of DVT. Answer 2 is incorrect because heparin reduces
because the parathyroid is responsible for calcium absorption. the risk of DVT in clients who will be on bedrest after sur-
Alteration in urine output is not characteristic of this disorder. gery. There is no need for this client to be on bedrest.
Answer 4 is incorrect because the parathyroid is responsible for Answer 3 is incorrect because alternating compression leg
calcium absorption. Muscle spasms, stiffness, and tetany are wraps reduce the risk of DVT in clients who will be on
signs of hypocalcemia, not muscle weakness. bedrest after surgery. There is no need for this client to be
TEST-TAKING TIP: Know the effect of the parathyroid on on bedrest. Answer 4 is incorrect because DVTs generally
calciumhypocalcemia. form in the legs of the client who is immobilized, and would
Content Area: Adult Health, Endocrine; Integrated not be affected by wrist surgery. In addition, the wrist
Process: Nursing Process, Assessment; Cognitive Level: will need to be immobilized for a period of time after this
Application; Client Need/Subneed: Physiological Integrity/ surgery, not exercised.
Reduction of Risk Potential/Potential for Complications from TEST-TAKING TIP: K.I.S.Keep It Simpleif the client is
Surgical Procedures and Health Alterations not on bedrest, ambulation is the best action.
Content Area: Adult Health, Vascular; Integrated Process:
91. CORRECT ANSWER: 2. Answer 1 is incorrect because 5% Nursing Process, Planning; Cognitive Level: Application;
dextrose in water (D5W) is not an appropriate fluid for volume
Client Need/Subneed: Physiological Integrity/Reduction of
resuscitation. When the sugar is metabolized, all that remains
Risk Potential
is water and almost two thirds of the infused volume will
transfer into cells, which will not increase circulating volume. 94. CORRECT ANSWER: 3. Answer 1 is incorrect because,

ANSWERS
Answer 2 is correct because this client is in shock and although the oxygen saturation is 100%, shivering does
requires rapid volume infusion with IV normal saline (NS). increase oxygen consumption and supplemental O2 will be
Answer 3 is incorrect because, while it may be useful to have a needed, not removed. Answer 2 is incorrect because warm
second large-bore IV, it does not take priority over starting IV blankets need to be applied to treat the complaints and
fluids with normal saline. Answer 4 is incorrect because, shivering regardless of the clients temperature. Answer 3 is
regardless of the anion gap, the client will need normal saline to correct because shivering increases oxygen consumption.
raise the BP. Hypothermia associated with the perioperative period
TEST-TAKING TIP: This is a priority question. The client may cause arrhythmia, clotting problems, and impaired
shows signs of shock. Without additional data, choose an wound healing. Answer 4 is incorrect because the pain level
isotonic solution. is low at 2/10 and does not require treatment.
Content Area: Adult Health, Cardiovascular; Integrated Process: TEST-TAKING TIP: Think patient comfort. Care for the client
Nursing Process, Implementation; Cognitive Level: Analysis; first, and then check the temperature.
Client Need/Subneed: Physiological Integrity/Physiological Content Area: Geriatrics, Musculoskeletal; Integrated Process:
Adaptation/Medical Emergencies Nursing Process, Implementation; Cognitive Level: Application;
Client Need/Subneed: Physiological Integrity/Reduction of Risk
92. CORRECT ANSWER: 1. Answer 1 is correct because Potential/Potential for Complications from Surgical Procedures
generally low serum sodium results from excessive
and Health Alterations
water intake or retention. This client has adequate
2164_Ch06_347-578 29/03/12 12:31 PM Page 576

576 chapter 6 Physiological Integrity

95. CORRECT ANSWER: 4. Answer 1 is incorrect because 98. CORRECT ANSWER: 4. Answer 1 is incorrect because
rapid heart rate is more commonly associated with dehydra- blood and mucus in the stools are not characteristic of
tion or hypovolemia, not fluid overload. Answer 2 is incorrect gallstones. Answer 2 is incorrect because gallstones can
because pulmonary crackles that clear with deep breathing cause pain that is usually located in the epigastrum or the
are an indication of atelectasis, not heart failure from volume RUQ. It may be precipitated by eating, not relieved by
overload. Answer 3 is incorrect because concentrated urine is eating. Answer 3 is incorrect because low hemoglobin is
more often seen in dehydration as the body tries to conserve not a typical feature of gallstones. Answer 4 is correct
fluid, not in fluid overload. Answer 4 is correct because because gallstones are asymptomatic in over 80% of
excess fluid in the lung is decreasing oxygen transfer clients.
into the tissues. TEST-TAKING TIP: Look for a hint in the stem. The
TEST-TAKING TIP: Look for an option that occurs when the stem says uncomplicated. Three of the options are
heart is unable to effectively pump the circulating volume. complications.
Content Area: Adult Health, Fluid and Electrolyte Content Area: Adult Health, Gastrointestinal; Integrated
Imbalances; Integrated Process: Nursing Process, Analysis; Process: Teaching and Learning; Cognitive Level: Application;
Cognitive Level: Application; Client Need/Subneed: Client Need/Subneed: Physiological Integrity/Physiological
Physiological Integrity/Physiological Adaptation/ Adaptation/Pathophysiology
Fluid and Electrolyte Imbalances
99. CORRECT ANSWER: 2. Answer 1 is incorrect because
96. CORRECT ANSWER: 1. Answer 1 is correct because any infection growing from surgery would not be showing
padding is very important before surgery begins. During up on postoperative day 1. This fever is from atelectasis.
surgery, the client will be immobilized on a hard surface. Answer 2 is correct because low-grade fever on postopera-
Answer 2 is incorrect because turning is an excellent preventa- tive day 1 is almost always an indication of atelectasis.
tive strategy in the unit, but not during the surgery. Answer 3 This low O2 saturation supports this. Have the client take
is incorrect because range of motion prevents contractures, not deep breaths on the incentive spirometer to open up the
pressure sores. In addition, ROM is not done during surgery. alveoli. Answer 3 is incorrect because the fever and low O2
Answer 4 is incorrect because an alternating pressure mattress Sat are most likely from atelectasis, not dehydration.
makes the clients body move. This is not reasonable during Dehydration might be reflected in the rapid heart rate,
surgery. but it would be accompanied by a lower BP. Answer 4 is
TEST-TAKING TIP: Choose the one option that is different incorrect because, when the pain level is 2/10, medications
than the others (before surgery). Three of the options will move are not indicated.
the client during surgery, which is not feasible. TEST-TAKING TIP: Look for the abnormal findings. The first
Content Area: Adult Health, Integumentary; Integrated Process: action to take with a lower than normal O2 saturation is deep
Nursing Process, Implementation; Cognitive Level: Application; breathing and coughing.
Client Need/Subneed: Physiological Integrity/Reduction of Content Area: Adult Health, Respiratory; Integrated Process:
Risk Potential/Potential for Complications from Surgical Nursing Process, Implementation; Cognitive Level: Analysis;
Procedures and Health Alterations Client Need/Subneed: Physiological Integrity/Reduction of
Risk Potential/Potential for Complications from Surgical
97. CORRECT ANSWER: 1. Answer 1 is correct because
ANSWERS

Procedures and Health Alterations


an elevated blood sugar (over 200) is associated with an
increased risk of infection and poor wound healing. 100. CORRECT ANSWER: 2. Answer 1 is incorrect because
Answer 2 is incorrect because clients are not placed in the absence of bowel sounds is expected on the first postoper-
contact isolation to prevent infection; it is done to prevent ative day. No action is required. Answer 2 is correct because
transmission of a known infection to other clients. Answer 3 is paralytic ileus is expected after major abdominal surgery.
incorrect because, to minimize the risk of bacteria develop- The absence should be recorded in the medical record.
ing resistance to antibiotics, preoperative doses are given no Answer 3 is incorrect because the absence of bowel sounds is
more than 2 hours before surgery. Answer 4 is incorrect expected. It does not indicate impaction. Answer 4 is incorrect
because glucocorticoids increase the risk of infection by because the absence of bowels sounds is expected. Massage is
masking symptoms. not indicated.
TEST-TAKING TIP: Prevention is the goal. Choose the answer TEST-TAKING TIP: The key word is expected. Select an
that monitors a key serum level. Diabetes in the stem goes option that is realistic 24 hours after having been NPO
with blood sugar in the correct option. Look for an option (no stool will have formed in this short time period) for
that reduces the susceptibility of the client to an infection. abdominal surgery (and massage would be painful).
Content Area: Adult Health, Integumentary; Integrated Process: Content Area: Adult Health, Gastrointestinal; Integrated
Nursing Process, Implementation; Cognitive Level: Application; Process: Nursing Process, Implementation; Cognitive
Client Need/Subneed: Physiological Integrity/Reduction of Level: Application; Client Need/Subneed: Physiological
Risk Potential/Potential for Complications from Surgical Integrity/Physiological Adaptation/Alterations in Body
Procedures and Health Alterations Systems
2164_Ch06_347-578 29/03/12 12:31 PM Page 577

Answers/Rationales/Tips 577
101. CORRECT ANSWER: 3. Answer 1 is incorrect because TEST-TAKING TIP: Think of the Latin meaning of the
African Americans have a lower risk of gallstones than do problemdys means difficulty and phagia means to eat
whites or Hispanics. Answer 2 is incorrect because women feed through tube.
are two to three times more likely to develop gallstones than Content Area: Adult Health, Cardiovascular; Integrated
men. Answer 3 is correct because rapid weight loss is a Process: Nursing Process, Implementation; Cognitive Level:
risk factor for gallstones due to bile that becomes super- Application; Client Need/Subneed: Physiological Integrity/
saturated with cholesterol. Also, frequent changes in Physiological Adaptation/Alterations in Body Systems
weight increases the risk. Answer 4 is incorrect because 104. CORRECT ANSWER: 1. Answer 1 is correct because,
coffee consumption has been associated with a lower risk of in aphasia, clients have difficulty understanding and/or
gallstones. using language. Nonverbal communication strategies
TEST-TAKING TIP: Remember the 4 Fs of increased risk may help this. Answer 2 is incorrect because, in aphasia,
for gallbladder diseasefemale, fair, fat, and forty. clients have difficulty understanding and/or using language.
Content Area: Adult Health, Gastrointestinal; Integrated
Making the communication more complex with additional
Process: Teaching and Learning; Cognitive Level: Analysis;
words is unlikely to help. Answer 3 is incorrect because, in
Client Need/Subneed: Physiological Integrity/Physiological
aphasia, clients have difficulty understanding and/or using
Adaptation/Pathophysiology language. They are not hard of hearing. Answer 4 is incorrect
102. CORRECT ANSWER: 2. Answer 1 is incorrect because because dysphagia refers to difficulty swallowing. Dysphasia
a stroke in the right brain would cause paralysis or weakness refers to difficulty speaking. Aphasia is absence or impair-
on the left side. It is not necessary to perform range of ment of ability to communicate through speech, writing or
motion on the unaffected extremity. Answer 2 is correct signs.
because a stroke in the right brain would cause paralysis TEST-TAKING TIP: Define the wordnot speaking. The
or weakness on the left side. Getting out of bed is easiest question asks for a way to facilitate communication for the
on the strong side. Answer 3 is incorrect because the client client. Select the answers with communication.
should be mobile as soon as possible. It is likely that only Content Area: Adult Health, Sensory; Integrated Process:
the left leg will be paralyzed or weak, and this may be a Nursing Process, Implementation; Cognitive Level:
permanent condition. A client will need to learn mobility Comprehension; Client Need/Subneed: Physiological
techniques despite the weakness. Answer 4 is incorrect Integrity/Physiological Adaptation/Alterations in Body
because it is important to maintain range of motion for Systems
the joints on the affected side, rather than immobilize the 105. CORRECT ANSWER: 3. Answer 1 is incorrect because
left side. neither standard nor droplet precautions require regular use
TEST-TAKING TIP: Right brain attack leads to left-sided of the N95 respirator. Answer 2 is incorrect because the
weakness/paralysis. Nursing actions should focus on preventing N95, if used, is not shared and is recommended for single
problems of immobility. use. Answer 3 is correct because the N95 is recommended
Content Area: Adult Health, Cardiovascular; Integrated Process:
when treatment may produce the potential of sputum
Nursing Process, Planning; Cognitive Level: Application; spray, such as intubation or suctioning. Only a medical-
Client Need/Subneed: Physiological Integrity/Basic Care and
surgical mask is necessary when entering the client room.

ANSWERS
Comfort/Mobility/Immobility Some hospitals may provide all direct-care providers with
103. CORRECT ANSWER: 4. Answer 1 is incorrect because facemasks or respirators. Answer 4 is incorrect because the
dysphagia refers to difficulty swallowing. Dysphasia (impaired N95 is intended for use when there is a potential for an
speech) may be helped by eliminating distractions when giving aerosol spray of sputum.
directions, not dysphagia. Answer 2 is incorrect because dys- TEST-TAKING TIP: Key point: potential for aerosol spray of
phagia refers to difficulty swallowing. Keeping the head elevat- sputum.
ed might help reduce risk of aspiration. Answer 3 is incorrect Content Area: Adult Health, Infectious Disease; Integrated
because thin liquids are more difficult to swallow than thick liq- Process: Teaching and Learning; Cognitive Level: Application;
uids and pureed foods. Answer 4 is correct because dysphagia Client Need/Subneed: Safe, Effective Care Environment/
refers to difficulty swallowing. Nutrition may need to be Management of Care, Supervision
provided through a feeding tube to prevent aspiration.
2164_Ch06_347-578 29/03/12 12:31 PM Page 578
2164_Ch07_579-602 29/03/12 12:32 PM Page 579

CHAPTER 7

Physiological Integrity
Nursing Care of the Geriatric Client

Robyn Marchal Nelson

579
2164_Ch07_579-602 29/03/12 12:32 PM Page 580

580 chapter 7 Physiological Integrity

Introduction 3. Decreased blood flow to skin and decreased


estrogen production baldness.
This chapter is unique in that the primary objective is to a. Hair distribution: thin on scalp, axilla, pubic
present practical, concise information of clinical rele- area, upper and lower extremities.
vance for the beginning practitioner that is not covered b. Decreased facial hair in men.
elsewhere. The focus is on 12 significant problems and 4. Decreased estrogen production increased
concerns associated with the older adult: falls, use of restraints, facial (chin, upper lip) hair in women.
thermoregulation, sleep disturbance, skin breakdown, polyphar- D. Eyes:
macy, specific types of hearing changes, age-related macular 1. Loss of soluble protein with loss of lens
degeneration, incontinence, sexual neglect, caregiver burden, transparency development of cataracts.
warning signs of poor nutrition. 2. Decrease in pupil size limits amount of light
The chapter begins with health assessment of the older entering the eye elderly need more light
adult, system by system, highlighting the most common to see.
changes associated with the aging process; it ends with 3. Decreased pupil reactivity decrease in rate of
several functional rating scales to assist in management of light changes to which a person can readily adapt.
care in the home or community. 4. Decreased accommodation to darkness and dim
Covered elsewhere in this book are other age-relevant light diminished night vision.
problems and test questions. Refer to Chapter 10 for 5. Loss of orbital fat sunken appearance.
mental health conditions common in the older adult 6. Blink reflexslowed.
(e.g., dementia, Alzheimers disease, depression). Chapter 6 7. Eyelidsloose.
covers common age-relevant problems related to immo- 8. Visual acuitydiminished.
bility, bladder and bowel dysfunction, hip fracture, use 9. Peripheral visiondiminished.
of assistive devices, cardiovascular conditions (stroke, 10. Visual fieldsdiminished (e.g., macular
hypertension), cataracts, and glaucoma. Also covered in degeneration).
Chapter 6 are health problems that have more serious 11. Lens accommodationdecreased; requires
consequences in the older adult, such as pneumonia and corrective lenses.
osteoporosis. Nutritional needs of the elderly are included 12. Presbyopialens may lose ability to become
in Chapter 9. convex enough to accommodate to nearby
I. HEALTH ASSESSMENT OF THE GERIATRIC objects; starts at age 40 (farsightedness).
CLIENT (see also Chapter 6) 13. Color of irisfades.
A. Skin: 14. Conjunctivathins, looks yellow.
1. Decrease in elasticity wrinkles and lines, 15. Increased intraocular pressure glaucoma.
dryness. 16. Previous corrective surgery.
2. Loss of fullness sagging. E. Ears:
3. Generalized loss of adipose and muscle tissue 1. Changes in cochlea decrease in average pitch
wasting appearance. of sound.
4. Decrease of adipose tissue on extremities, redis- 2. Hearing lossgreater in left ear than right;
tributed to hips and abdomen in middle age. greater in higher frequencies than in lower.
5. Bony prominences become visible. 3. Tympanic membraneatrophied, thickened
6. Excessive pigmentation age spots. hearing loss.
7. Dry skin and deterioration of nerve fibers and 4. Presbycusisprogressive loss of hearing in
sensory endings pruritus. old age.
8. Decreased blood flow pallor and 5. Use of hearing devices.
6. Predisposed to wax buildup (cerumen).
GERIATRICS

blotchiness.
9. Overgrowth of epidermal tissue lesions F. Mouth/dental health:
(some benign, some premalignant, some 1. Dental caries.
malignant). 2. Poor-fitting dentures; no dentures.
B. Nails: 3. Cancer of the mouthincreased risk.
1. Dry, brittle, peeling, ridges. 4. Decrease in taste buds inability to taste
2. Increased susceptibility to fungal infections. sweet/salty foods.
3. Decreased growth rate. 5. Olfactory bulb atrophies decreased ability
4. Toenails thick, difficult to cut, ingrown. to smell due to blockage or disease of olfactory
C. Hair: receptors in the upper sinus decreased
1. Loss of pigment graying, white. awareness of body odor, smoke, fumes,
2. Decreased density of hair follicles thinning spoiled food.
of hair. 6. Coating of tongue.
2164_Ch07_579-602 29/03/12 12:32 PM Page 581

Health Assessment of the Geriatric Client 581


G. Cardiovascular: 2. Cold intolerance.
1. Lack of elasticity of vessels increased resistance 3. Women: decreased ovarian function
to blood flow; decreased diameter of arteries increased gonadotropins.
increased blood pressure. 4. Decreased renal sensitivity to antidiuretic
2. Atherosclerotic and calcium plaques hormone (ADH) unable to concentrate
thrombosis. urine as effectively as younger persons.
3. Valves become sclerotic, less pliable reduced 5. Decreased clearance of blood glucose
filling and emptying. after meals elevated postprandial blood
4. Diastolic murmurs heard at base of heart. glucose.
5. Loss of elasticity, decreased contractility 6. Risk of diabetes mellitus increases with age.
decreased cardiac output. L. Urinary:
6. Changes in the coronary arteries pooling 1. Renal functionimpaired due to poor
of blood in systemic veins and shortness of perfusion.
breath reduced pumping action of the 2. Filtrationimpaired due to reduction in
heart. number of functioning nephrons decrease
7. Disturbance of the autonomic nervous system in urine concentration.
dysrhythmias. 3. Urgency and frequency: menoften due to
8. Extremitiesarteriosclerotic changes weaker prostatic hyperplasia; womendue to perineal
pedal pulses, colder extremities, mottled color; muscle weakness.
pain with ambulation. 4. Nocturiaboth men and women.
H. Respiratory: 5. Urinary tract infection (e.g., cystitis)increased
1. Efficiency reduced with age. incidence.
2. Greater residual air in lungs after expiration. 6. Incontinenceespecially with dementia;
3. Decreased vital capacity. stress/exercise induced.
4. Weaker expiratory muscles decreased 7. Retentiondue to incomplete bladder
capacity to cough infections of lower emptying.
respiratory tract. M. Musculoskeletal:
5. Decreased ciliary activity stasis of secretions 1. Muscle massdecreased. Loss of lower limb
susceptibility to infections. strength.
6. Oxygen debt in the muscles dyspnea on 2. Bony prominencesincreased.
exertion (DOE), sleep apnea due to O2 to 3. Demineralization of bone.
the brain. 4. Narrowing of intervertebral space shortening
7. Reduced chest wall compliance decreased of trunk loss of height.
expiratory excursion, affecting inspiratory and 5. Posturenormal; some kyphosis.
expiratory volumes. 6. Range of motionlimited.
I. Breasts: 7. Osteoarthritisrelated to extensive physical
1. Atrophy. activities and joint use.
2. Cancer riskincreased with age. 8. Gaitaltered; use of cane or walker.
J. Gastrointestinal: 9. Osteoporosis related to menopause, immobiliza-
1. Lack of intrinsic factor pernicious anemia. tion, elevated levels of cortisone increase in
2. Gastric motilitydecreased poorer, slower fractures.
digestion. 10. Calcium, phosphorus, and vitamin D
3. Esophageal peristalsisdecreased. decreased.
4. Hiatal herniaincreased incidence. N. Neurological:
5. Digestive enzymesgradual decrease of ptyalin 1. Voluntary, automatic reflexesslowed. GERIATRICS
(which converts starch), pepsin and trypsin 2. Sleep patternchanges.
(which digest protein), lipase (fat-splitting 3. Mental acuitychanges.
enzyme). 4. Sensory interpretation and movement
6. Absorptiondecreased. changes.
7. Improper diet constipation. 5. Pain perceptiondiminished.
8. Decreased thirst sensation risk for 6. Dexterity and agilitylessened.
dehydration. 7. Reaction timeslowed.
9. Decreased saliva dysphagia. 8. Memorypast more vivid than recent memory
K. Endocrine: due to loss of neurons from CNS.
1. Basal metabolism rate lowered decreased 9. Depression.
temperature. 10. Alzheimers disease.
2164_Ch07_579-602 29/03/12 12:32 PM Page 582

582 chapter 7 Physiological Integrity

O. Sexuality: b. Postural instabilitytendency to lose balance;


1. Women: older adults take steps to correct balance and
a. Estrogen productiondecreased with increase possibility of falling.
menopause. c. Impaired muscular controlinability to recover
b. Breastsatrophy. from trip or unexpected step; weaker muscle
c. Vaginal secretionsreduced lubricants. cushioning and slowed righting reflexes.
d. Sexualitydrive continues; sexual activity d. Deterioration of vision and hearing
declines. impaired ability to avoid obstacles.
2. Men: e. Loss of short-term memoryprone to trip
a. Testosterone productiondecreased. over forgotten objects.
b. Testesdecrease in size; decreased sperm f. Environmental:
count. (1)Home: unstable furniture and appliances;
c. Libido and sexual satisfactionno changes. stairs with poor rails; throw rugs and
P. Mental status: the 3 Ds: frayed carpets; poor lighting; low beds
1. Delirium: confusion/agitation with time and place and toilets; pets; objects on floor;
disorientation, illusions and/or hallucinations. medications.
2. Dementia: cognitive deficits (memory, reasoning, (2) Institutions: recent admission or transfer;
judgment). furniture; slick, hard floors; unsupervised
3. Depression: decreased interest/pleasure in activities; mealtimes; absence of handrails;
activities. inadequate lighting, long hallways.
Q. Immune system: 2. Nursing goal/implementation: reduce risk of
1. Immune response: decreased decreased T-cell falling in the older adult with fall prevention
activity and decrease in cell-mediated immunity. measures.
2. Increased risk of nosocomial infections a. Treat underlying condition (e.g., osteoporosis,
(e.g., Pseudomonas, staphylococci, enterococci, muscle weakness, imbalance, pain).
and fungi), pneumonia, cancer, reactivation of b. Reduce risk factors (e.g., visual problems,
varicella-zoster virus and tuberculosis (TB). orthostasis).
3. Atypical inflammatory response; no elevated c. Reduce environmental hazards (e.g., provide
temperatures or white blood cell counts. adequate lighting and night lamps, avoid
II. GENERAL NURSING DIAGNOSES IN THE cluttered areas, no throw rugs, provide bath
GERIATRIC CLIENT and shower support bars).
d. Increase leg range-of-motion (ROM) exercises.
A. Risk for loneliness related to isolation and loss of
e. Develop an individualized exercise plan.
many friends, family members, and pets due to
f. Support adequate nutrition (e.g., calcium
separation and death.
intake).
B. Self-care deficit related to inability to complete
B. Use of restraints:
activities of daily living.
1. Definitionany device, material, or equipment
C. Impaired verbal communication related to
attached to the client that cannot be easily
hearing loss.
removed by the client; restricts free movement;
D. Fluid volume deficit related to low fluid intake.
includes leg restraints, arm restraints, hand mitts,
E. Impaired skin integrity related to prolonged
soft ties or vests, wheelchair safety bars,
back-lying position and inability to turn self.
geri-chairs.
F. Body image disturbance related to physical changes
2. Assessment of problems resulting from use of
associated with the aging process.
restraints:
GERIATRICS

G. Sleep pattern disturbance related to concern about


a. Increased agitation, confusion.
outcomes of pending diagnostic tests.
b. Falls.
III. PROBLEMS ASSOCIATED WITH THE c. Pressure sores.
GERIATRIC CLIENT* d. Bone density loss (demineralization).
A. Falls: e. Immobility hazard.
1. Assessment. Risk factors: f. Death from strangulation.
a. Gait changesprone to trip and stumble; do 3. Nursing goal/implementation: provide
not pick feet up as high. restraint-free care with alternatives to restraints.
a. Physical: recliners; medications for pain
relief; seating adaptations (physical therapy/
occupational therapy); chairs with deep
*Source: Cheryl Osborne, EdD, RN, Professor of Nursing, Director of Gerontology,
California State University, Sacramento.
seats.
2164_Ch07_579-602 29/03/12 12:32 PM Page 583

Problems Associated with the Geriatric Client 583


b. Psychosocial: encourage expression of feelings, g. Avoid/minimize drugs that negatively influ-
giving time for client response; encourage ence sleep, such as ranitidine, diltiazem,
positive self-concept; offer hope; active listen- atenolol, nifedipine.
ing; increased or decreased sensory stimula- h. Create a restful environment: turn off lights,
tion; increased visiting; reality orientation; reduce or eliminate noise, minimize disrup-
clocks; animals. tions for therapy or monitoring.
c. Activity: structured daily routines; wandering/ E. Skin breakdown.
pacing permitted; physical exercise; nighttime 1. Assessment:
activities. a. Age-related changes: slower rate of epidermal
d. Environmental: door buzzers; limb bracelet proliferation; thinner dermis; decreased
alarms; signs, call bells. dermal blood supply, melanocytes (gray
e. Structural: exit alarms; increased lighting; hair), moisture, sweat and sebaceous
enclosed courtyards. glands.
f. Supervision: family, nursing; volunteer; security. b. Predisposing risk factors: exposure to ultravi-
g. Sedation. olet (UV) rays (sunlight, excessive tanning);
C. Thermoregulationnormal oral temperature for adverse medication effects; personal hygiene
greater than 75 years of age: oral, 96.9 to 98.3F; habits (too frequent bathing); limited
rectal, 98 to 99F. Decreased or absence of activity, heredity.
increased temperature in infection or dehydration. c. Functional consequences: dry skin; skin wrin-
1. Assessment of causes: kles; delayed wound healing; increased
a. Factors affecting the hypothalmus: decreased susceptibility to burns, injury, infection,
muscle activity, metabolic rate, food and fluid altered thermoregulation, skin cancer,
intake, subcutaneous fat; changes in peripher- cracking nails.
al blood flow; diseases; medications. 2. Nursing goal/implementation: treatment for
b. External factors: environmental temperature; pressure sores.
humidity; airflow; type and amount of Stage Ireddened broken skin: cover and protect
clothing. (use sprays, gels, transparent films, transparent
2. Nursing goal/implementation: prevent occlusive wafers).
hypothermia and hyperthermia. Stage IIblister or partial-thickness skin loss:
a. Ensure and monitor adequate fluid and food cover, protect, hydrate, insulate, and absorb
intake. exudate (use transparent films, occlusive
b. Maintain constancy in environmental temper- wafer dressings, calcium alginate for absorbing
ature: avoid drafts, overheating, prolonged exudate; polyurethane foam, moistened gauze
exposure to cold. dressing).
c. Monitor ventilation: provide airflow (air con- Stage IIIfull-thickness skin loss: cover,
ditioners, fans; safe sources of heat). protect, hydrate, insulate, absorb, cleanse,
d. Use layered clothing (remove when warm; prevent infection, and promote granulation
add when cold). (use occlusive wafer dressings, absorption
D. Sleep disturbances. dressing, calcium alginate, and moistened
1. Nursing goal/implementation: promote restful gauze dressings).
sleep and prevent sleep deprivation with sleep care Stage IVfull-thickness skin loss involving
strategies. muscle, tendon, and bone: same as stage III
a. Maintain normal sleep pattern: arrange except before promoting granulation, dead
medications and therapies to minimize sleep eschar (tissue) is removed (use absorption
interruptions. dressing, calcium alginate, and moistened GERIATRICS
b. Encourage daytime activity; discourage gauze dressings).
daytime naps. F. Polypharmacyconcurrent use of several drugs
c. Support bedtime routines/rituals: bedtime increasing the potential for adverse reactions, drug
reading, listening to music, quiet television. interactions, and self-medication errors (Table 7.1).
d. Promote comfort: mattress, pillows, 1. Reasons for polypharmacy:
wrinkle-free linens, loose bed covering. a. Lack of communication among multiple
e. Promote relaxation: warm milk or soup if not health-care providers.
contraindicated, back rub. b. Lack of information about over-the-counter
f. Avoid/minimize stimulation before bedtime: drug use.
no caffeine after dinner, reduce fluid intake c. Lack of information about client
before sleep, refrain from smoking. noncompliance.
2164_Ch07_579-602 29/03/12 12:32 PM Page 584

584 chapter 7 Physiological Integrity

Table 7.1 b. Stress incontinence: pelvic muscle exercises,


biofeedback.
Medications That Should Not Be Used by c. Anxiety, depression: counseling, exercise,
Geriatric Clients meditation, relaxation techniques; touch,
Medication Category Contraindicated Medications music, and pet therapy.
Antidepressants Elavil
d. Arthritis: acupuncture, heat therapy, thera-
Triavil peutic exercise, postural or alignment aids;
touch, music, and pet therapy.
Cardiovascular Aldomet
Catapres e. Chronic neuromuscular problems: massage,
Cyclospasmol body work, touch and music therapy.
Pavabid f. Sleep problem (see D. Sleep disturbances).
Serpasil G. Hearing changesexternal auditory canal atro-
Trental
phies, resulting in thinner walls and increased
Pain and arthritis medications Bufferin cerumen buildup; degenerative changes in
Darvocet ossicular joints leading to slower/stiffer move-
Darvon
Feldene
ments; loss of hair cells and cochlear neurons
Talwin in inner ear.
1. Types:
Gastrointestinal Colace
Dialose-Plus a. Presbycusisbilateral loss of high-pitched
Donnatal tones; slightly less severe in women than
Doxidan in men.
Mylanta b. Impaired pitch discriminationafter age 55,
Surfak
makes localizing and understanding sounds
Tigan
difficult; impaired ability to understand
Tranquilizers and hypnotics Ativan consonants.
Dalmane
Halcion
c. Decline in speech discriminationafter age
Librium 60, speech intelligibility declines; slowing of
Nembutal memory and slowing of mental processes
Restoril with advancing age may also affect speech,
Valium hearing, and understanding.
Xanax
d. Diminished vestibular functiondeficits in
Neurological Artane equilibrium and greater fall risk.
Cogentin
2. Nursing goal/implementation: improve
Source: Cheryl Osborne, EdD, RN, Professor of Nursing, Director of communication for person who is hearing
Gerontology, California State University, Sacramento. impaired.
a. Compensate with other sensesface listener;
d. Use of complementary (alternative, folk medi- make eye contact if culturally acceptable;
cine) therapies and fear of telling health-care get persons attention before talking; use
provider. touch if culturally appropriate; help with
e. Assumption that, once medication is started, hearing aid; avoid walking around room
it should be continued indefinitely and not while talking; write down key words if
changed. person can read.
f. Assumption that, if there are no early side b. Alter stimulus and behaviorspeak in normal
effects, there will not be any later. volume of voice or slightly lower, avoid shout-
GERIATRICS

g. Changes in daily habits (smoking, activity, ing; use short sentences; separate important
diet/fluid intake). words with pauses; allow more time for com-
h. Changes in mental-emotional status that may munication; have person repeat to show
affect consumption patterns. understanding; repeat; teach person to be
i. Changes in health status. more assertive about impairment.
j. Financial limitations (drug substitution). c. Modify environmenteliminate or reduce
2. Nursing goal/implementation: assist the older background noise (turn off running water,
adult to use medications safely and try nonphar- close doors, lower TV or radio); select areas
macological interventions for common health with sound-absorbing abilities (carpets,
problems. drapes) for conversations; amplify tele-
a. Constipation: exercise, relaxation, biofeedback; phone; allow for adequate light on
increase fluid and fiber intake. speakers face.
2164_Ch07_579-602 29/03/12 12:32 PM Page 585

Problems Associated with the Geriatric Client 585


H. Visual impairment. assistance, use of Velcro closures on cloth-
1. Age-related macular degeneration (AMD) ing, raised toilet seats with safety bars.
leading cause of irreversible and legal blindness c. Monitor fluid intake; adequate hydration
for those over 65 years; blurred far and near 1 to 2 L during day; avoid alcohol and
vision; loss of central vision; difficulty going caffeine and restrict fluids at bedtime.
up and down steps and stairs; parts of words d. Avoid medications contributing to
and letters disappear when reading; straight incontinence.
lines appear to be wavy. Acuity: 20/200 or less. e. Avoid use of indwelling catheters if
a. Assessment. Risk factors: increased age, possible.
women, Caucasian, smoking, UV light f. Use usual undergarments; be positive about
exposure, diabetes, diet (low in leafy green continence; use absorbent pads to improve
vegetables and antioxidants). perineal hygiene if other measures fail.
b. Prevention: UV light filters, aspirin, g. Treat constipation.
vitamins A and B, beta-carotene, zinc. h. Observe for signs of UTI.
c. Prognosis: no treatment or cure; laser may J. Sexual neglectmany myths exist about sexuality
prevent spread of AMD in some clients. and aging.
d. Nursing goal: assist client to maintain 1. Facts about sexuality in the older adult:
independent functioning. a. Sexual desire can and does exist in
2. See Chapter 6 for other causes of visual advanced age.
problems in the older adult, such as stroke, b. If in good health, a satisfying sex life can
pp. 382383; glaucoma, pp. 388389; extend into the 80s and beyond; if sexually
cataracts, pp. 389390; and diabetes, active in youth and middle age, vigor and
pp. 467470. interest will be retained into old age.
I. Loss of urinary controlneurological mechanism c. Sexual attractiveness has little to do with
controlling bladder emptying does not work age and the appearance of partner.
effectively and results in incontinence; a symp- d. Less than 1% of sudden coronary deaths
tom, not a diagnosis. occur during sexual intercourse; greater
1. Assessment. Risk factors: anxiety and tension exists if sex is restricted.
a. Immobility. e. Vaginal lubrication is decreased because of
b. Cognitive and functional impairments menopause, but sexual pleasure still exists.
Parkinsons disease, Alzheimers disease, f. Sex may actually be better in later years;
multiple sclerosis, alcoholism, stroke, partners have an appreciation of intimate
vitamin B deficiency, inability to walk and sharing and caring.
transfer to toilet. g. Sexual activity is a good form of exercise
c. Medications. and helps maintain flexibility and stamina.
d. Institutionalization. h. Older adults have a strong interest in sexual
e. Pathological conditionsmen: hyperplasia activity and physical and mental well-being;
leading to infection, incomplete emptying, older adults should be encouraged to con-
urgency, and frequency; women: weakening tinue sexual interests without guilt.
of pelvic floor postmenopause, leading to i. Male erections can continue into the 80s
residual urine and infection; atrophy of and beyond.
vaginal and trigonal tissue, leading to fre- 2. Nursing goal: assist the client to reduce
quency, urgency, and incontinence. barriers to a satisfying sexual experience.
f. Childbirth. a. Barriers to a satisfying sexual experience:
2. Incontinencepossible symptom of urinary (1)Loss of sexual responsivenesscauses: GERIATRICS
tract infection (UTI), impaction, chronic con- monotony of a same old sexual rela-
stipation, dementia, inability to walk and tionship; mental or physical fatigue;
transfer by self to toilet, dehydration. overindulgence in food or drink; preoc-
3. Nursing goal/implementation: minimize cupation with career or economic pur-
incontinence episodes and reduce urinary tract suits; physical or mental infirmities of
infections. either partner; performance anxiety;
a. Regular toileting scheduleevery 2 hours, lack of privacy.
with some exceptions at night; or personal- (2)Changes with aginghormonal;
ize regimen based on assessment. decreased muscle tone and elasticity;
b. Modify environmentlocation of toilet, prostate hyperplasia; sclerosing arteries
good lighting, prompt response to calls for and veins; increased time needed for
2164_Ch07_579-602 29/03/12 12:32 PM Page 586

586 chapter 7 Physiological Integrity

arousal and rearousal; medications (2)Concentrate on the present.


(e.g., antihypertensives); surgery (3)Talk about the reasons for being a
(e.g., prostatectomy); response to caregiver.
menopause; availability of a partner. (4)Use respite care as needed.
K. Caregiver burden80% to 90% of the care given L. Poor nutrition.
to adults who are dependent is given by family 1. Assessment of warning signs:
and friends, especially middle-aged women. a. Disease, illness, or chronic condition that
1. Negative consequences of caregiving: changes eating habits/pattern; also emotion-
a. Infringement on privacy. al problems: confusion, depression, or
b. Decreased social contact. sadness.
c. Loss of income and assets. b. Eating too little or too much, skipping
d. Increased family conflict and distress. meals, drinking more than 1 to 2 alcoholic
e. Little or no time for personal or recreational beverages daily.
activities. c. Missing, loose, or rotten teeth, or ill-fitting
f. Increased use of alcohol and psychotropic dentures.
drugs. d. Spending less than 25 to 30 dollars per week
g. Changes in living arrangements (sharing on food.
households). e. Living alone.
h. Likelihood of decreasing or giving up job f. Taking multiple medications with increased
responsibilities. chance for side effects (change in taste, con-
i. Increased risk of clinical depression. stipation, weakness, drowsiness, diarrhea,
j. Feelings of anger, guilt, anxiety, grief, nausea).
depression, helplessness, chronic fatigue, g. Unplanned weight loss or gain.
emotional exhaustion. h. Problems with self-care (walking, shopping,
k. Poor health and increased stress-related buying and preparing food).
illness and injuries. i. Age greater than 80 (frail elderly).
2. Positive consequences of caregiving: 2. Nursing goal/implementation: promote
a. Family becomes closer. optimal nutritional health for the older adult
b. Making a difference in the quality of care. (see Chapter 9 for additional information on
c. Companionship for adult who is dependent nutrition in the older adult).
and caregiver. a. Encourage fluids during meals.
d. Better understanding of the needs of the b. 5 to 6 small meals per day.
adult who is dependent. c. Advise not to lie down for 1 hour after a
e. Feeling useful and worthwhile. meal.
f. Improved relationship between caregiver and d. Avoid overuse of salt and sweets.
adult who is dependent. e. Use alternate seasonings: herbs, garlic,
3. Impact of hospitalization of older adult on lemon.
caregiver: M. See Chapter 10 for discussion of depression,
a. Frustration with delays in older adult being elder abuse, and causes of impaired cognition,
admitted. such as dementia, Alzheimers disease, and
b. Perception of poor care; complaints about delirium.
rudeness; upset that call lights and questions N. See Chapter 6 for discussion of osteoarthritis and
are not answered in a timely manner. Parkinsons disease.
c. Lack of involvement in decision making.
GERIATRICS

IV. SUMMARY OF MOST FREQUENT ISSUES


d. Lack of preparation for discharge; too much IN THE CARE OF THE OLDER GERIATRIC
information given too quickly; problems CLIENT (Table 7.2)
coordinating services (e.g., home visits,
needed equipment). V. Use the functional rating scale for the geriatric
e. Fatigue/stress from going back and forth to client (Table 7.3) to rate social resources, economic
hospital. resources, mental health, physical health, and activi-
4. Nursing goal/implementation: assist the ties of daily living.
caregiver in achieving control and a sense of VI. Use the functional screening examination (Table 7.4)
satisfaction. to help determine dependent/independent status for
a. Strategies to minimize caregiver burden: planning home care.
(1)Develop and maintain a routine.
2164_Ch07_579-602 29/03/12 12:32 PM Page 587

Functional Rating Scale for the Geriatric Client 587

Table 7.2
Most Frequent Issues in the Care of the Geriatric Client
Complication Cause
Loss of functional independence Immobility, lack of time to perform task, lack of expectations by staff or family
Falling Postural hypotension, dizziness, medication effects, unfamiliar surroundings
Confusion Drug effects, unfamiliar environment, sensory deficits
Skin breakdown Immobility, skin changes with aging, inadequate turning
Incontinence Cognitive changes, inability to access toilet, need for assistance, loss of sphincter muscle
control
Constipation Drug effects, inactivity, dietary changes, changes in GI motility
Heatstroke Altered thermoregulation response
Hypothermia Loss of subcutaneous tissue decreased insulation
Self-injury Reduced tactile sensation
Source: Cheryl Osborne, EdD, RN, Professor of Nursing, Director of Gerontology, California State University,
Sacramento.

Table 7.3
Functional Rating Scale for the Geriatric Client
Social Resources Rating Scale
Rate the current social resources of the person being evaluated along the six-point scale presented below. Circle the one
number that best describes the persons present circumstances.
1. Excellent social resources. Social relationships are very satisfying and extensive; at least one person would take care of him or
her indefinitely.
2. Good social resources. Social relationships are fairly satisfying and adequate, and at least one person would take care of him
or her indefinitely; or social relationships are very satisfying and extensive, and only short-term help is available.
3. Mildly socially impaired. Social relationships are unsatisfactory, of poor quality, few, but at least one person would take care of
him or her indefinitely; or social relationships are fairly satisfactory, adequate, and only short-term help is available.
4. Moderately socially impaired. Social relationships are unsatisfactory, of poor quality; few, and only short-term care is available;
or social relationships are at least adequate or satisfactory; but help would only be available now and then.
5. Severely socially impaired. Social relationships are of poor quality, few; and help would only be available now and then; or social
relationships are at least satisfactory or adequate; but help is not even available now and then.
6. Totally socially impaired. Social relationships are unsatisfactory, of poor quality, few, and help is not even available now and then.
Economic Resources Rating Scale
Rate the current economic resources of the person being evaluated along the six-point scale presented below. Circle the
one number that best describes the persons present circumstances.
1. Economic resources are excellent. Income is ample and person has reserves.
2. Economic resources are satisfactory. Income is ample but person has no reserves; or income is adequate and person has reserves.
3. Economic resources are mildly impaired. Income is adequate but person has no reserves; or income is somewhat inadequate
but person has reserves.
4. Economic resources are moderately impaired. Income is somewhat inadequate and person has no reserves.
5. Economic resources are severely impaired. Income is totally inadequate and person may or may not have reserves.
6. Economic resources are completely impaired. Person is destitute, completely without income or reserves. GERIATRICS
Mental Health Rating Scale
Rate the current mental functioning of the person being evaluated along the six-point scale presented below. Circle the one
number that best describes the persons present functioning.
1. Outstanding mental health. Intellectually alert and clearly enjoying life. Manages routine and major problems with ease and is
free from any psychiatric symptoms.
2. Good mental health. Handles both routine and major problems satisfactorily and is intellectually intact and free of psychiatric
symptoms.
3. Mildly mentally impaired. Has mild psychiatric symptoms and/or mild intellectual impairment. Continues to handle routine,
though not major, problems satisfactorily.
4. Moderately mentally impaired. Has definite psychiatric symptoms and/or moderate intellectual impairment. Able to make routine
commonsense decisions, but unable to handle major problems.
Continued
2164_Ch07_579-602 29/03/12 12:32 PM Page 588

588 chapter 7 Physiological Integrity

Table 7.3
Functional Rating Scale for the Geriatric Clientcontd
5. Severely mentally impaired. Has severe psychiatric symptoms and/or severe intellectual impairment that interferes with routine
judgments and decision making in everyday life.
6. Completely mentally impaired. Grossly psychotic or completely impaired intellectually. Requires either intermittent or constant
supervision because of clearly abnormal or potentially harmful behavior.
Physical Health Rating Scale
Rate the current physical functioning of the person being evaluated along the six-point scale presented below. Circle the
one number that best describes the persons present functioning.
1. In excellent physical health. Engages in vigorous physical activity, either regularly or at least from time to time.
2. In good physical health. No significant illnesses or disabilities. Only routine medical care such as annual checkups required.
3. Mildly physically impaired. Has only minor illnesses and/or disabilities that might benefit from medical treatment or corrective
measures.
4. Moderately physically impaired. Has one or more diseases or disabilities that are either painful or require substantial medical
treatment.
5. Severely physically impaired. Has one or more illnesses or disabilities that are either severely painful or life-threatening or require
extensive medical treatment.
6. Totally physically impaired. Confined to bed and requires full-time medical assistance or nursing care to maintain vital bodily
functions.
Performance Rating Scale For Activities of Daily Living (ADL)
Rate the current performance of the person being evaluated on the six-point scale presented below. Circle the one number
that best describes the persons present performance.
1. Excellent ADL capacity. Can perform all ADL without assistance and with ease.
2. Good ADL capacity. Can perform all ADL without assistance.
3. Mildly impaired ADL capacity. Can perform all but one to three ADL. Some help is required with one to three, but not necessarily
every day. Can get through any single day without help. Is able to prepare own meals.
4. Moderately impaired ADL capacity. Regularly requires assistance with at least four ADL but is able to get through any single day
without help; or regularly requires help with meal preparation.
5. Severely impaired ADL capacity. Requires help each day but not necessarily throughout the day or night with many ADL.
6. Completely impaired ADL capacity. Requires help throughout the day and/or night to carry out ADL.
Summary of Ratings: Projected Outcome
Social resources _________________ >15 Requires help
Economic resources _________________ 1215Intermediate supports
Mental health _________________ <11 Independent living
Physical health _________________
Activities of daily living _________________
Cumulative impairment score _________________
(sum of the five ratings)
Source: Adapted from University of California Davis Medical Center, Alzheimers Center, Sacramento, California.

Table 7.4
Functional Screening Examination
Test Request Function Tested Able, Normal Limited but Able Unable
GERIATRICS

Put both hands together, Shoulder external rotation, flexion, abduction; elbow o o o
behind your head flexion; wrist extension; gross strength (managing
clothing, hair combing, washing back)
Put both hands together, Shoulder internal rotation, adduction; elbow flexion; o o o
behind your back gross strength (managing clothing, washing lower
back)
Sitting, touch great toe Back, hip, knee flexion, gross strength, balance o o o
with opposite hand (lower extremity dressing and hygiene)
Squeeze my two fingers, Grasp; approximately 20-lb grasp needed for o o o
each hand functional activities (e.g., holding a pan)
2164_Ch07_579-602 29/03/12 12:32 PM Page 589

Functional Screening Examination 589

Table 7.4
Functional Screening Examinationcontd
Test Request Function Tested Able, Normal Limited but Able Unable
Hold paper between thumb Pinch; approximately 3-lb grip needed for functional o o o
and lateral side of index activities (e.g., turning a key)
finger (while examiner tries
to pull it out)
Show me your medicines Cognition; knowledge; visual acuity, hand function; o o o
compliance; polypharmacy; effective management
of health problems; access to health care; support
system; finances; polyproviders
Get up from the chair Hip strength; gait, symmetry in posture velocity, o o o
without using your hands, step height; balance, ability to withstand sudden
walk 10 ft down hall, turn changes in position; fall risk; cardiovascular and
around, stand with your pulmonary status; transfer ability; motion; fear of
eyes closed (examiner falls; judgment and cognition; environmental
nudges to determine awareness; need for assistance/equipment;
balance), sit down flexibility; endurance; ability to walk and function
without using hands at home
Take off your shoes and Balance; fine motor skills; dexterity; hearing and o o o
socks receptive language; visual acuity; judgment; regis-
tration and attention span; low back flexibility and
mobility; weight shifting; safety and adequacy of
footware; social support if nails are cut by someone
else; pain and comfort on movement
Observe a meal being Nutritional intake; intactness of reflexes; use of o o o
prepared/eaten utensils; hand/oral motor function; sequencing skills,
visual acuity; use of utensils and equipment; ability
to get needed help; ability to handle food and liquid;
ability to compensate for limitations
Summary and conclusions:

GERIATRICS

Source: Dr. Cheryl Osborne, EdD, RN, Professor of Nursing, Director of Gerontology, California State University,
Sacramento.
NOTE: Totally independenttotal score of 9 in Able or Limited columns; dependent > 1 in Unable column.
2164_Ch07_579-602 29/03/12 12:32 PM Page 590

590 chapter 7 Physiological Integrity

Questions wheelchair for 2 hours twice a day. What is the correct


explanation for the frequent diarrhea stools?
Select the one answer that is best for each question, unless 1. Inadequate roughage in the diet.
otherwise directed. 2. Inactivity from sedentary lifestyle.
3. Gastrointestinal virus.
1. The nurse should ensure that a healthy 89-year-old client
4. Fecal impaction.
admitted to a skilled nursing facility has received which
vaccinations? Select all that apply. 7. An elderly client has a suspected medical diagnosis of
1. Pneumococcal. cataracts. Which symptom is consistent with this finding?
2. Hepatitis B (series of 3). 1. Objects are distorted and blurry.
3. Tetanus. 2. Vision is improved in bright lights.
4. Influenza. 3. Objects have a halo around them.
5. Meningococcal. 4. Single objects seem to be doubled.
6. Zoster.
7. Diphtheria.
8. What is the most appropriate footwear that a nurse
should recommend to a client with Parkinsons disease?
2. An elderly client has a noncemented total hip joint placed. 1. Open-toed sandals.
Postoperative activity for this client should include: 2. Double-knotted leather wing tips.
1. Bedrest for 6 weeks with continuous passive motion. 3. High-top sneakers.
2. Touch-down weight-bearing starting first postopera- 4. Slip-on rubber soles.
tive day.
3. Head of bed flat for 48 hours.
9. What are normal changes associated with aging? Select all
that apply.
4. Hip immobilization for 3 to 4 weeks with no
1. Decreased salivation.
weight-bearing.
2. Decreased ability to hear low-frequency sounds.
3. A client with Alzheimers disease has superficial skin 3. Impaired healing of tissues.
breakdown related to functional incontinence. The 4. Multiple nighttime voiding.
correct nursing intervention includes: 5. Increased sensitivity to pain.
1. Inserting a continuous indwelling catheter per order. 6. Higher than normal baseline body temperature.
2. Assisting to the toilet and protecting bed with pads.
3. Limiting oral fluid intake to 250 mL/day.
10. An RN makes a home visit to a client who is 62 years
old, and finds that the client is not responding to verbal
4. Administering a loop diuretic, such as furosemide, as
stimuli and has no pulse or respirations, and the skin is
ordered.
cool to touch. The nurses first action should be to:
4. An elderly client, who has had a stroke, is receiving full- 1. Do mouth-to-mouth resuscitation, 2 breaths.
strength tube feedings of 75 mL/hr, and an IV of D5 1/2NS 2. Do chest compression.
at 50 mL/hr. The Na+ level is 150. Which action should be 3. Call emergency response (911).
taken by a nurse? 4. Call the family first.
1. The IV should be changed to a higher Na+ content such
as D5NS.
11. In reviewing the laboratory and x-ray reports of an elderly
client, which findings should a nurse identify as being
2. The rate of the IV should be slowed.
consistent with a diagnosis of emphysema?
3. Tap water should be given through the feeding tube.
1. Increased PCO2, hypoinflated alveoli, and decreased PO2.
4. A diuretic should be given.
2. Decreased PCO2, decreased PO2, and decreased
5. An elderly client with kidney disease is admitted with hematocrit.
dementia. The laboratory values are: Ca++ at 7 mg/dL, PO4 3. Increased PCO2, hyperinflated alveoli, and decreased
QUESTIONS

at 7 mg/dL, albumin at 2 gm, creatinine at 8 mg/dL. The hematocrit.


laboratory findings indicate: 4. Increased PCO2, increased hematocrit, and hyperinflated
1. Elevated phosphates from kidney failure. alveoli.
2. High risk for seizures from low Ca++. Treatment is a
priority.
12. When a nurse shines a penlight into an elderly clients
eyes to check pupil reaction, the nurse notes that one of
3. Need for dialysis to raise phosphate level.
the eyes is very cloudy and the pupil does not react. The
4. Need for a diet high in dairy to increase albumin.
nurse should:
6. An elderly client continues to have fecal incontinence with 1. Check vision in the cloudy eye.
6 to 7 small brown liquid stools each day. The client eats a 2. Notify the MD for CT evaluation.
soft diet and does not receive any stool softeners or laxa- 3. Keep the room lights dim.
tives. The clients primary form of activity is sitting in the 4. Restrict dietary protein.
2164_Ch07_579-602 29/03/12 12:32 PM Page 591

Questions 591
13. An elderly client with a history of congestive heart failure 19. Which condition is most likely to cause chronic pain in
(CHF) and hypertension (HTN) had a hip prosthesis an older adult?
inserted. Blood pressure is 80/50 mm Hg; heart rate is 80; 1. Osteoarthritis.
urine output is 15 mL during the last hour. The most 2. An old fracture.
appropriate nursing action would be to: 3. Sinus headaches.
1. Document these findings and continue to monitor 4. Peripheral neuropathy.
vital signs.
20. The care plan for an older adult with asthma, chronic
2. Give NS 250 mL over 15 minutes, check vital signs
obstructive pulmonary disease (COPD), and chronic
(VS) and lung sounds.
anxiety should include:
3. Check Na+ and K+ levels before selecting IV fluids.
Select all that apply.
4. Give 1 L lactated Ringers as fast as possible, then
1. Inhalation therapy and instruction about methods of
check VS, neck circumference, and lab reports.
conserving energy.
14. Following general anesthesia for a hip replacement, 2. An exercise program to increase the vital capacity of
an elderly clients vital signs are: P: 80, R: 14, BP the lungs.
110/78 mm Hg; O2 saturation is 100% on 40% mask; 3. Respiratory exercises with emphasis on forced inhalation.
and pain is 2/10. The client is shivering and complains 4. Oxygen therapy at 4 L/min as needed, and deep
of being cold. The first nursing priority should be to: breathing for relaxation.
1. Remove the oxygen. 5. Teaching the use of the diaphragm to improve
2. Check the temperature. breathing.
3. Apply warm blankets.
21. Which sign/symptom in an older adult is most indicative
4. Give pain medication.
of a urinary tract infection?
15. A 78-year-old client with CHF is being discharged home 1. Confusion.
where his wife, who has chronic obstructive pulmonary 2. Painful urination.
disease (COPD), plans to care for him. The wife indi- 3. Fever above 102F.
cates that the family will help. The correct action by a 4. Urinary frequency.
nurse would be to:
22. A nurse should include the risk of developing vaginitis
1. Recommend a nursing home placement for the couple.
when teaching an older adult woman about:
2. Contact a health-care practitioner to determine if the
1. Anticoagulation therapy.
wife is able to provide care.
2. Resuming sexual activity.
3. Determine if the family members will help.
3. Effects of poor nutrition.
4. Offer to make referrals for community resources to
4. Prolonged antibiotic therapy.
assist the couple.
23. An elderly client, who underwent a total hip replacement
16. A client with age-related hearing loss is complaining to a 3 days ago, asks why crossing my legs is prohibited.
nurse about being frustrated with hearing difficulties.
The correct response by a nurse is based on the knowl-
Which sound is most difficult for the client to hear?
edge that:
1. Recording of a march played softly.
1. Abduction can cause dislocation of the prosthesis.
2. Young children talking.
2. Adduction can cause dislocation of the prosthesis.
3. Motorcycle on the street.
3. Pressure on arteries in the legs can cause blood clots.
4. A mans voice in an elevator.
4. Avoiding acute flexion of the hip prevents contractures.
17. Which conditions increase the risk of the older adult 24. An older adult with osteoarthritis reports pain, stiffness,
client developing a decubitus ulcer? Select all that apply.
and deformities of the fingers. Which nursing actions
1. Osteoarthritis.
2. Impaired circulation.
would be appropriate? Select all that apply. QUESTIONS
1. Application of cold packs.
3. Incontinence.
2. Teaching hand exercises.
4. Malnutrition.
3. Scheduling meditation therapy.
5. Excessive sun exposure.
4. Giving prescribed vitamin therapy.
18. An older client complains of colored rings around the 5. Limiting use of affected hand to minimize pain.
lightbulbs in the room. The correct response by a nurse
25. Which older adult client is at greatest risk of developing
would be:
hypothermia?
1. Is there a history of cataracts in your family?
1. A client with a sedentary lifestyle.
2. I dont see any colored rings in the lightbulbs.
2. A client who is shivering.
3. Have you had your eyes checked for glaucoma?
3. A client who complains of being cold.
4. Have you fallen recently and hit your head?
4. A client with increased body fat.
2164_Ch07_579-602 29/03/12 12:32 PM Page 592

592 chapter 7 Physiological Integrity

26. A nurse is teaching an elderly client with right hemiplegia 33. A 90-year-old clients condition is one of lethargy, poor
and severe speech impairment how to dress. An appropri- capillary perfusion, and urinary incontinence. These
ate first step in the teaching would be to: findings should indicate to a nurse that this client is at
1. Ask the client to put on a shirt. greatest risk for:
2. Demonstrate the correct way to put on a shirt. 1. Aspiration.
3. Explain the difficulties in putting on a shirt. 2. Contractures.
4. Give verbal instructions on dressing techniques. 3. Dehydration.
4. Skin breakdown.
27. To prevent injury, a nurse should advise an older adult
who is taking tricyclic antidepressants to: 34. An 80-year-old client complains of sleeping less and
1. Eat a diet high in roughage. awakens several times during the night. The client takes
2. Get an additional night-light. acetaminophen at bedtime and does not drink caffeine.
3. Provide lubrication for the oral mucosa. Which response by a nurse is most appropriate?
4. Stand up slowly from sitting or lying positions. 1. Tell the client to eliminate fluids after 6:00 p.m.
2. Recommend that the client go to bed 1 hour earlier.
28. A nurse should monitor an older adult more closely for 3. Recommend a sleep study to diagnose sleep apnea.
adverse reactions to medications if the client history
4. Tell the client that the sleeping pattern is a normal
includes:
age-related change.
1. CHF and COPD.
2. Colitis and polycythemia vera. 35. Assessment of an older adult with pneumonia will often
3. Hepatitis and glomerulonephritis. reveal:
4. Diabetes and cholecystitis. Select all that apply.
1. Anorexia and changes in behavior.
29. After reviewing an older adult clients history, a nurse 2. Headache and difficulty breathing.
should include teaching about fall prevention if the client
3. Muscle aches and fever.
has been taking:
4. Nonproductive cough and chest pain.
1. Diphenhydramine (Benadryl).
5. Afebrile and productive cough.
2. Ferrous sulfate.
3. Guaifenesin (Robitussin) 36. A 73-year-old client is admitted for rehabilitation after a
4. Loratidine (Claritin) mild stroke. The client complains of not feeling rested,
begins to nap during the day, and sleeps only 4 to 5 hours
30. While waiting to be seen in an emergency department nightly. The most appropriate nursing action is to:
(ED) for possible CHF, an elderly client with moderate
1. Assess the clients sleep-wake cycle to determine
dementia jumps up and says, I have to go feed my
necessary interventions.
chickens now. A triage nurses most appropriate
2. Do nothing as the sleep pattern is associated with
response is:
normal aging.
1. All right, you may leave.
3. Determine if the client takes a medication for sleep at
2. Please tell us about your chickens.
bedtime and request an order.
3. That noise was the TV, not chickens.
4. Relocate the client to a different room that is quieter.
4. You are not on the farm anymore.
37. An 87-year-old man, who has been living independently,
31. To improve an older adults sense of security, a nurse is entering a nursing home. To help him adjust, the most
should:
effective action is to:
1. Focus on increasing socialization skills in the client.
1. Involve him in as many activities as possible so he can
2. Connect the past with the future through pictures.
meet other residents.
3. Provide praise and recognition for past
2. Move him as quickly as possible so that he does not
QUESTIONS

accomplishments.
have time to think.
4. Review comforting memories using reminiscence.
3. Restrict family visits for the first 2 weeks to give him
32. A healthy, older adult client complains of dry, itchy skin. time to adjust.
The correct response by the nurse is: 4. Suggest that he bring his favorite things from home to
1. Avoid scratching the skin to minimize the risk of make his room seem familiar.
infection.
38. When preparing to perform discharge teaching with an
2. Drink fluids and shower instead of taking a bath.
older adult client, which nursing actions will facilitate
3. Take fewer baths, use soap sparingly, and apply lotion
learning? Select all that apply.
afterward.
1. Involving family members in the teaching sessions.
4. Wear cotton clothing and use a hypoallergenic
2. Keeping the pace slow, presenting a small amount of
soap.
material at a time.
2164_Ch07_579-602 29/03/12 12:32 PM Page 593

Questions 593
3. Using repetition as well as providing reinforcement, 43. The incidence of tuberculosis in the older adult is
such as written material. significantly increased among individuals who:
4. Providing ample opportunity for repeated practice 1. Are physically inactive.
sessions. 2. Are cigarette smokers.
5. Scheduling a group session with other clients. 3. Have received the bacille Calmette-Gurin (BCG)
39. In preparing a health education presentation for older vaccine.
adults, a nurse should keep in mind that: 4. Reside in institutions.
1. Older adults generally prefer group teaching. 44. The daughter of a client with Alzheimers disease
2. Older adult learners need more than a single teaching becomes frustrated when talking to her father. What
session. should a nurse suggest to improve communication?
3. Presentations should use more written materials than 1. Answer his questions simply even if the question is
lectures. asked repeatedly.
4. Videotapes and pamphlets ensure teaching 2. Finish his sentences before he becomes agitated.
effectiveness. 3. Focus the conversation on future events.
40. Which statement by the family caregiver of an older 4. Play word games to stimulate his mind and slow
woman with incontinence requires teaching by a nurse? progression of the disease.
1. It is too bad incontinence occurs with aging. 45. Which suggestion would be most accurate for a nurse to
2. Incontinence has been so embarrassing for make to the family of a client who has Alzheimers disease?
my mother. 1. Be sure to correct the client if the client is experiencing
3. Mother says her incontinence is related to the delusions.
number of children she had. 2. Avoid getting upset in front of the client.
4. I am relieved to know Mothers incontinence may be 3. Repeat requests to the client in order to increase
reversible. compliance.
41. A 63-year-old client is returning home after being hospi- 4. Activities should be done quickly to reduce client
talized for injuries received during a home invasion and anxiety.
robbery. Although neighborhood robberies are rampant, 46. A family member asks a nurse about possible treatments
the client has lived in the same house for 50 years and for Stage III Alzheimers disease (AD). Which statement
does not want to move. The client receives a monthly by the nurse is correct?
social security check in the mail. The most appropriate 1. High doses of vitamin E may slow disease progression.
action by a home health nurse would be to: 2. The benefits of NSAIDs have been proven.
Select all that apply. 3. The risks of herbals, such as ginkgo biloba, are greater
1. Advise the client to arrange for someone to visit than any benefit.
regularly or move into the home. 4. Recent research has shown hormone replacement
2. Advise the client to have the social security check improves cognition.
deposited directly to the bank.
3. Suggest the client get a dog. 47. Which assessment findings are consistent with a diagnosis
4. Advise the client to get a locking mail box to prevent of Alzheimers disease (AD)? Select all that apply.
mail theft. 1. Remote and recent memory impairment.
5. Take no action since the client has a right to autonomy. 2. No memory impairment but the client is unable to
recognize familiar persons.
42. A nurse in a senior adult day care program observes that 3. Cognitive decline has been rapid.
the participants have long toenails, corns, calluses, and 4. B12 deficiency or thyroid disease is often present.
other problems indicating a need for foot care. What is a 5. Impaired recall of words.
nurses correct action? 6. Loss of ability to recognize objects. QUESTIONS
1. Schedule podiatry services at the site after a class on
foot care. 48. A 76-year-old client with hypertension, type 2 diabetes,
2. Establish a regular foot care plan whereby the partici- and a known allergy to seafood is scheduled for a cardiac
pants toenails would be cut and corns and calluses catheterization. Which orders should a nurse recognize as
removed. appropriate for this client? Select all that apply.
3. Instruct family members in the proper methods of cut- 1. Administration of methylprednisolone (Solu-Medrol).
ting toenails and using commercial foot care products. 2. Draw blood for a liver panel.
4. Recommend that the participants soak their feet for 3. Draw blood for a renal panel.
10 minutes before cutting their toenails, using safe 4. Start NS IV at 125 mL/hr.
toenail clippers. 5. Hold metformin (Glucophage).
2164_Ch07_579-602 29/03/12 12:32 PM Page 594

594 chapter 7 Physiological Integrity

49. A home health nurse is visiting an older client. The client 2. CORRECT ANSWER: 2. Answer 1 is incorrect because, as
asks the nurse about splitting the simvastatin tablet that long as bones and joints are stabilized, ambulation with
has been ordered. Which response should client teaching weight-bearing improves healing. Prolonged bedrest is
include? associated with many damaging complications, including
1. All medications can be split safely. deep vein thrombosis, pressure ulcers, and pneumonia.
2. Let the pharmacy split an extended-release tablet. Continuous passive motion maintains range of motion, but
3. Wash and dry the tablet-splitting device after each use. does not protect from other complications of immobility.
4. If the drug is enteric-coated, splitting changes the ther- Answer 2 is correct because early mobilization improves
apeutic response. healing and reduces complications resulting from
immobility. Because the joint is not cemented, surgeons
50. A home health nurse is making a follow-up visit to an often request less than full weight-bearing for initial
older adult following discharge from a hospital. Which
mobilization. Answer 3 is incorrect because modern
action by the nurse is most important regarding medica-
surgical procedures for hip replacement minimize damage
tion administration in the home setting?
to surrounding soft tissues; therefore, the head of the bed
1. Give the ordered medication during the visit.
can be elevated with all surgical approaches. Answer 4 is
2. Provide comprehensive teaching.
incorrect because immobilization of the hip is not
3. Count the number of pills remaining.
necessary and would result in complications of immobility.
4. Prepare a list of medications for client.
Weight-bearing, as long as the joint is stable, improves
healing.
Answers/Rationales/Tips TEST-TAKING TIP: The question asks for an activity. Only
one option describes a mobility activity.
1. CORRECT ANSWERS: 1, 3, 4, 6. Answer 1 is correct Content Area: Geriatrics, Musculoskeletal; Integrated Process:
because pneumococcus is an important cause of pneumo- Nursing Process, Planning; Cognitive Level: Application;
nia in the elderly. Clients who were vaccinated more than Client Need/Subneed: Physiological Integrity/Basic Care and
5 years previously or who were under 65 years old when Comfort/Mobility/Immobility
they received the vaccination should receive a booster.
Answer 2 is incorrect because, unless there are clients with 3. CORRECT ANSWER: 2. Answer 1 is incorrect because
developmental disabilities residing at the skilled nursing indwelling catheters are associated with a risk for infection,
facility, vaccination of residents is not necessary. Answer 3 and should not be used as a nursing convenience. Answer 2
is correct because fewer than 100 people per year become is correct because, in functional incontinence, as in
infected with tetanus in the United States, thanks to Alzheimers disease, the client is usually unable to per-
the efficiency of this vaccine. The wounds that become form self-care because of alteration in mental status.
infected are often inconsequential, such as pricking Assisting the client to use a toilet and cleaning after
your finger on a rosebush thorn. Boosters are required any incontinence are appropriate. Answer 3 is incorrect
every 10 years. Answer 4 is correct because influenza because this severe fluid restriction would dehydrate the
virus is an important cause of morbidity and mortality client and is inappropriate. Answer 4 is incorrect because
in the elderly. Although the vaccine does not always diuretics are appropriate for clients whose cardiorenal func-
prevent disease, it lessens the severity of disease that tion is compromised. There is no evidence here to support
does occur. Frequent mutation of the virus necessitates a use of this drug.
new vaccination each year preceding or during the flu TEST-TAKING TIP: Choose the option that supports normal
season. Answer 5 is incorrect because a skilled nursing functioning (toileting). Three of the choices alter intake or nor-
facility is not considered a high-risk setting for contracting mal output.
meningococcal infections. Answer 6 is correct because Content Area: Geriatrics, Integumentary; Integrated Process:
varicella vaccination should be administered once, to Nursing Process, Implementation; Cognitive Level:
ANSWERS

reduce the incidence of shingles. Answer 7 is incorrect Comprehension; Client Need/Subneed: Physiological
because, although a skilled nursing facility in the Integrity/Basic Care and Comfort/Elimination
United States is an unlikely place to contract diphtheria, 4. CORRECT ANSWER: 3. Answer 1 is incorrect because
the product most often used to provide tetanus contains the serum sodium is high. The client needs relatively more
different vaccines as well (Tdtetanus/diphtheria). water and less sodium. Answer 2 is incorrect because slowing
TEST-TAKING TIP: Know the high-risk groups for each virus the IV rate would contribute to dehydration and raise the
and the effect of aging. serum sodium further. Answer 3 is correct because hyperna-
Content Area: Geriatrics, Immunological; Integrated Process:
tremia can result from the solute load in tube feedings.
Nursing Process, Analysis; Cognitive Level: Application; Additionally, the IV contains normal saline, which
Client Need/Subneed: Health Promotion and Maintenance/
could be removed. The client needs free water to dilute
Immunizations
2164_Ch07_579-602 29/03/12 12:32 PM Page 595

Answers/Rationales/Tips 595
the sodium level down. The enteral route is appropriate Client Need/Subneed: Physiological Integrity/Basic Care and
for this delivery. Answer 4 is incorrect because a diuretic Comfort/Elimination
would contribute to dehydration and raise the serum sodium
7. CORRECT ANSWER: 1. Answer 1 is correct because
further.
the opacity and cloudiness of the lens cause blurry and
TEST-TAKING TIP: Eliminate the two answers that focus
dimmer vision. Answer 2 is incorrect because in bright
on the IV (Answers 1 and 2). Recognize the effects of tube
lights the pupil constricts and the client with cataracts has
feedings on fluid and electrolyte levels.
more difficulty seeing. Answer 3 is incorrect because the
Content Area: Geriatrics, Fluid and Electrolyte Imbalances;
presence of a halo is characteristic of glaucoma. Answer 4 is
Integrated Process: Nursing Process, Implementation;
incorrect because double vision is not a symptom of
Cognitive Level: Application; Client Need/Subneed:
cataracts.
Physiological Integrity/Pharmacological and Parenteral
TEST-TAKING TIP: Only one option describes impaired vision
Therapies/Parenteral/Intravenous Therapies
related to cataracts.
5. CORRECT ANSWER: 1. Answer 1 is correct because Content Area: Geriatrics, Sensory; Integrated Process: Nursing
the kidneys normally excrete phosphate, which is usually Process, Assessment; Cognitive Level: Comprehension;
maintained at a level of 2.5 to 4.5 mg/dL in adults Client Need/Subneed: Physiological Integrity/Physiological
(up to 6 mg/dL in infants and children). A low total Adaptation/Pathophysiology
calcium in the presence of low albumin is expected in
8. CORRECT ANSWER: 4. Answer 1 is incorrect because
kidney failure. Physiologically important nonbound
the client characteristically has a shuffling gait, and the
calcium is likely to be normal so that seizure risk is not
foot would be prone to injury with an open-toed shoe.
elevated. Answer 2 is incorrect because the significance of
Answer 2 is incorrect because the shoes should be easy
low total serum calcium must be interpreted with considera-
to put on so that the client could maintain some degree
tion of the low albumin that is seen with kidney failure.
of independence, which would be difficult to do with
Less than 50% of the plasma calcium is bound to serum
double-knotted shoes. Answer 3 is incorrect because the
protein, primarily albumin. Risk for seizures is not increased.
shoes should be easy to put on so that the client could
Answer 3 is incorrect because the phosphate level is already
maintain some degree of independence, which would be
elevated. Dialysis might be used to bring it down. Answer 4
difficult to do with high-top sneakers. Answer 4 is correct
is incorrect because a diet high in dairy products contains
because the client will be able to maintain a level of
too much phosphate to be consumed by clients with kidney
independence with a slip-on shoe. The rubber soles will
disease.
lessen the risk of falls.
TEST-TAKING TIP: Focus on the elevated phosphate level.
TEST-TAKING TIP: Visualize a shuffling gait and tremor.
Know the normal ranges of laboratory tests in order to recog-
Which shoe could the client put on independently and which
nize levels that place the client at risk.
would not contribute to falls?
Content Area: Geriatrics, Renal; Integrated Process: Nursing
Content Area: Adult Health, Neuromuscular; Integrated
Process, Analysis; Cognitive Level: Analysis; Client Need/Subneed:
Process: Nursing Process, Implementation; Cognitive Level:
Physiological Integrity/Physiological Adaptation/Fluid and
Application; Client Need/Subneed: Health Promotion and
Electrolyte Imbalances
Maintenance/Self-care
6. CORRECT ANSWER: 4. Answer 1 is incorrect because 9. CORRECT ANSWERS: 1, 3, 4. Answer 1 is correct
inadequate roughage would not cause diarrhea. Fecal
because medications and some diseases common in
impaction is likely causing the diarrhea. Fiber can be
the older adult affect salivation in 30% of clients.
added to a soft diet to prevent constipation. Answer 2 is
Answer 2 is incorrect because the older adult has difficulty
incorrect because the inactivity contributes to the potential
with high-pitched sounds. Answer 3 is correct because the
for constipation and fecal impaction, but the impaction
skin becomes more susceptible to infection and injury.
is causing the liquid stools. Answer 3 is incorrect because
Answer 4 is correct because men often have prostate
this client is more at risk to develop constipation and ANSWERS
enlargement, which leads to the need to urinate several
fecal impaction, not a GI virus. Answer 4 is correct
times during the night. Answer 5 is incorrect because pain
because the client is at risk to develop fecal impaction
sensitivity may be decreased because nerve cells are altered.
secondary to constipation. Pressure on the colonic
Answer 6 is incorrect because the temperature of the older
mucosa causes seepage of liquid stool around the area
adult is often subnormal.
of impaction.
TEST-TAKING TIP: Ask if each option will occur in most
TEST-TAKING TIP: Without a fever or positive stool culture,
clients with aging.
impaction should be ruled out.
Content Area: Geriatrics; Integrated Process: Nursing Process,
Content Area: Geriatrics, Gastrointestinal; Integrated Process:
Assessment; Cognitive Level: Analysis; Client Need/Subneed:
Nursing Process, Analysis; Cognitive Level: Application;
Health Promotion and Maintenance/Aging Process
2164_Ch07_579-602 29/03/12 12:32 PM Page 596

596 chapter 7 Physiological Integrity

10. CORRECT ANSWER: 3. Answer 1 is incorrect because intervention is needed. Answer 2 is correct because the
activating the emergency medical services (EMS) is first, fol- low blood pressure and low urine output suggest fluid
lowed by the CAB sequence (compression, airway, breath- volume deficit, which should be treated with normal
ing) of cardiopulmonary resuscitation (CPR). Answer 2 is saline. Although it is usual to expect tachycardia in
incorrect because activating the EMS is first, then the clients with hypotension, it is likely that the CHF and
CAB sequence of CPR. Answer 3 is correct because the HTN are being managed with a beta blocker, which
emergency response system needs to be activated before prevents the reflex tachycardia. Due to the clients age
beginning CPR so that support services can arrive as and heart problems, a small dose of normal saline should
quickly as possible. Answer 4 is incorrect because resuscita- be given and evaluated. Answer 3 is incorrect because,
tion of the client is a priority. Calling the family would while it is good to know these laboratory values, it is not
delay starting CPR. necessary before starting some normal saline to correct the
TEST-TAKING TIP: All answers are actions that need to be hypotension. Answer 4 is incorrect because the nurse needs
taken. Know the steps to take when a client is discovered who is to use caution when administering rapid rates of IV fluids to
unresponsive. elderly clients with CHF.
Content Area: Geriatrics, Cardiopulmonary; Integrated Process: TEST-TAKING TIP: Analysis of the data is consistent with
Nursing Process, Implementation; Cognitive Level: Analysis; shock. Choose an action that is safest for the older adult.
Client Need/Subneed: Physiological Integrity/Physiological Content Area: Geriatrics, Cardiovascular; Integrated Process:
Adaptation/Medical Emergencies Nursing Process, Analysis; Cognitive Level: Analysis;
Client Need/Subneed: Physiological Integrity/Physiological
11. CORRECT ANSWER: 4. Answer 1 is incorrect because Adaptation/Fluid and Electrolyte Imbalances
alveoli are hyper-inflated, trapping air. Answer 2 is incorrect
because CO2 increases. Hypoxemia causes the body to 14. CORRECT ANSWER: 3. Answer 1 is incorrect because,
increase red blood cell (RBC) production and hematocrit although the oxygen saturation is 100%, shivering does
(Hct) increases. Answer 3 is incorrect because hematocrit increase oxygen consumption and supplemental O2 will
will be increased in response to hypoxemia. Answer 4 is be needed, not O2 removed. Answer 2 is incorrect because
correct because emphysema is an obstructive pulmonary warm blankets need to be applied to treat the complaints
disease that traps air in the alveoli. The gas exchange is and shivering regardless of the clients temperature. Answer 3
impaired as CO2 increases. The body attempts to is correct because shivering increases oxygen consump-
improve oxygenation by producing more red blood tion. Hypothermia associated with the perioperative
cells. There are immature RBCs that result in a more period may cause arrhythmia, clotting problems,
sluggish circulation. and impaired wound healing. Answer 4 is incorrect
TEST-TAKING TIP: Review the physiological changes with because the pain level is low at 2/10 and does not require
emphysematrapped air, poor gas exchange, production of treatment.
immature RBCs to carry O2. TEST-TAKING TIP: Think client comfort. Care for the client
Content Area: Geriatrics, Respiratory; Integrated Process: first, and then check the temperature.
Nursing Process, Analysis; Cognitive Level: Analysis; Content Area: Geriatrics, Musculoskeletal; Integrated Process:
Client Need/Subneed: Physiological Integrity/Physiological Nursing Process, Implementation; Cognitive Level: Application;
Adaptation/Pathophysiology Client Need/Subneed: Physiological Integrity/Reduction of Risk
Potential/Potential for Complications from Surgical Procedures
12. CORRECT ANSWER: 1. Answer 1 is correct because and Health Alterations
these are signs of cataracts, which can affect the clients
vision. Answer 2 is incorrect because these are signs of cataracts. 15. CORRECT ANSWER: 4. Answer 1 is incorrect
A CT scan is not needed. Answer 3 is incorrect because these are because remaining in the home with support is a better
signs of cataracts. Increased lighting is generally needed to assist option. Answer 2 is incorrect because this is a decision for
with vision, not decreased lighting. Answer 4 is incorrect because the client and spouse to make. Answer 3 is incorrect because
ANSWERS

these are signs of cataracts. Management is not related to dietary this casts doubt on wifes statement. Answer 4 is correct
protein. because this action supports the client and wife in this
TEST-TAKING TIP: The stem and correct option include the decision.
same worldcloudy. TEST-TAKING TIP: Choose the option that supports the
Content Area: Geriatrics, Sensory; Integrated Process: independence of the client and family in decisions about care.
Nursing Process, Assessment; Cognitive Level: Analysis; Content Area: Geriatrics, Cardiovascular; Integrated Process:
Client Need/Subneed: Physiological Integrity/Reduction of Nursing Process, Implementation; Cognitive Level: Application;
Risk Potential/System Specific Assessments Client Need/Subneed: Safe and Effective Care Environment/
Management of Care/Referrals
13. CORRECT ANSWER: 2. Answer 1 is incorrect because
the findings for BP and urine output are not normal. 16. CORRECT ANSWER: 2. Answer 1 is incorrect because
They indicate a client who has volume depletion, and an volume is not as much of a problem as pitch. Answer 2 is
2164_Ch07_579-602 29/03/12 12:32 PM Page 597

Answers/Rationales/Tips 597
correct because higher pitched sounds typically made by Content Area: Geriatrics, Musculoskeletal; Integrated Process:
children are more difficult to hear. Answer 3 is incorrect Nursing Process, Analysis; Cognitive Level: Application;
because a motorcycle would likely be a loud, low-pitched Client Need/Subneed: Health Promotion and Maintenance/
sound. Answer 4 is incorrect because a mans voice tends to Aging Process
be a lower pitch.
20. CORRECT ANSWERS: 1, 5. Answer 1 is correct because
TEST-TAKING TIP: Three of the options are low pitched.
the actions are appropriate for the older adult and will
Only one choice is typically high pitched.
improve ventilation. Answer 2 is incorrect because lung
Content Area: Geriatrics, Sensory; Integrated Process:
capacity diminishes with age and this is not reversible.
Nursing Process, Analysis; Cognitive Level: Application;
Answer 3 is incorrect because COPD traps air, so the client
Client Need/Subneed: Health Promotion and Maintenance/
should learn pursed-lip breathing, not forced inhalation.
Aging Process
Answer 4 is incorrect because the oxygen level is higher
17. CORRECT ANSWERS: 2, 3, 4. Answer 1 is incorrect than recommended for COPD. The stimulus to breathe is
because osteoarthritis does not predispose a client to hypoxia. Answer 5 is correct because the diaphragm
decubitus ulcer or skin breakdown. The client would need should be used to facilitate inspiration and expiration
to be immobilized by the arthritis. Answer 2 is correct rather than the accessory muscles of breathing.
because impaired circulation will lead to skin break- TEST-TAKING TIP: Think about the pathophysiology involved
down and a decubitus ulcer. Answer 3 is correct because and the impact of aging.
incontinence will cause skin breakdown and a decubitus Content Area: Geriatrics, Respiratory; Integrated Process:
ulcer. Answer 4 is correct because malnutrition will Nursing Process, Planning; Cognitive Level: Application;
impair skin integrity leading to breakdown and decubiti. Client Need/Subneed: Physiological Integrity/Physiological
Answer 5 is incorrect because years of sun exposure Adaptation/Alterations in Body Systems
causes the appearance of the skin to age, but the integrity
21. CORRECT ANSWER: 1. Answer 1 is correct because,
of the skin is affected by circulation, moisture, and
by the time a UTI is diagnosed, the older adult may
malnutrition.
begin to show signs of urosepsis, with altered mental
TEST-TAKING TIP: Choose the options that directly affect
status. Answer 2 is incorrect because the older adult may be
skin integrity.
asymptomatic. Answer 3 is incorrect because aging alters the
Content Area: Geriatrics, Integumentary; Integrated Process:
thermoregulatory response, and the older adult may not
Nursing Process, Analysis; Cognitive Level: Application;
have fever with a lower urinary tract infection. Answer 4 is
Client Need/Subneed: Physiological Integrity/Reduction of
incorrect because the older adult may already have a problem
Risk Potential/Potential for Alterations in Body Systems
with frequency, as part of the aging process.
18. CORRECT ANSWER: 3. Answer 1 is incorrect because TEST-TAKING TIP: Choose the option that is different
cataracts would appear as an opaque cloudiness in the eye. from the others (not urine or fever), and unique to the
Answer 2 is incorrect because the response does not address older adult.
the cause. Answer 3 is correct because colored rings or Content Area: Geriatric, Urological; Integrated Process:
halos are characteristic of glaucoma. Answer 4 is incorrect Nursing Process, Assessment; Cognitive Level: Analysis;
because colored rings or halos would not be consistent with Client Need/Subneed: Physiological Integrity/Reduction of
head injury. Risk Potential/System Specific Assessments
TEST-TAKING TIP: Consider which condition would
22. CORRECT ANSWER: 4. Answer 1 is incorrect because
most likely cause these symptoms in the eye: cataracts
vaginitis is not associated with anticoagulation. Answer 2
or glaucoma?
is incorrect because vaginitis is associated with increased
Content Area: Geriatrics, Sensory; Integrated Process:
sexual activity or multiple partners, not just resuming sex.
Communication and Documentation; Cognitive Level: Analysis;
Answer 3 is incorrect because vaginitis occurs because of a
Client Need/Subneed: Physiological Integrity/Physiological
yeast, fungal, or bacterial infection. Poor nutrition may ANSWERS
Adaptation/Alterations in Body Systems
lead to decreased resistance. Answer 4 is correct because
19. CORRECT ANSWER: 1. Answer 1 is correct because antibiotic therapy alters the normal protective organisms
osteoarthritis is a common cause of chronic pain related and allows fungal or yeast infections to develop.
to the aging process. Answer 2 is incorrect because an TEST-TAKING TIP: More than one option may increase the
old fracture may or may not be a source of chronic pain. risk for vaginitis, but select the one that is designed to destroy
Answer 3 is incorrect because this condition is not due to organismseven the protective ones.
aging. Answer 4 is incorrect because this condition is related Content Area: Geriatrics, Genitourinary; Integrated Process:
to diabetes or liver disease, not aging. Teaching and Learning; Cognitive Level: Application;
TEST-TAKING TIP: Choose the option that can occur in any Client Need/Subneed: Physiological Integrity/Reduction of
client from normal agingnot from injury or illness. Risk Potential/Potential for Alterations in Body Systems
2164_Ch07_579-602 29/03/12 12:32 PM Page 598

598 chapter 7 Physiological Integrity

23. CORRECT ANSWER: 2. Answer 1 is incorrect because incorrect because there may be some processing problems
abduction prevents dislocation. Answer 2 is correct because following a stroke; verbal instructions may not be clear to
crossing the legs, or bringing the legs together, exerts the client.
pressure on the hip and increases the risk of dislocation. TEST-TAKING TIP: Choose the option that is different from
Answer 3 is incorrect because blood clots develop in the the others. Three options involve talk; only one shows how
veins, not arteries. Answer 4 is incorrect because acute to put on the shirt.
flexion (greater than 90 degrees) increases the risk of Content Area: Geriatrics, Cardiovascular; Integrated Process:
dislocation. Nursing Process, Implementation; Cognitive Level: Application;
TEST-TAKING TIP: Visualize the position of the client after Client Need/Subneed: Health Promotion and Maintenance/
surgery with the abductor pillow to prevent dislocation. Self-care
Think: the opposite of abductionadduction. 27. CORRECT ANSWER: 4. Answer 1 is incorrect because
Content Area: Adult Health, Musculoskeletal; Integrated
constipation is a side effect but not an imminent cause
Process: Teaching and Learning; Cognitive Level: Application;
for injury. Answer 2 is incorrect because blurred vision
Client Need/Subneed: Physiological Integrity/Reduction of Risk
is a side effect; however, the improved lighting is not the
Potential/Potential for Complications from Surgical Procedures best answer. Answer 3 is incorrect because dry mouth is a
and Health Alterations side effect but not a cause of injury. Answer 4 is correct
24. CORRECT ANSWERS: 1, 2, 3. Answer 1 is correct because tricyclic antidepressants cause orthostatic
because cold or heat will help the pain, but not flexibility. hypotension.
Answer 2 is correct because range of motion maintains TEST-TAKING TIP: Select the option that provides the greatest
flexibility and functional ability. Answer 3 is correct client safety.
because meditation may help with chronic pain. Answer 4 Content Area: Geriatrics, Vascular; Integrated Process: Nursing
is incorrect because vitamins or herbal therapies do not Process, Implementation; Cognitive Level: Application;
produce consistent results. Anti-inflammatory drugs may Client Need/Subneed: Physiological Integrity/Pharmacological
help, but exercises will be needed to maintain function. and Parenteral Therapies/Adverse Effects/Contraindications/
Answer 5 is incorrect because exercise is important to pre- Interactions
vent contractures of the hand. 28. CORRECT ANSWER: 3. Answer 1 is incorrect because
TEST-TAKING TIP: Look for actions that will manage pain clients with cardiac and respiratory problems would be
and increase flexibility. more prone to side effects, but kidney and liver problems
Content Area: Geriatrics, Musculoskeletal; Integrated Process:
will cause toxic responses. Answer 2 is incorrect because
Nursing Process, Implementation; Cognitive Level: Application; neither condition will interfere with the detoxification or
Client Need/Subneed: Physiological Integrity/Basic Care and
breakdown of drugs. Answer 3 is correct because drugs
Comfort/Non-pharmacological Comfort Interventions are detoxified by the liver or the kidney. The client would
25. CORRECT ANSWER: 1. Answer 1 is correct because be at risk for adverse drug effects when the function of
the older client who is inactive is likely to have poor the liver or kidneys is impaired. Answer 4 is incorrect
circulation and loss of subcutaneous fat. Answer 2 is because a client who has diabetes and gallbladder problems
incorrect because shivering increases body heat. Answer 3 is would not pose the greatest concern for reaction to medica-
incorrect because this client is aware of the cold, is likely tions detoxified by the liver or kidneys.
to do something about it, and therefore is less likely to TEST-TAKING TIP: Think about the role each condition has
develop a problem. Answer 4 is incorrect because fat acts in drug metabolism and detoxification.
as insulation. Content Area: Geriatrics, Pharmacology; Integrated Process:
TEST-TAKING TIP: Three options protect from hypothermia. Nursing Process, Analysis; Cognitive Level: Application;
Select the one option that would not protect the client from Client Need/Subneed: Physiological Integrity/Pharmacological
hypothermia. and Parenteral Therapies/Adverse Effects/Contraindications/
ANSWERS

Content Area: Geriatrics, Thermoregulatory; Integrated Process: Interactions


Nursing Process, Analysis; Cognitive Level: Application; 29. CORRECT ANSWER: 1. Answer 1 is correct because
Client Need/Subneed: Health Promotion and Maintenance/
Benadryl causes drowsiness. Answer 2 is incorrect
Health Screening because there is no effect from iron that would interfere
26. CORRECT ANSWER: 2. Answer 1 is incorrect because with balance. Answer 3 is incorrect because there is no
the client needs to see how to manage the right-sided effect from Robitussin that would affect the clients balance.
hemiplegia. Answer 2 is correct because demonstration Answer 4 is incorrect because Claritin is a nondrowsy allergy
is the best approach to teaching this client. Answer 3 is formula.
incorrect because the client does not need to know the TEST-TAKING TIP: Look for a drug that would interfere with
difficulties but rather the best approach. Answer 4 is balance.
2164_Ch07_579-602 29/03/12 12:32 PM Page 599

Answers/Rationales/Tips 599
Content Area: Geriatrics, Pharmacology; Integrated Process: 33. CORRECT ANSWER: 4. Answer 1 is incorrect because
Teaching and Learning; Cognitive Level: Application; Client these conditions do not cause aspiration, for which risk
Need/Subneed: Safe and Effective Care Environment/Safety factors would come from secretions, food, or fluid.
and Infection Control/Accident Prevention and Injury Answer 2 is incorrect because these conditions do not
Prevention cause contractures. Answer 3 is incorrect because these
30. CORRECT ANSWER: 2. Answer 1 is incorrect because conditions do not cause dehydration. Answer 4 is correct
the client needs to be seen in the ED. Engage the client because all of the conditions are risk factors for skin
in conversation, which will hopefully have a calming breakdowninactivity, poor circulation, and skin wet
effect. Answer 2 is correct because, unlike techniques from urine.
used with the client who is mentally ill, the nurse would TEST-TAKING TIP: Ask what the three presenting conditions
not attempt to orient the client with moderate or severe will cause. Eliminate the options where the clients condition
dementia to person, place, or thing. Answer 3 is incorrect is not a risk factor: tracheobronchial disorders (Answer 1),
because the client with moderate dementia usually cannot musculoskeletal problems (Answer 2), and fluid and electrolyte
be oriented to person, place, or thing. Answer 4 is incorrect imbalances (Answer 3).
Content Area: Geriatrics, Integumentary; Integrated Process:
because the client with moderate dementia usually cannot
be oriented to person, place, or thing. Nursing Process, Analysis; Cognitive Level: Application;
Client Need/Subneed: Physiological Integrity/Reduction of
TEST-TAKING TIP: Select an option that engages the client
without attempting to orient the client with dementia. Risk Potential/Potential for Alterations in Body Systems
Content Area: Geriatrics, Neurological; Integrated Process: 34. CORRECT ANSWER: 4. Answer 1 is incorrect because
Communication and Documentation; Cognitive Level: the problem is not due to urinary frequency. Answer 2 is
Analysis; Client Need/Subneed: Psychosocial Integrity/ incorrect because the action will not promote sleep.
Therapeutic Communications Answer 3 is incorrect because the complaint of waking

31. CORRECT ANSWER: 4. Answer 1 is incorrect because up at night is not indicative of sleep apnea. The client with
effects of aging may be interfering with increasing the sleep apnea doesnt usually wake up at night. Answer 4 is
social skills of the client; focusing on this area could cause correct because the older adult does experience changes
more difficulties in maintaining self-esteem. Answer 2 is in sleeping. It is important to rule out any pathological
incorrect because the older adult may be feeling that the patterns.
future is hopeless. Answer 3 is incorrect because the older TEST-TAKING TIP: Recognize the normal changes of aging
adult does not need praise for past accomplishments as affecting the sleep-wake cycle.
Content Area: Geriatrics; Integrated Process: Nursing
much as a focus on remembering happy events for self-
esteem. Answer 4 is correct because focusing on long- Process, Implementation; Cognitive Level: Application;
Client Need/Subneed: Health Promotion and Maintenance/
term memory helps maintain self-worth and self-esteem
in the client. Aging Process
TEST-TAKING TIP: Talking about the past is a source of com- 35. CORRECT ANSWERS: 1, 5. Answer 1 is correct
fort for the older adult, not talking about the future (Answers because the signs of pneumonia are more subtle and
1 and 2). subjective and are related to developing sepsis. Answer 2
Content Area: Geriatrics, Mental Health; Integrated Process: is incorrect because headache is not common; respiratory
Nursing Process, Implementation; Cognitive Level: Application; rate increases. Answer 3 is incorrect because the older
Client Need/Subneed: Psychosocial Integrity/Mental Health adult typically has no fever with pneumonia. Answer 4 is
Concepts incorrect because the client may have increased sputum
32. CORRECT ANSWER: 3. Answer 1 is incorrect and no chest (pleuritic) pain. Answer 5 is correct because
because this response does not provide relief. Answer 2 is sputum will be increased, causing a productive cough.
incorrect because this response would help with dryness, The older adult often has a subnormal temperature, so
that the temperature with an infection is often not ANSWERS
but offers no relief. Answer 3 is correct because this
response will lessen the dryness and relieve the itching. elevated.
Answer 4 is incorrect because this response does not
TEST-TAKING TIP: The typical sign of pneumonia is often
provide relief. not present in the older adult (high fever). Look for less specific
TEST-TAKING TIP: Look for actions that do not contribute to symptoms (Answer 1) and a choice consistent with the altered
the dryness and provide relief. thermoregulatory system in the older adult.
Content Area: Geriatrics, Respiratory; Integrated Process:
Content Area: Geriatrics, Integumentary; Integrated Process:
Nursing Process, Implementation; Cognitive Level: Application; Nursing Process, Assessment; Cognitive Level: Application;
Client Need/Subneed: Physiological Integrity/Physiological
Client Need/Subneed: Physiological Integrity/Physiological
Adaptation/Alterations in Body Systems Adaptation/Alterations in Body Systems
2164_Ch07_579-602 29/03/12 12:32 PM Page 600

600 chapter 7 Physiological Integrity

36. CORRECT ANSWER: 1. Answer 1 is correct because adequate. Learning takes time. Follow-up sessions are
these are strategies to promote better sleep. Answer 2 is important to building confidence in the client and to
incorrect because more information is needed to ensure there assess the effectiveness of prior teaching. Answer 3 is
is not a pathological cause. Answer 3 is incorrect because incorrect because the teaching techniques vary with older
more information is needed before an intervention is selected. adults. The nurse needs to assess each client. A combination
Answer 4 is incorrect because this may be a solution but more of lecture, visual aids, and discussion is commonly used.
information is needed. Answer 4 is incorrect because there is no substitute for
TEST-TAKING TIP: First priorityAssess. human interaction. Teaching aids are invaluable, but there
Content Area: Geriatrics; Integrated Process: Nursing must be discussion following use of such aids.
Process, Implementation; Cognitive Level: Application; TEST-TAKING TIP: Eliminate the options that imply a one
Client Need/Subneed: Physiological Integrity/Basic Care and size fits all approach to teaching older adults.
Comfort/Rest and Sleep Content Area: Geriatrics; Integrated Process: Teaching and
Learning; Cognitive Level: Application; Client Need/Subneed:
37. CORRECT ANSWER: 4. Answer 1 is incorrect because this Health Promotion and Maintenance/Principles of Teaching
strategy will be important but not the most effective. Answer 2 and Learning
is incorrect because the speed of the move is irrelevant. The
client will still think about the change. Answer 3 is incorrect 40. CORRECT ANSWER: 1. Answer 1 is correct because
because family will be important in making this transition. incontinence is not a normal consequence of aging.
Answer 4 is correct because it is important to create a famil- Age-related changes in the urinary tract predispose
iar environment. the older client to incontinence, but incontinence is a
TEST-TAKING TIP: Eliminate the options with the words symptom of many possible disordersnot just aging.
many (Answer 1), quickly (Answer 2), and restrict Answer 2 is incorrect because incontinence is embarrassing.
(Answer 3). The most effective approach is to make the Many clients conceal the problem and fail to seek help
environment familiar. that may reverse the problem. Answer 3 is incorrect because
Content Area: Geriatrics, Mental Health; Integrated Process: the number of vaginal births does increase the incidence
Nursing Process, Implementation; Cognitive Level: Application; of incontinence in womenyoung and old. Answer 4 is
Client Need/Subneed: Psychosocial Integrity/Mental Health incorrect because collaboration with the family, the client,
Concepts and the health-care team may reverse incontinence.
TEST-TAKING TIP: The stem asks for what is not correct
38. CORRECT ANSWERS: 1, 2, 3, 4. Answer 1 is correct information and needs further teaching by the nurse.
because family members provide a source for reinforce- Incontinence is not a fact of aging. Look for an option
ment as well as valuable assessment information that implies it is an expected consequence of aging and
about the clients living situation and learning needs. therefore is an incorrect statement that requires
Answer 2 is correct because a slower pace and more
teaching.
frequent sessions with smaller amounts of information Content Area: Geriatrics, Renal; Integrated Process: Nursing
will contribute to better understanding. Answer 3 is Process, Evaluation; Cognitive Level: Analysis; Client Need/
correct because reinforcement of the content in writing Subneed: Physiological Integrity/Basic Care and Comfort/
is an appropriate action when teaching an older adult. Elimination
Answer 4 is correct because learning improves with the
opportunity to practice. Answer 5 is incorrect because 41. CORRECT ANSWERS: 2, 3. Answer 1 is incorrect
visual problems may require special lighting, and difficulty because this will not necessarily stop the robbery attempts,
hearing compounded by background noises make a group which are targeting people who are vulnerable and may
session a less effective teaching approach. have money. Answer 2 is correct because removing the
TEST-TAKING TIP: When all of the options are accepted target (the Social Security check) reduces the risk.
teaching strategies, look for the options that accommodate the Answer 3 is correct because a dog will likely deter a
ANSWERS

sensory and possible cognitive limitations seen with aging. robber. Answer 4 is incorrect because the Social Security
Content Area: Geriatrics; Integrated Process: Teaching and check would still be delivered to the home. Answer 5 is
Learning; Cognitive Level: Application; Client Need/Subneed: incorrect because the client is at risk for a repeat home
Health Promotion and Maintenance/Principles of Teaching invasion. The nurse needs to address strategies to reduce
and Learning or eliminate the risk.
TEST-TAKING TIP: Think safety; reduce the risk of being
39. CORRECT ANSWER: 2. Answer 1 is incorrect because targeted and offer a deterrenta dog.
not everyone relates to or learns well in groups. If group Content Area: Geriatrics, Safety; Integrated Process: Caring;
teaching is used, follow-up is essential to ensure that an Cognitive Level: Application; Client Need/Subneed: Safe
individual client has attained sufficient knowledge. Answer 2 and Effective Care Environment/Safety and Infection
is correct because a single teaching strategy is rarely Control/Home Safety
2164_Ch07_579-602 29/03/12 12:32 PM Page 601

Answers/Rationales/Tips 601
42. CORRECT ANSWER: 1. Answer 1 is correct because and may increase agitation. Attempting to orient the
ideally foot care should be provided by podiatric care client to person, place, or time increases agitation in the
providers. The service is covered by insurance. Answer 2 client with dementia. Answer 2 is correct because, if the
is incorrect because the calluses should only be removed family becomes upset and angry, the clients agitation
by a podiatric specialist. This option does not indicate who and anxiety will likely increase. Answer 3 is incorrect
is doing the foot care. Answer 3 is incorrect because this because repetition will not improve the clients condition.
choice does not include the importance of being seen by a Answer 4 is incorrect because this will increase client
podiatric specialist. Answer 4 is incorrect because the older anxiety. The client with dementia cannot process rapid
adult should not cut his or her own toenails. Care should stimuli.
be provided by an expert. TEST-TAKING TIP: Think of the effect that each option will
TEST-TAKING TIP: Only one option includes the importance have on the client.
of being seen by an expert in foot care. Content Area: Geriatrics, Neurological; Integrated Process:
Content Area: Geriatrics, Integumentary; Integrated Process: Teaching and Learning; Cognitive Level: Application;
Nursing Process, Implementation; Cognitive Level: Application; Client Need/Subneed: Psychosocial Integrity/Sensory/
Client Need/Subneed: Physiological Integrity/Basic Care and Perceptual Alterations
Comfort/Personal Hygiene 46. CORRECT ANSWER: 3. Answer 1 is incorrect because
43. CORRECT ANSWER: 4. Answer 1 is incorrect because only drugs that are reversible inhibitors of cholinesterase
inactivity does not increase susceptibility. The client needs have benefitted in slowing disease progression in mild to
to be in close contact with someone who has the disease. moderate Alzheimers disease. Stage III is a late stage
Answer 2 is incorrect because smoking does not increase with severe dementia. Answer 2 is incorrect because only
susceptibility, but the potential for lung damage may be drugs that are reversible inhibitors of cholinesterase have
greater. Answer 3 is incorrect because recipients of BCG benefitted mild to moderate Alzheimers disease, not
should be less likely to contract the infection. Answer 4 is Stage III. Answer 3 is correct because families are often
correct because the spread of the infection increases in desperate to find a treatment, and there may be serious
crowded, close living conditions since TB is spread side effects. They need to discuss alternative therapies
from person to person by airborne transmission with the physician. Answer 4 is incorrect because only
droplets from talking, coughing, sneezing, laughing, drugs that are reversible inhibitors of cholinesterase have
or singing. benefitted cognition in mild to moderate Alzheimers
TEST-TAKING TIP: Think about how tuberculosis is disease, not Stage III.
spreadairborne from person to person. TEST-TAKING TIP: Look for an option that is different
Content Area: Geriatrics, Infectious Disease; Integrated Process: from the othersonly one option raises the potential risks
Nursing Process, Analysis; Cognitive Level: Comprehension; of a drug. Drug therapy does not work consistently in all
Client Need/Subneed: Safe and Effective Care Environment/ AD clients.
Safety and Infection Control/Standard/Transmission-based/ Content Area: Geriatrics, Neurological; Integrated Process:
Other Precautions Teaching and Learning; Cognitive Level: Application;
Client Need/Subneed: Physiological Integrity/Physiological
44. CORRECT ANSWER: 1. Answer 1 is correct because Adaptation/Alterations in Body Systems
the client with Alzheimers disease typically has lost
short-term memory. Understanding of the fathers 47. CORRECT ANSWERS: 1, 5, 6. Answer 1 is correct because
disease will decrease the daughters frustration. Answer 2 there is recent and remote memory impairment, as well as
is incorrect because this inappropriate action does not anomia, agnosia, and aphasia. Answer 2 is incorrect because
address daughters frustration nor does it improve communi- the client loses short-term memory and may not remember
cation. Answer 3 is incorrect because the client will not younger family members, like grandchildren. Answer 3 is
recall the conversation. Answer 4 is incorrect because the incorrect because the cognitive decline is slow and gradual.
cognitive problem is not reversible. Word games, such as Answer 4 is incorrect because these causes of cognitive impair- ANSWERS
crossword puzzles, may delay development of the disease. ment would have been ruled out before a diagnosis of AD was
TEST-TAKING TIP: Only one option gives the family member made. Answer 5 is correct because it describes anomia,
a strategy to address the frustration. which occurs with AD. Answer 6 is correct because it
Content Area: Geriatric, Neurological; Integrated Process: describes agnosia, which is consistent with AD.
Nursing Process, Implementation; Cognitive Level: Application; TEST-TAKING TIP: AD is the leading cause of dementia
Client Need/Subneed: Psychosocial Integrity/Therapeutic memory impairment as well as language disorders.
Communications Content Area: Geriatrics, Neurological; Integrated Process:
Nursing Process, Analysis; Cognitive Level: Application;
45. CORRECT ANSWER: 2. Answer 1 is incorrect because Client Need/Subneed: Physiological Integrity/Physiological
correcting the client will not increase the clients compliance, Adaptation/Pathophysiology
2164_Ch07_579-602 29/03/12 12:32 PM Page 602

602 chapter 7 Physiological Integrity

48. CORRECT ANSWERS: 1, 3, 4, 5. Answer 1 is correct incorrect because, while splitting enteric-coated tablets
because antihistamines (Solu-Medrol) may be ordered defeats the protective function of the enteric coating, it does
before the procedure because of the known allergy to not change the drug effect. Enteric-coated tablets should
seafood. Answer 2 is incorrect because the contrast dye in never be split; gastric irritation would be likely.
this procedure will have the greatest effect on the kidneys, TEST-TAKING TIP: Review the rules for safely splitting
not the liver. Answer 3 is correct because the clients age tablets.
and history of diabetes combined with the nephrotoxic Content Area: Geriatrics, Pharmacology; Integrated Process:
effects of the contrast dye may lead to acute renal failure. Teaching and Learning; Cognitive Level: Application;
Answer 4 is correct because IV hydration is a key to Client Need/Subneed: Physiological Integrity/Pharmacological
preventing acute renal failure secondary to the contrast and Parenteral Therapies/Medication Administration
dye used during catheterization. Answer 5 is correct 50. CORRECT ANSWER: 2. Answer 1 is incorrect because
because metformin is excreted in the kidneys. If the giving the medication without ensuring that the client under-
client develops renal failure secondary to the contrast stands the orders after the nurse leaves would not be the
media, the drug will accumulate and the client may best action. Answer 2 is correct because effectiveness of
develop fatal lactic acidosis. the discharge orders will depend on the client understand-
TEST-TAKING TIP: Think safety first. Protect the kidneys. ing the regimen. Answer 3 is incorrect because this may
Content Area: Geriatric, Endocrine; Integrated Process:
indicate a problem that teaching will correct. Answer 4 is
Nursing Process, Analysis; Cognitive Level: Analysis; incorrect because this will most likely have been part of the
Client Need/Subneed: Physiological Integrity/Reduction of
predischarge teaching.
Risk Potential/Therapeutic Procedures TEST-TAKING TIP: Teaching is an umbrella answer that
49. CORRECT ANSWER: 3. Answer 1 is incorrect because encompasses the other options.
this is an incorrect statement. Answer 2 is incorrect because Content Area: Geriatrics, Pharmacology; Integrated Process:
extended-release tablets should never be split. Answer 3 is Nursing Process, Implementation; Cognitive Level: Application;
correct because, if splitting can be done, an approved Client Need/Subneed: Physiological Integrity/Pharmacological
device should be used. Clean between usages. Answer 4 is and Parenteral Therapies/Medication Administration
ANSWERS
2164_Ch08_603-688 29/03/12 12:33 PM Page 603

CHAPTER 8

Physiological Integrity
Pharmacological and Parenteral Therapies

Sally Lambert Lagerquist Janice Lloyd McMillin Robyn Marchal Nelson


Kathleen E. Snider

603
2164_Ch08_603-688 29/03/12 12:33 PM Page 604

604 Conversions and Calculations in Medication Administration

CONVERSIONS Example: the order is for 50 mg of phenobarbital,


A N D C A L C U L AT I O N S which comes in 20 mg per 5 mL.
I N M E D I C AT I O N 50 mg (Desired) 5 mL (Quantity)
A D M I N I S T R AT I O N =x
20 mg (Available) 1
I. COMMON METRIC CONVERSION Solve for x:
A. Weight: 1
50 5
1 gram (gm) = 1,000,000 micrograms (mcg) x=
20 1
1 milligram (mg) = 1,000 micrograms (mcg) 4
1,000 milligrams (mg) = 1 gram (gm)
50 1
1 kilogram (kg) = 1,000 grams (gm) x=
41
B. Volume:
50
1 liter (L) = 1,000 milliliters (mL) x=
4
1 kiloliter (kL) = 1,000 liters (L)
x = 12.5 mL
1 milliliter (mL) = 1 cubic centimeter (cc)
II. APOTHECARY SYSTEM (Note: metric is preferred VI. DIMENSIONAL ANALYSIS FOR
for medication administration.) CONVERSION AND CALCULATION
A. Weight: Another format for dosage calculation that uses only one
60 grains (gr) = 1 dram (dr) equation even when conversion to like units is needed.
8 drams (dr) = 1 ounce (oz) Example with like units: penicillin 500 mg comes in
B. Liquid: 250-mg capsules. You would give _______ capsules.
60 minims = 1 fluidram (fl dr) On the left side of the equation is the drug form you are
8 fluidrams (fl dr) = 1 fluidounce (fl oz) solving for:
16 fluidounces (fl oz) = 1 pint (pt) x capsules =
2 pints (pt) = 1 quart (qt) On the right side, place the known information in a
4 quarts (qt) = 1 gallon (gal) common fraction with the numerator matching the x
III. EQUIVALENT HOUSEHOLD CONVERSIONS quantity (capsules). The amount in each capsule that is
20 drops (gtt) = 1 milliliter (mL) known is the denominator (250 mg):
1 teaspoon (tsp) = 5 milliliters (mL) x capsules =
1 capsule
1 ounce (oz) = 30 milliliters (mL) 250 mg
1 cup (C) = 240 milliliters (mL) Next set up a proportion with the information that
1 liter (L) = 1,000 milliliters (mL) matches the denominator (500 mg):
1 capsule 500 mg
IV. DETERMINING EQUIVALENCY FROM ONE x capsules =
250 mg 1
SYSTEM TO ANOTHER
500
Use a proportion. On the left side of the proportion is x=1
250
what you know to be an equivalent (e.g., 30 milliliters x = 2 capsules
equals 1 ounce). On the right side of the proportion
is what you need to determine (e.g., you want to give Example with different units: Keflin 400 mg IM q10h.
1.5 ounces and your medicine cup is in millilitersyou The drug is available in 0.5 gm per 2 mL.
would write x milliliters equals 1.5 ounces). Rewrite the 2 mL
x mL =
proportion without using the symbols. 0.5 g
Because the available drug is in grams (gm), an
Example: additional fraction is added for the conversion from grams
1 oz : 30 mL :: x mL : 1.5 oz to milliliters. Remember: the numerator of each fraction
Solve for x: must match the denominator of the fraction before.
1x = 30 1.5 2 mL 1g
x = 45 x mL =
0.5 g 1,000 mg
1.5 ounces equals 45 milliliters (mL) 2 mL 1g 400 mg
x mL =
V. ALTERNATE FORMULA FOR CALCULATION 0.5 g 1,000 mg 1
CALCULATIONS

WITH LIKE UNITS (amount of drug ordered)


D (Desired Amount) Cancel out the like abbreviations on the right side of the
Q (Quantity Available) = x equation. If the equation is correct, the only abbreviation
A (Available Amount)
2164_Ch08_603-688 29/03/12 12:33 PM Page 605

chapter 8 Physiological Integrity 605


remaining on the right side will match the abbreviation on cups can be used for older infants and children.
the left side (mL): Never pinch infant or childs nostrils to force the
2 mL 1g 400 mg child to open mouth.
x mL =
0.5 g 1,000 mg 1 D. When giving tablets or capsules (that are not
Solve for x: enteric coated), crush and mix into smallest possi-
2 400 ble amount of food or liquid to ensure that child
x=
0.5 1000 takes entire dose. Do not mix with essential food or
800 liquid (e.g., milk); select an optional food, such
x=
500 as applesauce.
x = 1.6 mL IV. OPHTHALMIC INSTILLATIONS
VII. DETERMINING FLOW RATES FOR A. Position: supine or sitting with head extended.
PARENTERAL INFUSION (see Intravenous B. For eyedrops: hold dropper 1 to 2 cm above middle
Therapy, II. B, p. 606). of conjunctival sac.
C. For eye ointment: squeeze 2 cm of ointment from
tube onto conjunctival sac.
M E D I C AT I O N D. After giving drops or ointment, encourage child to
A D M I N I S T R AT I O N : keep eyes closed briefly, to maximize contact with
I N FA N T S A N D eyes. Child should be asked to look in all directions
CHILDREN (with eyes closed) to enhance even distribution of
I. DEVELOPMENTAL CONSIDERATIONS medication.
A. Be honest. Do not bribe or threaten child to obtain E. Whenever possible, administer eye ointments at
cooperation. nap time or bedtime, due to possible blurred
B. When administering medication, do so in the least vision. Administer eyedrops prior to ointment
traumatic manner possible. (if both are ordered for the same time). Eyedrops
C. Describe any sensations child may expect to may roll off the slick surface of the eye
experience (e.g., pinch of needle during IM). ointment.
D. Explain how child can help nurse (e.g., Lie as V. OTIC INSTILLATIONS
still as you can). A. Position: head to side so that affected ear is
E. Tell child that it is OK to cry; provide privacy. uppermost.
F. Offer support, praise, and encouragement during B. For child under 3 years of age: pull pinna gently
and after giving medication. down and back.
G. Allow child opportunity for age-appropriate C. For child 3 years of age and over: pull pinna gently
therapeutic play to work through feelings and up and back.
experiences, to clarify any misconceptions, and D. After administering eardrops, encourage child to
to teach child more effective coping strategies. remain with head to side with affected ear upper-
II. SAFETY CONSIDERATIONS most, to maximize contact with entire external
A. Be absolutely sure dose you are giving is both canal to reach eardrum. Gentle massage of area in
safe (check recommended mg/kg) and accurate front of ear will facilitate entry of eardrops into
(have another nurse check your calculations). canal.
Remember: dose should generally be smaller E. If eardrops are kept in refrigerator, allow to warm
than adult dose. to room temperature before instilling.
B. Check identification band or ask parent or another VI. DERMATOLOGICAL INSTILLATIONS
nurse for childs first and last name. A. Remember: young childs skin is more permeable;
C. Restrain child to avoid injury while giving medica- therefore, there is increased risk for medication
tion; a second person is often required to help hold absorption and resultant systemic effects. Monitor
child. for systemic effects.
III. ORAL MEDICATIONS B. Apply thin layer of cream or ointment, and confine
A. Use syringe without needle to draw up medication. it to portions of skin where it is essential.
B. Position: upright or semireclining. VII. RECTAL MEDICATIONS
MEDS-CHILD

C. Place tip of syringe along the side of infants A. Prepare child emotionally and physically; rectal
tongue, and give medication slowly, in small route is invasive and embarrassing, particularly for
amounts, allowing infant to swallow. Medicine children.
2164_Ch08_603-688 29/03/12 12:33 PM Page 606

606 Intravenous Therapy/Blood Administration

B. Position: side-lying with upper leg flexed. 3. Controlled-volume setspermit accurate infusion
C. Lubricate rounded end of suppository and of measured volumes of fluids.
insert past anal sphincter with gloved fingertip a. Particularly valuable when piggybacked into
(wear double gloves when inserting rectal primary infusion.
medication). b. Solutions containing drugs can then be
D. Remove fingertip but hold childs buttocks gently administered intermittently.
together until child no longer strains or indicates 4. Y-type administration setsallow for simultaneous
urge to expel medication. or alternate infusion of two fluids.
VIII. INTRAMUSCULAR MEDICATIONS a. May contain filter and pressure unit for blood
transfusions.
A. Because the infant or child is much smaller
b. Air embolism significant hazard with this type
physically than an adult, the nurse should select
of administration set.
a shorter needle, generally 5/8 inch (infant) to
5. Positive-pressure setsdesigned for rapid infusion
1 inch (child).
of replacement fluids.
B. Preferred injection sites in infants and young
a. In emergency, built-in pressure chamber
children include the vastus lateralis and ventrog-
increases rate of blood administration.
luteal muscles. The deltoid muscle, though
b. Pump chamber must be filled at all times to
small, provides easy access and can be used in
avoid air embolism.
children over 12 months of age with adequate
c. Application of positive pressure to infusion
muscle mass.
fluids is responsibility of physician.
C. Usually 1 mL is the maximum volume that
6. Infusion pumpsused to deliver small volumes
should be administered in a single site to infants
of fluid or doses of high-potency drugs.
and children.
a. Used primarily in neonatal, pediatric, and
D. Because of vast differences in size, muscle mass,
adult intensive care units.
and subcutaneous tissue, it is especially important
b. Have increased the safety of parenteral therapy
to note bony prominences as landmarks for intra-
and reduced nursing time.
muscular injections.
D. Long-term delivery systemscentrally placed
E. Have a second adult present to help restrain
venous access catheters and ports for the
the child.
administration of drugs (e.g., chemotherapy),
F. Once the nurse has told the child he or she is to
blood and blood products, antibiotics, analgesics,
receive an injection, the procedure should be carried
antiemetics, and total parenteral nutrition
out as quickly and skillfully as possible.
(TPN). Types: Hickman/Broviac, Groshong,
venous access port (VAP).
I N T R AV E N O U S 1. Inserted under strict aseptic conditions using
THERAPY/BLOOD local or general anesthesia.
A D M I N I S T R AT I O N 2. Major concern: prevention of infection.
I. INFUSION SYSTEMS II. FLUID ADMINISTRATION
A. Plastic bag: A. Factors influencing rate:
1. Contains no vacuumneeds no air to replace 1. Clients size.
fluid as it flows from container. 2. Clients physical condition.
2. Medication can be added with syringe through a 3. Age of client.
resealable latex port. 4. Type of fluid.
a. During infusion, administration set should be 5. Clients tolerance to fluid.
completely clamped before medications are 6. Clients position.
added. B. Flow rates for parenteral infusions can be computed
b. Prevents undiluted, and perhaps toxic, dose using the following formula:
from entering administration set. gtt/mL of given set
B. Closed system: total volume/hr = gtt/min
60 min/hr
1. Requires partial vacuumhowever, only filtered
air enters container. Example: if 1,000 mL is to be infused in an
IV THERAPY

2. Medication may be added during infusion 8-hr period (125 mL/hr) and the administration
through air vent in administration set. set delivers 15 gtt/mL, the rate is 31.2 gtt/min:
C. Administration sets: 15 1
1. Standard setsdeliver 10 to 15 drops (gtt)/mL. 125 = 125 = 31.2 gtt/min
60 4
2. Pediatric or minidrop setsdeliver 60 drops (gtt)/mL.
2164_Ch08_603-688 29/03/12 12:33 PM Page 607

chapter 8 Physiological Integrity 607


C. Generally the type of fluid administration set 3. Replacement therapyinfusions to replace current
determines its rate of flow. losses in fluid and electrolytes.
1. Fluid administration setsapproximately B. Types of intravenous fluids (Table 8.1):
15 gtt/min. 1. Isotonic solutionsfluids that approximate the
2. Blood administration setsapproximately osmolarity (280 to 300 mOsm/L) of normal
10 gtt/min. blood plasma.
3. Pediatric administration setsapproximately a. Sodium chloride (0.9%)normal saline.
60 gtt/min. (1) Indications:
4. Always check information on the administration (a) Extracellular fluid replacement when
set box to determine the number of gtt/mL Cl loss is equal to or greater than
before calculating; varies with manufacturer. Na+ loss.
D. Factors influencing flow rates: (b) Treatment of metabolic alkalosis.
1. Gravitya change in the height of the infusion (c) Na+ depletion.
bottle will increase or decrease the rate of flow; (d) Initiating and terminating blood
for example, raising the bottle higher will increase transfusions.
the rate of flow, and vice versa. (2) Possible side effects:
2. Blood clot in needlestopping the infusion for (a) Hypernatremia.
any reason or an increase in venous pressure may (b) Acidosis.
result in partial or total obstruction of needle by (c) Hypokalemia.
clot due to: (d) Circulatory overload.
a. Delay in changing infusion bottle. b. Five percent dextrose in water (D5W).
b. Blood pressure cuff on, or restraints on or (1) Provides calories for energy, sparing body
above infusion needle. protein and preventing ketosis resulting
c. Client lying on arm in which infusion is being from fat breakdown.
made. (a) 3.75 calories are provided per gram
3. Change in needle positionagainst or away from of glucose.
vein wall. (b)United States Pharmacopeia (USP)
4. Venous spasmdue to cold blood or irritating standards require use of monohydrated
solution. glucose, so only 91% is actually
5. Plugged ventcauses infusion to stop. glucose.
III. FLUID AND ELECTROLYTE THERAPY (c) D5W yields 170.6 calories; D5W means
5 grams glucose/100 mL water.
A. Types of therapy:
50 3.75 = 187.5 calories/L
1. Maintenance therapyprovides water, elec-
0.91 187.5 = 170.6 calories/L
trolytes, glucose, vitamins, and in some instances
(2) Indications:
protein to meet daily requirements.
(a) Dehydration.
2. Restoration of deficitsin addition to mainte-
(b) Hypernatremia.
nance therapy, fluid and electrolytes are added to
(c) Drug administration.
replace previous losses.

Table 8.1
Commonly Used Intravenous Fluids
IV Solution Na+ (mEq/L) K+ (mEq/L) Ca++ (mEq/L) Cl (mEq/L) Lactate (mEq/L) Calories/L
0.9% NaCl (NS) 154 0 0 154 0 0
5% Dextrose (D5W) 0 0 0 0 0 170
5% Dextrose + 0.9% NaCl (D5NS) 154 0 0 154 0 170
5% Dextrose + 0.45% NaCl (D5 1/2NS) 77 0 0 77 0 170
Lactated Ringers solution 130 4 3 109 28 9
IV THERAPY

3% NaCl 462 0 0 462 0 0


5% NaCl 770 0 0 770 0 0
0.45% NaCl 69.3 0 0 69.3 0 0
10% Dextrose (D10W) 0 0 0 0 0 340
2164_Ch08_603-688 29/03/12 12:33 PM Page 608

608 Intravenous Therapy/Blood Administration

(3) Possible side effects: (b) Encourage diuresis in clients who are
(a) Hypokalemia. dehydrated.
(b) Osmotic diuresisdehydration. (c) Evaluate kidney status before instituting
(c) Transient hyperinsulinism. electrolyte infusions.
(d) Water intoxication. (2) Possible side effects:
c. Five percent dextrose in normal saline (D5NS). (a) Hypernatremia.
(1) Prevents ketone formation and loss of (b) Circulatory overload.
potassium and intracellular water. (c) Use with caution in clients who are
(2) Indications: edematous with cardiac, renal, or
(a) Hypovolemic shocktemporary hepatic disease.
measure. (d) After adequate renal function is estab-
(b) Burns. lished, appropriate electrolytes should
(c) Acute adrenocortical insufficiency. be given to avoid hypokalemia.
(3) Same side effects as normal saline. 4. Alkalizing agentsfluids used in the treatment of
d. Isotonic multiple-electrolyte fluidsused for metabolic acidosis:
replacement therapy; ionic composition a. Ringers:
approximates blood plasma. (1) Administrationrate usually not more
(1) TypesPlasmanate, Polysol, and lactated than 300 mL/hr.
Ringers. (2) Side effectsobserve carefully for signs of
(2) Indicated in: vomiting, diarrhea, excessive alkalosis.
diuresis, and burns. b. Sodium bicarbonate:
(3) Possible side effectcirculatory overload. (1) Indications:
(4) Lactated Ringers is contraindicated in (a) Replace excessive loss of bicarbonate
severe metabolic acidosis and/or alkalosis ion.
and liver disease. (b) Emergency treatment of life-threatening
(5) Same side effects as normal saline. acidosis.
2. Hypertonic solutionsfluids with an osmolarity (2) Administration:
much higher than 310 mOsm (+50 mOsm); (a) Depends on clients weight, condition,
increase osmotic pressure of blood plasma, and carbon dioxide level.
thereby drawing fluid from the cells. (b) Usual dose is 500 mL of a 1.5%
a. Ten percent dextrose in normal saline (D10NS). solution (89 mEq).
(1) Administered in large vein to dilute and (3) Side effects:
prevent venous trauma. (a) Alkalosis.
(2) Used for: nutrition and to replenish (b) Hypocalcemic tetany.
Na+ and Cl. (c) Rapid infusion may induce cellular
(3) Possible side effects: acidity and death.
(a) Hypernatremia (excess Na+). 5. Acidifying solutionsfluids used in treatment of
(b) Acidosis (excess Cl). metabolic alkalosis.
(c) Circulatory overload. a. Types:
b. Sodium chloride solutions, 3% and 5%. (1) Normal saline (see B.1. Isotonic solutions,
(1) Slow administration essential to prevent pp. 607608).
overload (100 mL/hr). (2) Ammonium chloride.
(2) Indicated in water intoxication and severe b. Administrationdosage depends on clients
sodium depletion. condition and serum laboratory values.
3. Hypotonic solutionsfluids whose osmolarity c. Side effects:
is significantly less than that of blood plasma (1) Hepatic encephalopathy in presence of
(50 mOsm); these fluids lower plasma decreased liver function because ammonia
osmotic pressures, causing fluid to enter cells. is metabolized by liver.
a. 0.45% sodium chlorideused for replacement (2) Toxic effects: irregular respirations,
when requirement for Na+ use is questionable. twitching, and bradycardia.
b. 2.5% dextrose (D) in 0.45% saline, also (3) Contraindicated with renal failure.
IV THERAPY

5% D in 0.2% NaClcommon rehydrating 6. Blood and blood products (Table 8.2).


solution. a. Indications:
(1) Indications: (1) Maintenance of blood volume.
(a) Fluid replacement when some Na+ (2) Supply red blood cells to maintain
replacement is also necessary. oxygen-carrying capacity.
2164_Ch08_603-688 29/03/12 12:33 PM Page 609

chapter 8 Physiological Integrity 609

Table 8.2
Transfusion with Blood or Blood Products
Nursing Care Plan/
Blood or Blood Product Indications Assessment: Side Effects Implementation
Whole blood 1. Acute hemorrhage 1. Hemolytic reaction 1. See Chapter 6, Table 6.9
2. Hypovolemic shock 2. Fluid overload (p. 374), for complete discus-
3. Febrile reaction sion of nursing responsibilities
4. Pyrogenic reaction 2. Protocol for checking blood
5. Allergic reaction before transfusion is begun
varies with each institution;
however, at least two RNs
must verify that the unit of
blood has been crossmatched
for a specific client
Red blood cells, packed 1. Acute anemia with hypoxia See Whole blood See Whole blood
2. Aplastic anemia
3. Bone marrow failure due to
malignancy
4. Clients who need red
blood cells but not volume
Red blood cells, frozen 1. See Red blood cells, 1. Less likely to cause antigen See Whole blood
packed reaction
2. Clients sensitized by previ- 2. Decreased possibility of
ous transfusions transmitting hepatitis
White blood cells Currently being used in 1. Elevated temperature 1. Careful monitoring of
(leukocytes) severe leukopenia with 2. Graft-versus-host disease temperature
infection (research still being 2. Must be given as soon as
done) collected
Platelet concentrate 1. Severe deficiency 1. Fever, chills Monitor temperature
2. Clients who have thrombo- 2. Hives
cytopenia and are bleed- 3. Development of antibodies
ing, with platelet counts that will destroy platelets in
below 10,000/mm3 future transfusions
Contraindications:
1. Idiopathic thrombocytopenic
purpura
2. Disseminated intravascular
coagulation (DIC)
Single-donor fresh 1. Clotting deficiency or 1. Side effects rare Use sterile, pyrogen-free filters
plasma concentrates not available 2. Heart failure
or deficiency not fully 3. Possible hepatitis
diagnosed
2. Shock
Plasma removed from 1. Shock due to loss of See Single-donor fresh plasma See Single-donor fresh plasma
whole blood (up to plasma
5 days after expiration 2. Burns
date, which is 21 days) 3. Peritoneal injury
4. Hemorrhage
5. While awaiting blood
crossmatch results
Freeze-dried plasma See Plasma removed from See Single-donor fresh plasma Must be reconstituted with sterile
whole blood water before use
Single-donor fresh 1. See Single-donor fresh See Single-donor fresh plasma 1. Notify blood bank to thaw
frozen plasma plasma about 30 min before
TRANSFUSION

2. Inherited or acquired administration


disorders of coagulation 2. Give immediately
3. Preoperatively for
hemophilia
Continued
2164_Ch08_603-688 29/03/12 12:33 PM Page 610

610 Intravenous Therapy/Blood Administration

Table 8.2
Transfusion with Blood or Blood Productscontd
Nursing Care Plan/
Blood or Blood Product Indications Assessment: Side Effects Implementation
Cryoprecipitate For hemophilia: Rare 0.55 mL of cryoprecipitate
concentrate (factor 1. Prevention concentrate has same effect on
VIIIantihemophilic 2. Preoperatively serum level as 1,600 mL of fresh
factor) 3. During bleeding episodes frozen plasma
Factors II, VII, IX, and Specific deficiencies Hepatitis Commercially prepared
X compiled
Fibrinogen (factor I) Fibrinogen deficiency Increased risk of hepatitis 1. Reconstitute with sterile water
because the hepatitis virus 2. Do not warm fibrinogen or use
combines with fibrinogen during hot water to reconstitute
fractionation 3. Do not shake
4. Must be given with a filter
Albumin or salt-poor 1. Shock due to hemorrhage, None; these are heat-treated Commercially prepared
albumin trauma, infection, surgery, products
or burns
2. Treatment of cerebral
edema
3. Low serum protein levels

(3) Supply clotting factors to maintain coagu- VI. TOTAL PARENTERAL NUTRITION (TPN):
lation properties. nutrition through a central venous line to clients who
(4) Exchange transfusion. are in a catabolic state; are malnourished and cannot
IV. INTRAVENOUS CANCER CHEMOTHERAPY tolerate food by mouth or enteral nutrition; are in
negative nitrogen balance; or have conditions that
A. Usual sites: forearm, dorsum of hand, wrist, ante-
interfere with protein ingestion, digestion, and absorp-
cubital fossa.
tion (e.g., Crohns disease, major burns, and side
B. Procedure:
effects of radiation therapy of abdomen). Least desir-
1. Normal saline infusion usually started first,
able route for nutrition.
to verify vein patency, position of needle.
Chemotherapy piggybacked into IV line A. Types of solutions:
that is running. 1. Hydrolyzed proteins (Hyprotein, Amigen).
2. Rate: usually 1 mL/min. Running slowly 2. Synthetic amino acids (Freamine).
decreases nausea, vomiting, and the degree of 3. Usual components:
vein damage. a. 3% to 8% amino acid.
3. Check vein patency every 3 to 5 minutes. b. 10% to 25% glucose.
4. If more than one drug is to be infused, normal c. Multivitamins.
saline should be infused between drugs. d. Electrolytes.
5. Never infuse against resistance. 4. Supplements that can be added:
6. Stop treatment if client reports pain at needle a. Fructose.
site. Extravasation (infiltration of toxic drugs b. Alcohol.
into tissue surrounding vessel) may be present c. Minerals: iron, copper, calcium.
7. If extravasation present, begin protocol appropri- d. Trace elements: iodine, zinc, magnesium.
ate to drug administered (e.g., flushing of line e. Vitamins: A, B, C.
with saline, applying ice or heat, local injection f. Androgen hormone therapy.
of site with antidote drugs, topical application of g. Insulin.
steroid creams). h. Heparin.
IV THERAPY

8. Once treatment is completed, remove needle, i. Fats (lipid or fat emulsions) with prolonged
apply Band-Aid, exert pressure to prevent use. Lipid emulsions are contraindicated if
hematoma formation. client has allergy to eggs or is on lipid-
containing medications such as propofol
V. COMPLICATIONS OF IV THERAPY (Diprivan).
(Table 8.3). (text continues on page 612)
2164_Ch08_603-688 29/03/12 12:33 PM Page 611

chapter 8 Physiological Integrity 611

Table 8.3
Complications of IV Therapy
Complication Assessment Nursing Care Plan/Implementation
Subjective Data Objective Data
Infiltrationfluid infusing into Pain around needle 1. Infusion rate slow 1. Stop IV
surrounding tissue rather insertion 2. Swelling, hardness, cool- 2. Apply warm towel to area
than into vessel ness, blanching of tissue 3. Restart at another site
at site of needle 4. Record
3. Blood does not return
into tubing when
bag/bottle lowered
4. Puffiness under surface
of arm
Thrombophlebitis Pain along the vein Redness, swelling around 1. Stop IV
inflammatory changes in affected area (red line) 2. Notify physician
vessel; thromboemboli 3. Cold compresses or warm towel, as
the development of venous ordered
clots within the inflamed 4. Restart in another site
vessel 5. Rest affected limb; do not rub
6. See nursing care of throm-
bophlebitis, Chapter 6, p. 434
Pyrogenic reaction 1. Headache 1. Temperature 1. Discontinue IV
contaminated equipment/ 2. Backache 2. Chills 2. Vital signs
solution 3. Nausea 3. Face flushed 3. Send equipment for culture/analysis
4. Anxiety 4. Vomiting 4. Antibiotic ointment, as ordered, at
5. BP injection site
6. Cyanosis 5. Prevention: change tubing q2448h;
meticulous sterile technique; check
for precipitation, expiration dates,
damage to containers, tubings, etc.;
refrigerate hyperalimentation fluids;
discard hyperalimentation fluids that
have been at room temperature for
812 hr and use new bag regardless
of amount left in first bag (change, to
prevent infectionexcellent medium
for bacterial growth)
Fluid overloadexcessive 1. Headache 1. Pulse, venous 1. Stop IV
amount of fluid infused; 2. Shortness of pressure 2. Semi-Fowlers position
infants/elderly at risk breath 2. Venous distention 3. Notify physician
3. Syncope 3. Flushed skin 4. Be prepared for diuretic therapy
4. Dyspnea 4. Coughing 5. Preventive measures: monitor
5. Respirations flow rate and clients response to
6. Cyanosis, pulmonary IV therapy (see Chapter 6, Fluid
edema volume excess, p. 358, for
7. Shock subjective and objective data)
Air emboliair in circulatory Loss of 1. Hypotension, cyanosis 1. Turn on left side, with head
system consciousness 2. Tachycardia down
3. Venous pressure 2. Administer oxygen therapy
4. Tachypnea 3. Medical emergencycall
physician
Nerve damageimproper Numbness: fingers, Unusual position for limb 1. Untie
position of limb during hands 2. Passive ROM exercises
infusion or tying limb down 3. Monitor closely for return of function
too tight during infusion 4. Record limb status
damage to nerve
IV THERAPY

Pulmonary embolismblood Dyspnea 1. Orthopnea 1. Slow IV to keep vein open


clot enters pulmonary 2. Signs of circulatory and (rate: 56 drops/min)
circulation and obstructs cardiac collapse 2. Notify physician
pulmonary artery 3. Medical emergency
4. Be prepared for lifesaving measures
and anticoagulation therapy
2164_Ch08_603-688 29/03/12 12:33 PM Page 612

612 Total Parenteral Nutrition

B. Administration: 3. Risk for injury related to possible complications.


1. Dosage varies with clinical condition; 1 to 2 L 4. Altered nutrition, more or less than body require-
over 24 hours at a constant IV drip rate. If ments, related to ability to tolerate parenteral
TPN is discontinued, the rate must be tapered nutrition.
over 4 to 8 hours to avoid fluid, electrolyte D. Nursing care plan/implementation:
abnormalities. 1. Goal: prevent infection.
2. Solution prepared under laminar flow hood a. Dressing change:
(usually in pharmacy); solution must be refrig- (1) Strict aseptic technique.
erated; when refrigerated, expires in 24 hours; (2) Nurse and client wear mask during
once removed from refrigerator, expires in dressing change.
12 hours. (3) Cleanse skin with solution as ordered:
3. Incompatible with most medications; check with (a) Acetone to defat the skin, destroy the
pharmacy. Give in a dedicated TPN line. Do not bacterial wall.
inject IV push medications into TPN line. (b) Iodine 1% solution as antiseptic agent.
4. Route: Must be given via central line catheter (4) Dressing changed q4872h; transparent
double- or triple-lumen catheter or infusion port polyurethane dressings may be changed
of pulmonary artery (PA) catheter inserted by weekly.
physician into internal jugular or subclavian (5) Mark with nurses initials, date and time
vein. More commonly given via peripherally of change.
inserted central catheter (PICC line), inserted by (6) Air-occlusive dressing.
specially trained IV nurses into brachial or b. Attach final filter on tubing setup, to prevent
cephalic veins. Placement must be confirmed by air embolism.
x-ray before beginning infusion. Catheter tip in c. Solution: change q24h to prevent infection.
superior vena cava or right atrium. d. Culture wound and catheter tip if signs of
5. Management of PICC line: infection appear.
a. Always wash hands before handling. e. Monitor temperature q4h.
b. Do not take BP in PICC arm. f. Use lumen line for feeding only (not for CVP
c. No needle sticks near or above PICC. If or medications).
possible draw blood from opposite arm. 2. Goal: prevent fluid and electrolyte imbalance.
d. Avoid excessive shoulder use; if sent home a. Daily weights.
with PICC, cautious use of backpacks, b. I&O.
playing basketball, shoveling, weight c. Blood glucose q6h for 24 hours using
lifting. glucometer; may need insulin coverage. If
e. Cover PICC arm before bathing/showering. normal range, change to daily.
f. If dressing becomes wet, soiled, or loose, d. Monitor Chem 20 electrolytes biweekly
change as soon as possible. initially.
g. If catheter breaks, secure with tape and call e. Infusion pump to maintain constant infusion
care provider. rate.
h. If sudden chest pain, shortness of breath 3. Goal: prevent complications.
(SOB), or gurgling sensation heard near a. Warm TPN solution to room temperature to
ear with catheter breakage, clamp or pinch prevent chills.
catheter, have client lie on left side with b. Monitor for signs of complications
head down. Call physician. (Table 8.4).
6. Side effects: (1) Infiltration.
a. Hyperosmolar coma. (2) Thrombophlebitis.
b. Hyperglycemia greater than 130 mg/dL. (3) Fever.
c. Septicemia. (4) Hyperglycemia.
d. Thrombosis/sclerosis of vein. (5) Fluid and electrolyte imbalance.
e. Air embolus. c. Have client perform Valsalva maneuver or
f. Pneumothorax. apply a plastic-coated clamp when changing
C. Analysis/nursing diagnosis: tubing to prevent air embolism.
1. Fluid volume excess, potential, related to d. Tape tubings together to prevent accidental
TPN

inability to tolerate amount and consistency separation.


of solution. E. Evaluation/outcome criteria:
2. Fluid volume deficit related to state of 1. No signs of infection.
malnutrition. 2. Blood sugar less than 130 mg/dL.
2164_Ch08_603-688 29/03/12 12:33 PM Page 613

chapter 8 Physiological Integrity 613

Table 8.4
Complications Associated with Total Parenteral Nutrition
Problem Nursing Interventions
Infection
Local infection (pain, redness, edema) Sterile dressings; administer antibiotics as ordered; general
Generalized, systemic infection (elevated temperature, WBC comfort measures
count)
Arterial Puncture
Artery is punctured instead of vein Needle is withdrawn and pressure is applied
Physician aspirates bright-red blood that is pulsating strongly
Air Embolus
Air enters venous system during catheter insertion or tubing STAT arterial blood gases (ABGs), chest x-ray, ECG
changes; or catheter/tubing pull apart Connect catheter to sterile syringes, and aspirate air
Chest pain, dizziness, cyanosis, confusion Clean catheter tip, connect to new tubing
Place client on left side with head lowered (left
Trendelenburg prevents air from going into pulmonary
artery)
Prevention: have client perform Valsalva maneuver or use
plastic-coated clamp on catheter at insertion or tubing changes
Catheter Embolus
Catheter must be checked for placement by x-ray and Careful observation of catheter
observed when removed to be sure it is intact Monitor for signs of distress
Pneumothorax
If needle punctures pleura, client reports dyspnea, chest pain May seal off or may need chest tubes

3. Electrolytes within normal limits. both positive and negative symptoms of schizo-
4. Wounds begin to heal. phrenia. Risperidone also available as long-
5. Weight: no further loss, begins to gain. acting injection (Risperdal Consta): IM q
2 wk in alternate gluteal muscle site; delayed
onset of action. Benefit: lower incidence of
PSYCHOTROPIC extrapyramidal symptoms.
M E D I C AT I O N S b. Assessmentside effects: same as for typical
I. ANTIPSYCHOTICS (also called neuroleptics, major antipsychotics but with higher incidence of
tranquilizers) weight gain.
A. Typical antipsychotics: 2. Clozapine (Clozaril).
1. Phenothiazines (prochlorperazine [Compazine], a. Usethose who do not respond to other
promazine HCl [Sparine], chlorpromazine HCl neuroleptic antipsychotic drugs; offers relief
[Thorazine], thioridazine [Mellaril], trifluoper- from schizophrenic symptoms: hallucinations,
azine HCl [Stelazine], perphenazine [Trilafon], delusions, flat affect, apathy.
triflupromazine HCl [Vesprin], fluphenazine b. Assessmentside effects:
enanthate [Prolixin]). (1) Most serious is agranulocytosis (potentially
2. Butyrophenones (haloperidol [Haldol, fatal; reversible if diagnosed within 1 to
Serenace], droperidol fentanyl citrate [Innovar]). 2 weeks of onset).
3. Thiothixenes (chlorprothixene [Taractan], (a) Symptoms of agranulocytosis:
thiothixene [Navane])chemically related to infection, high fever, chills, sore
phenothiazines, with similar therapeutic effects. throat, malaise, ulceration of
4. Dibenzoxazepinesloxapine succinate mucous membranes.
(b) Laboratory value: Discontinue drug
PSYCHOTROPIC

[Loxitane])
B. Atypical antipsychotics: with white blood cell (WBC) count less
1. Risperidone (Risperdal), olanzapine (Zyprexa), than 2,000 mcg/L or granulocyte count
quetiapine (Seroquel), ziprasidone (Geodon). less than 1,000 mcg/L.
a. Useincremental increases for first 3 days to (2) Other side effects: seizures, tachycardia,
manage psychotic symptoms. Effective for orthostatic hypotension.
2164_Ch08_603-688 29/03/12 12:33 PM Page 614

614 Psychotropic Medications

(3) Caution: must have weekly blood tests for start, after change, or dose increase, when
WBC count; do not resume clozapine used with combination of medications.
once it is discontinued, due to side effects. b. Assessment:
C. General use of antipsychotic medicationsacute (1) Vital signs: extreme temperature of
and chronic psychoses; most useful in cases of dis- 107F (leading to seizures, diaphoresis,
organization of thought or behavior; to decrease confusion, stupor, coma), fluctuating
panic, fear, hostility, restlessness, aggression, and BP, pulse: irregular, tachycardia.
withdrawal. (2) Laboratory values: increased creatine phos-
D. General assessmentside effects: phokinase (CPK), increased potassium,
1. Hypersensitivity effects: leukocytosis ( WBC).
a. Blood dyscrasiaagranulocytosis, leukopenia, (3) Renal failure.
granulocytopenia. (4)Muscular: parkinsonian rigidity (lead-
b. Skin reactionssolar sensitivity, allergic pipe skeletal muscle rigidity) that leads
dermatitis, flushing, blue-gray skin. to dyspnea and dysphagia, tremors,
c. Obstructive jaundice. dyskinesia.
2. Extrapyramidal symptoms (EPS) affecting (5) At risk: clients with organic brain disorders
voluntary movement and skeletal muscles and severe dehydration.
a. Parkinsonism (also called pseudoparkinsonism) c. Medical treatment:
tremors, cogwheel rigidity, shuffling gait, (1) Discontinue all drugs STAT.
pill-rolling, masklike facies, salivation, and (2) Institute supportive care.
difficulty starting muscular movement (3) Administer bromocriptine (Parlodel) or
(dyskinesia). dantrolene (Dantrium), dopamine
b. Dystonialimb and neck spasms (torticollis), functionenhancing substances (e.g.,
extensive rigidity of back muscles levodopa, carbidopa, bromocriptine,
(opisthotonos), oculogyric crisis, speech and amantadine).
swallowing difficulties, and protrusion of d. Health teaching: avoid overheating or dehydra-
tongue. tion; diet: fluids, fiber, hard candy.
c. Akathisiamotor restlessness, pacing, foot E. Antipsychotic agentscomparison of side effects
tapping, inner tremulousness, and agitation. (Table 8.5).
d. Tardive dyskinesia (TD)excessive blinking; F. General nursing care plan/implementation:
vermiform tongue movement; stereotyped, 1. Goal: anticipate and check for side effects.
abnormal, involuntary sucking, chewing, lick- a. Protect the persons skin from sunburn when
ing, and pursing movements of tongue and outside.
mouth; grimacing, frowning, rocking. b. For hypotension: take BP and have person
(1) Causelong-term use of high doses of lie down for 30 minutes, especially after an
antipsychotic drugs. injection.
(2) Predisposing factorsage, women, organic c. Watch for signs of blood dyscrasia: sore throat,
brain syndrome (OBS); history of electro- fever, malaise.
convulsive therapy (ECT) or use of tricyclic d. Observe for symptoms of hypothermic or
antidepressants or anti-Parkinson drugs. hyperthermic reaction due to effect on heat-
3. Potentiates central nervous system depressants. regulating mechanism.
4. Orthostatic hypotension (less with butyrophenones). e. Observe for, withhold drug for, and report
5. Anticholinergic effects (atropine-like)dry early symptoms of: jaundice and bile tract
mouth, stuffy nose, blurred vision, urinary obstruction; high fever; upper abdominal
retention, and constipation. pain, nausea, diarrhea; rash; monitor liver
6. Pigmentary retinopathyocular changes function.
(lens and corneal opacity). f. Relieve excessive mouth dryness: mouth
7. Photosensitivity. rinse, increase fluid intake, gum or hard
8. Weight gain (especially true of the atypical candy.
antipsychotic medications). g. Relieve gastric irritation, constipation:
PSYCHOTROPIC

9. Neuroleptic malignant syndrome (NMS): take with and increase fluids and roughage
a. Description: a rare complication of antipsy- in diet.
chotic drugs, with a rapid progression (1 to h. Observe for and report changes in carbohy-
3 days) and a 20% mortality rate. It is a drate metabolism (glycosuria, weight gain,
serious medical emergency for which early polyphagia); change diet.
recognition of symptoms is critical. Onset: at i. Check blood sugar levels periodically.
2164_Ch08_603-688 29/03/12 12:33 PM Page 615

chapter 8 Physiological Integrity 615

Table 8.5
Antipsychotic AgentsComparison of Side Effects
Extrapyramidal Orthostatic
Classification Name Symptoms (EPS)* Sedation* Anticholinergic* Hypotension*
Generic Trade
Typical antipsychotics
Phenothiazine Chlorpromazine Thorazine 3 4 3 4
Fluphenazine Prolixin 5 2 2 2
Perphenazine Trilafon 4 2 2 2
Prochlorperazine Compazine 4 3 2 2
Promazine Sparine 3 3 4 3
Thioridazine Mellaril 2 4 4 4
Triflupromazine Vesprin 3 4 4 3
Butyrophenone Haloperidol Haldol 5 1 1 1
Thioxanthene Thiothixene Navane 4 2 2 2
Dibenzoxazepine Loxapine Loxitane 4 3 2 3
Atypical antipsychotics Clozapine Clozaril 1 5 5 4
Olanzapine Zyprexa 1 3 2 1
Quetiapine Seroquel 1 3 2 1
Risperidone Risperdal 1 1 1 3
Ziprasidone Geodon 1 2 1 1
*Key: 1 = very low; 2 = low; 3 = moderate; 4 = high; 5 = very high.

2. Goal: health teaching. (Desyrel), amoxapine (Asendin)effective in 2 to


a. Dangers of drug potentiation with alcohol 4 weeks.
or sleeping pills. 1. Useelevate mood in depression, increase phys-
b. Advise about driving or occupations where ical activity and mental alertness; may bring
blurred vision may be a problem. relief of symptoms of depression so that client
c. Caution against abrupt cessation at high doses. can attend individual or group therapy; bipolar
d. Warn regarding dark urine (sign of jaundice, disorder, depressed; dysthymic disorder; sleep
urinary retention). disturbance; agitation.
e. Have client with respiratory disorder 2. Assessmentside effects:
breathe deeply and cough (drug is a cough a. Behavioralactivation of latent schizophrenia
depressant). (hallucinations); hypomania; suicide attempts;
f. Need for continuous use of drug and mental confusion. Withhold drug if observed.
follow-up care. b. Central nervous system (CNS)tremors,
g. Prompt reporting of hypersensitivity symp- seizures, ataxia, jitteriness, irritability.
toms: fever, laryngeal edema; abdominal c. Autonomic nervous system (ANS)dry
distention (constipation, urinary retention); mouth, nasal congestion, aggravation of
jaundice; blood dyscrasia. glaucoma (blurred vision), constipation,
G. General evaluation/outcome criteria: urinary retention, edema, paralysis, ECG
1. Behavior is less agitated. changes (flattened T waves; arrhythmia
2. Knows side effects to observe for, lessen, and/or severe in overdose).
prevent. 3. Nursing care plan/implementation:
3. Continues to use drug. a. Goal: assess risk of suicide during initial
II. ANTIDEPRESSANTS improvementcareful, close observation.
b. Goal: prevent risk of tachycardia and cardiac
A. Tricyclic antidepressants (TCAs) (imipramine
arrhythmias and orthostatic hypotensionuse
HCl [Tofranil], desipramine HCl [Norpramine,
caution with client with cardiovascular dis-
PSYCHOTROPIC

Pertofrane], nortriptyline HCl [Pamelor, Aventyl],


ease, hyperthyroidism, having ECT or surgery
trimipramine [Surmontil], clomipramine
(gradually discontinue 2 to 3 days before
[Anafranil], amitriptyline HCl [Elavil, Endep],
surgery). Monitor BP, pulse twice a day;
amitriptyline HCl/perphenazine [Triavil], pro-
ECGs, 2 to 3/wk until dose adjusted.
triptyline HCl [Vivactil], doxepin HCl [Sinequan,
(1) Avoid long hot showers or baths.
Adapin]), bupropion (Wellbutrin), trazodone
2164_Ch08_603-688 29/03/12 12:33 PM Page 616

616 Psychotropic Medications

c. Goal: observe for signs of urinary retention, con- overripe fruit, sausages, salami, bologna,
stipation: monitor I&O and weight gain liver, herring, soy sauce, meat tenderizers,
(encourage exercise). red wine, beer, caffeine, cola, yeast,
(1) Fiber diet. chocolate.
(2) Reduced calories. (3) Change position gradually to prevent
d. Goal: cautious drug use with glaucoma or history postural hypotension.
of seizures. Observe seizure precautions due to (4) Report any stiff neck, palpitations, chest
lowered seizure threshold. pain, headaches because of possible
e. Goal: health teaching. hypertensive crises (can be fatal).
(1) Advise against driving car or participating (5) Take no nonprescribed drugs.
in activities requiring mental alertness, due 3. Evaluation/outcome criteria:
to sedative effects. a. Improvement in sleep, appetite, activity,
(2) Encourage increased fluid intake and interest in self and surroundings.
frequent mouth rinsing to combat dry b. Lessening of anxiety and complaints.
mouth; use candy, ice; take with food. C. Selective serotonin reuptake inhibitors
(3) Avoid smoking, which decreases drug (SSRIs) (fluoxetine [Prozac], paroxetine [Paxil],
effects. sertraline [Zoloft], fluvoxamine [Luvox],
(4) Avoid use of alcohol and other drugs, citalopram hydrobromide [Celexa])SSRIs
due to adverse interactions, especially are generally the first-line choice because they
over-the-counter (OTC) drugs (e.g., have fewer side effects, do not require blood
antihistamines). monitoring, and are safe in overdose.
(5) Advise of delay in desired effect (2 to 1. Assessmentside effects: CNS stimulation
4 weeks). insomnia, agitation, headache (especially with
(6) Instruct gradual discontinuance to avoid Prozac); weight loss; sexual dysfunction (men:
withdrawal symptoms. impotence; women: loss of orgasm, decreased
4. Evaluation/outcome criteria: diminished libido). Other side effects are similar to TCAs
symptoms of agitated depression and anxiety. (dry mouth, sedation, nausea).
B. Monoamine oxidase inhibitors (MAOIs) 2. Nursing care plan/implementation:
(phenelzine sulfate [Nardil], isocarboxazid a. Goal: reduce insomnia, agitationtake early in
[Marplan], tranylcypromine sulfate [Parnate], day; avoid alcohol, caffeine; teach relaxation
iproniazid [Marsilid], pargiline HCl [Eutonyl], before sleep time.
nialamide [Niamid]). b. Goal: reduce headachesgive analgesics.
1. Assessmentside effects: c. Goal: prevent weight lossweigh every day or
a. Behavioralmay activate latent schizophrenia, every other day, same time and scale; do not
mania, excitement. use with clients who are anorexic.
b. CNStremors; hypertensive crisis (avoid: 3. Evaluation/outcome criteria:
cheese, colas, caffeine, red wine, beer, yeast, a. Improvement in mood and hygiene, thought
chocolate, chicken liver, and other substances and communication patterns.
high in tyramine or pressor amines [e.g., amphet- b. Has not harmed self.
amines and cold and hay fever medication]); c. No significant anticholinergic and cardiovas-
intracerebral hemorrhage; hyperpyrexia. cular side effects.
c. ANSdry mouth, aggravation of glaucoma, III. ANTIANXIETY AGENTS (also called anxiolytics,
bowel and bladder control problems; edema, minor tranquilizers) (Table 8.6)
paralysis, ECG changes (severe arrhythmia in
A. Benzodiazepines, beta-adrenergic blockers, bus-
overdose).
pirone HCl, diphenylmethane antihistamines
d. Allergic hepatocellular jaundice.
1. Useacute alcohol withdrawal, tension, and
2. Nursing care plan/implementation:
irrational fears; anxiety disorders, preoperative
a. Goal: reduce risk of hypertensive crisisdiet
sedation; have muscle relaxant and anticonvul-
restrictions of foods high in tyramine content.
sant properties.
b. Goal: observe for urinary retention
2. Assessmentside effects: hypotension,
PSYCHOTROPIC

measure I&O.
drowsiness, motor uncoordination, confusion,
c. Goal: health teaching.
skin eruptions, edema, menstrual irregularities,
(1) Therapeutic response takes 2 to 3 weeks.
constipation, extrapyramidal symptoms,
(2) Food and alcohol restrictions: avocados,
blurred vision, lethargy; increased or
bananas, raisins, licorice, chocolate, aged
decreased libido.
cheese, yogurt, sour cream, papaya, figs,
2164_Ch08_603-688 29/03/12 12:33 PM Page 617

chapter 8 Physiological Integrity 617

Table 8.6
Antianxiety Agents: Comparison
Drug Use Side Effects/Cautions
Benzodiazepines For specific stress: Drowsiness, mental alertness, BP, headache
Ativan (lorazepam) Medium acting Lethal overdose with alcohol
Klonopin (clonazepam) Long acting High risk for dependency with long-term use; taper gradually
Librium (chlordiazepoxide) Long acting over 26 wk
Serax (oxazepam) Symptoms with abrupt withdrawal: irritability,
Valium (diazepam) Short acting dizziness
Tranxene (clorazepate) Do not use in pregnancy
Xanax (alprazolam) Short acting Paradoxical reactions (euphoria, excitement)
Beta-adrenergic blocker Stage fright Hypotension
Inderal (propranolol hydrochloride) Relief of physical signs Bradycardia
of anxiety (tachycardia) No risk of dependence or abuse
Buspirone hydrochloride Long-term use Less sedation; no risk of dependence
BuSpar Delayed effect (3 wk); no CNS depression
Diphenylmethane antihistamines Safe for long-term use No risk of physical dependency or abuse; minimal toxicity
Atarax (hydroxyzine hydrochloride) May cause drowsiness
Vistaril (hydroxyzine pamoate)
Adapted from Lagerquist S: Nurse Notes: Psychiatric-Mental Health. Philadelphia, Lippincott.

3. Nursing care plan/implementation: no seizures or confusion; relief of tension, anxi-


a. Goal: administer cautiously, because drug may: ety, skeletal muscle spasm.
(1) Be habituating (causing withdrawal con- IV. ANTIMANIC AGENTS
vulsions; therefore, gradual withdrawal
A. Lithium (Eskalith, Lithane, Lithobid)effect
necessary).
occurs 1 to 3 weeks after first dose.
(2) Potentiate CNS depressants.
1. Useacute manic attack and prevention of
(3) Have adverse effect on pregnancy.
recurrence of cyclic manic-depressive episodes
(4) Be dangerous for those: with suicidal ten-
of bipolar disorders.
dencies or severe psychoses, narrow-angle
2. Assessment:
glaucoma, elderly or debilitated.
a. Side effectslevels from 1.6 to 2.0 mEq/L
b. Goal: reduce GI effectscrush tablet or take
may cause: blurred vision, tinnitus, tremors,
with meals or milk; give antacids 1 hour
nausea and vomiting, severe diarrhea,
before.
polyuria, polydipsia; ataxia. Levels greater
c. Goal: monitor effects on liver function: Periodic
than 2 mEq/L may cause: motor weakness,
liver function tests and blood counts, especial-
headache, edema, and lethargy. Levels greater
ly with upper respiratory infection, hepatic or
than 2.5 mEq/L may exhibit signs of severe
renal dysfunction.
toxicity: arrhythmias, myocardial infarction
d. Goal: reduce risk ofhypotension, respiratory
(MI), cardiovascular collapse, oliguria/anuria;
depression, phlebitis, venous thrombosis.
neurological (twitching, marked drowsiness,
e. Goal: health teaching.
slurred speech, dysarthria, athetotic move-
(1) Advise against suddenly stopping drug
ments, convulsions, delirium, stupor, coma).
(withdrawal symptoms begin in 5 to
b. Precautionscautious use with clients on
7 days).
diuretics; with abnormal electrolytes (sweating,
(2) Talk with physician if plans to be or is
dehydrated, and clients who are postopera-
pregnant or lactating.
tive); with thyroid problems, on low-salt diets;
(3) Urge to drink fluids.
with heart failure; with impaired renal function;
(4) Avoid: alcohol, OTC drugs (due to poten-
PSYCHOTROPIC

with pregnancy and lactation; risk of suicide.


tiation of other CNS depressants), and
c. Dosagetherapeutic level 0.8 to 1.6 mEq/L;
heavy smoking and caffeine.
dose for maintenance 300 to 1,200 mg/day;
(5) Problem with habituation.
toxic level greater than 2.0 mEq/L; blood
4. Evaluation/outcome criteria: decreased alcohol
sample drawn in acute phase 10 to 14 hours
withdrawal symptoms or preoperative anxiety;
after last dose, taken three times a day.
2164_Ch08_603-688 29/03/12 12:33 PM Page 618

618 Psychotropic Medications

3. Nursing care plan/implementation: 2. Assessment:


a. Goal: anticipate and check for signs and symptoms a. Trihexyphenidyl HCl:
of toxicity. (1)Side effectsanticholinergic: dry mouth,
(1) Reduce GI symptoms: take with meals. blurred vision, dizziness, nausea, constipa-
(2) Check for edema: daily weight, I&O. tion, drowsiness, urinary hesitancy or
(3) Monitor blood levels greater than retention; pupil dilation; headache;
2.0 mEq/L for side effects and signs weakness; tachycardia.
of toxicity: nausea, vomiting, diarrhea, (2) Precautionscautious use with: cardiac,
anorexia, ataxia, weakness, drowsiness, liver, or kidney disease or obstructive gas-
fine tremor or muscle twitching, slurred trointestinal-genitourinary disease, benign
speech. prostatic hyperplasia (BPH), or myasthenia
(4) Monitor results from repeat thyroid and gravis. Do not give if glaucoma present.
kidney function tests. b. Benztropine mesylateside effects: same as for
(5) Withhold drug and notify physician trihexyphenidyl HCl plus:
when 1.5 mEq/L is reached. (1) Effect on body temperature (hyperpyrexia)
(6)Monitor vital signs 2 to 3 times/day may result in life-threatening state
(pulse irregularities, hypotension, (heatstroke).
arrhythmias). (2) GI distress.
b. Goal: report fever, diarrhea, prolonged vomiting (3)Inability to concentrate, memory
immediately. difficulties, and mild confusion (often
c. Goal: monitor effect (therapeutic and toxic) mistaken for senility); drowsiness.
through blood samples taken: (4) May lead to toxic psychotic reactions.
(1) 10 to 14 hours after last dose. (5) Subjective sensationslight or heavy
(2) Every 2 to 3 days until 1.6 mEq/L is feelings in legs, numbness and tingling
reached. of extremities, light-headedness or tightness
(3) Once a week while in hospital. of head, and giddiness.
(4) Every 2 to 3 months to maintain blood 3. Nursing care plan/implementation:
levels less than 2 mEq/L. a. Goal: relieve GI distress by giving after or with
d. Goal: health teaching. meals or at bedtime.
(1) Advise client of 1- to 3-week lag time for b. Goal: monitor adverse effects.
effect. (1) Hypotension, tachycardia: check pulse,
(2) Urge to drink adequate liquids (2 to blood pressure; increased temperature;
3 L/day), ice; strict oral hygiene. decreased sweating.
(3) Report: polyuria and polydipsia. (2) Constipation and fecal impaction: add
(4) Diet: avoid caffeine, crash diets, diet roughage to diet.
pills, self-prescribed low-salt diet, alcohol, (3) Dry mouth: increase fluid intake; encour-
antacids, high-sodium foods (which age frequent mouth rinsing; offer sugarless
increase lithium excretion and reduce candy or gum, ice.
drug effect); take with meals. Use (4) Blurred vision: suggest reading glasses.
sugarless candy. (5) Dizziness: assist with ambulation; use
(5) Caution against driving, operating side rails.
machinery that requires mental (6) Urinary retention: maintain I&O.
alertness until drug is effective. c. Health teaching:
(6) Warn not to change or omit dose. (1) Avoid driving, and limit activities requiring
4. Evaluation/outcome criteria: alertness.
a. Changed facial affect. (2) Delayed drug effect (2 to 3 days).
b. Improved posture, ability to concentrate, (3) Potential abuse due to hallucinogenic effects.
sleep patterns; mood is stabilized. (4) Avoid alcohol and other CNS depressants;
c. Assumption of self-care. avoid hot weather.
d. No signs of lithium toxicity. (5) Take with food.
PSYCHOTROPIC

V. ANTIPARKINSONIAN AGENTS (6) Do not stop drug abruptly.


4. Evaluation/outcome criteria:
A. Trihexyphenidyl HCl (Artane) and benztropine
a. Less rigidity, drooling, and oculogyric crisis.
mesylate (Cogentin).
b. Improved gait, balance, posture.
1. Usecounteract drug-induced extrapyramidal
c. Has not experienced symptoms of
reactions.
hyperthermia.
M I N D - A LT E R I N G S U B S T A N C E S
Major substances used by the public to alter mental states are compared in Table 8.7.

Table 8.7
Major Substances Used for Mind Alteration
Usual Single Adult Legitimate Medical
Official Name Slang Name Dose/Duration UsesPresent and Projected Short-Term Effects Long-Term Effects
1
Alcoholwhisky, gin, Booze, hooch, suds, 1 /2 oz gin or whiskey, Rare: sometimes used as a CNS depressant; relax- Diversion of energy and
beer, wine moonshine, firewater, 12 oz beer/24 hr sedative (for tension) ation (sedation); euphoria; money from more cre-
nightcap drowsiness; impaired: ative and productive
judgment, reaction time, pursuits; habituation;
2164_Ch08_603-688 29/03/12 12:33 PM Page 619

coordination, and possible obesity with


emotional control; chronic excessive use;
frequent aggressive irreversible damage to
behavior and driving brain and liver; addiction
accidents; loss of with severe withdrawal
inhibitions; slurred illness (delirium tremens
speech [DTs]) with heavy use;
many deaths
Caffeinecoffee, tea, Java, mud, brew 12 cups, 1 bottle, Mild stimulant; treatment of CNS stimulant; increased Sometimes insomnia,
cola, No-Doz, APC 5 mg/24 hr some forms of coma alertness; reduction of restlessness, or gastric
fatigue irritation; habituation
Nicotine (and coal tar) Fags, nails, weeds, butts, 12 cigarettes/12 hr None (used as an CNS stimulant; relaxation Lung (and other) cancer,
cigarettes, cigars, pipe chaw, cancer sticks insecticide) or distraction heart and blood vessel
tobacco, snuff disease, cough, etc.;
higher infant mortality
rate; many deaths; habit-
uation; diversion of ener-
gy and money; air pollu-
tion; fire
SEDATIVES Downers
chapter 8

Alcoholsee above
Barbiturates Barbs, blue, angel, blue 50100 mg Treatment of insomnia and CNS depressants; sleep Irritability, weight loss,
amobarbital (Amytal, devils, yellow jackets, tension induction; relaxation addiction with severe
Tuinal), pentobarbital yellow birds, dolls, (sedation); sometimes withdrawal illness (like
Induction of anesthesia
(Nembutal), secobarbital red devils, red birds, euphoria; drowsiness; DTs); diversion of energy
(Seconal), phenobarbital phennies, goofers, jelly As an anticonvulsant impaired: judgment, reac- and money; habituation,
beans, tooies tion time, coordination, addiction
and emotional control;
relief of anxiety/tension;
muscle relaxation
Glutethimide (Doriden) Gorilla pills, CBs, D 500 mg
Physiological Integrity

Chloral hydrate (Noctec) Peter, Mickey 500 mg


Meprobamate (Miltown, Dolls, dollies 400 mg/4 hr*
619

Equanil)
Continued
MIND-ALTERING
MIND-ALTERING

620

Table 8.7
Major Substances Used for Mind Alterationcontd
Usual Single Adult Legitimate Medical
Official Name Slang Name Dose/Duration UsesPresent and Projected Short-Term Effects Long-Term Effects
STIMULANTS Uppers
Caffeinesee above
Nicotinesee above
Amphetamines Pep pills, wake-ups, 2.515.0 mg Treatment of: obesity, CNS stimulants; Restlessness, weight
Methamphetamine Bennies, splash, narcolepsy, fatigue, increased alertness; loss, toxic psychosis
2164_Ch08_603-688 29/03/12 12:33 PM Page 620

(Methedrine, Pesoxyn) peaches depression, hyperkinesia reduction of fatigue; loss (mainly paranoid);
Water crystal, speed, of appetite; insomnia; diversion of energy
meth often euphoria, agitation and money; habituation;
Mind-Altering Substances

extreme irritability, toxic


psychosis
Dextroamphetamine Dexies, uppers Symptoms of overdose:
(Dexedrine) Diet pills 25 mg hallucinations, convul-
sions; cardiac, respiratory
failure; coma; pulmonary
edema
Phenmetrazine HCl Coke, snow, blow, happy Variable/4 hr* Anesthesia of the eyes and
(Preludin) gold dust, toot, lady, throat
Cocaine flake, crack, cecil, girl, C
TRANQUILIZERS
Chlordiazepoxide Green and whites, 525 mg Treatment of: anxiety, Selective CNS depres- Sometimes drowsiness,
(Librium) roaches tension, alcoholism, sants; relaxation, relief of dryness of mouth,
Phenothiazines: neurosis, psychosis, anxiety/tension; suppres- blurring of vision,
Chlorpromazine HCl 1050 mg psychosomatic disorders, sion of hallucinations or skin rash, tremor;
(Thorazine) and vomiting delusions, improved occasionally jaundice,
Perchlorperazine 510 mg functioning agranulocytosis, or
(Compazine) death
Trifluoperazine HCl 25 mg
(Stelazine)
Reserpine (rauwolfia) 0.100.25 mg/46 hr*
Marijuana or cannabis Pot, grass, Texas tea, Variable1 cigarette or Treatment of: depression, Relaxation, euphoria, Memory impairment;
weed, stuff, joint, reefers, pipe, or 1 drink or cake tension, loss of appetitie, increased appetite; some possible diversion of
Mary Jane, MJ, hay, (India)/4 hr* high blood pressure, nausea alteration of time percep- energy and money;
locoweed and vomiting from tion; possible impairment habituation; occasional
chemotherapy of judgment and coordi- acute panic reactions
nation; mixed CNS (with paranoia, delu-
depressant-stimulant; sions, hallucinations)
lowered inhibitions
Hashish Hash, rope, Sweet Lucy
ANTIDEPRESSANTS
Dibenzazepine 25 mg, 10 mg Treatment of moderate to Relief of depression Basically the same as
(imipramine [Tofranil], severe depression (elevation of mood), tranquilizers above
amitriptyline HCl [Elavil]) stimulation
MAO inhibitors 10 mg, 15 mg/46 hr*
(phenelzine sulfate
[Nardil], tranylcypromine
sulfate [Parnate])
NARCOTICS (OPIATES, ANALGESICS)
Opium Op, black, poppy, tar, 1012 pipes Treatment of severe pain, CNS depressants; Constipation, loss of
Big O (Asia)/4 hr* diarrhea, and cough sedation, euphoria, relief appetite and weight,
of pain, impaired intellec- temporary impotency
2164_Ch08_603-688 29/03/12 12:33 PM Page 621

tual functioning and or sterility; habituation,


coordination addiction with unpleas-
ant and painful with-
drawal illness
Heroin Horse, H, smack, shit, Variablebag or paper
junk, brown, pcag, TNT with 5%10% heroin
Morphine (Astramorph) M, white stuff, Miss 1015 mg
Emma
Codeine Terp, robo, romo, syrup 1530 mg Topical anesthetic
Aspirin/oxycodone HCl Perks, Perkies 1 tablet
(Percodan)
Meperidine HCl (Demerol) Doctors 50100 mg
Methadone (Dolophine) Dollies, done 2.540.0 mg
Cough syrups (Cheracol, 24 oz (for euphoria)/
Hycodan, Romilar, etc.) 46 hr*
Dilaudid DLs, 4s, lords, little D
HALLUCINOGENS
chapter 8

LSD (Lysergic acid Acid, sugar cube, trip, 150 mcg/1012 hr Experimental study of mind Production of visual Usually none; sometimes
diethylamide) big D and brain function; imagery, increased precipitates or intensifies
enhancement of creativity sensory awareness, an already existing psy-
and problem-solving; treat- anxiety, nausea, impaired chosis (hallucinations);
ment of alcoholism, mental coordination; sometimes more commonly can
illness, and reduction of consciousness expansion produce a panic reaction
intractable pain in the (extreme hyperactivity)
person who is dying;
chemical warfare
Psilocybin Magic mushrooms, Gods 25 mg
flesh, rooms
Physiological Integrity

STP Serenity, tranquility, peace 6 mg


DMT Businessmans Trip
621

Continued

MIND-ALTERING
MIND-ALTERING

622

Table 8.7
Major Substances Used for Mind Alterationcontd
Usual Single Adult Legitimate Medical
Official Name Slang Name Dose/Duration UsesPresent and Projected Short-Term Effects Long-Term Effects
2164_Ch08_603-688 29/03/12 12:33 PM Page 622

Mescaline (peyote) Cactus, mesc, mescal, 350 mg/1214 hr


half moon, big chief, bad
seed
Mind-Altering Substances

PCP (Phencyclidine) Angel dust, hog, rocket


fuel
MDMA Ecstasy, XTC, Adam
MISCELLANEOUS
Glue, gasoline, and Variable None except for antihista- When used for mind alter- Variablesome of the
solvents mines used for allergy and ation generally produces substances can serious-
amyl nirite for fainting a high (euphoria) with ly damage the liver or
impaired coordination and kidney, and some pro-
judgment duce hallucinations
Amyl nitrite Pearls, poppers 12 ampules
Antihistamines 2550 mg
Nutmeg Variable/2 hr
Nonprescription
sedatives (Compoze)
Catnip
Nitrous oxide
*Time given pertains to all drugs listed.
Hashish or charas is a more concentrated form of the active ingredient THC (tetrahydrocannabinol) and is consumed in smaller doses, analogous to vodka-beer
ratios.
Copyright Joel Fort, MD, author of Alcohol: Our Biggest Drug Problems (McGraw-Hill) and The Pleasure Seekers (Grove Press); founder, The National Center for
Solving Special and Health ProblemFORT HELP and the Violence Prevention Program, San Francisco; and formerly Lecturer, School of Criminology, University
of California, Berkeley, and Consultant, World Health Organization. Used by permission.
2164_Ch08_603-688 29/03/12 12:33 PM Page 623

chapter 8 Physiological Integrity 623

A B S O R P T I O N R AT E S B Y D I F F E R E N T R O U T E S
Table 8.8 lists the rates of absorption in a healthy person with normal perfusion by various routes.

Table 8.8
Rates of Absorption by Different Routes
Route of Administration Time Until Drug Takes Effect*
Topical Hours to days
Oral 3090 min
Rectal 530 min (unpredictable)
Subcutaneous injection 1530 min
Intramuscular injection 1020 min
Sublingual tablet 35 min
Inhalation 3 min
Endotracheal 3 min
Intravenous 3060 sec
Intracardiac 15 sec
*In a healthy person with normal perfusion.

REGIONAL ANALGESIA-ANESTHESIA IN LABOR


AND BIRTH
The characteristics and nursing implications for regional analgesia-anesthesia in labor and delivery are detailed in Table 8.9.

Table 8.9
Regional Analgesia-Anesthesia for Labor and Birth
Types Characteristics Nursing Implications
Common Agents in 0.5%1.0% Solution
Lidocaine (Xylocaine) Used with epinephrine (or other vasocon- Note any history of allergy; note response:
Bupivacaine HCl (Marcaine) strictor drug) to delay absorption, prolong allergic reaction, hypotension, and lack of
Tetracaine HCl (Pontocaine) anesthetic effect, and decrease chance of wearing off of anesthetic effect; observe
Mepivacaine HCl (Carbocaine) hypotension for hypertensive crisis if agent combined
Chloroprocaine HCl (Nesacaine) with epinephrine and oxytocin is also
Ropivacaine (Norapin) being given
Peripheral Nerve Block
Pudendal (510 mL each side) Perineal anesthesia of short duration To get cooperation, give explanation
anesthetizes lower two thirds of (30 min); local anesthesia; simple and during procedure
vagina and perineum safe; does not depress neonate; may
inhibit bearing-down reflex
Paracervical (uterosacral) block When the cervix is dilated, may be given Explain: especially length and type of
(510 mL given into each side) between 3 and 8 cm by physician when needle; take maternal vital signs and fetal
anesthetizes cervix and upper two woman is having at least 3 contractions in heart rate (FHR); have her void; help posi-
thirds of vagina; note: used more 10 min; lasts 4590 min; can be repeated; tion; monitor FHR continuously; monitor
for gynecological surgery than for can be followed by local, epidural, or other; contractions; and watch for return of pain
labor may cause temporary fetal bradycardia
Local infiltration Useful for perineal repairs No special nursing care
LABOR-BIRTH

Epidural lumbar block Useful during first and second stages; can Hypotension (with resultant fetal
be given one shot or continuously; T10 to bradycardia): (a) turn from supine to
S5 for vaginal birth; T8 to S1 for abdominal lateral, or elevate legs, (b) administer
birth; complete anesthesia for labor and humidified oxygen by mask at 810 L/min,
birth (c) increase rate of IV fluids (use infusate
without oxytocin); will need coaching to
push, and low forceps or vacuum may be
required
Continued
2164_Ch08_603-688 29/03/12 12:33 PM Page 624

624 Food/FluidMedication Interactions

Table 8.9
Regional Analgesia-Anesthesia for Labor and Birthcontd
Types Characteristics Nursing Implications
Peripheral Nerve Block
Subarachnoid spinal
Continuous Useful during first or second stage of labor, Instruct when to bear down
or for abdominal surgery
Low spinal (saddle, one Same as continuous Same as continuous
shot) block
Intrathecal (spinal): morphine, 0.5 mg produces marked analgesia for Side effects: respiratory depression, pruri-
fentanyl, sufentanil 1224 hr; onset in 2030 min tus, nausea, vomiting, sleepiness, urinary
Fentanyl and sufentanil produce short- retention; keep naloxone 0.4 mg at bed-
acting analgesia (11/231/2 hr) side and respiratory support equipment
readily available

F O O D / F L U I D M E D I C AT I O N I N T E R A C T I O N S
Food and fluid considerations with various drugs are detailed in Table 8.10.

Table 8.10
Food and Fluid Considerations with Drugs
Key:
1. Take with food or milk Co-trimoxazole 4, 6 Indinavir 2, 4
2. Take on empty stomach (1 hr ac or Cyclosporine 1 Indomethacin 1
2 hr pc) Isoniazid 2
3. Dont drink milk or eat dairy products D
4. Take with full glass of water K
Demeclocycline 2, 3
5. Take 1/2 hr before meals Diltiazem 1 Keflex 6
6. Take with or without food Diuril 1 Ketoconazole 1
Donnatal 5 Ketoprofen 1
A
Dopar 1 K-Lor 1
Albuterol 1 Doxycycline hyclate 3, 6
Allopurinol 1 Dulcolax 3 L
Aminophylline 1 Levodopa 1
Amiodarone 1 E
Librium 1
Amlodipine 6 Ecotrin 3 Lisinopril 6
Amoxicillin 6 Enalapril 6 Lovastatin 1
Ampicillin 2 Erythromycin stearate 2
Aspirin 1 Ethosuximide 1 M
Augmentin 6 Etodolac 2 Macrodantin 1
Azo Gantrisin 4, 6 Marax 1
F
B Metronidazole 1
Famotidine 6 Mexiletine 1
Baclofen 1 Feldene 1 Mycophenolate 2
Bactrim 4, 6 Ferrous sulfate 3
Belladonna 5 Flecainide 6 N
Benemid 1, 4 Fluoxetine 6 Nafcillin 2
Bisacodyl 3 Naproxen 4
G
C Neostigmine 5
Gantrisin 4, 6 Niacin 1
Captopril 2 Gemfibrozil 5 Nicardipine 5
Cefaclor 6 Glycopyrrolate 5 Nitrofurantoin 1
Cefuroxime axetil 6
FOOD-FLUID

Cephalexin 6 H P
Chlorothiazide 1 Hismanal 2 Parlodel 1
Cholestyramine 5 Hydrocodone/acetaminophen 1 Penicillamine 2
Cimetidine 1 Hydromorphone 1 Penicillin G 2
Ciprofloxacin 4, 6 Penicillin V 2
Clonazepam 1 I Persantine 2
Clopidogrel 6 Phenylbutazone 1
Ibuprofen 1, 2
Cloxacillin 2 Phenytoin 1
Ilosone 6
2164_Ch08_603-688 29/03/12 12:33 PM Page 625

chapter 8 Physiological Integrity 625

Table 8.10
Food and Fluid Considerations with Drugscontd
Key:
Prednisone 1 S Theophylline 1, 4
Pro-Banthine 5 Ticlopidine 1
Simvastatin 6
Probenecid 4 Trimethoprim 2
Sinemet 1
Q Slow-K 1 V
Sucralfate 2
Quinidine 1 Sulfisoxazole 4, 6 Vasotec 6
R T Z
Ranitidine 6 Tedral 1 Zestril 6
Rifampin 2 Tetracycline 2, 3
Robinul 5

A N T I N E O P L A S T I C D R U G C L A S S I F I C AT I O N S
Table 8.11 provides a classification of the antineoplastic drugs.

Table 8.11
Antineoplastic Drug Classifications
I. Alkylating agents IV. Cellular growth factors
A. Alkyl sulfonates A. Filgrastim (Neupogen)
1. Busulfan (Myleran) B. Sargramostim (Leukine, Prokine)
B. Ethylenimines V. Enzymes
1. Thiotepa A. L-Asparaginase (Elspar)
C. Nitrogen mustards VI. Hormonal agents
1. Chlorambucil (Leukeran) A. Androgens/antiandrogens
2. Cyclophosphamide (Cytoxan) 1. Flutamide (Eulexin)
3. Ifosfamide (Ifex) B. Estrogens/antiestrogens
4. Mechlorethamine hydrochloride (Mustargen, HN2, 1. Estramustine phosphate sodium (Emcyt)
nitrogen mustard) 2. Tamoxifen citrate (Nolvadex)
5. Melphalan (Alkeran, L-PAM, L-phenylalanine mustard) C. Glucocorticoids
D. Nitrosoureas D. Luteinizing hormonereleasing hormone (LH-RH)
1. Carmustine (BCNU, BiCNU) antagonists
2. Lomustine (CCNU, CeeNu) 1. Buserelin (Suprefact)
3. Semustine (methyl-CCNU) 2. Leuprolide (Lupron)
4. Streptozocin (Zanosar, streptozotocin) E. Octreotide acetate (Sandostatin)
E. Triazenes VII. Immunomodulating agents
1. Dacarbazine (DTIC-Dome) A. Interferons
II. Antibiotics 1. Interferon alfa-2a (Roferon-A)
A. Anthracyclines 2. Interferon alfa-2b (Intron A)
1. Daunorubicin (daunomycin, Cerubidine) B. Interleukins: aldesleukin (interleukin-2, IL-2, Proleukin)
2. Doxorubicin hydrochloride (Adriamycin) C. Levamisole (Ergamisol)
3. Idarubicin (Idamycin) VIII. Miscellaneous agents
B. Bleomycins A. Carboplatin (Paraplatin)
1. Bleomycin sulfate (Blenoxane) B. Cisplatin (cis-platinum II, Platinol)
C. Dactinomycin (actinomycin D, Cosmegen) C. Hexamethylmelamine (HMM)
D. Mitomycin (mitomycin C, Mutamycin) D. Hydroxyurea (Hydrea)
E. Plicamycin (Mithracin) E. Mitotane (o,p-DDD, Lysodren)
III. Antimetabolites F. Mitoxantrone (Novantrone)
A. Folate antagonist G. Procarbazine (N-methylhydrazine, Matulane, Natulan)
1. Methotrexate (Folex, Mexate) IX. Monoclonal antibodies
ANTINEOPLASTICS

B. Purine analogues X. Plant-derived products


1. Cladribine (2-chlorodeoxyadenosine, Leustatin) A. Epipodophyllotoxins
2. Fludarabine (Fludara) 1. Etoposide (VePesid)
3. Mercaptopurine (6-MP, Purinethol) 2. Teniposide (Vumon)
4. Pentostatin (deoxycoformycin, Nipent) B. Taxanes: paclitaxel (Taxol)
5. Thioguanine (6-TG, 6-thioguanine) C. Vinca alkaloids
C. Pyrimidine analogues 1. Vinblastine (Velban)
1. Cytarabine (cytosine arabinoside, Cytosar-U, ara-C) 2. Vincristine (Oncovin)
2. Fluorouracil (5-FU, 5-fluorouracil)
2164_Ch08_603-688 29/03/12 12:33 PM Page 626

626 Common Side Effects of Chemotherapeutic Agents

COMMON SIDE EFFECTS OF CHEMOTHERAPEUTIC


AGENTS
The common side effects of various chemotherapeutic agents are detailed in Table 8.12.

Table 8.12
Common Side Effects of Chemotherapeutic Agents*
Nausea and Vomiting Mucositis Diarrhea Skin Reactions Lung Neurological
Alkylating Agents
Busulfan 0 + 0 Hyperpigmentation 0 0
Carboplatin + 0 0 0 0 0
Chlorambucil 0 0 0 0 + 0
Cisplatin + 0 0 Alopecia 0 +
Cyclophosphamide + 0 0 0 + 0
DTIC + 0 0 0 0 0
Ifosfamide + 0 0 Alopecia 0 Encephalopathy
Mechlorethamine + + + Alopecia, rash 0 0
Melphalan 0 0 0 0 0 0
Antimetabolites
2-Chlorodeoxyadenosine + 0 0 0 0 0
Cytosine arabinoside 0 0 + 0 + 0
2-Deoxycoformycin + 0 0 Erythema 0 Lethargy, coma
Fludarabine + 0 0 0 0 0
5-Fluorouracil 0 + + Phlebitis 0 Cerebellar
Hydroxyurea 0 0 0 Skin atrophy 0 0
6-Mercaptopurine + 0 0 Rash 0 0
Methotrexate + + + Dermatitis + 0
with leucovorin + + + Dermatitis + 0
Thioguanine + + 0 0 0 0
Antitumor Antibiotics
Bleomycin 0 0 0 Erythema + 0
Dactinomycin + + 0 Alopecia, rash 0 0
Daunorubicin + + + Alopecia, vesicant 0 0
Doxorubicin + + + Alopecia, vesicant 0 0
Idarubicin + + + Alopecia, vesicant 0 0
Mitomycin C + + 0 Vesicant + 0
Mitoxantrone + + + Alopecia, vesicant 0 0
Nitrosoureas
Carmustine (BCNU) + 0 0 0 + 0
Lomustine (CCNU) + 0 0 0 0 0
Streptozocin + 0 0 0 0 0
CHEMOTHERAPY

Thiotepa + + + Alopecia, rash 0 0


Plant Alkaloids
Etoposide (VP-16) 0 0 0 0 0 0
Taxol + 0 0 Alopecia 0 Neuropathy
Vinblastine + + 0 Alopecia, vesicant 0 +
Vincristine 0 0 0 Vesicant 0 + Neuropathy
*Key: +, present; 0, absent.
Adapted from Ewald G, McKenzie C: Manual of Medical Therapeutics. Little, Brown, Boston (out of print).
2164_Ch08_603-688 29/03/12 12:33 PM Page 627

chapter 8 Physiological Integrity 627

PROPERTIES OF SELECTED ANALGESIC AGENTS


Table 8.13 lists the properties of some nonopioid and nonsteroidal analgesic agents.

Table 8.13
Properties of Selected Analgesic Agents
Specific Group Analgesic Antipyretic Anti-inflammatory
Nonopioid
Acetaminophen * * 0
Fenoprofen * * *
Flurbiprofen * * *
Ibuprofen * * *
Ketoprofen * * *
Naproxen * * *
Nonsteroidal
Aspirin * * *
Diclofenac * * *
Diflunisal * 0 *
Etodolac * 0 *
Indomethacin * * *
Meclofenamate * 0 *
Nabumetone * * *
Piroxicam * * *
Sulindac * 0 *
Tolmetin * * *
* = possesses the property assigned; 0 = lacks the property assigned.

EQUIANALGESIC DOSING FOR OPIOID ANALGESICS


Table 8.14 provides information on equianalgesic dosing for the opioid analgesics.

Table 8.14
Equianalgesic Dosing for Opioid Analgesics
Approximate Equianalgesic Recommended Starting Dose Recommended Starting Dose
Dose (adults >50 kg body weight) (children and adults >50 kg body weight)*
Drug Oral Parenteral Oral Parenteral Oral Parenteral
Opioid Agonist
Morphine 30 mg q34h 10 mg q34h 30 mg q34h 10 mg q34h 0.3 mg/kg q34h 0.1 mg/kg q34h
(around-the-
clock dosing)
60 mg q34h
(single dose
or intermittent
dosing)
Codeine 180200 mg 130 mg q34h 60 mg q34h 60 mg q2h 1 mg/kg q34h Not recommended
ANALGESICS

q34h (intramuscular/
subcutaneous)
Hydrocodone 30 mg q34h Not available 10 mg q34h Not available 0.2 mg/kg q34h Not available
(in Lorcet,
Lortab, Vicodin,
others)
Continued
2164_Ch08_603-688 29/03/12 12:33 PM Page 628

628 Equianalgesic Dosing for Opioid Analgesics

Table 8.14
Equianalgesic Dosing for Opioid Analgesicscontd
Approximate Equianalgesic Recommended Starting Dose Recommended Starting Dose
Dose (adults >50 kg body weight) (children and adults >50 kg body weight)*
Drug Oral Parenteral Oral Parenteral Oral Parenteral
Opioid Agonist
Hydromorphone 7.5 mg q34h 1.5 mg q34h 6 mg q34h 1.5 mg q34h 0.06 mg/kg q34h 0.015 mg/kg q34h
(Dilaudid)
Levorphanol 4 mg q68h 2 mg q68h 4 mg q68h 2 mg q68h 0.04 mg/kg q68h 0.02 mg/kg q68h
(Levo-Dromoran)
Meperidine 300 mg q23h 100 mg q3h Not 100 mg q3h Not recommended 0.75 mg/kg q23h
(Demerol) recommended
Methadone 20 mg q68h 10 mg q68h 20 mg q68h 10 mg q68h 0.2 mg/kg q68h 0.1 mg/kg q68h
(Dolophine,
others)
Oxycodone 30 mg q34h Not available 10 mg q34h Not available 0.2 mg/kg q34h Not available
(Roxicodone,
also in Percocet,
Percodan, Tylox,
others)
Oxymorphone Not available 1 mg q34h Not available 1 mg q34h Not recommended Not recommended
(Numorphan)
Opioid Agonist-Antagonist and Partial Agonist
Buprenorphine Not available 0.30.4 mg Not available 0.4 mg q68h Not available 0.004 mg/kg
(Buprenex) q68h q68h
Butorphanol Not available 2 mg q34h Not available 2 mg q34h Not available Not recommended
(Stadol)
Dezocine Not available 10 mg q34h Not available 10 mg q34h Not available Not recommended
(Dalgan)
Nalbuphine Not available 10 mg q34h Not available 10 mg q34h Not available 0.1 mg/kg q34h
(Nubain)
Pentazocine 150 mg q34h 60 mg q34h 50 mg q46h Not Not recommended Not recommended
(Talwin, others) recommended
Note: Published tables vary in the suggested doses that are equianalgesic to morphine. Clinical response is the
criterion that must be applied for each client; titration to clinical response is necessary. Because there is not
complete cross-tolerance among these drugs, it is usually necessary to use a lower-than-equianalgesic dose when
changing drugs and to retitrate to response.
Caution: Recommended doses do not apply to clients with renal or hepatic insufficiency or other conditions
affecting drug metabolism and kinetics.
*Caution: Doses listed for clients with body weight less than 50 kg cannot be used as initial starting doses in

babies less than 6 mo of age.


For morphine, hydromorphone, and oxymorphone, rectal administration is an alternate route for clients unable

to take oral medications, but equianalgesic doses may differ from oral and parenteral doses because of
pharmacokinetic differences.
Caution: Codeine doses above 65 mg often are not appropriate because of diminishing incremental analgesia

with increasing doses but continually increasing constipation and other side effects. Oral doses refer to
combination with aspirin or acetaminophen.
Caution: Doses of aspirin and acetaminophen in combination opioid/NSAID preparations must also be adjusted to

the clients body weight.


Adapted from Acute Pain Management Guideline Panel: Acute Pain Management in Adults: Operative Procedures.
Quick Reference Guide for Clinicians. Agency for Health Care Policy and Research, Department of Health and
Human Services, Rockville, MD. AHCPR Publication No. 920019.
ANALGESICS
2164_Ch08_603-688 29/03/12 12:33 PM Page 629

chapter 8 Physiological Integrity 629

D I E TA R Y S U P P L E M E N T S A N D H E R B S
Information on dietary supplements and herbal products used for psychiatric conditions is provided in Table 8.15, and
Table 8.16 lists the potential dangers of various herbs.

Table 8.15
Dietary Supplements and Herbal Products Used for Psychiatric Conditions
Herb Action/Use Caution Drug Interactions
DHEA Depression Hx of prostatic or breast Antidiabetic agents
(dehydroepiandrosterone) Fatigue carcinoma Estrogen
Hepatic dysfunction Corticosteroids
Diabetes
Ginkgo biloba Mental function Contraindication: Blood-thinning medications
(e.g., Alzheimers disease) Discontinue 2 wk before (e.g., Coumadin, heparin,
Antiplatelet agent surgery NSAIDs, ASA)
CNS stimulant Blood clotting disorder Hormonal
Renal failure Monoamine oxidase (MAO)
Acute infection inhibitors
or BP Antihypertensives
Caffeine
Decongestants
Ginseng Fatigue, stress Contraindicated with: MAO inhibitors
Mental alertness Peptic ulcer Antihypertensives
Physical endurance Intracranial bleeding Barbiturates
Appetite Elderly with cardiovascular Anticoagulants
Antidepressant disease Insulin and oral hypoglycemics
Immune response Diabetes Digoxin
Kava Anxiety Discontinue after 3 mo Alcohol/CNS depressants
Stress (yellow hair, skin, nails) Barbiturates
Restlessness Reaction: Antipsychotics (e.g., Haldol)
Sleep ( insomnia) Euphoria Antianxiety drugs (Valium, Xanax,
Relaxation Depression Ativan)
Somnolence Effects of levodopa
Eye accommodation (Parkinsons disease)
disturbance Statins (liver injury)
Muscle weakness
SAMe Depression (not bipolar) Bleeding Other antidepressants (MAO
(S-adenosylmethionine) inhibitors, tricyclic antidepres-
sants [TCAs], selective serotonin
reuptake inhibitors [SSRIs])
Potentiates St. Johns wort
St. Johns wort Mild depression Panic, agitation, Serotonin level if on antidepres-
confusion sants (e.g., MAO inhibitors,
Skin sensitivity in sun SSRIs, TCAs)
(if also on sulfa, Feldane, Reduces effect of:
Prilosec, Prevacid) Digoxin
Immunosuppressants
Protease inhibitors (HIV)
Oral contraceptives
Coumadin
Theophylline (asthma)
Antipsychotics (e.g., clozapine)
Chemotherapy
Valerian Minor tranquilizer Light-headedness Potentiates other CNS
DIET SUPPS-HERBS

Anxiety, panic attacks Restless depressants (e.g., alcohol)


Sleep disorders (insomnia) Fatigue Benzodiazepines (e.g., Xanax,
Nausea Valium)
Tremor Anticonvulsants (e.g., Dilantin,
Blurred vision phenobarbital)
Antidepressants (e.g., Elavil,
Tofranil, Prozac)
Avoid taking with substances that
cause drowsiness
2164_Ch08_603-688 29/03/12 12:33 PM Page 630

630 Dietary Supplements and Herbs

Table 8.16
Herbs and Potential Dangers
Herb Use Adverse Effects/Drug Interactions/Cautions
Aloe External: Adverse effect:
Heal burns, wounds Contact dermatitis
Anti-infective
Moisturizer
Internal: Adverse effect:
Laxative Hypokalemia
General healing Caution:
Avoid oral use in GI conditions (e.g., inflammation,
ulcers, pain of unknown origin)
Arnica External: Adverse effect:
Inflammation caused by insect bites Toxic skin reaction
After muscular and joint injuries Avoid use on broken skin
(e.g., bruises)
Black cohosh Premenstrual syndrome (PMS) Interaction with:
Menopause Oral contraceptives
Anticoagulants (e.g., Coumadin)
Hormone replacement therapy (HRT)
Caution with:
Estrogen-sensitive cancers (breast, ovarian)
Salicylate allergies
History of thromboembolic disease
Burdock leaf and root Severe skin problems Adverse effect:
Arthritis Contaminated with belladonna
Chamomile External: Caution with:
Topical anti-inflammatory agent Asthma
(for eczema, hemorrhoids, mastitis,
leg ulcers)
Internal:
Indigestion
Antispasmodic (irritable bowel
syndrome)
Echinacea Immune system stimulant at onset Interaction with:
of cold Immunosuppressants (e.g., cyclosporine, anabolic
Anti-infective steroids)
Antipyretic Caution with:
Lupus
Multiple sclerosis
Evening primrose oil Asthma Interaction with:
Diabetic neuropathy Phenothiazines
Eczema ASA
Hyperglycemia NSAIDs
Irritable bowel syndrome Anticoagulants
Menopause Antiplatelet agents
Multiple sclerosis Anticonvulsants
Rheumatoid arthritis Caution with:
Peptic ulcer
Intracranial bleeding
Epilepsy
Schizophrenia
Feverfew (see also Migraines Adverse effect:
Appendix E) Increase bleeding
Spontaneous abortion in early pregnancy
Interaction with:
HERBS

Anticoagulants
Antiplatelets
NSAIDs
Caution:
Do not use before or after surgery
2164_Ch08_603-688 29/03/12 12:33 PM Page 631

chapter 8 Physiological Integrity 631

Table 8.16
Herbs and Potential Dangerscontd
Herb Use Adverse Effects/Drug Interactions/Cautions
Ginger Antiemetic Interaction with:
Colds, flu Anticoagulants
Headaches Antiplatelets
Cardiac glycosides
Hypoglycemic agents
Caution with:
Diabetes
Gallstones
Goldenseal Topical treatment of: Interaction with:
Sores Tretinoins (Retin-A)
Inflamed mucous membranes Anticoagulants
Adverse effect:
Causes uterine contractions
Caution with:
Women who are pregnant
Lactation
Hawthorne Dementia Interaction with:
Congestive heart failure (CHF) Angiotensin-converting enzyme (ACE) inhibitors
Angina Anticoagulants
Coronary vasodilators
Caution with:
Lactation
Melaleuca oil Topical use for burns Adverse effect:
Bactericidal CNS depression
Fungicidal Allergic dermatitis
Milk thistle Liver: hepatitis and cirrhosis Interaction with:
Gallstones Oral contraceptives
Psoriasis Adverse effect:
Laxative effect
Nettle leaf Kidney, bladder stones Caution with:
UTI Pregnancy
Lactation
Pennyroyal External: Use not recommended because of hepatotoxicity
Skin disease
Internal:
Digestive disorders
Liver and gallbladder disorders
Gout
Sassafras oil Lice Adverse effect:
Insect bites Carcinogenesis (e.g., hepatic tumors)
Saw palmetto Antiandrogen therapy Interaction with:
Benign prostatic hyperplasia (BPH) HRT
Proscar
Senna Catharsis Adverse effect:
Laxative Electrolyte imbalance (decreased K+)
Caution with:
Acute intestinal inflammation (e.g., Crohns disease,
ulcerative colitis, appendicitis)
HERBS
2164_Ch08_603-688 29/03/12 12:33 PM Page 632

632 Common Medications

C O M M O N M E D I C AT I O N S
Dosages, uses, mechanism of action, and assessment of side effects of common medications are detailed in Table 8.17.

Table 8.17
Common Medications
Drug and Dosage Use Action Assessment: Side Effects
Adrenergics
Alpha and Beta Agonists
Epinephrine (Adrenalin)Subcu Asystole, bronchospasm, Stimulates pacemaker cells; Ventricular arrhythmias, fear,
or IM 0.11 mg in 1:1000 solu- anaphylaxis, glaucoma inhibits histamine and medi- anxiety, anginal pain,
tion; IVintracardiac 1:10,000 ates bronchial relaxation; decreased renal blood flow,
solution; ophthalmic intraocular pressure burning of eyes, headache
1:10001:50,000 solution
NURSING IMPLICATIONS: Use TB syringe for greater accuracy; massaging injection site hastens action; repeated injections may
cause tissue necrosis; avoid injection in buttocks because bacteria in area may lead to gas gangrene; may make mucous plugs in
lungs more difficult to dislodge
Norepinephrine (Levophed)IV Acute hypotension, cardio- Increases rate and strength Palpitations, pallor, headache,
812 mcg/min titrated to desired genic shock of heartbeat; increases hypertension; anxiety, insom-
response vasoconstriction nia; dilated pupils, nausea,
vomiting; glycosuria, tissue
sloughing
NURSING IMPLICATIONS: Observe vital signs, mentation, skin temperature, and color (earlobes, lips, nailbeds); tissue necrosis
occurs with infiltration; antidote is phentolamine 510 mg in 1015 mL normal saline
Beta Agonists
Dobutamine (Dobutrex)IV Acute heart failure Stimulates cardiac contractile Tachycardia, arrhythmias
1.510 mcg/kg/min force (positive inotropy); fewer
changes in heart rate than
dopamine or isoproterenol
NURSING IMPLICATIONS: Mix with 5% dextrose; do not dilute until ready to use; protect from light; administer with infusion pump;
check vital signs constantly; extravasation can produce tissue necrosis; see Norepinephrine
Dopamine (Inotropin)IV Acute heart failure Cardiac contractility; renal Ectopic beats; nausea, vomit-
15 mcg/kg/min titrated to blood flow ing; tachycardia, anginal pain,
desired response; gradually dyspnea, hypotension
increase in 1- to 4-mcg/kg/min
increments at 10- to 30-min
intervals; must maintain at
20 mcg/kg/min or less
NURSING IMPLICATIONS: Monitor vital signs, urine output, and signs of peripheral ischemia; will cause tissue sloughing if infiltra-
tion occurs
Isoproterenol (Isuprel) Cardiogenic shock, heart Cardiac contractility; Tachyarrhythmias, hypoten-
sublingual 1015 mg; block, bronchospasm facilitates atrioventricular (AV) sion, headache, flushing of
IV 0.55.0 mcg/min in solution asthma, emphysema conduction and pacemaker skin; nausea; tremor; dizziness
automaticity
NURSING IMPLICATIONS: Monitor vital signs, ECG; oral inhalation solutions must not be injected
Analgesics (see Table 8.14 for Equianalgesic Dosing)
Acetaminophen (Tylenol, Datril, Simple fever or pain Analgesic and antipyretic No remarkable side effects
Panadol)PO 325650 mg q4h actions; no anti-inflammatory when taken for a short period
or anticoagulant effects
NURSING IMPLICATIONS: Consult with physician if no relief after 4 days of therapy
Aspirin (acetylsalicylic acid)PO Minor aches and pains; Selectively depresses subcorti- Erosive gastritis with bleeding;
ANALGESICS

or rectal 0.30.6 gm fever from colds and cal levels of CNS coryza, urticaria; nausea, vom-
Ecotrin (enteric-coated aspirin) influenza; rheumatoid iting; tinnitus, impaired hearing,
PO 81325 mg arthritis; anticoagulant and respiratory alkalosis
therapy
2164_Ch08_603-688 29/03/12 12:33 PM Page 633

chapter 8 Physiological Integrity 633

Table 8.17
Common Medicationscontd
Drug and Dosage Use Action Assessment: Side Effects
NURSING IMPLICATIONS: Administer with food or after meals; observe for nasal, oral, or subcutaneous bleeding; push
fluids; check hematocrit (Hct), hemoglobin (Hgb), prothrombin times frequently; avoid use in children with flu; do not take
ecotrin with milk or dairy
Butorphanol (Stadol)IM Obstetric use: Control 3040 times more potent than No excessive sedation.
12 mg; IV 0.52 mg pain, especially during meperidine; onset: 215 min;
Neonatal respiratory
labor. Avoid in clients who duration; 0.52 hr
depression less than with
are opioid addicted.
meperidine.
Pseudo-sinusoidal FHR pattern
after administration.
NURSING IMPLICATIONS: Care must be taken that drug is not given with meperidine, as it may potentiate effects of meperidine;
avoid in clients who are opioid addicted
Celecoxib (Celebrex)PO Arthritis; acute pain; NSAID, anti-inflammatory, Back pain; peripheral edema;
100200 mg twice daily or dysmenorrhea analgesic, antipyretic; inhibits diarrhea; dizziness; rash,
200 mg daily prostaglandin synthesis rhinitis
NURSING IMPLICATIONS: Monitor for signs of fluid retention and edema; report promptly unexplained weight gain, rash
CodeinePO, IM, Subcu Control pain; may be used Nonsynthetic narcotic Of little use during labor;
1560 mg (gr 1/4 to 1) during the puerperium analgesic allergic response; constipation;
GI upset
NURSING IMPLICATIONS: Note response to the medication; less respiratory depression; preferred for client with head injury
Etodolac (Lodine)PO Management of Inhibits prostaglandin Dyspepsia; asthma; drowsi-
4001,200 mg/day, osteoarthritis; mild to synthesis; suppression of ness, dizziness; rash, tinnitus,
300400 q68h moderate pain inflammation and pain (NSAID) anaphylaxis
NURSING IMPLICATIONS: Give 30 min before or 2 hr after meals for rapid effect; may be taken with food to decrease GI irritation
Fentanyl transdermal Chronic pain; not recom- Binds to opiate receptors in Drowsiness, confusion, weak-
(Duragesic)25100 mcg/hr mended for postoperative the CNS to alter response to ness; constipation, dry mouth,
pain and perception of pain nausea, vomiting, anorexia;
sweating
Obstetric use: Short-
acting pain control during
labor
NURSING IMPLICATIONS: Apply to upper torso, flat, nonirritated surface; when applying, hold firmly with palm of hand 1020 sec
Hydrocodone/Acetaminophen Moderate to severe pain Bind to opiate receptors in Confusion, sedation, hypoten-
(Vicodin)PO 510 mg q46h CNS; alter perception of and sion, constipation
Obstetric use: in postpar- response to painful stimuli
tum to control pain
NURSING IMPLICATIONS: Assess: type, location, and intensity of pain before and 1 hr (peak) after giving; prolonged use may lead
to physical and psychological dependence; give with food or milk to reduce GI irritation
Hydromorphone (Dilaudid) Moderate to severe pain; Binds to opiate receptors in Sedation, confusion, hypoten-
PO 24 mg q36h; IM 12 mg antitussive the CNS; alters perception and sion, constipation
q36h up to 24 mg q46h; response to pain
IV 0.51.0 mg q3h
NURSING IMPLICATIONS: Give PO with food or milk; give IV 2 mg over 35 min; fluids, bulk, and laxatives to minimize constipation
Ibuprofen (Motrin)PO Nonsteroid anti-inflamma- Inhibition of prostaglandin GI upset, leukopenia;
300800 mg 34 times/day (not tory, antirheumatic used in synthesis or release sodium/water retention
to exceed 3,200 mg/day) chronic arthritis pain

Obstetric use: uterine


ANALGESICS

cramping postpartum
(not used during
pregnancy)
Continued
2164_Ch08_603-688 29/03/12 12:33 PM Page 634

634 Common Medications

Table 8.17
Common Medicationscontd
Drug and Dosage Use Action Assessment: Side Effects
NURSING IMPLICATIONS: Give on empty stomach for best result; may mix with food if GI upset severe; teach caution when using
other medications
Indomethacin (Indocin)PO Rheumatoid arthritis; Antipyretic/anti-inflammatory GI distress, bleeding; rash;
25 mg 34 times/day; increase to bursitis; gouty arthritis action; inhibits prostaglandin headache; blood dyscrasias;
max. 200 mg daily in divided biosynthesis corneal changes
Closure of patent ductus
doses
arteriosus (PDA) in prema-
ture infants and some
newborn
NURSING IMPLICATIONS: Monitor GI side effects; administer after meals for best effect or with food, milk, or antacids if GI
symptoms severe
Ketorolac (Toradol)PO, IV Short-term management of Inhibits prostaglandin Drowsiness, dizziness, dysp-
10 mg q46h; IM 3060 mg pain; ocular itching due to synthesis, producing peripher- nea; prolonged bleeding time;
initially, then 1530 mg q6h; allergies (NSAID) ally mediated analgesia; dyspepsia
ophthalmic 1 gtt 4 times daily antipyretic/anti-inflammatory
for 1 wk
NURSING IMPLICATIONS: May be given routinely or prn; advise dentist or physician before any procedure
Meperidine HCl (Demerol)PO, Pain due to trauma or sur- Acts on CNS to produce anal- Palpitations, bradycardia,
IM 50100 mg q34h gery; allay apprehension gesia, sedation, euphoria, and hypotension; nausea, vomiting;
before surgery respiratory depression syncope, sweating, tremors,
convulsions
NURSING IMPLICATIONS: Check respiratory rate and depth before giving drug; give IM, because subcutaneous administration is
painful and can cause local irritation
Obstetric use: maternal Acts as a uterine irritant; Maternal side effects same as
relaxation may either slow depresses CNS, maternal and above; also can depress fetus
labor or speed up labor fetal; allays apprehension; PO
peak action 12 hr; IM peak
action first hour
NURSING IMPLICATIONS: Monitor maternal vital signs, contractions, progress of labor, and response to drug; fetal heart rate; if
delivery occurs during peak action, prepare to give narcotic antagonist to mother or neonate or both
Morphine SO4PO 1030 mg Control pain and relieve Depresses CNS reception of Nausea, vomiting; flushing;
(Roxanol, MS Contin); Subcu fear, apprehension, rest- pain and ability to interpret confusion; urticaria; depressed
815 mg; IV 410 mg; rectal lessness, as in pulmonary stimuli; depresses respiratory rate and depth of respirations,
1020 mg edema center in medulla decreased blood pressure
NURSING IMPLICATIONS: Check rate and depth of respirations before administering drug; observe for gas pains and abdominal
distention; smaller doses for aged; monitor vital signs; observe for postural hypotension
Obstetric use: preeclamp- Increases cerebral blood flow; Respiratory and circulatory
sia, eclampsia; uterine provides antihypertensive depression in mother and
dysfunction; pain relief action; CNS depressant neonate; may depress
contractions
NURSING IMPLICATIONS: Observe for level of sedation, respirations, arousability, and deep tendon reflex; give narcotic antagonist
as necessary; check I&O (urinary retention possible)
Nalbuphine (Nubain)IV 1 mg at Obstetric use: pain relief Onset: 215 min; duration Increased sedation and limited
6- to 10-min intervals during labor. Avoid in 14 hr ceiling effect; ceiling effect
clients who are opioid for neonatal respiratory
addicted. depression
NURSING IMPLICATIONS: Less nausea and vomiting and sedation if given with patient-controlled analgesia (PCA). Prophylactic
nalbuphine seems unable to prevent pruritus
Naproxen (Naprosyn)PO Mild to moderate pain; Inhibits prostaglandin Headache, drowsiness,
ANALGESICS

250500 mg twice daily dysmenorrhea; rheumatoid synthesis; suppression of dizziness, nausea, dyspepsia,
arthritis; osteoarthritis inflammation constipation, bleeding
(NSAID)
2164_Ch08_603-688 29/03/12 12:33 PM Page 635

chapter 8 Physiological Integrity 635

Table 8.17
Common Medicationscontd
Drug and Dosage Use Action Assessment: Side Effects
NURSING IMPLICATIONS: Take with a full glass of water; avoid exposure to sun
Oxycodone HCl (OxyContin) Postpartum use: control Less potent and addicting than See Morphine SO4
PO 320 mg; Subcu 5 mg pain; may be used during morphine; for moderate pain
puerperium; 56 times episiotomy and afterpains;
more potent than codeine peak action 1 hr
NURSING IMPLICATIONS: Administer per order and observe for effect
Pentazocine (Talwin) Relief of moderate to Narcotic antagonist, opioid Respiratory depression,
PO 50100 mg; IM 3060 mg severe pain antagonist properties, equiva- nausea, vomiting, dizziness,
q34h lent to codeine light-headedness, seizures
NURSING IMPLICATIONS: Monitor respirations, BP; caution with client with myocardial infarction (MI), head injuries, chronic
obstructive pulmonary disease (COPD)
Antacids (see Antiulcer Agents)
Antiadrenal Agent
Aminoglutethimide (Cytadren) Cushings syndrome Inhibits enzymatic conversion Cortical hypofunction,
PO 250 mg q6h of cholesterol to pregnenolone, hypotension, hypothyroidism
thus reducing synthesis of
adrenal glucocorticoids/
mineralocorticoids
NURSING IMPLICATIONS: Monitor: vital signs, laboratory values, disease signs and symptoms; observe frequently during periods
of stress
Antianemic Agents
Ferrous sulfate (Feosol, Iron-deficiency anemia; Corrects nutritional Nausea, vomiting, anorexia,
Fer-In-Sol)adults, PO prophylactically during iron-deficiency anemia constipation, diarrhea; yellow-
300 mg1.2 gm daily; children infancy, childhood, brown discoloration of eyes,
under 6 yr, PO 75225 mg daily; pregnancy teeth
612 yr, PO 120600 mg daily
NURSING IMPLICATIONS: To minimize GI distress, give with meals; do not give with antacids or tea; liquid form should be taken
through straw to prevent staining of teeth; causes dark-green/black stool
Filgrastim (Neupogen)Subcu, Neutropenia from cancer, Stimulates white blood cell Bone pain, fever
IV 5-mcg/kg/day increments chemotherapy, bone production, specifically
marrow transplantation neutrophil count; a biologic
response modifier
NURSING IMPLICATIONS: Monitor WBC count as indication of effectiveness
Lenalidomide (Revlimid) Slow progression of Works in bone marrow to slow Deep vein thrombosis, neu-
PO 525 mg daily for 21 days multiple myeloma; growth of cancerous myeloma tropenia, thrombocytopenia,
transfusion-dependent cells birth defects
anemia
NURSING IMPLICATIONS: Clients with multiple myeloma take the drug with dexamethasone
Pegfilgrastim (Neulasta)Subcu Neutropenia from Stimulates bone marrow and Infection, skin rash, splenic
6 mg per chemotherapy cycle chemotherapy promotes neutrophils (WBCs) rupture, acute respiratory dis-
tress syndrome (ARDS)
NURSING IMPLICATIONS: Do not give 14 days before or 24 hours after chemotherapy. Instruct client to report immediately
sudden, severe pain in left upper stomach spreading to shoulder
Antianginals
Atenolol (Tenormin)PO Hypertension: angina; Blocks (cardiac) beta-adrenergic Fatigue, weakness,
50150 mg daily; IV 5 mg initially, arrhythmias receptors bradycardia, heart failure,
ANTIANGINALS

wait 10 min, then another 5 mg pulmonary edema


NURSING IMPLICATIONS: Give 1 mg/min IV; check vital signs; assess for signs of fluid overload

Continued
2164_Ch08_603-688 29/03/12 12:33 PM Page 636

636 Common Medications

Table 8.17
Common Medicationscontd
Drug and Dosage Use Action Assessment: Side Effects
Isosorbide dinitrate (Isordil, Acute angina: long-term Produces vasodilation, Headache, dizziness,
Isorbid)sublingual 2.510.0 mg prophylaxis for angina; decreases preload hypotension, tachycardia
q510 min for 3 doses; heart failure
PO 510 mg initially,
1040 mg q6h
NURSING IMPLICATIONS: Avoid eating, drinking, or smoking until sublingual tablets are dissolved; change positions slowly; aspirin
or acetaminophen for headache. Determine if client is taking drugs for erectile dysfunctionserious drug interaction!
Nitroglycerinsublingual Angina pectoris; adjunctive Directly relaxes smooth Faintness, throbbing
0.250.6 mg prn; transdermal treatment in MI, heart fail- muscle, dilating blood vessels; headache; vomiting;
(patch) 2.515.0 mg/day; ure, hypertension (IV form) lowers peripheral vascular flushing, hypotension; visual
topical 23 inches q8h; resistance; increases blood disturbances
IV 1020 mcg/min flow
NURSING IMPLICATIONS: Instruct client to sit or lie down when taking drug, to reduce hypotensive effect; onset 13 min;
may take 13 doses at 5-min intervals to relieve pain; up to 10/day may be allowed; if headache occurs, tell client to expel
tablet as soon as pain relief occurs; keep drug at bedside or on person; watch expiration datestablets lose potency with
exposure to air and humidity; alcohol ingestion soon after taking may produce shocklike syndrome from drop in BP; smoking causes
vasoconstricting effect; causes burning under tongue; may crush between teeth to absorption
Antiarrhythmics (also see Calcium Channel Blockers)
Amiodarone hydrochloride Life-threatening ventricular Class III antiarrhythmic; Muscle weakness, fatigue,
(Cordarone)PO 800 arrhythmias; supraventricu- antianginal, antiadrenergic dizziness, hypotension with IV;
1,600 mg/day loading dose; lar arrhythmias (e.g., atrial corneal microdeposits;
400600 mg/day maintenance fibrillation) anorexia, nausea, vomiting;
photosensitivity
NURSING IMPLICATIONS: Monitor vital signs (VS). Expect possible CNS symptoms 1 wk after beginning drug. Instruct client to
protect eyes and skin from sun
Bretylium (Bretylol)IV Ventricular fibrillation; Inhibits norepinephrine release Worsening of arrhythmia;
0.510 mg/kg q6h; IM ventricular tachycardia from sympathetic nerve end- tachycardia and increased BP
510 mg/kg (max. 250 mg ings; increased fibrillation initially; nausea, vomiting;
in one site) threshold hypotension later
NURSING IMPLICATIONS: Monitor BP and cardiac status closely; rotate IM injection sites; no more than 5 mL/site
Diphenylhydantoin (Dilantin) Digitalis toxicity; ventricular Depresses pacemaker activity Severe pain if administered in
PO 100200 mg 34 times daily; ectopy in sinoatrial (SA) node and small vein; ataxia, vertigo, nys-
IV loading dose 1015 mg/kg Purkinje tissue without slowing tagmus, seizures, confusion;
(not to exceed 50 mg/min), conduction velocity skin eruptions, hypotension if
50100 mg over 510 min administered too fast
NURSING IMPLICATIONS: With IV use, monitor vital signs; observe for CNS side effects; have O2 on hand; seizure precautions
(padded side rails, nonmetal airway; suction, mouth gag); also see Anticonvulsants
Lidocaine HClIV 50100 mg, Ventricular tachycardia; Depresses myocardial Drowsiness, dizziness,
bolus; 14 mg/kg/min, IV drip premature ventricular response to abnormally nervousness, confusion,
contractions (PVCs) generated impulses paresthesias
NURSING IMPLICATIONS: Monitor vital signs; observe for signs of CNS toxicity; monitor ECG for prolonged P-R interval
Metoprolol tartrate (Lopressor) Hypertension; angina; MI Beta-adrenergic blocker; Dizziness, fatigue, insomnia;
PO 25 mg/day; range cardiac output (CO), heart bradycardia, shortness of
25300 mg/day rate (HR) breath (SOB)
NURSING IMPLICATIONS: Monitor: vital signs carefully; I&O, daily weight; check for rales. Give with or without foodbe
consistent
Procainamide HCI (Pronestyl) Atrial and ventricular Depresses myocardium; Polyarthralgia; fever, chills,
ANTIARRHYTHMICS

PO, IM 5001,000 mg 46 times arrhythmias; PVCs; over- lengthens conduction time urticaria; nausea, vomiting;
daily; IV 1 gm dose of digitalis; general between atria and ventricles psychoses; rapid decrease
anesthesia in BP
NURSING IMPLICATIONS: Check pulse rate before giving; monitor heart action during IV administration
2164_Ch08_603-688 29/03/12 12:33 PM Page 637

chapter 8 Physiological Integrity 637

Table 8.17
Common Medicationscontd
Drug and Dosage Use Action Assessment: Side Effects
Propranolol HCI (Inderal)IV Ventricular ectopy; angina Beta-adrenergic blocker, Bradycardia, hypotension,
push 0.53.0 mg (up to 3 mg); unresponsive to nitrites, cardiac contractility, vertigo, paresthesia of hands
PO 2060 mg 34 times daily paroxysmal atrial tachycar- heart rate, myocardial
dia (PAT); hypertension oxygen requirements
NURSING IMPLICATIONS: Instruct client to take pulse before each dose; do not give to clients with history of asthma or chronic
obstructive pulmonary disease; no smoking, because hypertension may occur
Quinidine SO4PO 0.20.6 gm Atrial fibrillation; PAT; Lengthens conduction time in Nausea, vomiting, diarrhea;
q2h loading dose; maintenance: ventricular tachycardia; atria and ventricles; blocks vertigo, tremor, headache;
4001,000 mg 34 times daily; PVCs vagal stimulation of heart abdominal cramps; AV block,
IV 510 mg/kg over 3060 min cardiac arrest
NURSING IMPLICATIONS: Count pulse before giving; report changes in rate, quality, or rhythm; give drug with food; monitor
BP daily
Antiasthmatic
Cromolyn sodiuminhalation Perennial bronchial asthma Inhibits release of Cough, hoarseness, wheezing;
1 capsule 4 times daily (not acute asthma or status bronchoconstrictors dry mouth, bitter aftertaste,
asthmaticus) histamine and slow-reacting urticaria; urinary frequency
substance of anaphylaxis
(SRS-A); suppresses allergic
response
NURSING IMPLICATIONS: Instruct on use of inhalerexhale; tilt head back; inhale rapidly, deeply, steadily; remove inhaler,
exhalerepeat until dose is taken; gargle or drink water after treatment
Anticholelithic
Ursodeoxycholic (Actigall)PO Gallstones Dissolves gallstones Diarrhea; avoid concurrent use
300600 mg with bile acid sequestrants
NURSING IMPLICATIONS: Monitor for pain, GI distress
Anticholinergics (Antimuscarinics)
Atropine SO4PO, Subcu, IM, Peptic ulcer; spasms of Blocks parasympathomimetic Dry mouth, tachycardia,
IV 0.31.2 mg; ophthalmic GI tract, Stokes-Adams effects of acetylcholine on blurred vision, drowsiness,
0.5%1.0% up to 6 times daily syndrome; control effector organs skin flushing; urinary
excessive secretions retention; contraindications:
during surgery glaucoma and paralytic
ileus
NURSING IMPLICATIONS: Observe for postural hypotension in clients who are ambulating; administer cautiously in aged; and
monitor vital signs for pulse and respiratory rate changes
Dicyclomine (Bentyl)PO Diverticulosis Antispasmodic Blurred vision; constipation;
1020 mg 34 times/day dry eyes, dry mouth
NURSING IMPLICATIONS: Report symptoms listed under side effects, abdominal pain
Glycopyrrolate (Robinul)PO Inhibit salivation and respi- Inhibits action of acetylcholine Tachycardia; dry mouth;
12 mg 23 times per day; ratory secretions; peptic (antimuscarinic action) urinary hesitancy
IM, IV 4.4 mcg/kg (preop); ulcer disease
100200 mcg 4 times a day
(ulcer)
NURSING IMPLICATIONS: Give 3060 min before meals; do not give within 1 hr of antacids; IM or IV may be given undiluted
Hyoscyamine (Levsinex)PO Diverticulosis Manages spasms of irritable Constipation; dry mouth,
0.1250.5 mg 34 times/day; or bowel syndrome blurred vision
PO 0.375 mg twice a day in
ANTICHOLINERGICS

extended-released tablets
NURSING IMPLICATIONS: Monitor for: pain, bleeding, constipation, drug reaction

Continued
2164_Ch08_603-688 29/03/12 12:34 PM Page 638

638 Common Medications

Table 8.17
Common Medicationscontd
Drug and Dosage Use Action Assessment: Side Effects
Propantheline bromide Decreases hypertonicity Blocks neural transmission Nausea, gastric fullness,
(Pro-Banthine)PO 15 mg and hypersecretion of GI at ganglia of autonomic constipation; mydriasis
4 times daily; IM, IV 30 mg tract; ulcerative colitis; nervous system and at
peptic ulcer parasympathetic effector
organs
NURSING IMPLICATIONS: Give before meals; observe urinary output to avoid retention, particularly in elderly; mouth care to relieve
dryness; contraindicated with glaucoma
Tincture of belladonnaPO Hypermotility of stomach; Blocks parasympathomimetic Dry mouth, thirst; dilated
0.30.6 mL 3 times daily bowel, biliary, and renal effects of acetylcholine pupils; skin flushing; elevated
colic; prostatitis temperature; delirium
NURSING IMPLICATIONS: Administer 3060 min before meals; observe for side effects; physostigmine salicylate is antidote
Tolterodine tartrate (Detrol LA) Overactive bladder Selective muscarinic urinary Dry mouth; back pain; fatigue,
PO 2 mg twice daily; 4 mg bladder receptor antagonist weight gain
sustained release (SR) daily
NURSING IMPLICATIONS: Report eye pain promptly. Blurred vision, sensitivity to light, and dry mouth should be reported if
bothersome
Anticoagulants
Enoxaparin (Lovenox)Subcu Deep vein thrombosis; Potentiates inhibitory effect of Bleeding, anemia,
3040 mg twice daily, 710 days; prophylaxis before knee antithrombin thrombocytopenia
or 1 mg/kg q12h and abdominal surgery
NURSING IMPLICATIONS: Observe injection sites for hematomas, ecchymosis, or inflammation; ice cube massage at site prior to
injection may lessen bruising
Heparininitial dose: IV bolus of Acute thromboembolic Prevents thrombin formation Hematuria, bleeding gums,
5,000 units, followed by infusion emergencies ecchymosis
of 20,00040,000 units over
24 hr
NURSING IMPLICATIONS: Observe clotting timesshould be 2030 min; antagonist is protamine sulfate
Warfarin sodium (Coumadin) Venous thrombosis; atrial Depresses liver synthesis of Minor or major hemorrhage;
initial dose: PO 1015 mg; fibrillation with emboliza- prothrombin and factors VII, alopecia; fever; nausea,
maintenance dose: PO 210 mg tion; pulmonary emboli; IX, and X diarrhea; dermatitis
daily myocardial infarction
NURSING IMPLICATIONS: Drug effects last 34 days; antagonist is vitamin K; avoid foods high in vitamin K; no aspirin

Anticoagulant Antidotes
Protamine sulfate 1%IV Overdose of heparin Positive electrostatic charge Excessive coagulation;
10 mg/mL slowly; 1 mg/100 inactivates negatively charged hypotension, bradycardia,
units heparin heparin molecules dyspnea
NURSING IMPLICATIONS: Slow IV; no more than 50 mg in 10-min period; monitor VS continuously; check activated partial throm-
boplastin time (aPTT) for effectiveness
Vitamin K (Aquamephyton, Warfarin (Coumadin) Counteracts the inhibitory Flushing, hypotension, allergic
Konakion)PO, IM, Subcu hypoprothrombinemia; effects of oral anticoagulants reactions; reappearance of
2.525.0 mg; 0.51.0 mg in hemorrhagic disease in on hepatic synthesis of vitamin clotting problems (high doses)
newborns newborns Kdependent clotting factors
NURSING IMPLICATIONS: Give IV only if absolutely necessary; dilute with preservative-free 0.9% NaCl, D5W, or 5% dextrose in
NaCl; protect solution from light; repeat injection may cause redness and pain; check prothrombin time (PT) for drug effect
ANTICONVULSANTS

Anticonvulsants
Carbamazepine (Tegretol)PO Multiple sclerosis Decreases synaptic transmis- Contraindicated in hypersensi-
200 mg twice/day (range: sion in CNS by affecting tivity, bone marrow depression;
6001,200/day in divided doses) sodium channels in neurons drowsiness, ataxia, agranulo-
cytosis, aplastic anemia
NURSING IMPLICATIONS: Monitor: complete blood count (CBC), liver function, electrocardiogram (ECG). Therapeutic levels should
be 612 mcg/mL; take medication as directed around the clock; use sunscreen due to photosensitivity
2164_Ch08_603-688 29/03/12 12:34 PM Page 639

chapter 8 Physiological Integrity 639

Table 8.17
Common Medicationscontd
Drug and Dosage Use Action Assessment: Side Effects
Clonazepam (Klonopin)PO Absence seizures (petit Produces anticonvulsant and Drowsiness, ataxia, behavioral
1.5 mg in 3 doses initially, mal seizures, myoclonic sedative effects in the CNS; changes
0.51.0 mg every third day seizures) mechanism unknown
NURSING IMPLICATIONS: Give with food; evaluate liver enzymes, CBC, and platelets; avoid abrupt withdrawalmay cause status
epilepticus
Diazepam (Valium)PO 210 mg All types of seizures Induces calming effect on Drowsiness, ataxia,
24 times daily; IM, IV 510 mg limbic system, thalamus, and paradoxical increase in CNS
Obstetric use: eclampsia hypothalamus excitability
NURSING IMPLICATIONS: IV may cause phlebitis; give IV injection slowly, because respiratory arrest can occur; inject IM deeply
into tissue
Ethosuximide (Zarontin)PO Absence seizures Depresses motor cortex and GI distress: nausea, vomiting,
500 mg/day, increase by reduces CNS sensitivity to cramps, diarrhea, anorexia;
250 mg/day until effective convulsive nerve stimuli blood dyscrasias
NURSING IMPLICATIONS: Administer with meals; regular CBC; precautions to avoid injury from drowsiness
Magnesium saltsIV infusion Cirrhosis Osmotically active in GI tract Use caution in renal failure;
45 gm contraindicated in: hypermag-
nesemia, anuria, heart block
NURSING IMPLICATIONS: Monitor: vital signs, seizure precautions, I&O; respiratory rate must be 16 before administration
Magnesium sulfateIM or IV Control seizures in preg- Depresses CNS and smooth, Flushing, sweating, extreme
1- to 4-gm loading dose, nancy, epilepsy; relief of cardiac, and skeletal muscle; thirst; complete heart block;
followed by continuous infusion acute constipation; promotes osmotic retention of dehydration; depressed or
(23 gm/hr) reduces edema, inflamma- fluid absent reflexes, respirations
tion, and itching of skin;
may inhibit preterm
contractions
NURSING IMPLICATIONS: If given IV, monitor vital signs continuously; I&O; use with caution in clients with impaired renal functions;
observe mother and newborn for signs of toxicity if given near birth; antidote: calcium gluconate
Phenytoin or diphenylhydantoin Psychomotor epilepsy, Depresses motor cortex by Nervousness, ataxia; gastric
(Dilantin) SO4PO 30100 mg convulsive seizures; preventing spread of abnormal distress; nystagmus; slurred
34 times daily; IM 100200 mg ventricular arrhythmias electrical impulses speech; hallucinations; gingival
34 times daily; IV 150250 mg hyperplasia
NURSING IMPLICATIONS: Give with meals or after meals (pc); frequent and diligent mouth care; advise client that urine may turn
pink to red-brown; teach client signs of adverse reactions; mix IV with normal saline (precipitates with D5W)
Primidone (Mysoline)PO Tonic-clonic, focal, or local Inhibits abnormal brain electri- Excessive sedation or ataxia,
100250 mg, increase over 10 seizures cal activity; dose-dependent vertigo
days (not to exceed 2 gm/day) CNS depression
NURSING IMPLICATIONS: Careful neurological, cardiovascular, and respiratory assessment; have resuscitation equipment available
Valproic acid (Depakene)PO Absence, tonic-clonic, Inhibits spread of abnormal Nausea, vomiting, diarrhea
15 mg/kg/day, increase up to myoclonic, focal, or local discharges through brain (disappear over time); drowsi-
60 mg/kg/day seizures ness or sedation if taken
in combination with other
anticonvulsants
NURSING IMPLICATIONS: Assess responses; monitor blood levels; precautions against excessive sedation; discourage alcohol use
Antidiarrheals
Diphenoxylate HCl with atropine Diarrhea Increases intestinal tone Rash; drowsiness, dizziness;
sulfate (Lomotil)PO 510 mg and decreases propulsive depression; abdominal disten-
ANTIDIARRHEALS

34 times daily peristalsis tion; headache, blurred vision;


nausea
NURSING IMPLICATIONS: May potentiate action of barbiturates, opiates, and other depressants; closely observe clients receiving
these drugs, and administer narcotic antagonists such as levallorphan tartrate (Lorfan), naloxone HCl (Narcan), and nalorphine HCl
(Naline) as ordered; administer cautiously to clients with hepatic dysfunctionmay precipitate hepatic coma

Continued
2164_Ch08_603-688 29/03/12 12:34 PM Page 640

640 Common Medications

Table 8.17
Common Medicationscontd
Drug and Dosage Use Action Assessment: Side Effects
Kaolin with pectin (Kaopectate) Diarrhea Reported to absorb irritants Granuloma of the stomach
adults, PO 60120 mL after each and soothe
bowel movement (BM); children
over 12, PO 60 mL; 612 yr, PO
3060 mL; 36 yr, PO 1530 mL
after each BM
NURSING IMPLICATIONS: Do not administer for more than 2 days, in presence of fever, or to children younger than 3 yr
Paregoric or camphorated opium Diarrhea Acts directly on intestinal Occasional nausea; prolonged
tincturePO 510 mL q2h, not smooth muscle to increase use may produce dependence
more than 4 times daily tone and decrease propulsive
peristalsis
NURSING IMPLICATIONS: Contains approximately 1.6 mg morphine or 16 mg opium and is subject to federal narcotic regulations;
administer with partial glass of water to facilitate passage into stomach; observe number and consistency of stoolsdiscontinue
drug as soon as diarrhea is controlled; keep in tight light-resistant bottles
Antiemetics
Chlorpromazine (Thorazine) Nausea, vomiting, hiccups, Alters the effects of dopamine Sedation, extrapyramidal reac-
preop IM 12.525.0 gm 12 hr preoperative sedation, in the CNS; anticholinergic, tions; dry eyes, blurred vision;
before; IV 2550 mg; PO psychoses alpha-adrenergic blocking hypotension; constipation, dry
1025 mg q46h; IM 2550 mg mouth; photosensitivity
q34h; suppository 50100 mg
q68h
NURSING IMPLICATIONS: Keep flat 30 min after IM; change positions slowly; frequent mouth care; may turn urine pink to
red-brown
Prochlorperazine dimaleate Nausea, vomiting, and See Trimethobenzamide HCl Drowsiness, orthostatic
(Compazine)PO, IM, rectal retching hypotension, palpitations,
530 mg 4 times daily blurred vision, diplopia,
headache
NURSING IMPLICATIONS: Use cautiously in children, women who are pregnant, and clients with liver disease
Trimethobenzamide HCl Nausea; vomiting Suppresses chemoreceptors in Drowsiness, vertigo; diarrhea,
(Tigan)PO, IM, rectal 250 mg the trigger zone located in the headache, hypotension;
4 times daily medulla oblongata jaundice; blurred vision; rigid
muscles
NURSING IMPLICATIONS: Give deep IM to prevent escape of solution; can cause edema, pain, and burning
Antifungals
Amphotericin B (Fungizone)IV Severe fungal infections; Fungistatic or fungicidal; binds Febrile reactions; chills,
5 mg/250 mL dextrose over histoplasmosis to sterols in fungal cell mem- nausea, vomiting; muscle/joint
46 hr (to 1 mg/kg body weight) brane, altering cell permeability pain; renal damage; hypoten-
sion, tachycardia, arrhythmias;
hypokalemia
NURSING IMPLICATIONS: Monitor for side effects, thrombophlebitis at IV site; blood urea nitrogen (BUN) >40; creatinine >3; stop
drug because of nephrotoxicity
Ketoconazole (Nizoral)PO Histoplasmosis, systemic Antifungal Headache, fatigue, dizziness;
200400 mg daily fungal infections nausea, vomiting; decreased
libido, impotence; gynecomas-
tia, especially in men
NURSING IMPLICATIONS: Administer with food; avoid concomitant use of antacids, histamine2 (H2) blockers; advise to report side
effects
ANTIFUNGALS

Nystatin (Nilstat, Mycostatin) Skin, mucous membrane Fungistatic and fungicidal; Nausea, vomiting, Gl distress,
PO, rectal, vaginal 100,000 infections (Candida albi- binds to sterols in fungal cell diarrhea
1,000,000 units 34 times daily cans); oral thrush, vaginitis; membrane
intestinal candidiasis

Table 8.17
2164_Ch08_603-688 29/03/12 12:34 PM Page 641

chapter 8 Physiological Integrity 641

Table 8.17
Common Medicationscontd
Drug and Dosage Use Action Assessment: Side Effects
NURSING IMPLICATIONS: Oral useclear mouth of food; keep medication in mouth several minutes before swallowing; vaginal
usually requires 2 wk therapy; continue use during menses; consult physician before using anti-infective douches; determine predis-
posing factors to infection (diabetes, pregnancy, antibiotics, tight-fitting nylon pantyhose)
Antigout Agents
Allopurinol (Lopurin, Zyloprim) Primary hyperuricemia, Lowers plasma and urinary uric Rash, itching; nausea, vomit-
PO 100 mg initially, 300 mg daily secondary hyperuricemia acid levels; no analgesic, anti- ing; anemia, drowsiness
with meals or pc with cancer therapy inflammatory, or uricosuric
actions
NURSING IMPLICATIONS: Report side effects, particularly rash, because drug must be stopped; avoid driving or other complex
tasks until drug effects known; give at least 3,000 mL fluid daily; minimum urine output of 2,000 mL/day; keep urine neutral or alka-
line with sodium bicarbonate or potassium citrate; use cautiously with: liver disease, impaired renal function, history of peptic ulcers,
lower GI disease, or bone marrow depression
ColchicinePO 1.01.2 mg Gouty arthritis, acute gout Inhibits leukocyte migration Nausea, vomiting, diarrhea,
acute phase; 0.52.0 mg nightly and phagocytosis in gouty abdominal pain; peripheral
with milk or food; IV 12 mg joints; nonanalgesic, neuritis; bone marrow depres-
initially nonuricosuric sion (sore throat, bleeding
gums, sore mouth); tissue
and nerve necrosis with
IV use
NURSING IMPLICATIONS: Do not dilute IV form with normal saline or 5% dextroseuse sterile water to prevent precipitation;
infuse over 35 min IV; potentiate drug action with alkali-ash foods (milk, most fruits and vegetables)
Probenecid (Benemid)PO Chronic gouty arthritis (no Inhibits renal tubular Headache; nausea, vomiting,
0.250.50 gm twice daily pc value in acute); adjuvant resorption of uric acid; no anorexia, sore gums; urinary
therapy with penicillin to analgesic or anti-inflammatory frequency, flushing
increase plasma levels activity; competitively inhibits
renal tubular secretion of
penicillin and many weak
organic acids
NURSING IMPLICATIONS: Give with food, milk, or prescribed antacid; 3,000 mL/day fluids; avoid alcohol, which increases serum
urates; do not take with aspirininhibits action of drug; renal function and hematology should be evaluated frequently; during acute
gout, give with colchicine
Antihistamines
Astemizole (Hismanal)PO Relief of allergic symptoms Blocks the effects of histamine Drowsiness, headache, fatigue;
10 mg/day (rhinitis, urticaria); less stimulation; dry mouth; rash;
sedating increased appetite (none of
these are frequent)
NURSING IMPLICATIONS: Take 1 hr before or 2 hr after eating; good oral hygiene; may need to reduce calories
Cetirizine (Zyrtec)PO Seasonal and perennial Potent histamine1 (H1) receptor Drowsiness, sedation,
510 mg/day; 10 mg daily allergic rhinitis; chronic antagonist headache
or twice daily for urticaria urticaria
NURSING IMPLICATIONS: Sedation more common in older adult. Do not take with OTC antihistamines
Chlorpheniramine maleate Asthma; hay fever; serum Inhibits action of histamine Nausea, gastritis, diarrhea;
(Chlor-Trimeton)PO 24 mg reactions; anaphylaxis headache; dryness of mouth
3 or 4 times daily; Subcu, IM, and nose; nervousness,
IV 1020 mg irritability
NURSING IMPLICATIONS: IV may drop BP; give slowly; caution client about drowsiness
Diphenhydramine HCl Allergic and pyrogenic Inhibits action of histamine on Sedation, dizziness, inability
ANTIHISTAMINES

(Benadryl)PO 2550 mg 3 or reactions; motion receptor cells; decreases to concentrate; headache;


4 times daily; IM, IV 1020 mg sickness; radiation action of acetylcholine anorexia; dermatitis; nausea;
sickness; hay fever; diplopia, insomnia
Parkinsons disease
Continued
2164_Ch08_603-688 29/03/12 12:34 PM Page 642

642 Common Medications

Table 8.17
Common Medicationscontd
Drug and Dosage Use Action Assessment: Side Effects
NURSING IMPLICATIONS: Avoid use in newborn or preterm infants and clients with glaucoma; supervise ambulation; caution
against driving or operating mechanical devices; excitation or hallucinations may occur in children
Fexofenadine (Allegra)PO Seasonal allergic rhinitis; Antagonizes histamine at the Headache, drowsiness, fatigue;
50 mg twice daily chronic urticaria H1 receptor site dyspepsia; throat irritation
NURSING IMPLICATIONS: Monitor effectiveness; well tolerated; consult physician if breastfeeding
Loratidine (Claritin)PO 10 mg Seasonal allergic rhinitis; Selective peripheral H1 recep- Dizziness; dry mouth, thirst;
daily on empty stomach idiopathic chronic urticaria tor sites, blocking histamine fatigue; headache
release
NURSING IMPLICATIONS: Causes significant drowsiness in older adult
Nedocromil (Tilade)inhalation Asthma Prevents the release of Hypersensitivity, acute
2 sprays histamine from sensitized asthma attacks; does not
mast cells relieve (may accelerate)
bronchospasm
NURSING IMPLICATIONS: Monitor pulmonary function; use only as prescribed; if symptoms increase, notify health-care professional;
teach proper use of metered-dose inhaler
Antihyperglycemics
Alpha-Glucosidase Inhibitor
Acarbose (Precose)PO 25 mg Type 2 diabetes Alpha-glucosidase inhibitors/ Overdose signs: flatulence,
3 times daily; may be increased oral hypoglycemic diarrhea, abdominal discomfort
q48 wk (range: 50100 mg
3 times daily)
NURSING IMPLICATIONS: Observe for side effects of hypoglycemia; take same time each day; do not double dose; therapy is
long term
Biguanide
Metformin (Glucophage)PO Type 2 diabetes Decreases hepatic production Lactic acidosis; nausea,
500 mg twice daily; may increase of glucose; decreases intes- vomiting, diarrhea, abdominal
by 500 mg to 2,000 mg/day Obstetric Use: Also used tinal absorption of glucose bloating
for infertility when the
client has polycystic ovary
syndrome and gestational
diabetes after first
trimester.
NURSING IMPLICATIONS: Monitor glucose levels; take medication as directed; dietary compliance; metallic taste resolves
spontaneously
Sulfonylureas
Acetohexamide (Dymelor) Oral hypoglycemic; Lowers blood glucose by Hypoglycemia (profuse sweat-
PO 2001,500 mg daily; antidiabetic stimulating insulin release ing, hunger, headache, nausea,
12 doses/day; from beta cells; effective only confusion, ataxia, coma; skin
duration 1224 hr Obstetric Use: May be if pancreas has ability to pro- rashes; bone marrow depres-
used after first trimester of duce insulin sion, liver toxicity
pregnancy for controlling
blood sugar in gestational
diabetes
NURSING IMPLICATIONS: Drug therapy must be combined with diet therapy, weight control, and planned, graded exercise; alcohol
intolerance may occur (disulfiram reactionflushing, pounding headache, sweating, nausea, vomiting); should not be taken at bed-
time unless specifically ordered (nocturnal hypoglycemia more likely); take at same time each day; contraindicated in liver disease,
ANTIHYPERGLYCEMIC

renal disease, pregnancy


Chlorpropamide (Diabinese)PO See Acetohexamide See Acetohexamide See Acetohexamide
100500 mg maintenance; not to
exceed 750 mg/day; duration
3060 hr
2164_Ch08_603-688 29/03/12 12:34 PM Page 643

chapter 8 Physiological Integrity 643

Table 8.17
Common Medicationscontd
Drug and Dosage Use Action Assessment: Side Effects
NURSING IMPLICATIONS: See Acetohexamide
Glimepiride (Amaryl), glipizide Type 2 diabetes mellitus Stimulate release of insulin Photosensitivity; hypoglycemia;
(Glucotrol)PO 12 mg once from pancreas ingestion of alcohol may induce
daily; increase slowly up to 8 mg reaction
in one dose; 15 mg/day in
divided dose
NURSING IMPLICATIONS: Observe for signs of hypoglycemia; long-term therapy; avoid aspirin, alcohol; follow prescribed
diet/exercise program
Glyburide (Micronase)PO See Acetohexmide See Acetohexamide See Acetohexamide
2.55 mg initially;
1.2520 mg/day
NURSING IMPLICATIONS: See Acetohexamide
Tolazamide (Tolinase)PO See Acetohexamide See Acetohexamide See Acetohexamide
100500 mg; once/day;
duration 1014 hr
NURSING IMPLICATIONS: See Acetohexamide
Tolbutamide (Orinase)PO See Acetohexamide See Acetohexamide See Acetohexamide
5003,000 mg; 23 doses/day;
duration 612 hr
NURSING IMPLICATIONS: See Acetohexamide
InsulinRapid Acting
Crystalline zinc insulin (regular) Poorly controlled diabetes; Enhances transmembrane Hypoglycemia (profuse sweat-
(Humulin R) (clear)onset trauma; surgery, coma passage of glucose into cells; ing, nausea, hunger, headache,
0.51.0 hr; peak 24 hr; promotes carbohydrate, fat, confusion, ataxia, coma); aller-
duration 68 hr and protein metabolism gic reaction at injection site
Insulin analogue: lispro More rapid absorption and
(Humalog), aspart (Novolog) shorter duration
NURSING IMPLICATIONS: Monitor blood and urine for glucose and acetone levels; insulin currently being used can be kept at
room temperature for 1 mo; refrigerate stock insulin only; rotate injection sites; cold insulin leads to lipodystrophy, reduced absorp-
tion, and local reaction; only form of insulin that is given IV; need to eat within 15 min of insulin analogue injection
InsulinIntermediate Acting Basal
NPH insulin (isophane insulin Clients who can be See Crystalline zinc insulin See Crystalline zinc insulin
suspension) (cloudy)onset controlled by one dose (regular) (regular)
12 hr; peak 58 hr; duration per day
1020 hr
NURSING IMPLICATIONS: Gently rotate vial between palms, invert several times to mix; do not shake; see Crystalline zinc insulin
(regular)
Insulin zinc suspension (lente Clients allergic to NPH See Crystalline zinc insulin See Crystalline zinc insulin
insulin) (cloudy)onset 13 hr; (regular) (regular)
peak 612 hr; duration 1824 hr
NURSING IMPLICATIONS: See Crystalline zinc insulin (regular)
InsulinSlow Acting
Extended insulin zinc suspension Often mixed with semilente See Crystalline zinc insulin See Crystalline zinc insulin
(ultralente) (Humulin U)onset for 24-hr curve (regular) (regular)
ANTIHYPERGLYCEMIC

48 hr; peak 1224 hr; duration


36 hr
NURSING IMPLICATIONS: See Crystalline zinc insulin (regular)

Continued
2164_Ch08_603-688 29/03/12 12:34 PM Page 644

644 Common Medications

Table 8.17
Common Medicationscontd
Drug and Dosage Use Action Assessment: Side Effects
InsulinLong Acting Basal
Insulin glargine (Lantus) (clear) Closely imitates pancreass Absorbs equally over 24-hr
onset 12 hr; duration 24 hr basal insulin release period
NURSING IMPLICATIONS: See Crystalline zinc insulin (regular); glargine may cause mild pain at injection site and cannot be mixed
with other insulins
Antihypertensives
Captopril (Capoten)PO 50 mg Hypertension, heart failure Prevents production of Hypotension; loss of taste
3 times daily angiotensin II; vasodilation perception; proteinuria; rashes
NURSING IMPLICATIONS: Monitor VS and weight; take 1 hr before or 2 hr after meals; change positions slowly; avoid salt and salt
substitutes
Clonidine (Catapres)PO Mild to moderate hyperten- Stimulates alpha-adrenergic Drowsiness; dry mouth;
200600 mcg/day; transdermal sion; also used for epidural receptors in CNS postural hypotension
100300 mcg applied every 7 pain management
days
NURSING IMPLICATIONS: Instruct client to take same time each day; avoid sudden position changes; report: mental depression,
swelling of feet, paleness or cold feeling in fingertips or toes, vivid dreams or nightmares
Guanadrel (Hylorel)PO Moderate to severe Prevents release of norepi- Confusion; fainting; fatigue;
5 mg twice daily (range: hypertension nephrine in response to diarrhea; orthostatic hypoten-
2075 mg/day) sympathetic stimulation sion; dizziness
NURSING IMPLICATIONS: Give with diuretics to minimize tolerance and fluid retention; weigh twice weekly; fewer side effects after
8 wk of therapy
Guanethidine SO4 (Ismelin)PO Severe to moderately Blocks norepinephrine at Orthostatic hypotension; diar-
1050 mg daily in divided doses severe hypertension postganglionic synapses rhea; inhibition of ejaculation

NURSING IMPLICATIONS: Postural hypotension is marked in the morning, accentuated by hot weather, alcohol, and exercise;
teach to rise slowly, with assistance
Guanfacine hydrochloride Hypertension, in combina- Centrally acting alpha- Drowsiness; weakness; dizzi-
(Tenex)PO 1 mg daily to tion with thiazide-like adrenergic receptor agonist ness; dry mouth; constipation;
maximum dose of 3 mg/day diuretics impotence
NURSING IMPLICATIONS: Warn client not to drive or perform activities requiring alertness; take at bedtime to minimize sedation;
monitor BP and pulse
Hydralazine HCl (Apresoline) Moderate hypertension Dilates peripheral blood Palpitations, tachycardia,
PO 1050 mg 4 times daily vessels, increases renal blood angina pectoris, tremors;
flow depression
NURSING IMPLICATIONS: Encourage moderation in exercise and identification of stressful stimuli
Obstetric use: preeclamp- Relaxes peripheral blood Headache, heart palpitation;
sia, eclampsia vessels (opens vascular gastric irritation; coronary
bedphysiological insufficiency; edema; chills,
dehydration) fever; severe depression
NURSING IMPLICATIONS: Side rails up; must not stand without assistance; may be given with diuretics; observe carefully; IM route
only; monitor BP; IV for severe hypertension and preeclampsia; in pregnancy, must not decrease arterial pressure too much or too
rapidly; otherwise, will jeopardize uteroplacental perfusion
Lisinopril (Prinivil)PO Hypertension; heart failure; Inhibits angiotensin-converting Headache, fatigue, hypoten-
10 mg/day, up to 2040 mg post-MI enzyme; alters hemodynamics sion; cough, rash; BUN and
daily or twice daily without reflex tachycardia creatinine
ANTIHYPERTENSIVE

NURSING IMPLICATIONS: Notify physician if sudden drop in BP with supine positioning 15 hr after initial drug dose. Check BP
before dose to determine 24-hr control
Methyldopa (Aldomet)PO Severe to moderately Inhibits formation of dopamine, Initial drowsiness, depression
500 mg2 gm in divided doses severe hypertension a precursor of norepinephrine with feelings of unreality,
edema, jaundice, dry mouth
2164_Ch08_603-688 29/03/12 12:34 PM Page 645

chapter 8 Physiological Integrity 645

Table 8.17
Common Medicationscontd
Drug and Dosage Use Action Assessment: Side Effects
NURSING IMPLICATIONS: Contraindicated in acute and chronic liver disease; encourage not to drive car if drowsy
Minoxidil (Loniten)PO Severe hypertension; Relaxes vascular smooth ECG changes; tachycardia;
5 mg/day (range: 1040 mg/day) end-stage organ failure muscle Na+, water retention;
hypertrichosis
NURSING IMPLICATIONS: Depilatory cream may minimize increased hair growth; report resting pulse increase more than 20 beats/min;
caution client to change positions slowly
Phentolamine hydrochloride Prevents dermal necrosis; Blocks alpha-adrenergic Weakness, dizziness, orthosta-
(Regitine)PO 50 mg 46 doses hypertensive crisis; receptors tic hypotension; nausea,
daily; IV, IM, local 510 mg, pheochromocytoma vomiting, abdominal pain
diluted in minimum 10 mL
normal saline
NURSING IMPLICATIONS: When giving parenterally, client should be supine; monitor for overdosage (precipitous drop in BP); do
not give with epinephrine
Prazosin (Minipress)PO 615 Mild to moderate Dilates arteries and veins by Dizziness, headache, weak-
mg 2 or 3 times daily hypertension blocking postsynaptic ness, palpitations; orthostatic
alpha1-adrenergic receptors hypotension with first dose
NURSING IMPLICATIONS: Monitor: I&O, VS, and weight at beginning of therapy; teach client/family to check BP at least weekly;
need to comply with other interventions (weight reduction, diet, smoking cessation, exercise)
Reserpine (Serpasil) PO Mild to moderate Depletes catecholamines and Depression; nasal stuffiness;
0.25 mg daily hypertension decreases peripheral vasocon- increased gastric secretions;
striction, heart rate, and BP rash, pruritus
NURSING IMPLICATIONS: Watch for signs of mental depression; closely monitor pulse rates of clients also receiving digitalis; avoid
alcohol
Obstetric use: CNS-depressant tranquilizer; Low level of toxicity; weight
Preeclampsia-eclampsia sedation is major effect; gain; diarrhea; allergic
decreases neural transmission reactionsdry mouth, itching,
to nerves; decreases tone in skin eruptions
blood vessels
NURSING IMPLICATIONS: Side rails up; must not stand up without assistance; observe carefully; monitor BP
Timolol (Timoptic)PO Hypertension, migraine, Blocks stimulation of myocar- Fatigue, weakness; depres-
2040 mg/day; ophthalmic 1 gtt glaucoma dial (beta1) and pulmonary/ sion; insomnia; peripheral
12 times/day vascular (beta2) receptors vasoconstriction; diarrhea,
nausea, vomiting
NURSING IMPLICATIONS: Check VS and evidence of heart failure; do not take if pulse <50; avoid OTC cold remedies, coffee, tea,
and cola
Antihypotensives
Metaraminol (Aramine)IM Adrenalectomy Elevated systolic and diastolic Hypertension, cardiac arrhyth-
210 mg, or IV 15100 mg in BP by vasoconstriction mias; may cause hypertensive
500 mL NaCl crisis, if on MAO inhibitors or
antidepressants
NURSING IMPLICATIONS: Monitor VS frequently (q35 min); observe for side effects; have phentolamine available as antidote
Anti-infectives
Amoxicillin (Amoxil)PO Ear, nose, and throat (ENT) Broad spectrum; bactericidal Rash, diarrhea; anaphylaxis
250500 mg q8h; sexually infections; soft tissue
transmitted infection (STI): 3-gm infections (cellulitis);
single dose with probenecid gonorrhea
ANTI-INFECTIVES

NURSING IMPLICATIONS: Determine allergies to penicillin, cephalosporins. Instruct client to take drug around the clock, and to
complete therapy

Continued
2164_Ch08_603-688 29/03/12 12:34 PM Page 646

646 Common Medications

Table 8.17
Common Medicationscontd
Drug and Dosage Use Action Assessment: Side Effects
Cefazolin (Ancef, Kefzol)IM, IV Staphylococcus Bactericidal Allergic reaction; urticaria,
250 mg1.5 gm q612h aureus; Escherichia coli; rash; abnormal bleeding
Klebsiella; group A and B
Streptococcus;
Pneumococcus
NURSING IMPLICATIONS: See Penicillin; may cause false-positive laboratory tests (Coombs, urine glucose); oral probenecid may
be taken concurrently to prolong effects of drug
Cephalexin (Keflex)PO 14 gm Infections caused by Bactericidal effects on suscep- Nausea, vomiting; urticaria;
daily in 24 equally divided gram-positive cocci; tible organisms, inhibition of toxic paranoid reactions;
doses infections: respiratory, bacterial cell wall synthesis dizziness; increased
biliary, urinary, bone, alkaline phosphatase;
septicemia, abdominal; nephrotoxicity; bone marrow
surgical prophylaxis suppression
NURSING IMPLICATIONS: Peak blood levels delayed when given with food; report: nausea, flushing, tachycardia, headache;
monitor for nephrotoxicity and for bleeding
Cephalothin (Keflin, Seffin)IM, Same as cephalexin, See Cephalexin See Cephalexin
IV 212 gm/day in 46 equally except not recommended
divided doses for biliary tract infections
NURSING IMPLICATIONS: See Cephalexin; pain at site of IM injection; given in large muscle; rotate sites
Ciprofloxacin (Cipro)PO Lower respiratory tract Inhibits bacterial DNA Restlessness; nausea,
250750 mg q12h; IV infections; skin, bone, and synthesis diarrhea, vomiting, abdominal
200400 mg q12h; ophthalmic joint infections; urinary pain
12 gtt q1530 min, then tract infection (UTI)
46 times daily
NURSING IMPLICATIONS: Give PO on an empty stomach unless GI irritation occurs, then take with food; do not take with
milk or yogurt; IV over 60 min
Clarithromycin (Biaxin)PO Peptic ulcer Inhibits protein synthesis of Hypersensitivity to other
7.5 mg/kg q12h; usual 500 mg bacterial ribosome antibiotics; use with caution
q12h in liver/renal impairment
NURSING IMPLICATIONS: Take medication around the clock; report signs of superinfection, diarrhea; do not take during pregnancy
Cloxacillin (Tegopen)PO Penicillinase-producing Binds to bacterial cell wall, Nausea, vomiting, diarrhea;
250500 mg q6h staphylococci infections: leading to cell death rashes, allergic reactions;
respiratory, sinus, and skin seizures (high doses)
NURSING IMPLICATIONS: Give around the clock on an empty stomach; observe for signs of superinfection (black, furry tongue,
vaginal itching, loose stools)
Co-trimoxazole (Bactrim, Acute otitis media; urinary Bacteriostatic; anti-infective; Hypersensitivity; see
Septra)PO 160 mg twice daily tract infection; shigellosis; antagonizes folic acid Sulfisoxazole
or 20 mg/kg/day for P. jiroveci P. jiroveci pneumonia; production; combination of
pneumonia prostatitis sulfamethoxazole and
trimethoprim
NURSING IMPLICATIONS: IV administration can cause phlebitis and tissue damage with extravasation
Doxycycline (Vibramycin)PO Chlamydia, other STIs; Inhibits bacterial protein of the Urticaria, diarrhea; nausea,
100 mg q12h on day 1; then 30S bacterial ribosome vomiting; photosensitivity
Lyme disease
100 mg/day  7 days; IV 200 mg
in 12 infusions on day 1; then
100200 mg/day
NURSING IMPLICATIONS: Take prescribed medications as ordered; use sunscreen or protective clothing due to photosensitivity;
ANTI-INFECTIVES

report signs of superinfection, if original symptoms not improved


Erythromycinadults, PO Pneumonia; pelvic inflam- Inhibits protein synthesis of Abdominal cramping,
250 mg q6h; children, PO matory disease; intestinal microorganism; more effective distention, diarrhea
3050 mg/kg daily amebiasis; ocular infec- against gram-positive
tions; used if allergic to organisms
penicillin
2164_Ch08_603-688 29/03/12 12:34 PM Page 647

chapter 8 Physiological Integrity 647

Table 8.17
Common Medicationscontd
Drug and Dosage Use Action Assessment: Side Effects
NURSING IMPLICATIONS: Be sure culture and sensitivity done before treatment; give on empty stomach 1 hr before or 3 hr
after meals; do not crush or chew tablets; do not give with fruit juice
Gentamicin (Garamycin)IM, Serious gram-negative Bactericidal effects on Serious toxic effects:
IV 35 mg/kg/day in 34 divided infections; possible susceptible gram-positive and kidneys, ear; causes muscle
doses; topical; skin, eye S. aureus, uncomplicated gram-negative organisms and weakness/paralysis
urinary tract infections mycobacteria
NURSING IMPLICATIONS: Monitor plasma levels (peak is 410 mcg/mL); clients with burns, cystic fibrosis may need higher doses
Metronidazole (Flagyl)PO Diverticulosis Disrupts DNA and protein Cimetidine may decrease
7.5 mg/kg q6h (not to exceed synthesis in susceptible metabolism; seizures,
4 gm/day) organisms dizziness, abdominal pain
NURSING IMPLICATIONS: Administer with food or milk to decrease GI symptoms; avoid alcohol; has unpleasant metallic taste;
avoid OTC medication while on this drug
Penicillin: penicillin G, penicillin G Streptococcus; Primarily bactericidal Dermatitis and delayed or
potassium, penicillin G procaine, Staphylococcus; immediate anaphylaxis
ampicillinIM 400,0001.2 million Pneumococcus;
units q8h  10 days, or single Gonococcus;
dose of 2.4 million units Treponema pallidum

Obstetric use: group B Prophylaxis


-hemolytic streptococcus
prophylaxis
NURSING IMPLICATIONS: Outpatients should be observed for 20 min after injection; hospitalized clients should be observed at
frequent intervals for 20 min after injection
Pentamidine (Pentam)IV Prevention or treatment of Appears to disrupt DNA or Anxiety, headache; bron-
4 mg/kg once daily; inhalation P. jiroveci pneumonia RNA synthesis to protozoa chospasm, cough; hypotension,
300 mg via nebulizer arrhythmias; nephrotoxicity;
hypoglycemia; leukopenia,
thrombocytopenia, anemia,
chills
NURSING IMPLICATIONS: Assess for infection and respiratory status; unpleasant metallic taste may occur (not significant)
Sulfisoxazole (Gantrisin), sul- Acute, chronic, and Bacteriostatic and bactericidal Nausea, vomiting; oliguria,
famethizole (Thiosulfil), and sulfi- recurrent urinary tract anuria; anemia, leukopenia;
somidine (Elkosin)PO 2.4-gm infections dizziness; jaundice, skin
loading dose, then 12 gm rashes, and photosensitivity
4 times daily  710 days
NURSING IMPLICATIONS: Maintenance of blood levels is important; encourage fluids to prevent crystal formation in kidney
tubulespush up to 3,000 mL/day; avoid in later stage of pregnancy and first month of life for newborn (may cause kernicterus)
Tetracyclines: doxycycline Broad-spectrum antibiotic Primarily bacteriostatic GI upsets such as diarrhea,
hyclate (Vibramycin), nausea, vomiting; sore throat;
oxytetracycline (Terramycin), black, hairy tongue; glossitis;
tetracycline HCl (Sumycin) inflammatory lesions in
PO 250500 mg q6h; IM anogenital region
250 mg/day; IV 250500 mg
q812h
NURSING IMPLICATIONS: Phototoxic reactions have been reported; clients should be advised to stay out of direct sunlight;
medication should not be given with milk or snacks, because food interferes with absorption of tetracyclines; do not give to
women who are pregnant and children under 8 yr
Trimethoprim (TMP)/sulfamethox- Bronchitis, Shigella enteri- Combination inhibits the Nausea, vomiting; rashes;
ANTI-INFECTIVES

azole (SMZ) (Bactrim)PO tis, otitis media, P. jiroveci metabolism of folic acid in phlebitis at IV site; aplastic
160 mg TMP or 800 mg SMZ pneumonia, UTI, travelers bacteria; bactericidal anemia; hepatic necrosis
q12h for 1421 days; IV diarrhea
810 mg/kg (TMP 4050 mg/kg)
q612h
Continued
2164_Ch08_603-688 29/03/12 12:34 PM Page 648

648 Common Medications

Table 8.17
Common Medicationscontd
Drug and Dosage Use Action Assessment: Side Effects
NURSING IMPLICATIONS: Check IV site frequently; do not give IM; give PO on empty stomach; take around the clock; avoid
exposure to sun
Anti-inflammatory Agents
Gastrointestinal
Infliximab (Remicade)IV Autoimmune disorders Blocks the action of tumor Allergic reactions, bleeding
3 mg/kg up to 10 mg/kg (e.g., Crohns disease, necrosis factor-alpha, which problems
ulcerative colitis, psoriatic attacks normal healthy parts
arthritis) of the body
NURSING IMPLICATIONS: Infuse over 2 hr; use in-line filter. Medication to treat hypersensitivity should be available
Mesalamine (Asacol, Pentusa, Ulcerative colitis Local-acting anti-inflammatory Contraindicated if client has
Rowasa)PO 800 mg 3 times/day in GI tract allergy to sulfonamides or
for 6 wk salicylates; acceleration of
symptoms; abdominal pain
NURSING IMPLICATIONS: Monitor for: abdominal pain, diarrhea, blood in stools, superinfection; if no improvement notify health-
care professional; may need diagnostic studies
Olsalazine (Dipentum)PO See Mesalamine See Mesalamine See Mesalamine
1 gm/day in 2 divided doses
NURSING IMPLICATIONS: See Mesalamine
Sulfasalazine (Azulfidine)PO See Mesalamine See Mesalamine See Mesalamine
34 gm/day in divided doses
NURSING IMPLICATIONS: See Mesalamine
Inhalants
Beclomethasone (Beclovent, Emphysema Glucocorticoid (inhalation) Headache, hypersensitivity to
Vanceril)4250 mcg/spray; fluorocarbon propellant; used
2 metered sprays 34 times/day with caution in diabetes or
glaucoma
NURSING IMPLICATIONS: Do not exceed 1 mg/day
Flunisolide (AeroBid) Emphysema Potent locally acting anti- Headache, dysphonia; fungal
4250 mcg/spray; 2 metered inflammatory and immune infections; adrenal suppression
sprays 24 times/day modifier with long-term use
NURSING IMPLICATIONS: Monitor respiratory status and lung sounds; assess clients changing from systemic to inhaled glucocorti-
coids for signs of adrenal insufficiency (anorexia, nausea, weakness, hypotension, hypoglycemia)
Triaminolone (Azmacort)See Asthma See Beclomethasone See Beclomethasone
Beclomethasone
NURSING IMPLICATIONS: See Beclomethasone
Systemics
Cortisone acetatePO, IM Adrenocorticotropic Anti-inflammatory effect of Moon facies, hirsutism,
20100 mg daily in single or hormone (ACTH) unknown action thinning of skin, striae;
divided doses insufficiency; rheumatoid hypertension; menstrual
arthritis; allergies; ulcera- irregularities, delayed healing;
tive colitis; nephrosis psychoses
NURSING IMPLICATIONS: Give oral form pc, with snack at bedtime; give deep IM (never deltoid); monitor vital signs;
observe for behavior changes; skin care and activity to tolerance; dietsalt restricted, high protein, KCl supplement; pro-
tect from injury
ANTIINFLAMMATORY

Desoxycorticosterone acetate Addisons disease; burns; Promotes reabsorption of Edema, hypertension,


(hydrocortisone)IM, IV surgical shock; adrenal sodium; restores plasma pulmonary congestion,
100500 mg q26h; PO surgery volume, BP, and electrolyte hypokalemia
35 mg 24 times daily balance
2164_Ch08_603-688 29/03/12 12:34 PM Page 649

chapter 8 Physiological Integrity 649

Table 8.17
Common Medicationscontd
Drug and Dosage Use Action Assessment: Side Effects
NURSING IMPLICATIONS: Salt restriction according to BP readings; monitor vital signs; weigh daily

Dexamethasone (Decadron)PO Addisons disease; allergic Anti-inflammatory effect See Cortisone acetate
0.55.0 mg daily; IM, IV 420 mg reactions; leukemia;
daily Hodgkins disease; iritis;
dermatitis; rheumatoid
arthritis

Obstetric use: stimulate


surfactant production
in client with premature
labor
NURSING IMPLICATIONS: Contraindicated in tuberculosis; see Cortisone acetate for nursing care
Methylprednisolone sodium Glucocorticoid, See Dexamethasone See Dexamethasone
(Solu-Medrol)IV, IM 1040 mg, corticosteroid
slowly
NURSING IMPLICATIONS: See Dexamethasone
Prednisolone (Deltsone)PO Multiple sclerosis Suppress inflammation and Depression, euphoria; peptic
560 mg/day single or divided normal immune response ulceration; thromboembolism
doses
NURSING IMPLICATIONS: Monitor for: Homans sign, abdominal pain, vomiting, tarry stools, mental status or mood changes
PrednisonePO 2.515.0 mg Rheumatoid arthritis; Anti-inflammatory effect of Insomnia and gastric distress
daily cancer therapy unknown action
NURSING IMPLICATIONS: See Cortisone acetate
Rituximab (Rituxan)IV Non-Hodgkins lymphoma; Monoclonal antibody that inter- Infusion reaction; tumor
1,000-mg doses separated rheumatoid arthritis feres with growth and spread lysis syndrome, skin reaction,
by 2 wk of certain B cells fever, headache, chills,
sneezing
NURSING IMPLICATIONS: Give slowly IV. Avoid vaccines during treatment
Antilipemics
Atorvastatin calcium (Lipitor) Reduce: low-density Increases number of hepatic Back pain, myalgia; constipa-
PO 10 mg daily up to 80 mg/day lipoprotein (LDL), LDL receptors tion, diarrhea; liver function
cholesterol, triglycerides tests
NURSING IMPLICATIONS: Assess for: muscle pain, tenderness, or weaknesscheck creatine phosphokinase (CPK) level. Use
cautiously with: antifungals, niacin, erythromycin
Cholestyramine (Questran)PO Hypercholesterolemia; Binds bile acids in GI tract, Nausea, constipation,
4 gm 14 times/day pruritus from increased increased clearance of abdominal discomfort
bile cholesterol
NURSING IMPLICATIONS: Take before meals; do not take with other medications; give others 1 hr before or 46 hr after
Gemfibrozil (Lopid)PO Hypercholesterolemia May inhibit peripheral lipolysis GI upset (abdominal pain,
1,200 mg/day and reduce triglyceride epigastric pain, diarrhea,
synthesis in liver nausea, vomiting); rash;
headache, dizziness, blurred
vision
NURSING IMPLICATIONS: Use caution when driving or doing tasks requiring alertness; take before meals
Niacin (vitamin B3, nicotinic Hypercholesterolemia Decreases livers production GI upset, flushing; pruritus,
ANTILIPEMICS

acid)PO 1.56.0 gm of LDLs and synthesis of hyperuricemia, hyperglycemia


triglycerides
Continued
2164_Ch08_603-688 29/03/12 12:34 PM Page 650

650 Common Medications

Table 8.17
Common Medicationscontd
Drug and Dosage Use Action Assessment: Side Effects
NURSING IMPLICATIONS: Take the drug with meals; prevent flushing by taking an aspirin 30 min before; monitor closely during
first year of therapy
Simvastatin (Zocor)PO Lipid-lowering Inhibits enzyme responsible Abdominal cramps, constipa-
1020 mg daily for increased synthesis of tion, diarrhea, flatus; heartburn;
cholesterol elevation of liver enzymes,
pancreatitis
NURSING IMPLICATIONS: Give once in the evening with or without food
Antineoplastics
6-Mercaptopurine (Purinethol) Leukemia (ulcerative) Antineoplastic Bone marrow depression,
PO 2.5 mg/kg/day; may increase severe hepatotoxicity
to 5 mg/kg after 4 wk; mainte-
nance 1.52.5 mg/kg/day
NURSING IMPLICATIONS: Monitor: bleeding, infection, liver function tests, skin care, nausea and vomiting
Methotrexate (Rheumatrex)PO, Leukemia Interferes with folic acid Pulmonary fibrosis; hepatotoxi-
IM 1530 mg/day for 5 days; Breast, lung cancer metabolism, resulting in city; stomatitis, signs and
repeat after 1 wk for 35 courses inhibition of DNA synthesis symptoms of chemotherapy
and cell reproduction side effects
NURSING IMPLICATIONS: Monitor: liver function; assess respiratory status; see nursing care of clients receiving chemotherapy,
pp. 526527
Mitotane (Lysodren)PO Adrenocortical carcinoma Affects pituitary disorders GI complaints; lethargy,
910 gm/day in 34 divided somnolence
doses initially; decrease for
maintenance
NURSING IMPLICATIONS: Monitor: signs of disease progression, GI abnormalities, abdominal pain
Anti-Parkinson Agents
Benztropine (Cogentin)PO Parkinsons disease Blocks cholinergic activity Blurred vision, dry eyes;
12 mg/day (range: in CNS constipation; caution with anti-
0.56 mg/day) histamines, phenothiazines
NURSING IMPLICATIONS: Take as directed for best results; monitor for drowsiness, dizziness, orthostatic hypotension; good
mouth care; use caution with OTC drugs, overheating (drug causes a decrease in perspiration)
Biperiden (Akineton)PO 2 mg Parkinsons disease Restores natural balance of Same as Benztropine
34 times/day; not to exceed neurotransmitters in CNS
16 mg/day
NURSING IMPLICATIONS: See Benztropine
Carbidopa/levodopa (Sinemet) Parkinsons disease Levodopa is converted to Use with MAO inhibitors may
PO (10/100, 25/100 dopamine in CNS, where it result in hypertensive crisis;
carbidopa/levodopa ratio) serves as a neurotransmitter some drugs can reverse
1 tablet 3 times/day up to effects (e.g., phenothiazines,
8 tablets daily reserpine)
NURSING IMPLICATIONS: Rate of dosage increase determined by client response; make accurate observations. May experience
on-off phenomenonloss of drug effects and sudden return. Prevent falling from postural hypotension, leg freezing, instability
Trihexyphenidyl (Artane) PO Parkinsons disease Inhibits action of acetylcholine Dizziness, nervousness,
12 mg daily; usual maintenance blurred vision, mydriasis
dose: 515 mg/day in divided
dose
NURSING IMPLICATIONS: Monitor symptoms; avoid orthostatic hypotension; mouth care
ANTIPLATELETS

Antiplatelet Agents
Clopidogrel (Plavix)PO 75 mg Reduce risk of atheroscle- Inhibits platelet aggregation Bleeding; neutropenia;
daily rotic event (MI, stroke) reactions similar to aspirin
NURSING IMPLICATIONS: Give once daily without regard to food; avoid aspirin, NSAIDs; observe for symptoms of: stroke, peripheral
vascular disease, MI
2164_Ch08_603-688 29/03/12 12:34 PM Page 651

chapter 8 Physiological Integrity 651

Table 8.17
Common Medicationscontd
Drug and Dosage Use Action Assessment: Side Effects
Dipyridamole (Persantine)PO Prevent thromboembolism; Decreases platelet aggrega- Headache, dizziness, hypoten-
70100 mg 4 times/day; IV surgical graft patency; tion; coronary vasodilator sion; nausea; MI, arrhythmias
570 mcg/kg diagnostic agent in with IV
myocardial perfusion
studies
NURSING IMPLICATIONS: Take at evenly spaced intervals; avoid use of alcohol; if no GI irritation, take 1 hr before or 2 hr after
meals
Eptifibatide (Integrilin)IV Severe chest pain; small Binds to glycoprotein receptor Bleeding
180-mcg/kg bolus; 2 mcg/kg/min heart attacks; unstable sites of platelets
until discharge or up to 72 hr angina; before balloon
angioplasty
NURSING IMPLICATIONS: Minimize any vascular trauma during treatment. Monitor laboratory tests for indications of bleeding or
clotting disorders; stop drug immediately if bleeding occurs
Ticlopidine (Ticlid)PO 250 mg Stroke prevention; Inhibits platelet aggregation; Diarrhea, rashes, bleeding,
twice daily prevent early restenosis prolongs bleeding time intracerebral bleeding
of coronary stents
NURSING IMPLICATIONS: Give with food or immediately after eating
Antituberculosis Agents*
First-Line Drugs
EthambutolPO 1525 mg/kg See Isoniazid See Isoniazid Optic neuritis (reversible with
discontinuation of drug; very
rare at 15 mg/kg); skin rash
NURSING IMPLICATIONS: Use with caution with renal disease or when eye testing is not feasible; used in combination with
other drug
IsoniazidPO, IM 510 mg/kg Tuberculosis Suppresses or interferes with Peripheral neuritis; hepatitis;
up to 300 mg biosynthesis; bacteriostatic hypersensitivity
NURSING IMPLICATIONS: Give pyridoxine (vitamin B6) 10 mg as prophylaxis for neuritis; 50100 mg as treatment
PyrazinamidePO 1530 mg/kg Tuberculosis Bactericidal Hyperuricemia, hepatotoxicity
up to 3 gm
NURSING IMPLICATIONS: Loss of glycemic control in those with diabetes
RifampinPO 1020 mg/kg; Tuberculosis, meningitis, Impairs RNA synthesis; Hepatitis, febrile reaction,
IV 600 mg Haemophilus influenzae bactericidal purpura (rare)
Rifamate combination (isoniazid
with rifampin)
Rifater combination (isoniazid
and pyrazinamide with
rifampin)see Rifampin
NURSING IMPLICATIONS: Orange urine color; negates effect of birth control pills
StreptomycinIM 1520 mg/kg Tuberculosis, endocarditis, Inhibits protein synthesis; Eighth cranial nerve damage,
up to 1 gm tularemia bactericidal nephrotoxicity
NURSING IMPLICATIONS: Use with caution in older clients or those with renal disease
Antiulcer Agents
Aluminum hydroxide gel Gastric acidity; peptic Buffers HCl in gastric juices Constipation and fecal
(Amphojel)PO 510 mL q24h ulcer; phosphatic urinary without interfering with impaction
or 1 hr pc calculi; phosphorus level electrolyte balance
in chronic renal failure
ANTIULCER

NURSING IMPLICATIONS: Shake well before administering; encourage fluids to prevent impaction and milk-alkali syndrome
Calcium carbonate (Titralac)PO Peptic ulcer and chronic Reduces hyperacidity Constipation or laxative effect
12 gm taken with H2O after gastritis
meals and at bedtime
*To minimize resistant strains, combination therapy is used long-term.
Continued
2164_Ch08_603-688 29/03/12 12:34 PM Page 652

652 Common Medications

Table 8.17
Common Medicationscontd
Drug and Dosage Use Action Assessment: Side Effects
NURSING IMPLICATIONS: See Aluminum hydroxide gel
Cimetidine (Tagamet)PO Duodenal ulcers, gastroe- Inhibits action of histamine at Confusion, dizziness; nausea;
300600 mg q6h; IM, sophageal reflux disease H2 receptor site, inhibits gas- rash; diarrhea, constipation;
IV 300 mg q6h (GERD), gastric hyper- tric acid secretion; neutralizes hypermagnesemia
secretion; peptic acid and absorbs excess acid
gastritis, heartburn;
esophagitis
NURSING IMPLICATIONS: Give with meals or immediately after; avoid smoking; avoid prolonged administration to clients with renal
insufficiency
Famotidine (Pepcid)PO Peptic ulcer Inhibits gastric acid secretion Use with caution: elderly, renal
40 mg/day at bedtime or by inhibiting histamine at H2 failure; confusion, arrhythmias,
20 mg twice a day receptor site agranulocytosis, aplastic
anemia
NURSING IMPLICATIONS: Assess for: epigastric pain, blood in stool, or emesis; monitor laboratory values to avoid side effects;
take medication for full course of treatment; avoid self-medication with OTC drugs; avoid smoking
Lansoprazole (Prevacid)PO Peptic ulcer Binds to an enzyme in pres- Hypersensitivity; use with
15 mg/day; with Helicobacter ence of gastric acid; prevents caution in elderly; dosage
pylori: 30 mg twice a day with final transport of hydrogen ions change needed with hepatic
amoxicillin into gastric lumen impairment
NURSING IMPLICATIONS: Avoid alcohol; report signs of: GI bleeding, cramping, abdominal pain, vomiting
Magnesium and aluminum Gastric hyperacidity; peptic Neutralizes and binds gastric Constipation from aluminum
hydroxides (Maalox ulcer; heartburn; reflux acids, heals ulcers hydroxide; diarrhea from
suspension)PO 530 mL esophagitis magnesium hydroxide
q13h pc and at bedtime
NURSING IMPLICATIONS: Encourage fluid intake; contraindicated for clients who are debilitated or those with renal insufficiency
Misoprostol (Cytotec)PO Peptic ulcer Prostaglandin analogue Hypersensitivity to
200 mcg 4 times/day; rectal prostaglandins; diarrhea;
800 mcg; vaginal 2550 mcg Obstetric use: labor avoid during pregnancy
induction, to control
postpartum hemorrhage
NURSING IMPLICATIONS: Monitor effectiveness of the drug; report increase in abdominal pain, bleeding
Nizatidine (Axid) See Cimetidine See Cimetidine See Cimetidine
see Cimetidine
NURSING IMPLICATIONS: See Cimetidine
Omeprazole (Prilosec)PO Peptic ulcer Gastric acidpump inhibitor Abdominal pain; contraindicated:
20 mg once daily GERD hypersensitivity
NURSING IMPLICATIONS: May cause drowsiness, dizziness; avoid: alcohol, aspirin, NSAIDs, foods that cause GI irritation
Ranitidine (Zantac)PO 150 mg Duodenal ulcer, gastric Inhibits action of histamine Headache, malaise; nausea,
twice daily; IM 50 mg q68h; ulcer, GERD, gastric at H2 receptor site, inhibits constipation, diarrhea
IV 50 mg q68h hypersecretion gastric acid secretion
NURSING IMPLICATIONS: Food does not affect absorption; give 1 hr apart from antacids; smoking interferes with action
Sucralfate (Carafate)PO 1 gm Prevention and treatment Reacts with gastric acid to Constipation
4 times daily of duodenal ulcer form a thick paste that adheres
to the ulcer surface
NURSING IMPLICATIONS: Give 1 hr before meals and at bedtime; do not crush or chew tablets; take antacids 30 min before
or 1 hr after sucralfate; increase fluids and dietary bulk
ANTIVIRALS

Antivirals
Acyclovir (Zovirax)PO 200 mg Herpes simplex (types 1, 2) Converts to an active cytotoxic Headache, nausea, vomiting,
35 times/day; IV 5 mg/kg q8h metabolite that inhibits viral diarrhea; increased serum BUN
over 1 hr; topical 6 times daily DNA replication and creatinine
for 1 wk
2164_Ch08_603-688 29/03/12 12:34 PM Page 653

chapter 8 Physiological Integrity 653

Table 8.17
Common Medicationscontd
Drug and Dosage Use Action Assessment: Side Effects
NURSING IMPLICATIONS: Measure I&O q8h; ensure adequate hydration; assess for common side effects; apply topical with finger
cot or rubber glove; refer for counseling
Indinavir (Crixivan)PO 800 mg HIV; possible prevention in Inhibits action of HIV Ketoacidosis; kidney stones;
q8h combination therapy protease and cleavage of hyperglycemia; altered taste;
viral polyproteins acid regurgitation
NURSING IMPLICATIONS: Give with water or other liquids (skim milk, coffee, tea, juice) 1 hr before or 2 hr after meal; avoid
high-fat, high-protein meal within 2 hr; with concurrent therapy, may need to take drugs 1 hr apart; capsules are sensitive to
moisturekeep desiccant in bottle
Nevirapine (Viramune)PO HIV; delays disease Binds to the enzyme reverse Rash; Stevens-Johnson
200 mg daily for 2 wk, then progression transcriptase; disrupts DNA syndrome (severe erythema
twice daily synthesis multiforme)
NURSING IMPLICATIONS: Give with or without food; drug does not cure AIDSdecreases opportunistic infections
Zidovudine (AZT, Retrovir)PO AIDS and related disorders Inhibits HIV replication Blood disorders, especially
200 mg q4h anemia and granulocytopenia;
headache; nausea; insomnia;
myalgia
NURSING IMPLICATIONS: Monitor for signs of opportunistic infection and adverse drug effects; drug must be taken around the
clock; regular blood tests (q2 wk)
Blood ViscosityReducing Agent
Pentoxifylline (Trental)PO Peripheral vascular disease Increases flexibility of red Nausea, vomiting, GI upset;
400 mg 3 times daily (intermittent claudication) blood cells (RBCs); inhibits drowsiness; dizziness;
platelet aggregation headache
NURSING IMPLICATIONS: Give with meals to minimize GI irritation; take tablets whole; report persistent GI or CNS side effects
Bronchodilators
Albuterol (Ventolin)PO 26 mg Bronchodilator Results in accumulation Nervousness, restlessness,
34 times/day; inhalation q46h of cyclic adenosine tremor, hypertension, nausea
or 2 puffs 15 min before exercise monophosphate (cAMP) at
beta-adrenergic receptors
NURSING IMPLICATIONS: Give with meals; allow 1 min between inhalations; rinse mouth with water after inhalation
AminophyllinePO 250 mg Rapid relief of bron- Relaxes smooth muscles and Nausea, vomiting; cardiac
24 times daily; rectal chospasm; asthma; increases cardiac contractility; arrhythmias; intestinal
250500 mg; IV 250500 mg pulmonary edema interferes with reabsorption of bleeding; insomnia, restless-
over 1020 min Na+ and Cl in proximal tubules ness; rectal irritation from
suppository
NURSING IMPLICATIONS: Give oral with or after meals; monitor vital signs for changes in BP and pulse; weigh daily; IM injections
are painful
Ephedrine SO4PO, Subcu, Asthma: allergies; brady- Relaxes hypertonic muscles in Wakefulness, nervousness,
IM 25 mg 3 or 4 times daily cardia; nasal decongestant bronchioles and GI tract dizziness, palpitations,
hypertension
NURSING IMPLICATIONS: Monitor vital signs; avoid giving dose near bedtime; check urine output in older adults
Ipratropium (Atrovent) Emphysema Inhibits cholinergic receptors in Nasal dryness, epistaxis;
14 inhalations bronchial smooth muscle fluorocarbon toxicity when
used with other inhalants
NURSING IMPLICATIONS: Instruct in proper use of inhaler; do not use more than 12 metered doses in 12 hr; if severe bron-
chospasm occurs, consult physician/nurse practitioner
BRONCHODILATORS

Isoproterenol HCl (Isuprel) Mild to moderately severe Relaxes hypertonic bronchioles Nervousness, tachycardia;
inhalation of 1:100 or asthma attack; bronchitis; hypertension; insomnia
1:200 solution pulmonary emphysema
NURSING IMPLICATIONS: Monitor vital signs before and after treatment; teach client how to use nebulizer

Continued
2164_Ch08_603-688 29/03/12 12:34 PM Page 654

654 Common Medications

Table 8.17
Common Medicationscontd
Drug and Dosage Use Action Assessment: Side Effects
Metaproterenol (Alupent)PO Asthma Bronchodilation Nervousness, restlessness,
20 mg 34 times/day; inhalation tremor, paradoxical
23 metered-dose inhalations bronchospasm
NURSING IMPLICATIONS: Assess lung sounds, observe for wheezing (paradoxical bronchospasm); observe clients respiratory
signs and symptoms; refer to health-care professional if symptoms persist or shortness of breath continues
Salmeterol (Serevent)inhalation Asthma Produces accumulation of Headache, palpitations,
50 mcg twice a day (2 inhalations) cAMP at beta-adrenergic abdominal pain; avoid use
receptors when on beta-adrenergic
agents (decreases
effectiveness)
NURSING IMPLICATIONS: Assess lung sounds; monitor pulmonary function; do not use for acute symptoms
Terbutaline (Brethine)PO Bronchospasm See Isoproterenol See Isoproterenol
2.55.0 mg q6h (not to exceed
20 mg/24 hr); Subcu 0.25 mg, Obstetric use: may be If used in pregnancy: maternal
repeat 1530 min (not to exceed used for premature labor; and fetal tachycardia
0.5 mg/hr); inhalation 2 puffs not Food and Drug
(0.2 mg each) q46h Administration (FDA)
approved for use as
tocolytic agent, but
widely used
NURSING IMPLICATIONS: See Isoproterenol
TheophyllinePO 400 mg/day in Treatment/prevention of Bronchodilation Restlessness; increased respi-
divided doses; max. adult dose: emphysema, asthma ration, heart rate; palpitations,
900-mg divided dose (bronchoconstriction); arrhythmias; nausea, vomiting;
chronic bronchitis increased urine output
dehydration
NURSING IMPLICATIONS: Monitor theophylline levels: 1020 mcg/mL; monitor signs of toxicity; take with 8 oz water or with meals
to decrease GI symptoms
Zafirlukast (Accolate)PO 20 mg Asthma Antagonizes the effects of Headache, fever, infection,
twice daily leukotrienes pain
NURSING IMPLICATIONS: Monitor lung sounds, respiratory function
Zileuton (Zyflo)PO 600 mg Asthma Enzyme inhibitor Headache, dizziness; chest
4 times daily pain; increases blood level
of theophylline, propranolol,
warfarin
NURSING IMPLICATIONS: Assess lung sounds; do not discontinue or reduce dosage without consulting health-care professional;
not used to treat acute attack
Calcium Channel Blockers
Amlodipine (Norvasc)PO Hypertension, angina Inhibits the transport of Arrhythmias, peripheral edema,
510 mg daily pectoris, vasospastic calcium into myocardial and heart failure; photosensitivity;
(Prinzmetals) angina, vascular smooth muscle gingival hyperplasia
muscle cells
NURSING IMPLICATIONS: Instruct client to: monitor pulse, report rate <50 beats/min, take with or without food
Diltiazem (Cardizem)PO Angina, hypertension, atrial Inhibits excitation-contraction Headache, fatigue, arrhyth-
30120 mg 34 times/day; arrhythmias coupling; decreased SA, AV mias, edema, hypotension;
IV 515 mg/hr node conduction constipation; rash
CA+ CHANNEL BLOCK

NURSING IMPLICATIONS: May take with meals; take pulse; do not give drug if pulse <50 beats/min; change positions slowly; may
take nitroglycerin sublingually concurrently
Metoprolol (Lopressor)PO Hypertension Blocks stimulation of Contraindicated in: congestive
100 mg/day up to 450 mg/day myocardial adrenergic heart failure (CHF), pulmonary
receptors (beta1) edema; caution with renal
impairment
2164_Ch08_603-688 29/03/12 12:34 PM Page 655

chapter 8 Physiological Integrity 655

Table 8.17
Common Medicationscontd
Drug and Dosage Use Action Assessment: Side Effects
NURSING IMPLICATIONS: Monitor vital signs, especially BP; prevent orthostatic hypotension; avoid self-medication with OTC; may
cause drowsiness; medication may increase sensitivity to cold
Nicardipine (Cardene)PO Angina, hypertension Inhibits excitation-contraction Dizziness, light-headedness,
2040 mg 3 times daily coupling headache; peripheral edema;
flushing
NURSING IMPLICATIONS: Give on an empty stomach; chest pain may occur 30 min after dose (temporary, from reflex tachycardia)
Nifedipine (Procardia)PO Angina, hypertension Vasodilation Dizziness, light-headedness,
1030 mg 3 times daily (not to giddiness, headache, nervous-
exceed 180 mg/day); sublingual Obstetric use: may be ness; nasal congestion; sore
10 mg repeated in 15 min used for premature labor; throat, dyspnea, cough,
not FDA approved for use wheezing; nausea; flushing,
as tocolytic agent, but warmth
widely used
NURSING IMPLICATIONS: May take with meals; make position changes slowly; angina may occur 30 min after dose (temporary)
Verapamil (Calan, Isopten)PO Angina; supraventricular Inhibits calcium movement into Constipation, AV block,
240480 mg, 34 times/day; arrhythmias; essential smooth-muscle cells; lowers hepatotoxicity
IV 75150 mcg/kg over 2 min hypertension pressure by reducing cardiac
contractility
NURSING IMPLICATIONS: Monitor VS and ECG for bradycardia and arrhythmias; observe for jaundice, abdominal pain;
encourage fluids and bulk-forming foods
Cardiac Glycosides
Digitoxindigitalizing dose: PO Heart failure: atrial Increases force of cardiac Arrhythmias; nausea, vomiting;
200 mcg twice/day; IM, IV fibrillation and flutter; contractility, slows heart rate, anorexia, malaise; color vision,
200400 mcg; maintenance supraventricular decreases right atrial pres- yellow or blue
dose: PO 50300 mcg daily tachycardia sures, promotes diuresis
NURSING IMPLICATIONS: Hold medication if pulse rate less than 50 or over 120; encourage foods high in potassium
(e.g., bananas, orange juice); observe for signs of electrolyte depletion, apathy, disorientation, and anorexia
Digoxin (Lanoxin)digitalizing See Digitoxin See Digitoxin See Digitoxin
dose: PO 1015 mcg/kg; IV
0.61 mg/kg; maintenance dose:
PO 0.1250.375 mg daily
NURSING IMPLICATIONS: See Digitoxin
Central Nervous System Stimulants
Amphetamine SO4PO 560 mg Mild depressive states; Raises BP, decreases sense of Restlessness, dizziness,
daily in divided doses narcolepsy; posten- fatigue, elevates mood tremors, insomnia; increased
cephalitic parkinsonism; libido; suicidal and homicidal
obesity control; minimal tendencies; palpitations;
brain dysfunction in anginal pain
children (attention deficit
disorder)
NURSING IMPLICATIONS: Give before 4 p.m. to avoid sleep disturbance; dependence on drug may develop; contraindicated with:
MAO inhibitors, hyperthyroidism, and psychotic states
Methylphenidate hydrochloride Childhood hyperactivity; Mild CNS and respiratory Anorexia; dizziness, drowsi-
(Ritalin)PO 0.3 mg/kg/day or narcolepsy; attention stimulation ness, insomnia, nervousness;
adults 2060 mg in divided deficit disorder in children BP and pulse changes
doses
NURSING IMPLICATIONS: To avoid insomnia, take last dose 45 hr before bedtime; monitor vital signs; check weight 23 times
CNS STIMULANTS

weekly and report weight losses

Continued
2164_Ch08_603-688 29/03/12 12:34 PM Page 656

656 Common Medications

Table 8.17
Common Medicationscontd
Drug and Dosage Use Action Assessment: Side Effects
Chemotherapy and Antineoplastics (see Table 8.11 and Table 8.12)
Cholinergics
Bethanechol chloride Postoperative abdominal Increases GI and bladder tone; Belching, abdominal cramps,
(Urecholine)PO 530 mg; atony and distention; blad- decreases sphincter tone diarrhea, nausea, vomiting;
Subcu 2.55.0 mg der atony with retention; incontinence; profuse sweat-
postsurgical or postpartum ing, salivation; respiratory
urinary retention; myasthe- depression
nia gravis
NURSING IMPLICATIONS: Check respirations; have urinal or bedpan close at hand and answer calls quickly; atropine SO4 is the
antidote for cholinergic drugs
Edrophonium (Tensilon)IV Diagnosis of myasthenia Inhibits breakdown of acetyl- Excess secretions; bron-
2 mg; IM 10 mg gravis; reversal of neuro- choline; anticholinesterase chospasm; bradycardia;
muscular blockers abdominal cramps, vomiting,
diarrhea; excess salivation;
sweating
NURSING IMPLICATIONS: Effects last up to 30 min; give IV undiluted with TB syringe
Neostigmine (Prostigmin)PO Myasthenia gravis, postop- Inhibits breakdown of Excess secretions; bron-
1530 mg q34h; Subcu, erative bladder distention, acetylcholine chospasm; bradycardia;
IM 0.251 mg urinary retention, reversal abdominal cramps, nausea,
of neuromuscular blockers vomiting, diarrhea; excess
salivation; sweating
NURSING IMPLICATIONS: Take oral form with food or milk; with chewing difficulty, take 30 min before eating
Physostigmine salicylate Glaucoma; reverse effects Increases concentration of Sweating; marked miosis,
(Eserine)ophthalmic 0.1 mL of of: tricyclic antidepres- acetylcholine lacrimation; nausea, diarrhea
0.25%10% solution: not more sants, overdose, toxic
than 1 drop 4 times daily; IM, effects of atropine and
IV 0.53 mg diazepam
NURSING IMPLICATIONS: If brown eyed, may require stronger solution or more frequent dose
Pilocarpine HClophthalmic Chronic open-angle and Contraction of the sphincter Brow ache, headache, ocular
12 gtt 1%2% solution up to acute closed-angle muscle of iris, resulting in pain, blurring and dimness of
6 times/day glaucoma miosis vision, allergic conjunctivitis;
nausea, vomiting, profuse
sweating; bronchoconstriction
in clients with bronchial
asthma
NURSING IMPLICATIONS: Initially the medication may be irritating; teach proper sterile technique for instilling dropswipe excess
solution to prevent systemic symptoms; discard cloudy solutions
Pyridostigmine (Mestinon)PO Myasthenia gravis; reversal Inhibits breakdown of acetyl- See Neostigmine
60 mg/day  3 days; IM, IV 2 mg of neuromuscular blockers choline and prolongs its effects
q23h; maintenance:
60150 mg/day
NURSING IMPLICATIONS: See Neostigmine
Decongestants
Phenylephrine (Neo-Synephrine, Shock, hypotension, Constricts blood vessels by Dizziness, restlessness, dysp-
Sinex)Subcu, IM 25 mg decongestant, adjunct stimulating alpha-adrenergic nea, tachycardia, arrhythmias;
q1015 min, not to exceed 5 mg; to spinal anesthesia, receptors ophthalmological: burning,
IV 4060 mcg/min; nasal 23 gtt mydriatic photophobia, tearing
or 12 sprays q34h; ophthalmic
DECONGESTANTS

3 gtt/day
NURSING IMPLICATIONS: Check for correct concentration; protect eyes from light sensitivity; blow nose before using; rebound
congestion will occur with prolonged use
Pseudoephedrine (Sudafed)PO Nasal congestion, allergies, Produces vasoconstriction in Nervousness, anxiety, palpita-
60 mg q46h chronic ear infections respiratory tract and possibly tions, anorexia
bronchodilation
2164_Ch08_603-688 29/03/12 12:34 PM Page 657

chapter 8 Physiological Integrity 657

Table 8.17
Common Medicationscontd
Drug and Dosage Use Action Assessment: Side Effects
NURSING IMPLICATIONS: Give at least 2 hr before bedtime to minimize insomnia
Electrolyte and Water Balance Agents
Acetazolamide (Diamox)PO Glaucoma; heart failure; Weak diuretic; produces Electrolyte depletion
2501,000 mg/day; IV 500 mg convulsive disorders acidosis; self-limiting effect; symptomatologylassitude,
increases bicarbonate apathy, decreased urinary
excretion output, mental confusion
NURSING IMPLICATIONS: Weigh daily, I&O; assess edema; give early in day to allow sleep at night; observe for side effects;
replace electrolytes as ordered
Ethacrynic acid (Edecrin)PO Pulmonary edema; ascites; Inhibits the reabsorption of Nausea, vomiting, diarrhea;
50200 mg daily in divided doses edema of heart failure Na+ in the ascending loop of hypokalemia; hypotension;
Henle gout; dehydration; deafness;
metabolic acidosis
NURSING IMPLICATIONS: Assess for dehydrationskin turgor, neck veins; hypotension; KCl supplement
Furosemide (Lasix)PO Edema and associated Inhibits Na+ and Cl reabsorp- Dermatitis, pruritus; paresthe-
4080 mg daily in divided doses heart failure; cirrhosis; tion in the loop of Henle sia; blurring of vision; postural
renal disease; nephrotic hypotension; nausea, vomiting,
syndrome: hypertension diarrhea, dehydration, elec-
trolyte depletion; hearing loss
(usually reversible)
NURSING IMPLICATIONS: Assess for weakness, lethargy, leg cramps, anorexia; peak action in 12 hr; duration 68 hr; do not give
at bedtime; supplemental KCl indicated; may induce digitalis toxicity
Hydrochlorothiazide Edema; heart failure; Na+ Inhibits NaCl and water reab- Hypokalemia, nausea, vomit-
(HydroDIURIL, Esidrix, retention in steroid therapy; sorption in the distal ascending ing, diarrhea; dizziness;
Microzide)PO 25100 mg hypertension loop and the distal convoluted paresthesias; may exacerbate
3 times daily tubule of the kidneys diabetes
Chlorothiazide (Diuril)PO
5001,000 mg daily oral
suspension
NURSING IMPLICATIONS: Watch for muscle weakness; give well-diluted potassium chloride supplement; monitor urine for changes
in sugar and acetone
Mannitol (Osmitrol)IV Cerebral edema, oliguria, Hypertonic solution that kidney Diarrhea; water intoxication,
50100 mg/24 hr as acute renal failure, drug tubules cannot reabsorb, thirst, headache
15%20% solution intoxication thereby causing obligatory
water loss
NURSING IMPLICATIONS: Usually Foley catheter required; monitor cardiac and respiratory status; a rate of infusion to elicit 3050 mL
urine/hr
Sevelamer hydrochloride Reduction of serum Polymer binds intestinal Headache; infection; pain;
(Renagel)PO 2 capsules or phosphorus level phosphate diarrhea, constipation; cough
tablets 3 times daily
NURSING IMPLICATIONS: Important to take multivitamin supplement approved by physician. Do not take after printed expiration
date
Spironolactone (Aldactone)PO Cirrhosis of liver; when other Inhibits effects of aldosterone Headache; lethargy; diarrhea;
25 mg 24 times daily diuretics are ineffective in distal tubules of kidney ataxia; skin rash; gynecomastia
NURSING IMPLICATIONS: Potassium-sparing drug; do not give supplemental KCl; monitor for signs of electrolyte imbalance
Emetic
Ipecac syrupAdults, elderly Emergency emetic for NH4 ions cause gastric Violent emesis, tachycardia,
adults, children >12 yr: PO poison ingestion irritation decreased BP, dyspnea
1530 mL followed by
EMETIC

34 glasses of water.
Children 112 yr: PO 15 mL
followed by 12 glasses of water.
Infants 6 mo1 yr: PO 510 mL
followed by 1 glass of water
Continued
2164_Ch08_603-688 29/03/12 12:34 PM Page 658

658 Common Medications

Table 8.17
Common Medicationscontd
Drug and Dosage Use Action Assessment: Side Effects
NURSING IMPLICATIONS: Contraindicated in liver and renal disease; if given for emesis, follow dose with as much water as client
will drink. Should not be used in the home. Dosage and usage are controversial, as is repetition of dose if vomiting has not occurred
in 2030 min after giving first dose.
Enzyme
Pancrelipase (Viokase)adults, Chronic pancreatitis; cystic Assists in digestion of starch, Anorexia, nausea, vomiting,
PO 325 mg1 gm daily, during fibrosis; gastrectomy; protein, and fats; decreases diarrhea; buccal/anal soreness
meals; children, PO 300600 mg pancreatectomy; sprue nitrogen and fat content of (infants); sneezing; skin rashes;
3 times daily stool diabetes
NURSING IMPLICATIONS: May be taken with antacid or cimetidine; do not crush or chew tablets; monitor I&O, weight; be alert for
signs of diabetes; children may use sprinkles
Erectile Dysfunction
Sildenafil citrate (Viagra)PO Erectile dysfunction Vasodilation of nitric oxide in Headache; cardiac arrest
50 mg 0.54 hr before sexual the corpus cavernosum of with sudden death; exfoliative
activity penis dermatitis
NURSING IMPLICATIONS: High-fat meal delays drug action. Report to physician: headaches, flushing, chest pain, indiges-
tion, blurred vision, changes in color vision; safety issue if taking nitrates
Expectorants
Guaifenesin (Robitussin)PO Respiratory congestion Increases expectoration Low incidence of Gl upset;
100400 mg q4h by causing irritation of drowsiness
gastric mucosa; reduces
adhesiveness/surface tension
of respiratory tract fluid
NURSING IMPLICATIONS: Encourage to stop smoking; increase fluid intake; respiratory hygiene
Terpin hydratePO 510 mL Bronchitis; emphysema Liquefies bronchial secretions Nausea, vomiting, gastric
q34h irritation
NURSING IMPLICATIONS: Give undiluted; push fluids
Fibrinolytic Agents
Alteplase, recombinant (Activase, Acute MI; under investiga- Promotes conversion of plas- Internal or local bleeding;
tPA)IV bolus 100 mg over tion for pulmonary emboli, minogen to plasmin, which is urticaria; dysrhythmias related
90 min: 15-mg bolus given deep vein thrombosis, fibrinolytic to reperfusion; hypotension;
over 12 min; then 50 mg and peripheral artery nausea, vomiting
over 30 min; then 35 mg over thrombosis
60 min
NURSING IMPLICATIONS: Assess for signs of reperfusion (relief of chest pain, no ST-segment elevation); observe for bleeding;
avoid IM injection; do not mix other medications in line
Streptokinase (Stretase)IV Lysis of pulmonary or Reacts with plasminogen, Prolonged coagulation; allergic
1.5 million units initially diluted; systemic emboli or dissolves fibrin clots reactions; mild fever
infused over 60 min thrombi; acute MI
NURSING IMPLICATIONS: Monitor for signs of excessive bleeding, particularly at injection sites; avoid nonessential handling of
client
UrokinaseIV 4,400 IU/kg Massive pulmonary emboli, Directly activates plasminogen Bleeding, anaphylaxis, rash
over 10 min; 1.01.8 mL of coronary artery thrombi,
5,000 IU/mL into catheter occluded IV catheter
NURSING IMPLICATIONS: Vital signs; check q15 min for bleeding during first hr; q1530 min for 8 hr; have epinephrine ready
Fibrinolytic Antidote
FIBRINOLYTIC

Aminocaproic acid (Amicar)PO, Management of streptoki- Inhibits plasminogen activator Hypotension, bradycardia,
IV 5-gm loading dose; 1 gm/hr to nase or urokinase and antagonizes plasmin cardiac arrhythmias
30 gm in 24 hr overdose
NURSING IMPLICATIONS: Give slowly IV to prevent side effects; not recommended for disseminated intravascular coagulation (DIC)
2164_Ch08_603-688 29/03/12 12:34 PM Page 659

chapter 8 Physiological Integrity 659

Table 8.17
Common Medicationscontd
Drug and Dosage Use Action Assessment: Side Effects
Hematopoietic Agents
Darbepoetin alfa (Aranesp)IV, Treatment of anemia with Erythropoiesis-stimulating Peripheral edema; infection;
Subcu 25500 mcg/kg once chronic renal failure, protein headache, hypertension,
weekly chemotherapy hypotension; arrhythmias;
nausea, vomiting, diarrhea;
myalgia, respiratory infection
NURSING IMPLICATIONS: See Epoetin
Epoetin alfa (Epogen, Procrit) Anemia secondary to Glycoprotein that stimulates Headache, hypertension; iron
Subcu, IV 3500 units/kg/dose chronic kidney failure, RBC production deficiency, clotting of AV fistula
3 times/wk chemotherapy, HIV
treatment
NURSING IMPLICATIONS: Monitor BP closely. Headache is common; report severe or persistent problem
Hormones
Chlorotrianisene (TACE), Prostatic cancer; Nonsteroidal synthetic Rare after one course of
estrogenPO 1250 mg menopause estrogen treatment; thromboembolism;
impotence and gynecomastia
in men
NURSING IMPLICATIONS: Supply client with package insert; contraindicated in blood coagulation disorders
Diethylstilbestrol (DES)PO, IM Prostate carcinoma; Synthetic nonsteroidal Anorexia, nausea, vomiting,
0.25.0 mg daily; vaginal sup- menopausal symptoms; compound with estrogenic headache, diarrhea, dizziness,
pository 0.10.5 mg at bedtime osteoporosis; pain; effects on pituitary, ovaries, faintingmany side effects
mammary carcinoma; myometrium, endometrium, with long-term use
atrophic vaginitis and other tissues
NURSING IMPLICATIONS: Never give if woman is pregnantpredisposes to vaginal cancer in female offspring at puberty
EstradiolPO 12 mg for Menopausal symptoms; Inhibits release of pituitary Anorexia, nausea, vomiting,
replacement therapy; up to osteoporosis; hypogenital- gonadotropins; promotes diarrhea, fluid retention; mental
10 mg for cancer daily and up to ism; sexual infantilism; growth of female genital depression; headache, throm-
3 times daily; cyclic (on 3 wk, off breast and prostatic tissues boembolism; feminization
1 wk) carcinoma in men
NURSING IMPLICATIONS: Baseline VS; weigh daily; encourage frequent physical checkups to check serum lipids; teach breast
self-examination
Medroxyprogesterone Amenorrhea; functional Similar to progesterone, but Drowsiness; cyclic menstrual
(Provera)PO 2.510.0 mg; uterine bleeding; threat- can be taken orally; thickens withdrawal bleeding; GI upset;
IM 4001,000 mg weekly ened abortion; dysmenor- uterine decidua headache; edema; breast
rhea; adjunctive and congestion
palliative with renal cancer
and endometriosis;
premenstrual syndrome
(PMS)
NURSING IMPLICATIONS: Teach client regarding self-administration; breast self-examination for possible breast changes
Menotropins (Pergonal)IM Infertility use: treatment Human gonadotropic Abortions occur in 25%; failure
1 ampule (FSH + LH)/day for of secondary anovulation; responses; induces rate 55%80% of clients:
912 days (followed by stimulation of ovulation; sperm stimulation possible multiple births;
5,00010,000 units human spermatogenesis ovarian enlargement;
chorionic gonadotropin (HCG); gynecomastia in men
if ovulation does not occur,
repeat with 2 ampules)
NURSING IMPLICATIONS: Assist in collection of urine to assess estrogen levels; counsel regarding couples need to have daily
intercourse from day of HCG injection until ovulation
HORMONES

ProgesteroneSubcu, IM Amenorrhea; dysmenorrhea; Converts endometrium into Nausea, vomiting; dizziness,


510 mg daily; sublingual endometriosis; habitual secreting structure; prevents edema, headache; protein
510 mg abortion ovulation; stimulates growth of metabolism
mammary tissue
Continued
2164_Ch08_603-688 29/03/12 12:34 PM Page 660

660 Common Medications

Table 8.17
Common Medicationscontd
Drug and Dosage Use Action Assessment: Side Effects
NURSING IMPLICATIONS: Give deep IM and rotate sites; weigh daily to ascertain fluid retention
TestosteronePO 510 mg daily; Hypogonadism; Growth of sex organs and Nausea, dyspepsia; masculin-
IM 2550 mg 23 times/wk; eunuchism; impotence, appearance of secondary ization; hypercalcemia;
200400 mg IM q24 wk for advanced cancer of male sex characteristics; menstrual irregularities; renal
breast cancer breast counteracts excessive calculi; Na+, K+, and H2O
amounts of estrogen retention
NURSING IMPLICATIONS: Observe for edema; weigh daily; I&O; push fluids for clients who are bedridden, to prevent renal calculi
Vasopressin (Pitressin)IM, Diabetes insipidus Antidiuretic Nausea, flatus; tremor; pound-
Subcu 510 mcg 24 times daily; ing in head; water intoxication
IM, Subcu 2.55 mcg 23 times
daily for chronic therapy
NURSING IMPLICATIONS: Monitor fluid balance, vital signs
Immune Modifiers
Glatiramer acetate (Copaxone) Multiple sclerosis (MS) Management of relapsing- Allergic reactions; anxiety;
Subcu 20 mg/day remitting MS chest pain; weight gain; cough
NURSING IMPLICATIONS: Monitor for side effects; new drug to attempt to control signs and symptoms of progressing multiple
sclerosis
Interferon beta-1b (Betaseron) Multiple sclerosis Reduce incidence of relapse Seizures; GI symptoms; flu-like
Subcu 30 mcg once a week symptoms; neutropenia, injec-
tion site reactions
NURSING IMPLICATIONS: Monitor progress of multiple sclerosis; determine if drug is effective in reducing incidence of relapse;
provide information about multiple side effects
Immunosuppressant Agents
Azathioprine (Imuran)PO, IV Prevent renal transplant Antagonizes purine Hypersensitivity; additive
35 mg/kg/day initially; mainte- rejection metabolism/inhibition of myelosuppression with anti-
nance: 13 mg/kg/day Ulcerative colitis DNA/RNA synthesis neoplastics; hepatotoxicity,
Rheumatoid arthritis leukopenia
NURSING IMPLICATIONS: Assess for infection; monitor I&O and daily weight
Cyclosporine (Sandimmune)PO Prevent organ rejection Inhibits interleukin-2, which is Hypersensitivity; caution with
510 mg/kg several hours before Ulcerative colitis; necessary for initiation of T-cell alcohol, renal impairment
surgery; daily for 2 wk; reduce Rheumatoid arthritis activity
dosage by 2.5 mg/kg/wk to
510 mg/kg/day; IV 56 mg/kg
several hours before surgery;
switch to PO form as soon as
possible
NURSING IMPLICATIONS: Monitor: I&O, daily weight, blood pressure; take as directed (time of day and with food); observe for
nephrotoxicity, gingival hyperplasia; avoid self-administration of OTC drugs
Muromonab-CD 3 (Orthoclone Renal, liver, or cardiac Immunosuppression of T-cell Tremor; dyspnea, shortness of
OKT3)IV 5 mg/day, 1014 days transplant rejection function breath, wheezing; chest pain;
diarrhea, nausea, vomiting;
chills, fever
NURSING IMPLICATIONS: Draw solution into filter; give IV push over <1 min; do not give as IV infusion; do not mix with other
medications
Mycophenolate (CellCept)PO Prevent renal transplant Inhibits purine synthesis; sup- Diarrhea; vomiting; leukopenia;
1 gm twice daily rejection presses T and B lymphocytes sepsis
IMMUNOSUPPRES

NURSING IMPLICATIONS: Give initial dose within 72 hr of transplant surgery; give on empty stomach 1 hr before or 2 hr after
meals; do not open, crush, or chew capsules; do not give with magnesium or aluminum antacids
TacrolimusPO 0.0750.15 mg/kg Prevent rejection of liver Inhibits T-lymphocyte Headache, insomnia, tremor;
q12h; IV 0.050.1 mg/kg/day transplant activation ascites; hypertension; periph-
eral edema; abdominal pain;
rash; hyperglycemia, anemia
2164_Ch08_603-688 29/03/12 12:34 PM Page 661

chapter 8 Physiological Integrity 661

Table 8.17
Common Medicationscontd
Drug and Dosage Use Action Assessment: Side Effects
NURSING IMPLICATIONS: Give 6 hr after transplant surgery; oral therapy is preferred because of anaphylaxis with IV; therapy is
lifelong
Mineralocorticoid Agent
Fludrocortisone (Florinef)PO Addisons disease Causes sodium reabsorption, Heart failure; adrenal suppres-
0.10.2 mg daily hydrogen and potassium sion, hypertension
excretion, and water retention
NURSING IMPLICATIONS: Do not miss dose; therapy is lifelong; high potassium; report: weight gain, muscle weakness, cramps,
nausea, anorexia, dizziness
Mucolytic Agent
Acetylcysteine (Mucomyst) Emphysema; pneumonia; Lowers viscosity of respiratory Stomatitis, nausea; rhinorrhea;
inhalation 15 mL of 20% solu- tracheostomy care; atelec- secretions by opening disulfide bronchospasm
tion or 110 mL of 10% solution tasis; cystic fibrosis linkages in mucus
per nebulizer 3 times daily Antidote to acetaminophen
overdose, to protect
against hepatotoxicity
NURSING IMPLICATIONS: Observe respiratory rate; maintain open airway with suctioning as necessary; observe clients with asthma
carefully for increased bronchospasm; discontinue treatment immediately if this occurs; odor disagreeable initially
Muscle Relaxants
Baclofen (Lioresal)PO 5 mg Relief of spasticity of multi- Centrally acting skeletal Pruritis, tinnitus; nausea, vom-
3 times daily up to 1020 mg ple sclerosis, spinal cord muscle relaxant; depresses iting; diarrhea or constipation;
4 times/day maintenance dose injury polysynaptic afferent reflex drowsiness
activity at spinal cord level
NURSING IMPLICATIONS: Administer with food if GI symptoms; monitor for safety when ambulating; do not discontinue abruptly
Dantrolene sodium See Baclofen See Baclofen See Baclofen
(Dantrium)PO 25 mg/day
25 mg 23 times daily to 100 mg
4 times daily max.
NURSING IMPLICATIONS: See Baclofen
Narcotic Antagonist
Naloxone HCl (Narcan)IV Reverses respiratory Reverses respiratory depres- No common side effects;
0.10.2 mg repeated q23 min depression due to sion of morphine SO4, meperi- tachycardia, hypertension;
prn up to 0.42 mg for narcotic- narcotics dine HCl, and methadone nausea and vomiting with high
induced respiratory depression HCl; does not itself cause res- doses
piratory depression, sedation,
or analgesia
NURSING IMPLICATIONS: Note time, type of narcotic, dosage received; not useful with CNS depression from other drugs; respiratory
depression may return; monitor closely
Sedatives and Hypnotics
Chloral hydratePO 250 mg Sedation for elderly; Depresses sensorimotor areas Nausea, vomiting, gastritis;
3 times daily; hypnotic: PO delirium tremens; pruritus; of cerebral cortex pinpoint pupils; delirium; rash;
0.51.0 gm; rectal suppository mania; barbiturate and decreased BP, pulse, respira-
0.30.9 gm alcohol withdrawal tions, temperature; hepatic
damage
NURSING IMPLICATIONS: Caution: should not be taken in combination with alcohol; dependency is possible
Chlordiazepoxide HCl (Librium) Psychoneuroses; CNS depressant resulting in Ataxia, fatigue; blurred vision,
SEDATIVES-HYPNO

PO 510 mg; IM, IV 50100 mg preoperative apprehension; mild sedation; appetite stimu- diplopia; lethargy; nightmares;
chronic alcoholism; anxiety lant; anticonvulsant confusion
NURSING IMPLICATIONS: Ensure anxiety relief by allowing client to verbalize feelings; advise client to avoid driving and alcoholic
beverages

Continued
2164_Ch08_603-688 29/03/12 12:34 PM Page 662

662 Common Medications

Table 8.17
Common Medicationscontd
Drug and Dosage Use Action Assessment: Side Effects
Diazepam (Valium)PO 210 mg Anxiety disorders; alcohol Induces calming effect on CNS depressionsedation
34 times daily; IM, IV 210 mg withdrawal; adjunctive limbic system, thalamus, and or ataxia (dose related); dry
q34h therapy in seizure disor- hypothalamus mouth; blurred vision;
ders; status epilepticus; mydriasis; constipation;
eclamptic seizures; urinary retention
tetanus, preoperative or
preprocedural sedation
(also see Midazolam)
NURSING IMPLICATIONS: Do not mix with other drugs; IM injection painful; observe for phlebitis; monitor response; measures to
ensure client safety (e.g., falls); high potential for abuse; contraindicated in acute closed-angle glaucoma and porphyria
Flurazepam (Dalmane)PO Hypnotic Fastest acting; see Diazepam See Diazepam
>15 yr, 30 mg at bedtime; elderly
or debilitated, 15 mg at bedtime
NURSING IMPLICATIONS: See Diazepam
Hydroxyzine pamoate (Vistaril) See Chlordiazepoxide CNS relaxant with sedative Drowsiness; headache; itching;
PO 25100 mg 4 times daily (Librium); antiemetic in effect on limbic system and dry mouth; tremor
postoperative conditions; thalamus
adjunctive therapy

Obstetric use: prodromal Allows relaxation


labor
NURSING IMPLICATIONS: Give deep IM only; potentiates action of: warfarin (Coumadin), narcotics, and barbiturates
Lorazepam (Ativan)PO 12 mg Anxiety disorders; See Diazepam See Diazepam
23 times daily (up to 10 mg); insomnia; alternative to
24 mg at bedtime; IM 4 mg diazepam for status
max.; IV 2 mg max. epilepticus; preanesthesia
NURSING IMPLICATIONS: See Diazepam
Meprobamate (Equanil, Anxiety; stress; absence See Hydroxyzine pamoate Voracious appetite; dryness of
Miltown)PO 400 mg seizures (Vistaril) mouth; ataxia
34 times daily
NURSING IMPLICATIONS: Older clients prone to drowsiness and hypotension; observe for jaundice
Midazolam (Versed)IM Preanesthesia; prediagnos- Penetrates blood-brain barrier Respiratory depression, apnea;
0.050.08 mg/kg; tic procedures; induction of to produce sedation and disorientation, behavioral
IV 0.10.15 mg/kg general anesthesia amnesia excitement
NURSING IMPLICATIONS: Monitor ventilatory status and oxygenation; prevent injuries from CNS depression; nonirritating to vein
Phenobarbitalsedative, Preoperative sedation; Depresses CNS, promoting Cough, hiccups; restlessness;
PO 2030 mg 3 times daily; emergency control of drowsiness pain; hangover; CNS and
hypnotic, PO 50100 mg; IV, IM convulsions; absence circulatory depression
100300 mg; also butabarbital seizures
(Butisol), pentobarbital
(Nembutal), secobarbital
(Seconal)
NURSING IMPLICATIONS: Observe for hypotension during IV administration; put up side rails on bed of older clients; observe for
increased tolerance
Promethazine (Phenergan)IV, Preoperative sedation; Antihistaminic; sedative, Drowsiness; coma; hypoten-
IM, PO 2550 mg postoperative sedation antiemetic, antimotion sion; hypertension; leukopenia;
sickness photosensitivity; irregular respi-
SEDATIVES-HYPNO

rations; blurred vision; urinary


retention; dry mouth, nose,
throat
NURSING IMPLICATIONS: Administer PO with food, milk; IM deep into large muscles, rotate sites; verify compatibility with other
drugs; safety concerns due to sedative effect
2164_Ch08_603-688 29/03/12 12:34 PM Page 663

chapter 8 Physiological Integrity 663

Table 8.17
Common Medicationscontd
Drug and Dosage Use Action Assessment: Side Effects
Zolpidem (Ambien)PO 510 mg Short-term treatment of Nonbenzodiazepine hypnotic; Headache on awakening;
at bedtime, limited to 710 days insomnia preserves deep sleep (stages drowsiness, fatigue; confusion
3 and 4) and falls in elderly; myalgia
NURSING IMPLICATIONS: Monitor for: compromised respiratory status, increased depression, cognitive impairment. Onset more
rapid on empty stomach. Do not breastfeed while taking this medication
Serum
Immune globulinIM 0.02 mL/kg Hepatitis Provides passive immunity Do not use if client is
hypersensitive
NURSING IMPLICATIONS: Should be administered within 2 wk of exposure
Thyroid Hormone Inhibitors
PropylthiouracilPO Hyperthyroidism; return Inhibits functional thyroid hor- Blood dyscrasias,
300400 mg/day, divided client to euthyroid state; mone synthesis by blocking hepatotoxicity,
initial dose; 100150 mg/day also used preoperatively reactions; responsible for hypothyroidism
maintenance dose; methimazole iodide conversion to iodine;
(Tapazole) 1560 mg/day inhibition of thyroxine (T4) con-
initial dose; 515 mg/day version to triiodothyronine (T3)
maintenance dose
NURSING IMPLICATIONS: Teach importance of compliance with medication protocol; avoid iodine-rich foods (seafood,
iodized salt); caution when using other drugs
Saturated potassium iodide To reduce size, vascularity Inhibits thyroid hormone GI distress; stained teeth;
(SSKI)PO 300 mg 3 or of thyroid before thyroid secretion, synthesis increased respiratory
4 times daily surgery; emergency treat- secretions; rashes; acne
ment of thyroid storm; or
control of hyperthyroid
symptoms after radioiodine
(131I) therapy
NURSING IMPLICATIONS: Dilute in juice, give through straw; bloody diarrhea/vomiting indicates acute poisoning

Thyroid Hormone Replacement


Levothyroxine (Levothroid, Hypothyroidism Replacement therapy to Symptoms of hyperthyroidism
Synthroid)PO Myxedema coma alleviate symptoms
0.050.10 mg/day;
Emergency replacement
IV 200500 mcg
therapy
NURSING IMPLICATIONS: Teach signs and symptoms of hyperthyroidism, hypothyroidism; monitor bowel activity; teach diet
to combat constipation; keep medication in tight light-proof container; avoid foods that inhibit thyroid secretion (turnips, cab-
bage, carrots, peaches, peas, strawberries, spinach, radishes)
Liothyronine (Cytomel)PO Mild hypothyroidism in Replacement therapy See Levothyroxine
25 mcg/day to maintenance adults
dose 2575 mcg
NURSING IMPLICATIONS: See Levothyroxine
Uterine Contractants
Ergonovine maleate (Ergotrate) Postabortal or postpartum Stimulates uterine contractions Nausea, vomiting; occasional
PO, IM, IV 0.2 mg (gr 1/320) hemorrhage; promotes for 3 hr or more transient hypertension, espe-
involution after delivery of cially if given IV; cramping
placenta
UTERINE CONTRACT

NURSING IMPLICATIONS: Store in cool place; monitor maternal BP and pulse; do not use in labor
Methylergonovine maleate Postpartum hemorrhage, Stimulates stronger and longer Nausea, vomiting; transient
(Methergine)PO 0.2 mg; IM, after delivery of placenta contractions than ergonovine hypertension, dizziness, tachy-
IV 0.2 mg (gr 1/320) maleate (Ergotrate) cardia; cramping
NURSING IMPLICATIONS: Do not give if mother is hypertensive; do not use if solution is discolored; do not use in labor

Continued
2164_Ch08_603-688 29/03/12 12:34 PM Page 664

664 Questions

Table 8.17
Common Medicationscontd
Drug and Dosage Use Action Assessment: Side Effects
Oxytocin (Pitocin, Syntocinon) Stimulates rhythmic Induces labor; augments con- Tetanic contractions, uterine
IV 2 mL (15 units) in 250-mL contractions of uterus tractions; prevents or controls rupture; cardiac arrhythmias,
solution; 1 mL IM after delivery postpartum atony; antidiuretic FHR deceleration
effect
NURSING IMPLICATIONS: Contraindicated if cervix is unripe, in cephalopelvic disproportion (CPD), abruptio placentae, and cardio-
vascular disease; monitor: FHR, contractions, maternal BP, pulse, I&O; watch for signs of water intoxication with prolonged IV use;
drug of choice in presence of hypertension; never use undiluted; discontinue if tetanic contractions occur; antidote: propranolol
Volume Expander
Albumin 5%, 25%IV 500 mL Cirrhosis Provides colloidal oncotic Hypersensitivity, CHF, hepatic
as needed Nephrotic syndrome pressure, which mobilizes fluid failure
from extracellular back into
intravascular space
NURSING IMPLICATIONS: Monitor progress of disease; monitor for signs of shock; if fever, tachycardia, or hypotension occur, stop
treatment and call physician

Questions 5. Managing a sodium level of 120 in a client with diabetic


ketoacidosis is correctly accomplished with:
Select the one answer that is best for each question, unless 1. Oral salt tablets.
otherwise directed. 2. Intravenous 3% sodium chloride solution.
1. A nursing priority when administering a blood transfusion 3. Fluid restriction.
is to: 4. Insulin and 0.9% sodium chloride.
1. Check the fibrinogen level before infusing. 6. The correct time for a nurse to administer pancrelipase
2. Infuse blood slowly the first 20 minutes. (Pancrease) is:
3. Warm the blood prior to administration. 1. One hour after meals.
4. Infuse the blood over 1 to 2 hours. 2. With meals.
2. Blood sugar management for a client who has type 2 3. At bedtime.
diabetes with nausea and decreased appetite should 4. With insulin.
include: 7. Before administering furosemide (Lasix), a nurse should
1. Continuing insulin even if the client is vomiting. verify that:
2. Continuing insulin if blood sugar is less than 80 mg/dL. 1. The blood pressure is under 180/90 mm Hg.
3. Stopping insulin if urine is positive for ketones. 2. The potassium is over 4 mg/dL.
4. Stopping insulin until the client is able to tolerate 3. There are no crackles in the lungs.
food. 4. The client has an indwelling catheter in place.
3. Which finding indicates that epoetin alfa (Epogen) has 8. One hour after starting the insulin drip at 10 units/hr,
been effective? the clients blood sugar has fallen from 899 to 750.
1. Negative Homans sign, aPTT of 35 to 55. Which action should a nurse take?
2. Guaiac-negative gastric secretions and stool. 1. Increase the insulin drip rate to 20 units/hr.
3. Hematocrit of 33%. 2. Decrease the insulin drip rate to 7 units/hr.
4. Creatinine of 1.4. 3. Give a bolus of 10 units of regular insulin and increase
4. A nurse should locate and remove a clients clonidine the drip rate to 8 units/hr.
patch if: 4. Keep the rate the same and do not bolus.
1. The client complains of numbness and tingling down 9. Which IV fluid order is most appropriate for a client on
the arm that is nearest to the patch. dialysis?
QUESTIONS

2. The blood pressure is 80/50 mm Hg. 1. D5 1/2NS at 20 mL/hr.


3. The client tells the nurse about being allergic to sulfa 2. NS at 150 mL/hr.
products. 3. 1/4NS with 20 mEq KCl at 75 mL/hr.
4. The potassium level is 3.0 mg/dL. 4. D10W with 40 mEq KCl at 50 mL/hr.
2164_Ch08_603-688 29/03/12 12:34 PM Page 665

chapter 8 Physiological Integrity 665


10. Teaching for a client starting on spironolactone 17. A client who is postoperative needs to walk, but even
(Aldactone) should include: turning in bed is very painful. A patient-controlled anal-
1. The importance of removing the patch at bedtime. gesia (PCA) pump is set with a lock-out of 6 minutes.
2. Food sources to replace lost potassium. A nurse notes 45 attempts to get medication and 2 injec-
3. The purpose of this drug is to prevent fibrosis in tions received in the last 2 hours. The correct nursing
the heart. action is to:
4. Taking the medication just before bed. 1. Get client out of bed now while pain level is low.
2. Teach client how and why to use the pump more
11. A nurse should hold administration of a nitrate if: effectively.
1. Pulse rate is under 60.
3. Record clients refusal to get out of bed.
2. Client is allergic to sulfa.
4. Push pain button before getting client out of bed.
3. Blood pressure is less than 90/50 mm Hg.
4. Drug therapy has exceeded 2 weeks. 18. A client is due to receive Novolog insulin. When should
it be administered?
12. A client has a blood pressure of 145/83 mm Hg and 1. 60 minutes before the meal.
an ejection fraction of 50%. Before discharge from the
2. 15 minutes before the meal.
hospital, the client will need instructions on taking
3. As soon as the meal is finished.
which drug(s)?
4. Without regard to meals.
1. A thiazide diuretic.
2. Digoxin and furosemide. 19. A client with a duodenal ulcer has been taking sucralfate
3. A beta blocker and an ACE inhibitor. (Carafate). Which statement by the client would be a
4. An anticoagulant. priority for a nurse to address?
1. I dont like the taste, so I mix it in pudding.
13. A client with a sexually transmitted infection (STI) is to 2. I wish I could take Carafate at the same time as my
receive azithromycin. Before the medication is given,
other pills.
which nursing assessment is a priority?
3. I have found that stewed prunes help prevent
1. Obtain a CBC.
constipation from the drug.
2. Ask the client about allergies.
4. Taking Carafate with H2O has helped my fluid
3. Check the blood pressure.
intake.
4. Ask the client about sexual contact.
20. A client in diabetic ketoacidosis (DKA) has an order for
14. Which intervention would most likely prevent nausea in an insulin drip at 4 units/hr. The pharmacy prepared a
a client receiving cisplatin chemotherapy?
solution of 10 units of insulin in 100 mL normal saline.
1. Administering trimethobenzamide prn nausea.
The nurse correctly sets the infusion pump at _______
2. Administering dexamethasone and ondansetron prior
mL/hr. Fill in the blank.
to chemotherapy.
3. Serving all food warm or hot. 21. A nurse is concerned that a client may be at risk
4. Keeping client NPO 24 hours before for oversedation from opioid therapy using a
chemotherapy. patient-controlled analgesia pump. The most
reliable assessment for possible oversedation would
15. The most important nursing assessment to make before a be to check:
client is started on indomethacin is:
1. Changes in the level of pain reported by the client.
1. Asking if there is a history of sulfa allergies.
2. The oxygen saturation level recorded by pulse
2. Asking if there is a history of gastric bleeding.
oximetry.
3. Checking blood pressure.
3. How easily the client can be roused from sleeping.
4. Checking blood sugar.
4. The level of carbon dioxide in the blood using
16. A client, who is postoperative, has the anesthetic bupiva- capnography.
caine infusing into an epidural catheter for pain control.
22. For optimal effects from thrombolytic therapy given for a
On assessment, a nurse notes the client has decreased
brain attack (cerebrovascular accident [CVA]), treatment
sensation and numbness in the lower abdomen. The pain
should be given:
level is 3/10. The nurse should:
1. Within 3 hours of the onset of symptoms.
1. Record this expected finding.
2. When signs of increased intracranial pressure
2. Increase the epidural infusion rate.
QUESTIONS

occur.
3. Turn the epidural infusion off and raise the head
3. As pulse pressure begins to increase.
of bed.
4. No later than 24 hours after the onset of
4. Turn the epidural infusion off and lower the head
symptoms.
of bed.
2164_Ch08_603-688 29/03/12 12:34 PM Page 666

666 Questions

23. What observation should indicate to a nurse that the ther- 3. Urine output 30 mL/hr (previously 40 to 50 mL/hr).
apeutic effect of calcium gluconate has been achieved? 4. Diminished crackles in the lungs.
1. Thyroid gland shrinks. 30. To prevent a common adverse effect of prolonged use
2. Blood loss is curtailed during surgery. of phenytoin sodium (Dilantin), a client should be
3. Trousseaus sign is absent. instructed to:
4. Chvosteks sign is positive. 1. Obtain an annual influenza vaccination.
24. Which common systemic side effect of chemotherapy 2. Drink at least 8 to 10 glasses of water daily.
will a nurse note in an oncology client? 3. Eat a diet low in sodium and high in potassium.
1. Ascites. 4. Report gum swelling or bleeding to the physician.
2. Septicemia. 31. A client is being discharged and instructed to take
3. Polycythemia. furosemide (Lasix) every morning. A nurse tells the
4. Leukopenia. client to notify the health provider in the event of:
25. Which order by a physician should a nurse question? 1. Increased appetite.
1. A client with COPD who is prescribed metoprolol 2. Disruption in sleep patterns.
(Lopressor). 3. Increased urinary frequency.
2. A client with chronic renal failure receiving an 4. Leg cramps.
aluminum hydroxide gel (Amphogel). 32. A client is planning a cruise and wants to use the scopo-
3. A client with an abdominal aortic aneurysm (AAA) lamine patch. Which frequent side effect should a nurse
taking diltiazem (Cardizem). tell the client to expect?
4. A client on long-term hemodialysis receiving epoetin 1. Irregular pulse.
alfa (Epogen). 2. Dry mouth.
26. Which prescribed drugs would a nurse most likely give 3. Drop in blood pressure.
the client for respiratory stridor, with wheezing, and 4. Slurred speech.
hypotension after a bee sting? Select all that apply. 33. A nurse instructs a client about taking an angiotensin-
1. Epinephrine. converting enzyme (ACE). The client should be warned
2. Diphenhydramine (Benadryl). to continue therapy even when which common side
3. Corticosteroid (Solu-Medrol). effect occurs?
4. Furosemide (Lasix). 1. A dry, persistent cough.
5. Acetaminophen (Tylenol). 2. Exacerbation of heart failure.
6. Ranitidine (Zantac). 3. Sedation.
27. During cardiopulmonary resuscitation (CPR) for ventric- 4. Urinary incontinence.
ular fibrillation, which drug would an RN most likely 34. Which solution will a client most likely receive for fluid
prepare first? replacement in the first 24 hours after a burn injury that
1. Atropine sulfate. covers a large area?
2. Epinephrine. 1. D5W.
3. Furosemide (Lasix). 2. Lactated Ringers.
4. Lidocaine. 3. Dextran.
28. A nurse should teach a client with angina about the com- 4. Normal saline with added K+.
mon side effects of nitroglycerin, including: 35. A client is to receive oxycodone for acute pain. Adverse
Select all that apply. effects of opioid analgesics include:
1. Headache. Select all that apply.
2. Hypotension. 1. Agitation.
3. Shock. 2. Constipation.
4. Flushing. 3. Diarrhea.
5. Dizziness. 4. Vomiting
6. Shortness of breath. 5. Pruritus
7. Abdominal cramps. 6. Sedation.
29. The cardiac glycosides are essential in the therapy of a 36. The treatment for a client suffering from hypovolemic
QUESTIONS

client with a myocardial infarction (MI). What should shock following extensive burns will focus on controlling
indicate to a nurse that a therapeutic response to this fluid loss and replacing fluid volume. A nurse should
medication has been achieved? expect the client with burns to receive solutions that
1. A 15% increase in apical pulse rate. expand volume. To pull fluid from the interstitial spaces,
2. A rise in CVP from 12 to 15 cm.
2164_Ch08_603-688 29/03/12 12:34 PM Page 667

chapter 8 Physiological Integrity 667


which solutions should the nurse prepare? Select all 43. The correct nursing actions when administering a
that apply. vesicant chemotherapeutic agent are to:
1. A unit of whole blood. Select all that apply.
2. Hespan (hetastarch). 1. Assess the site for swelling, redness, or presence of
3. Albumin. vesicles.
4. Lactated Ringers. 2. Administer the drug by IV push.
5. Packed red blood cells. 3. Give the drug slowly to minimize cardiac toxicity.
37. During a blood transfusion, a client reports a throb- 4. Apply ice to the site if infiltration occurs.
bing headache, chills, and nausea. What ordered drugs 5. Apply heat to the site if extravasation occurs.
should a nurse prepare for administration? Select all 44. A client with diabetic ketoacidosis is to receive regular
that apply. insulin IV at 5 units/hr. The solution received from
1. Epinephrine. the pharmacy includes 100 units of regular insulin in
2. Sodium bicarbonate. 500 mL of NS. A nurse carefully calculates that the
3. Theophylline. insulin dip should infuse at a rate of _______ mL/hr.
4. Solu-Cortef. Fill in the blank.
5. Dopamine.
45. A nurse is to give metoclopramide (Reglan) 10 mg IV
38. Upon discharge from a hospital, teaching for a client over 15 minutes. The dose on hand is 10 mg metoclo-
with osteoporosis about proper administration of calcium pramide in 50 mL D5W. The nurse should infuse this
supplements should include: medication at a rate of _______ mL/hr. Fill in the blank.
1. Taking calcium with vitamin D to increase absorption.
2. Taking calcium citrate at meal time. 46. A nurse is to give 0.01 mcg/kg/min of nesiritide
3. Taking a calcium carbonate on an empty stomach. (Natrecor) to a client weighing 110 pounds. The pre-
4. Taking calcium once a day. mixed solution contains 1.5 mg of the drug in 250 mL
of D5W. The nurse correctly calculates that the client
39. A common symptom for Sjgrens syndrome, an autoim- should receive _______ mL/hr of this medication. Fill
mune disorder in women, is dry eyes. A client asks a in the blank.
nurse what she can do for relief. Which suggestion given
by the nurse would be correct for initial treatment? 47. A client with hyperglycemia is to receive an insulin drip
1. Use artificial tears 3 to 4 times a day. at 12 units/hr. The premixed bag contains 50 units of
2. Use cyclosporine A (Restasis) ophthalmic drops. regular insulin in 100 mL of normal saline. The nurse
3. Schedule punctal occlusion. should set the infusion pump to administer the drip at a
4. Use pilocarpine drops 3 times a day. rate of _______ mL/hr. Fill in the blank.

40. A nurse is to administer nitroprusside (Nipride) 48. A clients blood pressure drops to 70/40 mm Hg.
4 mcg/kg/min intravenously to a client with severe Albumin 500 mL is to be given IV over 30 minutes.
hypertension who weighs 75 kg .The pharmacy prepares Using a macrodrip system (10 gtt/min), the IV rate
a solution of 50 mg nitroprusside in 500 mL D5W. should be _______ gtt/min. Fill in the blank.
The nurse correctly calculates that _______ mL/hr of 49. A client reports unexplained weight loss and palpitations.
this medication should be administered to the client. Assessment findings include: exophthalmos, tachycardia,
Fill in the blank. decreased thyroid-stimulating hormone (TSH), and
41. Epidural analgesia has been started for a client who is increased triiodothyronine (T3) and thyroxine (T4).
experiencing chronic pain that is unrelieved by other Which treatments would be most appropriate for this
methods. Proper positioning of the client should include: client? Select all that apply.
Select all that apply. 1. Levothyroxine (Synthroid).
1. Side-lying, semi-Fowlers. 2. Radioactive iodine.
2. Supine, head of bed flat, thin pillow. 3. Propylthiouracil (PTU).
3. Side-lying, head of bed flat. 4. Methimazole (Tapazole).
4. High Fowlers, supine. 5. Liothyronine (Cytomel).
5. Prone. 50. A client with diabetes is admitted with symptoms of
42. Cefotetan disodium (Cefotan) 2 grams IV over diabetic ketoacidosis. Which type of insulin IV should a
QUESTIONS

30 minutes is to be administered to a client 1 hour nurse expect to administer?


before surgery. The available IV solution contains 1. NPH.
2 grams Cefotan in 100 mL of normal saline. A nurse 2. Regular.
calculates that the client should receive _______ mL/hr 3. Lantus.
of this medication. Fill in the blank. 4. Intermediate- and short-acting mix.
2164_Ch08_603-688 29/03/12 12:34 PM Page 668

668 Questions

51. Which combination of insulins, if ordered, should a 3. Meperidine.


nurse question? 4. Fentanyl.
1. Regular and NPH. 59. A client weighing 70 kg has received 10 mg of morphine
2. Regular and Humulin-N. sulfate (MS) IM for pain rated at 5/10 postappendectomy.
3. Lantus and Lispro. Which outcome should a nurse expect?
4. Humulin-R and Lente. 1. Blood pressure and pulse will increase.
52. When drawing up two different insulins in one syringe, 2. Moderate pain will be relieved for 4 hours.
which insulin should a nurse draw up first? 3. Dose is insufficient for the clients size.
1. Lispro. 4. Another dose will be needed in 2 hours.
2. NPH. 60. A nursing student asks an RN why clients who have had
3. Lantus. myocardial infarctions (MIs) should receive medications
4. Novolin 70/30. intravenously rather than intramuscularly. The best
53. When administering nitroglycerin to a client for chest response by the RN would be:
pain, a nurse knows that a second dose can be safely 1. A larger dose of medication would be required to
given in: bring relief from chest pain.
1. 5 minutes. 2. The trauma from more frequent needle sticks would
2. 10 minutes. cause stress and increase the serum enzymes.
3. 15 minutes. 3. Tissue perfusion is poor and venous return is
4. 20 minutes. diminished following an MI, which affects results.
4. Intramuscular injections should be avoided if
54. A client is to start on an enteral tube feeding. What the client is also receiving an anticoagulant drug
should a nurse do to make this as comfortable as possible post-MI.
for the client?
1. Keep the head of the bed flat. 61. A client with a history of alcohol abuse returns to a
2. Give the feeding as quickly as possible. postanesthesia care unit (PACU) following abdominal
3. Chill the feeding to improve the taste. surgery. A nurse knows pain management may be more
4. Start the tube feeding slowly. difficult because of:
1. Potential diminished drug effects related to impaired
55. A physician orders furosemide (Lasix) for a client. What liver blood flow.
should a nurse tell the client to expect if the medication 2. Possible drug tolerance requiring higher dosages to
is having the desired effect? produce desired effects.
1. Elevated blood pressure. 3. The need for higher dosages of medications because
2. Elevated heart rate. of accelerated liver detoxification.
3. More frequent urination. 4. Increased sensitivity to the anesthetic agents used,
4. Nighttime urination will decrease. requiring higher dosages than normal.
56. When administering a beta blocker, a nurse should first: 62. A client, who is gravida 6, para 4, has blood drawn for
1. Take the clients temperature and heart rate. routine prenatal first-trimester laboratory work. Upon
2. Place the client in an upright position before the client reviewing the clients laboratory results, which result
swallows the drug. should a nurse consider within normal limits for this
3. Take the clients blood pressure and heart rate. stage of pregnancy?
4. Ask the client if there have been any adverse effects. 1. Hemoglobin of 11 gm/dL.
57. Maintaining the infusion rate of total parenteral nutrition 2. Hemoglobin of 18 gm/dL.
(TPN) is a nursing responsibility. If the infusion rate is 3. Fasting serum glucose of 110 mg/dL.
too rapid, which side effects would the nurse observe? 4. White blood cell count of 18,000/mL.
1. Serum sodium and potassium excess. 63. A client, who is gravida 1, para 0, is admitted with a
2. Increased blood pressure and hyperglycemia. diagnosis of severe preeclampsia. A nurse starts the client
3. Hypoglycemia and low blood pressure. on intravenous magnesium sulfate. For which expected
4. Potassium excess and congestive heart failure. side effect should the nurse closely observe the client?
58. An older client complains that, in the past, some anal- 1. Decreased urinary output.
2. Hypersomnolence.
QUESTIONS

gesics have caused urinary retention and constipation.


Which analgesic should a nurse tell the client would 3. Absence of knee jerk reflex.
not cause those problems? 4. Decreased respiratory rate.
1. Lomotil. 64. A client, who is gravida 5, para 3, requests epidural anes-
2. Morphine. thesia to relieve the discomfort of labor. After placement
2164_Ch08_603-688 29/03/12 12:34 PM Page 669

chapter 8 Physiological Integrity 669


of her epidural and a loading dose of medication, which prescribed by a physician for pain. A nurse should know
assessment should be a nurses priority? that the most important barrier to the use of opioids for
1. Checking for cervical dilation. pain is:
2. Placing the client in a supine position. 1. Clients fear becoming addicted.
3. Obtaining a fetal heart rate. 2. Physicians concern about drug abuse.
4. Checking the clients blood pressure. 3. Possible side effects of the drug.
4. Lack of accurate pain screening tools.
65. A client, who is gravida 2, para 1, is at 42 weeks gesta-
tion with gestational diabetes that is controlled with diet. 70. A client with intractable chest pain is to receive
A physician orders an intravenous infusion of oxytocin nitroglycerin 50 mg/250 mL and the rate is to be
for the induction of labor due to postdates pregnancy. titrated until chest pain is relieved. The IV is infusing
When caring for the client who is receiving intravenous at 27 mL/hr and the client is pain-free. A nurse deter-
oxytocin, the nurse should monitor for: mines that the client is receiving nitroglycerin at a rate
1. Maternal hypoglycemia. of _______ mcg/min. Fill in the blank.
2. Fetal movement.
71. A nurse enters a clients room to administer an IV medica-
3. Maternal hyperreflexia.
tion and finds that the IV site is swollen, cool, and pale.
4. Fetal bradycardia.
The client reports discomfort at the insertion site. The
66. A client, who is gravida 2, para 1 at 14 weeks gestation, nurse recognizes that this may be:
has a routine ultrasound that shows no fetal cardiac 1. Infiltration.
motion. Which medication should a nurse anticipate 2. Phlebitis.
being ordered for this client? 3. Sepsis.
1. Magnesium sulfate. 4. An allergic reaction.
2. Clomiphene citrate (Clomid).
72. Medications are absorbed differently with aging. A nurse
3. Calcium gluconate.
should expect the dosage for an older adult to be:
4. Misoprostol (Cytotec).
1. Doubled to achieve the same effectiveness.
67. A client, who is gravida 1, para 0 with severe preeclamp- 2. Reduced to prevent excessive systemic levels.
sia, has been on a magnesium sulfate infusion at 2 gm/hr 3. Increased slightly to make up for diminished liver
for the past 6 hours. Her recent assessment reveals a function.
blood pressure 150/88 mm Hg, pulse 88, respirations 16, 4. Monitored for reduced clinical effectiveness.
1+ deep tendon reflexes, with 80 mL of urinary output
73. A client is scheduled for an inguinal herniorrhaphy in the
during the past hour. Which action should a nurse take?
morning and wants to know when to take the blood
1. Decrease the infusion rate to 1 gm/hr and turn the
pressure pill before surgery. The best response by a nurse
client on her left side.
would be:
2. Continue the infusion of magnesium sulfate and mon-
1. No medications are taken by mouth the morning of
itor the clients blood pressure.
surgery.
3. Stop the infusion of magnesium sulfate and notify the
2. Since you will not be taking anything by mouth after
physician.
midnight, your drug wont be absorbed as well in the
4. Administer calcium gluconate IV push and notify the
stomach.
physician.
3. You will probably take your medicine with a small
68. A client is having severe pain in the lower back and left amount of water in the morning as usual.
leg. The client has sensation in all extremities, but is not 4. Your medication will be given to you intravenously
moving the left leg. While waiting for a diagnostic x-ray, just before surgery.
the client asks for pain medication. The correct response
74. A client, who is newly diagnosed with tuberculosis, is
by the nurse would be to:
being seen in a clinic. Current treatment includes isoni-
1. Tell the client nothing can be given until a diagnosis is
azid (INH) and rifampin (Rifadia). During the assess-
made.
ment, the client states that there has been blood in the
2. Give half of the usual dose of analgesic to minimize
urine since starting treatment. The best response by a
pain.
nurse would be:
3. Request order for an opioid to achieve maximum
1. One of your medications changes the color of body
relief.
fluids.
QUESTIONS

4. Offer two acetaminophen tablets.


2. This is expected. There is nothing to worry about.
69. A 35-year-old man has been experiencing low back pain 3. I will send a urine specimen to the lab for analysis.
for 2 days with no relief from ibuprofen 400 mg 4 times 4. Have you had any pain on urination or more fre-
daily. His pain is 7 on a scale of 1 to 10. An opioid is quent urination?
2164_Ch08_603-688 29/03/12 12:34 PM Page 670

670 Questions

75. A child is hospitalized for moderate respiratory distress terbutaline. A physician discharges the client with a
caused by asthma. A physician orders a corticosteroid. A prescription for terbutaline 5 mg PO q4h. When
nurse knows that the main rationale for this medication giving discharge instructions, which instruction
is to: should a nurse include?
1. Improve circulation. 1. Call your physician if your heart rate is higher than
2. Cause bronchodilation. 120 before you take your terbutaline.
3. Reduce inflammation. 2. You can double your dose of terbutaline if you are
4. Thin respiratory secretions. having more than 2 contractions per hour.
3. Call your physician if your fetus moves more than
76. A child with gastroenteritis is admitted to a pediatric 10 times per hour.
unit. The child is severely dehydrated and anuric. After 4. You can increase your activity each day if you are not
the child receives a fluid bolus, a physician orders contin- fatigued.
uous IV fluids. Which IV fluid order should a nurse
question? 83. A client, who is gravida 1, para 0, is at 39 weeks
1. 0.45% NaCl with KCl 20 mmol/L. gestation. She has an uncomplicated prenatal history.
2. 0.45% NaCl with 5% glucose. Her recent cervical examination reveals that she is 4 to
3. 0.9% NaCl. 5 cm dilated, 100% effaced, and at a +1 station. She is
4. 0.9% NaCl with 5% glucose. asking for an epidural for labor pain control. Which
order should a nurse anticipate to prevent hypotension
77. A pediatric nurse prepares to infuse a fluid bolus of in this client?
300 mL NS over 1 hour. A nurse plans to use micro- 1. Administer a 500- to 2,000-mL bolus of IV fluid.
drip IV tubing. The nurse calculates that the drip 2. Administer oxygen at 12 L/min via face mask.
rate for this infusion should be _______ gtt/min. 3. Place the client in high Fowlers position.
Fill in the blank. 4. Administer epinephrine to raise her blood pressure.
78. A nurse admits a preschooler with sickle cell anemia. 84. A client, who is a 17-year-old gravida 1, para 0,
Which medications should the nurse expect that the is at 35 weeks gestation with a blood pressure of
child has been receiving at home? Select all that apply. 164/116 mm Hg on her left side, 4+ deep tendon reflexes,
1. Fer-In-Sol. and 3+ proteinuria. Her physician orders a 4-gram load-
2. Penicillin. ing dose of magnesium sulfate over 30 minutes, followed
3. Ibuprofen. by 2 gm/hr. After administration of the bolus, the client
4. Folic acid. complains of chest heaviness. A nurse assesses the clients
5. Desferal. deep tendon reflexes, which are now absent. The nurse
79. A child recovering from abdominal surgery is receiving should anticipate that the physician will most likely
morphine for pain via PCA at a concentration of 1 mg/mL. order:
The child receives a basal rate of 1 mg/hr and is able to self- 1. Butorphanol tartrate.
administer up to four 1-mg boluses each hour. A nurse 2. Furosemide.
hangs a 50-mL syringe at the start of the shift and reviews 3. Diazepam.
the childs PCA record 4 hours later to find that the child 4. Calcium gluconate.
self-administered a total of 7 boluses during that time. The 85. A client, who is a 44-year-old gravida 2, para 1, is at
nurse calculates that _______ mL of morphine should be 18 weeks gestation with consistent blood pressures
left in the syringe. Fill in the blank. ranging from 130 to 140/80 to 90 mm Hg during her
80. A 44-pound child is to receive a 20-mL/kg bolus of last two prenatal visits. She has no proteinuria or hyper-
IV fluid over 2 hours. A nurse correctly sets the infusion reflexia. Which medication should a nurse anticipate
pump at a rate of _______ mL/hr. Fill in the blank. being prescribed for this client?
1. Vasotec.
81. A client, who is gravida 4, para 3 at 28 weeks gestation, 2. Aldomet.
is dilated 2 cm. A physician has prescribed nifedipine to 3. Diazepam.
treat her preterm labor. For which common maternal side 4. Magnesium sulfate.
effect of the medication should a nurse assess?
1. Decreased deep tendon reflexes. 86. A client, who is gravida 6, para 5 and in the fourth stage
2. Bradycardia. of labor, receives an injection of carboprost (Hemabate)
QUESTIONS

3. Hypotension. 250 mcg IM. The nurse knows that the expected out-
4. Tachycardia. comes of this medication are:
Select all that apply.
82. A client, who is gravida 2, para 0 at 31 weeks 1. Increased uterine atony.
gestation, has been hospitalized for 5 days on oral 2. Delayed uterine involution.
2164_Ch08_603-688 29/03/12 12:34 PM Page 671

chapter 8 Physiological Integrity 671


3. Increased myometrial contractions. 3. Stadol 1 mg IV push every 4 hours as needed for
4. Decreased blood pressure. pain.
5. Increased contractility of gastrointestinal smooth 4. Ancef 2 gm IVPB every 6 hours.
muscle.
93. A client, who is gravida 1, para 0, delivers an infant at
87. A physician orders oxytocin IV for the augmentation 39 weeks gestation. The clients blood type is O negative
of labor. The premixed IV has 20 units of oxytocin in and the baby is O positive. A physician orders an injec-
1,000 mL of lactated Ringers solution. The oxytocin tion of Rho(D) immune globulin. To provide adequate
is ordered to begin at 6 milliunits/min. To administer postpartum prophylaxis, when should a nurse administer
the correct dose, a nurse should begin the infusion at Rho(D) immune globulin?
_______ mL/hr via infusion pump. Fill in the blank. 1. Within 72 hours of delivery.
2. Within 1 week of delivery.
88. A client, who is gravida 3, para 1, is being induced for 3. Before 12 hours after delivery.
pregnancy-induced hypertension using IV oxytocin.
4. Within 2 weeks of delivery.
Under which circumstances should a nurse discontinue
infusion of oxytocin? Select all that apply.
1. Three contractions in a 10-minute window of time. Answers/Rationales/Tips
2. Contractions every 3 minutes, lasting 90 seconds
each. 1. CORRECT ANSWER: 2. Answer 1 is incorrect because there
3. Decrease in fetal heart rate after the peak of is no risk to the client from fibrinogen, which is a protein that
the contraction during each of the last three is converted into fibrin to form a thrombus or clot. Answer 2
contractions. is correct because a hemolytic reaction will most likely
4. Five contractions in the past 8 minutes. occur during the first 20 minutes. Answer 3 is incorrect
5. Client is completely dilated. because warming is not required, but may be more comfortable
for the client. Answer 4 is incorrect because the rate of infusion
89. The nursing instructor questions a nursing student could be up to 4 hours.
about an analgesic that may be ordered for a child with TEST-TAKING TIP: This is a priority question; therefore,
otitis media who is complaining of mild ear pain. Which other options could be correct. Look for the action that protects
medication should the nursing student correctly expect the client from injurythink safety.
to be ordered? Content Area: Adult Health, Hematological; Integrated Process:
1. Ibuprofen. Nursing Process, Analysis; Cognitive Level: Application;
2. Acetaminophen. Client Need/Subneed: Physiological Integrity/Pharmacological
3. Codeine. and Parenteral Therapies/Blood and Blood Products
4. Dilaudid.
2. CORRECT ANSWER: 1. Answer 1 is correct because during
90. An adolescent who has attention deficit-hyperactivity illness the cells still need insulin. Failure to give it will like-
disorder (ADHD) complies with taking the prescribed ly put the client into a hyperosmolar, if nonketotic state
once-a-day medication. The adolescents academic classes (type 1 diabetes is more likely to cause ketoacidosis). Since
(e.g., math, science, etc.) should be scheduled by the the client will not be eating very much, the insulin dose
school nurse for the: may be reduced, and it is hard to predict by how much.
1. Morning. Clients with diabetes need to prepare a sick day strategy,
2. Midday. which will include checking blood sugar, and possibly
3. Afternoon. ketone level, more frequently. Answer 2 is incorrect because
4. Intermingled with high-interest classroom insulin should not be administered if the blood glucose is low.
activities. Answer 3 is incorrect because ketone-positive urine indicates
91. A client is to receive 5 mcg/min of isoproterenol that there is inadequate insulin carrying glucose into the cells.
hydrochloride (Isuprel). The pharmacy prepares Insulin is needed. Answer 4 is incorrect because, when unable
1 mg of Isuprel in 500 mL of D5W. A nurse should to ingest food, the liver breaks down glycogen, which provides
regulate the infusion rate to _______ mL/hr. Fill in glucose for the body. Insulin is still required to take this glu-
the blank. cose into the cell.
TEST-TAKING TIP: Totally stopping insulin is not advised as a
92. A client, who is an 18-year-old primigravida with gesta- general management strategy; therefore, eliminate Answers 3
tional diabetes, is admitted to a labor and delivery unit at and 4.
ANSWERS

34 weeks gestation in preterm labor. Which physicians Content Area: Adult Health, Endocrine; Integrated Process:
order for this client should a nurse question? Nursing Process, Planning; Cognitive Level: Application;
1. Magnesium sulfate 4 gm IV bolus over Client Need/Subneed: Physiological Integrity/Pharmacological
30 minutes. and Parenteral Therapies/Expected Effects/Outcomes
2. Brethine 10 mcg IV.
2164_Ch08_603-688 29/03/12 12:34 PM Page 672

672 Answers/Rationales/Tips

3. CORRECT ANSWER: 3. Answer 1 is incorrect because diabetic ketoacidosis need fluid replacement with
Epogen is given to treat anemia. Negative Homans sign normal saline and insulin to return the blood sugar
and elevated activated partial thromboplastin time (aPTT) to a normal level.
are used to monitor thromboembolism. Answer 2 is incorrect TEST-TAKING TIP: Select the option with insulin. Think
because Epogen is given to treat anemia. Guaiac-negative about the fluid and electrolyte imbalances that occur from a
gastric secretions might indicate effectiveness of stress ulcer high blood glucose: Blood glucose is elevated (>200 mg/dL) in
prevention. Answer 3 is correct because epoetin alfa diabetic ketoacidosis. Knowing why the low sodium (<135) has
(Epogen) stimulates the development of red blood occurred is important for selecting the correct action.
cells (RBCs). It is given to anemic clients to increase Content Area: Adult Health, Endocrine; Integrated Process:
hematocrit (Hct). Answer 4 is incorrect because Epogen is Nursing Process, Implementation; Cognitive Level: Analysis;
given to treat anemia. It is not measured by creatinine. Client Need/Subneed: Physiological Integrity/Pharmacological
TEST-TAKING TIP: Think: Epogen = erythropoietin = erythro and Parenteral Therapies/Expected Effects/Outcomes
(red) increased RBCs increased Hct. 6. CORRECT ANSWER: 2. Answer 1 is incorrect because
Content Area: Adult Health, Hematological; Integrated Process:
taking this medication 1 hour after a meal does not replace
Nursing Process, Analysis; Cognitive Level: Application; Client enzymes that would be released by the pancreas in response
Need/Subneed: Physiological Integrity/Pharmacological and
to food intake. Taking it with meals stimulates the normal
Parenteral Therapies/Expected Effects/Outcomes pancreas activity. Answer 2 is correct because this medica-
4. CORRECT ANSWER: 2. Answer 1 is incorrect because tion is replacing enzymes that would be released by the
clonidine patches are used to manage pain or hypertension. pancreas in response to food intake. Taking it with meals
Numbness or tingling down the arm is not related to the stimulates the normal pancreatic activity and helps with
patch. Answer 2 is correct because clonidine is an alpha-2 the digestion of proteins, starches, and fats. Answer 3 is
receptor stimulator, working at the presynaptic receptor incorrect because taking this medication at bedtime does
of the sympathetic nervous system. Thus, it reduces the not replace enzymes that would be released by the pancreas
output of norepinephrine and is used to treat hyperten- in response to food intake. Taking it with meals stimulates
sion. If the BP is too low, antihypertensive medications the normal pancreatic activity. Answer 4 is incorrect because
should not be given. It is important for the nurse to clients with deficiencies in pancreatic enzymes do not
remember that drugs in patches are in a constant state necessarily have any deficit in insulin secretion. Pancrease
of being given, and the patch would need to be should be administered with the meal; insulin (if adminis-
removed. Answer 3 is incorrect because clonidine patches tered) would need to be given before the meal to allow for
are not sulfa-based. Answer 4 is incorrect because clonidine absorption.
patches do not affect serum potassium. TEST-TAKING TIP: Knowing the importance of pancreatic
TEST-TAKING TIP: If the drug is an antihypertensive, select enzymes in facilitating digestion makes the choice easierwith
the answer that relates to blood pressure, not neurology meals, not before or after.
(Answer 1) or allergy (Answer 3). Content Area: Adult Health, Gastrointestinal; Integrated
Content Area: Adult Health, Cardiovascular; Integrated Process: Nursing Process, Implementation; Cognitive Level:
Process: Nursing Process, Analysis; Cognitive Level: Application; Client Need/Subneed: Physiological Integrity/
Application; Client Need/Subneed: Physiological Integrity/ Pharmacological and Parenteral Therapies/Medication
Pharmacological and Parenteral Therapies/Adverse Effects/ Administration
Contraindications/Interactions 7. CORRECT ANSWER: 2. Answer 1 is incorrect because
5. CORRECT ANSWER: 4. Answer 1 is incorrect because high blood pressure (e.g., 180/90) is treated with diuretics
clients in diabetic ketoacidosis are dehydrated. They need and does not need to be verified (although furosemide is
salt and fluids, which are generally given intravenously. not usually the first choice). Answer 2 is correct because
Answer 2 is incorrect because hypertonic saline solutions are this diuretic will cause potassium to be lost in the urine.
not appropriate. Fluids must run fast in these clients who Clients receiving furosemide often need K+ replacement.
are dehydrated, and this would change sodium levels too Answer 3 is incorrect because crackles that are present
quickly. Rapid changes in serum sodium can precipitate a with pulmonary edema may be treated with furosemide.
dangerous fluid shift. Answer 3 is incorrect because clients Answer 4 is incorrect because, although the urine volume
with diabetic ketoacidosis are dehydrated. Fluid restriction will be increased, there is no need for a bladder catheter if
is inappropriate. Answer 4 is correct because high blood the client can void.
sugar increases the osmolarity of the blood and stimu- TEST-TAKING TIP: Only one of the options relates to the
ANSWERS

lates the body to secrete antidiuretic hormone (ADH) to importance of the potassium level. The other options relate to
help the kidneys to retain free water, which dilutes the assessing and managing the impact of fluid in the client.
sodium. Obligate diuresis from the osmotic action of Content Area: Adult Health, Fluid and Electrolyte Imbalances;
sugar in the tubules of the kidneys forces water and sodi- Integrated Process: Nursing Process, Assessment; Cognitive
um loss even though the client is dehydrated. Clients in Level: Application; Client Need/Subneed: Physiological
2164_Ch08_603-688 29/03/12 12:34 PM Page 673

chapter 8 Physiological Integrity 673


Integrity/Pharmacological and Parenteral Therapies/ because generally diuretics are taken in the morning to prevent
Medication Administration having to wake up from sleep to urinate.
TEST-TAKING TIP: Three of the options focus on administra-
8. CORRECT ANSWER: 2. Answer 1 is incorrect because tion and adverse effects of the medication. Only one option
rapid reduction in blood glucose reduces serum osmolarity
states a desired outcome. Choose the option that is different.
and may shift fluids into cells, causing cerebral edema. The
Review the common classifications of diureticsthiazides, loop,
blood glucose level should not fall by more than 15% an
and potassium-sparingand know examples for each type.
hour. The insulin infusion rate should be reduced, not
Content Area: Adult Health, Cardiology; Integrated Process:
increased. Answer 2 is correct because rapid reduction in
Teaching and Learning; Cognitive Level: Application; Client
blood glucose reduces serum osmolarity and may shift
Need/Subneed: Physiological Integrity/Pharmacological and
fluids into cells, causing cerebral edema. The blood
Parenteral Therapies/Medication Administration
glucose should not fall by more than 15% an hour.
Slow the insulin infusion rate. Answer 3 is incorrect 11. CORRECT ANSWER: 3. Answer 1 is incorrect because
because rapid reduction in blood glucose reduces serum nitrates vasodilate veins but do not decrease heart rate.
osmolarity and may shift fluids into cells, causing cerebral This would not be a reason to hold the drug. Answer 2 is
edema. The blood glucose level has fallen by more than incorrect because nitrates are not sulfa drugs. Answer 3 is
15% an hour. The insulin infusion rate is too fast and no correct because nitrates are potent vasodilators. This
bolus is needed. Answer 4 is incorrect because blood glucose reduces the venous return, cardiac output, and BP. The
level has dropped by more than 15% in 1 hour. This drug should be held if the BP is too low. Answer 4 is
reduces serum osmolarity and may shift fluids into cells, incorrect because, with prolonged therapy, the client may
causing cerebral edema. The insulin infusion rate should develop tolerance to nitrates. Efficacy can be retained by
be reduced. providing a nitrate-free interval each day to allow the
TEST-TAKING TIP: Recognize a therapeutic drop in blood receptor to regain sensitivity. Therefore, therapy can
glucose versus a change that will potentially cause a problem. proceed over weeks and years.
Content Area: Adult Health, Endocrine; Integrated Process: TEST-TAKING TIP: An example of a nitrate is nitroglycerin.
Nursing Process, Implementation; Cognitive Level: Analysis; Note the similarity to the drug classificationnitrate and
Client Need/Subneed: Physiological Integrity/Pharmacological nitro-. Nitroglycerin is a vasodilator, leading to BP.
and Parenteral Therapies/Adverse Effects/Contraindications/ Content Area: Adult Health, Cardiovascular; Integrated Process:
Interactions Nursing Process, Implementation; Cognitive Level: Application;
Client Need/Subneed: Physiological Integrity/Pharmacological
9. CORRECT ANSWER: 1. Answer 1 is correct because and Parenteral Therapies/Medication Administration
clients on dialysis should receive minimal fluids and no
potassium, since these agents cannot be removed until 12. CORRECT ANSWER: 3. Answer 1 is incorrect because,
the next dialysis treatment. Answer 2 is incorrect because although thiazide diuretics are often first-line therapy for
this IV rate is too fast for a client who is unable to excrete hypertension, the heart failure is correctly managed with
fluids via the kidney. Answer 3 is incorrect because potassi- beta blockers and angiotensin-converting enzyme (ACE)
um is rarely given to clients requiring dialysis since they are inhibitors. Diuretics would be added only if there was
unable to secrete any excess. Answer 4 is incorrect because insufficient clinical response to the other drugs. Answer 2 is
potassium is rarely given to clients requiring dialysis since incorrect because digoxin and furosemide, long considered
they are unable to secrete any excess. the treatment of choice in heart failure, were found not to
TEST-TAKING TIP: First, eliminate the two options with reduce mortality. Heart failure should be treated with beta
potassium (Answers 3 and 4). Then, select the option with the blockers and/or ACE inhibitors. Answer 3 is correct because
slowest rate. Look for the option that best addresses the fluid the ejection fraction is low, indicating a problem with
and sodium restrictions usually required for the client with cardiac output. The ejection fraction of a healthy heart is
chronic renal failure. 60% to 70%. Beta blockers and ACE inhibitors reduce
Content Area: Adult Health, Renal; Integrated Process: Nursing morbidity and mortality in heart failure. Answer 4 is
Process, Analysis; Cognitive Level: Analysis; Client Need/Subneed: incorrect because, although heart failure can increase the
Physiological Integrity/Pharmacological and Parenteral Therapies/ risk of thromboembolism, the usefulness of anticoagulation
Parenteral/Intravenous Therapies in clients without atrial fibrillation or a history of blood
clots is not well established.
10. CORRECT ANSWER: 3. Answer 1 is incorrect because this TEST-TAKING TIP: The problem is with the pumping
diuretic is taken in pill form. Answer 2 is incorrect because
effectiveness of the heart. Select the option with the greatest
spironolactone is a potassium-sparing diuretic. Supplemental
ANSWERS

effect on improving cardiac output and client survival.


potassium is not advised. Answer 3 is correct because spirono-
Content Area: Adult Health, Cardiovascular; Integrated Process:
lactone has antiproliferative effects and prevents fibrotic
Nursing Process, Analysis; Cognitive Level: Analysis; Client
remodeling of heart tissue. It is the only diuretic that
Need/Subneed: Physiological Integrity/Pharmacological and
reduces mortality in heart failure. Answer 4 is incorrect
Parenteral Therapies/Medication Administration
2164_Ch08_603-688 29/03/12 12:34 PM Page 674

674 Answers/Rationales/Tips

13. CORRECT ANSWER: 2. Answer 1 is incorrect because and reducing pain. This is the desired effect and the
data from a complete blood count (CBC) are rarely required data should be recorded. Answer 2 is incorrect because
in the assessments of clients with STIs, and would not be a pain levels of 3/10 rarely require additional medication.
priority. Answer 2 is correct because clients should always Answer 3 is incorrect because the medication is working
be questioned about allergies before medications are well and the client is not showing adverse effects. Raising
administered. Answer 3 is incorrect because none of the the head of the bed may be helpful when too much medica-
adverse effects, contradictions, or cautions associated with tion has been administered, as gravity would help keep the
azithromycin involves blood pressure. Answer 4 is incorrect medicine from anesthetizing nerves involved in breathing.
because, although it is important to get information about Answer 4 is incorrect because the medication is working
sexual contacts, this information does not alter the decision well and the client is not showing adverse effects. There
to treat the current client, and thus is not a priority before is no reason to turn the infusion off. Lowering the head
giving azithromycin. would facilitate movement of the medication to a higher
TEST-TAKING TIP: With priority questions, look for the spinal level. This is not needed for this client with good
option that provides the client with the greatest safety. pain control.
Content Area: Adult Health, Communicable Disease; Integrated TEST-TAKING TIP: Know the normal level of numbness with
Process: Nursing Process, Assessment; Cognitive Level: an epidural. Select the only option that does not reequire an
Application; Client Need/Subneed: Physiological Integrity/ active intervention (e.g., increase, raise, lower).
Pharmacological and Parenteral Therapies/Medication Content Area: Adult Health, Pain Control; Integrated
Administration Process: Nursing Process, Implementation; Cognitive Level:
Application; Client Need/Subneed: Physiological Integrity/
14. CORRECT ANSWER: 2. Answer 1 is incorrect because Pharmacological and Parenteral Therapies/Expected Effects/
nausea is prevented by giving the antidote before chemother- Outcomes
apy. Anticholinergics, such as trimethobenzamide, have
limited usefulness in chemotherapy-induced nausea. 17. CORRECT ANSWER: 2. Answer 1 is incorrect because, if
Answer 2 is correct because there is often specific match- the pain level is too high for the client to turn in bed, the client
ing of nausea treatments and chemotherapeutic agents. will not do well with walking. Answer 2 is correct because,
Nausea is better prevented than treated by giving the with lock-out of 6 minutes, the client could have received
antiemetic before the chemotherapy. Answer 3 is incorrect 20 injections over the last 2 hours, yet only received 2. The
because heat increases the odor of foods, which may high pain level will prevent the client from engaging in
worsen nausea. Answer 4 is incorrect because there is no health promoting activities: 45 attempts indicates that the
reason to have a client NPO for 24 hours before chemother- client wants more medicine. Teaching is required to help the
apy. This would promote dehydration and malnutrition client spread out the attempts. Answer 3 is incorrect because
unnecessarily. the nurse might record it if the client refuses ambulation, but it
TEST-TAKING TIP: Look for an action prior to chemotherapy. would be better to help the client control the pain to make
Content Area: Adult Health, Oncology; Integrated Process: ambulation possible. Answer 4 is incorrect because only the client
Nursing Process, Implementation; Cognitive Level: should push the PCA button. This helps prevent overdoses.
Application; Client Need/Subneed: Physiological Integrity/ TEST-TAKING TIP: Look for the option that is different from
Pharmacological and Parenteral Therapies/Expected Effects/ the others. Three of the options refer to getting out of bed.
Outcomes Content Area: Adult Health, Pain Control; Integrated Process:
Nursing Process, Implementation; Cognitive Level: Application;
15. CORRECT ANSWER: 2. Answer 1 is incorrect because Client Need/Subneed: Physiological Integrity/Pharmacological
indomethacin is not a sulfa-based drug and may be used safely and Parenteral Therapies/Pharmacological Pain Management
in clients with sulfa allergies. Answer 2 is correct because a
side effect of indomethacin is gastrointestinal bleeding. 18. CORRECT ANSWER: 2. Answer 1 is incorrect because, if
This risk is greater in clients with history of GI bleeding. given 60 minutes before a meal, this rapidly absorbing insulin
Answer 3 is incorrect because, even though indomethacin can could cause hypoglycemia before the client begins to eat.
worsen hypertension, a greater risk is hemorrhage. Answer 4 is Answer 2 is correct because the log ending indicates this
incorrect because indomethacin does not affect blood sugar. drug is one of the insulin analogues. Very small changes in
TEST-TAKING TIP: Only one option addresses a side effect of the amino acid sequence facilitate rapid absorption of this
the drug. drug. Answer 3 is incorrect because regular and rapid-acting
Content Area: Adult Health, Pharmacology; Integrated Process: insulin should be given before meals so that they are active in
Nursing Process, Assessment; Cognitive Level: Application; the blood as sugars are being absorbed from the intestine.
ANSWERS

Client Need/Subneed: Physiological Integrity/Pharmacological Answer 4 is incorrect because regular and rapid-acting insulin
and Parenteral Therapies/Medication Administration should be given before meals so that they are active in the
blood as sugars are being absorbed from the intestine.
16. CORRECT ANSWER: 1. Answer 1 is correct because TEST-TAKING TIP: The goal is to prevent hyperglycemia
bupivacaine blocks neuron transmission, numbing tissue give before eating. Choose between short (15 minutes) and
2164_Ch08_603-688 29/03/12 12:34 PM Page 675

chapter 8 Physiological Integrity 675


long (60 minutes) interval. Know the peak effect time for Client Need/Subneed: Physiological Integrity/Pharmacological
rapid-acting insulin. Choose the shortest time period. and Parenteral Therapies/Pharmacological Pain Management
Content Area: Adult Health, Endocrine; Integrated Process:
22. CORRECT ANSWER: 1. Answer 1 is correct because
Nursing Process, Implementation; Cognitive Level: Application;
administration of therapy within 3 hours decreases the size
Client Need/Subneed: Physiological Integrity/Pharmacological
of the stroke and improves client outcome. Answer 2 is
and Parenteral Therapies/Medication Administration
incorrect because the typical signs of a stroke occur earlier. By
19. CORRECT ANSWER: 1. Answer 1 is correct because the time the signs of increased intracranial pressure (ICP) are
Carafate should be given alone 30 to 60 minutes before present, there has been distortion of brain centers. Answer 3 is
mealtime. It is activated by stomach acid and coats a stom- incorrect because this is a sign of increased intracranial pressure
ach ulcer. Answer 2 is incorrect because the drug may be taken and the Cushing response. Numbness, trouble speaking, or loss
with other drugs. Only digoxin and tetracycline should be of balance would be earlier signs of brain attack. Answer 4 is
taken at different times. Answer 3 is incorrect because consti- incorrect because clinical research has shown that therapy is
pation is not a side effect. Diarrhea would be more common. most effective when given within 3 hours after the onset of
Answer 4 is incorrect because the drug should not be diluted to symptoms.
obtain maximum effect; water does not coat the stomach. TEST-TAKING TIP: Think the sooner the better.
TEST-TAKING TIP: Look for the option that would maximize Content Area: Adult Health, Cardiovascular; Integrated Process:
the therapeutic coating effect of Carafate. Nursing Process, Implementation; Cognitive Level: Application;
Content Area: Adult Health, Gastrointestinal; Integrated Process: Client Need/Subneed: Physiological Integrity/Pharmacological
Nursing Process, Evaluation; Cognitive Level: Application; and Parenteral Therapies/Medication Administration
Client Need/Subneed: Physiological Integrity/Pharmacological
23. CORRECT ANSWER: 3. Answer 1 is incorrect because cal-
and Parenteral Therapies/Medication Administration
cium therapy is indicated for hypocalcemia, not hyperthy-
20. CORRECT ANSWER: 40. Calculate the rate using the roidism. Answer 2 is incorrect because calcium gluconate is
following formula: used to treat hypocalcemia, not blood loss. Answer 3 is correct
10 units 4 units because Trousseaus sign is present with hypocalcemia. The

100 mL x mL carpal spasm will not occur if treatment has been effective.
TEST-TAKING TIP: Memorize the formulas for calculating Answer 4 is incorrect because Chvosteks sign, the twitching of
infusion rates. facial muscles when tapped, indicates hypocalcemia.
Content Area: Adult Health, Endocrine; Integrated Process: TEST-TAKING TIP: Know why calcium gluconate is givenfor
Nursing Process, Implementation; Cognitive Level: Analysis; low calcium, which causes tetany as evidenced by Trousseaus sign.
Client Need/Subneed: Physiological Integrity/Pharmacological Content Area: Adult Health, Fluid and Electrolyte Imbalances;
and Parenteral Therapies/Dosage Calculation Integrated Process: Nursing Process, Evaluation; Cognitive
Level: Application; Client Need/Subneed: Physiological
21. CORRECT ANSWER: 4. Answer 1 is incorrect because Integrity/Pharmacological and Parenteral Therapies/Expected
there may be improvement in the clients perception of pain;
Effects/Outcomes
however, the concern is about oversedation by the drug. There
are sedation scales available that do help the nurse identify 24. CORRECT ANSWER: 4. Answer 1 is incorrect because
the onset of sedation in a client. Answer 2 is incorrect because ascites is the accumulation of fluid in the peritoneal space.
the client may be receiving supplemental oxygen. Answer 3 is This may occur as the result of a disease process, such as liver
incorrect because a client who is oversedated can be awakened failure, not from chemotherapy. Answer 2 is incorrect because
and will respond to questions appropriately. Answer 4 is cor- septicemia is the presence of an infection in the blood.
rect because opioids will depress ventilation, causing a Chemotherapy can increase the clients risk of infection
buildup of carbon dioxide in the blood. If hypoventilation because of the decreased WBCs, which if not treated could
is present, measuring the level of carbon dioxide is the result in septicemia. Answer 3 is incorrect because poly-
most reliable indication of possible respiratory depression. cythemia is an increase in immature RBCs. Chemotherapy
Capnography measures breath by breath the level of CO2 suppresses RBC production in the bone marrow. Answer 4 is
and is most often used during anesthesia, in critical care, correct because chemotherapy causes myelosuppression
and in emergency medicine to prevent respiratory depres- (depression of bone marrow function). WBCs, RBCs and
sion. If capnography is not available with the client who is platelets are decreased.
on PCA, monitoring the clients respiratory rate and depth TEST-TAKING TIP: Think of a common side effectone that
and assessing sedation levels is essential. all clients who are on chemotherapy usually experience.
TEST-TAKING TIP: This question is asking for the most Content Area: Adult Health, Oncological; Integrated Process:
ANSWERS

reliable indication. If a client is oversedated, ventilation will Nursing Process, Assessment; Cognitive Level: Application;
be affected. Hypoventilation affects CO2. Client Need/Subneed: Physiological Integrity/Pharmacological
Content Area: Adult Health, Pain Control; Integrated Process: and Parenteral Therapies/Adverse Effects/Contraindications/
Nursing Process, Assessment; Cognitive Level: Application; Interactions
2164_Ch08_603-688 29/03/12 12:34 PM Page 676

676 Answers/Rationales/Tips

25. CORRECT ANSWER: 1. Answer 1 is correct because 28. CORRECT ANSWERS: 1, 2, 4, 5. Answer 1 is correct
beta blockers (metoprolol) cause bronchoconstriction. because the vasodilating effect of the drug causes
Note that the client already has a respiratory problem. increased cerebral blood flow and headache. Answer 2
Answer 2 is incorrect because an aluminum hydroxide gel is is correct because the drug causes vasodilation, which
used to reduce the elevated phosphate level, which results in decreases venous return, cardiac output, and BP.
renal failure. Answer 3 is incorrect because calcium channel Answer 3 is incorrect because the effects reduce the O2
blockers (diltiazem) are used to control blood pressure in the demand on the heart, but there is no change in volume or
client with an AAA. Answer 4 is incorrect because clients pumping effectiveness. Answer 4 is correct because the
on hemodialysis are often anemic. Epoetin stimulates RBC vasodilating effect causes flushing. Answer 5 is correct
production. because vasodilation and the drop in BP could cause
TEST-TAKING TIP: Three of the answers produce the desired dizziness and light-headedness. Answer 6 is incorrect
effect for the condition; one does not. Think about the drug because the drug should decrease the O2 demand, and
action and the potential for an adverse effect as the result of the workload to cardiac function would be improved. Shortness
drugs normal action. of breath indicates heart failure. Answer 7 is incorrect
Content Area: Adult Health, Respiratory; Integrated because the drug does not interfere with gastric or
Process: Nursing Process, Analysis; Cognitive Level: intestinal functions.
Analysis; Client Need/Subneed: Physiological Integrity/ TEST-TAKING TIP: Focus on the vasodilation effect of nitro-
Pharmacological and Parenteral Therapies/Adverse glycerin on blood vessels. Consider if vasodilation will produce
Effects/Contraindications/Interactions each of the signs or symptoms.
Content Area: Adult Health, Cardiology; Integrated Process:
26. CORRECT ANSWERS: 1, 2, 3. Answer 1 is correct
Nursing Process, Assessment; Cognitive Level: Application;
because subcutaneous epinephrine is the first line of
Client Need/Subneed: Physiological Integrity/Pharmacological
treatment. Clients with bee sting allergies should carry
and Parenteral Therapies/Expected Effects/Outcomes
an EpiPen. Answer 2 is correct because Benadryl is a
standard treatment for an allergic reaction. The client 29. CORRECT ANSWER: 4. Answer 1 is incorrect because an
should chew the tablets. Answer 3 is correct because increase in rate would not be desired post-MI. Answer 2 is
Solu-Medrol is a standard anti-inflammatory drug incorrect because an increase in central venous pressure
for anaphylaxis or an allergic reaction. Answer 4 is (CVP) would signal increasing demand on the heart. Answer 3
incorrect because the diuretic effects of Lasix will not help is incorrect because this would indicate a decrease in renal
anaphylaxis. Answer 5 is incorrect because the analgesic or perfusion. Answer 4 is correct because improved cardiac
antipyretic effects of Tylenol will not help anaphylaxis. contractions will increase cardiac output. Crackles would
Answer 6 is incorrect because a histamine2 (H2) blocker be heard if heart failure were present.
such as Zantac is not a first line of defense, although the TEST-TAKING TIP: Only one of the options indicates an
drug may be given. improvement in cardiac function.
TEST-TAKING TIP: Anaphylaxis is the concern here. Know the Content Area: Adult Health, Cardiology; Integrated Process:
first-line drugs for anaphylaxis. Eliminate the drugs used for Nursing Process, Evaluation; Cognitive Level: Application;
diuretic and antipyretic effects. Client Need/Subneed: Physiological Integrity/Pharmacological
Content Area: Adult Health, Respiratory; Integrated Process: and Parenteral Therapies/Expected Effects/Outcomes
Nursing Process, Analysis; Cognitive Level: Application;
30. CORRECT ANSWER: 4. Answer 1 is incorrect because
Client Need/Subneed: Physiological Integrity/Pharmacological
there is no increased incidence of viral infections while
and Parenteral Therapies/Expected Effects/Outcomes
taking the drug. A need for an annual influenza vaccination
27. CORRECT ANSWER: 2. Answer 1 is incorrect because is not related to taking this drug. Answer 2 is incorrect
atropine is used for bradycardia or first-degree atrioventricular because the drug does not affect renal function or fluid
(AV) blocks. Answer 2 is correct because epinephrine balance. Answer 3 is incorrect because the drug does not
(adrenalin) is a first-line drug used during CPR to stimu- affect electrolyte levels. Answer 4 is correct because gingival
late cardiac activity. Answer 3 is incorrect because Lasix is a hyperplasia is a noninflammatory enlargement of the
diuretic, not an antiarrhythmic. Answer 4 is incorrect because gingiva from an increase in normal cells. Good dental
lidocaine is indicated for ventricular arrhythmias, specifically hygiene may reduce the swelling.
premature ventricular contractions (PVCs). TEST-TAKING TIP: The question asks for a common side
TEST-TAKING TIP: Consider the cardiac effects of each drug effect. Choose the option that is different from the othersonly
and the nature of the arrhythmia. The best choice is a drug one has the client report an adverse effect.
ANSWERS

that will stimulate normal cardiac activity. Content Area: Adult Health, Neurological; Integrated
Content Area: Adult Health, Cardiology; Integrated Process: Process: Teaching and Learning; Cognitive Level:
Nursing Process, Implementation; Cognitive Level: Application; Application; Client Need/Subneed: Physiological Integrity/
Client Need/Subneed: Physiological Integrity/Pharmacological Pharmacological and Parenteral Therapies/Adverse
and Parenteral Therapies/Expected Effects/Outcomes Effects/Contraindications/Interactions
2164_Ch08_603-688 29/03/12 12:34 PM Page 677

chapter 8 Physiological Integrity 677


31. CORRECT ANSWER: 4. Answer 1 is incorrect because the client with burns. Answer 3 is incorrect because
this loop diuretic does not affect appetite. Answer 2 is colloids, such as Dextran or albumin, are not used for
incorrect because taking the diuretic in the morning should primary fluid replacement. Answer 4 is incorrect because
not increase nighttime urination, with sleep disturbances. this client would be hyperkalemic and should not receive
Answer 3 is incorrect because this is a desired outcome of additional K+. NS without K+ would be appropriate for
the drug, not an adverse effect. Answer 4 is correct fluid replacement.
because leg cramps could indicate excessive loss of TEST-TAKING TIP: Look for a solution that will not contribute
potassium. to any shift of fluids or electrolytes in the first 24 hours.
TEST-TAKING TIP: Choose an option that has potentially the Content Area: Adult Health, Fluid and Electrolyte Imbalances;
greatest risk if not corrected. Eliminate options that involve the Integrated Process: Nursing Process, Analysis; Cognitive Level:
GI (Answer 1) and urinary (Answer 3) systems because, in this Application; Client Need/Subneed: Physiological Integrity/
case, they do not cause at-risk effects. Pharmacological and Parenteral Therapies/Parenteral/
Content Area: Adult Health, Fluid and Electrolyte Imbalances; Intravenous Therapies
Integrated Process: Nursing Process, Implementation;
35. CORRECT ANSWERS: 2, 4, 5, 6. Answer 1 is incorrect
Cognitive Level: Application; Client Need/Subneed:
because opioids act on the brain to decrease painagitation
Physiological Integrity/Pharmacological and Parenteral would not be expected. Answer 2 is correct because opioids
Therapies/Adverse Effects/Contraindications/Interactions do affect the GI tract. Constipation will persist even
32. CORRECT ANSWER: 2. Answer 1 is incorrect because with long-term use of opioids. Answer 3 is incorrect
the drug increases heart rate. Irregularity is not expected. because opioids are constipating. Answer 4 is correct
Answer 2 is correct because scopolamine has the same because nausea and vomiting are common, and may
effects as atropine, which is used to dry secretions subside in a few days. Fluids and an antiemetic may
preoperatively. Answer 3 is incorrect because the drug help. Answer 5 is correct because itching does occur.
increases the heart rate, so hypotension is not expected. However, it is not considered an allergy to the drug.
Answer 4 is incorrect because the drug affects salivation, Antihistamines are given. Answer 6 is correct because
not speech. the drug acts directly on the brain and produces
TEST-TAKING TIP: Look for a side effect that would be sedation. However, the client will quickly develop
annoying but not have a severe consequence. tolerance to sedation.
Content Area: Adult Health, Gastrointestinal; Integrated TEST-TAKING TIP: When two options are contradictory
Process: Nursing Process, Implementation; Cognitive Level: (e.g., Answers 1 and 6, and Answers 2 and 3), one of them
Application; Client Need/Subneed: Physiological Integrity/ is incorrect.
Pharmacological and Parenteral Therapies/Adverse Effects/ Content Area: Adult Health, Pain Control; Integrated Process:
Contraindications/Interactions Nursing Process, Assessment; Cognitive Level/Subneed:
Analysis; Client Need: Physiological Integrity/Pharmacological
33. CORRECT ANSWER: 1. Answer 1 is correct because a and Parenteral Therapies/Adverse Effects/Contraindications/
dry cough might be annoying, but would not place the Interactions
client at increased risk for injury or debilitation.
Answer 2 is incorrect because progression of heart failure 36. CORRECT ANSWERS: 2, 3. Answer 1 is incorrect
would not be an acceptable side effect. Answer 3 is incorrect because volume will increase, but fluid will not be pulled
because sedation would be an unsafe consequence of the from the tissue. Answer 2 is correct because this solution is
drug. Answer 4 is incorrect because incontinence would not classified as a hypertonic volume expander, pulling fluids
be an acceptable side effect for an adult. from the tissues. Answer 3 is correct because albumin is a
TEST-TAKING TIP: The question focuses on what is an hypertonic solution and will increase the osmotic pres-
OK side effect so that the drug doesnt have to be stopped. sure, pulling fluid into the vascular space. Answer 4 is
Three of the choices have the potential to incapacitate the incorrect because lactated Ringers is considered to be iso-
client. tonic, not causing any shift of fluids. Volume will increase,
Content Area: Adult Health, Cardiovascular; Integrated but fluid will remain in the tissues. Answer 5 is incorrect
Process: Teaching and Learning; Cognitive Level: Application; because packed red blood cells improve oxygen-carrying
Client Need/Subneed: Physiological Integrity/Pharmacological capacity, but not volume.
and Parenteral Therapies/Adverse Effects/Contraindications/ TEST-TAKING TIP: Look for a solution that will increase
Interactions osmotic (pull) pressure in the intravascular (venous) system.
Content Area: Adult Health, Fluid and Electrolyte Imbalances;
34. CORRECT ANSWER: 2. Answer 1 is incorrect because
ANSWERS

Integrated Process: Nursing Process, Analysis; Cognitive Level:


dextrose can cause a pseudodiabetes during the first 24 hours Application; Client Need/Subneed: Physiological Integrity/
after a burn. Answer 2 is correct because a crystalloid, Pharmacological and Parenteral Therapies/Expected Effects/
such as lactated Ringers, will correct the volume deficit, Outcomes
sodium loss, and metabolic acidosis that is present in
2164_Ch08_603-688 29/03/12 12:34 PM Page 678

678 Answers/Rationales/Tips

37. CORRECT ANSWERS: 2, 4, 5. Answer 1 is incorrect 40. CORRECT ANSWER: 180. This is a three-step problem.
because epinephrine is the first line of treatment for an First, calculate the ordered dose. Second, convert milligrams
allergic reaction. This client is experiencing a hemolytic to micrograms. Third, calculate the flow rate.
reaction. Epinephrine blocks the effects of the histamine
1. 4 mcg/min  75 kg  300 mcg/min
released during an allergic reaction. Answer 2 is correct
2. 50 mg  1,000 mcg  50,000 mcg
because the client is experiencing hemolytic shock and
3. 50,000 mcg divided by 500 mL  100 mcg/mL; then
sodium bicarbonate will be needed to reverse metabolic
acidosis. Answer 3 is incorrect because theophylline is a 300 mcg/min  3 mL/min  60 min  180 mL/hr
bronchodilator, and would improve wheezing, but it would 100 mcg/mL
not reverse the antibody effects and the potential for shock, TEST-TAKING TIP: Dont forget to convert mg to mcg.
which is causing the signs and symptoms. Answer 4 is cor- Content Area: Adult Health, Pharmacology; Integrated Process:
rect because hydrocortisone will be given to control the Nursing Process, Implementation; Cognitive Level: Analysis;
effects of the antigen-antibody reaction that has occurred Client Need/Subneed: Physiological Integrity/Pharmacological
from apparent ABO incompatibility. Answer 5 is correct and Parenteral Therapies/Dosage Calculation
because this client is experiencing a hemolytic reaction
41. CORRECT ANSWERS: 1, 4. Answer 1 is correct
that can progress to septic shock. A vasopressor will be
because the clients head is elevated, which prevents
needed to maintain the blood pressure.
the analgesia from affecting the respiratory muscles.
TEST-TAKING TIP: Recognize the difference between a
Answer 2 is incorrect because the clients head needs to be
hemolytic reaction (ABO incompatibility) and an allergic
elevated, not flat, to prevent the drug from affecting the
reaction to the donor protein (likely a hereditary component
respiratory muscles. Answer 3 is incorrect because the
to this reaction).
clients head needs to be elevated, not flat, to prevent
Content Area: Adult Health, Pharmacology; Integrated
the drug from affecting the respiratory muscles. Answer 4
Process: Nursing Process, Implementation; Cognitive Level:
is correct because head elevation prevents migration
Application; Client Need/Subneed: Physiological Integrity/
of the drug toward the respiratory muscles. Answer 5 is
Pharmacological and Parenteral Therapies/Blood and Blood
incorrect because the clients head needs to be elevated
Products
and not face down, to prevent the drug from affecting the
38. CORRECT ANSWER: 1. Answer 1 is correct because respiratory muscles.
400 to 800 IU of vitamin D is needed a day for proper TEST-TAKING TIP: Eliminate the two answers with flat.
absorption and metabolism of calcium. Answer 2 is incor- Picture prone and eliminate Answer 5. Choose the two
rect because calcium citrate can be taken anytime. Answer 3 answers that maximize analgesia without affecting the
is incorrect because calcium carbonate should be taken with respiratory muscles.
food. Answer 4 is incorrect because calcium is frequently Content Area: Adult Health, Pain Control; Integrated Process:
taken in divided doses. Nursing Process, Implementation; Cognitive Level: Application;
TEST-TAKING TIP: Choose the answer that is different from Client Need/Subneed: Physiological Integrity/Pharmacological
the others. Three options deal with the time that calcium is and Parenteral Therapies/Pharmacological Pain Management
taken.
42. CORRECT ANSWER: 200. The solution contains the
Content Area: Adult Health, Musculoskeletal; Integrated
prescribed dose. Infuse the 100 mL at a rate of 200 mL/hr
Process: Teaching and Learning; Cognitive Level: Application;
and the drug will be received in 30 minutes as ordered.
Client Need/Subneed: Physiological Integrity/Pharmacological
TEST-TAKING TIP: To calculate the flow rate, double the
and Parenteral Therapies/Medication Administration
volume in order to infuse in half the time.
39. CORRECT ANSWER: 1. Answer 1 is correct because over- Content Area: Adult Health, Pharmacology; Integrated Process:
the-counter artificial tears are effective for dry eyes initially. Nursing Process, Analysis; Cognitive Level: Application;
Answer 2 is incorrect because conservative treatment, such as Client Need/Subneed: Physiological Integrity/Pharmacological
artificial tears, is indicated before use of a prescriptive treat- and Parenteral Therapies/Dosage Calculation
ment to increase tear production and reduce inflammation.
43. CORRECT ANSWERS: 1, 5. Answer 1 is correct
Answer 3 is incorrect because this would slow the rate of tear
because these are local signs of tissue damage from
drainage and more tears would stay on the eyes. Answer 4 is
infiltration of a vesicant, which causes blistering or
incorrect because pilocarpine drops are ordered to treat dry
necrosis with infiltration. Answer 2 is incorrect because
mouth, not dry eyes.
IV push increases the chance of infiltration. Nonvesicants,
TEST-TAKING TIP: Think conservative treatment initially.
which may cause little to no irritation if infiltration occurs,
ANSWERS

Content Area: Adult Health, Sensory; Integrated Process:


are given safely by IV push. Answer 3 is incorrect because
Nursing Process, Implementation; Cognitive Level: Application;
not all vesicants are cardiotoxic. Answer 4 is incorrect
Client Need/Subneed: Physiological Integrity/Pharmacological
because ice or a cold pack should not be used with vinca
and Parenteral Therapies/Expected Effects/Outcomes
2164_Ch08_603-688 29/03/12 12:34 PM Page 679

chapter 8 Physiological Integrity 679


alkaloids. Answer 5 is correct because heat is specifically 48. CORRECT ANSWER: 167. Calculate the rate using the fol-
recommended to prevent ulceration with vincristine or lowing formula:
vinblastine if extravasation of the drug occurs. gtt  10 gtt  500 mL  5000 gtt  166.6 or 167 gtt/min
TEST-TAKING TIP: Assess and heat are the best answers. min mL 30 min 30 min
Content Area: Adult Health, Oncological; Integrated Process: TEST-TAKING TIP: Use a ratio formula when calculating.
Nursing Process, Assessment; Cognitive Level: Application; Content Area: Adult Health, Pharmacology; Integrated
Client Need/Subneed: Physiological Integrity/Pharmacological Process: Nursing Process, Analysis; Cognitive Level: Analysis;
and Parenteral Therapies/Medication Administration Client Need/Subneed: Physiological Integrity/Pharmacological

44. CORRECT ANSWER: 25. Calculate the rate using the fol- and Parenteral Therapies/Dosage Calculation
lowing formula: 49. CORRECT ANSWER: 2, 3, 4. Answer 1 is incorrect
Desired amount  Quantity available  x because Synthroid would be ordered for hypothyroidism.
Available amount Answer 2 is correct because radioactive iodine is an
5 units  500 mL  2500x  25 appropriate drug treatment for hyperthyroidism.
100 units 1 100 Answer 3 is correct because PTU is used in the treatment
TEST-TAKING TIP: Memorize the formula to solve for of hyperthyroidism. Answer 4 is correct because Tapazole
mL/hr. is an appropriate drug for the treatment of hyperthy-
Content Area: Adult Health, Pharmacology; Integrated Process: roidism. Answer 5 is incorrect because Cytomel is used in
Nursing Process, Analysis; Cognitive Level: Application; the treatment of hypothyroidism.
Client Need/Subneed: Physiological Integrity/Pharmacological TEST-TAKING TIP: The symptoms are for hyperthyroidism.
and Parenteral Therapies/Dosage Calculation Therefore, eliminate the drugs that appear to include a thyroid
45. CORRECT ANSWER: 200. The solution contains the replacementthyro in the name.
Content Area: Adult Health, Endocrine; Integrated Process:
prescribed dose. Simply calculate the infusion rate. Since
15 minutes is one fourth of an hour, multiply the volume Nursing Process, Analysis; Cognitive Level: Analysis; Client
Need/Subneed: Physiological Integrity/Pharmacological and
times 4.
TEST-TAKING TIP: No formula is needed. Dont make this Parenteral Therapies/Expected Effects/Outcomes
question more difficult. 50. CORRECT ANSWER: 2. Answer 1 is incorrect because
Content Area: Adult Health, Pharmacology; Integrated Process: NPH is an intermediate-acting insulin given subcutaneously.
Nursing Process, Analysis; Cognitive Level: Application; Answer 2 is correct because regular insulin is the only
Client Need/Subneed: Physiological Integrity/Pharmacological form of insulin that can be given IV. Answer 3 is incorrect
and Parenteral Therapies/Dosage Calculation because Lantus is a long-acting insulin. It should be used to
46. CORRECT ANSWER: 5. maintain a normal blood sugar, and is given subcutaneously.
Answer 4 is incorrect because Lispro, a rapid-acting insulin,
Step 1: Convert the lb to kg: 110 lb  1 kg/2.2 lb  50 kg
cannot be given IV.
Step 2: Calculate drug delivery rate for this client, who
TEST-TAKING TIP: The stem asks for IV administration. Only
weighs 50 kg:
regular insulin can be given both IV and subcutaneously.
0.01 mcg/kg/min  50 kg  0.5 mcg/min
Content Area: Adult Health, Endocrine; Integrated Process:
Step 3: Calculate volume delivery:
Nursing Process, Implementation; Cognitive Level: Application;
x mL  250 mL  1 mg  0.5 mcg  60 min  5 mL/hr
Client Need/Subneed: Physiological Integrity/Pharmacological
hr 1.5 mg 1000 mcg min 1 hr
and Parenteral Therapies/Expected Effects/Outcomes
TEST-TAKING TIP: Use dimensional analysis to solve for x.
Content Area: Adult Health, Pharmacology; Integrated Process: 51. CORRECT ANSWER: 3. Answer 1 is incorrect because regular
Nursing Process, Analysis; Cognitive Level: Analysis; Client insulin is safe to mix with NPH. Answer 2 is incorrect because
Need/Subneed: Physiological Integrity/Pharmacological and NPH is safe to mix with Humulin-N, another name for NPH
Parenteral Therapies/Dosage Calculation or Lente insulin. Answer 3 is correct because, when Lantus is
mixed with other insulins, it is rendered useless. Lantus is a
47. CORRECT ANSWER: 24. Calculate the rate using the
long-acting insulin with no peak. Answer 4 is incorrect because
following formula:
Humulin-R is safe to mix with Lente (or NPH).
mL  100 mL  12 units  24 mL/hr
TEST-TAKING TIP: Select the option that includes Lantus,
hr 50 units 1 hour
which is not in any other option.
TEST-TAKING TIP: Use the correct formula to compute the
Content Area: Adult Health, Endocrine; Integrated Process:
rate.
Nursing Process, Analysis; Cognitive Level: Application;
ANSWERS

Content Area: Adult Health, Pharmacology; Integrated


Client Need/Subneed: Physiological Integrity/Pharmacological
Process: Nursing Process, Analysis; Cognitive Level: Analysis;
and Parenteral Therapies/Adverse Effects/Contraindications/
Client Need/Subneed: Physiological Integrity/Pharmacological
Interactions
and Parenteral Therapies/Dosage Calculation
2164_Ch08_603-688 29/03/12 12:34 PM Page 680

680 Answers/Rationales/Tips

52. CORRECT ANSWER: 1. Answer 1 is correct because is taken near bedtime, the increased need to urinate will
Lispro is a rapid-acting insulin and should always be disrupt sleep.
drawn up first, to avoid contaminating the bottle of the TEST-TAKING TIP: The question is asking for the desired
fast-acting insulin with intermediate- or slow-acting effect of a diureticlook for a description that shows fluid is
insulin. Answer 2 is incorrect because NPH could contami- being removed (i.e., excreted).
nate the Lispro. Answer 3 is incorrect because Lantus should Content Area: Adult Health, Renal; Integrated Process: Nursing
never be mixed. It is rendered useless when mixed. Answer 4 Process, Implementation; Cognitive Level: Application;
is incorrect because Novolin 70/30 is a mixture of long Client Need/Subneed: Physiological Integrity/Pharmacological
lasting (70 units) with rapid (30 units); therefore, further and Parenteral Therapies/Expected Effects/Outcomes
mixings should be questioned. 56. CORRECT ANSWER: 3. Answer 1 is incorrect because a
TEST-TAKING TIP: It is more important to remember to clients temperature has nothing to do with giving a beta
draw the rapid-acting insulin first, rather than remembering blocker. Answer 2 is incorrect because there are no required
clear to cloudy. positions for ingestion. The client chooses a position that
Content Area: Adult Health, Endocrine; Integrated Process:
facilitates swallowing. Answer 3 is correct because a beta
Nursing Process, Implementation; Cognitive Level: Application; blocker can decrease blood pressure and heart rate to
Client Need/Subneed: Physiological Integrity/Pharmacological
unsafe levels. Answer 4 is incorrect because the client may
and Parenteral Therapies/Medication Administration not be aware of the effects of the drug on heart rate and
53. CORRECT ANSWER: 1. Answer 1 is correct because, blood pressure. Also, the nurse needs objective data to assess
following the first dose, a second or third dose can be the response, not subjective data.
given in 5-minute intervals. Answer 2 is incorrect TEST-TAKING TIP: Ask yourself: Which vital sign is most rele-
because, if the pain is due to angina, the greatest effect vant to the drugs antihypertensive effect, BP or temperature?
occurs with 5-minute intervals. Answer 3 is incorrect Content Area: Adult Health, Cardiovascular; Integrated Process:
because waiting too long may result in ischemic damage. Nursing Process, Implementation; Cognitive Level: Application;
Answer 4 is incorrect because repeated doses in a shorter Client Need/Subneed: Physiological Integrity/Pharmacological
period of time are needed to determine the cause of the and Parenteral Therapies/Medication Administration
chest pain. 57. CORRECT ANSWER: 2. Answer 1 is incorrect because the
TEST-TAKING TIP: Remember: the maximum effect of increase in vascular volume might result in hemodilution of
nitroglycerin is achieved in 5 minutes . . .take five. sodium, not excess. Answer 2 is correct because TPN is a
Content Area: Adult Health, Cardiovascular; Integrated Process:
high-glucose and hyperosmolar solution that would result
Nursing Process, Planning; Cognitive Level: Comprehension; in increased vascular volume. Answer 3 is incorrect because
Client Need/Subneed: Physiological Integrity/Pharmacological
the blood glucose level and blood pressure would increase with
and Parenteral Therapies/Medication Administration rapid infusion, not decrease. Answer 4 is incorrect because an
54. CORRECT ANSWER: 4. Answer 1 is incorrect because increase in blood pressure and hyperglycemia would occur
the head of the bed should be elevated, not flat, to prevent before hyperkalemia and heart failure.
aspiration. Answer 2 is incorrect because the feeding should TEST-TAKING TIP: Focus on two contradictory options:
be given slowly to prevent diarrhea and vomiting. Answer 3 is BP and hyperglycemia vs. BP and hypoglycemia. Think
incorrect because the feeding should be warm. Also, clients too fast too high.
cannot taste the feedings; improving taste is not a concern. Content Area: Adult Health, Total Parenteral Nutrition;
Answer 4 is correct because giving the feeding too fast Integrated Process: Nursing Process, Analysis; Cognitive Level:
could upset the clients stomach and cause diarrhea. Application; Client Need/Subneed: Physiological Integrity/
TEST-TAKING TIP: Choose an option that minimizes vomit- Pharmacological and Parenteral Therapies/Total Parenteral
ing and aspiration. Nutrition
Content Area: Adult Health, Gastrointestinal; Integrated Process:
58. CORRECT ANSWER: 3. Answer 1 is incorrect because
Nursing Process, Implementation; Cognitive Level: Application; Lomotil, an opioid, is used to control diarrhea. Answer 2 is
Client Need/Subneed: Physiological Integrity/Pharmacological
incorrect because constipation and urinary retention are
and Parenteral Therapies/Total Parenteral Nutrition common with morphine. Answer 3 is correct because
55. CORRECT ANSWER: 3. Answer 1 is incorrect because meperidine produces less constipation. However, all
furosemide lowers blood pressure, not elevates. Answer 2 is opioids used in the management of pain can cause
incorrect because furosemide does not usually cause tachy- constipation. Answer 4 is incorrect because fentanyl does
cardia; however, the heart rate might increase if too much cause constipation. This drug is usually reserved for
ANSWERS

fluid is removed. Answer 3 is correct because the drug is anesthesia or cancer-related pain.
a diuretic and there should be an increase in frequency TEST-TAKING TIP: Look at the two frequently used opioid
and volume of urine if furosemide is working. Answer 4 is analgesics (morphine and meperidine). Select the option that
incorrect because the client needs to be told that, if the drug has fewer GI problems.
2164_Ch08_603-688 29/03/12 12:34 PM Page 681

chapter 8 Physiological Integrity 681


Content Area: Geriatrics, Pain Control; Integrated Process: Need/Subneed: Physiological Integrity/Pharmacological and
Nursing Process, Implementation; Cognitive Level: Application; Parenteral Therapies/Pharmacological Pain Management
Client Need/Subneed: Physiological Integrity/Pharmacological
62. CORRECT ANSWER: 1. Answer 1 is correct because
and Parenteral Therapies/Adverse Effects/Contraindications/
blood volume increases by 30% to 50% in pregnancy. This
Interactions
causes hemodilution of RBCs and physiological anemia.
59. CORRECT ANSWER: 2. Answer 1 is incorrect because Normal hemoglobin levels in pregnancy range from 11 to
blood pressure and pulse should decrease since MS is a 12 gm/dL. Answer 2 is incorrect because plethora (hemoglobin
vasodilator. Answer 2 is correct because, without any >15.5 gm/dL for women) is not a normal finding in pregnancy.
further client information, this is an appropriate dosage Answer 3 is incorrect because normal fasting serum glucose in
for relief of moderate pain. Answer 3 is incorrect because pregnancy is less than 95 mg/dL. Answer 4 is incorrect because
the dose is sufficient for a client whose weight is 154 lb. a normal WBC for first trimester pregnancy is 6,600 to
Answer 4 is incorrect because the pain relief should be 14,100/mL.
near 4 hours. TEST-TAKING TIP: This question asks to identify the normal
TEST-TAKING TIP: Focus on two options with a time frame. value for the first trimester of pregnancy. Know that laboratory
Select the option with the longer lasting effect when the pain is values change during pregnancy.
moderate. Content Area: Maternity, Antepartum; Integrated Process:
Content Area: Adult Health, Pain Control; Integrated Process: Nursing Process, Analysis; Cognitive Level: Application;
Nursing Process, Analysis; Cognitive Level: Application; Client Client Need/Subneed: Physiological Integrity/Reduction of
Need/Subneed: Physiological Integrity/Pharmacological and Risk Potential/Laboratory Values
Parenteral Therapies/Expected Effects/Outcomes
63. CORRECT ANSWER: 2. Answer 1 is incorrect because
60. CORRECT ANSWER: 3. Answer 1 is incorrect because decreased urinary output is not an expected side effect of mag-
increasing the dosage to compensate for the poor perfusion nesium sulfate; it is an indicator of worsening preeclampsia.
would be unsafe. There would be an increased risk of side Answer 2 is correct because hypersomnolence (excessive
effects with a larger dose. Answer 2 is incorrect because these sleepiness) is an expected side effect not associated with
are enzymes that are cardiac specific and that do not increase magnesium toxicity, an unexpected side effect related to a
from other causes of muscle trauma such as injections. Answer high serum level. Answer 3 is incorrect because the absence of
3 is correct because drug effectiveness would be less pre- knee jerk reflex is a symptom of magnesium toxicity. Answer 4
dictable with the change in perfusion. Answer 4 is incorrect is incorrect because decreased respiratory rate is a symptom of
because anticoagulant therapy does not prevent the use of IM magnesium toxicity.
injections. TEST-TAKING TIP: This question is asking for the expected
TEST-TAKING TIP: Look for the answer that refers to tissue side effect of magnesium sulfate administration. Eliminate the
perfusion after an MI. Remember: tissue perfusion is needed answers that are indicative of magnesium toxicity.
for drug absorption with the IM route. Content Area: Maternity, Intrapartum; Integrated Process:
Content Area: Adult Health, Cardiovascular; Integrated Process: Nursing Process, Assessment; Cognitive Level: Application;
Teaching and Learning; Cognitive Level: Application; Client Need/Subneed: Physiological Integrity/Pharmacological
Client Need/Subneed: Physiological Integrity/Pharmacological and Parenteral Therapies/Expected Effects/Outcomes
and Parenteral Therapies/Medication Administration
64. CORRECT ANSWER: 4. Answer 1 is incorrect because the
61. CORRECT ANSWER: 2. Answer 1 is incorrect because side effect from epidural anesthesia that the nurse needs to
effects of the drug may increase with liver damage because monitor is hypotension. Cervical dilation is an assessment that
the liver is responsible for detoxification of drugs. Answer 2 is common to all clients in labor. Answer 2 is incorrect because
is correct because the client will likely need a higher dose placing the client in a supine position will worsen the hypoten-
because alcohol is a CNS depressant. The client will have sion. Answer 3 is incorrect because obtaining a fetal heart rate
developed a drug tolerance to a standard does of pain is a common assessment to all clients in labor, and does not
medication. Answer 3 is incorrect because the dosage should address the specific side effect of epidural anesthesia. Answer 4
be lowered if there is liver damage, where there would be is correct because checking the clients blood pressure is
impaired ability of the liver to detoxify medications. Answer 4 most important with epidural anesthesia.
is incorrect because the client will likely have a decreased TEST-TAKING TIP: This question is asking about a side effect
sensitivity that requires a higher dosage. that is common in epidural anesthesia. Regular assessments of
TEST-TAKING TIP: Eliminate the two answers that focus on cervical dilation and fetal heart rate are common to all clients in
the liver (Answers 1 and 3). The reason for a higher dose is labor and do not address the specific condition in the question.
ANSWERS

because alcohol depresses the normal nervous system response Content Area: Maternity, Intrapartum; Integrated Process:
to pain. Nursing Process, Assessment; Cognitive Level: Application;
Content Area: Adult Health, Pain Control; Integrated Process: Client Need/Subneed: Physiological Integrity/Pharmacological
Nursing Process, Analysis; Cognitive Level: Application; Client and Parenteral Therapies/Pharmacological Pain Management
2164_Ch08_603-688 29/03/12 12:34 PM Page 682

682 Answers/Rationales/Tips

65. CORRECT ANSWER: 4. Answer 1 is incorrect because the client has a normal assessment, there is no need for
maternal hypoglycemia is unrelated to the administration intervention.
of oxytocin, either for induction of labor or for use after Content Area: Maternity, Antepartum; Integrated Process:
delivery. Answer 2 is incorrect because fetal movement is a Nursing Process, Implementation; Cognitive Level:
general assessment for all clients in labor; it is not specific Application; Client Need/Subneed: Physiological Integrity/
to the administration of oxytocin. Answer 3 is incorrect Pharmacological and Parenteral Therapies/Expected Effects/
because maternal hyperreflexia is an assessment specifically Outcomes
for a client with preeclampsia, not Pitocin induction. 68. CORRECT ANSWER: 3. Answer 1 is incorrect because
Answer 4 is correct because oxytocin administration can
treatment of pain is a priority and will not interfere with
be associated with tetanic contractions and inadequate the ability to diagnose a fracture of back or limb.
resting period between contractions, resulting in fetal Answer 2 is incorrect because the pain needs to be relieved,
bradycardia. not minimized. Answer 3 is correct because pain needs
TEST-TAKING TIP: Look for an assessment specific to the to be treated immediately and maximally, to avoid
side effects of oxytocin induction, not general assessments or potential complications such as shock. Answer 4 is
assessment of a condition that the client does not have. The incorrect because this analgesic choice will not relieve
effect on the fetus (Answer 4) is most significant for the nurse severe pain.
to monitor. TEST-TAKING TIP: Relief of pain is a priority. Unrelieved
Content Area: Maternity, Intrapartum; Integrated Process:
pain can cause shock.
Nursing Process, Assessment; Cognitive Level: Application; Content Area: Adult Health, Pain Control; Integrated Process:
Client Need/Subneed: Physiological Integrity/Pharmacological
Nursing Process, Implementation; Cognitive Level: Application;
and Parenteral Therapies/Adverse Effects/Contraindications/ Client Need/Subneed: Physiological Integrity/Pharmacological
Interactions and Parenteral Therapies/Pharmacological Pain Management
66. CORRECT ANSWER: 4. Answer 1 is incorrect because 69. CORRECT ANSWER: 2. Answer 1 is incorrect because,
magnesium sulfate is a treatment for preterm labor and while a specific client may have concerns about becoming
preeclampsia; it is not a treatment to induce labor. addicted to opioids, it is more often physicians who are
Answer 2 is incorrect because Clomid is a treatment to
concerned about abuse. Answer 2 is correct because
induce ovulation, not delivery. Answer 3 is incorrect physicians often undermedicate clients because of fear
because calcium gluconate is an antidote for magnesium that the client is abusing a narcotic opioid. Answer 3 is
toxicity, not a treatment for inducing labor. Answer 4 is incorrect because the side effects can be managed if they
correct because this client has a fetus that has died. occur. Answer 4 is incorrect because there are accurate
Cytotec is an appropriate treatment for inducing assessment tools.
delivery of a fetal demise. TEST-TAKING TIP: Choose the option that is the greatest
TEST-TAKING TIP: The question is asking how to induce barrier . . . staff s concern.
delivery in a client with a fetal demise or a missed abortion. Content Area: Adult Health, Pain Control; Integrated
Eliminate the drugs that do not induce labor. Process: Nursing Process, Analysis; Cognitive Level:
Content Area: Maternity, Intrapartum; Integrated Process:
Application; Client Need/Subneed: Physiological
Nursing Process, Planning; Cognitive Level: Application; Integrity/Pharmacological and Parenteral Therapies/
Client Need/Subneed: Physiological Integrity/Pharmacological
Pharmacological Pain Management
and Parenteral Therapies/Expected Effects/Outcomes
70. CORRECT ANSWER: 90. Calculate the delivery rate using
67. CORRECT ANSWER: 2. Answer 1 is incorrect because the following formula:
the assessment is normal for this client; there is no reason 1000 mcg  50 mg  27 mL  1 hour  90 mcg/min
to decrease the infusion. Decreasing the magnesium level 1 mg 250 mL 1 hr 60 min
may put the client at risk for eclampsia. Answer 2 is correct TEST-TAKING TIP: First, convert mg to mcg, then use the
because the assessment findings are normal for this formula.
client; continuing the infusion is the appropriate action Content Area: Adult Health, Pharmacology; Integrated
for the nurse. Answer 3 is incorrect because the assessment Process: Nursing Process, Analysis; Cognitive Level: Analysis;
is normal for this client; there is no reason to stop the infu- Client Need/Subneed: Physiological Integrity/Pharmacological
sion. Decreasing the magnesium level may put the client at and Parenteral Therapies/Dosage Calculation
risk for eclampsia. Answer 4 is incorrect because the assess-
ment is normal for this client; there is no reason to adminis- 71. CORRECT ANSWER: 1. Answer 1 is correct because
ANSWERS

ter an antidote to the client. Decreasing the magnesium level it is the only one that fits all four symptoms. Answer 2 is
may put the client at risk for eclampsia. incorrect because with phlebitis the skin would be warm
TEST-TAKING TIP: This question is asking if the client is and red. Answer 3 is incorrect because sepsis is a systemic
having symptoms consistent with magnesium toxicity. Since problem, not a localized complication. Answer 4 is incorrect
2164_Ch08_603-688 29/03/12 12:34 PM Page 683

chapter 8 Physiological Integrity 683


because an allergic response is usually systemic, or if Content Area: Adult Health, Infectious Disease; Integrated
localized, it would produce redness. Process: Teaching and Learning; Cognitive Level: Application;
TEST-TAKING TIP: What would be the expected color and Client Need/Subneed: Physiological Integrity/Pharmacological
skin temperature for each option? Only one of the options and Parenteral Therapies/Expected Effects/Outcomes
would be cool and pale.
75. CORRECT ANSWER: 3. Answer 1 is incorrect because
Content Area: Adult Health, Integumentary; Integrated Process:
a corticosteroid is not given to improve circulation.
Nursing Process, Analysis; Cognitive Level: Application; Client
Answer 2 is incorrect because albuterol, not a corticosteroid,
Need/Subneed: Physiological Integrity/Pharmacological and
is administered to cause bronchodilation during asthma
Parenteral Therapies/Adverse Effects/Contraindications/
attacks. Answer 3 is correct because corticosteroids are
Interactions
used in the treatment of acute asthma exacerbation to
72. CORRECT ANSWER: 2. Answer 1 is incorrect because reduce inflammation of the airways. Answer 4 is incorrect
older adults usually require lower dosages. Answer 2 is because a corticosteroid is given to reduce inflammation,
correct because changes in kidney and/or liver function not thin secretions.
will affect excretion of drugs. The chance of toxicity TEST-TAKING TIP: Corticosteroids are used in the treatment
increases. Answer 3 is incorrect because liver impairment of many diseases; do not let the diagnosis of asthma fool you
would result in a greater risk of toxicity from too high of a into choosing the answer option that is specific to the lungs.
dosage. Answer 4 is incorrect because the monitoring Two of the answer options are specific to respiration and can be
should be for clinical effectiveness, not for reduced eliminated; only one option has a general actionreduce
effectiveness. inflammationwhich is the best response.
TEST-TAKING TIP: Remember: older adults are more sensitive Content Area: Child Health, Respiratory; Integrated Process:
to drug effects; therefore, eliminate Answers 1 and 3. Nursing Process, Planning; Cognitive Level: Analysis;
Content Area: Geriatrics, Aging Process; Integrated Process: Client Need/Subneed: Physiological Integrity/Pharmacological
Nursing Process, Analysis; Cognitive Level: Application; and Parenteral Therapies/Expected Effects/Outcomes
Client Need/Subneed: Physiological Integrity/Pharmacological
76. CORRECT ANSWER: 1. Answer 1 is correct because
and Parenteral Therapies/Medication Administration
the nurse should question an order to administer an
73. CORRECT ANSWER: 3. Answer 1 is incorrect because IV solution containing potassium to a child who is
oral medications may be taken because of the type of anes- anuric. Potassium should not be added to a childs
thesia used for the surgery. Answer 2 is incorrect because this IV until kidney function is established by the return of
is not an accurate statement. Nothing has affected absorp- urine output. Answer 2 is incorrect because this would be
tion yet. Answer 3 is correct because antihypertensive an acceptable order for continuous IV fluids. Answer 3 is
medications may still be given because the procedure incorrect because this would be an acceptable order for con-
is usually done by using local, spinal, or epidural tinuous IV fluids. Answer 4 is incorrect because this would
anesthesia. Answer 4 is incorrect because IV drugs would be an acceptable order for continuous IV fluids.
only be given for an emergency. TEST-TAKING TIP: Look for the answer that is different from
TEST-TAKING TIP: Consider the type of surgery (GI) and the group. Answer 1 is the only one containing potassium.
the anesthesia (not general). Eliminate the two options with Content Area: Child Health, Gastrointestinal; Integrated
no medications . . . by mouth and not . . . anything by Process: Nursing Process, Analysis; Cognitive Level: Analysis;
mouth. Client Need/Subneed: Physiological Integrity/Pharmacological
Content Area: Adult Health, Gastrointestinal; Integrated Process: and Parenteral Therapies/Parenteral/Intravenous Therapies
Nursing Process, Implementation; Cognitive Level: Application;
77. CORRECT ANSWER: 300. To arrive at the correct
Client Need/Subneed: Physiological Integrity/Pharmacological
answer, first recall that microdrip tubing has a drip
and Parenteral Therapies/Medication Administration
rate of 60 mL/min. Next, multiply the volume to
74. CORRECT ANSWER: 1. Answer 1 is correct because be infused by the drop factor (300  60) to arrive at
rifampin changes tears, sweat, saliva, and urine to an 18,000. Then, divide this answer by the infusion time
orange-red color. This is an expected effect. Answer 2 is (18,000/60) to arrive at the correct response of
incorrect because this response does not explain the cause or 300 drops per minute.
address the clients worry. Answer 3 is incorrect because this TEST-TAKING TIP: Determine the infusion time in
color change is normal. Answer 4 is incorrect because this minute rather than hour increments to arrive at the
change does not indicate a urinary tract infection. correct response.
TEST-TAKING TIP: Eliminate the trite reassurance response Content Area: Child Health, Pharmacology; Integrated Process:
ANSWERS

(Answer 2), and the other two answers that relate to possible Nursing Process, Implementation; Cognitive Level: Analysis;
problems (Answers 3 and 4). Select the answer about an Client Need/Subneed: Physiological Integrity/Pharmacological
expected effect. and Parenteral Therapies/Dosage Calculation
2164_Ch08_603-688 29/03/12 12:34 PM Page 684

684 Answers/Rationales/Tips

78. CORRECT ANSWERS: 2, 3, 4, 5. Answer 1 is incorrect Client Need/Subneed: Physiological Integrity/Pharmacological


because iron supplementation is typically not prescribed and Parenteral Therapies/Dosage Calculation
for children with sickle cell anemia, because there already 81. CORRECT ANSWER: 3. Answer 1 is incorrect because
is a risk of excess iron in the blood. Answer 2 is correct decreased deep tendon reflexes are a side effect of
because children with sickle cell anemia frequently take magnesium sulfate therapy. Answer 2 is incorrect because
prophylactic antibiotics because of an increased risk for bradycardia is not a common side effect of a calcium
infection. Answer 3 is correct because these children channel blocker, such as nifedipine. Answer 3 is correct
may take NSAIDs for pain relief. Answer 4 is correct because nifedipine is a calcium channel blocker, more
because folic acid is commonly prescribed to assist in commonly used to treat high blood pressure and heart
the production of healthy red blood cells. Answer 5 is disease. Nifedipine blocks the passage of calcium into
correct because these children are at risk for iron over- certain tissues, relaxing the uterine muscles and smooth
load and are commonly prescribed chelation agents muscles of blood vessels throughout the body. Use of
such as Desferal. nifedipine for the treatment of preterm labor is an
TEST-TAKING TIP: Recall that children who are likely to unlabeled use of the drug. Hypotension is a side effect
receive multiple blood transfusions (as in sickle cell anemia) of nifedipine. Answer 4 is incorrect because tachycardia
are at risk for hemosiderosis, or iron overload. Select the is a side effect of beta-sympatholytic medications, such as
options that will not worsen this condition. terbutaline.
Content Area: Child Health, Hematological; Integrated
TEST-TAKING TIP: All tocolytics are smooth muscle relaxers
Process: Nursing Process, Analysis; Cognitive Level: Analysis;
and act on the smooth muscles throughout the body. None of
Client Need/Subneed: Physiological Integrity/Pharmacological
the tocolytics cause bradycardia; therefore, eliminate Answer 2.
and Parenteral Therapies/Medication Administration Eliminate also two other options that relate to other medica-
79. CORRECT ANSWER: 39. To solve this calculation, tions: magnesium sulfate (Answer 1) and a beta sympatholytic
first determine how many mL of morphine has been (Answer 4).
administered over the past 4 hours. The child has self- Content Area: Maternity, Intrapartum; Integrated Process:
administered 7 boluses of 1 mg each. If the syringe has a Nursing Process, Assessment; Cognitive Level: Application;
concentration of 1 mg/1 mL, the child has received 7 mL. Client Need/Subneed: Physiological Integrity/Pharmacological
The basal rate is 1 mg/hr; 4 hours at this rate is equal and Parenteral Therapies/Expected Effects/Outcomes
to 4 mL. Add the self-administered doses to the basal rate 82. CORRECT ANSWER: 1. Answer 1 is correct because
to get a total of 11 mL administered in 4 hours. Finally, terbutaline is considered to be working at a therapeutic
subtract 11 mL from the 50-mL syringe starting volume level when a womans resting heart rate is between 90
to arrive at the correct response of 39 mL left in the and 105 beats per minute. If the heart rate is higher
syringe after 4 hours. than 120, the physician should be notified before taking
TEST-TAKING TIP: Do not be confused by extra informa- the terbutaline. Answer 2 is incorrect because a client
tion in pharmacology questions. For this item, the informa- should not be doubling the dose of terbutaline without
tion states the child can only self-administer up to 4 boluses consulting her physician. Also, the client can have up to
per hour. This information is not to be considered; no infor- 5 contractions per hour; more than 6 contractions per hour
mation is given regarding the number of boluses the child is a reason to notify the physician. Answer 3 is incorrect
administered in a single houronly the total amount for a because normal fetal movement is 10 or more movements
4-hour period. per hour. There is no need to call the physician for normal
Content Area: Child Health, Pharmacology; Integrated Process:
fetal movement. Answer 4 is incorrect because clients
Nursing Process, Analysis; Cognitive Level: Analysis; Client with preterm labor should be resting as much as possible.
Need/Subneed: Physiological Integrity/Pharmacological and
It is not appropriate to instruct a client to increase her
Parenteral Therapies/Dosage Calculation activity.
80. CORRECT ANSWER: 200. First, determine the childs TEST-TAKING TIP: Focus on a side effect of the drug:
weight in kilograms by taking 44 and dividing by 2.2 to heart rate.
arrive at a weight of 20 kilograms. Then, determine the Content Area: Maternity, Antepartum; Integrated Process:
total amount of fluid to be administered by multiplying Nursing Process, Implementation; Cognitive Level: Application;
20 kg by 20 mL/kg to arrive at 400 mL. If the 400 mL is Client Need/Subneed: Physiological Integrity/Pharmacological
to be delivered over 2 hours, divide 400 mL by 2 to arrive and Parenteral Therapies/Expected Effects/Outcomes
at a rate of 200 mL/hr. 83. CORRECT ANSWER: 1. Answer 1 is correct because
ANSWERS

TEST-TAKING TIP: Always perform medication calculations administering a fluid bolus before beginning the
twice, especially when caring for children. epidural can prevent hypotension that is associated
Content Area: Child Health, Pharmacology; Integrated Process:
with anesthesia administration. Answer 2 is incorrect
Nursing Process, Implementation; Cognitive Level: Analysis; because administering oxygen can assist in providing
2164_Ch08_603-688 29/03/12 12:34 PM Page 685

chapter 8 Physiological Integrity 685


additional oxygen to the fetus, but does not prevent hypoten- hypertension, not preeclampsia, and magnesium is not an
sion. Answer 3 is incorrect because placing the client in antihypertensive agent.
high Fowlers position will not prevent hypotension, and TEST-TAKING TIP: This key to this question is that the
may actually decrease the effectiveness of the epidural client has chronic hypertension and is in the second trimester.
anesthesia by decreasing the dermatome level. Answer 4 is Preeclampsia generally begins in the third trimester with
incorrect because administering epinephrine is not an proteinuria and hyperreflexia; this client has no proteinuria
appropriate prevention measure. It is used to treat, not or hyperreflexia.
prevent, hypotension. Content Area: Maternity, Antepartum, At-risk; Integrated
TEST-TAKING TIP: The key word in this question is Process: Nursing Process, Planning; Cognitive Level:
prevent. Preventing a side effect is different than treating Application; Client Need/Subneed: Physiological Integrity/
the condition. An IV fluid bolus prevents hypotension; Pharmacological and Parenteral Therapies/Expected
epinephrine treats hypotension after it has developed. Effects/Outcomes
Content Area: Maternity, Intrapartum; Integrated Process:
86. CORRECT ANSWERS: 3, 5. Answer 1 is incorrect
Nursing Process, Planning; Cognitive Level: Application;
because Hemabate decreases uterine atony, by increasing
Client Need/Subneed: Physiological Integrity/Pharmacological
smooth muscle contractions. Answer 2 is incorrect
and Parenteral Therapies/Expected Effects/Outcomes
because Hemabate increases uterine contractions, not
84. CORRECT ANSWER: 4. Answer 1 is incorrect because decreases uterine contractions (the process of involution).
butorphanol tartrate is a pain medication, and does not Answer 3 is correct because the major action of Hemabate
treat the clients condition of magnesium toxicity. is to increase uterine (myometrial) contractions and
Answer 2 is incorrect because furosemide works by blocking decrease uterine bleeding. Answer 4 is incorrect because
the absorption of salt and fluid in the kidney tubules, Hemabate increases smooth muscle contractions, including
causing a profound increase in urine output (diuresis). The vascular smooth muscles. Hemabate usually results in
diuretic effect of furosemide can cause body water and an increase in blood pressure, not a decrease in blood
electrolyte depletion and is not used to treat pregnancy- pressure. Answer 5 is correct because Hemabate causes
induced hypertension, and does not treat magnesium increased smooth muscle contractions, including
toxicity. Answer 3 is incorrect because diazepam is used to gastrointestinal smooth muscle, which can lead to
treat seizures related to eclampsia. Diazepam is not used diarrhea.
to reverse magnesium toxicity. Answer 4 is correct because TEST-TAKING TIP: A memory tip: the name of the medica-
this client most likely is experiencing magnesium tion is heme abate; heme means blood and abate
toxicity. Calcium gluconate is the antidote for means to decrease or reduce. This medication is used to
magnesium overdose. reduce bleeding after delivery.
TEST-TAKING TIP: The key is to recognize that the client Content Area: Maternity, Intrapartum; Integrated Process:
is experiencing magnesium toxicity. Do not get distracted Nursing Process, Analysis; Cognitive Level: Analysis;
by other possible drugs used in treating preeclampsia. Client Need/Subneed: Physiological Integrity/Pharmacological
Remember that the antidote for magnesium is another and Parenteral Therapies/Expected Effects/Outcomes
electrolyte, calcium.
87. CORRECT ANSWER: 18 mL/hr. The answer is calculated
Content Area: Maternity, Intrapartum; Integrated Process:
as follows:
Nursing Process, Analysis; Cognitive Level: Application;
20 units/1,000 mL  0.02 units/mL. This is the concentra-
Client Need/Subneed: Physiological Integrity/Pharmacological
tion of the drug.
and Parenteral Therapies/Medication Administration
A pump rate of 1 mL/hr  0.02 units/hr.
85. CORRECT ANSWER: 2. Answer 1 is incorrect because Pitocin is most commonly expressed as milliunits/minute.
Vasotec is an angiotensin-converting enzyme (ACE) Convert 6 milliunits/min to units/min by dividing by
inhibitor. ACE inhibitors are not used in pregnancy due 1,000 mL (volume of infusion). The result is 0.006
to the potential for certain fetal defects. Answer 2 is unit/min. Multiply by 60 minutes  0.36 units/hr 
correct because this client is in the second trimester 0.02 units/hr (amount delivered in 1 mL/hr). Pump
of pregnancy and has a history of high blood pressure would be set to a rate of 18 mL/hr.
during the pregnancy, an indicator of chronic hyperten- TEST-TAKING TIP: The initial concentration of oxytocin is in
sion. Aldomet is an antihypertensive drug that is safe units/mL, which needs to be converted to milliunits/min. Pay
during pregnancy, and is commonly used for chronic attention to the complex conversion from units/mL to milliunits
hypertension during pregnancy. Answer 3 is incorrect per minute.
ANSWERS

because diazepam is used to treat seizures in eclampsia. Content Area: Maternity, Intrapartum; Integrated Process:
This client has chronic hypertension, not preeclampsia. Nursing Process, Implementation; Cognitive Level: Application;
Answer 4 is incorrect because magnesium sulfate is used Client Need/Subneed: Physiological Integrity/Pharmacological
to prevent seizures in eclampsia. This client has chronic and Parenteral Therapies/Dosage Calculation
2164_Ch08_603-688 29/03/12 12:34 PM Page 686

686 Answers/Rationales/Tips

88. CORRECT ANSWERS: 2, 3, 4. Answer 1 is incorrect the adolescent to focus more readily on difficult content,
because 3 contractions in a 10-minute window is a normal and to avoid sleep disturbances. Answer 2 is incorrect
pattern for contractions, and not a reason to discontinue the because the adolescents schedule should be arranged so
oxytocin infusion. Answer 2 is correct because contractions that academic subjects (e.g., math, science, etc.) are taught
that occur every 3 minutes, lasting 90 seconds, do not in the morning, when the adolescent is experiencing the
allow adequate resting time for the uterus, and the oxy- effect of the morning dose of medication, not midday.
tocin should be discontinued. Answer 3 is correct because This will allow the adolescent to focus more readily on
this pattern describes late decelerations, which are a sign difficult content. Answer 3 is incorrect because the adoles-
of fetal distress, and the oxytocin should be discontinued. cents schedule should be arranged so that academic subjects
Answer 4 is correct because more than 5 contractions in a (e.g., math, science, etc.) are taught in the morning, when
10-minute window is hyperstimulation, and the oxytocin the adolescent is experiencing the effect of the morning
should be discontinued. Answer 5 is incorrect because dose of medication, not in the afternoon. This will allow
being completely dilated is not a reason to discontinue the the adolescent to focus more readily on difficult content,
oxytocin infusion. Contractions are still needed for the sec- and to avoid sleep disturbances. Answer 4 is incorrect
ond stage of labor. Discontinuing the oxytocin may decrease because, while intermingling high-interest classroom
the number of contractions and increase the length of the activities with the adolescents academic classes (e.g., math,
second stage. science, etc.) will assist in maintaining the adolescents
TEST-TAKING TIP: Draw out the pattern of contractions attention and interest in learning, it does not take into
described to see if the contractions are too frequent or too long account the timing of the effectiveness of the morning
as reasons for stopping the oxytocin. dose of medication. This would not allow the
Content Area: Maternity, Intrapartum; Integrated Process: adolescent to more readily focus on difficult content.
Nursing Process, Evaluation; Cognitive Level: Analysis; TEST-TAKING TIP: Once-a-day medication typically means
Client Need/Subneed: Physiological Integrity/Pharmacological morning.
and Parenteral Therapies/Adverse Effects/Contraindications/ Content Area: Pediatrics, Pharmacology; Integrated Process:
Interactions Nursing Process, Planning; Cognitive Level: Application;
Client Need/Subneed: Physiological Integrity/Pharmacological
89. CORRECT ANSWER: 1. Answer 1 is correct because and Parenteral Therapies/Medication Administration
ibuprofen has a longer duration of action (about
6 hours) than acetaminophen (about 4 hours), which 91. CORRECT ANSWER: 150. First, convert milligrams to
is a distinct advantage for nighttime comfort or when micrograms. Second, calculate micrograms per mL, and
a child is resistant to taking medication. Selecting third, calculate the flow rate.
ibuprofen would require giving fewer doses to the child. TEST-TAKING TIP: Be sure to convert the order into the same
Answer 2 is incorrect because acetaminophen has a shorter unit of measure as the available solution.
duration of action (about 4 hours) than ibuprofen (about Content Area: Adult Health, Pharmacology; Integrated
6 hours). This is a distinct disadvantage for nighttime Process: Nursing Process, Analysis; Cognitive Level: Analysis;
comfort or when a child is resistant to taking medication Client Need/Subneed: Physiological Integrity/Pharmacological
because acetaminophen would result in giving more doses and Parenteral Therapies/Dosage Calculation
to the child. Answer 3 is incorrect because, while codeine is 92. CORRECT ANSWER: 2. Answer 1 is incorrect because
recommended as an analgesic in the treatment of otitis magnesium sulfate 4 gm IV bolus over 30 minutes is
media, it is reserved for severe pain rather than the mild an appropriate order for this client for preterm labor.
pain about which the child is complaining. Answer 4 is Answer 2 is correct because the correct dosage and route
incorrect because, while Dilaudid is used in the pediatric for Brethine is 0.25 mg SQ. Both the dosage and route
population, it is reserved for the severe pain associated are wrong for this drug. Answer 3 is incorrect because
with cancer or major surgeries. Stadol 1 mg IV push every 4 hours as needed for pain is
TEST-TAKING TIP: duration = doses. an appropriate order. Answer 4 is incorrect because
Content Area: Pediatrics, Pharmacology; Integrated Process:
Ancef 2 gm intravenous piggyback (IVPB) every 6 hours
Nursing Process, Implementation; Cognitive Level: Analysis; is an appropriate order.
Client Need/Subneed: Physiological Integrity/Pharmacological
TEST-TAKING TIP: This question is asking for the order that
and Parenteral Therapies/Expected Effects/Outcomes is wrong or inappropriate for this client. Look for the order
90. CORRECT ANSWER: 1. Answer 1 is correct because that is incorrect.
the adolescents schedule should be arranged so that Content Area: Maternity, Intrapartum; Integrated Process:
ANSWERS

academic subjects (e.g., math, science, etc.) are taught in Nursing Process, Analysis; Cognitive Level: Application;
the morning when the adolescent is experiencing the Client Need/Subneed: Physiological Integrity/Pharmacological
effect of the morning dose of medication. This will allow and Parenteral Therapies
2164_Ch08_603-688 29/03/12 12:34 PM Page 687

chapter 8 Physiological Integrity 687


93. CORRECT ANSWER: 1. Answer 1 is correct because TEST-TAKING TIP: Since Rho(D) immune globulin is admin-
Rho(D) immune globulin needs to be administered before istered in the hospital, eliminate the time frames that would be
72 hours after delivery to be effective. Answer 2 is incor- longer than the average hospital stay for delivery. Is it hours or
rect because 1 week is too long after delivery for adequate weeks? 72 hours is a reasonable length of time.
prophylaxis. Answer 3 is incorrect because Rho(D) immune Content Area: Maternity, Postpartum; Integrated Process:
globulin does not need to be administered by 12 hours of Nursing Process, Implementation; Cognitive Level: Application;
age. Adequate prophylaxis is obtained if administration is Client Need/Subneed: Physiological Integrity/Pharmacological
within 72 hours after delivery. Answer 4 is incorrect because and Parenteral Therapies/Medication Administration
2 weeks is too long after delivery for adequate prophylaxis.

ANSWERS
2164_Ch08_603-688 29/03/12 12:34 PM Page 688
2164_Ch09_689-710 29/03/12 12:35 PM Page 689

CHAPTER 9

Physiological Integrity:
Basic Care and ComfortNutrition

Sally Lambert Lagerquist Janice Lloyd McMillin Robyn Marchal Nelson


Denise Wall Parilo Kathleen E. Snider

689
2164_Ch09_689-710 29/03/12 12:35 PM Page 690

690 chapter 9 Physiological Integrity


NUTRITION

P O P U L AT I O N - B. Lactationfour to five servings.


SPECIFIC C. Count as one serving1 cup milk; 1/2 cup undilut-
NUTRITIONAL NEEDS ed evaporated milk; 1/4 cup dry milk; 11/4 cups
cottage cheese; 2 cups low-fat cottage cheese;
Nutrition During Pregnancy 11/2 oz cheddar or Swiss cheese; or 11/2 cups
ice cream.
and Lactation
II. MEAT, POULTRY, FISH, DRY BEANS, NUTS,
Table 9.1 gives a summary of nutrient needs (maternal AND EGGS GROUPimportant for protein, iron,
and fetal) during pregnancy. and many B vitamins.
I. MILK GROUPimportant for calcium, protein A. Pregnancythree servings.
of high biological value, and other vitamins and B. Lactationthree servings.
minerals. C. Count as one serving1/2 cup cooked dry beans,
A. Pregnancythree to four servings (four to five for 1 egg, or 11/2 tbsp peanut butter is equivalent to
adolescents). 1 oz meat; use peanut butter or nuts rarely to avoid

Table 9.1
Nutrient Needs During Pregnancy
Nutrient Maternal Need Fetal Need Food Source
Protein75 gm/day Maternal tissue growth: uterus, Rapid fetal growth Milk and milk products; animal
breasts, blood volume, storage meatsmuscle, organs; grains,
legumes; eggs
Calories2,500/day Increased basal metabolic Primary energy source for Carbohydrates: 4 kcal/gm
rate (BMR) growth of fetus Proteins: 4 kcal/gm
Fats: 9 kcal/gm
Minerals1,200 mg/ Increase in maternal Skeleton and tooth formation Milk and milk products,
day (calcium and Ca++ metabolism especially natural cheese*
phosphorus)
Iron30 mg/day Increase in red blood cell (RBC) Liver storage (especially in third Organ meatsliver, animal meat;
mass trimester) egg yolk; whole or enriched
grains; green leafy vegetables;
Prevent anemia
nuts
Decrease infection risk
Vitamins
A Tissue growth Cell developmenttissue and Butter, cream, fortified margarine;
bone growth and tooth bud green and yellow vegetables
formation
B Coenzyme in many metabolic Coenzyme in many metabolic Animal meats, organ meats; milk
processes processes and cheese; beans, peas, nuts;
enriched grains
C Tissue formation and integrity Tissue formation and integrity Citrus fruits, berries, melons,
papaya, kiwi, strawberries; pep-
Increase iron absorption
pers; green, leafy vegetables;
broccoli, brussels sprouts, snow
pea pods; potatoes

D Absorption of Ca++, phosphorus Mineralization of bone tissue Fortified milk and margarine
and tooth buds
E Tissue growth; cell wall integrity; Tissue growth; cell integrity; Widely distributed: meat, milk,
RBC integrity RBC integrity eggs, grains, leafy vegetables,
vegetable oils
Folic acid Meet increased metabolic Meet increased metabolic Liver; deep-green, leafy vegeta-
demands in pregnancy demands, including production bles; asparagus; avocado
of cell nucleus material
Production of blood products
Prevent neural tube defects
*Natural cheese contains less lactose; therefore, it is a good source for those with lactose intolerance.
Tofu (soybean cake) is also high in calcium and contain no lactose.
2164_Ch09_689-710 29/03/12 12:35 PM Page 691

Religious Considerations in Meal Planning 691


excessive fat intake; limit eggs to reduce cholesterol Nutritional Needs of the Geriatric

NUTRITION
intake; trim fat from meat, and remove skin from
poultry. Client
III. VEGETABLE AND FRUIT GROUPvitamins I. CALORIES1,500 to 2,000 kcal/day to maintain
and minerals (especially vitamins A and C) and ideal weight; 12% of calories from protein sources;
roughage. 50% to 60% of calories from carbohydrates; 20% to
A. Vegetables: 30% of calories from fats.
1. Pregnancythree to four servings. II. HIGH FIBERprevent or alleviate constipation and
2. Lactationthree to five servings. dependence on laxatives.
3. Count as one serving1 cup raw leafy greens,
1
/2 cup of others. III. SODIUM3 to 4 gm/day according to cardiac and
B. Fruits: renal status.
1. Pregnancytwo to four servings. IV. FATSlimit to help retard the development of
2. Lactationtwo to four servings. cancer, atherosclerosis, obesity, and other diseases.
3. Count as one serving1/2 medium grapefruit; V. FLUIDS6 to 8 glasses/day.
1 medium apple, banana, or orange; 3/4 cup
fruit juice. VI. COMMON DEFICIENCIES: calories; calcium;
C. Good sources of vitamin Ccitrus, cantaloupe, folic acid; iron; thiamine; vitamins A, B12, C, and D;
mango, papaya, strawberries, broccoli, and green niacin; zinc.
and red bell peppers. VII. FACTORS CONTRIBUTING TO FOOD
D. Fair sources of vitamin Ctomatoes, honeydew PREFERENCES:
melon, asparagus tips, raw cabbage, collards, kale, A. Physical ability to prepare, shop for, and eat food.
mustard greens, potatoes (white and sweet), B. Income.
spinach, and turnip greens. C. Availability of food if dependent on others.
E. Good sources of vitamin Adark-green or deep- D. Food intolerances.
yellow vegetables and a few fruits (apricots,
broccoli, pumpkin, sweet potato, spinach, VIII. COMMON EATING PROBLEMS: Table 9.2
cantaloupe, carrots, and winter squash). provides interventions for common eating problems of
F. Good sources of folic aciddark-green foliage-type the geriatric client.
vegetables.
IV. BREAD AND CEREAL GROUPgood for
C U LT U R A L F O O D
thiamine, iron, niacin, and other vitamins and
P AT T E R N S
minerals. Table 9.3 gives a summary of the cultural food patterns of
A. Pregnancy6 to 11 servings. various ethnic groups, and Table 9.4 details the hot ver-
B. Lactation6 to 11 servings. sus cold theory of disease treatment.
C. Count as one serving1 slice bread, 1 oz ready-to-
eat cereal, 1/2 to 3/4 cup cooked cereal, cornmeal,
grits, macaroni, noodles, rice, or spaghetti. RELIGIOUS
C O N S I D E R AT I O N S
V. NOTE: use dark-green leafy and deep-yellow vegeta- IN MEAL
bles often; eat dry beans and peas often; count 1/2 cup PLANNING
cooked dry beans or peas as a serving of vegetables or
1 oz from meat group. I. ORTHODOX JEWS:
A. Kosher meat and poultry.
B. No shellfish, eels, or pork products.
Nutritional Needs of the Newborn C. Milk and dairy products cannot be consumed with
I. CALORIES108 kcal/kg/day. meat or poultry; requires separate utensils.
D. No eggs with a blood spot may be eaten.
II. PROTEIN2.2 gm/kg/day (1 gm protein = 1 oz milk). Eggs may be used with either meat or dairy
III. FLUIDS3.5 oz/kg/day. meals.
IV. VITAMIN D400 IU daily for infants who are II. CONSERVATIVE AND REFORM JEWS: dietary
bottle-fed after week 2. practices may vary from religious laws.
V. FLUORIDE0.25 mg daily up to 3 years old when III. MUSLIMS: no pork or alcohol.
local water supply has less than 0.3 ppm content. IV. HINDUS: vegetarianism (cows are sacred).
2164_Ch09_689-710 29/03/12 12:35 PM Page 692

692 chapter 9 Physiological Integrity


NUTRITION

Table 9.2
Eating Problems of the Geriatric Client and Ways to Improve Nutrition
Problem Cause Dietary Interventions
Within the mouth Impaired taste buds Referral for correction of dental problems
Dental caries Mouthwash/oral care before meals
Chewing difficulty due to: Food prepared to meet clients needs (chopped,
Poorly fitting dentures pureed, soft)
No dentures
No saliva
Paralysis
Metallic taste from medications
In the upper GI tract Swallowing difficulty Thickened and jellied liquids
Paralysis due to stroke Soft, chopped foods
Food causing GI distress Small, frequent meals
Presentation important (how food looks on serving
plate)
Avoid expression such as baby food (negative
connotation)
In the lower GI tract Constipation Add fruits/liquids to restore bowel function (see
Diarrhea Table 11.12, p. 844)
Bloating Avoid foods that cause diarrhea
Plan meals at appropriate times for ease of
digestion
Avoid salt
Psychosocial Loneliness Encourage attendance at meal program if available
Meals no longer social event Arrange group meals/activities when possible
Depression Emphasize best nutrition when appetite is at peak
Anorexia (breakfast)
Allow time to complete meal
Interact with others whenever possible
In the environment Inability to shop for, prepare, or cook Refer to Meals-on-Wheels
food Encourage family involvement; describe how food
Impaired vision is arranged on plate
Lack of resources Refer to social services
Difficulty in feeding self Assess need for assistive devices
Acute or chronic illnesses present Assess ability to grasp utensils and guide food to
(e.g., arthritis, stroke) mouth
Open packages and milk cartons; butter the
bread; cut meat and vegetables
From Thomas, CL (ed): Tabers Cyclopedic Medical Dictionary, ed 18. FA Davis, Philadelphia, 1997.

Table 9.3
Cultural Food Patterns
Ethnic Group Cultural Food Patterns Dietary Excesses or Omissions
Mexican (native) Basic sources of proteindried beans, Limited meats, milk, and milk products
flan, cheese, many meats, fish, eggs Some are using flour tortillas more than the more
Chili peppers and many deep-green and nutritious corn tortillas
yellow vegetables Excessive use of lard (manteca), sugar
Fruits include: zapote, guava, papaya, Tendency to boil vegetables for long periods of
mango, citrus time
Tortillas (corn, flour); sweet bread; fideo;
tacos, burritos, enchiladas
2164_Ch09_689-710 29/03/12 12:35 PM Page 693

Cultural Food Patterns 693

Table 9.3

CULTURAL
Cultural Food Patternscontd
Ethnic Group Cultural Food Patterns Dietary Excesses or Omissions
Filipino (Spanish-Chinese Most meats, eggs, nuts, legumes May limit meat, milk, and milk products (the latter
influence) Many different kinds of vegetables may be due to lactose intolerance)
Large amounts of rice and cereals Tend to prewash rice
Tend to fry many foods
Chinese (mostly Cantonese) Cheese, soybean curd (tofu), many meats, Tendency among some immigrants to use excess
chicken and pigeon eggs, nuts, legumes grease in cooking.
Many different vegetables, leaves, bam- May be low in protein, milk, and milk products
boo sprouts (the latter may be due to lactose intolerance)
Rice and rice-flour products; wheat, corn, Often wash rice before cooking
millet seed; green tea Large amounts of soy and oyster sauces, both of
Mixtures of fish, pork, and chicken with which are high in salt; monosodium glutamate
vegetablesbamboo shoots, broccoli, (MSG)
cabbage, onions, mushrooms, pea pods
Puerto Rican Milk with coffee Use large amounts of lard for cooking
Pork, poultry, eggs, dried fish; beans Limited use of milk and milk products
(habichuelas) Limited amounts of pork and poultry
Viandas (starchy vegetables; starchy ripe
fruits)
Avocados, okra, eggplant, sweet yams
Rice, cornmeal
African American Milk with coffee Limited use of milk group (lactose intolerance)
Pork, poultry, eggs Extensive use of frying, smothering, simmering
Fruit: strawberries, watermelon for cooking
Vegetables: turnip, collard, mustard Large amounts of fat: salt pork, bacon drippings,
greens; kale, okra, sweet potatoes lard, gravies
Cereals (including grits, hominy, corn- May have limited use of citrus and enriched
bread, hot breads) breads
Molasses (dark molasses is especially
good source of calcium, iron, vitamins B1
and B2, and niacin)
Middle Eastern (Greek, Yogurt Tend to use excessive sweeteners, lamb fat,
Syrian, Armenian) Predominantly lamb, nuts, dried peas, olive oil
beans, lentils Tend to fry meats and vegetables
Deep-green leaves and vegetables; dried Insufficient milk and milk products (almost no
fruits butteruse olive oil, which has no nutritive value
Dark breads and cracked wheat except for calories)
Deficiency in fresh fruits
Middle European (Polish) Many milk products Tend to use excessive sweets and to overcook
Pork, chicken vegetables
Root vegetables (potatoes), cabbage, fruits Limited amounts of fruits (citrus), raw vegetables,
Wheat products and meats
Sausages, smoked and cured meats, noo-
dles, dumplings, bread, cream with coffee
Native American If Americanized, use milk and milk Nutrition-related problems: obesity, diabetes, den-
(American Indianmuch products tal problems, iron-deficiency anemia; alcoholism
variation) Variety of meats: game, fowl, fish; nuts, Limited quantities of high-protein foods depending
seeds, legumes on availability (flocks of game, fowl) and economic
Variety of vegetables, some wild situation
Variety of fruits, some wild, rose hips; roots Excessive use of sugar
Variety of breads, including tortillas, corn-
meal, rice
Italian Staples are pasta with sauces; bread; eggs; Limited use of whole grains
cheese; tomatoes; and vegetables such as Insufficient servings from milk group
artichokes, eggplant, greens, and zucchini Tendency to overcook vegetables
Only small amount of meat is used Enjoy sweets
2164_Ch09_689-710 29/03/12 12:35 PM Page 694

694 chapter 9 Physiological Integrity


SPECIAL DIETS

Table 9.4
Hot-Cold Theory of Disease Treatment*
Hot Diseases or Cold Diseases or Hot Medicines Cold Medicines
Conditions Conditions Hot Foods Cold Foods and Herbs and Herbs
Constipation Cancer Beverages, Barley water Anise Bicarbonate of
Diarrhea Common cold aromatic Cod Aspirin soda
Infections Earache Cereal grains Dairy products Castor oil Linden
Kidney diseases Headache Cheese Fruits, tropical Cinnamon Milk of magnesia
Liver complaints Joint pain Chili peppers Honey Cod liver oil Orange flower
Skin eruptions Malaria Chocolate Meats (goat, fish, Garlic water
Throat (sore) Menstrual period Coffee chicken) Ginger root Sage
Ulcers Paralysis Eggs Milk, bottled Iron preparations
Warts Pneumonia Fruits, temperate Raisins Mint
Rheumatism zone Vegetables, fresh Penicillin
Stomach cramps Goat milk Tobacco
Teething Hard liquor Vitamins
Tuberculosis Meats (beef,
water fowl,
mutton)
Oils
Onions
Peas
*A Latin American, particularly Puerto Rican, approach to treating diseases. A hot disease is treated with cold
treatments (foods, medicines) and vice versa.
Adapted from Wilson, HS, & Kneisl, CR: Psychiatric Nursing, ed 2. Addison-Wesley, Menlo Park, CA. (out of print)

V. SEVENTH-DAY ADVENTISTS: SPECIAL DIETS


A. Vegetarianism is common (lacto-ovo).
B. No shellfish or pork products. I. LOW-CARBOHYDRATE DIETketogenic: low
C. Avoid stimulants (coffee, tea, other caffeine sources). carbohydrate, high fat; dumping syndrome: low carbo-
D. No alcohol. hydrate, high fat, high protein.
VI. MORMONS (LATTER-DAY SAINTS [LDS]): no II. GLUTEN-FREE DIETelimination of all foods
coffee, tea, or alcohol. made from oats, barley, wheat, and rye; may have corn
and rice. Used for celiac disease.
VII. CATHOLICS: some still adhere to meatless Fridays
and fasting during Lent. III. HIGH-PROTEIN DIETlean meat, cheese, and
green vegetables.
A. Nephrotic syndrome (may also be on
C O M M O N V I TA M I N S low-sodium diet).
AND NUTRIENTS B. Acute leukemia (combined with high-calorie and
A N D R E L AT E D soft food diets).
DEFICIENCIES C. Neoplastic disease.
Table 9.5 gives a summary of the physiological functions IV. LOW-PROTEIN DIET:
of common vitamins and the results of deficiencies. A. Usually accompanied by high-carbohydrate diet
Table 9.6 gives a similar summary of essential nutrients. and normal fats and calories.
B. Renal failure, uremia, anuria, acute
MEDICAL glomerulonephritis.
CONDITIONS V. LOW-SODIUM DIET:
W I T H D I E TA R Y A. Heart failure.
MANAGEMENT B. Nephrotic syndrome.
The dietary management of various medical conditions is C. Acute glomerulonephritis (varies with degree of
detailed in Table 9.7. oliguria).
(text continues on page 699)
2164_Ch09_689-710 29/03/12 12:35 PM Page 695

Physiological Functions of Common Vitamins 695

Table 9.5

VITAMINS
Physiological Functions of Common Vitamins and Related Deficiencies
Vitamin Chief Functions Results of Deficiency Characteristics Good Sources
Vitamin A
Retinol (animal Essential for maintain- Mild: Fat soluble* Liver
source) ing the integrity of Retarded growth Not destroyed by ordinary Animal fats:
Carotene epithelial membranes Increased susceptibility cooking temperatures Butter
Beta-carotene Helps maintain resist- to infection Is destroyed by high Cheese
ance to infections Abnormal function temperatures when Cream
Necessary for the for- of gastrointestinal, oxygen is present Egg yolk
mation of rhodopsin genitourinary, and Marked capacity for Whole milk
and prevention of night respiratory tracts due storage in the liver Fish liver oil
blindness to altered epithelial NOTE: Excessive intake Liver
Necessary for proper membranes of carotene, from which Vegetables:
bone growth Skin dries, shrivels, vitamin A is formed, Green leafy, especially
Facilitates RNA thickens; sometimes may produce yellow escarole, kale,
formation from DNA pustule formation discoloration of the skin parsley
Thought to be cancer Night blindness (carotenemia); excessive Yellow, especially
preventive because of Severe: vitamin A intake causes carrots
antioxidant properties Xerophthalmia, a symptoms similar to Artificial:
associated with control characteristic eye those of deficiency Concentrates in
of free radical damage disease, and other conditions several forms
to DNA and cell local infections Irradiated fish oils
membranes
Vitamin B6
Pyridoxine Used in hemoglobin Anemias Soluble in water and Meats
Pyridoxal synthesis Depressed immunity alcohol Cereal grains
Pyridoxamine Essential for metabo- Dermatitis around eyes Rapidly inactivated in Some fruits
lism of tryptophan to and mouth presence of heat, sunlight, Nuts
niacin Neuritis or air
Needed for utilization Anorexia, nausea, and
of certain other amino vomiting
acids
Vitamin B12
Cyanocobalamin Necessary for myelin Pernicious anemia Soluble in water or Found only in animal
Hydroxycobalamin synthesis Neurological disorders alcohol products (e.g., meats,
Essential for normal Unstable in hot alkaline or eggs, dairy products)
development of red acid solutions Most of vitamin B12
blood cells required by humans
Associated with folate is synthesized by
metabolism intestinal bacteria;
can also be recycled
Vitamin C
Ascorbic acid Essential to formation Mild: Soluble in water Abundant in most
of intracellular cement Lowered resistance to Easily destroyed by fresh fruits and
substances in a variety infections oxidation; heat hastens vegetables, especially
of tissues, including Joint tenderness the process citrus fruit and juices,
skin, dentin, cartilage, Susceptibility to dental Lost in cooking, particu- tomato and orange
and bone matrix caries, pyorrhea, and larly if water in which food Artificial:
Important in healing of bleeding gums was cooked is discarded; Ascorbic acid
wounds and bone Delayed wound healing loss is greater if cooked in Cevitamic acid
fractures Bruising iron or copper utensils
Prevents scurvy Severe: Quick-frozen foods lose
Facilitates absorption Hemorrhage little of their vitamin C
of iron Anemia Stored in the body to a
Protects folate Scurvy limited extent
Antioxidant NOTE: Many drugs
Promotes capillary affect availability
permeability
Continued
2164_Ch09_689-710 29/03/12 12:35 PM Page 696

696 chapter 9 Physiological Integrity

Table 9.5
VITAMINS

Physiological Functions of Common Vitamins and Related Deficienciescontd


Vitamin Chief Functions Results of Deficiency Characteristics Good Sources
Vitamin D
Calciferol Hormone-like regulation Mild: Soluble in fats and organic Formed in the skin by
Ergocalciferol of calcium and phospho- Interferes with utilization solvents* exposure to sunlight
Cholecalciferol rus metabolism by of calcium and phospho- Fortified milk and
Relatively stable under
Calcitriol promotion of: rus in bone and tooth dairy products
refrigeration
Antirachitic factor Gastrointestinal formation Egg yolk
absorption Stored in liver Fish liver oils
Irritability
Fish having fat distrib-
Bone and tooth Often associated with
Weakness uted through the flesh,
mineralization vitamin A
Severe: salmon, tuna, herring
Renal reabsorption Rickets in young sardines
children Liver
Skeletal reserves
Oysters
Osteomalacia in adults
Antirachitic Artifically prepared
forms
Vitamin E
Alpha-tocopherol Important antioxidant Red blood cell Soluble in fat* Vegetable oils
Beta-tocopherol that: resistance to rupture
Stable to heat in absence of Margarine
Gamma-tocopherol Prevents red blood cell is decreased, but
oxygen and ultraviolet light
hemolysis deficiency seldom Whole-grain or fortified
Protects vitamin A and occurs except in Unstable under freezing and cereals
unsaturated fatty acids premature infants processing
Wheat germ
from oxidation and people with chronic
Promotes cell fat malabsorption Green leafy vegetables
membrane integrity
Improves immune
response
May protect against
cancer
Folate
Folacin Essential for normal Anemia Slightly soluble in water Liver
Folic acid functioning of Eggs
NOTE: Neural tube Easily destroyed by heat in
hematopoietic system Fish
defects (e.g., spina bifi- presence of acid
Important coenzyme Green leafy vegetables
da) are associated with
for RNA and DNA Decreases when food is Asparagus
maternal deficiency;
synthesis stored at room temperature
Peas, beans, legumes
Important in fetal alcohol and contracep-
NOTE: A large dose may
development tives interfere with Nuts, seeds, wheat
prevent appearance of ane-
absorption germ
Functions interrelated mia in a case of pernicious
with those of vitamin anemia but still permit neu- Avocado
B12 rological symptoms to Some fruits
develop
Vitamin K
Menadione Promotes synthesis of Prolonged clotting time, Soluble in fat* Green leafy vegetables
Phylloquinone clotting factors resulting in bleeding Stable to heat Meats
Menaquinone Dairy products
NOTE: Seldom occurs
Intestinal bacteria
in the absence of
anticoagulant drugs;
in addition to food
sources, intestinal
bacteria manufacture
approximately half the
bodys requirement
Fat malabsorption can
cause deficiency
Synthetic form (mena-
dione) does not depend
on fat absorption
2164_Ch09_689-710 29/03/12 12:35 PM Page 697

Physiological Functions of Common Vitamins 697

Table 9.5

VITAMINS
Physiological Functions of Common Vitamins and Related Deficienciescontd
Vitamin Chief Functions Results of Deficiency Characteristics Good Sources
Niacin
Nicotinic acid As the component of Pellagra Soluble in hot water and Milk
Nicotinamide two important enzymes, alcohol Eggs
Gastrointestinal distur-
Antipellagra it is important in glycoly- Meats
bances Not destroyed by heat,
vitamin sis, tissue respiration, fat Legumes
light, air, or alkali
synthesis, and cellular Mental disturbances Enriched foods
energy production Not destroyed in ordinary Whole-grain cereals
NOTE: Associated with
cooking Nuts
Nicotinic acid, but not alcoholism
NOTE: Also formed in
nicotinamide, causes
the body from dietary
vasodilation and flushing
tryptophan (amino acid)
Prevents pellagra
Riboflavin
Vitamin B2 Important as coenzyme Impaired growth Water soluble Milk and milk products
in cellular oxidation Lassitude and weakness
Alcohol soluble Enriched foods
Cheilosis
Essential to normal
Glossitis Not destroyed by heat in Whole-grain breads and
growth
Dermatitis cooking unless with alkali cereals
Participates in light Anemia
Unstable in light, especially Liver
adaptation Photophobia
in presence of alkali Meats
Cataracts
Vital to protein Eggs
metabolism
Associated with niacin
and vitamin B6 functions
Thiamine
Vitamin B1 Important role in carbo- Mild: Soluble in water Widely distributed in
hydrate metabolism Loss of appetite plant and animal
Not readily destroyed
tissues but seldom
Essential for mainte- Impaired metabolism of by ordinary cooking
occurs in high concen-
nance of normal appetite starches and sugars temperatures
tration, except in
Essential for normal Emaciation Destroyed by exposure to brewers yeast
functioning of nervous heat, alkali, or sulfites
Irritability Enriched or whole-grain
tissue
Severe: Not stored in body cereals
Coenzyme for cellular Various nervous NOTE: Deficiency is often
Pork
energy production disorders associated with alcoholism
Peas, beans
Loss of coordinating Nuts
power of muscles
Artificial:
Beriberi Concentrates from
yeast
Paralysis in humans
Rice polishings
Wheat germ
*Vitamins A, D, E, and K are available in water-soluble forms and are used in children with cystic fibrosis and celiac disease.
Adapted from Venes, D (ed): Tabers Cyclopedic Medical Dictionary, ed 20. FA Davis, Philadelphia, 2005.

Table 9.6
Essential Nutrients and Potential Deficiencies
Nutrient Function Deficiency Leads to
Calcium Aids in formation and maintenance of Neuromuscular irritability
bones and teeth; permits healthy nerve
Impaired blood clotting
functioning and normal blood clotting
Phosphorus Bone building Rickets

Continued
2164_Ch09_689-710 29/03/12 12:35 PM Page 698

698 chapter 9 Physiological Integrity


NUTRITION

Table 9.6
Essential Nutrients and Potential Deficienciescontd
Nutrient Function Deficiency Leads to
Magnesium Cellular metabolism of carbohydrates and Cellular metabolism of carbohydrates and protein
protein Tetany
Sodium Fluid and electrolyte balance; acid-base balance; Fluid and electrolyte imbalance
electrochemical impulses of nerves and muscles Muscle contraction
Potassium Osmotic pressure and water balance Fluid and electrolyte imbalance
Cardiac and skeletal muscular contractility
Chloride Fluid and electrolyte balance; acid-base balance Fluid imbalances; alkalosis
Digestion
Iron Hemoglobin formation Anemia; risk of infection
Cellular oxidation
Iodine Synthesis of thyroid hormone; overall body metabolism Goiter
Zinc Constituent of cell enzyme system; CO2 carrier in Metabolism of protein and carbohydrates
RBCs Delayed wound healing; risk of infection
Vitamin A Collagen synthesis Poor healing; scaly skin
Vitamin C Capillary integrity Poor healing; bruising
Vitamin K Coagulation Bruising and hemorrhage
Pyridoxine and Antibody, RBC, and WBC formation Risk of infection
thiamine Anemia
Protein Wound repair; clotting Poor healing; edema
WBC production
Phagocytosis
Fats Cellular energy Impaired tissue repair
Cell membrane integrity
Carbohydrates Cellular energy; spare protein Interference with healing

Table 9.7
Medical Conditions with Dietary Management
Condition Recommended Diet
Celiac sprue Avoid glutens (wheat, buckwheat, rye, oats, barley)
Cholelithiasis Avoid fatty food
Cirrhosis Sodium; limit protein
Diverticulosis Low residue
Esophagitis Thick liquids; avoid alcohol
Gastroesophageal reflux Avoid: late meals, chocolate, caffeine, mints
disease (GERD)
Gout Alcohol, purine (organ meats, anchovies, sardines, consomm, gravies, fish roes, herring); fluid
Hyperhomocysteinemia Vitamin B12; folates
Hypertension DASH (Dietary Approaches to Stop Hypertension) diet: saturated fats and cholesterol; total
fats and dairy fats; fruits, vegetables, whole grains, fish, poultry, and nuts
Iron-deficiency anemia Vitamin C with iron supplements
Irritable bowel syndrome (IBS) Fiber, dairy products
Nephrotic syndrome Sodium
Osteoporosis Alcohol; supplement calcium and vitamin D
Pernicious anemia Vitamin B12 supplements
Renal failure Sodium, potassium, protein, fluids
2164_Ch09_689-710 29/03/12 12:35 PM Page 699

Common Therapeutic Diets 699

THERAPEUTIC DIETS
VI. HIGH-PHOSPHORUS DIET: C. Foods allowed: very tender minced, ground, baked,
A. Use when serum phosphorus level is less than broiled, roasted, stewed, or creamed beef, lamb,
2.7 mg/dL due to: veal, liver, poultry, or fish; crisp bacon or sweet-
1. Insufficient intake (e.g., malnutrition, starvation). breads; cooked vegetables; pasta; all fruit juices;
2. Increased phosphorus excretion due to: soft raw fruits; soft breads and cereals; all desserts
a. Renal failure. that are soft; and cheeses.
b. Hyperparathyroidism. D. Foods avoided: coarse whole-grain cereals and
c. Malignancy. breads; nuts; raisins; coconut; fruits with small
d. Antacids that are aluminum hydroxidebased seeds; fried foods; high-fat gravies or sauces; spicy
or magnesium-based. salad dressings; pickled meat, fish, or poultry;
B. Sources of phosphorus: beef, chicken, fish, organ strong cheeses; brown or wild rice; raw vegetables,
meats, pork, nuts, legumes, whole-grain breads and as well as lima beans and corn; spices such as
cereals, milk, egg, cheese, ice cream, carbonated horseradish, mustard, and catsup; and popcorn.
beverages. IV. SODIUM-RESTRICTED DIET
A. Purpose: reduce sodium content in the tissues and
COMMON promote excretion of water.
THERAPEUTIC B. Use: heart failure, hypertension, renal disease, cir-
DIETS rhosis, toxemia of pregnancy, and cortisone therapy.
C. Modifications: mildly restrictive 2-gm sodium diet
I. CLEAR LIQUID DIET to extremely restricted 200-mg sodium diet.
A. Purpose: relieve thirst and help maintain fluid D. Foods avoided: table salt; all commercial soups,
balance. including bouillon; gravy, catsup, mustard, meat
B. Use: postsurgically and following acute vomiting sauces, and soy sauce; buttermilk, ice cream, and
or diarrhea. sherbet; sodas; beet greens, carrots, celery, chard,
C. Foods allowed: carbonated beverages; coffee sauerkraut, and spinach; all canned vegetables;
(caffeinated and decaffeinated); tea; fruit-flavored frozen peas; all baked products containing salt,
drinks; strained fruit juices; clear, flavored gelatins; baking powder, or baking soda; potato chips and
broth, consomm; sugar; popsicles; commercially popcorn; fresh or canned shellfish; all cheeses;
prepared clear liquids; and hard candy. smoked or commercially prepared meats; salted
D. Foods avoided: milk and milk products, fruit juices butter or margarine; bacon; olives; and commer-
with pulp, and fruit. cially prepared salad dressings.
II. FULL LIQUID DIET V. RENAL DIET
A. Purpose: provide an adequately nutritious diet A. Purpose: control protein, potassium, sodium, and
for clients who cannot chew or who are too ill fluid levels in body.
to do so. B. Use: acute and chronic renal failure, hemodialysis.
B. Use: acute infection with fever, gastrointestinal C. Foods allowed: high-biological proteins such as
upsets, after surgery as a progression from clear meat, fowl, fish, cheese, and dairy productsrange
liquids. between 20 and 60 mg/day. Potassium is usually
C. Foods allowed: clear liquids, milk drinks, cooked limited to 1,500 mg/day. Vegetables such as cab-
cereals, custards, ice cream, sherbets, eggnog, all bage, cucumber, and peas are lowest in potassium.
strained fruit juices, vegetable juices, creamed Sodium is restricted to 500 mg/day. Fluid intake is
vegetable soups, puddings, mashed potatoes, restricted to the daily urine volume plus 500 mL,
instant breakfast drinks, yogurt, mild cheese which represents insensible water loss. Fluid intake
sauce or pureed meat, and seasonings. measures water in fruit, vegetables, milk, and meat.
D. Foods avoided: nuts, seeds, coconut, fruit, jam, D. Foods avoided: cereals, bread, macaroni, noodles,
and marmalade. spaghetti, avocados, kidney beans, potato chips,
III. SOFT DIET raw fruit, yams, soybeans, nuts, gingerbread, apri-
A. Purpose: provide adequate nutrition for those who cots, bananas, figs, grapefruit, oranges, percolated
have trouble chewing. coffee, Coca-Cola, Orange Crush, sport drinks,
B. Use: clients with no teeth or ill-fitting dentures; and breakfast drinks such as Tang or Awake.
transition from full liquid to general diet; and for VI. HIGH-PROTEIN, HIGH-CARBOHYDRATE
those who cannot tolerate highly seasoned, fried, DIET
or raw foods following acute infections or gastroin- A. Purpose: corrects large protein losses and raises the
testinal disturbances, such as gastric ulcer or level of blood albumin. May be modified to include
cholelithiasis. low-fat, low-sodium, and low-cholesterol diets.
2164_Ch09_689-710 29/03/12 12:35 PM Page 700

700 chapter 9 Physiological Integrity


THERAPEUTIC DIETS

B. Use: burns, hepatitis, cirrhosis, pregnancy, hyper- D. Foods avoided: remember to avoid the 5 Cs
thyroidism, mononucleosis, protein deficiency due of cholesterolcookies, cream, cake, coconut,
to poor eating habits, geriatric clients with poor chocolate; whole milk and whole-milk or cream
food intake, nephritis, nephrosis, and liver and products, avocados, olives, commercially prepared
gallbladder disorders. baked goods such as donuts and muffins, poultry
C. Foods allowed: general diet with added protein. In skin, highly marbled meats, shellfish, fish canned in
adults, high-protein diets usually contain 135 to oil, nuts, coconut, commercially prepared meats,
150 gm protein. butter, ordinary margarines, olive oil, lard, pudding
D. Foods avoided: restrictions depend on modifications made with whole milk, ice cream, candies with
added to the diet. These modifications are deter- chocolate, cream, sauces, gravies, and commercially
mined by the clients condition. fried foods.
VII. PURINE-RESTRICTED DIET X. DIABETIC DIET
A. Purpose: designed to reduce intake of uric A. Purpose: maintain blood glucose as near normal
acidproducing foods. as possible; prevent or delay onset of diabetic
B. Use: high uric acid retention, uric acid renal stones, complications.
and gout. B. Use: diabetes mellitus.
C. Foods allowed: general diet plus 2 to 3 quarts of C. Foods allowed: choose foods with low glycemic index;
liquid daily. composed of 45% to 55% carbohydrates, 30% to
D. Foods avoided: cheese containing spices or nuts, 35% fats, and 10% to 25% protein. Foods are divid-
fried eggs, meat, liver, seafood, lentils, dried peas ed into groups from which exchanges can be made.
and beans, broth, bouillon, gravies, oatmeal Coffee, tea, broth, bouillon, spices, and flavorings
and whole wheats, pasta, noodles, and alcoholic can be used as desired. Exchange groups include:
beverages. Limited quantities of meat, fish, and milk, vegetables, fruit, starch/bread (includes starchy
seafood allowed. vegetables), meat (divided into lean, medium fat,
VIII. BLAND DIET and high fat), and fat exchanges. The number of
A. Purpose: provision of a diet low in fiber, roughage, exchanges allowed from each group is dependent on
mechanical irritants, and chemical stimulants. the total number of calories allowed. Nonnutritive
B. Use: gastritis, hyperchlorhydria, functional GI dis- sweeteners (aspartame) if desired. Nutritive sweeten-
orders, gastric atony, diarrhea, spastic constipation, ers (sorbitol) in moderation for those who have
biliary indigestion, and hiatus hernia. controlled diabetes and have normal weight.
C. Foods allowed: varied to meet individual needs and D. Foods avoided: concentrated sweets or regular soft
food tolerances. drinks.
D. Foods avoided: fried foods, including eggs, meat, XI. ACID-ASH AND ALKALI-ASH DIETS
fish, and seafood; cheese with added nuts or spices;
A. Purpose: furnish a well-balanced diet in which the
commercially prepared luncheon meats; cured
total acid ash is greater than the total alkali ash
meats such as ham; gravies and sauces; raw vegeta-
each day.
bles; potato skins; fruit juices with pulp; figs;
B. Use: retard the formation of renal calculi. The type
raisins; fresh fruits; whole wheats; rye bread; bran
of diet chosen depends on laboratory analysis of
cereals; rich pastries; pies; chocolate; jams with
the stones.
seeds; nuts; seasoned dressings; caffeinated coffee;
C. Acid-ash and alkali-ash food groups:
strong tea; cocoa; alcoholic and carbonated
1. Acid ash: meat, whole grains, eggs, cheese,
beverages; and pepper.
cranberries, prunes, plums.
IX. LOW-FAT, CHOLESTEROL-RESTRICTED 2. Alkali ash: milk, vegetables, fruit (except
DIET cranberries, prunes, and plums).
A. Purpose: reduce hyperlipidemia, provide dietary 3. Neutral: sugars, fats, beverages (coffee and tea).
treatment for malabsorption syndromes and clients D. Foods allowed: all the client wants of the following:
having acute intolerance for fats. 1. Breads: any, preferably whole grain; crackers; rolls.
B. Use: hyperlipidemia, atherosclerosis, pancreatitis, 2. Cereals: any, preferably whole grain.
cystic fibrosis, sprue, gastrectomy, massive resection 3. Desserts: angel food or sunshine cake; cookies
of the small intestine, and cholecystitis. made without baking powder or soda; cornstarch
C. Foods allowed: nonfat milk; low-carbohydrate, low- pudding, cranberry desserts, custards, gelatin
fat vegetables; most fruits; breads; pastas; cornmeal; desserts, ice cream, sherbet, plum or prune
lean meats; unsaturated fats such as corn oil; desserts; rice or tapioca pudding.
desserts made without whole milk; and unsweet- 4. Fats: any, such as butter, margarine, salad dress-
ened carbonated beverages. ings, Crisco, Spry, lard, salad oils, olive oil, etc.
2164_Ch09_689-710 29/03/12 12:35 PM Page 701

Anticancer Nutrients and Nonnutritive Compounds 701

ANTICANCER
5. Fruits: cranberries, plums, prunes. juice (except prune); cooked or canned apples, apri-
6. Meat, eggs, cheese: any meat, fish, or fowl, cots, peaches, pears; ripe bananas; strained vegetable
two servings daily; at least one egg daily. juice; canned, cooked, or strained asparagus, beets,
7. Potato substitutes: corn, hominy, lentils, green beans, pumpkin, acorn squash, spinach; white
macaroni, noodles, rice, spaghetti, vermicelli. bread; refined cereals (Cream of Wheat).
8. Soup: broth as desired; other soups from foods XIV. LACTOSE-FREE DIET
allowed.
A. Purpose: decrease symptoms that occur after having
9. Sweets: cranberry or plum jelly; sugar; plain
milk products: diarrhea, cramps, abdominal pain,
sugar candy.
increased flatus.
10. Miscellaneous: cream sauce, gravy, peanut
B. Use: in lactose intolerance, where there is an inability
butter, peanuts, popcorn, salt, spices, vinegar,
to tolerate lactose because of absence or deficiency of
walnuts.
lactase, an enzyme found in the secretions of the
E. Restricted foods: no more than the amount allowed
small intestine required for digestion of lactose.
each day.
C. Foods avoided: milk products.
1. Milk: 1 pint daily (may be used in other ways
D. Foods allowed: soy-based milk foods; hard cheese,
than as beverage).
cottage cheese, yogurt (contains inactive lactose
2. Cream: 1/3 cup or less daily.
enzyme).
3. Fruits: one serving of fruit daily (in addition to
the prunes, plums, and cranberries); certain
fruits listed under Foods avoided (following) are ANTICANCER
not allowed at any time. NUTRIENTS AND
4. Vegetables, including potatoes: two servings NONNUTRITIVE
daily; certain vegetables listed under Foods COMPOUNDS
avoided (following) are not allowed at any time.
I. USE: To enhance immune function, promote wellness
F. Foods avoided:
as an anti-inflammatory agent; to reduce risk of
1. Carbonated beverages, such as ginger ale, cola,
chronic diseases, including cancer.
root beer.
2. Cakes or cookies made with baking powder or II. SOURCE FOODS:
soda. A. Berries.
3. Fruits: dried apricots, bananas, dates, figs, 1. Rich in: vitamin C, fiber, potassium, phyto-
raisins, rhubarb. chemicals (flavonoids).
4. Vegetables: dried beans, beet greens, dandelion 2. Effect:
greens, carrots, chard, lima beans. a. Antioxidants: protect against cell damage.
5. Sweets; chocolate or candies other than those b. Anti-inflammatory.
listed under Foods allowed (preceding); syrups. c. Antiulcerative.
6. Miscellaneous: other nuts, olives, pickles. d. Antiviral.
XII. HIGH-FIBER DIET e. May help to inhibit tumor formation in: liver,
colon, esophageal, and oral cancer.
A. Purpose: soften stool; exercise digestive tract mus- 3. Servings: daily.
cles; speed passage of food through digestive tract B. Citrus fruits.
to prevent exposure to cancer-causing agents in 1. Rich in: vitamin C, folic acid, phytochemicals
food; lower blood lipids; prevent sharp rise in (e.g., beta-carotene, limonoids, monoterpenes,
blood glucose after eating. phenols).
B. Use: diabetes, hyperlipidemia, constipation, 2. Effect:
diverticulosis, anticarcinogenic (colon). a. Inhibit activation of cancer cells; detoxify can-
C. Foods allowed: recommended intake about 6 gm cer promoters.
crude fiber daily: all bran cereals; watermelon, b. Aid protective enzymes.
prunes, dried peaches, apple with skin; parsnips, c. Stimulate cancer-killing immune cells.
peas, brussel sprouts; sunflower seeds. d. May reduce risk of: breast, skin, colon cancers.
XIII. LOW-RESIDUE (LOW-FIBER) DIET 3. Servings: daily.
A. Purpose: reduce stool bulk and slow transit time. C. Cruciferous vegetable family.
B. Use: bowel inflammation during acute diverticulitis 1. Rich in: antioxidants (e.g., beta-carotene and
or ulcerative colitis, preparation for bowel surgery, vitamin C); also isothiocyanates; including
esophageal and intestinal stenosis. sulforaphane.
C. Foods allowed: eggs; ground or well-cooked tender 2. Group includes: broccoli, kale, cauliflower,
meat, fish, poultry; milk; mild cheeses; strained fruit cabbage, bok choy, brussels sprouts.
2164_Ch09_689-710 29/03/12 12:35 PM Page 702

702 chapter 9 Physiological Integrity


ANTICANCER

3. Effect: 2. Turmeric and cumin: rich in curcumin that


a. Isothiocyanates: interfere with tumor growth. may reduce risk of leukemia, skin, and liver
b. Neutralize cancer-causing chemicals. cancers.
c. Stimulate enzymes (e.g., glutathione 3. Chili peppers: rich in capsaicin that helps to pre-
S-transferase) that inactivate carcinogens. vent formation of nitrosamines; may reduce risk
d. May help reduce risk of hormone-related of colon, gastric and rectal cancers.
cancers (e.g., breast, prostate, and thyroid). I. Legumes/beans.
e. May help reduce risk of: lung, GI, oral, 1. Rich in: low-fat source of protein and dietary
pharyngeal, and esophageal cancers. fiber, iron, folic acid, calcium, zinc, phytochemi-
4. Servings: 3 servings/week. cals (e.g., phytosterols, saponins, phytic acid,
D. Fatty fish (salmon, trout, herring, bluefish, isoflavones).
sardines). 2. Effect: prevent DNA damage provide
1. Rich in: omega-3 fatty acids, iron, B vitamins, anticancer activity.
selenium, vitamin D. 3. Servings: 3 to 4/week.
2. Effect: J. Nuts.
a. Inhibit growth of cancer cells. 1. Rich in: dietary fiber, vitamin E, healthy fats.
b. Stimulate the immune system. a. Brazil nuts: high amount of selenium.
c. Help prevent or reduce muscle wasting. b. Walnuts: high in omega-3 fatty acids.
d. May reduce the risk of cancers of: breast, 2. Effect: encourage suicide (apoptosis) of cancer
prostate, endometrium, colon. cells.
3. Servings: twice/week or more often. 3. Servings: 2 Brazil nuts daily; 2 tbsp of walnuts
E. Flaxseed. daily.
1. Rich in: omega-3 fatty acids, lignans, fiber, K. Olive oil (extra virgin).
protein, calcium, potassium, vitamin B, iron. 1. Rich in: monounsaturated fats, polyphenols.
2. Effect: 2. Effect:
a. Block tumor growth. a. Inhibit oxidative stress.
b. Inhibit angiogenesis. b. Arrest cell proliferation.
c. Enhance immune system. c. Induce cell death (apoptosis).
d. Anti-inflammatory. 3. May reduce risk of: breast, prostate, and colorec-
3. Caution: Not to be used by clients with tal cancers.
hormone-sensitive breast cancer and/or if L. Vegetables/fruitsorange-colored (carrots, sweet
using tamoxifen or other antiestrogenic drugs. potatoes, winter squash, mangoes, papayas).
F. Garlic and onions. 1. Rich in: vitamin C, folic acid, other B vitamins,
1. Effect: fiber and beta-carotene.
a. Increase detoxification of enzymes. 2. Effect: carotenoids strengthen immune system.
b. Stimulate cancer-fighting immune cells. 3. May reduce risk of: lung, colorectal, uterine,
c. May decrease risk of: prostate, breast, gastric, prostate, and breast cancers.
and colon cancer. 4. Servings: daily servings of dark-orange and
d. May also have antiasthma and cardioprotec- green foods.
tive effects. M. Soy foods.
2. Servings: use in any recipe. 1. Rich in: protein, fiber, calcium, variety of
G. Green tea. vitamins and minerals; phytoestrogens
1. Rich in: polyphenols (antioxidants that prevent (e.g., isoflavones, lignans).
DNA damage). 2. Dietary sources: soybeans (edamame), tofu,
2. Effect: tempeh, miso, soy nuts, soy milk.
a. Catechins (a type of polyphenol) may help to 3. Caution: Those with breast cancer are advised
rid body of carcinogens. to limit soy consumption to 3 to 4 servings/
b. May neutralize cell-damaging free radicals. week.
c. Stimulates immune response. 4. May reduce risk of cancers: breast, prostate,
d. May suppress the growth of cancer cells. thyroid, and colon.
e. May reduce the risk of: colorectal, prostate, 5. Servings: 1 to 2 daily soy servings (10 to
breast, bladder, gastric, esophageal, liver, lung, 30 gm soy protein and 30 to 90 mg
skin, and head and neck cancers. isoflavones). Avoid excesses of more than
3. Servings: 2 to 4 cups daily. 200 mg isoflavones daily.
H. Herbs and spices. N. Tomatoes.
1. Rosemary: rich in carnosol and ursolic acid with 1. Rich in: lycopene, vitamin C, beta-carotene,
strong antioxidant activity. potassium.
2164_Ch09_689-710 29/03/12 12:35 PM Page 703

Food List for Menu Planning 703


2. Effect: IV. FOODS HIGH IN B VITAMINS

FOOD LIST
a. Antioxidants that scavenge free radicals. A. Thiamine: pork, dried beans, dried peas, liver,
b. Reduce tissue damage. lamb, veal, nuts, peas.
c. Protect cell membranes. B. Riboflavin: liver, poultry, milk, yogurt, whole-grain
d. Prevent formation of nitrosamines. cereals, beef, oysters, tongue, fish, cottage cheese, veal.
3. Have been shown to inhibit growth of: prostate, C. Niacin: liver, fish, poultry, peanut butter, whole
colon, bladder, cervical, stomach, esophageal, grains and enriched breads, lamb, veal, beef, pork.
and lung cancers.
V. FOODS HIGH IN VITAMIN C: oranges, strawber-
4. Servings: 4 or more daily.
ries, dark-green leafy vegetables, potatoes, grapefruit,
O. Whole grains.
tomato, cabbage, broccoli, melon, liver.
1. Rich in: fiber, vitamins, trace minerals, antioxi-
dants, plant sterols, phytoestrogens, phytases, VI. FOODS HIGH IN IRON, CALCIUM,
tocotrienols, lignans, ellagic acid, saponins. AND RESIDUE
2. Dietary sources: oats, barley, mullet, brown rice, A. Iron: breads (brown, corn, ginger); fish (tuna);
bulgur. poultry; organ meats; whole-grain cereals; shellfish;
3. Effect: may reduce risk of various cancers, as well as egg yolk; fruits (apples, berries); dried fruits (dates,
hypertension, heart disease, obesity, and diabetes. prunes, apricots, peaches, raisins); vegetables (dark-
4. Servings: 3 times daily. green leafy, potatoes, tomatoes, rhubarb, squash);
P. Yogurt with live, active cultures. molasses; dried beans and peas; peanut butter;
1. Rich in: protein, calcium, and probiotics. brown sugar; noodles; rice.
2. Effect: B. Calcium: milk (dry, skim, whole, evaporated, butter-
a. Boost immunity. milk); cheese (American, Swiss, hard); dark green
b. Inhibit cell proliferation. leafy vegetables (kale, turnip greens, mustard greens,
c. Induce cell death. collards); black-eyed peas; tofu; canned fish with
3. May protect against colorectal cancer. bones; figs.
C. Residue: whole-grain cereals (oatmeal, bran, shredded
wheat); breads (whole wheat, cracked wheat, rye,
FOOD LIST FOR bran muffins); vegetables (lettuce, spinach, Swiss
MENU PLANNING chard, raw carrots, raw celery, corn, cauliflower,
I. HIGH-CHOLESTEROL FOODSover 50 mg/ eggplant, sauerkraut, cabbage); fruits (bananas, figs,
100-gm portion: beef, butter, cheese, egg yolks, shell- apricots, oranges).
fish, kidney, liver, pork, veal. VII. FOODS TO BE USED IN LOW-PROTEIN
II. HIGH-SODIUM FOODSover 500 mg/100-gm AND LOW-CARBOHYDRATE DIETS
portion: bacon (cured, Canadian); beef (corned, A. Low protein (these proteins are allowed in various
cooked, canned, dried, creamed); biscuits, baking amounts in controlled-protein diets for renal
powder; bouillon cubes; bran, added sugar and malt; decompensation): milk (buttermilk, reconstituted
bran flakes with thiamine; breads (wheat, French, evaporated, low sodium, skim, powdered); meat
rye, white, whole wheat); butter, cheese (cheddar, (chicken, lamb, turkey, beef [lean], veal); fish (sole,
Parmesan, Swiss, pasteurized American); cocoa; flounder, haddock, perch); cheese (cheddar,
cookies, gingersnaps; cornflakes; cornbread; crackers American, Swiss, cottage); eggs; fruits (apples,
(graham, saltines); margarine; milk (dry, skim); grapes, pears, pineapple); vegetables (cabbage,
mustard; oat products; olives (green, ripe); peanut cucumbers, lettuce, tomatoes); cereals (cornflakes,
butter; pickles, dill; popcorn with oil and salt; puffed rice, puffed wheat, farina, rolled oats).
raisins; salad dressing (blue cheese, Roquefort, B. Low carbohydrate: all meats; cheese (hard, soft,
French, Thousand Island); sausages (bologna, cottage); eggs; shellfish (oysters, shrimp); fats
frankfurters); soy sauce; tomato catsup; tuna in oil. (bacon, butter, French dressing, salad oil, mayon-
III. HIGH-POTASSIUM FOODSmore than naise, margarine); vegetables (asparagus, green beans,
400 mg/100-gm portion: almonds; bacon, Canadian; beet greens, broccoli, brussels sprouts, cabbage,
baking powder, low sodium; beans (white, lima); beef, celery, cauliflower, cucumber, lettuce, green pepper,
hamburger; bran with sugar and malt; cake (fruitcake, spinach, squash, tomatoes); fruits (avocados, straw-
gingerbread); cashew nuts; chicken, light meat; cocoa; berries, cantaloupe, lemons, rhubarb).
coffee, instant; cookies, gingersnaps; dates; fruits, fruit VIII. DIETARY GUIDEMyPlate has replaced the Food
juices; garlic; milk (skim, powdered); peanuts, roasted; Pyramid as a guide to daily food selection (Fig. 9.1)
peanut butter; peas; pecans; potatoes, boiled in skin; A. Balance calories:
scallops; tea, instant; tomato puree; turkey, light meat; 1. Enjoy food, but eat less.
veal; walnuts, black; yeast, brewers. 2. Avoid oversized portions.
2164_Ch09_689-710 29/03/12 12:35 PM Page 704

704 chapter 9 Physiological Integrity


QUESTIONS

4. A nurse should determine that a client understands the


dietary restrictions with a low-cholesterol diet if the
client selects a larger portion of:
1. Cottage cheese.
2. Cheese.
3. Beef.
4. Brown rice.
5. A nurse instructs a client to eat a low-iodine diet
before a thyroid study. A client should be told to
avoid:
1. Shellfish.
2. Coffee and chocolate.
3. Dark-green leafy vegetables.
4. Tap water unless it is run through a filter.
6. A client would demonstrate awareness of dietary
influences in the prevention of dumping syndrome if
the intake were adjusted by:
Figure 9.1 MyPlateguide to daily food choices. Select all that apply.
(www.choosemyplate.gov.) 1. Increasing fats as tolerated.
2. Eating more dry items than liquid.
B. Increase the following foods: 3. Consuming fluids before or after mealtimes.
1. Half the plate should be fruits and vegetables. 4. Avoiding concentrated carbohydrates.
2. Grains should be whole grains (e.g., whole 5. Increasing simple carbohydrates.
wheat, oatmeal, brown rice). 7. In preparation for surgery, a client is placed on a low-
3. Switch to fat-free or low-fat milk. residue diet. In discussing the kinds of foods that will
C. Reduce the following foods: be allowed, a nurse should list:
1. Sodiumread the label. 1. Ground lean beef, soft-boiled eggs.
2. Sodadrink water instead of sugary drinks. 2. Lettuce, spinach, corn.
3. Prunes, grapes, apples.
4. Bran cereal, whole-wheat toast, milk.
Questions 8. In Mnires disease, the diet is modified as a part of
Select the one answer that is best for each question, unless the treatment plan. Which diet is most appropriate?
otherwise directed. 1. Low sodium.
1. A client is discharged on a 2-gm sodium-restricted diet. 2. High protein.
Which menu choice would indicate the client understands 3. Low carbohydrate.
the restriction? 4. Low fat.
1. Chow mein, egg roll, hot tea. 9. The results of a clients complete blood count are:
2. Onion soup, unsalted crackers, 1 oz cheddar cheese. RBC 3.2 million/mm3, Hgb 10 gm/dL, Hct 29%, and
3. Corned beef sandwich, dill pickle, ginger ale. mean corpuscular hemoglobin (MCH) 25 pg/cell.
4. Turkey breast Divan, rice pilaf, iced tea. Which foods should the nurse recommend to the
2. A client with Parkinsons disease will likely have the client? Select all that apply.
greatest difficulty swallowing: 1. Peas.
1. Apple juice. 2. Cauliflower.
2. Cottage cheese. 3. Milk.
3. Milk shake. 4. Eggs.
4. Ground beef. 5. Raisins.

3. Which statement by a client with hypokalemia indicates 10. A client is admitted with a diagnosis of brain attack
a need for further teaching by a nurse? (stroke). A physician orders a diet as tolerated. Before
1. I will eat more bananas and cantaloupes. feeding the client, a nurse should first:
2. I will eat more bran. 1. Wait for a specific diet order.
3. I will take my K-Lor on a full stomach. 2. Elevate the head of the bed 30 degrees.
4. I will call my provider for weakness, confusion, 3. Evaluate the clients swallow reflex.
abdominal cramping, or anxiety. 4. Delegate feeding to the nursing assistant.
2164_Ch09_689-710 29/03/12 12:35 PM Page 705

Questions 705

QUESTIONS
11. A client is admitted to a hospital for pneumonia stroke and heart attack with a higher than recom-
and it is discovered that the client also has a stage II mended Na+ intake?
pressure ulcer. When planning a diet for this client, 1. A woman.
what foods should a nurse encourage the client 2. A client who is overweight.
to eat? 3. A man.
1. Fruits and vegetables. 4. A client who is Hispanic.
2. Whole grains. 18. Which food would be restricted in a low-sodium diet?
3. Milk, yogurt, and cheese. 1. Whole milk.
4. Lean meats and low-fat milk. 2. Ginger ale.
12. A 16-year-old client, who is gravida 1, para 0, is 3. Orange juice.
attending a class on pregnancy and nutrition. Which 4. Black coffee.
menu selection should a nurse recommend that 19. A client is attending an informational session for in-
would best meet the nutritional needs of the client vitro fertilization. The client asks a nurse, What foods
who is pregnant? can I eat to increase my intake of folic acid? The nurse
1. Hamburger patty, green beans, French fries, suggests that the best source of folic acid is:
and iced tea. 1. Green vegetables and liver.
2. Roast beef sandwich, potato chips, baked beans, 2. Yellow vegetables and red meat.
and cola. 3. Carrots and onions.
3. Baked chicken, fruit cup, potato salad, coleslaw, 4. Milk and eggs.
yogurt, and herbal tea.
4. Fish sandwich, gelatin with fruit, and coffee. 20. A 40-year-old client at 9 weeks gestation presents to
an obstetric clinic complaining of nausea and vomiting
13. A client, who is 23 years old, gravida 3, para 1, has a every morning when she awakens. Which intervention
hemoglobin of 9 gm/dL. She has been prescribed an should a nurse recommend for this client?
exogenous iron supplement, but tells a nurse that she 1. Eat a handful of peanuts before getting out of bed.
cannot take pills. Which food should the nurse recom- 2. Increase her intake of high-fat foods.
mend that is high in iron? 3. Drink a carbonated beverage before bedtime.
1. Dried apricots. 4. Eat dry, unsalted crackers before getting out of bed.
2. Lima beans.
3. Tofu.
21. A nurse prepares to give maternal breast milk to an
infant via a bottle. Which steps should the nurse take
4. Milk.
when preparing and administering breast milk? Select
14. A neonatal nurse is caring for a newborn who is breast- all that apply and place in priority order.
feeding. How many kilocalories are present in an ounce 1. Review the diet orders.
of maternal breast milk? 2. Compare the infants identification band against the
1. 10 kcal/ounce. label on the milk.
2. 20 kcal/ounce. 3. Discard thawed milk after 1 hour.
3. 30 kcal/ounce. 4. Thaw frozen milk slowly in the microwave.
4. 40 kcal/ounce. 5. Put on gloves.
15. A child is diagnosed with cystic fibrosis. Which food(s) 22. A client, who has not been eating or drinking, is
should the child avoid? Select all that apply. admitted to a hospital. Which sign of dehydration
1. Low-fat yogurt. should a nurse expect?
2. Nuts. 1. Tachycardia.
3. Ham. 2. Hypertension.
4. Hot dogs. 3. Shortness of breath.
5. Sugar-free Jello. 4. Crackles in the lungs.
6. Low-salt crackers. 23. A client, who is 24 years old and has gestational dia-
16. What would be allowed in the diet of a client with betes, is getting instructions about her diet. A nurse
peptic ulcer disease (PUD)? knows that her teaching has been successful when the
1. Milk. client correctly identifies that she should avoid:
2. Caffeinated coffee. Select all that apply.
3. Alcohol. 1. White rice.
4. Tomato juice. 2. Fruit juice.
3. Cold cereal.
17. Sodium intake is recommended to be no more than 4. Hot cereal.
2,400 mg each day. Which client is most at risk for 5. Milk.
2164_Ch09_689-710 29/03/12 12:35 PM Page 706

706 chapter 9 Physiological Integrity

24. A client, who is a 17-year-old gravida 2, para 1, prac- 3. CORRECT ANSWER: 2. Answer 1 is incorrect because
ANSWERS

tices a strict vegetarian diet. She has been prescribed an bananas and cantaloupes are good sources of potassium.
iron supplement during her second trimester of preg- Answer 2 is correct because bran has fiber but little
nancy. To promote better absorption, a nurse should potassium for a client with low potassium. Answer 3 is
instruct the client to take the iron supplement with incorrect because potassium is irritating and is correctly
which beverage? taken with food or plenty of liquids. Answer 4 is incorrect
1. Milk. because these are signs of hypokalemia.
2. Iced tea. TEST-TAKING TIP: The question calls for selecting the wrong
3. Soy milk. statement as the best answer. Review good sources of potassium
4. Orange juice. and signs/symptoms of low potassium.
Content Area: Adult Health, Fluid and Electrolyte Imbalances;
25. A client is attending a class on prenatal nutrition. The Integrated Process: Nursing Process, Evaluation; Cognitive Level:
client asks why vitamin C is important during preg-
Application; Client Need/Subneed: Physiological Integrity/
nancy. A nurses best response would be:
Basic Care and Comfort/Nutrition and Oral Hydration
1. Vitamin C is needed to promote collagen formation.
2. Eating foods high in vitamin C helps metabolize 4. CORRECT ANSWER: 4. Answer 1 is incorrect because cot-
fats and carbohydrates. tage cheese is a source of protein (which comprises only 15%
3. Supplemental vitamin C in large doses can prevent of calories) and contains more cholesterol. Answer 2 is incor-
neural tube defects. rect because cheese, a source of protein, should be limited
4. Studies have shown that vitamin C deficiency leads because of fat and calories. Answer 3 is incorrect because beef,
to pregnancy-induced hypertension. higher in fat than fish or chicken, should be limited. Answer 4
is correct because 50% to 60% of the calories should come
Answers/Rationales/Tips from a carbohydrate source such as brown rice.
TEST-TAKING TIP: Only brown rice is a low-carbohydrate and
1. CORRECT ANSWER: 4. Answer 1 is incorrect because low-fat choice. The other three options are sources of protein.
ethnic foods, such as Chinese, are high in sodium. Content Area: Adult Health, Nutrition; Integrated Process:
Answer 2 is incorrect because the broth used in onion soup, Nursing Process, Evaluation; Cognitive Level: Application;
and cheeses, are both high in sodium. Answer 3 is incorrect Client Need/Subneed: Physiological Integrity/Basic Care
because the brining processes for corned beef, as well as and Comfort/Nutrition and Oral Hydration
the sodium contained in ginger and dill pickle, all make 5. CORRECT ANSWER: 1. Answer 1 is correct because
this a poor choice for the diet. Answer 4 is correct because shellfish are high in iodine. Also, some brands of table
turkey and rice are lowest in sodium. The sauce used salt should be avoided. Answer 2 is incorrect because coffee
on the turkey Divan can be made with a low-sodium and chocolate are not rich in iodine. Answer 3 is incorrect
soup. because dark-green leafy vegetables are not rich in iodine.
TEST-TAKING TIP: Eliminate the choices that are known to Answer 4 is incorrect because water is not rich in iodine.
be high in sodiumethnic foods, canned or processed foods, or TEST-TAKING TIP: Eliminate caffeine and water as sources of
those prepared with large amounts of salt. iodine (Answers 2 and 4). Shellfish are the most obvious source
Content Area: Adult Health, Nutrition; Integrated Process:
of iodine.
Nursing Process, Evaluation; Cognitive Level: Application; Content Area: Adult Health, Endocrine; Integrated Process:
Client Need/Subneed: Physiological Integrity/Basic Care
Nursing Process, Implementation; Cognitive Level:
and Comfort/Nutrition and Oral Hydration Comprehension; Client Need/Subneed: Physiological Integrity/
2. CORRECT ANSWER: 1. Answer 1 is correct because any Basic Care and Comfort/Nutrition and Oral Hydration
thin liquid or food increases the chance of aspiration. 6. CORRECT ANSWERS: 1, 2, 3, 4. Answer 1 is correct
Answer 2 is incorrect because thicker foods will reduce the
because dietary fat does not contribute to dumping
risk of aspiration. Answer 3 is incorrect because thick liquids syndrome like carbohydrates do. Answer 2 is correct
are less likely to be aspirated. Answer 4 is incorrect because because fluid with meals or near mealtime (less than
ground beef is soft, does not required chewing, and the risk 1 hour before or 1 hour after) contributes to dumping
of aspiration is minimized. syndrome. Answer 3 is correct because fluid should not be
TEST-TAKING TIP: Look for the option that is different from consumed with meals. Fluids, 1 hour before or 1 hour
the othersone is a thin consistency, and like saliva, is more after a meal, reduce the potential for dumping syndrome.
likely to cause aspiration. Answer 4 is correct because carbohydrates increase the
Content Area: Adult Health, Neuromuscular; Integrated Process:
chance of dumping syndrome because they speed up
Nursing Process, Analysis; Cognitive Level: Application; Client gastric emptying. Answer 5 is incorrect because the diet
Need/Subneed: Physiological Integrity/Basic Care and Comfort/
should keep carbohydrate intake low.
Nutrition and Oral Hydration
2164_Ch09_689-710 29/03/12 12:35 PM Page 707

Answers/Rationales/Tips 707
TEST-TAKING TIP: Ask how each option will affect gastric count indicates iron-deficiency anemia, and dried fruits

ANSWERS
emptying. Will it speed or slow the dumping of gastric are a rich source of iron.
contents? TEST-TAKING TIP: Choose common food sources of iron.
Content Area: Adult Health, Gastrointestinal; Integrated Content Area: Adult Health, Hematological; Integrated
Process: Nursing Process, Evaluation; Cognitive Level: Analysis; Process: Teaching and Learning; Cognitive Level: Application;
Client Need/Subneed: Physiological Integrity/Basic Care and Client Need/Subneed: Physiological Integrity/Basic Care and
Comfort/Nutrition and Oral Hydration Comfort/Nutrition and Oral Hydration
7. CORRECT ANSWER: 1. Answer 1 is correct because 10. CORRECT ANSWER: 3. Answer 1 is incorrect because
residue is produced by high-fiber foods. Lean beef, the RN can offer the client any preferred foods. Answer 2 is
well cooked, and eggs are not good sources of fiber. incorrect because this would not be the first nursing action.
Low residue reduces the amount of stool in the large Answer 3 is correct because the client may be at risk for
bowel. Answer 2 is incorrect because leafy greens aspiration if the ability to swallow has been affected.
(particularly in the cruciferous family) are high in roughage Answer 4 is incorrect because the RN must assess the client
and fiber. Answer 3 is incorrect because only grapes would and determine ability to swallow before delegating care.
be allowed on a low-residue diet. Generally fruit juices TEST-TAKING TIP: Focus on a step of the nursing process:
(except prune) are permitted. Answer 4 is incorrect because evaluate before any action (wait, elevate, delegate). Think
grains such as bran and whole wheat are high fiber sources. client safety.
Dairy may be permitted. In a low-residue diet, refined, plain Content Area: Adult Health, Cardiovascular; Integrated Process:
foods are permitted. Nursing Process, Planning; Cognitive Level: Application;
TEST-TAKING TIP: Know that high-fiber foods influence stool Client Need/Subneed: Physiological Integrity/Basic Care and
production. The goal before surgery is to reduce the amount of Comfort/Nutrition and Oral Hydration
stool. Eliminate most vegetables (Answer 2), fruits (Answer 3),
11. CORRECT ANSWER: 4. Answer 1 is incorrect because
and bran and whole wheat (Answer 4).
fruits and vegetables are mostly carbohydrates. Answer 2 is
Content Area: Adult Health, Gastrointestinal; Integrated
incorrect because whole grains are not complete proteins.
Process: Nursing Process, Implementation; Cognitive Level:
Answer 3 is incorrect because this choice has a lot of fats as
Comprehension; Client Need/Subneed: Physiological Integrity/
well as carbohydrates. Answer 4 is correct because proteins
Basic Care and Comfort/Nutrition and Oral Hydration
help to repair and build cells.
8. CORRECT ANSWER: 1. Answer 1 is correct because TEST-TAKING TIP: The client is likely in a catabolic state and
fluid retention is thought to aggravate the symptoms of cells are being destroyed. Look for the option with the most
Mnires disease. Foods high in sodium contribute to amount of protein.
fluid retention. Answer 2 is incorrect because foods high Content Area: Adult Health, Integumentary; Integrated Process:
in protein do not contribute to fluid retention, which is the Nursing Process, Implementation; Cognitive Level: Application;
concern. Answer 3 is incorrect because reducing the carbohy- Client Need/Subneed: Physiological Integrity/Basic Care and
drate content of the diet will not decrease fluid retention, Comfort/Nutrition and Oral Hydration
which is the concern. Answer 4 is incorrect because reducing
12. CORRECT ANSWER: 3. Answer 1 is incorrect because
the fat intake in the diet will not decrease fluid retention,
the hamburger patty and French fries are choices that are
which is the concern.
high in fat and have fewer nutrients. Caffeine-containing
TEST-TAKING TIP: Consider which option could have a direct
beverages such as iced tea should be avoided in pregnancy.
effect on the earsodium would increase fluid retention.
Answer 2 is incorrect because this option has no fresh
Content Area: Adult Health, Sensory; Integrated Process:
vegetable or fruit choices, and cola with caffeine should be
Nursing Process, Planning; Cognitive Level: Application;
avoided in pregnancy. Answer 3 is correct because this
Client Need/Subneed: Physiological Integrity/Basic Care and
menu has a variety of lean protein, carbohydrates, fruits,
Comfort/Nutrition and Oral Hydration
vegetables, and dairy. While the choices within the
9. CORRECT ANSWERS: 1, 4, 5. Answer 1 is correct categories are not ideal, it is the most balanced of the
because the clients red blood cell (RBC), hemoglobin options. Answer 4 is incorrect because this option has no
(Hgb), and hematocrit (Hct) levels are low and indicate fresh vegetable or fruit choices, and coffee with caffeine
iron-deficiency anemia, and legumes are a good source should be avoided in pregnancy.
of iron. Answer 2 is incorrect because the green vegetables TEST-TAKING TIP: Three of the options have beverages with
(particularly leafy) are iron-rich food sources. Answer 3 is caffeine, which should be avoided in pregnancy.
incorrect because milk is not an iron-rich food source. Content Area: Maternity, Antepartum; Integrated Process:
Answer 4 is correct because the laboratory results reflect Nursing Process, Implementation; Cognitive Level: Analysis;
iron-deficiency anemia, and animal protein is an excel- Client Need/Subneed: Physiological Integrity/Basic Care and
lent source of iron. Answer 5 is correct because the RBC Comfort/Nutrition and Oral Hydration
2164_Ch09_689-710 29/03/12 12:35 PM Page 708

708 chapter 9 Physiological Integrity

13. CORRECT ANSWER: 1. Answer 1 is correct because PUD diet. Answer 2 is incorrect because coffee is known to
ANSWERS

dried apricots are a good source of iron, require no cook- increase acid production and is restricted in a PUD diet.
ing, and are palatable for a young mother. Answer 2 is Answer 3 is incorrect because alcohol is known to increase
incorrect because, even though lima beans are a good source acid production and is restricted in a PUD diet. Answer 4 is
of iron, they require preparation and are not usually a correct because, although tomato juice may be acidic, it
primary choice for a younger client. Answer 3 is incorrect does not increase acid production. The guideline for the
because, even though tofu is a good source of iron, it diet with PUD is to avoid foods that increase symptoms,
requires preparation and is not usually a primary choice for and these vary from client to client.
a younger client. Answer 4 is incorrect because milk is not a TEST-TAKING TIP: The key concept is avoiding food/fluid
good source of iron; it is a good source of calcium. You also that increases acid production.
need to consider if the client has lactose intolerance. Content Area: Adult Health, Gastrointestinal; Integrated
TEST-TAKING TIP: The key to this question is the clients age. Process: Nursing Process, Planning; Cognitive Level:
Iron-rich foods can be less palatable. Even if a food is a good Application; Client Need/Subneed: Physiological Integrity/
source of iron, the young adult may not eat it or take time to Basic Care and Comfort/Nutrition and Oral Hydration
prepare anything. Choose the best food for this young busy 17. CORRECT ANSWER: 2. Answer 1 is incorrect because,
mother. based on current research, the only population having an
Content Area: Maternity, Antepartum; Integrated Process:
association between salt intake and hypertension are clients
Nursing Process, Implementation; Cognitive Level: Application; who are obese. Answer 2 is correct because high sodium
Client Need/Subneed: Physiological Integrity/Basic Care and
intake increases the risk of cardiovascular disease and
Comfort/Nutrition and Oral Hydration death in clients who are obese. Answer 3 is incorrect
14. CORRECT ANSWER: 2. Answer 1 is incorrect because it is because, based on current research, the only population having
too few kilocalories. Answer 2 is correct because maternal an association between salt intake and hypertension are clients
breast milk contains 20 kilocalories per ounce. Answer 3 is who are obese. Answer 4 is incorrect because, based on current
incorrect because it is too many kilocalories. Answer 4 is incor- research, the only population having an association between
rect because it is too many kilocalories. salt intake and hypertension are clients who are obese.
TEST-TAKING TIP: Recall that maternal breast milk contains TEST-TAKING TIP: Eliminate gender and ethnic factors.
the same number of kilocalories per ounce as regular infant Choose the client who will be most affected by fluid retention
formula. associated with an increase in sodium intake.
Content Area: Child Health, Newborn; Integrated Process: Content Area: Adult Health, Cardiovascular; Integrated Process:
Nursing Process, Analysis; Cognitive Level: Comprehension; Nursing Process, Analysis; Cognitive Level: Comprehension;
Client Need/Subneed: Physiological Integrity/Basic Care and Client Need/Subneed: Physiological Integrity/Basic Care and
Comfort/Nutrition and Oral Hydration Comfort/Nutrition and Oral Hydration
15. CORRECT ANSWERS: 1, 5, 6. Answer 1 is correct because 18. CORRECT ANSWER: 1. Answer 1 is correct because
the child with cystic fibrosis should avoid low-fat foods and whole milk has 120 mg of sodium per 8 oz. Answer 2 is
eat foods that are high in calories instead, such as full-fat incorrect because 8 oz of ginger ale has only 6 mg of sodium.
yogurt. Answer 2 is incorrect because children with cystic fibro- Answer 3 is incorrect because the sodium content of orange juice
sis should eat foods high in calories, such as nuts. Answer 3 is is so low it is not even listed in the nutrition books. Answer 4 is
incorrect because children with cystic fibrosis should eat meats, incorrect because 8 oz of coffee has only 1 mg of sodium.
such as ham. Answer 4 is incorrect because children with cystic TEST-TAKING TIP: Recall that dairy products are high in
fibrosis should eat foods high in calories, such as hot dogs. sodium. Review therapeutic diets for restricted foods.
Answer 5 is correct because children with cystic fibrosis Content Area: Adult Health, Cardiovascular; Integrated Process:
should eat foods high in calories, such as puddings. Answer 6 Nursing Process, Analysis; Cognitive Level: Application; Client
is correct because children with cystic fibrosis should eat Need/Subneed: Physiological Integrity/Basic Care and
foods that contain salt, as they tend to lose salt. Comfort/Nutrition and Oral Hydration
TEST-TAKING TIP: Note the key word in the stem: avoid. 19. CORRECT ANSWER: 1. Answer 1 is correct because green
These children need high-calorie diets to maintain body weight vegetables and liver are rich sources of folic acid, an essen-
because children with cystic fibrosis are likely to have failure to tial nutrient to prevent neural tube defects in the develop-
thrive and require nighttime gastrostomy feedings. ing fetus. Answer 2 is incorrect because green vegetables are the
Content Area: Child Health, Respiratory; Integrated Process:
best source of folic acid. Yellow vegetables are a source of folic
Nursing Process, Analysis; Cognitive Level: Application; acid, but red meat is not a good source of folic acid. Answer 3
Client Need/Subneed: Physiological Integrity/Basic Care and
is incorrect because carrots are a good source of vitamin A, but
Comfort/Nutrition and Oral Hydration not folic acid, and onions are not a particularly rich source of
16. CORRECT ANSWER: 4. Answer 1 is incorrect because milk folic acid. Answer 4 is incorrect because milk and eggs are not
is known to increase acid production and is restricted in a the best source of folic acid.
2164_Ch09_689-710 29/03/12 12:35 PM Page 709

Answers/Rationales/Tips 709
TEST-TAKING TIP: Folic acid is highest in leafy green vegeta- Client Need/Subneed: Physiological Integrity/Basic Care and

ANSWERS
bles. Even though many foods have some folic acid, the option Comfort/Nutrition and Oral Hydration
with the green vegetables is the best choice.
22. CORRECT ANSWER: 1. Answer 1 is correct because the
Content Area: Maternity, Antepartum; Integrated Process:
body is trying to compensate for the fluid loss by stimulat-
Teaching and Learning; Cognitive Level: Application; Client
ing the heart rate to increase cardiac output and perfusion.
Need/Subneed: Physiological Integrity/Basic Care and
Answer 2 is incorrect because hypotension would be more com-
Comfort/Nutrition and Oral Hydration
mon with dehydration or hypovolemia. Answer 3 is incorrect
20. CORRECT ANSWER: 4. Answer 1 is incorrect because because shortness of breath would be more likely with volume
eating peanuts before getting out of bed avoids having an overload. Answer 4 is incorrect because crackles in the lungs are
empty stomach, but the peanuts are high in fat and slowly a sign of increased fluid in the lungs, not dehydration.
digested, which can add to the hyperemesis symptoms. TEST-TAKING TIP: Look for the option that would be the
Answer 2 is incorrect because a client with hyperemesis compensatory response to the hypovolemia resulting from dehy-
should eat foods that are easy to digest, such as: toast, dration. Three of the options are seen with hypervolemia or vol-
crackers, bagels, pretzels, cereal, rice, pasta, and potatoes. ume overload, not dehydration.
A high-carbohydrate, low-fat diet is best tolerated by clients Content Area: Adult Health, Fluid and Electrolyte Imbalances;
with hyperemesis. Answer 3 is incorrect because drinking Integrated Process: Nursing Process, Assessment; Cognitive
carbonated beverages can assist in preventing the dehydra- Level: Application; Client Need/Subneed: Physiological Integrity/
tion associated with hyperemesis, but it will not help with Basic Care and Comfort/Nutrition and Oral Hydration
nausea and vomiting that the client experiences upon
23. CORRECT ANSWERS: 1, 2, 3. Answer 1 is correct
waking. She should be drinking throughout the day to avoid
because white rice has a high glycemic index and results in
dehydration. Suggestions include: tea, juices, water and
a rapid increase in the clients blood sugar, and it should
carbonated beverages. Answer 4 is correct because eating a
be avoided. Answer 2 is correct because fruit juice has a
small amount of easily digested carbohydrate can help
high glycemic index and results in a rapid increase in the
decrease nausea and vomiting. To avoid having an empty
clients blood sugar, and it should be avoided. Answer 3 is
stomach, the crackers should be eaten before getting out
correct because cold cereal has a high glycemic index and
of bed.
results in a rapid increase in the clients blood sugar, and
TEST-TAKING TIP: The key to this question is assisting the
it should be avoided. Answer 4 is incorrect because hot
client to manage her symptoms that occur upon waking. Choose
cereal does not have a high glycemic index, does not result in
the option that best treats the clients major concern (nausea
a rapid increase in the clients blood sugar, and can be part
and vomiting).
of the diet in gestational diabetes. Answer 5 is incorrect
Content Area: Maternity, Antepartum; Integrated Process:
because milk does not have a high glycemic index, does not
Nursing Process, Implementation; Cognitive Level: Application;
result in a rapid increase in the clients blood sugar, and can
Client Need/Subneed: Physiological Integrity/Basic Care and
be part of the diet in gestational diabetes.
Comfort/Nutrition and Oral Hydration
TEST-TAKING TIP: Clients with gestational diabetes need to
21. CORRECT ANSWERS: 1, 2, 5, 3. Answer 1 is correct have blood sugar that is well controlled; foods that cause a spike
because the first step is to confirm the diet order. in blood sugar are to be avoided.
Answer 2 is correct because the second step should be to Content Area: Maternity, Antepartum; Integrated Process:
verify that the correct client is receiving the breast milk. Nursing Process, Evaluation; Cognitive Level: Application;
Answer 3 is correct because the final step is to discard Client Need/Subneed: Physiological Integrity/Basic Care and
thawed breast milk after 1 hour. Answer 4 is incorrect Comfort/Nutrition and Oral Hydration
because breast milk should never be warmed in the
24. CORRECT ANSWER: 4. Answer 1 is incorrect because
microwave; this destroys the immune properties and creates
cows milk contains little iron and can reduce iron absorption.
hot spots in the bottle, which could burn the infant.
Answer 2 is incorrect because iced tea contains tannins, which
Answer 5 is correct because the third step is to put on
are known to reduce the absorption of iron. Answer 3 is incor-
gloves, since breast milk is a body fluid and may be a
rect because studies have shown soy to have an inhibitory effect
route of exposure to infectious organisms. Universal
on iron absorption. Answer 4 is correct because orange juice
precautions (applying protective barriers such as gloves,
is high in vitamin C, which can increase the absorption of
gowns, and mask) should be applied when handling
iron and is needed for the production of hemoglobin.
blood or body fluids.
TEST-TAKING TIP: Remember: acidic beverages can assist in
TEST-TAKING TIP: Breast milk is a body fluid that has the
absorption of iron supplements. Milk, antacids, soy, and tan-
potential for transmitting infectious disease. A nurse should take
nins decrease absorption.
similar steps as when administering blood products, including
Content Area: Maternity, Antepartum; Integrated Process:
confirming the five rights, and following standard precautions.
Nursing Process, Implementation; Cognitive Level: Application;
Content Area: Child Health, Nutrition; Integrated Process:
Client Need/Subneed: Physiological Integrity/Basic Care and
Nursing Process, Implementation; Cognitive Level: Application;
Comfort/Nutrition and Oral Hydration
2164_Ch09_689-710 29/03/12 12:35 PM Page 710

710 chapter 9 Physiological Integrity

25. CORRECT ANSWER: 1. Answer 1 is correct because tube defects. Answer 4 is incorrect because there are no
ANSWERS

vitamin C is important in the synthesis of collagen, which studies that have shown that vitamin C deficiency leads to
is the main structural component of the skin as well as pregnancy-induced hypertension.
many other body tissues. Vitamin C also works as a pow- TEST-TAKING TIP: Since the cause of pregnancy-induced
erful antioxidant, aids in the absorption of iron, is critical hypertension has not been found, eliminate the response that
in fighting off infections, helps alleviate allergic reactions, correlates vitamin C deficiency and pregnancy-induced hyper-
and aids in wound healing. Vitamin C helps the fetus tension (Answer 4). Eliminate Answers 2 and 3, as they relate
grow and builds strong bones and teeth. Answer 2 is incor- to vitamin B complex and folic acid.
rect because vitamin C does not assist in metabolizing fats Content Area: Maternity, Antepartum; Integrated Process:
and carbohydrates. Vitamin B complex, especially thiamine, Nursing Process, Implementation; Cognitive Level: Application;
helps with metabolism of fats and carbohydrates. Answer 3 is Client Need/Subneed: Physiological Integrity/Basic Care and
incorrect because folic acid, not vitamin C, prevents neural Comfort/Nutrition and Oral Hydration
2164_Ch10_711-824 29/03/12 1:01 PM Page 711

CHAPTER 10

Psychosocial Integrity
Behavioral/Mental Health Care
Throughout the Life Span

Sally Lambert Lagerquist Janis Ryan Wisherop (Reviewer and


Contributor to Selected Sections)

711
2164_Ch10_711-824 29/03/12 1:01 PM Page 712

712 chapter 10 Psychosocial Integrity

The chief objective of this chapter is to highlight the terms and changing behavior under structured
most commonly observed behavioral and emotional conditions.
problems and disorders in the mental health field. The 1. Structure of the mind: id, ego, superego;
emphasis is on (a) main points for assessment, (b) analysis unconscious, preconscious, conscious.
of data based on underlying basic concepts and general 2. Stages of psychosexual development
principles drawn from a psychodynamic and interper- (Table 10.1).
sonal theoretical framework, and (c) nursing interven- 3. Defense mechanisms (see pp. 772774).
MENTAL HEALTH

tions based on the therapeutic use of self as the corner- II. PSYCHOSOCIAL DEVELOPMENT MODEL
stone of a helping process. Nursing actions are listed in (Erikson, Maslow, Piaget, Duvall)
priority whenever possible. Hence the nursing process
A. Erik EriksonEight Stages of Man (1963)
framework is followed throughout. Note that nursing
1. Psychosocial developmentinterplay of biology
interventions are divided into planning and implementa-
with social factors, encompassing total life
tion (covering long-term and short-term goals and stress-
span, from birth to death, in progressive
ing priority of actions) and health teaching. Evaluation of
developmental tasks.
results is listed separately, although this step of the nurs-
2. Stages of life cyclelife consists of a series of
ing process is circular and relates back to assessment
developmental phases (Table 10.2 and
and goals.
Table 10.3).
The categorization of psychiatric-emotional disorders
a. Universal sequence of biological, social,
can be complex and controversial. For purposes of
psychological events.
clarity and simplicity, an attempt has been made here to
b. Each person experiences a series of normative
capsulize many theoretical principles and component
conflicts and crises and thus needs to accom-
skills of the helping process that these disorders have in
plish specific psychosocial tasks.
common. That the term client is often used in place of
c. Two opposing energies (positive and negative
patient reflects the interpersonal rather than medical
forces) coexist and must be synthesized.
model of psychiatric nursing. The diagnostic categoriza-
d. How each age-specific task is accomplished
tion of disorders (based on a synthesis of the North
influences the developmental progress of
American Nursing Diagnosis Association [NANDA],
the next phase and the ability to deal
Psychiatric Nursing Diagnosis [PND-I], and American
with life.
Nurses Association [ANA] classification system for psy-
B. Abraham MaslowHierarchy of Needs (1962)
chiatric nursing diagnoses) is included here to update
1. Beliefs regarding emotional health based on a
the reader in current terminology in the mental health
comprehensive, multidisciplinary approach to
field.
human problems, involving all aspects of
The underlying organizational framework for this
functioning.
chapter is based on applicable categories of client needs
a. Premise: mental illness cannot be
and subneeds from the official NCLEX-RN Test Plan.
understood without prior knowledge
of mental health.
PSYCHOSOCIAL
b. Focus: positive aspects of human behavior
GROWTH AND
(e.g., contentment, joy, happiness).
DEVELOPMENT (text continues on page 714)
Major Theoretical Models
Table 10.1
I. PSYCHODYNAMIC MODEL (Freud)
A. Assumptions and key ideas: Freuds Stages of Psychosexual Development
1. No human behavior is accidental; each psychic Stage Age Behaviors
event is determined by preceding ones. Oral Birth1 yr Dependency and oral gratification
2. Unconscious mental processes occur with great
Anal 13 yr Creativity, stinginess, cruelty, clean-
frequency and significance. liness, self-control, punctuality
3. Psychoanalysis is used to uncover childhood
Phallic or 36 yr Sexual, aggressive feelings; guilt
trauma, which may involve conflict and oedipal
repressed feelings.
4. Psychoanalytic methods are used: therapeutic Latency 612 yr Reactivation of pregenital impulses;
intellectual and social growth
alliance, transference, regression, dream associa-
tion, catharsis. Genital 1218 yr Displacement of pregenital impuls-
es; learns responsibility for self;
B. Freudshifted from classification of behavior to establishes identity
understanding and explaining in psychological
2164_Ch10_711-824 29/03/12 1:01 PM Page 713

Psychosocial Growth and Development 713

Table 10.2
Eriksons Stages of the Life Cycle
Conflict Areas
Age and Stage of Development Needing Resolution Evaluation: Result of Resolution/Nonresolution
Infancy (birth18 mo) Trust Shows affection, gratification, recognition; trusts self and
others; begins to tolerate frustrations; develops hope. Uses

MENTAL HEALTH
vs. primary caregiver as a base for exploration.
Mistrust Withdrawn, alienated.
Early childhood (18 mo3 yr) Autonomy Cooperative, self-controlled, self-expressive, can delay
vs. gratification; develops will.
Shame and doubt Exaggerated self-restraint; defiance; compulsive; overly
compliant.
Late childhood (35 yr) Initiative Realistic goals; can evaluate self; explorative; imitates adult,
shows imagination; tests reality; anticipates roles; develops
vs. purpose, self-motivation.
Guilt Self-imposed restrictions relative to jealousy, guilt, and denial.
School age (512 yr) Industry Sense of duty; acquires self-confidence from social and school
vs. competencies; persevering in real tasks.
Inferiority School and social dropout; social loner; incompetent.
Adolescence (1218 yr) Identity Has ideological commitments, self-actualizing; sense of self;
experiments with roles; experiences sexual polarizations;
vs. develops fidelity.
Role diffusion Ambivalent, confused, indecisive; may act out (antisocial acts).
Young adulthood (1825 yr) Intimacy, solidarity Makes commitments to love, work relationships, a cause or
vs. creative effort; able to sustain mutual love relationships.
Isolation Superficial, impersonal, biased.
Adulthood (2560 yr) Generativity Productive, creative, procreative, concerned for others;
vs. develops care.
Self-absorption, stagnation Self-indulgent.
Late adulthood (60 yrdeath) Ego integrity Appreciates past, present, and future; self-acceptance of own
contribution to others, of own self-worth, and of changes in
vs. lifestyle and life cycle; can face not being; develops wisdom.
Despair Preoccupied with loss of hope, of purpose; contemptuous,
fears death.
From Lagerquist, S: Nursing Examination Review, ed 4. Addison-Wesley, Redwood City, CA.

Table 10.3
Summary of Theories of Psychosocial Development Throughout the Life Cycle
Freud Piaget Sullivan Erikson
Emphasis on:
Pathology (intrapsychic) Normal children Pathology (interpersonal) Both health and illness
Anxiety No emphasis on ego, anxiety, Anxiety
identity, libido
Unconscious, uncontrollable drives Cognitive development Unconscious, uncontrollable Problems are manageable
drives and can be solved
Ego needing defense Tasks can be accomplished Self-system needing defense Need to integrate individual
through learning process and society
Pathological Development Influenced by:
Early feelings; repressed Individual differences and Unconscious mind and inter- Ego, anxiety, identity, libido
experiences in unconscious mind social influences on the mind personal relationships (IPRs) concepts combined with
social forces
Continued
2164_Ch10_711-824 29/03/12 1:01 PM Page 714

714 chapter 10 Psychosocial Integrity

Table 10.3
Summary of Theories of Psychosocial Development Throughout the Life Cyclecontd
Freud Piaget Sullivan Erikson
Change Possible With:
Understanding content and Socialization process to facil- Improved IPRs and under- Integration of attitudes,
meaning of unconscious itate cognitive development standing basic good-bad libido, and social roles for
MENTAL HEALTH

transformations strong ego identity


Age Group:
First 5 yr of life Middle childhood years Adolescence Middle age, old age
Focus on:
Emotional development Cognitive skills Emotional and interpersonal Emotional, interpersonal,
development; relationships spiritual
with opposite sex
Psychosexual aspects Cognitive, interactive Psychosocial aspects; Psychosocial aspects
aspects developing sense of
identity
Cause of Conflicts and Problems:
Oral, anal, genital stage problems Faulty adaptation between Threats to self-system; Unresolved conflicts, crises
(especially unresolved oedipal/ individual and environment disturbed communication in 8 successive life cycle
castration conflicts) for intellectual development process; 7 stages not stages
complete
Prognosis:
Few changes possible after age 5 Little change in adult Change usually possible with Change not only possible
cognitive structure improved IPRs but expected throughout
after middle life
adolescence
Sexual problems part of Sex as a variable in Sexual problems are only one Sexual identity as one of
disturbed behavior learning (age, IQ) type of faulty IPRs affecting many problems solved by
behavior interaction of desire and
social process

2. Hierarchy of needsphysiological, safety, love e. Stress management: can delay seeking


and belonging, self-esteem and self-recognition, gratification and relief; does not blame or
self-actualization, aesthetic. As each stage is dwell on past; assumes self-responsibility;
mastered, the next stage becomes dominant either modifies own expectations, seeks
(Fig. 10.1). substitutes, or withdraws from stressful
3. Characteristics of optimal mental healthkeep in situation when cannot reduce stress.
mind that wellness is on a continuum with C. Jean PiagetCognitive and Intellectual Development
cultural variations. (1963)
a. Self-esteem: entails self-confidence and 1. Assumptionschild development is
self-acceptance. steered by interaction of environmental
b. Self-knowledge: involves accurate self- and genetic influences; therefore, focus is
perception of strengths and limitations. on environmental and social forces (see
c. Satisfying interpersonal relationships: able to Table 10.3 for comparison with other
meet reciprocal emotional needs through theories).
collaboration rather than by exploitation or 2. Key concepts:
power struggles or jealousy; able to make full a. Assimilation: process of acquiring new knowl-
commitments in close relationships. edge, skills, and insights by using what the
d. Environmental mastery: can adapt, change, and child already knows and has.
solve problems effectively; can make decisions, b. Accommodation: adjusts to change by solving
choose from alternatives, and predict conse- previously unsolvable problems because of
quences. Actions are conscious, not impulsive. newly assimilated knowledge.
2164_Ch10_711-824 29/03/12 1:01 PM Page 715

Psychosocial Growth and Development 715

MENTAL HEALTH
AESTHETIC
(Beauty, spirituality)

SELF-
ACTUALIZATION
(The individual
possesses a
feeling of self-
fulfillment and
the realization of
his or her highest
potential.)

SELF-ESTEEM
ESTEEM-OF-OTHERS
(The individual seeks self-respect
and respect from others, works to
achieve success and recognition in
work, and desires prestige from
accomplishments.)

LOVE AND BELONGING


(Needs are for: giving and receiving of
affection, companionship, satisfactory
interpersonal relationships, and
identification with a group.)

SAFETY AND SECURITY


(Needs at this level are for: avoiding harm, maintaining
comfort, order, structure, physical safety, freedom from
fear, and protection.)

PHYSIOLOGICAL NEEDS
(Basic fundamental needs include: food, water, oxygen, air, sleep, exercise,
elimination, shelter, and sexual expression.)

Figure 10.1 Maslows hierarchy of needs. (Modified from Townsend M: Essentials of Psychiatry/Mental Health Nursing. FA Davis, Philadelphia.)

c. Adaptation: coping process to handle environ- e. Negotiating boundaries of couple with families.
mental demands. f. Discussing issue of childbearing.
3. Age-specific developmental levelssensorimotor, 2. Childbearing years:
preconceptual, intuitive, concrete, formal oper- a. Working out authority, responsibility, and
ational thought (Table 10.4). caregiver roles.
D. E. M. DuvallFamily Development (1971) b. Having children and forming new unit.
developmental tasks are family oriented, presented c. Facilitating childs trust.
in eight stages throughout the life cycle. d. Need for personal time and space while
1. Married couple: sharing with each other and child.
a. Establishing relationship. 3. Preschool-age years:
b. Defining mutual goals. a. Experiencing changes in energy.
c. Developing intimacy: issues of dependence- b. Continuing development as couple, parents,
independence-interdependence. family.
d. Establishing mutually satisfying c. Establishing own family traditions without
relationship. guilt related to breaks with tradition.
2164_Ch10_711-824 29/03/12 1:01 PM Page 716

716 chapter 10 Psychosocial Integrity

Table 10.4 influences by providing anticipatory guidance and


maximizing strengths.
Piagets Age-Specific Development Levels B. Secondary preventiondecrease disability by
Age Stage Abilities shortening its duration and reducing its severity
Infancy2 yr Sensorimotor Preverbal; uses all senses; through detection of early-warning signs and
coordinates simple motor effective intervention following case-finding.
actions C. Crisis intervention (see pp. 795796).
MENTAL HEALTH

24 yr Preconceptual Can use language; egocentric; D. Tertiary preventionavoid permanent disorder


imitation in play, parallel play through rehabilitation.
47 yr Intuitive Asks questions; can use sym- IV. BEHAVIORAL MODEL (Pavlov, Watson, Wolpe,
bols and associate subjects Skinner)
with concepts
A. Assumptions:
711 yr Concrete Sees relationships, aware of 1. Roots in neurophysiology (i.e., neurotransmitter
viewpoints; understands
cause and effect, can make
functions versus effects).
conclusions; solves concrete 2. Stimulus-response learning can be conditioned
problems through reinforcement.
11 yr and Formal Abstract and conceptual 3. Behavior is what one does.
older operational thinking; can check ideas, 4. Behavior is observable, describable, predictable,
thought thoughts, and beliefs; lives in and controllable.
present and nonpresent; can 5. Classification of mental disease is clinically
use formal logic and scientific
useless, only provides legal labels.
reasoning
B. Aim: change observable behavior. There is no
From Lagerquist, S: Nursing Examination Review, ed 4. Addison-Wesley, underlying cause, no internal motive.
Redwood City, CA.
V. COMPARISON OF MODELS: see Table 10.3 for
comparison of four theories.
4. School-age years:
a. Establishing new roles in work.
b. Childrens school activities interfering with Body Image Development
family activities.
5. Teenage years: and Disturbances Throughout
a. Parents continue to develop roles in
community other than with children.
the Life Cycle
b. Children experience freedom while accepting I. DEFINITION Mental picture of bodys appear-
responsibility for actions. ance; an interrelated phenomenon which includes
c. Struggle with parents in emancipation the surface, depth, internal and postural picture
process. of the body, as well as the attitudes, emotions,
d. Family value system is challenged. and personality reactions of the individual in
e. Couple relationships may be strong or weak relation to his body as an object in space, apart
depending on responses to needs. from all others.*
6. Families as launching centers: II. OPERATIONAL DEFINITION
a. Young adults launched with rites of passage. A. Body image is created by social interaction.
b. Changes in couples relationship due to empty 1. Approval given for normal and proper
nest and increased leisure time. appearance, gestures, posture, etc.
c. Changes in relationship with children away 2. Behavioral and physical deviations from
from home. normality not given approval.
7. Middle-aged parents: Dealing with issues of aging 3. Body image formed by the persons response to
of own parents. the approval and disapproval of others.
8. Aging family members: 4. Persons values, attitudes, and feelings about
a. Sense of accomplishment and desire to self continually evolving and unconsciously
continue to live fully. integrated.
b. Coping with bereavement and living alone.
III. COMMUNITY MENTAL HEALTH MODEL
(Gerald Kaplan)levels of prevention *Adapted from Kolb, L: Disturbances in body image. In Arieti, S (ed): American Handbook
of Psychiatry. Basic Books, New York.
A. Primary preventionlower the risk of mental Adapted from Norris, C: Body image. In Carlson, C, & Blackwell, B (eds): Behavioral
illness and increase capacity to resist contributory Concepts and Nursing Intervention, ed 2. JB Lippincott, Philadelphia.
2164_Ch10_711-824 29/03/12 1:01 PM Page 717

Body Image Development and Disturbances Throughout the Life Cycle 717
B. Self-image, identity, personality, sense of self, and therefore, there is variability in degree of threat,
body image are interdependent. personality integrity, and coping behavior.
C. Behavior is determined by body image. K. Attitudes concerning the self will influence and be
III. CONCEPTS RELATED TO PERSONS WITH influenced by persons physical appearance and
PROBLEMS OF BODY IMAGE ability. Society has developed stereotyped ideas
regarding outer body structure (body physique)
A. Image of self changes with changing posture
and inner personalities (temperament). Current

MENTAL HEALTH
(walking, sitting, gestures).
stereotypes are:
B. Mental picture of self may not correspond with
1. Endomorphtalkative, sympathetic, good
the actual body; subject to continual but slow
natured, trusting, dependent, lazy, fat.
revision.
2. Mesomorphadventuresome, self-reliant,
C. The degree to which people like themselves
strong, tall.
(good self-concept) is directly related to how
3. Ectomorphthin, tense and nervous, suspicious,
well defined they perceive their body image
stubborn, pessimistic, quiet.
to be.
L. Person with a firm ego boundary or body image is
1. Vague, indefinite, or distorted body image corre-
more likely to be independent, striving, goal
lates with the following personality traits:
oriented, influential. Under stress, may develop
a. Sad, empty, hollow feelings.
skin and muscle disease.
b. Mistrustful of others; poor peer relations.
M. Person with poorly integrated body image and weak
c. Low motivation.
ego boundary is more likely to be passive, less goal
d. Shame, doubt, sense of inferiority, poor
oriented, less influential, more prone to external
self-concept.
pressures. Under stress, may develop heart and
e. Inability to tolerate stress.
GI diseases.
2. Integrated body image tends to correlate positively
N. Any situation, illness, or injury that causes a
with the following personality traits:
change in body image is a crisis, and the person
a. Happy, good self-concept.
will go through the phases of crisis in an attempt to
b. Good peer relations.
reintegrate the body image (Table 10.5).
c. Sense of initiative, industry, autonomy,
identity. IV. ASSESSMENT: (Table 10.6).
d. Able to complete tasks. V. ANALYSIS/NURSING DIAGNOSISbody image
e. Assertive. development disturbance may be related to:
f. Academically competent; high achievement. A. Obvious loss of a major body partamputation of
g. Able to cope with stress. an extremity; hair, teeth, eye, breast.
D. Childs concept of body image can indicate degree B. Surgical procedures in which the relationship of
of ego strength and personality integration; vague, body parts is visibly disturbedcolostomy,
distorted self-concept may indicate schizophrenic ileostomy, gastrostomy, ureteroenterostomy.
processes. C. Surgical procedures in which the loss of body
E. Successful completion of various developmental parts is not visible to othershysterectomy, lung,
phases determines body concept and degree of body gallbladder, stomach.
boundary definiteness. D. Repair procedures (plastic surgery) that do not
F. Physical changes of height, weight, and body build reconstruct body image as assumedrhinoplasty,
lead to changes in perception of body appearance plastic surgery to correct large ears, breasts.
and of how body is used. E. Changes in body size and proportionobesity,
G. Success in using ones body (motor ability) emaciation, acromegaly, gigantism, pregnancy,
influences the value one places on self pubertal changes (too early, too late, too big, too
(self-evaluation). small, too tall, too short).
H. Secondary sex characteristics are significant aspects F. Other changes in external body surfacehirsutism
of body image (too much, too little, too early, too in women, mammary glands in men.
late, in the wrong place, may lead to disturbed G. Skin color changeschronic dermatitis, Addisons
body image). Sexual differences in body image disease.
are in part related to differences in anatomical H. Skin texture changesscars, thyroid disease,
structure and body function, as well as to excoriative dermatitis, acne.
contrasts in lifestyles and cultural roles. I. Crippling changes in bones, joints, muscles
I. Different cultures and families value bodily traits arthritis, multiple sclerosis, Parkinsons disease.
and bodily deviations differently. J. Failure of a body part to functionquadriplegia,
J. Different body parts (e.g., hair, nose, face, stature, paraplegia, stroke (brain attack).
shoulders) have varying personal significance;
2164_Ch10_711-824 29/03/12 1:01 PM Page 718

718 chapter 10 Psychosocial Integrity

Table 10.5
Four Phases of Body Image Crisis
Phase Assessment Nursing Care Plan/Implementation
Acute shock Anxiety, numbness, helplessness. Provide sustained support, be available to listen, express
interest and concern.
Allow time for silence and privacy.
MENTAL HEALTH

Denial Retreats from reality; fantasy about the Accept denial without reinforcing it. Avoid arguing and
wholeness and capability of the body; overloading with reality. Gradually raise questions, reply
euphoria; rationalization; refusal to with doubt to convey unrealistic ideas.
participate in self-care. Follow clients suggestions for personal-care routine to help
increase feelings of adequacy and to decrease helplessness.
Acknowledgment Grief over loss of valued body part, Expect and accept displacement onto nurse of anger,
of reality function, or role; depression, apathy; resentment, projection of clients inadequacy.
agitation, bitterness; physical symptoms Examine own behavior to see if clients remarks are justified.
(insomnia, anorexia, nausea, crying) Simply listen if this is the only way the client can handle
serve as outlet for feeling; redefinition feelings at this time.
of body structure and function, with Offer sustained, nonjudgmental listening without being
implications for change in lifestyle; defensive or taking remarks personally.
acceptance of and cooperation with Help dispel anger by encouraging its ventilation.
realistic goals for care and treatment; Encourage self-care activities.
preoccupation with body functions. Support family members as they cope with changes in
clients health or body image, role changes, treatment plans.
Resolution and Perceives crisis in new light; increased Teaching and counseling by same nurse in warm, supportive
adaptation mastery leads to increased self-worth; relationship.
can look at, feel, and ask questions Assess level of knowledge; begin at that level.
regarding altered body part; tests Consider motivational state.
others reactions to changed body; Provide gradual, nontechnical medical information and
repetitive talk on painful topic of specific facts.
changed self; concentration on normal Repeat instructions frequently, patiently, consistently.
functions in order to increase sense of Support sense of mastery in self-care; draw on inner
control. resources.
Do not discourage dependence while gradually encouraging
independence.
Focus on necessary adaptations of lifestyle due to realistic
limitations.
Provide follow-up care via referral to community resources
after client is discharged.
From Lagerquist, S: Nursing Examination Review, ed 4. Addison-Wesley, Redwood City, CA.

Table 10.6
Body Image Development and Disturbance Throughout the Life Cycle: Assessment
Age Group Development of Body Image Developmental Disturbances in Body Image
Infant and toddler Becomes aware of body boundaries Infant
and separateness of own external Inadequate somatosensory stimulation impaired ego
body from others through sensory development, increased anxiety level, poor foundation for
stimulation. reality testing.
Explores external body parts; handles Continues to see external objects as extension of self
and controls the environment and unrealistic, distorted perceptions of significant persons,
body through play, bathing, and inability to form normal attachments to others (possessive,
eating. engulfing, autistic, withdrawn).
Experiences pain, shame, fear, and Toddler
pleasure. Feels doubt or power in If body fails to meet parental expectations shameful,
mastery of motor skills and strives self-deprecating feelings.
for autonomy. Learns who one is in Failure to master environment and control own body
relation to the world. helplessness, inadequacy, and doubt.
2164_Ch10_711-824 29/03/12 1:01 PM Page 719

Body Image Development and Disturbances Throughout the Life Cycle 719

Table 10.6
Body Image Development and Disturbance Throughout the Life Cycle: Assessmentcontd
Age Group Development of Body Image Developmental Disturbances in Body Image
Preschool and Preschool
school age Experiences praise, blame, derogation, Distortion of body image of genital area due to conflict over
or criticism for body, its parts, or use pleasure versus punishment.

MENTAL HEALTH
(pleasure, pain, doubt, or guilt).
Explores genitalsdiscovers If body build does not conform to sex-typed expectations
anatomical differences between and sex role identification body image confusion.
sexes with joy, pride, or shame.
Begins awareness of sexual identity.
Differentiates self as a body and self as
a mind.
School age
Beginning of self-concept; of self as Physical impairments (speech, poor vision, poor hearing)
man or woman. feelings of inadequacy and inferiority.
Learns mastery of the body (to do, Overly self-conscious about, and excessive focus on, body
to protect self, to protect others) and changes in puberty.
environment (run, skip, skate, swim);
feels pleasure, competence, worth, or
inadequacy.
Adolescent Physical self is of more concern than at Growth and changes may produce distorted view of self
any other time except old age. overemphasis on defects with compensations; inflated ideas
Forced body awareness due to physical of body ability, beauty, perfection; preoccupation with body
changes (new senses, proportions, appearance or body processes, women more likely than men
features); feelings of pleasure, power, to see body fatter than it is; egocentrism.
confidence, or helplessness, pain,
inadequacy, doubt, and guilt.
Adult body proportions emerge.
Anxiety over ideal self versus emerging/
emerged physical self; body is compared
competitively with same-sex peers.
Use of body (adolescents values and
attitudes) to relate with opposite sex.
Body image crucial for self-concept
formation, status achievement, and
adequate social relations.
Physical changes need to be integrated
into evolving body image (strong, com-
petent, powerful, or weak and helpless).
Early adulthood Learns to accept own body without
undue preoccupation with its functions
or control of these functions.
Stability of body image. Less dependable, less likable body regression to adolescent
behavior and dress due to denial of aging, defeat, depression,
self-pity, egocentrism due to fear of loss of sexual identity, with-
drawal to early old age.
Middle age New challenges due to differential rates Women more likely to judge themselves uglier than do men
of aging in various body parts. or younger and older women.
Body not functioning as well; unre-
solved fears, misconceptions, and
experiences in relation to body image
persist and become recognized.
Old age Accelerated physical decline with influ- Ill health fear of invalidism, hypochondriasis.
ence on self-concept and lifestyle. Denial related to feelings of threatened incapacity and fear of
Can accept self and personality as a declining functions.
whole; continued emphasis on physical Despair over loss of beauty, strength, and youthfulness, with
self, with increased emphasis on inner, self-disgust about body projection of criticism onto others.
emotional self. Regression.
Isolation (separation of affect and thought) leads to less
intense response to death, disease, aging.
Compartmentalization (focus on one thing at a time) causes
narrowing of consciousness, resistance, rigidity, repetitiveness.
Resurgence of egocentrism.
2164_Ch10_711-824 29/03/12 1:01 PM Page 720

720 chapter 10 Psychosocial Integrity

K. Distorted ideas of structure, function, and signifi- VII. EVALUATION/OUTCOME CRITERIA:


cance stemming from symbolism of disease seen in A. Able to resume function in activities of daily living
terms of life and death when heart or lungs are rather than prolonging illness.
afflictedheart attacks, asthmatic attacks, B. Able to accept limits imposed by physical
pneumonia. or mental conditions and not attempt
L. Side effects of drug therapymoon facies, hir- unrealistic tasks.
sutism, striated skin, changes in body contours. C. Can shift focus from reminiscence about the
MENTAL HEALTH

M. Violent attacks against the bodyincest, rape, healthy past to present and future.
shooting, knifing, battering. D. Less verbalized discontent with present body;
N. Mental, emotional disordersschizophrenia diminished display of self-displeasure, despair,
with depersonalization, somatic delusions, and weeping, and irritability.
hallucinations about the body; anorexia nervosa,
hypochondriasis; hysteria, malingering.
O. Diseases requiring isolation may convey attitude Body Image DisturbanceSelected
that body is undesirable, unacceptable
tuberculosis, AIDS, malodorous conditions Examples
(e.g., gangrene, cancer). I. DEFINITIONa body image disturbance arises
P. Womens movement and sexual revolutionuse of when a person is unable to accept the body as is and
body for pleasure, not just procreation, sexual to adapt to it; a conflict develops between the body as
freedom, wide range of normality in sex practices, it actually is and the body that is pictured mentally
legalized abortion. that is, the ideal self.
Q. Medical technologyorgan transplants, lifesaving
but scar-producing burn treatment, alive but hope- II. ANALYSIS/NURSING DIAGNOSIS: body image
less, alive but debilitated with chronic illnesses. disturbance may be related to:
A. Sensation of size change due to obesity, pregnancy,
VI. GENERAL NURSING CARE PLAN/
weight loss.
IMPLEMENTATION:
B. Feelings of being dirtymay be imaginary due to
A. Protect from psychological threat related to impaired hallucinogenic drugs, psychoses.
self-attitudes. C. Dual change of body structure and function due to
1. Emphasize persons normal aspects. trauma, amputation, stroke, etc.
2. Encourage self-performance. D. Progressive deformities due to chronic illness, burns,
B. Maintain warm, communicating relationship. arthritis.
1. Encourage awareness of positive responses from E. Loss of body boundaries and depersonalization
others. due to sensory deprivation, such as blindness,
2. Encourage expression of feelings. immobility, fatigue, stress, anesthesia. May also
C. Increase reality perception. be due to psychoses or hallucinogenic drugs.
1. Provide reliable information about health status.
2. Provide kinesthetic feedback to paralyzed part III. ASSESSMENT: see Table 10.6.
(e.g., I am raising your leg.). Body Image Disturbance Caused
3. Provide perceptual feedback (e.g., touch,
describe, look at scar). by Amputation
4. Support a realistic assessment of the situation. A. Assessment:
5. Explore with the client his or her strengths and 1. Loss of self-esteem; feelings of helplessness,
resources. worthlessness, shame, and guilt.
D. Help achieve positive feelings about self, about 2. Fear of abandonment may lead to appeals for
adequacy. sympathy by exhibiting helplessness and
1. Support strengths despite presence of handicaps. vulnerability.
2. Assist client to look at self in totality rather than 3. Feelings of castration (loss of self ) and symbolic
focus on limitations. death; loss of wholeness.
E. Health teaching: 4. Existence of phantom pain (most clients).
1. Teach client and family about expected changes 5. Passivity, lack of responsibility for use of disabled
in functioning. body parts.
2. Explain importance of maintaining a positive B. Nursing care plan/implementation:
self-attitude. 1. Avoid stereotyping person as being less compe-
3. Advise that negative responses from others be tent now than previously by not referring to
regarded with minimum significance. client as the amputee.
2164_Ch10_711-824 29/03/12 1:01 PM Page 721

Body Image DisturbanceSelected Examples 721


2. Foster independence; encourage self-care by c. Direct visual contact via mirrors and grooming.
assessing what client can do for himself or d. Use of safety features (e.g., Posey belt).
herself. 3. Health teaching: control of bowel and bladder
3. Help person set realistic short-term and function; how to prevent problems of
long-term goals by exploring with the client immobility.
his or her strengths and resources. C. Evaluation/outcome criteria: dignity is main-
4. Health teaching: tained while relearning to control elimination.

MENTAL HEALTH
a. Encourage family members to work through
their feelings, to accept person as he or she Body Image Disturbance
presents self. in Myocardial Infarction
b. Teach how to set realistic goals and
Emotional problems (e.g., anxiety, depression, sleep
limitations.
disturbance, fear of another myocardial infarction
c. Explain what phantom pain is; that it is a
[MI]) during convalescence can seriously hamper reha-
normal experience.
bilitation. The adaptation and convalescence are influ-
d. Explain role and function of prosthetic
enced by the multiple symbolic meanings of the heart,
devices, where and how to obtain them, and
for example:
how to find assistance in their use.
C. Evaluation/outcome criteria: 1. Seat of emotions (love, pride, fear, sadness).
1. Can acknowledge the loss and move 2. Center of the body (one-of-a-kind organ).
through three stages of mourning (shock 3. Life itself (can no longer rely on the heart;
and disbelief, developing awareness, and failure of the heart means failure of life).
resolution).
A. Assessment:
2. Can discuss fears and concerns about loss
1. Attitudeoverly cautious and restrictive; may
of body part, its meaning, the problem of
result in boredom, weakness, insomnia,
compensating for the loss, and reaction of
exaggerated dependency.
persons (repulsion, rejection, and sympathy).
2. Acceptance of illnessuse of denial may result in
noncompliance.
Body Image Disturbance 3. Behaviorself-destructive.
in Brain Attack (Stroke) 4. Family conflictsrelated to activity, diet.
A. Assessment: 5. Effects of MI on:
1. Feelings of shame (personal, private, self- a. Changes in lifestyleeating, smoking,
judgment of failure) due to loss of bowel drinking; activities, employment, sex.
and bladder control, speech function. b. Family membersmay be anxious,
2. Body image boundaries disrupted; contact with overprotective.
environment is hindered by inability to ambulate c. Role in familyrole reversal may result in loss
or manipulate environment physically; may result of incentive for work.
in personality deterioration due to diminished d. Dependence-independenceissues related to
number of sensory experiences. Loses orientation family conflicts (especially restrictive attitudes
to body sphere; feels confused, trapped in own about desirable activity and dietary regimen).
body. e. Jobsocial pressure to slow down may
B. Nursing care plan/implementation: result in loss of job, reassignment, forced early
1. Reduce frustration and infantilism due to retirement, has-been social status.
communication problems by: B. Nursing care plan/implementation:
a. Rewarding all speech efforts. 1. Prevent cardiac cripple by shaping persons and
b. Listening and observing for all familys attitude toward damaged organ.
nonverbal cues. a. Instill optimism.
c. Restating verbalizations to see if correct b. Encourage productive living rather than
meaning is understood. inactivity.
d. Speaking slowly, using two- to three-word 2. Set up a physical and mental activity program
sentences. with client and mate.
2. Assist reintegration of body parts and function; 3. Provide anticipatory guidance regarding expected
help regain awareness of paralyzed side by: weakness, fear, uncertainty.
a. Tactile stimulation. 4. Health teaching: nature of coronary disease,
b. Verbal reminders of existence of affected interpretation of medical regimen, effect on
parts. sexual behavior.
2164_Ch10_711-824 29/03/12 1:01 PM Page 722

722 chapter 10 Psychosocial Integrity

C. Evaluation/outcome criteria: 6. Ethnic and family practices.


1. Adheres to medical regimen. 7. Psychological:
2. Modifies lifestyle without becoming overly a. Compensation for feelings of helplessness and
dependent on others. inadequacy.
b. Maternal overprotection; overfed and force-fed,
Body Image and Obesity especially infants who are formula-fed.
(see Chapter 6) c. Food offered and used to relieve anxiety, frus-
MENTAL HEALTH

A. Definition: body weight exceeding 20% above the tration, anger, and rage can lead to difficulty in
norm for persons age, sex, and height constitutes differentiating between hunger and other needs.
obesity. Body mass index (BMI) is also used. d. As a child, food offered instead of love.
Although a faulty adaptation, obesity may serve as 8. Social:
a protection against more severe illness; it repre- a. Food easily available.
sents an effort to function better, be powerful, stay b. Use of motorized transportation and
well, or be less sick. The problem may not be diffi- labor-saving devices.
culty in losing weight; reducing may not be the c. Refined carbohydrates.
appropriate cure. d. Social aspects of eating.
B. Assessmentcharacteristics: e. Restaurant meals high in salt, sugar,
1. Ageone out of three persons under 30 years trans-fats, and larger portions.
of age is more than 10% overweight. D. Nursing care plan/implementation:
2. Increase risks for stroke, MI, diabetes. 1. Encourage prevention of lifelong body image
3. Feelings: self-hate, self-derogation, failure, problems.
helplessness; tendency to avoid clothes a. Support breastfeeding, where infant determines
shopping and mirror reflections. quantity consumed, not mother; work
4. Viewed by others as ugly, repulsive, lacking in through her feelings against breastfeeding
will power, weak, unwilling to change, neurotic. (fear of intimacy, dependence, feelings of
5. Discrepancy between actual body size (real self ) repulsion, concern about confinement, and
and persons concept of it (ideal self ). inability to produce enough milk).
6. Pattern of successful weight loss followed b. Help mothers to not overfeed the infant if
quickly and repetitively by failure; that is, formula-fed: suggest water between feedings;
weight gain. do not start solids until 6 months old or
7. Eating in response to outer environment 14 pounds; do not enrich the prescribed
(e.g., food odor, time of day, food availability, formula.
degree of stress, anger); not inner environment c. Help mothers differentiate between hunger
(hunger, increased gastric motility). and other infant cries; help mothers to try
8. Experiences less pleasure in physical activity; out different responses to the expressed needs
less active than others. other than offering food.
9. All people who are obese are not the same. 2. Use case findings of infants who are obese,
a. In newborns and infants who are obese, there as well as young children, and adolescents.
is an increased number of adipocytes via 3. Assess current eating patterns.
hyperplastic process. 4. Identify need to eat, and relate need to preceding
b. In adults who are obese, there may be events, hopes, fears, or feelings.
increased body fat deposits, resulting in 5. Employ behavior modification techniques.
increased size of adipocytes via hypertrophic 6. Encourage outside interests not related to food
process. or eating.
c. When an infant who is obese becomes an 7. Alleviate guilt, reduce stigma of being obese.
adult who is obese, the result may be an 8. Health teaching:
increased number of cells available for a. Promote awareness of certain stressful periods
fat storage. that can produce maladaptive responses such
10. Loss of control of own body or eating behavior. as obesity (e.g., puberty, postnuptial, postpar-
C. Analysis/nursing diagnosis: defensive coping relat- tum, menopause).
ed to eating disorder. Contributing factors: b. Assist in drawing up a meal plan for slow,
1. Genetic. steady weight loss.
2. Thermodynamic. c. Advise eating five small meals a day and
3. Endocrine. increase exercises.
4. Neuroregulatory. E. Evaluation/outcome criteria: goal for desired
5. Biochemical factors in metabolism. weight is reached; weight-control plan is continued.
2164_Ch10_711-824 29/03/12 1:01 PM Page 723

Human Sexuality Throughout the Life Cycle 723

Human Sexuality Throughout and femaleness; also includes gender


identity.
the Life Cycle C. Gender identitya sense of masculinity and
femininity shaped by biological, environmental,
Human sexuality refers to all the characteristics of an indi- and intrapsychic forces, as well as cultural tradi-
vidual (social, personal, and emotional) that are manifest tions and education.
in his or her relationships with others and that reflect D. Sex role behaviorincludes components of both

MENTAL HEALTH
gender-genital orientation. sexual identity and gender identity. Aim: sexual
I. COMPONENTS OF SEXUAL SYSTEM fulfillment through masturbation, heterosexual,
A. Biological sexualityrefers to chromosomes, or homosexual experiences. Selection of behavior
hormones, primary and secondary sex is influenced by personal value system and sexual,
characteristics, and anatomical structure. gender, and biological identity. Gender identity
B. Sexual identitybased on own feelings and and roles are learned and constantly reinforced by
perceptions of how well traits correspond input and feedback regarding social expectations
with own feelings and concepts of maleness and demands (Table 10.7).

Table 10.7
Sexual Behavior Throughout the Life Cycle
Age Development of Sexual Behavior
First 18 mo Major source of pleasure from touch and oral exploration.
18 mo3 yr Pleasurable and sexual feelings are associated with genitals (acts of urination and defecation). Masturbation
without fantasy or eroticism.
36 yr Beginning resolution of Oedipus and Electra complexes; foundation for heterosexual relationships; masturba-
tion with curiosity about genitals of opposite sex.
612 yr Peer relations with same sex; onset of sex play; morality and sexual attitudes taught and learned; phase of
sexual tranquility.
1218 yr Onset of puberty with biological development of secondary sex characteristics; menstruation and ejaculation occur.
(adolescence)
Frequent masturbation. Intense anxiety and guilt may occur over heterosexual or homosexual behavior
(petting, coitus, masturbation, STI, pregnancy, genital size).
1823 yr (early Maximum interpersonal and intrapsychic self-consciousness about sexuality. Issues: premarital coitus, sexual
adulthood) freedom.
Anxiety about: sexual competency, genital size, impotence, fear of pregnancy, rejection.
2330 yr Focus on sexual activity in coupling and parenthood; mutual masturbation.
3045 yr (middle For womenpeak sexuality without new sexual experiences. Conflict regarding extramarital sex may increase.
adulthood) Purpose of Intercourse:
Need for body contact (and procreation until age 35+).
Physical expression of trust, love, and affection.
Reaffirmation of self-concept, as sexually desirable and sexually competent due to worry about effects of aging.
Sexual Dysfunctions:
Men: erectile dysfunction, premature ejaculation, decreasing libido.
Women: intermittent lack of orgasmic response, vaginismus, dyspareunia.
For either or both: changes or divergences in degree of sexual interest.
Causes of Sexual Dysfunction (Men):
Overindulgence in food or drink.
Preoccupation with career and economic pursuits.
Mental or physical fatigue.
Boredom with monotony of relationship.
Drug dependency: alcohol, tobacco, certain medications.
Fear of failure.
Chronic illness: diabetes, alcoholism peripheral neuropathy impotence (smoking and drinking may result
in decreased testosterone production); excessive smoking vascular constriction decreased libido; spinal
cord injuries; prostatectomy, androgen deprivation therapy (for prostate cancer).
Self-devaluation due to accumulation of role function losses, sexual self-image, and body image.
Past history of lack of sexual enjoyment in younger years.
Causes of Sexual Dysfunction (Women):
Belief in myths regarding shoulds and should nots of frequency, variations, and enjoyment.
Widowhood: inhibition and loyalty to deceased.
Continued
2164_Ch10_711-824 29/03/12 1:01 PM Page 724

724 chapter 10 Psychosocial Integrity

Table 10.7
Sexual Behavior Throughout the Life Cyclecontd
Age Development of Sexual Behavior
4565 yr (later Menopause occurs.
adulthood)
Little or no fear of pregnancy; evidence of sexual activity differences in men and women: women may have
MENTAL HEALTH

increased pleasure, men take longer to reach orgasm; may prefer less strenuous mutual masturbation.
Over 65 yr Activity depends on earlier sexual attitude.
(old age) May suffer guilt and shame when engaging in sex.
Can have active and enjoyable sex life with continuing sex needs.
Age is not a barrier provided there is opportunity for sexual activity with a partner or for sublimated activities.
Women in this age group outnumber men; single women outnumber single men by an even larger margin.
From Lagerquist, S: Nursing Examination Review, ed 4. Addison-Wesley, Redwood City, CA.

II. CONCEPTS AND PRINCIPLES OF HUMAN F. Masturbationa natural behavior at all ages; can fulfill
SEXUAL RESPONSE a variety of needs (see Masturbation, p. 726).
A. Human sexual response involves not only the G. Availability of contraception for minorsthe right
genitals but the total body. of access to medical contraceptive care should be
B. Factors in early postnatal and childhood periods available to all ages.
influence gender identity, gender role, sex typing, H. Abortionconfidentiality for minors.
and sexual responses in later life. I. Treatment for sexually transmitted infections (STIs)
C. Cultural and personally subjective variables influ- naming of partners as part of STI control.
ence ways of sexual expression and perception of J. Sex and the elderlyneed opportunity for sexual
what is satisfying. expression; need privacy when in communal living
D. Healthy sexual expressions vary widely. setting.
E. Requirements for human sexual response: K. Sex and the disabledneed to have possible means
1. Intact central and peripheral nervous system to available for rewarding sexual expressions.
provide sensory perception, motor reaction. II. SEXUAL MYTHS*
2. Intact circulatory system to produce vasoconges- A. Myth: Ignorance is bliss.
tive response. Fact: What you dont know can hurt you (note the
3. Desirable and interested partner, if sex outlet high frequency of STI and abortions); myths can
involves mutuality. perpetuate fears and such misinformation as:
4. Freedom from guilt, anxiety, misconceptions, 1. Masturbation causes mental illness.
and interfering conditioned responses. 2. Women dont or shouldnt have orgasms.
5. Acceptable physical setting, usually private. 3. Tampons cause STI.
4. Plastic wrap works better than condoms.
Sexual-Health Counseling 5. Coca-Cola is an effective douche.
Fact: Lack of knowledge during initial experiences
General Issues may result in fear and set precedent for future
I. ISSUES in sexual practices with implications for sexual reactions.
counseling: B. Myths: The planned sex act is not OK and is
immoral for nice girls. If a woman gets pregnant,
A. Sex educationneed to provide accurate and it is her own fault. Contraceptives are solely a
complete information on all aspects of sexuality womans responsibility.
to all people. Fact: Sex and contraception are the prerogative and
B. Sexual-health careshould be part of total health- responsibility of both partners.
care planning for all. C. Myth: A good relationship is harmonious, free of
C. Sexual orientationneed to avoid discrimination conflict and disagreement (which are signs of
based on sexual orientation (such as homosexuality); rejection and incompatibility).
the right to satisfying, nonexploitive relationships Fact: Conflict can induce growth in self-
with others, regardless of gender. understanding and in understanding of others.
D. Sex and the lawsex between consenting adults
not a legal concern.
E. Explicit sexual material (pornography)can be useful *Adapted from Sedgwick, R: Myths in human sexuality: a social-psychological perspective.
in fulfilling various needs in life, as in quadriplegia. Nurs Clin North Am 10(3):539550. Philadelphia, WB Saunders.
2164_Ch10_711-824 29/03/12 1:01 PM Page 725

Sexual-Health Counseling 725


D. Myth: Sexual deviance (such as homosexuality) is a 2. Use language related to sexual behavior that is
sign of personality disturbance. mutually comfortable and understood between
Fact: No single sexual behavior is the most desir- client and nurse.
able, effective, or satisfactory. Personal sexual a. Use alternative terms for definitions
choice is a fundamental right. (e.g., being intimate vs. having sex).
E. Myth: A womans sexual needs and gratification b. Determine exact meaning of words and phrases
should be secondary to her partners; a womans because sexual words and expressions have

MENTAL HEALTH
role is to satisfy others. different meanings to people with different
Fact: A woman has as much right to sexual freedom backgrounds and experiences.
and experience as a man. 3. Desensitize own stress reaction to the emotional
F. Myth: Menopause is an affliction signifying the end component of taboo topics.
of sex. a. Increase awareness of own sexual values,
Fact: Many women do not suffer through biases, prejudices, stereotypes, and fears.
menopause, and many report renewed sexual b. Avoid overreacting, underreacting.
interest. 4. Become sensitively aware of interrelationships
G. Myth: Sexual activity past 60 years of age is not between sexual needs, fears, and behaviors and
essential. other aspects of living.
Fact: Sexual activity is therapeutic because it: 5. Begin with commonly discussed areas (such as
1. Affirms identity. menstruation) and progress to discussion of indi-
2. Provides communication. vidual sexual experiences (such as masturbation).
3. Provides companionship. Move from areas where there is less voluntary con-
4. Meets intimacy needs. trol (nocturnal emissions) to more responsibility
H. Myth: A womans sex drive decreases in and voluntary behavior (premature ejaculation).
postmenopausal period. 6. Offer educational information to dispel fears,
Fact: The strength of the sex drive becomes greater myths; give tacit permission to explore sensitive
as androgen overcomes the inhibitory action of areas.
estrogen. 7. Bring into awareness possibly repressed feelings
I. Myth: Men over age 60 cannot achieve an of guilt, anger, denial, and suppressed sexual
erection. feelings.
Fact: According to Masters and Johnson, a major 8. Explore possible alternatives of sexual expression.
difference between the aging man and the 9. Determine interrelationships among mental,
younger man is the duration of each phase of social, physical, and sexual well-being.
the sexual cycle. The older man is slower in B. Assessment parameters:
achieving an erection. 1. Self-awareness of body image, values, and
J. Myth: Regular sexual activity cannot help the aging attitudes toward human sexuality; comfort
persons loss of function. with own sexuality.
Fact: Research is revealing that disuse atrophy 2. Ability to identify sex problems on basis of own
may lead to loss of sexual capacity. Regular satisfaction or dissatisfaction.
sexual activity helps preserve sexual function. 3. Developmental history, sex educatio

You might also like